Вы находитесь на странице: 1из 432

&kPDUD/HJLVODWLYDGR'LVWULWR)HGHUDO

 Ì1',&(
Ì1',&(

LÍNGUA PORTUGUESA
Interpretação de texto. .................................................................................................................................. 1

Argumentação. ............................................................................................................................................. 2

Pressupostos e subentendidos. ...................................................................................................................... 5

Níveis de linguagem. .................................................................................................................................... 7

Articulação do texto: coesão e coerência. ................................................................................................... 14

Termos da oração. Processos de coordenação e subordinação. ................................................................... 19

Discurso direto e indireto. ........................................................................................................................... 30

Tempos, modos e vozes verbais. Classes de palavras. ................................................................................. 32

Flexão nominal e verbal. ............................................................................................................................. 41

Concordância nominal e verbal. Regência nominal e verbal. ...................................................................... 43

Ocorrência da Crase. ................................................................................................................................... 49

Ortografia e acentuação. ............................................................................................................................. 50

Pontuação. .................................................................................................................................................. 56

Equivalência e transformação de estruturas. ............................................................................................... 57

Redação. ..................................................................................................................................................... 59
Ì1',&(  &kPDUD/HJLVODWLYDGR'LVWULWR)HGHUDO Ì1',&(

RACIOCÍNIO
LÓGICO-MATEMÁTICO
MATEMÁTICA
Números inteiros e racionais: operações (adição, subtração, multiplicação, divisão,
potenciação); expressões numéricas; múltiplos e divisores de números naturais; problemas. ...................... 1
Frações e operações com frações. ............................................................................................................... 13
Números e grandezas proporcionais ............................................................................................................ 30
Razões e proporções; .................................................................................................................................. 23
Divisão em partes proporcionais; ................................................................................................................ 30
Regra de três ................................................................................................................................................ 33
Porcentagem e problemas. .......................................................................................................................... 36

RACIOCÍNIO LÓGICO
Estrutura lógica de relações arbitrárias entre pessoas, lugares, objetos ou eventos fictícios; ........................ 1

Deduzir novas informações das relações fornecidas e avaliar as condições usadas para
estabelecer a estrutura daquelas relações. .................................................................................................... 2

Compreensão e elaboração da lógica das situações por meio de: raciocínio verbal,
raciocínio matemático, raciocínio sequencial, orientação espacial e temporal,
formação de conceitos, discriminação de elementos. .................................................................................. 24

Compreensão do processo lógico que, a partir de um conjunto de hipóteses, conduz, de forma válida,
a conclusões determinadas .......................................................................................................................... 10

REALIDADE DO DISTRITO FEDERAL


A realidade étnica, social, histórica, geográfica, cultural, política e econômica do Distrito Federal
e da Região Integrada de Desenvolvimento do Distrito Federal e Entorno – RIDE, instituída
pela Lei Complementar federal nº 94/1998. ............................................................................................ 1 a 6
&kPDUD/HJLVODWLYDGR'LVWULWR)HGHUDO
 Ì1',&(
Ì1',&(

12d¯(6'(
',5(,72
&2167,78&,21$/
'(/(,25*Ç1,&$'2',675,72)('(5$/
('(352&(662/(*,6/$7,92

3ULQFtSLRVIXQGDPHQWDLVGD&RQVWLWXLomR)HGHUDOGH                          
/3ULQFtSLRVIXQGDPHQWDLVGD&RQVWLWXLomR)HGHUDOGH
'LUHLWRVHJDUDQWLDVIXQGDPHQWDLV                                           
2UJDQL]DomRGR(VWDGR                                                  
3RGHU/HJLVODWLYR                                                       
/HLIHGHUDOQžTXHUHJXODRDFHVVRDLQIRUPDo}HVHPkPELWRQDFLRQDO
/HLIHGHUDOQž
//HLIHGHUDOQž     
/HLGLVWULWDOQžTXHUHJXODRDFHVVRDLQIRUPDo}HVQR'LVWULWR)HGHUDO     
/HL&RPSOHPHQWDUGLVWULWDOQƒ                                       
//HL&RPSOHPHQWDUGLVWULWDOQƒ
5HJLPHQWR,QWHUQRGD&kPDUD/HJLVODWLYDGR'LVWULWR)HGHUDO
5HJLPHQWR,QWHUQRGD&kPDUD/HJLVODWLYDGR'LVWULWR)HGHUDO
/5HJLPHQWR,QWHUQRGD&kPDUD/HJLVODWLYDGR'LVWULWR)HGHUDO
5HVROXomRQžFRQVROLGDGDSHOD5HVROXomRQƒ                 VLWH

37(67(6FRP            

',5(,72$'0,1,675$7,92
&RQFHLWRGH$GPLQLVWUDomR3~EOLFD                                           
/&RQFHLWRGH$GPLQLVWUDomR3~EOLFD
$WRVDGPLQLVWUDWLYRV                                                       
/$WRVDGPLQLVWUDWLYRV
&RQWUDWRV$GPLQLVWUDWLYRV                                                 
/&RQWUDWRV$GPLQLVWUDWLYRV
3RGHUHVGD$GPLQLVWUDomR                                                 
/3RGHUHVGD$GPLQLVWUDomR
/HLQž                                                       
//HLQž
/HLQž                                                       
//HLQž
/HJLVODomRGRVHUYLGRUS~EOLFRGR'LVWULWR)HGHUDO
//HJLVODomRGRVHUYLGRUS~EOLFRGR'LVWULWR)HGHUDO
/HL&RPSOHPHQWDUGLVWULWDOQž                                     
/HL&RPSOHPHQWDUGLVWULWDOQž                                     
/HLIHGHUDOQžUHFHSFLRQDGDSHOD/HLGLVWULWDOQž
/HLIHGHUDOQž
//HLIHGHUDOQž            

37(67(6FRP           

2VVtPERORVHPEOHPDVGLVWLQWLYRVHGHQRPLQDo}HVXWLOL]DGRVVmRPHUDPHQWHLOXVWUDWLYRV 'DWDGD(GLomR
QmRVLJQLILFDQGRDSRLRFRODERUDomRDXWRUL]DomRRXDERQRGDUHVSHFWLYD(QWLGDGH
HVyIRUDPHPSUHJDGRVSDUDEHPLGHQWLILFDUR&RQFXUVRDTXHVHUHIHUHHVWD$SRVWLOD 
Ì1',&(  &kPDUD/HJLVODWLYDGR'LVWULWR)HGHUDO Ì1',&(

NOÇÕES DE ADMINISTRAÇÃO
Noções de Administração: Ética no serviço público. . . . . . . . . . . . . . . . . . . . . . . . . . . . . . . . . . . . . . . . . . . .1

Modelos de gestão pública. . . . . . . . . . . . . . . . . . . . . . . . . . . . . . . . . . . . . . . . . . . . . . . . . . . . . . . . . . . . . .3

Gestão estratégica (planejamento estratégico no serviço público). . . . . . . . . . . . . . . . . . . . . . . . . . . . . . . .7

Qualidade no serviço público. Excelência no serviço público. . . . . . . . . . . . . . . . . . . . . . . . . . . . . . . . . . .11

Conceitos de eficiência, eficácia e efetividade aplicados à Administração Pública. . . . . . . . . . . . . . . . . . .17

Avaliação e mensuração do desempenho. . . . . . . . . . . . . . . . . . . . . . . . . . . . . . . . . . . . . . . . . . . . . . . . . .19

Caracterização das organizações: tipos de estruturas organizacionais. . . . . . . . . . . . . . . . . . . . . . . . . . . .23

Aspectos comportamentais: liderança, motivação, clima e cultura organizacional. . . . . . . . . . . . . . . . . . .29

Desenvolvimento Institucional. . . . . . . . . . . . . . . . . . . . . . . . . . . . . . . . . . . . . . . . . . . . . . . . . . . . . . . . . .39

Abordagens da mudança organizacional. . . . . . . . . . . . . . . . . . . . . . . . . . . . . . . . . . . . . . . . . . . . . . . . . . .43

Processo decisório: tipos de decisões. . . . . . . . . . . . . . . . . . . . . . . . . . . . . . . . . . . . . . . . . . . . . . . . . . . . .48

Gerenciamento de conflitos. . . . . . . . . . . . . . . . . . . . . . . . . . . . . . . . . . . . . . . . . . . . . . . . . . . . . . . . . . . . .50

Novas formas de gestão de serviços públicos: formas de supervisão e contratualização de resultados;


prestação de serviços públicos e novas tecnologias. . . . . . . . . . . . . . . . . . . . . . . . . . . . . . . . . . . . . . . . . . .53

Instrumentos gerenciais contemporâneos: gestão de processos, gestão de projetos,


gestão por resultados. . . . . . . . . . . . . . . . . . . . . . . . . . . . . . . . . . . . . . . . . . . . . . . . . . . . . . . . . . . . . . . . . .60
LÍNGUA PORTUGUESA
com a anterior e/ou com a posterior, criando condições para a
estruturação do conteúdo a ser transmitido. A essa interligação
dá-se o nome de contexto.

Intertexto - comumente, os textos apresentam referências


diretas ou indiretas a outros autores através de citações. Esse
tipo de recurso denomina-se intertexto.

Interpretação de Texto - o primeiro objetivo de uma


interpretação de um texto é a identificação de sua ideia
principal. A partir daí, localizam-se as ideias secundárias, ou
fundamentações, as argumentações, ou explicações, que levem
Interpretação de texto. ao esclarecimento das questões apresentadas na prova.

Normalmente, numa prova, o candidato é convidado a:


Apreensão e Compreensão do texto
Identificar - é reconhecer os elementos fundamentais
de uma argumentação, de um processo, de uma época (neste
Apreensão - conceito: o entendimento do texto fica
caso, procuram-se os verbos e os advérbios, os quais definem
restrito aos limites físicos. Para entendermos a informação não
o tempo).
recorremos às informações externas.
Comparar - é descobrir as relações de semelhança ou de
diferenças entre as situações do texto.
Compreensão - conceito: fazemos confronto de sentidos
Comentar - é relacionar o conteúdo apresentado com uma
com o texto lido e outros elementos produzidos anteriormente
realidade, opinando a respeito.
no texto. Para um bom entendidmento há o uso de elementos
Resumir - é concentrar as ideias centrais e/ou secundárias
adicionais.
em um só parágrafo.
Parafrasear - é reescrever o texto com outras palavras.
Apreensão + Compreensão = Entendimento do texto
Condições Básicas para Interpretar
Para ler e entender um texto é preciso atingir dois níveis de
leitura: informativa e de reconhecimento.
Faz-se necessário:
A primeira deve ser feita cuidadosamente por ser o primeiro
- Conhecimento Histórico – literário (escolas e gêneros
contato com o texto, extraindo-se informações e se preparando
literários, estrutura do texto), leitura e prática;
para a leitura interpretativa. Durante a interpretação grife
- Conhecimento gramatical, estilístico (qualidades do texto)
palavras-chave, passagens importantes; tente ligar uma palavra
e semântico; Na semântica (significado das palavras) incluem-
à ideia central de cada parágrafo.
se: homônimos e parônimos, denotação e conotação, sinonímia
A última fase de interpretação concentra-se nas perguntas
e antonímia, polissemia, figuras de linguagem, entre outros.
e opções de respostas. Marque palavras como não, exceto,
- Capacidade de observação e de síntese e
respectivamente, etc., pois fazem diferença na escolha adequada.
- Capacidade de raciocínio.
Retorne ao texto mesmo que pareça ser perda de tempo.
Leia a frase anterior e posterior para ter ideia do sentido global
Algumas dicas para interpretar um texto:
proposto pelo autor.
- O autor escreveu com uma intenção - tentar descobrir qual
Um texto para ser compreendido deve apresentar ideias
é, qual é a chave.
seletas e organizadas, através dos parágrafos que é composto
- Leia todo o texto uma primeira vez de forma despreocupada
pela ideia central, argumentação e/ou desenvolvimento e a
- assim você verá apenas os aspectos superficiais primeiro.
conclusão do texto.
- Na segunda leitura observe os detalhes, visualize em sua
A alusão histórica serve para dividir o texto em pontos
mente o cenário, os personagens - Quanto mais real for a leitura
menores, tendo em vista os diversos enfoques.
na sua mente, mais fácil será para interpretar o texto.
Uma das partes bem distintas do parágrafo é o tópico frasal,
- Duvide do(a) autor(a), leia as entrelinhas, perceba o que
ou seja, a ideia central extraída de maneira clara e resumida.
o(a) autor(a) te diz sem escrever no texto.
Atentando-se para a ideia principal de cada parágrafo,
- Não tenha medo de opinar - Já vi terem medo de dizer o
asseguramos um caminho que nos levará à compreensão do
que achavam e a resposta estaria correta se tivessem dito.
texto.
- Visualize vários caminhos, várias opções e interpretações,
O ato de escrever toma de empréstimo uma série de palavras
só não viaje muito na interpretação. Veja os caminhos apontados
e expressões amarrando, conectando uma palavra uma oração,
pela escrita do(a) autor(a). Apegue-se aos caminhos que lhe
uma ideia à outra. O texto precisa ser coeso e coerente.
são mostrados.
- Identifique as características físicas e psicológicas dos
Interpretação
personagens - Se um determinado personagem tem como
característica ser mentiroso, por exemplo, o que ele diz no
A interpretação é o alargamento dos horizontes. E esse
texto poderá ser mentira não é mesmo? Analisar e identificar os
alargamento acontece justamente quando há leitura.
personagens são pontos necessários para uma boa interpretação
Texto – é um conjunto de ideias organizadas e relacionadas
de texto.
entre si, formando um todo significativo capaz de produzir
- Observe a linguagem, o tempo e espaço, a sequência dos
interação comunicativa (capacidade de codificar e decodificar).
acontecimentos, o feedback, conta muito na hora de interpretar.
- Analise os acontecimentos de acordo com a época do
Contexto – um texto é constituído por diversas frases. Em
texto - É importante que você saiba ou pesquise sobre a época
cada uma delas, há uma certa informação que a faz ligar-se

1
LÍNGUA PORTUGUESA
narrada no texto, assim, certas contradições ou estranhamentos Do ponto de vista da localização institucional, há grande
vistos por você podem ser apenas a cultura da época sendo diversidade de situações no que se refere aos Ministérios
demonstrada. Públicos dos demais países da América Latina. Encontra-se, por
- Leia quantas vezes achar que deve - Não entendeu? Leia exemplo, Ministério Público dependente do Poder Judiciário
de novo. Nem todo dia estamos concentrados e a rapidez na na Costa Rica, na Colômbia e, no Paraguai, e ligado ao Poder
leitura vem com o hábito. Executivo, no México e no Uruguai.
Constata-se, entretanto, que, apesar da maior extensão de
Para ler e entender um texto é preciso atingir dois níveis de obrigações do Ministério Público brasileiro, a relação entre o
leitura: Informativa e de reconhecimento; número de integrantes da instituição e a população é uma das
mais desfavoráveis no quadro latino-americano. De fato, dados
Seguem algumas dicas para você analisar, compreender e recentes indicam que, no Brasil, com 4,2 promotores para cada
interpretar com mais proficiência. 100 mil habitantes, há uma situação de clara desvantagem
- Crie o hábito da leitura e o gosto por ela. Quando nós no que diz respeito ao número relativo de integrantes. No
passamos a gostar de algo, compreendemos melhor seu Panamá, por exemplo, o número é de 15,3 promotores para
funcionamento. Nesse caso, as palavras tornam-se familiares a cada cem mil habitantes; na Guatemala, de 6,9; no Paraguai,
nós mesmos. Não se deixe levar pela falsa impressão de que ler de 5,9; na Bolívia, de 4,5. Em situação semelhante ou ainda
não faz diferença. Também não se intimide caso alguém diga mais crítica do que o Brasil, estão, (l.11) por exemplo, o Peru,
que você lê porcaria. Leia tudo que tenha vontade, pois com com 3,0; a Argentina, com 2,9; e, por fim, o Equador, com a
o tempo você se tornará mais seleto e perceberá que algumas mais baixa relação: 2,4. É correto dizer que há nações (l.12)
leituras foram superficiais e, às vezes, até ridículas. Porém elas proporcionalmente com menos promotores que o Brasil. No
foram o ponto de partida e o estímulo para se chegar a uma entanto, as atribuições do Ministério Público brasileiro são
leitura mais refinada. Existe tempo para cada tempo de nossas muito mais (l.13) extensas do que as dos Ministérios Públicos
vidas. desses países.
- Seja curioso, investigue as palavras que circulam em seu
meio. Maria Tereza Sadek. A construção de um novo Ministério
- Aumente seu vocabulário e sua cultura. Além da leitura, um Públicoresolutivo. Internet: <https://aplicacao.mp.mg.gov.br>
bom exercício para ampliar o léxico é fazer palavras cruzadas. (l.11) – linha 11 no texto original
- Faça exercícios de sinônimos e antônimos. (l.12) – linha 12 no texto original
- Leia verdadeiramente. (l.13) – linha 13 no texto original
- Leia algumas vezes o texto, pois a primeira impressão pode
ser falsa. É preciso paciência para ler outras vezes. Antes de Julgue os itens seguintes com (C) quando a afirmativa
responder as questões, retorne ao texto para sanar as dúvidas. estiver Correta e com (E) quando a afirmativa estiver Errada.
- Atenção ao que se pede. Às vezes a interpretação está Itens relativos às ideias e a aspectos linguísticos do texto acima.
voltada a uma linha do texto e por isso você deve voltar ao
parágrafo para localizar o que se afirma. Outras vezes, a 01. Os dados expostos no terceiro parágrafo indicam que os
questão está voltada à ideia geral do texto. profissionais do Ministério Público brasileiro são mais eficientes
- Fique atento a leituras de texto de todas as áreas do que os dos órgãos equivalentes nos demais países da América
conhecimento, porque algumas perguntas extrapolam ao que do Sul.
está escrito.
02. Com base nos dados apresentados no texto, é correto
Tome cuidado com as vírgulas. Veja por exemplo a diferença concluir que a situação do Brasil, no que diz respeito ao número
de sentido nas frases a seguir. de promotores existentes no Ministério Público por habitante,
- Só, o Diego da M110 fez o trabalho de artes. está pior que a da Guatemala, mas melhor que a do Peru.
- Só o Diego da M110 fez o trabalho de artes.
- Os alunos dedicados passaram no vestibular. 03. Seriam mantidas a coerência e a correção gramatical
- Os alunos, dedicados, passaram no vestibular. do texto se, feitos os devidos ajustes nas iniciais maiúsculas e
- Marcão, canta Garçom, de Reginaldo Rossi. minúsculas, o período “É correto (...) o Brasil” (l.11-12) fosse
- Marcão canta Garçom, de Reginaldo Rossi. iniciado com um vocábulo de valor conclusivo, como logo, por
conseguinte, assim ou porquanto, seguido de vírgula.
Explicações:
- Diego fez sozinho o trabalho de artes. Respostas
- Apenas o Diego fez o trabalho de artes. 01. E (Afirmativa Errada) / 02. C (Afirmativa
- Havia, nesse caso, alunos dedicados e não dedicados e, Correta) / 03. E (Afirmativa Errada)
passaram no vestibular, somente, os que se dedicaram, restringindo
o grupo de alunos.
- Nesse outro caso, todos os alunos eram dedicados.
- Marcão é chamado para cantar.
- Marcão pratica a ação de cantar.
Argumentação.
Questões

Se considerarmos o panorama internacional, perceberemos Argumentação


que o Ministério Público brasileiro é singular. Em nenhum
outro país, há um Ministério Público que apresente perfil O ato de comunicação não visa apenas transmitir uma
institucional semelhante ao nosso ou que ostente igual conjunto informação a alguém. Quem comunica pretende criar uma
de atribuições. imagem positiva de si mesmo (por exemplo, a de um sujeito

2
LÍNGUA PORTUGUESA
educado, ou inteligente, ou culto), quer ser aceito, deseja que que o objetivo do texto seja comprovar alguma delas. Parte da
o que diz seja admitido como verdadeiro. Em síntese, tem a ideia de que o consenso, mesmo que equivocado, corresponde
intenção de convencer, ou seja, tem o desejo de que o ouvinte ao indiscutível, ao verdadeiro e, portanto, é melhor do que
creia no que o texto diz e faça o que ele propõe. aquilo que não desfruta dele. Em nossa época, são consensuais,
Se essa é a finalidade última de todo ato de comunicação, por exemplo, as afirmações de que o meio ambiente precisa ser
todo texto contém um componente argumentativo. A protegido e de que as condições de vida são piores nos países
argumentação é o conjunto de recursos de natureza linguística subdesenvolvidos. Ao confiar no consenso, porém, correse o
destinados a persuadir a pessoa a quem a comunicação se risco de passar dos argumentos válidos para os lugarescomuns,
destina. Está presente em todo tipo de texto e visa a promover os preconceitos e as frases carentes de qualquer base científica.
adesão às teses e aos pontos de vista defendidos.
Para compreender claramente o que é um argumento, é Argumento de Existência
bom voltar ao que diz Aristóteles, filósofo grego do século lV É aquele que se fundamenta no fato de que é mais fácil
a.C., numa obra intitulada “Tópicos: os argumentos são úteis aceitar aquilo que comprovadamente existe do que aquilo que
quando se tem de escolher entre duas ou mais coisas”. é apenas provável, que é apenas possível. A sabedoria popular
Se tivermos de escolher entre uma coisa vantajosa e uma enuncia o argumento de existência no provérbio “Mais vale um
desvantajosa, como a saúde e a doença, não precisamos pássaro na mão do que dois voando”.
argumentar. Suponhamos, no entanto, que tenhamos de
escolher entre duas coisas igualmente vantajosas, a riqueza e Argumento quase lógico
a saúde. Nesse caso, precisamos argumentar sobre qual das É aquele que opera com base nas relações lógicas, como
duas é mais desejável. O argumento pode então ser definido causa e efeito, analogia, implicação, identidade, etc. Esses
como qualquer recurso que torna uma coisa mais desejável que raciocínios são chamados quase lógicos porque, diversamente
outra. Isso significa que ele atua no domínio do preferível. Ele dos raciocínios lógicos, eles não pretendem estabelecer relações
é utilizado para fazer o interlocutor crer que, entre duas teses, necessárias entre os elementos, mas sim instituir relações
uma é mais provável que a outra, mais possível que a outra, prováveis, possíveis, plausíveis. Por exemplo, quando se diz “A
mais desejável que a outra, é preferível à outra. é igual a B”, “B é igual a C”, “então A é igual a C”, estabelece-
O objetivo da argumentação não é demonstrar a verdade de se uma relação de identidade lógica. Entretanto, quando se
um fato, mas levar o ouvinte a admitir como verdadeiro o que afirma “Amigo de amigo meu é meu amigo” não se institui uma
o enunciador está propondo. identidade lógica, mas uma identidade provável.
Há uma diferença entre o raciocínio lógico e a argumentação.
O primeiro opera no domínio do necessário, ou seja, pretende Argumento do Atributo
demonstrar que uma conclusão deriva necessariamente das
premissas propostas, que se deduz obrigatoriamente dos É aquele que considera melhor o que tem proriedades
postulados admitidos. No raciocínio lógico, as conclusões não típicas daquilo que é mais valorizado socialmente, por exemplo,
dependem de crenças, de uma maneira de ver o mundo, mas o mais raro é melhor que o comum, o que é mais refinado é
apenas do encadeamento de premissas e conclusões. melhor que o que é mais grosseiro, etc.
Já vimos diversas características dos argumentos. É preciso Por esse motivo, a publicidade usa, com muita frequência,
acrescentar mais uma: o convencimento do interlocutor, o celebridades recomendando prédios residenciais, produtos
auditório, que pode ser individual ou coletivo, será tanto mais de beleza, alimentos estéticos, etc., com base no fato de que
fácil quanto mais os argumentos estiverem de acordo com suas o consumidor tende a associar o produto anunciado com
crenças, suas expectativas, seus valores. Não se pode convencer atributos da celebridade.
um auditório pertencente a uma dada cultura enfatizando coisas Uma variante do argumento de atributo é o argumento da
que ele abomina. Será mais fácil convencêlo valorizando coisas competência linguística. A utilização da variante culta e formal
que ele considera positivas. No Brasil, a publicidade da cerveja da língua que o produtor do texto conhece a norma linguística
vem com frequência associada ao futebol, ao gol, à paixão socialmente mais valorizada e, por conseguinte, deve produzir
nacional. Nos Estados Unidos, essa associação certamente não um texto em que se pode confiar. Nesse sentido é que se diz que
surtiria efeito, porque lá o futebol não é valorizado da mesma o modo de dizer dá confiabilidade ao que se diz.
forma que no Brasil. O poder persuasivo de um argumento está A argumentação é a exploração de recursos para fazer
vinculado ao que é valorizado ou desvalorizado numa dada parecer verdadeiro aquilo que se diz num texto e, com isso,
cultura. levar a pessoa a que texto é endereçado a crer naquilo que ele
diz.
Tipos de Argumento A persuasão pode ser válida e não válida. Na persuasão
válida, expõemse com clareza os fundamentos de uma ideia
Argumento de Autoridade ou proposição, e o interlocutor pode questionar cada passo
É a citação, no texto, de afirmações de pessoas reconhecidas do raciocínio empregado na argumentação. A persuasão não
pelo auditório como autoridades em certo domínio do saber, válida apoiase em argumentos subjetivos, apelos subliminares,
para servir de apoio àquilo que o enunciador está propondo. Esse chantagens sentimentais, com o emprego de “apelações”, como
recurso produz dois efeitos distintos: revela o conhecimento do a inflexão de voz, a mímica e até o choro.
produtor do texto a respeito do assunto de que está tratando; Alguns autores classificam a dissertação em duas
dá ao texto a garantia do autor citado. É preciso, no entanto, modalidades, expositiva e argumentativa. Esta exige
não fazer do texto um amontoado de citações. A citação precisa argumentação, razões a favor e contra uma ideia, ao passo
ser pertinente e verdadeira. que a outra é informativa, apresenta dados sem a intenção
de convencer. Na verdade, a escolha dos dados levantados,
Argumento de Quantidade a maneira de expôlos no texto já revelam uma “tomada de
É aquele que valoriza mais o que é apreciado pelo maior posição”, a adoção de um ponto de vista na dissertação, ainda
número de pessoas, o que existe em maior número, o que tem que sem a apresentação explícita de argumentos. Desse ponto
maior duração, o que tem maior número de adeptos, etc. O de vista, a dissertação pode ser definida como discussão, debate,
fundamento desse tipo de argumento é que mais = melhor. A questionamento, o que implica a liberdade de pensamento,
publicidade faz largo uso do argumento de quantidade. a possibilidade de discordar ou concordar parcialmente. A
liberdade de questionar é fundamental, mas não é suficiente
Argumento do Consenso para organizar um texto dissertativo. É necessária também a
É uma variante do argumento de quantidade. Fundamentase exposição dos fundamentos, os motivos, os porquês da defesa
em afirmações que, numa determinada época, são aceitas como de um ponto de vista.
verdadeiras e, portanto, dispensam comprovações, a menos

3
LÍNGUA PORTUGUESA
Para discutir um tema, para confrontar argumentos e causa, a falsa analogia são algumas causas do sofisma. O
posições, é necessária a capacidade de conhecer outros pontos sofisma pressupõe má fé, intenção deliberada de enganar
de vista e seus respectivos argumentos. Uma discussão impõe, ou levar ao erro; quando o sofisma não tem essas intenções
muitas vezes, a análise de argumentos opostos, antagônicos. propositais, costuma-se chamar esse processo de argumentação
Como sempre, essa capacidade aprendese com a prática. Um de paralogismo.
bom exercício para aprender a argumentar e contraargumentar Exemplos de sofismas:
consiste em desenvolver as seguintes habilidades:
- argumentação: anotar todos os argumentos a favor Dedução
de uma ideia ou fato; imaginar um interlocutor que adote a Todo professor tem um diploma (geral, universal)
posição totalmente contrária; Fulano tem um diploma (particular)
Logo, fulano é professor (geral – conclusão falsa)
- contraargumentação: imaginar um diálogodebate e
quais os argumentos que essa pessoa imaginária possivelmente Indução
apresentaria contra a argumentação proposta; O Rio de Janeiro tem uma estátua do Cristo Redentor.
(particular)
- refutação: argumentos e razões contra a argumentação Taubaté (SP) tem uma estátua do Cristo Redentor.
oposta. (particular)
Rio de Janeiro e Taubaté são cidades.
A argumentação tem a finalidade de persuadir, portanto, Logo, toda cidade tem uma estátua do Cristo Redentor.
argumentar consiste em estabelecer relações para tirar (geral – conclusão falsa)
conclusões válidas, como se procede no método dialético. O Nota-se que as premissas são verdadeiras, mas a conclusão
método dialético não envolve apenas questões ideológicas, pode ser falsa. Nem todas as pessoas que têm diploma são
geradoras de polêmicas. Tratase de um método de investigação professores; nem todas as cidades têm uma estátua do
da realidade pelo estudo de sua ação recíproca, da contradição Cristo Redentor. Cometese erro quando se faz generalizações
inerente ao fenômeno em questão e da mudança dialética que apressadas ou infundadas. A “simples inspeção” é a ausência
ocorre na natureza e na sociedade. de análise ou análise superficial dos fatos, que leva a
Descartes (15961650), filósofo e pensador francês, criou pronunciamentos subjetivos, baseados nos sentimentos não
o método de raciocínio silogístico, baseado na dedução, que ditados pela razão.
parte do simples para o complexo. Para ele, verdade e evidência
são a mesma coisa, e pelo raciocínio tornase possível chegar a Análise e síntese são dois processos opostos, mas
conclusões verdadeiras, desde que o assunto seja pesquisado interligados; a análise parte do todo para as partes, a síntese,
em partes, começandose pelas proposições mais simples até das partes para o todo. A análise precede a síntese, porém,
alcançar, por meio de deduções, a conclusão final. Para a linha de certo modo, uma depende da outra. A análise decompõe
de raciocínio cartesiana, é fundamental determinar o problema, o todo em partes, enquanto a síntese recompõe o todo pela
dividilo em partes, ordenar os conceitos, simplificandoos, reunião das partes. Sabese, porém, que o todo não é uma
enumerar todos os seus elementos e determinar o lugar de cada simples justaposição das partes. Se alguém reunisse todas as
um no conjunto da dedução. peças de um relógio, não significa que reconstruiu o relógio,
A lógica cartesiana, até os nossos dias, é fundamental para pois fez apenas um amontoado de partes. Só reconstruiria todo
a argumentação dos trabalhos acadêmicos. Descartes propôs se as partes estivessem organizadas, devidamente combinadas,
quatro regras básicas que constituem um conjunto de reflexos seguida uma ordem de relações necessárias, funcionais, então,
vitais, uma série de movimentos sucessivos e contínuos do o relógio estaria reconstruído.
espírito em busca da verdade: Síntese, portanto, é o processo de reconstrução do todo por
- evidência; meio da integração das partes, reunidas e relacionadas num
- divisão ou análise; conjunto. Toda síntese, por ser uma reconstrução, pressupõe
- ordem ou dedução; a análise, que é a decomposição. A análise, no entanto, exige
- enumeração. uma decomposição organizada, é preciso saber como dividir o
todo em partes. As operações que se realizam na análise e na
A enumeração pode apresentar dois tipos de falhas: a síntese podem ser assim relacionadas:
omissão e a incompreensão. Qualquer erro na enumeração Análise: penetrar, decompor, separar, dividir.
pode quebrar o encadeamento das ideias, indispensável para o Síntese: integrar, recompor, juntar, reunir.
processo dedutivo.
A forma de argumentação mais empregada na redação A análise tem importância vital no processo de coleta de
acadêmica é o silogismo, raciocínio baseado nas regras ideias a respeito do tema proposto, de seu desdobramento e da
cartesianas, que contém três proposições: duas premissas, maior criação de abordagens possíveis. A síntese também é importante
e menor, e a conclusão. As três proposições são encadeadas de na escolha dos elementos que farão parte do texto.
tal forma, que a conclusão é deduzida da maior por intermédio A análise decompõe o todo em partes, a classificação
da menor. A premissa maior deve ser universal, emprega todo, estabelece as necessárias relações de dependência e hierarquia
nenhum, pois alguns não caracteriza a universalidade. entre as partes. Análise e classificação ligam-se intimamente,
Há dois métodos fundamentais de raciocínio: a dedução a ponto de se confundir uma com a outra, contudo são
(silogística), que parte do geral para o particular, e a indução, procedimentos diversos: análise é decomposição e classificação
que vai do particular para o geral. A expressão formal do é hierarquisação.
método dedutivo é o silogismo. A dedução é o caminho das Para a clareza da dissertação, é indispensável que, logo na
consequências, baseiase em uma conexão descendente (do geral introdução, os termos e conceitos sejam definidos, pois, para
para o particular) que leva à conclusão. Segundo esse método, expressar um questionamento, devese, de antemão, expor
partindo-se de teorias gerais, de verdades universais, pode-se clara e racionalmente as posições assumidas e os argumentos
chegar à previsão ou determinação de fenômenos particulares. que as justificam. É muito importante deixar claro o campo
O percurso do raciocínio vai da causa para o efeito. da discussão e a posição adotada, isto é, esclarecer não só o
Quanto a seus aspectos formais, o silogismo pode ser válido assunto, mas também os pontos de vista sobre ele.
e verdadeiro; a conclusão será verdadeira se as duas premissas A definição tem por objetivo a exatidão no emprego da
também o forem. Se há erro ou equívoco na apreciação dos linguagem e consiste na enumeração das qualidades próprias
fatos, pode-se partir de premissas verdadeiras para chegar a de uma ideia, palavra ou objeto. Definir é classificar o elemento
uma conclusão falsa. Tem-se, desse modo, o sofisma. Uma conforme a espécie a que pertence, demonstra: a característica
definição inexata, uma divisão incompleta, a ignorância da que o diferencia dos outros elementos dessa mesma espécie.

4
LÍNGUA PORTUGUESA
Entre os vários processos de exposição de ideias, a definição Apoio na consensualidade: Certas afirmações
é um dos mais importantes, sobretudo no âmbito das ciências. dispensam explicação ou comprovação, pois seu conteúdo é
A definição científica ou didática é denotativa, ou seja, atribui aceito como válido por consenso, pelo menos em determinado
às palavras seu sentido usual ou consensual, enquanto a espaço sociocultural. Nesse caso, incluem-se:
conotativa ou metafórica emprega palavras de sentido figurado. - A declaração que expressa uma verdade universal (o
Segundo a lógica tradicional aristotélica, a definição consta de homem, mortal, aspira à imortalidade);
três elementos: - A declaração que é evidente por si mesma (caso dos
- o termo a ser definido; postulados e axiomas);
- o gênero ou espécie; - Quando escapam ao domínio intelectual, ou seja, é de
- a diferença específica. natureza subjetiva ou sentimental (o amor tem razões que
a própria razão desconhece); implica apreciação de ordem
Procedimentos Argumentativos: Constituem os estética (gosto não se discute); diz respeito à fé religiosa, aos
procedimentos argumentativos mais empregados para dogmas (creio, ainda que parece absurdo).
comprovar uma afirmação: exemplificação, explicitação,
enumeração, comparação. Comprovação pela experiência ou observação: A
verdade de um fato ou afirmação pode ser comprovada por
Exemplificação: Procura justificar os pontos de vista por meio de dados concretos, estatísticos ou documentais.
meio de exemplos, hierarquizar afirmações. São expressões
comuns nesse tipo de procedimento: mais importante que, Comprovação pela fundamentação lógica: A
superior a, de maior relevância que. Empregamse também dados comprovação se realiza por meio de argumentos racionais,
estatísticos, acompanhados de expressões: considerando os baseados na lógica: causa/efeito; consequência/causa;
dados; conforme os dados apresentados. Fazse a exemplificação, condição/ocorrência.
ainda, pela apresentação de causas e consequências, usandose
comumente as expressões: porque, porquanto, pois que, uma Refutação pelo absurdo: refutase uma afirmação
vez que, visto que, por causa de, em virtude de, em vista de, por demonstrando o absurdo da consequência. Exemplo clássico é
motivo de. a contraargumentação do cordeiro, na conhecida fábula “O lobo
e o cordeiro”;
Explicitação: O objetivo desse recurso argumentativo é
explicar ou esclarecer os pontos de vista apresentados. Podese Refutação por exclusão: consiste em propor várias
alcançar esse objetivo pela definição, pelo testemunho e pela hipóteses para eliminá-las, apresentandose, então, aquela que
interpretação. Na explicitação por definição, empregamse se julga verdadeira;
expressões como: quer dizer, denominase, chamase, na verdade,
isto é, haja vista, ou melhor; nos testemunhos são comuns Desqualificação do argumento: atribuise o argumento
as expressões: conforme, segundo, na opinião de, no parecer à opinião pessoal subjetiva do enunciador, restringindose a
de, consoante as ideias de, no entender de, no pensamento de. universalidade da afirmação;
A explicitação se faz também pela interpretação, em que são
comuns as seguintes expressões: parece, assim, desse ponto de Ataque ao argumento pelo testemunho de
vista. autoridade: consiste em refutar um argumento empregando
os testemunhos de autoridade que contrariam a afirmação
Enumeração: Fazse pela apresentação de uma sequência apresentada;
de elementos que comprovam uma opinião, tais como a
enumeração de pormenores, de fatos, em uma sequência Desqualificar dados concretos apresentados:
de tempo, em que são frequentes as expressões: primeiro, consiste em desautorizar dados reais, demonstrando que o
segundo, por último, antes, depois, ainda, em seguida, então, enunciador baseouse em dados corretos, mas tirou conclusões
presentemente, antigamente, depois de, antes de, atualmente, hoje, falsas ou inconsequentes. Por exemplo, se na argumentação
no passado, sucessivamente, respectivamente. Na enumeração de afirmouse, por meio de dados estatísticos, que “o controle
fatos em uma sequência de espaço, empregamse as seguintes demográfico produz o desenvolvimento”, afirma-se que a
expressões: cá, lá, acolá, ali, aí, além, adiante, perto de, ao redor conclusão é inconsequente, pois se baseia em uma relação de
de, no Estado tal, na capital, no interior, nas grandes cidades, no causaefeito difícil de ser comprovada. Para contraargumentar,
sul, no leste... propõese uma relação inversa: “o desenvolvimento é que gera o
controle demográfico”.
Comparação: Analogia e contraste são as duas maneiras
de se estabelecer a comparação, com a finalidade de comprovar
uma ideia ou opinião. Na analogia, são comuns as expressões:
da mesma forma, tal como, tanto quanto, assim como, igualmente.
Para estabelecer contraste, empregamse as expressões: mais
que, menos que, melhor que, pior que.
Entre outros tipos de argumentos empregados para Pressupostos e subentendidos.
aumentar o poder de persuasão de um texto dissertativo
encontram-se:
Informações Explícitas e Implícitas
Argumento de autoridade: O saber notório de uma
autoridade reconhecida em certa área do conhecimento dá Texto:
apoio a uma afirmação. Dessa maneira, procura-se trazer para
o enunciado a credibilidade da autoridade citada. Lembre- “Neto ainda está longe de se igualar a qualquer um desses
se que as citações literais no corpo de um texto constituem craques (Rivelino, Ademir da Guia, Pedro Rocha e Pelé), mas
argumentos de autoridade. Ao fazer uma citação, o enunciador ainda tem um longo caminho a trilhar (...).”
situa os enunciados nela contidos na linha de raciocínio que Veja São Paulo, 26/12/1990, p. 15.
ele considera mais adequada para explicar ou justificar um
fato ou fenômeno. Esse tipo de argumento tem mais caráter Esse texto diz explicitamente que:
confirmatório que comprobatório. - Rivelino, Ademir da Guia, Pedro Rocha e Pelé são craques;
- Neto não tem o mesmo nível desses craques;
- Neto tem muito tempo de carreira pela frente.

5
LÍNGUA PORTUGUESA
O texto deixa implícito que: da continuidade do diálogo se alguém interviesse com uma
- Existe a possibilidade de Neto um dia aproximar-se dos pergunta deste tipo:
craques citados; “Mas quem disse que é certa a mudança do curso do rio?”
- Esses craques são referência de alto nível em sua
especialidade esportiva; A aceitação do pressuposto estabelecido pelo emissor
- Há uma oposição entre Neto e esses craques no que diz permite levar adiante o debate; sua negação compromete o
respeito ao tempo disponível para evoluir. diálogo, uma vez que destrói a base sobre a qual se constrói a
argumentação, e daí nenhum argumento tem mais importância
Todos os textos transmitem explicitamente certas ou razão de ser. Com pressupostos distintos, o diálogo não é
informações, enquanto deixam outras implícitas. Por exemplo, possível ou não tem sentido.
o texto acima não explicita que existe a possibilidade de A mesma pergunta, feita para pessoas diferentes, pode
Neto se equiparar aos quatro futebolistas, mas a inclusão do ser embaraçosa ou não, dependendo do que está pressuposto
advérbio ainda estabelece esse implícito. Não diz também com em cada situação. Para alguém que não faz segredo sobre a
explicitude que há oposição entre Neto e os outros jogadores, mudança de emprego, não causa o menor embaraço uma
sob o ponto de vista de contar com tempo para evoluir. A pergunta como esta:
escolha do conector “mas” entre a segunda e a primeira oração “Como vai você no seu novo emprego?”
só é possível levando em conta esse dado implícito. Como se vê,
há mais significados num texto do que aqueles que aparecem O efeito da mesma pergunta seria catastrófico se ela se
explícitos na sua superfície. Leitura proficiente é aquela capaz dirigisse a uma pessoa que conseguiu um segundo emprego
de depreender tanto um tipo de significado quanto o outro, o e quer manter sigilo até decidir se abandona o anterior. O
que, em outras palavras, significa ler nas entrelinhas. Sem essa adjetivo novo estabelece o pressuposto de que o interrogado
habilidade, o leitor passará por cima de significados importantes tem um emprego diferente do anterior.
ou, o que é bem pior, concordará com ideias e pontos de vista
que rejeitaria se os percebesse. Marcadores de Pressupostos
Os significados implícitos costumam ser classificados em
duas categorias: os pressupostos e os subentendidos. - Adjetivos ou palavras similares modificadoras do
substantivo
Pressupostos: são ideias implícitas que estão implicadas
logicamente no sentido de certas palavras ou expressões Julinha foi minha primeira filha.
explicitadas na superfície da frase. Exemplo: “Primeira” pressupõe que tenho outras filhas e que as outras
“André tornou-se um antitabagista convicto.” nasceram depois de Julinha.
Destruíram a outra igreja do povoado.
A informação explícita é que hoje André é um antitabagista “Outra” pressupõe a existência de pelo menos uma igreja
convicto. Do sentido do verbo tornarse, que significa “vir a ser”, além da usada como referência.
decorre logicamente que antes André não era antitabagista
convicto. Essa informação está pressuposta. Ninguém se torna - Certos verbos
algo que já era antes. Seria muito estranho dizer que a palmeira
tornou-se um vegetal. Renato continua doente.
“Eu ainda não conheço a Europa.” O verbo “continua” indica que Renato já estava doente no
momento anterior ao presente.
A informação explícita é que o enunciador não tem Nossos dicionários já aportuguesaram a palavrea copydesk.
conhecimento do continente europeu. O advérbio ainda deixa O verbo “aportuguesar” estabelece o pressuposto de que
pressuposta a possibilidade de ele um dia conhecê-la. copidesque não existia em português.
As informações explícitas podem ser questionadas pelo
receptor, que pode ou não concordar com elas. Os pressupostos, - Certos advérbios
porém, devem ser verdadeiros ou, pelo menos, admitidos
como tais, porque esta é uma condição para garantir a A produção automobilística brasileira está totalmente nas
continuidade do diálogo e também para fornecer fundamento mãos das multinacionais.
às afirmações explícitas. Isso significa que, se o pressuposto é O advérbio totalmente pressupõe que não há no Brasil
falso, a informação explícita não tem cabimento. Assim, por indústria automobilística nacional.
exemplo, se Maria não falta nunca a aula nenhuma, não tem Você conferiu o resultado da loteria?
o menor sentido dizer “Até Maria compareceu à aula de hoje”. Hoje não.
Até estabelece o pressuposto da inclusão de um elemento A negação precedida de um advérbio de tempo de âmbito
inesperado. limitado estabelece o pressuposto de que apenas nesse intervalo
Na leitura, é muito importante detectar os pressupostos, pois (hoje) é que o interrogado não praticou o ato de conferir o
eles são um recurso argumentativo que visa a levar o receptor resultado da loteria.
a aceitar a orientação argumentativa do emissor. Ao introduzir
uma ideia sob a forma de pressuposto, o enunciador pretende - Orações adjetivas
transformar seu interlocutor em cúmplice, pois a ideia implícita Os brasileiros, que não se importam com a coletividade, só
não é posta em discussão, e todos os argumentos explícitos se preocupam com seu bemestar e, por isso, jogam lixo na rua,
só contribuem para confirmá-la. O pressusposto aprisiona o fecham os cruzamentos, etc.
receptor no sistema de pensamento montado pelo enunciador. O pressuposto é que “todos” os brasileiros não se importam
A demonstração disso pode ser feita com as “verdades com a coletividade.
incontestáveis” que estão na base de muitos discursos políticos, Os brasileiros que não se importam com a coletividade só
como o que segue: se preocupam com seu bemestar e, por isso, jogam lixo na rua,
“Quando o curso do rio São Francisco for mudado, será fecham os cruzamentos, etc.
resolvido o problema da seca no Nordeste.” Nesse caso, o pressuposto é outro: “alguns” brasileiros não
se importam com a coletividade.
O enunciador estabelece o pressuposto de que é certa a No primeiro caso, a oração é explicativa; no segundo, é
mudança do curso do São Francisco e, por consequência, a restritiva. As explicativas pressupõem que o que elas expressam
solução do problema da seca no Nordeste. O diálogo não teria se refere à totalidade dos elementos de um conjunto; as
continuidade se um interlocutor não admitisse ou colocasse restritivas, que o que elas dizem concerne apenas a parte dos
sob suspeita essa certeza. Em outros termos, haveria quebra elementos de um conjunto. O produtor do texto escreverá uma

6
LÍNGUA PORTUGUESA
restritiva ou uma explicativa segundo o pressuposto que quiser distinguirem-se várias espécies ou tipos: visual, auditiva, tátil,
comunicar. etc., ou, ainda, outras mais complexas, constituídas, ao mesmo
tempo, de elementos diversos. Os elementos constitutivos da
Subentendidos: são insinuações contidas em uma frase linguagem são, pois, gestos, sinais, sons, símbolos ou palavras,
ou um grupo de frases. Suponhamos que uma pessoa estivesse usados para representar conceitos de comunicação, ideias,
em visita à casa de outra num dia de frio glacial e que uma significados e pensamentos. Embora os animais também se
janela, por onde entravam rajadas de vento, estivesse aberta. Se comuniquem, a linguagem verbal pertence apenas ao Homem.
o visitante dissesse “Que frio terrível”, poderia estar insinuando Não se devem confundir os conceitos de linguagem e de
que a janela deveria ser fechada. língua. Enquanto aquela (linguagem) diz respeito à capacidade
Há uma diferença capital entre o pressuposto e o ou faculdade de exercitar a comunicação, latente ou em ação
subentendido. O primeiro é uma informação estabelecida ou exercício, esta última (língua ou idioma) refere-se a um
como indiscutível tanto para o emissor quanto para o receptor, conjunto de palavras e expressões usadas por um povo, por
uma vez que decorre necessariamente do sentido de algum uma nação, munido de regras próprias (sua gramática).
elemento linguístico colocado na frase. Ele pode ser negado, O estudo da linguagem, que envolve os signos, de uma
mas o emissor colocao implicitamente para que não o seja. Já forma geral, é chamado semiótica. A linguística é subordinada à
o subentendido é de responsabilidade do receptor. O emissor semiótica porque seu objeto de estudo é a língua, que é apenas
pode esconder-se atrás do sentido literal das palavras e negar um dos sinais estudados na semiótica.
que tenha dito o que o receptor depreendeu de suas palavras.
Assim, no exemplo dado acima, se o dono da casa disser que é Para que serve a linguagem?
muito pouco higiênico fechar todas as janelas, o visitante pode
dizer que também acha e que apenas constatou a intensidade (...)
do frio. Ai, palavras, ai, palavras,
O subentendido serve, muitas vezes, para o emissor que estranha potência, a vossa!
protegerse, para transmitir a informação que deseja dar a Todo o sentido da vida
conhecer sem se comprometer. Imaginemos, por exemplo, que principia à vossa porta;
um funcionário recémpromovido numa empresa ouvisse de um o mel do amor cristaliza
colega o seguinte: seu perfume em vossa rosa;
“Competência e mérito continuam não valendo nada como sois o sonho e sois a audácia,
critério de promoção nesta empresa...” calúnia, fúria, derrota...

Esse comentário talvez suscitasse esta suspeita: A liberdade das almas,


“Você está querendo dizer que eu não merecia a promoção?” ai! Com letras se elabora...
E dos venenos humanos
Ora, o funcionário preterido, tendo recorrido a um sois a mais fina retorta:
subentendido, poderia responder: frágil, frágil como o vidro
“Absolutamente! Estou falando em termos gerais.” e mais que o aço poderosa!
Reis, impérios, povos, tempos,
pelo vosso impulso rodam...
(...)

Cecília Meireles.
Romanceiro da Inconfidência. In: Obra poética.
Níveis de linguagem. Rio de Janeiro, Nova Aguilar, 1985, p. 442.

Esses versos foram extraídos do poema “Romance LIII ou


Linguagem das palavras aéreas”, em que Cecília Meireles fala sobre o poder
da palavra. Mostram que a palavra, apesar de frágil, por ser
Como instrução geral, podemos dizer que uma hipótese constituída de sons, é ao mesmo tempo extremamente forte,
interpretativa é aceitável sempre que o texto apresenta pista ou porque, com seu significado, derruba reis e impérios; serve
pistas que a confirmam e sustentam. O texto abaixo é bastante para construir a liberdade do ser humano e também para
apropriado. envenenar a sua vida; serve para sussurrar declarações de
amor, para exprimir os sonhos, para impulsionar os desejos
“Aquela senhora tem um piano. mais grandiosos, mas também para caluniar, para expor a raiva,
Que é agradável, mas não é o correr dos rios. para impor a derrota.
Nem o murmúrio que as árvores fazem...
Por que é preciso ter um piano? - A linguagem é o traço definidor do ser humano, é a aptidão
O melhor é ter ouvidos que o distingue dos animais.
E amar a Natureza.”
O provérbio popular “Palavra não quebra osso”, contrapondo
Que simboliza o piano no poema? a palavra à ação, insinua que a linguagem não tem nenhum
Dentro do contexto que se insere o piano, representa um poder: um golpe, mas não uma palavra, é capaz de quebrar
bem cultural, o que se percebe pela oposição que o texto osso. Ora podemos desfazer facilmente essa visão simplista das
estabelece entre o som do piano (bem cultural) e o correr coisas, analisando para que serve a linguagem.
dos rios e o murmúrio das árvores (bens naturais). O poema
descarta a necessidade do piano, dando preferência à fruição - A linguagem é uma maneira de perceber o mundo.
dos sons da Natureza.
“Este deve ser o bosque”, murmurou pensativamente (Alice),
O que é a linguagem? “onde as coisas não têm nomes”. (...)
Ia devaneando dessa maneira quando chegou à entrada do
É qualquer e todo sistema de signos que serve de meio bosque, que parecia muito úmido e sombrio. “Bom, de qualquer
de comunicação de ideias ou sentimentos através de signos modo é um alívio”, disse enquanto avançava em meio às árvores,
convencionados, sonoros, gráficos, gestuais etc., podendo ser “depois de tanto calor, entrar dentro do... dentro do... dentro
percebida pelos diversos órgãos dos sentidos, o que leva a do quê?” Estava assombrada de não poder se lembrar do nome.

7
LÍNGUA PORTUGUESA
“Bom, isto é, estar debaixo das... debaixo das... debaixo disso substituir as palavras, porque a língua é bem mais que um
aqui, ora!”, disse, colocando a mão no tronco da árvore. “Como é sistema de demonstração de objetos ou mera cópia do mundo
que essa coisa se chama? É bem capaz de não ter nome nenhum... natural. As coisas não designam tudo que uma língua pode
ora, com certeza não tem mesmo!” expressar.
Ficou calada durante um minuto, pensando. Então, de repente, Mostrar um objeto, por exemplo, não indica sua inclusão
exclamou: - Ah, então isso terminou acontecendo! E agora quem numa dada classe. No léxico de uma língua, agrupamos os
sou eu? Eu quero me lembrar, se puder. nomes em classes. Maçã, pera, banana e laranja pertencem à
Lewis Carroll. Aventuras de Alice. classe das frutas. Ao mostrar uma fruta qualquer, não consigo
Trad. Sebastião Uchôa Leite. exprimir a ideia da classe fruta; não posso, então, expressar
3ª ed. São Paulo, Summus, p 165-166 ideias mais gerais. Não produzimos palavras somente para
designar as coisas, mas para estabelecer relações entre elas e
Esse texto, reproduzido do livro “Através do espelho e o que para comentá-las. Mostrar um objeto não exprime as categorias
Alice encontrou lá”, mostra que a protagonista, ao entrar no de quantidade, de gênero (masculino e feminino), de número
bosque em que as coisas não têm nome, é incapaz de apreender (singular e plural); não permite indicar sua localização no espaço
a realidade em torno dela, de saber o que as coisas são. Isso (aqui/aí/lá), etc. A língua não é um sistema de demonstração
significa que as coisas do mundo exterior só têm existência para de objetos, pois permite falar do que está presente e do que está
os homens quando são nomeadas. A linguagem é uma forma ausente, do que existe e do que não existe; permite até criar
de apreender a realidade: só percebemos aquilo a que a língua novas realidades, mundos não existentes.
dá nome. A linguagem é uma atividade simbólica, o que significa
Roberto Pompeu de Toledo, articulista da Veja, comenta que as palavras criam conceitos, e eles ordenam a realidade,
essa questão na edição de 26 de junho de 2002 (p. 130), ao categorizam o mundo. Por exemplo, criamos o conceito de pôr-
falar da expressão “risco país”, usada para traduzir o grau do-sol. Sabemos que, do ponto de vista científico, o Sol não “se
de confiabilidade de um país entre credores ou investidores põe”, uma vez que é a Terra que gira em torno dele. Contudo
internacionais: esse conceito, criado pela linguagem, determina uma realidade
que encanta a todos. Outro exemplo: apagar uma coisa escrita
(...) As coisas não são coisas enquanto não são nomeadas. O no computador é uma atividade diferente de apagar o que foi
que não se expressa não se conhece. Vive na inocência do limbo, escrito a lápis, a caneta ou mesmo a máquina. Por isso, surgiu
no sono profundo da inexistência. Uma vez identificado, batizado uma nova palavra para denominar essa nova realidade, deletar.
e devidamente etiquetado, o “risco país” passou a existir. E lá é No entanto, se essa palavra não existisse, não perceberíamos a
possível viver num país em risco? Lá é possível dormir em paz num atividade de apagar no computador como uma ação diferente
país submetido à medição do perigo que oferece com a mesma de apagar o que foi escrito a lápis. Uma nova realidade, uma
assiduidade com que a um paciente se tira a pressão? É como nova invenção, uma nova ideia exigem novas palavras, e estas
viajar num navio onde se apregoasse, num escandaloso placar é que lhes conferem existência para toda a comunidade de
luminoso, sujeito a tantas oscilações como as das ondas do mar, falantes.
o “risco naufrágio”. As palavras formam um sistema independente das coisas
nomeadas por elas, tanto é que cada língua pode ordenar o
- A linguagem é uma forma de interpretar a realidade. mundo de maneira diversa, exprimir diferentes modos de
ver a realidade. O inglês, por exemplo, para expressar o que
O segundo projeto era representado por um plano de abolir denominamos carneiro, tem duas palavras: sheep, que designa
completamente todas as palavras, fossem elas quais fossem o animal, e mutton, que significa a carne do carneiro preparada
(...). Em vista disso, propôs-se que, sendo as palavras apenas e servida à mesa. Em português, dizemos as duas coisas numa
nomes para as coisas, seria mais conveniente que todos os palavra só: Este carneiro tem muita lã e Este carneiro está
homens trouxessem consigo as coisas de que precisassem falar apimentado, ou seja, não aplicamos a distinção que os falantes
ao discorrer sobre determinado assunto (...). ...muitos eruditos da língua inglesa têm incorporada à sua visão de mundo.
e sábios aderiram ao novo plano de se expressarem por meio de Isso mostra que a linguagem é uma maneira de interpretar
coisas, cujo único inconveniente residia em que, se um homem o universo natural e segmentá-lo em categorias, segundo as
tivesse que falar sobre longos assuntos e de vária espécie, ver-se-ia particularidades de cada cultura. Por essa razão, a linguagem
obrigado, em proporção, a carregar nas costas um grande fardo modela nossa maneira de perceber e de ordenar a realidade.
de coisas, a menos de poder pagar um ou dois criados robustos A linguagem expressa também as diferentes maneiras de
para acompanhá-lo (...). interpretar uma ocorrência. Querendo desculpar-se, o filho diz
Outra grande vantagem oferecida pela invenção consiste em para a mãe: O jarro de porcelana caiu e quebrou. A mãe replica:
que ela serviria de língua universal, compreendida em todas as Você derrubou o jarro e, por isso, ele quebrou. Observe que, na
nações civilizadas, cujos utensílios e objetos são geralmente da primeira formulação, não existe um responsável pela queda
mesma espécie, ou tão parecidos que o seu emprego pode ser e pela quebra do objeto. É como se isso se devesse ao acaso.
facilmente percebido. Na segunda formulação, atribui-se a responsabilidade pelo
acontecimento a um agente.
Jonathan Swift. Viagens de Gulliver.
Rio de Janeiro/São Paulo, Ediouro/Publifolha, p. 194-195. - A linguagem é uma forma de ação.

Esse trecho do livro “Viagens de Gulliver” narra um projeto Existem certas fórmulas linguísticas que servem para agir
dos sábios de Balnibarbi: substituir as palavras – que, no seu no mundo. Quando um padre diz aos noivos “Eu vos declaro
entender, têm o inconveniente de variar de língua para língua marido e mulher”, quando alguém diz “Prometo estar aqui
– pelas coisas. Quando alguém quisesse falar de uma cadeira, amanhã”, quando um leiloeiro proclama “Arrematado por mil
mostraria uma cadeira, quem desejasse discorrer sobre uma reais”, quando o presidente de alguma câmara municipal afirma
bolsa, mostraria uma bolsa, etc. Trata-se de uma ironia de Swift “Declaro aberta a sessão”, eles não estão constatando alguma
às concepções vulgares de que a compreensão da realidade coisa do mundo, mas realizando uma ação. O ato de abrir uma
independe da língua que a nomeia, como se as palavras fossem sessão realiza-se quando seu presidente a declara aberta; o
etiquetas aplicadas a coisas classificadas independentemente ato da promessa realiza-se quando se diz “Prometo”. Em casos
da linguagem, quando, na verdade, a língua é uma forma de como esses, o dizer se confunde com a própria ação e serve para
categorizar o mundo, de interpretá-lo. demonstrar que a linguagem não é algo sem consequência,
O que inviabiliza o sistema imaginado pelos sábios de porque ela também é ação.
Balnibarbi não é apenas o excesso de peso das coisas que cada
falante precisaria carregar: é o fato de que as coisas não podem

8
LÍNGUA PORTUGUESA
Funções da Linguagem Emprega-se a expressão função conativa da linguagem
quando esta é usada para interferir no comportamento das
Quando se pergunta a alguém para que serve a linguagem, pessoas por meio de uma ordem, um pedido ou uma sugestão.
a resposta mais comum é que ela serve para comunicar. Isso A palavra conativo é proveniente de um verbo latino (conari)
está correto. No entanto, comunicar não é apenas transmitir que significa “esforçar-se” (para obter algo).
informações. É também exprimir emoções, dar ordens, falar
apenas para não haver silêncio. Para que serve a linguagem? - A linguagem serve para expressar a subjetividade:
Função Emotiva.
- A linguagem serve para informar: Função
Referencial. “Eu fico possesso com isso!”

“Estados Unidos invadem o Iraque” Nessa frase, quem fala está exprimindo sua indignação
com alguma coisa que aconteceu. Com palavras, objetivamos
Essa frase, numa manchete de jornal, informa-nos sobre um e expressamos nossos sentimentos e nossas emoções.
acontecimento do mundo. Exprimimos a revolta e a alegria, sussurramos palavras de
Com a linguagem, armazenamos conhecimentos na amor e explodimos de raiva, manifestamos desespero, desdém,
memória, transmitimos esses conhecimentos a outras pessoas, desprezo, admiração, dor, tristeza. Muitas vezes, falamos para
ficamos sabendo de experiências bem-sucedidas, somos exprimir poder ou para afirmarmo-nos socialmente. Durante o
prevenidos contra as tentativas mal sucedidas de fazer alguma governo do presidente Fernando Henrique Cardoso, ouvíamos
coisa. Graças à linguagem, um ser humano recebe de outro certos políticos dizerem “A intenção do Fernando é levar o país à
conhecimentos, aperfeiçoa-os e transmite-os. prosperidade” ou “O Fernando tem mudado o país”. Essa maneira
Condillac, um pensador francês, diz: “Quereis aprender informal de se referirem ao presidente era, na verdade, uma
ciências com facilidade? Começai a aprender vossa própria maneira de insinuarem intimidade com ele e, portanto,
língua!” Com efeito, a linguagem é a maneira como aprendemos de exprimirem a importância que lhes seria atribuída pela
desde as mais banais informações do dia a dia até as teorias proximidade com o poder. Inúmeras vezes, contamos coisas
científicas, as expressões artísticas e os sistemas filosóficos mais que fizemos para afirmarmo-nos perante o grupo, para mostrar
avançados. nossa valentia ou nossa erudição, nossa capacidade intelectual
A função informativa da linguagem tem importância ou nossa competência na conquista amorosa.
central na vida das pessoas, consideradas individualmente Por meio do tipo de linguagem que usamos, do tom de voz
ou como grupo social. Para cada indivíduo, ela permite que empregamos, etc., transmitimos uma imagem nossa, não
conhecer o mundo; para o grupo social, possibilita o acúmulo raro inconscientemente.
de conhecimentos e a transferência de experiências. Por meio Emprega-se a expressão função emotiva para designar a
dessa função, a linguagem modela o intelecto. utilização da linguagem para a manifestação do enunciador,
É a função informativa que permite a realização do trabalho isto é, daquele que fala.
coletivo. Operar bem essa função da linguagem possibilita que
cada indivíduo continue sempre a aprender. - A linguagem serve para criar e manter laços
A função informativa costuma ser chamada também de sociais: Função Fática.
função referencial, pois seu principal propósito é fazer com que
as palavras revelem da maneira mais clara possível as coisas ou __Que calorão, hein?
os eventos a que fazem referência. __Também, tem chovido tão pouco.
__Acho que este ano tem feito mais calor do que nos outros.
- A linguagem serve para influenciar e ser __Eu não me lembro de já ter sentido tanto calor.
influenciado: Função Conativa.
Esse é um típico diálogo de pessoas que se encontram num
“Vem pra Caixa você também.” elevador e devem manter uma conversa nos poucos instantes
em que estão juntas. Falam para nada dizer, apenas porque o
Essa frase fazia parte de uma campanha destinada a silêncio poderia ser constrangedor ou parecer hostil.
aumentar o número de correntistas da Caixa Econômica Federal. Quando estamos num grupo, numa festa, não podemos
Para persuadir o público alvo da propaganda a adotar esse manter-nos em silêncio, olhando uns para os outros. Nessas
comportamento, formulou-se um convite com uma linguagem ocasiões, a conversação é obrigatória. Por isso, quando não se
bastante coloquial, usando, por exemplo, a forma vem, de tem assunto, fala-se do tempo, repetem-se histórias que todos
segunda pessoa do imperativo, em lugar de venha, forma de conhecem, contam-se anedotas velhas. A linguagem, nesse caso,
terceira pessoa prescrita pela norma culta quando se usa você. não tem nenhuma função que não seja manter os laços sociais.
Pela linguagem, as pessoas são induzidas a fazer Quando encontramos alguém e lhe perguntamos “Tudo bem?”,
determinadas coisas, a crer em determinadas ideias, a sentir em geral não queremos, de fato, saber se nosso interlocutor
determinadas emoções, a ter determinados estados de alma está bem, se está doente, se está com problemas. A fórmula é
(amor, desprezo, desdém, raiva, etc.). Por isso, pode-se dizer uma maneira de estabelecer um vínculo social.
que ela modela atitudes, convicções, sentimentos, emoções, Também os hinos têm a função de criar vínculos, seja entre
paixões. Quem ouve desavisada e reiteradamente a palavra alunos de uma escola, entre torcedores de um time de futebol
negro pronunciada em tom desdenhoso aprende a ter ou entre os habitantes de um país. Não importa que as pessoas
sentimentos racistas; se a todo momento nos dizem, num tom não entendam bem o significado da letra do Hino Nacional, pois
pejorativo, “Isso é coisa de mulher”, aprendemos os preconceitos ele não tem função informativa: o importante é que, ao cantá-
contra a mulher. lo, sentimo-nos participantes da comunidade de brasileiros.
Não se interfere no comportamento das pessoas apenas com Na nomenclatura da linguística, usa-se a expressão função
a ordem, o pedido, a súplica. Há textos que nos influenciam fática para indicar a utilização da linguagem para estabelecer
de maneira bastante sutil, com tentações e seduções, como ou manter aberta a comunicação entre um falante e seu
os anúncios publicitários que nos dizem como seremos bem interlocutor.
sucedidos, atraentes e charmosos se usarmos determinadas
marcas, se consumirmos certos produtos. - A linguagem serve para falar sobre a própria
Com essa função, a linguagem modela tanto bons cidadãos, linguagem: Função Metalinguística.
que colocam o respeito ao outro acima de tudo, quanto
espertalhões, que só pensam em levar vantagem, e indivíduos Quando dizemos frases como “A palavra ‘cão’ é um
atemorizados, que se deixam conduzir sem questionar. substantivo”; “É errado dizer ‘a gente viemos’”; “Estou usando

9
LÍNGUA PORTUGUESA
o termo ‘direção’ em dois sentidos”; “Não é muito elegante usar Observe-se que a maior concentração de sons oclusivos
palavrões”, não estamos falando de acontecimentos do mundo, ocorre no segundo verso, quando se afirma que o barulho dos
mas estamos tecendo comentários sobre a própria linguagem. É cavalos aumenta.
o que chama função metalinguística. A atividade metalinguística Quando se usam recursos da própria língua para acrescentar
é inseparável da fala. Falamos sobre o mundo exterior e o sentidos ao conteúdo transmitido por ela, diz-se que estamos
mundo interior e ao mesmo tempo, fazemos comentários sobre usando a linguagem em sua função poética.
a nossa fala e a dos outros. Quando afirmamos como diz o
outro, estamos comentando o que declaramos: é um modo de Para melhor compreensão das funções de linguagem, torna-
esclarecer que não temos o hábito de dizer uma coisa tão trivial se necessário o estudo dos elementos da comunicação.
como a que estamos enunciando; inversamente, podemos usar Antigamente, tinha-se a ideia que o diálogo era desenvolvido
a metalinguagem como recurso para valorizar nosso modo de de maneira “sistematizada” (alguém pergunta - alguém espera
dizer. É o que se dá quando dizemos, por exemplo, Parodiando ouvir a pergunta, daí responde, enquanto outro escuta em
o padre Vieira ou Para usar uma expressão clássica, vou dizer silêncio, etc). Exemplo:
que “peixes se pescam, homens é que se não podem pescar”.
- A linguagem serve para criar outros universos. Elementos da comunicação
- Emissor - emite, codifica a mensagem;
A linguagem não fala apenas daquilo que existe, fala também - Receptor - recebe, decodifica a mensagem;
do que nunca existiu. Com ela, imaginamos novos mundos, - Mensagem - conteúdo transmitido pelo emissor;
outras realidades. Essa é a grande função da arte: mostrar que - Código - conjunto de signos usado na transmissão e
outros modos de ser são possíveis, que outros universos podem recepção da mensagem;
existir. O filme de Woody Allen “A rosa púrpura do Cairo” - Referente - contexto relacionado a emissor e receptor;
(1985) mostra isso de maneira bem expressiva. Nele, conta-se - Canal - meio pelo qual circula a mensagem.
a história de uma mulher que, para consolar-se do cotidiano
sofrido e dos maus-tratos infligidos pelo marido, refugia-se no Porém, com os estudos recentes dos linguistas, essa
cinema, assistindo inúmeras vezes a um filme de amor em que teoria sofreu uma modificação, pois, chegou-se a conclusão
a vida é glamorosa, e o galã é carinhoso e romântico. Um dia, que quando se trata da parole, entende-se que é um veículo
ele sai da tela e ambos vão viver juntos uma série de aventuras. democrático (observe a função fática), assim, admite-se um
Nessa outra realidade, os homens são gentis, a vida não é novo formato de locução, ou, interlocução (diálogo interativo):
monótona, o amor nunca diminui e assim por diante. - locutor - quem fala (e responde);
- locutário - quem ouve e responde;
- A linguagem serve como fonte de prazer: Função - interlocução - diálogo
Poética.
As respostas, dos “interlocutores” podem ser gestuais,
Brincamos com as palavras. Os jogos com o sentido e os faciais etc. por isso a mudança (aprimoração) na teoria.
sons são formas de tornar a linguagem um lugar de prazer. As atitudes e reações dos comunicantes são também
Divertimo-nos com eles. Manipulamos as palavras para delas referentes e exercem influência sobre a comunicação
extrairmos satisfação. Lembramo-nos:
Oswald de Andrade, em seu “Manifesto antropófago”, diz
“Tupi or not tupi”; trata-se de um jogo com a frase shakespeariana - Emotiva (ou expressiva): a mensagem centra-se no “eu”
“To be or not to be”. Conta-se que o poeta Emílio de Menezes, do emissor, é carregada de subjetividade. Ligada a esta função
quando soube que uma mulher muito gorda se sentara no banco está, por norma, a poesia lírica.
de um ônibus e este quebrara, fez o seguinte trocadilho: “É a - Função apelativa (imperativa): com este tipo de mensagem,
primeira vez que vejo um banco quebrar por excesso de fundos”. o emissor atua sobre o receptor, afim de que este assuma
A palavra banco está usada em dois sentidos: “móvel comprido determinado comportamento; há frequente uso do vocativo
para sentar-se” e “casa bancária”. Também está empregado em e do imperativo. Esta função da linguagem é frequentemente
dois sentidos o termo fundos: “nádegas” e “capital”, “dinheiro”. usada por oradores e agentes de publicidade.
Observe-se o uso do verbo bater, em expressões diversas, - Função metalinguística: função usada quando a língua
com significados diferentes, nesta frase do deputado Virgílio explica a própria linguagem (exemplo: quando, na análise de
Guimarães: um texto, investigamos os seus aspectos morfo-sintáticos e/ou
semânticos).
“ACM bate boca porque está acostumado a bater: bateu - Função informativa (ou referencial): função usada quando
continência para os militares, bateu palmas para o Collor e quer o emissor informa objetivamente o receptor de uma realidade,
bater chapa em 2002. Mas o que falta é que lhe bata uma dor de ou acontecimento.
consciência e bata em retirada.” - Função fática: pretende conseguir e manter a atenção
(Folha de S. Paulo) dos interlocutores, muito usada em discursos políticos e textos
publicitários (centra-se no canal de comunicação).
Verifica-se que a linguagem pode ser usada utilitariamente - Função poética: embeleza, enriquecendo a mensagem com
ou esteticamente. No primeiro caso, ela é utilizada para figuras de estilo, palavras belas, expressivas, ritmos agradáveis,
informar, para influenciar, para manter os laços sociais, etc. No etc.
segundo, para produzir um efeito prazeroso de descoberta de Também podemos pensar que as primeiras falas conscientes
sentidos. Em função estética, o mais importante é como se diz, da raça humana ocorreu quando os sons emitidos evoluiram
pois o sentido também é criado pelo ritmo, pelo arranjo dos para o que podemos reconhecer como “interjeições”. As
sons, pela disposição das palavras, etc. primeiras ferramentas da fala humana.
Na estrofe abaixo, retirada do poema “A Cavalgada”, de
Raimundo Correia, a sucessão dos sons oclusivos /p/, /t/, /k/, A função biológica e cerebral da linguagem é aquilo que
/b/, /d/, /g/ sugere o patear dos cavalos: mais profundamente distingue o homem dos outros animais.
Podemos considerar que o desenvolvimento desta função
E o bosque estala, move-se, estremece... cerebral ocorre em estreita ligação com a bipedia e a libertação
Da cavalgada o estrépito que aumenta da mão, que permitiram o aumento do volume do cérebro, a par
Perde-se após no centro da montanha... do desenvolvimento de órgãos fonadores e da mímica facial.
Devido a estas capacidades, para além da linguagem falada
Apud: Lêdo Ivo. Raimundo Correia: Poesia. 4ª ed. e escrita, o homem, aprendendo pela observação de animais,
Rio de Janeiro, Agir, p. 29. Coleção Nossos Clássicos. desenvolveu a língua de sinais adaptada pelos surdos em

10
LÍNGUA PORTUGUESA
diferentes países, não só para melhorar a comunicação entre Mamães. Todos aqueles que quiserem se tornar uma Jovem
surdos, mas também para utilizar em situações especiais, Mamãe, devem contatar padre Cavalcante em seu escritório. (...)
como no teatro e entre navios ou pessoas e não animais que
se encontram fora do alcance do ouvido, mas que se podem (Jornal da USP, 9, p. 15)
observar entre si.
Humor à parte, esses exemplos comprovam que aprender
Potencialidades da Linguagem não só a norma culta da língua, mas também os mecanismos de
estruturação do texto.
Depois de analisar as funções da linguagem, conclui-se
que ela é onipresente na vida de todos nós. Cerca-nos desde A palavra texto é bastante usada na escola e também em
o despertar da consciência, ainda no berço, segue-nos durante outras instituições sociais que trabalham com a linguagem. É
toda a vida e acompanha-nos até a hora da morte. Sem a comum ouvirmos expressões como “O texto constitucional desceu
linguagem, não se pode estruturar o mundo do trabalho, pois a detalhes que deveriam estar em leis ordinárias”; “Seu texto ficou
é ela que permite a troca de informações e de experiências e muito bom”; “O texto da prova de Português era muito longo e
a cooperação entre os homens. Sem ela, o homem não pode complexo”; “Os atores de novelas devem decorar textos enormes
conhecer-se nem conhecer o mundo. Sem ela, não se exerce a todos os dias”. Apesar de corrente, porém, o termo não é de
cidadania, porque os eleitores não podem influenciar o governo. fácil definição: quando perguntamos qual é o seu significado,
Sem ela não se pode aprender, expressar os sentimentos, percebemos que a maioria das pessoas é incapaz de responder
imaginar outras realidades, construir as utopias e os sonhos. com precisão e clareza.
No entanto, a linguagem parece-nos uma coisa natural. Não Texto é um todo organizado de sentido, delimitado por dois
prestamos muita atenção a ela. Nem sempre dedicamos muito brancos e produzido por um sujeito num dado tempo e num
tempo ao seu estudo. Conhecer bem a língua materna e línguas determinado espaço.
estrangeiras é uma necessidade. O texto é um todo organizado de sentido, isso quer dizer
Que é saber bem uma língua? Evidentemente, não é saber que ele não é um amontoado de frases simplesmente colocadas
descrevê-la. A descrição gramatical de uma língua é um meio de umas depois das outras, mas um conjunto de frases costuradas
adquirir sobre ela um domínio crescente. Saber bem uma língua entre si. Por isso o sentido de cada parte depende da sua relação
é saber usá-la bem. No entanto, o emprego de palavras raras e com as outras partes, isto é, o sentido de uma palavra ou de uma
a correção gramatical não são sinônimos do uso adequado da frase depende das outras palavras ou frases com que mantêm
língua. Falar bem é atingir os propósitos de comunicação. Para relação. Em síntese, o sentido depende do contexto, entendido
isso, é preciso usar um nível de língua adequado, é necessário como a unidade maior que compreende uma unidade menor, a
construir textos sem ambiguidades, coerentes, sem repetições oração é contexto da palavra, o período é contexto da oração
que não acrescentam nada ao sentido. e assim sucessivamente. O contexto pode ser explícito (quando
O texto que segue foi dito por um locutor esportivo: é exposto em palavras) ou implícito (quando é percebido na
situação em que o texto é produzido). Observe os três pequenos
“Adentra o tapete verde o facultativo esmeraldino a fim de textos abaixo:
pensar a contusão do filho do Divino Mestre, mola propulsora do
eleven periquito.” - Todos os dias ele fazia sua fezinha. Na noite de segunda-feira
(Álvaro da Costa e Silva. In: Bundas, p.33.) sonhou com um deserto e jogou seco no camelo.
- Nos desertos da Arábia, o camelo é ainda o principal meio de
O que o locutor quis dizer foi: Entra em campo o médico transporte dos beduínos.
do Palmeiras a fim de cuidar da contusão de Ademir da Guia - O camelo aqui carrega a família inteira nas costas, porque
(filho de Domingos da Guia), jogador de meio de campo do lá ninguém trabalha.
time do Parque Antártica. Certamente, aquele texto não seria
entendido pela maioria dos ouvintes. Portanto não é um bom Em cada uma dessas frases a palavra camelo tem um sentido
texto, porque não usa um nível de língua adequado à situação diferente. Na primeira, significa “o oitavo grupo do jogo no
de comunicação. Outros exemplos: bicho, que corresponde ao número 8 e inclui as dezenas 29, 30,
31 e 32”; na segunda, “animal originário das regiões desérticas,
“As videolocadoras de São Carlos estão escondendo suas fitas de grande porte, quadrúpede, de cor amarelada, de pescoço longo
de sexo explícito. A decisão atende a uma portaria de dezembro e com duas saliências no dorso”; na terceira, “pessoa que trabalha
de 1991, do Juizado de Menores, que proíbe que as casas de vídeo muito”. O que determina essa diferença de sentido da palavra
aluguem, exponham e vendam fitas pornográficas a menores de é exatamente o contexto, o todo em que ela está inserida. No
18 anos. A portaria proíbe ainda os menores de 18 anos de irem texto, portanto, o sentido de cada parte não é independente,
a motéis e rodeios sem a companhia ou autorização dos pais.” tudo são relações. Aliás, a palavra texto significa “tecido”,
que não é um amontoado de fios, mas uma trama arranjada
(Jornal Folha do Sudoeste) de maneira organizada. O sentido não é solitário, é solidário.
Certamente a portaria não deveria obrigar os pais Vejamos outros dois períodos:
a acompanhar os filhos aos motéis nem a dar-lhes uma
autorização por escrito para ser exibida na entrada desse tipo - Marcelinho é um bom atacante, mas é desagregador.
de estabelecimento. - Marcelinho é desagregador, mas é um bom atacante.
O jornal da USP publicou uma série de textos encontrados
em comunicados de paróquias e templos. Todos são mal Esses períodos relacionam diferentemente as orações. No
escritos, embora neles não se encontrem erros de ortografia, primeiro, a oração é “desagregador” é introduzida por “mas”,
concordância, etc.: enquanto no segundo é a oração “é um bom atacante” que é
iniciada por essa conjunção. O sentido é completamente
- Não deixe a preocupação acabar com você. Deixe que a Igreja diferente, pois o “mas” introduz o argumento mais forte e, por
ajude. conseguinte, determina a orientação argumentativa da frase.
- Terça-feira à noite: sopão dos pobres, depois oração e Isso significa que, quando afirmo, “não quero o jogador no meu
medicação. time”; quando digo, “acredito que todos os seus defeitos devem
- (...) lembre-se de todos que estão tristes e cansados de nossa ser desculpados”.
igreja e de nossa comunidade. Observe agora o poema “Canção do Exílio” de Murilo
- Para aqueles que têm filhos e não sabem, nós temos uma Mendes:
creche no segundo andar.
- Quinta-feira às 5h haverá reunião do Clube das Jovens Minha terra tem macieiras da Califórnia

11
LÍNGUA PORTUGUESA
onde cantam gaturamos de Veneza. dependendo do contexto em que está inserida.
Os poetas da minha terra Que é que faz perceber que um conjunto de frases
são pretos que vivem em torres de ametista, compõe um texto? O primeiro fator é a coerência, ou seja, a
os sargentos de exército são monistas, cubistas, compatibilidade de sentido entre elas, de modo que não haja
os filósofos são polacos vendendo a prestações. nada ilógico, nada contraditório, nada desconexo. Outro fator
A gente não pode dormir é a ligação das frases por certos elementos que recuperam
com os oradores e os pernilongos. passagens já ditas ou garantem a concatenação entre as partes.
Os sururus em família têm por testemunha a Gioconda. Assim, em “Não chove há vários meses. Os pastos não poderiam,
Eu morro sufocado pois, estar verdes”, a palavra “pois” estabelece uma relação de
em terra estrangeira. decorrência lógica entre uma e outra frase. O segundo fator,
Nossas flores são mais bonitas entretanto, é menos importante que o primeiro, pois mesmo
nossas frutas mais gostosas sem esses elementos de conexão, um conjunto de frases pode
mas custam cem mil réis a dúzia. ser coerente e, portanto, um todo organizado de sentido.
Ai quem me dera chupar uma carambola de verdade
e ouvir um sabiá com certidão de idade! Estudo do Significado

Poesias (1925-1953). Rio de Janeiro, O papel essencial da linguagem, em quaisquer das suas
José Olympio, 1959, p. 5. formas, é a produção de significado: ao se relacionar com as
coisas, o homem faz uso de sinais que as representam.
Tomando apenas os dois primeiros versos, pode-se pensar A linguagem pode ser usada pelo homem para múltiplas
que esse poema seja uma apologia do caráter universalista e finalidades: para ajudá-lo a compreender a si mesmo, o mundo
cosmopolita da brasilidade: macieiras e gaturamos representam físico e as pessoas que o rodeiam, para influenciar o outro e até
a natureza vegetal e animal, respectivamente; Califórnia e para trapacear, fingindo uma intenção para esconder outra. A
Veneza são a imagem do espaço estrangeiro, e minha terra, a do linguagem pode também manifestar-se sob grande variedade
solo pátrio. No Brasil, até a natureza acolhe o que é estrangeiro. de formas:
Pode-se ainda acrescentar, em apoio a essa tese, que esses - Sons produzidos pela voz (linguagem verbal)
versos são calcados nos dois primeiros do poema homônimo de - Cores, formas e volumes (linguagem visual)
Gonçalves Dias, que é uma glorificação da terra pátria: - Movimentos do corpo (linguagem corporal, dança)
- Sons produzidos por instrumentos (linguagem musical)
Minha terra tem palmeiras, - Imagens em movimento (cinema), etc.
Onde canta o Sabiá; Para resumir, costumamos distinguir duas grandes divisões
para definir as formas de linguagem:
Apud: Manuel Bandeira.
Gonçalves Dias: Poesia. 7ª ed. Rio de Janeiro - Linguagem Verbal: mais especificamente constituída
Agir, 1976, p. 18. Coleção Nossos Clássicos. pela língua, seja ela oral, seja escrita.
Essa hipótese de leitura, se não é absurda quando isolamos - Linguagens não-verbais: constituídas por todas as
os versos em questão, não encontra amparo quando os outras modalidades diferentes da língua: pintura, escultura,
confrontamos com o restante do texto. Murilo Mendes mostra, música, dança, cinema, etc.
na verdade, que as características da brasilidade não têm valor
positivo, não concorrem para a exaltação da pátria: o poeta Hoje em dia, graças, sobretudo à facilidade de reprodução
denuncia que a cultura brasileira é postiça, é uma miscelânea de sons, cores, movimentos e imagens, é muito comum
de elementos advindos de vários países. Ele mostra que os a exploração conjunta de várias formas de linguagem: a
“poetas são pretos que vivem em torres de ametista”, alienados linguagem do cinema e da televisão é uma demonstração
num mundo idealizado, evitando as mazelas do mundo real, eloquente da exploração conjunta de sons musicais e da voz
sem se preocupar com os negros, que vivem, em geral, em humana, de cores, de imagens em movimento.
condições muito precárias (trata-se de uma referência irônica Qualquer que seja a forma de manifestação, toda linguagem
ao Simbolismo e, principalmente, a Cruz e Sousa); que “os tem um ponto em comum: nenhuma opera com a realidade tal
sargentos do exército são monistas, cubistas”, ou seja, em vez que ela é, mas com representações da realidade. Dizendo de
da preocupação com seu ofício de garantir a segurança do outra maneira, toda linguagem é constituída de signos. E o que
território nacional, têm pretensões de incursionar por teorias são signos?
filosóficas e estéticas; que “os filósofos são polacos vendendo Signos são qualquer forma material (sons, linhas, cores,
a prestações”, são prostituídos (polaca é termo designativo volumes, imagens em movimento) que representam alguma
de prostituta) pela venalidade barata; que “os oradores” se coisa diferente dela mesma. Em outras palavras, todo signo
identificam com “os pernilongos” em sua oratória repetitiva; é constituído de algo material, perceptível pelos órgãos dos
que o romantismo gonçalvino estava certo ao afirmar que a sentidos (ouvido, olho) e de algo imaterial, uma representação
natureza brasileira é pródiga, só que essa prodigalidade não é mental, inteligível. A dimensão material do signo costuma ser
acessível à maioria da população. A exclamação do final é, ao designada por dois nomes: plano de expressão ou significante.
mesmo tempo, a manifestação do desejo de ter contato com A dimensão imaterial e inteligível é chamada por dois
coisas genuinamente brasileiras e um lamento, pois o poeta nomes: plano de conteúdo ou significado. Por uma questão
sabe que não se tornará realidade. de simplificação, usaremos a seguinte nomenclatura:
O texto de Murilo faz referência ao de Gonçalves Dias,
mas, diferentemente do poema gonçalvino, não celebra Signo: qualquer tipo de sinal material usado para
ufanisticamente a pátria. Ao contrário, ironiza-a, lamenta a representar algo, isto é, tornar presente alguma coisa ausente.
invasão estrangeira. O exílio é a própria terra, desnaturada a
ponto de parecer estrangeira. Uma árvore plantada no bosque não é um signo, porque
Desse modo, os dois primeiros versos não podem ser não passa de uma árvore, não representa nada além de si
interpretados como um elogio ao caráter cosmopolita da cultura mesma. Prova disso é que não podemos trazer para este livro a
brasileira. Ao contrário, devem ser lidos como uma crítica ao árvore real, apenas uma representação dela, formada de cores
caráter postiço da nossa cultura. Isso porque só a segunda e formas sobre uma superfície de papel.
interpretação se encaixa coerentemente dentro do contexto. Que o signo não passa de representação da realidade é um
Por exemplo, comprova-se que o significado das frases tema que tem sido objeto de debate entre os homens. O célebre
não é autônomo. Num texto, o significado das partes depende pintor surrealista belga René Magritte (1898-1967), pintou
do todo. Por isso, cada frase tem um significado distinto, um cachimbo com requintes de pormenores, dando a máxima

12
LÍNGUA PORTUGUESA
impressão de realismo. Surpreendentemente, num jogo de Resumindo tudo, para quem estuda uma língua do ponto
ironia, escreveu abaixo da pintura a frase Ceci n’est pás une de vista de quem vai conviver e trabalhar com ela, o que mais
pipe (“Isto não é um cachimbo”). importa é a capacidade de produzir e compreender significados.
Aos que o contestavam, achando absurda a ideia de negar Todos os demais tipos de aprendizado linguístico estão
que aquilo fosse um cachimbo, conta-se que ele desafiava: subordinados a essas duas competências mais amplas e mais
altas.
__ Então acenda-o e comece a fumá-lo. A Semântica é um ramo da Linguística que se ocupa
do significado das formas linguísticas em geral. Por formas
Após essas considerações, vamos fazer duas observações de linguísticas vamos entender tanto as mínimas unidades de
ordem terminológica: significado constituintes das palavras (os prefixos e sufixos, por
- a representação do cachimbo é um signo do cachimbo, não exemplo) quanto enunciados maiores, como orações e períodos.
o objeto cachimbo; Analisar, pois, uma palavra ou uma frase sob o ponto de vista
- ao objeto chamamos de referente, ou a coisa real. semântico equivale a tentar decifrar o que elas significam ou o
que querem dizer.
Podemos, então, após esses dados, montar um esquema Dado que a finalidade última de qualquer linguagem é a
daquilo que os estudiosos chamam de signo: produção de significado, não é preciso destacar a importância
fundamental da Semântica dentro dos estudos linguísticos.
Signo: é qualquer objeto, forma ou fenômeno material que Nem é preciso também falar da importância desse tópico nas
representa a ideia de algo diferente dele mesmo. provas de concursos na matéria de língua portuguesa em geral.
Para facilitar a compreensão de certas particularidades
Assim sendo, o signo apresenta três dimensões: relativas ao significado das palavras e das formas linguísticas
- Significante (ou plano de expressão): é a parte material em geral, uma noção primária se impõe como necessária: a de
do signo (um objeto, uma forma ou um fenômeno perceptível que o significado de um signo não é constituído por uma peça
pelos sensores do corpo humano). única, mas por um punhado de significados menores que se
- Significado (ou plano de conteúdo): é o conceito, ou a combinam entre si para criar a noção com que representamos
forma mental criada no intelecto pelo significante. as coisas ou os eventos do mundo.
- Referente: é a coisa representada pelo signo. É dado de Dizemos de outra maneira, o significado das palavras não
realidade trazido à mente por meio do signo. é simples, mas complexo, constituído de um feixe de unidades
menores a que os estudiosos chamam de traços semânticos ou
São os signos que nos permitem trazer para a lembrança traços de significado.
referentes que já deixaram de existir. As palavras, por exemplo, São os traços semânticos que usamos para definir o
são signos e, por meio delas, podemos trazer para o presente significado das palavras. Tomemos um exemplo que já ficou
pessoas e fatos que já desapareceram. Tomemos, por exemplo, clássico nos estudos de Semântica, usado pelo linguista
uma palavra como Camões. francês contemporâneo Bernard Pottier. Segundo ele, a palavra
Trata-se de um signo, pois o referente (o poeta em carne “cadeira” é um móvel doméstico que contém os seguintes traços
e osso) não existe mais. Significante: uma conjunto de sons, semânticos:
representado pelo espectro de uma onda sonora. Significado: - com encosto.
o conceito associado no intelecto quando ouvimos essa - sobre pernas.
combinação de sons. O referente é o famoso poeta português - para uma só pessoa.
Luís Vaz de Camões, que, como se sabe, morreu faz tempo. - para sentar-se.
Observação: A rigor, não é exato usar um retrato de
Camões para ilustrar o significado da palavra Camões, pois Se aos quatro traços da palavra “cadeira” acrescentarmos
este, na verdade, é de instância intelectual. Excluindo esse mais um, “com braços”, teremos a palavra “poltrona”. Uma
inconveniente, a pintura, serve para sugerir o conceito que mesma palavra pode, num dado contexto, trocar um traço
a combinação de sons (k – a – m – õ – e – s) cria no nosso semântico por outro e ganhar novo sentido. É o que acontece,
intelecto. por exemplo, em:
A conclusão mais importante de tudo isso é que usamos
os signos no lugar das coisas e, pela linguagem, construímos Peixe não vive fora d’água.
um universo paralelo ao universo real. Se levarmos em conta
que as relações entre os homens são determinadas mais pelas A palavra “peixe” é marcada, nesse contexto, pelo traço
representações que fazemos das coisas do que pelas coisas em semântico “não humano”. Esse traço pode ser trocado, por
si, vamos compreender que interpretar e produzir significados é exemplo, por um traço “humano”, noutro contexto como este:
a competência de maior importância para quem deseja dominar
os segredos da linguagem. Na festa de aniversário da minha prima, eu era um peixe fora
Relacionando o aprendizado do português com esses dados d’água.
preliminares, podemos encadear os seguintes raciocínios:
- A principal função de qualquer forma de linguagem é “Peixe”, nesse contexto, não fará sentido se não trocarmos o
a construção de significados para atingir certos resultados traço semântico “não humano” por “humano”.
planejados pelo construtor. Em síntese, concluímos que, por ser constituído de feixes de
- O português é uma forma de linguagem. traços semânticos, o sentido da palavra não é estável, podendo
- Portanto a competência mais importante para os falantes sofrer variações de época para época, de lugar para lugar, de
da língua portuguesa é saber construir significados e decifrar os contexto para contexto.
significados produzidos por meio dela. Há contextos em que uma palavra não pode ser interpretada
com todos os traços semânticos que comumente a definem.
Esse é um dado de extrema importância tanto para quem Por isso é que os bons dicionários costumam dar os diferentes
ensina quanto para quem aprende não só o português como sentidos possíveis de uma palavra, acompanhados do contexto
qualquer outra língua, com o propósito de usá-la para o mundo em que ela adquire cada um dos seus vários sentidos.
do trabalho, para o exercício da cidadania e para aquisição de Tomemos como exemplo uma palavra como “cadeia” e
novos conhecimentos. alguns de seus múltiplos sentidos no português:
Para quem aprende uma língua com esse tipo de interesse, o
que mais importa é adquirir a capacidade de compreender, com - Prisão: Sonegar imposto dá cadeia.
a máxima proficiência, os significados direcionados para atingir - Rede, conjunto de emissoras: O presidente falará em cadeia
os resultados programados. nacional.

13
LÍNGUA PORTUGUESA
- Sequência: As ruínas são cercadas por uma cadeia de bloquinho comum de anotações e segurava na mão, retomando
montanhas. na segunda um dos termos da primeira: bloquinho. O pronome
Levando esses dados em consideração, torna-se mais relativo é um elemento coesivo, e a conexão entre as duas
fácil compreender as particularidades sobre o significado das orações, um fenômeno de coesão. Há dois tipos principais de
palavras. mecanismos de coesão: retomada ou antecipação de palavras,
expressões ou frases e encadeamento de segmentos.
Delimitações e Sujeito do Texto
Retomada ou Antecipação por meio de uma
O texto é delimitado por dois brancos. Se ele é um todo palavra gramatical - (pronome, verbos ou advérbios)
organizado de sentido, ele pode ser verbal, visual (um quadro),
verbal e visual (um filme), sonoro (uma música), etc. Mas em “No mercado de trabalho brasileiro, ainda hoje não há total
todos esses casos ele será delimitado por dois espaços de não- igualdade entre homens e mulheres: estas ainda ganham menos
sentido, dois brancos, um antes de começar o texto e outro do que aqueles em cargos equivalentes.”
depois. É o branco do papel; é o tempo de espera para que um
filme comece e o que está depois da palavra FIM; é o silêncio Nesse período, o pronome demonstrativo “estas” retoma o
que precede os primeiros acordes de uma melodia e que sucede termo mulheres, enquanto “aqueles” recupera a palavra homens.
às notas finais, etc. Os termos que servem para retomar outros são denominados
O texto é produzido por um sujeito num dado tempo e num anafóricos; os que servem para anunciar, para antecipar outros
determinado espaço. Esse sujeito, por pertencer a um grupo são chamados catafóricos. No exemplo a seguir, desta antecipa
social que vive num dado tempo e num certo espaço, expõe em abandonar a faculdade no último ano:
seus textos as ideias, os anseios, os temores, as expectativas
desse grupo. Todo texto, assim, relaciona-se com o contexto “Já viu uma loucura desta, abandonar a faculdade no último
histórico e geográfico em que foi produzido, refletindo a ano?”
realidade apreendida por seu autor, que sobre ela se pronuncia.
O poema de Murilo Mendes que comentamos anteriormente São anafóricos ou catafóricos os pronomes demonstrativos,
mostra o anseio de uma geração, no Brasil, em certa época, de os pronomes relativos, certos advérbios ou locuções adverbiais
conhecer bem o país e revelar suas mazelas para transformá-lo. (nesse momento, então, lá), o verbo fazer, o artigo definido, os
Não há texto que não reflita o seu tempo e o seu lugar. pronomes pessoais de 3ª pessoa (ele, o, a, os, as, lhe, lhes), os
Cabe lembrar, no entanto, que uma sociedade não produz uma pronomes indefinidos. Exemplos:
única forma de ver a realidade, um modo único de analisar os
problemas estabelecidos num dado contexto. Como a sociedade “Ele era muito diferente de seu mestre, a quem sucedera na
é dividida em grupos sociais, que têm interesses muitas vezes cátedra de Sociologia na Universidade de São Paulo.”
antagônicos, ela produz ideias divergentes entre si. A mesma
sociedade que gera a ideia de que é preciso pôr abaixo a O pronome relativo “quem” retoma o substantivo mestre.
floresta amazônica para explorar suas riquezas, produz a ideia
de que preservá-la é mais rentável. É bem verdade, no entanto, “As pessoas simplificam Machado de Assis; elas o veem como
que algumas ideias, em certas épocas, exercem domínio sobre um descrente do amor e da amizade.”
outras.
É necessário entender as concepções correntes na época e O pronome pessoal “elas” recupera o substantivo pessoas; o
na sociedade em que o texto foi produzido, para não correr o pronome pessoal “o” retoma o nome Machado de Assis.
risco de entendê-lo de maneira distorcida. Como não há ideias
puras, todas as ideias estão materializadas em textos, analisar “Os dois homens caminhavam pela calçada, ambos trajando
a relação de um texto com sua época é estudar a sua relação roupa escura.”
com outros textos.
É preciso que fiquem bem claras estas conclusões: O numeral “ambos” retoma a expressão os dois homens.
- No texto, o sentido não é solitário, mas solidário.
- O texto está delimitado por dois espaços de não-sentido. “Fui ao cinema domingo e, chegando lá, fiquei desanimado
- O texto revela ideais, concepções, anseios, expectativas com a fila.”
e temores de um grupo social numa determinada época, em
determinado lugar. O advérbio “lá” recupera a expressão ao cinema.

“O governador vai pessoalmente inaugurar a creche dos


funcionários do palácio, e o fará para demonstrar seu apreço aos
servidores.”

Articulação do texto: coesão e A forma verbal “fará” retoma a perífrase verbal vai inaugurar
coerência. e seu complemento.

- Em princípio, o termo a que “o” anafórico se refere deve


Coesão estar presente no texto, senão a coesão fica comprometida,
como neste exemplo:
Uma das propriedades que distinguem um texto de um
amontoado de frases é a relação existente entre os elementos “André é meu grande amigo. Começou a namorá-la há vários
que os constituem. A coesão textual é a ligação, a relação, meses.”
a conexão entre palavras, expressões ou frases do texto. Ela
manifesta-se por elementos gramaticais, que servem para A rigor, não se pode dizer que o pronome “la” seja um
estabelecer vínculos entre os componentes do texto. Observe: anafórico, pois não está retomando nenhuma das palavras
citadas antes. Exatamente por isso, o sentido da frase fica
“O iraquiano leu sua declaração num bloquinho comum de totalmente prejudicado: não há possibilidade de se depreender
anotações, que segurava na mão.” o sentido desse pronome.
Pode ocorrer, no entanto, que o anafórico não se refira a
Nesse período, o pronome relativo “que” estabelece conexão nenhuma palavra citada anteriormente no interior do texto,
entre as duas orações. O iraquiano leu sua declaração num mas que possa ser inferida por certos pressupostos típicos da

14
LÍNGUA PORTUGUESA
cultura em que se inscreve o texto. É o caso de um exemplo Referência à força física que caracteriza o herói grego
como este: Hércules.

“O casamento teria sido às 20 horas. O noivo já estava “Um presidente da República tem uma agenda de trabalho
desesperado, porque eram 21 horas e ela não havia comparecido.” extremamente carregada. Deve receber ministros, embaixadores,
visitantes estrangeiros, parlamentares; precisa a todo o momento
Por dados do contexto cultural, sabe-se que o pronome “ela” tomar graves decisões que afetam a vida de muitas pessoas;
é um anafórico que só pode estar-se referindo à palavra noiva. necessita acompanhar tudo o que acontece no Brasil e no mundo.
Num casamento, estando presente o noivo, o desespero só pode Um presidente deve começar a trabalhar ao raiar do dia e terminar
ser pelo atraso da noiva (representada por “ela” no exemplo sua jornada altas horas da noite.”
citado).
A repetição do termo presidente estabelece a coesão entre o
- O artigo indefinido serve geralmente para introduzir último período e o que vem antes dele.
informações novas ao texto. Quando elas forem retomadas,
deverão ser precedidas do artigo definido, pois este é que tem a “Observava as estrelas, os planetas, os satélites. Os astros
função de indicar que o termo por ele determinado é idêntico, sempre o atraíram.”
em termos de valor referencial, a um termo já mencionado.
Os dois períodos estão relacionados pelo hiperônimo astros,
“O encarregado da limpeza encontrou uma carteira na sala que recupera os hipônimos estrelas, planetas, satélites.
de espetáculos. Curiosamente, a carteira tinha muito dinheiro
dentro, mas nem um documento sequer.” “Eles (os alquimistas) acreditavam que o organismo do homem
era regido por humores (fluidos orgânicos) que percorriam,
- Quando, em dado contexto, o anafórico pode referir-se a ou apenas existiam, em maior ou menor intensidade em nosso
dois termos distintos, há uma ruptura de coesão, porque ocorre corpo. Eram quatro os humores: o sangue, a fleuma (secreção
uma ambiguidade insolúvel. É preciso que o texto seja escrito pulmonar), a bile amarela e a bile negra. E eram também estes
de tal forma que o leitor possa determinar exatamente qual é a quatro fluidos ligados aos quatro elementos fundamentais: ao
palavra retomada pelo anafórico. Ar (seco), à Água (úmido), ao Fogo (quente) e à Terra (frio),
respectivamente.”
“Durante o ensaio, o ator principal brigou com o diretor por
causa da sua arrogância.” Ziraldo. In: Revista Vozes, nº3, abril de 1970, p.18.

O anafórico “sua” pode estar-se referindo tanto à palavra Nesse texto, a ligação entre o segundo e o primeiro períodos
ator quanto a diretor. se faz pela repetição da palavra humores; entre o terceiro e o
segundo se faz pela utilização do sinônimo fluidos.
“André brigou com o ex-namorado de uma amiga, que É preciso manejar com muito cuidado a repetição de
trabalha na mesma firma.” palavras, pois, se ela não for usada para criar um efeito de
sentido de intensificação, constituirá uma falha de estilo.
Não se sabe se o anafórico “que” está se referindo ao termo No trecho transcrito a seguir, por exemplo, fica claro o uso
amiga ou a ex-namorado. Permutando o anafórico “que” por “o da repetição da palavra vice e outras parecidas (vicissitudes,
qual” ou “a qual”, essa ambiguidade seria desfeita. vicejam, viciem), com a evidente intenção de ridicularizar a
condição secundária que um provável flamenguista atribui ao
Retomada por palavra lexical - (substantivo, Vasco e ao seu Vice-presidente:
adjetivo ou verbo)
“Recebi por esses dias um e-mail com uma série de piadas
Uma palavra pode ser retomada, que por uma repetição, sobre o pouco simpático Eurico Miranda. Faltam-me provas, mas
quer por uma substituição por sinônimo, hiperônimo, hipônimo tudo leva a crer que o remetente seja um flamenguista.”
ou antonomásia.
Sinônimo é o nome que se dá a uma palavra que possui Segundo o texto, Eurico nasceu para ser vice: é vice-
o mesmo sentido que outra, ou sentido bastante aproximado: presidente do clube, vice-campeão carioca e bi-vice-campeão
injúria e afronta, alegre e contente. mundial. E isso sem falar do vice no Carioca de futsal, no Carioca
Hiperônimo é um termo que mantém com outro uma de basquete, no Brasileiro de basquete e na Taça Guanabara.
relação do tipo contém/está contido; São vicissitudes que vicejam. Espero que não viciem.
Hipônimo é uma palavra que mantém com outra uma
relação do tipo está contido/contém. O significado do termo José Roberto Torero. In: Folha de S. Paulo, 08/03/2000, p.
rosa está contido no de flor e o de flor contém o de rosa, pois 4-7.
toda rosa é uma flor, mas nem toda flor é uma rosa. Flor é, pois,
hiperônimo de rosa, e esta palavra é hipônimo daquela. A elipse é o apagamento de um segmento de frase que
Antonomásia é a substituição de um nome próprio por pode ser facilmente recuperado pelo contexto. Também
um nome comum ou de um comum por um próprio. Ela ocorre, constitui um expediente de coesão, pois é o apagamento de um
principalmente, quando uma pessoa célebre é designada por termo que seria repetido, e o preenchimento do vazio deixado
uma característica notória ou quando o nome próprio de uma pelo termo apagado (=elíptico) exige, necessariamente, que se
personagem famosa é usado para designar outras pessoas que faça correlação com outros termos presentes no contexto, ou
possuam a mesma característica que a distingue: referidos na situação em que se desenrola a fala.
Vejamos estes versos do poema “Círculo vicioso”, de
“O rei do futebol (=Pelé) só podia ser um brasileiro.” Machado de Assis:

“O herói de dois mundos (=Garibaldi) foi lembrado numa (...)


recente minissérie de tevê.” Mas a lua, fitando o sol, com azedume:

Referência ao fato notório de Giuseppe Garibaldi haver “Mísera! Tivesse eu aquela enorme, aquela
lutado pela liberdade na Europa e na América. Claridade imorta, que toda a luz resume!”
Obra completa. Rio de Janeiro, Nova Aguilar, 1979, v.III,
“Ele é um Hércules.” (=um homem muito forte). p. 151.

15
LÍNGUA PORTUGUESA
Nesse caso, o verbo dizer, que seria enunciado antes daquilo Arrolam-se três argumentos em favor da tese que é o
que disse a lua, isto é, antes das aspas, fica subentendido, é interlocutor quem pode tomar uma dada decisão. O último
omitido por ser facilmente presumível. deles é introduzido por “e também”, que indica um argumento
Qualquer segmento da frase pode sofrer elipse. Veja que, final na mesma direção argumentativa dos precedentes.
no exemplo abaixo, é o sujeito meu pai que vem elidido (ou Esses operadores introduzem novos argumentos; não
apagado) antes de sentiu e parou: significam, em hipótese nenhuma, a repetição do que já foi dito.
Ou seja, só podem ser ligados com conectores de conjunção
“Meu pai começou a andar novamente, sentiu a pontada no segmentos que representam uma progressão discursiva.
peito e parou.” É possível dizer “Disfarçou as lágrimas que o assaltaram e
continuou seu discurso”, porque o segundo segmento indica
Pode ocorrer também elipse por antecipação. No exemplo um desenvolvimento da exposição. Não teria cabimento
que segue, aquela promoção é complemento tanto de querer usar operadores desse tipo para ligar dois segmentos como
quanto de desejar, no entanto aparece apenas depois do “Disfarçou as lágrimas que o assaltaram e escondeu o choro
segundo verbo: que tomou conta dele”.

“Ficou muito deprimido com o fato de ter sido preferido. - Disjunção Argumentativa: há também operadores
Afinal, queria muito, desejava ardentemente aquela promoção.” que indicam uma disjunção argumentativa, ou seja, fazem uma
conexão entre segmentos que levam a conclusões opostas, que
Quando se faz essa elipse por antecipação com verbos têm orientação argumentativa diferente: ou, ou então, quer...
que têm regência diferente, a coesão é rompida. Por exemplo, quer, seja... seja, caso contrário, ao contrário.
não se deve dizer “Conheço e gosto deste livro”, pois o verbo
conhecer rege complemento não introduzido por preposição, “Não agredi esse imbecil. Ao contrário, ajudei a separar a
e a elipse retoma o complemento inteiro, portanto teríamos briga, para que ele não apanhasse.”
uma preposição indevida: “Conheço (deste livro) e gosto deste
livro”. Em “Implico e dispenso sem dó os estranhos palpiteiros”, O argumento introduzido por ao contrário é diametralmente
diferentemente, no complemento em elipse faltaria a preposição oposto àquele de que o falante teria agredido alguém.
“com” exigida pelo verbo implicar.
Nesses casos, para assegurar a coesão, o recomendável é - Conclusão: existem operadores que marcam uma
colocar o complemento junto ao primeiro verbo, respeitando conclusão em relação ao que foi dito em dois ou mais enunciados
sua regência, e retomá-lo após o segundo por um anafórico, anteriores (geralmente, uma das afirmações de que decorre a
acrescentando a preposição devida (Conheço este livro e conclusão fica implícita, por manifestar uma voz geral, uma
gosto dele) ou eliminando a indevida (Implico com estranhos verdade universalmente aceita): logo, portanto, por conseguinte,
palpiteiros e os dispenso sem dó). pois (o pois é conclusivo quando não encabeça a oração).

Coesão por Conexão “Essa guerra é uma guerra de conquista, pois visa ao
controle dos fluxos mundiais de petróleo. Por conseguinte, não é
Há na língua uma série de palavras ou locuções que são moralmente defensável.”
responsáveis pela concatenação ou relação entre segmentos
do texto. Esses elementos denominam-se conectores ou Por conseguinte introduz uma conclusão em relação à
operadores discursivos. Por exemplo: visto que, até, ora, no afirmação exposta no primeiro período.
entanto, contudo, ou seja.
Note-se que eles fazem mais do que ligar partes do texto: - Comparação: outros importantes operadores discursivos
estabelecem entre elas relações semânticas de diversos tipos, são os que estabelecem uma comparação de igualdade,
como contrariedade, causa, consequência, condição, conclusão, superioridade ou inferioridade entre dois elementos, com vistas
etc. Essas relações exercem função argumentativa no texto, a uma conclusão contrária ou favorável a certa ideia: tanto...
por isso os operadores discursivos não podem ser usados quanto, tão... como, mais... (do) que.
indiscriminadamente.
Na frase “O time apresentou um bom futebol, mas não “Os problemas de fuga de presos serão tanto mais graves
alcançou a vitória”, por exemplo, o conector “mas” está quanto maior for a corrupção entre os agentes penitenciários.”
adequadamente usado, pois ele liga dois segmentos com
orientação argumentativa contrária. Se fosse utilizado, nesse O comparativo de igualdade tem no texto uma função
caso, o conector “portanto”, o resultado seria um paradoxo argumentativa: mostrar que o problema da fuga de presos
semântico, pois esse operador discursivo liga dois segmentos cresce à medida que aumenta a corrupção entre os agentes
com a mesma orientação argumentativa, sendo o segmento penitenciários; por isso, os segmentos podem até ser permutáveis
introduzido por ele a conclusão do anterior. do ponto de vista sintático, mas não o são do ponto de vista
argumentativo, pois não há igualdade argumentativa proposta,
- Gradação: há operadores que marcam uma gradação “Tanto maior será a corrupção entre os agentes penitenciários
numa série de argumentos orientados para uma mesma quanto mais grave for o problema da fuga de presos”.
conclusão. Dividem-se eles, em dois subtipos: os que indicam Muitas vezes a permutação dos segmentos leva a conclusões
o argumento mais forte de uma série: até, mesmo, até mesmo, opostas: Imagine-se, por exemplo, o seguinte diálogo entre o
inclusive, e os que subentendem uma escala com argumentos diretor de um clube esportivo e o técnico de futebol:
mais fortes: ao menos, pelo menos, no mínimo, no máximo,
quando muito. “__Precisamos promover atletas das divisões de base para
reforçar nosso time.
- Conjunção Argumentativa: há operadores que __Qualquer atleta das divisões de base é tão bom quanto os
assinalam uma conjunção argumentativa, ou seja, ligam um do time principal.”
conjunto de argumentos orientados em favor de uma dada
conclusão: e, também, ainda, nem, não só... mas também, Nesse caso, o argumento do técnico é a favor da promoção,
tanto... como, além de, a par de. pois ele declara que qualquer atleta das divisões de base
tem, pelo menos, o mesmo nível dos do time principal, o que
“Se alguém pode tomar essa decisão é você. Você é o diretor da significa que estes não primam exatamente pela excelência em
escola, é muito respeitado pelos funcionários e também é muito relação aos outros.
querido pelos alunos.”

16
LÍNGUA PORTUGUESA
Suponhamos, agora, que o técnico tivesse invertido os - Argumento Decisivo: há operadores discursivos
segmentos na sua fala: que introduzem um argumento decisivo para derrubar a
argumentação contrária, mas apresentando-o como se fosse
“__Qualquer atleta do time principal é tão bom quanto os das um acréscimo, como se fosse apenas algo mais numa série
divisões de base.” argumentativa: além do mais, além de tudo, além disso, ademais.

Nesse caso, seu argumento seria contra a necessidade da “Ele está num período muito bom da vida: começou a namorar
promoção, pois ele estaria declarando que os atletas do time a mulher de seus sonhos, foi promovido na empresa, recebeu um
principal são tão bons quanto os das divisões de base. prêmio que ambicionava havia muito tempo e, além disso, ganhou
uma bolada na loteria.”
- Explicação ou Justificativa: há operadores que
introduzem uma explicação ou uma justificativa em relação ao O operador discursivo introduz o que se considera a prova
que foi dito anteriormente: porque, já que, que, pois. mais forte de que “Ele está num período muito bom da vida”; no
entanto, essa prova é apresentada como se fosse apenas mais
“Já que os Estados Unidos invadiram o Iraque sem autorização uma.
da ONU, devem arcar sozinhos com os custos da guerra.”
- Generalização ou Amplificação: existem operadores
Já que inicia um argumento que dá uma justificativa para que assinalam uma generalização ou uma amplificação do que
a tese de que os Estados Unidos devam arcar sozinhos com o foi dito antes: de fato, realmente, como aliás, também, é verdade
custo da guerra contra o Iraque. que.

- Contrajunção: os operadores discursivos que assinalam “O problema da erradicação da pobreza passa pela geração de
uma relação de contrajunção, isto é, que ligam enunciados empregos. De fato, só o crescimento econômico leva ao aumento
com orientação argumentativa contrária, são as conjunções de renda da população.”
adversativas (mas, contudo, todavia, no entanto, entretanto,
porém) e as concessivas (embora, apesar de, apesar de que, O conector introduz uma amplificação do que foi dito antes.
conquanto, ainda que, posto que, se bem que).
Qual é a diferença entre as adversativas e as concessivas, “Ele é um técnico retranqueiro, como aliás o são todos os que
se tanto umas como outras ligam enunciados com orientação atualmente militam no nosso futebol.
argumentativa contrária?
Nas adversativas, prevalece a orientação do segmento O conector introduz uma generalização ao que foi
introduzido pela conjunção. afirmado: não “ele”, mas todos os técnicos do nosso futebol são
retranqueiros.
“O atleta pode cair por causa do impacto, mas se levanta mais
decidido a vencer.” - Especificação ou Exemplificação: também
há operadores que marcam uma especificação ou uma
Nesse caso, a primeira oração conduz a uma conclusão exemplificação do que foi afirmado anteriormente: por exemplo,
negativa sobre um processo ocorrido com o atleta, enquanto a como.
começada pela conjunção “mas” leva a uma conclusão positiva.
Essa segunda orientação é a mais forte. “A violência não é um fenômeno que está disseminado apenas
Compare-se, por exemplo, “Ela é simpática, mas não é entre as camadas mais pobres da população. Por exemplo, é
bonita” com “Ela não é bonita, mas é simpática”. No primeiro crescente o número de jovens da classe média que estão envolvidos
caso, o que se quer dizer é que a simpatia é suplantada pela falta em toda sorte de delitos, dos menos aos mais graves.”
de beleza; no segundo, que a falta de beleza perde relevância
diante da simpatia. Quando se usam as conjunções adversativas, Por exemplo assinala que o que vem a seguir especifica,
introduz-se um argumento com vistas à determinada conclusão, exemplifica a afirmação de que a violência não é um fenômeno
para, em seguida, apresentar um argumento decisivo para uma adstrito aos membros das “camadas mais pobres da população”.
conclusão contrária.
Com as conjunções concessivas, a orientação argumentativa - Retificação ou Correção: há ainda os que indicam uma
que predomina é a do segmento não introduzido pela conjunção. retificação, uma correção do que foi afirmado antes: ou melhor,
de fato, pelo contrário, ao contrário, isto é, quer dizer, ou seja, em
“Embora haja conexão entre saber escrever e saber gramática, outras palavras. Exemplo:
trata-se de capacidades diferentes.”
“Vou-me casar neste final de semana. Ou melhor, vou passar a
A oração iniciada por “embora” apresenta uma orientação viver junto com minha namorada.”
argumentativa no sentido de que saber escrever e saber
gramática são duas coisas interligadas; a oração principal O conector inicia um segmento que retifica o que foi dito
conduz à direção argumentativa contrária. antes.
Quando se utilizam conjunções concessivas, a estratégia Esses operadores servem também para marcar um
argumentativa é a de introduzir no texto um argumento que, esclarecimento, um desenvolvimento, uma redefinição do
embora tido como verdadeiro, será anulado por outro mais conteúdo enunciado anteriormente. Exemplo:
forte com orientação contrária.
A diferença entre as adversativas e as concessivas, portanto, “A última tentativa de proibir a propaganda de cigarros nas
é de estratégia argumentativa. Compare os seguintes períodos: corridas de Fórmula 1 não vingou. De fato, os interesses dos
fabricantes mais uma vez prevaleceram sobre os da saúde.”
“Por mais que o exército tivesse planejado a operação O conector introduz um esclarecimento sobre o que foi dito
(argumento mais fraco), a realidade mostrou-se mais complexa antes.
(argumento mais forte).” Servem ainda para assinalar uma atenuação ou um reforço
“O exército planejou minuciosamente a operação (argumento do conteúdo de verdade de um enunciado. Exemplo:
mais fraco), mas a realidade mostrou-se mais complexa
(argumento mais forte).” “Quando a atual oposição estava no comando do país, não fez
o que exige hoje que o governo faça. Ao contrário, suas políticas
iam na direção contrária do que prega atualmente.

17
LÍNGUA PORTUGUESA
O conector introduz um argumento que reforça o que foi A língua tem um grande número de conectores e
dito antes. sequenciadores. Apresentamos os principais e explicamos
sua função. É preciso ficar atento aos fenômenos de coesão.
- Explicação: há operadores que desencadeiam uma Mostramos que o uso inadequado dos conectores e a utilização
explicação, uma confirmação, uma ilustração do que foi inapropriada dos anafóricos ou catafóricos geram rupturas na
afirmado antes: assim, desse modo, dessa maneira. coesão, o que leva o texto a não ter sentido ou, pelo menos, a
não ter o sentido desejado. Outra falha comum no que tange
“O exército inimigo não desejava a paz. Assim, enquanto se a coesão é a falta de partes indispensáveis da oração ou do
processavam as negociações, atacou de surpresa.” período. Analisemos este exemplo:

O operador introduz uma confirmação do que foi afirmado “As empresas que anunciaram que apoiariam a campanha de
antes. combate à fome que foi lançada pelo governo federal.”
Coesão por Justaposição
O período compõe-se de:
É a coesão que se estabelece com base na sequência dos - As empresas
enunciados, marcada ou não com sequenciadores. Examinemos - que anunciaram (oração subordinada adjetiva restritiva da
os principais sequenciadores. primeira oração)
- que apoiariam a campanha de combate à fome (oração
- Sequenciadores Temporais: são os indicadores de subordinada substantiva objetiva direta da segunda oração)
anterioridade, concomitância ou posterioridade: dois meses - que foi lançada pelo governo federal (oração subordinada
depois, uma semana antes, um pouco mais tarde, etc. (são adjetiva restritiva da terceira oração).
utilizados predominantemente nas narrações).
Observe-se que falta o predicado da primeira oração. Quem
“Uma semana antes de ser internado gravemente doente, escreveu o período começou a encadear orações subordinadas
ele esteve conosco. Estava alegre e cheio de planos para o futuro.” e “esqueceu-se” de terminar a principal.
Quebras de coesão desse tipo são mais comuns em períodos
- Sequenciadores Espaciais: são os indicadores de longos. No entanto, mesmo quando se elaboram períodos curtos
posição relativa no espaço: à esquerda, à direita, junto de, etc. é preciso cuidar para que sejam sintaticamente completos e
(são usados principalmente nas descrições). para que suas partes estejam bem conectadas entre si.
Para que um conjunto de frases constitua um texto, não
“A um lado, duas estatuetas de bronze dourado, basta que elas estejam coesas: se não tiverem unidade de
representando o amor e a castidade, sustentam uma cúpula oval sentido, mesmo que aparentemente organizadas, elas não
de forma ligeira, donde se desdobram até o pavimento bambolins passarão de um amontoado injustificado. Exemplo:
de cassa finíssima. (...) Do outro lado, há uma lareira, não
de fogo, que o dispensa nosso ameno clima fluminense, ainda na “Vivo há muitos anos em São Paulo. A cidade tem excelentes
maior força do inverno.” restaurantes. Ela tem bairros muito pobres. Também o Rio de
José de Alencar. Senhora. São Paulo, FTD, 1992, p. 77. Janeiro tem favelas.”

- Sequenciadores de Ordem: são os que assinalam Todas as frases são coesas. O hiperônimo cidade retoma
a ordem dos assuntos numa exposição: primeiramente, em o substantivo São Paulo, estabelecendo uma relação entre o
segunda, a seguir, finalmente, etc. segundo e o primeiro períodos. O pronome “ela” recupera a
palavra cidade, vinculando o terceiro ao segundo período.
“Para mostrar os horrores da guerra, falarei, inicialmente, O operador também realiza uma conjunção argumentativa,
das agruras por que passam as populações civis; em seguida, relacionando o quarto período ao terceiro. No entanto, esse
discorrerei sobre a vida dos soldados na frente de batalha; conjunto não é um texto, pois não apresenta unidade de sentido,
finalmente, exporei suas consequências para a economia mundial isto é, não tem coerência. A coesão, portanto, é condição
e, portanto, para a vida cotidiana de todos os habitantes do necessária, mas não suficiente, para produzir um texto.
planeta.”
Questões
- Sequenciadores para Introdução: são os que, na
conversação principalmente, servem para introduzir um tema 1. Texto 1 – Bem tratada, faz bem
ou mudar de assunto: a propósito, por falar nisso, mas voltando
ao assunto, fazendo um parêntese, etc. Sérgio Magalhães, O Globo

“Joaquim viveu sempre cercado do carinho de muitas pessoas. O arquiteto Jaime Lerner cunhou esta frase premonitória:
A propósito, era um homem que sabia agradar às mulheres.” “O carro é o cigarro do futuro.” Quem poderia imaginar a
reversão cultural que se deu no consumo do tabaco?
- Operadores discursivos não explicitados: se o
texto for construído sem marcadores de sequenciação, o leitor Talvez o automóvel não seja descartável tão facilmente. Este
deverá inferir, a partir da ordem dos enunciados, os operadores jornal, em uma série de reportagens, nestes dias, mostrou o
discursivos não explicitados na superfície textual. Nesses casos, privilégio que os governos dão ao uso do carro e o desprezo ao
os lugares dos diferentes conectores estarão indicados, na transporte coletivo. Surpreendentemente, houve entrevistado
escrita, pelos sinais de pontuação: ponto-final, vírgula, ponto- que opinou favoravelmente, valorizando Los Angeles – um caso
e-vírgula, dois-pontos. típico de cidade rodoviária e dispersa.
Ainda nestes dias, a ONU reafirmou o compromisso desta
“A reforma política é indispensável. Sem a existência da geração com o futuro da humanidade e contra o aquecimento
fidelidade partidária, cada parlamentar vota segundo seus global – para o qual a emissão de CO2 do rodoviarismo é
interesses e não de acordo com um programa partidário. Assim, agente básico. (A USP acaba de divulgar estudo advertindo que
não há bases governamentais sólidas.” a poluição em São Paulo mata o dobro do que o trânsito.)
O transporte também esteve no centro dos protestos de
Esse texto contém três períodos. O segundo indica a causa junho de 2013. Lembremos: ele está interrelacionado com a
de a reforma política ser indispensável. Portanto o ponto-final moradia, o emprego, o lazer. Como se vê, não faltam razões
do primeiro período está no lugar de um porque. para o debate do tema.

18
LÍNGUA PORTUGUESA
“Como se vê, não faltam razões para o debate do tema.” Assinale a opção em que se indica, INCORRETAMENTE, o
referente do termo em destaque.
Substituindo o termo sublinhado por uma oração (A) “quase US$ 1 bilhão de seu orçamento bianual” (5º§)
desenvolvida, a forma correta e adequada seria: – organização
(A) para que se debatesse o tema; (B) “A agência passou a dar mais ênfase” (6º§) – OMS
(B) para se debater o tema; (C) “Pesa contra o órgão da ONU”(7º§) – OMS
(C) para que se debata o tema; (D) “Seus esforços iniciais foram limitados” (7º§) –
(D) para debater-se o tema; gravidade da situação
(E) para que o tema fosse debatido. (E) “A comunidade tem diante de si” (10º§) – comunidade
internacional
02. “A USP acaba de divulgar estudo advertindo que a Respostas
poluição em São Paulo mata o dobro do que o trânsito”. 01. (C) / 02. (C) / 03. (D)
A oração em forma desenvolvida que substitui correta e
adequadamente o gerúndio “advertindo” é:
(A) com a advertência de;
(B) quando adverte;
(C) em que adverte;
(D) no qual advertia; Termos da oração. Processos de
(E) para advertir. coordenação e subordinação.
03.
Texto III - Corrida contra o ebola
Análise Sintática
Já faz seis meses que o atual surto de ebola na África
Ocidental despertou a atenção da comunidade internacional, A Análise Sintática examina a estrutura do período,
mas nada sugere que as medidas até agora adotadas para divide e classifica as orações que o constituem e reconhece a
refrear o avanço da doença tenham sido eficazes.
função sintática dos termos de cada oração.
Ao contrário, quase metade das cerca de 4.000
contaminações registradas neste ano ocorreram nas últimas
três semanas, e as mais de 2.000 mortes atestam a força da Oração: é todo enunciado linguístico dotado de sentido,
enfermidade. A escalada levou o diretor do CDC (Centro de porém há, necessariamente, a presença do verbo. A oração
Controle e Prevenção de Doenças) dos EUA, Tom Frieden, a encerra uma frase (ou segmento de frase), várias frases ou
afirmar que a epidemia está fora de controle. um período, completando um pensamento e concluindo o
O vírus encontrou ambiente propício para se propagar. enunciado através de ponto final, interrogação, exclamação e,
De um lado, as condições sanitárias e econômicas dos países em alguns casos, através de reticências.
afetados são as piores possíveis. De outro, a Organização Em toda oração há um verbo ou locução verbal (às vezes
Mundial da Saúde foi incapaz de mobilizar com celeridade elípticos). Não têm estrutura sintática, portanto não são
um contingente expressivo de profissionais para atuar nessas
orações, não podem ser analisadas sintaticamente frases como:
localidades afetadas.
Verdade que uma parcela das debilidades da OMS se explica Socorro!
por problemas financeiros. Só 20% dos recursos da entidade Com licença!
vêm de contribuições compulsórias dos países-membros – o Que rapaz impertinente!
restante é formado por doações voluntárias.
A crise econômica mundial se fez sentir também nessa área, Na oração as palavras estão relacionadas entre si, como
e a organização perdeu quase US$ 1 bilhão de seu orçamento partes de um conjunto harmônico: elas formam os termos
bianual, hoje de quase US$ 4 bilhões. Para comparação, o CDC ou as unidades sintáticas da oração. Cada termo da oração
dos EUA contou, somente no ano de 2013, com cerca de US$ desempenha uma função sintática. Geralmente apresentam dois
6 bilhões. grupos de palavras: um grupo sobre o qual se declara alguma
Os cortes obrigaram a OMS a fazer escolhas difíceis. A agência
coisa (o sujeito), e um grupo que apresenta uma declaração
passou a dar mais ênfase à luta contra enfermidades globais
crônicas, como doenças coronárias e diabetes. O departamento (o predicado), e, excepcionalmente, só o predicado. Exemplo:
de respostas a epidemias e pandemias foi dissolvido e integrado
a outros. Muitos profissionais experimentados deixaram seus A menina banhou-se na cachoeira.
cargos. A menina – sujeito
Pesa contra o órgão da ONU, de todo modo, a demora banhou-se na cachoeira – predicado
para reconhecer a gravidade da situação. Seus esforços iniciais
foram limitados e mal liderados. O sujeito é o termo da frase que concorda com o verbo em
O surto agora atingiu proporções tais que já não é mais número e pessoa. É normalmente o «ser de quem se declara
possível enfrentá-lo de Genebra, cidade suíça sede da OMS. algo”, “o tema do que se vai comunicar”.
Tornou-se crucial estabelecer um comando central na África
O predicado é a parte da oração que contém “a informação
Ocidental, com representantes dos países afetados.
Espera-se também maior comprometimento das potências nova para o ouvinte”. Normalmente, ele se refere ao sujeito,
mundiais, sobretudo Estados Unidos, Inglaterra e França, constituindo a declaração do que se atribui ao sujeito.
que possuem antigos laços com Libéria, Serra Leoa e Guiné,
respectivamente. Núcleo de um termo é a palavra principal (geralmente
A comunidade internacional tem diante de si um desafio um substantivo, pronome ou verbo), que encerra a essência
enorme, mas é ainda maior a necessidade de agir com rapidez. de sua significação. Nos exemplos seguintes as palavras
Nessa batalha global contra o ebola, todo tempo perdido conta “amigo” e “revestiu” são o núcleo do sujeito e do predicado,
a favor da doença. respectivamente
(Disponível em: http://www1.folha.uol.com.br/
Os termos da oração da língua portuguesa são classificados
opiniao/2014/09/1512104-editorial-corrida-contra-o-ebola.shtml:
Acesso em: 08/09/2014)
em três grandes níveis:

19
LÍNGUA PORTUGUESA
- Termos Essenciais da Oração: Sujeito e Predicado. O núcleo (isto é, a palavra base) do sujeito é, pois, um
- Termos Integrantes da Oração: Complemento substantivo ou pronome. Em torno do núcleo podem aparecer
Nominal e Complementos Verbais (Objeto Direto, Objeto palavras secundárias (artigos, adjetivos, locuções adjetivas,
indireto e Agente da Passiva). etc.) Exemplo:
- Termos Acessórios da Oração: Adjunto Adnominal, “Todos os ligeiros rumores da mata tinham uma voz para
Adjunto Adverbial, Aposto e Vocativo. a selvagem filha do sertão.” (José de Alencar)

Termos Essenciais da Oração: São dois os termos O sujeito pode ser:


essenciais (ou fundamentais) da oração: sujeito e predicado. Simples: quando tem um só núcleo: As rosas tem
espinhos; “Um bando de galinhas-d’angola atravessa a rua em
Sujeito: é equivocado dizer que o sujeito é aquele que fila indiana.”
pratica uma ação ou é aquele (ou aquilo) do qual se diz alguma Composto: quando tem mais de um núcleo: “O burro e o
coisa. Ao fazer tal afirmação estamos considerando o aspecto cavalo nadavam ao lado da canoa.”
semântico do sujeito (agente de uma ação) ou o seu aspecto Expresso: quando está explícito, enunciado: Eu viajarei
estilístico (o tópico da sentença). Já que o sujeito é depreendido amanhã.
de uma análise sintática, vamos restringir a definição apenas Oculto (ou elíptico): quando está implícito, isto é,
ao seu papel sintático na sentença: aquele que estabelece quando não está expresso, mas se deduz do contexto: Viajarei
concordância com o núcleo do predicado. Quando se trata amanhã. (sujeito: eu, que se deduz da desinência do verbo);
de predicado verbal, o núcleo é sempre um verbo; sendo um “Um soldado saltou para a calçada e aproximou-se.” (o sujeito,
predicado nominal, o núcleo é sempre um nome. Então têm por soldado, está expresso na primeira oração e elíptico na segunda:
características básicas: e (ele) aproximou-se.); Crianças, guardem os brinquedos.
- estabelecer concordância com o núcleo do predicado; (sujeito: vocês)
- apresentar-se como elemento determinante em relação ao Agente: se faz a ação expressa pelo verbo da voz ativa: O
predicado; Nilo fertiliza o Egito.
- constituir-se de um substantivo, ou pronome substantivo Paciente: quando sofre ou recebe os efeitos da ação
ou, ainda, qualquer palavra substantivada. expressa pelo verbo passivo: O criminoso é atormentado
pelo remorso; Muitos sertanistas foram mortos pelos índios;
Exemplo: Construíram-se açudes. (= Açudes foram construídos.)
A padaria está fechada hoje. Agente e Paciente: quando o sujeito realiza a ação
está fechada hoje: predicado nominal expressa por um verbo reflexivo e ele mesmo sofre ou recebe
fechada: nome adjetivo = núcleo do predicado os efeitos dessa ação: O operário feriu-se durante o trabalho;
a padaria: sujeito Regina trancou-se no quarto.
padaria: núcleo do sujeito - nome feminino singular Indeterminado: quando não se indica o agente da ação
verbal: Atropelaram uma senhora na esquina. (Quem atropelou
No interior de uma sentença, o sujeito é o termo determinante, a senhora? Não se diz, não se sabe quem a atropelou.); Come-
ao passo que o predicado é o termo determinado. Essa posição se bem naquele restaurante.
de determinante do sujeito em relação ao predicado adquire
sentido com o fato de ser possível, na língua portuguesa, uma Sem Sujeito: constituem a enunciação pura e absoluta de
sentença sem sujeito, mas nunca uma sentença sem predicado. um fato, através do predicado; o conteúdo verbal não é atribuído
Exemplo: a nenhum ser. São construídas com os verbos impessoais, na 3ª
pessoa do singular: Havia ratos no porão; Choveu durante o
As formigas invadiram minha casa. jogo.
as formigas: sujeito = termo determinante Observação: São verbos impessoais: Haver (nos sentidos
invadiram minha casa: predicado = termo determinado de existir, acontecer, realizar-se, decorrer), Fazer, passar, ser
e estar, com referência ao tempo e Chover, ventar, nevar, gear,
O sujeito sempre se manifesta em termos de sintagma relampejar, amanhecer, anoitecer e outros que exprimem
nominal, isto é, seu núcleo é sempre um nome. Quando esse fenômenos meteorológicos.
nome se refere a objetos das primeira e segunda pessoas,
o sujeito é representado por um pronome pessoal do caso Predicado: assim como o sujeito, o predicado é um
reto (eu, tu, ele, etc.). Se o sujeito se refere a um objeto da segmento extraído da estrutura interna das orações ou das
terceira pessoa, sua representação pode ser feita através de frases, sendo, por isso, fruto de uma análise sintática. Nesse
um substantivo, de um pronome substantivo ou de qualquer sentido, o predicado é sintaticamente o segmento linguístico
conjunto de palavras, cujo núcleo funcione, na sentença, como que estabelece concordância com outro termo essencial
um substantivo. da oração, o sujeito, sendo este o termo determinante
(ou subordinado) e o predicado o termo determinado (ou
Além dessas formas, o sujeito também pode se constituir principal). Não se trata, portanto, de definir o predicado como
de uma oração inteira. Nesse caso, a oração recebe o nome de “aquilo que se diz do sujeito” como fazem certas gramáticas
oração substantiva subjetiva: da língua portuguesa, mas sim estabelecer a importância do
É difícil optar por esse ou aquele doce... fenômeno da concordância entre esses dois termos essenciais
É difícil: oração principal da oração. Então têm por características básicas: apresentar-se
optar por esse ou aquele doce: oração substantiva subjetiva como elemento determinado em relação ao sujeito; apontar um
atributo ou acrescentar nova informação ao sujeito. Exemplos:
O sujeito é constituído por um substantivo ou pronome, ou Carolina conhece os índios da Amazônia.
por uma palavra ou expressão substantivada. Exemplos: sujeito: Carolina = termo determinante
O sino era grande. predicado: conhece os índios da Amazônia = termo
Ela tem uma educação fina. determinado

20
LÍNGUA PORTUGUESA
Todos nós fazemos parte da quadrilha de São João. Outros verbos, pelo contrário, para integrarem o predicado
sujeito: todos nós = termo determinante necessitam de outros termos: são os verbos de predicação
predicado: fazemos parte da quadrilha de São João = termo incompleta, denominados transitivos. Exemplos:
determinado
João puxou a rede.
Nesses exemplos podemos observar que a concordância é “Não invejo os ricos, nem aspiro à riqueza.” (Oto Lara
estabelecida entre algumas poucas palavras dos dois termos Resende)
essenciais. No primeiro exemplo, entre “Carolina” e “conhece”;
no segundo exemplo, entre “nós” e “fazemos”. Isso se dá porque Observe que, sem os seus complementos, os verbos puxou,
a concordância é centrada nas palavras que são núcleos, isto invejo, aspiro, etc., não transmitiriam informações completas:
é, que são responsáveis pela principal informação naquele puxou o quê? Não invejo a quem? Não aspiro a quê?
segmento. No predicado o núcleo pode ser de dois tipos: um Os verbos de predicação completa denominam-se
nome, quase sempre um atributo que se refere ao sujeito da intransitivos e os de predicação incompleta, transitivos.
oração, ou um verbo (ou locução verbal). No primeiro caso, Os verbos transitivos subdividem-se em: transitivos diretos,
temos um predicado nominal (seu núcleo significativo é um transitivos indiretos e transitivos diretos e indiretos
nome, substantivo, adjetivo, pronome, ligado ao sujeito por um (bitransitivos).
verbo de ligação) e, no segundo, um predicado verbal (seu Além dos verbos transitivos e intransitivos, quem encerram
núcleo é um verbo, seguido, ou não, de complemento(s) ou uma noção definida, um conteúdo significativo, existem os
termos acessórios). Quando, num mesmo segmento o nome e o verbos de ligação que entram na formação do predicado
verbo são de igual importância, ambos constituem o núcleo do nominal, relacionando o predicativo com o sujeito.
predicado e resultam no tipo de predicado verbo-nominal Quanto à predicação classificam-se, pois os verbos em:
(tem dois núcleos significativos: um verbo e um nome).
Exemplos: Intransitivos: são os que não precisam de complemento,
Minha empregada é desastrada. pois têm sentido completo.
predicado: é desastrada “Três contos bastavam, insistiu ele.” (Machado de Assis)
núcleo do predicado: desastrada = atributo do sujeito “Os guerreiros Tabajaras dormem.” (José de Alencar)
tipo de predicado: nominal
Observações: Os verbos intransitivos podem vir
O núcleo do predicado nominal chama-se predicativo acompanhados de um adjunto adverbial e mesmo de um
do sujeito, porque atribui ao sujeito uma qualidade ou predicativo (qualidade, características): Fui cedo; Passeamos
característica. Os verbos de ligação (ser, estar, parecer, etc.) pela cidade; Cheguei atrasado; Entrei em casa
funcionam como um elo entre o sujeito e o predicado. aborrecido. As orações formadas com verbos intransitivos
não podem “transitar” (= passar) para a voz passiva. Verbos
A empreiteira demoliu nosso antigo prédio. intransitivos passam, ocasionalmente, a transitivos quando
predicado: demoliu nosso antigo prédio construídos com o objeto direto ou indireto.
núcleo do predicado: demoliu = nova informação sobre o - “Inutilmente a minha alma o chora!” (Cabral do
sujeito Nascimento)
tipo de predicado: verbal - “Depois me deitei e dormi um sono pesado.” (Luís
Jardim)
Nos predicados verbais e verbo-nominais o verbo é - “Morrerás morte vil da mão de um forte.” (Gonçalves
responsável também por definir os tipos de elementos que Dias)
aparecerão no segmento. Em alguns casos o verbo sozinho - “Inútil tentativa de viajar o passado, penetrar no
basta para compor o predicado (verbo intransitivo). Em outros mundo que já morreu...” (Ciro dos Anjos)
casos é necessário um complemento que, juntamente com
o verbo, constituem a nova informação sobre o sujeito. De Alguns verbos essencialmente intransitivos: anoitecer,
qualquer forma, esses complementos do verbo não interferem crescer, brilhar, ir, agir, sair, nascer, latir, rir, tremer, brincar,
na tipologia do predicado. chegar, vir, mentir, suar, adoecer, etc.
Entretanto, é muito comum a elipse (ou omissão) do verbo,
quando este puder ser facilmente subentendido, em geral por Transitivos Diretos: são os que pedem um objeto direto,
estar expresso ou implícito na oração anterior. Exemplos: isto é, um complemento sem preposição. Pertencem a esse
grupo: julgar, chamar, nomear, eleger, proclamar, designar,
“A fraqueza de Pilatos é enorme, a ferocidade dos algozes considerar, declarar, adotar, ter, fazer, etc. Exemplos:
inexcedível.” (Machado de Assis) (Está subentendido o verbo é Comprei um terreno e construí a casa.
depois de algozes) “Trabalho honesto produz riqueza honrada.” (Marquês de
“Mas o sal está no Norte, o peixe, no Sul” (Paulo Moreira da Maricá)
Silva) (Subentende-se o verbo está depois de peixe) “Então, solenemente Maria acendia a lâmpada de sábado.”
(Guedes de Amorim)
Chama-se predicação verbal o modo pelo qual o verbo
forma o predicado. Dentre os verbos transitivos diretos merecem destaque os
Há verbos que, por natureza, tem sentido completo, que formam o predicado verbo nominal e se constrói com o
podendo, por si mesmos, constituir o predicado: os verbos complemento acompanhado de predicativo. Exemplos:
de predicação completa são denominados intransitivos. Consideramos o caso extraordinário.
Exemplo: Inês trazia as mãos sempre limpas.

As flores murcharam. Observações: Os verbos transitivos diretos, em geral,


Os animais correm. podem ser usados também na voz passiva; Outra característica

21
LÍNGUA PORTUGUESA
desses verbos é a de poderem receber como objeto direto, os vez uma princesa.; Eu não estava em casa.; Fiquei à sombra.;
pronomes o, a, os, as: convido-o, encontro-os, incomodo-a, Anda com dificuldades.; Parece que vai chover.
conheço-as; Os verbos transitivos diretos podem ser construídos
acidentalmente com preposição, a qual lhes acrescenta novo Os verbos, relativamente à predicação, não têm classificação
matiz semântico: arrancar da espada; puxar da faca; pegar de fixa, imutável. Conforme a regência e o sentido que apresentam
uma ferramenta; tomar do lápis; cumprir com o dever; Alguns na frase, podem pertencer ora a um grupo, ora a outro.
verbos transitivos diretos: abençoar, achar, colher, avisar,
abraçar, comprar, castigar, contrariar, convidar, desculpar, Predicativo: Há o predicativo do sujeito e o predicativo
dizer, estimar, elogiar, entristecer, encontrar, ferir, imitar, levar, do objeto.
perseguir, prejudicar, receber, saldar, socorrer, ter, unir, ver, etc.
Predicativo do Sujeito: é o termo que exprime um
Transitivos Indiretos: são os que reclamam um atributo, um estado ou modo de ser do sujeito, ao qual se prende
complemento regido de preposição, chamado objeto indireto. por um verbo de ligação, no predicado nominal. Exemplos:
Exemplos: A bandeira é o símbolo da Pátria.
“Ninguém perdoa ao quarentão que se apaixona por uma A mesa era de mármore.
adolescente.” (Ciro dos Anjos)
“Populares assistiam à cena aparentemente apáticos e Além desse tipo de predicativo, outro existe que entra na
neutros.” (Érico Veríssimo) constituição do predicado verbo-nominal. Exemplos:
“Lúcio não atinava com essa mudança instantânea.” (José O trem chegou atrasado. (=O trem chegou e estava
Américo) atrasado.)
“Do que eu mais gostava era do tempo do retiro espiritual.” O menino abriu a porta ansioso.
(José Geraldo Vieira)
Observações: O predicativo subjetivo às vezes está
Observações: Entre os verbos transitivos indiretos importa preposicionado; Pode o predicativo preceder o sujeito e até
distinguir os que se constroem com os pronomes objetivos mesmo ao verbo: São horríveis essas coisas!; Que linda
lhe, lhes. Em geral são verbos que exigem a preposição a: estava Amélia!; Completamente feliz ninguém é.; Raros são
agradar-lhe, agradeço-lhe, apraz-lhe, bate-lhe, desagrada-lhe, os verdadeiros líderes.; Quem são esses homens?; Lentos
desobedecem-lhe, etc. Entre os verbos transitivos indiretos e tristes, os retirantes iam passando.; Novo ainda, eu não
importa distinguir os que não admitem para objeto indireto entendia certas coisas.; Onde está a criança que fui?
as formas oblíquas lhe, lhes, construindo-se com os pronomes
retos precedidos de preposição: aludir a ele, anuir a ele, assistir Predicativo do Objeto: é o termo que se refere ao objeto
a ela, atentar nele, depender dele, investir contra ele, não ligar de um verbo transitivo. Exemplos:
para ele, etc. O juiz declarou o réu inocente.
Em princípio, verbos transitivos indiretos não comportam a O povo elegeu-o deputado.
forma passiva. Excetuam-se pagar, perdoar, obedecer, e pouco
mais, usados também como transitivos diretos: João paga Observações: O predicativo objetivo, como vemos dos
(perdoa, obedece) o médico. O médico é pago (perdoado, exemplos acima, às vezes vem regido de preposição. Esta, em
obedecido) por João. Há verbos transitivos indiretos, como certos casos, é facultativa; O predicativo objetivo geralmente
atirar, investir, contentar-se, etc., que admitem mais de uma se refere ao objeto direto. Excepcionalmente, pode referir-se
preposição, sem mudança de sentido. Outros mudam de sentido ao objeto indireto do verbo chamar. Chamavam-lhe poeta;
com a troca da preposição, como nestes exemplos: Trate de Podemos antepor o predicativo a seu objeto: O advogado
sua vida. (tratar=cuidar). É desagradável tratar com gente considerava indiscutíveis os direitos da herdeira.; Julgo
grosseira. (tratar=lidar). Verbos como aspirar, assistir, dispor, inoportuna essa viagem.; “E até embriagado o vi muitas
servir, etc., variam de significação conforme sejam usados como vezes.”; “Tinha estendida a seus pés uma planta rústica da
transitivos diretos ou indiretos. cidade.”; “Sentia ainda muito abertos os ferimentos que
aquele choque com o mundo me causara.”
Transitivos Diretos e Indiretos: são os que se usam com
dois objetos: um direto, outro indireto, concomitantemente. Termos Integrantes da Oração
Exemplos:
No inverso, Dona Cléia dava roupas aos pobres. Chamam-se termos integrantes da oração os que completam
Ceda o lugar aos mais velhos. a significação transitiva dos verbos e nomes. Integram (inteiram,
completam) o sentido da oração, sendo por isso indispensável à
De Ligação: Os que ligam ao sujeito uma palavra ou compreensão do enunciado. São os seguintes:
expressão chamada predicativo. Esses verbos entram na - Complemento Verbais (Objeto Direto e Objeto
formação do predicado nominal. Exemplos: Indireto);
A Terra é móvel. - Complemento Nominal;
A água está fria. - Agente da Passiva.
A Lua parecia um disco.
Objeto Direto: é o complemento dos verbos de predicação
Observações: Os verbos de ligação não servem apenas de incompleta, não regido, normalmente, de preposição. Exemplos:
anexo, mas exprimem ainda os diversos aspectos sob os quais As plantas purificaram o ar.
se considera a qualidade atribuída ao sujeito. O verbo ser, por “Nunca mais ele arpoara um peixe-boi.” (Ferreira Castro)
exemplo, traduz aspecto permanente e o verbo estar, aspecto Procurei o livro, mas não o encontrei.
transitório: Ele é doente. (aspecto permanente); Ele está
doente. (aspecto transitório). Muito desses verbos passam à O objeto direto tem as seguintes características:
categoria dos intransitivos em frases como: Era (=existia) uma - Completa a significação dos verbos transitivos diretos;

22
LÍNGUA PORTUGUESA
- Normalmente, não vem regido de preposição; confrade conheço desde os seus mais tenros anos”.
- Traduz o ser sobre o qual recai a ação expressa por um - Sendo objeto direto o numeral ambos(as): “O aguaceiro
verbo ativo: Caim matou Abel. caiu, molhou a ambos.”; “Se eu previsse que os matava a
- Torna-se sujeito da oração na voz passiva: Abel foi morto ambos...”.
por Caim. - Com certos pronomes indefinidos, sobretudo referentes a
pessoas: Se todos são teus irmãos, por que amas a uns e odeias
O objeto direto pode ser constituído: a outros?; Aumente a sua felicidade, tornando felizes também
- Por um substantivo ou expressão substantivada: O lavrador aos outros.; A quantos a vida ilude!.
cultiva a terra.; Unimos o útil ao agradável. - Em certas construções enfáticas, como puxar (ou arrancar)
- Pelos pronomes oblíquos o, a, os, as, me, te, se, nos, vos: da espada, pegar da pena, cumprir com o dever, atirar com
Espero-o na estação.; Estimo-os muito.; Sílvia olhou-se ao os livros sobre a mesa, etc.: “Arrancam das espadas de aço
espelho.; Não me convidas?; Ela nos chama.; Avisamo-lo a fino...”; “Chegou a costureira, pegou do pano, pegou da
tempo.; Procuram-na em toda parte.; Meu Deus, eu vos amo.; agulha, pegou da linha, enfiou a linha na agulha e entrou a
“Marchei resolutamente para a maluca e intimei-a a ficar coser.”; “Imagina-se a consternação de Itaguaí, quando soube
quieta.”; “Vós haveis de crescer, perder-vos-ei de vista.” do caso.”
- Por qualquer pronome substantivo: Não vi ninguém
na loja.; A árvore que plantei floresceu. (que: objeto direto Observações: Nos quatro primeiros casos estudados a
de plantei); Onde foi que você achou isso? Quando vira as preposição é de rigor, nos cinco outros, facultativa. A substituição
folhas do livro, ela o faz com cuidado.; “Que teria o homem do objeto direto preposicionado pelo pronome oblíquo átono,
percebido nos meus escritos?” quando possível, se faz com as formas o(s), a(s) e não lhe,
lhes: amar a Deus (amá-lo); convencer ao amigo (convencê-
Frequentemente transitivam-se verbos intransitivos, dando- lo); O objeto direto preposicionado, é obvio, só ocorre com
se-lhes por objeto direto uma palavra cognata ou da mesma verbo transitivo direto; Podem resumir-se em três as razões
esfera semântica: ou finalidades do emprego do objeto direto preposicionado: a
“Viveu José Joaquim Alves vida tranquila e patriarcal.” clareza da frase; a harmonia da frase; a ênfase ou a força da
(Vivaldo Coaraci) expressão.
“Pela primeira vez chorou o choro da tristeza.” (Aníbal
Machado) Objeto Direto Pleonástico: Quando queremos dar
“Nenhum de nós pelejou a batalha de Salamina.” destaque ou ênfase à idéia contida no objeto direto, colocamo-
(Machado de Assis) lo no início da frase e depois o repetimos ou reforçamos por
Em tais construções é de rigor que o objeto venha meio do pronome oblíquo. A esse objeto repetido sob forma
acompanhado de um adjunto. pronominal chama-se pleonástico, enfático ou redundante.
Exemplos:
Objeto Direto Preposicionado: Há casos em que o O dinheiro, Jaime o trazia escondido nas mangas da
objeto direto, isto é, o complemento de verbos transitivos camisa.
diretos, vem precedido de preposição, geralmente a preposição O bem, muitos o louvam, mas poucos o seguem.
a. Isto ocorre principalmente: “Seus cavalos, ela os montava em pêlo.” (Jorge Amado)
- Quando o objeto direto é um pronome pessoal tônico:
Deste modo, prejudicas a ti e a ela.; “Mas dona Carolina Objeto Indireto: É o complemento verbal regido de
amava mais a ele do que aos outros filhos.”; “Pareceu-me que preposição necessária e sem valor circunstancial. Representa,
Roberto hostilizava antes a mim do que à ideia.”; “Ricardina ordinariamente, o ser a que se destina ou se refere à ação verbal:
lastimava o seu amigo como a si própria.”; “Amava-a tanto “Nunca desobedeci a meu pai”. O objeto indireto completa a
como a nós”. significação dos verbos:
- Quando o objeto é o pronome relativo quem: “Pedro
Severiano tinha um filho a quem idolatrava.”; “Abraçou - Transitivos Indiretos: Assisti ao jogo; Assistimos à
a todos; deu um beijo em Adelaide, a quem felicitou pelo missa e à festa; Aludiu ao fato; Aspiro a uma vida calma.
desenvolvimento das suas graças.”; “Agora sabia que podia
manobrar com ele, com aquele homem a quem na realidade - Transitivos Diretos e Indiretos (na voz ativa
também temia, como todos ali”. ou passiva): Dou graças a Deus; Ceda o lugar aos mais
- Quando precisamos assegurar a clareza da frase, evitando velhos; Dedicou sua vida aos doentes e aos pobres; Disse-
que o objeto direto seja tomado como sujeito, impedindo lhe a verdade. (Disse a verdade ao moço.)
construções ambíguas: Convence, enfim, ao pai o filho amado.;
“Vence o mal ao remédio.”; “Tratava-me sem cerimônia, como O objeto indireto pode ainda acompanhar verbos de outras
a um irmão.”; A qual delas iria homenagear o cavaleiro? categorias, os quais, no caso, são considerados acidentalmente
- Em expressões de reciprocidade, para garantir a clareza e a transitivos indiretos: A bom entendedor meia palavra basta;
eufonia da frase: “Os tigres despedaçam-se uns aos outros.”; Sobram-lhe qualidades e recursos. (lhe=a ele); Isto não lhe
“As companheiras convidavam-se umas às outras.”; “Era o convém; A proposta pareceu-lhe aceitável.
abraço de duas criaturas que só tinham uma à outra”.
- Com nomes próprios ou comuns, referentes a pessoas, Observações: Há verbos que podem construir-se com dois
principalmente na expressão dos sentimentos ou por amor da objetos indiretos, regidos de preposições diferentes: Rogue a
eufonia da frase: Judas traiu a Cristo.; Amemos a Deus sobre Deus por nós.; Ela queixou-se de mim a seu pai.; Pedirei
todas as coisas. “Provavelmente, enganavam é a Pedro.”; “O para ti a meu senhor um rico presente; Não confundir o
estrangeiro foi quem ofendeu a Tupã”. objeto direto com o complemento nominal nem com o adjunto
- Em construções enfáticas, nas quais antecipamos o objeto adverbial; Em frases como “Para mim tudo eram alegrias”,
direto para dar-lhe realce: A você é que não enganam!; Ao “Para ele nada é impossível”, os pronomes em destaque podem
médico, confessor e letrado nunca enganes.; “A este ser considerados adjuntos adverbiais.

23
LÍNGUA PORTUGUESA
O objeto indireto é sempre regido de preposição, expressa Observações: Frase de forma passiva analítica sem
ou implícita. A preposição está implícita nos pronomes objetivos complemento agente expresso, ao passar para a ativa, terá
indiretos (átonos) me, te, se, lhe, nos, vos, lhes. Exemplos: sujeito indeterminado e o verbo na 3ª pessoa do plural: Ele
Obedece-me. (=Obedece a mim.); Isto te pertence. (=Isto foi expulso da cidade. (Expulsaram-no da cidade.);
pertence a ti.); Rogo-lhe que fique. (=Rogo a você...); Peço- As florestas são devastadas. (Devastam as florestas.);
vos isto. (=Peço isto a vós.). Nos demais casos a preposição Na passiva pronominal não se declara o agente: Nas ruas
é expressa, como característica do objeto indireto: Recorro a assobiavam-se as canções dele pelos pedestres. (errado);
Deus.; Dê isto a (ou para) ele.; Contenta-se com pouco.; Ele Nas ruas eram assobiadas as canções dele pelos pedestres.
só pensa em si.; Esperei por ti.; Falou contra nós.; Conto (certo); Assobiavam-se as canções dele nas ruas. (certo)
com você.; Não preciso disto.; O filme a que assisti agradou
ao público.; Assisti ao desenrolar da luta.; A coisa de Termos Acessórios da Oração
que mais gosto é pescar.; A pessoa a quem me refiro você a
conhece.; Os obstáculos contra os quais luto são muitos.; As Termos acessórios são os que desempenham na oração
pessoas com quem conto são poucas. uma função secundária, qual seja a de caracterizar um ser,
determinar os substantivos, exprimir alguma circunstância.
Como atestam os exemplos acima, o objeto indireto é São três os termos acessórios da oração: adjunto adnominal,
representado pelos substantivos (ou expressões substantivas) adjunto adverbial e aposto.
ou pelos pronomes. As preposições que o ligam ao verbo são: a,
com, contra, de, em, para e por. Adjunto adnominal: É o termo que caracteriza ou
determina os substantivos. Exemplo: Meu irmão veste roupas
Objeto Indireto Pleonástico: à semelhança do objeto vistosas. (Meu determina o substantivo irmão: é um adjunto
direto, o objeto indireto pode vir repetido ou reforçado, por adnominal – vistosas caracteriza o substantivo roupas: é
ênfase. Exemplos: “A mim o que me deu foi pena.”; “Que me também adjunto adnominal).
importa a mim o destino de uma mulher tísica...? “E, aos O adjunto adnominal pode ser expresso: Pelos adjetivos:
brigões, incapazes de se moverem, basta-lhes xingarem-se à água fresca, terras férteis, animal feroz; Pelos artigos: o
distância.” mundo, as ruas, um rapaz; Pelos pronomes adjetivos: nosso
tio, este lugar, pouco sal, muitas rãs, país cuja história
Complemento Nominal: é o termo complementar conheço, que rua?; Pelos numerais: dois pés, quinto ano,
reclamado pela significação transitiva, incompleta, de certos capítulo sexto; Pelas locuções ou expressões adjetivas que
substantivos, adjetivos e advérbios. Vem sempre regido de exprimem qualidade, posse, origem, fim ou outra especificação:
preposição. Exemplos: A defesa da pátria; Assistência às - presente de rei (=régio): qualidade
aulas; “O ódio ao mal é amor do bem, e a ira contra o mal, - água da fonte, filho de fazendeiros: origem
entusiasmo divino.”; “Ah, não fosse ele surdo à minha voz!” - fio de aço, casa de madeira: matéria
- casa de ensino, aulas de inglês: fim, especialidade
Observações: O complemento nominal representa o - criança com febre (=febril): característica
recebedor, o paciente, o alvo da declaração expressa por um
nome: amor a Deus, a condenação da violência, o medo de Observações: Não confundir o adjunto adnominal formado
assaltos, a remessa de cartas, útil ao homem, compositor por locução adjetiva com complemento nominal. Este
de músicas, etc. É regido pelas mesmas preposições usadas representa o alvo da ação expressa por um nome transitivo:
no objeto indireto. Difere deste apenas porque, em vez de a eleição do presidente, aviso de perigo, declaração de
complementar verbos, complementa nomes (substantivos, guerra, empréstimo de dinheiro, plantio de árvores,
adjetivos) e alguns advérbios em –mente. Os nomes que colheita de trigo, destruidor de matas, descoberta de
requerem complemento nominal correspondem, geralmente, a petróleo, amor ao próximo, etc. O adjunto adnominal
verbos de mesmo radical: amor ao próximo, amar o próximo; formado por locução adjetiva representa o agente da ação, ou
perdão das injúrias, perdoar as injúrias; obediente aos a origem, pertença, qualidade de alguém ou de alguma coisa:
pais, obedecer aos pais; regresso à pátria, regressar à o discurso do presidente, aviso de amigo, declaração do
pátria; etc. ministro, empréstimo do banco, a casa do fazendeiro,
folhas de árvores, farinha de trigo, beleza das matas,
Agente da Passiva: é o complemento de um verbo na voz cheiro de petróleo, amor de mãe.
passiva. Representa o ser que pratica a ação expressa pelo verbo
passivo. Vem regido comumente pela preposição por, e menos Adjunto adverbial: É o termo que exprime uma
frequentemente pela preposição de: Alfredo é estimado pelos circunstância (de tempo, lugar, modo, etc.) ou, em outras
colegas; A cidade estava cercada pelo exército romano; palavras, que modifica o sentido de um verbo, adjetivo ou
“Era conhecida de todo mundo a fama de suas riquezas.” advérbio. Exemplo: “Meninas numa tarde brincavam de
roda na praça”. O adjunto adverbial é expresso: Pelos
O agente da passiva pode ser expresso pelos substantivos advérbios: Cheguei cedo.; Ande devagar.; Maria é mais
ou pelos pronomes: alta.; Não durma ao volante.; Moramos aqui.; Ele fala bem,
As flores são umedecidas pelo orvalho. fala corretamente.; Volte bem depressa.; Talvez esteja
A carta foi cuidadosamente corrigida por mim. enganado.; Pelas locuções ou expressões adverbiais: Às vezes
viajava de trem.; Compreendo sem esforço.; Saí com
O agente da passiva corresponde ao sujeito da oração na meu pai.; Júlio reside em Niterói.; Errei por distração.;
voz ativa: Escureceu de repente.

A rainha era chamada pela multidão. (voz passiva) Observações: Pode ocorrer a elipse da preposição antes
A multidão aclamava a rainha. (voz ativa) de adjuntos adverbiais de tempo e modo: Aquela noite,
não dormi. (=Naquela noite...); Domingo que vem não

24
LÍNGUA PORTUGUESA
sairei. (=No domingo...); Ouvidos atentos, aproximei-me “Elesbão? Ó Elesbão! Venha ajudar-nos, por favor!” (Maria
da porta. (=De ouvidos atentos...); Os adjuntos adverbiais de Lourdes Teixeira)
classificam-se de acordo com as circunstâncias que exprimem: “A ordem, meus amigos, é a base do governo.” (Machado
adjunto adverbial de lugar, modo, tempo, intensidade, causa, de Assis)
companhia, meio, assunto, negação, etc; É importante saber “Correi, correi, ó lágrimas saudosas!” (fagundes Varela)
distinguir adjunto adverbial de adjunto adnominal, de objeto “Ei-lo, o teu defensor, ó Liberdade!” (Mendes Leal)
indireto e de complemento nominal: sair do mar (ad.adv.);
água do mar (adj.adn.); gosta do mar (obj.indir.); ter medo Observação: Profere-se o vocativo com entoação
do mar (compl.nom.). exclamativa. Na escrita é separado por vírgula(s). No exemplo
inicial, os pontos interrogativo e exclamativo indicam um
Aposto: É uma palavra ou expressão que explica ou chamado alto e prolongado. O vocativo se refere sempre à
esclarece, desenvolve ou resume outro termo da oração. 2ª pessoa do discurso, que pode ser uma pessoa, um animal,
Exemplos: uma coisa real ou entidade abstrata personificada. Podemos
D. Pedro II, imperador do Brasil, foi um monarca sábio. antepor-lhe uma interjeição de apelo (ó, olá, eh!):
“Nicanor, ascensorista, expôs-me seu caso de consciência.” O vocativo é um tempo à parte. Não pertence à estrutura
(Carlos Drummond de Andrade) da oração, por isso não se anexa ao sujeito nem ao predicado.

O núcleo do aposto é um substantivo ou um pronome Questões


substantivo:
Foram os dois, ele e ela. 01. O termo em destaque é adjunto adverbial de intensidade
Só não tenho um retrato: o de minha irmã. em:
(A) pode aprender e assimilar MUITA coisa
O aposto não pode ser formado por adjetivos. Nas frases (B) enfrentamos MUITAS novidades
seguintes, por exemplo, não há aposto, mas predicativo do (C) precisa de um parceiro com MUITO caráter
sujeito: (D) não gostam de mulheres MUITO inteligentes
Audaciosos, os dois surfistas atiraram-se às ondas. (E) assumimos MUITO conflito e confusão
As borboletas, leves e graciosas, esvoaçavam num balé
de cores. 02. Assinale a alternativa correta: “para todos os males, há
dois remédios: o tempo e o silêncio”, os termos grifados
Os apostos, em geral, destacam-se por pausas, indicadas, na são respectivamente:
escrita, por vírgulas, dois pontos ou travessões. Não havendo (A) sujeito – objeto direto;
pausa, não haverá vírgula, como nestes exemplos: (B) sujeito – aposto;
Minha irmã Beatriz; o escritor João Ribeiro; o romance (C) objeto direto – aposto;
Tóia; o rio Amazonas; a Rua Osvaldo Cruz; o Colégio (D) objeto direto – objeto direto;
Tiradentes, etc. (E) objeto direto – complemento nominal.
“Onde estariam os descendentes de Amaro vaqueiro?”
(Graciliano Ramos) 03. Assinale a alternativa em que o termo destacado é
O aposto pode preceder o termo a que se refere, o qual, às objeto indireto.
vezes, está elíptico. Exemplos: (A) “Quem faz um poema abre uma janela.” (Mário
Rapaz impulsivo, Mário não se conteve. Quintana)
Mensageira da idéia, a palavra é a mais bela expressão (B) “Toda gente que eu conheço e que fala comigo /
da alma humana. Nunca teve um ato ridículo / Nunca sofreu enxovalho (...)”
(Fernando Pessoa)
O aposto, às vezes, refere-se a toda uma oração. Exemplos: (C) “Quando Ismália enlouqueceu / Pôs-se na torre a sonhar
Nuvens escuras borravam os espaços silenciosos, sinal de / Viu uma lua no céu, / Viu uma lua no mar.” (Alphonsus de
tempestade iminente. Guimarães)
O espaço é incomensurável, fato que me deixa atônito. (D) “Mas, quando responderam a Nhô Augusto: ‘– É a
Simão era muito espirituoso, o que me levava a preferir sua jagunçada de seu Joãozinho Bem-Bem, que está descendo para
companhia. a Bahia.’ – ele, de alegre, não se pôde conter.” (Guimarães Rosa)

Um aposto pode referir-se a outro aposto: Respostas


“Serafim Gonçalves casou-se com Lígia Tavares, filha do 01. D / 02. C / 03. D
velho coronel Tavares, senhor de engenho.” (Ledo Ivo)
Período: Toda frase com uma ou mais orações constitui
O aposto pode vir precedido das expressões explicativas isto um período, que se encerra com ponto de exclamação, ponto
é, a saber, ou da preposição acidental como: de interrogação ou com reticências.
Dois países sul-americanos, isto é, a Bolívia e o O período é simples quando só traz uma oração, chamada
Paraguai, não são banhados pelo mar. absoluta; o período é composto quando traz mais de uma
oração.
O aposto que se refere a objeto indireto, complemento Existe uma maneira prática de saber quantas orações há
nominal ou adjunto adverbial vem precedido de preposição: num período: é contar os verbos ou locuções verbais. Num
O rei perdoou aos dois: ao fidalgo e ao criado. período haverá tantas orações quantos forem os verbos ou as
locuções verbais nele existentes. Exemplos:
Vocativo: (do latim vocare = chamar) é o termo (nome, Pegou fogo no prédio. (um verbo, uma oração)
título, apelido) usado para chamar ou interpelar a pessoa, o Quero que você aprenda. (dois verbos, duas orações)
animal ou a coisa personificada a que nos dirigimos: Está pegando fogo no prédio. (uma locução verbal, uma

25
LÍNGUA PORTUGUESA
oração) - Orações coordenadas sindéticas adversativas:
Deves estudar para poderes vencer na vida. (duas mas, porém, todavia, contudo, entretanto, no entanto.
locuções verbais, duas orações) Estudei bastante / mas não passei no teste.
OCA OCS Adversativa
Há três tipos de período composto: por coordenação, por
subordinação e por coordenação e subordinação ao mesmo Observe que a 2ª oração vem introduzida por uma conjunção
tempo (também chamada de misto). que expressa ideia de oposição à oração anterior, ou seja, por
uma conjunção coordenativa adversativa.
Período Composto por Coordenação – Orações
Coordenadas A espada vence, mas não convence.
“É dura a vida, mas aceitam-na.” (Cecília Meireles)
Considere, por exemplo, este período composto: Tens razão, contudo não te exaltes.
Passeamos pela praia, / brincamos, / recordamos os tempos Havia muito serviço, entretanto ninguém trabalhava.
de infância.
1ª oração: Passeamos pela praia - Orações coordenadas sindéticas conclusivas:
2ª oração: brincamos portanto, por isso, pois, logo.
3ª oração: recordamos os tempos de infância Ele me ajudou muito, / portanto merece minha gratidão.
OCA OCS Conclusiva
As três orações que compõem esse período têm sentido
próprio e não mantêm entre si nenhuma dependência sintática: Observe que a 2ª oração vem introduzida por uma conjunção
elas são independentes. Há entre elas, é claro, uma relação de que expressa ideia de conclusão de um fato enunciado na oração
sentido, mas, como já dissemos, uma não depende da outra anterior, ou seja, por uma conjunção coordenativa conclusiva.
sintaticamente.
As orações independentes de um período são chamadas Vives mentindo; logo, não mereces fé.
de orações coordenadas (OC), e o período formado só de Ele é teu pai: respeita-lhe, pois, a vontade.
orações coordenadas é chamado de período composto por Raimundo é homem são, portanto deve trabalhar.
coordenação.
As orações coordenadas são classificadas em assindéticas e - Orações coordenadas sindéticas alternativas:
sindéticas. ou,ou... ou, ora... ora, seja... seja, quer... quer.
Seja mais educado / ou retire-se da reunião!
- As orações coordenadas são assindéticas (OCA) quando OCA OCS Alternativa
não vêm introduzidas por conjunção. Exemplo:
Os torcedores gritaram, / sofreram, / vibraram. Observe que a 2ª oração vem introduzida por uma conjunção
OCA OCA OCA que estabelece uma relação de alternância ou escolha com
referência à oração anterior, ou seja, por uma conjunção
“Inclinei-me, apanhei o embrulho e segui.” (Machado de coordenativa alternativa.
Assis)
“A noite avança, há uma paz profunda na casa deserta.” Venha agora ou perderá a vez.
(Antônio Olavo Pereira) “Jacinta não vinha à sala, ou retirava-se logo.” (Machado
“O ferro mata apenas; o ouro infama, avilta, desonra.” de Assis)
(Coelho Neto) “Em aviação, tudo precisa ser bem feito ou custará preço
muito caro.” (Renato Inácio da Silva)
- As orações coordenadas são sindéticas (OCS) quando “A louca ora o acariciava, ora o rasgava
vêm introduzidas por conjunção coordenativa. Exemplo: freneticamente.” (Luís Jardim)
O homem saiu do carro / e entrou na casa.
OCA OCS - Orações coordenadas sindéticas explicativas: que,
porque, pois, porquanto.
As orações coordenadas sindéticas são classificadas de Vamos andar depressa / que estamos atrasados.
acordo com o sentido expresso pelas conjunções coordenativas OCA OCS Explicativa
que as introduzem. Pode ser:
Observe que a 2ª oração é introduzida por uma conjunção
- Orações coordenadas sindéticas aditivas: e, nem, que expressa ideia de explicação, de justificativa em relação
não só... mas também, não só... mas ainda. à oração anterior, ou seja, por uma conjunção coordenativa
Saí da escola / e fui à lanchonete. explicativa.
OCA OCS Aditiva
Leve-lhe uma lembrança, que ela aniversaria amanhã.
Observe que a 2ª oração vem introduzida por uma conjunção “A mim ninguém engana, que não nasci ontem.” (Érico
que expressa ideia de acréscimo ou adição com referência à Veríssimo)
oração anterior, ou seja, por uma conjunção coordenativa “Qualquer que seja a tua infância, conquista-a, que te
aditiva. abençoo.” (Fernando Sabino)
O cavalo estava cansado, pois arfava muito.
A doença vem a cavalo e volta a pé.
As pessoas não se mexiam nem falavam. Período Composto por Subordinação
“Não só findaram as queixas contra o alienista, mas até
nenhum ressentimento ficou dos atos que ele praticara.” Observe os termos destacados em cada uma destas orações:
(Machado de Assis) Vi uma cena triste. (adjunto adnominal)

26
LÍNGUA PORTUGUESA
Todos querem sua participação. (objeto direto) Cumpriremos nosso dever, ainda que (ou mesmo
Não pude sair por causa da chuva. (adjunto adverbial quando ou ainda quando ou mesmo que) todos nos
de causa) critiquem.
Por mais que gritasse, não me ouviram.
Veja, agora, como podemos transformar esses termos em
orações com a mesma função sintática: - Conformativas: Expressam a conformidade de um fato
Vi uma cena / que me entristeceu. (oração subordinada com outro. Conjunções: conforme, como (=conforme), segundo.
com função de adjunto adnominal) O trabalho foi feito / conforme havíamos planejado.
Todos querem / que você participe. (oração subordinada OP OSA Conformativa
com função de objeto direto)
Não pude sair / porque estava chovendo. (oração O homem age conforme pensa.
subordinada com função de adjunto adverbial de causa) Relatei os fatos como (ou conforme) os ouvi.
Como diz o povo, tristezas não pagam dívidas.
Em todos esses períodos, a segunda oração exerce uma O jornal, como sabemos, é um grande veículo de
certa função sintática em relação à primeira, sendo, portanto, informação.
subordinada a ela. Quando um período é constituído de pelo
menos um conjunto de duas orações em que uma delas (a - Temporais: Acrescentam uma circunstância de tempo
subordinada) depende sintaticamente da outra (principal), ele ao que foi expresso na oração principal. Conjunções: quando,
é classificado como período composto por subordinação. As assim que, logo que, enquanto, sempre que, depois que, mal
orações subordinadas são classificadas de acordo com a função (=assim que).
que exercem: adverbiais, substantivas e adjetivas. Ele saiu da sala / assim que eu cheguei.
OP OSA Temporal
Orações Subordinadas Adverbiais
Formiga, quando quer se perder, cria asas.
As orações subordinadas adverbiais (OSA) são “Lá pelas sete da noite, quando escurecia, as casas se
aquelas que exercem a função de adjunto adverbial da oração esvaziam.” (Carlos Povina Cavalcânti)
principal (OP). São classificadas de acordo com a conjunção “Quando os tiranos caem, os povos se levantam.”
subordinativa que as introduz: (Marquês de Maricá)
Enquanto foi rico, todos o procuravam.
- Causais: Expressam a causa do fato enunciado na oração
principal. Conjunções: porque, que, como (= porque), pois que, - Finais: Expressam a finalidade ou o objetivo do que foi
visto que. enunciado na oração principal. Conjunções: para que, a fim de
Não fui à escola / porque fiquei doente. que, porque (=para que), que.
OP OSA Causal Abri a porta do salão / para que todos pudessem entrar.
OP OSA Final
O tambor soa porque é oco.
Como não me atendessem, repreendi-os severamente. “O futuro se nos oculta para que nós o imaginemos.”
Como ele estava armado, ninguém ousou reagir. (Marquês de Maricá)
“Faltou à reunião, visto que esteve doente.” (Arlindo Aproximei-me dele a fim de que me ouvisse melhor.
de Sousa) “Fiz-lhe sinal que se calasse.” (Machado de Assis) (que
= para que)
- Condicionais: Expressam hipóteses ou condição para a “Instara muito comigo não deixasse de frequentar as
ocorrência do que foi enunciado na principal. Conjunções: se, recepções da mulher.” (Machado de Assis) (não deixasse =
contanto que, a menos que, a não ser que, desde que. para que não deixasse)
Irei à sua casa / se não chover.
OP OSA Condicional - Consecutivas: Expressam a consequência do que foi
enunciado na oração principal. Conjunções: porque, que, como
Deus só nos perdoará se perdoarmos aos nossos (= porque), pois que, visto que.
ofensores. A chuva foi tão forte / que inundou a cidade.
Se o conhecesses, não o condenarias. OP OSA Consecutiva
“Que diria o pai se soubesse disso?” (Carlos Drummond
de Andrade) Fazia tanto frio que meus dedos estavam endurecidos.
A cápsula do satélite será recuperada, caso a experiência “A fumaça era tanta que eu mal podia abrir os olhos.”
tenha êxito. (José J. Veiga)
De tal sorte a cidade crescera que não a reconhecia
- Concessivas: Expressam ideia ou fato contrário ao da mais.
oração principal, sem, no entanto, impedir sua realização. As notícias de casa eram boas, de maneira que pude
Conjunções: embora, ainda que, apesar de, se bem que, por mais prolongar minha viagem.
que, mesmo que.
Ela saiu à noite / embora estivesse doente. - Comparativas: Expressam ideia de comparação com
OP OSA Concessiva referência à oração principal. Conjunções: como, assim como,
tal como, (tão)... como, tanto como, tal qual, que (combinado
Admirava-o muito, embora (ou conquanto ou posto com menos ou mais).
que ou se bem que) não o conhecesse pessoalmente. Ela é bonita / como a mãe.
Embora não possuísse informações seguras, ainda OP OSA Comparativa
assim arriscou uma opinião.

27
LÍNGUA PORTUGUESA
A preguiça gasta a vida como a ferrugem consome o - depois de expressões na voz passiva, como sabe-se, conta-
ferro.” (Marquês de Maricá) se, diz-se, etc. Ex.: Sabe-se que ele saiu da cidade.
Ela o atraía irresistivelmente, como o imã atrai o ferro. - depois de verbos como convir, cumprir, constar, urgir,
Os retirantes deixaram a cidade tão pobres como vieram. ocorrer, quando empregados na 3ª pessoa do singular e
Como a flor se abre ao Sol, assim minha alma se abriu seguidos das conjunções que ou se. Ex.: Convém que todos
à luz daquele olhar. participem da reunião.

Obs.: As orações comparativas nem sempre apresentam É necessário que você colabore. (= Sua colaboração é
claramente o verbo, como no exemplo acima, em que está necessária.)
subentendido o verbo ser (como a mãe é). Parece que a situação melhorou.
Aconteceu que não o encontrei em casa.
- Proporcionais: Expressam uma ideia que se relaciona Importa que saibas isso bem.
proporcionalmente ao que foi enunciado na principal.
Conjunções: à medida que, à proporção que, ao passo que, - Oração Subordinada Substantiva Completiva
quanto mais, quanto menos. Nominal: É aquela que exerce a função de complemento
Quanto mais reclamava / menos atenção recebia. nominal de um termo da oração principal. Observe: Estou
OSA Proporcional OP convencido de sua inocência. (complemento nominal)
Estou convencido / de que ele é inocente.
À medida que se vive, mais se aprende. OP OSS Completiva Nominal
À proporção que avançávamos, as casas iam rareando.
O valor do salário, ao passo que os preços sobem, vai Sou favorável a que o prendam. (= Sou favorável à
diminuindo. prisão dele.)
Estava ansioso por que voltasses.
Orações Subordinadas Substantivas Sê grato a quem te ensina.
“Fabiano tinha a certeza de que não se acabaria tão
As orações subordinadas substantivas (OSS) são cedo.” (Graciliano Ramos)
aquelas que, num período, exercem funções sintáticas próprias
de substantivos, geralmente são introduzidas pelas conjunções - Oração Subordinada Substantiva Predicativa:
integrantes que e se. Elas podem ser: É aquela que exerce a função de predicativo do sujeito da
oração principal, vindo sempre depois do verbo ser. Observe: O
- Oração Subordinada Substantiva Objetiva Direta: importante é sua felicidade. (predicativo)
É aquela que exerce a função de objeto direto do verbo da O importante é / que você seja feliz.
oração principal. Observe: O grupo quer a sua ajuda. (objeto OP OSS Predicativa
direto)
O grupo quer / que você ajude. Seu receio era que chovesse. (Seu receio era a chuva.)
OP OSS Objetiva Direta Minha esperança era que ele desistisse.
Meu maior desejo agora é que me deixem em paz.
O mestre exigia que todos estivessem presentes. (= Não sou quem você pensa.
O mestre exigia a presença de todos.)
Mariana esperou que o marido voltasse. - Oração Subordinada Substantiva Apositiva: É
Ninguém pode dizer: Desta água não beberei. aquela que exerce a função de aposto de um termo da oração
O fiscal verificou se tudo estava em ordem. principal. Observe: Ele tinha um sonho: a união de todos
em benefício do país. (aposto)
- Oração Subordinada Substantiva Objetiva Ele tinha um sonho / que todos se unissem em benefício
Indireta: É aquela que exerce a função de objeto indireto do do país.
verbo da oração principal. Observe: Necessito de sua ajuda. OP OSS Apositiva
(objeto indireto)
Necessito / de que você me ajude. Só desejo uma coisa: que vivam felizes. (Só desejo uma
OP OSS Objetiva Indireta coisa: a sua felicidade)
Só lhe peço isto: honre o nosso nome.
Não me oponho a que você viaje. (= Não me oponho à “Talvez o que eu houvesse sentido fosse o presságio disto:
sua viagem.) de que virias a morrer...” (Osmã Lins)
Aconselha-o a que trabalhe mais. “Mas diga-me uma cousa, essa proposta traz algum
Daremos o prêmio a quem o merecer. motivo oculto?” (Machado de Assis)
Lembre-se de que a vida é breve.
As orações apositivas vêm geralmente antecedidas de dois-
- Oração Subordinada Substantiva Subjetiva: pontos. Podem vir, também, entre vírgulas, intercaladas à oração
É aquela que exerce a função de sujeito do verbo da oração principal. Exemplo: Seu desejo, que o filho recuperasse a
principal. Observe: É importante sua colaboração. (sujeito) saúde, tornou-se realidade.
É importante / que você colabore.
OP OSS Subjetiva Orações Subordinadas Adjetivas

A oração subjetiva geralmente vem: As orações subordinadas Adjetivas (OSA) exercem


- depois de um verbo de ligação + predicativo, em a função de adjunto adnominal de algum termo da oração
construções do tipo é bom, é útil, é certo, é conveniente, etc. principal. Observe como podemos transformar um adjunto
Ex.: É certo que ele voltará amanhã. adnominal em oração subordinada adjetiva:

28
LÍNGUA PORTUGUESA
Desejamos uma paz duradoura. (adjunto adnominal) Reduzida: Temos apenas um filho, criado com muito amor.
Desejamos uma paz / que dure. (oração subordinada Desenvolvida: Temos apenas um filho, que criamos com
adjetiva) muito amor. (Oração Subordinada Adjetiva Explicativa)

As orações subordinadas adjetivas são sempre introduzidas Reduzida: A criança sequestrada foi resgatada.
por um pronome relativo (que , qual, cujo, quem, etc.) e podem Desenvolvida: A criança que sequestraram foi resgatada.
ser classificadas em: (Oração Subordinada Adjetiva Restritiva)

- Subordinadas Adjetivas Restritivas: São restritivas Orações Reduzidas de Gerúndio:


quando restringem ou especificam o sentido da palavra a que Gerúndio: terminação –ndo.
se referem. Exemplo:
Reduzida: Não enviando o relatório a tempo, perdeu a
O público aplaudiu o cantor / que ganhou o 1º lugar. bolsa de estudos.
OP OSA Restritiva Desenvolvida: Porque não enviou o relatório a tempo,
perdeu a bolsa de estudos. (Oração Subordinada Adverbial
Nesse exemplo, a oração que ganhou o 1º lugar Causal)
especifica o sentido do substantivo cantor, indicando que Questões
o público não aplaudiu qualquer cantor mas sim aquele que
ganhou o 1º lugar. 01. Assinale a opção que apresenta explicação correta para
a inserção de “que é” antes do segmento grifado no texto.
Pedra que rola não cria limo.
Os animais que se alimentam de carne chamam-se A Secretaria de Aviação Civil da Presidência da República
carnívoros. divulgou recentemente a pesquisa O Brasil que voa – Perfil dos
Rubem Braga é um dos cronistas que mais belas páginas Passageiros, Aeroportos e Rotas do Brasil, o mais completo
escreveram. levantamento sobre transporte aéreo de passageiros do País.
“Há saudades que a gente nunca esquece.” (Olegário Mais de 150 mil passageiros, ouvidos durante 2014 nos 65
Mariano) aeroportos responsáveis por 98% da movimentação aérea do
País, revelaram um perfil inédito do setor.
- Subordinadas Adjetivas Explicativas: São
explicativas quando apenas acrescentam uma qualidade à <http://www.anac.gov.br/Noticia.aspx?ttCD_
palavra a que se referem, esclarecendo um pouco mais seu CHAVE=1957&slCD_ ORIGEM=29>. Acesso em: 13/12/2015 (com
sentido, mas sem restringi-lo ou especificá-lo. Exemplo: adaptações).
O escritor Jorge Amado, / que mora na Bahia, / lançou um
novo livro. a) Prejudica a correção gramatical do período, pois provoca
OP OSA Explicativa OP truncamento sintático.
b) Transforma o aposto em oração subordinada adjetiva
Deus, que é nosso pai, nos salvará. explicativa.
Valério, que nasceu rico, acabou na miséria. c) Altera a oração subordinada explicativa para oração
Ele tem amor às plantas, que cultiva com carinho. restritiva.
Alguém, que passe por ali à noite, poderá ser assaltado. d) Transforma o segmento grifado em oração principal do
período.
Orações Reduzidas e) Corrige erro de estrutura sintática inserido no período.

As orações reduzidas são caracterizadas por possuírem o 02. É indiscutível que no mundo contemporâneo o
verbo nas formas de gerúndio, particípio ou infinitivo. ambiente do futebol é dos mais intensos do ponto de vista
Ao contrário das demais orações subordinadas, as orações psicológico. Nos estádios a concentração é total. Vive-se ali
reduzidas não são ligadas através dos conectivos situação de incessante dialética entre o metafórico e o literal,
entre o lúdico e o real. O que varia conforme o indivíduo
Há três tipos de orações reduzidas: considerado é a passagem de uma condição a outra. Passagem
- Orações reduzidas de infinitivo rápida no caso do torcedor, cuja regressão psíquica do lúdico
- Orações reduzidas de gerúndio dura algumas horas e funciona como escape para as pressões do
- Orações reduzidas de particípio cotidiano. Passagem lenta no caso do futebolista profissional,
que vive quinze ou vinte anos em ambiente de fantasia, que
Orações Reduzidas de Infinitivo: geralmente torna difícil a inserção na realidade global quando
Infinitivo: terminações –ar, -er, -ir. termina a carreira. A solução para muitos é a reconversão em
técnico, que os mantém sob holofote. Lothar Matthäus, por
Reduzida: É preciso comer frutas e legumes. exemplo, recordista de partidas em Copas do Mundo, com a
Desenvolvida: É preciso que se coma frutas e legumes. seleção alemã, Ballon d’Or de 1990, tornou-se técnico porque
(Oração Subordinada Substantiva Subjetiva) “na verdade, para mim, o futebol é mais importante do que a
família”. [...]
Reduzida: Meu desejo era ganhar uma viagem. Sendo esporte coletivo, o futebol tem implicações e
Desenvolvida: Meu desejo era que eu ganhasse uma significações psicológicas coletivas, porém calcadas, pelo
viagem. (Oração Subordinada Substantiva Predicativa) menos em parte, nas individualidades que o compõem. O jogo
é coletivo, como a vida social, porém num e noutra a atuação
Orações Reduzidas de Particípio: de um só indivíduo pode repercutir sobre o todo. Como em
Particípio: terminações –ado, -ido. qualquer sociedade, na do futebol vive-se o tempo inteiro em

29
LÍNGUA PORTUGUESA
equilíbrio precário entre o indivíduo e o grupo. O jogador busca As marcas do discurso são:
o sucesso pessoal, para o qual depende em grande parte dos
companheiros; há um sentimento de equipe, que depende das - A fala das personagens é, de princípio, anunciada por
qualidades pessoais de seus membros. O torcedor lúcido busca um verbo (disse e interrompeu no caso do filho e perguntou
o prazer do jogo preservando sua individualidade; todavia, a e começou a dizer no caso do pai) denominado “verbo de
dizer” (como: recrutar, retorquir, afirmar, declarar e outros do
própria condição de torcedor acaba por diluí-lo na massa.
mesmo tipo), que pode vir antes, no meio ou depois da fala das
personagens (no nosso caso, veio depois);
(JÚNIOR, Hilário Franco. A dança dos deuses: futebol, cultura, - A fala das personagens aparece nitidamente separada da
sociedade. São Paulo: Companhia das letras, 2007, p. 303-304, com fala do narrador, por aspas, dois pontos, travessão ou vírgula;
adaptações) - Os pronomes pessoais, os tempos verbais e as palavras
que indicam espaço e tempo (por exemplo, pronomes
*Ballon d’Or 1990 - prêmio de melhor jogador do ano demonstrativos e advérbios de lugar e de tempo) são usados em
relação à pessoa da personagem, ao momento em que ela fala
O jogador busca o sucesso pessoal ... diz “eu”, o espaço em que ela se encontra é o aqui e o tempo
em que fala é o agora.
A mesma relação sintática entre verbo e complemento,
Discurso Indireto
sublinhados acima, está em:
(A) É indiscutível que no mundo contemporâneo... Observemos um fragmento do mesmo conto de Machado
(B) ... o futebol tem implicações e significações psicológicas de Assis:
coletivas ...
(C) ... e funciona como escape para as pressões do cotidiano. “Um dia, Serafina recebeu uma carta de Tavares dizendo-
(D) A solução para muitos é a reconversão em técnico ... lhe que não voltaria mais à casa de seu pai, por este lhe haver
(E) ... que depende das qualidades pessoais de seus mostrado má cara nas últimas vezes que ele lá estivera.”
membros. Idem. Ibidem, p. 48.
Respostas
Nesse caso o narrador para citar que Tavares disse a
01. B / 02. B
Serafina, usa o outro procedimento: não reproduz literalmente
as palavras de Tavares, mas comunica, com suas palavras, o
que a personagem diz. A fala de Tavares não chega ao leitor
diretamente, mas por via indireta, isto é, por meio das palavras
do narrador. Por essa razão, esse expediente é chamado discurso
indireto.
Discurso direto e indireto. As principais marcas do discurso indireto são:

- As falas das personagens também vêm introduzidas por


um verbo de dizer;
Discurso Direto, Indireto e Indireto Livre
- As falas das personagens constituem oração subordinada
substantiva objetiva direta do verbo de dizer e, portanto, são
Num texto, as personagens falam, conversam entre si,
separadas da fala do narrador por uma partícula introdutória
expõem ideias. Quando o narrador conta o que elas disseram,
normalmente “que” ou “se”;
insere na narrativa uma fala que não é de sua autoria, cita o
- Os pronomes pessoais, os tempos verbais e as palavras
discurso alheio. Há três maneiras principais de reproduzir a
que indicam espaço e tempo (como pronomes demonstrativos
fala das personagens: o discurso direto, o discurso indireto e o
e advérbios de lugar e de tempo) são usados em relação ao
discurso indireto livre.
narrador, ao momento em que ele fala e ao espaço em que está.
Discurso Direto
Passagem do Discurso Direto para o Discurso
Indireto
“Longe do olhos...”
Pedro disse:
- Meu pai! Disse João Aguiar com um tom de ressentimento
- Eu estarei aqui amanhã.
que fez pasmar o comendador.
- Que é? Perguntou este.
No discurso direto, o personagem Pedro diz “eu”; o “aqui” é
João Aguiar não respondeu. O comendador arrugou a testa
o lugar em que a personagem está; “amanhã” é o dia seguinte
e interrogou o roto mudo do filho. Não leu, mais adivinhou
ao que ele fala. Se passarmos essa frase para o discurso indireto
alguma coisa desastrosa; desastrosa, entenda-se, para os cálculos
ficará assim:
conjunto-políticos ou políticos-conjugais, como melhor nome haja.
- Dar-se-á caso que... começou a dizer comendador.
Pedro disse que estaria lá no dia seguinte.
- Que eu namore? Interrompeu galhofeiramente o filho.
No discurso indireto, o “eu” passa a ele porque á alguém
Machado de Assis. Contos. 26ª Ed. São Paulo, Ática, 2002,
de quem o narrador fala; estaria é futuro do pretérito: é um
p. 43.
tempo relacionado ao pretérito da fala do narrador (disse), e
não ao presente da fala do personagem, como estarei; lá é o
O narrador introduz a fala das personagens, um pai e um
espaço em que a personagem (e não o narrador) havia de estar;
filho, e, em seguida, como quem passa a palavra a elas e as deixa
no dia seguinte é o dia que vem após o momento da fala da
falar. Vemos que as partes introdutórias pertencem ao narrador
personagem designada por ele.
(por exemplo, disse João Aguiar com um tom de ressentimento
Na passagem do discurso direto para o indireto, deve-
que faz pasmar o comendador) e as falas, às personagens, (por
se observar as frases que no discurso direto tem as formas
exemplo, Meu pai!).
interrogativas, exclamativa ou imperativa convertem-se, no
O discurso direto é o expediente de citação do discurso
discurso indireto, em orações declarativas.
alheio pela qual o narrador introduz o discurso do outro e,
depois, reproduz literalmente a fala dele.
Ela me perguntou: quem está ai?

30
LÍNGUA PORTUGUESA
Ela me perguntou quem estava lá. Graciliano Ramos. Vidas secas.
28ª Ed. São Paulo, Martins, 1971, p. 136.
As interjeições e os vocativos do discurso direto desaparecem
no discurso indireto ou tem seu valor semântico explicitado, Nesse texto, duas vozes estão misturadas: a do narrador e a
isto é, traduz-se o significado que elas expressam. de Fabiano. Não há indicadores que delimitem muito bem onde
começa a fala do narrador e onde se inicia a da personagem.
O papagaio disse: Oh! Lá vem a raposa. Não se tem dúvida de que no período inicial está traduzida
O papagaio disse admirado (explicitação do valor a fala do narrador. A bem verdade, até não se conformou
semântico da interjeição oh!) que ao longe vinha a raposa. (início do segundo parágrafo), é a voz do narrador que está
comandando a narrativa. Na oração devia haver engano, já
Se o discurso citado (fala da personagem) comporta um “eu” começa haver uma mistura de vozes: sob o ponto de vista das
ou um “tu” que não se encontram entre as pessoas do discurso marcas gramaticais, não há nenhuma pista para se concluir, que
citante (fala do narrador), eles são convertidos num “ele”, se o a voz de Fabiano é que esteja sendo citada; sob o ponto de
discurso citado contém um “aqui” não corresponde ao lugar em vista do significado, porém, pode-se pensar numa reclamação
que foi proferido o discurso citante, ele é convertido num “lá”. atribuída a ele.
Tomemos agora esse trecho: “Ele era bruto, sim senhor, via-
Pedro disse lá em Paris: - Aqui eu me sinto bem. se perfeitamente que era bruto, mas a mulher tinha miolo. Com
certeza havia um erro no papel do branco.” Pelo conteúdo de
Eu (pessoa do discurso citado que não se encontra no verdade é pelo modo de dizer, tudo nos induz a vislumbrar aí
discurso citante) converte-se em ele; aqui (espaço do discurso a voz de Fabiano ecoando por meio do discurso do narrador.
citado que é diferente do lugar em que foi proferido o discurso É como se o narrador, sem abandonar as marcas linguísticas
citante) transforma-se em lá: próprias de sua fala, estivesse incorporando as reclamações
- Pedro disse que lá ele se sentia bem. e suspeitas da personagem, a cuja linguagem pertencem
expressões do tipo bruto, sim senhor e a mulher tinha miolo.
Se a pessoa do discurso citado, isto é, da fala da personagem Até a repetição de palavras e certa entonação presumivelmente
(eu, tu, ele) tem um correspondente no discurso citante, ela exclamativa confirmam essa inferência.
ocupa o estatuto que tem nesse último. Para perceber melhor o que é o discurso indireto livre,
confrontemos uma frase do texto com a correspondente em
Maria declarou-me: - Eu te amo. discurso direito e indireto:

O “te” do discurso citado corresponde ao “me” do citante. - Discurso Indireto Livre


Por isso, “te” passa a “me”: Estava direito aquilo?

- Maria declarou-me que me amava. - Discurso Direto


Fabiano perguntou: - Está direito isto?
No que se refere aos tempos, o mais comum é o que o verbo
de dizer esteja no presente ou no pretérito perfeito. Quando o - Discurso Indireto
verbo de dizer estiver no presente e o da fala da personagem Fabiano perguntou se aquilo estava direito
estiver no presente, pretérito ou futuro do presente, os tempos
mantêm-se na passagem do discurso direto para o indireto. Se Essa forma de citação do discurso alheio tem características
o verbo de dizer estiver no pretérito perfeito, as alterações que próprias que são tanto do discurso direto quanto do indireto. As
ocorrerão na fala da personagem são as seguintes: características do discurso indireto livre são:

- Não há verbos de dizer anunciando as falas das


Discurso Direto – Discurso Indireto
personagens;
Presente – Pretérito Imperfeito - Estas não são introduzidas por partículas como “que” e
“se” nem separadas por sinais de pontuação;
Pretérito Perfeito – Pretérito mais-que-perfeito
- O discurso indireto livre contém, como o discurso direto,
Futuro do Presente – Futuro do Pretérito orações interrogativas, imperativas e exclamativas, bem como
interjeições e outros elementos expressivos;
Joaquim disse: - Compro tudo isso. - Os pronomes pessoais e demonstrativos, as palavras
- Joaquim disse que comprava tudo isso. indicadoras de espaço e de tempo são usadas da mesma forma
que no discurso indireto. Por isso, o verbo estar, do exemplo
Joaquim disse: - Comprei tudo isso. acima, ocorre no pretérito imperfeito, e não no presente
- Joaquim disse que comprara tudo isso. (está), como no discurso direto. Da mesma forma o pronome
demonstrativo ocorre na forma aquilo, como no discurso
Joaquim disse: - Comprarei tudo isso. indireto.
- Joaquim disse que compraria tudo isso. Quanto ao discurso indireto, pode ser de dois tipos e cada
um deles cria um efeito de sentido diverso.
Discurso Indireto Livre
- Discurso Indireto que analisa o conteúdo: elimina
“(...) No dia seguinte Fabiano voltou à cidade, mas ao fechar os elementos emocionais ou afetivos presentes no discurso
o negócio notou que as operações de Sinhá Vitória, como de direto, assim como as interrogações, exclamações ou formas
costume, diferiam das do patrão. Reclamou e obteve a explicação imperativas, por isso produz um efeito de sentido de objetividade
habitual: a diferença era proveniente de juros. analítica. Com efeito, nele o narrador revela somente o
Não se conformou: devia haver engano. Ele era bruto, sim conteúdo do discurso da personagem, e não o modo como ela
senhor, via-se perfeitamente que era bruto, mas a mulher tinha diz. Com isso estabelece uma distância entre sua posição e a da
miolo. Com certeza havia um erro no papel do branco. Não se personagem, abrindo caminho para a réplica e o comentário.
descobriu o erro, e Fabiano perdeu os estribos. Passar a vida Esse tipo de discurso indireto despersonaliza discurso citado em
inteira assim no toco, entregando o que era dele de mão beijada! nome de uma objetividade analítica. Cria, assim, a impressão
Estava direito aquilo? Trabalhar como negro e nunca arranjar de que o narrador analisa o discurso citado de maneira racional
carta de alforria!” e isenta de envolvimento emocional. O discurso indireto, nesse
caso, não se interessa pela individualidade do falante no modo

31
LÍNGUA PORTUGUESA
como ele diz as coisas. Por isso é a forma preferida nos textos
de natureza filosófica, científica, política, etc., quando se expõe
as opiniões dos outros com finalidade de criticá-las, rejeitá-las
ou acolhê-las.

- Discurso Indireto que analisa a expressão: serve Tempos, modos e vozes verbais. Classes
T
para destacar mais o modo de dizer do que o que se diz; por de palavras.
exemplo, as palavras típicas do vocabulário da personagem
citada, a sua maneira de pronunciá-las, etc. Nesse caso, as
palavras ou expressões ressaltadas aparecem entre aspas. Classe de Palavras
O discurso indireto livre fica a meio caminho da
subjetividade e da objetividade. Tem muitas funções. Por ARTIGO
exemplo, dá verossimilhança a um texto que pretende
manifestar pensamentos, desejos, enfim, a vida interior de uma Artigo é uma palavra que antepomos aos substantivos para
personagem. determiná-los, indicando, ao mesmo tempo, gênero e número.
Em síntese, demonstra um envolvimento tal do narrador Dividem-se os artigos em: definidos: o, a, os, as e
com a personagem, que as vozes de ambos se misturam indefinidos: um, uma, uns, umas.
como se eles fossem um só ou, falando de outro modo, como Os definidos determinam os substantivos de modo preciso,
se o narrador tivesse vestido completamente a máscara da particular: Viajei com o médico.
personagem, aproximando-a do leitor sem a marca da sua Os indefinidos determinam os substantivos de modo vago,
intermediação. impreciso, geral: Viajei com um médico.
Veja-se como, neste trecho: “O tímido José”, de Antônio
de Alcântara Machado, o narrador, valendo-se do discurso - Ambas as mãos. Usa-se o artigo entre o numeral ambas e
indireto livre, leva o leitor a partilhar do constrangimento da o substantivo: Ambas as mãos são perfeitas.
personagem, simulando estar contaminado por ele:
- Estou em Paris / Estou na famosa Paris. Não se usa artigo
(...) Mais depressa não podia andar. Garoar, garoava sempre. antes dos nomes de cidades, a menos que venham determinados
Mas ali o nevoeiro já não era tanto felizmente. Decidiu. Iria por adjetivos ou locuções adjetivas.
indo no caminho da Lapa. Se encontrasse a mulher bem. Se não Vim de Paris
encontrasse paciência. Não iria procurar. Iria é para casa. Afinal Vim da luminosa Paris.
de contas era mesmo um trouxa. Quando podia não quis. Agora
que era difícil queria. Mas com alguns nomes de cidades conservamos o artigo.
O Rio de Janeiro, O Cairo, O Porto.
Laranja-da-china. In: Novelas Paulistanas.
1ª Ed. Belo Horizonte, Itatiaia/ São Paulo, Edusp, 1998, p. 184. Pode ou não ocorrer crase antes dos nomes de cidade,
conforme venham ou não precedidos de artigo.
Questões Vou a Paris.
Vou à Paris dos museus.
01. Sobre o discurso indireto é correto afirmar, EXCETO:
- Toda cidade / toda a cidade. Todo, toda designam
(A) No discurso indireto, o narrador utiliza suas próprias qualquer, cada.
palavras para reproduzir a fala de um personagem. Toda cidade pode concorrer (qualquer cidade).
(B) O narrador é o porta-voz das falas e dos pensamentos Todo o, toda a designam totalidade, inteireza.
das personagens. Conheci toda a cidade (a cidade inteira).
(C) Normalmente é escrito na terceira pessoa. As falas são
iniciadas com o sujeito, mais o verbo de elocução seguido da No plural, usa-se todos os, todas as, exceto antes de
fala da personagem. numeral não seguido de substantivo.
(D) No discurso indireto as personagens são conhecidas Todas as cidades vieram.
através de seu próprio discurso, ou seja, através de suas pró- Todos os cinco clubes disputarão o título.
prias palavras. Todos cinco são concorrentes.
02. Assinale a alternativa que melhor complete o seguinte - Tua decisão / a tua decisão. De maneira geral, é facultativo
trecho: o uso do artigo antes dos possessivos.
Aplaudimos tua decisão.
No plano expressivo, a força da ____________ em Aplaudimos a tua decisão.
_____________ provém essencialmente de sua capacidade de
_____________ o episódio, fazendo ______________ da situa- Se o possessivo não vier seguido de substantivo explícito é
ção a personagem, tornando-a viva para o ouvinte, à maneira obrigatória a ocorrência do artigo.
de uma cena de teatro __________ o narrador desempenha a Aplaudiram a tua decisão e não a minha.
mera função de indicador de falas.
- Decisões as mais oportunas / as mais oportunas decisões.
(A) narração - discurso indireto - enfatizar - ressurgir – No superlativo relativo, não se usa o artigo antes e depois do
onde; substantivo.
(B) narração - discurso onisciente - vivificar - demonstrar- Tomou decisões as mais oportunas.
-se – donde; Tomou as decisões mais oportunas.
(C) narração - discurso direto - atualizar - emergir - em que; É errado: Tomou as decisões as mais oportunas.
(D) narração - discurso indireto livre - humanizar - imergir
- na qual; - Faz uns dez anos. O artigo indefinido, posto antes de um
(E) dissertação - discurso direto e indireto - dinamizar - pro- numeral, designa quantidade aproximada: Faz uns dez anos que
tagonizar - em que. saí de lá.
Respostas - Em um / num. Os artigos definidos e indefinidos contraem-
01. D / 02. C / 03. B se com preposições: de + o= do, de + a= da, etc. As formas

32
LÍNGUA PORTUGUESA
de + um e em + um podem-se usar contraídas (dum e num) Composto - formado de mais de um radical. Por exemplo:
ou separadas (de um, em um). Estava em uma cidade grande. franco-brasileiro.
Estava numa cidade grande. Pátrio - é o adjetivo que indica a naturalidade ou a naciona-
lidade do ser. Por exemplo: brasileiro, cambuiense, etc.
SUBSTANTIVO
Locução Adjetiva
Substantivo é tudo o que nomeia as “coisas” em geral.
Substantivo é tudo o que pode ser visto, pego ou sentido. É toda expressão formada de uma preposição mais um
Substantivo é tudo o que pode ser precedido de artigo. substantivo, equivalente a um adjetivo. Por exemplo, homens
com aptidão (aptos), bandeira da Irlanda (irlandesa).
Classificação e Formação
Gêneros dos Adjetivos
Substantivo Comum: Substantivo comum é aquele que
designa os seres de uma espécie de forma genérica. Por exemplo: Biformes - têm duas formas, sendo uma para o masculino e
pedra, computador, cachorro, homem, caderno. outra para o feminino. Por exemplo, mau e má, judeu e judia.
Se o adjetivo é composto e biforme, ele flexiona no feminino so-
Substantivo Próprio: Substantivo próprio é aquele que mente o último elemento. Por exemplo, o motivo sócio-literário
designa um ser específico, determinado, individualizando-o. e a causa sócio-literária. Exceção = surdo-mudo e surda-muda.
Por exemplo: Maxi, Londrina, Dílson, Ester. O substantivo
próprio sempre deve ser escrito com letra maiúscula. NUMERAL

Substantivo Concreto: Substantivo concreto é aquele


      
2 
que designa seres que existem por si só ou apresentam-se em
nossa imaginação como se existissem por si. Por exemplo: ar,   
   
2 
som, Deus, computador, Ester.
3    
Substantivo Abstrato: Substantivo abstrato é aquele    
2 
que designa prática de ações verbais, existência de
     
2 
qualidades ou sentimentos humanos. Por exemplo:
saída (prática de sair), beleza (existência do belo), saudade.     2 
 2      
2 
Formação dos substantivos
      2 
Substantivo Primitivo: É primitivo o substantivo que
     2 
não se origina de outra palavra existente na língua portuguesa.
Por exemplo: pedra, jornal, gato, homem.  2    !   2 

Substantivo Derivado: É derivado o substantivo que Faz-se a leitura do numeral cardinal, dispondo-se a
provém de outra palavra da língua portuguesa. Por exemplo: palavra “e” entre as centenas e as dezenas e entre as dezenas
pedreiro, jornalista, gatarrão, homúnculo. e unidades. Por exemplo, 1.203.726 = um milhão duzentos e
três mil setecentos e vinte e seis.
Substantivo Simples: É simples o substantivo formado
por um único radical. Por exemplo: pedra, pedreiro, jornal, PRONOME
jornalista.
A palavra que acompanha (determina) ou substitui um
Substantivo Composto: É composto o substantivo nome é denominada pronome. Ex.: Ana disse para sua irmã: -
formado por dois ou mais radicais. Por exemplo: pedra- Eu preciso do meu livro de matemática. Você não o encontrou?
sabão, homem-rã, passatempo. Ele estava aqui em cima da mesa.
- eu substitui “Ana”
Substantivo Coletivo: É coletivo o substantivo no - meu acompanha “o livro de matemática”
singular que indica diversos elementos de uma mesma espécie. - o substitui “o livro de matemática”
- abelha - enxame, cortiço, colmeia - ele substitui “o livro de matemática”
- acompanhante - comitiva, cortejo, séquito
- alho - (quando entrelaçados) réstia, enfiada, cambada Flexão: Quanto à forma, o pronome varia em gênero,
- aluno - classe número e pessoa:
- amigo - (quando em assembleia) tertúlia
Gênero (masculino/feminino)
ADJETIVO Ele saiu/Ela saiu
Meu carro/Minha casa
É a classe gramatical de palavras que exprimem qualidade,
defeito, origem, estado do ser. Número (singular/plural)
Eu saí/Nós saímos
Classificação dos Adjetivos Minha casa/Minhas casas

Explicativo - exprime qualidade própria do ser. Por exemplo: Pessoa (1ª/2ª/3ª)


neve fria. Eu saí/Tu saíste/Ele saiu
Restritivo - exprime qualidade que não é própria do ser. Ex: Meu carro/Teu carro/Seu carro
fruta madura.
Primitivo - não vem de outra palavra portuguesa. Por exem- Função: O pronome tem duas funções fundamentais:
plo: bom e mau.
Derivado - tem origem em outra palavra portuguesa. Por Substituir o nome: Nesse caso, classifica-se como
exemplo: bondoso. pronome substantivo e constitui o núcleo de um grupo nominal.
Simples - formado de um só radical. Por exemplo: brasileiro. Ex.: Quando cheguei, ela se calou. (ela é o núcleo do sujeito da

33
LÍNGUA PORTUGUESA
segunda oração e se trata de um pronome substantivo porque Pronomes Possessivos: São aqueles que indicam ideia
está substituindo um nome) de posse. Além de indicar a coisa possuída, indicam a pessoa
gramatical possuidora.
Referir-se ao nome: Nesse caso, classifica-se como
pronome adjetivo e constitui uma palavra dependente do grupo
nominal. Ex.: Nenhum aluno se calou. (o sujeito “nenhum
aluno” tem como núcleo o substantivo “aluno” e como palavra
dependente o pronome adjetivo “nenhum”)

Pronomes Pessoais: São aqueles que substituem os


nomes e representam as pessoas do discurso:
1ª pessoa - a pessoa que fala - eu/nós
2ª pessoa - a pessoa com que se fala - tu/vós
3ª pessoa - a pessoa de quem se fala - ele/ela/eles/elas

Pronomes pessoais retos: são os que têm por função Existem palavras que eventualmente funcionam como
principal representar o sujeito ou predicativo. pronomes possessivos. Ex.: Ele afagou-lhe (seus) os cabelos.

Pronomes pessoais oblíquos: são os que podem exercer Pronomes Demonstrativos: Os pronomes
função de complemento. demonstrativos possibilitam localizar o substantivo em relação
às pessoas, ao tempo, e sua posição no interior de um discurso.

Pronomes Espaço Tempo Ao dito Enumeração

‡”–‘†‡ ”‡•‡–‡ ‡ˆ‡”‡–‡ ‡ˆ‡”‡–‡


“—‡ ƒ“—‹Ž‘“—‡ ƒ‘‹‘
ˆƒŽƒȋͳ͐ ƒ‹†ƒ ‘ ‡Ž‡‡–‘
’‡••‘ƒȌǤ ˆ‘‹†‹–‘Ǥ …‹–ƒ†‘‡
—ƒ‡—‡Ǧ
este, ”ƒ­ ‘Ǥ
esta, šǤǣ ‘ šǤǣ‡•–‡ šǤǣ•–ƒ šǤǣ
isto, ‰‘•–‡‹ ƒ‘ǡ–‡Š‘ ƒϐ‹”ƒ­ ‘ Š‘‡‡ƒ
estes, †‡•–‡ ”‡ƒŽ‹œƒ†‘ ‡†‡‹š‘— —ŽŠ‡”• ‘
estas Ž‹˜”‘ „‘• •—”’”‡•ƒǣ ƒ••ƒǦ
Pronomes Oblíquos ƒ“—‹Ǥ ‡‰×…‹‘•Ǥ ‰‘•–ƒ˜ƒ†‡ …”ƒ†‘•
“—À‹…ƒǤ ’‡Žƒ…—Ž–—”ƒ
- Associação de pronomes a verbos: Os pronomes oblíquos ƒ–—ƒŽǡƒ•
o, a, os, as, quando associados a verbos terminados em -r, -s, -z, ‡•–ƒ±ƒ‹•
assumem as formas lo, la, los, las, caindo as consoantes. Ex.: ‘’”‹‹†ƒǤ
Carlos quer convencer seu amigo a fazer uma viagem; Carlos ‡”–‘†‡ ƒ••ƒ†‘ ‡ˆ‡”‡–‡ 
quer convencê-lo a fazer uma viagem. “—‡ ‘—ˆ—–—”‘ ƒ“—‹Ž‘“—‡
‘—˜‡ȋʹ͐ ’”ך‹‘• Œžˆ‘‹†‹–‘Ǥ
- Quando associados a verbos terminados em ditongo nasal ’‡••‘ƒȌǤ
(-am, -em, -ão, -õe), assumem as formas no, na, nos, nas. Ex.: esse,
Fizeram um relatório; Fizeram-no. essa, šǤǣ ‘ šǤǣ‡••‡ šǤǣ 
esses, ‰‘•–‡‹ ‹‘ƒ‘ǡ
‘•–ƒ˜ƒ†‡
- Os pronomes oblíquos podem ser reflexivos e quando isso essas †‡••‡ ”‡ƒŽ‹œ‡‹„‘• “—À‹…ƒǤ
ocorre se referem ao sujeito da oração. Ex.: Maria olhou-se no Ž‹˜”‘ ‡‰×…‹‘• ••ƒ
espelho; Eu não consegui controlar-me diante do público. “—‡‡•–ž ƒϐ‹”ƒ­ ‘
‡–—ƒ• ‡†‡‹š‘—
- Antes do infinitivo precedido de preposição, o pronome  ‘•Ǥ •—”’”‡•ƒ
usado deverá ser o reto, pois será sujeito do verbo no infinitivo. ‡”–‘ ƒ••ƒ†‘‘—  ‡ˆ‡”‡–‡
Ex.: O professor trouxe o livro para mim. (pronome oblíquo, †ƒ͵͐ ˆ—–—”‘ ƒ‘’”‹‡‹”‘
pois é um complemento); O professor trouxe o livro para eu ler. ’‡••‘ƒǡ ”‡‘–‘• ‡Ž‡‡–‘
(pronome reto, pois é sujeito) †‹•–ƒ–‡ …‹–ƒ†‘
†‘• ‡—ƒ
Pronomes de Tratamento: São aqueles que substituem ‹–‡”Ž‘…—Ǧ ‡—‡”ƒ­ ‘Ǥ
a terceira pessoa gramatical. Alguns são usados em tratamento aquele, –‘”‡•Ǥ
cerimonioso e outros em situações de intimidade. Conheça aquela,
alguns: aquilo, šǤǣ ‘ šǤǣ‡Š‘  šǤǣ
- você (v.): tratamento familiar aqueles, ‰‘•–‡‹ „‘ƒ• Š‘‡‡ƒ
- senhor (Sr.), senhora (Srª.): tratamento de respeito aquelas †ƒ“—‡Ž‡ ”‡…‘”†ƒ­Ù‡• —ŽŠ‡”• ‘
- senhorita (Srta.): moças solteiras Ž‹˜”‘“—‡ †‡ͳͻ͸Ͳǡ ƒ••ƒ…”ƒ†‘•
- Vossa Senhoria (V.Sª.): para pessoa de cerimônia ƒ‘„‡”–ƒ ’‘‹•ƒ“—‡Ž‡ ’‡Žƒ…—Ž–—”ƒ
- Vossa Excelência (V.Exª.): para altas autoridades –”‘—š‡Ǥ ƒ‘”‡ƒŽ‹œ‡‹ ƒ–—ƒŽǡƒ•
- Vossa Reverendíssima (V. Revmª.): para sacerdotes „‘• ‡•–ƒ±ƒ‹•
- Vossa Eminência (V.Emª.): para cardeais ‡‰×…‹‘•Ǥ ‘’”‹‹†ƒ“—‡
- Vossa Santidade (V.S.): para o Papa ƒ“—‡Ž‡Ǥ
- Vossa Majestade (V.M.): para reis e rainhas
- Vossa Majestade Imperial (V.M.I.): para imperadores Pronomes Indefinidos: São pronomes que acompanham
- Vossa Alteza (V.A.): para príncipes, princesas e duques o substantivo, mas não o determinam de forma precisa: algum,

34
LÍNGUA PORTUGUESA
bastante, cada, certo, diferentes, diversos, demais, mais, -se a uma ação futura, vinculada a um momento já passado. Ex:
menos, muito nenhum, outro, pouco, qual, qualquer, quanto, Aprenderia tocar violão, se tivesse ouvido para a música (indica
tanto, todo, tudo, um, vários. condição); Eles gostariam de convidá-la para a festa.

Algumas locuções pronominais indefinidas: cada Modos Verbais


qual, qualquer um, tal e qual, seja qual for, sejam quem for,
todo aquele, quem (que), quer uma ou outra, todo aquele - Indicativo. Apresenta o fato de manei-
(que), tais e tais, tal qual, seja qual for. ra real, certa, positiva. Ex: Eu estudo geografia
Iremos ao cinema; Voltou para casa.
VERBO - subjuntivo. Pode exprimir um desejo e apresenta o fato
como possível ou duvidoso, hipotético. Ex: Queria que me le-
Quando se pratica uma ação, a palavra que representa essa vasses ao teatro; Se eu tivesse dinheiro, compraria um carro;
ação e indica o momento em que ela ocorre é o verbo. Exem- Quando o relógio despertar, acorda-me.
plos: - Imperativo. Exprime ordem, conselho ou súplica. Ex:
- Aquele pedreiro trabalhou muito. (ação – pretérito) Limpa a cozinha, Maria; Descanse bastante nestas férias; Se-
- Venta muito na primavera. (fenômeno – presente) nhor tende piedade de nós.
- Ana ficará feliz com a tua chegada. (estado - futuro)
- Maria enviuvou na semana passada. (mudança de estado As formas nominais do verbo são Três: infinitivo, gerún-
– pretérito) dio e particípio.
- A serra azula o horizonte. (qualidade – presente)
Infinitivo:
Conjugação Verbal: Existem 3 conjugações verbais: Pessoal - cantar (eu), cantares (tu), vender (eu), venderes
- A 1ª que tem como vogal temática o ‘’a’’. Ex: cantar, pular, (tu), partir (eu), partires (tu)
sonhar etc... Impessoal - cantar, vender, partir.
- A 2ª que tem como vogal temática o ‘’e’’. Ex: vender, comer, Gerúndio - cantando, vendendo, partindo.
chover, sofrer etc... Particípio - cantado, vendido, partido.
- A 3ª que tem como vogal temática o ‘’i’’. Ex: partir, dividir,
sorrir, abrir etc... Impessoal: Uma forma em que o verbo não se refere a
nenhuma pessoa gramatical: é o infinitivo impessoal quando
2º COJUGAÇÃO 3º não se refere às pessoas do discurso. Exemplos: viver é bom. (a
1º COJUGAÇÃO vida é boa); É proibido fumar. (é proibido o fumo)
verbos terminados CONJUGAÇÃO
verbos terminados
em ER verbos terminados
em AR Pessoal: Quando se refere às pessoas do discurso. Neste
em IR
cantar vender partir caso, não é flexionado nas 1ª e 3ª pessoas do singular e flexio-
amar chover sorrir nadas nas demais:
sonhar sofrer abrir Falar (eu) – não flexionado
OBS: O verbo pôr, assim como seus derivados (compor, Falares (tu) – flexionado
repor, depor, etc.), pertence à 2º conjugação, porque na sua Falar (ele) – não flexionado
forma antiga a sua terminação era em er: poer. A vogal “e”, ape- Falarmos (nós) – flexionado
sar de haver desaparecido do infinitivo, revela-se em algumas Falardes (voz) – flexionado
formas de verbo: põe, pões, põem etc. Falarem (eles) – flexionado

Pessoas: 1ª, 2ª e 3ª pessoa são abordadas em 2 situações: Ex: É conveniente estudares (é conveniente o estudo); É útil
singular e plural. pesquisarmos (é útil a nossa pesquisa)
Primeira pessoa do singular – eu; ex: eu canto
Segunda pessoa do singular – tu; ex: tu cantas Aspecto: Aspecto é a maneira de ser ação.
Terceira pessoa do singular – ele; ex ele: canta
Primeira pessoa do plural – nós; ex: nós cantamos O Pretérito Perfeito Composto: indica um fato con-
Segunda pessoa do plural – vós; ex: vós cantais cluído, revela de certa forma a ideia de continuidade. Ex: Eu te-
Terceira pessoa do plural – eles; ex: eles cantam nho estudado (eu estudei até o presente momento). Os verbos
invocativos (terminados em “ecer” ou “escer”) indica uma con-
Tempos e Modo de Verbo tinuidade gradual. Ex: embranquecer é começar a ficar grisalho
e envelhecer é ir ficando velho.
- Presente. Fato ocorrido no momento em que se fala. Ex:
Faz O Presente do Indicativo pode:
- Pretérito. Fato ocorrido antes. Ex: Fez - indicar frequência. Ex: O sol nasce para todos.
- Futuro. Fato ocorrido depois. Ex: Fará - ser empregado no lugar do futuro. Ex: amanhã vou ao tea-
tro. (irei); Se continuam as indiretas, perco a paciência. (conti-
O pretérito subdivide-se em perfeito, imperfeito e mais- nuarem; perderei)
-que-perfeito. - ser empregado no lugar do pretérito (presente histórico).
- Perfeito. Ação acabada. Ex: Eu li o ultimo romance de Ex: É 1939: alemães invadem o território polonês (era; inva-
Rubens Fonseca. diram)
- Imperfeito. Ação inacabada no momento a que se refere
à narração. Ex: Ele olhava o mar durante horas e horas. O Pretérito Imperfeito do Indicativo pode:
- Mais-que-perfeito. Ação acabada, ocorrida antes de ou- - Substituir o futuro do pretérito. Ex: Se eu soubesse, não
tro fato passado. Ex: Para poder trabalhar melhor, ela dividira dizia aquilo. (diria)
a turma em dois grupos. - Expressar cortesia ou timidez. Ex: O senhor podia fazer o
favor de me emprestar uma caneta? (pode)
O futuro subdivide-se em futuro do presente e futuro do
pretérito. Futuro do Presente pode:
- Futuro do Presente. Refere-se a um fato imediato e - Indicar probabilidade. Ex: Ele terá, no máximo, uns 70
certo. Ex: Comprarei ingressos para o teatro. quilos.
- Futuro do Pretérito. Pode indicar condição, referindo- - Substituir o imperativo. Ex: Não matarás. (não mates)

35
LÍNGUA PORTUGUESA
Tempos Simples e Tempos Compostos: Os tempos vendido, ter (teres) partido.
são simples quando formados apenas pelo verbo principal. Gerúndio pretérito composto - tendo cantado, tendo vendi-
do, tendo partido.
Indicativo: Regulares: Regulares são verbos que se conjugam de acor-
Presente - canto, vendo, parto, etc. do com o paradigma (modelo) de cada conjugação. Cantar (1ª
Pretérito perfeito - cantei, vendi, parti, etc. conjugação) vender (2ª conjugação) partir (3ª conjugação) to-
Pretérito imperfeito - cantava, vendia, partia, etc. dos que se conjugarem de acordo com esses verbos serão re-
Pretérito mais-que-perfeito - cantara, vendera, partira, etc. gulares.
Futuro do presente - cantarei, venderei, partirei, etc. ADVÉRBIO
Futuro do pretérito - cantaria, venderia, partiria, etc.
Palavra invariável que modifica essencialmente o verbo,
Subjuntivo: exprimindo uma circunstância.
Presente - cante, venda, parta, etc.
Pretérito imperfeito - cantasse, vendesse, partisse, etc. Advérbio modificando um verbo ou adjetivo:
Futuro - cantar, vender, partir. Ocorre quando o advérbio modifica um verbo ou um adjetivo
acrescentando a eles uma circunstância. Por circunstância
Imperativo: Ao indicar ordem, conselho, pedido, o fato entende-se qualquer particularidade que determina um fato,
verbal pode expressar negação ou afirmação. São, portanto, ampliando a informação nele contida. Ex.: Antônio construiu
duas as formas do imperativo: seu arraial popular ali; Estradas tão ruins.
- Imperativo Negativo: Não falem alto.
- Imperativo Afirmativo: Falem mais alto. Advérbio modificando outro advérbio: Ocorre
quando o advérbio modifica um adjetivo ou outro advérbio,
Imperativo negativo: É formado do presente do geralmente intensificando o significado. Ex.: Grande parte da
subjuntivo. população adulta lê muito mal.

Advérbio modificando uma oração inteira: Ocorre


1º 2º 3º quando o advérbio está modificando o grupo formado por todos
CONJUGAÇAO CONJUGAÇÃO CONJUGAÇÃO os outros elementos da oração, indicando uma circunstância.
CANT - AR VEND - ER PART - IR Ex.: Lamentavelmente o Brasil ainda tem 19 milhões de
analfabetos.
Não cantes Não vendas Não partas
Locução Adverbial: É um conjunto de palavras que pode
Não cante Não venda Não parta exercer a função de advérbio. Ex.: De modo algum irei lá.

Não cantemos Não vendamos Não partamos Tipos de Advérbios

Não canteis Não vendais Não partais - de modo: Ex.: Sei muito bem que ninguém deve passar
atestado da virtude alheia. Bem, mal, assim, adrede, melhor,
Não cantem Não vendam Não partam pior, depressa, acinte, debalde, devagar, ás pressas, às claras,
às cegas, à toa, à vontade, às escondas, aos poucos, desse jeito,
Imperativo afirmativo: Também é formado do presente desse modo, dessa maneira, em geral, frente a frente, lado a
do subjuntivo, com exceção da 2º pessoa do singular e da 2º lado, a pé, de cor, em vão e a maior parte dos que terminam
pessoa do plural, que são retiradas do presente do indicativo em -mente: calmamente, tristemente, propositadamente,
sem o “s”. Ex: Canta – Cante – Cantemos – Cantai – Cantem pacientemente, amorosamente, docemente, escandalosamente,
bondosamente, generosamente.
O imperativo não possui a 1º pessoa do singular, pois não se - de intensidade: Ex.: Acho que, por hoje, você já ouviu
prevê a ordem, o pedido ou o conselho a si mesmo. bastante. Muito, demais, pouco, tão, menos, em excesso,
bastante, pouco, mais, menos, demasiado, quanto, quão, tanto,
Tempos são compostos quando formados pelos auxilia- assaz, que (equivale a quão), tudo, nada, todo, quase, de todo,
res ter ou haver. de muito, por completo, bem (quando aplicado a propriedades
graduáveis).
Indicativo: - de tempo: Ex.: Leia e depois me diga quando pode sair
Pretérito perfeito composto - tenho cantado, tenho vendido, na gazeta. Hoje, logo, primeiro, ontem, tarde, outrora, amanhã,
tenho partido, etc. cedo, dantes, depois, ainda, antigamente, antes, doravante,
Pretérito mais-que-perfeito composto - tinha cantado, tinha nunca, então, ora, jamais, agora, sempre, já, enfim, afinal,
vendido, tinha partido, etc. amiúde, breve, constantemente, entrementes, imediatamente,
Futuro do presente composto - terei cantado, terei vendido, primeiramente, provisoriamente, sucessivamente, às vezes,
terei partido, etc. à tarde, à noite, de manhã, de repente, de vez em quando,
Futuro do pretérito composto - teria cantado, teria vendido, de quando em quando, a qualquer momento, de tempos em
teria partido, etc. tempos, em breve, hoje em dia.
- de lugar: Ex.: A senhora sabe aonde eu posso encontrar
Subjuntivo: esse pai-de-santo? Aqui, antes, dentro, ali, adiante, fora, acolá,
Pretérito perfeito composto - tenha cantado, tenha vendido, atrás, além, lá, detrás, aquém, cá, acima, onde, perto, aí, abaixo,
tenha partido, etc. aonde, longe, debaixo, algures, defronte, nenhures, adentro,
Pretérito mais-que-perfeito composto - tivesse cantado, ti- afora, alhures, nenhures, aquém, embaixo, externamente, a
vesse vendido, tivesse partido, etc. distancia, a distancia de, de longe, de perto, em cima, à direita,
Futuro composto - tiver cantado, tiver vendido, tiver parti- à esquerda, ao lado, em volta.
do, etc. - de negação: Ex.: De modo algum irei lá. Não, nem,
nunca, jamais, de modo algum, de forma nenhuma, tampouco,
Infinitivo: de jeito nenhum.
Pretérito impessoal composto - ter cantado, ter vendido, ter - de dúvida: Ex.: Talvez ela volte hoje. Acaso, porventura,
partido, etc. possivelmente, provavelmente, quiçá, talvez, casualmente, por
Pretérito pessoal composto - ter (teres) cantado, ter (teres) certo, quem sabe.

36
LÍNGUA PORTUGUESA
- de afirmação: Ex.: Realmente eles sumiram. Sim, Quando se quer realçar o advérbio, pode-se antecipá-lo.
certamente, realmente, decerto, efetivamente, certo, Ex.: Imediatamente convoquei os alunos.
decididamente, realmente, deveras, indubitavelmente.
- de exclusão: Apenas, exclusivamente, salvo, senão, PREPOSIÇÃO
somente, simplesmente, só, unicamente.
- de inclusão: Ex.: Emocionalmente o indivíduo também É uma palavra invariável que serve para ligar termos ou
amadurece durante a adolescência. Ainda, até, mesmo, orações. Quando esta ligação acontece normalmente há uma
inclusivamente, também. subordinação do segundo termo em relação ao primeiro. As
- de ordem: Depois, primeiramente, ultimamente. preposições são muito importantes na estrutura da língua, pois
- de designação: Eis estabelecem a coesão textual e possuem valores semânticos
- de interrogação: Ex.: E então? Quando é que embarca? indispensáveis para a compreensão do texto.
onde? (lugar), como? (modo), quando? (tempo), porque?
(causa), quanto? (preço e intensidade), para que? (finalidade). Tipos de Preposição

Palavras Denotativas: Há, na língua portuguesa, - Preposições essenciais: palavras que atuam
uma série de palavras que se assemelham a advérbios. exclusivamente como preposições. A, ante, perante, após, até,
A Nomenclatura Gramatical Brasileira não faz nenhuma com, contra, de, desde, em, entre, para, por, sem, sob, sobre,
classificação especial para essas palavras, por isso elas são trás, atrás de, dentro de, para com.
chamadas simplesmente de palavras denotativas. - Preposições acidentais: palavras de outras classes
- Adição: Ex.: Comeu tudo e ainda queria mais. Ainda, gramaticais que podem atuar como preposições. Como,
além disso. durante, exceto, fora, mediante, salvo, segundo, senão, visto.
- Afastamento: Ex.: Foi embora daqui. Embora. - Locuções prepositivas: duas ou mais palavras valendo
- Afetividade: Ex.: Ainda bem que passei de ano. Ainda como uma preposição, sendo que a última palavra é uma delas.
bem, felizmente, infelizmente. Abaixo de, acerca de, acima de, ao lado de, a respeito de, de
- Aproximação: quase, lá por, bem, uns, cerca de, por acordo com, em cima de, embaixo de, em frente a, ao redor de,
volta de. graças a, junto a, com, perto de, por causa de, por cima de, por
- Designação: Ex.: Eis nosso novo carro. Eis. trás de.
- Exclusão: Ex.: Todos irão, menos ele. Apenas, salvo,
menos, exceto, só, somente, exclusive, sequer, senão. A preposição é invariável. No entanto pode unir-se a outras
- Explicação: Ex.: Viajaremos em julho, ou seja, nas férias. palavras e assim estabelecer concordância em gênero ou em
Isto é, por exemplo, a saber, ou seja. número. Ex: por + o = pelo; por + a = pela.
- Inclusão: Ex.: Até ele irá viajar. Até, inclusive, também, Vale ressaltar que essa concordância não é característica da
mesmo, ademais. preposição e sim das palavras a que se ela se une. Esse processo
- Limitação: Ex.: Apenas um me respondeu. Só, somente, de junção de uma preposição com outra palavra pode se dar a
unicamente, apenas. partir de dois processos:
- Realce: Ex.: E você lá sabe essa questão? É que, cá, lá,
não, mas, é porque, só, ainda, sobretudo. - Combinação: A preposição não sofre alteração.
- Retificação: Ex.: Somos três, ou melhor, quatro. Aliás, preposição a + artigos definidos o, os
isto é, ou melhor, ou antes. a + o = ao
- Situação: Ex.: Afinal, quem perguntaria a ele? Então, preposição a + advérbio onde
mas, se, agora, afinal. a + onde = aonde

Grau dos Advérbios: Os advérbios, embora pertençam à - Contração: Quando a preposição sofre alteração.
categoria das palavras invariáveis, podem apresentar variações Preposição + Artigos
com relação ao grau. Além do grau normal, o advérbio pode-se De + o(s) = do(s)
apresentar no grau comparativo e no superlativo. De + a(s) = da(s)
De + um = dum
- Grau Comparativo: quando a circunstância expressa De + uns = duns
pelo advérbio aparece em relação de comparação. O advérbio De + uma = duma
não é flexionado no grau comparativo. Para indicar esse grau De + umas = dumas
utilizam as formas tão…quanto, mais…que, menos…que. Pode Em + o(s) = no(s)
ser: Em + a(s) = na(s)
- comparativo de igualdade. Ex.: Chegarei tão cedo quanto Em + um = num
você. Em + uma = numa
- comparativo de superioridade. Ex.: Chegarei mais cedo Em + uns = nuns
que você. Em + umas = numas
- comparativo de inferioridade. Ex.: Chegaremos menos A + à(s) = à(s)
cedo que você. Por + o = pelo(s)
Por + a = pela(s)
- Grau Superlativo: nesse caso, a circunstância expressa
pelo advérbio aparecerá intensificada. O grau superlativo - Preposição + Pronomes
do advérbio pode ser formado tanto pelo processo sintético De + ele(s) = dele(s)
(acréscimo de sufixo), como pelo processo analítico (outro De + ela(s) = dela(s)
advérbio estará indicando o grau superlativo). De + este(s) = deste(s)
- superlativo (ou absoluto) sintético: formado com o De + esta(s) = desta(s)
acréscimo de sufixo. Ex.: Cheguei tardíssimo. De + esse(s) = desse(s)
- superlativo (ou absoluto) analítico: expresso com o auxilio De + essa(s) = dessa(s)
de um advérbio de intensidade. Ex.: Cheguei muito tarde. De + aquele(s) = daquele(s)
De + aquela(s) = daquela(s)
Quando se empregam dois ou mais advérbios terminados De + isto = disto
em –mente, pode-se acrescentar o sufixo apenas no ultimo. De + isso = disso
Ex.: Nada omitiu de seu pensamento; falou clara, franca e De + aquilo = daquilo
nitidamente. De + aqui = daqui

37
LÍNGUA PORTUGUESA
De + aí = daí idéia expressa por uma palavra (ou um conjunto de palavras
De + ali = dali - locução interjetiva) que poderia ser colocada em termos de
De + outro = doutro(s) uma sentença. Observe:
De + outra = doutra(s) - Bravo! Bravo! Bis! (bravo e bis: interjeição). Sentença
Em + este(s) = neste(s) sugestão: “Foi muito bom! Repitam!”
Em + esta(s) = nesta(s) - Ai! Ai! Ai! Machuquei meu pé. (ai: interjeição). Sentença
Em + esse(s) = nesse(s) sugestão: “Isso está doendo!” ou “Estou com dor!”
Em + aquele(s) = naquele(s)
Em + aquela(s) = naquela(s) O significado das interjeições está vinculado à maneira
Em + isto = nisto como elas são proferidas. Desse modo, o tom da fala é que dita
Em + isso = nisso o sentido que a expressão vai adquirir em cada contexto de
Em + aquilo = naquilo enunciação. Exemplos:
A + aquele(s) = àquele(s) - Psiu! (contexto: alguém pronunciando essa expressão na
A + aquela(s) = àquela(s) rua). Significado da interjeição (sugestão): “Estou te chaman-
A + aquilo = àquilo do! Ei, espere!”.
- Psiu! (contexto: alguém pronunciando essa expressão em
1. O “a” pode funcionar como preposição, pronome pessoal um hospital). Significado da interjeição (sugestão): “Por favor,
oblíquo e artigo. Como distingui-los? faça silêncio!”.
- Caso o “a” seja um artigo, virá precedendo a um - Puxa! Ganhei o maior prêmio do sorteio! (puxa: interjei-
substantivo. Ele servirá para determiná-lo como um substantivo ção) (tom da fala: euforia)
singular e feminino. - Puxa! Hoje não foi meu dia de sorte! (puxa: interjeição)
A dona da casa não quis nos atender. (tom da fala: decepção)
Como posso fazer a Joana concordar comigo?
- Quando é preposição, além de ser invariável, liga dois As interjeições são palavras invariáveis, isto é, não sofrem
termos e estabelece relação de subordinação entre eles. variação em gênero, número e grau como os nomes, nem de
Cheguei a sua casa ontem pela manhã. número, pessoa, tempo, modo, aspecto e voz como os verbos.
Não queria, mas vou ter que ir a outra cidade para procurar No entanto, em uso específico, algumas interjeições sofrem va-
um tratamento adequado. riação em grau. Deve-se ter claro, neste caso, que não se trata
- Se for pronome pessoal oblíquo estará ocupando o lugar de um processo natural dessa classe de palavra, mas tão só uma
e/ou a função de um substantivo. variação que a linguagem afetiva permite. Exemplos: oizinho,
Temos Maria como parte da família. / A temos como parte bravíssimo, até loguinho.
da família.
Creio que conhecemos nossa mãe melhor que ninguém. / Creio CONJUNÇÃO
que a conhecemos melhor que ninguém.
Conjunção é a palavra invariável que liga duas orações ou
2. Algumas relações semânticas estabelecidas por meio das dois termos semelhantes de uma mesma oração. Por exemplo:
preposições:
Destino: Irei para casa. A menina segurou a boneca e mostrou quando viu as
Modo: Chegou em casa aos gritos. amiguinhas.
Lugar: Vou ficar em casa; Deste exemplo podem ser retiradas três informações:
Assunto: Escrevi um artigo sobre adolescência.
Tempo: A prova vai começar em dois minutos. 1-) segurou a boneca 2-) a menina mostrou 3-) viu
Causa: Ela faleceu de derrame cerebral. as amiguinhas
Fim ou finalidade: Vou ao médico para começar o
tratamento. Cada informação está estruturada em torno de um verbo:
Instrumento: Escreveu a lápis. segurou, mostrou, viu. Assim, há nessa frase três orações:
Posse: Não posso doar as roupas da mamãe. 1ª oração: A menina segurou a boneca 2ª oração: e mostrou
Autoria: Esse livro de Machado de Assis é muito bom. 3ª oração: quando viu as amiguinhas.
Companhia: Estarei com ele amanhã. A segunda oração liga-se à primeira por meio do “e”, e a
Matéria: Farei um cartão de papel reciclado. terceira oração liga-se à segunda por meio do “quando”. As
Meio: Nós vamos fazer um passeio de barco. palavras “e” e “quando” ligam, portanto, orações.
Origem: Nós somos do Nordeste, e você?
Conteúdo: Quebrei dois frascos de perfume. Observe: Gosto de natação e de futebol.
Oposição: Esse movimento é contra o que eu penso. Nessa frase as expressões de natação, de futebol são partes
Preço: Essa roupa sai por R$ 50 à vista. ou termos de uma mesma oração. Logo, a palavra “e” está
ligando termos de uma mesma oração.
INTERJEIÇÃO
Conjunção é a palavra invariável que liga duas
É a palavra que expressa emoções, sentimentos ou pensa- orações ou dois termos semelhantes de uma mesma
mentos súbitos. Trata-se de um recurso da linguagem afetiva, oração.
em que não há uma ideia organizada de maneira lógica, como
são as sentenças da língua, mas sim a manifestação de um sus- Morfossintaxe da Conjunção
piro, um estado da alma decorrente de uma situação particular,
um momento ou um contexto específico. Exemplos: As conjunções, a exemplo das preposições, não exercem
- Ah, como eu queria voltar a ser criança! (ah: expressão de propriamente uma função sintática: são conectivos.
um estado emotivo = interjeição)
- Hum! Esse cuscuz estava maravilhoso! (hum: expressão Classificação - Conjunções Coordenativas- Conjunções
de um pensamento súbito = interjeição) Subordinativas

As sentenças da língua costumam se organizar de forma ló- Conjunções coordenativas


gica: há uma sintaxe que estrutura seus elementos e os distribui Dividem-se em:
em posições adequadas a cada um deles. As interjeições, por
outro lado, são uma espécie de palavra-frase, ou seja, há uma - ADITIVAS: expressam a ideia de adição, soma.

38
LÍNGUA PORTUGUESA
Ex. Gosto de cantar e de dançar. a) Temos uma CS Explicativa, que indica uma justificativa
Principais conjunções aditivas: e, nem, não só...mas ou uma explicação do fato expresso na oração anterior.
também, não só...como também. b) As orações são coordenadas e, por isso, independentes
- ADVERSATIVAS: Expressam ideias contrárias, de oposição, uma da outra. Neste caso, há uma pausa entre as orações que
de compensação. vêm marcadas por vírgula.
Ex. Estudei, mas não entendi nada. Não atravesse a rua. Você pode ser atropelado.
Principais conjunções adversativas: mas, porém, contudo, b) Outra dica é, quando a oração que antecede a OC
todavia, no entanto, entretanto. (Oração Coordenada) vier com verbo no modo imperativo, ela
- ALTERNATIVAS: Expressam ideia de alternância. será explicativa.
Ou você sai do telefone ou eu vendo o aparelho. Façam silêncio, que estou falando. (façam= verbo imperativo)
Principais conjunções alternativas: Ou...ou, ora...ora, quer...
quer, já...já. 2º) Na frase “Precisavam enterrar os mortos em outra cidade
- CONCLUSIVAS: Servem para dar conclusões às orações. porque não havia cemitério no local.”
Ex. Estudei muito, por isso mereço passar. a) Temos uma OSA Causal, já que a oração subordinada
Principais conjunções conclusivas: logo, por isso, pois (parte destacada) mostra a causa da ação expressa pelo
(depois do verbo), portanto, por conseguinte, assim. verbo da oração principal. Outra forma de reconhecê-
- EXPLICATIVAS: Explicam, dão um motivo ou razão. Ex. É la é colocá-la no início do período, introduzida pela
melhor colocar o casaco porque está fazendo muito frio lá fora. conjunção como - o que não ocorre com a CS Explicativa.
Principais conjunções explicativas: que, porque, pois (antes Como não havia cemitério no local, precisavam enterrar os mortos
do verbo), porquanto. em outra cidade.
b) As orações são subordinadas e, por isso, totalmente
Conjunções subordinativas dependentes uma da outra.

- CAUSAIS Questões
Principais conjunções causais: porque, visto que, já que,
uma vez que, como (= porque). 01. Assinale o par de frases em que as palavras sublinhadas
Ele não fez o trabalho porque não tem livro. são substantivo e pronome, respectivamente:
- COMPARATIVAS (A) A imigração tornou-se necessária. / É dever cristão
Principais conjunções comparativas: que, do que, tão... praticar o bem.
como, mais...do que, menos...do que. (B) A Inglaterra é responsável por sua economia. / Havia
Ela fala mais que um papagaio. muito movimento na praça.
- CONCESSIVAS (C) Fale sobre tudo o que for preciso. / O consumo de
Principais conjunções concessivas: embora, ainda que, drogas é condenável.
mesmo que, apesar de, se bem que. (D) Pessoas inconformadas lutaram pela abolição. /
Indicam uma concessão, admitem uma contradição, um Pesca-se muito em Angra dos Reis.
fato inesperado. Traz em si uma ideia de “apesar de”. (E) Os prejudicados não tinham o direito de reclamar. /
Embora estivesse cansada, fui ao shopping. (= apesar de estar Não entendi o que você disse.
cansada)
Apesar de ter chovido fui ao cinema. 02. Assinale o item que só contenha preposições:
- CONFORMATIVAS (A) durante, entre, sobre
Principais conjunções conformativas: como, segundo, (B) com, sob, depois
conforme, consoante (C) para, atrás, por
Cada um colhe conforme semeia. (D) em, caso, após
Expressam uma ideia de acordo, concordância, (E) após, sobre, acima
conformidade.
- CONSECUTIVAS 03. Observe as palavras grifadas da seguinte frase:
Expressam uma ideia de consequência. “Encaminhamos a V. Senhoria cópia autêntica do Edital
Principais conjunções consecutivas: que (após “tal”, “tanto”, nº 19/82.” Elas são, respectivamente:
“tão”, “tamanho”). (A) verbo, substantivo, substantivo
Falou tanto que ficou rouco. (B) verbo, substantivo, advérbio
- FINAIS (C) verbo, substantivo, adjetivo
Expressam ideia de finalidade, objetivo. (D) pronome, adjetivo, substantivo
Todos trabalham para que possam sobreviver. (E) pronome, adjetivo, adjetivo
Principais conjunções finais: para que, a fim de que, porque
(=para que), 04. Assinale a opção em que a locução grifada tem valor
- PROPORCIONAIS adjetivo:
Principais conjunções proporcionais: à medida que, quanto (A) “Comprei móveis e objetos diversos que entrei a utilizar
mais, ao passo que, à proporção que. com receio.”
À medida que as horas passavam, mais sono ele tinha. (B) “Azevedo Gondim compôs sobre ela dois artigos.”
- TEMPORAIS (C) “Pediu-me com voz baixa cinquenta mil réis.”
Principais conjunções temporais: quando, enquanto, logo (D) “Expliquei em resumo a prensa, o dínamo, as serras...”
que. (E) “Resolvi abrir o olho para que vizinhos sem escrúpulos
Quando eu sair, vou passar na locadora. não se apoderassem do que era delas.”

Importante: 05. O “que” está com função de preposição na alternativa:


Diferença entre orações causais e explicativas (A) Veja que lindo está o cabelo da nossa amiga!
Quando estudamos Orações Subordinadas Adverbiais (B) Diz-me com quem andas, que eu te direi quem és.
(OSA) e Coordenadas Sindéticas (CS), geralmente nos (C) João não estudou mais que José, mas entrou na
deparamos com a dúvida de como distinguir uma oração causal Faculdade.
de uma explicativa. Veja os exemplos: (D) O Fiscal teve que acompanhar o candidato ao banheiro.
(E) Não chore que eu já volto.
1º) Na frase “Não atravesse a rua, porque você pode ser
atropelado”:

39
LÍNGUA PORTUGUESA
06. “Saberão que nos tempos do passado o doce amor era Formação da Voz Passiva
julgado um crime.”
(A) 1 preposição A voz passiva pode ser formada por dois processos:
(B) 3 adjetivos Analítico e Sintético.
(C) 4 verbos
(D) 7 palavras átonas Voz Passiva Analítica: Verbo Ser + particípio (-ado /
(E) 4 substantivos -ido) do verbo principal.
O relógio será substituído.
07. As expressões sublinhadas correspondem a um adjetivo, O trabalho foi finalizado.
exceto em:
(A) João Fanhoso anda amanhecendo sem entusiasmo. *O agente da passiva geralmente é acompanhado da
(B) Demorava-se de propósito naquele complicado preposição por, mas pode ocorrer a construção com a preposição
banho. de: A casa ficou cercada de soldados.
(C) Os bichos da terra fugiam em desabalada carreira.
(D) Noite fechada sobre aqueles ermos perdidos da caatinga **Pode acontecer ainda que o agente da passiva não esteja
sem fim. explícito na frase: A exposição será aberta amanhã.
(E) E ainda me vem com essa conversa de homem da roça.
***A variação temporal é indicada pelo verbo auxiliar “ser”,
08. Em “__ como se tivéssemos vivido sempre juntos”, a pois o seu particípio é invariável.
forma verbal está no: Observe a transformação das frases seguintes:
(A) imperfeito do subjuntivo;
(B) futuro do presente composto; Ele fez o trabalho. (pretérito perfeito do indicativo)
(C) mais-que-perfeito composto do indicativo; O trabalho foi feito por ele. (pretérito perfeito do indicativo)
(D) mais-que-perfeito composto do subjuntivo;
(E) futuro composto do subjuntivo. Ele faz o trabalho. (presente do indicativo)
O trabalho é feito por ele. (presente do indicativo)
Respostas
01. E / 02. A / 03. C / 04. E / 05. D / 06. E / 07. B Ele fará o trabalho. (futuro do presente)
/ 08. D O trabalho será feito por ele. (futuro do presente)

Vozes dos Verbos ****Nas frases com locuções verbais, o verbo “ser” assume o
mesmo tempo e modo do verbo principal da voz ativa. Observe
Os verbos podem assumir diferentes formas nas orações. A a transformação da frase seguinte: O vento ia levando as
maneira como são apresentadas as ações expressas pelo verbo folhas. (gerúndio); As folhas iam sendo levadas pelo vento.
indicam se a frase apresenta voz ativa, voz passiva ou voz (gerúndio)
reflexiva.
Observe os exemplos abaixo: *****É menos frequente a construção da voz passiva analítica
(1) Amanda maquiava Clarice. V O Z com outros verbos que podem eventualmente funcionar como
ATIVA auxiliares: A aluna ficou marcada pela piada.
(2) Clarice era maquiada por Amanda. VOZ PASSIVA
(3) Amanda maquiava-se. VOZ REFLEXIVA Voz Passiva Sintética: verbo na 3ª pessoa, seguido do
pronome apassivador “se”:
- Voz Ativa: sujeito pratica a ação expressa pelo verbo, ele Abriram-se as portas do estabelecimento.
é agente. Vendeu-se o prédio da escola.
O grupo de arquitetos elaborou a maquete.
(O grupo de arquitetos – sujeito agente) (elaborou – ação) *O agente não costuma vir expresso na voz passiva sintética.
(a maquete – objeto paciente)
Transformação da Voz Ativa na Voz Passiva
- Voz Passiva: sujeito é paciente, recebe a ação expressa O objeto da voz ativa será o sujeito voz passiva. O sujeito
pelo verbo. da ativa passará a agente da passiva. O verbo assumirá a forma
A maquete foi elaborada pelo grupo de arquitetos. passiva, conservando-se o tempo verbal.
(A maquete – sujeito paciente) (foi elaborada – ação) (pelo
grupo de arquitetas – agente da passiva) Gutenberg inventou a imprensa. (Voz Ativa)
Gutenberg – sujeito da Ativa
- Voz Reflexiva: Há dois tipos: a imprensa – Objeto Direto

Reflexiva: Recebe apenas a nomenclatura de “reflexiva” A imprensa foi inventada por Gutenberg (Voz Passiva)
quando o sujeito praticar a ação sobre si mesmo. A imprensa – Sujeito da Passiva
Exemplos: por Gutenberg – Agente da Passiva
- Pedro machucou-se.
- Ela cortou-se com a faca. Os professores têm orientado os alunos.
- Roberta olhava-se no espelho do carro. Os alunos têm sido orientados pelos professores.
Eu o acompanharei.
Reflexiva Recíproca: Recebe a nomenclatura de Ele será acompanhado por mim.
“reflexiva recíproca” quando houver dois elementos como
sujeito: um pratica a ação sobre o outro, que pratica a ação Observações:
sobre o primeiro = ação mútua.
Exemplos: *Quando o sujeito da voz ativa for indeterminado, não
- Marta e Renato amam-se. haverá complemento agente na passiva.
- Os rivais agrediram-se durante a festa. Exemplo: Enganaram-nos - Fomos enganados
- Os jovens abraçaram-se.
**Os verbos que não são ativos nem passivos ou reflexivos,
são chamados de neutros.

40
LÍNGUA PORTUGUESA
Exemplos: O suco é bom; Aqui não neva. processo mudando alguns detalhes do início, seriam grandes as
chances de não chegarmos a nada parecido com a inteligência.
***Os verbos chamar-se, batizar-se, operar-se (no sentido
cirúrgico) e vacinar-se são considerados passivos, logo o sujeito (Adaptado de Hélio Schwartsman. Folha de S. Paulo,
é paciente. 28/10/2012)
Chamo-me Felipe.
Batizei-me na Igreja matriz. ...alguns animais transformaram a capacidade de resolver
Operou-se no período da manhã. problemas em estratégia de sobrevivência.
Vacinaram-se contra o tétano. Transpondo-se a frase acima para a voz passiva, a forma
verbal resultante será:
Questões (A) transformam-se.
(B) foi transformada.
01. Desde o desenvolvimento da linguagem, há 5.000 (C) foram transformados.
anos, a espécie humana passou a ter seu caminho evolutivo (D) é transformado.
direcionado pela cultura, cujos impulsos foram superando (E) era transformada.
a limitação da biologia e os açoites da natureza. Foi pela
capacidade de pensar e de se comunicar que a humanidade 03. Assinale a alternativa INCORRETA quanto à classificação
obteve os meios para escapar da fome e da morte prematura. das vozes verbais:
O atual empuxo tecnológico se acelerou de tal forma que (A) Consertam-se bicicletas. (Voz passiva sintética)
alguns felizardos com acesso a todos os recursos disponíveis (B) Machucou-se com o canivete. (Voz reflexiva)
na vanguarda dos avanços médicos, biológicos, tecnológicos (C) Estaremos aqui pelos mesmos motivos. (Voz ativa)
e metabólicos podem realisticamente pensar em viver em (D) Alugaram-se as casas daquele bairro. (Voz passiva
boa saúde mental e física bem mais do que 100 anos. O analítica)
prolongamento da vida saudável, em razão de uma velhice sem Respostas
doenças, já foi só um exercício de visionários. Hoje é um campo 01. E / 02. B / 03. D
de pesquisa dos mais sérios e respeitados.
Robert Fogel, o principal formulador do conceito da
evolução tecnofísica, e outros estudiosos estão projetando os
limites dessa fabulosa caminhada cultural na qualidade de
vida dos seres humanos. Quando se dedicam a essa tarefa, os
estudiosos esbarram, em primeiro lugar, nas desigualdades de
renda e de acesso às inovações. Fazem parte das conjecturas Flexão nominal e verbal.
dos estudiosos a questão ambiental e a necessidade urgente de
obtenção e popularização de novas formas de energia menos
agressivas ao planeta.
(Adaptado de Revista Veja, 25 de abril de 2012 p Flexão Nominal
141)
Flexão de número
... que a humanidade obteve os meios ... Os nomes (substantivo, adjetivo etc.), de modo geral,
admitem a flexão de número: singular e plural: animal –
Transpondo-se a frase acima para a voz passiva, a forma animais.
verbal resultante será:
(A) seria obtido. Na maioria das vezes, acrescenta-se S: ponte –
(B) tinham obtido. pontes; bonito – bonitos.
(C) foi obtida.
(D) teriam sido obtidos. Palavras terminadas em R ou Z: acrescenta-se ES:
(E) foram obtidos. éter – éteres; avestruz – avestruzes. O pronome qualquer faz o
plural no meio: quaisquer
02. Em um belo artigo, o físico Marcelo Gleiser, analisando
a constatação do satélite Kepler de que existem muitos planetas Palavras oxítonas terminadas em S: acrescenta-se
com características físicas semelhantes ao nosso, reafirmou sua ES: ananás – ananases.
fé na hipótese da Terra rara, isto é, a tese de que a vida complexa
(animal) é um fenômeno não tão comum no Universo. As paroxítonas e as proparoxítonas são invariáveis
Gleiser retoma as ideias de Peter Ward expostas de modo Exemplos:
persuasivo em “Terra Rara”. Ali, o autor sugere que a vida RSLUHVïRVSLUHVR{QLEXVïRV{QLEXV
microbiana deve ser um fenômeno trivial, podendo pipocar até
em mundos inóspitos; já o surgimento de vida multicelular na Palavras terminadas em IL:
Terra dependeu de muitas outras variáveis físicas e históricas, átono: trocam IL por EIS: fóssil – fósseis.
o que, se não permite estimar o número de civilizações tônico: trocam L por S: funil – funis.
extraterráqueas, ao menos faz com que reduzamos nossas
expectativas. Palavras terminadas em EL:
Uma questão análoga só arranhada por Ward é a da átono: plural em EIS: nível – níveis.
inexorabilidade da inteligência. A evolução de organismos tônico: plural em ÉIS: carretel – carretéis.
complexos leva necessariamente à consciência e à inteligência?
Robert Wright diz que sim, mas seu argumento é mais Palavras terminadas em X são invariáveis: o clímax
matemático do que biológico: complexidade engendra ïRVFOtPD[.
complexidade, levando a uma corrida armamentista entre
espécies cujo subproduto é a inteligência. Há palavras cuja sílaba tônica avança: M~QLRU ï
Stephen J. Gould e Steven Pinker apostam que não. Para juniores; caráter – caracteres.
eles, é apenas devido a uma sucessão de pré-adaptações e
coincidências que alguns animais transformaram a capacidade A palavra Caracteres é plural tanto de caractere quanto
de resolver problemas em estratégia de sobrevivência. Se de caráter.
rebobinássemos o filme da evolução e reencenássemos o

41
LÍNGUA PORTUGUESA
Palavras terminadas em ão fazem o plural em ãos, Quando há dois verbos de sentido oposto: o perde-ganha –
ães e ões. os perde-ganha.
Em ões: balões, corações, grilhões, melões, gaviões. Nas frases substantivas (frases que se transformam em
Em ãos: pagãos, cristãos, cidadãos, bênçãos, órgãos. Os VXEVWDQWLYRV 2PDULDYDLFRPDVRXWUDVïRVPDULDYDLFRP
paroxítonos, como os dois últimos, sempre fazem o plural em as-outras.
ãos.
São invariáveis: arco-íris, louva-a-deus, sem-vergonha,
Em ães: escrivães, tabeliães, capelães, capitães, alemães. sem-teto e sem-terra: Os sem-terra apreciavam os arco-íris.

Em ões ou ãos: corrimões/corrimãos, verões/verãos, Admitem mais de um plural: SDLQRVVR ï SDLVQRVVRV


anões/anãos. RXSDLQRVVRVSDGUHQRVVRïSDGUHVQRVVRVRXSDGUHQRVVRV
WHUUDQRYD ï WHUUDVQRYDV RX WHUUDQRYDV VDOYRFRQGXWR ï
Em ões ou ães: charlatões/charlatães, guardiões/ VDOYRVFRQGXWRV RX VDOYRFRQGXWRV [HTXHPDWH ï [HTXHV
guardiães, cirurgiões/cirurgiães. PDWHVRX[HTXHVPDWHIUXWDSmRïIUXWDVSmHVRXIUXWDVSmR
JXDUGDPDULQKD ï JXDUGDVPDULQKDV RX JXDUGDVPDULQKD
Em ões, ãos ou ães: anciões/anciãos/anciães, ermitões/ Casos especiais: palavras que não se encaixam nas regras:
ermitãos/ermitães R EHPPHTXHU ï RV EHPPHTXHUHV R MRmRQLQJXpP ï RV
MR}HVQLQJXpPROXJDUWHQHQWHïRVOXJDUWHQHQWHVRPDSD
Plural dos diminutivos com a letra z: Coloca-se P~QGLïRVPDSDVP~QGL
a palavra no plural, corta-se o s e acrescenta-se zinhos (ou
zinhas): coraçãozinho – corações – coraçõe – coraçõezinhos. Flexão de Gênero
Os substantivos e as palavras que o acompanham na frase
Plural com metafonia (ô - ó): Algumas palavras, admitem a flexão de gênero: masculino e feminino: Meu amigo
quando vão ao plural, abrem o timbre da vogal “o”, outras diretor recebeu o primeiro salário. Minha amiga diretora
não. Com metafonia singular (ô) plural (ó): coro-coros; corvo- recebeu a primeira prestação. A flexão de feminino pode
corvos; destroço-destroços. Sem metafonia singular (ô) plural ocorrer de duas maneiras.
(ô): adorno-adornos; bolso-bolsos; transtorno-transtornos. Com a troca de o ou e por a: lobo – loba; mestre –
mestra.
Casos especiais:DYDODYDOHVHDYDLVFDOïFDOHVHFDLVFyV Por meio de diferentes sufixos nominais de gênero:
ïFRVHVHFyV²fel, feles e féis – mal e males – cônsul e cônsules. muitas vezes comalterações do radical: ateu – ateia; bispo –
episcopisa; conde – condessa; duque – duquesa; frade – freira;
Os dois elementos variam: Quando os compostos são ilhéu – ilhoa; judeu – judia; marajá – marani; monje – monja;
formados por substantivo mais palavra variável (adjetivo, pigmeu – pigmeia; píton – pitonisa; sandeu – sandia; sultão –
VXEVWDQWLYR QXPHUDO SURQRPH  DPRUSHUIHLWR ï DPRUHV sultana.
SHUIHLWRV FRXYHIORU ï FRXYHVIORUHV VHJXQGDIHLUD ï
segundas-feiras. Alguns substantivos são uniformes quanto ao gênero, ou
seja, possuem uma única forma para masculino e feminino.
Só o primeiro elemento varia: Podem ser:
Quando há preposição no composto, mesmo que oculta: pé- Sobrecomuns: admitem apenas um artigo, podendo
GHPROHTXHïSpVGHPROHTXHFDYDORYDSRUïFDYDORVYDSRU designar os dois sexos: a pessoa, o cônjuge, a testemunha.
(de ou a vapor). Comuns de dois gêneros: admitem os dois artigos,
Quando o segundo substantivo determina o primeiro (fim SRGHQGRHQWmRVHUPDVFXOLQRVRXIHPLQLQRVRHVWXGDQWHïD
RXVHPHOKDQoD EDQDQDPDomïEDQDQDVPDom VHPHOKDQWHD HVWXGDQWHRFLHQWLVWDïDFLHQWLVWDRSDWULRWDïDSDWULRWD
PDom QDYLRHVFRODïQDYLRVHVFROD DILQDOLGDGHpDHVFROD  Epicenos: admitem apenas um artigo, designando os
Alguns autores admitem a flexão dos dois elementos. É uma animais: O jacaré, a cobra, o polvo
situação polêmica: mangas-espada (preferível) ou mangas- O feminino de elefante é elefanta, e não elefoa. Aliás, é
espadas. Quando dizemos (e isso vai ocorrer outras vezes) que correto, mas designa apenas uma espécie de elefanta.
é uma situação polêmica, discutível, convém ter em mente que Mamão, para alguns gramáticos, deve ser considerado
a questão do concurso deve ser resolvida por eliminação, ou epiceno. É algo discutível.
seja, analisando bem as outras opções. Há substantivos de gênero duvidoso, que as pessoas
costumam trocar: champanha, aguardente, dó, alface, eclipse,
Apenas o último elemento varia: calformicida, cataplasma, grama (peso), grafite, milhar libido,
Quando os elementos são adjetivosKLVSDQRDPHULFDQRï plasma, soprano, musse, suéter, preá, telefonema.
hispano-americanos. A exceção é surdo-mudo, em que os dois Existem substantivos que admitem os dois gêneros: diabetes
adjetivos se flexionam: surdos-mudos. (ou diabete), laringe, usucapião etc.
Nos compostos em que aparecem os adjetivos grão, grã e
bel JUmRGXTXH ï JUmRGXTXHV JUmFUX] ï JUmFUX]HV EHO Flexão de Grau
SUD]HUïEHOSUD]HUHV Grau do substantivo
Quando o composto é formado por verbo ou qualquer Normal ou Positivo: sem nenhuma alteração.
elemento invariável (advérbio, interjeição, prefixo etc.) mais Aumentativo: Sintético: chapelão. Analítico: chapéu
substantivo ou adjetivoDUUDQKDFpXïDUUDQKDFpXVVHPSUH grande, chapéu enorme etc.
YLYDïVHPSUHYLYDVVXSHUKRPHPïVXSHUKRPHQV Diminutivo: Sintético: chapeuzinho. Analítico: chapéu
Quando os elementos são repetidos ou onomatopaicos pequeno, chapéu reduzido etc.
UHSUHVHQWDPVRQV UHFRUHFRïUHFRUHFRVSLQJXHSRQJXHï Um grau é sintético quando formado por sufixo; analítico,
SLQJXHSRQJXHVEHPWHYLïEHPte-vis. por meio de outras palavras.
Como se vê pelo segundo exemplo, pode haver alguma
alteração nos elementos, ou seja, não serem iguais. Se forem Grau do adjetivo
verbos repetidos, admite-se também pôr os dois no plural: Normal ou Positivo: João é forte.
SLVFDSLVFDïSLVFDSLVFDVRXSLVFDVSLVFDV Comparativo: de superioridade: João é mais forte
que André. (ou do que); de inferioridade: João é menos
Nenhum elemento varia: forte que André. (ou do que); de igualdade: João é tão forte
Quando há verbo mais palavra invariável: O cola-tudo – os quanto André. (ou como)
cola-tudo.

42
LÍNGUA PORTUGUESA
Superlativo: Absoluto: sintético: João é fortíssimo; Questões
analítico: João é muito forte (bastante forte, forte demais etc.);
Relativo: de superioridade: João é o mais forte da turma; de 01. Assinale o par de vocábulos que formam o plural como
inferioridade: João é o menos forte da turma. órfão e mata-burro, respectivamente:
O grau superlativo absoluto corresponde a um aumento (A) cristão / guarda-roupa
do adjetivo. Pode ser expresso por um sufixo (íssimo, érrimo (B) questão / abaixo-assinado
ou imo) ou uma palavra de apoio, como muito, bastante, (C) alemão / beija-flor
demasiadamente, enorme etc. As palavras maior, menor, (D) tabelião / sexta-feira
melhor, e pior constituem sempre graus de superioridade: (E) cidadão / salário-família
O carro é menor que o ônibus; menor (mais pequeno):
comparativo de superioridade. Ele é o pior do grupo; pior (mais 02. Relativamente à concordância dos adjetivos compostos
mau): superlativo relativo de superioridade. indicativos de cor, uma, dentre as seguintes, está errada. Qual?
Alguns superlativos absolutos sintéticos que podem (A) saia amarelo-ouro
apresentar dúvidas: acre – acérrimo, amargo – amaríssimo; (B) papel amarelo-ouro
amigo – amicíssimo; antigo – antiqüíssimo; cruel – crudelíssimo; (C) caixa vermelho-sangue
doce – dulcíssimo; fácil – facílimo; feroz – ferocíssimo; fiel – (D) caixa vermelha-sangue
fidelíssimo; geral – generalíssimo; humilde – humílimo; magro (E) caixas vermelho-sangue
– macérrimo; negro – nigérrimo; pobre – paupérrimo; sagrado
– sacratíssimo; sério – seriíssimo; soberbo – superbíssimo. Respostas
01. A/ 02. D
Flexão Verbal
As flexões verbais são expressas por meio dos tempos,
modo e pessoa da seguinte forma: O tempo indica o momen-
to em que ocorre o processo verbal; O modo indica a atitude do
falante (dúvida, certeza, impossibilidade, pedido, imposição,
etc.); A pessoa marca na forma do verbo a pessoa gramatical Concordância nominal e verbal.
do sujeito.
Tempos: Há tempos do presente, do passado (pretérito) e
Regência nominal e verbal.
do futuro.

Modo Concordância Nominal e Verbal


Modo Indicativo: Indica uma certeza relativa do falante
com referência ao que o verbo exprime; pode ocorrer no tempo Concordância é o mecanismo pelo qual algumas palavras
presente, passado ou futuro: alteram suas terminações para adequar-se à terminação de
outras palavras. Em português, distinguimos dois tipos de
Presente: Processo simultâneo ao ato da fala, fato
corriqueiro, habitual: Compro livros nesta livraria. Usa-se concordância: nominal, que trata das alterações do artigo,
também o presente com o valor de passado, passado histórico do numeral, dos pronomes adjetivos e dos adjetivos para
(nos contos, narrativas) concordar com o nome a que se referem, e verbal, que trata
das alterações do verbo, para concordar com o sujeito.
Tempos do Pretérito (passado): Exprimem processos
anteriores ao ato da fala. São eles: Concordância Nominal
- Pretérito Imperfeito: Exprime um
processo habitual, ou com duração no tempo: - Regra Geral: o adjetivo e as palavras adjetivas (artigo,
Naquela época eu cantava como um pássaro. numeral, pronome adjetivo) concordam em gênero e número
- Pretérito Perfeito: Exprime uma ação acabada: Paulo
com o nome a que se referem.
quebrou meu violão de estimação.
- Pretérito Mais-que-Perfeito: Exprime um processo
anterior a um processo acabado: Embora tivera deixado a Esta / observação / curta / desfaz o equívoco.
escola, ele nunca deixou de estudar. Esta (pronome adjetivo – feminino – singular)
observação (substantivo – feminino – singular)
Tempos do Futuro: Indicam processos que irão aconte- curta (adjetivo – feminino – singular)
cer:
- Futuro do Presente: Exprime um processo que ainda - Um só adjetivo qualificando mais de um
não aconteceu: Farei essa viagem no fim do ano. substantivo.
- Futuro do Pretérito: Exprime um processo posterior a Adjetivo posposto: quando um mesmo adjetivo qualifica dois
um processo que já passou: Eu faria essa viagem se não tivesse
ou mais substantivos e vem depois destes, há duas construções:
comprado o carro.
Modo Subjuntivo: Expressa incerteza, possibilidade ou 1- o adjetivo vai para o plural: Agia com calma e
dúvida em relação ao processo verbal e não está ligado com a pontualidade britânicas.
noção de tempo. Há três tempos: presente, imperfeito e futuro. 2- o adjetivo concorda com o substantivo mais próximo:
Quero que voltes para mim; Não pise na grama; É possível que Agia com calma e pontualidade britânica.
ele seja honesto; Espero que ele fique contente; Duvido que
ele seja o culpado; Procuro alguém que seja meu companheiro Sempre que se optar pelo plural, é preciso notar o seguinte:
para sempre; Ainda que ele queira, não lhe será concedida a se entre os substantivos houver ao menos um no masculino, o
vaga; Se eu fosse bailarina, estaria na Rússia; Quando eu tiver adjetivo assumirá a terminação do masculino: Fez tudo com
dinheiro, irei para as praias do nordeste. entusiasmo e paixão arrebatadores.
Modo Imperativo: Exprime atitude de ordem, pedido ou
Quando o adjetivo exprime uma qualidade tal que só
solicitação: Vai e não voltes mais.
Pessoa: A norma da língua portuguesa estabelece três cabe ao último substantivo, é óbvio que a concordância
pessoas: Singular: eu , tu , ele, ela. Plural: nós, vós, eles, obrigatoriamente se efetuará com este último: Alimentavam-se
elas. No português brasileiro é comum o uso do pronome de de arroz e carne bovina.
tratamento você (s) em lugar do tu e vós.

43
LÍNGUA PORTUGUESA
Adjetivo anteposto: quando um adjetivo qualifica dois nas quais é difícil enxergar o céu./ Duvido que exista paisagens
ou mais substantivos e vem antes destes, concorda com o dominicais mais urbanas.
substantivo mais próximo: Escolhestes má ocasião e lugar. (C) Talvez seja programa de quem vive em cidades
cinzentas, na qual é difícil enxergar o céu./ Duvido que existam
Quando o adjetivo anteposto aos substantivos funcionar paisagens dominicais mais urbana.
como predicativo, pode concordar com o mais próximo ou ir (D) Talvez seja programa de quem vive em cidades
para o plural: cinzentas, nas quais são difíceis enxergar o céu./ Duvido que
Estava quieta a casa, a vila e o campo. existam paisagens dominical mais urbana.
Estavam quietos a casa, a vila e o campo. (E) Talvez seja programa de quem vive em cidades
cinzentas, nas quais é difícil enxergar o céu./ Duvido que
- Verbo ser + adjetivo: Nos predicados nominais em existam paisagens dominicais mais urbanas.
que ocorre o verbo ser mais um adjetivo, formando expressões
do tipo é bom, é claro, é evidente, etc., há duas construções 02. Assinale a alternativa correta quanto à concordância
possíveis: nominal.
1- se o sujeito não vem precedido de nenhum modificador, (A) Ela mesmo fez a entrega da encomenda.
tanto o verbo quanto o adjetivo ficam invariáveis: Pizza é (B) Estou meia preocupada, disse a jovem senhora ao seu
bom; É proibido entrada. colega de trabalho.
2- se o sujeito vem precedido de modificador, tanto o verbo (C) É proibida a entrada de mercadorias ilegais nesta
quanto o predicativo concordam regularmente: A pizza é boa; cidade.
É proibida a entrada. (D) “Muito obrigado”, respondeu-me aquela senhora.
(E) Custou muito caro, naquele contexto, as despesas
- Palavras adverbiais x palavras adjetivas: há assumidas pelo casal.
palavras que ora têm função de advérbio, ora de adjetivo. Respostas
Quando funcionam como advérbio são invariáveis: Há 01. E / 02. C
ocasiões bastante oportunas.
Quando funcionam como adjetivo, concordam com o nome Concordância Verbal
a que se referem: Há bastantes razões para confiarmos na
proposta. - Regra Geral: o verbo concorda com seu sujeito em
Estão nesta classificação palavras como pouco, muito, pessoa e número.
bastante, barato, caro, meio, longe, etc. Eu contarei convosco.
Tu estavas enganado.
- Expressões “anexo” e “obrigado”: são palavras
adjetivas e, como tais, devem concordar com o nome a que se - Sujeito composto anteposto ao verbo: quando
referem. Exemplos: o sujeito composto vem anteposto ao verbo, este vai para o
Seguem anexas as listas de preços. plural: O sol e a lua brilhavam. Há casos em que, mesmo com
Seguem anexos os planos de aula. o sujeito composto anteposto, justifica-se o singular. Isto ocorre
basicamente em três situações:
Podemos colocar sob a mesma regra palavras como incluso,
quite, leso, mesmo e próprio. 1- quando o sujeito é formado de palavras sinônimas ou que
formam unidade de sentido: A coragem e o destemor fez dele
1- Alerta e menos são sempre invariáveis: um herói. É bom notar que, no mesmo caso, vale também o
Estamos alerta. plural. O singular, aqui, talvez se explique pela facilidade que
Há situações menos complicadas. temos em juntar, numa só unidade, conceitos sinônimos.
Há menos pessoas no local. 2- quando o sujeito é formado por núcleos dispostos em
gradação (ascendente ou descendente): Uma palavra, um
2- Em anexo é sempre invariável: gesto, um mínimo sinal bastava. No mesmo caso, cabe
Seguem, em anexo, as fotografias. também o plural. A construção com o verbo no singular é
compreensível: nas sequências gradativas, o último elemento é
- Expressões só e sós: quando equivale a somente, é sempre mais enfático, o que leva o verbo a concordar com ele.
advérbio e invariável; quando equivale a sozinho, é adjetivo e 3- quando o sujeito vem resumido por palavras como
variável. alguém, ninguém, cada um, tudo, nada: Alunos, mestres,
Só eles não concordam. diretores, ninguém faltou. Aqui não ocorre plural. É que o
Eles saíram sós. valor sintetizante do pronome (ninguém) é tão marcante que só
nos fica a ideia do conjunto e não das partes que o compõem.
A expressão a sós é invariável: Gostaria de ficar a sós por
uns momentos. - Sujeito composto posposto ao verbo: quando o
Questões sujeito composto vem depois do verbo, há duas construções
igualmente certas.
01. Se usadas no plural as palavras destacadas nas frases
– Talvez seja programa de quem vive em uma cidade cinzenta, 1- o verbo vai para o plural: Brilhavam o sol e a lua.
na qual é difícil enxergar o céu. / Duvido que exista paisagem 2- o verbo concorda com o núcleo mais próximo: Brilhava
dominical mais urbana. – elas assumem versão correta em o sol e a lua.
(A) Talvez seja programa de quem vive em cidades cinzenta
na qual é difícil enxergar o céu./ Duvido que exista paisagens Quando, porém, o sujeito composto vem posposto e o núcleo
dominical mais urbanas. mais próximo está no plural, o verbo só pode, obviamente, ir
(B) Talvez seja programa de quem vive em cidades cinzentas, para o plural: Já chegaram as revistas e o jornal.

44
LÍNGUA PORTUGUESA
- Sujeito composto de pessoas gramaticais - Sujeito constituído pelo pronome relativo que:
diferentes: quando o sujeito composto é formado de pessoas quando o sujeito for o pronome relativo que, o verbo concorda
gramaticais diferentes, o verbo vai para o plural, sempre na com o antecedente desse pronome.
pessoa gramatical de número mais baixo. Assim, quando Fui eu que prometi.
ocorrer: Foste tu que prometeste.
1ª e 2ª – o verbo vai para a 1ª do plural.
2ª e 3ª – o verbo vai para a 2ª do plural. - Sujeito constituído pelo pronome relativo quem:
1ª e 3ª – o verbo vai para a 1ª do plural. quando o sujeito de um verbo for o pronome relativo quem, há
duas construções possíveis:
Eu, tu e ele ficaremos aqui. 1- o verbo fica na 3ª pessoa do singular, concordando
regularmente com o sujeito (quem): Fui eu quem falou.
Quando o sujeito é formado pelo pronome tu mais uma 3ª 2- o verbo concorda com o antecedente: Fui eu quem falei.
pessoa, o verbo pode ir também para a 3ª pessoa do plural. Isto
se deve à baixa frequência de uso da segunda pessoa do plural: - Pronome indefinido plural seguido de pronome
Tu e ele chegaram (ou chegastes) a tempo. pessoal preposicionado: quando o sujeito é formado de
um pronome indefinido (ou interrogativo) no plural seguido de
- Verbo acompanhado do pronome se apassivador: um pronome pessoal preposicionado, há possibilidade de duas
quando o pronome se funciona como partícula apassivadora, o construções:
verbo concorda regularmente com o sujeito, que estará sempre 1- o verbo vai para a 3ª pessoa do plural, concordando
presente na oração. com o pronome indefinido ou interrogativo: Alguns de nós
Vende-se apartamento. partiram.
Vendem-se apartamentos. 2- o verbo concorda com o pronome pessoal que se segue
ao indefinido (ou interrogativo): Alguns de nós partimos.
- Verbo acompanhado do pronome se indicador de
indeterminação do sujeito: quando a indeterminação do Quando o pronome interrogativo ou indefinido estiver
sujeito é marcada pelo pronome se, o verbo fica necessariamente no singular, o verbo ficará, necessariamente, na 3ª pessoa do
no singular: Precisa-se de reforços (sujeito indeterminado). singular.
Alguém de nós falhou?
- Verbos dar, bater, soar: na indicação de horas, Qual de nós sairá?
concordam com a palavra horas, que é o sujeito dos respectivos
verbos. - Expressões um dos que, uma das que: quando o
Bateram dez horas. sujeito de um verbo for o pronome relativo que, nas expressões
Soou uma hora. um dos que, uma das que, o verbo vai para o plural (construção
dominante) ou fica no singular.
Pode ser que o sujeito deixe de ser o número das horas e Ele foi um dos que mais falaram.
passe a ser outro elemento da oração, o instrumento que bate Ele foi um dos que mais falou.
as horas, por exemplo. No caso, a concordância mudará: O Cada uma das construções corresponde a uma interpretação
relógio bateu dez horas. diferente do mesmo enunciador.
- Sujeito coletivo: quando o sujeito é formado por um Ele é um dentre aqueles que mais falaram.
substantivo coletivo no singular, o verbo fica no singular, Ele é um que mais falou dentre aqueles.
concordando com a forma do substantivo e não com a ideia: A
multidão aplaudiu o orador. Nesse caso, pode ocorrer também - Expressões mais de, menos de: quando o sujeito for
o plural em duas situações: constituído das expressões mais de, menos de, o verbo concorda
1- quando o coletivo vier distanciado do verbo: O povo, com o numeral que se segue à expressão.
apesar de toda a insistência e ousadia, não conseguiram Mais de um aluno saiu.
evitar a catástrofe. Mais de dois alunos saíram.
2- quando o coletivo, antecipado ao verbo, vier seguido
de um adjunto adnominal no plural: A multidão dos Com a expressão mais de um pode ocorrer o plural em duas
peregrinos caminhavam lentamente. situações:
1- quando o verbo dá ideia de ação recíproca: Mais de um
- Nomes próprios plurais: quando o sujeito é formado veículo se entrechocaram.
por nomes próprios de lugar que só têm a forma plural, há duas 2- quando a expressão mais de vem repetida: Mais de
construções: um padre, mais de um bispo estavam presentes.
1- se tais nomes vierem precedidos de artigo, o verbo
concordará com o artigo. - Expressões um e outro, nem um nem outro: quando
Os Estados Unidos progrediram muito. o sujeito é formado pelas expressões um e outro, nem um nem
O Amazonas corre volumoso pela floresta. outro, o verbo fica no singular ou plural.
Nem um nem outro concordou.
2- se tais nomes não vierem precedidos de artigo, o verbo Nem um nem outro concordaram.
ficará sempre no singular: Ø Minas Gerais elegeu seu senador.
O substantivo que segue a essas expressões deve ficar no
Quanto aos títulos de livros e nomes de obras, mesmo singular: Uma e outra coisa me atrai.
precedidos de artigo, são admissíveis duas construções: Quando núcleos de pessoas diferentes vêm precedidos de
Os lusíadas foi a glória das letras lusitanas. nem, o mais usual é o verbo no plural, na pessoa gramatical
Os lusíadas foram a glória das letras lusitanas. prioritária (a de número mais baixo): Nem eu nem ele
faltamos com a palavra.

45
LÍNGUA PORTUGUESA
- Sujeito constituído por pronome de tratamento: Quando um dos dois (predicativo ou sujeito) é nome de
quando o sujeito é formado por pronomes de tratamento, o pessoa, a concordância se faz com a pessoa.
verbo vai sempre para a 3ª pessoa (singular ou plural). Você é suas decisões.
Vossa Excelência se enganou. Suas preocupações era a filha.
Vossas Excelências se enganaram.
O verbo concorda com o pronome pessoal, seja este sujeito,
- Núcleos ligados por ou: quando os núcleos do sujeito seja predicativo.
vêm ligados pela conjunção ou, há duas construções: O professor sou eu.
1- o verbo fica no singular quando o ou tem valor Eu sou o professor.
excludente: Pedro ou Paulo será eleito papa. (a eleição de um
implica necessariamente a exclusão do outro) Nas indicações de hora, data e distância, o verbo ser,
2- o verbo vai para o plural, quando o ou não for excludente: impessoal, concorda com o predicativo.
Maça ou figo me agradam à sobremesa. (ambas as frutas me É uma hora.
agradam) São duas horas.

- Silepse: ocorre concordância siléptica quando o verbo Neste último caso (dias do mês) o verbo ser admite duas
não concorda com o sujeito que aparece expresso na frase, construções:
mas com um elemento implícito na mente de quem fala: Os É (dia) treze de maio.
brasileiros somos improvisadores. Está implícito que o São treze (dias) de maio.
falante (eu ou nós) está incluído entre os brasileiros.
O verbo ser, seguido de um quantificador, nas expressões
- Expressão haja vista: na expressão haja vista, a de peso, distância ou preço, fica invariável.
palavra vista é sempre invariável. O verbo haja pode ficar Quinze quilos é bastante.
invariável ou concordar com o substantivo que se segue à
expressão. Questões
Haja vista os últimos acontecimentos.
Hajam vista os últimos acontecimentos. 01. Aponte a alternativa cuja concordância verbal está
correta:
Admite-se ainda a construção: (A) A alta dos preços dos combustíveis irritam o povo.
Haja vista aos últimos acontecimentos. (B) Os Estados Unidos fica na América do Norte.
(C) Minhas costas está doendo.
- Verbo parecer seguido de infinitivo: o verbo (D) Ela foi uma das que chegou a tempo
parecer, seguido de infinitivo, admite duas construções: (E) A maioria dos brasileiros gosta de futebol.
1- flexiona-se o verbo parecer e não se flexiona o infinitivo:
Os montes parecem cair. 02. “existe um protocolo para identificar os focos”. Se
2- flexiona-se o infinitivo e não se flexiona o verbo parecer: colocássemos o termo “um protocolo” no plural, uma forma
Os montes parece caírem. verbal adequada para a substituição da forma verbal “existe”
seria:
- Verbo impessoais: os verbos impessoais ficam sempre (A) hão.
na 3ª pessoa do singular. (B) haviam.
Haverá sóis mais brilhantes. (C) há.
Também não se flexiona o verbo auxiliar que se põe junto a (D) houveram.
um verbo impessoal, formando uma locução verbal. (E) houve.
Deve fazer umas cinco horas que estou esperando. Respostas
01. E / 02. C
O verbo haver no sentido de existir ou de tempo passado e
o verbo fazer na indicação de tempo transcorrido ou fenômeno Regência Nominal e Verbal
da natureza são impessoais.
O verbo existir nunca é impessoal: tem sempre sujeito, com Regência Nominal
o qual concorda normalmente: Existirão protestos; Poderão
existir dúvidas. Assim como há verbos de sentido incompleto (transitivos),
Quando o verbo haver funciona como auxiliar de há também nomes de sentido incompletos. Substantivos,
outro verbo, deve concordar normalmente com o sujeito: Os adjetivos, e, certos advérbios, também podem, como no caso dos
convidados já haviam saído. verbos, solicitarem um complemento (complemento nominal)
para ampliar, ou mesmo, completar seu sentido: Tenho amor
- Verbo ser: a concordância do verbo ser oscila (nome de sentido incompleto) aos livros (compl. Nominal).
frequentemente entre o sujeito e o predicativo. Entre tantos O substantivo amor rege um complemento nominal
casos, podemos ressaltar: precedido da preposição (a). Portanto, a relação particular,
Quando o sujeito e o predicativo são nomes de coisas de entre o nome e seu complemento, vem sempre marcada por
números diferentes, o verbo concorda, de preferência, com o uma preposição: Estava ansioso para ouvir música.
que está no plural. Contudo, cabe observar, que certos substantivos e adjetivos
Tua vida são essas ilusões. admitem mais de uma regência, ou seja, mais de uma
Essas vaidades são o teu segredo. preposição. A escolha desta ou daquela preposição deve,
no entanto, obedecer às exigências da clareza, da eufonia e
Nesse caso, muitas vezes, faz-se a concordância com o adequar-se as diferentes nuanças do pensamento.
elemento a que se quer dar destaque.

46
LÍNGUA PORTUGUESA
Ao aprender a regência do verbo, você estará praticamente (B) indigno, odioso, perito
aprendendo a regência do nome cognato (que vem da mesma (C) leal, limpo, oneroso
raiz do verbo). É o caso, por exemplo, do verbo obedecer e (D) orgulhoso, rico, sedento
do nome obediência. O verbo obedecer exige a preposição (a), (E) oposto, pálido, sábio
que é a mesma exigida pelo nome obediência. De maneira,
que a regência deste verbo e deste nome resume-se na mesma Respostas
preposição (a). 01. B / 02. A / 03. D
Na regência nominal, não há tantos desencontros entre a
norma culta e a fala popular. Em todo caso, segue aqui uma lista Regência Verbal
de alguns nomes acompanhados das respectivas preposições:
O estudo da regência verbal nos ajuda a escrever melhor.
Acesso (a) - acessível [a, para] - acostumado [a, com] - Quanto à regência verbal, os verbos podem ser:
adequado [a] - admiração [a, por] - afável [com, para com] - Verbos Transitivos: Exigem complemento (objetos) para
- afeição [a, por] - aflito [com, por] - alheio [a, de] - aliado que tenham sentido completo. Podem ser: Transitivos Diretos;
[a, com] - alusão [a] – amante [de] – amigo [de] - amor [a, Transitivos Indiretos; Transitivos Diretos e Indiretos.
de, para com, por] – amoroso [com] - análogo [a] - ansioso - Verbos Intransitivos.
[de, para, por] - antipatia [a, contra, por] – aparentado [com]
- apologia [de] - apto [a, para] - assíduo [a, em] - atenção [a] Transitivos Diretos: Não possuem sentido completo, logo
- atento [a, em] - atencioso [com, para com] - aversão [a, para, precisam de um complemento (objeto). Esses complementos
por] - avesso [a] - ávido [de, por] - benéfico [a] - benefício [a] (sem preposição) são chamados de objetos diretos. Ex.: Maria
– bom [para]. comprou um livro.
“Um livro” é o complemento exigido pelo verbo. Ele não
Equivalente [a] - empenho [de, em, por] – entendido [em] está acompanhado de preposição. “Um livro” é o objeto direto.
– erudito [em] – escasso [de] – essencial [para] – estreito [de] Note que se disséssemos: “Maria comprou.” a frase estaria
- exato [em] - fácil [a, de, para] - facilidade [de, em, para] - incompleta, pois quem compra, compra alguma coisa. O verbo
falho [de, em] - falta [a] - fanático [por] - favorável [a] - fiel comprar é transitivo direto.
[a] - feliz [de, com, em, por] - fértil [de, em] – forte [em] - fraco
[em, de] – furioso [com] - grato [a] - graduado [a] - guerra Transitivos Indiretos: Também não possuem sentido
[a] - hábil [em] - habituado [a] - horror [a, de, por] - hostil completo, logo precisam de um complemento, só que desta
[a, contra, para com] - ida [a] – idêntico [a] - impaciência vez este complemento é acompanhado de uma preposição. São
[com] – impossibilidade [de, em] - impotente [para, contra] chamados de objetos indiretos. Ex. Gosto de filmes.
- impróprio [para] - imune [a, de] - inábil [para] - inacessível “De filmes” é o complemento exigido pelo verbo gostar, e ele
[a] – incansável [em] - incapaz [de, para] – incerto [em] - está acompanhado por uma preposição (de). Este complemento
inconsequente [com] – indeciso [em] - indiferente [a] – indigno é chamado de objeto indireto. O verbo gostar é transitivo
[de] - indulgente [com, para com] - inerente [a] – infiel [a] – indireto
influência [sobre] - ingrato [com] – insensível [a] - intolerante
[com] - invasão [de] – inútil [para] - isento [de] - junto [a, de] Transitivos Diretos e Indiretos: Exigem 2
- leal [a] - lento [em] – liberal [com]. complementos. Um com preposição, e outro sem. Ex. O garoto
ofereceu um livro ao colega.
Regência de Advérbios: Merecem menção estes três O verbo oferecer é transitivo direto e indireto. Quem
advérbios: longe [de], perto [de] e proximamente [a, de]. Todos oferece, oferece alguma coisa a alguém. Ofereceu alguma coisa
os advérbios formados de adjetivos + sufixo [-mente], tendem = Um brinquedo (sem preposição). Ofereceu para alguém =
a apresentar a mesma preposição dos adjetivos: Compatível ao colega (com preposição). ao = combinação da preposição a
[com]; compativelmente [com]. Relativo [a]; relativamente [a] com o artigo definido o.

Questões Intransitivos: não possuem complemento. Ou seja,


os verbos intransitivos possuem sentido completo. Ex: “Ele
01. O projeto.....estão dando andamento é incompatível..... morreu.” O verbo morrer tem sentido completo. Algumas vezes
tradições da firma. o verbo intransitivo pode vir acompanhado de algum termo
(A) de que, com as que indica modo, lugar, tempo, etc. Estes termos são chamados
(B) a que, com as de adjuntos adverbiais. Ex. Ele morreu dormindo. Dormindo
(C) que, as foi a maneira, o modo que ele morreu. Dormindo é o adjunto
(D) à que, às adverbial de modo.
(E) que, com as
Existem verbos intransitivos que precisam vir acompanhados
02. Quanto a amigos, prefiro João.....Paulo,.....quem de adjuntos adverbiais apenas para darem um sentido completo
sinto......simpatia. para a frase. Ex. Moro no Rio de Janeiro.
(A) a, por, menos O verbo morar é intransitivo, porém precisa do complemento
(B) do que, por, menos “no RJ’ para que a frase tenha um sentido completo. “No RJ” é
(C) a, para, menos o adj. adverbial de lugar.
(D) do que, com, menos
(E) do que, para, menos Aspirar: O verbo aspirar pode ser transitivo direto ou tran-
sitivo indireto.
03. Assinale a opção em que todos adjetivos podem ser Transitivo direto: quando significa “sorver”, “tragar”, “inspi-
seguidos pela mesma preposição: rar” e exige complemento sem preposição.
(A) ávido, bom, inconsequente - Ela aspirou o aroma das flores.

47
LÍNGUA PORTUGUESA
- Todos nós gostamos de aspirar o ar do campo. Esquecer – Lembrar:
Transitivo indireto: quando significa “pretender”, “desejar”, - Lembrar algo – esquecer algo
“almejar” e exige complemento com a preposição “a”. - Lembrar-se de algo – esquecer-se de algo (pronominal)
- O candidato aspirava a uma posição de destaque. No 1º caso, os verbos são transitivos diretos, ou seja exigem
- Ela sempre aspirou a esse emprego. complemento sem preposição.
- Ele esqueceu o livro.
Quando é transitivo indireto não admite a substituição pelos No 2º caso, os verbos são pronominais (-se, -me, etc) e exi-
pronomes lhe(s). Devemos substituir por “a ele(s)”, “a ela(s)”. gem complemento com a preposição “de”. São, portanto, tran-
- Aspiras a este cargo? sitivos indiretos.
- Sim, aspiro a ele. (e não “aspiro-lhe”). - Ele se esqueceu do caderno.
- Eu me esqueci da chave.
Assistir: O verbo assistir pode ser transitivo indireto, tran- - Eles se esqueceram da prova.
sitivo direto e intransitivo. - Nós nos lembramos de tudo o que aconteceu.
Transitivo indireto: quando significa “ver”, “presenciar”,
“caber”, “pertencer” e exige complemento com a preposição Há uma construção em que a coisa esquecida ou lembrada
“a”. passa a funcionar como sujeito e o verbo sofre leve alteração
- Assisti a um filme. (ver) de sentido. É uma construção muito rara na língua contem-
- Ele assistiu ao jogo. porânea, porém, é fácil encontrá-la em textos clássicos tanto
- Este direito assiste aos alunos. (caber) brasileiros como portugueses. Machado de Assis, por exemplo,
Transitivo direto: quando significa “socorrer”, “ajudar” e fez uso dessa construção várias vezes.
exige complemento sem preposição. - Esqueceu-me a tragédia. (cair no esquecimento)
- O médico assiste o ferido. (cuida) - Lembrou-me a festa. (vir à lembrança)
O verbo lembrar também pode ser transitivo direto e in-
Nesse caso o verbo “assistir” pode ser usado com a prepo- direto (lembrar alguma coisa a alguém ou alguém de alguma
sição “a”. coisa).
- Assistir ao paciente.
Preferir: É transitivo direto e indireto, ou seja, possui
Intransitivo: quando significa “morar” exige a preposição um objeto direto (complemento sem preposição) e um objeto
“em”. indireto (complemento com preposição)
- O papa assiste no Vaticano. (no: em + o) - Prefiro cinema a teatro.
- Eu assisto no Rio de Janeiro. - Prefiro passear a ver TV.
“No Vaticano” e “no Rio de Janeiro” são adjuntos adverbiais Não é correto dizer: “Prefiro cinema do que teatro”.
de lugar.
Simpatizar: Ambos são transitivos indiretos e exigem a
Chamar: O verbo chamar pode ser transitivo direto ou preposição “com”.
transitivo indireto. - Não simpatizei com os jurados.
É transitivo direto quando significa “convocar”, “fazer vir” e
exige complemento sem preposição. Querer: Pode ser transitivo direto (no sentido de “desejar”)
- O professor chamou o aluno. ou transitivo indireto (no sentido de “ter afeto”, “estimar”).
É transitivo indireto quando significa “invocar” e é usado - A criança quer sorvete.
com a preposição “por”. - Quero a meus pais.
- Ela chamava por Jesus.
Com o sentido de “apelidar” pode exigir ou não a preposi- Namorar: É transitivo direto, ou seja, não admite
ção, ou seja, pode ser transitivo direto ou transitivo indireto. preposição.
Admite as seguintes construções: - Maria namora João.
- Chamei Pedro de bobo. (chamei-o de bobo) Não é correto dizer: “Maria namora com João”.
- Chamei a Pedro de bobo. (chamei-lhe de bobo)
- Chamei Pedro bobo. (chamei-o bobo) Obedecer: É transitivo indireto, ou seja, exige complemento
- Chamei a Pedro bobo. (chamei-lhe bobo) com a preposição “a” (obedecer a).
- Devemos obedecer aos pais.
Visar: Pode ser transitivo direto (sem preposição) ou tran- Embora seja transitivo indireto, esse verbo pode ser usado
sitivo indireto (com preposição). na voz passiva.
Quando significa “dar visto” e “mirar” é transitivo direto. - A fila não foi obedecida.
- O funcionário já visou todos os cheques. (dar visto)
- O arqueiro visou o alvo e atirou. (mirar) Ver: É transitivo direto, ou seja, não exige preposição.
Quando significa “desejar”, “almejar”, “pretender”, “ter em - Ele viu o filme.
vista” é transitivo indireto e exige a preposição “a”. Questões
- Muitos visavam ao cargo.
- Ele visa ao poder. 01. (IFC - Auxiliar Administrativo - IFC). Todas as
Nesse caso não admite o pronome lhe(s) e deverá ser substi- alternativas estão corretas quanto ao emprego correto da
tuído por a ele(s), a ela(s). Ou seja, não se diz: viso-lhe. regência do verbo, EXCETO:
(A) Faço entrega em domicílio.
Quando o verbo “visar” é seguido por um infinitivo, a pre- (B) Eles assistem o espetáculo.
posição é geralmente omitida. (C) João gosta de frutas.
- Ele visava atingir o posto de comando. (D) Ana reside em São Paulo.
(E) Pedro aspira ao cargo de chefe.

48
LÍNGUA PORTUGUESA
02. Assinale a opção em que o verbo o sentido ficaria ambíguo: a água inundou a rua até a casa
chamar é empregado com o mesmo sentido que de Maria (= inundou inclusive a casa). Quando até significa
apresenta em __ “No dia em que o chamaram de Ubirajara, “perto de”, é preposição; quando significa “inclusive”, é
Quaresma ficou reservado, taciturno e mudo”: partícula de inclusão.
(A) pelos seus feitos, chamaram-lhe o salvador da pátria; - Com expressões repetitivas: Tomamos o remédio gota
(B) bateram à porta, chamando Rodrigo; a gota; Enfrentaram-se cara a cara.
(C) naquele momento difícil, chamou por Deus e pelo - Com expressões tomadas de maneira
Diabo; indeterminada: O doente foi submetido a dieta leve (no
(D) o chefe chamou-os para um diálogo franco; masc. = foi submetido a repouso, a tratamento prolongado,
(E) mandou chamar o médico com urgência. etc.); Prefiro terninho a saia e blusa (no masc. = prefiro
terninho a vestido).
Respostas - Antes de pronome interrogativo, não ocorre crase:
01.B / 02. A A que artista te referes?
- Na expressão valer a pena (no sentido de valer o
sacrifício, o esforço), não ocorre crase, pois o “a” é
artigo definido: Parodiando Fernando Pessoa, tudo vale a
pena quando a alma não é pequena...

Ocorrência da Crase. A Crase é Facultativa


- Antes de nomes próprios feminino: Enviamos um
telegrama à Marisa; Enviamos um telegrama a Marisa. Em
Crase português, antes de um nome de pessoa, pode-se ou não
empregar o artigo “a” (“A Marisa é uma boa menina”. Ou
Crase é a superposição de dois “a”, geralmente a preposição “Marisa é uma boa menina”). Por isso, mesmo que a preposição
“a” e o artigo a(s), podendo ser também a preposição “a” e o esteja presente, a crase é facultativa.
pronome demonstrativo a(s) ou a preposição “a” e o “a” inicial Quando o nome próprio feminino vier acompanhado de
dos pronomes demonstrativos aqueles(s), aquela(s) e aquilo. uma expressão que o determine, haverá crase porque o artigo
Essa superposição é marcada por um acento grave (`). definido estará presente. Dedico esta canção à Candinha do
Para haver crase, é indispensável a presença da preposição Major Quevedo. [A (artigo) Candinha do Major Quevedo é
“a”, que é um problema de regência. Por isso, quanto mais fanática por seresta.]
conhecer a regência de certos verbos e nomes, mais fácil será - Antes de pronome adjetivo possessivo feminino
para ele ter o domínio sobre a crase. singular: Pediu informações à minha secretária; Pediu
informações a minha secretária. A explicação é idêntica à do
Não existe Crase item anterior: o pronome adjetivo possessivo aceita artigo,
mas não o exige (“Minha secretária é exigente.” Ou: “A minha
- Antes de palavra masculina: Chegou a tempo ao secretária é exigente”). Portanto, mesmo com a presença da
trabalho; Vieram a pé; Vende-se a prazo. preposição, a crase é facultativa.
- Antes de verbo: Ficamos a admirá-los; Ele começou a
ter alucinações. - Com o pronome substantivo possessivo feminino
- Antes de artigo indefinido: Levamos a mercadoria a singular, o uso de acento indicativo de crase não é facultativo
uma firma; Refiro-me a uma pessoa educada. (conforme o caso será proibido ou obrigatório): A minha cidade
- Antes de expressão de tratamento introduzida é melhor que a tua. O acento indicativo de crase é proibido
pelos pronomes possessivos Vossa ou Sua ou ainda da porque, no masculino, ficaria assim: O meu sítio é melhor que o
expressão Você, forma reduzida de Vossa Mercê: Enviei teu (não há preposição, apenas o artigo definido). Esta gravura
dois ofícios a Vossa Senhoria; Traremos a Sua Majestade, o rei é semelhante à nossa. O acento indicativo de crase é obrigatório
Hubertus, uma mensagem de paz; Eles queriam oferecer flores porque, no masculino, ficaria assim: Este quadro é semelhante
a você. ao nosso (presença de preposição + artigo definido).
- Antes dos pronomes demonstrativos esta e essa:
Não me refiro a esta carta; Os críticos não deram importância Casos Especiais
a essa obra.
- Antes dos pronomes pessoais: Nada revelei a ela; - Nomes de localidades: Dentre as localidades, há as
Dirigiu-se a mim com ironia. que admitem artigo antes de si e as que não o admitem. Por
- Antes dos pronomes indefinidos com exceção de aí se deduz que, diante das primeiras, desde que comprovada
outra: Direi isso a qualquer pessoa; A entrada é vedada a toda a presença de preposição, pode ocorrer crase; diante das
pessoa estranha. Com o pronome indefinido outra(s), pode segundas, não. Para se saber se o nome de uma localidade
haver crase porque ele, às vezes, aceita o artigo definido a(s): aceita artigo, deve-se substituir o verbo da frase pelos verbos
As cartas estavam colocadas umas às outras (no masculino, estar ou vir. Se ocorrer a combinação “na” com o verbo estar ou
ficaria “os cartões estavam colocados uns aos outros”). “da” com o verbo vir, haverá crase com o “a” da frase original.
- Quando o “a” estiver no singular e a palavra Se ocorrer “em” ou “de”, não haverá crase: Enviou seus
seguinte estiver no plural: Falei a vendedoras desta firma; representantes à Paraíba (estou na Paraíba; vim da Paraíba); O
Refiro-me a pessoas curiosas. avião dirigia-se a Santa Catarina (estou em Santa Catarina; vim
- Quando, antes do “a”, existir preposição: Ela de Santa Catarina); Pretendo ir à Europa (estou na Europa; vim
compareceu perante a direção da empresa; Os papéis estavam da Europa). Os nomes de localidades que não admitem artigo
sob a mesa. Exceção feita, às vezes, para até, por motivo passarão a admiti-lo, quando vierem determinados. Porto
de clareza: A água inundou a rua até à casa de Maria (= a Alegre indeterminadamente não aceita artigo: Vou a Porto
água chegou perto da casa); se não houvesse o sinal da crase, Alegre (estou em Porto Alegre; vim de Porto Alegre); Mas,

49
LÍNGUA PORTUGUESA
acompanhando-se de uma expressão que a determine, passará barulho de ontem, houve...); A segunda expressão não aceita
a admiti-lo: Vou à grande Porto Alegre (estou na grande Porto preposição “a” (o “a” que aparece é artigo definido, não
Alegre; vim da grande Porto Alegre); Iríamos a Madri para ficar havendo, pois, crase): Dada a questão primordial envolvendo
três dias; Iríamos à Madri das touradas para ficar três dias. tal fato (= dado o problema primordial...); Dadas as respostas,
- Pronomes demonstrativos aquele(s), aquela(s), o aluno conferiu a prova (= dados os resultados...).
aquilo: quando a preposição “a” surge diante desses
demonstrativos, devemos sobrepor essa preposição à primeira Questões
letra dos demonstrativos e indicar o fenômeno mediante um
acento grave: Enviei convites àquela sociedade (= a + aquela); 01. O trecho a seguir reproduz uma parte de verbete do
A solução não se relaciona àqueles problemas (= a + aqueles); Michaelis - Moderno Dicionário da Língua Portuguesa.
Não dei atenção àquilo (= a + aquilo). A simples interpretação As.sis.tir (lat assistere) vti 1 Comparecer, estar presente:
da frase já nos faz concluir se o “a” inicial do demonstrativo é Assistir a um ofício divino. Tendo como base somente a
simples ou duplo. Entretanto, para maior segurança, podemos informação gramatical acima, assinale a seguir a única
usar o seguinte artifício: Substituir os demonstrativos aquele(s), alternativa de acordo com a norma padrão para a língua escrita.
aquela(s), aquilo pelos demonstrativos este(s), esta(s), isto, (A) Eu assisti a queda das torres gêmeas em 2001.
respectivamente. Se, antes destes últimos, surgir a preposição (B) Quando ela gritou eu assisti a cobra subindo.
“a”, estará comprovada a hipótese do acento de crase sobre o (C) Ele assistiu o jogo do Brasil ontem.
“a” inicial dos pronomes aquele(s), aquela(s), aquilo. Se não (D) Ela assistiu à apresentação do balé municipal.
surgir a preposição “a”, estará negada a hipótese de crase. (E) Ele assistiu a criança brincando.
Enviei cartas àquela empresa./ Enviei cartas a esta empresa;
A solução não se relaciona àqueles problemas./ A solução não 02. Leia os períodos:
se relaciona a estes problemas; Não dei atenção àquilo./ Não I) Os alunos saíram mais cedo da aula para assistir ao novo
dei atenção a isto; A solução era aquela apresentada ontem./ episódio de Jornada nas Estrelas.
A solução era esta apresentada ontem. II) Sou Frankenstein, o terrível, e sempre consigo o que
- Palavra “casa”: quando a expressão casa significa “lar”, estou a fim.
“domicílio” e não vem acompanhada de adjetivo ou locução III) Prefiro rosas, meu amor, à espinhos.
adjetiva, não há crase: Chegamos alegres a casa; Assim que saiu IV) Depois da manifestação na prefeitura, os alunos
do escritório, dirigiu-se a casa; Iremos a casa à noitinha. Mas, retornaram à escola.
se a palavra casa estiver modificada por adjetivo ou locução V) A estrada corre paralelamente a ferrovia e chega a
adjetiva, então haverá crase: Levaram-me à casa de Lúcia; cidadezinha.
- Palavra “terra”: Não há crase, quando a palavra terra
significa o oposto a “mar”, “ar” ou “bordo”: Os marinheiros Assinale a alternativa CORRETA:
ficaram felizes, pois resolveram ir a terra; Os astronautas (A) somente a afirmativa I está correta.
desceram a terra na hora prevista. Há crase, quando a palavra (B) somente as afirmativas II, III e IV estão corretas.
significa “solo”, “planeta” ou “lugar onde a pessoa nasceu”: O (C) somente as afirmativas I, II, IV e V estão corretas.
colono dedicou à terra os melhores anos de sua vida. (D) somente as afirmativas I e IV estão corretas.
- Palavra “distância”: Não se usa crase diante da palavra (E) somente as afirmativas I e II estão corretas.
distância, a menos que se trate de distância determinada: Via-
se um monstro marinho à distância de quinhentos metros; 03. Leia a frase e assinale a alternativa que contém os
Estávamos à distância de dois quilômetros do sítio, quando termos que preenchem, correta e respectivamente, as lacunas.
aconteceu o acidente. Mas: A distância, via-se um barco Entre os brasileiros_____ frente de negócios próprios
pesqueiro; Olhava-nos a distância. abertos______ menos de quatro anos, a porcentagem dos
- Pronome Relativo: Todo pronome relativo tem um que___ de 45___ 54 anos dobrou nesta década – de 7% em
substantivo (expresso ou implícito) como antecedente. Para 2001 para 15% hoje. (Veja, 15.07.2009)
saber se existe crase ou não diante de um pronome relativo, (A) à … à … têm … à
deve-se substituir esse antecedente por um substantivo (B) a … a … tem … à
masculino. Se o “a” se transforma em “ao”, há crase diante do (C) à … há … têm … a
relativo. Mas, se o “a” permanece inalterado ou se transforma (D) a … a … tem … a
em “o”, então não há crase: é preposição pura ou pronome (E) a … há … têm … à
demonstrativo
Respostas
A Crase é Obrigatória 01. D / 02. D / 03. C
- Sempre haverá crase em locuções prepositivas, locuções
adverbiais ou locuções conjuntivas que tenham como núcleo
um substantivo feminino.
- Sempre haverá crase em locuções que exprimem hora
determinada.
- Quando a expressão “à moda de” (ou “à maneira de”) Ortografia e acentuação.
estiver subentendida.
- Quando as expressões “rua”, “loja”, “estação de rádio”, etc.
estiverem subentendidas. Ortografia
- Quando está implícita uma palavra feminina.
- Não confundir devido com dado (a, os, as): a primeira A palavra ortografia é formada pelos elementos gregos
expressão pede preposição “a”, havendo crase antes de palavra orto “correto” e grafia “escrita” sendo a escrita correta das
feminina determinada pelo artigo definido. Devido à discussão palavras da língua portuguesa, obedecendo a uma combinação
de critérios etimológicos (ligados à origem das palavras) e
de ontem, houve um mal-estar no ambiente (= devido ao
fonológicos (ligados aos fonemas representados).

50
LÍNGUA PORTUGUESA
Somente a intimidade com a palavra escrita, é que acaba Parônimos: Registramos alguns parônimos que se
trazendo a memorização da grafia correta. Deve-se também diferenciam pela oposição das vogais /e/ e /i/, /o/ e /u/.
criar o hábito de consultar constantemente um dicionário. Fixemos a grafia e o significado dos seguintes:

Emprego da letra H área = superfície


ária = melodia, cantiga
Esta letra, em início ou fim de palavras, não tem valor arrear = pôr arreios, enfeitar
fonético; conservou-se apenas como símbolo, por força da arriar = abaixar, pôr no chão, cair
etimologia e da tradição escrita. Grafa-se, por exemplo, hoje, comprido = longo
porque esta palavra vem do latim hodie. cumprido = particípio de cumprir
Emprega-se o H: comprimento = extensão
- Inicial, quando etimológico: hábito, hélice, herói, hérnia, cumprimento = saudação, ato de cumprir
hesitar, haurir, etc. costear = navegar ou passar junto à costa
- Medial, como integrante dos dígrafos ch, lh e nh: chave, custear = pagar as custas, financiar
boliche, telha, flecha companhia, etc. deferir = conceder, atender
- Final e inicial, em certas interjeições: ah!, ih!, hem?, hum!, diferir = ser diferente, divergir
etc. delatar = denunciar
- Algumas palavras iniciadas com a letra H: hálito, dilatar = distender, aumentar
harmonia, hangar, hábil, hemorragia, hemisfério, heliporto, descrição = ato de descrever
hematoma, hífen, hilaridade, hipocondria, hipótese, hipocrisia, discrição = qualidade de quem é discreto
homenagear, hera, húmus; emergir = vir à tona
- Sem h, porém, os derivados baiano, baianinha, baião, imergir = mergulhar
baianada, etc. emigrar = sair do país
imigrar = entrar num país estranho
Emprego das letras E, I, O e U emigrante = que ou quem emigra
imigrante = que ou quem imigra
Na língua falada, a distinção entre as vogais átonas /e/ e /i/, eminente = elevado, ilustre
/o/ e /u/ nem sempre é nítida. É principalmente desse fato que iminente = que ameaça acontecer
nascem as dúvidas quando se escrevem palavras como quase, recrear = divertir
intitular, mágoa, bulir, etc., em que ocorrem aquelas vogais. recriar = criar novamente
soar = emitir som, ecoar, repercutir
Escrevem-se com a letra E suar = expelir suor pelos poros, transpirar
sortir = abastecer
- A sílaba final de formas dos verbos terminados em –uar: surtir = produzir (efeito ou resultado)
continue, habitue, pontue, etc. sortido = abastecido, bem provido, variado
- A sílaba final de formas dos verbos terminados em –oar: surtido = produzido, causado
abençoe, magoe, perdoe, etc. vadear = atravessar (rio) por onde dá pé, passar a vau
- As palavras formadas com o prefixo ante– (antes, anterior): vadiar = viver na vadiagem, vagabundear, levar vida de
antebraço, antecipar, antedatar, antediluviano, antevéspera, vadio
etc.
- Os seguintes vocábulos: Arrepiar, Cadeado, Candeeiro, Emprego das letras G e J
Cemitério, Confete, Creolina, Cumeeira, Desperdício, Destilar,
Disenteria, Empecilho, Encarnar, Indígena, Irrequieto, Para representar o fonema /j/ existem duas letras; g e j.
Lacrimogêneo, Mexerico, Mimeógrafo, Orquídea, Peru, Quase, Grafa-se este ou aquele signo não de modo arbitrário, mas de
Quepe, Senão, Sequer, Seriema, Seringa, Umedecer. acordo com a origem da palavra. Exemplos: gesso (do grego
gypsos), jeito (do latim jactu) e jipe (do inglês jeep).
Emprega-se a letra I
Escrevem-se com G
- Na sílaba final de formas dos verbos terminados em –air/–
oer /–uir: cai, corrói, diminuir, influi, possui, retribui, sai, etc. - Os substantivos terminados em –agem, -igem, -ugem:
- Em palavras formadas com o prefixo anti- (contra): garagem, massagem, viagem, origem, vertigem, ferrugem,
antiaéreo, Anticristo, antitetânico, antiestético, etc. lanugem. Exceção: pajem
- Nos seguintes vocábulos: aborígine, açoriano, artifício, - As palavras terminadas em –ágio, -égio, -ígio, -ógio,
artimanha, camoniano, Casimiro, chefiar, cimento, crânio, -úgio: contágio, estágio, egrégio, prodígio, relógio, refúgio.
criar, criador, criação, crioulo, digladiar, displicente, erisipela, - Palavras derivadas de outras que se grafam com g:
escárnio, feminino, Filipe, frontispício, Ifigênia, inclinar, massagista (de massagem), vertiginoso (de vertigem),
incinerar, inigualável, invólucro, lajiano, lampião, pátio, ferruginoso (de ferrugem), engessar (de gesso), faringite (de
penicilina, pontiagudo, privilégio, requisito, Sicília (ilha), faringe), selvageria (de selvagem), etc.
silvícola, siri, terebintina, Tibiriçá, Virgílio. - Os seguintes vocábulos: algema, angico, apogeu, auge,
estrangeiro, gengiva, gesto, gibi, gilete, ginete, gíria, giz,
Grafam-se com a letra O hegemonia, herege, megera, monge, rabugento, sugestão,
tangerina, tigela.
abolir, banto, boate, bolacha, boletim, botequim, bússola,
chover, cobiça, concorrência, costume, engolir, goela, mágoa, Escrevem-se com J
mocambo, moela, moleque, mosquito, névoa, nódoa, óbolo,
ocorrência, rebotalho, Romênia, tribo. - Palavras derivadas de outras terminadas em –já: laranja
(laranjeira), loja (lojista, lojeca), granja (granjeiro, granjense),
Grafam-se com a letra U gorja (gorjeta, gorjeio), lisonja (lisonjear, lisonjeiro), sarja
(sarjeta), cereja (cerejeira).
bulir, burburinho, camundongo, chuviscar, cumbuca, - Todas as formas da conjugação dos verbos terminados em
cúpula, curtume, cutucar, entupir, íngua, jabuti, jabuticaba, –jar ou –jear: arranjar (arranje), despejar (despejei), gorjear
lóbulo, Manuel, mutuca, rebuliço, tábua, tabuada, tonitruante, (gorjeia), viajar (viajei, viajem) – (viagem é substantivo).
trégua, urtiga. - Vocábulos cognatos ou derivados de outros que têm j:

51
LÍNGUA PORTUGUESA
laje (lajedo), nojo (nojento), jeito (jeitoso, enjeitar, projeção, - Verbos derivados de palavras cujo radical termina em –s:
rejeitar, sujeito, trajeto, trejeito). analisar (de análise), apresar (de presa), atrasar (de atrás),
- Palavras de origem ameríndia (principalmente tupi- extasiar (de êxtase), extravasar (de vaso), alisar (de liso), etc.
guarani) ou africana: canjerê, canjica, jenipapo, jequitibá, - Formas dos verbos pôr e querer e de seus derivados: pus,
jerimum, jiboia, jiló, jirau, pajé, etc. pusemos, compôs, impuser, quis, quiseram, etc.
- As seguintes palavras: alfanje, alforje, berinjela, cafajeste, - Os seguintes nomes próprios de pessoas: Avis, Baltasar,
cerejeira, intrujice, jeca, jegue, Jeremias, Jericó, Jerônimo, Brás, Eliseu, Garcês, Heloísa, Inês, Isabel, Isaura, Luís, Luísa,
jérsei, jiu-jítsu, majestade, majestoso, manjedoura, manjericão, Queirós, Resende, Sousa, Teresa, Teresinha, Tomás, Valdês.
ojeriza, pegajento, rijeza, sabujice, sujeira, traje, ultraje, - Os seguintes vocábulos e seus cognatos: aliás, anis, arnês,
varejista. ás, ases, através, avisar, besouro, colisão, convés, cortês,
- Atenção: Moji, palavra de origem indígena, deve ser escrita cortesia, defesa, despesa, empresa, esplêndido, espontâneo,
com J. Por tradição algumas cidades de São Paulo adotam a evasiva, fase, frase, freguesia, fusível, gás, Goiás, groselha,
grafia com G, como as cidades de Mogi das Cruzes e Mogi- heresia, hesitar, manganês, mês, mesada, obséquio, obus,
Mirim. paisagem, país, paraíso, pêsames, pesquisa, presa, presépio,
presídio, querosene, raposa, represa, requisito, rês, reses,
Representação do fonema /S/ retrós, revés, surpresa, tesoura, tesouro, três, usina, vasilha,
vaselina, vigésimo, visita.
O fonema /s/, conforme o caso, representa-se por:
- C, Ç: acetinado, açafrão, almaço, anoitecer, censura, Emprego da letra Z
cimento, dança, dançar, contorção, exceção, endereço, Iguaçu,
maçarico, maçaroca, maço, maciço, miçanga, muçulmano, - Os derivados em –zal, -zeiro, -zinho, -zinha, -zito, -zita:
muçurana, paçoca, pança, pinça, Suíça, suíço, vicissitude. cafezal, cafezeiro, cafezinho, avezinha, cãozito, avezita, etc.
- S: ânsia, ansiar, ansioso, ansiedade, cansar, cansado, - Os derivados de palavras cujo radical termina em –z:
descansar, descanso, diversão, excursão, farsa, ganso, cruzeiro (de cruz), enraizar (de raiz), esvaziar (de vazio), etc.
hortênsia, pretensão, pretensioso, propensão, remorso, sebo, - Os verbos formados com o sufixo –izar e palavras cognatas:
tenso, utensílio. fertilizar, fertilizante, civilizar, civilização, etc.
- SS: acesso, acessório, acessível, assar, asseio, assinar, - Substantivos abstratos em –eza, derivados de adjetivos
carrossel, cassino, concessão, discussão, escassez, escasso, e denotando qualidade física ou moral: pobreza (de pobre),
essencial, expressão, fracasso, impressão, massa, massagista, limpeza (de limpo), frieza (de frio), etc.
missão, necessário, obsessão, opressão, pêssego, procissão, - As seguintes palavras: azar, azeite, azáfama, azedo,
profissão, profissional, ressurreição, sessenta, sossegar, sossego, amizade, aprazível, baliza, buzinar, bazar, chafariz, cicatriz,
submissão, sucessivo. ojeriza, prezar, prezado, proeza, vazar, vizinho, xadrez.
- SC, SÇ: acréscimo, adolescente, ascensão, consciência,
consciente, crescer, cresço, descer, desço, desça, disciplina, Sufixo –ÊS e –EZ
discípulo, discernir, fascinar, florescer, imprescindível,
néscio, oscilar, piscina, ressuscitar, seiscentos, suscetível, - O sufixo –ês (latim –ense) forma adjetivos (às vezes
suscetibilidade, suscitar, víscera. substantivos) derivados de substantivos concretos: montês (de
- X: aproximar, auxiliar, auxílio, máximo, próximo, monte), cortês (de corte), burguês (de burgo), montanhês (de
proximidade, trouxe, trouxer, trouxeram, etc. montanha), francês (de França), chinês (de China), etc.
- XC: exceção, excedente, exceder, excelência, excelente, - O sufixo –ez forma substantivos abstratos femininos
excelso, excêntrico, excepcional, excesso, excessivo, exceto, derivados de adjetivos: aridez (de árido), acidez (de ácido),
excitar, etc. rapidez (de rápido), estupidez (de estúpido), mudez (de mudo)
avidez (de ávido) palidez (de pálido) lucidez (de lúcido), etc.
Homônimos
Sufixo –ESA e –EZA
acento = inflexão da voz, sinal gráfico
assento = lugar para sentar-se Usa-se –esa (com s):
acético = referente ao ácido acético (vinagre) - Nos seguintes substantivos cognatos de verbos terminados
ascético = referente ao ascetismo, místico em –ender: defesa (defender), presa (prender), despesa
cesta = utensílio de vime ou outro material (despender), represa (prender), empresa (empreender),
sexta = ordinal referente a seis surpresa (surpreender), etc.
círio = grande vela de cera - Nos substantivos femininos designativos de títulos
sírio = natural da Síria nobiliárquicos: baronesa, dogesa, duquesa, marquesa, princesa,
cismo = pensão consulesa, prioresa, etc.
sismo = terremoto - Nas formas femininas dos adjetivos terminados em –ês:
empoçar = formar poça burguesa (de burguês), francesa (de francês), camponesa (de
empossar = dar posse a camponês), milanesa (de milanês), holandesa (de holandês),
incipiente = principiante etc.
insipiente = ignorante - Nas seguintes palavras femininas: framboesa, indefesa,
intercessão = ato de interceder lesa, mesa, sobremesa, obesa, Teresa, tesa, toesa, turquesa, etc.
interseção = ponto em que duas linhas se cruzam
ruço = pardacento Usa-se –eza (com z):
russo = natural da Rússia - Nos substantivos femininos abstratos derivados de
adjetivos e denotando qualidades, estado, condição: beleza (de
Emprego de S com valor de Z belo), franqueza (de franco), pobreza (de pobre), leveza (de
leve), etc.
- Adjetivos com os sufixos –oso, -osa: gostoso, gostosa,
gracioso, graciosa, teimoso, teimosa, etc. Verbos terminados em –ISAR e –IZAR
- Adjetivos pátrios com os sufixos –ês, -esa: português,
portuguesa, inglês, inglesa, milanês, milanesa, etc. Escreve-se –isar (com s) quando o radical dos nomes
- Substantivos e adjetivos terminados em –ês, feminino – correspondentes termina em –s. Se o radical não terminar em
esa: burguês, burguesa, burgueses, camponês, camponesa, –s, grafa-se –izar (com z): avisar (aviso + ar), analisar (análise
camponeses, freguês, freguesa, fregueses, etc. + ar), alisar (a + liso + ar), bisar (bis + ar), catalisar (catálise

52
LÍNGUA PORTUGUESA
+ ar), improvisar (improviso + ar), paralisar (paralisia + ar), Quando a um elemento de composição terminado em vogal
pesquisar (pesquisa + ar), pisar, repisar (piso + ar), frisar seguir, sem interposição do hífen, palavra começada com /r/
(friso + ar), grisar (gris + ar), anarquizar (anarquia + izar), ou /s/: arroxeado, correlação, pressupor, bissemanal, girassol,
civilizar (civil + izar), canalizar (canal + izar), amenizar minissaia, etc.
(ameno + izar), colonizar (colono + izar), vulgarizar (vulgar Questões
+ izar), motorizar (motor + izar), escravizar (escravo + izar),
cicatrizar (cicatriz + izar), deslizar (deslize + izar), matizar 01. Assinale a alternativa em que todas as palavras estão
(matiz + izar). grafadas segundo a ortografia oficial.
(A) Diante da paralização das atividades dos agentes dos
Emprego do X correios, pede-se a compreenção de todos, pois ouve exceções
na distribuição dos processos.
- Esta letra representa os seguintes fonemas: (B) O revesamento dos funcionarios entre o Natal e o
Ch – xarope, enxofre, vexame, etc. Ano Novo será feito mediante sorteio, para que não ocorra
CS – sexo, látex, léxico, tóxico, etc. descriminação.
Z – exame, exílio, êxodo, etc. (C) Durante o período de recessão, os chefes serão
SS – auxílio, máximo, próximo, etc. encumbidos de controlar a imissão de faxes e copias xerox.
S – sexto, texto, expectativa, extensão, etc. (D) A concessão de férias obedece a critérios legais, o
mesmo ocorrendo com os casos de rescisão contratual.
- Não soa nos grupos internos –xce- e –xci-: exceção, exceder, (E) É certo que os cuidados com o educando devem dobrar
excelente, excelso, excêntrico, excessivo, excitar, inexcedível, durante a adolecencia, para que o jovem haja sempre de acordo
etc. com a lei.
- Grafam-se com x e não com s: expectativa, experiente,
expiar, expirar, expoente, êxtase, extasiado, extrair, fênix, texto, 02.
etc. O “gilete” dos tablets
- Escreve-se x e não ch: Num mundo capitalista como este em que vivemos, onde
Em geral, depois de ditongo: caixa, baixo, faixa, feixe, as empresa concorrem para posicionar suas marcas e fixar
frouxo, ameixa, rouxinol, seixo, etc. Excetuam-se caucho e os logotipos e slogans na cabeça dos consumidores, a síndrome do
derivados cauchal, recauchutar e recauchutagem. “Gillete” pode ser decisiva para a perpetuação de um produto. É
Geralmente, depois da sílaba inicial en-: enxada, enxame, isso que preocupa a concorrência do iPad, tablet da Apple.
enxamear, enxaguar, enxaqueca, enxergar, enxerto, enxoval, Assim como a marca de Lâminas de barbear tornou-se
enxugar, enxurrada, enxuto, etc. Excepcionalmente, grafam- sinônimo de todas categoria de barbeadores, eclipsando o
se com ch: encharcar (de charco), encher e seus derivados nome das marcas que ofericiam produtos similares, o mesmo
(enchente, preencher), enchova, enchumaçar (de chumaço), pode estar acontecendo com o tablet, lançado por Steve Jobs. O
enfim, toda vez que se trata do prefixo en- + palavra iniciada maior temor do mercado é que as pessoas passem a se referir
por ch. aos tablets como “iPad” em geral, dizendo “iPad da Samsung” ou
Em vocábulos de origem indígena ou africana: abacaxi, “iPad da Motorola”, e assim por diante.
xavante, caxambu, caxinguelê, orixá, maxixe, etc. [...] O mesmo se deu com os lenços Kleenex, os curativos
Nas seguintes palavras: bexiga, bruxa, coaxar, faxina, graxa, Band-aid e as fotocopiadoras Xerox. Resta saber se os
lagartixa, lixa, lixo, mexer, mexerico, puxar, rixa, oxalá, praxe, consumidores se habituarão com outros nomes para o produto
vexame, xarope, xaxim, xícara, xale, xingar, xampu. tecnológico.

Emprego do dígrafo CH No texto I, a palavra “gilete” (com inicial minúscula e


apenas uma letra “L” na segunda sílaba) compõe o título, ao
Escreve-se com ch, entre outros os seguintes vocábulos: passo que no primeiro parágrafo tem-se a forma “Gillette” (com
bucha, charque, charrua, chavena, chimarrão, chuchu, cochilo, inicial maiúscula e duas letras “L” na segunda sílaba). Julgue as
fachada, ficha, flecha, mecha, mochila, pechincha, tocha. afirmativas a respeito dessa diferença.
I. A diferença de grafia entre as duas formas é fruto de um
Homônimos erro de ortografia.
II. A diferença de grafia se dá devido “gilete”, do título, ser
Bucho = estômago um nome comum e “Gillette”, do primeiro parágrafo, um nome
Buxo = espécie de arbusto próprio.
Cocha = recipiente de madeira III. Há diferença entre as formas por “Gillette” ser parte
Coxa = capenga, manco do nome de um problema recorrente em economia chamado
Tacha = mancha, defeito; pequeno prego; prego de cabeça síndrome do “Gillette”.
larga e chata, caldeira. IV. Há diferença entre as formas por “gilete” ser a designação
Taxa = imposto, preço de serviço público, conta, tarifa de qualquer lâmina descartável de barbear e “Gillette”, uma
Chá = planta da família das teáceas; infusão de folhas do lâmina descartável de uma marca específica.
chá ou de outras plantas Estão corretas apenas as afirmativas
Xá = título do soberano da Pérsia (atual Irã) (A) I e III.
Cheque = ordem de pagamento (B) II e III.
Xeque = no jogo de xadrez, lance em que o rei é atacado (C) II e IV.
por uma peça adversária (D) III e IV.

Consoantes dobradas 03. Assinale a opção em que todas as palavras são vocábulos
de sentidos iguais ou aproximados:
- Nas palavras portuguesas só se duplicam as consoantes (A) Escopo; Intento; Mira; Tronco.
C, R, S. (B) Adiado; Adiantado; Delongado; Moroso.
- Escreve-se com CC ou CÇ quando as duas consoantes soam (C) Dúctil; Madeira; Lenha; Brando.
distintamente: convicção, occipital, cocção, fricção, friccionar, (D) Branco; Níveo; Cândido; Alvo.
facção, sucção, etc. (E) Tangerina; Bergamota; Jambo; Mexerica.
- Duplicam-se o R e o S em dois casos: Quando,
intervocálicos, representam os fonemas /r/ forte e /s/ Respostas
sibilante, respectivamente: carro, ferro, pêssego, missão, etc. 01. D / 02. C / 03. D

53
LÍNGUA PORTUGUESA
Acentuação gráfica Acentuação dos Vocábulos Oxítonos
Acentuam-se com acento adequado os vocábulos oxítonos
Tonicidade terminados em:
Num vocábulo de duas ou mais sílabas, há, em geral, uma - a, e, o, seguidos ou não de s: xará, serás, pajé, freguês,
que se destaca por ser proferida com mais intensidade que as vovô, avós, etc. Seguem esta regra os infinitivos seguidos de
outras: é a sílaba tônica. Nela recai o acento tônico, também pronome: cortá-los, vendê-los, compô-lo, etc.
chamado acento de intensidade ou prosódico. Exemplos: café, - em, ens: ninguém, armazéns, ele contém, tu conténs, ele
janela, médico, estômago, colecionador. convém, ele mantém, eles mantêm, ele intervém, eles intervêm,
O acento tônico é um fato fonético e não deve ser confundido etc.
com o acento gráfico (agudo ou circunflexo) que às vezes o
assinala. A sílaba tônica nem sempre é acentuada graficamente. Acentuação dos Monossílabos
Exemplo: cedo, flores, bote, pessoa, senhor, caju, tatus, Acentuam-se os monossílabos tônicos: a, e, o, seguidos ou
siri, abacaxis. não de s: há, pá, pé, mês, nó, pôs, etc.
As sílabas que não são tônicas chamam-se átonas (=fracas),
e podem ser pretônicas ou postônicas, conforme apareçam antes Acentuação dos Ditongos
ou depois da sílaba tônica. Exemplo: montanha, facilmente, Acentuam-se a vogal dos ditongos abertos éi, éu, ói, quando
heroizinho. tônicos.
De acordo com a posição da sílaba tônica, os vocábulos com Segundo as novas regras os ditongos abertos “éi” e “ói” não
mais de uma sílaba classificam-se em: são mais acentuados em palavras paroxítonas: assembléia,
platéia, idéia, colméia, boléia, Coréia, bóia, paranóia, jibóia,
Oxítonos: quando a sílaba tônica é a última: café, rapaz, apóio, heróico, paranóico, etc. Ficando: Assembleia, plateia,
escritor, maracujá. ideia, colmeia, boleia, Coreia, boia, paranoia, jiboia, apoio,
Paroxítonos: quando a sílaba tônica é a penúltima: mesa, heroico, paranoico, etc.
lápis, montanha, imensidade. Nos ditongos abertos de palavras oxítonas terminadas
Proparoxítonos: quando a sílaba tônica é a em éi, éu e ói e monossílabas o acento continua: herói,
antepenúltima: árvore, quilômetro, México. constrói, dói, anéis, papéis, troféu, céu, chapéu.

Monossílabos são palavras de uma só sílaba, conforme a Acentuação dos Hiatos


intensidade com que se proferem, podem ser tônicos ou átonos. A razão do acento gráfico é indicar hiato, impedir a
ditongação. Compare: caí e cai, doído e doido, fluído e fluido.
Monossílabos tônicos são os que têm autonomia - Acentuam-se em regra, o /i/ e o /u/ tônicos em hiato
fonética, sendo proferidos fortemente na frase em que com vogal ou ditongo anterior, formando sílabas sozinhas ou
aparecem: é, má, si, dó, nó, eu, tu, nós, ré, pôr, etc. com s: saída (sa-í-da), saúde (sa-ú-de), faísca, caíra, saíra,
egoísta, heroína, caí, Xuí, Luís, uísque, balaústre, juízo, país,
Monossílabos átonos são os que não têm autonomia cafeína, baú, baús, Grajaú, saímos, eletroímã, reúne, construía,
fonética, sendo proferidos fracamente, como se fossem sílabas proíbem, influí, destruí-lo, instruí-la, etc.
átonas do vocábulo a que se apoiam. São palavras vazias de - Não se acentua o /i/ e o /u/ seguidos de nh: rainha, fuinha,
sentido como artigos, pronomes oblíquos, elementos de ligação, moinho, lagoinha, etc; e quando formam sílaba com letra que
preposições, conjunções: o, a, os, as, um, uns, me, te, se, lhe, não seja s: cair (ca-ir), sairmos, saindo, juiz, ainda, diurno,
nos, de, em, e, que. Raul, ruim, cauim, amendoim, saiu, contribuiu, instruiu, etc.
De acordo com as novas regras da Língua Portuguesa não se
Acentuação dos Vocábulos Proparoxítonos acentua mais o /i/ e /u/ tônicos formando hiato quando vierem
Todos os vocábulos proparoxítonos são acentuados na vogal depois de ditongo: baiúca, boiúna, feiúra, feiúme, bocaiúva,
tônica: etc. Ficaram: baiuca, boiuna, feiura, feiume, bocaiuva, etc.
- Com acento agudo se a vogal tônica for i, u ou a, e, o Os hiatos “ôo” e “êe” não são mais acentuados: enjôo, vôo,
abertos: xícara, úmido, queríamos, lágrima, término, déssemos, perdôo, abençôo, povôo, crêem, dêem, lêem, vêem, relêem.
lógico, binóculo, colocássemos, inúmeros, polígono, etc. Ficaram: enjoo, voo, perdoo, abençoo, povoo, creem, deem,
- Com acento circunflexo se a vogal tônica for fechada leem, veem, releem.
ou nasal: lâmpada, pêssego, esplêndido, pêndulo, lêssemos,
estômago, sôfrego, fôssemos, quilômetro, sonâmbulo etc. Acento Diferencial
Emprega-se o acento diferencial como sinal distintivo de
Acentuação dos Vocábulos Paroxítonos vocábulos homógrafos, nos seguintes casos:
Acentuam-se com acento adequado os vocábulos - pôr (verbo) - para diferenciar de por (preposição).
paroxítonos terminados em: - verbo poder (pôde, quando usado no passado)
- ditongo crescente, seguido, ou não, de s: sábio, róseo, - é facultativo o uso do acento circunflexo para diferenciar
planície, nódua, Márcio, régua, árdua, espontâneo, etc. as palavras forma/fôrma. Em alguns casos, o uso do acento
- i, is, us, um, uns: táxi, lápis, bônus, álbum, álbuns, jóquei, deixa a frase mais clara. Exemplo: Qual é a forma da fôrma do
vôlei, fáceis, etc. bolo?
- l, n, r, x, ons, ps: fácil, hífen, dólar, látex, elétrons, fórceps,
etc. Segundo as novas regras da Língua Portuguesa não existe
- ã, ãs, ão, ãos, guam, guem: ímã, ímãs, órgão, bênçãos, mais o acento diferencial em palavras homônimas (grafia igual,
enxáguam, enxáguem, etc. som e sentido diferentes) como:
Não se acentua um paroxítono só porque sua vogal tônica - côa(s) (do verbo coar) - para diferenciar de coa, coas (com
é aberta ou fechada. Descabido seria o acento gráfico, por + a, com + as);
exemplo, em cedo, este, espelho, aparelho, cela, janela, socorro, - pára (3ª pessoa do singular do presente do indicativo do
pessoa, dores, flores, solo, esforços. verbo parar) - para diferenciar de para (preposição);

54
LÍNGUA PORTUGUESA
- péla (do verbo pelar) e em péla (jogo) - para diferenciar dizer e começar a te seguir. Nós não nos conhecemos. Mas você
de pela (combinação da antiga preposição per com os artigos saberá quando eu o retuitar ou mencionar seu nome no sítio,
ou pronomes a, as); e poderá falar comigo. Meus seguidores também podem se
- pêlo (substantivo) e pélo (v. pelar) - para diferenciar de interessar pelos seus tuítes e começar a seguir você. Em suma,
pelo (combinação da antiga preposição per com os artigos o, nós continuaremos não nos conhecendo, mas as pessoas que
os); estão ________ nossa volta podem virar amigas entre si.
- péra (substantivo - pedra) - para diferenciar de pera
(forma arcaica de para - preposição) e pêra (substantivo); Adaptado de: COSTA, C. C.. Disponível em:
- pólo (substantivo) - para diferenciar de polo (combinação <http://super.abril.com.br/cotidiano/como-internet-estamudando-
popular regional de por com os artigos o, os); amizade-619645.shtml>.Acesso em: 1º de outubro de 2012.
- pôlo (substantivo - gavião ou falcão com menos de um
ano) - para diferenciar de polo (combinação popular regional Considere as seguintes afirmações sobre acentuação gráfica.
de por com os artigos o, os); I. A palavra têm recebe acento gráfico pela mesma regra
que prescreve o uso do acento em alguém.
Emprego do Til II. A palavra você é acentuada pela mesma regra que
O til sobrepõe-se às letras “a” e “o” para indicar vogal nasal. determina o uso do acento em saberá.
Pode figurar em sílaba: III. A palavra tuítes, distintamente da palavra fluida, recebe
- tônica: maçã, cãibra, perdão, barões, põe, etc; acento gráfico porque apresenta duas vogais contíguas que
- pretônica: ramãzeira, balõezinhos, grã-fino, cristãmente, pertencem a sílabas diferentes.
etc;
- átona: órfãs, órgãos, bênçãos, etc. Quais estão corretas
(A) Apenas I.
Trema (o trema não é acento gráfico) (B) Apenas II.
Desapareceu o trema sobre o /u/ em todas as palavras (C) Apenas III.
do português: Linguiça, averiguei, delinquente, tranquilo, (D) Apenas II e III.
linguístico. Exceto em palavras de línguas estrangeiras: Günter, (E) I, II e III.
Gisele Bündchen, müleriano.
02.
Questões A água, ingrediente essencial à vida, certamente é o
recurso mais precioso de que a humanidade dispõe. Embora
01. se observe pelo mundo tanta negligência e falta de visão com
Um estudo feito pela Universidade de Michigan constatou relação a esse bem vital, é de se esperar que os seres humanos
que o que mais se faz no Facebook, depois de interagir com procurem preservar e manter os reservatórios naturais desse
amigos, é olhar os perfis de pessoas que acabamos de conhecer. líquido precioso. De fato, o futuro da espécie humana e de
Se você gostar do perfil, adicionará aquela pessoa, e estará muitas outras espécies pode ficar comprometido, a menos
formado um vínculo. No final, todo mundo vira amigo de que haja uma melhora significativa no gerenciamento dos
todo mundo. Mas, não é bem assim. As redes sociais têm o recursos hídricos. Entre esses fatores que mais têm afetado esse
poder de transformar os chamados elos latentes (pessoas recurso estão o crescimento populacional e a grande expansão
que frequentam o mesmo ambiente social, mas não são suas dos setores produtivos, como a agricultura e a indústria. Essa
amigas) em elos fracos – uma forma superficial de amizade. situação, responsável pelo consumo e também pela poluição da
Pois é, por mais que existam exceções _______qualquer regra, água em escala exponencial, tem conduzido à necessidade de
todos os estudos mostram que amizades geradas com a ajuda reformulação do seu gerenciamento. No ambiente agrícola, as
da Internet são mais fracas, sim, do que aquelas que nascem perspectivas de mudança decorrem das alterações do clima,
e se desenvolvem fora dela. Isso não é inteiramente ruim. que afetarão sensivelmente não só a disponibilidade de água,
Os seus amigos do peito geralmente são parecidos com você: mas também a sobrevivência de diversas espécies de animais
pertencem ao mesmo mundo e gostam das mesmas coisas. Os e vegetais. O atual estado de conhecimento técnico-científico
elos fracos, não. Eles transitam por grupos diferentes do seu e, nesse âmbito já permite a adoção e implementação de ténicas
por isso, podem lhe apresentar novas pessoas e ampliar seus direcionadas para o equilíbrio ambiental, porém o desafio está
horizontes – gerando uma renovação de ideias que faz bem em colocá-las em prática, uma vez que isso implica mudança
a todos os relacionamentos, inclusive às amizades antigas. O de comportamento e de atitude por parte do produtor, aliadas
problema é que a maioria das redes na Internet é simétrica: se à necessidade de uma política pública que valorize a adoção
você quiser ter acesso às informações de uma pessoa ou mesmo dessas medidas.
falar reservadamente com ela, é obrigado a pedir a amizade
dela. Como é meio grosseiro dizer “não” ________ alguém que Marco Antonio Ferreira Gomes e Lauro Charlet Pereira. Água no século XXI:
você conhece, todo mundo acaba adicionando todo mundo. E desafios e oportunidade. Internet: www.agsolve.com.br (com adaptações)
isso vai levando ________ banalização do conceito de amizade.
É verdade. Mas, com a chegada de sítios como o Twitter, ficou As palavras “negligência”, “reservatórios”, “espécie” e
diferente. Esse tipo de sítio é uma rede social completamente “equilíbrio” apresentam acentuação gráfica em decorrência da
assimétrica. E isso faz com que as redes de “seguidores” e mesma regra gramatical.
“seguidos” de alguém possam se comunicar de maneira muito Certo ( ) Errado ( )
mais fluida. Ao estudar a sua própria rede no Twitter, o sociólogo
Nicholas Christakis, da Universidade de Harvard, percebeu 03.Observe as palavras acentuadas, em destaque no
que seus amigos tinham começado a se comunicar entre si seguinte texto:
independentemente da mediação dele. Pessoas cujo único A Itália empreende atualmente uma revolução em sua
ponto em comum era o próprio Christakis acabaram ficando indústria vinícola, apresentando modernos e dinâmicos vinhos,
amigas. No Twitter, eu posso me interessar pelo que você tem a não abandonando seu inigualável caráter gastronômico.

55
LÍNGUA PORTUGUESA
Assinale a alternativa cujas palavras são acentuadas, que sofria desde moço se foi transformando em opressora asma
respectivamente, segundo as regras que determinam a cardíaca; os lábios grossos, o inferior um tanto tenso (...)”
acentuação das palavras destacadas no texto. (Visconde de Taunay)
(A) Saída; mostrará; hífen.
(B) Comprá-la; político; nível. Dois-Pontos ( : )
(C) Ócio; fenômeno; inútil. - iniciar a fala dos personagens: Então o padre respondeu:
(D) Dá-lo; anônima; estéril. - Parta agora.
(E) Eólica; órfã; ninguém. - antes de apostos ou orações apositivas, enumerações ou
sequência de palavras que explicam, resumem ideias anteriores:
Respostas Meus amigos são poucos: Fátima, Rodrigo e Gilberto.
01. D / 02. “CERTA” / 03. C - antes de citação: Como já dizia Vinícius de Morais: “Que o
amor não seja eterno posto que é chama, mas que seja infinito
enquanto dure.”

Ponto de Interrogação ( ? )
- Em perguntas diretas: Como você se chama?
Pontuação. - Às vezes, juntamente com o ponto de exclamação: Quem
ganhou na loteria? Você. Eu?!

Pontuação Ponto de Exclamação ( ! )


- Após vocativo: “Parte, Heliel!” ( As violetas de Nossa Sra.-
Os sinais de pontuação são sinais gráficos empregados na Humberto de Campos).
língua escrita para tentar recuperar recursos específicos da - Após imperativo: Cale-se!
língua falada, tais como: entonação, jogo de silêncio, pausas, - Após interjeição: Ufa! Ai!
etc. - Após palavras ou frases que denotem caráter emocional:
Que pena!
Ponto ( . )
- indicar o final de uma frase declarativa: Lembro-me muito Reticências ( ... )
bem dele. - indicar dúvidas ou hesitação do falante: Sabe...eu queria
- separar períodos entre si: Fica comigo. Não vá embora. te dizer que...esquece.
- nas abreviaturas: Av.; V. Ex.ª - interrupção de uma frase deixada gramaticalmente
incompleta: Alô! João está? Agora não se encontra. Quem sabe
Vírgula ( , ): É usada para marcar uma pausa do enunciado se ligar mais tarde...
com a finalidade de nos indicar que os termos por ela separados, - ao fim de uma frase gramaticalmente completa com a
apesar de participarem da mesma frase ou oração, não formam intenção de sugerir prolongamento de ideia: “Sua tez, alva e
uma unidade sintática: Lúcia, esposa de João, foi a ganhadora pura como um foco de algodão, tingia-se nas faces duns longes
única da Sena. cor-de-rosa...” (Cecília- José de Alencar)
Podemos concluir que, quando há uma relação sintática - indicar supressão de palavra (s) numa frase transcrita:
entre termos da oração, não se pode separá-los por meio de “Quando penso em você (...) menos a felicidade.” (Canteiros -
vírgula. Não se separam por vírgula: Raimundo Fagner)
- predicado de sujeito;
- objeto de verbo; Aspas ( “ ” )
- adjunto adnominal de nome; - isolar palavras ou expressões que fogem à norma culta,
- complemento nominal de nome; como gírias, estrangeirismos, palavrões, neologismos, arcaísmos
- predicativo do objeto; e expressões populares: Maria ganhou um apaixonado “ósculo”
- oração principal da subordinada substantiva (desde que do seu admirador; A festa na casa de Lúcio estava “chocante”;
esta não seja apositiva nem apareça na ordem inversa). Conversando com meu superior, dei a ele um “feedback” do
serviço a mim requerido.
Ponto-e-Vírgula ( ; ) - indicar uma citação textual: “Ia viajar! Viajei. Trinta e
quatro vezes, às pressas, bufando, com todo o sangue na face,
- separar os itens de uma lei, de um decreto, de uma petição, desfiz e refiz a mala”. (O prazer de viajar - Eça de Queirós)
de uma sequência, etc:
Art. 127 – São penalidades disciplinares: Se, dentro de um trecho já destacado por aspas, se fizer
I- advertência; necessário a utilização de novas aspas, estas serão simples. (
II- suspensão; ‘ ‘)
III- demissão;
IV- cassação de aposentadoria ou disponibilidade; Parênteses ( () )
V- destituição de cargo em comissão; - isolar palavras, frases intercaladas de caráter explicativo
VI- destituição de função comissionada. (cap. V das e datas: Na 2ª Guerra Mundial (1939-1945), ocorreu inúmeras
penalidades Direito Administrativo) perdas humanas; “Uma manhã lá no Cajapió (Joca lembrava-
se como se fora na véspera), acordara depois duma grande
- separar orações coordenadas muito extensas ou orações tormenta no fim do verão”. (O milagre das chuvas no nordeste-
coordenadas nas quais já tenham tido utilizado a vírgula: Graça Aranha)
“O rosto de tez amarelenta e feições inexpressivas, numa
quietude apática, era pronunciadamente vultuoso, o que mais Os parênteses também podem substituir a vírgula ou o
se acentuava no fim da vida, quando a bronquite crônica de travessão.

56
LÍNGUA PORTUGUESA
Travessão (__ ) (D) Não existe sociedade civilizada sem lei. As
- dar início à fala de um personagem: O filho perguntou: transformações tecnológicas apenas tornam mais complexas as
__Pai, quando começarão as aulas? regras que, muitas vezes incomodam e atrapalham, mas que,
- indicar mudança do interlocutor nos diálogos. __Doutor, o continuarão sendo garantias fundamentais de desenvolvimento
que tenho é grave? __Não se preocupe, é uma simples infecção. com justiça.
É só tomar um antibiótico e estará bom. (E) As transformações tecnológicas apenas tornam
- unir grupos de palavras que indicam itinerário: A rodovia
Belém-Brasília está em péssimo estado. mais complexas as regras que, muitas vezes, incomodam
Também pode ser usado em substituição à virgula, em e atrapalham, mas que continuarão sendo uma garantia
expressões ou frases explicativas: Xuxa – a rainha dos baixinhos fundamental de desenvolvimento com justiça. Não existe
– é loira. sociedade civilizada sem lei.
Parágrafo
Constitui cada uma das secções de frases de um escritor; Respostas
começa por letra maiúscula, um pouco além do ponto em que 01. A / 02. C / 03. D
começam as outras linhas.

Colchetes ( [] )
Utilizados na linguagem científica.
Equivalência e transformação de
Asterisco ( * )
estruturas.
Empregado para chamar a atenção do leitor para alguma
nota (observação).
Equivalência e transformação de estruturas
Barra ( / )
Aplicada nas abreviações das datas e em algumas
“Ideias confusas geram redações confusas”. Esta frase leva-
abreviaturas.
nos a refletir sobre a organização das ideias em um texto.
Significa dizer que, antes da redação, naturalmente devemos
+tIHQ ï
dominar o assunto sobre o qual iremos tratar e, posteriormente,
Usado para ligar elementos de palavras compostas e para
planejar o modo como iremos expô-lo, do contrário haverá
unir pronomes átonos a verbos. Exemplo: guarda-roupa
dificuldade em transmitir ideias bem acabadas. Portanto,
a leitura, a interpretação de textos e a experiência de vida
Questões
antecedem o ato de escrever.
Obtido um razoável conhecimento sobre o que iremos
01. Assinale dentre as alternativas a frase que apresenta
escrever, feito o esquema de exposição da matéria, é necessário
pontuação adequada:
saber ordenar as ideias em frases bem estruturadas. Logo, não
(A) Mãe, venha até meu quarto.
basta conhecer bem um determinado assunto, temos que o
(B) Curitiba 27 de outubro de 2012.
transmitir de maneira clara aos leitores.
(C) O menino, sentia-se mal.
O estudo da pontuação pode se tornar um valioso aliado
(D) Onde estão os nossos: pais, vizinhos.
para organizarmos as ideias de maneira clara em frases. Para
(E) Assim permite-se roupas, curtas.
tanto, é necessário ter alguma noção de sintaxe. “Sintaxe”,
conforme o dicionário Aurélio, é a “parte da gramática que
02. Assinale a opção em que o uso da vírgula é utilizado em
estuda a disposição das palavras na frase e a das frases no
uma expressão conclusiva.
discurso, bem como a relação lógica das frases entre si”; ou
(A) O tempo está feio, isto é, choverá ainda esta manhã.
em outras palavras, sintaxe quer dizer “mistura”, isto é, saber
(B) Daqui a pouco, iremos todos ao mercado.
misturar as palavras de maneira a produzirem um sentido
(C) O tempo está feio, portanto, choverá em breve.
evidente para os receptores das nossas mensagens. Observe:
(D) Gabriela, a bonita garota, está cheia de alegria.
(E) Cheios de esperança, os meninos saíram alegres.
1) A desemprego globalização no Brasil e no na está Latina
América causando.
03. Indique a opção em que o trecho está incorreto
2) A globalização está causando desemprego no Brasil e na
gramaticalmente.
América Latina.
(A) As transformações tecnológicas, já que não existe
sociedade civilizada sem lei, apenas tornam mais complexas
Ora, no item 1 não temos uma ideia, pois não há uma
as regras que, muitas vezes, incomodam e atrapalham,
frase, as palavras estão amontoadas sem a realização de “uma
mas que continuarão sendo uma garantia fundamental de
sintaxe”, não há um contexto linguístico nem relação inteligível
desenvolvimento com justiça.
com a realidade; no caso 2, a sintaxe ocorreu de maneira perfeita
(B) Não existe sociedade civilizada sem lei e as
e o sentido está claro para receptores de língua portuguesa
transformações tecnológicas apenas tornam mais complexas
inteirados da situação econômica e cultural do mundo atual.
as regras que, muitas vezes, incomodam e atrapalham,
mas que continuarão sendo uma garantia fundamental de
A Ordem dos Termos na Frase
desenvolvimento com justiça.
(C) Não existe sociedade civilizada sem lei, por isso as
Leia novamente a frase contida no item 2. Note que ela é
transformações tecnológicas apenas tornam mais complexas
organizada de maneira clara para produzir sentido. Todavia,
as regras que, muitas vezes, incomodam e atrapalham, mas,
há diferentes maneiras de se organizar gramaticalmente tal
no entanto, continuarão sendo uma garantia fundamental de
frase, tudo depende da necessidade ou da vontade do redator
desenvolvimento com justiça.
em manter o sentido, ou mantê-lo, porém, acrescentado ênfase

57
LÍNGUA PORTUGUESA
a algum dos seus termos. Significa dizer que, ao escrever, - A globalização está causando desemprego no Brasil e na
podemos fazer uma série de inversões e intercalações em América Latina.
nossas frases, conforme a nossa vontade e estilo. Tudo depende
da maneira como queremos transmitir uma ideia, do nosso Todavia, ao repetir qualquer um dos termos da oração por
estilo. Por exemplo, podemos expressar a mensagem da frase três vezes ou mais, então é necessário usar a vírgula, mesmo
2 da seguinte maneira: que estejamos usando a ordem direta. Esta é a regra básica nº1
para a colocação da vírgula. Veja:
- No Brasil e na América Latina, a globalização está causando
desemprego. A globalização, a tecnologia e a “ciranda financeira” causam
desemprego…
Neste caso, a mensagem é praticamente a mesma, apenas (três núcleos do sujeito)
mudamos a ordem das palavras para dar ênfase a alguns termos A globalização causa desemprego no Brasil, na América
(neste caso: No Brasil e na A. L.). Repare que, para obter a Latina e na África.
clareza tivemos que fazer o uso de vírgulas. (três adjuntos adverbiais)
Entre os sinais de pontuação, a vírgula é o mais usado e
o que mais nos auxilia na organização de um período, pois A globalização está causando desemprego, insatisfação e
facilita as boas “sintaxes”, boas misturas, ou seja, a vírgula sucateamento industrial no Brasil e na América Latina. (três
ajuda-nos a não “embolar” o sentido quando produzimos frases complementos verbais)
complexas. Com isto, “entregamos” frases bem organizadas aos
nossos leitores. 2) Em princípio, não devemos, na ordem direta, separar com
O básico para a organização sintática das frases é a ordem vírgula o sujeito e o verbo, nem o verbo e o seu complemento,
direta dos termos da oração. Os gramáticos estruturam tal nem o complemento e as circunstâncias, ou seja, não devemos
ordem da seguinte maneira: separar com vírgula os termos da oração. Veja exemplos de tal
incorreção:
SUJEITO + VERBO+ COMPLEMENTO VERBAL+
CIRCUNSTÂNCIAS O Brasil, será feliz. A globalização causa, o desemprego.
A globalização + está causando+ desemprego +
no Brasil nos dias de hoje. Ao intercalarmos alguma palavra ou expressão entre os
termos da oração, cabe isolar tal termo entre vírgulas, assim o
Nem todas as orações mantêm esta ordem e nem todas sentido da ideia principal não se perderá. Esta é a regra básica
contêm todos estes elementos, portanto cabem algumas nº2 para a colocação da vírgula. Dito em outras palavras:
observações: quando intercalamos expressões e frases entre os termos da
oração, devemos isolar os mesmos com vírgulas. Vejamos:
1) As circunstâncias (de tempo, espaço, modo, etc.)
A globalização, fenômeno econômico deste fim de século
normalmente são representadas por adjuntos adverbiais
XX, causa desemprego no Brasil.
de tempo, lugar, etc. Note que, no mais das vezes, quando
queremos recordar algo ou narrar uma história, existe a
Aqui um aposto à globalização foi intercalado entre o
tendência a colocar os adjuntos nos começos das frases:
sujeito e o verbo.
“No Brasil e na América…” “Nos dias de hoje…” “Nas
Outros exemplos
minhas férias…”, “No Brasil…”. e logo depois os verbos e outros
elementos: “Nas minhas férias fui…”; “No Brasil existe…”
A globalização, que é um fenômeno econômico e cultural,
está causando desemprego no Brasil e na América Latina.
Observações
Neste caso, há uma oração adjetiva intercalada.
a) tais construções não estão erradas, mas rompem com a As orações adjetivas explicativas desempenham
ordem direta; frequentemente um papel semelhante ao do aposto explicativo,
b) é preciso notar que em Língua Portuguesa, há muitas por isto são também isoladas por vírgula.
frases que não têm sujeito, somente predicado. Por exemplo:
Está chovendo em Porto Alegre. Faz frio em Friburgo. São A globalização causa, caro leitor, desemprego no Brasil…
quatro horas agora;
c) Outras frases são construídas com verbos intransitivos, Neste outro caso, há um vocativo entre o verbo e o seu
que não têm complemento: complemento.
O menino morreu na Alemanha, (sujeito +verbo+ adjunto
adverbial) A globalização causa desemprego, e isto é lamentável, no
A globalização nasceu no século XX. (idem) Brasil…
d) Há ainda frases nominais que não possuem verbos: cada
macaco no seu galho. Nestes tipos de frase, a ordem direta faz- Aqui, há uma oração intercalada (note que ela não pertence
se naturalmente. Usam-se apenas os termos existentes nelas. ao assunto: globalização, da frase principal, tal oração é apenas
um comentário à parte entre o complemento verbal e os
Levando em consideração a ordem direta, podemos adjuntos.
estabelecer três regras básicas para o uso da vírgula:
Obs: a simples negação em uma frase não exige vírgula:
1) Se os termos estão colocados na ordem direta não haverá
a necessidade de vírgulas. A frase (2) é um exemplo disto: A globalização não causou desemprego no Brasil e na
América Latina.

58
LÍNGUA PORTUGUESA
Quando “quebramos” a ordem direta, invertendo-a, tal o enunciado trazer uma frase no singular, por exemplo, e pedir
quebra torna a vírgula necessária. que o aluno passe a frase para o plural, mantendo o sentido.
Esta é a regra nº3 da colocação da vírgula. Outro exemplo é o enunciado dar a frase em um tempo verbal,
e pedir para que o aluno passe-a para outro tempo verbal.
No Brasil e na América Latina, a globalização está causando
desemprego… http://ricardovigna.wordpress.com/2009/02/02/estudos-de-
No fim do século XX, a globalização causou desemprego -linguagem-1-estrutura-frasal-e-pontuacao/
no Brasil…

Nota-se que a quebra da ordem direta frequentemente se


dá com a colocação das circunstâncias antes do sujeito. Trata-
se da ordem inversa. Estas circunstâncias, em gramática, são
representadas pelos adjuntos adverbiais. Muitas vezes, elas Redação.
são colocadas em orações chamadas adverbiais que têm uma
função semelhante a dos adjuntos adverbiais, isto é, denotam
tempo, lugar, etc. Exemplos: Tipos Textuais

Quando o século XX estava terminando, a globalização Para escrever um texto, necessitamos de técnicas que
começou a causar desemprego. implicam no domínio de capacidades linguísticas. Temos dois
momentos: o de formular pensamentos (o que se quer dizer)
Enquanto os países portadores de alta tecnologia e o de expressá-los por escrito (o escrever propriamente dito).
desenvolvem-se, a globalização causa desemprego nos países Fazer um texto, seja ele de que tipo for, não significa apenas
escrever de forma correta, mas sim, organizar ideias sobre
pobres.
determinado assunto.
E para expressarmos por escrito, existem alguns modelos
Durante o século XX, a Globalização causou desemprego no de expressão escrita: Descrição – Narração – Dissertação.
Brasil.
Descrição
Obs1: alguns gramáticos, Sacconi, por exemplo,
consideram que as orações subordinadas adverbiais devem ser š’Ù‡…ƒ”ƒ…–‡”À•–‹…ƒ•†‘••‡”‡•‘—†ƒ•…‘‹•ƒ•ǡƒ’”‡•‡–ƒ—ƒ
isoladas pela vírgula também quando colocadas após as suas ˜‹• ‘Ǣ
orações principais, mas só quando a) a oração principal tiver
uma extensão grande: por exemplo: A globalização causa… 2—–‹’‘†‡–‡š–‘ϐ‹‰—”ƒ–‹˜‘Ǣ
, enquanto os países…(vide frase acima); b) Se houver uma ‡–”ƒ–‘†‡’‡••‘ƒ•ǡƒ„‹‡–‡•ǡ‘„Œ‡–‘•Ǣ
outra oração após a principal e antes da oração adverbial: A ”‡†‘À‹‘†‡ƒ–”‹„—–‘•Ǣ
globalização causa desemprego no Brasil e as pessoas aqui
estão morrendo de fome , enquanto nos países portadores de •‘†‡˜‡”„‘•†‡Ž‹‰ƒ­ ‘Ǣ
alta tecnologia…  ”‡“—‡–‡‡’”‡‰‘†‡‡–žˆ‘”ƒ•ǡ…‘’ƒ”ƒ­Ù‡•‡‘—–”ƒ•
ϐ‹‰—”ƒ•†‡Ž‹‰—ƒ‰‡Ǣ
Obs2: quando os adjuntos adverbiais são mínimos, isto é, ‡…‘‘”‡•—Ž–ƒ†‘ƒ‹ƒ‰‡ϐÀ•‹…ƒ‘—’•‹…‘Ž×‰‹…ƒǤ
têm apenas uma ou duas palavras não há necessidade do uso
da vírgula: Narração

Hoje a globalização causa desemprego no Panamá.


š’Ù‡—ˆƒ–‘ǡ”‡Žƒ…‹‘ƒ—†ƒ­ƒ•†‡•‹–—ƒ­ ‘ǡƒ’‘–ƒƒ–‡•ǡ
Ali a globalização também causou…
†—”ƒ–‡‡†‡’‘‹•†‘•ƒ…‘–‡…‹‡–‘•ȋ‰‡”ƒŽ‡–‡ȌǢ
A não ser que queiramos dar ênfase: Aqui, a globalização…
2—–‹’‘†‡–‡š–‘•‡“—‡…‹ƒŽǢ
Obs3: na língua escrita, normalmente, ao realizarmos a ‡Žƒ–‘†‡ˆƒ–‘•Ǣ
ordem inversa, emprestamos ênfase aos termos que principiam
”‡•‡­ƒ†‡ƒ””ƒ†‘”ǡ’‡”•‘ƒ‰‡•ǡ‡”‡†‘ǡ…‡ž”‹‘ǡ–‡’‘Ǣ
as frases. Veja este exemplo de Rui Barbosa destacado por
Garcia: ’”‡•‡–ƒ­ ‘†‡—…‘ϐŽ‹–‘Ǣ
•‘†‡˜‡”„‘•†‡ƒ­ ‘Ǣ
“A mim, na minha longa e aturada e continua prática do
escrever, me tem sucedido inúmeras vezes, depois de considerar 
‡”ƒŽ‡–‡ǡ±‡•…Žƒ†ƒ†‡†‡•…”‹­Ù‡•Ǣ
por muito tempo necessária e insuprível uma locução nova, †‹žŽ‘‰‘†‹”‡–‘±ˆ”‡“—‡–‡Ǥ
encontrar vertida em expressões antigas mais clara, expressiva
e elegante a mesma ideia.” Dissertação

Estas três regras básicas não solucionam todos os problemas š’Ù‡—–‡ƒǡ‡š’Ž‹…ƒǡƒ˜ƒŽ‹ƒǡ…Žƒ••‹ϐ‹…ƒǡƒƒŽ‹•ƒǢ


de organização das frases, mas já dão um razoável suporte
2—–‹’‘†‡–‡š–‘ƒ”‰—‡–ƒ–‹˜‘Ǥ
para que possamos começar a ordenar a expressão das nossas
ideias. Em suma: o importante é não separar os termos básicos ‡ˆ‡•ƒ†‡—ƒ”‰—‡–‘ǣ
das orações, mas, se assim o fizermos, seja intercalando ou a) ƒ’”‡•‡–ƒ­ ‘†‡—ƒ–‡•‡“—‡•‡”ž†‡ˆ‡†‹†ƒǡ
invertendo elementos, então devemos usar a vírgula. b) †‡•‡˜‘Ž˜‹‡–‘‘—ƒ”‰—‡–ƒ­ ‘ǡ
c) ˆ‡…Šƒ‡–‘Ǣ
Observação: quanto à equivalência e transformação de ”‡†‘À‹‘†ƒŽ‹‰—ƒ‰‡‘„Œ‡–‹˜ƒǢ
estruturas, outro exemplo muito comum cobrado em provas é

59
LÍNGUA PORTUGUESA
- Como na descrição o que se reproduz é simultâneo, não
”‡˜ƒŽ‡…‡ƒ†‡‘–ƒ­ ‘Ǥ
existe relação de anterioridade e posterioridade entre seus
enunciados;
Carta
- Devem-se evitar os verbos e, se isso não for possível, que
se usem então as formas nominais, o presente e o pretério
 ••‡ ± — –‹’‘ †‡ –‡š–‘ “—‡ •‡ …ƒ”ƒ…–‡”‹œƒ ’‘” ‡˜‘Ž˜‡” — imperfeito do indicativo, dando-se sempre preferência aos
”‡‡–‡–‡‡—†‡•–‹ƒ–ž”‹‘Ǣ verbos que indiquem estado ou fenômeno.
- Todavia deve predominar o emprego das comparações,
2‘”ƒŽ‡–‡‡•…”‹–ƒ‡’”‹‡‹”ƒ’‡••‘ƒǡ‡•‡’”‡˜‹•ƒ— dos adjetivos e dos advérbios, que conferem colorido ao texto.
–‹’‘†‡Ž‡‹–‘”Ǣ
2‡…‡••ž”‹‘“—‡•‡—–‹Ž‹œ‡—ƒŽ‹‰—ƒ‰‡ƒ†‡“—ƒ†ƒ…‘‘ Para transformar uma descrição numa narração, bastaria
–‹’‘†‡†‡•–‹ƒ–ž”‹‘‡“—‡†—”ƒ–‡ƒ…ƒ”–ƒ ‘•‡’‡”…ƒƒ˜‹• ‘ introduzir um enunciado que indicasse a passagem de um
†ƒ“—‡Ž‡’ƒ”ƒ“—‡‘–‡š–‘‡•–ž•‡†‘‡•…”‹–‘Ǥ estado anterior para um posterior. No caso do texto II inicial,
para transformá-lo em narração, bastaria dizer: Reunia a isso
Descrição grande medo do pai. Mais tarde, Iibertou-se desse medo...

É a representação com palavras de um objeto, lugar, situação Características Linguísticas:


ou coisa, onde procuramos mostrar os traços mais particulares
ou individuais do que se descreve. É qualquer elemento que O enunciado narrativo, por ter a representação de
seja apreendido pelos sentidos e transformado, com palavras, um acontecimento, fazer-transformador, é marcado pela
em imagens. temporalidade, na relação situação inicial e situação final,
Sempre que se expõe com detalhes um objeto, uma pessoa enquanto que o enunciado descritivo, não tendo transformação,
ou uma paisagem a alguém, está fazendo uso da descrição. Não é atemporal.
é necessário que seja perfeita, uma vez que o ponto de vista do Na dimensão linguística, destacam-se marcas sintático-
observador varia de acordo com seu grau de percepção. Dessa semânticas encontradas no texto que vão facilitar a
forma, o que será importante ser analisado para um, não será compreensão:
para outro. - Predominância de verbos de estado, situação ou
A vivência de quem descreve também influencia na hora indicadores de propriedades, atitudes, qualidades, usados
de transmitir a impressão alcançada sobre determinado objeto, principalmente no presente e no imperfeito do indicativo (ser,
pessoa, animal, cena, ambiente, emoção vivida ou sentimento. estar, haver, situar-se, existir, ficar).
Evidentemente, quando se diz que a ordem dos enunciados - Ênfase na adjetivação para melhor caracterizar o que é
pode ser invertida, está-se pensando apenas na ordem descrito.
cronológica, pois, como veremos adiante, a ordem em que - Emprego de figuras (metáforas, metonímias, comparações,
os elementos são descritos produz determinados efeitos de sinestesias).
sentido. - Uso de advérbios de localização espacial.
Quando alteramos a ordem dos enunciados, precisamos
fazer certas modificações no texto, pois este contém anafóricos Recursos:
(palavras que retomam o que foi dito antes, como ele, os, aquele,
etc. ou catafóricos (palavras que anunciam o que vai ser dito, - Usar impressões cromáticas (cores) e sensações térmicas.
como este, etc.), que podem perder sua função e assim não ser Ex: O dia transcorria amarelo, frio, ausente do calor alegre do
compreendidos. Se tomarmos uma descrição como As flores sol.
manifestavam todo o seu esplendor. O Sol fazia-as - Usar o vigor e relevo de palavras fortes, próprias, exatas,
brilhar, ao invertermos a ordem das frases, precisamos fazer concretas. Ex: As criaturas humanas transpareciam um céu
algumas alterações, para que o texto possa ser compreendido: sereno, uma pureza de cristal.
O Sol fazia as flores brilhar. Elas manifestavam todo - As sensações de movimento e cor embelezam o poder da
o seu esplendor. Como, na versão original, o pronome natureza e a figura do homem. Ex: Era um verde transparente
oblíquo as é um anafórico que retoma flores, se alterarmos que deslumbrava e enlouquecia qualquer um.
a ordem das frases ele perderá o sentido. Por isso, precisamos - A frase curta e penetrante dá um sentido de rapidez
mudar a palavra flores para a primeira frase e retomá-la com do texto. Ex: Vida simples. Roupa simples. Tudo simples. O
o anafórico elas na segunda. pessoal, muito crente.
Por todas essas características, diz-se que o fragmento do
conto de Machado é descritivo. Descrição é o tipo de texto A descrição pode ser apresentada sob duas formas:
em que se expõem características de seres concretos (pessoas,
objetos, situações, etc.) consideradas fora da relação de Descrição Objetiva: quando o objeto, o ser, a cena, a
anterioridade e de posterioridade. passagem são apresentadas como realmente são, concretamente.
Ex: “Sua altura é 1,85m. Seu peso, 70 kg. Aparência atlética,
Características: ombros largos, pele bronzeada. Moreno, olhos negros, cabelos
- Ao fazer a descrição enumeramos características, negros e lisos”.
comparações e inúmeros elementos sensoriais; Não se dá qualquer tipo de opinião ou julgamento. Exemplo:
- As personagens podem ser caracterizadas física e “ A casa velha era enorme, toda em largura, com porta central
psicologicamente, ou pelas ações; que se alcançava por três degraus de pedra e quatro janelas de
- A descrição pode ser considerada um dos elementos guilhotina para cada lado. Era feita de pau-a-pique barreado,
constitutivos da dissertação e da argumentação; dentro de uma estrutura de cantos e apoios de madeira-de-lei.
- é impossível separar narração de descrição; Telhado de quatro águas. Pintada de roxo-claro. Devia ser mais
- O que se espera não é tanto a riqueza de detalhes, mas velha que Juiz de Fora, provavelmente sede de alguma fazenda
sim a capacidade de observação que deve revelar aquele que que tivesse ficado, capricho da sorte, na linha de passagem
a realiza; da variante do Caminho Novo que veio a ser a Rua Principal,
- Utilizam, preferencialmente, verbos de ligação. Exemplo: depois a Rua Direita – sobre a qual ela se punha um pouco de
“(...) Ângela tinha cerca de vinte anos; parecia mais velha esguelha e fugindo ligeiramente do alinhamento (...).” (Pedro
pelo desenvolvimento das proporções. Grande, carnuda, Nava – Baú de Ossos)
sanguínea e fogosa, era um desses exemplares excessivos do
sexo que parecem conformados expressamente para esposas Descrição Subjetiva: quando há maior participação da
da multidão (...)” (Raul Pompéia – O Ateneu); emoção, ou seja, quando o objeto, o ser, a cena, a paisagem são

60
LÍNGUA PORTUGUESA
transfigurados pela emoção de quem escreve, podendo opinar Descrição de pessoas (I):
ou expressar seus sentimentos. Ex: “Nas ocasiões de aparato é
que se podia tomar pulso ao homem. Não só as condecorações - Introdução: primeira impressão ou abordagem de qualquer
gritavam-lhe no peito como uma couraça de grilos. Ateneu! aspecto de caráter geral.
Ateneu! Aristarco todo era um anúncio; os gestos, calmos, - Desenvolvimento: características físicas (altura, peso, cor
soberanos, calmos, eram de um rei...” (“O Ateneu”, Raul da pele, idade, cabelos, olhos, nariz, boca, voz, roupas).
Pompéia) - Desenvolvimento: características psicológicas
“(...) Quando conheceu Joca Ramiro, então achou outra (personalidade, temperamento, caráter, preferências,
esperança maior: para ele, Joca Ramiro era único homem, par- inclinações, postura, objetivos).
de-frança, capaz de tomar conta deste sertão nosso, mandando - Conclusão: retomada de qualquer outro aspecto de caráter
por lei, de sobregoverno.” geral.
(Guimarães Rosa – Grande Sertão: Veredas)
Descrição de pessoas (II):
Os efeitos de sentido criados pela disposição dos elementos - Introdução: primeira impressão ou abordagem de qualquer
descritivos: aspecto de caráter geral.
Como se disse anteriormente, do ponto de vista da progressão - Desenvolvimento: análise das características físicas,
temporal, a ordem dos enunciados na descrição é indiferente, associadas às características psicológicas (1ª parte).
uma vez que eles indicam propriedades ou características que - Desenvolvimento: análise das características físicas,
ocorrem simultaneamente. No entanto, ela não é indiferente do associadas às características psicológicas (2ª parte).
ponto de vista dos efeitos de sentido: descrever de cima para - Conclusão: retomada de qualquer outro aspecto de caráter
baixo ou viceversa, do detalhe para o todo ou do todo para o geral.
detalhe cria efeitos de sentido distintos.
Uma descrição deve privilegiar o uso frequente de adjetivos, A descrição, ao contrário da narrativa, não supõe ação. É uma
também denominado adjetivação. Para facilitar o aprendizado estrutura pictórica, em que os aspectos sensoriais predominam.
desta técnica, sugerese que o concursando, após escrever seu Porque toda técnica descritiva implica contemplação e
texto, sublinhe todos os substantivos, acrescentando antes ou apreensão de algo objetivo ou subjetivo, o redator, ao descrever,
depois deste um adjetivo ou uma locução adjetiva. precisa possuir certo grau de sensibilidade. Assim como o pintor
capta o mundo exterior ou interior em suas telas, o autor de
Descrição de objetos constituídos de uma só parte: uma descrição focaliza cenas ou imagens, conforme o permita
- Introdução: observações de caráter geral referentes à sua sensibilidade.
procedência ou localização do objeto descrito.
- Desenvolvimento: detalhes (lª parte) formato (comparação Conforme o objetivo a alcançar, a descrição pode ser
com figuras geométricas e com objetos semelhantes); dimensões não-literária ou literária. Na descrição não-literária, há
(largura, comprimento, altura, diâmetro etc.) maior preocupação com a exatidão dos detalhes e a precisão
- Desenvolvimento: detalhes (2ª parte) material, peso, cor/ vocabular. Por ser objetiva, há predominância da denotação.
brilho, textura.
- Conclusão: observações de caráter geral referentes a sua Textos descritivos não-literários: A descrição técnica
utilidade ou qualquer outro comentário que envolva o objeto é um tipo de descrição objetiva: ela recria o objeto usando uma
como um todo. linguagem científica, precisa. Esse tipo de texto é usado para
descrever aparelhos, o seu funcionamento, as peças que os
Descrição de objetos constituídos por várias partes: compõem, para descrever experiências, processos, etc.
- Introdução: observações de caráter geral referentes à
procedência ou localização do objeto descrito. Textos descritivos literários: Na descrição literária
- Desenvolvimento: enumeração e rápidos comentários das predomina o aspecto subjetivo, com ênfase no conjunto de
partes que compõem o objeto, associados à explicação de como associações conotativas que podem ser exploradas a partir de
as partes se agrupam para formar o todo. descrições de pessoas; cenários, paisagens, espaço; ambientes;
- Desenvolvimento: detalhes do objeto visto como um todo situações e coisas. Vale lembrar que textos descritivos também
(externamente) formato, dimensões, material, peso, textura, podem ocorrer tanto em prosa como em verso.
cor e brilho.
- Conclusão: observações de caráter geral referentes a sua Narração
utilidade ou qualquer outro comentário que envolva o objeto
em sua totalidade. A Narração é um tipo de texto que relata uma história
real, fictícia ou mescla dados reais e imaginários. O texto
Descrição de ambientes: narrativo apresenta personagens que atuam em um tempo e
- Introdução: comentário de caráter geral.
- Desenvolvimento: detalhes referentes à estrutura global em um espaço, organizados por uma narração feita por um
do ambiente: paredes, janelas, portas, chão, teto, luminosidade narrador. É uma série de fatos situados em um espaço e no
e aroma (se houver). tempo, tendo mudança de um estado para outro, segundo
- Desenvolvimento: detalhes específicos em relação a objetos relações de sequencialidade e causalidade, e não simultâneos
lá existentes: móveis, eletrodomésticos, quadros, esculturas ou como na descrição. Expressa as relações entre os indivíduos,
quaisquer outros objetos. os conflitos e as ligações afetivas entre esses indivíduos e o
- Conclusão: observações sobre a atmosfera que paira no mundo, utilizando situações que contêm essa vivência.
ambiente. Todas as vezes que uma história é contada (é narrada),
o narrador acaba sempre contando onde, quando, como e
Descrição de paisagens: com quem ocorreu o episódio. É por isso que numa narração
- Introdução: comentário sobre sua localização ou qualquer predomina a ação: o texto narrativo é um conjunto de ações;
outra referência de caráter geral. assim sendo, a maioria dos verbos que compõem esse tipo de
- Desenvolvimento: observação do plano de fundo texto são os verbos de ação. O conjunto de ações que compõem
(explicação do que se vê ao longe). o texto narrativo, ou seja, a história que é contada nesse tipo de
- Desenvolvimento: observação dos elementos mais texto recebe o nome de enredo.
próximos do observador explicação detalhada dos elementos As ações contidas no texto narrativo são praticadas pelas
que compõem a paisagem, de acordo com determinada ordem. personagens, que são justamente as pessoas envolvidas
- Conclusão: comentários de caráter geral, concluindo acerca no episódio que está sendo contado. As personagens são
da impressão que a paisagem causa em quem a contempla. identificadas (nomeadas) no texto narrativo pelos substantivos

61
LÍNGUA PORTUGUESA
próprios. - Apresentação: é a parte do texto em que são
Quando o narrador conta um episódio, às vezes (mesmo apresentados alguns personagens e expostas algumas
sem querer) ele acaba contando “onde” (em que lugar) as circunstâncias da história, como o momento e o lugar onde a
ações do enredo foram realizadas pelas personagens. O lugar ação se desenvolverá.
onde ocorre uma ação ou ações é chamado de espaço, - Complicação: é a parte do texto em que se inicia
representado no texto pelos advérbios de lugar. propriamente a ação. Encadeados, os episódios se sucedem,
A história contada, por isso, passa por uma introdução conduzindo ao clímax.
(parte inicial da história, também chamada de prólogo), pelo - Clímax: é o ponto da narrativa em que a ação atinge seu
desenvolvimento do enredo (é a história propriamente dita, momento crítico, tornando o desfecho inevitável.
o meio, o “miolo” da narrativa, também chamada de trama) e - Desfecho: é a solução do conflito produzido pelas ações
termina com a conclusão da história (é o final ou epílogo). dos personagens.
Aquele que conta a história é o narrador, que pode ser
pessoal (narra em 1ª pessoa: Eu) ou impessoal (narra em Tipos de Personagens:
3ª pessoa: Ele).
Assim, o texto narrativo é sempre estruturado por verbos de Os personagens têm muita importância na construção de
ação, por advérbios de tempo, por advérbios de lugar e pelos um texto narrativo, são elementos vitais. Podem ser principais
substantivos que nomeiam as personagens, que são os agentes ou secundários, conforme o papel que desempenham no
do texto, ou seja, aquelas pessoas que fazem as ações expressas enredo, podem ser apresentados direta ou indiretamente.
pelos verbos, formando uma rede: a própria história contada. A apresentação direta acontece quando o personagem
Tudo na narrativa depende do narrador, da voz que conta aparece de forma clara no texto, retratando suas características
a história. físicas e/ou psicológicas, já a apresentação indireta se dá
quando os personagens aparecem aos poucos e o leitor vai
Elementos Estruturais (I): construindo a sua imagem com o desenrolar do enredo, ou seja,
a partir de suas ações, do que ela vai fazendo e do modo como
- Enredo: desenrolar dos acontecimentos. vai fazendo.
- Personagens: são seres que se movimentam, se
relacionam e dão lugar à trama que se estabelece na ação. - Em 1ª pessoa:
Revelam-se por meio de características físicas ou psicológicas.
Os personagens podem ser lineares (previsíveis), complexos, Personagem Principal: há um “eu” participante que
tipos sociais (trabalhador, estudante, burguês etc.) ou tipos conta a história e é o protagonista.
humanos (o medroso, o tímido, o avarento etc.), heróis ou
antiheróis, protagonistas ou antagonistas. - Em 3ª pessoa:
- Narrador: é quem conta a história.
- Espaço: local da ação. Pode ser físico ou psicológico. Onisciente: não há um eu que conta; é uma terceira
- Tempo: época em que se passa a ação. Cronológico: pessoa.
o tempo convencional (horas, dias, meses); Psicológico: o
tempo interior, subjetivo. Narrador Objetivo: não se envolve, conta a história
como sendo vista por uma câmara ou filmadora.
Elementos Estruturais (II):
Tipos de Discurso:
Personagens Quem? Protagonista/Antagonista
Acontecimento O quê? Fato Discurso Direto: o narrador passa a palavra diretamente
Tempo Quando? Época em que ocorreu o fato para o personagem, sem a sua interferência.
Espaço Onde? Lugar onde ocorreu o fato
Modo Como? De que forma ocorreu o fato Discurso Indireto: o narrador conta o que o personagem
Causa Por quê? Motivo pelo qual ocorreu o fato diz, sem lhe passar diretamente a palavra.
Resultado - previsível ou imprevisível.
Final - Fechado ou Aberto. Discurso Indireto-Livre: ocorre uma fusão entre a fala
do personagem e a fala do narrador. É um recurso relativamente
Esses elementos estruturais combinam-se e articulam-se de recente. Surgiu com romancistas inovadores do século XX.
tal forma, que não é possível compreendê-los isoladamente,
como simples exemplos de uma narração. Há uma relação Sequência Narrativa:
de implicação mútua entre eles, para garantir coerência e
verossimilhança à história narrada. Uma narrativa não tem uma única mudança, mas várias:
Quanto aos elementos da narrativa, esses não estão, uma coordena-se a outra, uma implica a outra, uma subordina-
obrigatoriamente sempre presentes no discurso, exceto as se a outra.
personagens ou o fato a ser narrado. A narrativa típica tem quatro mudanças de situação:
- uma em que uma personagem passa a ter um querer ou
Existem três tipos de foco narrativo: um dever (um desejo ou uma necessidade de fazer algo);
- uma em que ela adquire um saber ou um poder (uma
- Narrador-personagem: é aquele que conta a história competência para fazer algo);
na qual é participante. Nesse caso ele é narrador e personagem - uma em que a personagem executa aquilo que queria ou
ao mesmo tempo, a história é contada em 1ª pessoa. devia fazer (é a mudança principal da narrativa);
- Narrador-observador: é aquele que conta a história - uma em que se constata que uma transformação se deu e
como alguém que observa tudo que acontece e transmite ao em que se podem atribuir prêmios ou castigos às personagens
leitor, a história é contada em 3ª pessoa. (geralmente os prêmios são para os bons, e os castigos, para
- Narrador-onisciente: é o que sabe tudo sobre o enredo os maus).
e as personagens, revelando seus pensamentos e sentimentos
íntimos. Narra em 3ª pessoa e sua voz, muitas vezes, aparece Toda narrativa tem essas quatro mudanças, pois elas se
misturada com pensamentos dos personagens (discurso pressupõem logicamente. Com efeito, quando se constata a
indireto livre). realização de uma mudança é porque ela se verificou, e ela
efetua-se porque quem a realiza pode, sabe, quer ou deve fazê-
Estrutura: la. Tomemos, por exemplo, o ato de comprar um apartamento:

62
LÍNGUA PORTUGUESA
quando se assina a escritura, realiza-se o ato de compra; Apresentação da Narrativa:
para isso, é necessário poder (ter dinheiro) e querer ou dever
comprar (respectivamente, querer deixar de pagar aluguel ou - visual: texto escrito; legendas + desenhos (história em
ter necessidade de mudar, por ter sido despejado, por exemplo). quadrinhos) e desenhos.
Algumas mudanças são necessárias para que outras se - auditiva: narrativas radiofonizadas; fitas gravadas e discos.
deem. Assim, para apanhar uma fruta, é necessário apanhar - audiovisual: cinema; teatro e narrativas televisionadas.
um bambu ou outro instrumento para derrubá-la. Para ter um
carro, é preciso antes conseguir o dinheiro. Dissertação

Narrativa e Narração A dissertação é uma exposição, discussão ou interpretação


de uma determinada ideia. É, sobretudo, analisar algum tema.
Existe alguma diferença entre as duas? Sim. A narratividade Pressupõe um exame crítico do assunto, lógica, raciocínio,
é um componente narrativo que pode existir em textos que não clareza, coerência, objetividade na exposição, um planejamento
são narrações. A narrativa é a transformação de situações. Por de trabalho e uma habilidade de expressão.
exemplo, quando se diz “Depois da abolição, incentivou-se a É em função da capacidade crítica que se questionam
imigração de europeus”, temos um texto dissertativo, que, no pontos da realidade social, histórica e psicológica do mundo
entanto, apresenta um componente narrativo, pois contém e dos semelhantes. Vemos também, que a dissertação no seu
uma mudança de situação: do não incentivo ao incentivo da significado diz respeito a um tipo de texto em que a exposição
imigração européia. de uma ideia, através de argumentos, é feita com a finalidade
Se a narrativa está presente em quase todos os tipos de de desenvolver um conteúdo científico, doutrinário ou artístico.
texto, o que é narração?
A narração é um tipo de narrativa. Tem ela três Dissertação Expositiva e Argumentativa:
características:
- é um conjunto de transformações de situação A dissertação expositiva é voltada para aqueles fatos que
- é um texto figurativo, isto é, opera com personagens e estão sendo focados e discutidos pela grande mídia. É um tipo
fatos concretos de acontecimento inquestionável, mesmo porque todos os
- as mudanças relatadas estão organizadas de maneira tal detalhes já foram expostos na televisão, rádio e novas mídias.
que, entre elas, existe sempre uma relação de anterioridade e Já o texto dissertativo argumentativo vai fazer uma reflexão
posterioridade. maior sobre os temas. Os pontos de vista devem ser declarados
em terceira pessoa, há interações entre os fatos que se aborda.
Essa relação de anterioridade e posterioridade é sempre Tais fatos precisam ser esclarecidos para que o leitor se sinta
pertinente num texto narrativo, mesmo que a sequência linear convencido por tal escrita. Quem escreve uma dissertação
da temporalidade apareça alterada. Assim, por exemplo, argumentativa deve saber persuadir a partir de sua crítica de
no romance machadiano Memórias póstumas de Brás Cubas, determinado assunto. A linguagem jamais poderá deixar de ser
quando o narrador começa contando sua morte para em objetiva, com fatos reais, evidências e concretudes.
seguida relatar sua vida, a sequência temporal foi modificada.
No entanto, o leitor reconstitui, ao longo da leitura, as relações São partes da dissertação: Introdução /
de anterioridade e de posterioridade. Desenvolvimento / Conclusão.
Resumindo: na narração, as três características explicadas
acima (transformação de situações, figuratividade e relações Introdução: em que se apresenta o assunto; se apresenta a
de anterioridade e posterioridade entre os episódios relatados) ideia principal, sem, no entanto, antecipar seu desenvolvimento.
devem estar presentes conjuntamente. Um texto que tenha só Tipos:
uma ou duas dessas características não é uma narração.
- Divisão: quando há dois ou mais termos a serem
Esquema que pode facilitar a elaboração de seu texto discutidos. Ex: “Cada criatura humana traz duas almas consigo:
narrativo: uma que olha de dentro para fora, outra que olha de fora para
dentro...”
- Introdução: citar o fato, o tempo e o lugar, ou seja, o que - Alusão Histórica: um fato passado que se relaciona a um
aconteceu, quando e onde. fato presente. Ex: “A crise econômica que teve início no começo
- Desenvolvimento: causa do fato e apresentação dos dos anos 80, com os conhecidos altos índices de inflação que
personagens. a década colecionou, agravou vários dos históricos problemas
- Desenvolvimento: detalhes do fato. sociais do país. Entre eles, a violência, principalmente a urbana,
- Conclusão: consequências do fato. cuja escalada tem sido facilmente identificada pela população
brasileira.”
Tipologia da Narrativa Ficcional: - Proposição: o autor explicita seus objetivos.
- Convite: proposta ao leitor para que participe de alguma
- Romance coisa apresentada no texto. Ex: Você quer estar “na sua”? Quer
- Conto se sentir seguro, ter o sucesso pretendido? Não entre pelo cano!
- Crônica Faça parte desse time de vencedores desde a escolha desse
- Fábula momento!
- Lenda - Contestação: contestar uma ideia ou uma situação. Ex:
- Parábola “É importante que o cidadão saiba que portar arma de fogo não
- Anedota é a solução no combate à insegurança.”
- Poema Épico - Características: caracterização de espaços ou aspectos.
- Estatísticas: apresentação de dados estatísticos. Ex:
Tipologia da Narrativa NãoFiccional: “Em 1982, eram 15,8 milhões os domicílios brasileiros com
televisores. Hoje, são 34 milhões (o sexto maior parque de
- Memorialismo aparelhos receptores instalados do mundo). Ao todo, existem
- Notícias no país 257 emissoras (aquelas capazes de gerar programas) e
- Relatos 2.624 repetidoras (que apenas retransmitem sinais recebidos).
- História da Civilização (...)”
- Declaração Inicial: emitir um conceito sobre um fato.
- Citação: opinião de alguém de destaque sobre o assunto do

63
LÍNGUA PORTUGUESA
texto. Ex: “A principal característica do déspota encontra-se no Vejamos:
fato de ser ele o autor único e exclusivo das normas e das regras Ideia central: A poluição atmosférica deve ser combatida
que definem a vida familiar, isto é, o espaço privado. Seu poder, urgentemente.
escreve Aristóteles, é arbitrário, pois decorre exclusivamente de
sua vontade, de seu prazer e de suas necessidades.” Desenvolvimento: A poluição atmosférica deve ser
- Definição: desenvolve-se pela explicação dos termos que combatida urgentemente, pois a alta concentração de elementos
compõem o texto. tóxicos põe em risco a vida de milhares de pessoas, sobretudo
- Interrogação: questionamento. Ex: “Volta e meia se faz daquelas que sofrem de problemas respiratórios:
a pergunta de praxe: afinal de contas, todo esse entusiasmo
pelo futebol não é uma prova de alienação?” - A propaganda intensiva de cigarros e bebidas tem levado
- Suspense: alguma informação que faça aumentar a muita gente ao vício.
curiosidade do leitor. - A televisão é um dos mais eficazes meios de comunicação
- Comparação: social e geográfica. criados pelo homem.
- Enumeração: enumerar as informações. Ex: “Ação - A violência tem aumentado assustadoramente nas cidades
à distância, velocidade, comunicação, linha de montagem, e hoje parece claro que esse problema não pode ser resolvido
triunfo das massas, Holocausto: através das metáforas e das apenas pela polícia.
realidades que marcaram esses 100 últimos anos, aparece a - O diálogo entre pais e filhos parece estar em crise
verdadeira doença do século...” atualmente.
- Narração: narrar um fato. - O problema dos sem-terra preocupa cada vez mais a
sociedade brasileira.
Desenvolvimento: é a argumentação da ideia inicial,
de forma organizada e progressiva. É a parte maior e mais
importante do texto. Podem ser desenvolvidos de várias formas: O parágrafo pode processar-se de diferentes maneiras:

- Trajetória Histórica: cultura geral é o que se prova Enumeração: Caracteriza-se pela exposição de uma
com este tipo de abordagem. série de coisas, uma a uma. Presta-se bem à indicação
- Definição: não basta citar, mas é preciso desdobrar a ideia de características, funções, processos, situações, sempre
principal ao máximo, esclarecendo o conceito ou a definição. oferecendo o complemento necessário à afirmação estabelecida
- Comparação: estabelecer analogias, confrontar na frase nuclear. Pode-se enumerar, seguindo-se os critérios de
situações distintas. importância, preferência, classificação ou aleatoriamente.
- Bilateralidade: quando o tema proposto apresenta
pontos favoráveis e desfavoráveis. Causa e Consequência: A frase nuclear, muitas vezes,
- Ilustração Narrativa ou Descritiva: narrar um fato encontra no seu desenvolvimento um segmento causal (fato
ou descrever uma cena. motivador) e, em outras situações, um segmento indicando
- Cifras e Dados Estatísticos: citar cifras e dados consequências (fatos decorrentes).
estatísticos.
- Hipótese: antecipa uma previsão, apontando para - O espírito competitivo foi excessivamente exercido entre
prováveis resultados. nós, de modo que hoje somos obrigados a viver numa sociedade
- Interrogação: Toda sucessão de interrogações deve fria e inamistosa.
apresentar questionamento e reflexão.
- Refutação: questiona-se praticamente tudo: conceitos, Tempo e Espaço: Muitos parágrafos dissertativos marcam
valores, juízos. temporal e espacialmente a evolução de ideias, processos.
- Causa e Consequência: estruturar o texto através dos
porquês de uma determinada situação. Explicitação: Num parágrafo dissertativo pode-se
- Oposição: abordar um assunto de forma dialética. conceituar, exemplificar e aclarar as ideias para torná-las mais
- Exemplificação: dar exemplos. compreensíveis.
Exemplo: “Artéria é um vaso que leva sangue proveniente do
Conclusão: é uma avaliação final do assunto, um coração para irrigar os tecidos. Exceto no cordão umbilical e na
fechamento integrado de tudo que se argumentou. Para ela ligação entre os pulmões e o coração, todas as artérias contém
convergem todas as ideias anteriormente desenvolvidas. sangue vermelho-vivo, recém-oxigenado. Na artéria pulmonar,
porém, corre sangue venoso, mais escuro e desoxigenado, que
- Conclusão Fechada: recupera a ideia da tese. o coração remete para os pulmões para receber oxigênio e
- Conclusão Aberta: levanta uma hipótese, projeta um liberar gás carbônico”.
pensamento ou faz uma proposta, incentivando a reflexão de
quem lê. Antes de se iniciar a elaboração de uma dissertação, deve
delimitar-se o tema que será desenvolvido e que poderá ser
Alguns pontos essenciais desse tipo de texto são: enfocado sob diversos aspectos. Se, por exemplo, o tema é
a questão indígena, ela poderá ser desenvolvida a partir das
- toda dissertação é uma demonstração, daí a necessidade seguintes ideias:
de pleno domínio do assunto e habilidade de argumentação;
- em consequência disso, impõem-se à fidelidade ao tema; - A violência contra os povos indígenas é uma constante na
- a coerência é tida como regra de ouro da dissertação; história do Brasil.
- impõem-se sempre o raciocínio lógico; - O surgimento de várias entidades de defesa das populações
- a linguagem deve ser objetiva, denotativa; qualquer indígenas.
ambiguidade pode ser um ponto vulnerável na demonstração - A visão idealizada que o europeu ainda tem do índio
do que se quer expor. Deve ser clara, precisa, natural, original, brasileiro.
nobre, correta gramaticalmente. O discurso deve ser impessoal - A invasão da Amazônia e a perda da cultura indígena.
(evitar-se o uso da primeira pessoa).
O parágrafo é a unidade mínima do texto e deve apresentar: Depois de delimitar o tema que você vai desenvolver, deve
uma frase contendo a ideia principal (frase nuclear) e uma ou fazer a estruturação do texto.
mais frases que explicitem tal ideia.
Exemplo: “A televisão mostra uma realidade idealizada A estrutura do texto dissertativo constitui-se de:
(ideia central) porque oculta os problemas sociais realmente Introdução: deve conter a ideia principal a ser
graves. (ideia secundária)”. desenvolvida (geralmente um ou dois parágrafos). É a abertura

64
LÍNGUA PORTUGUESA
do texto, por isso é fundamental. Deve ser clara e chamar a o enfoque a ser trabalhado assim como as fontes a serem
atenção para dois itens básicos: os objetivos do texto e o plano entrevistadas. Antes da entrevista o repórter costuma reunir
do desenvolvimento. Contém a proposição do tema, seus o máximo de informações disponíveis sobre o assunto a ser
limites, ângulo de análise e a hipótese ou a tese a ser defendida. abordado e sobre a pessoa que será entrevistada. Munido deste
Desenvolvimento: exposição de elementos que vão material, ele formula perguntas que levem o entrevistado a
fundamentar a ideia principal que pode vir especificada através fornecer informações novas e relevantes. O repórter também
da argumentação, de pormenores, da ilustração, da causa deve ser perspicaz para perceber se o entrevistado mente ou
e da consequência, das definições, dos dados estatísticos, manipula dados nas suas respostas, fato que costuma acontecer
da ordenação cronológica, da interrogação e da citação. principalmente com as fontes oficiais do tema. Por exemplo,
No desenvolvimento são usados tantos parágrafos quantos quando o repórter vai entrevistar o presidente de uma instituição
forem necessários para a completa exposição da ideia. E esses pública sobre um problema que está afetando o fornecimento de
parágrafos podem ser estruturados das cinco maneiras expostas serviços à população, ele tende a evitar as perguntas e a querer
acima. reverter a resposta para o que considera positivo na instituição.
Conclusão: é a retomada da ideia principal, que agora É importante que o repórter seja insistente. O entrevistador
deve aparecer de forma muito mais convincente, uma vez que deve conquistar a confiança do entrevistado, mas não tentar
já foi fundamentada durante o desenvolvimento da dissertação dominá-lo, nem ser por ele dominado. Caso contrário, acabará
(um parágrafo). Deve, pois, conter de forma sintética, o induzindo as respostas ou perdendo a objetividade.
objetivo proposto na instrução, a confirmação da hipótese As entrevistas apresentam com frequência alguns sinais
ou da tese, acrescida da argumentação básica empregada no de pontuação como o ponto de interrogação, o travessão,
desenvolvimento. aspas, reticências, parêntese e às vezes colchetes, que servem
para dar ao leitor maior informações que ele supostamente
Texto Literário: expressa a opinião pessoal do autor desconhece. O título da entrevista é um enunciado curto que
que também é transmitida através de figuras, impregnado chama a atenção do leitor e resume a ideia básica da entrevista.
de subjetivismo. Ex: um romance, um conto, uma poesia... Pode estar todo em letra maiúscula e recebe maior destaque
(Conotação, Figurado, Subjetivo, Pessoal). da página. Na maioria dos casos, apenas as preposições ficam
com a letra minúscula. O subtítulo introduz o objetivo principal
Texto Não-Literário: preocupa-se em transmitir uma da entrevista e não vem seguido de ponto final. É um pequeno
mensagem da forma mais clara e objetiva possível. Ex: uma texto e vem em destaque também. A fotografia do entrevistado
notícia de jornal, uma bula de medicamento. (Denotação, aparece normalmente na primeira página da entrevista e
Claro, Objetivo, Informativo). pode estar acompanhada por uma frase dita por ele. As frases
O objetivo do texto é passar conhecimento para o leitor. importantes ditas pelo entrevistado e que aparecem em
Nesse tipo textual, não se faz a defesa de uma ideia. Exemplos destaque nas outras páginas da entrevista são chamadas de
de textos explicativos são os encontrados em manuais de “olho”.
instruções.
Crônica: Assim como a fábula e o enigma, a crônica é
Informativo: Tem a função de informar o leitor a respeito um gênero narrativo. Como diz a origem da palavra (Cronos
de algo ou alguém, é o texto de uma notícia de jornal, de revista, é o deus grego do tempo), narra fatos históricos em ordem
folhetos informativos, propagandas. Uso da função referencial cronológica, ou trata de temas da atualidade. Mas não é só isso.
da linguagem, 3ª pessoa do singular. Lendo esse texto, você conhecerá as principais características
da crônica, técnicas de sua redação e terá exemplos.
Argumentativo: Os textos argumentativos, ao contrário, Uma das mais famosas crônicas da história da literatura luso-
têm por finalidade principal persuadir o leitor sobre o ponto brasileira corresponde à definição de crônica como “narração
de vista do autor a respeito do assunto. Quando o texto, além histórica”. É a “Carta de Achamento do Brasil”, de Pero Vaz de
de explicar, também persuade o interlocutor e modifica seu Caminha, na qual são narrados ao rei português, D. Manuel, o
comportamento, temos um texto dissertativo-argumentativo. descobrimento do Brasil e como foram os primeiros dias que
Exemplos: texto de opinião, carta do leitor, carta de os marinheiros portugueses passaram aqui. Mas trataremos,
solicitação, deliberação informal, discurso de defesa e acusação sobretudo, da crônica como gênero que comenta assuntos do
(advocacia), resenha crítica, artigos de opinião ou assinados, dia a dia.
editorial. Questões

Exposição: Apresenta informações sobre assuntos, expõe 01.


ideias, explica, avalia, reflete. Estrutura básica; ideia principal; (...) Há um pôr-do-sol de primavera e uma velha
desenvolvimento; conclusão. Uso de linguagem clara. Ex: casa abandonada. Está em ruínas.
ensaios, artigos científicos, exposições,etc. A velha casa não mais abriga vidas em seu interior.
Tudo é passado. Tudo é lembrança.
Injunção: Indica como realizar uma ação. É também Hoje, apenas almas juvenis brincam
utilizado para predizer acontecimentos e comportamentos. despreocupadas e felizes entre suas paredes
Utiliza linguagem objetiva e simples. Os verbos são, na sua trêmulas.
maioria, empregados no modo imperativo. Há também o uso Em seu chão, despido da madeira polida que a
do futuro do presente. Ex: Receita de um bolo e manuais. cobriam, brotam ervas daninhas. Entre a vegetação
que busca minimizar as doces recordações do
Diálogo: é uma conversação estabelecida entre duas ou passado, surge a figura amarela e suave da
mais pessoas. Pode conter marcas da linguagem oral, como margarida, flor-mulher. As nuanças de suas cores
pausas e retomadas. sorriem e denunciam lembranças de seus
ocupantes.
Entrevista: é uma conversação entre duas ou mais pessoas A velha casa está em ruínas. Pássaros saltitam e
(o entrevistador e o entrevistado), na qual perguntas são feitas gorjeiam nas amuradas que a cercam. Seus trinados
pelo entrevistador para obter informação do entrevistado. Os são melodias no altar do tempo à espera de
repórteres entrevistam as suas fontes para obter declarações redentoras orações. Raízes vorazes de grandes
que validem as informações apuradas ou que relatem situações árvores infiltraram-se entre as pedras do alicerce e
vividas por personagens. Antes de ir para a rua, o repórter abalam suas estruturas.
recebe uma pauta que contém informações que o ajudarão Agoniza a velha casa. Agora, somente imagens
a construir a matéria. Além das informações, a pauta sugere desfilam, ao longo das noites. As janelas são bocas

65
LÍNGUA PORTUGUESA
escancaradas. A casa velha em ruínas clama por 03.
vozes e movimentos...
(Geraldo M. de Carvalho)

De acordo com a tipologia textual, o texto acima:


(A) é descritivo, com traços dissertativos compondo um
ambiente nostálgico.
(B) possui descrição subjetiva apenas no trecho “A velha
casa não mais abriga vidas em seu interior. Tudo é passado.
Tudo é lembrança.”
(C) ocorre uma descrição objetiva narrativa no trecho todo.
(D) é formado basicamente de descrições subjetivas.

02.
Como cuidar de seu dinheiro em 2015
Gustavo
Cerbasi.
“Ao final dos anos 80, a Petrobras se encontrava diante do
Em 2015, cuidarei bem do meu dinheiro. Organizarei bem desafio de produzir petróleo em águas abaixo de 500 metros,
os números e as verbas. Esses números mudarão bastante ao feito não conseguido então por nenhuma companhia no
longo do ano. Um monstro chamado inflação ronda o país. Só
que, agora, ele usa um manto da invisibilidade, que ganhou mundo. Num gesto de ousadia, decidiu desenvolver no Brasil
de seu criador, o governo. Quando morder meu bolso, eu nem a tecnologia necessária para produzir em águas até mil metros.
saberei de onde terá vindo o ataque, não terei tempo de me O sucesso foi total. Menos de uma década depois, a Petrobras
defender. Por isso, deixarei boas gorduras no orçamento para dispõe de tecnologia comprovada para produção de petróleo
atirar a ele, quando aparecer. Essas gorduras serão chamadas em águas muito profundas. O último recorde foi obtido em
de verba para lazer e reservas de emergência. janeiro de 1999 no campo de Roncador, na bacia de Campos,
Em 2015, não farei apostas. Já há gente demais apostando produzindo a 1.853 metros de profundidade. Mas a escalada
não para. Ao encerrar-se a década, a empresa prepara-se para
em imóveis, ações e outros investimentos especulativos. Farei superar, mais uma vez, seus próprios limites. A meta, agora, são
escolhas certeiras. Deixarei a maior parte de meu investimento os 3 mil metros de profundidade, a serem alcançados mediante
na renda fixa. Ela está com uma generosidade única no mundo. projetos que aliam a inovação tecnológica à redução de custos.
Enquanto isso, estudo o desespero de especuladores que “
aguardarão a improvável recuperação dos imóveis, da Petrobras,
da credibilidade dos mercados. Quando esses especuladores Exposição PETROBRAS em 60 momentos. Agência
jogarem a toalha, usarei parte de minhas reservas para fazer Petrobras
investimentos bons e baratos. Mas não na Petrobras.
Muita gente fala que, com a inflação e a recessão, pode O tipo textual predominante que caracteriza o texto é a:
(A) narração.
perder o emprego ou os clientes. Faltará renda, faltarão (B) predição
consumidores. O ano de 2015 será, mais uma vez, ruim para (C) instrução
quem vende. Será um ano bom para quem pensa em comprar. (D) descrição.
Estarei atento aos bons negócios para quem tem dinheiro na (E) argumentação
mão. Se a renda fixa paga bem, a compra à vista tende a me
dar descontos maiores. É por esse mesmo motivo que, em 2015, 04.
evitarei as dívidas. Os juros estão altos e isso me convida a Por isso foi à luz de uma vela mortiça
poupar, e não a alugar dinheiro dos bancos. Dívidas de longo Que li, inserto na cama,
prazo são corrigidas pela inflação, também em alta. Por isso, O que estava à mão para ler --
(...)
aproveitarei os ganhos extras de fim de ano para liquidar Em torno de mim o sossego excessivo de noite de província
dívidas e me policiar para não contrair novas. Fazia um grande Barulho ao contrário,
No ano que começa, também não quero fazer papel de otário Dava-me uma tendência do choro para a desolação.
e deixar nas mãos do governo mais impostos do que preciso. A “Primeira Epístola aos Coríntios” ...
Não sonegarei. Mas aproveitarei o fim do ano para organizar Relia-a à luz de uma vela subitamente antiquíssima,
meus papéis e comprovantes, planejar a declaração de Imposto E um grande mar de emoção ouvia-se dentro de mim...
de Renda de março e tentar a maior restituição que puder, ou Sou nada...
o mínimo pagamento necessário. Listarei meus gastos com Sou uma ficção...
dependentes, educação e saúde, doarei para instituições que Que ando eu a querer de mim ou de tudo neste mundo?
“Se eu não tivesse a caridade.”
fazem o bem, aplicarei num PGBL o que for necessário para o
E a soberana luz manda, e do alto dos séculos,
máximo benefício. Entregarei minha declaração quanto antes,
A grande mensagem com que a alma é livre...
no início de março. Quero ver minha restituição na conta mais
“Se eu não tivesse a caridade...”
cedo, já que 2015 será um ano bom para quem tiver dinheiro
Meu Deus, e eu que não tenho a caridade.
na mão.
CAMPOS, Álvaro de. (Heterônimo de Fernando Pessoa). Ali não
Para quem lamenta, recomendo cuidado com o monstro e
havia eletricidade. In: “Poemas”. Disponível em:< http:// www.
com o governo. Para quem está atento às oportunidades, desejo
citador.pt/poemas/>. Acesso em: 5 jan. 2015, com adaptações.
boas compras.
(Disponívelem:http://epoca.globo.com/
A respeito da tipologia textual, é correto afirmar que o
colunaseblogs/gustavocerbasi/noticia/2015/01/
poema representa uma
comocuidardebseudinheirobem2015.html Acesso em:
(A) narração.
06/02/2015.)
(B) argumentação.
(C) descrição.
De acordo com a tipologia textual, o objetivo principal do
(D) caracterização.
autor é:
(E) dissertação.
(A) narrar.
(B) instruir.
Respostas
(C) descrever.
01. D / 02. D / 03. A. / 04. A.
(D) argumentar.

66
0DWHPiWLFD
a) Conjunto dos números inteiros não nulos ou diferentes
*
OPERAÇÕES (ADIÇÃO, SUBTRAÇÃO, de zero = ¯
MULTIPLICAÇÃO, DIVISÃO, POTENCIAÇÃO * *
¯ = ¯ – {0} ou ¯ = {..., –3, –2, –1, +1, +2, +3, ...}
E RADICIAÇÃO)
b) Conjunto dos números inteiros não negativos = ¯+  (¯+
= Û)
OPERAÇÕES FUNDAMENTAIS EM Ü
¯+ = {0, +1, +2, +3, ...} ou
As operações fundamentais em Ü abrangem: ¯+ = {0, 1, 2, 3, ...}
a) Conjunto dos números naturais (Conjunto Û) c) Conjunto dos números inteiros não positivos = ¯–
b) Conjunto dos números inteiros (Conjunto ¯)
c) Conjunto dos números racionais (Conjunto ¦) (¯– = {0, –1, –2, –3, ...})
d) Conjunto dos números irracionais (Conjunto ž)
*
d) Conjunto dos número inteiros positivos é igual a ¯+
Com o conjunto dos números naturais (Û), calculamos
todas as operações fundamentais em aritmética, mas, é bom *
 (¯+ = {+1, +2, +3, ... })
lembrar que ele é fechado em relação à adição, ou seja, a
soma de dois números naturais é sempre um número natural. *
e) Conjunto dos números inteiros negativos ¯–
Então, quando calculamos a sua operação inversa, a subtra-
ção, notamos que ela não possui a propriedade do fechamento, *
(¯– = {–1, –2, –3, ... })
ou seja:

8 – 6 = 2, mas 6 – 8 = ? REPRESENTAÇÃO GEOMÉTRICA:


Logo, para que essa operação fosse possível, tornou-se A RETA NUMÉRICA INTEIRA
necessário criar novos números que formaram o conjunto
chamado Conjunto dos Números Inteiros Relativos, ou Pode-se dar outra representação ao conjunto ¯. Para
apenas Conjunto dos Números Inteiros, cujo símbolo é ¯. isso desenhamos uma reta r e sobre ela marcamos o ponto O,
correspondendo ao número zero, dividindo-a em duas semi-
retas.
CONJUNTO DOS NÚMEROS INTEIROS RELATIVOS
A partir do ponto O, marcamos à sua direita e à sua
(CONJUNTO ¯) esquerda, segmentos consecutivos, com a mesma medida (1
cm, por exemplo) e façamos corresponder, a cada ponto à
É formado pelos elementos: direita de O, os números inteiros positivos e a cada ponto à
esquerda de O, os números inteiros negativos.
a) Números inteiros positivos, cujos numerais são:
+1, +2, +3, +4, +5, ..., que são lidos:
+1 (mais um ou um positivo),
+2 (mais dois ou dois positivos), etc.

b) Números inteiros negativos, cujos numerais são: –1, –2,


–3, –4, –5, ..., que são lidos: Deste modo, verificamos que cada número inteiro pode
–1 (menos um ou um negativo), ser associado a um ponto da reta r.
–2 (menos dois ou dois negativos), etc.
Esta representação é chamada Representação Geomé-
c) Número zero (0), que não é positivo nem negativo. trica de ¯

A reunião dos conjuntos dos números inteiros negativos, Os pontos F’, E’, D’, C’, B’, A’, O, A, B, C, D, E, F são as
do zero e dos números inteiros positivos, forma o Conjunto imagens geométricas, respectivamente, dos números –6, –5,
dos Números Inteiros, que é representado pela letra ¯ (lê-se: –4, –3, –2, –1, 0, 1, 2, 3, 4, 5, 6 e os números –6, –5, –4, –3, ...
zê) e é escrito: etc. são as abcissas dos pontos F’, E’, D’, C’, ..., etc.
¯ = {..., –5, –4, –3, –2, –1, 0, +1, +2, +3, +4, +5, ...} A reta r sobre a qual estão assinalados os pontos é a
reta numerada.
Nota: Pode-se dispensar o sinal + que acompanha os
números inteiros positivos, pois os mesmos se identificam com
os números naturais maiores que zero. Então: COMPARAÇÃO ENTRE NÚMEROS INTEIROS
+1=1,+2=2,+3=3, ..., +9=9, ..., +30=30, ..., +50=50, ...
Pode-se aceitar que qualquer movimento na reta
Desta forma, o conjunto ¯, pode ser escrito: numerada, para a direita, deve ser considerado como movi-
mento positivo, e, para a esquerda, um movimento negativo.
¯ = {..., –4, –3, –2, –1, 0, 1, 2, 3, 4, ...} Então conclui-se que: qualquer número localizado à esquerda,
na reta numerada, é menor que qualquer número localizado à
Então, nota-se que, todo número natural é também um direita, e vice-versa.
número inteiro, portanto:
Û G ¯ Û é subconjunto de ¯ Desta forma, observando a reta numerada da figura
acima, afirmamos que:
SUBCONJUNTOS DE ¯ a) –6 < –2 e) +2 < +5

Além do conjunto Û, podem ser identificados os seguin- b) –3 < +2 f) +5 > +3


tes: c) 0 < +5 g) –3 < +1
Nota: o símbolo *(asterisco) indica a ausência do zero d) 0 > –3 h) +2 > –6
no conjunto.

-1-
0DWHPiWLFD
MÓDULO OU VALOR ABSOLUTO a) Crédito com Crédito dá Crédito:
DE UM NÚMERO INTEIRO (+8) + (+5) = +13
b) Débito com Débito dá Débito:
O módulo ou valor absoluto de um número positivo ou
(–8) + (–5) = –13
negativo é o próprio número sem ser levado em consideração
o sinal de + ou –. Assim, o módulo de +5 é 5 ou o módulo de –7 c) Crédito com Débito dá o que for maior:
é 7. Indica-se o módulo colocando o número inteiro entre duas
barras. Exemplos:

a) –6 = 6 (lê-se: o módulo de –6 é igual a 6) Para estabelecer as regras, consideramos os seguintes


b) +3 =3 (lê-se: o módulo de +3 é igual a 3) casos:

NÚMEROS INTEIROS OPOSTOS OU SIMÉTRICOS Primeiro Caso: Um dos números dados é zero.
Exemplo:
São dois números inteiros que possuem o mesmo a) (+4) + 0 = +4 c) 0 + (+4) = +4
módulo e sinais contrários, como +5 e –5, 9 e –9, etc. O zero é b) (–4) + 0 = –4 d) 0 + (–4) = –4
oposto dele mesmo. Exemplo:
Regra: Quando um dos números é zero, a soma é igual
Número

ao outro número.
–4 +2 –15 7 0 –20 +1 –3 12 –11 –16
Segundo Caso: Os números dados têm o mesmo sinal.
Exemplos:
Oposto ou
Simétrico

+4 –2 +15 –7 0 +20 –1 +3 –12 +11 16 Crédito + Crédito = Crédito Maior


 (+7) + (+5) = +12
Débito + Débito = Débito Maior
 (–7) + (–5) = –12
DETERMINAÇÃO DE UM SUBCONJUNTO DE ¯
Regra: A soma de dois números de mesmo sinal é
Um novo símbolo: (lê-se: tal que) obtida conservando-se o sinal comum às parcelas e
somando-se seus módulos. Exemplos:
Seja determinar os seguintes subconjuntos de ¯ a) (+4) + (+7) = +11
b) (+9) + (+7) = +16
1) O conjunto dos números inteiros maiores que – 4: c) (–5) + (–9) = –14
pela nomeação dos seus elementos: d) (–1) + (–6) = –7
{–3, –2, –1, 0, +1, +2, ...} Terceiro Caso: Os números dados têm sinais diferentes.
simbolicamente: {x  ¯ x > –4} Exemplos:
Então: {x  ¯ x > –4} = Crédito + Débito Menor = Crédito Menor
(+12) + (–4) = +8
= {–3, –2, –1, 0, +1, +2, ...}
Crédito + Débito Maior = Débito Menor
2) O conjunto dos números inteiros menores ou iguais a –5: (+4) + (–12) = –8
pela nomeação dos seus elementos:
Regra: A soma de dois números de sinais diferentes é
{–5, –6, –7, –8, –9, ...} obtida dando-se o sinal da parcela que tem maior
módulo e subtraindo-se seus módulos. Exemplos:
simbolicamente: {x  ¯ x  –5}
a) (+16) + (–5) = +11
Então: {x  ¯ x  –5} = b) (–12) + (+7) = –5
= {–5, –6, –7, –8, –9, ...} c) (–4) + (+11) = +7
d) (+3) + (–15) = –12
3) O conjunto dos números inteiros maiores ou iguais a –3 e) (+7) + (–6) = +1
e menores que +3: (significa escrever os números f) (–7) + (+6) = –1
inteiros compreendidos entre –3 e +3, inclusive o –3)
pela nomeação de seus elementos: Quarto Caso: Os números dados são simétricos.
Exemplos:
{–3, –2, –1, 0, +1, +2}
Crédito + Débito igual = nada
simbolicamente: {x  ¯ –3  x <+3} (+5) + (–5) = 0
Então: {x  ¯ –3  x < +3} = Débito + Crédito igual = nada
= {–3, –2, –1, 0, +1, +2} (–5) + (+5) = 0

Regra: A soma de dois números opostos ou simétricos


OPERAÇÕES FUNDAMENTAIS EM ¯ é igual a zero.

ADIÇÃO DE NÚMEROS INTEIROS (ADIÇÃO EM ¯) PROPRIEDADES DA ADIÇÃO


Para que haja um melhor entendimento da adição em ¯, 1ª) Fechamento: A soma de dois números inteiros é sempre
devemos comparar os números inteiros com as operações de um número inteiro.
crédito e débito, ou seja:
Se a  ¯e b  ¯ _ (a + b)  ¯
Crédito = números positivos e Exemplo: (–7)+(+3)=–4 
Débito = números negativos.  Se (–7)  ¯ e (+3)  ¯
(–4)  ¯.
Assim, de maneira fácil se verifica que:

-2-
0DWHPiWLFD
2ª) Comutativa: A ordem das parcelas não altera a soma. Regra: A diferença de dois números inteiros é a soma do
primeiro com o simétrico do segundo. Exemplos:
Se a  ¯ e b  ¯, então a + b = b + a.
a) (+7) – (+9) =
Exemplo:
(transformando na soma do 1º + oposto do 2º)
= (+7) + (–9) =
(passando para notação simplificada)
3ª) Associativa: Não importa de que forma as parcelas = +7 – 9 =
sejam agrupadas ou associadas, a soma é sempre a
mesma. (calculando a soma algébrica)
Se a  ¯, b  ¯e c  ¯  = –2.
 (a+b) + c = a + (b+c).
b) (–5) – (–8) =
Exemplo: (transformando na soma do 1º + oposto do 2º)
= (–5) + (+8) =
(passando para notação simplificada)
= –5 +8 =
4ª) Elemento neutro: O zero é o elemento neutro da adição. (calculando a soma algébrica)
Se a  ¯ _ a + 0 =0 + a = a = +3.
Exemplo: (–8) + 0 = 0 + (–8) = –8 Por esses exemplos, nota-se que as operações em que
aparece o sinal negativo antes do parênteses, podem ser
5º) Elemento oposto ou simétrico: todo número inteiro realizadas facilmente por um raciocínio direto.
admite um oposto ou simétrico e a soma de qualquer
número inteiro com o seu oposto ou simétrico é sempre Observe nos exemplos a e b esses fatos:
igual a zero.
a) – (+9) = –9  – (+) = –
Se a  ¯, então existe o elemento oposto (–a) tal que b) – (–8) = +8  – (–) = +
(+a) + (–a) = 0.
Então, as subtrações podem ser passadas diretamente
para a notação simplificada (sem parênteses), aplicando o
ADIÇÃO DE TRÊS OU MAIS NÚMEROS INTEIROS raciocínio direto:
– (+) = – e – (–) = +. Exemplos:
É feita calculando-se, separadamente, a soma de todas
as parcelas positivas e a soma de todas as parcelas negativas a) (+4) – (+9) = +4 – 9 = –5
e em seguida soma-se os resultados obtidos. Exemplo: b) (–3) – (+4) = – 3 – 4 = – 7
c) (+2) – (–8) =+2 +8 = +10
d) (–5) – (–7) =– 5 + 7 = +2

PROPRIEDADES DA SUBTRAÇÃO EM ¯

1ª) Fechamento: a diferença de dois números inteiros é


NOTAÇÃO SIMPLIFICADA sempre um número inteiro.

Para simplificar a representação da soma de números Se a  ¯ e b  ¯ _ (a – b)  ¯.


inteiros, basta eliminar os sinais + da operação e os parênteses Exemplo: (–4) – (–7) = –4 +7 = +3 
das parcelas, escrevendo-se apenas as parcelas, uma em
seguida da outra, cada qual com o seu próprio sinal. Exemplos: Se (–4)  ¯ e (–7)  ¯ _ + 3  ¯

Notação Simplificada 2ª) A subtração em ¯ não possui as propriedades comutati-


Expressão
(Soma Algébrica) va e associativa e não tem elemento neutro.
a) (+5) + (–8) _ +5 – 8
b) (–3) + (+7) + (–6) _ – 3 +7 – 6 SOMA ALGÉBRICA

A notação simplificada chama-se soma algébrica. A adição algébrica é uma expressão numérica onde
aparecem somente as operações de adição e subtração, cujo
A soma algébrica é calculada da mesma forma que a resultado é chamado soma algébrica. Para resolvê-las, basta
soma de três ou mais números inteiros. Exemplo: eliminar os parênteses, passando-os para a notação simplifica-
da, usando o seguinte raciocínio direto:
+(+) = + ou +(–) = –
ou –(–) = + ou –(+) = –

Exemplo: Veja (–3) + (–5) – (–6) – (+9) + (+4) =


(o sinal que precede o 1º parêntese, quando não estiver
SUBTRAÇÃO DE NÚMEROS INTEIROS escrito, é sempre +)
(SUBTRAÇÃO EM ¯)

A subtração de dois números inteiros é a operação que


nos permite adicionar o minuendo ao oposto do subtraendo,
ou seja, toda subtração é substituída por uma adição.

-3-
0DWHPiWLFD
REGRAS PRÁTICAS PARA A ELIMINAÇÃO 2º) Se os fatores têm sinais contrários (um positivo e outro
DE PARÊNTESES negativo), então multiplicamos os módulos e damos ao
resultado o sinal negativo. Exemplos:
1ª) Parênteses precedidos do sinal +: podem ser elimina- a) (+3) # (–7) = –21
dos juntamente com esse sinal, conservando-se apenas b) (–4) # (+5) = –20
os sinais dos números contidos em seu interior. Exem-
plos: Em vista dos exemplos dados, podemos estabelecer o
seguinte resumo dos sinais do produto, que chamamos Regra
a) Prática dos Sinais do Produto:

SINAIS DOS FATORES SINAL DO PRODUTO


(+) # (+) 
b)
(–) # (–) 
(+) # (–) –
(–) # (+) –
2ª) Parênteses precedidos do sinal –: podem ser elimina-
dos juntamente com esse sinal, trocando-se os sinais Nota: A multiplicação por zero é sempre nula. Exemplos:
dos números contidos em seu interior. Exemplos: a) (+5) # 0 = 0 c) 0 # (+3) = 0
b) (–7) # 0 = 0 d) 0 # (–9) = 0
a)
MULTIPLICAÇÃO DE TRÊS OU MAIS
NÚMEROS INTEIROS

Na prática, calculamos o produto dos valores absolutos


b) de todos os fatores, contamos o número de fatores negati-
vos, e colocamos no produto o sinal, observando o seguinte
critério:

a) Se o total de fatores negativos for PAR, o produto é


POSITIVO.
EXPRESSÕES NUMÉRICAS ENVOLVENDO
ADIÇÃO E SUBTRAÇÃO EM ¯ b) Se o total de fatores negativos for ÍMPAR, o produto é
NEGATIVO.
São sentenças matemáticas envolvendo operações
apenas com números. Resolvê-las é fazer corretamente todas Exemplos:
as operações nelas contidas até se chegar num resultado final,
bastando para isso eliminar em primeiro lugar os parênteses, a) ² #  # ² #  # ² # ²  
depois os colchetes e por último as chaves, seguindo-se as (4 fatores negativos)
mesmas regras práticas para eliminação de parênteses,
calculando-se, finalmente, a soma algébrica obtida. Exemplo: b)  # ² #  # ² # ² #   ²
(3 fatores negativos)

(eliminam-se os parênteses) PROPRIEDADES ESTRUTURAIS


DA MULTIPLICAÇÃO EM ¯

1ª) Fechamento: o produto de dois números inteiros é


(eliminam-se os colchetes) sempre um número inteiro.
Se a  ¯ e b  ¯ _ (a # b)  ¯
(eliminam-se as chaves) Exemplo: (–3) # (+5) = –15 
Se (–3)  ¯ e (+5)  ¯ _
= –5+1–3+2+3+5–7–1 = _ –15  ¯.
(agrupam-se os negativos e os positivos)
2ª) Comutativa: a ordem dos fatores não altera o produto.
Se a  ¯ e b  ¯ _ a # b = b # a.
(calcula-se a soma algébrica de cada grupo) Exemplo:
= –16 +11 = –5
(calcula-se a soma algébrica final)

3ª) Associativa: não importa de que forma sejam agrupa-


MULTIPLICAÇÃO DE NÚMEROS INTEIROS dos ou associados os fatores, o produto é sempre o
(MULTIPLICAÇÃO EM ¯) mesmo.
Ao calcular o produto de dois números inteiros, podemos Se a  ¯, b  ¯ e c  ¯ _
observar que: _ (a # b) # c = a # (b # c).

1º) Se os fatores têm sinais iguais (ambos positivos ou 4ª) Elemento Neutro: o número +1 é o elemento neutro da
ambos negativos), então multiplicamos os módulos e multiplicação.
damos ao resultado o sinal positivo. Exemplos: Se a  ¯ _ a # (+1) = a
a) (+4) # (+7) = + 28 Exemplo: (–6) # (+1)=(+1) # (–6)=–6.
b) (–5) # (–7) = +35

-4-
0DWHPiWLFD
5ª) Distributiva em Relação à Adição e à Subtração: o DIVISÃO DE NÚMEROS INTEIROS (DIVISÃO EM ¯)
produto de um número inteiro por uma soma algébrica
pode ser obtido multiplicando-se esse número pelos O quociente de dois números inteiros, com o segundo
termos da soma e, em seguida, somando-se os produtos diferente de zero, é obtido dividindo-se o módulo do dividendo
parciais. pelo módulo do divisor, observando-se que:
Se a  ¯, b  ¯ e c  ¯ _
 a # (b+c) = ab + ac 1º) Se o dividendo e o divisor têm o mesmo sinal, o quoci-
ou a # (b–c) = ab – ac ente é sempre positivo. Exemplos:

Exemplos: a) (–3) # (2+5) =–6–15=–21 a) (+20) : (+5) = +4


b) (–15) : (–3) = +5
b) (–2) # (4–7) =–8+14=+6
c) (+4) # (–4+5) =–16+20=+4 2º) Se o dividendo e o divisor têm sinais contrários, o
quociente é sempre negativo. Exemplos:
EXPRESSÕES NUMÉRICAS COM ADIÇÃO, a) (+30) : (–5) = –6
SUBTRAÇÃO E MULTIPLICAÇÃO EM ¯ b) (–25) : (+5) = –5

Lembre-se que, além de resolver-se em primeiro lugar o Em vista dos exemplos dados, podemos estabelecer o
que está entre parênteses, depois o que está entre colchetes seguinte resumo dos sinais que chamaremos de Regra Prática
e por último o que está entre chaves, a operação multiplicação dos Sinais do Quociente.
deverá ser efetuada antes das operações adição ou subtra-
ção. Exemplos: SINAIS DOS NÚMEROS SINAL DO QUOCIENTE
a) (+) : (+) 
(primeiro as multiplicações) (–) : (–) 
(+) : (–) –
(eliminam-se os parênteses)
(–) : (+) –
= – 3 + 12 – 7 + 10 + 1 =
(agrupam-se os negativos e os positivos)
Nota: A divisão exata de dois números inteiros só é possível
quando o primeiro número é múltiplo do segundo e o segundo
é diferente de zero.
(calcula-se a soma algébrica)
= – 10 + 23 = + 13 PROPRIEDADES DA DIVISÃO EM ¯
b) É conveniente observar que a divisão nem sempre pode
ser realizada no conjunto ¯. Por exemplo, (+7) : (–5) ou (–1) :
(primeiro o que está entre parênteses)
(–4) não podem ser realizadas em ¯.

Então, não valem, em ¯, as propriedades do Fechamen-


(só as multiplicações) to, Comutativa, Associativa e Elemento Neutro.
= (–4) – (–12) =
(eliminam-se os parênteses) A propriedade Distributiva vale só à direita e quando
possível. Exemplo:
= – 4 + 12 = +8
(calcula-se a soma algébrica) (18 + 12) : (–6) = 18 : (–6) + 12 : (–6)
(+30) : (–6) = – 3 – 2
c) –5 = –5

Veja que a distributiva à esquerda, em relação à adição


(aplica-se a propriedade distributiva) e subtração, não é válida. Exemplo:
= 2x – 8 – 10x – 20 + 3 = 18 : (3+6) g 18 : 3 + 18 : 6
(agrupam-se os termos da mesma espécie) 18 : 9 g 6 + 3
2 g 9

EXPRESSÕES COM AS QUATRO OPERAÇÕES EM ¯


(calcula-se a soma algébrica)
Valem as mesmas regrinhas estudadas anteriormente,
agora lembrando que as operações multiplicação ou divisão
deverão ser efetuadas antes das operações adição ou subtra-
d) 4x – 3xy + 2y = quando x = –2 e y = +1. ção. Exemplo:
(basta substituir cada letra pelo valor atribuído e em
seguida calcular a expressão numérica)

Nota: O exercício “d” chama-se cálculo do valor numérico de


uma expressão literal.

-5-
0DWHPiWLFD
PRESTE MUITA ATENÇÃO:

Existe diferença entre (–2)2 e –22. Veja:

(–2)2 = (–2) (–2) = 4 –22 = –(2 # 2) = –4

• quando um número entre parênteses está elevado ao


quadrado, ou a qualquer outro expoente significa que é
o número todo, no caso (–2).

• quando se um sinal negativo fora do parênteses, ele-


va–se o número ao expoente e depois conserva–se o
sinal, no caso 2.

PROPRIEDADES OPERATÓRIAS DA POTENCIAÇÃO

POTENCIAÇÃO PRODUTOS DE POTÊNCIAS DE MESMA BASE

A potenciação foi criada para indicar multiplicações com Repete–se a base e somam–se os expoentes.
fatores iguais.
Ex.: (5)2 # (5)3 # (5)1 = 52 + 3 + 1 = 56 = 15625
Assim, 2 # 2 # 2 # 2 # 2 # 2 # 2 pode ser representado pelo 25 # 125 # 5 =
símbolo 27, que significa potência de base 2 e expoente 7. 3125 # 5 =
Então: 15625

27 = 2 # 2 # 2 # 2 # 2 # 2 # 2 = 128, (–2)3 # (–2)4 # (–2)2 = (–2)3 + 4 + 2 = (–2)9 = –512


(–8) # (+16) # (+4) =
onde lê–se 2 elevado à 7ª é igual a 128 (–128) # 4 =
–512
2 é a base _ base é o fator que se repete
Então: am # an = am + n (sendo a um número inteiro e
7 é o expoente _ expoente é o número de vezes m e n números naturais)
que repetimos a base

128 é a potência _ potência é o resultado da opera- QUOCIENTE DE POTÊNCIAS DE MESMA BASE


ção
Repete–se a base e subtraem–se os expoentes.
Para dois números inteiros a e n, temos que:
Ex.: (10)4 : (10)2 = (10)4–2 = (10)2 = 100
n
a = a # a # a # a ... 10000 : 100 =
¨«««««ª«««©
100
n fatores
(–5)8 : (–5)3 = (–5)8 – 3 = (–5)5 = –3125
Obs.: expoente 2 = quadrado 390625 : (–125) =
expoente 3 = cubo –3125
expoente 4 em diante = 4ª potência assim
por diante m n m–n
Então: a :a =a
(sendo a um número inteiro g 0 e
IMPORTANTE m e n números naturais onde m  n)
• Quando o expoente é 1, a potência é igual à base:
POTÊNCIA DE POTÊNCIA
a1 = a Ex.: 71 = 7 (–2)1 = –2
Repete–se a base e multiplicam–se os expoentes.
• Quando o expoente é zero, a potência será sempre igual
a 1. Ex.: [(–2)3]2 = (–2)3 # 2 = (–2)6 = 64
(–8)2 = 64
a0 = 1 Ex.: 70 = 1 (–2)0 = 1
[(4)3]3 = 43 # 3 = 49 = 262144
O sinal da potência de um número inteiro depende da 643 = 262144
base e do expoente.
m n m#n
Então: [(a) ] = a
• Potência de base positiva é sempre positiva, não
importando se o expoente é par ou ímpar. (sendo a um número inteiro e
m e n números naturais)
Ex.: 42 = 16 (4 # 4) 43 = 64 (4 # 4 # 4)

• Potência de base negativa e expoente par é sempre POTÊNCIA DE UM PRODUTO OU QUOCIENTE


positiva.
Repetem–se as bases elevando–se cada termo ao
Ex.: (–1)2 = +1 [(–1) (–1)] expoente indicado.
(–6)4 = 1296 [(–6) (–6) (–6) (–6)]
Ex.: (4 # 3)3 = 43 # 33 = 64 # 27 = 1728
• Potência de base negativa e expoente ímpar é sempre 123 = 1728
negativa.
[3 # (2)2]2 = [32 # (2)2]2 = [32 # 24] = 9 # 16 = 144
Ex.: (–2)3 = –8 [(–2) (–2) (–2)] [3 # 4]2 = [12]2 = 144
(–3)5 = –243 [(–3) (–3) (–3) (–3) (–3)]

-6-
0DWHPiWLFD
(–8 : 4)2 = (–8)2 : 42 = 64 : 16 = 4
(–2)2 = 4 Na igualdade

[9 : (–3)2]2 = 92 : 92 = 81 : 81 = 1 (lê–se raiz cúbica de 8 igual a 2, pois 23 = 8)


[9 : 9]2 = 12 = 1 3 < índice
Então: (a # b)m = am # bm 8 < radicando
(a : b)m = am # bm 2 < raiz
sendo a e b números inteiros e
m um número natural Para facilitar o cálculo das raízes veja uma tabela de
potências (extraída dos números naturais, ou seja, bases
positivas).
EXERCÍCIOS
Quarta Quinta
Número Quadrado Cubo
1) Classifique em verdadeiro (V) ou falso (F): Potência Potência
a) (–7)2 + (–2)2 = (–9)2 1 1 1 1 1
b) (–4)2 = 16 e –42 = –16 2 4 8 16 32
c) (–1)0 # (–1)2 # (–1)3 # (–1)4 = (–1)9 3 9 27 81 243
d) (–2)3 > (–3)2
4 16 64 256 1024
e) 32 # 30 # 33 = 35
5 25 125 625 3125
f) [5 – (–1)]2 = 52 – (–1)2
6 36 216 1296 7776
g) 32 + 22 = 52
7 49 343 2401 16807
h) (33)3 = 36
8 64 512 4096 32768
i) [(–5) (4)]2 = (–5)2 # 42
j) 20 = 1 e 0 1 = 0 9 81 729 6561 59049
10 100 1000 10000 100000
2) Qual é a base (pode haver 2 bases ou até mesmo
nenhuma) 11 121 1331 14641 161051
a) x3 = – 8 c) x2 = –9 e) x5 = –1 12 144 1728 20736 248832
b) x2 = 16 d) x0 = 1
13 169 2197 28561 371293
3) Qual é o expoente? 14 196 2744 38416 537824
a) 3x = 1 c) –3x = –27 e) 4x = 4 15 225 3375 50625 759375
b) –2x = 4 d) –3x = 9
Exemplos:
RESPOSTAS

1) a) F c) V e) V g) F i) V (consultar a tabela de potências na


b) V d) F f) F h) F j) V coluna do quadrado 92)
2) a) –2 d) qualquer número (consultar a tabela de potências na
b) 4 ou –4 e) inteiro e –1
c) não existe coluna da quinta potência 75)

3) a) 0 b) 2 c) 3 d) 2 e) 1 (consultar a tabela de potências na


coluna do cubo 113)

RADICIAÇÃO pois –23 = –8

Radiciação é a operação inversa à potenciação. Ex.: Obs.: não existe raiz quadrada de um número negativo, pois
Qual o número que elevado ao quadrado dá 9? Resp.: os todo número elevado ao quadrado é positivo. Para
números 3 e –3; pois (3)2 = 9 e (–3)2 = 9. obtermos uma raiz quadrada de um número negativo, o
sinal negativo vem antes do radical. Veja:
Simbolicamente fazemos assim:

.
EXPRESSÕES NUMÉRICAS COM AS QUATRO
Na igualdade OPERAÇÕES, POTENCIAÇÃO E RADICIAÇÃO EM ¯
(lê–se raiz quadrada de 9 igual a 3, pois 32 = 9), As expressões com números inteiros relativos, envolven-
do as operações estudadas devem obedecer à seguinte ordem
2 < índice de solução:
9 < radicando
1º) Potenciações ou radiciações;
3 < raiz
2º) Multiplicações ou divisões;
< radical 3º) Adições ou subtrações.

-7-
0DWHPiWLFD
Lembrando ainda que se deve fazer na ordem, as 4) Somando dois números negativos, temos como resulta-
operações contidas entre parênteses, entre colchetes e entre do:
chaves. Exemplo:
a) um número negativo
b) um número positivo
c) não podemos somá-los
d) nenhuma dessas afirmações

5) Qual é a igualdade verdadeira?


a) –22 = (–2)2 c) (–3)2 = 32
b) (–4)1 = 41 d) (–8)0 = –1

RESPOSTAS:
1) a) A = {–4, –3, –2, –1, ...}
b) B = {–5, –6, –7, –8, ...}
c) E = {–2, –1, +1, +2, +3, +4}
d) F = {+3, +2, +1, 0, –1, ...}
2) a) A = {–8, –4, –3, +3, +5}
b) B = {–1, +1, +2, +3, ...}
c) D = (–1, +1, +2, +3}
d) E = {–2, –1, 0, +1, +2, +3}
3) a) –2 b) –14 c) –6 d) +3
4) a) +12 b) +3 c) +22 d) +3

TESTES: 1) c 2) b 3) c 4) a 5) c

NÚMEROS PARES E ÍMPARES


EXERCÍCIOS A RESOLVER
A sucessão de números naturais 0, 2, 4, 6, 8, 10, 12, ...
1) Nomeando os elementos entre chaves, determinar os é chamada sucessão dos números pares. A sucessão de
conjuntos (pense na reta numerada). números naturais 1, 3, 5, 7, 9, 11, 13, ... é chamada sucessão
doso números ímpares.
a) A ={X  ¯ X > –5}
b) B = {X  ¯ X < –4} Nos números pares, o algarismo mais à direita é sempre
c) E = {X  ¯* –2  X < +5} 0, 2, 4, 6 ou 8. Nos números ímpares, o algarismo mais à
d) F = {X  ¯ X  +3} direita é sempre 1, 3, 5, 7 ou 9.

2) Escrever em ordem crescente os elementos dos conjun- MÚLTIPLOS E DIVISORES DE UM NÚMERO


tos:
a) A = {+5, –4, –3, –8, +3} MÚLTIPLO
b) B = {X  ¯* X  –1}
c) D = {X  ¯* –2 < X < +4} Dizemos que um número é múltiplo de outro, quando a
d) E = {X  ¯ –2  X < +4} sua divisão por esse outro é exata. Assim, 15 é múltiplo de 3 e
de 5, pois:
3) Elimine os parênteses, colchetes e chaves e efetue as
operações resultantes. a) 15 : 3 = 5 b) 15 : 5 = 3
a) (2–7+1) – (2+5–9) = Múltiplo de um número é o produto desse número por
b) –(–2+9) + (5–8) – 4 = um outro número qualquer. Então, para se obter os múltiplos de
c) –[(10–12)–(–8+9)]–[(4–10)+15] = um número, basta multiplicá-lo, sucessivamente, pela seqüên-
d) {3 – [(5–8) + 1]} – 2 = cia natural dos números, e, como essa seqüência é infinita,
conclui-se que:
4) Determine o valor das expressões:
a) Todo número tem uma infinidade de múltiplos.
a) [(–1–4) # (+3) – 4 # (–5)] – [(–2–8) : (–1–1) + 2 # (–6)] = b) Com exceção do zero, o menor múltiplo de um número
b) x # y2 + 3x + 2y = quando x = +1 e y = –2 é o próprio número.
c) (–2+3) # (–3–1)2 – [(–5–2)2 : (–1–6) + (–1)2 # (–4+5)3] =
d) 10–[5–(4–3)+( – )+(6–7)–(8–9)3+(–2)2] = Exemplo: os múltiplos de 3 são:
M(3)={0,3,6,9,12,15,18,...}
TESTES
DIVISOR
1) Assinale a afirmação verdadeira:
Dizemos que um número é divisor de outro, quando
a) 2  ¯x c) 0¯ divide esse outro exatamente, ou seja, sem deixar resto, e, se
b) –4  ¯+ d) 0¯ ele é divisor do outro, o outro é múltiplo dele.

2) Qual é a afirmação verdadeira? Exemplo: se 5 é divisor de 10 _


* 10 é múltiplo de 5.
a) ¯+ G ¯– c) ¯+ G ¯+
*
b) N =¯+ d) ¯+ = N Os divisores de um número formam sempre um conjunto
finito.
3) A afirmação verdadeira é:
a) –10 > +2 c) –10 > –20 Exemplo: os divisores de 15 são:
b) –10 > –6 d) –10 > 0 D(15) = {1, 3, 5, 15}.

-8-
0DWHPiWLFD
Observações: 157 2 157 3 157 5
17 78 07 52 07 31
a) O um é divisor de todos os números e é o seu menor 1 1 2
divisor.
b) O zero não é divisor de nenhum número, mas é múltiplo 157 7 157 11 157 13
de todos eles, e também o seu menor múltiplo. 17 22 047 14 027 12
c) O maior divisor de qualquer número é ele próprio. 3 03 01
d) O maior múltiplo de qualquer número é infinito. Resposta: O número 157 é primo, pois o quociente (12)
e) Qualquer número, com exceção do zero, é, ao mesmo da última divisão é menor que o divisor (13) e nenhuma das
tempo, múltiplo e divisor de si mesmo. divisões foi exata.

b) O número 161 é primo?


DIVISIBILIDADE
161 2 161 3 161 5 161 7
Os critérios de divisibilidade, são certas regras práticas 1 80 11 53 11 32 21 23
que nos permitem saber se um número é divisível por outro, 2 1 0
sem efetuar a divisão. Assim, um número é divisível:
Resposta: O número 161 é composto, pois a última
a) por 2: quando for par. divisão foi exata.
b) por 3: quando a soma de todos os seus algarismos
for divisível por 3.
c) por 5: quando terminar em zero ou 5. DECOMPOSIÇÃO EM FATORES PRIMOS
d) por 9: quando a soma de todos os seus algarismos
for divisível por 9. Na multiplicação 8 x 5 = 40, os números 8 e 5 são
e) por 10: quando terminar em zero. chamados fatores. Como 8 = 2 x 2 x 2, podemos escrever o
número 40 da seguinte forma:
Exemplo: o número 450 é divisível:
ou
a) por 2: porque é par.
b) por 3: porque 4 + 5 + 0 = 9, que é divisível por 3.
c) por 5: porque termina em zero. A expressão 23 x 5 chama-se forma fatorada do número
d) por 9: porque 4 + 5 + 0 = 9, que é divisível por 9. 40.
e) por 10: porque termina em zero.
Para se encontrar a forma fatorada de um número, faz-
se a sua decomposição em fatores primos, obedecendo à
DIVISIBILIDADE POR 10, 100 E 1 000 seguinte regra:

Um número natural é divisível por 10, 100, a) Divide-se o número dado pelo seu menor divisor primo.
1 000, ... se terminar por um zero, dois zeros, três zeros, ...
respectivamente. Exemplos: b) Procede-se da mesma maneira com o quociente obtido
até se encontrar o quociente 1.
450 é divisível por 10.
3 500 é divisível por 10 e por 100. Exemplo:
97 000 é divisível por 10, por 100 e por 1 000.
90 2 _ o menor divisor primo de 90 é 2  divide-se
3 90 por 2
NÚMEROS PRIMOS 45 3 _ o menor divisor primo de 45 é 3  divide-se
5 45 por 3
São números que possuem apenas dois divisores a 15 _ o menor divisor primo de 15 é 3  divide-se
unidade e eles mesmos. Exemplos: 15 por 3
5 _ o menor divisor primo de 5 é 5  divide-se
a) 2, pois  D (2)={1, 2}  a unidade e ele mesmo. 5 por 5
b) 3, pois  D (3)={1, 3}  a unidade e ele mesmo. 1 _ encontramos o quociente 1
c) 5, pois  D (5)={1, 5}  a unidade e ele mesmo, etc.
Então escrevemos:

NÚMEROS MÚLTIPLOS OU COMPOSTOS. ou


São números que possuem outros divisores além da
unidade e deles mesmos. Exemplos: Outros exemplos:
a) 4, pois D (4) = {1, 2, 4} a) 180 22
b) 6, pois D (6) = {1, 2, 3, 6}, etc... 180 = 2 x 2 x 3 x 3 x 5
90 33
Nota: Por convenção, o número 1 não é nem primo, nem 45 5 ou
composto. 15
5
1 180 = 22 x 32 x 5
RECONHECIMENTO DOS NÚMEROS PRIMOS

Para descobrir se um número é ou não primo, basta b) 198 2


dividi-lo sucessivamente pelos números primos (2, 3, 5, 7, 11, 198 = 2 x 3 x 3 x 11
99 3
...). Se a divisão não for exata até que o quociente fique 33 3 ou
menor que o divisor, o número é primo. Se a divisão for
11 11
exata, o número é composto. Exemplos: 198 = 2 x 32 x 11
1
a) O número 157 é primo?

-9-
0DWHPiWLFD
DETERMINAÇÃO DO CONJUNTO DOS TESTES
DIVISORES DE UM NÚMERO
7) Se a, b e c são números naturais diferentes de zero e
O processo que nos permite encontrar o conjunto de
todos os divisores de um número, obedece ao seguinte critério: a : b = c, então:
a) a é divisor de b c) a é múltiplo de b
a) Decompõe-se o número em fatores b) a é divisor de c d) c é múltiplo de a
primos:
8) Todo número que termina em zero, é divisível:
a) somente por 10 c) somente por 5
b) somente por 2 e 5 d) por 2, 5 e 10

9) Se um número é primo, ele tem:


b) Coloca-se outro traço vertical à direi-
ta da decomposição e escreve-se o a) apenas um divisor
número 1, que é divisor de todos os b) infinitos divisores
números, à direita do novo traço e c) apenas dois divisores distintos
acima do primeiro fator: d) apenas o número 1 como divisor

10) Se x = 23 # 3 # 5, então o valor de x é:


c) Multiplica-se o primeiro fator primo a) 30 b) 120 c) 90 d) 135
pelo divisor 1 e coloca-se o produto
obtido na linha correspondente ao 2:
RESPOSTAS:

1) 149, 311 e 421 são primos.


2) 48=24 # 3; 81=34; 300=22 # 3 # 52;
d) Multiplicam-se os demais fatores pelos divisores que 504=23 # 32 # 7
estiverem à direita do traço vertical e acima desses 3) D(28)={1, 2, 4, 7, 14, 28}
fatores. (Os produtos repetidos são eliminados): D(70)={1, 2, 5, 7, 10, 14, 35, 70}
D(192)={1,2,3,4,6,8,12,16,24,32,48,64,96,192}
D(250) = {1, 2, 5, 10, 25, 50, 125, 250}
4) a, c, d, e são verdadeiras.
5) M(4) = {0, 4, 8, ...}; M(6) = {0, 6, 12, ...};
M(5) = {0, 5, 10, ...}; M(11) = {0, 11, 22, ...}
6) a  14, 32, 36, 40, 120; b  36, 120
c  35, 40, 120; d  40, 120

e) Os divisores do número são os números colocados à TESTES: 7) c 8) d 9) c 10) b


direita do traço vertical, que deverão ser colocados em
ordem. Então:
D(90) = {1, 2, 3, 5, 6, 9, 10, 15, 18, 30, 45, 90} MÁXIMO DIVISOR COMUM

Outro exemplo: Qual é o conjunto dos divisores do Chama-se divisor comum o número que divide dois ou
número 132? mais números sem deixar resto.

Então, se: D(45) = {1, 3, 5, 9, 15, 45} e D(63) =


{1, 3, 7, 9, 21, 63} os divisores comuns desses números são:
D(45)  D(63) = {1, 3, 9}.

Como se pode ver, o maior dos divisores comuns de 45


e 63 é 9.
D(132)={1, 2, 3, 4, 6, 11, 12, 22, 33, 44, 66, 132}
O número 9 é chamado de máximo divisor comum
(m.d.c.) de 45 e 63.
EXERCÍCIOS A RESOLVER
Indica-se o máximo divisor comum assim:
1) Verificar quais, entre os números, são primos: 149, 275, m.d.c. (45, 63) = 9.
311, 372 e 421.
Então, conclui-se que o máximo divisor comum de dois
2) Decompor em fatores primos os números: 48, 81, 300 e ou mais números é o maior dos seus divisores comuns.
504.

3) Escreva o conjunto dos divisores de: 28, 70, 192 e 250. PROCESSOS PRÁTICOS DE CÁLCULO DO M.D.C.
4) Identifique as sentenças verdadeiras: O m.d.c. pode ser calculado por dois processos práticos:
a) 3 é divisor de 12. d) 15 é múltiplo de 1.
b) 24 é divisor de 12. e) 29 é número primo. 1º) Cálculo do m.d.c. pela decomposição em fatores
c) 24 é múltiplo de 12. primos – Obedece-se à regra:
a) Decompõe-se cada número em fatores primos.
5) Escreva o conjunto dos múltiplos de: 4, 5, 6 e 11.
b) O m.d.c. é o produto dos fatores primos comuns
6) Entre os números 14, 32, 35, 36, 40 e 120, verifique a esses números, elevados ao seu menor expoen-
quais são divisíveis por: te.
a) 2 b) 3 c) 5 d) 10
Exemplo: Calcular o m.d.c. de 60, 264 e 504.

- 10 -
0DWHPiWLFD
6) Virgínia deseja plantar 72 mudas de violeta, 24 de rosa,
36 de orquídeas e 48 de camélia no menor número
possível de canteiros. Sabendo-se que cada canteiro
deverá receber o mesmo número de plantas de uma só
espécie, pergunta-se:
a) qual o número de plantas que deve conter cada
canteiro?
b) quantos canteiros serão necessários?

TESTES
Os fatores primos comuns são 2 e 3.
7) Indique a alternativa que apresenta números primos
O menor expoente do fator 2 é 2 e do fator 3 é 1. entre si:
a) 20 e 15 b) 12 e 18 c) 13 e 26 d) 15 e 26
Então, m.d.c. (60, 264, 504) = 22 # 3 = = 4 # 3 = 12.
8) O m.d.c. dos números 4 e 12 é:
2º) Cálculo do m.d.c. pelas divisões sucessivas a) 2 b) 4 c) 12 d) 1

Obedece-se à regra: 9) O m.d.c. de dois números primos entre si é:


Divide-se o número maior pelo número menor. Em a) o menor deles c) o número um
seguida divide-se o número menor pelo primeiro resto; b) o maior deles d) o produto deles
depois divide-se o primeiro resto pelo segundo resto,
e assim sucessivamente até se encontrar uma divisão 10) Se x = 23 # 3 # 5 e y = 24 # 32 # 7, então o m.d.c. (x, y) é:
exata. O último divisor é o m.d.c. a) 210 b) 24 c) 360 d) 5040
Exemplo:
Calcular o m.d.c. (25, 60).
RESPOSTAS:
Faz-se as divisões sucessivas usando o dispositivo
abaixo, chamado Algoritmo de Euclides, onde os 1) a) 234 b) 266 c) 7 d) 42
quocientes são coloca-dos por cima dos respectivos di-
visores. 2) a) 17 b) 21 c) 35 d) 81
3) a) {1,2,3,6,9,18};
Linha dos quocientes  2 2 2 b) {1,2,3,4,6,12};
c) {1,3,5,15}
Linha dos divisores  60 25 10 É 4) 240 litros 5) 36 m
Linha dos restos  10 5 0 6) 12 plantas e 15 canteiros
M.D.C.(25,60) = 5
TESTES: 7) d 8) b 9) c 10) b
Obs.: 1ª) Chamam-se números primos entre si, a dois ou
mais números cujo m.d.c. = 1.
MÍNIMO MÚLTIPLO COMUM
2ª) O m.d.c. de dois números, em que um é múltiplo
do outro, é o menor deles. Sejam os múltiplos de 3 e 6. Excluindo o zero, que é
múltiplo de todos os números, temos:
EXERCÍCIOS A RESOLVER M(3) = { 3 6 9 12 15 18 21 24 27 30 33 ...}
1) Aplicando a decomposição em fatores primos, calcule o
m.d.c. de: M(6) = { 6 12 18 24 30 36 42 ...}
a) 468 e 702 c) 560, 623, 840
b) 798 e 1064 d) 504, 672, 882, 546 O conjunto intersecção nos fornece os múltiplos comuns:
M(3)  M(6) = {6, 12, 18, 24, 30, ... }
2) Aplicando as divisões sucessivas, calcule o m.d.c. de:
a) 85 e 153 c) 630 e 1085 O menor número desse conjunto é o 6 e chama-se
b) 63 e 105 d) 243 e 405 mínimo múltiplo comum, que é indicado assim:

3) Determine os divisores comuns de:


m.m.c. (3, 6) = 6
a) 90 e 198 c) 105, 135 e 165
b) 48 e 84
Então, podemos definir que: o mínimo múltiplo comum
4) As capacidades de dois reservatórios são de 6480 litros (m.m.c.) de dois ou mais números é o menor número, diferente
e 6000 litros respectivamente. Deseja-se construir um de zero, que é divisível por todos eles, ao mesmo tempo.
tanque que possa ser alimentado por esses reservatóri-
os. Calcular a maior capacidade desse tanque de
maneira que ele possa ser abastecido um número exato PROCESSOS PRÁTICOS PARA CÁLCULO DO M.M.C.
de vezes com a água de qualquer reservatório.
1º) Cálculo do m.m.c. pela decomposição em fatores
5) Três peças de fazenda medem, respectivamente, 180 m, primos
252 m e 324 m. Pretende-se dividi-las em retalhos de
igual comprimento. Qual deverá ser esse comprimento, Obedece-se à regra:
de modo que o número de retalhos seja o menor possí-
vel? a) Decompõem-se os números em fatores primos.

- 11 -
0DWHPiWLFD
b) Multiplicam-se todos fatores primos comuns e não 5ª) O produto do m.d.c. pelo m.m.c. de dois números
comuns, elevados aos seus maiores expoentes. naturais diferentes de zero é igual ao produto desses
Exemplo: Calcular o m.m.c.(36, 90, 120). mesmos números.

Então, se: m.d.c. (12, 30) = 6 e


m.m.c. (12, 30) = 60

Teremos:

EXERCÍCIOS A RESOLVER
Os fatores primos comuns e não comuns são 2, 3 e 5.
1) Determine, pela decomposição em fatores primos, o
O maior expoente do fator 2 é 3 e do fator 3 é 2. m.m.c. de:
a) 150 e 180 d) 18, 30 e 48
Então: m.m.c. (36, 90, 120) = b) 80 e 120 e) 132, 60 e 84
= 23 # 32 # 5 = 8 # 9 # 5 = 360. c) 500 e 750 f) 180, 90 e 450
2º) Cálculo do m.m.c. pela decomposição simultânea em 2) Determine, pela decomposição simultânea, o m.m.c. de:
fatores primos:
a) 20 e 36 d) 45 e 108
Obedece-se à regra: b) 42 e 54 e) 24, 27 e 30
c) 40 e 96 f) 12, 15 e 18
a) Decompõem-se, ao mesmo tempo, todos os números
em fatores primos. 3) Aplicando as propriedades do m.d.c. e do m.m.c.,
calcule:
b) O m.m.c. é o produto de todos os fatores primos obtidos. a) m.d.c.(2, 9) c) m.d.c.(2, 3, 5)
b) m.m.c.(2, 9) d) m.m.c.(4,8, 24)
Exemplo: calcular o m.m.c.(36, 90, 120).
4) Qual é o produto de dois números, se o seu m.d.c. é 8 e
o seu m.m.c. é 48?

5) Calcular os dois menores números pelos quais devemos


multiplicar os números 60 e 78, a fim de obter produtos
iguais.

6) Numa República, o Presidente deve permanecer durante


4 anos em seu cargo, os Senadores 6 anos e os Deputa-
dos, 3 anos. Se, em 1929 houve eleições para os 3
3 2 cargos, em que ano se realizarão novamente juntas as
m.m.c. (36, 90, 120 _ 2 # 3 # 5 _ 9 # 8 # 5 = 360
eleições para esses cargos?

7) Duas rodas de uma engrenagem têm, respectivamente,


PROPRIEDADES DO M.M.C. 14 e 21 dentes. Cada roda tem um dente estragado. Se
num dado instante estão em contato os dois dentes
1ª) O m.m.c. de dois ou mais números primos entre si é o estragados, depois de quantas voltas esse encontro se
produto deles. repetirá?
Exemplo: m.m.c. (5, 3, 2) = 30. TESTES
2ª) O m.m.c. entre dois números em que o maior é divisível
pelo menor, é o maior deles. 8) Se a e b são números naturais e a é múltiplo de b, então
m.m.c (a, b) é igual:
Exemplo: m.m.c. (12, 3) = 12.
a) 1 b) a c) b d) a#b
3ª) Se vários números forem multiplicados ou divididos por
um certo número diferente de zero, o seu m.m.c. também 9) Se a e b são números primos entre si, então m.m.c (a, b)
ficará multiplicado ou dividido por esse número. é igual a:
Então, sendo: m.m.c.(18, 12, 60) = 180, podemos dizer a) 1 b) a c) b d) a#b
que:
10) Se a = 2 # 32 # 5 e b = 2 # 3 # 7, então o m.m.c. (a, b), é:
a) m.m.c. (18 # 3, 12 # 3, 60 # 3) = 180 # 3
b) m.m.c. (18 : 3, 12 : 3, 60 : 3) = 180 : 3. a) 180 b) 6 c) 18 d) 630

4ª) Dividindo-se o m.m.c. de vários números por todos eles,


um por vez, os quocientes obtidos serão números primos RESPOSTAS:
entre si.
1) a) 900 c) 1500 e) 4620
Então, se m.m.c. (12, 18, 60) = 180, teremos, efe- b) 240 d) 720 f) 900
tuando as divisões: 2) a) 180 c) 480 e) 1080
180 : 12 = 15; 180 : 18 = 10 e b) 378 d) 540 f) 180
180 : 60 = 3 3) a) 1 b) 18 c) 1 d) 24
4) 384 5) 10 e 13 6) em 1941
onde os números 15, 10 e 3 são primos entre si, pois só 7) duas voltas da maior e três voltas da menor.
admitem a unidade como divisor comum. TESTES: 8) b 9) d 10) d

- 12 -
0DWHPiWLFD
AS QUATRO OPERAÇÕES COM Normalmente, um número misto é representado sem o
NÚMEROS FRACIONÁRIOS E DECIMAIS sinal “+” colocado entre o inteiro e a fração.

NOÇÃO DE FRAÇÃO Então, indica-se e lê-se: três inteiros e um

É obtida quando se divide uma unidade qualquer em quarto.


partes iguais, como por exemplo, uma pizza dividida em quatro
partes iguais. TRANSFORMAÇÃO DE NÚMEROS MISTOS
EM FRAÇÕES IMPRÓPRIAS:

_ Multiplica-se o inteiro pelo denominador, e ao produto


soma-se o numerador, obtendo, assim, o numerador da fração
procurada. O denominador é conservado o mesmo.

Exemplo:

_
TRANSFORMAÇÃO DE FRAÇÕES IMPRÓPRIAS
um quarto dois quartos três quartos EM NÚMEROS MISTOS (EXTRAÇÃO DE INTEIROS):

Divide-se o numerador pelo denominador. O quo-


REPRESENTAÇÃO DAS FRAÇÕES ciente é a parte inteira, o resto é o numerador da parte
fracionária e o divisor é o denominador da fração própria.
Os números um quarto, dois quartos, três quartos, são Exemplo:
chamados números fracionários ou racionais ou simples-
mente frações e são escritos assim: ou 1/4 (um quarto), Extrair os inteiros de .

ou 2/4 (dois quartos), ou 3/4 (três quartos). Solução: dividindo-se o numerador pelo denominador,
vem:
Então, para se representar uma fração são necessários
dois números naturais, com o segundo diferente de zero,
que são chamados termos, sendo que o primeiro é o numera-
dor e o segundo é o denominador. Logo, na fração 3/4, o 3 é
o numerador e o 4 é o denominador, e significam:
Então, onde:
O DENOMINADOR INDICA EM QUANTAS PARTES A
UNIDADE FOI DIVIDIDA.

FRAÇÕES EQUIVALENTES
O NUMERADOR INDICA O NÚMERO DESSAS PARTES
QUE FOI TOMADO. São duas ou mais frações que representam a mesma
parte do inteiro.

TIPOS DE FRAÇÕES Exemplo: Observe as figuras

Podem ser próprias, impróprias e aparentes. â 1/3

Fração própria: é quando o numerador é menor que o â 2/6


denominador e são todas menores que a unidade.

Exemplos: etc. â 3/9

Fração imprópria: é quando o numerador é maior que As frações , e representam a mesma parte do
o denominador e são todas maiores que a unidade.
inteiro e são chamadas de frações equivalentes.
Exemplos: etc.
PROPRIEDADE FUNDAMENTAL DAS FRAÇÕES

Fração aparente: é quando o numerador é igual ou Quando multiplicamos ou dividimos o numerador e o


múltiplo do denominador e todas representam números denominador de uma fração por um mesmo número natural
naturais que se obtêm dividindo o numerador pelo denomina- diferente de zero, obtemos uma fração equivalente à fração
dor. dada. Exemplo:
Exemplos: etc. Seja a fração 3/4. Se multiplicarmos os seus termos
(numerador e denominador) por 2, teremos a fração 6/8 e,
observando a figura seguinte, vê-se que essas frações são
NUMERO MISTO equivalentes.
A soma de um número inteiro com uma fração própria
chama-se número misto ou fração mista.

Exemplo: .

- 13 -
0DWHPiWLFD

Veja ainda que: ,

Logo:
CLASSES DE EQUIVALÊNCIA
(denominadores diferentes) (denominadores iguais)
É o conjunto de frações equivalentes à fração dada. Nota: Se houver números mistos ou inteiros, trans-
Para construí-las basta multiplicar ou dividir os seus dois formam-se esses números em frações impróprias ou
termos pelos números naturais 1, 2, 3, 4, 5, ... Exemplo: aparentes e faz-se como no exemplo anterior. Exemplo:
Seja construir a classe de equivalência de 2/3.
transformando, vem:
Teremos:

m.m.c. (2, 5, 10) = 10

ou

Ao conjunto dessas frações equivalentes dá-se o nome


de classes de equivalência da fração 2/3 e é indicada assim: COMPARAÇÃO DE FRAÇÕES

Devem ser considerados os casos:

1º) Frações com denominadores iguais: a maior é a que


tem o maior numerador. Exemplo:
SIMPLIFICAÇÃO DE FRAÇÕES

Significa reduzi-las a frações equivalentes cujos termos
sejam números primos entre si. São dois os processos
práticos para isso: 

1º) Simplificação pelas divisões sucessivas: consiste em


dividir sucessivamente os dois termos da fração por Logo:
um mesmo divisor comum diferente de 1. Exemplo:
2º) Frações com numeradores iguais: a maior é a que
tem o menor denominador. Exemplo:


Logo:

Nota: Os termos da fração 3/4 são primos entre si.
Então, dizemos que a fração é irredutível. 
2º) Simplificação pelo m.d.c.: consiste em calcular o m.d.c.
entre os dois termos da fração e em seguida dividi-los Logo:
pelo M.D.C. encontrado. Exemplo:
3º) Frações com numeradores e denominadores diferen-
m.d.c. (36, 48) = 12 tes: se as frações têm numera-dores e denominadores
diferentes é necessário reduzi-las ao mesmo denomina-
dor para então enquadrá-las no primeiro caso.
Logo:
EXERCÍCIOS A RESOLVER

REDUÇÃO DE FRAÇÕES AO MESMO DENOMINADOR 1) Simplificar as frações pelo processo das divisões suces-
sivas:
Consiste em transformar duas ou mais frações em
a) b) c) d) e)
outras frações equivalentes que tenham denominadores
iguais. Para isso, opera-se assim:
2) Simplificar as frações pelo processo do m.d.c.:
Seja reduzir e ao mesmo denominador. Faz-se:
a) b) c) d) e)
1º) Calcula-se o m.m.c. dos denominadores:
m.m.c. (4, 6, 2) = 12 3) Qual é o valor de x, em cada caso, para que as frações
sejam equivalentes:
2º) Divide-se o m.m.c. pelos denominadores das frações
dadas: 12 : 4=3, 12 : 6=2 e 12 : 2 = 6. a) c)
3º) Multiplicam-se esses quocientes pelos respectivos
numeradores: b) d)

- 14 -
0DWHPiWLFD
4) Reduzir as frações ao mesmo denominador: 2º) Frações com denominadores diferentes:

a) b) Reduzem-se as frações ao mesmo denominador e em


seguida aplica-se a regra anterior. Exemplos:

a)
5) Colocar em ordem crescente as frações:

a) b) m.m.c (3, 4, 6) = 12

TESTES
reduzindo ao mesmo denominador
6) Qual é a fração aparente que representa o número 3?

a) b) c) d)
somando, simplificando e extraindo os inteiros.
7) A fração equivalente a 2/3, cujo numerador é 6, é:
b)
a) b) c) d)
Nota: Havendo números mistos ou inteiros, deve-se
8) O número misto é igual a qual fração imprópria? reduzi-los a frações impróprias ou aparentes. Exemplo:

a) b) c) d)

EXERCÍCIOS A RESOLVER
9) Extraindo os inteiros de 11/4, obtemos:
Calcular, simplificando e extraindo os inteiros:
a) b) c) d)
1) 4)

10) Qual das frações é a maior: ? 2) 5)

a) b) c) d) 3) 6)

RESPOSTAS:
RESPOSTAS:
1) 2) 3) 4) 5) 6)
1) a) ; b) ; c) ; d) ; e) .

2) a) ; b) ; c) ; d) ; e) . MULTIPLICAÇÃO DE FRAÇÕES

3) a) x = 18; b) x = 20; c) x = 20; d) x = 4. O produto de frações é obtido pela multiplicação dos


numerados entre si e dos denominadores entre si. Exem-
4) a) ; b) ; plos:

a)
5) a) ; b) .

b)
TESTES: 6) d; 7) a; 8) b; 9) d; 10) a.

Cancelamento: sempre que possível, os produtos


OPERAÇÕES COM FRAÇÕES ORDINÁRIAS
devem ser simplificados antes de efetuarmos a multiplicação.
Essa simplificação, que só pode ser feita na multiplicação,
ADIÇÃO E SUBTRAÇÃO chama-se cancelamento. Exemplos:
Devem ser considerados dois casos:
a)
1º) Frações com o mesmo denominador:
Somam-se ou subtraem-se os numeradores, na ordem Cancela-se o fator 3 do numerador e do denominador.
que se apresentam, conservando-se o mesmo denomi-
nador. Simplifica-se o resultado encontrado, se for
possível. Exemplos: b)

a)
Cancelam-se os fatores 3 e os fatores 7.

b) c)

c)
Dividem-se os fatores 10 e 15 por 5.

- 15 -
0DWHPiWLFD
EXERCÍCIOS A RESOLVER
d)
Calcular os produtos:

Dividem-se os fatores 3 e 9 por 3 e os fatores 8 e 4 por 1) 4)


4.
2) 5)
FRAÇÃO DE FRAÇÃO
3) 6)
Seja calcular os .
Calcular:

7) Os de 8) Os de
Essa expressão pode ser substituída por , ou
seja, substituímos a preposição de pelo sinal de multiplica- 9) Os
ção. Então temos:

. 10) Os de 490 11) Os de 1 hora

12) Os de R$ 600,00
Portanto, para se calcular uma fração de fração,
multiplicam-se as duas frações.
Calcular os quocientes:
NÚMEROS RACIONAIS INVERSOS
(FRAÇÕES INVERSAS) 13) 16)
19)
Uma fração é inversa de outra fração diferente de zero
quando se troca de lugar o numerador com o denominador.
Exemplos: 20)
14) 17)
a) O inverso de é

b) O inverso de é4
15) 18)
c) O inverso de 6 é

Note-se que: RESPOSTAS:

a) 1) 6) 11) 45 minutos 16)

2) 7) 12) R$ 250,00 17)


b)
3) 8) 1 13) 18)

Quando multiplicamos uma fração pelo seu inverso, o 4) 9) 14) 19)


produto é sempre 1.
5) 10) 140 15) 20)
DIVISÃO DE FRAÇÕES

Divide-se uma fração por outra, ou uma fração por um POTENCIAÇÃO DE FRAÇÕES
número inteiro, ou ainda, um número inteiro por uma fração,
multiplicando-se a primeira pelo inverso da segunda. É obtida elevando-se tanto o numerador como o deno-
Exemplos: minador ao expoente indicado. Exemplos:

a) a) b)

b) Nota: As convenções adotadas para as potências de


expoente 1 e expoente zero com números naturais são
também válidas para as frações. Então:
c)
a) Potências de expoente 1 são iguais à própria base.
b) Potências de expoente zero e base diferente de zero
são iguais a 1. Exemplos:
d)
a) b)

- 16 -
0DWHPiWLFD
RADICIAÇÃO DE FRAÇÕES 6)

A raiz quadrada de uma fração que é quadrado perfeito


é obtida extraindo-se a raiz quadrada do numerador e do
denominador. Exemplos:

a) porque

b) porque

EXPRESSÕES NUMÉRICAS FRACIONÁRIAS

São feitas obedecendo-se às mesmas regras estabeleci-


das para o cálculo com números naturais e inteiros, ou seja:
1º) potenciações e radiciações;
2º) multiplicações e divisões, na ordem em que aparecem;
3º) adições e subtrações, na ordem em que aparecem.

Se a expressão tiver sinais de associação: ( ), [ ] e { },


estes devem ser eliminados na ordem:
1º) as operações contidas nos parênteses ( ); depois
2º) as operações contidas nos colchetes [ ]; e por último
3º) as operações contidas nas chaves { }.

Siga os exemplos:

1)

EXERCÍCIOS A RESOLVER

1) Calcular as potências:

a) c) e)

b) d) f)

2) Extrair a raiz quadrada de:


2)
a) c) e)

b) d) f)

3) Calcular o valor das expressões:


3)
a)

b)

c)
4)

d)

TESTES

4) A potência é igual a:
5)
a) 6/10 b) 9/10 c) 9/25 d) 6/25

5) O inverso da potência é:

a) b) c) d)

- 17 -
0DWHPiWLFD
13/72  correspondem a 26 exercícios
6) A soma do dobro de com a metade de é: 1/72  corresponderá a 2 exercícios 
 (26 exercícios : 13 = 2 exercícios)
a) b) c) 2 d) Paulo  exercícios

7) O valor da expressão Armando  exercícios

é um número compreendido entre: Resp.: Paulo conferiu 54 exercícios e Armando 64.


a) 3e4 b) 2e3 c) 1e2 d) 0e1
4) 2/3 de uma peça de fazenda custaram R$ 98,00. Qual
será o valor de 4/5 da mesma?
RESPOSTAS: Solução:

a) b) c) e) f) Se 2/3 custam R$98,00


1) d) 1
1/3 custará R$ 98,00 : 2 = R$ 49,00
3/3 (preço da peça) R$ 49,00 # 3 = R$ 147,00
2) a) b) c) d) e) f) 5/5 = R$ 147,00 (peça toda)
1/5 = R$ 147,00 : 5 = R$ 29,40
3) a) ; b) ; c) ; d) 4/5 = R$ 29,40 # 4 = R$ 117,60.
Resp.: O valor de 4/5 da peça será R$ 117,60.
TESTES: 4) c 5) c 6) a 7) b
5) Dividir a terça parte de 4/5 pela metade de 2/7.

PROBLEMAS ENVOLVENDO NÚMEROS RACIONAIS Solução:


ABSOLUTOS (FRAÇÕES ORDINÁRIAS) Terça parte de
PROBLEMAS RESOLVIDOS (MODELOS):
Metade de
1) Se 3/7 de uma estrada correspondem a 90 km, qual o
comprimento dessa estrada? Efetuando a divisão, teremos:
Solução:
3/7  correspondem a 90 km
1/7  corresponderá a 30 km _
_ (90 km : 3 = 30 km) Resp.: O resultado da divisão é
7/7  (estrada toda) tem:
7 # 30 km = 210 km
6) Um negociante vendeu 3/5 de uma peça de fazenda e
Resp.: A estrada tem 210 km de comprimento. ainda lhe restaram 32 metros. Quanto media essa peça?
2) Um automóvel já percorreu 2/5 da distância entre duas Solução:
cidades. Resta ainda percorrer 60 km. Qual é a distância
entre essas cidades? Vendendo 3/5 ainda lhe sobraram:
Solução:
Já percorreu  2/5
Se 2/5 correspondem a 32 metros
Resta percorrer  1/5 corresponderá a 16 metros  (32 : 2 = 16)
Os 5/5 que representam a peça inteira terão:
3/5  correspondem a 60 km 5 # 16 = 80 metros.
1/5  corresponderá a 20 km _
_ (60 km : 3 = 20 km) Resp.: A peça media 80 metros.
5/5  (distância total) é:
5 # 20 km = 100 km 7) Um estudante gastou 2/7 do seu dinheiro numa compra
de material escolar. Depois tornou a gastar mais 3/5 e
Resp.: A distância entre as duas cidades é de 100 ainda lhe restaram R$ 10,00. Quanto possuía esse
km. estudante?
3) Vários exercícios de um concurso foram conferidos por Solução:
três candidatos: Armando, Paulo e José. Armando Fração gasta nas duas compras:
conferiu 4/9 dos exercícios; Paulo 3/8 e José, os 26
exercícios restantes. Quantos foram os exercícios
conferidos por Paulo? e por Armando?
Solução: Fração do dinheiro que ainda resta:

Se 4/35 correspondem a R$ 10,00


Fração correspondente ao número de exercícios conferi- 1/35 corresponderá a R$ 2,50 
dos por Armando: 32/72 e Paulo: 27/72. (R$ 10,00 : 4 = R$ 2,50)
Fração correspondente aos Os 35/35, que representam todo o dinheiro desse
 exercícios conferidos por estudante serão: 35 # 2,50 = R$ 87,50.
José. Resp.: O estudante possuía R$ 87,50.

- 18 -
0DWHPiWLFD
8) Uma pessoa gastou 1/5 do que tinha; a seguir, metade PROBLEMAS PARA RESOLVER
do que lhe sobrou e depois R$ 600,00; ficou com R$
600,00. Quanto tinha primitiva-mente?
Solução: 1) Dividir a quinta parte de 3/5 pela terça parte de .
Gastando 1/5 do que tinha, ainda lhe restaram 4/5. R = 21/50
Gastando novamente 1/2 do resto 4/5, ou seja:
2) Calcular os de 120. R = 90
, ainda lhe sobram

3) Qual é o número cujos valem 40? R = 100

Como gastou R$ 600,00 e ainda ficou com R$ 600,00, 4) Um candidato acertou das questões de um concurso
então esses 2/5 são iguais a R$ 1.200,00 
(R$600,00 + R$600,00 = R$ 1.200,00) e com isso obteve 20 pontos. Quem acertou 3/4 das
Se 2/5 correspondem a R$ 1.200,00 questões, quantos pontos fez? R = 18 pontos
1/5 corresponderá a R$ 600,00 
(R$ 1.200,00 : 2 = R$ 600,00)
5) Uma piscina está com da capacidade total. Se
Os 5/5 que representam o que possuía, serão:
5 # 600,00 = R$ 3.000,00. colocarmos mais 9.000 litros de água, ela ficará comple-
Resp.: Tinha R$ 3.000,00. tamente cheia. Qual é a capacidade total?
R=24.000 litros

9) Uma peça de fazenda, depois de molhada, encolheu


3/14 do seu comprimento, ficando com 33 metros. 6) Um operário recebe, pelos de seu trabalho, a impor-
Quantos metros tinha a peça e qual foi o seu custo,
sabendo-se que o metro da fazenda valia R$ 7,25? tância de R$ 120,00. Quanto lhe resta ainda para
Solução: receber? R = R$ 200,00
Encolhendo 3/14, ainda restam:
7) Uma peça de fazenda é dividida em três partes. Uma é
igual a ; outra, . Que fração representa a terceira
Se 11/14 correspondem a 33 metros
1/14 corresponderão a 3 metros  parte? R=
 33 : 11 = 3
Os 14/14 serão: 14 # 3 = 42 metros
Se a peça tinha 42 metros e cada metro custa R$ 7,25, 8) Os mais de uma peça de fazenda somados
então o seu preço será de:
medem 44 metros. Calcular o comprimento da peça.
42 # R$ 7,25 = R$ 304,50.
R = 70 metros
Resp.: A peça tinha 42 metros e seu custo foi de R$
304,50.
9) Uma pessoa gastou do seu dinheiro e ainda ficou com
10) Duas torneiras enchem um tanque em 4 horas. Uma R$ 600,00. Quanto possuía? R = R$ 1.600,00
delas sozinha, enche-o em 7 horas. Em quantos minutos
a outra, sozinha, encheria o tanque?
Solução: 10) Um estudante tinha R$ 80,00. Gastou e depois mais
As duas torneiras juntas enchem o tanque em 4 horas.
Portanto, numa hora alimentarão 1/4 do tanque. do resto. Quanto ainda lhe restou? R = R$ 10,00
Da mesma forma, se uma das torneiras leva 7 horas
para encher o tanque, em 1 hora ela alimentará 1/7 do 11) Uma pessoa gastou num dia do seu dinheiro e no
tanque.
Portanto, a segunda torneira, em 1 hora, encherá 3/28 do
outro, . Ficou ainda com R$ 3.600,00. Quanto pos-
tanque, pois:
suía? R = R$ 7.000,00

Donde se conclui que esta segunda torneira sozinha 12) Um excursionista fez uma viagem de 360 km. Os do
encherá o tanque em 560 minutos, pois

em 60 min. _ em 60 min. : 3 = 20 min. percurso foram feitos de trem, a cavalo e o resto de


automóvel. Quantos quilômetros andou de automóvel? A
fração percorrida de automóvel, quanto representa da
e em 28 # 20 min. = 560 min. viagem total? R = 45 km e 1/8 do percurso

Resp.: Em 560 minutos.

- 19 -
0DWHPiWLFD
NÚMEROS DECIMAIS (FRAÇÕES DECIMAIS) TRANSFORMAÇÃO DE NÚMERO DECIMAL
EM FRAÇÃO DECIMAL
Fração decimal é toda fração cujo denominador é uma Procede-se assim:
potência de 10, como 10, 100, 1000, etc. Exemplos:
a) o numerador é o número decimal sem a vírgula e sem
os zeros iniciais.
a) (lê-se: sete décimos) b) o denominador é o número 1 seguido de tantos zeros
quantas forem as casas decimais do número decimal.
Exemplos:

a) 5,6 = (uma casa decimal  um zero)

b) (lê-se: quinze centésimos)


b) 0,35 = (duas casas decimais  dois
zeros)

c) 0,005 = (três casas decimais  três


zeros)
c) (lê-se: nove milésimos)

TRANSFORMAÇÃO DE FRAÇÃO DECIMAL


EM NÚMERO DECIMAL

NÚMEROS DECIMAIS Escreve-se o numerador da fração e separa-se por uma


vírgula, a partir da direita, tantas casas decimais quantos
As frações , e , podem ser repre-sentadas forem os zeros do denominador. Exemplos:

respectivamente, por 0,7; 0,15 e 0,009. Logo: a) = 6,5 (um zero  uma casa decimal)

b) = 3,47 (dois zeros  duas casas


decimais)
Os numerais 0,7; 0,15 e 0,009 são exemplos de nume-
rais decimais e são chamados simplesmente de números 
decimais. c) = 0,003 (três zeros três casas
decimais)

Nos números decimais, a vírgula separa a parte inteira


da parte decimal. Observe os exemplos:
PROPRIEDADES DOS NÚMEROS DECIMAIS:
a)
1ª) O valor de um número decimal não se altera quando se
acrescentam ou se suprimem zeros à direita desses
números.
b) Exemplo: 54,65 = 54,650 = 54,65000

2ª) Para multiplicar um número decimal por 10, 100, 1000,


... desloca-se a vírgula para a direita uma, duas, três, ...
casas. Se faltarem algarismos, acrescentam-se zeros.
LEITURA DE UM NÚMERO DECIMAL
Exemplos: 18,65 # 10 = 186,5; 3,582 # 10000 = 35820
É feita assim: primeiro a parte inteira, e em seguida a
parte decimal acompanhada das palavras: 3ª) Para dividir um número decimal por 10, 100, 1000, ...
desloca-se a vírgula para a esquerda uma, duas, três,
décimos  se houver uma casa decimal, ... casas. Se faltarem algarismos, serão eles supridos
com zeros.
centésimos  se houver duas casas decimais,
Exemplos: 341,68 : 10 = 34,168; 0,15 : 100 = 0,0015
milésimos  se houver três casas decimais,

e assim por diante. Exemplos: EXERCÍCIOS A RESOLVER


a) 3,4 (lê-se: três inteiros e quatro décimos)
1) Representar por algarismos os seguintes números:
b) 4,07 (lê-se: quatro inteiros e sete centésimos)
a) quarenta e dois inteiros e cento e cinco milésimos.
c) 12,0016 (lê-se: doze inteiros e dezesseis décimos de b) doze inteiros e três décimos milésimos.
c) quarenta e dois centésimos milésimos.
milésimos)
d) cinco inteiros e cento e trinta e sete milionésimos.
Nota: Quando a parte inteira é zero, pode ser lida
apenas a parte decimal. 2) Escrever por extenso os seguintes números:
Então: 0,013 (lê-se: treze milésimos) a) 41,35 c) 1,003497
b) 2,0835 d) 0,0001982

- 20 -
0DWHPiWLFD
3) Converter em números decimais as frações: MULTIPLICAÇÃO

a) b) c) d) Multiplicamos os números decimais como se fos-sem


números naturais, sem nos preocuparmos com as vírgulas
e separamos no produto, tantas casas decimais quantas
forem as do multiplicando mais as do multiplicador. Exem-
4) Efetuar as multiplicações: plo:
a) 31,5 # 10 c) 64,31 # 10 5,68 # 6,3 = temos:
b) 0,437 # 1000 d) 3,875 # 100

5) Efetuar as divisões:
a) 41,62 : 10 c) 4,73 : 1000
b) 385,3 : 100 d) 38,72 : 1000

RESPOSTAS: DIVISÃO

1) a) 42,105; b) 12,0003; c) 0,00042; Em geral, são observados dois casos:


d) 5,000137.
2) a) quarenta e um inteiros e trinta e cinco centésimos 1º) O divisor é inteiro: Efetua-se a divisão como se fossem
b) dois inteiros e oitocentos e trinta e cinco décimos números inteiros e, a seguir, separam-se no quociente,
milésimos tantas ordens decimais quantas existam no divisor.
c) um inteiro e três mil, quatrocentos e noventa e sete Exemplo:
milionésimos
d) um mil, novecentos e oitenta e dois décimos milioné- 157,92 : 42 =
simos
3) a) 0,03 b) 2,7 c) 0,0458 d) 48,57. a) Dividindo, como se fossem inteiros, teremos:
4) a) 315 b) 437 c) 643,1 d) 387,5.
5) a) 4,162 b) 3,853 c) 0,00473 d) 0,03872.

OPERAÇÕES COM NÚMEROS DECIMAIS

ADIÇÃO E SUBTRAÇÃO b) Separando duas casas decimais no quociente,


que são as existentes no dividendo vem: 157,92
Na prática procedemos assim: : 42 = 3,76

a) igualamos o número de casas decimais, acrescentan- 2º) O divisor é decimal: Quando o divisor é decimal,
do zeros. multiplicam-se o dividendo e o divisor por uma
potência de 10 (conforme a propriedade funda-mental
b) colocamos vírgula debaixo de vírgula. das frações), de modo que o divisor se torne inteiro e
assim, recaímos no caso anterior. Exemplo:
c) efetuamos a operação indicada.
22,016 : 4,3 =
Exemplos: multiplicando o dividendo e o divisor por 10, vem:
1º) 220,16 : 43 = 512 (como no caso anterior).
Logo: 220,16 : 43 = 5,12
(separando duas casas no quociente)

POTENCIAÇÃO DE NÚMEROS DECIMAIS


2º)
É calculada da mesma forma que a potenciação de
números naturais, inteiros ou fracionários. Exemplos:
a)
3º) b)

Nota: As convenções adotadas para as potências de


expoente 1 e expoente zero nos conjuntos N e Q, são válidas
4º) Resolver a expressão: para números decimais. Então:
a) ;
b) .

REPRESENTAÇÃO DECIMAL DE UMA


FRAÇÃO ORDINÁRIA
Cálculos: É feita dividindo-se o numerador pelo denominador da
fração. Podem acontecer dois casos:

1º) A divisão é exata: É quando o resto da divisão é zero


e o número decimal resultante é chamado decimal
exato.

- 21 -
0DWHPiWLFD
Exemplo: 14) 0,0132 : 6 =
15) 3,22 : 2,3 =
Converter em decimal a fração 16) 1,75 : 2,5 =
temos: 17) 3,591 : 0,95 =
18) 0,019 : 7,6 =

Calcular as potências e as expressões:


Logo, a forma decimal da fração é
3 2
19) (0,4) = 24) (0,05) =
2 2 2
 que é um número decimal exato, 20) (2,5) = 25) (2,3) +(0,9) =
porque o resto da divisão é zero. 4 3
21) (0,2) = 26) 10,2 – (1,2) =
2
2º) A divisão não é exata: É quando o resto da divisão 22) 6,2 27) (7,2) : 5,184 =
não é zero, e o número decimal resultante é chamado 0 2
23) (1,12) = 28) (2,3 – 1,75) =
dízima periódica ou decimal aproximado.

Exemplos: Converter em números decimais as frações:

a) 29) 31) 33)

30) 32) 34)

TESTES

b) 35) Qual é a sentença correta:


a) 3,5 = 0,35 c) 0,4 = 0,40
b) 0,03 = 0,3 d) 6,04 = 6,40

36) O número decimal 0,018 pode ser representado por:

a) c)

Então: e b) d)

Os números 2,666... e 1,1666... são chamados dízimas 37)


2
A potência (0,03) é igual a:
periódicas, pois as divisões não são exatas e o número 6,
chamado período, se repete infinitamente. a) 0,0009 c) 0,09
b) 0,009 d) 0,9
Nota: A dízima 2,666... é chamada dízima periódica
simples porque o período 6 começa logo depois da vírgula, 38) O quociente 0,01575 : 0,45 é igual a:
e a dízima 1,1666... é chamada dízima periódica composta
porque entre a vírgula e o período 6, existe uma parte a) 350 c) 0,35
decimal que não se repete (o número 1). b) 0,035 d) 3,5

EXERCÍCIOS A RESOLVER

Efetuar as operações:

1) 0,7+1,3+1,054+0,07 =
2) 4,3 – 2,07 =
3) 2 – 0,003 = RESPOSTAS:
4) (4,32 – 1,008) – (5,02 – 3,1) = 1) 3,124 13) 0,0343 25) 6,1
5) (2,1+1,3 – 1,7) – (3,14 – 2,8) = 2) 2,23 14) 0,0022 26) 8,472
3) 1,997 15) 1,4 27) 10
6) 2,5 # 0,157 = 4) 1,392 16) 0,7 28) 0,3025
7) 18,002 # 1,004 = 5) 1,36 17) 3,78 29) 0,6
6) 0,3925 18) 0,0025 30) 0,8333...
8) 3,1415 # 2,71 = 7) 18,074008 19) 0,064 31) 0,777...
9) (4,32+1,18) # 0,07 = 8) 8,513465 20) 6,25 32) 0,41666...
9) 0,385 21) 0,0016 33) 3,75
10) (7,2 – 1,3) # (4,2 – 1,6) = 10) 15,34 22) 6,2 34) 0,222...
11) 6,534 : 9 = 11) 0,726 23) 1
12) 10,657 24) 0,0025
12) 85,256 : 8 =
13) 0,2401 : 7 = TESTES: 35) c; 36) d; 37) a; 38) b.

- 22 -
0DWHPiWLFD
RAZÕES E PROPORÇÕES Nelas, devem ser observados os fatos:

a) O seu produto é sempre igual a 1. Exemplo:


RAZÃO: Considere a afirmação: "No concurso que você
vai prestar, o número de vagas está para o número de candida-
tos na razão de 1 para 5"

Essa afirmação diz que para cada vaga existente, b) Razão de antecedente zero não possui inversa.
correspondem 5 candidatos e pode ser representada em
matemática por 1/5 (lê-se: 1 para 5 ou 1 está para 5). RAZÕES IGUAIS: São duas razões em que as frações
que as representam são equivalentes, como por exemplo
Afirmações desse tipo, que comparam um número com
outro, por meio de uma divisão do primeiro pelo segundo, , pois se:
chamam-se razões. Então, dizemos que:

"Razão entre dois números racionais a e b, com b g a) e


0, é o quociente de a por b."
b) _ 
Indica-se: a:b ou e lê-se:
 (basta multiplicar os termos cruzados)

Em duas razões iguais observa-se sempre o seguinte:


"a está para b" ou "a para b". "os produtos do antecedente de uma pelo conseqüente da
outra são sempre iguais." Veja:
O número a chama-se antecedente e o número b
conseqüente. Exemplos: Calcular a razão do primeiro número
para o segundo: Se _ 

a) 9e3 _  (basta multiplicar os termos cruzados)

APLICAÇÕES: Verificar se as razões são iguais:


b) 12 e 20 _
a) , temos que:

c)
_

b) temos que:
d) 2,2 e 3,3 _ razão = 2,2 : 3,3 _

_ _

RAZÃO DE DUAS GRANDEZAS: Chama-se razão de ESCALA


uma grandeza a para uma grandeza b, da mesma espécie ao
quociente da divisão dos números que exprimem suas medidas, É uma razão especial que é usada na representação de
tomadas na mesma unidade. Exemplos: (as medidas devem mapas, maquetes, plantas de construções, etc., e a definimos
estar todas na mesma unidade) assim:

a) 10 m e 20 m _ razão = 10 m : 20 m _ "Escala de um desenho é a razão existente entre o


comprimento representado no desenho e o correspondente
comprimento real", medidos na mesma unidade de compri-
_ mento. Então:

b) 2 m e 5 cm _ razão = 2 m : 5 cm _

_ Assim sendo, na escala 1:800 ou 1/800 (lê-se: escala de


1 por 800 ), significa que os comprimentos reais são 800 vezes
maiores que os correspondentes comprimentos no desenho.
Exemplos de aplicações:
c) 4 kg e 5.000 g _ razão = 4 kg : 5.000 g _
a) Qual é a escala do desenho em que um comprimento
_ real de 60 cm está representado por um comprimento de
12 cm?

RAZÕES INVERSAS: São duas razões em que o


antecedente de uma é o conseqüente da outra e vice-versa,
tais como:

a) b)
Resposta: A escala é de 1:5 ou 1/5.

- 23 -
0DWHPiWLFD
b) Num desenho de escala 1:50, qual é o comprimento real Determine o antecedente das seguintes razões, sabendo
correspondente a um comprimento de 8 cm? que:
Tem-se:
21) o conseqüente é 5 e a razão vale
_

22) o conseqüente é e a razão vale




Então: _ 1 # x = 50 # 8 cm _ TESTES

23) Num concurso público concorreram 24.000 candidatos


_ x = 400 cm para 1.200 vagas. A razão entre o número de vagas e o
número de candidatos foi de:
Resp.: O comprimento real é de 400 cm.
a) b) c) d)
EXERCÍCIOS A RESOLVER
24) Um clube tem 1.600 sócios, dos quais 900 são moças. A
Calcular as razões de: razão entre o número de moços e o número de moças é:

1) 40 e 8 a) b) c) d)
2) 8 e 12 5) 1,2 e

3) 6) 4 m e 200 cm Respostas:
7) 2,5 m e 0,5 dam 1) 5 9) 1/5 17) 8 cm
8) 2 m2 e 6.000 dm2 2) 2/3 10) não 18) 20/28
4)
9) 20 5 e 100 dm3 3) 3/20 11) sim 19) 12/32
4) 14/3 12) sim 20) 56/128
5) 3/2 13) não 21) 15/2
6) 2 14) 1/4 22) 3/4
Verificar se são iguais os pares de razões: 7) 1/2 15) 135 m2
8) 1/30 16) 12 meninas
10) 12) 4:8e3:6
Testes: 23) c 24) d

11) 13)
PROPORÇÃO

Sejam os números 3, 6, 4 e 8 e nessa ordem, vamos


Resolver os problemas: calcular:
a) a razão do 1º para o 2º 
14) Dois quadrados têm respectivamente 3 cm e 6 cm de
lado. Qual é a razão entre as superfícies do primeiro
para o segundo? b) a razão do 3º para o 4º 

15) Numa residência, a razão entre a área construída e a Como a razão do 1º para o 2º é igual à razão do 3º para
área livre é de 2:3. Sabendo-se que a área cons- o 4º, escrevemos:
truída é de 90 m2, qual é a área livre?
3:6=4:8 ou
16) Em uma classe mista, a razão entre o número de meni-
nos e o número de meninas é 3:2. Sabendo-se que o
número de meninos é 18, qual o número de meninas? e dizemos que os números 3, 6, 4 e 8, nessa ordem, formam
uma proporção, donde se conclui que: "proporção é uma
igualdade de duas razões."
17) Num desenho de escala 1:100, qual é o comprimento, no
desenho, que corresponde a um comprimento real de 8 De um modo geral, representam-se as proporções das
m? maneiras:

Escreva razões equivalentes a: ou a:b = c:d ou a:b::c:d

18) , cujo antecedente seja 20. e destacamos:

Sugestão: Estabeleça uma igualdade de razões: I- A sua leitura é: a está para b assim como c está para d.

II - a, b, c e d são os termos, na ordem: a é o 1º, b é o 2º,


c é o 3º e d é o 4º termo.
19) , cujo conseqüente seja 32.
III - a e d são os extremos e b e c são os meios.
20) , cujo antecedente seja 56. IV - a e c são os antecedentes e b e d são os conseqüen-
tes.

- 24 -
0DWHPiWLFD
PROPRIEDADE FUNDAMENTAL  aplicando a propriedade fundamental:
DAS PROPORÇÕES
x # 9 = 3 # 15 _ 9x = 45 _ x = _
Por ser uma igualdade de razões, pode-se estabelecer _x=5
que:
"Em toda proporção a : b : : c : d , o produto dos 3) 5 : 6 : : x : 12
meios (b # c) é sempre igual ao produto dos extremos (a # d)  aplicando a propriedade fundamental:
e vice-versa", que é a sua propriedade fundamental.
6 # x = 5 # 12 _ 6x = 60 _ x = _
Então, na proporção: temos: _ x = 10

8 # 15 = 6 # 20
120 = 120 4)
onde: 8 # 15 = produto dos meios
6 # 20 = produto dos extremos
 aplicando a propriedade fundamental:

TRANSFORMAÇÕES DE UMA PROPORÇÃO 3 # x = 8 # 6 _ 3x = 48 _ x = _

Transformar uma proporção, significa dispor os seus _ x = 16


termos de modos diferentes de maneira que a igualdade dos
produtos dos meios e dos extremos não sofra alteração. Nota: Pode-se ver que o termo desconhecido pode ser qual-
Assim, trocando convenientemente a disposição dos termos quer dos meios ou qualquer dos extremos.
de uma proporção, podemos escrevê-la de oito maneiras Outros exemplos:
diferentes. Exemplo:

Escrever das oito maneiras diferentes, a proporção: 5)

1ª) _3 # 4 = 2 # 6

permutando os meios da 1ª:

2ª)
6)
permutando os extremos da 1ª:

3ª)

permutando os meios e os extremos da 1ª:

4ª)

invertendo as razões das proporções: 1ª, 2ª, 3ª e 4ª:

5ª)
7)
6ª)
 calculam-se as operações entre parênteses:

7ª)

8ª)

CÁLCULO DO TERMO DESCONHECIDO


DE UMA PROPORÇÃO (RESOLUÇÃO)

Resolver uma proporção significa encontrar o valor do


seu termo desconhecido e para isso basta aplicar a sua 8)
propriedade fundamental. Veja os exemplos seguintes.

Calcular o termo desconhecido em:


1) x:8=5:2
 aplicando a propriedade fundamental:
x # 2 = 8 # 5 _ 2x = 40 _ x = 40/2 _ 9)
_ x = 20

2)

- 25 -
0DWHPiWLFD
QUARTA PROPORCIONAL TERCEIRA PROPORCIONAL

Se quatro números, como por exemplo: 9, 4, 18 e 8, É o quarto termo de uma proporção contínua. Então,
formam, nessa ordem, uma proporção, diz-se que o 4º se numa proporção contínua ocorrer: 2 : 10 = 10 : x , vemos
número (8) é a quarta proporcional dos números 9, 4 e 18. que x é um terceiro elemento diferente, que com os outros dois
Então, para se encontrar a quarta proporcional dos números a, (2 e 10) formam essa proporção contínua. Diz-se, então, que x
b e c, basta formar com eles uma proporção, tal que: é a terceira proporcional dos números 2 e 10 e pode-se
a : b = c : x, onde x é a quarta proporcional de a, b e c. concluir que, para calcular a terceira proporcional de dois
Exemplos: números, a e b, basta formar com eles, nessa ordem, uma
proporção contínua, onde b é o meio igual, ou seja:
1) Calcular a quarta proporcional dos números 3, 6 e 16. a : b = b : x. Exemplos:
Forma-se com eles, nessa ordem, uma proporção onde
x é o quarto termo e passa-se a resolvê-la: 1) Calcular a terceira proporcional dos números 16 e 8.
3 : 6 = 16 : x _ 3 # x = 6 # 16 _ Forma-se a proporção contínua onde 8 é o meio igual.
_ 3x = 96 _ x = _ x = 32 Então: 16 : 8 = 8 : x _
_ 16 # x = 8 . 8 _ 16x = 64 _
_ _x=4
2) Qual é a quarta proporcional dos números
2) Qual é a terceira proporcional dos números
Forma-se a proporção:

Proporção contínua: é aquela que tem ou os meios ou PROPRIEDADES DAS PROPORÇÕES


os extremos iguais, tais como:
1ª) PROPRIEDADE DA SOMA DOS TERMOS
a) b)
Em qualquer proporção, a soma dos dois primeiros
termos está para o primeiro (ou para o segundo), assim como
a soma dos dois últimos termos está para o terceiro (ou para
MÉDIA PROPORCIONAL OU GEOMÉTRICA o quarto).
É o meio ou extremo igual de uma proporção contínua.
Então, em , temos:
Assim, nas proporções 36 : 12 = 12 : 4 e 9 : 3 = 27 : 9,
dizemos que 12 é a média proporcional ou geométrica dos
números 36 e 4 e 9 é a média proporcional ou geométrica 1 ou 2
dos números 3 e 27.

Então, quando se quer calcular a média geométrica de


dois números, basta formarmos com eles uma proporção 2ª) PROPRIEDADE DA DIFERENÇA DOS TERMOS
contínua, onde os números dados figurem ou como meios ou
como extremos, resolvendo-se a proporção obtida, em Em qualquer proporção, a diferença dos dois primeiros
seguida. Exemplos: termos está para o primeiro (ou para o segundo), assim como
a diferença dos dois últimos termos está para o terceiro (ou
1) Calcule a média proporcional dos números 20 e 5. para o quarto).
Forma-se a proporção contínua:
Então, em , temos:
20 : x = x : 5 , onde: x # x = 20 # 5 _
_ x2 = 100 _ _ x = 10
(1) ou (2)
2) Qual é a média geométrica dos números ?
APLICAÇÃO: Calcular dois termos de uma proporção,
Forma-se a proporção contínua: desde que sejam conhecidos ou a sua soma ou a sua diferen-
ça. Exemplos:

1) Encontrar dois números cuja soma é 48 e que estão


entre si na razão de 3 para 5.
Solução:
Chamando de x e y os números procurados, vem:

Nota: Pode-se concluir que a média proporcional ou geomé-


trica de dois números é igual à raiz quadrada do
produto desses números.

- 26 -
0DWHPiWLFD
Aplicando a propriedade da soma dos termos, vem
(1) ou (2)
, temos:
4ª) PROPRIEDADE DA DIFERENÇA DOS ANTECEDEN-
TES E DOS CONSEQÜENTES
(1) ou (2)
Em qualquer proporção a diferença dos antecedentes
está para a diferença dos conseqüentes, assim como cada
antecedente está para o seu conseqüente.
Como x + y = 48 e substituindo em (1) e (2), vem:

Então em , temos

(1) ou (2)

APLICAÇÃO: Cálculo de dois antecedentes ou dois


conseqüentes de uma proporção, desde que sejam conhecidos
ou a sua soma ou a sua diferença. Exemplo:

1) Resolver a proporção , sabendo-se que x+y=42.

Solução:
Resp.: Os números procurados são 18 e 30.
Aplicando a propriedade da soma dos antecedentes e
2) Calcular dois números cuja diferença entre eles é 20 e conseqüentes, vem:
que estão entre si na razão de 6 : 4.
Solução: Sejam a e b os números procurados. , temos:
Então:

(1) ou (2)

Como x + y = 42 , vem:
Aplicando a propriedade da diferença dos termos, vem:

, temos:

(1) ou (2)

Como a – b = 20 , vem:

Resp.: x = 18 e y = 24.

2) Resolva a proporção , sabendo-se que


a – b = 21.
Solução:
Aplicando a propriedade da diferença dos antecedentes
e conseqüentes, vem:

, temos:

Resp.: Os números procurados são 60 e 40.


(1) ou (2)
3ª) PROPRIEDADE DA SOMA DOS ANTECEDENTES E
DOS CONSEQÜENTES
Como a – b = 21 , vem:
Em qualquer proporção a soma dos antecedentes está
para a soma dos conseqüentes, assim como cada antece-
dente está para o seu conseqüente.

Então em , temos:

- 27 -
0DWHPiWLFD

2) Calcular x e y na proporção , sabendo-se que x2

+ y2 = 52.
Solução:

Resposta: a = 70 e b = 49. Forma-se o sistema:

5ª) PROPRIEDADE DO PRODUTO DOS ANTECEDENTES


E DOS CONSEQÜENTES
Aplicando a conseqüência, vem:
Em qualquer proporção o produto dos antecedentes
está para o produto dos conseqüentes, assim como o
quadrado de cada antecedente está para o quadrado do seu
conseqüente. Logo, o sistema fica:

Então em , temos onde, aplicando a propriedade da soma dos termos


(1ª), vem:

(1) ou (2) , temos:

CONSEQÜÊNCIA: Em qualquer proporção, os quadra- (1) ou (2)


dos de seus termos também formam uma proporção.

Então, se _ Como x2 + y2 = 52 , vem:

APLICAÇÃO: Cálculo de dois termos de uma propor-


ção, desde que seja conhecido seu produto ou o quadrado de
seus termos. Exemplos:

1) Determinar dois números, sabendo que seu produto é


180 e sua razão é .
Solução:
Chamando de a e b os números procurados e formando
o sistema, vem:
Resp.: x = 4 e y = 6.

SÉRIE DE RAZÕES IGUAIS (PROPORÇÃO MÚLTIPLA)

Se as razões forem todas iguais, pode-se


Aplicando a propriedade do produto dos antecedentes
e conseqüentes, vem:
escrever: , formando uma série de razões
, temos iguais que são chamadas de proporções múltiplas, nas
quais valem também as propriedades da soma (ou diferença)
dos antecedentes e dos conseqüentes, ou seja: "Em
qualquer proporção múltipla, a soma (ou diferença) dos
(1) ou (2) antecedentes está para a soma (ou diferença) dos conse-
qüentes, assim como cada antecedente está para o seu
conseqüente."
Como a # b = 180 , vem:
APLICAÇÃO: Calcular a, b e c em ,

sabendo-se que a – b + c = 33.


Solução:
Aplicando a propriedade da soma (ou diferença) dos
antecedentes e conseqüentes, vem:

_ (1)

ou (2) ou (3)
Resp.: Os números são 12 e 15.

- 28 -
0DWHPiWLFD
Como a – b + c = 33 , vem: Resolver os problemas:

26) A diferença entre dois números é 15 e a razão ente eles


é 8/5. Calcule-os.

27) A diferença dos quadrados de dois números é 144 e


estão entre si na razão de 5 para 3. Quais são os
números?
28) O produto de dois números é 96 e a sua razão é 2 para
3. Quais são eles?

29) A soma de dois números é 55 e o maior está para 8


assim como o menor está para 3. Calcule-os.

TESTES

30) A quarta proporcional dos números 1/2, 3/4 e 2/3 é:


a) 4 b) 1/2 c) 1/4 d) 1
Resp.: a = 24, b = 6 e c = 15.
31) A média proporcional dos números 27 e 3 é:
EXERCÍCIOS A RESOLVER a) 1458 b) 729 c) 81 d) 9

Resolver as proporções (aplicar a propriedade fundamen-


32) Na proporção , o valor de x é:
tal)

1) 5 : 6 = 2x : 3 2) x: a) 9 b) 3 c) –3 d) –9

3) 3x : = 0,4 : 4) 33) Sabendo-se que x – y = 40 e , então

x + y é igual a:
5) 6)
a) 72 b) 144 c) 36 d) 18

7) 34) Se , então o valor de x é:

8) a) b) c) 10 d) 5

Calcular a quarta proporcional de:


35) O valor de x na proporção , é:
9) 8, 12 e 10 10)
a) 8 b) 7 c) 6 d) 4
11) 0,4; 0,6 e 1,2
36) Um garoto de 1 m de altura projeta uma sombra de 0,5
Calcular a média proporcional de: m. No mesmo instante, um edifício de 18 m irá projetar
uma sombra de:
12) 8e2 13)
a) 12 m b) 9 m c) 8 m d) 6 m
14) 3e 15) 3,2 e 0,2 37) Sendo e x – y = 15, o valor de x + y é:

Calcular a terceira proporcional de: a) 41 b) 40 c) 39 d) 37


16) 3 e 12 17) 5 e 20 38) A razão entre a minha idade e a idade do meu primo é
19) de 2 para 5 e juntos temos 42 anos. Então, tenho:
18) 1 e 25
a) 16 anos b) 14 anos c) 12 anos d) 10 anos

Calcular os elementos desconhecidos nas proporções, 39) Cortaram 20 kg de carne em dois pedaços, cuja razão é
usando a propriedade cabível em cada caso: 2/3. O pedaço maior pesa:
a) 11 kg b) 12 kg c) 14 kg d) 15 kg
20) 21)

RESPOSTAS:

22) 23) 1) 5/4 11) 1,8 21) 35 e 7 31) d


2) 1 12) 4 22) 8 e 12 32) b
3) 4/75 13) 9/5 23) 9 e 15 33) a
4) 8 14) 2/3 24) 3,9 e 15 34) a
Calcular os termos desconhecidos em: 5) 12 15) 0,8 25) 25, 10 e 15 35) d
6) – 23 16) 48 26) 40 e 25 36) b
7) 182/27 17) 80 27) 15 e 9 37) c
8) 75/16 18) 625 28) 8 e 12 38) c
24) 25) 9) 15 19) 1/18 29) 40 e 15 39) b
10) 20/27 20) 8 e 12 30) d

- 29 -
0DWHPiWLFD
NÚMEROS E GRANDEZAS PROPORCIONAIS DIVISÃO DE UM NÚMERO EM PARTES
DIRETAMENTE PROPORCIONAIS
NÚMEROS DIRETAMENTE PROPORCIONAIS
1) Dividir o número 180 em partes diretamente propor-
cionais a 3, 4 e 11.
Sejam os conjuntos A e B de números racionais que
estão em correspondência biunívoca (mesma quantidade de Solução:
elementos): a) representar os números por a, b e c.
b) considerar as sucessões (a, b, c) e (3, 4, 11)
A = {2, 5, 8, 11} e B = {6, 15, 24, 33}
como diretamente proporcionais
Formando as razões entre os elementos corresponden-
tes de A e B, temos:
Então:
, onde:

e Calcula-se o coeficiente de proporcionalidade,


aplicando-se a propriedade das proporções múltiplas, ou
seja:
ou seja, essas razões são constantes e todas iguais a
, donde se pode escrever: 

 coeficiente de proporcionalidade, e o valor de cada


uma das partes a, b e c será encontrado pelo produto
de cada um dos números 3, 4 e 11 pelo coeficiente de
Diz-se então que os elementos dos conjuntos A e B são proporcionalidade.
diretamente proporcionais ou simplesmente proporcionais
e conclui-se que: Então, vem:

"Duas sucessões de números são diretamente


proporcionais quando as razões existentes entre um Resp.: As partes são 30, 40 e 110.
elemento qualquer da primeira e o seu correspondente na
segunda sucessão são constantes (iguais)."
2) Dividir o número 372 em partes diretamente propor-
A razão constante que existe entre os dois conjuntos
chama-se fator de proporcionalidade ou coeficiente de cionais a
proporcionalidade.
Solução:
APLICAÇÃO: O número 372 deve ser dividido em três parcelas: a, b,
c (a+b+c = 372) em que a série de razões iguais será:
Calcular os valores de a, b e c dos conjuntos de números
diretamente proporcionais:

A = {2, b, 4, 7} e B = {a, 9, c, 21}.

Solução:
Como são diretamente propor-cionais os dois conjun-
tos, forma-se as razões iguais entre os elementos Daí o sistema:
correspondentes de A e B:

de onde vem:
Então:

 coeficiente de proporcionalidade
Como uma proporcionalidade não se altera quando se
multiplica todos os números do conjunto por um mesmo
Igualando cada razão a (coeficiente) e calculando número, pode-se então reduzir as frações ao mesmo
denominador e desprezar, em seguida, o denomina-
o termo desconhecido, temos: dor, a fim de que as partes sejam substituídas por
números inteiros. Então, a expressão anterior fica:

m.m.c. (2, 3, 5) = 30 (despreza-se o denominador)

E, por conseguinte, o problema agora consiste em dividir


372 em partes diretamente proporcionais a 15, 10 e 6,
Resp.: Os valores são: a = 6, b = 3 e c = 12. ou seja, como no primeiro exemplo:

- 30 -
0DWHPiWLFD
de onde vem:

3 # 40 = 120  coeficiente de proporcionalidade

Igualando cada produto a 120 (coeficiente) e calculan-


do o termo desconhecido, temos:

Então:

Resp.: As partes são 180, 120 e 72.

3) (Solução direta)  Dividir 183 em partes diretamente


Resp.: Os valores são: a = 30, b = 6 e c = 12.
proporcionais a

DIVISÃO DE UM NÚMERO EM PARTES


INVERSAMENTE PROPORCIONAIS

Dividir um número em partes inversamente proporcio-


nais a números dados, significa dividi-lo em partes direta-
mente proporcionais aos inversos dos números dados.
Exemplo:

1) Dividir o número 18 em partes inversamente proporcio-


nais a 2, 3 e 6.
Solução:
Então: O problema consiste em dividir 18 em partes direta-
mente proporcionais aos inversos de 2, 3 e 6, que
são: 1/2, 1/3 e 1/6. Então:
Resp.: As partes são 84, 63 e 36.

NÚMEROS INVERSAMENTE PROPORCIONAIS

Sejam agora os conjuntos C e D de números racionais


que também estão em correspondência biunívoca:
C = {1, 3, 5, 10} e D = {60, 20, 12, 6}

Calculando os produtos entre os números do conjunto


C e os correspondentes do conjunto D, temos:
1 # 60; 3 # 20; 5 # 12; 10 # 6, vê-se que esses produtos são
constantes e todos iguais a 60, donde se pode escrever: Logo:

1 # 60 = 3 # 20 = 5 # 12 = 10 # 6 = 60

Diz-se então que os elementos dos conjuntos C e D são


inversamente proporcionais e conclui-se que: Resp.: As partes são 9, 6 e 3.
"Duas sucessões de números são inversamente
proporcionais quando os produtos entre um elemento 2) Dividir o número 200 em partes inversamente proporcio-
qualquer da primeira pelos correspondentes na segunda
sucessão são constantes (iguais)." nais a

Esses produtos iguais chamam-se também fator de Solução:


proporcionalidade ou coeficiente de proporcionalidade.
É o mesmo que dividir 200 em partes diretamente
proporcionais aos inversos de , que são: 3 e 5.
APLICAÇÃO:
Então:
Determinar os valores de a, b e c dos conjuntos de
números inversamente proporcionais: A = {3, 4, b, 10} e B =
{40, a, 20, c}.

Solução:
Como os dois conjuntos são inversamente proporcio-
nais, forma-se os produtos iguais entre os elementos
correspondentes de A e B: Logo:

Resp.: As partes são 75 e 125.

- 31 -
0DWHPiWLFD
EXERCÍCIOS A RESOLVER TESTES

Verificar se são direta ou inversamente proporcionais a


sucessão de números que formam os conjuntos A e B de 18) As sucessões de números são diretamen-
cada exercício: te proporcionais. Então, o coeficiente de proporcionali-
dade é:
1) A={1, 5, 2} 3) A={2, 3, 4, 6}
B={3, 15, 6} B={48, 32, 24, 16} a) 135 c) 75
b) 25 d)
2) A={6, 4, 12, 2}
B={8, 12, 4, 24}

19) As sucessões de números são inversamen-


Calcular x e y sabendo-se que os conjuntos A e B são
diretamente proporcionais: te proporcionais. Então, o coeficiente de proporcionalida-
de é:
4) A = {5, x, 20} 6) A = {1, x, 7}
B = {3, 6, y} B = {5, 15, y} a) 60 c)

5) A = {x, 12, 15} b) d)


B = {28, y, 20}

Calcular m e n sabendo-se que os conjuntos A e B são 20) As sucessões de números são inversa-
inversamente proporcionais:
mente proporcionais. Então, a+b é igual a:
7) A = {m, 2, 3} 9) A = {2, 10, n} a) 63 c) 28
B = {7, n, 14} B = {m, 9, 15} b) d) 21
8) A = {3, m, 10}
B = {5, 25, n}
21) Quero repartir o número 380 em parcelas que são
inversamente proporcionais aos números 2, 5 e 4,
respectivamente. Essas parcelas serão:
Dividir:
a) 200, 80 e 100
b) 150, 130 e 100
10) 888 em partes proporcionais a 18, 11, 21 e 24. c) 180, 60 e 140
d) 200, 60 e 120
11) 3.250 em partes proporcionais a 0,4; 1,2 e 3,4.
22) Uma pessoa divide R$ 13.000,00 proporcionalmente às
idades de seus 3 filhos, que têm respectivamente 3, 4 e
12) 4.000 em partes proporcionais a 6 anos. Quanto receberão o filho mais novo e o mais
velho?
13) 380 em partes inversamente proporcionais a 2, 5 e 4. a) R$ 2.000,00 e R$ 6.000,00
b) R$ 3.000,00 e R$ 6.000,00
c) R$ 3.000,00 e R$ 4.000,00
14) 459 em partes inversamente proporcionais a 3, 4, 10 e 6. d) R$ 4.000,00 e R$ 6.000,00

PROBLEMAS:

15) Um pai distribuiu R$ 5.000,00 aos seus três filhos em RESPOSTAS:


partes diretamente proporcionais às suas idades, que
são 4, 7 e 9 anos. Quanto coube a cada um? 1) diretamente proporcionais
2) inversamente proporcionais
3) inversamente proporcionais
4) x = 10 e y = 12
16) Um tio oferece R$ 6.000,00 para serem repartidos entre 5) x = 21 e y = 16
seus três sobrinhos, em partes inversamente proporcio- 6) x = 3 e y = 35
nais às faltas à escola que deram durante o mês. Quanto 7) m = 6 e n = 21
coube a cada sobrinho, sabendo que dois deles faltaram 8) m = 3/5 e n = 3/2
2 vezes cada um e outro faltou 5 vezes? 9) m = 45 e n = 6
10) 216, 132, 252 e 288
11) 260, 780 e 2.210
12) 960, 1.440 e 1.600
17) Um reservatório de 25.200 m3 de capacidade foi comple- 13) 200, 80 e 100
tamente cheio por 3 torneiras que despejaram por minuto 14) 180, 135, 54 e 90
12 5, 8 5 e 16 5 de água respectivamente. Determinar o 15) R$ 1.000,00; R$ 1.750,00 e R$ 2.250,00.
volume de água que cada torneira despejou. 16) R$ 2.500,00; R$ 2.500,00 e R$ 1.000,00.
17) 8.400 5; 5.600 5 e 11.200 5
18) d 19) a 20) c 21) a 22) b

- 32 -
0DWHPiWLFD
REGRA DE TRÊS SIMPLES E COMPOSTA 1) Se 5 operários tecem 800 m de fazenda por dia, quantos
metros tecerão 9 operários?
GRANDEZAS DIRETAMENTE PROPORCIONAIS: Solução:
Indicando por x a quantidade de metros que farão os 9
São duas grandezas que, quando o valor de uma delas operários, temos a seguinte disposição prática:
aumenta ou diminui, o valor da outra aumenta ou diminui o
mesmo número de vezes. Então, se: 5 operários . . . . . . . . . . . 800 m
9 operários . . . . . . . . . . . x
5 m de tecido custam . . . . . . . . . . . . R$ 20,00
10 m custarão . . . . . . . . . . . . . . . . . . . R$ 40,00 Se 5 operários tecem 800 m, mais operários tecerão
15 m custarão . . . . . . . . . . . . . . . . . . . R$ 60,00 mais metros.
Como nesse exemplo as grandezas: número de operári-
Nota-se que quando o valor da primeira (comprimento) os e quantidade de metros são diretamente proporcio-
torna-se o dobro, o triplo, etc., o mesmo ocorre com o valor da nais, assinalamos essa variação na disposição prática,
outra (custo) e por isso mesmo essas duas grandezas são através de flechas no mesmo sentido. A proporção
diretamente proporcionais. resultante e sua solução é:
A propriedade que caracteriza a existência de grandezas
diretamente proporcionais é: "a razão entre os valores de
uma é igual à razão entre os valores correspondentes da
outra." No exemplo acima, temos:

ou ou Resp.: 9 operários tecerão 1.440 metros de fazenda.

2) Se 12 operários demoram 15 dias para executar um


trabalho, 10 operários, em quanto tempo farão o mesmo
trabalho?

onde as flechas de mesmo sentido indicam que as razões 12 operários . . . . . . 15 dias


10 operários . . . . . . x
resultaram de grandezas diretamente proporcionais.
É óbvio que, se 12 operários demoram 15 dias, menos
operários demorarão mais dias para fazer o mesmo
GRANDEZAS INVERSAMENTE PROPORCIONAIS: trabalho.
São duas grandezas que, quando o valor de uma delas Como o tempo necessário para efetuar um trabalho é
aumenta ou diminui, o valor da outra diminui ou aumenta o inversamente proporcional ao número de operá-
mesmo número de vezes. Então, se: rios empregados, assinalamos essa variação, na disposi-
ção prática, com flechas de sentidos contrários.
3 operários fazem um serviço em 36 dias
Invertendo a primeira razão , para que as flechas
6 operários farão o mesmo serviço em 18 dias
9 operários o farão em 12 dias tomem o mesmo sentido, temos a proporção:

Nota-se que quando o valor da primeira (operários) torna-


se o dobro, o triplo, etc., o valor da outra (tempo) torna-se a
metade, um terço, etc., e por isso mesmo essas duas grande-
zas são inversamente proporcionais.

A propriedade que caracteriza a existência de grandezas Resp.: 10 operários farão o mesmo trabalho em 18
inversamente proporcionais é: "a razão entre os valores de dias.
uma é igual ao inverso da razão entre os valores corres-
pondentes da outra." No exemplo acima, temos: 3) Certo automóvel percorre 330 km em 5 horas. Conser-
vando a mesma velocidade quantos quilômetros percor-
rerá em 9 horas?
ou ou
330 km . . . . . . . . 5 horas
x . . . . . . . . . . . . 9 horas
onde as flechas de sentidos contrários indicam que as
razões resultaram de grandezas inversamente proporcionais. Se, em 5 horas percorre 330 km, em mais horas percor-
rerá mais km. Portanto, a regra de três é direta 
flechas de mesmo sentido. Então:
REGRA DE TRÊS SIMPLES

São problemas que envolvem duas grandezas direta ou


inversamente proporcionais. Resolvê-los, consiste em formar
com os três valores conhecidos e a incógnita procurada, uma
proporção e dela tirar o valor desejado. Para isso faz-se:
Resp.: Percorrerá 594 km.
1º) Escreve-se numa mesma coluna as grandezas de
mesma espécie. 4) Um avião, com a velocidade de 320 km/h, vence a
distância entre duas cidades em 6 horas. Outro avião,
2º) Identifica-se se as grandezas são direta ou inversamen- com a velocidade de 360 km/h, em quanto tempo
te proporcionais. percorrerá essa mesma distância?

3º) Escreve-se a proporção correspondente e passa-se a 320 km/n . . . . . . . . . . 6 horas


resolvê-la. Exemplos: 360 km/h . . . . . . . . . . x

- 33 -
0DWHPiWLFD
Se, voando a 320 km/h demora 6 horas, voando a mais Ora, se para fazer 40 metros de muro são necessários 8
km/h, demorará menos horas. Então, a regra de três é pedreiros, para fazer mais metros de muro (70) serão
inversa  flechas de sentidos contrários. necessários mais pedreiros. Regra de três direta 
flechas de mesmo sentido.
Invertendo a razão e resolvendo a proporção, vem:
Resumindo as letras a e b, vem:
6 dias
f 8x pedreiros
pedreiros
 14 dias  40 m / comp.
70 m / comp.
Notamos, na disposição prática, que as flechas que têm
o mesmo sentido da do grupo que contém x, indicam grande-
zas diretamente proporcionais e as de sentido contrário
Transformando 16/3h em horas e minutos, vem: indicam grandezas inversamente proporcionais. Então, a
razão desse grupo de grandezas inversamente proporcio-
nais (6 / 14) deve ser invertida, a fim de tomar o mesmo
_ sentido das grandezas diretamente proporcionais.

No grupo que contiver x, não se faz alterações. Somen-


te escreve-o como se encontrar, na posição do 1º grupo, e,
_ em seguida, transcreve-se os demais grupos, fazendo a
inversão dos grupos que forem inversamente proporcionais.
Resp.: O outro avião percorrerá a mesma distância em Tem-se então:
5 horas e 20 minutos.

REGRA DE TRÊS COMPOSTA


Feito isto, conserva-se a razão que tem x e
São problemas que envolvem três ou mais grandezas multiplicam-se entre si as demais razões, simplificando-as
direta ou inversamente proporcionais. Para resolvê-los, faz-se: se for possível. Então vem:
1º) Escreve-se numa mesma coluna as grandezas de
mesma espécie.

2º) Identifica-se se as grandezas são direta ou inversamen-


te proporcionais, considerando as colunas duas a duas,
sendo que uma delas deve conter o termo desconheci-
do.

3º) Escreve-se a proporção correspondente, igualando a


razão que contém o termo desconhecido com o produ- Resp.: Serão necessários 6 pedreiros.
to das outras razões, e passa-se a resolvê-la. Exem-
plos: 2) Sete operários, em 5 dias de 8 horas, fazem 2.800 m de
tecido. Quantos operários serão necessários para fazer
1) Se 8 pedreiros constroem em 6 dias um muro de 40 m 2.160 m do mesmo tecido em 9 dias de 6 horas?
de comprimento, quantos pedreiros serão necessários
para construir, em 14 dias, um muro de 70 m de compri- (1º Grupo) (2º Grupo) (3º Grupo) (4º Grupo)
mento? 7 operários 5 dias 8 horas 2.800 m
Solução f x operários  9 dias  6 horas  2.160 m
Temos a seguinte disposição prática:
a) Comparação do 2º com o 1º grupo
(1º Grupo) (2º Grupo) (3º Grupo)
8 pedreiros 6 dias 40 m/comp.
f x pedreiros  14 dias  70 m/comp.  5 dias
mais dias f 7 operários
menos operários
Para resolvermos o problema proposto, comparamos
cada grupo de valores com o grupo em que estiver o x (no Regra de três inversa  flechas de sentidos contrá-
exemplo, o 1º grupo), colocando-lhe à esquerda, uma flecha de rios.
formato diferente das demais para servir como termo de
b) Comparação do 3º com o 1º grupo
comparação. Nessa comparação deveremos observar apenas
o grupo comparado com o que tem x, sem preocupação com
qualquer outro grupo, para observarmos se esses valores
formam regra de três direta ou inversa. Desta forma, temos:
 8 horas
menos horas f 7 operários
mais operários

a) Comparação do 2º com o 1º grupo: Regra de três inversa  flechas de sentidos contrá-


rios.
Se em
em 146 dias são necessários
dias serão necessáriosf
8 pedreiros
x pedreiros c) Comparação do 4º com o 1º grupo
Ora, se em 6 dias são necessários 8 pedreiros para fazer
o muro, em mais dias (14) serão necessários menos  2.800 m
menos metros f 7 operários
menos operários
pedreiros. Regra de três inversa  flechas de sentidos
contrários. Regra de três direta  flechas de mesmo sentido.
b) Comparação do 3º com o 1º grupo: Então:
7 operários 5 dias 8 horas 2.800 m
Se para fazer
para fazer  40 m de muro são necessários
70 m de muro serão necessários f
8 pedreiros
x pedreiros f x operários  9 dias  6 horas  2.160 m
- 34 -
0DWHPiWLFD
4) Para forrar as paredes de uma sala, são necessárias 30
Invertendo as razões peças de papel de 60 cm de largura. Quantas peças de
90 cm de largura seriam necessárias para forrar a
f    mesma sala? R.: 20 peças.

Conservando-se a razão com x e multiplicando-se as 5) Se, de cada 30 kg de café cru resultam 25 kg de café
torrado, quantos kg de café cru serão necessários para
demais, vem:
se obter 200 kg de café torrado? R.: 240 kg.
Simplificando fica:
6) Um circo é armado por 15 homens que trabalham 10
horas por dia, em 3 dias. Em quanto tempo armariam
esse circo, 10 homens que trabalhassem 9 horas por
dia? R.: 5 dias.
Resp.: Serão necessários 4 operários.
7) Com uma bomba elétrica, eleva-se 4.200 litros de água
3) Foram empregados 24 kg de fio para tecer 120 m de à altura de 12 m, em 1 hora e 20 minutos. Quanto tempo
fazenda de 0,82 m de largura. Quantos metros da empregará essa bomba para elevar 12.600 litros a altura
mesma fazenda, de 1,23 m de largura serão tecidos com de 8 metros? R.: 2 h 40 min.
30 kg do mesmo fio?
8) Com 15 operários, em 18 dias gastou-se R$ 405,00 para
24 kg / fio 120 m
 30 kg / fio f x  0,82 m / larg.
1,23 m / larg.
fazer certo trabalho. Quanto se gastaria para um trabalho
semelhante, dispensando-se 8 operários, sendo que os
restantes fariam o trabalho em 12 dias? R.: R$ 126,00.

_  f  9) Um automóvel com a velocidade média de 60 km / h,


rodando 7 horas por dia, leva 20 dias para fazer certo
Simplificando fica: percurso. Quantos dias levaria o mesmo automóvel, para
fazer aquele percurso, se viajasse 12 horas por dia, com
a velocidade média de 50 km / h? R.: 14 dias.

10) Um livro tem 250 páginas de 40 linhas cada, sendo cada


linha composta por 66 letras. Reimprimindo-o com os
mesmos caracteres, porém com páginas de 30 linhas de
50 letras cada uma, quantas páginas terá o novo livro?
Resp.: Serão tecidos 100 metros. R.: 440 páginas.

4) Se o transporte, por estrada de ferro, de 15 toneladas de 11) Se 80 operários, trabalhando 10 horas por dia teceram
certa mercadoria, à distância de 400 km custa R$ 90,00, 7.500 m de fazenda em 25 dias, quantos metros do
qual será o frete de 32 toneladas, ao mesmo preço por mesmo tecido farão 54 operários trabalhando 8 horas
km, em 250 km? por dia, durante 30 dias? R.: 4.860 m.
15 toneladas 400 km R$ 90,00
 32 toneladas  250 km f x 12) Um automóvel gasta 10 litros de gasolina para percorrer
65 km. Quantos litros gastará num percurso de 910 km?
R.: 140 litros.
_ f  
TESTES
Simplificando fica:
12) Uma máquina produz 600 peças em 20 minutos. Quan-
tas peças produzirá em 50 minutos?
a) 675 b) 1500 c) 2000 d) 3000

13) Se 8 tratores realizam certo trabalho em 15 dias, 10


tratores realizariam o mesmo trabalho em:
Resp.: O frete será de R$ 120,00. a) 12 dias b) 16 dias c) 6 dias d) 8 dias

PROBLEMAS PARA RESOLVER: 14) Na construção de um muro de 24 m de comprimento


foram utilizados 3120 tijolos. Para construir um muro de
60 m de comprimento serão necessários quantos tijolos:
1) Duas rodas dentadas, engrenadas uma na outra, têm
respectivamente, 24 e 108 dentes. Quantas voltas dará a) 7728 b) 5184 c) 5400 d) 7800
a menor, enquanto a maior dá 16? R.: 72 voltas.
15) Em 3 dias, 4 máquinas produzem 600 peças. Para
2) Numa cocheira existem 30 cavalos, para os quais uma produzir 900 peças em 2 dias, serão necessárias quan-
certa quantidade de feno dura 40 dias. Tendo sido tas máquinas:
retirados 10 cavalos, quanto tempo durará agora aquela
quantidade de feno? R.: 60 dias. a) 24 b) 15 c) 9 d) 6

3) Uma pessoa, dando 51 passos por minuto, demora 15


minutos para percorrer certa distância. Que tempo RESPOSTAS:
demorará para percorrer a mesma distância, se, em
cada minuto, der 45 passos? R.: 17 minutos. 12) b 13) a 14) d 15) c

- 35 -
0DWHPiWLFD
PORCENTAGEM
Então: .

A razão entre dois valores quaisquer de uma grandeza


pode ser representado com um conseqüente ou denominador O caso inverso é:
qualquer. Suponha então que numa caixa de frutas, contendo
laranjas e mexericas, num total de 90 frutas, 27 delas sejam (após simplificada).
laranjas. A razão entre o número de laranjas e o total de
frutas será 27/90, que pode ser representada de várias Logo: .
formas, como por exemplo:
No comércio, para simplicidade nos cálculos, usa-se
, etc. . . determinar as comissões, os lucros, os prejuízos, os abatimen-
tos, os juros, as corretagens, etc., em proporções a 100
unidades de outra grandeza da mesma espécie. Isto significa
Então, se pode dizer, com o mesmo sentido, que na que, quando se diz que um corretor recebeu 7% de comissão,
caixa de frutas, 27/90 das frutas são laranjas; ou 3/10 das quer-se dizer que, em cada 100 reais, a parte que lhe coube foi
frutas são laranjas; ou 12/40 das frutas são laranjas; ou 30/100 7 reais.
das frutas são laranjas, etc.

RAZÃO CENTESIMAL (OU PERCENTUAL) TERMOS DA PORCENTAGEM

É a razão representada com o denominador ou conse- Em todo problema de porcentagem, deve-se distinguir
qüente 100 e é chamada de percentagem ou porcentagem. quatro elementos
No exemplo acima, a razão com forma de porcentagem é
30/100, que pode também ser escrita "30%", em que o símbo- 1º) O PRINCIPAL que é o número total sobre o qual se quer
lo "%" indica porcentagem. calcular a porcentagem. (todo em espécie). É represen-
tado por P.
O numerador ou antecedente "30" da razão chama-se
taxa de porcentagem e o número total de frutas "90" é 2º) A PORCENTAGEM que é a parte que se quer encontrar
chamado principal. do principal e é da mesma espécie do principal (parte
do principal). É representada por p.
É bom notar que o número de laranjas "27", é uma
fração do todo "90", ou seja, vale de 90, ou simplesmente 30% 3º) O NÚMERO FIXO 100 que representa o total em %
de 90. Então se diz que 27 é 30% de 90, ou, 30% das frutas (todo em %). Nunca aparece no problema e é represen-
são laranjas. tado por 100.

Pode-se representar uma razão sob a forma de porcen- 4º) A TAXA DE PORCENTAGEM que é o número de partes
tagem, e, reciprocamente, representar uma porcentagem sob que devem ser tomadas em cada 100 partes do principal
a forma de fração irredutível. Observe: (parte em %). É representada por i.

1º) Representar a razão sob a forma de porcentagem.


RESOLUÇÃO DE PROBLEMAS
Solução:
Consiste em achar uma razão igual a 3/5 e de conseqüen- Os problemas relativos a porcentagem são resolvidos
te 100. Então, representando por x o antecedente da facilmente, por meio de regra de três simples e direta,
razão procurada, forma-se a proporção seguindo-se o critério:

Ao principal (todo em espécie)  corresponde 


, de onde vem: 100% (todo em %) e à porcentagem (parte do principal) 
corresponde  taxa de porcentagem (parte em %).

PROBLEMAS RESOLVIDOS
Logo, a porcentagem procurada será:
Siga, com muita atenção, os modelos:

1º) Calcular quanto deve receber um corretor pela venda de


Reciprocamente, teríamos: representar 60% sob a forma um terreno no valor de R$ 50.000,00, se a comissão foi
estipulada em 3,5%.
de fração irredutível.
Solução:
(após simplificada). Distinguindo os quatro elementos do problema, temos:

Então, a fração irredutível correspondente a 60% é (P)  PRINCIPAL = 50.000,00 (todo em espécie) 
é o valor total.
(p)  PORCENTAGEM = x (parte do prin-cipal) 
2º) Representar sob a forma de porcentagem. é o que vai ser calculado.
Solução: (100)  NÚMERO FIXO = 100% (todo em %) 
Segundo o mesmo critério anterior, vem: nunca aparece escrito no problema.
(i)  TAXA DE PORCENTAGEM = 3,5% (parte em
%)  é a parte do 100% que o corretor vai
ganhar de comissão.

Com esses quatro elementos, arma-se o dispositivo


(regra de três simples direta):

- 36 -
0DWHPiWLFD
VALOR CORRESPONDE PORCENTAGEM 4) Sobre uma compra de R$ 68.000,00, se concede um
abatimento de R$ 3.400,00. Qual é a taxa do abatimen-
(P) 50.000,00 (todo) . . . 100 100% (todo) to?
(p) x (parte) . . . . . (i) 3,5% (parte)

Estabelecendo a proporção e resolvendo-a, vem:  

Resp.: A comissão do corretor é de R$ 1.750,00.


Resp.: A taxa do abatimento foi de 5%.
2) Ao pagar uma conta de R$ 48.000,00, uma pessoa tem
um abatimento de 4%. Quanto pagou pela conta? 5) Em 35 g de uma solução de iodo, a porção de iodo pesa
Solução: 0,7 g. Qual a taxa percentual de iodo da solução?
Distinguindo os quatro elementos, temos:

(P)  PRINCIPAL = 48.000,00 (todo em espécie)


 valor total.
 
(p)  PORCENTAGEM = x (parte do principal) 
o que se vai calcular.
(100)  NÚMERO FIXO = 100% (todo em %)  não
aparece no problema.
(i)  TAXA = 4% (parte do 100%)  é o que vai
ser abatido.
Resp.: A taxa percentual de iodo da solução é de 2%.
Dispositivo da regra de três:
6) Um rapaz, comprando uma motoneta, conseguiu um
desconto de 3% sobre o preço marcado, e assim obteve
VALOR CORRESPONDE PORCENTAGEM um desconto de R$ 18,00. Qual o preço marcado?

(P) 48.000,00 (todo) . . . 100 100% (todo)


(p) x (parte) . . . . . (i) 4% (parte)  
Proporção e cálculo:

Resp.: O preço marcado é de R$ 600,00.

O abatimento foi de R$ 1.920,00, então, a pessoa pagou: 7) Em um recipiente contendo álcool puro, derramam-se
7,5 5 de água para se obter uma mistura que contivesse
R$ 48.000,00 – R$ 1.920,00 = R$ 46.080,00 25% de água. Qual o volume da mistura?
Resp.: A pessoa pagou R$ 46.080,00 pela conta.

OUTROS EXEMPLOS (RESOLUÇÃO DIRETA)


 
3) Em uma classe de 35 alunos, 40% são meninos. Quan-
tas são as meninas?

Resp.: O volume da mistura é de 30 litros.

TAXA MILESIMAL

Se, em lugar de tanto por cento, se tiver tanto por mil,


será uma taxa milesimal, cujo símbolo é "%o" e para calculá-
  la, basta substituir nos problemas que a contiver, o número
100 por 1.000 e se efetuar os cálculos resultantes como nos
problemas anteriores.

EXERCÍCIOS A RESOLVER

O número de meninos é 14; logo o número de meninas Exprimir, sob a forma de porcentagem, as razões:
é: 35 – 14 = 21
Resp.: As meninas são em número de 21. 1) 2) 3) 4) 5) 6)

- 37 -
0DWHPiWLFD
Representar, sob a forma de fração irredutível, as porcenta- TESTES
gens:
39) A razão 7/20 na forma percentual é:
7) 15% 9) 7,5% 11) 38,25% a) 7% b) 25% c) 30% d) 35%
8) 4,5% 10) 24,8% 12) 21,875%
40) 9% de 0,8 é igual a:
Calcular as porcentagens ou taxa milesimal: a) 0,72 b) 0,072 c) 7,2 d) 72
13) 8% de 175 18) 2%o de 200 g 41) Numa classe de 50 alunos, 30 são moças. A taxa
14) 0,2% de 938 19) 5%o de 500 g percentual de rapazes é:
15) de 600 20) 3%o de 145 g a) 25% b) 30% c) 40% d) 75%

16) 5% de 3/4 21) 12% de R$ 60.000,00 42) Em sua composição, o feijão tem 22% de proteínas.
Quantos gramas de proteínas fornecem 300 g de feijão?
17) de 22) 6% de R$180.000,00
a) 66 b) 6,6 c) 17,6 d) 176

43) Uma jóia contém em seu peso 65% de ouro. Se essa jóia
Determinar quanto por cento é: pesa 15,4 gramas, a quantidade, em gramas, de ouro
que esta jóia tem é:
23) 35 de 700 25) 5 dm3 de 50 da5
24) 3 m de 24 m 26) 16 kg de 80 kg a) 10,10 g c) 10,01 g
b) 10,00 g d) 10,11 g

Resolver os problemas: 44) Sabe-se que 140 representam 35% de um número x.


Este número x é:
27) O transporte de um objeto custa R$ 864,00 e esta a) 400 b) 500 c) 600 d) 300
importância representa 8% do valor do objeto. Qual é o
valor desse objeto?
45) A taxa percentual que corresponde à fração 3/4 é:
28) Uma conta, ao ser paga à vista, sofre um abatimento de a) 80% b) 75% c) 70% d) 65%
5% no valor de R$ 200,00. Qual é o valor da conta?
46) Numa cidade, as tarifas de ônibus passaram de R$
29) Qual o valor de uma fatura pela qual se pagou R$ 16,00 para R$ 24,00. O percentual de aumento foi de:
1.900,00, sabendo-se que o vendedor concordou em a) 50% b) 40% c) 60% d) 70%
fazer um abatimento de 5%?
47) Em certo país, a população atual é de 80 milhões de
30) Um aluno, ao fazer uma composição de 420 palavras, habitantes. Sabendo-se que a taxa de crescimento
cometeu 21 erros de ortografia. Qual é a taxa percentual populacional é de 40% ao ano, a população daqui a 2
de erros da composição? anos será:
31) Um negociante comprou 156 kg de mercadorias por R$ a) 83,200 milhões c) 156,8 milhões
171,60. Por quanto deve revender o quilo, se pretende b) 112 milhões d) 168,5 milhões
ganhar 30% sobre o preço da compra?
48) Em um colégio, 38% dos alunos são meninos e as
32) Um comerciante compra 310 toneladas de minério a R$ meninas são 155. Neste colégio, o número de alunos é:
45,00 a tonelada. Vende um quinto com lucro de 25%; a) 140 b) 240 c) 150 d) 250
dois quintos com lucro de 15% e o resto com lucro de
10%. Quanto recebe ao todo e qual o seu lucro?
RESPOSTAS:
33) Uma pessoa compra uma propriedade por R$ 1) 75% 20) 435 mg
30.000,00. Paga de taxa, comissões e escritura, R$ 2) 12,5%
7.200,00. Por quanto deve revendê-la, para lucrar 12%? 21) R$ 7.200,00
3) 40% 22) R$ 10.800,00
4) 46,87% 23) 5%
34) Um comerciante adquiriu 120 kg de certa mercadoria à 5) 160% 24) 12,5%
razão de R$ 2,40 o quilograma. Obteve um desconto de 6) 302,5% 25) 1%
1% e teve uma despesa de transporte de R$ 18,80. 7) 3/20 26) 20%
Revendendo a mercadoria a R$ 3,00 o quilograma, qual 8) 9/200 27) R$ 10.800,00
será sua taxa percentual de lucro? 9) 3/40 28) R$ 4.000,00
10) 31/125 29) R$ 2.000,00
35) Uma betoneira, depois de trabalhar na construção de um 11) 153/400 30) 5%
edifício sofre uma depreciação de 27% de seu valor e é, 31) R$ 1,43
então, avaliada em R$ 3.650,00. Qual é o valor primitivo? 12)
32) R$ 16.042,50 e
13) 14 R$ 2.092,50
36) Uma pessoa compra um apartamento por R$ 14) 1,876 33) R$ 41.664,00
170.000,00 e o revende com o lucro de 15% sobre o 15) 50 34) 18,45%
preço de venda. Qual é o preço de venda? 16) 0,0375 35) R$ 5.000,00
17) 10/9 36) R$ 200.000,00
37) Numa cidade, a população adulta é de 18.300 pessoas, 18) 4 dg 37) 10.614
42% das quais são analfabetas. Quantos são os adultos 19) 2,5 g 38) 4 alunos
alfabetizados dessa cidade?
TESTES:
38) Em uma classe com 40 alunos, a taxa de porcentagem
de comparecimento, certo dia, foi de 90%. Quantos 39) d 41) c 43) c 45) b 47) c
alunos faltaram nesse dia? 40) b 42) a 44) a 46) a 48) d

- 38 -
5DFLRFtQLR/yJLFR0DWHPiWLFR

t: No Brasil, fala-se espanhol.


ESTRUTURA LÓGICA DE RELAÇÕES u: Toda ave voa.
ARBITRÁRIAS ENTRE PESSOAS, v: O número 3 é par.
LUGARES, OBJETOS OU EVENTOS x: O número 7 é primo.
FICTÍCIOS. z: O número 7 é ímpar.

Perceba que estamos utilizando uma letra minúscula


INTRODUÇÃO AO RACIOCÍNIO LÓGICO para nomearmos as proposições, desta forma, quando nos
referirmos na proposição r, estamos nos referindo a "O
Lógica é a ciência que trata dos princípios válidos do número 2 é primo. "
raciocínio e da argumentação. Seu estudo trata das formas
do pensamento em geral e das operações intelectuais que
visam à determinação do que é verdadeiro ou não, ou seja, Valores Lógicos das Proposições
um encadeamento coerente de alguma coisa que obedece
a certas convenções ou regras. Assim, o estudo da Lógica Valor lógico é a classificação da proposição em
é um esforço no sentido de determinar as condições que verdadeiro (V) ou falso (F), pelos princípios da não contradi-
permitem tirar de determinadas proposições (ponto ou idéia ção e do terceiro excluído. Sendo assim, a classificação é
de que se parte para estruturar um raciocínio), também única, ou seja, a proposição só pode ser verdadeira ou falsa.
chamadas de premissas, uma conclusão delas derivada.
Para as proposições utilizadas no tópico anterior, os
valores lógicos são:
ESTRUTURAS LÓGICAS r: O número 2 é primo. (Verdadeiro)
CONECTIVOS s: Marte é o planeta vermelho. (Verdadeiro)
t: No Brasil, fala-se espanhol. (Falso)
Para iniciarmos o estudo de conectivos é necessário
entendermos previamente alguns conceitos básicos. u: Toda ave voa. (Falso)
v: O número 3 é par. (Falso)
Conceitos Básicos x: O número 7 é primo. (Verdadeiro)
Proposições z: O número 7 é ímpar. (Verdadeiro)

A proposição é todo o enunciado com palavras e/ou


símbolos que representam um pensamento de sentido Conectivos
completo.
Conectivo é tudo aquilo que estabelece uma cone-
Toda proposição é uma representação lógica do juízo xão, isto é, que une uma coisa a outra. Na lógica, o conecti-
que afirma (valor lógico verdadeiro) ou nega (valor lógico vo é um termo ou símbolo dele, que relaciona proposições
falso) a identidade representativa de dois conceitos. As de modo tal que a verdade ou inverdade da afirmação
regras que determinam quais as proposições que devem resultante é determinada pela verdade ou inverdade dos
ser verdadeiras constituem a lógica matemática. seus componentes.

Assim sendo, têm-se exemplos de proposições As proposições podem ser conectadas através dos
abaixo: seguintes conectivos: e, ou, não, se... então..., ... se e
somente se...
5 > 1 (valor lógico verdadeiro)
5 = 1 (valor lógico falso) Os conectivos são representados por símbolos, como
mostra a tabela abaixo:
No caso das proposições, a lógica matemática tem
como base dois princípios: Conectivo Símbolo
& Princípio da não-contradição: "Uma proposição não e Y
pode ser verdadeira e falsa ao mesmo tempo".
& Princípio do terceiro excluído: "Toda proposição ou é ou Z
falsa ou é verdadeira, não existe uma terceira opção".
não ~ou ¬
Outros exemplos de proposições:

r: O número 2 é primo.
se... então 
s: Marte é o planeta vermelho. ... se e somente se... 


5DFLRFtQLR/yJLFR0DWHPiWLFR
Traduza as proposições moleculares abaixo.
DEDUZIR NOVAS INFORMAÇÕES DAS
RELAÇÕES FORNECIDAS E AVALIAR a) PQ
b) P~Q
AS CONDIÇÕES USADAS PARA
c) ~ S Y (Q Y R)
ESTABELECER A ESTRUTURA
d) ~PS
DAQUELAS RELAÇÕES.
e) P Y (Q Z R)
f) (P  Q) Y R
PROPOSIÇÕES SIMPLES E COMPOSTAS g) (P  Q) Y (~R Y S)
As proposições são classificadas em simples e h) P  ((Q Y Q) Z S)
compostas:
Resolução:
Proposições Simples ou Atômica: são as proposi- a) Se há nuvens, choverá.
ções formadas por uma única proposição, ou seja, não
contém nenhuma outra como parte integrante de si mesma. b) Se aparecerem nuvens hoje, amanhã não teremos
Essas proposições são nomeadas por letras minúsculas do bom tempo.
alfabeto:
c) Não teremos bom tempo amanhã pois choverá e
a, b, c,..., p, q,... ventará.
Proposições Composta ou Molecular: São as d) Se não surgirem nuvens hoje, amanhã teremos bom
proposições formadas por combinações de duas ou mais tempo.
proposições simples. Tais combinações são feitas através
dos conectivos. Essas proposições são nomeadas por e) Há nuvens, de modo que teremos chuva ou vento.
letras maiúsculas do alfabeto: A, B, C,..., P, Q,...
f) Se há nuvens, choverá; ou teremos vento.
A partir das proposições simples citadas acima, pode-
se gerar, utilizando conectivos, outras compostas como: g) Teremos nuvens hoje se e somente se chover; mas
não teremos vento e teremos bom tempo amanhã.
W: O número 2 é primo e o número 7 é ímpar.
h) Nuvens hoje se e só se amanhã chover, mas sem
Y: O número 3 é par ou o número 7 é primo. vento, ou fizer bom tempo.
D: Se o número 7 é primo então ele é ímpar.
Exemplo 2: Considerando as proposições simples:
K: O número 3 é par se e somente se for múltiplo de 2.
P: O estudante comete erros.
Utilizando os símbolos que representam os conecti-
vos tem-se que as proposições compostas acima podem Q: Há motivação para o estudo.
ser escritas como:
R: O estudante aprende a matéria.
W: rYz
Simbolizar:
Y: vZx
xz
a) Se o estudante não comete erros, aprende a matéria.
D:
va
b) Se há motivação para o estudo, o estudante não
K: comete erros.
Os valores lógicos das proposições W, Y, D e K são c) Se não há motivação para o estudo, então o estudan-
respectivamente V, V, F, V. te comete erros ou não aprende a matéria.
Para se formar proposições compostas utiliza-se d) Se o estudante comete erros, então, se não há
apenas os conectivos, e, ou, se... então, se e somente se; motivação para o estudo, o estudante não aprende a
não se utiliza o conectivo não; basicamente, a partir de um matéria.
proposição é possível construir uma negação e com duas
ou mais proposições pode-se formar estruturas conhecidas e) O estudante não comete erros ou aprende a matéria
como: se há motivação para o estudo.

& conjunções (r e z) f) O estudante comete erros; além disso, há motivação


para o estudo e o estudante aprende a matéria.
& disjunções (r ou z)
Resolução:
& condicionais (se r, então z)
& bicondicionais (r se e somente se z) a) ~PR

Exemplo 1: Considerando as proposições simples: b) Q~P

P: há nuvens hoje c) ~ Q (P Z ~ R)

Q: choverá d) P  (~ Q ~ R)

R: ventará e) Q  (~ P Z R)

S: fará bom tempo amanhã f) P Y (Q Y R)


5DFLRFtQLR/yJLFR0DWHPiWLFR
EXERCÍCIOS DE FIXAÇÃO: p q
Conectivos
1 V V
1) Considerando as proposições simples:
2 V F
P: Paulo é aprovado no exame.
Q: Paulo conclui a sua tese. 3 F V
R: Paulo recebe o título de doutor.
S: Paulo lecionará na faculdade. 4 F F
T: Paulo ensinará no colégio.

Traduza as proposições moleculares abaixo.


Teorema do número de linhas da tabela-verdade
a) (P Y Q) Z ~ R
A tabela-verdade lista todas as possíveis combinações
b) (~ P Y ~ Q)  ~ S de valores-verdade V e F para as variáveis envolvidas na
c) (P Y Q Y R)  ~ T expressão cujo valor lógico deseja-se deduzir. A tabela-
verdade de uma proposição composta com n proposições
d) R  (S Y ~ T) simples componentes contém 2n linhas. Ou seja, cada
e) R  (P Y Q) proposição simples tem dois valores V ou F, que se excluem.
f) ~ P  (~ R Y S) Para n proposição simples (atômicas) distintas, há
g) S  (P Y Q Y R) tantas possibilidades quantos são os arranjos com repeti-
ção de (V e F) elementos n a n. Segue-se que o número de
h) T~R linhas da tabela-verdade é 2n. Assim, para duas proposi-
ções são 4 linhas; para três proposições são 8; etc.

2) Considerando as proposições simples: Então, para se construir uma tabela-verdade procede


se da seguinte maneira:
P: Paulo diminui os erros cometidos.
Q: Há motivação para o estudo. 1- Determina-se o número de linhas da tabela-verdade
R: Paulo aprendeu a matéria. que se quer construir;
S: O professor é bom.
2- Observa-se a precedência entre os conectivos, isto
Simbolizar: é, determina-se a forma das proposições que ocor-
rem no problema;
a) Se o professor é bom, Paulo aprende a maté-
ria. 3- Aplicam-se as definições das proposições Lógicas
que o problema exigir.
b) Se o professor não é bom, não há motivação
para ao estudar.
Operações lógicas sobre as proposições
c) O professor é bom, há motivação para estudar e sua tabela-verdade
e, além disso, Paulo aprende a matéria
Uma série de operações é realizada quando se
d) Paulo não aprendeu a matéria, ele não diminu- analisam as proposições e seus respectivos conectivos.
iu os erros cometidos.

e) Se Paulo não diminuiu os erros cometidos, o a- Y)


Conjunção (Y
professor não era bom ou não havia motivação
para estudar.
A conjunção de duas proposições p e q, indicada por
f) Paulo aprende a matéria ou diminui os erros p Y q (lê-se: "p e q") é, por definição, a proposição
cometidos. que é verdadeira só quando o forem verdadeiras as
proposições componentes. A tabela-verdade para a
conjunção de duas proposições é dada a seguir:
A TABELA-VERDADE
p q pYq
Da mesma forma que as proposições simples podem
ser verdadeiras ou falsas, as proposições compostas V V V
podem também ser verdadeiras ou falsas. O valor-verdade
de uma expressão que representa uma proposição compos- F V F
ta depende dos valores-verdade das subexpressões que a
compõem e também a forma pela qual elas foram compos- V F F
tas. F F F
As tabelas-verdade explicitam a relação entre os
valores-verdade de uma expressão composta em termos p Y q: Evandro é estudioso e passará no con-
dos valores-verdade das subexpressões e variáveis que a curso.
compõem.

Na tabela seguinte, encontra-se todos os valores b- Z)


Disjunção (Z
Lógicos possíveis de uma proposição composta correspon-
dente das proposições simples abaixo: A disjunção de duas proposições p e q, indicada por
pZq (lê-se: "p ou q"), é, por definição, a proposição
p: Evandro é estudioso. que é verdadeira sempre que pelo menos uma das
proposições componentes o for. A tabela-verdade
q: Ele passará no concurso. para a disjunção de duas proposições é dada a
seguir:


5DFLRFtQLR/yJLFR0DWHPiWLFR
p q pZq g) 3 + 5 = 10 ou 5 + 3 = 10.
h) Se 2 + 2 = 4, então a Lua é cilíndrica.
V V V
i) 6 + 6 = 8 se e somente se a Lua é cilíndrica.
V F V j) 5 é o dobro de 2 se e somente se 9 é o triplo de
3.
F V V
F F F
RESPOSTAS
p Z q: Evandro é estudioso ou ele passará no a) F c) F e) V g) F i) V
concurso.
b) V d) V f) V h) F j) F
c- Condicional ()
NEGAÇÕES
A proposição condicional, indicada por p  q (lê-se:
"Se p então q") é, por definição, a proposição que é A negação de uma proposição p, indicada por ~ p (lê-
falsa quando p é verdadeira e q falsa, mas ela é se: "não p") é, por definição, a proposição que é verdadeira
verdadeira nos demais casos. A tabela-verdade para ou falsa conforme p é falsa ou verdadeira, de maneira que
a proposição condicional é dada a seguir: se p é verdade então ~ p é falso, e vice-versa. Os possíveis
valores lógicos para a negação são dados pela tabela-
pq
verdade abaixo:
p q
V V V p ~p

V F F F V
F V V V F
F F V
p: Evandro é estudioso.
~ p: Evandro não é estudioso.
p  q: Se Evandro é estudioso, então ele passa-
rá no concurso. Ou seja, p é condição Exemplo 1: Representar as sentenças abaixo
suficiente para q e q é condição necessá- usando símbolos adequados:
ria para p.
a) Pedro não foi à festa.
b) Não é fato que as baleias sejam peixes.
d- Bicondicional () c) Não se dá que haja prisioneiros.
d) Não é verdade que 2+2=5.
A proposição bicondicional, indicada por p  q (lê-se:
Resolução:
"p se e somente se q") é, por definição, a proposição
que é verdadeira somente quando p e q têm o a) não (Pedro foi à festa). ~ F
mesmo valor lógico. A tabela-verdade para a proposi- b) não (as baleias são peixes). ~ B
ção bicondicional é dada a seguir:
c) não (haja prisioneiros). ~ P
d) não (2+2=5). ~ C
p q pq

V V V EXERCÍCIOS DE FIXAÇÃO:
Negações
V F F
F V F 1) Passe as proposições para a negativa e simbolize-
as.
F F V
a) João é bonito.
p  q: Evandro é estudioso se e somente se ele b) O Brasil foi campeão.
passar no concurso. Ou seja, p é condi- c) Pedro e Ana são namorados.
ção necessária e suficiente para q.
d) Marta não perdeu a prova.
e) Aline é muito simpática.
EXERCÍCIOS DE FIXAÇÃO:
Tabela-Verdade f) 2 + 5 = 12.

1) Determinar o valor verdade de cada qual dos seguin- RESPOSTAS


tes compostos:
a) ~ J onde, J: João é bonito.
a) 2 + 2 = 5 e 3 + 4 = 10.
b) ~ B onde, B: O Brasil foi campeão.
b) O dobro de 3 é 6 ou o triplo de 4 é 10.
c) ~ P onde, P: Pedro e Ana são namorados.
c) Se 2 + 2 = 4, então 3 + 3 = 9.
d) M onde, M: Marta perdeu a prova.
d) Se 2 + 2 = 4, então 5 + 5 = 10.
e) ~ A onde, A: Aline é muito simpática.
e) 3 + 3 = 6 e o triplo de 5 é 15.
f) ~ C onde, C: 2 + 5 = 12.
f) 3 + 5 é igual a 12 ou 3 + 55 é diferente de 12.


5DFLRFtQLR/yJLFR0DWHPiWLFR
TAUTOLOGIA p ~p p Ú (~p)
Etimologicamente, a palavra tautologia é formada por F V V
dois radicais gregos: taut (o) - que significa "o mesmo" e V F F
–logia que significa "o que diz a mesma coisa já dita”. Para
a lógica, a tautologia é uma proposição analítica que
permanece sempre verdadeira, uma vez que o atributo é Resumidamente, temos:
uma repetição do sujeito, ou seja, o uso de palavras
diferentes para expressar uma mesma idéia; redundância, - tautologia contendo apenas V na última coluna da
pleonasmo. sua tabela-verdade;

Exemplo: O sal é salgado. - contradição contendo apenas F na última coluna de


sua tabela-verdade;
Uma proposição composta P (p, q, r, ...) é uma
tautologia se P (p, q, r, ...) tem valor lógico V quaisquer que - contingência contendo V e F na última coluna de
sejam os valores lógicos das proposições componentes p, sua tabela-verdade.
q, r, ..., ou seja, uma tautologia conterá apenas V na última
coluna da sua tabela-verdade. Tautologia V

Exemplo: Contradição F
Contigência V/F
A proposição "p ou não p”, isto é, p Z (~p) é uma
tautologia. De fato, a tabela-verdade de p Z (~p) é:
Algumas Questões de Concursos Anteriores:
p ~p p Z (~p)
V F V 1) (TRT-9R-2004-FCC) Considere a seguinte proposi-
ção: "na eleição para a prefeitura, o candidato A será
F V V eleito ou não será eleito”. Do ponto de vista lógico, a
afirmação da proposição caracteriza:

CONTRADIÇÃO a) um silogismo.
b) uma tautologia.
A contradição é uma relação de incompatibilidade c) uma equivalência.
entre duas proposições que não podem ser simultaneamen- d) uma contingência.
te verdadeiras nem simultaneamente falsas, por apresenta- e) uma contradição.
rem o mesmo sujeito e o mesmo predicado, mas diferirem
ao mesmo tempo em quantidade e qualidade. Resolução: Com a finalidade de montarmos a tabela
verdade para verificar se a proposição apresentada no
Exemplo: Todos os homens são mortais e alguns enunciado da questão é uma tautologia ou uma contradi-
homens não são mortais. ção, definiremos a seguinte proposição simples:

Há uma relação de incompatibilidade entre dois p: o candidato A será eleito


termos em que a afirmação de um implica a negação do
outro e reciprocamente. Então, a sentença "o candidato A será eleito OU
não será eleito” passará ser representada simbolicamente
Uma proposição composta P (p, q, r, ...) é uma como: p Z ~p.
contradição se P (p, q, r, ...) tem valor lógico F quaisquer
que sejam os valores lógicos das proposições componentes Construindo a tabela- verdade, teremos que:
p, q, r, ..., ou seja, uma contradição conterá apenas F na
última coluna da sua tabela-verdade. p ~p p ~p

Exemplo: V F V
F V V
A proposição "p e não p”, isto é, p Y (~p) é uma
contradição. De fato, a tabela-verdade de p Y (~p) é: Como a última linha desta tabela-verdade só apre-
senta o valor lógico Verdadeiro, temos uma tautologia.
p ~p p Y (~p) Então, a alternativa correta é a letra b.
V F F
F V F 2) (Fiscal Trabalho 98 ESAF) Um exemplo de tautologia
é:
a) se João é alto, então João é alto ou Guilherme
CONTINGÊNCIA é gordo
b) se João é alto, então João é alto e Guilherme é
Chama-se contingência toda a proposição composta gordo
em cuja última coluna de sua tabela-verdade figuram as c) se João é alto ou Guilherme é gordo, então
letras V e F cada uma pelo menos uma vez. Em outros Guilherme é gordo
termos, contingência é toda proposição composta que não d) se João é alto ou Guilherme é gordo, então
é tautologia nem contradição. As contingências são também João é alto e Guilherme é gordo
denominadas proposições contingentes ou proposições e) se João é alto ou não é alto, então Guilherme
indeterminadas. é gordo
Exemplo: Resolução: Para simplificar e facilitar esta resolução,
assumi-se as seguintes proposições simples:
& A proposição "se p então ~p”, isto é, p Ú (~p) é uma
contingência. De fato, a tabela-verdade de p Ú (~p) p: João é alto.
é: q: Guilherme é gordo.


5DFLRFtQLR/yJLFR0DWHPiWLFR
Daí, utilizando essas definições feitas acima para as PROPRIEDADES DAS IMPLICAÇÕES LÓGICAS
proposições p e q, as alternativas da questão poderão ser
reescritas, simbolicamente, como: As propriedades das implicações lógicas são:
p Ú (p Z q) (=se João é alto, então João é alto ou p _ (p Y q)
(p Y q) _ p
a) 1)

(p Ú q) _ ~p (q deve ser uma contradição)


Guilherme é gordo) 2)

[p Y (p Ú q) ] _ q
3)
p Ú (p Y q) (=se João é alto, então João é alto e
[ (p Ú q) Y ~q] _ ~p
b) 4)

[ (p Z q) Y ~p] _ q
Guilherme é gordo) 5)

p _ [qÚ (p Y q) ]
6)
(p Z q) Ú q (=se João é alto ou Guilherme é gordo,
[ (p  q) Y (q  r) ] _ (p  r)
c) 7)

[ (p Ú q) Y (q Ú r) ] _ (p Ú r)
então Guilherme é gordo) 8)
9)
d) (p Z q) Ú (p Z q) (=se João é alto ou Guilherme é
gordo, então João é alto e Guilherme é gordo)
RELAÇÕES ENTRE IMPLICAÇÕES
e) (p Z ~p) Ú q (=se João é alto ou não é alto, então
Guilherme é gordo) Existem três tipos de relações entre as implicações
lógicas, são elas:
O que resta ser feito agora é testar as alternativas,
procurando por aquela que seja uma Tautologia. Para isso, 1. A implicação recíproca: É aquela que o anteceden-
constrói-se a tabela-verdade de cada opção de resposta. te torna-se o conseqüente e vice-versa.
Teste da alternativa "a”: p Ú (p Z q) Exemplo: Se nadei, então me molhei.
Recíproca: Se me molhei, então nadei.
p q (p Z q) pÚq
2. A implicação contrária: É aquela que nega o
V V V V antecedente e o conseqüente.
V F V V
Exemplo: Se nadei, então me molhei.
F V V V Contrária: Se não nadei, então não me molhei.
F F F V
3. A implicação contrarrecíproca: É aquela que o
antecedente e trocado pela negação do conseqüente
Já chegamos à resposta! Observe que a última
vice-versa.
coluna da tabela-verdade acima só apresentou valores
lógicos verdadeiros. Com isso, concluímos: a proposição Exemplo: Se nadei, então me molhei.
da opção a – Se João é alto, então João é alto ou Guilher- Contrarrecíproca: Se não me molhei, então não
me é gordo – é uma tautologia. nadei.
Resposta: letra a
EQUIVALÊNCIAS LÓGICAS
IMPLICAÇÕES LÓGICAS Definição: Tem-se uma equivalência lógica entre
duas proposições se uma implicar na outra e vice-versa.
Dizemos que uma expressão p implica logicamente
uma expressão q se, e somente se, cada atribuição de valor
Exemplo: Uma pessoa muda não fala.
também. Utilizamos a notação p _ q para dizer que x
às variáveis que torna x verdadeira torna q verdadeira
Uma pessoa que não fala é muda.
implica logicamente q. Note que uma pessoa muda (antecedente) implica
em não falar (conseqüente) e que não falar (conseqüente)
Teorema: Uma expressão p implica logicamente q implica em ser muda (antecedente), ou seja, o antecedente
se, e somente se, p Ú q é uma tautologia. implica no conseqüente da mesma maneira que o conse-
qüente implica no antecedente.
Prova: x implica logicamente y se, e somente se,
sempre que x for verdadeiro, y também seja. Dessa manei-
ra, x implica logicamente y se, e somente se, nunca ocorre EQUIVALÊNCIA ENTRE PROPOSIÇÕES
no caso em que x é verdadeiro e y é falso. Pois isso
significa que a expressão x Ú y nunca é falsa, ou seja, que
x Ú y é uma tautologia. A equivalência lógica ocorre quando a tabela-verdade
de duas proposições forem idênticas. Essas proposições
são separadas pelo símbolo de equivalência @. Em
IMPLICAÇÕES LÓGICAS ENTRE PROPOSIÇÕES linguagem comum a equivalência é freqüentemente assina-
lada, entre outros modos, utilizando palavras como "é
A implicação lógica entre duas proposições é constru- equivalente”, "se, e só se” ou "é condição necessária e
ída a partir da primeira (antecedente) e da segunda (conse- suficiente”.
qüente), de maneira que a nova proposição será verdadeira
nos casos que: Por exemplo, o caráter de equivalência das proposi-
& O antecedente e o conseqüente são verdadeiros; ções:
& O antecedente é falso e o conseqüente é verdadeiro;
& O antecedente e o conseqüente são falsos. Um ângulo é reto se, e só se, o ângulo for de 90º.
Uma pessoa é rica se, e só se, ela possuir muito
E será falsa no caso em que: dinheiro.
& O antecedente é verdadeiro e o conseqüente é falso.
Uma condição necessária e suficiente para que
Regra da negação de uma implicação: O valor de x (y + 1) = 0 é que x = 0 ou y = –1
verdade da negação de uma implicação é o mesmo que o
da conjunção entre o antecedente e a negação do conse- Ou seja,
qüente.


5DFLRFtQLR/yJLFR0DWHPiWLFR
(o ângulo é reto) @ (o ângulo for de 90º) Equivalências da Tautologia e Contradição:
(a pessoa é rica) @ (a pessoa possui muito dinheiro) 1. (V Z p) @ V

(x (y + 1) = 0 @ (x = 0 Z y = –1) 2. (V Y p) @ p
3. (F Z p) @ p
(F Y p) @ F
Lembre-se: < implica logicamente
4.
@ logicamente equivalente Equivalências do Contrapositivo:
1. ~p Ú q @ ~q Ú ~p

EQUIVALÊNCIA ENTRE SENTENÇAS ABERTAS


LEIS DE DE MORGAN
Sabe-se que em sentenças abertas sempre temos
alguma incógnita (valor desconhecido), que é representado, 1. ~ (p Z q) @ (~p Y ~q)
normalmente, por uma letra do alfabeto. Dessa maneira,
duas sentenças abertas são equivalentes se possuírem o 2. ~ (p Y q) @ (~p Z ~q)
mesmo valor para a incógnita que as tornem verdadeiras.

Exemplo: x + 2 = 7 implica em x = 5, assim como DIAGRAMAS LÓGICOS


x = 5 implica em x + 2 = 7.

Simbolicamente: x + 2 = 7 _ x = 5x = 5 _ x + 2 = 7
Diagramas lógicos são estruturas auxiliadoras na
resolução de problemas que envolvem relações entre as
proposições. Para entender como se utilizar esse mecanis-
Ou seja, para que x + 2 = 7 seja verdade, x deve ser mo é necessário estudar a teoria dos conjuntos.
5 e se x for igual a 5, x + 2 deve ser igual a 7, logo:

x + 2 = 7 @ x = 5. TÓPICOS SOBRE A TEORIA DOS CONJUNTOS

Definição: Um conjunto é uma "coleção" de elemen-


PROPRIEDADES DAS EQUIVALÊNCIAS LÓGICAS tos que possuem uma ou mais características em comum,
são essas características que define o conjunto e o distin-
gue dos outros. Exemplos:
As propriedades das equivalências lógicas são:
1) O conjunto das vogais é formado por cinco elemen-
Leis Comutativas: tos: {a, e, i, o, u}.
1. pYq@qYp
pZq@qZp
2) O conjunto dos números pares é formado por infinitos
2. elementos.
3. pq@qp
3) O conjunto dos países que falam a língua portuguesa
4. pÚqgqÚp possui nove elementos: {Angola, Brasil, Cabo Verde,
Timor Leste, Guiné-Bissau, Macau, Moçambique,
Leis Associativas: Portugal, São Tomé e Príncipe}.
1. (p Y q) Y r @ p Y (q Y r)
4) O conjunto dos números naturais é formado por
2. (p Z q) Z r @ p Z (q Z r) infinitos elementos.

Leis Distributivas: Reescrevendo os quatro exemplos na forma de uma


1. p Y (q Z r) @ (p Y q) Z (p Y r) tabela:

2. p Z (q Y r) @ (p Z q) Y (p Z r) Nº de ele-
Ex. Característica Elementos
mentos
Lei da Negação:
1 Ser vogal. 5 a, e, i, o, u.
1. ~ (~p) @ p
2 Ser um número Infinitos Todos os números
Equivalências da Conjunção e da Disjunção: par. múltiplos de 2.
1. p Z q @ ~p Ú q @ ~q Ú p 3 País que fala lín- 9 Angola, Brasil, Ca-
p Y q @ ~ (p Ú ~q) @ ~ (q Ú ~p)
gua portuguesa. bo Verde, Timor
2. Leste, Guiné-Bis-
sau, Macau, Mo-
Equivalências da Condicional: çambique, Portu-
1. p Ú q @ ~q Ú ~p @ ~p Z q gal, São Tomé e
Príncipe.
Equivalências da Idempotência: 4 Ser um número Infinitos Todos os números
1. pZp@p natural. inteiros e positi-
vos.
2. pYp@p

Equivalências da Absorção: REPRESENTAÇÃO DE UM CONJUNTO


1. p Y (p Z q) @ p
Existem algumas formas de representar um conjunto,
2. p Z (p Y q) @ p em todas elas os conjuntos são nomeados por uma letra
(p Y q) Z ~q @ p Z ~q
maiúscula do alfabeto latino e os elementos por uma letra
3.
minúscula de qualquer alfabeto, não necessariamente o
4. (p Z q) Y ~q @ p Y ~q latino.


5DFLRFtQLR/yJLFR0DWHPiWLFR
A representação que é utilizada para descrever os
diagramas lógicos é conhecida como diagrama de Venn-
Euller.

& Diagrama de Venn-Euller: Coloca-se os elementos


do conjunto dentro de uma figura plana fechada,
normalmente uma circunferência.

Exemplo: Conjunto das vogais

P a e Onde:

i I- conjunto das pessoas inteligentes;

A-
E- conjunto das pessoas espertas;
o u conjunto de todas as pessoas que existem.

A analise do diagrama resulta em:

& Se Pedro estiver na condição 1, ele é inteligente,


Através da representação de Venn-Euller, pode-se então p ∨ q é verdadeira.
executar algumas operações com os conjuntos. Essas
operações estão intimamente ligadas com a relação entre & Se Pedro estiver na condição 2, ele é inteligente e
as proposições. esperto, então p ∨ q é verdadeira.

& Se Pedro estiver na condição 3, ele é esperto, então


DIAGRAMAS LÓGICOS
p ∨ q é verdadeira.
A correspondência entre as operações com conjuntos
e as operações lógicas são: & Se Pedro estiver na condição 4, ele não é inteligente
nem esperto, então p ∨ q é falsa.
Disjunção (∨) Ù União (A)
O resultado obtido foi o mesmo da tabela verdade da
Conjunção (∧) Ù Intersecção (B) disjunção, representado abaixo:
Negação (~) Ù Complementação ( )
p q p∨q

A)
∨) - UNIÃO (A
DISJUNÇÃO (∨ V V V
V F V
A disjunção é representada pela operação de união.

novo conjunto (A A B) formado por todos os elementos de


A união entre dois conjuntos A e B consiste em montar um F V V
A e de B. F F F

Definição:
B)
∧) - INTERSECÇÃO (B
CONJUNÇÃO (∧
Exemplo 1:
A conjunção é representada pela operação da
A = {1, 2, 3, 4, 5}
te em montar um novo conjunto (A B B), formado pelos
intersecção. A intersecção entre os conjuntos A e B consis-
A A B = {1, 2, 3, 4, 5, 6, 7}
B = {3, 5, 6, 7}
elementos que pertencem a A e a B simultaneamente.
Na forma de diagrama, tem-se: Definição:

Exemplo 1:

A = {1, 2, 3, 4, 5}

A B B = {3, 5}
B = {3, 5, 6, 7}

No diagrama, a intersecção é a parte que coincide


entres os dois conjuntos:
Observação: A A B = B A A

Exemplo 2:

Dada a disjunção - "Pedro é estudioso ou esperto",


escreva e analise o diagrama lógico associado a
disjunção.

Solução: A disjunção pode ser escrita, simbolica- Observação: A B B = B B A


mente, pela proposição composta p ∨ q, onde p
representa estudioso e q esperto. Exemplo 2:
Pode-se representar essa disjunção pela união dos Dada a conjunção - "Pedro é estudioso e esperto",
conjuntos das pessoas inteligentes com o das pesso- escreva e analise o diagrama lógico associado a
as espertas. disjunção.


5DFLRFtQLR/yJLFR0DWHPiWLFR
Solução: A conjunção pode ser escrita, simbolica- Onde:
mente, pela proposição composta p ∧ q, onde p
A-
representa estudioso e q esperto. I- conjunto das pessoas inteligentes;
conjunto de todas as pessoas que existem.
Pode-se representar essa conjunção pela intersec-
ção dos conjuntos das pessoas inteligentes com o A análise do diagrama resulta em:
das pessoas espertas.
& Se Pedro estiver na condição 1, ele não é inteligente,
Onde: então ~p é verdadeira.

I- conjunto das pessoas inteligentes; & Se Pedro estiver na condição 2, ele é inteligente,
E- conjunto das pessoas espertas; então ~p é falsa.
U- conjunto de todas as pessoas que existem.
O resultado obtido foi o mesmo da tabela verdade da
A análise do diagrama resulta em: negação, demonstrado abaixo:

& Se Pedro estiver na condição 1, ele é inteligente, p ~p


mas não é esperto, então p ∧ q é falsa.
V F
& Se Pedro estiver na condição 2, ele é inteligente e
esperto, então p ∧ q é verdadeira. F V

& Se Pedro estiver na condição 3, ele é esperto, mas


não é inteligente, então p ∧ q é falsa.

& Se Pedro estiver na condição 4, ele não é inteligente


nem esperto, então p ∧ q é falsa.

O resultado obtido foi o mesmo da tabela verdade da


conjunção, demonstrado abaixo:

p q p∧q
V V V
F V F
V F F
F F F

NEGAÇÃO (~) - COMPLEMENTAÇÃO ( )

A negação é representada pela operação de comple-


mentação. A complementação é tudo aquilo que não faz
parte do conjunto.

O complemento de A é denotado por A .

Exemplo 1:

O conjunto complementar dos números pares são os


números ímpares.

Exemplo 2:

Dada a negação - "Pedro não é estudioso", escreva


e analise o diagrama lógico associado a essa nega-
ção.

Solução: A negação pode ser escrita, simbolicamen-


te, pela proposição ~p, onde p representa estudioso.

Pode-se representar essa negação pela complemen-


tação do conjunto das pessoas inteligentes.


5DFLRFtQLR/yJLFR0DWHPiWLFR
Neste caso, não temos um argumento, porque não há
COMPREENSÃO DO PROCESSO nenhuma pretensão de justificar uma proposição com base
nas outras. Nem há nenhuma pretensão de apresentar um
LÓGICO QUE, A PARTIR DE UM conjunto de proposições com alguma relação entre si. Há
CONJUNTO DE HIPÓTESES, CONDUZ, apenas uma seqüência de afirmações. E um argumento é,
como já vimos, um conjunto de proposições em que se
DE FORMA VÁLIDA, A CONCLUSÕES pretende que uma delas seja sustentada ou justificada
DETERMINADAS. pelas outras - o que não acontece no exemplo anterior.

CONCLUSÕES
Sabe-se que o objetivo da lógica consiste no estudo
das formas de argumentação válidas, pois ela estuda e Um argumento pode ter uma ou mais premissas,
sistematiza a validade ou invalidade da argumentação. mas só pode ter uma conclusão.
Dessa maneira, o objeto de estudo da lógica é Exemplos de argumentos com uma só premissa:
determinar se a conclusão de um argumento é ou não uma
conseqüência lógica das proposições. Lembre-se que uma 1) Premissa: Todos os brasileiros são sul-americanos.
proposição (declaração/afirmação) é uma sentença que Conclusão: Logo, alguns sul-americanos são
pode ser verdadeira ou falsa. brasileiros.

2) Premissa: O Victor e o André são alunos do


ARGUMENTO 11º ano.
Um argumento possui uma estrutura de rigor Conclusão: Logo, o Victor é aluno do 11º ano.
constituída por proposições em uma seqüência na qual
uma delas é a conclusão e as demais são premissas Exemplos de argumentos com duas premissas:
(etimologicamente, "que foram colocadas antes”). Assim
sendo, a última proposição (conclusão) é, de alguma 1) Premissa 1: Se o André é um aluno do 11º ano,
forma, justificada pelas anteriores (premissas). A verda- então estuda filosofia.
de das premissas de algum modo leva a crer que a Premissa 2: O André é um aluno do 11º ano.
conclusão é verdadeira.
Conclusão: Logo, o André estuda filosofia.
Assim, o conjunto de proposições p1, p2, p3, ..., pn que
tem como conseqüência outra proposição q. Chamaremos 2) Premissa 1: Todos os homens são imortais.
as proposições p1, p2, p3, ..., pn de premissas do argumento, Premissa 2: Sócrates é homem.
e a proposição q de conclusão do argumento.
Conclusão: Portanto, Sócrates é imortal.
O argumento pode ser representado por:
Nas proposições, há alguns indicadores de conclu-
são. Os dois mais utilizados são "logo" e "portanto". Um
indicador é um articulador do discurso, é uma palavra ou
expressão que utilizamos para introduzir uma razão (uma
premissa) ou uma conclusão. O quadro seguinte apresenta
alguns indicadores de premissa e de conclusão:

Indicadores de premissa Indicadores de conclusão


pois por isso
porque por conseguinte
dado que implica que
Por exemplo:
como foi dito logo
p1: Toda baleia é mamífero.  premissa visto que portanto
p2: Ora, nenhum mamífero é peixe.  premissa devido a então
q: Logo, a baleia não é peixe.  conclusão a razão é que daí que
admitindo que segue-se que
No exemplo, há três proposições em que a última, a sabendo-se que pode-se inferir que
conclusão, deriva logicamente das duas anteriores (premissas).
assumindo que conseqüentemente
Nem sempre a argumentação se formaliza claramen-
te como no exemplo citado. Quando expomos nossas  É claro que nem sempre as premissas e a conclusão
idéias, seja oralmente ou por escrito, às vezes começamos são precedidas por indicadores.
pela conclusão, além do mais, com freqüência, omitimos
premissas, deixando-as subentendidas. Costumamos
também concatenar argumentos de modo que a conclusão PROPOSIÇÕES E FRASES
de um pode ser a premissa de outro. Por isso, um dos
trabalhos do lógico é montar o raciocínio redescobrindo sua
estrutura e avaliando a validade da conclusão. Um argumento é um conjunto de proposições. Quer
as premissas quer a conclusão de um argumento são
É importante salientar que embora um argumento proposições. Mas o que é mesmo uma proposição?
seja um conjunto de proposições, nem todos os conjuntos
de proposições são argumentos. Por exemplo, o seguinte Uma proposição é o pensamento que uma frase
conjunto de proposições não é um argumento: declarativa exprime literalmente.

- Eu almoço na minha mãe, mas o Léo não. Não confunda proposições com frases. Uma frase é
uma entidade lingüística, é a unidade gramatical mínima de
- O Gabriel come pipocas no cinema. sentido. Por exemplo, o conjunto de palavras "O Brasil é um"
- O Rodrigo foi ao museu. não é uma frase. Mas o conjunto de palavras "O Brasil é um


5DFLRFtQLR/yJLFR0DWHPiWLFR
país” é uma frase, pois já se apresenta com sentido gramati- A verdade é uma propriedade das proposições. A
cal. Há vários tipos de frases: declarativas, interrogativas, validade é uma propriedade dos argumentos. É incorreto
imperativas e exclamativas. Mas só as frases declarativas falar em proposições válidas. As proposições não são
exprimem proposições. Uma frase só exprime uma proposi- válidas nem inválidas. As proposições só podem ser
ção quando o que ela afirma tem valor de verdade. verdadeiras ou falsas. Também é incorreto dizer que os
argumentos são verdadeiros ou que são falsos. Os argu-
Por exemplo, as seguintes frases não exprimem mentos não são verdadeiros nem falsos. Os argumentos
proposições, porque não têm valor de verdade, isto é, não dizem-se válidos ou inválidos.
são verdadeiras nem falsas:
Diz-se que um argumento é válido na circunstância
1) Que horas são? em que: se as suas premissas são todas verdadeiras, então
a conclusão não pode ser falsa. Repare que, para um
2) Traz a apostila.
argumento ser válido, não basta que as premissas e a
3) Prometo ir ao shopping. conclusão sejam verdadeiras. Pois, sendo as premissas
4) Quem me dera gostar de Matemática. verdadeiras, a conclusão jamais seja falsa.
Mas as frases seguintes exprimem proposições, A validade de um argumento dedutivo depende da
porque têm valor de verdade, isto é, são verdadeiras ou conexão lógica entre as premissas e a conclusão do
falsas, ainda que, acerca de algumas, não saibamos, neste argumento e não do valor de verdade das proposições que
momento, se são verdadeiras ou falsas: constituem o argumento. Como já foi dito, a validade é uma
propriedade diferente da verdade. A verdade é uma proprie-
1) O Brasil fica na América do Norte. dade das proposições que constituem os argumentos (mas
2) Brasília é a capital do Brasil. não dos argumentos) e a validade é uma propriedade dos
3) A neve é branca. argumentos (mas não das proposições). Sendo assim,
4) Há seres extraterrestres inteligentes. pode-se ter as seguintes combinações para os argumentos
válidos dedutivos:
A frase 1 é falsa, a 2 e a 3 são verdadeiras. E a 4? a) Premissas verdadeiras e conclusão verdadeira.
Bem, não sabemos qual é o seu valor de verdade, não Exemplo:
sabemos se é verdadeira ou falsa, mas sabemos que tem Todos os apartamentos são pequenos. (V)
de ser verdadeira ou falsa. Por isso, também exprime uma Todos os apartamentos são lares. (V)
proposição. j Alguns lares são pequenos. (V)
Uma proposição é uma entidade abstrata, é o b) Algumas ou todas as premissas falsas e uma
pensamento que uma frase declarativa exprime literalmen- conclusão verdadeira. Exemplo:
te. Ora, um mesmo pensamento pode ser expresso por Todos os peixes têm asas. (F)
diferentes frases. Por isso, a mesma proposição pode ser Todos os pássaros são peixes. (F)
expressa por diferentes frases. Por exemplo, as frases "O j Todos os pássaros têm asas. (V)
governo demitiu o presidente da TAP" e "O presidente da
TAP foi demitido pelo governo" exprimem a mesma proposi- c) Algumas ou todas as premissas falsas e uma
ção. As frases seguintes também exprimem a mesma
conclusão falsa. Exemplo:
proposição: "A neve é branca" e "Snow is white".
Todos os peixes têm asas. (F)
Todos os cães são peixes. (F)
j Todos os cães têm asas. (F)
VALIDADE DE UM ARGUMENTO -
INFERÊNCIAS Todos os argumentos acima são válidos, pois se
suas premissas fossem verdadeiras então as conclusões
Todos os seres humanos têm algo a dizer sobre a também as seriam.
realidade que os rodeia e um conjunto de crenças (nem
sempre verdadeiras) acerca do mundo que pretendem Lembre-se que um argumento é válido somente
transmitir e partilhar com os seus próximos. Não cabe à quando todas as suas premissas forem verdadeiras o que
lógica estabelecer critérios para aceitar uma proposição acarretará numa conclusão também verdadeira. Portanto,
como verdadeira, compete-lhe esclarecer em que medida um argumento é não válido se existir a possibilidade de
uma proposição é uma conseqüência de um certo conjunto suas premissas serem verdadeiras e sua conclusão falsa.
de outras proposições. Caso o veredicto seja negativo algo
exige revisão. Observe que a validade do argumento depende
apenas da estrutura dos enunciados. Exemplos:
Este fato permite explicar o interesse de algumas
pessoas particularmente conscientes da importância da Todas os mulheres são bonitas.
argumentação em propor um método que permitisse Todas as rainhas são mulheres.
determinar as circunstâncias em que uma inferência j Todas as rainhas são bonitas.
merece ser considerada válida. A primeira pessoa a fazê-lo
de uma forma sistemática foi Aristóteles, um filósofo grego Observe que não precisamos de nenhum conheci-
da Antiguidade. O seu exemplo foi seguido por vários outros mento aprofundado sobre o assunto para concluir que o
filósofos, entre os quais um lógico medieval português argumento acima é válido. Substituindo mulheres, bonitas
chamado Pedro Hispano. Durante o século XX o tema e princesas por A, B e C teremos respectivamente:
sofreu um desenvolvimento imenso devido, em particular,
à descoberta da lógica moderna por Frege. Todos os A são B.
Todos os C são A.
j Todos os C são B.
Nesse sentido, o estudo da lógica desenvolveu-se em
torno de uma idéia principal: a idéia de validade. Esta é Logo o que é importante é a forma do argumento e
uma idéia notável porque nos permite compreender, entre não o conhecimento de A, B e C, isto é, este argumento é
outras coisas, a razão pela qual, em certas circunstâncias, válido para quaisquer A, B e C e, portanto a validade é
podemos confiar nas conclusões a que chegamos ao conseqüência da forma do argumento. O atributo Validade
efetuar uma inferência, ou seja, a operação intelectual por aplica-se apenas aos argumentos dedutivos.
meio da qual se afirma a verdade de uma proposição em
decorrência de sua ligação com outras já reconhecidas
como verdadeiras.


5DFLRFtQLR/yJLFR0DWHPiWLFR
DEDUÇÕES • Se aumentamos os meios de pagamentos, então
ARGUMENTOS DEDUTIVOS E INDUTIVOS haverá inflação.

Os argumentos são divididos em dois grupos: • Não há inflação


j Não aumentamos os meios de pagamentos.
• dedutivos
• indutivos Esse argumento é evidentemente válido e sua forma
pode ser escrita da seguinte maneira:
O argumento será dedutivo quando suas premissas
fornecerem prova conclusiva da veracidade da conclusão, Existe também um tipo de argumento válido conheci-
isto é, o argumento é dedutivo quando a conclusão é do pelo nome de dilema.
completamente derivada das premissas.
Geralmente, este argumento ocorre quando
Exemplo: Todo ser humano têm pai. alguém é forçado a escolher entre duas alternativas
Todos os homens são humanos. indesejáveis.
j Todos os homens têm mãe.
Exemplo: José se inscreveu no concurso da Prefei-
O argumento será indutivo quando suas premissas tura Municipal de São Paulo, porém não
não fornecerem o apoio completo para ratificar as conclu- gostaria de sair de Ribeirão Preto, e seus
sões. colegas de trabalho estão torcendo por
ele.
Exemplo: O São Paulo é um bom time de futebol.
O Palmeiras é um bom time de futebol. Eis o dilema de José:
O Corinthians é um bom time de futebol.
O Santos é um bom time de futebol. • Ou José passa ou não passa no concurso.
j Todos os times paulistas de futebol são - Se José passar no concurso vai ter que ir
bons. embora de Ribeirão Preto.
Portanto nos argumentos indutivos a conclusão - Se José não passar no concurso ficará com
possui informações que ultrapassam as fornecidas nas vergonha diante dos colegas de trabalho.
premissas. Sendo assim, não se aplica, então, a definição j Ou José vai embora de Ribeirão Preto ou João
de argumentos válidos ou não válidos para argumentos ficará com vergonha dos colegas de trabalho.
indutivos.
Este argumento é evidentemente válido e sua forma
pode ser escrita da seguinte maneira:
ARGUMENTOS DEDUTIVOS VÁLIDOS

A noção de argumentos válidos ou não válidos aplica- ARGUMENTOS DEDUTIVOS NÃO VÁLIDOS
se apenas aos argumentos dedutivos, e também que a
validade depende apenas da forma do argumento e não dos Os argumentos dedutivos não válidos podem combi-
respectivos valores verdades das premissas. Vimos também nar verdade ou falsidade das premissas de qualquer
que não podemos ter um argumento válido com premissas maneira com a verdade ou falsidade da conclusão.
verdadeiras e conclusão falsa. A seguir exemplificaremos
alguns argumentos dedutivos válidos importantes. Assim, podemos ter, por exemplo, argumentos não-
válidos com premissas e conclusões verdadeiras, porém as
O primeiro argumento dedutivo válido que discutire- premissas não sustentam a conclusão.
mos chama-se "afirmação do antecedente”, (também
conhecido como modus ponens). Exemplo: Todos os mamíferos são mortais. (V)
Todos os gatos são mortais. (V)
Então vejamos: j Todos os gatos são mamíferos. (V)
Se Paulo for reprovado no concurso, então será Este argumento tem a forma:
demitido do serviço.
Todos os A são B
Paulo foi reprovado no concurso. Todos os C são B
j Paulo será demitido do serviço. j Todos os C são A

Este argumento é evidentemente válido e sua forma Podemos, facilmente, mostrar que este argumento é
pode ser escrita da seguinte forma: não-válido, pois as premissas não sustentam a conclusão,
e veremos então que podemos ter as premissas verdadei-
ras e a conclusão falsa, nesta forma, bastando substituir A
por mamífero, B por mortais e C por cobra.

Todos os mamíferos são mortais. (V)


Todos os as cobras são mortais. (V)
j Todas as cobras são mamíferas. (F)
Outro argumento dedutivo válido é a "negação do Com as premissas verdadeiras e a conclusão falsa
conseqüente” (também conhecido como modus tollens). nunca pode ocorrer que o argumento seja válido, então este
No exemplo anterior, p _ q é equivalente a ~p _ ~q.
argumento é não-válido, chamaremos os argumentos não-
válidos de falácias.
Essa equivalência é chamada de contra-positiva.
A falácia é um tipo de raciocínio incorreto, embora
Exemplo: "Se ele me ama, então casa comigo” é tenha a aparência de correção. É conhecida também como
equivalente a "Se ele não casa comigo, sofisma ou paralogismo, e alguns estudiosos fazem a
então ele não me ama”. distinção entre eles, dando ao sofisma o sentido pejorativo
decorrente da intenção de enganar o interlocutor, enquanto
Então, vejamos o exemplo do modus tollens. no paralogismo não haveria essa intenção.


5DFLRFtQLR/yJLFR0DWHPiWLFR
A seguir examinaremos algumas falácias conhecidas Vejamos esse raciocínio
que ocorrem com muita freqüência. representado no esquema ao
lado.
O primeiro caso de argumento dedutivo não-válido
que veremos é o que chamamos de "falácia da afirmação Outro exemplo:
do conseqüente”.
Todos os homens são mortais.
Por exemplo: Os gregos são homens.
Se ele me ama então ele casa comigo. Logo, os gregos são mortais.
Ele casa comigo.
j Ele me ama.
Podemos ainda dizer que o silogismo é um raciocínio
Podemos escrever este argumento como: que parte de uma proposição geral e conclui outra proposi-
ção geral (que também pode ser particular).

Uma proposição é geral quando o sujeito da proposi-


ção é tomado na sua totalidade. Por exemplo: "Toda baleia
é mamífero”. É preciso prestar atenção, pois às vezes
usamos apenas o artigo definido (o, a) para indicar a
totalidade: "O homem é livre”. Observe também que não
Este argumento é uma falácia, podemos ter as importa se nos referimos a uma parte de outra totalidade; se
premissas verdadeiras e a conclusão falsa. na proposição tomamos todos os elementos que a constitu-
em, trata-se de uma proposição geral.
Outra falácia que ocorre com freqüência é a conheci-
da por "falácia da negação do antecedente”. Na proposição "Os paulistas são sul-americanos”,
não importa que os paulistas sejam uma parte dos brasilei-
Exemplo: Se João parar de fumar ele engordará. ros, mas que nesse caso estamos nos referindo à totalidade
João não parou de fumar. dos paulistas.
j João não engordará.
Uma proposição é particular quando o sujeito da
Observe que temos a forma: proposição é tomado em apenas uma parte indeterminada:
"Alguns homens são injustos”; "Certas pessoas são curio-
sas”. Uma proposição particular pode ser singular quando
o sujeito se refere a um indivíduo: "Esta flor é bonita”; "São
Paulo é uma bela cidade”; "Mário é estudante”.

No exemplo, a seguir, a primeira dedução tem conclu-


são geral; e no segundo caso, a conclusão é particular:
Esse argumento é uma falácia, pois podemos ter as
premissas verdadeiras e a conclusão falsa. Todo brasileiro é sul-americano.
Algum brasileiro é índio
Algum índio é sul-americano.
ANALOGIA
É verdade que a dedução é um modelo de rigor. Mas
A analogia (ou raciocínio por semelhança) é uma também é estéril, na medida em que não nos ensina nada
indução parcial ou imperfeita, na qual passamos de um ou de novo, e apenas organiza o conhecimento já adquirido.
de alguns fatos singulares não a uma conclusão universal, Portanto, ela não inova, o que não significa que a dedução
mas a uma outra enunciação singular ou particular, inferida não tenha valor algum.
em virtude da comparação entre objetos que, embora
diferentes, apresentam pontos de semelhança: Em síntese, chamaremos de silogismo o argumento
formado por duas premissas e uma conclusão, de modo
Exemplo: Pedro sarou de suas dores de cabeça que todas as premissas envolvidas são categóricas de
com este remédio. forma típica.
Logo, Antônio há de sarar de suas dores
de cabeça com este mesmo remédio. Têm-se também os três termos:

É claro que o raciocínio por semelhança fornece • Termo menor – sujeito da conclusão.
apenas uma probabilidade, não uma certeza. Grande parte • Termo maior – predicado da conclusão.
de nossas conclusões diárias baseia-se na analogia.
• Termo médio – é o termo que aparece uma vez em
cada premissa e não aparece na conclusão.
SILOGISMO
Chamaremos de premissa maior a que contém o
Segundo o aristotelismo, silogismo ("ligação”) é o termo maior, e premissa menor a que contém o termo
raciocínio dedutivo estruturado formalmente a partir de duas menor.
proposições, ditas premissas, das quais, por inferência, se
obtém necessariamente uma terceira, chamada conclusão. Exemplo: Todas as mulheres são bonitas.
Todas as rainhas são mulheres.
Por exemplo, quando dizemos "se x = y, e y = z, j Todas as princesas são bonitas.
então x = z”, há um termo médio (y), que estabelece a
ligação entre x e z, de modo que a conclusão se torna Termo menor: as rainhas
necessária, ou seja, tem de ser esta e não outra. Além Termo maior: bonitas
disso, o enunciado da conclusão não excede o conteúdo Termo médio: mulheres
das premissas, isto é, não diz mais na conclusão do que já
foi dito. Premissa menor:
todas as rainhas são mulheres.
Assim, quando dizemos: "Todos os homens são
mortais / Sócrates é homem / Logo, Sócrates é mortal”, a Premissa maior:
conclusão é necessária porque deriva das premissas. todas as mulheres são bonitas.


5DFLRFtQLR/yJLFR0DWHPiWLFR
Algumas regras para a validade de um silogismo: F) logo o antecedente deverá ser falso, ou seja, eu vejo
Carlos. Da p3 sendo o conseqüente falso, logo o antece-
1. Todo silogismo deve conter somente três termos; dente será falso, então (não faz calor e passeio = F Y V =
F), logo faz calor.
2. O termo médio deve ser universal pelo menos uma
vez; Dentre as alternativas formadas pelo conectivo "e”
será verdadeira aquelas em que todas as proposições
3. O termo médio não pode constar na conclusão; lógicas forem verdadeiras, ou seja, (vejo Carlos, e não
estou deprimida, e não chove, e faz calor = V Y V Y V Y V =
4. Nenhum silogismo categórico de forma típica que V)
tenha duas premissas negativas é válido.
ALTERNATIVA C
5. De duas premissas particulares não poderá haver
conclusão;
2) (Questão 68 da Prova de Técnico do MPU) Se
6. Se há uma premissa particular, a conclusão será Fulano é culpado, então Beltrano é culpado. Se
particular; Fulano é inocente, então ou Beltrano é culpado, ou
Sicrano é culpado, ou ambos, Beltrano e Sicrano,
7. Se há uma premissa particular negativa a conclusão são culpados. Se Sicrano é inocente, então Beltrano
é inocente. Se Sicrano é culpado, então Fulano é
será particular negativa. culpado. Logo,

a) Fulano é inocente, e Beltrano é inocente, e


EXERCÍCIOS: Sicrano é inocente.
LÓGICA DE ARGUMENTAÇÃO b) Fulano é culpado, e Beltrano é culpado, e
Sicrano é inocente.
1) (Questão 66 da Prova de Técnico do MPU) Quando c) Fulano é culpado, e Beltrano é inocente, e
não vejo Carlos, não passeio ou fico deprimida. Sicrano é inocente.
Quando chove, não passeio e fico deprimida. Quan- d) Fulano é inocente, e Beltrano é culpado, e
do não faz calor e passeio, não vejo Carlos. Quando Sicrano é culpado.
não chove e estou deprimida, não passeio. Hoje, e) Fulano é culpado, e Beltrano é culpado, e
passeio. Sicrano é culpado.
Portanto, hoje
a) vejo Carlos, e não estou deprimida, e chove, e Resolução:
faz calor.
b) não vejo Carlos, e estou deprimida, e chove, e Temos as seguintes premissas:
faz calor.
c) vejo Carlos, e não estou deprimida, e não p1: Se Fulano é culpado, então Beltrano é culpado
chove, e faz calor.
d) não vejo Carlos, e estou deprimida, e não p2: Se Fulano é inocente, então ou Beltrano é culpado,
chove, e não faz calor. ou Sicrano é culpado, ou ambos, Beltrano e Sicrano,
e) vejo Carlos, e estou deprimida, e não chove, e são culpados
faz calor.
p3: Se Sicrano é inocente, então Beltrano é inocente.
Resolução:
p4: Se Sicrano é culpado, então Fulano é culpado
Observe o conectivo "se então” trata-se de um
condicional, que pode ser substituído por Quando Para que o argumento seja válido todas as premissas
condição suficiente então condição necessária. devem ser verdadeiras e a conclusão deverá ser verdadei-
ra.
O argumento é formado pelas seguintes premissas:
Vamos elaborar uma hipótese inicial: Fulano é
p1: Quando não vejo Carlos então não passeio ou fico culpado, se isto ocorrer, é condição suficiente para que
deprimida Beltrano seja culpado, logo Beltrano é culpado.

p2: Quando chove então não passeio e fico deprimida Da p3 tem-se que negando o conseqüente do "se
então” devemos negar seu antecedente, logo: Sicrano é
p3: Quando não faz calor e passeio então não vejo culpado. Da p4, ao afirmar o antecedente, também afirmo
Carlos seu conseqüente, logo Fulano é culpado. Da p2, conhece-
mos alguns valores lógicos, logo: (Se Fulano é inocente,
p4: Quando não chove e estou deprimida então não então ou Beltrano é culpado, ou Sicrano é culpado, ou
passeio ambos, Beltrano e Sicrano, são culpados = F Ú V Z V = V).

p5: Hoje, passeio Portanto, ao afirmar que os três são culpados,


obtemos todas as premissas verdadeiras, tornado o
Começaremos da p5 onde sabe-se que passeio. Da argumento válido.
p2 sabe-se que o conseqüente do "se então” é negado o
antecedente também o será, ou seja, não passeio é falso, ALTERNATIVA E
o conseqüente (não passeio e fico deprimida), terá valor
falso, independentemente do valor lógico da proposição
"fico deprimida”, pois se trata de um "e” que só será 3) (Questão 70 da Prova de Técnico do MPU - Proble-
verdadeiro se todas as proposições que o formarem forem mas com Verdades e Mentiras) Você está à frente de
verdadeiras, logo chove é falso, portanto, não chove. Da p4 duas portas. Uma delas conduz a um tesouro; a
tem-se a negação do conseqüente do "se então” logo o outra, a uma sala vazia. Cosme guarda uma das
antecedente do mesmo deverá ser falso, como se trata de portas, enquanto Damião guarda a outra. Cada um
um "e” a já sabemos que "não chove” restou para que o "e” dos guardas sempre diz a verdade ou sempre mente,
seja falso que estou deprimida seja falso, logo não estoude- ou seja, ambos os guardas podem sempre mentir,
primida. Da p1 tem-se que o conseqüente do "se então” tem ambos podem sempre dizer a verdade, ou um sem-
valor lógico falso (não passeio ou fico deprimida = F Z F = pre dizer a verdade e o outro sempre mentir. Você


5DFLRFtQLR/yJLFR0DWHPiWLFR
não sabe se ambos são mentirosos, se ambos são EXERCÍCIOS:
verazes, ou se um é veraz e o outro é mentiroso.
Mas, para descobrir qual das portas conduz ao ESTRUTURAS LÓGICAS,
tesouro, você pode fazer três (e apenas três) pergun- LÓGICA SENTENCIAL
tas aos guardas, escolhendo-as da seguinte relação:
E LÓGICA DE ARGUMENTAÇÃO.
p1: O outro guarda é da mesma natureza que você
(isto é, se você é mentiroso ele também o é, e 1) (AFC-STN/2005) Se Marcos não estuda, João não
se você é veraz ele também o é)? passeia. Logo:
a) Marcos estudar é condição necessária para
p2: Você é o guarda da porta que leva ao tesouro? João não passear.
b) Marcos estudar é condição suficiente para João
p3: O outro guarda é mentiroso? passear.
c) Marcos não estudar é condição necessária
p4: Você é veraz? para João não passear.
d) Marcos não estudar é condição suficiente para
Então, uma possível seqüência de três perguntas João passear.
que é logicamente suficiente para assegurar, seja e) Marcos estudar é condição necessária para
qual for a natureza dos guardas, que você identifique João passear.
corretamente a porta que leva ao tesouro, é
2) (Fiscal Recife/2003) Pedro, após visitar uma aldeia
a) p2 a Cosme, p2 a Damião, p3 a Damião. distante, afirmou: “Não é verdade que todos os
b) p3 a Damião, p2 a Cosme, p3 a Cosme. aldeões daquela aldeia não dormem a sesta”. A
c) p3 a Cosme, p2 a Damião, p4 a Cosme. condição necessária e suficiente para que a afirma-
d) p1 a Cosme, p1 a Damião, p2 a Cosme. ção de Pedro seja verdadeira é que seja verdadeira
e) p4 a Cosme, p1 a Cosme, p2 a Damião. a seguinte proposição:
Resolução: a) No máximo um aldeão daquela aldeia não
dorme a sesta.
A melhor dica para problemas com verdades e b) Todos os aldeões daquela aldeia dormem a
mentiras é iniciar pela pessoa que diz a verdade, no sesta.
entanto, tem-se a dificuldade de termos dois guardas c) Pelo menos um aldeão daquela aldeia dorme a
onde cada um deles sempre diz a verdade ou sem- sesta.
pre mente, ou seja, ambos os guardas podem sem- d) Nenhum aldeão daquela aldeia não dorme a
pre mentir, ambos podem sempre dizer a verdade, ou sesta.
um sempre dizer a verdade e o outro sempre mentir. e) Nenhum aldeão daquela aldeia dorme a sesta.

Nesse caso as perguntas 3 e 4 não ajudam muito, 3) (AFC/2002) Dizer que não é verdade que Pedro é
pois se você não sabe se Cosme ou Damião estão dizendo pobre e Alberto é alto, é logicamente equivalente a
verdade ou mentira perguntar se o outro é mentiroso, ou dizer que é verdade que:
você é veraz fica sem efeito.
a) Pedro não é pobre ou Alberto não é alto.
Mas a pergunta p1 que afirma que "O outro guarda é b) Pedro não é pobre e Alberto não é alto.
da mesma natureza que você” é interessante, observe os c) Pedro é pobre ou Alberto não é alto.
quadros comparativos abaixo: d) se Pedro não é pobre, então Alberto é alto.
e) e Pedro não é pobre, então Alberto não é alto.
O outro guarda é da mesma natureza que você?
4) (MPOG/2001) Dizer que “André é artista ou Bernardo
Cosme Damião não é engenheiro” é logicamente equivalente a dizer
que:
Fala a verdade (responde sim) Fala a verdade (responde sim)
a) André é artista se e somente se Bernardo não
Fala a verdade (responde não) Fala a verdade (responde sim) é engenheiro.
Fala mentira (responde sim) Fala mentira (responde não) b) Se André é artista, então Bernardo não é
engenheiro.
Fala mentira (responde não) Fala mentira (responde não) c) Se André não é artista, então Bernardo é
engenheiro
d) Se Bernardo é engenheiro, então André é
Observe que as respostas de Cosme e Damião para artista.
tais perguntas já são esperadas, se os dois falam a verdade e) André não é artista e Bernardo é engenheiro
os dois respondem sim, se os dois mentem os dois respon-
dem não, se um fala a verdade e o outro mente, o que fala 5) (CVM/2000) Dizer que a afirmação “todos os econo-
a verdade diz não e o que mente diz sim, portanto de mistas são médicos” é falsa, do ponto de vista lógico,
acordo com a resposta de ambos a esta mesma pergunta, equivale a dizer que a seguinte afirmação é verdadei-
já saberei se Cosme e Damião estão falando verdade ou ra:
mentira, só falta perguntar a qualquer um deles se "Você é
o guarda da porta que leva ao tesouro”, basta perguntar ao a) pelo menos um economista não é médico
Cosme e obteremos a resposta. b) nenhum economista é médico
c) nenhum médico é economista
ALTERNATIVA D d) pelo menos um médico não é economista
e) todos os não médicos são não economistas

6) (Fiscal Trabalho/98) Dizer que "Pedro não é pedreiro


ou Paulo é paulista" é, do ponto de vista lógico, o
mesmo que dizer que:

a) se Pedro é pedreiro, então Paulo é paulista


b) se Paulo é paulista, então Pedro é pedreiro
c) se Pedro não é pedreiro, então Paulo é paulista


5DFLRFtQLR/yJLFR0DWHPiWLFR
d) se Pedro é pedreiro, então Paulo não é paulista 12) (Técnico MPU/2004-2/Esaf) Se Pedro é pintor ou
e) se Pedro não é pedreiro, então Paulo não é Carlos é cantor, Mário não é médico e Sílvio não é
paulista sociólogo. Dessa premissa pode-se corretamente
concluir que:
7) (Fiscal Trabalho/98) A negação da afirmação condi-
cional "se estiver chovendo, eu levo o guarda-chuva" a) se Pedro é pintor e Carlos não é cantor, Mário
é: é médico ou Sílvio é sociólogo.
b) se Pedro é pintor e Carlos não é cantor, Mário
a) se não estiver chovendo, eu levo o guarda- é médico ou Sílvio não é sociólogo.
chuva c) Se Pedro é pintor e Carlos é cantor, Mário é
b) não está chovendo e eu levo o guarda-chuva médico e Sílvio não é sociólogo.
c) não está chovendo e eu não levo o guarda- d) se Pedro é pintor e Carlos é cantor, Mário é
chuva médico ou Sílvio é sociólogo.
d) se estiver chovendo, eu não levo o guarda- e) se Pedro não é pintor ou Carlos é cantor,
chuva Mário não é médico e Sílvio não é sociólogo.
e) está chovendo e eu não levo o guarda-chuva
13) (AFC/STN-2005/Esaf) A afirmação “Alda é alta, ou
8) (SERPRO/96) Uma sentença logicamente equiva- Bino não é baixo, ou Ciro é calvo” é falsa. Segue-se,
lente a “Pedro é economista, então Luísa é soltei- pois, que é verdade que:
ra” é:
a) se Bino é baixo, Alda é alta, e se Bino não é
a) Pedro é economista ou Luísa é solteira. baixo, Ciro não é calvo.
b) Pedro é economista ou Luísa não é solteira. b) se Alda é alta, Bino é baixo, e se Bino é bai-
c) Se Luísa é solteira,Pedro é economista; xo, Ciro é calvo.
d) Se Pedro não é economista, então Luísa não é c) se Alda é alta, Bino é baixo, e se Bino não é
solteira; baixo, Ciro não é calvo.
e) Se Luísa não é solteira, então Pedro não é d) se Bino não é baixo, Alda é alta, e se Bino é
economista. baixo, Ciro é calvo.
e) se Alda não é alta, Bino não é baixo, e se Ciro
9) (TRT-9R-2004-FCC) Considere a seguinte proposi- é calvo, Bino não é baixo.
ção: "na eleição para a prefeitura, o candidato A será
eleito ou não será eleito”. Do ponto de vista lógico, a
afirmação da proposição caracteriza: EXERCÍCIOS:
ESTRUTURAS LÓGICAS
(A) um silogismo.
(B) uma tautologia. 14) (AFC 2002 ESAF) Se Carina é amiga de Carol, então
(C) uma equivalência. Carmem é cunhada de Carol. Carmem não é cunha-
(D) uma contingência. da de Carol. Se Carina não é cunhada de Carol,
(E) uma contradição. então Carina é amiga de Carol. Logo,
a) Carina é cunhada de Carmem e é amiga de
10) (Fiscal Trabalho 98 ESAF) Um exemplo de tautologia Carol.
é: b) Carina não é amiga de Carol ou não é cunha-
da de Carmem.
a) se João é alto, então João é alto ou Guilher- c) Carina é amiga de Carol ou não é cunhada
me é gordo de Carol.
b) se João é alto, então João é alto e Guilherme d) Carina é amiga de Carmem e é amiga de
é gordo Carol.
c) se João é alto ou Guilherme é gordo, então e) Carina é amiga de Carol e não é cunhada de
Guilherme é gordo Carmem.
d) se João é alto ou Guilherme é gordo, então
João é alto e Guilherme é gordo 15) (ANEEL 2004 ESAF) Surfo ou estudo. Fumo ou não
e) se João é alto ou não é alto, então Guilherme surfo. Velejo ou não estudo. Ora, não velejo. Assim,
é gordo
a) estudo e fumo.
b) não fumo e surfo.
11) (Gestor Fazendário MG/2005/Esaf) Considere a c) não velejo e não fumo.
afirmação P: d) estudo e não fumo.
e) fumo e surfo.
P: “A ou B
16) (Fiscal Recife 2003 ESAF) André é inocente ou Beto
Onde A e B, por sua vez, são as seguintes afirma- é inocente. Se Beto é inocente, então Caio é culpa-
ções: do. Caio é inocente se e somente se Dênis é culpa-
do. Ora, Dênis é culpado. Logo:
A: “Carlos é dentista”
a) Caio e Beto são inocentes
B: “Se Enio é economista, então Juca é arquiteto”. b) André e Caio são inocentes
c) André e Beto são inocentes
Ora, sabe-se que a afirmação P é falsa. Logo: d) Caio e Dênis são culpados
e) André e Dênis são culpados,
a) Carlos não é dentista; Enio não é economista;
Juca não é arquiteto. 17) (Oficial de Chancelaria MRE 2004 ESAF) Se a
b) Carlos não é dentista; Enio é economista; professora de matemática foi à reunião, nem a
Juca não é arquiteto. professora de inglês nem a professora de francês
c) Carlos não é dentista; Enio é economista; deram aula. Se a professora de francês não deu
Juca é arquiteto. aula, a professora de português foi à reunião. Se a
d) Carlos é dentista; Enio não é economista; professora de português foi à reunião, todos os
Juca não é arquiteto. problemas foram resolvidos. Ora, pelo menos um
e) Carlos é dentista; Enio é economista; Juca problema não foi resolvido. Logo,
não é arquiteto.


5DFLRFtQLR/yJLFR0DWHPiWLFR
a) a professora de matemática não foi à reunião 23) (MPU_Admnistrativa_2004 ESAF) Quando não vejo
e a professora de francês não deu aula. Carlos, não passeio ou fico deprimida. Quando
b) a professora de matemática e a professora de chove, não passeio e fico deprimida. Quando não faz
português não foram à reunião. calor e passeio, não vejo Carlos. Quando não chove
c) a professora de francês não deu aula e a e estou deprimida, não passeio. Hoje, passeio.
professora de português não foi à reunião. Portanto, hoje
d) a professora de francês não deu aula ou a
professora de português foi à reunião. a) vejo Carlos, e não estou deprimida, e chove, e
e) a professora de inglês e a professora de fran- faz calor.
cês não deram aula. b) não vejo Carlos, e estou deprimida, e chove, e
faz calor.
18) (AFC-SFC 2001 ESAF) Se Vera viajou, nem Camile c) vejo Carlos, e não estou deprimida, e não
nem Carla foram ao casamento. Se Carla não foi ao chove, e faz calor.
casamento, Vanderléia viajou. Se Vanderléia viajou, d) não vejo Carlos, e estou deprimida, e não
o navio afundou. Ora, o navio não afundou. Logo, chove, e não faz calor.
e) vejo Carlos, e estou deprimida, e não chove, e
a) Vera não viajou e Carla não foi ao casamento faz calor.
b) Camile e Carla não foram ao casamento
c) Carla não foi ao casamento e Vanderléia não 24) (MPU Controle Interno 2004 ESAF) Sabe-se que
viajou João estar feliz é condição necessária para Maria
d) Carla não foi ao casamento ou Vanderléia sorrir e condição suficiente para Daniela abraçar
viajou Paulo. Sabe-se, também, que Daniela abraçar Paulo
e) Vera e Vanderléia não viajaram é condição necessária e suficiente para a Sandra
abraçar Sérgio. Assim, quando Sandra não abraça
19) (MPOG 2002 ESAF) Se M = 2x + 3y, então M = 4p + Sérgio,
3r. Se M = 4p + 3r, então M = 2w– 3r. Por outro lado,
M = 2x + 3y, ou M = 0. Se M = 0, então M+H = 1. Ora, a) João está feliz, e Maria não sorri, e Daniela
M+H g 1. Logo, abraça Paulo.
a) 2w – 3r = 0 b) João não está feliz, e Maria sorri, e Daniela
b) 4p + 3r g 2w – 3r não abraça Paulo.
c) M g 2x + 3y c) João está feliz, e Maria sorri, e Daniela não
d) 2x + 3y g 2w – 3r abraça Paulo.
e) M = 2w – 3r d) João não está feliz, e Maria não sorri, e Dani-
ela não abraça Paulo.
20) (Fiscal Trabalho 98 ESAF) Se Frederico é francês, e) João não está feliz, e Maria sorri, e Daniela
então Alberto não é alemão. Ou Alberto é alemão, ou abraça Paulo.
Egídio é espanhol. Se Pedro não é português, então
Frederico é francês. Ora, nem Egídio é espanhol nem 25) (AFTN 1996 ESAF) José quer ir ao cinema assistir
Isaura é italiana. Logo: ao filme "Fogo contra Fogo" , mas não tem certeza
se o mesmo está sendo exibido. Seus amigos, Maria,
a) Pedro é português e Frederico é francês Luís e Júlio têm opiniões discordantes sobre se o
b) Pedro é português e Alberto é alemão filme está ou não em cartaz. Se Maria estiver certa,
c) Pedro não é português e Alberto é alemão então Júlio está enganado. Se Júlio estiver engana-
d) Egídio é espanhol ou Frederico é francês do, então Luís está enganado. Se Luís estiver enga-
e) Se Alberto é alemão, Frederico é francês nado, então o filme não está sendo exibido. Ora, ou
o filme "Fogo contra Fogo" está sendo exibido, ou
21) (ACExt TCU 2002 ESAF) O rei ir à caça é condição José não irá ao cinema. Verificou-se que Maria está
necessária para o duque sair do castelo, e é condi- certa. Logo:
ção suficiente para a duquesa ir ao jardim. Por outro
lado, o conde encontrar a princesa é condição
a) o filme "Fogo contra Fogo" está sendo exibido
necessária e suficiente para o barão sorrir e é condi-
ção necessária para a duquesa ir ao jardim. O barão b) Luís e Júlio não estão enganados
não sorriu. Logo: c) Júlio está enganado, mas não Luís
d) Luís está engando, mas não Júlio
a) A duquesa foi ao jardim ou o conde encontrou e) José não irá ao cinema
a princesa.
b) Se o duque não saiu do castelo, então o con- 26) (TFC-SFC 2001 ESAF) Ou Anaís será professora, ou
de encontrou a princesa. Anelise será cantora, ou Anamélia será pianista. Se
c) O rei não foi à caça e o conde não encontrou Ana for atleta, então Anamélia será pianista. Se
a princesa. Anelise for cantora, então Ana será atleta. Ora,
d) O rei foi à caça e a duquesa não foi ao jardim. Anamélia não será pianista. Então:
e) O duque saiu do castelo e o rei não foi à caça.
a) Anaís será professora e Anelise não será
22) (AFC 2002 ESAF) Se Iara não fala italiano, então cantora
Ana fala alemão. Se Iara fala italiano, então ou Ching b) Anaís não será professora e Ana não será
fala chinês ou Débora fala dinamarquês. Se Débora atleta
fala dinamarquês, Elton fala espanhol. Mas Elton fala c) Anelise não será cantora e Ana será atleta
espanhol se e somente se não for verdade que d) Anelise será cantora ou Ana será atleta
Francisco não fala francês. Ora, Francisco não fala e) Anelise será cantora e Anamélia não será
francês e Ching não fala chinês. Logo, pianista
a) Iara não fala italiano e Débora não fala dina- 27) (Assistente de Chancelaria MRE 2004 ESAF) No final
marquês. de semana, Chiquita não foi ao parque. Ora, sabe-se
b) Ching não fala chinês e Débora fala dinamar- que sempre que Didi estuda, Didi é aprovado. Sabe-
quês. se, também, que, nos finais de semana, ou Dadá vai
c) Francisco não fala francês e Elton fala espa- à missa ou vai visitar tia Célia. Sempre que Dadá vai
nhol. visitar tia Célia, Chiquita vai ao parque, e sempre que
d) Ana não fala alemão ou Iara fala italiano. Dadá vai à missa, Didi estuda. Então, no final de
e) Ana fala alemão e Débora fala dinamarquês. semana,


5DFLRFtQLR/yJLFR0DWHPiWLFR
a) Dadá foi à missa e Didi foi aprovado. 33) (ANEEL 2004 ESAF) Se não leio, não compreendo.
b) Didi não foi aprovado e Dadá não foi visitar tia Se jogo, não leio. Se não desisto, compreendo. Se é
Célia. feriado, não desisto. Então,
c) Didi não estudou e Didi foi aprovado.
d) Didi estudou e Chiquita foi ao parque. a) se jogo, não é feriado.
e) Dadá não foi à missa e Didi não foi apro- b) se não jogo, é feriado.
vado. c) se é feriado, não leio.
d) se não é feriado, leio.
28) (Oficial de Chancelaria MRE 2004 ESAF) Se X  Y, e) se é feriado, jogo.
então Z > P ou Q  R. Se Z > P, então S  T. Se S 
T, então Q  R. Ora, Q > R, logo: 34) (AFTN 1998 ESAF) Considere as afirmações: A) se
Patrícia é uma boa amiga, Vítor diz a verdade; B) se
a) S>TeZP Vítor diz a verdade, Helena não é uma boa amiga; C)
b) STeZ>P se Helena não é uma boa amiga, Patrícia é uma boa
c) XYeZP amiga. A análise do encadeamento lógico dessas
d) X>YeZP três afirmações permite concluir que elas:
e) X<YeS<T
a) são equivalentes a dizer que Patrícia é uma
29) (Fiscal do Trabalho 2003 ESAF) Se não durmo, bebo. boa amiga
Se estou furioso, durmo. Se durmo, não estou furio- b) implicam necessariamente que Patrícia é
so. Se não estou furioso, não bebo. Logo, uma boa amiga
a) não durmo, estou furioso e não bebo c) implicam necessariamente que Vítor diz a
b) durmo, estou furioso e não bebo verdade e que Helena não é uma boa amiga
c) não durmo, estou furioso e bebo d) são consistentes entre si, quer Patrícia seja
d) durmo, não estou furioso e não bebo uma boa amiga, quer Patrícia não seja uma
e) não durmo, não estou furioso e bebo boa amiga
e) são inconsistentes entre si
30) (MPU Administrativa 2004 ESAF) Se Fulano é
culpado, então Beltrano é culpado. Se Fulano é 35) (Fiscal Trabalho 98 ESAF) Maria tem três carros: um
inocente, então ou Beltrano é culpado, ou Sicrano Gol, um Corsa e um Fiesta. Um dos carros é branco,
é culpado, ou ambos, Beltrano e Sicrano, são o outro é preto, e o outro é azul. Sabe-se que: 1) ou
culpados. Se Sicrano é inocente, então Beltrano é o Gol é branco, ou o Fiesta é branco, 2) ou o Gol é
inocente. Se Sicrano é culpado, então Fulano é preto, ou o Corsa é azul, 3) ou o Fiesta é azul, ou o
culpado. Logo, Corsa é azul, 4) ou o Corsa é preto, ou o Fiesta é
preto. Portanto, as cores do Gol, do Corsa e do
a) Fulano é inocente, e Beltrano é inocente, e Fiesta são, respectivamente,
Sicrano é inocente. a) branco, preto, azul
b) Fulano é culpado, e Beltrano é culpado, e b) preto, azul, branco
Sicrano é inocente. c) azul, branco, preto
c) Fulano é culpado, e Beltrano é inocente, e d) preto, branco, azul
Sicrano é inocente. e) branco, azul, preto
d) Fulano é inocente, e Beltrano é culpado, e
Sicrano é culpado. 36) (Fiscal Trabalho 98 ESAF) De três irmãos – José,
e) Fulano é culpado, e Beltrano é culpado, e Adriano e Caio –, sabe-se que ou José é o mais
Sicrano é culpado. velho, ou Adriano é o mais moço. Sabe-se, também,
que ou Adriano é o mais velho, ou Caio é o mais
31) (AFC/CGU 2003/2004 ESAF) Homero não é honesto, velho. Então, o mais velho e o mais moço dos três
ou Júlio é justo. Homero é honesto, ou Júlio é justo, irmãos são, respectivamente:
ou Beto é bondoso. Beto é bondoso, ou Júlio não é
justo. Beto não é bondoso, ou Homero é honesto. a) Caio e José
Logo, b) Caio e Adriano
c) Adriano e Caio
a) Beto é bondoso, Homero é honesto, Júlio não d) Adriano e José
é justo. e) José e Adriano
b) Beto não é bondoso, Homero é honesto, Júlio
não é justo. 37) (Técnico MPU_Admnistrativa_2004 ESAF) Ricardo,
c) Beto é bondoso, Homero é honesto, Júlio é Rogério e Renato são irmãos. Um deles é médico,
justo. outro é professor, e o outro é músico. Sabe-se que:
d) Beto não é bondoso, Homero não é honesto, 1) ou Ricardo é médico, ou Renato é médico, 2) ou
Júlio não é justo. Ricardo é professor, ou Rogério é músico; 3) ou
e) Beto não é bondoso, Homero é honesto, Júlio Renato é músico, ou Rogério é músico, 4) ou Rogério
é justo. é professor, ou Renato é professor. Portanto, as
profissões de Ricardo, Rogério e Renato são, respec-
32) (Fiscal Trabalho 98 ESAF) Se Luís estuda História, tivamente,
então Pedro estuda Matemática. Se Helena estuda a) professor, médico, músico.
Filosofia, então Jorge estuda Medicina. Ora, Luís b) médico, professor, músico.
estuda História ou Helena estuda Filosofia. Logo, c) professor, músico, médico.
segue-se necessariamente que: d) músico, médico, professor.
e) médico, músico, professor.
a) Pedro estuda Matemática ou Jorge estuda
Medicina 38) (Fiscal do Trabalho 2003 ESAF) Investigando uma
b) Pedro estuda Matemática e Jorge estuda fraude bancária, um famoso detetive colheu evidênci-
Medicina as que o convenceram da verdade das seguintes
c) Se Luís não estuda História, então Jorge não afirmações:
estuda Medicina
d) Helena estuda Filosofia e Pedro estuda Mate- 1) Se Homero é culpado, então João é culpado.
mática 2) Se Homero é inocente, então João ou Adolfo
e) Pedro estuda Matemática ou Helena não estu- são culpados.
da Filosofia 3) Se Adolfo é inocente, então João é inocente.
4) Se Adolfo é culpado, então Homero é culpado.


5DFLRFtQLR/yJLFR0DWHPiWLFR
As evidências colhidas pelo famoso detetive indi- a) a conclusão não é decorrência necessária das
cam, portanto, que: premissas.
a) Homero, João e Adolfo são inocentes. b) a segunda premissa não é decorrência lógica
da primeira.
b) Homero, João e Adolfo são culpados.
c) Homero é culpado, mas João e Adolfo são c) a primeira premissa pode ser falsa, embora a
inocentes. segunda possa ser verdadeira.
d) Homero e João são inocentes, mas Adolfo é d) a segunda premissa pode ser falsa, embora a
culpado. primeira possa ser verdadeira.
e) Homero e Adolfo são culpados, mas João é e) o argumento só é válido se Soninha na realida-
inocente. de não sorri.

39) (AFRE MG 2005 ESAF) Se André é culpado, então 44) Assinale a alternativa que contém um argumento
Bruno é inocente. Se André é inocente, então Bruno válido.
é culpado. Se André é culpado, Leo é inocente. Se
André é inocente, então Leo é culpado. Se Bruno é a) Alguns atletas jogam xadrez.
inocente, então Leo é culpado. Logo, André, Bruno e Todos os intelectuais jogam xadrez.
Leo são, respectivamente: Conclusão: Alguns atletas são intelectuais.
b) Se estudasse tudo, eu passaria.
a) Culpado, culpado, culpado. Eu não passei.
b) Inocente, culpado, culpado. Conclusão: Eu não estudei tudo.
c) Inocente, culpado, inocente.
d) Inocente, inocente, culpado. 45) Considere as premissas:
e) Culpado, culpado, inocente.
P1. Os bebês são ilógicos.
40) (AFC/STN 2005 ESAF) Se Pedro não bebe, ele visita P2. Pessoas ilógicas são desprezadas.
Ana. Se Pedro bebe, ele lê poesias. Se Pedro não P3. Quem sabe amestrar um crocodilo não é des-
visita Ana, ele não lê poesias. Se Pedro lê poesias, prezado.
ele não visita Ana. Segue se, portanto que, Pedro:
Assinale a única alternativa que não é uma conse-
a) bebe, visita Ana, não lê poesias.
qüência lógica das três premissas apresentadas.
b) não bebe, visita Ana, não lê poesias.
c) bebe, não visita Ana, lê poesias.
d) não bebe, não visita Ana, não lê poesias. a) Bebês não sabem amestrar crocodilos.
e) não bebe, não visita Ana, lê poesias. b) Pessoas desprezadas são ilógicas.
c) Pessoas desprezadas não sabem amestrar
41) (Fiscal Trabalho 98 ESAF) Se Pedro é inocente, crocodilos.
então Lauro é inocente. Se Roberto é inocente, então d) Pessoas ilógicas não sabem amestrar crocodi-
Sônia é inocente. Ora, Pedro é culpado ou Sônia é los.
culpada. Segue-se logicamente, portanto, que: e) Bebês são desprezados.

a) Lauro é culpado e Sônia é culpada


b) Sônia é culpada e Roberto é inocente EXERCÍCIOS:
c) Pedro é culpado ou Roberto é culpado DIAGRAMAS LÓGICOS
d) Se Roberto é culpado, então Lauro é culpado
e) Roberto é inocente se e somente se Lauro é
inocente 46) (Especialista em Políticas Públicas Bahia 2004 FCC)
Considerando “todo livro é instrutivo” como uma
proposição verdadeira, é correto inferir que:
EXERCÍCIOS: a) “Nenhum livro é instrutivo” é uma proposição
LÓGICA DE ARGUMENTAÇÃO necessariamente verdadeira.
b) “Algum livro é instrutivo” é uma proposição
42) (TRT-9ª Região/2004/FCC) Observe a construção de necessariamente verdadeira.
um argumento: c) “Algum livro não é instrutivo” é uma proposi-
ção verdadeira ou falsa.
Premissas: d) “Algum livro é instrutivo” é uma proposição
verdadeira ou falsa.
Todos os cachorros têm asas. e) “Algum livro não é instrutivo” é uma proposi-
Todos os animais de asas são aquáticos. ção necessariamente verdadeira.
Existem gatos que são cachorros.
47) (TTN-98 ESAF) Se é verdade que "Alguns A são R"
Conclusão: e que "Nenhum G é R", então é necessariamente
Existem gatos que são aquáticos. verdadeiro que:
a) algum A não é G;
Sobre o argumento A, as premissas P e a conclusão b) algum A é G
C, é correto dizer que: c) nenhum A é G;
d) algum G é A;
(A) A não é válido, P é falso e C é verdadeiro. e) nenhum G é A;
(B) A não é válido, P e C são falsos.
(C) A é válido, P e C são falsos.
48) (Fiscal Trabalho 98 ESAF) Sabe-se que existe pelo
(D) A é válido, P ou C são verdadeiros.
menos um A que é B. Sabe-se, também, que todo B
(E) A é válido se P é verdadeiro e C é falso. é C. Segue-se, portanto, necessariamente que
43) SERPRO-2001/ESAF) Considere o seguinte argu- a) todo C é B
mento: “Se Soninha sorri, Sílvia é miss simpatia. Ora, b) todo C é A
Soninha não sorri. Logo, Sílvia não é miss simpatia”. c) algum A é C
Este não é um argumento logicamente válido, uma d) nada que não seja C é A
vez que: e) algum A não é C49)


5DFLRFtQLR/yJLFR0DWHPiWLFR
49) (SERPRO 2001 ESAF) Todos os alunos de matemá- d) todos os professores de piano são professo-
tica são, também, alunos de inglês, mas nenhum res de canto
aluno de inglês é aluno de história. Todos os alunos e) todos os professores de piano são professo-
de português são também alunos de informática, e res de violão
alguns alunos de informática são também alunos de
história. Como nenhum aluno de informática é aluno 54) (MPOG 2002 ESAF) Em um grupo de amigas, todas
de inglês, e como nenhum aluno de português é as meninas loiras são, também, altas e magras, mas
aluno de história, então: nenhuma menina alta e magra tem olhos azuis.
a) pelo menos um aluno de português é aluno de Todas as meninas alegres possuem cabelos crespos,
inglês. e algumas meninas de cabelos crespos têm também
b) pelo menos um aluno de matemática é aluno olhos azuis. Como nenhuma menina de cabelos
de história. crespos é alta e magra, e como neste grupo de
c) nenhum aluno de português é aluno de mate- amigas não existe nenhuma menina que tenha
mática. cabelos crespos, olhos azuis e seja alegre, então:
d) todos os alunos de informática são alunos de
matemática. a) pelo menos uma menina alegre tem olhos
e) todos os alunos de informática são alunos de azuis.
português. b) pelo menos uma menina loira tem olhos azu-
is.
50) (AFCE TCU 99 ESAF) Em uma comunidade, todo c) todas as meninas que possuem cabelos cres-
trabalhador é responsável. Todo artista, se não for pos são loiras.
filósofo, ou é trabalhador ou é poeta. Ora, não há d) todas as meninas de cabelos crespos são
filósofo e não há poeta que não seja responsável. alegres.
Portanto, tem-se que, necessariamente, e) nenhuma menina alegre é loira.

a) todo responsável é artista 55) (SERPRO 2001 ESAF) Todos os alunos de matemá-
b) todo responsável é filósofo ou poeta tica são, também, alunos de inglês, mas nenhum
c) todo artista é responsável aluno de inglês é aluno de história. Todos os alunos
d) algum filósofo é poeta de português são também alunos de informática, e
e) algum trabalhador é filósofo alguns alunos de informática são também alunos de
história. Como nenhum aluno de informática é aluno
51) (AFCE TCU 99 ESAF) Se é verdade que "Alguns de inglês, e como nenhum aluno de português é
escritores são poetas" e que "Nenhum músico é aluno de história, então:
poeta", então, também é necessariamente verdade
que a) pelo menos um aluno de português é aluno
de inglês.
a) nenhum músico é escritor b) pelo menos um aluno de matemática é aluno
b) algum escritor é músico de história.
c) algum músico é escritor c) nenhum aluno de português é aluno de mate-
d) algum escritor não é músico mática.
e) nenhum escritor é músico d) todos os alunos de informática são alunos de
matemática.
52) (MPOG 2002 ESAF) Na formatura de Hélcio, todos e) todos os alunos de informática são alunos de
os que foram à solenidade de colação de grau português.
estiveram, antes, no casamento de Hélio. Como nem
todos os amigos de Hélcio estiveram no casamento 56) (SERPRO 2001 ESAF) Todas as amigas de Aninha
de Hélio, conclui-se que, dos amigos de Hélcio: que foram à sua festa de aniversário estiveram,
antes, na festa de aniversário de Betinha. Como nem
a) todos foram à solenidade de colação de grau todas amigas de Aninha estiveram na festa de
de Hélcio e alguns não foram ao casamento aniversário de Betinha, conclui-se que, das amigas
de Hélio. de Aninha,
b) pelo menos um não foi à solenidade de cola-
ção de grau de Hélcio. a) todas foram à festa de Aninha e algumas não
c) alguns foram à solenidade de colação de foram à festa de Betinha.
grau de Hélcio, mas não foram ao casamento b) pelo menos uma não foi à festa de Aninha.
de Hélio. c) todas foram à festa de Aninha e nenhuma foi
d) alguns foram à solenidade de colação de à festa de Betinha.
grau de Hélcio e nenhum foi ao casamento de d) algumas foram à festa de Aninha mas não
Hélio. foram à festa de Betinha.
e) todos foram à solenidade de colação de grau e) algumas foram à festa de Aninha e nenhuma
de Hélcio e nenhum foi ao casamento de Hé- foi à festa de Betinha.
lio.
53) (AFC-STN 2000 ESAF) Uma escola de arte oferece
aulas de canto, dança, teatro, violão e piano. Todos EXERCÍCIOS GERAIS:
os professores de canto são, também, professores
de dança, mas nenhum professor de dança é profes- 57) (AFTN 96 ESAF) Os carros de Artur, Bernardo e
sor de teatro. Todos os professores de violão são, César são, não necessariamente nesta ordem, uma
também, professores de piano, e alguns professores Brasília, uma Parati e um Santana. Um dos carros é
de piano são, também, professores de teatro. Sabe- cinza, um outro é verde, e o outro é azul. O carro de
se que nenhum professor de piano é professor de Artur é cinza; o carro de César é o Santana; o carro
dança, e como as aulas de piano, violão e teatro não de Bernardo não é verde e não é a Brasília. As cores
têm nenhum professor em comum, então: da Brasília, da Parati e do Santana são, respectiva-
mente:
a) nenhum professor de violão é professor de
canto a) cinza, verde e azul
b) pelo menos um professor de violão é profes- b) azul, cinza e verde
sor de teatro c) azul, verde e cinza
c) pelo menos um professor de canto é profes- d) cinza, azul e verde
sor de teatro e) verde, azul e cinza


5DFLRFtQLR/yJLFR0DWHPiWLFR
58) Um jantar reúne 13 pessoas de uma mesma Na primeira partida, Celina joga contra Alberto. Na
família. Das afirmações a seguir, referentes às segunda, Ana joga contra o marido de Júlia. Na
pessoas reunidas, a única necessariamente terceira, a esposa de Alberto joga contra o marido de
verdadeira é: Ana. Na quarta, Celina joga contra Carlos. E na
quinta, a esposa de Gustavo joga contra Alberto. A
a) pelo menos uma delas tem altura superior a
esposa de Tiago e o marido de Helena são, respecti-
1,90m.
vamente:
b) pelo menos duas delas são do sexo feminino.
c) pelo menos duas delas fazem aniversário no a) Celina e Alberto
mesmo mês. b) Ana e Carlos
d) pelo menos uma delas nasceu num dia par. c) Júlia e Gustavo
e) pelo menos uma delas nasceu em janeiro ou d) Ana e Alberto
fevereiro. e) Celina e Gustavo
59) (ANEEL 2004 ESAF) Fátima, Beatriz, Gina, Sílvia e 62) (MPOG 2003 ESAF) Três amigos, Beto, Caio e
Carla são atrizes de teatro infantil, e vão participar de Dario, juntamente com suas namoradas,
uma peça em que representarão, não necessaria- sentaram-se, lado a lado, em um teatro, para
mente nesta ordem, os papéis de Fada, Bruxa, assistir a um grupo de dança. Um deles é carioca,
Rainha, Princesa e Governanta. Como todas são outro é nordestino, e outro catarinense. Sabe-se,
atrizes versáteis, o diretor da peça realizou um também, que um é médico, outro é engenheiro e
sorteio para determinar a qual delas caberia cada outro é professor. Nenhum deles sentou-se ao
papel. Antes de anunciar o resultado, o diretor lado da namorada, e nenhuma pessoa sentou-se
reuniu-as e pediu que cada uma desse seu palpite ao lado de outra do mesmo sexo. As namoradas
sobre qual havia sido o resultado do sorteio. Disse chamam-se, não necessariamente nesta ordem,
Fátima: “Acho que eu sou a Governanta, Beatriz é a Lúcia, Samanta e Teresa. O médico sentou-se em
Fada, Sílvia é a Bruxa e Carla é a Princesa”. um dos dois lugares do meio, ficando mais próxi-
mo de Lúcia do que de Dario ou do que do cario-
Disse Beatriz: “Acho que Fátima é a Princesa ou a ca. O catarinense está sentado em uma das
Bruxa”. pontas, e a namorada do professor está sentada
Disse Gina: “Acho que Silvia é a Governanta ou a à sua direita. Beto está sentado entre Teresa, que
Rainha”. está à sua esquerda, e Samanta. As namoradas
Disse Sílvia: “Acho que eu sou a Princesa”. de Caio e de Dario são, respectivamente:
Disse Carla: “Acho que a Bruxa sou eu ou Bea-
triz”. a) Teresa e Samanta
b) Samanta e Teresa
Neste ponto, o diretor falou: “Todos os palpites c) Lúcia e Samanta
estão completamente errados; nenhuma de vocês d) Lúcia e Teresa
acertou sequer um dos resultados do sorteio”! Um e) Teresa e Lúcia
estudante de Lógica, que a tudo assistia, concluiu
então, corretamente, que os papéis sorteados 63) (Fiscal do Trabalho 2003 ESAF) Três amigas
para Fátima, Beatriz, Gina e Sílvia foram, respecti- encontram-se em uma festa. O vestido de uma delas
vamente, é azul, o de outra é preto, e o da outra é branco. Elas
calçam pares de sapatos destas mesmas três cores,
a) rainha, bruxa, princesa, fada. mas somente Ana está com vestido e sapatos de
b) rainha, princesa, governanta, fada. mesma cor. Nem o vestido nem os sapatos de Júlia
c) fada, bruxa, governanta, princesa. são brancos. Marisa está com sapatos azuis. Desse
d) rainha, princesa, bruxa, fada. modo,
e) fada, bruxa, rainha, princesa.
a) o vestido de Júlia é azul e o de Ana é preto.
60) (AFC 2002 ESAF) Um agente de viagens atende três b) o vestido de Júlia é branco e seus sapatos
amigas. Uma delas é loura, outra é morena e a outra são pretos.
é ruiva. O agente sabe que uma delas se chama
c) os sapatos de Júlia são pretos e os de Ana
Bete, outra se chama Elza e a outra se chama Sara.
são brancos.
Sabe, ainda, que cada uma delas fará uma viagem a
um país diferente da Europa: uma delas irá à Alema- d) os sapatos de Ana são pretos e o vestido de
nha, outra irá à França e a outra irá à Espanha. Ao Marisa é branco.
agente de viagens, que queria identificar o nome e o e) o vestido de Ana é preto e os sapatos de Ma-
destino de cada uma, elas deram as seguintes risa são azuis.
informações:
64) (AFC-SFC 2001 ESAF) Os cursos de Márcia, Bereni-
A loura: “Não vou à França nem à Espanha”. ce e Priscila são, não necessariamente nesta ordem,
A morena: “Meu nome não é Elza nem Sara”. Medicina, Biologia e Psicologia. Uma delas realizou
A ruiva: “Nem eu nem Elza vamos à França”. seu curso em Belo Horizonte, a outra em Florianópo-
lis, e a outra em São Paulo. Márcia realizou seu
O agente de viagens concluiu, então, acertadamente, curso em Belo Horizonte. Priscila cursou Psicologia.
que: Berenice não realizou seu curso em São Paulo e não
fez Medicina. Assim, cursos e respectivos locais de
a) A loura é Sara e vai à Espanha. estudo de Márcia, Berenice e Priscila são, pela
b) A ruiva é Sara e vai à França. ordem:
c) A ruiva é Bete e vai à Espanha.
d) A morena é Bete e vai à Espanha. a) Medicina em Belo Horizonte, Psicologia em
e) A loura é Elza e vai à Alemanha. Florianópolis, Biologia em São Paulo
b) Psicologia em Belo Horizonte, Biologia em
61) (Fiscal do Trabalho 2003 ESAF) Quatro casais Florianópolis, Medicina em São Paulo
reúnem-se para jogar xadrez. Como há apenas um c) Medicina em Belo Horizonte, Biologia em
tabuleiro, eles combinam que: Florianópolis, Psicologia em São Paulo
d) Biologia em Belo Horizonte, Medicina em São
a) nenhuma pessoa pode jogar duas partidas Paulo, Psicologia em Florianópolis
seguidas; e) Medicina em Belo Horizonte, Biologia em São
b) marido e esposa não jogam entre si. Paulo, Psicologia em Florianópolis


5DFLRFtQLR/yJLFR0DWHPiWLFR
65) (Analista MPU 2004 ESAF) Caio, Décio, Éder, 69) (MPU 2004 ESAF) Cinco irmãos exercem, cada um,
Felipe e Gil compraram, cada um, um barco. uma profissão diferente. Luís é paulista, como o
Combinaram, então, dar aos barcos os nomes de agrônomo, e é mais moço do que o engenheiro e
suas filhas. Cada um tem uma única filha, e todas mais velho do que Oscar. O agrônomo, o economista
têm nomes diferentes. Ficou acertado que nenhum e Mário residem no mesmo bairro. O economista, o
deles poderia dar a seu barco o nome da própria matemático e Luís são, todos, torcedores do Flamen-
filha e que a cada nome das filhas corresponderia go. O matemático costuma ir ao cinema com Mário e
um e apenas um barco. Décio e Éder desejavam, Nédio. O economista é mais velho do que Nédio e
ambos, dar a seus barcos o nome de Laís, mas mais moço do que Pedro; este, por sua vez, é mais
acabaram entrando em um acordo: o nome de moço do que o arquiteto.
Laís ficou para o barco de Décio e Éder deu a seu Logo,
barco o nome de Mara. Gil convenceu o pai de
Olga a pôr o nome de Paula em seu barco (isto é, a) Mário é engenheiro, e o matemático é mais
no barco dele, pai de Olga). Ao barco de Caio, velho do que o agrônomo, e o economista é
coube o nome de Nair, e ao barco do pai de Nair, mais novo do que Luís.
coube o nome de Olga. As filhas de Caio, Décio, b) Oscar é engenheiro, e o matemático é mais
Éder, Felipe e Gil são, respectivamente, velho do que o agrônomo, e Luís é mais velho
a) Mara, Nair, Paula, Olga, Laís. do que o matemático.
b) Laís, Mara, Olga, Nair, Paula. c) Pedro é matemático, e o arquiteto é mais velho
c) Nair, Laís, Mara, Paula, Olga. do que o engenheiro, e Oscar é mais velho do
d) Paula, Olga, Laís, Nair, Mara. que o agrônomo.
e) Laís, Mara, Paula, Olga, Nair. d) Luís é arquiteto, e o engenheiro é mais velho
do que o agrônomo, e Pedro é mais velho do
que o matemático.
66) (Assistente de Chancelaria MRE 2004 ESAF) Quatro e) Nédio é engenheiro, e o arquiteto é mais velho
meninas que formam uma fila estão usando blusas do que o matemático, e Mário é mais velho do
de cores diferentes, amarelo, verde, azul e preto. A que o economista.
menina que está imediatamente antes da menina que
veste blusa azul é menor do que a que está imediata-
mente depois da menina de blusa azul. A menina que 70) (AFC/CGU 2003/2004 ESAF) Três homens são
está usando blusa verde é a menor de todas e está levados à presença de um jovem lógico. Sabe-se
depois da menina de blusa azul. A menina de blusa que um deles é um honesto marceneiro, que
amarela está depois da menina que veste blusa sempre diz a verdade. Sabe-se, também, que um
preta. As cores das blusas da primeira e da segunda outro é um pedreiro, igualmente honesto e traba-
menina da fila são, respectivamente: lhador, mas que tem o estranho costume de
sempre mentir, de jamais dizer a verdade. Sabe-
a) amarelo e verde. se, ainda, que o restante é um vulgar ladrão que
b) azul e verde. ora mente, ora diz a verdade. O problema é que
c) preto e azul. não se sabe quem, entre eles, é quem. À frente do
d) verde e preto. jovem lógico, esses três homens fazem, ordenada-
e) preto e amarelo. mente, as seguintes declarações:

O primeiro diz: “Eu sou o ladrão.”


67) (MPU_Admnistrativa_2004 ESAF) Em torno de uma O segundo diz: “É verdade; ele, o que acabou de
mesa quadrada, encontram-se sentados quatro falar, é o ladrão.”
sindicalistas. Oliveira, o mais antigo entre eles, é O terceiro diz: “Eu sou o ladrão.”
mineiro. Há também um paulista, um carioca e um
baiano. Paulo está sentado à direita de Oliveira.
Norton, à direita do paulista. Por sua vez, Vasconce- Com base nestas informações, o jovem lógico pode,
los, que não é carioca, encontra-se à frente de Paulo. então, concluir corretamente que:
Assim,
a) O ladrão é o primeiro e o marceneiro é o tercei-
a) Paulo é paulista e Vasconcelos é baiano. ro.
b) Paulo é carioca e Vasconcelos é baiano. b) O ladrão é o primeiro e o marceneiro é o se-
c) Norton é baiano e Vasconcelos é paulista. gundo.
d) Norton é carioca e Vasconcelos é paulista. c) O pedreiro é o primeiro e o ladrão é o segundo.
e) Paulo é baiano e Vasconcelos é paulista. d) O pedreiro é o primeiro e o ladrão é o terceiro.
e) O marceneiro é o primeiro e o ladrão é o se-
gundo.
68) (Analista MPU 2004 ESAF) Ana, Bia, Clô, Déa e Ema
estão sentadas, nessa ordem e em sentido horário,
em torno de uma mesa redonda. Elas estão reunidas 71) (CGM RJ 2003 FJG) Juca, João e José fizeram as
para eleger aquela que, entre elas, passará a ser a seguintes afirmações:
representante do grupo. Feita a votação, verificou-se
que nenhuma fora eleita, pois cada uma delas havia Juca: Eu fui aprovado no concurso ou José foi
recebido exatamente um voto. Após conversarem aprovado no concurso.
sobre tão inusitado resultado, concluíram que cada João: Se José não foi aprovado no concurso, então
uma havia votado naquela que votou na sua vizinha eu fui aprovado no concurso.
da esquerda (isto é, Ana votou naquela que votou na José: Eu fui aprovado no concurso ou João foi
vizinha da esquerda de Ana, Bia votou naquela que aprovado no concurso.
votou na vizinha da esquerda de Bia, e assim por
diante). Os votos de Ana, Bia, Clô, Déa e Ema foram, Admitindo-se que apenas uma das três afirmações
respectivamente, para, acima seja verdadeira, é correto concluir que:
a) Ema, Ana, Bia, Clô, Déa. A) José foi aprovado no concurso
b) Déa, Ema, Ana, Bia, Clô. B) Juca foi aprovado no concurso
c) Clô, Bia, Ana, Ema, Déa. C) Juca e João foram aprovados no concurso
d) Déa, Ana, Bia, Ema, Clô. D) José e João foram aprovados no concurso
e) Clô, Déa, Ema, Ana, Bia.


5DFLRFtQLR/yJLFR0DWHPiWLFR
72) (MPOG 2002) Cinco amigas, Ana, Bia, Cati, Dida e e) o jovem poderia ser vingo ou mingo, e a estra-
Elisa, são tias ou irmãs de Zilda. As tias de Zilda da levava à Aldeia Azul
sempre contam a verdade e as irmãs de Zilda sem-
pre mentem. Ana diz que Bia é tia de Zilda. Bia diz
que Cati é irmã de Zilda. Cati diz que Dida é irmã de 75) Há três suspeitos de um crime: o cozinheiro, a
Zilda. Dida diz que Bia e Elisa têm diferentes graus governanta e o mordomo. Sabe-se que o crime foi
de parentesco com Zilda, isto é: se uma é tia a outra efetivamente cometido por um ou por mais de um
é irmã. Elisa diz que Ana é tia de Zilda. Assim, o deles, já que podem ter agido individualmente ou
número de irmãs de Zilda neste conjunto de cinco não. Sabe-se, ainda, que: A) se o cozinheiro é
amigas é dado por: inocente, então a governanta é culpada; B) ou o
mordomo é culpado ou a governanta é culpada, mas
a) 1 não os dois; C) o mordomo não é inocente. Logo:
b) 2
c) 3 a) a governanta e o mordomo são os culpados
d) 4 b) somente o cozinheiro é inocente
e) 5 c) somente a governanta é culpada
d) somente o mordomo é culpado
e) o cozinheiro e o mordomo são os culpados
73) (Analista MPU/ESAF) Fernanda atrasou-se e chega
ao estádio da Ulbra quando o jogo de vôlei já está
em andamento. Ela pergunta às suas amigas, que
estão assistindo à partida, desde o início, qual o
resultado até o momento. Suas amigas dizem-lhe:

Amanda: “Neste set, o escore está 13 a 12”.


Berenice: “O escore não está 13 a 12, e a Ulbra já
ganhou o primeiro set”.
Camila: “Este set está 13 a 12, a favor da Ulbra”.
Denise: “O escore não está 13 a 12, a Ulbra está
perdendo este set, e quem vai sacar é a equipe
visitante”.
Eunice: “Quem vai sacar é a equipe visitante, e a
Ulbra está ganhando este set”.

Conhecendo suas amigas, Fernanda sabe que duas


delas estão mentindo e que as demais estão dizendo
a verdade. Conclui, então, corretamente, que

a) o escore está 13 a 12, e a Ulbra está perden-


do este set, e quem vai sacar é a equipe visi-
tante.
b) o escore está 13 a 12, e a Ulbra está vencen- RESPOSTAS
do este set, e quem vai sacar é a equipe visi-
tante. 1-e 16 - b 31 - c 46 - b 61 - a
c) o escore não está 13 a 12, e a Ulbra está
vencendo este set, e quem vai sacar é a equi-
pe visitante. 2-c 17 - b 32 - a 47 - a 62 - b
d) o escore não está 13 a 12, e a Ulbra não está
vencendo este set, e a Ulbra venceu o primei- 3-a 18 - e 33 - a 48 - c 63 - c
ro set.
e) o escore está 13 a 12, e a Ulbra vai sacar, e a 4-d 19 - e 34 - d 49 - c 64 - c
Ulbra venceu o primeiro set.
5-a 20 - b 35 - e 50 - c 65 - e
74) (CVM – 2000) Beatriz encontrava-se em viagem por
um país distante, habitado pelos vingos e pelos 6-a 21 - c 36 - b 51 - d 66 - c
mingos. Os vingos sempre dizem a verdade; já os
mingos sempre mentem. Certo dia, vendo-se perdida
em uma estrada, Beatriz dirigiu-se a um jovem que 7-e 22 - a 37 - e 52 - b 67 - a
por ali passava e perguntou-lhe: “Esta estrada leva à
Aldeia Azul?”. O jovem respondeu-lhe: “Sim, esta 8-e 23 - c 38 - b 53 - a 68 - b
estrada leva à Aldeia Azul”. Como não soubesse se
o jovem era vingo ou mingo, Beatriz fez-lhe outra 9-b 24 - d 39 - c 54 - e 69 - a
pergunta: “E se eu te perguntasse se és mingo, o que
me responderias?”. E o jovem respondeu: “Respon-
deria que sim”. 10 - a 25 - e 40 - b 55 - c 70 - b

Dadas as respostas do jovem, Beatriz pôde conclu- 11 - b 26 - a 41 - c 56 - b 71 - b


ir corretamente que
12 - b 27 - a 42 - c 57 - d 72 - c
a) o jovem era mingo e a estrada não levava à
Aldeia Azul
b) o jovem era mingo e a estrada levava à Alde- 13 - c 28 - a 43 - a 58 - c 73 - b
ia Azul
c) o jovem era vingo e a estrada não levava à 14 - b 29 - d 44 - b 59 - d 74 - a
Aldeia Azul
d) o jovem era vingo e a estrada levava à Aldeia 15 - d 30 - e 45 - b 60 - e 75 - e
Azul


5DFLRFtQLR/yJLFR0DWHPiWLFR
Energia elétrica é o que faz a televisão funcionar
COMPREENSÃO E ELABORAÇÃO DA assim como combustível faz o carro funcionar.
LÓGICA DAS SITUAÇÕES POR MEIO
DE: RACIOCÍNIO VERBAL, RACIOCÍNIO RACIOCÍNIO NUMÉRICO
MATEMÁTICO, RACIOCÍNIO SEQUEN- Definição: É a capacidade de compreender proble-
CIAL, ORIENTAÇÃO ESPACIAL E TEM- mas que utilizam operações que envolvam números, bem
como o domínio das operações aritméticas básicas.
PORAL, FORMAÇÃO DE CONCEITOS,
DISCRIMINAÇÃO DE ELEMENTOS. As questões relativas a raciocínio numérico são
apresentadas sob a forma de seqüências de números.
Deve-se encontrar a lei de formação da seqüência para dar
continuidade à mesma. Exemplos:
Esses elementos são constituídos por alguns tipos de
raciocinios, como: verbal, numérico, abstrato e espacial.
Essas relações contribuem para a compreensão e elabora- 1) Escreva o próximo termo da seqüência:
ção do processo lógico de uma situação.
1 2 3 4 5 6 ?
RACIOCÍNIO VERBAL A resposta é 7. Essa é a seqüência dos números
naturais.
Definição: É a capacidade de compreender e usar os
conceitos verbais para organizar o pensamento e estabele-
cer relações abstratas entre conceitos verbais. 2) Escreva o próximo termo da seqüência:

As questões relativas ao raciocínio verbal são 2 4 6 8 10 12 ?


apresentadas sob a forma de analogias. Após a percepção
da relação entre um primeiro par de palavras, deve-se A resposta é 14. Essa é a seqüência dos números
encontrar uma quarta palavra que mantenha relação com pares.
uma terceira palavra apresentada. Exemplos:
3) Escreva o próximo termo da seqüência:
1) Homem está para Menino, como Mulher está para
1 2 4 8 16 32 ?
_________.
a) Senhora d) Boneca A resposta é 64. A lei de formação da seqüência é
b) Menina e) Nenê dada pelo dobro do número anterior, perceba que o
c) Jovem segundo número é o dobro do primeiro e o terceiro o
dobro do segundo e assim por diante, então o
A resposta é Menina. próximo número será o dobro de 32, ou seja, 64.
Os homens na infância são chamados de meninos e
as mulheres de meninas. 4) Escreva o próximo termo da seqüência:

0 1 4 9 25 36 ?
2) Presidente está para o país assim como o Papa está
para ___________. A resposta é 49. A lei de formação dessa seqüência
é a multiplicação do número por ele mesmo, perceba:
a) Igreja d) Missa
b) Templo e) Europa 0x0 = 0 1x1 = 1 2x2 = 4 3x3=9 4x4=16
c) Mundo 5x5=25 6x6=36 7x7=49
A resposta é Igreja.
Pode-se dizer também que a lei de formação é elevar
O presidente é o representante do país assim como o número ao quadrado, alias elevar o número ao
o Papa é o representante da Igreja. quadrado é o mesmo que multiplicar ele por ele
mesmo.
3) Pelé está para o futebol assim como Michael Jordan
está para __________. RACIOCÍNIO ABSTRATO
a) Handball d) Basquete Definição: É a capacidade de compreender e
b) Vôlei e) Automobilismo estabelecer relações entre objetos e situações similares,
c) Golf comparando símbolos, idéias e conceitos.
A resposta é Basquete. As questões relativas a raciocínio abstrato exigem a
Pelé foi o maior jogador de futebol de todos os tempos análise de certa relação de figuras, objetos, etc. Exemplos:
assim como Michael Jordan foi o de basquete.

1) Qual das cinco representa a melhor comparação?


4) Televisão está para energia elétrica assim como
carro está para _________. está para assim como está para:
a) Combustível d) Rodas
b) Farol e) Óleo a) d)
c) Volante
b) e)
A resposta é Combustível.
c)


5DFLRFtQLR/yJLFR0DWHPiWLFR
A resposta é C. Inicialmente temos um circulo dividi- gulos iguais AMN, ADN e BMN, BCN; segue mais
do em duas partes, então a quadrado também deve uma vez que a área da região sombreada é metade
ser dividido em duas partes. da área do quadrado. Finalmente, a área do triângulo
sombreado na opção V é maior do que a área do
triângulo sombreado da opção II, ou seja, é maior do
2) Qual das cinco se parece menos com as outras
que metade da área do quadrado. Comentário:
quatro?
observamos que na opção IV o ponto N não precisa
ser o ponto médio do lado CD. De fato, o argumento
a) d) usado acima para analisar essa opção não depende
da posição de N ao longo de CD.
b) e)
c)

A resposta é D. Todos as figuras são compostas por


segmentos retos, exceto o círculo.

3) Complete a seqüência abaixo?


2) Cinco discos de papelão foram colocados um a um
sobre uma mesa, conforme mostra a figura. Em que
? ordem os discos foram colocados na mesa?

a) V, R, S, U, T
b) U, R, V, S, T
c) R, S, U, V, T
d) T, U, R, V, S
A B C D e) V, R, U, S, T

A resposta é D. Perceba que a figura externa gira de


90º graus no sentido horário. A barra interna fica A resposta é A Solução 1: Na figura vê-se que V está
trocando de lado, esquerda – direita, e a outra figura abaixo de R, que está abaixo de S, que está abaixo
interna gira em sentido anti-horário de 90º. de U, que está abaixo de T. Logo a ordem em que os
discos foram colocados sobre a mesa é V, R, S, U, T.
4) Qual das cinco se parece menos com as outras
quatro?
EXERCÍCIOS PROPOSTOS:
Barco – Avião – Carro – Ônibus – Pneu
RACIOCÍNIO VERBAL, NUMÉRICO, ABSTRATO
A resposta é pneu. Todas as palavras são meios de E ESPACIAL
transportes, exceto pneu.
1) Livro está para ler assim como caderno está para
_________:
RACIOCÍNIO ESPACIAL
a) Escrever d) Ler
Definição: É a aptidão para visualizar relações de b) Copiar e) Colorir
espaço, de dimensão, de posição e de direção, bem como c) Pintar
julgar visualmente formas geométricas.

Exemplos: 2) Surf está para o prancha assim como tênis está para
__________.
1) Os quadrados abaixo têm todos o mesmo tamanho. a) Quadra d) Shorts
b) Bola e) Rede
c) Raquete

3) Trinco está para tranco assim como brinco está para


I II III IV V ___________.

Em qual deles a região sombreada tem a maior a) Banco d) Branca


área? b) Banca e) Branco
c) Brama
a) I d) IV
b) II e) V
4) Rádio está para som assim como televisão está para
c) III
__________.
A resposta é E. Na opção I o quadrado está dividido
em quatro triângulos iguais, de modo que a área da a) Imagem d) Tela
região sombreada é a metade da área do quadrado. b) Figura e) Canal
Na opção II, a diagonal divide o quadrado em dois c) Antena
triângulos iguais, e outra vez a área da região som-
breada é metade da área do quadrado. Na opção III 5) Espetáculo está para teatro assim como filme está
o triângulo sombreado tem área menor do que o para __________.
triângulo sombreado da opção II, ou seja, menor que
metade da área do quadrado. Na opção IV, observa- a) Câmera d) Cinema
mos na figura ao lado que a perpendicular MN ao b) Radio e) Televisão
segmento AB divide o quadrado nos pares de triân- c) Antena


5DFLRFtQLR/yJLFR0DWHPiWLFR
6) Deputado Federal está para o congresso assim como 15) Qual dos cinco se parece menos com os outros?
senador está para _____.

a) Câmara d) Ministério
b) Prefeitura e) Senado
c) Assembléia

7) Sapato está para pé assim como luva está para


__________.

a) Pescoço d) Dedo
b) Mão e) Perna
c) Braço

16) Complete a seqüência com uma das seis figuras


8) Botânico está para sociólogo assim como planta esta dadas.
para _________.

a) Mulher d) Sociedade
b) Problema e) Social
c) Sociologia

9) Calculadora está para calcular assim como tesoura


está para __________.

a) Rasgar d) Abrir
b) Cortar e) Colar
c) Fatiar

10) Vaca está para estábulo assim como homem está


para _____________.

a) Celeiro d) Fazenda
b) Leite e) Restaurante 17) Para montar um cubo, Guilherme recortou um peda-
c) Casa ço de cartolina branca e pintou de cinza algumas
partes, como na figura ao lado. Qual das figuras
abaixo representa o cubo construído por Guilherme?
11) Escreva o próximo termo da seqüência

a) 0 1 8 27 64 ? a)
b) 2 10 16 17 18 19 ?
c) 2 12 21 29 36 42 47 ?
d) 68 67 64 59 52 43 32 ? b)
e) 1 11 21 1211 111221 312211 ?
c)
12) Qual dos quatro se parece menos com os outros?

ǻ * E M d)

e)
13) Qual dos quatro se parece menos com os outros?

18) I II III

14) Complete a seqüência com uma das quatro figuras


dadas.
IV V

Paulo usou quatro peças diferentes dentre as cinco


acima para montar a figura indicada. Em qual das
peças está o quadradinho marcado com X?
a) I
b) II
c) III
d) IV
e) V


5DFLRFtQLR/yJLFR0DWHPiWLFR
GABARITO COMENTADO 12) Resposta: Triângulo. Todas as figuras são formadas
DOS EXERCÍCIOS PROPOSTOS: RACIOCÍNIO por quatro segmentos retos, exceto pelo triangulo
VERBAL, NUMÉRICO, ABSTRATO E ESPACIAIS que é formado por três.

1) Alternativa A. O Livro é feito ler e o caderno feito para 13) Resposta: Todas as figuras são formadas por dois
escrever. quadrados e um circulo exceto pela primeira que
possui dois círculos e um quadrado, portanto a
2) Alternativa C. Prancha é um item necessário para primeira figura é a que menos parece com as outras.
praticar Surf e raquete é necessário para jogar tênis.
14) Resposta: figura 4. Na última linha todas figuras
3) Alternativa E. Basta trocar o I pelo A. possuem apenas uma cor.

4) Alternativa A. A principal característica do rádio é a 15) Resposta: figura 2. A figura 1 e 5 são idênticas, bem
geração de som e da televisão a geração de imagem. como a 3 e a 4; a única que não possui par é a 2.

5) Alternativa D. Os espetáculos são apresentados no 16) Resposta: figura 6. Em cada fileira a cabeça, corpo,
teatro, já os filmes no cinema. perna e rabo devem ser distintos.

6) Alternativa E. O congresso é o local de trabalho dos 17) Resposta: figura C. Ao montar o cubo, a face branca
deputados e o senado o dos senadores. e a face cinza ficam opostas; logo as alternativas (A)
e (B) estão excluídas. As alternativas (D) e (E) estão
7) Alternativa B. Sapato se usa nos pés e luva nas excluídas pois no cubo não podem aparecer um
mãos. retângulo branco e outro cinza com um lado menor
em comum.
8) Alternativa D. Botânico estuda as plantas e o soció-
logo a sociedade. 18) Resposta A. Por tentativa e erro vemos que há
apenas duas maneiras de cobrir a figura com quatro
9) Alternativa B. A função da calculadora é executar peças, conforme mostrado abaixo. Em ambas, a
cálculos e da tesoura é cortar. casa com o X é coberta pela peça I.

10) Alternativa C. Estábulo é o local onde as vacas


moram e casa é onde os homens moram.

11) a) Seqüência dos cubos dos números – 03,13,23,


33, 43... logo o próximo número é 53 = 125.
X X
b) Seqüência dos números que começam com a
letra D. Dois, Dez, Dezesseis, Dezessete,
Dezoito, Dezenove.... O próximo será 200
(DUZENTOS) EXERCÍCIOS GERAIS
c) Seqüência que se obtém partindo do numero 4
e somando 10,9,8,7,.... 1) Qual dos cinco se parece menos com os outros
02 + 10 =12 quatro?
12 + 09= 21
21 + 08= 29 a) urso d) cachorro
29 + 07= 36 b) cobra e) tigre
36 + 06= 42 c) vaca
42 + 05= 47
47 + 04= 51 2) Se você reordenar as letras "ECHOOL” o resultado
Assim o próximo número da seqüência é: 51 será o nome de um:
d) Seqüência que subtrai os números ímpares do a) oceano d) estado
anterior para obter o próximo: b) país e) animal
68 - 01 = 67 c) cidade
67 – 03= 64
64 – 05= 59
59 – 07= 52 3) Qual dos cinco desenhos representa a melhor
52 – 09= 43 comparação?
43 – 11= 32
32 – 13= 19
Assim o próximo número da seqüência é: 19 está para assim como está para:

e) A lógica dessa seqüência é contar a formação


do número anterior. A seqüência começa com a) d)
o número 1, o segundo número diz que o
anterior é formado por um número um (11), o b) e)
próximo diz que o anterior (o onze) é formado
por dois números um (21), o próximo diz que o
anterior (o vinte e um) é formado por um núme- c)
ro dois e um número um (1211) e assim por
diante; então o próximo da seqüência tem que
contar a formação do número 312211, que é 4) Qual dos cinco se parece menos com os outros?
formado por um número 3, um número 1, dois
números 2 e dois números 1, assim o próximo a) batata d) feijão
número será: 13112221. b) milho e) maçã
c) cenoura


5DFLRFtQLR/yJLFR0DWHPiWLFR
5) Qual dos cinco desenhos representa a melhor 13) Qual dos cinco desenhos se parece menos com os
comparação? outros quatro?

está para assim como está para:


a) d)

a) d)

b) e) b) e)

c)
c)

6) João, que tem doze anos, é três vezes mais velho


que o irmão. Qual será a idade de João quando ele
for duas vezes mais velho que o irmão? 14) Qual dos cinco representa a melhor comparação?
a) 15 b) 16 c) 18 d) 20 e) 21 Árvore está para chão assim como chaminé está
para:

7) Qual dos cinco representa a melhor comparação? a) fumaça d) garagem


b) tijolo e) casa
Irmão está para irmã assim como sobrinha está para: c) céu
a) mãe d) tia
b) tio e) sobrinho 15) Qual dos números não pertence a série?
c) filha
9-7-8-6-7-5-6-3
8) Qual das cinco letras se parece menos com as a) 9 e) 7
outras? b) 7 f) 5
c) 8 g) 6
a) A b) Z c) F d) N e) E d) 6 h) 3

9) Qual das cinco representa a melhor comparação? 16) Qual dos cinco se parece menos com os outros
quatro?
Leite está para vidro assim como carta está para:
a) tato d) sorriso
a) selo d) correio b) paladar e) visão
b) caneta e) envelope c) audição
c) livro

17) Qual dos cinco desenhos representa a melhor


10) Qual dos cinco desenhos se parece menos com os comparação?
outros quatro?
está para assim como está
a) d) para:

a) d)
b) e)
b) e)

c) c)

18) João é mais alto que Pedro, e Antônio é mais baixo


11) Qual das cinco alternativas representa a melhor que João.
comparação?
AMOR está para ROMA assim como 5232 está para: Qual das alternativas abaixo estaria mais correta?

a) 2523 d) 3225 a) Antônio é mais alto que Pedro.


b) 3252 e) 5223 b) Antônio é mais baixo que Pedro.
c) 2325 c) Antônio tem a mesma altura que Pedro.
d) É impossível dizer quem é mais alto, se Antô-
nio ou Pedro.
12) Se alguns Smaugs são Thors e alguns Thors são
Thrains, então alguns Smaugs serão obrigatoriamen-
te Thrains. Esta afirmação é: 19) Qual dos cinco se parece menos com os outros
quatro?
a) falsa a) bolsa d) sapato
b) verdadeira b) meia e) vestido
c) nenhuma das duas c) calça


5DFLRFtQLR/yJLFR0DWHPiWLFR
20) Qual das cinco alternativas representa a melhor 28) O preço de um produto foi reduzido em 20% numa
comparação? liquidação, qual deverá ser a percentagem de au-
CAACCAC está para 3113313 assim como CACAA- mento do preço do mesmo produto para que ele volte
CAC está para: a ter o preço original?

a) 13133131 d) 31311313 a) 15% c) 25% e) 40%


b) 13133313 e) 31313113 b) 20% d) 30%
c) 31311131
29) Qual dos cinco se parece menos com os outros
21) Se você reordenar as letras " RAPIS ” o resultado quatro?
será o nome de um: a) cobre c) bronze e) chumbo
a) oceano c) estado e) animal b) ferro d) estanho
b) país d) cidade
30) Qual dos cinco desenhos representa a melhor
22) Qual dos cinco desenhos se parece menos com os comparação?
outros quatro?
está para assim como está
a) d)
para:
b) e)
a) d)
c)
b) e)

23) Qual dos cinco representa a melhor comparação? c)


Bala está para revólver assim como bomba está
para:
31) Qual das cinco se parece menos com as outras
a) atiradeira c) canhão e) aríete quatro?
b) pé d) catapulta
a) garrafa c) copo e) funil
b) banheira d) xícara
24) Se alguns Bifurs são Bofurs e todos os Gloins são
Bofurs, então alguns Bifurs são obrigatoriamente
Gloins. Esta afirmativa é: 32) Maria tinha vários biscoitos. Depois de comer um, ela
deu a metade para a irmã. Depois de comer mais
a) verdadeira um, a irmã deu a metade do que sobrou para o
b) falsa irmão. O irmão de Maria ganhou cinco biscoitos.
c) nenhuma das duas Quantos biscoitos tinha Maria no início?
a) 11 c) 23 e) 46
25) Qual das cinco figuras se parece menos com as
outras quatro? b) 22 d) 45

a) c) e) 33) Qual dos cinco se parece menos com os outros


quatro?
a) trigo c) aveia e) centeio
b) d) b) feno d) arroz

34) Qual dos números a seguir não pertence a série?


26) Qual das letras a seguir não pertence a série? 2 – 3 – 6 – 7 – 8 – 14 – 15 – 30
A–D–G–I–J–N–Q–T
a) A e) J a) 2 e) 8
b) D f) N b) 3 f) 14
c) G g) Q c) 6 g) 15
d) I h) T d) 7 h) 30

27) Qual dos cinco desenhos representa a melhor 35) Qual dos cinco desenhos representa a melhor
comparação? comparação?

está para assim como está para: está para assim como está para:

a) d) a) d)

b) e) b) e)

c) c)


5DFLRFtQLR/yJLFR0DWHPiWLFR
36) Uma espaçonave recebeu três mensagens na língua 44) Se todos os Wargs são Twerps e nenhum Twerp é
estranha de um planeta distante. Os astronautas Gollum, então nenhum Gollum poderá ser Warg. Esta
estudaram as três mensagens e descobriram que " afirmativa é:
ELROS ALDARION LENDIL ” significava " PERIGO
FOGUETE EXPLOSÃO ”, que "EDAIN MINYATUR a) verdadeira
ELROS ” significava " PERIGO ESPAÇONAVE b) falsa
FOGO ” e que " ALDARION GIMILZOR GONDOR ” c) nenhuma das duas
significava " HORRÍVEL GÁS EXPLOSÃO ”. O que
significa " ELENDIL ”?
a) perigo d) nada 45) Qual dos cinco se parece menos com os outros
b) explosão e) gás quatro?
c) foguete a) cavalo d) gato
b) canguru e) burro
c) zebra
37) Qual dos cinco se parece menos com os outro
quatro?
46) Qual das figuras a seguir não pertence a série?

47) Qual dos cinco representa a melhor comparação?


Dedo está para mão assim como folha está para:
a) árvore d) ramo
38) Qual dos cinco representa a melhor comparação? b) fruto e) casca
Cinto está para fivela assim como sapato está para: c) flor
a) meia d) cadarço
b) dedo e) sola
c) pé 48) A mãe de João o mandou ao supermercado comprar
9 latas grandes de pêssego. João só consegue levar
duas latas de cada vez. Quantas viagens ao super-
39) Qual dos cinco desenhos se parece menos com os mercado João teve que fazer?
outros? a) 4 d) 5½
b) 4½ e) 6
c) 5

40) João recebeu 41 cruzeiros de troco no supermerca- 49) Qual das cinco figuras se parece menos com as
do. Sabendo-se que ele recebeu 7 moedas, uma outras quatro?
delas terá sido de:
a) 1 centavo d) 5 cruzeiros
b) 20 centavos e) 10 cruzeiros
c) 1 cruzeiro

41) Qual dos cinco desenhos se parece menos com os 50) Qual dos cinco representa a melhor comparação?
outros quatro?
Pé está para joelho assim como mão está para:
a) dedo da mão d) braço
b) dedo do pé e) perna
c) cotovelo
42) Se você reordenar as letras " M A N L A A H E ” o
resultado será o nome de um:
a) oceano d) cidade 51) Qual das cinco figuras se parece menos com as
b) país e) animal outras quatro?
c) estado

43) Qual dos cinco desenhos representa a melhor


comparação?

52) Maria foi ao mesmo tempo a décima terceira melhor


classificada e a décima terceira classificada de um
concurso. Quantos eram os concorrentes?
a) 13 d) 27
b) 25 e) 28
c) 26


5DFLRFtQLR/yJLFR0DWHPiWLFR
53) Qual dos cinco representa a melhor comparação? GABARITO COMENTADO
Água está para gelo assim como leite está para: DOS EXERCÍCIOS GERAIS PROPOSTOS:
a) mel d) café
b) queijo e) biscoito 1) COBRA. Todos os outros têm pernas, os outros são
c) mingau mamíferos.

2) ANIMAL - COELHO.
54) Qual dos números a seguir não pertence a série?
1 – 2 – 5 – 10 – 13 – 26 – 48 3) D. Os triângulos e quadrados mudam de lugar, e a
figura vertical se torna horizontal.
a) 1 e) 13
b) 2 f) 26
4) MAÇÃ. Os outros são legumes.
c) 5 g) 48
d) 10
5) B. Como esta é uma comparação de inversos, o
triângulo de linhas contínuas é o inverso do quadrado
55) Qual dos cinco se parece menos com os outros de linhas interrompidas.
quatro?
6) 16. O irmão de João tem 4 anos. Em 4 anos, o irmão
a) presunto d) carne de porco terá 8 e João terá 16, ou seja, o dobro.
b) fígado e) bife
c) badejo
7) SOBRINHO. Irmão e irmã, sobrinha e sobrinho são
antônimos.
56) Se todos os Fleeps são Sloops e todos os Sloops
são Loopies, então todos os Fleeps serão obrigato- 8) E. Todas as outras têm apenas 3 linhas. "E” tem 4
riamente Loopies. Esta afirmativa é: linhas.
a) verdadeira
b) falsa 9) ENVELOPE. O leite vai dentro do copo, a carta vai
c) Nenhuma das duas dentro do envelope.

10) E. O círculo maior tem um pequeno círculo dentro, os


57) Qual dos cinco desenhos representa a melhor outros tem figuras pequenas diferentes das figuras
comparação? maiores.

11) C. AMOR é o contrário de ROMA, o inverso de 5232


é 2325.

12) FALSA. Exemplo: Se alguns gatos são animais e


alguns animais são cachorros, então alguns gatos
serão obrigatoriamente cachorros. Evidentemente
esta alternativa não pode ser feita.
58) Qual dos cinco se parece menos com os outros
quatro?
13) D. É o único que se compõe de círculos apenas.
a) metro d) decímetro
b) quilômetro e) milímetro 14) CASA. A árvore se ergue do chão, a chaminé se
c) hectare ergue da casa.

15) TRÊS. A ordem é "menos dois, mais um, menos dois,


59) Qual das cinco figuras representa a melhor compara-
mais um, etc. ”. Três não se encaixa na seqüência.
ção?

16) SORRISO. Todos os outros são sentidos, sorriso é


uma expressão facial.

17) B. As duas primeiras figuras apontam na mesma


direção, da mesma forma que o triângulo e a figura B
apontam na mesma direção.
60) Um peixe tem 9 centímetros de cabeça. A cauda tem
o tamanho da cabeça mais a metade do tamanho do
corpo. O corpo tem o tamanho da cabeça mais a 18) D. Sem mais informações, nada se pode concluir.
cauda. Qual o comprimento do peixe? Sabemos apenas que tanto Pedro quanto Antônio
são mais baixos que João.
a) 27 cm d) 72 cm
b) 54 cm e) 81 cm 19) BOLSA. Todos os outros são peças de vestuário.
c) 63 cm
20) D. Troque letras por números, C = 3 e A = 1.

21) CIDADE. "RAPIS” = PARIS.


5DFLRFtQLR/yJLFR0DWHPiWLFR
22) B. Não se compõe de linhas retas. 43) E. A mão entra na luva, o pé entra no sapato.

23) CANHÃO. O revólver solta balas, o canhão solta 44) VERDADEIRA. Exemplo: Se todos os cachorros são
bombas. Nenhum dos outros tem qualquer relação animais e nenhum animal é planta, então nenhuma
com uma força de explosão. planta será obrigatoriamente animal. Evidentemente
que esta afirmativa pode ser feita.
24) FALSA. Exemplo: Se alguns carros são verdes e
todas as folhas são verdes, então alguns carros 45) CANGURU. Os outros andam sobre 4 patas.
serão obrigatoriamente folhas. Evidentemente, esta
afirmativa não pode ser feita.
46) D. As letras de todos os outros quadrados se movi-
mentam em sentido horário.
25) B. Os números indicam a posição das letras no alfabe-
to. "E” é a quinta letra do alfabeto, e não a sexta.
47) RAMO. Os dedos saem da mão, as folhas saem dos
ramos.
26) D.
I. A ordem é "A, pulam-se duas letras, D, pulam-se
duas letras, G”, etc. I não se encaixa na seqüência.
48) 5.9 dividido por 2 = 4,5, mas serão necessários 5
viagens, pois numa delas João levará só uma lata.
27) E. As figura geométricas se invertem, e a posição
das cruzes e setas por dentro ou por fora das figuras
se inverte também. 49) E. A única que possui um número ímpar de pontas.

28) 25%. Exemplo: Um produto de 10 cruzeiros com 50) COTOVELO. O pé se liga à perna e o joelho é uma
desconto de 20% será vendido por 8 cruzeiros. Para junta da perna. A mão se liga ao braço e o cotovelo
que ele seja vendido outra vez por 10 cruzeiros, o é uma junta do braço.
aumento terá que ser de 2 cruzeiros.2 cruzeiros são
25% de 8 cruzeiros. 51) D. Tem duas faixas pretas. As outras tem uma.

29) BRONZE. Todos os outros são metais. Bronze é uma 52) Há 12 alunos abaixo e 12 acima de Maria, que com
liga (combinação de dois metais). ela somam 25.

30) E.3 figuras se transformam em 3 figuras diferentes, 53) QUEIJO. A água se transforma em gelo, o leite em
e 5 figuras se transformam em 5 figuras diferentes. queijo.

31) FUNIL. Os outros são recipientes, os líquidos pas- 54) QUARENTA E OITO. A ordem é "vezes dois, mais
sam por dentro do funil. três, vezes dois, mais três”, etc. 48 está fora da
seqüência.
32) 23.23 – 1 = 22. A metade de 22 = 11.11 – 1 = 10. A
metade de 10 = 5, que é o número de biscoitos que 55) BADEJO. Os outros são carnes de mamíferos.
restam. Badejo é um peixe.

33) FENO. Todos os outros são grãos, cereais. 56) VERDADEIRA. Exemplo: Se todos os cachorros são
mamíferos e todos os mamíferos são animais, então
34) OITO. A ordem é "mais um, vezes dois, mais um, todos os cachorros serão obrigatoriamente animais.
vezes dois”, etc. Oito sai fora da seqüência. Evidentemente, esta afirmativa pode ser feita.

35) B. As figuras são invertidas, e um dos lados não 57) D. Os círculos se transformam em quadrados, e as
aparece na figura menor. sombras aparecem invertidas.

36) FOGUETE. Elros = Perigo, Aldarion = Explosão, 58) HECTARE. Hectare é medida de área. Todos os
portanto Elendil significa foguete. outros são medidas de distância.

37) B. Todos os objetos começam com a letra C, só faca 59) C. O Quadrado, que tem 4 lados, está dividido em 4
começa com F. partes, o triângulo, que tem 3 lados, está dividido em
3 partes.
38) CADARÇO. A fivela fecha o cinto, o cadarço fecha o
sapato. 60) 72 cm. A cabeça tem 9 cm. A cauda tem 18 + 9 = 27.
O corpo tem 9 + 18 + 9 = 36.9 + 27 + 36 = 72. Este
problema também pode ser resolvido algebrica-
39) D. Está dividido em 4 partes, as outras estão dividi- mente.
das em três partes.

40) UM CRUZEIRO. A única moeda que pode completar


os 41 cruzeiros.
Prezado(a) Candidato(a):
41) D. As outras figuras se subdividem no mesmo núme-
Toda e qualquer dúvida referente a esta matéria poderá ser
ro de triângulos que o numero de lados. esclarecida através do e-mail
VIVEIROS@APOSTILASOLUCAO.COM.BR
42) PAÍS. "MANLAAHE” = ALEMANHA.


Realidade do Distrito Federal
O Brasil era dividido em lotes; a parte litorânea era dos
portugueses, e eles começavam a explorar, e a outra dos
espanhóis. No entanto, os colonos de Portugal decidiram
ampliar suas terras e saíram a conhecer e tomar conta de uma
região maior. Aventuravam-se em expedições e bandeiras, a fim
de descobrir o que mais o território indígena tinha a oferecer.

Na região Centro-Oeste, como dito anteriormente, foram os


primeiros habitantes dessa região. Nela, havia muitas minas de
ouro que, quando descobertas pelos bandeirantes, começaram
a ser exploradas. Desse modo, deram início às primeiras vilas:
a Vila Real do Bom Jesus de Cuiabá, território atual da capital
A realidade étnica, social, histórica, do estado do Mato Grosso, Vila Boa, o atual estado de Goiás e
geográfica, cultural, política e econômica a Meya Ponte, pertence, hoje, à cidade de Pirenópolis.
do Distrito Federal e da Região
Uma das primeiras atividades econômicas da região foi a
Integrada de Desenvolvimento do
criação de gado. Ora, os fazendeiros de Minas Gerais e de São
Distrito Federal e Entorno – RIDE, Paulo também criaram grandes fazendas no território da atual
instituída pela Lei Complementar região Centro-Oeste. Os grandes coronéis se estabeleceram
federal nº 94/1998. nessa terra e, até hoje, é possível ver as imensas fazendas nas
áreas verdes de Goiás.

Como estratégia de defesa de nossas fronteiras contra as


outras nações, os habitantes da região Centro-Oeste
construíram um forte, chamado de Forte de Coimbra. Essa
Olá candidato(a). Muitos dos dados sociais, étnicos e fortaleza foi edificada onde hoje se localiza o município de
geográficos apresentados a seguir tem um período de Corumbá, no Mato Grosso do Sul. Em Goiás, também existe um
tempo maior do que gostaríamos de trabalhar. Acontece município de mesmo nome; porém, para que não haja dúvida,
que o próprio IBGE não disponibiliza dados mais o Forte Novo de Coimbra, como é chamado, está no Mato
recentes para a Região, estes apenas são disponíveis para Grosso do Sul.
cada município individual. Como o edital pede uma
análise regional, mesmo que antigos, trabalharemos com A partir da construção dele, surgiram novos aglomerados e
os últimos apresentados – que podem ser conferidos nas povoados. A população da região Centro-Oeste aumentou, na
referências -. Em caso de dúvidas, nossa equipe de medida em que novas estradas de ferro, rodovias e hidrovias
tutores está à disposição. davam acesso à região.

O povoamento da região não teria aumentado muito, se não


História regional1 fosse pela transferência da Capital Federal para, vamos dizer, o
A região Centro-Oeste faz parte das cinco subdivisões do “centro do país”. Em meados da década de 1950, atendendo a
Brasil. As outras são: região norte, região Nordeste, região pedidos da época do Brasil Império, iniciariam a construção de
sudeste e região sul. Os estados que compõem o Centro-Oeste Brasília, exatamente no estado do Goiás.
são: Distrito Federal, Mato Grosso, Mato Grosso do Sul e Goiás.
A soma das áreas dos quatro estados resulta num total de Grande parte da população migrou para a futura capital. Em
1.606.371,505 km². Essa área lhe dá o status de segunda maior sua maioria, os nordestinos vieram para trabalhar na
região do Brasil, em superfície territorial. construção do antigo sonho de Marquês de Pombal, que
desejava a capital no interior do país, e para José Bonifácio, o
A segunda maior região também é uma das menos Patriarca da Independência e idealizador do nome “Brasília”.
populosas. A densidade populacional dela, segundo o Instituto O presidente do Brasil, na época, Juscelino Kubitschek, tinha o
Brasileiro de Geografia e Estatística (IBGE), é de 8,26 projeto chamado de “Cinquenta anos em cinco”, em que
habitantes/km². E mais, de acordo com as estatísticas do órgão, construiria a moderna capital, em pouco tempo – e fez jus ao
a região Centro-Oeste possui cerca de 14 milhões de habitantes. nome do projeto. Ela foi inaugurada em 21 de abril de 1960 e
Se fizermos um paralelo com a região Sudeste, que tem sua as obras tinham começado em 1955.
população contabilizada em cerca de 80 milhões de habitantes, Além do povoamento, a nova capital pode contribuir para o
percebemos o porquê da região Centro-Oeste estar nessa desenvolvimento socioeconômico da região Centro-Oeste, que
posição, com menos população. até então era pouco povoada e com baixas taxas de
crescimento. Um exemplo disso é o território que hoje pertence
No início do século XVI, o Brasil recebeu os europeus, ao estado do Tocantins – a antiga parte norte do Goiás. Ele era
oriundos de Portugal. Antes, no país, só habitavam os indígenas de responsabilidade também do governo, fazia parte do
e as terras não haviam sido tocadas pelo o homem branco. O território goiano, mas sempre houve maior foco na parte sul.
território onde se localiza a região Centro-Oeste, pelo menos,
ainda não. No período da colonização, bem no começo, os
lusitanos só podiam se alojar nas terras que pertenciam ao
litoral brasileiro.

1
REGIÃO CENTRO-OESTE. Disponível em: < http://regiao-centro-oeste.info/>

1
Realidade do Distrito Federal
História de Brasília2 Asa Norte. A outra linha, que representava o Eixo
Monumental, abrigaria os prédios públicos e o palácio do
As primeiras ideias Governo Federal no lado leste; a Rodoviária e a Torre de TV no
Na época do Brasil Colônia, já havia a ideia de levar a capital centro, e os prédios do governo local no lado oeste.
do país para a região central, para evitar ataques pelo mar. Mas
a possibilidade só começou a ganhar força no Império. Em Lúcio Costa não só desenhou os traços que definiram a
1823, José Bonifácio de Andrada e Silva, conhecido como capital do país, mas também previu como seria a alma de
“Patriarca da Independência”, reforçou a proposta de levar a Brasília, como afirmou no livro “Memória descritiva do Plano
sede das decisões brasileiras para o interior do território Piloto”:
e sugeriu pela primeira vez o nome “Brasília“.
Cidade planejada para o trabalho ordenado e eficiente, mas
Em 1883, o sacerdote católico italiano Dom Bosco sonhou ao mesmo tempo cidade viva e aprazível, própria ao devaneio
que visitava a América do Sul e, em seu relato, publicado no e à especulação intelectual, capaz de tornar-se, com o tempo,
livro “Memórias Biográficas de São João Bosco”, relatou o que além de centro de governo e administração, num foco de
viu: cultura dos mais lúcidos e sensíveis do país.

Entre os graus 15 e 20 havia uma enseada bastante longa e Com o projeto urbanístico aprovado, Juscelino
bastante larga, que partia de um ponto onde se formava um escolheu Oscar Niemeyer como o arquiteto responsável pela
lago. Disse, então, uma voz repetidamente: construção dos monumentos. O carioca foi autor das principais
– Quando se vierem a escavar as minas escondidas no meio estruturas da cidade: o Congresso Nacional, os Palácios da
destes montes, aparecerá aqui a terra prometida, de onde Alvorada e do Planalto, o Supremo Tribunal Federal e
jorrará leite e mel. Será uma riqueza inconcebível. a Catedral de Brasília. Além da dupla Oscar e Lúcio,
completavam o time que fez desta cidade um museu a céu
A visão acabou sendo interpretada como uma premonição aberto figuras inspiradas como Burle Marx, com jardins e
do local em que deveria ser construída a nova capital do Brasil. praças, e Athos Bulcão, com os painéis de azulejos que são
Mas ela começou a ser viabilizada somente em 1891, quando a marca registrada da capital.
determinação de sua área foi incluída na primeira Constituição
da República brasileira. No ano seguinte, um grupo de O nascimento
cientistas foi enviado para explorar o Planalto Central e
demarcar a área. Chefiada por Louis Ferdinand Cruls, a Em 21 de abril de 1960, Brasília nascia para o mundo e para
expedição ficou conhecida como “Missão Cruls”. Médicos, a sua gente. Com os projetos urbanístico de Lúcio Costa e o
geólogos e botânicos compunham a equipe que fez um arquitetônico de Oscar Niemeyer, surgia uma cidade sob formas
levantamento sobre a topografia, o clima, a geologia, a flora, a inovadoras, diferente de tudo já feito até então. A data de seu
fauna e os recursos materiais da região. A área ficou conhecida nascimento, não foi coincidência: marcava o dia da morte de
como Quadrilátero Cruls, a primeira versão do “quadradinho”, Tiradentes, um dos líderes mineiros que defendeu a
como todo brasiliense chama o mapa da cidade. independência do Brasil no século XVIII. O simbolismo ajudou
a fortalecer em Brasília o ideal de liberdade de um povo e a
A pedra fundamental do novo centro do poder brasileiro foi coragem de uma nação, associando a inauguração à ideia de
lançada em 1922, no centenário da Independência, próximo a independência e rendendo homenagem aos inconfidentes que
Planaltina, atual região administrativa do DF. haviam sonhado com um Brasil livre.

Em 1956, com nova demarcação da futura capital, o então Conforme a construção de Brasília seguia em frente, foram
presidente da República, Juscelino Kubitschek, deu início de surgindo pequenos acampamentos ao redor do Plano Piloto
fato à realização do projeto que durou séculos. Na mesma área para abrigar os trabalhadores que vieram para construir a nova
das coordenadas que Dom Bosco apontou e às margens do Lago capital. O primeiro acampamento foi chamado de Cidade Livre,
Paranoá, Brasília começou a ser erguida. que hoje é o Núcleo Bandeirante. Os demais agrupamentos
mais tarde tornaram-se inicialmente as cidades satélites que
O conceito agora são as 31 regiões administrativas que compõem o Distrito
Para organizar a logística da obra, foi criada a Companhia Federal.
Urbanizadora da Nova Capital (Novacap), que lançou no
mesmo ano o “Concurso Nacional do Plano Piloto da Nova A consagração
Capital do Brasil” com o objetivo de selecionar projetos Não são só os monumentos que fazem visitantes e turistas
urbanísticos para a construção da cidade. se renderam à grandeza da capital. Graças ao território plano e
à ausência de grandes construções verticais, o céu de
Dentre dezenas de propostas, a vencedora, do arquiteto e Brasília acabou conhecido como um dos mais bonitos do país,
urbanista Lúcio Costa, foi escolhida justamente pela que, para muitos, praticamente substitui o mar ao emoldurar as
simplicidade: a ideia, entregue em uma folha branca e construções de traços modernos e os largos espaços verdes que
desenhada a lápis, partiu do traçado de dois eixos cruzando-se completam a paisagem.
em ângulo reto, como o sinal da cruz.
Por conta desse conjunto de beleza e da importância
Uma dessas linhas, o Eixo Rodoviário, tinha o traço arquitetônica, Brasília recebeu em 1987 o título de Patrimônio
levemente inclinado, o que dava à cruz a forma de um avião. Cultural da Humanidade, concedido pela Unesco. Foi o
Ele seria a via que leva às áreas residenciais – hoje, Asa Sul e primeiro bem cultural contemporâneo a entrar nessa lista,

2
GOVERNO DE BRASÍLIA. História. Governo de Brasília. Disponível em: <
http://www.brasilia.df.gov.br/index.php/2015/10/21/historia/>

2
Realidade do Distrito Federal
figurando no mesmo patamar de importância das Pirâmides do Distrito Federal, buscando interesses comuns nos respectivos
Egito, a Grande Muralha da China, a Acrópole de Atenas, o setores da administração pública mencionadas anteriormente.
Centro Histórico de Roma e o Palácio de Versalhes.
RIDE-DF4
Brasília é multicultural
A forma como Brasília foi povoada tornou-a plural, A Região Integrada de Desenvolvimento do Distrito Federal
miscigenada e sincrética, representando a identidade de todo o e Entorno é uma região integrada de desenvolvimento
Brasil. Na busca por dias e futuro melhores, milhares de econômico, criada pela Lei Complementar nº 94, de 19 de
brasileiros de diversos cantos do país, em especial do Nordeste fevereiro de 1998 e regulamentada pelo Decreto nº 2.710, de 4
e de Minas, vieram para construir a capital e buscar uma vida de agosto de 1998, alterado pelos Decreto nº 3.445, de 4 de
nova. Eles ficaram conhecidos como candangos. Os pioneiros, maio de 2000 e Decreto nº 4.700, de 20 de maio de 2003.
que fixaram moradia na cidade entre 1960 e 1965, ainda
guardam histórias e casos daquela época. O Decreto nº 7.469, de 5 de maio de 2011 revogou os
anteriores e deu novas interpretações legais à RIDE do Distrito
Essa mistura de tanta gente diferente fez da nossa cidade Federal e Entorno.
um rico caldeirão de sotaques, sons e cores. Nossas comidas têm
todos os sabores brasileiros, nossas feiras são coloridas e É constituída pelo Distrito Federal, alguns municípios de
sortidas e aqui tem cultura para todos os gostos. E tudo isso Goiás e de Minas Gerais. Ocupa uma região de 55.434,99
espalhado por todas as regiões administrativas, que têm quilômetros quadrados, sendo pouco menor que a Croácia e sua
características e histórias próprias, cheias de surpresas. população é de aproximadamente 4 milhões de habitantes (tem
o tamanho aproximado do estado da PB). Apenas 2 municípios
O que são RIDEs?3 separam a RIDE de ser a primeira englobando área de mais da
metade das regiões do país (o município do extremo oeste
Criada pela Lei Federal 11.445/2007 a RIDE (Região mineiro e do extremo sudoeste nordestino).
Integrada de Desenvolvimento) é um conceito para áreas de
conurbação metropolitana de municípios distintos que No centro do território está a área mais densa, composta
compartilham espaços em regiões de Estados da Federação pelo Distrito Federal, detentor de 69% da população com os
diferentes. seus municípios limítrofes. Há uma grande área conurbada na
direção sul BR-040, incluindo-se nessa região os municípios de
Segundo os termos da Lei Federal, o Brasil atualmente Valparaíso de Goiás, Cidade Ocidental, Novo Gama e Luziânia
possui três RIDEs, cada uma criada por Lei complementar que representam 12,3% da população da RIDE do Distrito
específica e regulamentada por decretos distintos. Elas estão Federal e Entorno. Outros municípios que podem-se também
situadas nas regiões conurbadas do Distrito Federal e entorno considerar como área densamente povoada são: Águas Lindas
(Lei 94/98), Grande Teresina (Lei 112/91) e em de Goiás (margens da BR-070), Formosa (margens da BR-020),
Petrolina/Juazeiro (Lei 113/01). Planaltina (BR-010), Santo Antônio do Descoberto (BR-060).
Esses municípios somam uma população de aproximadamente
O grande desafio dessas regiões é buscar ações de 3,7 milhões de pessoas.
integração para que objetivos comuns sejam alcançados em
áreas compartilhadas, atenuando o conflito de interesses entre Consideram-se de interesse da RIDE os serviços públicos
as administrações Estaduais, do Distrito Federal, Municipais e comuns ao Distrito Federal, Estados de Goiás, e aos Municípios
as comunidades locais. que a integram, relacionados com as seguintes áreas:

Para motivar a superação de eventuais conflitos, a - infraestrutura;


legislação criadora dessas RIDEs busca dar preferência na - geração de empregos e capacitação profissional;
obtenção de recursos federais e estaduais para os municípios - saneamento básico, em especial o abastecimento de água,
dessas áreas conurbadas que trabalhem de forma integrada no a coleta e o tratamento de esgoto e o serviço de limpeza pública;
planejamento e execução de serviços comuns nos setores de - uso, parcelamento e ocupação do solo;
desenvolvimento social, saneamento básico, transporte, uso do - transportes e sistema viário;
solo, aproveitamento de recursos hídricos e controle da - proteção ao meio ambiente e controle da poluição
poluição ambiental. ambiental;
- aproveitamento de recursos hídricos e minerais;
Quando o planejamento articulado e integrado acontece o - saúde e assistência social;
resultado tem maior possibilidade de ser bom para todos. O - educação e cultura;
Governo Federal poupa recursos financeiros e os municípios das - produção agropecuária e abastecimento alimentar;
RIDEs tem mais facilidade de acesso aos meios de - habitação popular;
financiamento nesses setores da administração pública. - serviços de telecomunicação;
- turismo; e
Para isso, a RIDE tem um Conselho Administrativo da - segurança pública.
Região Integrada de Desenvolvimento (COARIDE). Seus
membros são responsáveis por articular ações administrativas Municípios pertencentes a RIDE
entre o Governo Federal, os Estaduais, Municipais e o do Área de abrangência

3 4
RIDESABE. O que são RIDEs? RIDEsab. Disponível em: < CORSAP. RIDE/DF. Consórcio Público de Manejo dos Resíduos Sólidos
http://ridesab.com.br/o-que-sao-rides/> e das Águas Pluviais da região integrada do Distrito Federal e
Goiás. Disponível em: < http://corsapdfgo.eco.br/ridedf/>

3
Realidade do Distrito Federal
Municípios que compõem a Região Integrada de O Índice de Desenvolvimento Humano Municipal
Desenvolvimento do Distrito Federal e Entorno (RIDE): (IDHM) na Ride-DF5
Distrito Federal.
Em 2000, a Ride-DF apresentava Índice de
Municípios do Estado de Goiás: Abadiânia, Água Fria de Desenvolvimento Humano Municipal (IDHM) igual a 0,680,
Goiás, Águas Lindas de Goiás, Alexânia, Cabeceiras, Cidade situando-se na faixa de Médio Desenvolvimento Humano. Já
Ocidental, Cocalzinho de Goiás, Corumbá de Goiás, Cristalina, em 2010, esta apresentava IDHM de 0,792, passando para a
Formosa, Luziânia, Mimoso de Goiás, Novo Gama, Padre faixa de Alto Desenvolvimento Humano.
Bernardo, Pirenópolis, Planaltina, Santo Antônio do
Descoberto, Valparaíso de Goiás e Vila Boa. O IDHM Educação, em 2000, era 0,516, passando, em 2010,
para 0,701. O IDHM Longevidade era de 0,791 e, em 2010,
Municípios do Estado de Minas Gerais: Cabeceira Grande, correspondeu a 0,857. Já o IDHM Renda era de 0,769, tendo
Buritis e Unaí. passado para 0,826.

Dados socioeconômicos dos Municípios que Entre 2000 e 2010, a dimensão que mais evoluiu, em termos
compõem a RIDE absolutos, foi a dimensão Educação, que registrou um aumento
de 0,185.
Área, População, PIB e IDH:
Evolução do IDHM na Ride-DF

Em 2000, 10% das Unidades de Desenvolvimento Humano


(UDH) da Ride-DF encontravam-se na faixa de Muito Alto
Desenvolvimento Humano, enquanto 20% apresentavam Alto
Desenvolvimento Humano. Em 2010, essas proporções
correspondiam respectivamente, a 35% e 39%. No mesmo
período, o percentual de UDHs nas faixas de Baixo e Muito
Baixo Desenvolvimento Humano passou, respectivamente, de
29% e 9%, para 0% em ambos os casos, não havendo UDHs
nessas faixas em 2010.

Economia6
Em 2008 a RIDE do Distrito Federal e Entorno teve
um produto interno bruto de R$ 125.7 Bilhões IBGE/2008.
Pode-se dizer que é a terceira região mais rica do Brasil, ainda
que a região não siga os mesmo parâmetros técnicos de
organização espacial de outras regiões do país. Não é
uma região metropolitana, nem mesorregião, tão pouco pode
Fonte: IBGE e PNUD ser classificada como uma microrregião, quando na verdade é
composta por parte da Microrregião do Entorno do Distrito
IDHM, População Residente Alfabetizada, Índice de Gini e Federal no estado de Goiás, excluído o município de Vila
Incidência da Pobreza: Propício, três municípios da Microrregião de Unaí no estado
de Minas Gerais e do próprio Distrito Federal. Somente o
Distrito Federal, independente dos municípios que compõem
essa RIDE, só tem PIB menor que a Mesorregião Metropolitana
de São Paulo e a Mesorregião Metropolitana do Rio de Janeiro.

Pelo exposto anteriormente percebe-se que é uma região


que tem a circulação financeira muito concentrada em seu
núcleo, acentuando a dependência para a geração de emprego
e renda. Existem grandes movimentos pendulares diários de
alguns municípios fronteiriços de Goiás, Minas Gerais e de
algumas regiões administrativas do Distrito Federal para à
região administrativa de Brasília, que concentra a maioria dos
serviços e postos de trabalho dessa RIDE.

O Distrito Federal concentra 93,58% do PIB, sobretudo no


setor de serviços, mas tem participação expressiva também nos
demais setores, apresentando o 10° maior PIB industrial e o 14°
maior PIB agropecuário entre os municípios brasileiros segundo
dados do IBGE/2008.
Fonte: Fonte – IBGE; IBGE, Censo Demográfico 2000 e Pesquisa de
Orçamentos Familiares - POF 2002/2003.

5 6
REGIÃO INTEGRADADE DESENVOLVIMENTO DO DISTRITO FEDERAL E Aspectos econômicos. Disponível em: <
ENTORNO. Disponível em: < https://pt.wikipedia.org/wiki/Regi%C3%A3o_Integrada_de_Desenvolvimento_do_
http://www.ipea.gov.br/agencia/images/stories/PDFs/livros/livros/141125_atlas Distrito_Federal_e_Entorno#Economia>
_df>

4
Realidade do Distrito Federal
Agropecuária (B) fazer do Governo Federal o principal responsável pela
Unaí teve em 2008 um PIB agropecuário de R$ 512 geração de empregos nos municípios goianos e mineiros
milhões IBGE/2008, sendo o maior da RIDE do Distrito Federal limítrofes ao Distrito Federal.
e Entorno e o 6ª maior do (C) assegurar que o trabalho de proteção ao meio ambiente
país. Brasília, Cristalina, Buritis e Luziânia também aparecem e de controle da poluição ambiental seja arcado integralmente
no ranking nacional entre os 100 melhores colocados, pelos municípios integrantes da RIDE.
ocupando respectivamente a 14ª, 16ª, 68ª e 85ª posições, (D) transferir à União a responsabilidade direta pela
despontando a região como uma das mais importantes para a operação do sistema de transporte público da região, incluindo
agropecuária no Brasil. a fixação das tarifas a serem pagas pelos usuários.
(E) articular, em termos de serviços públicos comuns, a ação
Indústria administrativa da União, dos estados de Goiás e Minas Gerais e
A grande força industrial da região é a construção civil. do Distrito Federal na região.
No Distrito Federal esse nicho corresponde a 56,6% do
setor IBGE/2008 e movimentou em 2008 R$ 3.7 Bilhões. 02. (Papiloscopista Policial – PC-DF – FUNIVERSA-
Luziânia é o único município do entorno a ter um parque 2015) A região integrada de desenvolvimento do Distrito
industrial de destaque, sobretudo devido à indústria de Federal (DF) e entorno (RIDE/DF) é uma região integrada
alimentos. Em 2008 teve movimentação industrial de de desenvolvimento econômico criada pela Lei
aproximadamente R$ 600 milhões IBGE/2008. Complementar n.º 94, de 19 de fevereiro de 1998, e
regulamentada pelo Decreto n.º 7.469, de 4 de maio de
Serviços 2011, para efeitos de articulação da ação administrativa da
A Administração Pública tem grande peso na formação União, dos estados de Goiás e Minas Gerais e do Distrito
do Produto Interno Bruto do Distrito Federal e por vezes Federal.
promove um falso julgamento de que toda a economia do
Distrito Federal se resume a esse nicho. Na verdade, assim como Internet: < www.sudeco.gov.br>.
a economia fluminense tem grande dependência da indústria Acesso em 18/4/2015.
extrativista, por exemplo, para dinamizar outros tantos setores
de sua economia, no Distrito Federal o setor público funciona (A) Na periferia goiana, a iniciativa estatal repetiu, a partir
como combustível, mas outros serviços se desenvolveram de meados da década de 1970, o polinucleamento urbano que
fortemente, destacando-se serviços financeiros, serviços de se verifica no interior do Distrito Federal, com o surgimento de
informação, atividades imobiliárias, serviços prestados a novos núcleos de povoamento.
empresas e comércio de uma forma geral IBGE/2008. (B) Urge buscar soluções para o uso compartilhado e
racional da água pela população que compõe a RIDE, com
Comércio Exterior especial atenção às represas que cumprem o papel de divisoras
Luziânia destaca-se como maior Exportador da RIDE do entre o Distrito Federal e Goiás, como a Barragem do
Distrito Federal e Entorno, ocupando em novembro de 2010 a Descoberto, a de Corumbá IV e a de Serra da Mesa.
102ª posição no Ranking Nacional de acordo com publicação (C) 19 municípios de Goiás, 2 de Minas Gerais e 33 do
mensal do Ministério do Desenvolvimento, Indústria e Distrito Federal integram a RIDE.
Comércio Exterior, totalizando R$ 348 milhões de reais nos (D) Não sendo uma unidade limítrofe com Minas Gerais, o
primeiros 11 meses do ano. O principal bem exportado Distrito Federal mantém com aquele estado relações
são: soja e seus subprodutos e milho e seus subprodutos. econômicas bem menos intensas que as que estabelece com o
estado de Goiás.
Investimentos nos integrantes da RIDE do Distrito (E) A cada ano, constata-se que a população do Distrito
Federal e Entorno Federal teve um incremento superior à média nacional,
Alguns investimentos de grande envergadura estão situação que também se verifica com o contingente
extrapolando as fronteiras do Distrito Federal e se instalando populacional de diversos municípios do entorno.
nos municípios vizinhos. É o caso do Alphaville Brasília
Residencial 1 e do Damha Residencial Brasília 1 que se situam 03. (Perito Criminal – PC-DF – FUNIVERSA)
no município de Cidade Ocidental, no estado de Goiás, além do Considerando aspectos geográficos, sociais, econômicos,
Outlet Premium Brasília, que foi construído no município políticos e culturais referentes ao Distrito Federal (DF) e à
de Alexânia, no estado de Goiás. Rede Integrada de Desenvolvimento do Entorno (RIDE),
assinale a alternativa correta.
Questões7
01 – (Agente de Trânsito – DETRAN-DF – (A) Característico da construção de Brasília, o
FUNIVERSA) A criação da Região Integrada de planejamento urbano foi reproduzido na ocupação das cidades
Desenvolvimento do Entorno (RIDE) subordinou-se, entre que compõem o Distrito Federal.
outros, ao objetivo de (B) Vitorioso em vários pontos, o projeto de construção da
nova capital no Planalto Central falhou no objetivo de
(A) promover o desenvolvimento econômico da porção do interiorizar o desenvolvimento nacional.
território goiano que não se integrou ao estado de Tocantins,
criado pela Constituição de 1988.

7
QCONCURSOS. Disponível em: < https://www.qconcursos.com/questoes-de- ssui_comentarios_gerais=&possui_gabarito_comentado_texto_e_video=&product_i
concursos/questoes/search?ano_publicacao=&area=&assunto=&caderno_id=&ca d=1&prova=&q=RIDE-
rgo=&data=&data_comentario_texto=&disciplina=&escolaridade=&esfera=&ins DF&resolvidas=&resolvidas_certas=&resolvidas_erradas=&sem_anuladas=&sem_
tituto=&migalha=&minissimulado_id=&modalidade=&nao_resolvidas=&nivel_d anuladas_impressao=&sem_desatualizadas=&sem_desatualizadas_impressao=&s
ificuldade=&order=questao_aplicada_em+desc&organizadora=&page=1&per_pa em_dos_meus_cadernos=&todas=on&url_solr=slave&user_id=4727204&utf8=%
ge=5&periodo_ate=&periodo_de=&possui_anotacoes=&possui_comentarios=&po E2%9C%93>

5
Realidade do Distrito Federal
(C) Com graves problemas estruturais, como transporte e
segurança, o entorno do DF teve sua população bastante
ampliada nas últimas décadas.
(D) Por sua especificidade, a RIDE omitiu a crucial questão
da segurança pública na configuração das áreas de interesse
para sua atuação.
(E) A atração exercida pelo agronegócio, particularmente
em Mato Grosso e Mato Grosso do Sul, fez cessar o fluxo
migratório para o entorno do DF.

04 (Perito Criminal. PC-DF – IADES – 2016). A


Região Integrada de Desenvolvimento do Distrito Federal e
Entorno (RIDE/DF) foi criada pela Lei Complementar nº
94/1998 e regulamentada pelo Decreto nº 2.710/1998,
alterado pelo Decreto nº 3.445/2000.

Disponível em:
<http://www.mi.gov.br/regioes_integradas_df_rides>.
Acesso em: 5 abr. 2016, com adaptações.

Em relação à RIDE/DF, assinale a alternativa correta.

(A) Além do Distrito Federal, participam da RIDE/DF os


estados da Bahia, de Goiás e de Minas Gerais.
(B) O objetivo comum dos municípios participantes da
RIDE/DF é a construção de infraestruturas de interesse comum,
visando a reduzir as desigualdades sociais.
(C) Segundo os preceitos que orientam a RIDE/DF, os
municípios limítrofes devem promover política de saúde
pública independentemente dos respectivos vizinhos.
(D) Dentro da RIDE/DF, o município só poderá utilizar-se
dos recursos hídricos cujas nascentes estejam dentro do próprio
território.
(E) A equidade econômica entre os diversos municípios
goianos e mineiros e o Distrito Federal facilitou o processo de
integração da RIDE/DF.

Respostas

01. E / 02. E / 03.C / 04.B

6
&kPDUD/HJLVODWLYDGR
  'LVWULWR)HGHUDO 'LU&RQVWLWXFLRQDO 

',5(,72
&2167,78&,21$/
'(/(,25*Ç1,&$'2',675,72)('(5$/
('(352&(662/(*,6/$7,92

,9 RV
RVYDORUHV
YDORUHVVRFLDLV
VRFLDLVGRWUDEDOKRHGD
GRWUDEDOKRHGDOLYUH
OLYUHLQLFLDWL
YD
3ULQFtSLRVIXQGDPHQWDLV
GD
1RWUDEDOKRVHWHPRSRQWRGHSDUWLGDjPHOKRUDGD
&RQVWLWXLomR)HGHUDOGH TXDOLGDGH GH YLGD DR DOFDQFH GR SURJUHVVR H SD] QD
FRQYLYrQFLDVRFLDO$OLYUHLQLFLDWLYDpXPWUDoRPDUFDQWH
GHQRVVDGHPRFUDFLDTXHPPDLVWUDEDOKDHWRPDLQLFLDWL
$UWž $ 5HS~EOLFD
5HS~EOLFD )HGHUDWLYDGR%UDVLO
)HGHUDWLYDGR%UDVLOIRUPDGDSHOD
LYDGR%UDVLO YDVFRQTXLVWDXPDYLGDPHOKRU
XQLmRLQGLVVRO~YHOGRV(VWDGRVH0XQLFtSLRVHGR
'LVWULWR)HGHUDOFR
FRQVWLWXLVHHP(VWDGR'HPRFUi
FRQVWLWXLVHHP(VWDGR'HPRFUi 9 RSOXUDOLVPRSROtWLFR
WLFRGH'LUHLWRHWHPFRPRIXQGDPHQWRV
WLFRGH'LUHLWRHWHPFRPRIXQGDPHQWRV
6HXP(VWDGRTXHUVHDILUPDUGHPRFUiWLFRHGH'LUHL
6HU 5HS~EOLFD VLJQLILFD VHU D QDomR JRYHUQDGD SRU WRGHYHSURWHJHUDOLEHUGDGHGHFRQYLFomR&RPRHVVDVH
SHVVRDVHVFROKLGDVQRVHLRGRSUySULRSRYRVHPYRFDomR H[HUFLWDDWUDYpVGRVSDUWLGRVSROtWLFRVQmRVHSRGHOLPLWDU
KHUHGLWiULD FRPRQDVPRQDUTXLDV )HGHUDWLYDpDIRUPD RQ~PHURGHSDUWLGRV
GR (VWDGR RX VHMD IUDFLRQDGR HP SURYtQFLDV RV
(VWDGRVPHPEURV HPXQLFtSLRVFRPDXWRQRPLDDGPLQLV †~QLFR 7RGRRSRGHU
7RGRRSRGHUHPDQD
HPDQDGR
GRSRYRTXHRH[HUFHSRU
WUDWLYD H ILQDQFHLUD 8P JRYHUQR FHQWUDO GD 8QLmR PHLR GH
PHLR GH UHSUHVHQWDQWHV
UHSUHVHQWDQWHV HOHLWRV
HOHLWRV RX
RX GLUHWDPHQWH
DGPLQLVWUDRVLQWHUHVVHVGDWRWDOLGDGH(VWDGR'HPRFUiWL QRVWHUPRVGHVWD&RQVWLWXLomR
FRGH'LUHLWRVLQWHWL]DDVXEPLVVmRGR%UDVLOjYRQWDGH
VREHUDQD GR SRYR VHJXQGR OHLV HGLWDGDV SHOR SUySULR $TXLQR%UDVLORH[HUFtFLRpLQGLUHWRRXVHMDHVFROKH
SRYRQRH[HUFtFLRGHVVDVXDVREHUDQLD PRV YRWDPRV QRVVRVUHSUHVHQWDQWHVTXHSDUWLFLSDUmRGD
DGPLQLVWUDomR GRV LQWHUHVVHV QDFLRQDLV 0DV WDPEpP
, DVREHUDQLD
DVREHUDQLD SRGHUHPRVSDUWLFLSDUGLUHWDPHQWHD&RQVWLWXLomRSUHYr
SRUH[HPSORSOHELVFLWRSDUDTXHDSURYHPRVRXUHMHLWH
2%UDVLOGHYHUiVHPSUHREVHUYDUVXDPi[LPDLQGHSHQ PRVFHUWRVWHPDV
GrQFLDQmRSHUPLWLQGRTXH3DtVHVTXDLVTXHULQWHUILUDPQD
DGPLQLVWUDomRGRVLQWHUHVVHVGHQRVVRSRYR $UWž 6mR
6mR3RGHUHV
3RGHUHVGD
GD8QLmR
8QLmRLQGHSHQGHQWHVHKDUP{QL
LQGHSHQGHQWHVHKDUP{QL
FRV HQWUH VL R /HJLVODWLYR R ([HFXWLYR H R
,, DFLGDGDQLD -XGLFLiULR

$HIHWLYDSDUWLFLSDomRQDJHVWmRGRVQHJyFLRVHLQWHUHV 2VSRGHUHVGR(VWDGRQDPRGHUQLGDGHVmRWULSDUWLGRV
VHV VRFLDLV VHUi GHIHULGD D WRGRV RV FLGDGmRV RX VHMD OLomR FRQVDJUDGD FRP 0RQWHVTXLHX  DR /HJLVODWLYR
jTXHOHVTXH QRVWHUPRVGDOHL WHQKDPFDSDFLGDGHSDUD -XGLFLiULRMXOJDURVFDVRVFRQFUH
FXPSUHID]HUDVOHLVDR-XGLFLiULR
-XGLFLiULR
DVVXPLUVXDVREULJDo}HVFtYLFDV SROtWLFDVHVRFLDLV  ([HFXWLYRID]HUFXPSULUWDLV
WRVVREUHVXDDSOLFDomRHDR([HFXWLYR
([HFXWLYR
OHLVHWRPDUDVLQLFLDWLYDVSDUDVDWLVID]HURVLQWHUHVVHVGD
,,, DGLJQLGDGHGDSHVVRDKXPDQD FROHWLYLGDGH-iDKDUPRQLD
KDUPRQLD
KDUPRQLDHQWUH
HQWUHRV
RVSRGHUHVFDUDFWHUL]D
SRGHUHV
VHSHODFRQYLYrQFLDUHVSHLWRVDHPTXHP~WXDHUHFLSUR
1HQKXPDQDomRFRQYLYHUiHPSD]SUyVSHUDHRUGHLUD FDPHQWH FDGD GRV SRGHUHV REVHUYD DV SUHUURJDWLYDV H
PHQWHVHQmRHVWLYHUDWHQWDjGLJQLGDGHGHVHXVHPHOKDQ IDFXOGDGHVFRQIHULGDVSHODRUGHPMXUtGLFDDVLFRPRDRV
WH7XGRTXDQWRRIHQGHUjGLJQLGDGHGRVHUKXPDQRVHUi GHPDLV SRGHUHV ³ ]HODQGR SDUD TXH XP QmR LQYDGD D
FRQVLGHUDGRLQFRQVWLWXFLRQDO HVIHUDGHFRPSHWrQFLDVHDWULEXLo}HVGRRXWUR
 'LU&RQVWLWXFLRQDO &kPDUD/HJLVODWLYDGR'LVWULWR)HGHUDO


$UWž &RQVWLWXHPREMHWLYRV
REMHWLYRV
REMHWLYRVIXQGDPHQWDLVGD5HS~EOLFD
IXQGDPHQWDLV ; FRQFHVVmRGHDVLORSROtWLFR
)HGHUDWLYDGR%UDVLO
$QRWHVH HQWUHWDQWR TXH D FRQFHVVmR GH DVLOR Vy VH
, FRQVWUXLUXPDVRFLHGDGHOLYUHMXVWDHVROLGiULD
FRQVWUXLUXPDVRFLHGDGHOLYUHMXVWDHVROLGiULD GDUiTXDQGRDSHUVHJXLomRIRUSROtWLFDQmRVHSURWHJHUmR
FULPLQRVRVFRPXQV DVVDVVLQRVODGU}HVHWF DLQGDTXH
,, JDUDQWLURGHVHQYROYLPHQWRQDFLRQDO SHUVHJXLGRVWDPEpPSRUPRWLYRVSROtWLFRV

,,, HUUDGLFDU
HUUDGLFDUDSREUH]DHDPDUJLQDOL]DomRHUHGX
SREUH]DHDPDUJLQDOL]DomRHUHGX †~QLFR $5HS~EOLFD
5HS~EOLFD)HGHUDWLYD
)HGHUDWLYDGR
GR%UDVLO
%UDVLOEXVFDUiDLQWH
]LUDVGHVLJXDOGDGHVVRFLDLVHUHJLRQDLV JUDomR
JUDomRHFRQ{PLFD
HFRQ{PLFDSROtWLFD
SROtWLFDVRFLDO
VRFLDOHFXOWXUDO
FXOWXUDOGRV
SRYRV
SRYRVGD
GD$PpULFD
$PpULFD/DWLQD YLVDQGRjIRUPDomR
/DWLQDYLVDQGR IRUPDomRGH
,9 SURPRYHUR
SURPRYHUREHP
EHPGH
GHWRGRV
WRGRVVHP
VHPSUHFRQFHLWRVGH XPDFRPXQLGDGHODWLQRDPHULFDQDGHQDo}HV
RULJHP
RULJHPUDoD
UDoDVH[R
VH[RFRU
FRULGDGH
LGDGHHTXDLVTXHU
TXDLVTXHURXWUDV
IRUPDVGHGLVFULPLQDomR

$UWž $UHS~EOLFD)HGHUDWLYDGR%UDVLOUHJHVH
UHJHVH
UHJHVHQDV
QDVVXDV
UHODo}HVLQWHUQDFLRQDLVSHORVVHJXLQWHVSULQFtSLRV
UHODo}HVLQWHUQDFLRQDLV  'LUHLWRVH*DUDQWLDV)XQGDPHQWDLV

, LQGHSHQGrQFLDQDFLRQDO GLUHLWRVHGHYHUHVLQGLYLGXDLVHFROHWLYRV

6RPRVVREHUDQRV1mRSRGHPRVYLQFXODUQRVVRSDtVD
TXDOTXHU RXWUR SDtV RX TXDOTXHU WLSR GH LQWHUHVVH RX 2 RUGHQDPHQWR MXUtGLFRFRQVWLWXFLRQDO EUDVLOHLUR
FRQGLomR FRQVDJUD D LQYLRODELOLGDGH GH FLQFR
FLQFR GLUHLWRV IXQGDPHQ
WDLV GLUHLWRjYLGD
WDLV YLGD GLUHLWRjOLEHUGDGH
YLGD OLEHUGDGH GLUHLWRj
OLEHUGDGH
,, SUHYDOrQFLDGRVGLUHLWRVKXPDQRV LJXDOGDGH GLUHLWRjV
LJXDOGDGH VHJXUDQoDH GLUHLWRjS
HJXUDQoD SURSULH
GDGH
GDGH
$OLEHUGDGHDYLGDDLQWHJULGDGHItVLFDGDVSHVVRDVQmR
SRGHUmR VHU VDFULILFDGDV SRU QDGD QHP SDUD DWHQGHU D $FUHVoDVH DLQGD TXH D 'HFODUDomR 8QLYHUVDO GRV
TXDOTXHUWLSRGHLQWHUHVVHDFRUGRFRQYHQomRRXQHJyFLR
'LUHLWRV+XPDQRVµ 218 LPS}HRUHFRQKHFLPHQWR
GH GLUHLWRV IXQGDPHQWDLV GD SHVVRD KXPDQD D  R GR
,,, DXWRGHWHUPLQDomRGRVSRYRV
GLUHLWRjYLGD LWHQV,,,H9,  E GLUHLWRjOLEHUGDGH LWHQV
,9,;;,,,;9,,,;,;;;H;;9,,  F GLUHLWRjLJXDOGDGH
6H TXHUHPRV SUHVHUYDU QRVVD OLEHUGDGH FRPR QDomR
GHYHPRVWDPEpPUHVSHLWDUDOLEHUGDGHGRVRXWURVSRYRV LWHQV,,,H9,,  G GLUHLWRjMXVWLoD LWHQV9,,,;;,H
$SULPHLUDGHVVDVOLEHUGDGHVpDGHHVFROKHUVHXSUySULR ;;9,,,  H GLUHLWRjVHJXUDQoD LWHQV9;,,;,9;;,,
GHVWLQRVHXPRGRGHYLGDVXDIRUPDGHJRYHUQR3RU ;;,;H;;;  I GLUHLWRjIDPtOLD LWHP;9,  J GLUHLWR
LVVRTXHGHYHPRVUHVSHLWDURV(VWDGRVWHRFUiWLFRVPHVPR jSURSULHGDGH LWHP;9,,  K GLUHLWRDRWUDEDOKR LWHQV
TXHUDGLFDLVRVVRFLDOLVWDVRVFRPXQLVWDVRVFDSLWDOLVWDV ;;,,,H;;,9  L GLUHLWRjVD~GH LWHQV;;9  M GLUHLWRj
RVDQDUTXLVWDVHWF HGXFDomR LWHP;;9, H O GLUHLWRjFLGDGDQLD LWHQV;9H
;;, 
,9 QmRLQWHUYHQomR $QRWDR0HVWUH3,172)(55(,5$TXH$JDUDQWLDGD
LQYLRODELOLGDGHDLQGDVHHVWHQGHDRVHVWUDQJHLURVUHVLGHQWHV
6HQmRDGPLWLPRVTXHVHLQWURPHWDPHPQRVVRSDtV QR3DtVFRQIRUPHVHYHULILFDQRWH[WRFRQVWLWXFLRQDOYLJHQWH
QmRGHYHPRVWDPEpPTXHUHULQWHUIHULUQRVRXWURV3DtVHV 0DV WDO JDUDQWLD DLQGD VH DPSOLD DRV HVWUDQJHLURV QmR
$LQWHUYHQomR5XVVDQR$IHJDQLVWmRRXD$PHULFDQDQR UHVLGHQWHV QR %UDVLO SRLV D GHFODUDomR GH GLUHLWRV SRVVXL
3DQDPiMDPDLVVHUiSUDWLFDGDSHOR%UDVLO FDUiWHU XQLYHUVDO 2 VHQWLGR GD H[SUHVVmR HVWUDQJHLUR
UHVLGHQWH GHYH VHU LQWHUSUHWDGR SDUD VLJQLILFDU TXH D
9 LJXDOGDGHHQWUHRV(VWDGRV YDOLGDGH H D IUXLomR OHJDO GRV GLUHLWRV IXQGDPHQWDLV VH
H[HUFHP GHQWUR GR WHUULWyULR EUDVLOHLUR 57-  
(VVHSULQFtSLRIXQGDPHQWDOH[LJHTXHQRVVRVJRYHUQDQ 1HVVHVHQWLGRRSLQDP&OiXGLR3DFKHFRH-RVp&HOVRGH0HOOR
WHVFRQVLGHUHPLJXDLVWDQWRXPSHTXHQRSDtVGRTXLQWR )LOKR$VVLPVHQGRRVHVWUDQJHLURVQmRUHVLGHQWHVQR%UDVLO
PXQGRTXDQWRRV(8$RX)UDQoD,QJODWHUUDHWF SRVVXHPLJXDOPHQWHDFHVVRjVDo}HVFRPRRPDQGDGRGH
VHJXUDQoD H GHPDLV UHPpGLRV SURFHVVXDLV 5)  57
9, GHIHVDGDSD] 5'$H&RQWUD5'$ ³ LQ&RPHQWiUL
RVj&RQVWLWXLomR%UDVLOHLUDHG6DUDLYDžYROS 
9,, VROXomRSDFtILFDGRVFRQIOLWRV
(GHVWDFDDLQGDRFRQVDJUDGR-XULVWDTXHDSURWHomRGR
9,,, UHS~GLRDRWHUURULVPRHDRUDFLVPR
UHJLPHMXUtGLFRGDVOLEHUGDGHVS~EOLFDVDOFDQoDWDPEpP
DVSHVVRDVMXUtGLFDVMiTXHWDPEpPHVWDVWrPGLUHLWRj
,; FRRSHUDomR
FRRSHUDomR HQWUH
HQWUH RV
RV SRYRV
SRYRV SDUD
SDUD R SURJUHVVRGD
SURJUHVVRGD
H[LVWrQFLDjVHJXUDQoDjSURSULHGDGHjSURWHomRWULEXWiULD
KXPDQLGDGH
HDRVUHPpGLRVFRQVWLWXFLRQDLV REUDFLWDGDS 
&kPDUD/HJLVODWLYDGR
  'LVWULWR)HGHUDO 'LU&RQVWLWXFLRQDO 

GLUHLWRV
&RQILUDFRPRRDUWž&)SURFODPDRVGLUHLWRVGLUHLWRV ,; pOLYUH
OLYUHDH[SUHVVmR
H[SUHVVmRGDDWLYLGDGHLQWHOHFWXDODUWtVWL
GDDWLYLGDGHLQWHOHFWXDODUWtVWL
HGHYHUHVLQGLYLGXDLVHFROHWLYRV
HGHYHUHVLQGLYLGXDLVHFROHWLYRV FD
FDFLHQWtILFD
FLHQWtILFDHGH
GHFRPXQLFDomR
FRPXQLFDomRLQGHSHQGHQWHPHQ
WHGHFHQVXUDRXOLFHQoD
$UWž 7RGRV
7RGRVVmR
VmRLJXDLV
LJXDLVSHUDQWH
SHUDQWHDOHL
OHLVHP
VHPGLVWLQomRGH
TXDOTXHUQDWXUH]D
TXDOTXHU QDWXUH]DJDUDQWLQGRVH
JDUDQWLQGRVHDRV
DRVEUDVLOHLURV ; VmR LQYLROiYHLV D LQWLPLGDGH D YLGDSULYDGDD
HDRV
DRVHVWUDQJHLURV
HVWUDQJHLURVUHVLGHQWHV
UHVLGHQWHVQRQR3DtV
3DtVDLQYLRODEL KRQUD H D LPDJHP GDV SHVVRDV DVVHJXUDGR R
OLGDGH
OLGDGHGR
GRGLUHLWR
GLUHLWRjYLGD
YLGDjOLEHUGDGHjLJXDOGD
OLEHUGDGHjLJXDOGD GLUHLWRDLQGHQL]DomR
GLUHLWR LQGHQL]DomRSHOR
SHORGDQR
GDQRPDWHULDO
PDWHULDORX
RXPRUDO
GH
GH j VHJXUDQoD
VHJXUDQoD H  j SURSULHGDGH QRV WHUPRV GHFRUUHQWHGHVXDYLRODomR
VHJXLQWHV
;, D FDVD
FDVD p  DVLOR LQYLROiYHO GRLQGLYtGXRQLQJXpP
2 SULQFtSLR GD LJXDOGDGH p IXQGDPHQWDO QD YLGD QHOD
QHOD SRGHQGR SHQHWUDU  VHP FRQVHQWLPHQWR GR
GHPRFUiWLFD$VSHVVRDVGHYHPVHUWUDWDGDVFRPLJXDO PRUDGRU
PRUDGRU VDOYR
VDOYR HP
HP FDVR
FDVR GH
GH IOD
IODJUDQWH
JUDQWH GHOLWR
GHOLWR RX
GDGH SHOD OHL  e D FKDPDGD LJXDOGDGH MXUtGLFD GDV GHVDVWUH
GHVDVWUH RX
RX SDUD
SDUD SUHVWDU VRFRUUR RX GXUDQWH R
SHVVRDV  1DWXUDOPHQWH Ki GLIHUHQoD HQWUH DV SHVVRDV GLDSRUGHWHUPLQDomRMXGLFLDO
(VVDVGLIHUHQoDVVHUmRREVHUYDGDVDSHQDVHWmRVRPHQWH
TXDQGRDOJXPDGLIHUHQoDIRUHVVHQFLDODXPDGHWHUPLQD ;,, p LQYLROiYHO R VLJLOR GD FRUUHVSRQGrQFLDHGDV
GDVLWXDomR FRPXQLFDo}HV
FRPXQLFDo}HVWHOHJUiILFDV
WHOHJUiILFDVGH
GHGDGRV
GDGRVHGDV
GDVFRPXQL
FDo}HV
o}HV WHOHI{QLFDV
WHOHI{QLFDV VDOYR
VDOYR QR
QR ~OWLPR
~OWLPR FDVR
FDVR SRU
3HOR SULQFtSLR GD LJXDOGDGH p REULJDWyULR R WUDEDOKR RUGHP
RUGHPMXGLFLDO
MXGLFLDOQDV
QDVKLSyWHVHV
KLSyWHVHVHQD
QDIRUPD
IRUPDTXH
TXHDOHL
HVWDEHOHF
HVWDEHOHFHU
HU SDUD
SDUD ILQV
ILQV GH
GH LQYHVWLJDomR
LQYHVWLJDomR FULPLQDO
FULPLQDO RX
LJXDOGHSHVVRDVLJXDLV3HVVRDVHPFRQGLo}HVGHVLJXDLV
LQVWUXomRSURFHVVXDOSHQDO
WHUmRREVHUYDGDVHVVDVGHVLJXDOGDGHV&ODURTXHHVVD
GHVLJXDOGDGHQmRSRGHUiDGYLUGDQDWXUH]D FRUILOLDomR
;,,, pOLYUH
pOLYUHRH[HUFtFLRGHTXDOTXHUWUDEDOKRRItFLRRX
VH[RHWF 
SURILVVmR
SURILVVmR DWHQGLGDV
DWHQGLGDV DV
DV TXDOLILFDo}HV
DOLILFDo}HV SURILVVLRQDLV
TXHDOHLHVWDEHOHFHU
$&RQVWLWXLomRJDUDQWHDWRGRVRVEUDVLOHLURV UHVLGDP
DTXL RX IRUD GR 3DtV  H D WRGRV RV HVWUDQJHLURV HVWHV ;,9 p DVVHJXUDGR D WRGRV R DFHVVR j LQIRUPDomRH
GHVGHTXHUHVLGDPDTXLQR%UDVLO TXHDVHXVVXDYLGD UHVJXDUGDGR
UHVJXDUGDGR R VLJLOR
VLJLOR GD
GD IRQWH
IRQWH TXDQGR
TXDQGR QHFHVViULR
EHQV GLUHLWRV H LQWHUHVVHV VHUmR VHPSUH SURWHJLGRV 2V DRH[HUFtFLRSURILVVLRQDO
yUJmRVS~EOLFRVGHYHUmRID]HUWRGRHPSHQKRSDUDSURWH
JHUDYLGDGHWRGRVWDOTXDOjOLEHUGDGHjLJXDOGDGHj ;9 p OLYUH
OLYUH D ORFRPRomR QR WHUULWyULR QDFLRQDOHP
VHJXUDQoDHjSURSULHGDGH WHPSR
WHPSR GHGH SD]
SD] SRGHQGR
SRGHQGR TXDOTXHU
TXDOTXHU SHVVRD
SHVVRD QRV
WHUPRV GDOHL
WHUPRVGD OHLQHOH
QHOHHQWUDU
HQWUDUSHUPDQHFHU
SHUPDQHFHURX
RXGHOH
GHOHVDLU
, KRPHQV
KRPHQVHPXOKHUHV
PXOKHUHVVmR
VmRLJXDLV
LJXDLVHP
HPGLUHLWRVH
GLUHLWRVHREULJD FRPVHXVEHQV
o}HVQRVWHUPRVGHVWD&RQVWLWXLomR
;9, WRGRV
WRGRV SRGHP
SRGHP UHXQLUVH
UHXQLUVH SDFLILFDPHQWHVHPDUPDV
SDFLILFDPHQWHVHPDUPDV
,, QLQJXpP
QLQJXpP VHUi
VHUi REULJDGRDID]HURXGHL[DUGHID]HU HP
HP ORFDLV
ORFDLV DEHUWRV
DEHUWRV DR
DR S~EOLFR
S~EOLFR LQGHSHQGHQWHPHQWH
LQGHSHQGHQWHPHQWH
DOJXPDFRLVDVHQmRHPYLUWXGHGHOHL GH DXWRUL]DomR GHVGH TXH QmR IUXVWUHP RXWUD
UHXQLmR
UHXQLmR DQWHULRUPHQWH FRQYRFDGD SDUD R PHVPR
,,, QLQJXpP
QLQJXpP VHUi VXEPHWLGR D WRUWXUD QHPDWUDWD ORFDO
ORFDO VHQGR
VHQGR DSHQDV
DSHQDV H[LJLGR
H[LJLGR SUpYLRDYLVRjDXWRUL
SUpYLRDYLVRjDXWRUL
PHQWRGHVXPDQRRXGHJUDGDQWH GDGHFRPSHWHQWH
;9,, pSOHQDD
pSOHQDDOLEHUGDGHGHDVVRFLDomRSDUDILQVOtFLWRV
,9 p OLYUH D PDQLIHVWDomR GRSHQVDPHQWRVHQGR YHGDGDDGHFDUiWHUSDUDPLOLWDU
YHGDGRRDQRQLPDWR
;9,,, D FULDomRGHDVVRFLDo}HVHQDIRUPDGD OHL DGH
9 pDVVHJXUDGR
DVVHJXUDGRRGLUHLWR
GLUHLWRGH
GHUHVSRVWDSURSRUFLRQDODR FRRSHUDWLYDV
FRRSHUDWLYDV  LQGHSHQGHP GH DXWRUL]DomR VHQGR
DJUDYR
DJUDYR DOpP
DOpP GD
GD LQGHQL]DomR
LQGHQL]DomR SRU
SRU GDQR
GDQR PDWHULDO YHGDGDDLQWHUIHUrQFLDHVWDWDOHPVHXIXQFLRQDPHQ
YHGDGD LQWHUIHUrQFLDHVWDWDOHPVHXIXQFLRQDPHQ
PRUDORXjLPDJHP WR

9, pLQYLROiYHO
LQYLROiYHODOLEHUGDGH
OLEHUGDGHGH
GHFRQVFLrQFLD
FRQVFLrQFLDHGHFUHQoD ;,; DVDVVRFLDo}HVVySRGHUmRVHUFRPSXOVRULDPHQWH
VHQGR
VHQ GR DVVHJXUDGR R OLYUH H[HUFtFLR GRV FXOWRV GLVVROYLGDV
GLVVROYLGDV RX
RX WHU
WHU VXDV DWLYLGDGHV VXVSHQVDV SRU
UHOLJLRVRV
UHOLJLRVRVHJDUDQWLGD
JDUDQWLGDQD
QDIRUPDGDOHLD
IRUPDGDOHLDSURWHomR GHFLVmR
GHFLVmR MXGLFLDO
MXGLFLDO H[LJLQGRVH
H[LJLQGRVH QR
QR SULPHLUR
SULPHLUR FDVR
FDVR R
DRVORFDLVGHFXOWRHDVXDVOLWXUJLDV
WUkQVLWRHPMXOJDGR
9,, p DVVHJXUDGD
DVVHJXUDGD QRV
QRV WHUPRV
WHUPRV GD OHL D SUHVWDomRGH
;; QLQJXpP
QLQJXpP SRGHUi
SRGHUi VHU
VHU FRPSHOLGRDDVVRFLDUVHRXD
FRPSHOLGRDDVVRFLDUVHRXD
DVVLVWrQFLD
DVVLVWrQFLDUHOLJLRVD
UHOLJLRVDQDV
QDVHQWLGDGHV
HQWLGDGHVFLYLV
FLYLVHPLOLWDUHV
GHLQWHUQDomRFROHWLYD SHUPDQHFHUDVVRFLDGR

9,,, QLQJXpP VHUi SULYDGR GH GLUHLWRV SRU PRWLYRGH ;;, DV HQWLGDGHV DVVRFLDWLYDV TXDQGRH[SUHVVDPHQWH
FUHQoD UHOLJLRVD RX GH FRQYLFomR ILORVyILFD RX DXWRUL]DGDV
DXWRUL]DGDV WrP
WrP OHJLWLPLGDGH SDUD UHSUHVHQWDU
SROtWLFD
SROtWLFD VDOYR
VDOYR VH
VH DV
DV LQYRFDU
LQYRFDU SDUD
SDUD H[LPLUVH
H[LPLUVH GH VHXVILOLDGRVMXGLFLDORXH[WUDMXGLFLDOPHQWH
REULJDomR
REULJDomR OHJDO D WRGRV LPSRVWD H UHFXVDUVH D
FXPSULUSUHVWDomRDOWHUQDWLYDIL[DGDHPOHL ;;,,pJDUDQWLGRRGLUHLWRGHSURSULHGDGH
 'LU&RQVWLWXFLRQDO &kPDUD/HJLVODWLYDGR'LVWULWR)HGHUDO


;;,,, DSURSULHGDGHDWHQGHUiDVXDIXQomRVRFLDO ;;;,9 VmR


mR D WRGRV
WRGRV DVVHJXUDGRV
DVVHJXUDGRV LQGHSHQGHQWHPHQWHGR
LQGHSHQGHQWHPHQWHGR
SDJDPHQWRGHWD[DV
;;,9 D OHL
OHL HVWDEHOHFHUi R SURFHGLPHQWR SDUDGHVDSUR
SULDomR
SULDomR SRUQHFHVVLGDGHRXXWLOLGDGHS~EOLFDRX
SRUQHFHVVLGDGHRXXWLOLGDGHS~EOLFDRX D RGLUHLWR
GLUHLWRGH
GHSHWLomR
SHWLomRDRV
DRV3RGHUHV
3RGHUHV3~EOLFRV
3~EOLFRVHP
HPGHIH
SRU
RU LQWHUHVVH
LQWHUHVVH VRFLDO
VRFLDO PHGLDQWH
PHGLDQWH MXVWD
MXVWD H SUpYLD
SUpYLD VD GH
GH GLUHLWR
GLUHLWR RX
RX FRQWUD
FRQWUD LOHJDOLGDGH
LOHJDOLGDGH RX
RX DEXVR
DEXVR GH
LQGHQL]D
LQGHQL]DomR
omR HP
HP GLQKHLUR
GLQKHLUR UHVVDOYDGRV
UHVVDOYDGRV RV
RV FDVRV SRGHU
SUHYLVWRVQHVWD&RQVWLWXLomR
E D REWHQomR
REWHQomR GH
GH FHUWLG}HV
FHUWLG}HV HP
HP UHSDUWLo}HVS~EOLFDV
UHSDUWLo}HVS~EOLFDV
;;9 QR
QRFDVRGHLPLQHQWHSHULJR
FDVRGHLPLQHQWHSHULJRS~EOLFR
S~EOLFRDDXWRULGDGH SDUD GHIHVD GH
SDUD GHIHVD GH GLUHLWRV
GLUHLWRV H HVFODUHFLPHQWR
HVFODUHFLPHQWR GH
GH VL
FRPSHWHQWH
FRPSHWHQWH SRGHUi
SRGHUi XVDU
XVDU GH
GH SURSULHGDGHSDUWLFX
SURSULHGDGHSDUWLFX WXDo}HVGHLQWHUHVVHSHVVRDO
ODU
ODUDVVHJXUDGD
DVVHJXUDGDDR
DRSURSULHWiULR
SURSULHWiULRLQGHQL]DomR
LQGHQL]DomRXOWHUL
RUVHKRXYHUGDQR ;;;9 D OHL QmR H[FOXLUi GD DSUHFLDomR GR3RGHU
-XGLFLiULROHVmRRXDPHDoDDGLUHLWR
;;9, DSHTXHQD
SHTXHQDSURSULHGDGH
SURSULHGDGHUXUDODVVLPGHILQLGDHP
OHL
OHL GHVGH
GHVGH TXH WUDEDOKDGD SHOD IDPtOLD QmR VHUi ;;;9, D OHLQmRSUHMXGLFDUiRGLUHLWRDGTXLULGRRDWR
REMHWR
REMHWR GH SHQKRUD SDUD SDJDPHQWR GH GpELWRV MXUtGLFRSHUIHLWRHDFRLVDMXOJDGD
GHFRUUHQWHV
GHFRUUHQWHVGH GHVXD
VXDDWLYLGDGH
DWLYLGDGHSURGXWLYD
SURGXWLYDGLVSRQGR
D OHL
OHL VREU
VREUH RV
RV PHLRV
PHLRV GH
GH ILQDQFLDU
ILQDQFLDU R VHX
VHX GHVHQYRO ;;;9,, QmRKDYHUiMXt]RRXWULEXQDOGHH[FHomR
YLPHQWR
;;;9,,, pUHFRQKHFLGD
UHFRQKHFLGDD LQVWLWXLomR
LQVWLWXLomRGR
GRM~UL
M~ULFRP
FRPDRUJD
;;9,, DRV
DRVDXWRUHV
DXWRUHVSHUWHQFH
SHUWHQFHRGLUHLWR
GLUHLWRH[FOXVLYR
H[FOXVLYRGHXWLOL]D QL]DomRTXHOKHGHUDOHLDVVHJXUDGRV
omR SXEOLFDomR
omR SXEOLFDomR RX
RX UHSURGXomR
UHSURGXomR GHGH VXDV
VXDV REUDV
WUDQVPLVVtYHO
WUDQVPLVVtYHO DRV
DRV KHUGHLURV
KHUGHLURV SHOR
SHOR WHPSR
WHPSR TXHDOHL
TXHDOHL D DSOHQLWXGHGHGHIHVD
IL[DU E RVLJLORGDVYRWDo}HV
F DVREHUDQLDGRVYHUHGLFWRV
DVREHUDQLDGRVYHUHGLFWRV
;;9,,,VmRDVVHJXUDGRVQRVWHUPRVGDOHL
G DFRPSHWrQFLD
FRPSHWrQFLDSDUD
SDUDRMXOJDPHQWR
MXOJDPHQWRGRV
GRVFULPHV
GRORVRVFRQWUDDYLGD
GRORVRVFRQWUDDYLGD
D D SURWHomR
SURWHomR jV
jV SDUWLFLSDo}HV
SDUWLFLSDo}HV LQGLYLGXDLV
GLYLGXDLV HP
HP REUDV
;;;,; QmR
QmR Ki
Ki FULPH
FULPH VHP
VHP OHL
OHL DQWHULRUTXHRGHILQDQHP
FROHWLYDVHj
FROHWLYDVHjUHSURGXomRGDLPDJHPHYR]KXPD
SHQDVHPSUpYLDFRPLQDomROHJDO
QDVLQFOXVLYHQDVDWLYLGDGHVGHVSRUWLYDV
;/ DOHL
OHLSHQDO
SHQDOQmR
QmRUHWURDJLUiVDOYR
UHWURDJLUiVDOYRSDUD
SDUDEHQHILFLDU
EHQHILFLDUR
E RGLUHLWR
GLUHLWRGH
GHILVFDOL]DomR
ILVFDOL]DomRGR
GRDSURYHLWDPHQWR
DSURYHLWDPHQWRHFRQ{
UpX
PLFR
PLFR GDV
GDV REUDV
REUDV TXH
TXH FULDUHP
FULDUHP RX
RX GHTXHSDUWLFLSD
GHTXHSDUWLFLSD
UHP
UHPDRV
DRVFULDGRUHV
FULDGRUHVDRV
DRVLQWpUSUHWHV
LQWpUSUHWHVHjV
jVUHVSHFWLYDV
;/, D OHL SXQLUi TXDOTXHUGLVFULPLQDomRDWHQWDWyULD
UHSUHVHQWDo}HVVLQGLFDLVHDVVRFLDWLYDV
GRVGLUHLWRVHOLEHUGDGHVIXQGDPHQWDLV
;/,, D SUiWLFD
SUiWLFD GR
GR UDFLVPRFRQVWLWXLFULPHLQDILDQoiYHO
UDFLVPRFRQVWLWXLFULPHLQDILDQoiYHO
;;,; DOHL
OHLDVVHJXUDUi
DVVHJXUDUiDRV
DRVDXWRUHV
DXWRUHVGH
GHLQYHQWRV
LQYHQWRVLQGXVWULD
H LPSUHVFULWtYHO
LPSUHVFULWtYHO VXMHLWR
VXMHLWR j SHQD
SHQD GH
GH UHFOXVmR
UHFOXVmR QRV
LV
LV SULYLOpJLR
SULYLOpJLR WHPSRUiULR
WHPSRUiULR SDUD
SDUD VXD
VXD XWLOL]DomR
XWLOL]DomR EHP
WHUPRVGDOHL
FRPR SURWHomR jV FULDo}HV LQGXVWULDLV j
FRPR SURWHomR
SURSULHGDGH
SURSULHGDGHGDV GDVPDUFDVDRVQRPHVGHHPSUHVDV
PDUFDVDRVQRPHVGHHPSUHVDV
;/,,, DOHL
OHLFRQVLGHUDUiFULPHVLQDILDQoiYHLVHLQVXVFHWt
FRQVLGHUDUiFULPHVLQDILDQoiYHLVHLQVXVFHWt
HDRXWURV
RXWURVVLJQRV
VLJQRVGLVWLQWLYRV
GLVWLQWLYRVWHQGR
WHQGRHP
HPYLVWD
YLVWDRLQWH
YHLV
YHLVGH
GHJUDoD
JUDoDRX
RXDQLVWLD
DQLVWLDDSUiWLFD
SUiWLFDGD
GDWRUWXUD
WRUWXUDRWUi
UHVVH
UHVVH VRFLDO
VRFLDO H R GHVHQYROYLPHQWR
GHVHQYROYLPHQWR WHFQROyJLFR
WHFQROyJLFR H
ILFR
ILFR LOtFLWR
LOtFLWR GH
GH HQWRUSHFHQWHV
HQWRUSHFHQWHV H GURJDV
GURJDV DILQV R
HFRQ{PLFRGR3DtV
WHUURULVPR
WHUURULVPRHRV RVGHILQLGRV
GHILQLGRVFRPR
FRPRFULPHVKHGLRQGRV
FULPHVKHGLRQGRV
SRU
SRUHOHV
HOHVUHVSRQGHQGR
UHVSRQGHQGRRVRVPDQGDQWHVRVH[HFXWRUHV
PDQGDQWHVRVH[HFXWRUHV
;;; pJDUDQWLGRRGLUHLWRGHKHUDQoD
pJDUDQWLGRRGLUHLWRGHKHUDQoD
HRVTXHSRGHQGRHYLWiORVVHRPLWLUHP
;;;, D VXFHVVmR GH EHQV GH HVWUDQJHLURV VLWXDGRVQR
3DtV
3DtVVHUi
VHUiUHJXODGD
UHJXODGDSHOD
SHODOHL
OHLEUDVLOHLUD
EUDVLOHLUDHP
HPEHQHItFLR ;/,9 FRQVWLWXL
FRQVWLWXLFULPH
FULPHLQDILDQoiYHOHLPSUHVFULWtYHODDomR
LQDILDQoiYHOHLPSUHVFULWtYHODDomR
GR
GR F{QMXJH
F{QMXJH RX
RX GRV
GRV ILOKRV
ILOKRV EUDVLOHL
EUDVLOHLURV
URV VHPSUH
VHPSUH TXH GH
GHJUXSRV
JUXSRVDUPDGRV
DUPDGRVFLYLV
FLYLVRX
RXPLOLWDUHV
PLOLWDUHVFRQWUD
FRQWUDDRU
QmR
QmR OKHV
OKHV VHMD
VHMD PDLV
PDLV IDYRUiYHO
IDYRUiYHO D OHL
OHL SHVVRDO GR GH
SHVVRDO GR GHPFRQVWLWXFLRQDOHR(VWDGR'HPRFUiWLFR
FXMXV
FXMXV
;/9 QHQKXPD
QHQKXPDSHQ SHQDSDVVDUi
SDVVDUiGD
GDSHVVRD
SHVVRDGRFRQGHQDGR
;;;,, R(VWDGR
(VWDGRSURPRYHUi
SURPRYHUiQD
QDIRUPD
IRUPDGD
GDOHL
OHLDGHIHVD
GHIHVDGR SRGHQGR
SRGHQGRDREULJDomR
REULJDomRGH
GHUHSDUDU
UHSDUDURGDQR
GDQRHDGHFUH
FRQVXPLGRU
FRQVXPLGRU WDomR
WDomR GR
GR SHUGLPHQWRGHEHQVVHUQRVWHUPRVGD
SHUGLPHQWRGHEHQVVHUQRVWHUPRVGD
OHL
OHLHVWHQGLGDV
HVWHQGLGDVDRV
DRVVXFHVVRUHV
VXFHVVRUHVHFRQWUD
FRQWUDHOHV
HOHVH[HFX
;;;,,, WRGRV
WRGRVWrP
WrPGLUHLWR
GLUHLWRDUHFHEHU
UHFHEHUGRV
GRVyUJmRV
yUJmRVS~EOLFRVLQ WDGDV
WDGDV DWp
DWp R OLPLWH
OLPLWH GR
GR YDORU
YDORU GR
GR SDWULP{QLR
IRUPDo}HV
IRUPDo}HV GH
GH VHX
VHX LQWHUHVVH
LQWHUHVVH SDUWLFXODU
SDUWLFXODU RX
RX GH WUDQVIHULGR
WUDQVIHULGR
LQWHUHVVH
LQWHUHVVHFROHWLYRRXJHUDO
FROHWLYRRX JHUDOTXH
TXHVHUmR
VHUmRSUHVWDGDV
SUHVWDGDVQR
SUD]R
SUD]RGD
GDOHL
OHLVRE
VRESHQD
SHQDGH
GHUHVSRQVDELOLGDGH
UHVSRQVDELOLGDGHUHVVDO ;/9, DOHL
OHLUHJXODUi
UHJXODUiDLQGLYLGXDOL]DomR
LQGLYLGXDOL]DomRGD
GDSHQD
SHQDHDGRWD
YDGDV
YDGDV DTXHODV
DTXHODV FXMR
FXMR VLJLOR
VLJLOR VHMD LPSUHVFLQGtYHO j UiHQWUHRXWUDVDVVHJXLQWHV
VHJXUDQoDGDVRFLHGDGHHGR(VWDGR
D SULYDomRRXUHVWULomRGDOLEHUGDGH
&kPDUD/HJLVODWLYDGR
  'LVWULWR)HGHUDO 'LU&RQVWLWXFLRQDO 

E SHUGDGHEHQV /;, QLQJXpP


QLQJXpPVHUiSUHVRVHQmRHPIODJUDQWHGHOLWRRX
VHUiSUHVRVHQmRHPIODJUDQWHGHOLWRRX
F PXOWD SRU
SRURUGHP HVFULWDHIXQGDPHQWDGD
RUGHPHVFULWD GHDXWRULGDGH
IXQGDPHQWDGDGH
G SUHVWDomRVRFLDODOWHUQDWLYD MXGLFLiULD
MXGLFLiULD FRPSHWHQWH VDOYR QRV FDVRV GH WUDQV
H VXVSHQVmRRXLQWHUGLomRGHGLUHLWRV JUHVVmR
JUHVVmR PLOLWDU
PLOLWDU RX
RX FULPH
FULPH SURSULDPHQWH PLOLWDU
GHILQLGRVHPOHL
;/9,,QmRKDYHUiSHQDV
QmRKDYHUiSHQDV
/;,, D SULVmR GH TXDOTXHU SHVVRD H R ORFDO RQGHVH
D GH
GHPRUWH
PRUWHVDOYR
VDOYRHP
HPFDVR
FDVRGH
GHJXHUUD
JXHUUDGHFODUDGD HQFRQWUH
HQFRQWUH VHUmR
VHUmR FRPXQLFDGRV
FRPXQLFDGRV LPHGLDWDPHQWH
LPHGLDWDPHQWH DR
QRVWHUPRVGRDUW;,; MXL]
MXL]FRPSHWHQWHHjIDPtOLDGRSUHVRRXjSHVVRD
FRPSHWHQWHHjIDPtOLDGRSUHVRRXjSHVVRD
E GHFDUiWHUSHUSpWXR SRUHOHLQGLFDGD
F GHWUDEDOKRVIRUoDGRV
G GHEDQLPHQWR /;,,,R SUHVRVHUi
RSUHVR VHUiLQIRUPDGR
LQIRUPDGRGH
GHVHXV
VHXVGLUHLWRVHQWUHRV
GLUHLWRVHQWUHRV
H FUXpLV TXDLV
TXDLVRGH
GHSHUPDQHFHUFDODGRVHQGROKHDVVHJXUD
SHUPDQHFHUFDODGRVHQGROKHDVVHJXUD
GDDDVVLVWrQFLDGDIDPtOLDHGHDGYRJDGR
;/9,,, DSHQD
SHQDVHUi
VHUiFXPSULGD
FXPSULGDHP
HPHVWDEHOHFLPHQWRV
HVWDEHOHFLPHQWRVGLVWLQ
WRV
WRV GH
GH DFRUGRFRPDQDWXUH]DGRGHOLWRDLGDGH
DFRUGRFRPDQDWXUH]DGRGHOLWRDLGDGH /;,9R
RSUHVR
SUHVRWHP
WHPGLUHLWR
GLUHLWRjLGHQWLILFDomR
LGHQWLILFDomRGRVUHVSRQVi
GRVUHVSRQVi
HRVH[RGRDSHQDGR YHLV SRU VXD  SULVmR RX SRU VHX LQWHUURJDWyULR
YHLV
SROLFLDO
;/,;p
p DVVHJXUDGR
DVVHJXUDGR DRV
DRV SUHVRV
SUHVRV R UHVSHLWR
UHVSHLWR jLQ
jLQWHJULGDGH
WHJULGDGH
/;9 D SULVmR
SULVmR LOHJDO
LOHJDO VHUi
VHUi LPHGLDWDPHQWH
LPHGLDWDPHQWH UHOD[DGDSHOD
UHOD[DGDSHOD
ItVLFDHPRUDO
DXWRULGDGHMXGLFLiULD
/ jVSUHVLGLiULDVVHUmRDVVHJXUDGDVFRQGLo}HVSDUD
jVSUHVLGLiULDVVHUmRDVVHJXUDGDVFRQGLo}HVSDUD
/;9,QLQJXpP
QLQJXpP
QLQJXpP VHUi
VHUi OHYDGR
OHYDGR j SULVmR RX QHODPDQWLGR
TXH SRVVDP
TXH SRVVDP SHUPDQHFHU
SHUPDQHFHU FRP
FRP VHXVILOKRVGXUDQWH
VHXVILOKRVGXUDQWH TXDQGR OHL DGPLWLU
TXDQGR D OHL DGPLWLU D OLEHUGDGH
OLEHUGDGH SURYLVyULD
RYLVyULD FRP
RSHUtRGRGHDPDPHQWDomR RXVHPILDQoD
/, QHQKXP EUDVLOHLUR VHUiH[WUDGLWDGRVDOYRR /;9,, QmRKDYHUiSULVmRFLYLOSRU GtYLGDVDOYRDGRUHV
QmRKDYHUiSULVmRFLYLOSRUGtYLGD
QDWXUDOL]DGRHPFDVRGHFULPHFRPXPSUDWLFDGR
QDWXUDOL]DGRHPFDVRGHFULPHFRPXPSUDWLFDGR SRQViYHOSHORLQDGLPSOHPHQWRYROXQWiULRHLQHVFXVi
DQWHV
DQWHVGD
GDQDWXUDOL]DomR
QDWXUDOL]DomRRX
RXGH
GHFRPSURYDGR
FRPSURYDGRHQYRO YHOGHREULJDomRDOLPHQWtFLDHDGRGHSRVLWiULRLQILHO
YLPHQWR
YLPHQWR HP
HP WUiILFR
WUiILFR LOtFLWR
LOtFLWR GH
GH HQWRUSHFHQWHV
HQWRUSHFHQWHV H
GURJDVDILQVQDIRUPDGDOHL 2EV 6XFHGHHQWUHWDQWRTXHSRUKDYHUR%UDVLODVVLQDGRj&RQYHQomR
$PHULFDQDGH'LUHLWRV+XPDQRVFRQKHFLGDFRPR´3DFWRGH6mR
-RVpGD&RVWD5LFDµTXHQR$UWžQžFFRDUWSURtEHR
/,, QmR
QmRVHUi
VHUiFRQFHGLGD
FRQFHGLGDH[WUDGLomR
H[WUDGLomRGHHVWUDQJHLURSRU DSULVLRQDPHQWR GR GHSRVLWiULR LQILHO ³ QmR PDLV VXEVLVWH QR
FULPHSROtWLFRRXGHRSLQLmR VLVWHPD QRUPDWLYR EUDVLOHLUR D SULVmR FLYLO SRU LQILGHOLGDGH
GHSRVLWiULD LQGHSHQGHQWHPHQWH GD PRGDOLGDGH GH GHSyVLWR
WUDWHVH GH GHSyVLWR YROXQWiULR FRQYHQFLRQDO  RX FXLGHVH GH
/,,, QLQJXpP
QLQJXpPVHUi
VHUiSURFHVVDGR
SURFHVVDGRQHP
QHPVHQWHQFLDGRVHQmR GHSyVLWRQHFHVViULR
SHODDXWRULGDGHFRPSHWHQWH
(VVDGLVSRVLomRLQFRUSRUDVHDRRUGHQDPHQWRFRQVWLWXFLRQDOQD
/,9 QLQJXpP
QLQJXpP VHUi
VHUi SULYDGR GD OLEHUGDGH RX GHVHXV IRUPDGRV††žHžGRDUWž&)UHYRJDGDDVVLPD6~PXODQž
67)
EHQVVHPRGHYLGRSURFHVVROHJDO
7mRDVVHQWHpDH[HJHVHTXHR6XSUHPR7ULEXQDO)HGHUDOHGLWRX
/9 DRV
DRVOLWLJDQWHV
OLWLJDQWHVHP
HPSURFHVVR
SURFHVVRMXGLFLDORX
MXGLFLDORXDGPLQLVWUD D6~PXOD9LQFXODQWH
9LQFXODQWHQžHVWDEHOHFHQGRTXH´eL
9LQFXODQWH LOtFLWDDSULVmR
OtFLWD
WLYR
WLYR H DRV
DRV DFXVDGRV
DFXVDGRV HPJHUDOVmRDVVHJXUDGRVR
HPJHUDOVmRDVVHJXUDGRVR GHGHSRVLWiULRLQILHOTXDOTXHUTXHVHMDDPRGDOLGDGHGRGHSyVLWRµ
FRQWUDGLWyULR
FRQWUDGLWyULR H DPSOD
DPSOD GHIHVD FRP RV PHLRV H $6~PXOD9LQFXODQWHWHPIRUoDGHOHLHGHYHVHUFXPSULGDSRU
UHFXUVRVDHODLQHUHQWHV WRGRVRV-Xt]RVH7ULEXQDLV

/9, VmR
VmRLQDGPLVVtYHLV
LQDGPLVVtYHLVQR
QRSURFHVVR
SURFHVVRDVSURYDVREWLGDV FRQFHGHUVHiKDEHDVFRUSXV
/;9,,, FRQFHGHUVHi KDEHDVFRUSXVVHPSUH
VHPSUHTXHDOJXpP
SRUPHLRVLOtFLWRV VRIUHU
VRIUHU RX
RX VH
VH DFKDU
DFKDU DPHDoDGR
DPHDoDGR GH
GH VRIUHUYLROrQFLD
VRIUHUYLROrQFLD
RX
RX FRDomR
FRDomR HP
HP VXD OLEHUGDGH GH
VXD OLEHUGDGH GH ORFRPRomR SRU
/9,, QLQJXpP
QLQJXpPVHUi
VHUiFRQVL
FRQVLGHUDGR
GHUDGRFXOSDGR
FXOSDGRDWp
DWpRWUkQVLWR LOHJDOLGDGHRXDEXVRGHSRGHU
HPMXOJDGRGHVHQWHQoDSHQDOFRQGHQDWyULD
/;,; FRQFHGHUVHi
FRQFHGHUVHiPDQGDGR
PDQGDGRGHGHVHJXUDQoD
VHJXUDQoDSDUDSURWH
R FLYLOPHQWH
/9,,,R FLYLOPHQWH LGHQWLILFDGR
LGHQWLILFDGR  QmR VHUi VXEPHWLGRD JHU
JHU GLUHLWR
GLUHLWR OtTXLGR H FHUWR QmR DPSDUDGR SRU
LGHQWLILFDomR
LGHQWLILFDomRFULPLQDO
FULPLQDOVDOYR
VDOYRQDV
QDVKLSyWHVHV
KLSyWHVHVSUHYLV KDEHDVFRUSXV
KDEHDVFRUSXV RX
RX KDEHDVGDWD
KDEHDVGDWD TXDQGR R UHV
 TXDQGR
WDVHPOHL SRQViYHO
SRQViYHOSHODLOHJDOLGDGHRXDEXVRGHSRGHUIRU
SHODLOHJDOLGDGHRXDEXVRGHSRGHUIRU
DXWRULGDGH
DXWRULGDGH S~EOLFD
S~EOLFD RXDJHQWHGHSHVVRDMXUtGLFD
RXDJHQWHGHSHVVRDMXUtGLFD
/,; VHUi
VHUi DGPLWLGD
DGPLWLGD DomR SULYDGD QRV FULPHV GHDomR QRH[HUFtFLRGHDWULEXLo}HVGR3RGHU3~EOLFR
S~EOLFDVHHVWDQmRIRULQWHQWDGDQRSUD]ROHJDO
/;; RPDQGDGRGH
RPDQGDGRGHVHJXUDQoDFROHWLYRSRGHVHULPSH
/; D OHL
OHL Vy
Vy SRGHUi
SRGHUi UHV
UHVWULQJLUDSXEOLFLGDGHGRVDWRV
WULQJLUDSXEOLFLGDGHGRVDWRV WUDGRSRU
SURFHVVXDLV TXDQGR
SURFHVVXDLV TXDQGR D GHIH
GHIHVD
VD GD
GD LQWLPLGDGH
LQWLPLGDGH RX
RX R
LQWHUHVVHVRFLDORH[LJLUHP D SDUWLGRSROtWLFRFRPUHSUHVHQWDomRQR&RQJUHVVR
1DFLRQDO
 'LU&RQVWLWXFLRQDO
 &kPDUD/HJLVODWLYDGR'LVWULWR)HGHUDO
E RUJDQL]DomR
UJDQL]DomRVLQGLFDO
VLQGLFDOHQWLGDGH
HQWLGDGHGH
GHFODVVH
FODVVHRXDVVR
RXDVVR †ž 2V WUDWDGRV H FRQYHQo}HVLQWHUQDFLRQDLVVREUH
FLDomROHJDOPHQWHFRQVWLWXtGDHHPIXQFLRQDPHQWR
FLDomROHJDOPHQWHFRQVWLWXtGDHHPIXQFLRQDPHQWR GLUHLWRVKXPDQRVTXHIRUHPDSURYDGRVHPFDGD
GLUHLWRVKXPDQRVTXHIRUHPDSURYDGRVHPFDGD
Ki
KiSHOR
SHORPHQRV
PHQRVXP
XPDQR
DQRHPGHIHVDGRVLQWHUHVVHV
HPGHIHVDGRVLQWHUHVVHV &DVDGR&RQJUHVVR1DFLRQDO
&DVDGR&RQJUHVVR1DFLRQDOHPGRLVWXUQRVSRU
GHVHXVPHPEURVRXDVVRFLDGRV WUrV
WUrV TXLQWRV
TXLQWRV GRV
GRV YRWRV
YRWRV GRV UHVSHFWLYRV PHPEURV
GRV UHVSHFWLYRV
VHUmR
VHUmR HTXLYDOHQWHV
HTXLYDOHQWHV jV
jV HPHQGDV
HPHQGDV FRQVWLWXFLRQDLV
DFUHVFLGRSHOD(PHQGD&RQVWLWXFLRQDOQž5HIRUPDGR-XGLFLiULR 
/;;, FRQFHGHUVHi
FRQFHGHUVHiPDQGDGR
PDQGDGRGHGHLQMXQomR
LQMXQomRVHPSUH
VHPSUHTXHD
IDOWD
IDOWDGHQRUPD
GHQRUPDUHJXODPHQWDGRUD
UHJXODPHQWDGRUDWRUQH
WRUQHLQYLiYHO
LQYLiYHOR †ž 2%UDVLO
%UDVLOVH
VHVXEPHWH
VXEPHWHjMXULVGLomRGH
MXULVGLomRGH7ULEXQDO3HQDO
H[HUFtFLR
H[HUFtFLRGRV
GRVGLUHLWRV
GLUHLWRVHOLEHUGDGHV
OLEHUGDGHVFRQVWLWXFLRQDLV ,QWHUQDFLRQDO
,QWHUQDFLRQDO D FXMD FULDomR WHQKD PDQLIHVWDGR
H GDV
GDV SUHUURJDWLYDV
SUHUURJDWLYDV LQHUHQWHV
LQHUHQWHV j QDFLRQDOLGDGH
QDFLRQDOLGDGH j DGHVmR DFUHVFLGRSHOD(&Qž5HIRUPDGR-XGLFLiULR 
VREHUDQLDHjFLGDGDQLD

FRQFHGHUVHiKDEHDVGDWD
/;;,,FRQFHGHUVHi
FRQFHGHUVHiKDEHDVGDWD
KDEHDVGDWD
'LUHLWRV6RFLDLV
D SDUDDVVHJXUDURFRQKHFLPHQWRGHLQIRUPDo}HV
SDUDDVVHJXUDURFRQKHFLPHQWRGHLQIRUPDo}HV
UHODWLYDV
UHODWLYDV jSHVVRDGRLPSHWUDQWHFRQVWDQWHVGH $UWž 6mR
6mRGLUHLWRV
GLUHLWRVVRFLDLV
VRFLDLVDHGXFDomR
HGXFDomRDVD~GH
VD~GHDDOLPHQ
UHJLVWURV
UHJLVWURV RX EDQFRV GH GDGRV GH HQWLGDGHV WDomRRWUDEDOKR
WDomR PRUDGLDRWUDQVSRUWH
WUDEDOKRDPRUDGLD WUDQVSRUWHROD]HU
JRYHUQDPHQWDLVRXGHFDUiWHUS~EOLFR D VHJXUDQoD
VHJXUDQoD D SUH
SUHYLGrQFLD
YLGrQFLD VRFLDO
VRFLDO D SURWHomR
SURWHomR j
PDWHUQLGDGH
PDWHUQLGDGHHjLQIkQFLD
LQIkQFLDDDVVLVWrQFLD
DVVLVWrQFLDDRV
DRVGHVDP
E SDUD
SDUDDUHWLILFDomR
UHWLILFDomRGH
GHGDGRV
GDGRVTXDQGR
TXDQGRQmR
QmRVH
VHSUHILUD SDUDGRV
SDUDGRVQD
QDIRUPD
IRUPDGHVWD
GHVWD&RQVWLWXLomR D(&Qž
ID]rOR
ID]rORSRU
SRUSURFHVVR
SURFHVVRVLJLORVR
VLJLORVRMXGLFLDO
MXGLFLDORX
RXDGPLQLV DFUHVFHXD´PRUDGLDµD(&QžDFUHVFHXD´DOLPHQWDomRµHD(&Qž
DFUHVFHQWRXR´WUDQVSRUWHµGHQWUHRV'LUHLWRV6RFLDLV
WUDWLYR
7RGD VRFLHGDGH GHYH WHU SRU REMHWLYR DWHQGHU DRV
/;;,,, TXDOTXHU FLGDGmR p SDUWH OHJtWLPD SDUDSURSRU LQWHUHVVHVFRPXQV$VSHVVRDVUH~QHPVHSDUDVHDMXGD
DomR
DomRSRSXODU
SRSXODUTXH
TXHYLVH
YLVHDDQXODU
DQXODUDWR
DWROHVLYR
OHVLYRDR
DRSDWUL UHPP~WXDHUHFLSURFDPHQWH1RVVDVQHFHVVLGDGHVEiVLFDV
P{QLLR S~EOLFR
P{Q S~EOLFR RX
RX GH
GH HQWLGDGH
HQWLGDGH GH
GH TXH
TXH R (VWDGR VRPHQWHVHUmRDWHQGLGDVFRPRDX[tOLRGRSUy[LPR3RU
SDUWLFLSH
SDUWLFLSH j PRUDOLGDGH
PRUDOLGDGH DGPLQLVWUDWLYD
DGPLQLVWUDWLYD DR
DR PHLR LVVRRVHUKXPDQRpJUHJiULRXPSUHFLVDGRRXWUR1HVVD
DPELHQWH
DPELHQWH H DR
DR SDWULP{QLR
SDWULP{QLR KLVWyULFR
KLVWyULFR H FXOWXUDO YLGD FRPXP p SUHFLVR TXH ILTXH FODUR TXDLV VmR QRVVDV
ILFDQ
ILFDQGR
GR R DXWRU
DXWRU VDOYR
VDOYR FRPSURYDGD
FRPSURYDGD PiIp
PiIp LVHQWR QHFHVVLGDGHV EiVLFDV H TXH GHYHUmR VHU DWHQGLGDV H
GHFXVWDVMXGLFLDLVHGR{QXVGDVXFXPErQFLD UHVSHLWDGDV SHORV DGPLQLVWUDGRUHV S~EOLFRV  SULQFLSDO
PHQWH
/;;,9 R (VWDGRSUHVWDUiDVVLVWrQFLDMXUtGLFDLQWHJUDOH
(VWDGRSUHVWDUiDVVLVWrQFLDMXUtGLFDLQWHJUDOH
JUDWXLWDDRVTXHFRPSURYDUHPLQVXILFLrQFLDGH $UWž 6mR
6mRGLUHLWRV
GLUHLWRVGRV
GRVWUDEDOKDGRUHV
WUDEDOKDGRUHVXUEDQRV
XUEDQRVHUXUDLV
UHFXUVRV DOpP
DOpP GH
GH RXWURV
RXWURV TXH
TXH YLVHP
YLVHP j PHOKRULD
PHOKRULD GH VXD
FRQGLomRVRFLDO
/;;9 R (VWDGR LQGHQL]DUi R FRQGHQDGR SRUHUUR
MXGLFLiULR
MXGLFLiULRDVVLP
DVVLPFRPR
FRPRRTXHILFDUSUHVRDOpPGR
TXHILFDUSUHVRDOpPGR $&RQVWLWXLomRSURWHJHWRGRVRVWUDEDOKDGRUHVXUEDQRV
WHPSRIL[DGRQDVHQWHQoD WUDEDOKDPQDVFLGDGHV HUXUDLV WUDEDOKDPQRFDPSR
DJULFXOWXUDRXSHFXiULD HWRUQDREULJDWyULDDREVHUYkQFLD
/;;9, VmRJUDWXLWRVSDUDRVUHFRQKHFLGDPHQWHSREUHV
VmRJUDWXLWRVSDUDRVUHFRQKHFLGDPHQWHSREUHV GHVVHV VHXV GLUHLWRV H GH RXWURV GLUHLWRV TXH VHMDP
QDIRUPDGDOHL HVWDEHOHFLGRVSRUOHLSDUDPHOKRUDUVXDTXDOLGDGHGHYLGD
HFRQYLYrQFLDVRFLDO
D RUHJLVWURFLYLOGHQDVFLPHQWR
E DFHUWLGmRGHyELWR , UHODomR
UHODomRGH
GHHPSUHJR
HPSUHJRSURWHJLGD
SURWHJLGDFRQWUD
FRQWUDGHVSHGLGD
DUELWUiULDRXVHP
DUELWUiULDRXVHPMXVWD
MXVWDFDXVD
FDXVDQRV
QRVWHUPRV
WHUPRVGHOHL
/;;9,, VmRJUDWXLWDVDVDo}HVGH
VmRJUDWXLWDVDVDo}HVGHKDEHDVFRUSXV
RJUDWXLWDVDVDo}HVGH  HKD
H FRPSOHPHQWDU
FRPSOHPHQWDU TXH SUHYHUi LQGHQL]DomR FRP
EHDVGDWD

H
HQD
QDIRUPD
IRUPDGD
GDOHL
OHLRV
RVDWRV
DWRVQHFHVViULRV SHQVDWyULDGHQWUHRXWURVGLUHLWRV
DRH[HUFtFLRGDFLGDGDQLD
,, VHJXURGHVHPSUHJR HPFDVRGHGHVHPSUHJR
/;;9,,, DWRGRV
WRGRVQR
QRkPELWR
kPELWRMXGLFLDO
MXGLFLDOHDGPLQLVWUDWLYRVmR
LQYROXQWiULR
DVVHJXUDGRV
DVVHJXUDGRVDUD]RiYHO
UD]RiYHOGXUDomR
GXUDomRGR GRSURFHVVR
SURFHVVRHRV
PHLRV
PHLRVTXH
TXHJDUDQWDP
JDUDQWDPDFHOHULGDGH
FHOHULGDGHGH GHVXD
VXDWUDPLWD
,,, IXQGRGHJDUDQWLDGRWHPSRGHVHUYLoR
omR
omR DFUHVFLGRSHOD(PHQGD&RQVWLWXFLRQDOQž

†ž $V QRUPDV GHILQLGRUDV GRV GLUHLWRVHJDUDQWLDV ,9 VDOiULR PtQLPR IL[DGR HPOHLQDFLRQDOPHQWH
IXQGDPHQWDLVWrPDSOLFDomRLPHGLDWD XQLILFDGRFDSD]
XQLILFDGR FDSD]GH
GHDWHQGHU
DWHQGHUDVXDV
VXDVQHFHVVLGDGHV
YLWDLV
YLWDLVEiVLFDV
EiVLFDVHjV
jVGH
GHVXD
VXDIDPtOLD
IDPtOLDFRPPRUDGLD
FRPPRUDGLD
†ž 2VGLUHLWRVHJDUDQWLDV
2VGLUHLWRVHJDUDQWLDVH[SUHVVRVQHVWD&RQVWLWXL DOLPHQWDomR
DOLPHQWDomR HGXFDomR
HGXFDomR VD~GH
VD~GH OD]HU
OD]HU YHVWXiULR
omR
omR QmR
QmR H[FOXHP
H[FOXHP RXWURV
RXWURV GHFRUUHQWHV
GHFRUUHQWHV GR
GR UHJLPH
UHJLPH H KLJLHQH
KLJLHQH WUDQVSRUWH
WUDQVSRUWH H SUHYLGrQFLD VRFLDO FRP
GRV
GRVSULQFtSLRV
SULQFtSLRVSRU
SRUHOD
HODDGRWDGRVRXGRVWUDWDGRV
DGRWDGRVRX GRVWUDWDGRV UHDMXVWHVSHULyGLFRVTXHOKHSUHVHUYHPRSRGHU
UHDMXVWHVSHULyGLFRVTXHOKHSUHVHUYHPRSRGHU
LQWHUQDFLRQDLVHP
LQWHUQDFLRQDLVHPTXH
TXHD5HS~EOLFD
5HS~EOLFD)HGHUDWLYD
)HGHUDWLYDGR DTXLVLWLYR
DTXLVLWLYR VHQGR
VHQGR YHGDGD
YHGDGD VXD YLQFXODomR SDUD
%UDVLOVHMDSDUWH TXDOTXHUILP
&kPDUD/HJLVODWLYDGR
  'LVWULWR)HGHUDO 'LU&RQVWLWXFLRQDO 

9 SLVR
SLVRVDODULDOSURSRUFLRQDOjH[WHQVmRHjFRP
VDODULDOSURSRUFLRQDOjH[WHQVmRHjFRP ;;,,,DGLFLRQDO
DGLFLRQDO
DGLFLRQDO GH UHPXQHUDomR SDUD DVDWLYLGDGHV
SOH[LGDGHGRWUDEDOKR SHQRVDVLQVDOXEUHVRXSHULJRVDVQDIRUPDGD
SHQRVDVLQVDOXEUHVRXSHULJRVDVQDIRUPDGD
OHL
9, LUUHGXWLELOLGDGH
LUUHGXWLELOLGDGHGR
GRVDOiULR
VDOiULRVDOYR
VDOYRRGLVSRVWRHP
FRQYHQomRRXDFRUGRFROHWLYR ;;,9 DSRVHQWDGRULD

9,, JDUDQWLD
JDUDQWLDGHVDOiULRQXQFDLQIHULRUDRPtQLPR
GHVDOiULRQXQFDLQIHULRUDRPtQLPR ;;9 DVVLVWrQFLD JUDWXLWD DRV ILOKRV HGHSHQGHQWHV
SDUDRVTXHSHUFHEHPUHPXQHUDomRYDULiYHO GHVGH QDVFLPHQWRDWp
GHVGHRQDVFLPHQWR DWp FLQFR 
FLQFR DQRV
DQRVGH GHLGDGH
HP
HP FUHFKHV
FUHFKHV H SUpHVFRODV
SUpHVFRODV UHGDomRGHVWHLQFLVRIRLDOWHUDGD
9,,, GpFLPR
GpFLPRWHUFHLUR
WHUFHLURVDOiULR
VDOiULRFRP
FRPEDVH
EDVHQD
QDUHPXQHUD SHOD(&Qž

omRLQWHJUDORXQRYDORUGDDSRVHQWDGRULD ;;9, UHFRQKHFLPHQWR GDV FRQYHQo}HV HDFRUGRV


FROHWLYRVGHWUDEDOKR
,; UHPXQHUDomR
UHPXQHUDomR GR
GR WUDEDOKR QRWXUQRVXSHULRUj
GRGLXUQR ;;9,, SURWHomRHPIDFHGDDXWRPDomRQDIRUPDGD
OHL
; SURWHomR
SURWHomRGR
GRVDOiULR
VDOiULRQD
QDIRUPDGDOHL
IRUPDGDOHLFRQVWLWXLQ
GRFULPHVXDUHWHQomRGRORVD ;;9,,, VHJXUR
VHJXURFRQWUD
FRQWUDDFLGHQWHV
DFLGHQWHVGH
GHWUDEDOKR
WUDEDOKRDFDUJR
FDUJRGR
HPSUHJDGRU VHP
HPSUHJDGRU VHP H[FOXLU
H[FOXLU D LQGHQL]DomR
LQGHQL]DomR D TXH
;, SDUWLFLSDomR QRV OXFURV RXUHVXOWDGRVGHV HVWH
HVWHHVWiREULJDGRTXDQGR
HVWiREULJDGR TXDQGRLQFRUUHU
LQFRUUHUHP
HPGROR
GRORRX
YLQFXODGDGD
YLQFXODGD GDUHPXQHUDomR
UHPXQHUDomRH
HH[FHSFLRQDOPHQ FXOSD
WH
WHSDUWLFLSDomRQDJHVWmR
SDUWLFLSDomRQDJHVWmRGD
GDHPSUHVD
HPSUHVDFRQIRU
PHGHILQLGRHPOHL
;;,; DomR
DomRTXDQWR
TXDQWRDFUpGLWRV
FUpGLWRVUHVXOWDQWHV
UHVXOWDQWHVGDVUHODo}HV
GH
GH WUDEDOKR
WUDEDOKR FRP
FRP SUD]R
SUD]R SUHVFULFLRQDOGHFLQFR
SUHVFULFLRQDOGHFLQFR
;,, VDOiULRIDPtOLDSDJRHPUD]mRGRGHSHQGHQWH
VDOiULRIDPtOLDSDJRHPUD]mRGRGHSHQGHQWH
DQRV
DQRV SDUD
SDUD RV
RV WUDEDOKDGRUHV
WUDEDOKDGRUHV XUEDQRV
XUEDQRV H UXUDLV
GRWUDEDOKDGRUGHEDL[DUHQGDQRVWHUPRVGD
DWp
DWp R OLPLWH
OLPLWH GH GRLV DQRV DSyV D H[WLQomR GR
OHL UHGDomRDOWHUDGDSHOD(PHQGD&RQVWLWXFLRQDOQž
FRQWUDWR
FRQWUDWR GH
GH WUDEDOKR
WUDEDOKR UHGDomR GHVWH LQFLVR PRGLILFDGD SHOD
(PHQGD&RQVWLWXFLRQDOQž
;,,, GXUDomR
GXUDomRGR
GRWUDEDOKR
WUDEDOKRQRUPDO
QRUPDOQmR
QmRVXSHULRU
VXSHULRUDRLWR
KRUDV
KRUDV GLiULDV H TXDUHQWD H TXDWUR VHPDQDLV ;;; SURLELomR
SURLELomRGH
GHGLIHUHQoD
GLIHUHQoDGH
GHVDOiULRV
VDOiULRVGHH[HUFtFLR
IDFXOWDGD
IDFXOWDGDDFRPSHQVDomR
FRPSHQVDomRGH
GHKRUiULRV
KRUiULRVHDUHGX GH
GH IXQo}HV
IXQo}HV H GH FULWpULRV GH DGPLVVmR SRU
omRGH
omR GHMRUQDGD
MRUQDGDPHGLDQWH
PHGLDQWHDFRUGR
DFRUGRRX
RXFRQYHQomR PRWLYRGHVH[RLGDGHFRURXHVWDGRFLYLO
FROHWLYDGHWUDEDOKR
;;;, SURLELomR
SURLELomRGH
GHTXDOTXHUGLVFULPLQDomRQRWRFDQWH
TXDOTXHUGLVFULPLQDomRQRWRFDQWH
;,9 MRUQDGD
MRUQDGDGH
GHVHLV
VHLVKRUDV
KRUDVSDUD
SDUDRWUDEDOKRUHDOL]DGR DVDOiULR
VDOiULRHFULWpULRVGHDGPLVVmR
FULWpULRVGHDGPLVVmRGR
GRWUDEDOKDGRU
HP
HPWXUQRV
WXUQRVLQLQWHUUXSWRV
LQLQWHUUXSWRVGH
GHUHYH]DPHQWRVDOYR
UHYH]DPHQWRVDOYR SRUWDGRUGHGHILFLrQFLD
QHJRFLDomRFROHWLYD
;;;,, SURLELomRGHGLVWLQomRHQWUHWUDEDOKRPDQXDO
SURLELomRGHGLVWLQomRHQWUHWUDEDOKRPDQXDO
;9 UHSRXVR
UHSRXVR VHPDQDO UHPXQHUDGRSUHIHUHQFLDO WpFQLFRHLQWHOHFWXDORXHQWUHRVSURILVVLRQDLV
WpFQLFR HLQWHOHFWXDORXHQWUHRVSURILVVLRQDLV
PHQWHDRVGRPLQJRV UHVSHFWLYRV

;9, UHPXQHUDomR
UHPXQHUDomRGR
GRVHUYLoR
VHUYLoRH[WUDRUGLQiULRVXSHUL ;;;,,, SURLELomR GH WUDEDOKR QRWXUQR SHULJRVRRX
RU
RU QR
QR PtQLPR
PtQLPR HP
HP FLQTHQWD
FLQTHQWD SRU FHQWR j GR LQVDOXEUHDPHQRUHVGHGH]RLWRHGHTXDOTXHU
QRUPDO WUDEDOKR D PHQRUHV
WUDEDOKR PHQRUHV GH GH GH]HVVHLVDQRV
GH]HVVHLVDQRVVDOYRQD
VDQRVVDOYRQD
FRQGLomR
FRQGLomR GH GH DSUHQGL]
DSUHQGL] D SDUWLU
SDUWLU GH GH TXDWRU]H
TXDWRU]H
;9,, JR]R
R]R GH
GH IpULDV
IpULDV DQXDLV
DQXDLV UHPXQHUDGDVFRPSHOR
UHPXQHUDGDVFRPSHOR DQRV
DQRV UHGDomRDOWHUDGDSHOD(PHQGD&RQVWLWXFLRQDOQž
PHQRV
PHQRV XP
XP WHUoR
WHUoR D PDLV
PDLV GR
GR TXHRVDOiULRQRU
TXHRVDOiULRQRU
;;;,9 LJXDOGDGH
LJXDOGDGHGH
GHGLUHLWRVHQWUHRWUDEDOKDGRU
GLUHLWRVHQWUHRWUDEDOKDGRUFRP
PDO
YtQFXOR
YtQFXORHPSUHJDWtFLR
HPSUHJDWtFLRSHUPDQHQWH
SHUPDQHQWHHRWUDEDOKD
;9,,, OLFHQoDj
OLFHQoDjJHVWDQWH
JHVWDQWHVHPSUHMXt]RGRHPSUHJRH GRUDYXOVR
GR
GRVDOiULR
VDOiULRFRP
FRPDGXUDomR
GXUDomRGH
GHFHQWR
FHQWRHYLQWH
YLQWHGLDV
†~QLFR 6mR
6mR DVVHJXUDGRVjFDWHJRULDGRVWUDEDOKDGRUHV
DVVHJXUDGRVjFDWHJRULDGRVWUDEDOKDGRUHV
;,; OLFHQoDSDWHUQLGDGH
OLFHQoDSDWHUQLGDGHQRV
QRVWHUPRV
WHUPRVIL[DGRVHPOHL GRPpVWLFRVRVGLUHLWRVSUHYLVWRVQRVLQFLVRV,9
GRPpVWLFRVRVGLUHLWRVSUHYLVWRVQRVLQFLVRV,9
9,
, 9,,
9,, 9,,,
9,,, ;
; ;,,,
;,,, ;9
;9 ;9,
;9, ;9,,
;9,, ;9,,,
;9,,, ;,;;,;
;; SURWHomR
SURWHomR GR PHUFDGR GHWUDEDOKRGDPXOKHU ;;,
;;, ;;,,
;;,, ;;,9
;;,9 ;;9,
;;9, ;;; ;;;, H ;;;,,, H
PHGLDQWH
PHGLDQWHLQFHQWLYRV
LQFHQWLYRVHVSHFtILFRV
HVSHFtILFRVQRV
QRVWHUPRV
WHUPRVGD DWHQGLGDV
DWHQGLGDV DV FRQGLo}HV HVWDEHOHFLGDV HP OHL H
OHL REVHUYDGD
REVHUYDGD D VLPSOLILFDomR
VLPSOLILFDomR GR GR FXPSULPHQWRGDV
FXPSULPHQWRGDV
REULJDo}HV
REULJDo}HV WULEXWiULDV SULQFLSDLV H DFHVVyULDV
;;, DYLVR
DYLVRSUpYLR
SUpYLRSURSRUFLRQDO
SURSRUFLRQDODR
DRWHPSRGHVHUYLoR GHFRUUHQWHV
GHFRUUHQWHVGDGDUHODomR
UHODomRGHGHWUDEDOKR
WUDEDOKRHVXDV VXDVSHFXOL
VHQGR
VHQGRQRPtQLPRGHWULQWDGLDVQRVWHUPRVGD
QRPtQLPRGHWULQWDGLDVQRVWHUPRVGD DULGDGHV
DULGDGHVRV
RVSUHYLVWRV
SUHYLVWRVQRV
QRVLQFLVRV
LQFLVRV, ,,,
,,,,,
,,,,;
,;;,,
OHL ;;9 H ;;9,,, EHP FRPR D VXD LQWHJUDomR j
SUHYLGrQFLD
SUHYLGrQFLD VRFLDO
VRFLDO UHGDomR GDGD SHOD (PHQGD &RQVWLWXFLRQDO Qž

;;,, UHGXomR
UHGXomR GRV
GRV ULVFRV
ULVFRV LQHUHQWHV
LQHUHQWHV DRWUDEDOKRSRU
PHLR
PHLRGH
GHQRUPDV
QRUPDVGHGHVD~GH
VD~GHKLJLHQH
KLJLHQHHVHJXUDQoD $V FRQKHFLGDV HPSUHJDGDV GRPpVWLFDV DJRUD SRU
GLVSRVLomR FRQVWLWXFLRQDO WrP GLUHLWR D ,9  VDOiULR
 'LU&RQVWLWXFLRQDO &kPDUD/HJLVODWLYDGR'LVWULWR)HGHUDO


PtQLPR 9, LUUHGXWLELOLGDGHGHVDOiULR 9,, ´JDUDQWLD 9, p REULJDWyULDDSDUWLFLSDomRGRVVLQGLFDWRVQDV


GHVDOiULRµ 9,,, žVDOiULR ; ´SURWHomRGRVDOiULRµ QHJRFLDo}HVFROHWLYDVGHWUDEDOKR
;,,, ´MRUQDGDGHWUDEDOKRGHKRUDVGLiULDVHKRUDV
VHPDQDLVµ ;9  UHSRXVR VHPDQDO UHPXQHUDGR ;9, 9,, R DSRVHQWDGR
DSRVHQWDGR ILOLDGR
ILOLDGR WHP
WHP GLUHLWR
GLUHLWR D YRWDU HVHU
´UHPXQHUDomR GR VHUYLoR H[WUDRUGLQiULRµ  KRUDV H[WUDV YRWDGRQDVRUJDQL]Do}HVVLQGLFDLV
SDJDVFRPGHDGLFLRQDO ;9,, IpULDVD ;9,,,
OLFHQoDPDWHUQLGDGHGHGLDV ;,; OLFHQoDSDWHUQLG 9,,, pYHGDGD
YHGDGDDGLVSHQVD
GLVSHQVDGR
GRHPSUHJDGRVLQGLFDOL]DGR
DGH VHIRUXPFDVHLURRXXPGRPpVWLFR ¢GRPpVWLFR" D SDUWLU
SDUWLU GR
GR UHJLVWUR
UHJLVWUR GDFDQGLGDWXUDDFDUJRGH
QmR HVWUDQKH SRLV Ki PXLWRV WUDYHVWLV HPSUHJDQGRVH GLUHomR
GLUHomR RX
RX UHSUHVHQWDomR
UHSUHVHQWDomR VLQGLFDO H VH HOHLWR
HP FDVD GH IDPtOLD H HOHV SRGHP VHU SDLV DSHVDU GD DLQGD
DLQGDTXH
TXHVXSOHQWH DWpXPDQRDSyVRILQDOGR
VXSOHQWHDWp
RSomRVH[XDOSDUWLFXODUL]DGD  ;;, DYLVRSUpYLR ;;,, PDQGDWR
PDQGDWR VDOYR
VDOYR VH
VH FRPHWHU
FRPHWHU IDOWD
IDOWD JUDYH
JUDYH QRV
QRV WHU
´VHJXUDQoDQRWUDEDOKRµ ;;,9 DSRVHQWDGRULD ;;9, PRVGDOHL
´DSOLFDomR GH FRQYHQo}HV H DFRUGRV FROHWLYRVµ ;;;
´LJXDOGDGHVDODULDOµ ;;;, ´LJXDOGDGHSDUDRSRUWDGRUGH †~QLFR $V
$VGLVSRVLo}HV
GLVSRVLo}HVGHVWH
GHVWHDUWLJR
DUWLJRDSOLFDPVHj
DSOLFDPVHjRUJDQL
GHILFLrQFLDµ H ;;;,,,  ´SURLELomR GH WUDEDOKR QRWXUQR ]DomR
]DomRGH
GHVLQGLFDWRV
VLQGLFDWRVUXUDLV
UXUDLVHGH
GHFRO{QLDV
O{QLDVGH
GHSHV
SHULJRVRRXLQVDOXEUHDPHQRUHVGHGH]RLWRHGHTXDOTXHU FDGRUHV
FDGRUHV DWHQGLGDV
DWHQGLGDV DV
DV FRQGLo}HV
FRQGLo}HV TXH D OHL
WUDEDOKRDPHQRUHVGHGH]HVVHLVDQRVµDOpPGHVHILOLDUHP HVWDEHOHFHU
DRVLVWHPDGH3UHYLGrQFLD6RFLDOHWDPEpPJR]DUHPGH
,  ´LQGHQL]DomR HP FDVR GH GHVSHGLGD DUELWUiULDµ ,, $UWž eDVVHJXUDGR
DVVHJXUDGRRGLUHLWR
GLUHLWRGH
GHJUHYHFRPSHWLQGRDRV
´VHJXURGHVHPSUHJRµ ,,, ´)*76µ )XQGRGH*DUDQWLDSRU WUDEDOKDGRUHV
WUDEDOKDGRUHV GHFLGLUVREUHDRSRUWXQLGDGHGH
GHFLGLUVREUHDRSRUWXQLGDGHGH
7HPSRGH6HUYLoR  ,; ´UHPXQHUDomRGHVHUYLoRQRWXUQR H[HUFrOR
H[HUFrOR H VREUH
VREUH RV
RV LQWHUHVVHV
LQWHUHVVHV TXH GHYDP SRU
VXSHULRU j GR GLXUQRµ ;,,  ´VDOiULRIDPtOLDµ SDJR HP PHLRGHOHGHIHQGHU
UD]mR GR GHSHQGHQWH GD WUDEDOKDGRUD ;;9  ´FUHFKH H
SUpHVFRODµJUDWXLWRVDRVILOKRVHGHSHQGHQWHVGDWUDEDOKD 2WmRFRQKHFLGRGLUHLWRGHJUHYHKRMHHPGLDpXP
GRUDH ;;9,,, ´VHJXURFRQWUDDFLGHQWHVGHWUDEDOKRµ GLUHLWRFRQVWLWXFLRQDOGRWUDEDOKDGRU4XDQGRHQWUDUHP
JUHYHRXSRUTXHHQWUDUHPJUHYHpXPDGHFLVmRVREHUD
$UWž eOLYUH
OLYUHDDVVRFLDomR
DVVRFLDomRSURILVVLRQDO
SURILVVLRQDORX
RXVLQGLFDORE QDHH[FOXVLYDGRVWUDEDOKDGRUHV6yRWUDEDOKDGRUWHPR
VHUYDGRRVHJXLQWH GLUHLWRSRGHUGHGHFLGLUVREUHDUHDOL]DomRGHXPDJUHYH

$ &RQVWLWXLomR DVVHJXUD D WRGRV RV WUDEDOKDGRUHV R †ž $OHL


OHLGHILQLUi
GHILQLUiRV
RVVHUYLoRV
VHUYLoRVRXDWLYLGDGHVHVVHQFLDLV
GLUHLWRGHVHDVVRFLDUHPDXPDDVVRFLDomRSURILVVLRQDORX HGLVSRUi
GLVSRUiVREUH
VREUHRDWHQGLPHQWR
DWHQGLPHQWRGDV
GDVQHFHVVLGDGHV
D XP VLQGLFDWR  e LPSRUWDQWH OHPEUDU TXH DWp EHP LQDGLiYHLVGDFRPXQLGDGH
SRXFR WHPSR DOJXQV HPSUHJDGRUHV PDQGDYDP VHXV
HPSUHJDGRVHPERUDTXDQGRVDELDPTXHHVWHVKDYLDPVH 9HMDTXDLVVmRRVVHUYLoRVRXDWLYLGDGHVHVVHQFLDLVVH
ILOLDGR D DOJXP VLQGLFDWR RX DVVRFLDomR SURILVVLRQDO GD JXQGRDOHLGHJUHYH
OHLGHJUHYH /HLQž
OHLGHJUHYH 
 GH-81 
FDWHJRULD
, WUDWDPHQWRHDEDVWHFLPHQWRGHiJXDSURGXomRHGLVWULEXL
, D OHL QmR SRGHUi H[LJLU DXWRUL]DomRGR(VWDGR omRGHHQHUJLDHOpWULFDJiVHFRPEXVWtYHLV
SDUD
SDUD D IXQ
IXQGDomR
GDomR GH
GH VLQGLFDWR
VLQGLFDWR UHVVDOYDGR
UHVVDOYDGR R ,, DVVLVWrQFLDPpGLFDHKRVSLWDODU
UHJLVWUR
UHJLVWURQR
QRyUJmR
yUJmRFRPSHWHQWH
FRPSHWHQWHYHGDGDV
YHGDGDVDR
DR3RGHU ,,, GLVWULEXLomRHFRPHUFLDOL]DomRGHPHGLFDPHQWRVHDOLPHQ
3~EOLFR
3~EOLFR D LQWHUIHUrQFLD
LQWHUIHUrQFLD H D LQWHUYHQomR
LQWHUYHQomR QD WRV
RUJDQL]DomRVLQGLFDO
RUJDQL]DomRVLQGLFDO ,9 IXQHUiULRV
9 WUDQVSRUWHFROHWLYR
 ,, pYHGDGD
YHGDGDDFULDomR
FULDomRGH
GHPDLV
PDLVGH
GHXPD
XPDRUJDQL]DomR 9, FDSWDomRHWUDWDPHQWRGHHVJRWRHOL[R
VLQGLFDO
VLQGLFDO HP TXDOTXHU JUDX UHSUHVHQWDWLYD GH 9,, WHOHFRPXQLFDo}HV
FDWHJRULDSURILVVLRQDORXHFRQ{PLFDQDPHVPD 9,,, JXDUGDXVRHFRQWUROHGHVXEVWkQFLDVUDGLRDWLYDVHTXLSD
EDVHWHUULWRULDO
EDVH WHUULWRULDOTXH
TXHVHUi
VHUiGHILQLGD
GHILQLGDSHORV
SHORVWUDEDOKD PHQWRVHPDWHULDLVQXFOHDUHV
GRUHVRXHPSUHJDGRUHVLQWHUHVVDGRVQmRSRGHQ ,; SURFHVVDPHQWRGHGDGRVOLJDGRVDVHUYLoRVHVVHQFLDLV
GRVHULQIHULRUjiUHDGHXP0XQLFtSLR ; FRQWUROHGHWUiIHJRDpUHR
;, FRPSHQVDomREDQFiULD
,,, DR
DRVLQGLFDWR
VLQGLFDWRFDEH
FDEHDGHIHVD
GHIHVDGRV
GRVGLUHLWRV
GLUHLWRVHLQWHUHV
VHV FROHWLYRV
VHV FROHWLYRV RX
RX LQGLYLGXDLV
LQGLYLGXDLV GD
GD FDWHJRULD
FDWHJRULD LQFOX †ž 2V
2VDEXVRV
DEXVRVFRPHWLGRV
FRPHWLGRVVXMHLWDPRVUHVSRQViYHLVjV
VLYHHPTXHVW}HVMXGLFLDLVRXDGPLQLVWUDWLYDV SHQDVGDOHL

,9 DDVVHPEOpLD
DVVHPEOpLDJHUDO
JHUDOIL[DUi
IL[DUiDFRQWULEXLomR
FRQWULEXLomRTXH
TXHHP ePDLVTXHMXVWDXPDSXQLomRDRVDEXVRVVHPpGLFRV
VH
VH WUDWDQGR
WUDWDQGR GH
GH FDWHJRULD
FDWHJRULD SURILVVLRQDO
SURILVVLRQDO VHUi
VHUi GHV RXHQIHUPHLURV HPJUHYHIHFKDUHPXP3URQWR6RFRUUR
FRQWDGD HP IROKD SDUD FXVWHLR GR VLVWHPD RXXPD87,GHXPKRVSLWDORXLPSHGLUHPTXHRXWURV
FRQIHGHUDWLYRGDUHSUHVHQWDomRVLQGLFDOUHVSHFWL FROHJDVSUHVWHPWDLVVHUYLoRVMXVWRVHUiTXHUHVSRQGDP
YD
YDLQGHSHQGHQWHPHQWH
LQGHSHQGHQWHPHQWHGDGDFRQWULEXLomR
FRQWULEXLomRSUHYLVWD QmRDSHQDVSHORVSUHMXt]RVFDXVDGRV SDJDQGRSHQVmRDR
HPOHL GHSHQGHQWHVGHDOJXPGHIXQWR FRPRWDPEpPLQGRSDUD
DFDGHLD9DOHUiRPHVPRUDFLRFtQLRSDUDRVJUHYLVWDVGH
9 QLQJXpP
QLQJXpPVHUi
VHUiREULJDGR
REULJDGRDILOLDUVH
ILOLDUVHRXDPDQWHUVH WRGDVDVGHPDLVFDWHJRULDVGHVHUYLoRVHVVHQFLDLVDYLGD
ILOLDGRDVLQGLFDWR FRPXQLWiULD
&kPDUD/HJLVODWLYDGR
  'LVWULWR)HGHUDO 'LU&RQVWLWXFLRQDO 

$UW eDVVHJXUDGD
DVVHJXUDGDDSDUWLFLSDomR
SDUWLFLSDomRGRV
GRVWUDEDOKDGRUHVH $~QLFDH[LJrQFLDpTXHRSDL EUDVLOHLUR RXDPmH EUD
HPSUHJDGRUHVQRVFROHJLDGRVGRVyUJmRVS~EOL
HPSUHJDGRU HVQRVFROHJLDGRVGRVyUJmRVS~EOL VLOHLUD HVWLYHVVHQRH[WHULRUDVHUYLoRGR%UDVLO6HHVWDYD
FRV
RV HP
HP TXH
TXH VHXV
VHXV LQWHUHVVHV
LQWHUHVVHV SURILVVLRQDLV
SURILVVLRQDLV RX HPSDVVHLR WXULVPR QmRVHUi DXWRPDWLFDPHQWH EUDVLOHLUR
SUHYLGHQFLiULRV
SUHYLGHQFLiULRV VHMDP
VHMDP REMHWR
REMHWR GH
GH GLVFXVVmR
GLVFXVVmR H VHUmRQHFHVViULDVDOJXPDVSURYLGrQFLDVSDUDTXHWDOFUL
GHOLEHUDomR DQoDSRVVDVHUFRQVLGHUDGDEUDVLOHLUDFRPRVHYr DVH
JXLU
$UW 1DV
1DVHPSUHVDV
HPSUHVDVGH
GHPDLV
PDLVGH
GHGX]HQWRVHPSUHJDGRV F RV
RV QDVFLGRVQRHVWUDQJHLURGHSDLEUDVLOHLUR
QDVFLGRVQRHVWUDQJHLURGHSDLEUDVLOHLUR
pDVVHJXUDGD
DVVHJXUDGDDHOHLomR
HOHLomRGH
GHXP
XPUHSUHVHQWDQWH
UHSUHVHQWDQWHGHV RX
RXGH
GHPmH EUDVLOHLUDGHVGH
PmHEUDVLOHLUD GHVGHTXH
TXHVHMDP
VHMDPUHJLVWUD
WHV
WHVFRP
FRPDILQDOLGDGH
ILQDOLGDGHH[FOXVLYD
H[FOXVLYDGH
GHSURPRYHUOKHV GRV
GRV HP
HP UHSDUWLomR
UHSDUWLomR EUDVLOHLUD
EUDVLOHLUD FRPSHWHQWH
RPSHWHQWH RX
RHQWHQGLPHQWRGLUHWRFRPRVHPSUHJDGRUHV YHQKDP
YHQKDPDUHVLGLU
UHVLGLUQD
QD5HS~EOLFD
5HS~EOLFD)HGHUDWLYD
)HGHUDWLYDGR
%UDVLO
%UDVLO H RSWHPHPTXDOTXHUWHPSRGHSRLV
GH
GHDWLQJLGD
DWLQJLGDDPDLRULGDGH
PDLRULGDGHSHOD
SHODQDFLRQDOLGDGH
EUDVLOHLUD
1DFLRQDOLGDGH 2EVUHGDomRGDGDSHOD(&Qž

6mR GXDV KLSyWHVHV SDUD R ILOKR GH SDL RX GH PmH
1DFLRQDLVVmRSHVVRDVFRPXPDPHVPDLGHQWLGDGHGH EUDVLOHLURQDVFLGRQRH[WHULRU D UHTXHUHUDR´YLUUHVLGLU
VROR H VDQJXH TXH FRPS}HP XP FRQFHLWR PDLV DPSOR QR%UDVLORX E PHVPRPRUDQGRQR([WHULRUSURYDUTXH
FKDPDGRSiWULD$QDomREUDVLOHLUDpDVVLPRDJUXSDPHQ DRQDVFHUIRL´UHJLVWUDGRHPUHSDUWLomREUDVLOHLUDFRPSH
WRGHWRGRVRVTXHVmRFRQVLGHUDGRVEUDVLOHLURV WHQWHµ

1RUPDOPHQWHHPFHUWRVDVSHFWRVR(VWDGR%UDVLOHLUR 6DWLVIHLWDV HVVDV FRQGLo}HV RV UHTXLVLWRV VmR D VXD


UHFRQKHFHUi DOJXQV HVSHFLDLV SULYLOpJLRV H SURWHomR DRV PDLRULGDGH SDUD PDQLIHVWDU VXD YRQWDGH GH DGTXLULU D
EUDVLOHLURV GLUHLWRGHFLGDGDQLDLVWRpYRWDUHVHUYRWDGR QDFLRQDOLGDGHEUDVLOHLUD
QDVHOHLo}HVRFXSDUFHUWRVFDUJRVS~EOLFRVVHUMXOJDGR
SHOD-XVWLoDEUDVLOHLUDQmRVHUH[WUDGLWDGRHWF3RULVVR $WHQomR D (&Qž  GLVFLSOLQRX D VLWXDomRGRV
p LPSRUWDQWH VDEHU TXHP p EUDVLOHLUR RX SRGH VHU ´ILOKRVGHEUDVLOHLURVµ SDLRXPmH QDVFLGRV
FRQVLGHUDGREUDVLOHLUR HQWUHPHLRDVGXDV(PHQGDV&RQVWLWXFLRQDLV
RX VHMD HQWUH D ´(PHQGD &RQVWLWXFLRQDO GH 5HYLVmR Qž
(VVHpRWHPDGHTXHWUDWDHVWHFDStWXORFRQVWLWXFLRQDO HD(&Qž³IDFXOWDQGRTXHHOHVDLQGDTXH
GHVLJQLILFDWLYDLPSRUWkQFLDSRLVFHUWRVFDUJRVS~EOLFRVVy WDUGLDPHQWH SRVVDP VHU UHJLVWVUDGRV HP UHSDUWLomR
VHUmRRFXSDGRVSRUEUDVLOHLURVQDWRVRXWURVWDPEpPSRU GLSORPiWLFDRXFRQVXODUEUDVLOHLUD³RXDWpPHVPRHP
EUDVLOHLURQDWXUDOL]DGR &DUWyULREUDVLOHLURVHYLHUHPDPRUDUQR%UDVLO

$UW6mREUDVLOHLURV
6mREUDVLOHLURV 3DUDWDQWRD(&Qž DUWž PDQGRXDFUHVFHU
RDUWDR$'&7³$WRGDV'LVSRVLo}HV&RQVWLWXFLRQDLV
, QDWRV 7UDQVLWyULDVTXHHVWDEHOHFH

D RV
RV QDVFLGRVQD5HS~EOLFD)HGHUDWLYDGR%UDVLO
QDVFLGRVQD5HS~EOLFD)HGHUDWLYDGR%UDVLO $UW 2VQDVFLGRVQRHVWUDQJHLURHQWUHGHMXQKRGHHD
DLQGDTXH
DLQGD TXHGH
GHSDLV
SDLVHVWUDQJHLURV
HVWUDQJHLURVGHVGH
GHVGHTXHHVWHV
TXHHVWHV GDWDGDSURPXOJDomRGHVWD(PHQGD&RQVWLWXFLRQDOILOKRV
QmRHVWHMDPDVHUYLoRGHVHXSDtV  GH SDL EUDVLOHLUR RX PmH EUDVLOHLUD SRGHUmR VHU UHJLVWUDGRV HP
UHSDUWLomRGLSORPiWLFDRXFRQVXODUEUDVLOHLUDFRPSHWHQWHRXHPRItFLR
eRFULWpULRGRMXVVROXVWHUULWRULDO&RPRVHYrSDUD GHUHJLVWURVHYLHUHPDUHVLGLUQD5HS~EOLFD)HGHUDWLYDGR%UDVLO
QRVVD&RQVWLWXLomREDVWDTXHDSHVVRDQDVoDQRWHUULWyULR 
QDFLRQDOSDUDVHUFRQVLGHUDGRXPEUDVLOHLURQDWR1mR
LPSRUWDVHpILOKRGHUXVVRRXFXEDQRRXDPHULFDQRRX $UW6mREUDVLOHLURV
6mREUDVLOHLURV
LQJOrVHWF 

1DVFHXDTXLpEUDVLOHLUR6yQmRVHUiEUDVLOHLURVHRV ,,QDWXUDOL]DGR
QDWXUDOL]DGR
SDLVIRUHPHVWUDQJHLURVTXHHVWHMDPQR%UDVLODVHUYLoRGH
VHX3DtV1HVVDKLSyWHVH R FDVDOHVWiUHSUHVHQWDQGRRV 1DWXUDOL]DGRVVmRSHVVRDVTXHQmRVmREUDVLOHLUDVQHP
LQWHUHVVHVGHVHX3DtVGHRULJHP1mRHVWiQR%UDVLOSRU SRUWHUHPQDVFLGRQRWHUULWyULRQDFLRQDOQHPSRUVHUHP
YRQWDGH SUySULD RX DPRU j WHUUD 3RU LVVR PHVPR TXH ILOKDVGHEUDVLOHLURVVmRHVWUDQJHLURVTXHVHWUDQVIRUPD
WHQKDQDVFLGRDTXLQmRVHUiEUDVLOHLUR UmRHPEUDVLOHLURVSRUXPDWRGHVXDYRQWDGH RSomR 

E RV
RVQDVFLGRVQRHVWUDQJHLURGHSDLEUDVLOHLURRX
QDVFLGRVQRHVWUDQJHLURGHSDLEUDVLOHLURRX $&RQVWLWXLomRRVFRORFDHPSRVLomRMXUtGLFDXPSRXFR
PmHEUDVLOHLUD
PmH EUDVLOHLUDGHVGH
GHVGHTXH
TXHTXDOTXHU
TXDOTXHUGHOHV
GHOHVHVWHMD
HVWHMDD LQIHULRUjGREUDVLOHLURQDWRFRPRYHUHPRVDGLDQWH
VHUYLoRGD5HS~EOLFD)HGHUDWLYDGR%UDVLO
D RV
RVTXH
TXHQD
QDIRUPD
IRUPDGD
GDOHL
OHLDGTXLUDP
DGTXLUDPDQDFLRQD
1mRVHGHVSUH]RXSRUpPRFULWpULRGRMXVVDQJLQLV OLGDGH
OLGDGH EUDVLOHLUD H[LJLGD DRV RULJLQiULRV GH
RXVHMDGRVDQJXHEUDVLOHLUR$VVLPVHDFULDQoDQDVFHX SDtVHVGHOtQJXDSRUWXJXHVDDSHQDVUHVLGrQ
QRHVWUDQJHLURPDVRSDLRXDPmHpEUDVLOHLURRILOKR FLD
FLD SRU
SRU XP
XP DQR
DQR LQLQWHUUXSWR
LQLQWHUUXSWR H LGRQHLGDGH
LGRQHLGDGH
VHUiWDPEpPEUDVLOHLUR PRUDO
 'LU&RQVWLWXFLRQDO
 &kPDUD/HJLVODWLYDGR'LVWULWR)HGHUDO
E RV
RV HVWUDQJHLURV
HVWUDQJHLURV GH
GH TXDOTXHU
TXDOTXHU QDFLRQDOLGDGH D GH
GH UHFRQKHFLPHQWR
UHFRQKHFLPHQWR GH QDFLRQDOLGDGH
UHVLGHQWHV
UHVLGHQWHVQD
QD5HS~EOLFD
5HS~EOLFD)HGHUDWLYD
)HGHUDWLYDGR
GR%UDVLO RULJLQiULDSHODOHLHVWUDQJHLUD
RULJLQiULDSHODOHLHVWUDQJHLUD
Ki
Ki PDLV GH TXLQ]H
PDLV GH TXLQ]H DQRV
DQRV LQLQWHUUXSWRVHVHP
LQLQWHUUXSWRVHVHP
FRQGHQDomR
FRQGHQDomR SHQDO
SHQDO GHVGH
GHVGH TXH
TXH UHTXHLUDP
UHTXHLUDP D E GH LPSRVLomR GH
GH LPSRVLomR GH QDWXUDOL]DomR
QDWXUDOL]DomR SHOD
SHOD QRU
QDFLRQDOLGDGHEUDVLOHLUD PD
PDHVWUDQJHLUD
HVWUDQJHLUDDR
DREUDVLOHLUR
EUDVLOHLURUHVLGHQWH
UHVLGHQWHHP
2EVHVWHWH[WRIRLLQWURGX]LGRSHOD(PHQGD&RQVWLWXFLRQDOGH5HYLVmRQž (VWDGR
VWDGR HVWUDQJHLUR
HVWUDQJHLUR FRPR
FRPR FRQGLomR
FRQGLomR SDUD
5HGX]LXVHRSUD]RGHUHVLGrQFLDSDUDDQRVMiTXHRYHOKR
DH[LJLDSRUPDLVGHWULQWDDQRV
SHUPDQrQFLD
SHUPDQrQFLD HPHP VHX
VHX WHUU
WHUULWyULR
LWyULR RX
RX SDUD
SDUD R
H[HUFtFLRGHGLUHLWRVFLYLV
†ž $RV
$RVSRUWXJXHVHV
SRUWXJXHVHVFRP
FRPUHVLGrQFLDSHUPDQHQWHQR
2EVHVWHWH[WRIRLLQWURGX]LGRSHOD(PHQGD
2EV (PHQGD
(PHQGD&RQVWLWXFLRQDO
&RQVWLWXFLRQDOGH
GH5HYLVmR
5HYLVmRQž
Qž
GH
3DtV
3DtVVH
VHKRXYHU
KRXYHUUHFLSURFLGDGH
UHFLSURFLGDGHHPHPIDYRU
IDYRUGH
GHEUDVL  2 YHOKR WH[WR QmR
 QmR  SRVVLELOLWDYD HP QHQKXPD KLSyWHVH TXH XP
OHLURV
OHLURV VHUmR
VHUmR DWULEXtGRV
DWULEXtGRV RV
RV GLUHLWRV
GLUHLWRV LQHUHQWHV
LQHUHQWHV DR EUDVLOHLURWLYHVVHGXSODQDFLRQDOLGDGH
GXSODQDFLRQDOLGDGHFRPRpDJRUDSHUPLWLGR
GXSODQDFLRQDOLGDGH

EUDVLOHLUR
EUDVLOHLURVDOYR
VDOYRRV
RVFDVRV
FDVRVSUHYLVWRV
SUHYLVWRVQHVWD
QHVWD&RQVWL $UW $OtQJXD
OtQJXDSRUWXJXHVD
SRUWXJXHVDpRLGLRPD
LGLRPDRILFLDO
RILFLDOGD
GD5HS~
WXLomR
2EV
2EV HVWHWH[WRIRLLQWURGX]LGRSHOD(PHQGD&RQVWLWXFLRQDO EOLFD)HGHUDWLYDGR%UDVLO
GH5HYLVmRQžGH

1RVVDUHODomRFRPDSiWULDPmHpULFDGHFDULQKRH †ž 6mR


6mRVtPERORV
VtPERORVGD
GD5HS~EOLFD
5HS~EOLFD)HGHUDWLYD
)HGHUDWLYDGR%UDVLO
UHVSHLWR UHFtSURFRV 9HMDVH TXH WDQWD p QRVVD FRQVL DEDQGHLUDRKLQRDVDUPDVHRVHORQDFLRQDLV
GHUDomR FRP RV SRUWXJXHVHV TXH QRVVD &RQVWLWXLomR
HVWDEHOHFH TXH VH HP 3RUWXJDO RV EUDVLOHLURV IRUHP †ž 2V (VWDGRV
2V (VWDGRV R 'LVWULWR )HGHUDO HRV0XQLFtSLRV
WUDWDGRVFRPLJXDOGDGHRVSRUWXJXHVHVDTXLVHUmRFRQVL SRGHUmRWHUVtPERORVSUySULRV
SRGHUmRWHUVtPERORVSUySULRV
GHUDGRVEUDVLOHLURV TXDOEUDVLOHLURQDWXUDOL]DGR
QDWXUDOL]DGR 
QDWXUDOL]DGR
6LJQLILFDTXHSRUWXJXrVSRGHUiYRWDUHVHUYRWDGRSDUD
3UHIHLWRHDWpSDUDR6HQDGR)HGHUDOVHUMXL]GHGLUHLWR
>H[FHWRGR6XSUHPR7ULEXQDO)HGHUDO@EDVWDQGRSDUDLVVRTXH 'LUHLWRV3ROtWLFRV
WHQKD UHVLGrQFLD SHUPDQHQWH QR %UDVLO H TXH 3RUWXJDO
RIHUHoDDVPHVPDVUHJDOLDVDRVEUDVLOHLURVTXHOiUHVLGDP
4XHPYLYHHPVRFLHGDGHGHYHWHURGLUHLWRGHSDUWLFL
†ž $ OHL QmR SRGHUi HVWDEHOHFHUGLVWLQomRHQWUH SDU GH WRGD VXD  RUJDQL]DomR DGPLQLVWUDWLYD HQILP GD
EUDVLOHLURV
EUDVLOHLURVQDWRV
QDWRVHQDWXUDOL]DGRV
QDWXUDOL]DGRVVDOYR
VDOYRQRV
QRVFDVRV JHUrQFLDGRVLQWHUHVVHVFRPXQV$HVVHGLUHLWRGHSDUWLFL
SUHYLVWRVQHVWD&RQVWLWXLomR SDomR GLUHWD VHQGR YRWDGR FRPR PDQGDWiULR GRV
FLGDGmRV RXLQGLUHWD SHORYRWRHVFROKHQGRVHXVPDQGD
8PDYH]QDWXUDOL]DGRRHVWUDQJHLURHOHSDVVDDVHUWmR WiULRV GiVHRQRPHGHGLUHLWRVSROtWLFRV
EUDVLOHLURTXDQWRREUDVLOHLURQDWR$&RQVWLWXLomRSURtEH
TXHHOHVRIUDTXDOTXHUGLVFULPLQDomR1HPPHVPRDOHL (QVLQD$/(;$1'5('( 025$(6TXHRVGLUHLWRVSROtWLFRV
SRGHUi HVWDEHOHFHU TXDOTXHU GLIHUHQoD >SULYLOpJLR RX VmR´GLUHLWRVS~EOLFRVVXEMHWLYRVTXHLQYHVWHPRLQGLYtGXRQR
WUDWDPHQWRGLIHUHQFLDGR@$SHQDVDSUySULD&RQVWLWXLomR VWDWXVDFWLY FLYLWDWLVSHUPLWLQGROKHRH[HUFtFLRFRQFUHWR
pTXHGHILQHDVUHVWULo}HVDRVEUDVLOHLURVQDWXUDOL]DGRVH GD OLEHUGDGH GH SDUWLFLSDomR QRV QHJyFLRV SROtWLFRV GR
DLQGDDVVLPSRUUD]}HVGRPDLRULQWHUHVVHQDFLRQDO>YHMD (VWDGRGHPDQHLUDDFRQIHULURVDWULEXWRVGDFLGDGDQLDµ³
RVFRPHQWiULRVDR†VHJXLQWH@ LQ´&RQVWLWXLomRGR%UDVLO,QWHUSUHWDGDµHG$7/$663LWHP
S 
†ž
 6mRSULYDWLYRVGHEUDVLOHLURQDWRRVFDUJRV
5HFRQKHFHPVHFRPRGLUHLWRVSROtWLFRV D RGLUHLWRGH
, GH3UHVLGHQWHH9LFH3UHVLGHQWHGD5HS~EOLFD VXIUiJLR E DDOLVWDELOLGDGHUHTXLVLWRSDUDRH[HUFtFLRGR
,, GH3UHVLGHQWHGD&kPDUDGRV'HSXWDGRV GLUHLWRGHYRWDUHPHOHLo}HVSOHELVFLWRVHUHIHUHQGRV F
,,, GH3UHVLGHQWHGR6HQDGR)HGHUDO DHOHJLELOLGDGHRXVHMDRGLUHLWRGHVHUFDQGLGDWRHYRWDU
,9 GH0LQLVWURGR6XSUHPR7ULEXQDO)HGHUDO G RGLUHLWRDSDUWLFLSDUHVHLQWHJUDUDLQLFLDWLYDSRSXODU
9 GHFDUUHLUDGLSORPiWLFD GHSURSRVLWXUDGHOHL H RGLUHLWRGHSURSRUDomRSRSX
9, GHRILFLDOGDV)RUoDV$UPDGDV ODU I  R GLUHLWR D RUJDQL]DU H SDUWLFLSDU GH SDUWLGRV
9,,GH0LQLVWURGH(VWDGRGD'HIHVD
GH0LQLVWURGH(VWDGRGD'HIHVD SROtWLFRV J HWF
2EV DOtQHDLQWURGX]LGDSHOD(PHQGD&RQVWLWXFLRQDOQž
0DVQmREDVWDTXHXPDSHVVRDYLYDHPVRFLHGDGHSDUD
2EV $SUySULD&RQVWLWXLomR)HGHUDOWDPEpPGHFODUDTXH TXHHODSRVVDSDUWLFLSDURXVHMDWHUGLUHLWRVSROtWLFRV
VyRVEUDVLOHLURVQDWRV
QDWRVSRGHUmRVHUQRPHDGRVSDUD
QDWRV 8PDSHVVRDQmRQDVFHFRPGLUHLWRVSROtWLFRVDGTXLUHRV
R&RQVHOKRGD5HS~EOLFD DUW9,,  DR ORQJR GH VXD YLGD VDWLVID]HQGR DOJXQV UHTXLVLWRV
OHJDLV PDVWDPEpPSRGHSHUGrORV
†ž 6HUi GHFODUDGD D SHUGD GDQDFLRQDOLGDGHGR
EUDVLOHLURTXH FLGDGDQLDQRGL]HUGR3URI0$12(/*21d$/9(6
$FLGDGDQLD
FLGDGDQLD
)(55(,5$),/+2pXPVWDWXVOLJDGRDRUHJLPHSROtWLFR
, WLYHUFDQFHODGDVXDQDWXUDOL]DomR
WLYHUFDQFHODGDVXDQDWXUDOL]DomRSRUVHQWHQoD $FLGDGDQLDDWLYDFRQVLVWHHPSRGHUHVFROKHUDSDVVLYDHP
MXGLFLDO
MXGLFLDO HP
HP YLUWXGH
YLUWXGH GH
GH DWLYLGDGH
DWLYLGDGH QRFLYD
QRFLYD DR DOpPGHHVFROKHUSRGHUVHUHVFROKLGR ³ LQ
&XUVRGH'LUHLWR
LQWHUHVVHQDFLRQDO &RQVWLWXFLRQDO
HG6DUDLYDS $VVLPRDQDOIDEHWRpDSHQDV
PHLRFLGDGmRSRLVWHPRGLUHLWRGHYRWDUPDVQmRp
DGTXLULURXWUDQDFLRQDOLGDGHVDOYRQRVFDVRV
,, DGTXLULURXWUDQDFLRQDOLGDGHVDOYRQRVFDVRV HOHJtYHOpGRWDGRDSHQDVGDFLGDGDQLDDWLYD
 &kPDUD/HJLVODWLYDGR
 'LVWULWR)HGHUDO 'LU&RQVWLWXFLRQDO 

$UW $VREHUDQLD
VREHUDQLDSRSXODU
SRSXODUVHUi
VHUiH[HUFLGD
H[HUFLGDSHOR
SHORVXIUiJLR SURLELDDUHHOHLomRQRVVHJXLQWHVWHUPRV´6mRLQHOHJtYHLVSDUDRVPHVPRVFDUJRVQR
SHUtRGR VXEVHTHQWH R 3UHVLGHQWH GD 5HS~EOLFD RV *RYHUQDGRUHV GH (VWDGR H GR
XQLYHUVDO
XQLYHUVDOHSHOR
SHORYRWR
YRWRGLUHWR
GLUHWRHVHFUHWR
VHFUHWRFRP
FRPYDORU 'LVWULWR)HGHUDORV3UHIHLWRVHTXHPRVKRXYHUVXFHGLGRRXVXEVWLWXtGRQRVVHLVPHVHV
DQWHULRUHVDRSOHLWRµ
LJXDO
LJXDOSDUD
SDUDWRGRV
WRGRVH
HQRV
QRVWHUPRV
WHUPRVGD
GDOHL
OHLPHGLDQWH
†ž 3DUD
3DUDFRQFRUUHUHP
FRQFRUUHUHPDRXWURVFDUJRVR3UHVLGHQWH
, SOHELVFLWR GD 5HS~EOLFD
GD 5HS~EOLFD RV *RYHUQDGRUHV GH (VWDGR H R
,, UHIHUHQGR 'LVWULWR
'LVWULWR)HGHUDO
)HGHUDOHRV
RV3UHIHLWRVGHYHPUHQXQFLDU
3UHIHLWRVGHYHPUHQXQFLDU
,,, LQLFLDWLYDSRSXODU DRV
DRVUHVSHFWLYRV
UHVSHFWLYRVPDQGDWRV
PDQGDWRVDWp
DWpVHLV
VHLVPHVHV
PHVHVDQWHV
DQWHVGR
SOHLWR
†ž2DOLVWDPHQWRHOHLWRUDOHRYRWRVmR
2DOLVWDPHQWRHOHLWRUDOHRYRWRVmR
†ž 6mR
6mRLQHOHJtYHLV
LQHOHJtYHLVQR
QRWHUULWyULR
WHUULWyULRGH
GHMXULVGLomR
MXULVGLomRGRWL
, REULJDWyULRVSDUDRVPDLRUHVGHGH]RLWRDQRV WXODURF{QMXJH
WXODU F{QMXJHHRV
RVSDUHQWHV
SDUHQWHVFRQVDQJtQHRV
FRQVDQJtQHRVRX
DILQV
DILQV DWp
DWp R VHJXQGR
VHJXQGR JUDX RX SRU DGRomR GR
,, IDFXOWDWLYRVSDUD 3UHVLGHQWH
3UHVLGHQWH GD 5HS~EOLFD GH *RYHUQDGRU GH
(VWDGR
(VWDGRRX
RX7HUULWyULR
7HUULWyULRGR
GR'LVWULWR
'LVWULWR)HGHUDO
)HGHUDOGH
GH3UH
D RVDQDOIDEHWRV IHLWR
IHLWRRX
RXGH
GHTXHP
TXHPRV
RVKDMD
KDMDVXEVWLWXtGR
VXEVWLWXtGRGHQWUR
GHQWURGRV
E RVPDLRUHVGHVHWHQWDDQRV
VHLV
VHLVPHVHV
PHVHVDQWHULRUHV
DQWHULRUHVDR
DRSOHLWR
SOHLWRVDOYR
VDOYRVH
VHMi
MiWLWXODU
F RV
RVPDLRUHV
PDLRUHVGH
GHGH]HVVHLV
GH]HVVHLVHPHQRUHV
PHQRUHVGH
GHGH]RLWR
GHPDQGDWRHOHWLYRHFDQGLGDWRjUHHOHLomR
DQRV
†ž 2 PLOLWDU DOLVWiYHO p HOHJtYHODWHQGLGDVDV
†ž 1mR
1mRSRGHP
SRGHPDOLVWDUVH
DOLVWDUVHFRPR
FRPRHOHLWRUHVRVHVWUDQJH
VHJXLQWHVFRQGLo}HV
LURV
LURV H GXUDQWH R SHUtRGR GR VHUYLoR PLOLWDU
REULJDWyULRRVFRQVFULWRV
, VHFRQWDUPHQRVGHGH]DQRVGHVHUYLoRGHYHUi
VHFRQWDUPHQRVGHGH]DQRVGH
DIDVWDUVHGDDWLYLGDGH
†ž 6mRFRQGLo}HVGHHOHJLELOLGDGHQDIRUPDGDOHL
6mRFRQGLo}HVGHHOHJLELOLGDGHQDIRUPDGDOHL
,, VH
VHFRQWDU
FRQWDUPDLV
PDLVGH
GHGH]
GH]DQRV
DQRVGH
GHVHUYLoR
VHUYLoRVHUiDJUH
, DQDFLRQDOLGDGHEUDVLOHLUD
JDGR
JDGR SHOD
SHOD DXWRULGDGH
DXWRULGDGH VXSHULRU
VXSHULRU H VH HOHLWR
,, RSOHQRH[HUFtFLRGRVGLUHLWRVSROtWLFRV
SDVVDUi
SDVVDUiDXWRPDWLFDPHQWH
DXWRPDWLFDPHQWHQRDWRGDGLSORPDomR
QRDWRGDGLSORPDomR
,,, RDOLVWDPHQWRHOHLWRUDO
SDUDDLQDWLYLGDGH
,9 RGRPLFtOLRHOHLWRUDOQDFLUFXQVFULomR
9 DILOLDomRSDUWLGiULD
†ž /HL
/HL FRPSOHPHQWDU
FRPSOHPHQWDU HVWDEHOHFHUi
HVWDEHOHFHUi RXWURVFDVRVGH
RXWURVFDVRVGH
9, DLGDGHPtQLPDGH
LQHOHJLELOLGDGHHRVSUD]RVGHVXDFHVVDomRD
LQHOHJLELOLGDGHHRVSUD]RVGHVXDFHVVDomRD
ILP  GH SURWHJHU D SURELGDGH
ILP URELGDGH DGPLQLVWUDWLYD
DGPLQLVWUDWLYD D
D WULQWD
WULQWD H FLQFR
FLQFR DQRV
DQRV SDUD 3UHVLGHQWH H 9L
PRUDOLGDGH
PRUDOLGDGHSDUD
SDUDRH[HUFtFLR GRPDQGDWR
H[HUFtFLRGR PDQGDWRFRQVLGH
FH3UHVLGHQWHGD5HS~EOLFDH6HQDGRU
UDGD
UDGDDYLGD
YLGDSUHJUHVVD
SUHJUHVVDGRFDQGLGDWRHD
GRFDQGLGDWRHD
DQRUPDOL
GDGH
GDGH H OHJLWLPLGDGH
OHJLWLPLGDGH  GDV HOHLo}HV FRQWUD D LQ
E WULQWD
WULQWD DQRV
DQRV SDUD *RYHUQDGRU H 9LFH*RYHU
IOXrQFLD
IOXrQFLD GR
GR SRGHU
SRGHU HFRQ{PLFR RX R DEXVR GR
QDGRUGH(VWDGRHGR'LVWULWR)HGHUDO
H[HUFtFLR
H[HUFtFLRGH
GHIXQomR
IXQomRFDUJR
FDUJRRX
RXHPSUHJR
HPSUHJRQDQDDGPL
QLVWUDomRGLUHWDRXLQGLUHWD
F  YLQWH H XP DQRV
DQRV SDUD 'HSXWDGR)HGHUDO
'HSXWDGR (VWDGXDO RX 'LVWULWDO 3UHIHLWR 9L 2EV FRPD(PHQGD&RQVWLWXFLRQDOGH5HYLVmRQžHVWH†žIRLDOWHUDGRSDUDQHOH
VHLQFOXLUGHQWUHDVFRQGLo}HVGHHOHJLELOLGDGHRREMHWLYRGHSURWHJHUWDPEpPD
FH3UHIHLWRHMXL]GHSD] SURELGDGHDGPLQLVWUDWLYDHDGHPRUDOLGDGHSDUDRH[HUFtFLRGRPDQGDWRFRQ
VLGHUDGDDYLGDSUHJUHVVDGRFDQGLGDWR

G GH]RLWRDQRVSDUD9HUHDGRU
GH]RLWRDQRV † 2PDQGDWR
PDQGDWRHOHWLYR
HOHWLYRSRGHUi
SRGHUiVHU
VHULPSXJQDGR
LPSXJQDGRDQWH
DQWHD
-XVWLoD (OHLWRUDOQRSUD]RGHTXLQ]HGLDVFRQ
-XVWLoD (OHLWRUDOQRSUD]RGHTXLQ]HGLDVFRQ
†ž 6mRLQHOHJtYHLVRVLQDOLVWiYHLVHRVDQDOIDEHWRV WDGRV
DGRV GD
GD GLSORPDomR
GLSORPDomR LQVWUXtGD
LQVWUXtGD D DomR
DomR FRP
SURYDVGH
SURYDV GHDEXVR
DEXVRGR
GRSRGHU
SRGHUHFRQ{PLFR
HFRQ{PLFRFRUUXSomR
,QHOHJtYHOTXHUGL]HUTXHDSHVVRDQmRSRGHUiQHPVHU RXIUDXGH
FDQGLGDWDQHPVHUHOHLWDSDUDTXDOTXHUFDUJRHOHWLYR'H
UHJUDWRGDSHVVRDTXHQmRSXGHUVHDOLVWDURXVHMDWLUDU † $DomR
DomRGH
GHLPSXJQDomR
LPSXJQDomRGHGHPDQGDWR
PDQGDWRWUDPLWDUi
WUDPLWDUiHP
RWtWXORGHHOHLWRUWDPEpPQmRSRGHUiVHUHOHLWD VHJUHGRGH
VHJUHGR GHMXVWLoD
MXVWLoDUHVSRQGHQGR
UHVSRQGHQGRRDXWRU
DXWRUQD
QDIRU
PDGDOHLVHWHPHUiULDRXGHPDQLIHVWDPiIp
$PDLRUFXULRVLGDGHSRUpPUHIHUHVHDRVDQDOIDEHWRV
D FDSDFLGDGH SROtWLFD GHVWHV OLPLWDVH D DSHQDV SRG
SRGHU $UW e YHGDGD D FDVVDomR GH GLUHLWRV SROtWLFRVFXMD
YRWDUQmRWrPWRGDYLDRGLUHLWRGHVHHOHJHU QHPGHVH
YRWDU SHUGDRXVXVSHQVmRVyVHGDUiQRVFDVRVGH
FDQGLGDWDUHP SDUDTXDOTXHUFDUJRHOHWLYR3RULVVRWrP
PHLDFLGDGDQLD , FDQFHODPHQWRGDQDWXUDOL]DomRSRUVHQWHQoD
WUDQVLWDGDHPMXOJDGR
†ž 23UHVLGHQWHGD5HS~EOLFD
3UHVLGHQWHGD5HS~EOLFDRV RV*RYHUQDGRUHV
*RYHUQDGRUHVGH
(VWDGRHGR
(VWDGR GR'LVWULWR
'LVWULWR)HGHUDO
)HGHUDORV
RV3UHIHLWRV
3UHIHLWRVHTXHP ,, LQFDSDFLGDGHFLYLODEVROXWD
RVKRXYHUVXFHGLGRRXVXEVWLWXtGRQRFXUVR
RVKRXYHUVXFHGLGRRXVXEVWLWXtGRQRFXUVRGRV
PDQGDWRV
PDQGDWRV SRGHUmR
SRGHUmR VHUVHU UHHOHLWRV
UHHOHLWRV SDUD
SDUD XP~QLFR eFDVRGHSHUGD6HDSHVVRDILFRXJDJiSRUFRPSOHWR
SHUtRGRVXEVHTHQWH REYLDPHQWHQHPWHUiFRQGLo}HVGHYRWDU QmRVDEHRTXH
2EV HVWH†žWHYHVXDUHGDomRDOWHUDGDSHOD(&Qž GH 2WH[WRDQWHULRU HVWi ID]HQGR  QHP GH VHU YRWDGD 2EVHUYHVH TXH D
 'LU&RQVWLWXFLRQDO &kPDUD/HJLVODWLYDGR'LVWULWR)HGHUDO


DEVROXWDTXHU
&RQVWLWXLomRUHIHUHVHjLQFDSDFLGDGHFLYLODEVROXWD
DEVROXWD ,,, SUHVWDomRGHFRQWDVj-XVWLoD(OHLWRUDO
GL]HUQmRKiSHUGDGRVGLUHLWRVSROtWLFRVFDVRDLQFDSDFL
GDGH VHMD UHODWLYD D LQWHUGLomR GH XP SUyGLJR SRU ,9 IXQFLRQDPHQWR
IXQFLRQDPHQWRSDUODPHQWDU
SDUODPHQWDUGHDFRUGRFRP
GHDFRUGRFRPDOHL
H[HPSORQmRRLPSHGLUiGHYRWDU 
1RXWUDVSDODYUDV3DUWLGRQmRSRGHWHUXPGRQR1HOH
,,, FRQGHQDomR FULPLQDO WUDQVLWDGDHPMXOJDGR GHYHUiKDYHUXPSDUODPHQWRLVWRpXP&ROHJLDGRHP
HQTXDQWRGXUDUHPVHXVHIHLWRV TXH RV ILOLDGRV RX VHXV 'HOHJDGRV YRWDUmR H WRPDUmR
GHFLV}HVVREUHRVGHVWLQRVGR3DUWLGR
,9 UHFXVD
UHFXVDGH
GHFXPSULU
FXPSULUREULJDomR
REULJDomRDWRGRVLPSRVWDRX
SUHVWDomR
SUHVWDomRDOWHUQDWLYD
DOWHUQDWLYDQRV
QRVWHUPRV
WHUPRVGR
GRDUW
DUWž
ž9,,, †ž e DVVHJXUDGD DRVSDUWLGRVSROtWLFRVDXWRQRPLD
SDUD
SDUD GHILQLUVXDHVWUXWXUDLQWHUQDRUJDQL]DomR
GHILQLUVXDHVWUXWXUDLQWHUQDRUJDQL]DomR
'HVVD IRUPD VH XPD SHVVRD VH UHFXVDU D SUHVWDU R H IXQFLRQDPHQWR
IXQFLRQDPHQWR H SDUD
SDUD DGRWDU
DGRWDU RV FULWpULRV GH
VHUYLoRPLOLWDUSHUGHUi RGLUHLWRGHYRWDUHVHUYRWDGR HVFROKD
HVFROKDHRUHJLPH
UHJLPHGH
GHVXDV
VXDVFROLJDo}HV
FROLJDo}HVHOHLWRUDLV
PDVUHDGTXLULUiVHXVGLUHLWRVSROtWLFRVDVVLPTXHKRXYHU VHP
VHP REULJDWRULHGDGH
REULJDWRULHGDGH GH
GH YLQFXODomR
YLQFXODomR HQW HQWUHUH DV
FXPSULGR FRP VHX GHYHU GH FLGDGmR $QRWHVH TXH WDO FDQGLGDWXUDV
FDQGLGDWXUDV HP kPELWR QDFLRQDO HVWDGXDO
VXVSHQVmRVyVHGDUiFDVRDUHFXVDGHFXPSULPHQWRVH GLVWULWDO
GLVWULWDO RX PXQLFLSDO GHYHQGR VHXV HVWDWXWRV
UHILUDDXPDREULJDomRLPSRVWDJHQHUDOL]DGDPHQWHDWRGR HVWDEHOHFHU
HVWDEHOHFHU QRUPDV
QRUPDV GH
GH GLVFLSOLQD
GLVFLSOLQD H ILGHOLGDGH
PXQGR0HVPRDVVLPXPDREULJDomRLPSRVWDSRUOHL SDUWLGiULD 2EVUHGDomRDOWHUDGDSHOD(&Qž

$WHQomR FDVRHPVHXFRQFXUVRVHMDIRUPXODGDTXHVWmR †ž 2V


2VSDUWLGRV
SDUWLGRVSROtWLFRV
SROtWLFRVDSyVDGTXLULUHPSHUVRQDOL
VREUHVHUFDVRGHVXVSHQVmRRXGHSHUGDUHV GDGH MXUtGLFD
GDGH MXUtGLFD QD
QD IRUPD
IRUPD GD
GD OHL
OHL FLYLOUHJLVWUDUmR
FLYLOUHJLVWUDUmR
SHUGDFRQFXUVRS~EOLFRQmRp
SRQGDWUDWDUVHGHFDVRGHSHUGD
SHUGD VHXVHVWDWXWRVQR7ULEXQDO6XSHULRU(OHLWRUDO
OXJDUSDUDGHIHVDGHWHVHVRXFRQYLFo}HVSHVVRDLVHVLP
SDUDUHVSRQGHUGHDFRUGRFRPRSHQVDPHQWRSUHGRPL †ž 2VSDUWLGRVSROtWLFRVWrPGLUHLWRDUHFXUVRVGR
QDQWH(QRFDVRDGRXWULQDWRGDDILUPDWUDWDUVHGHFDVR IXQGRSDUWLGiULRHDFHVVRJUDWXLWRDRUiGLRHj
IXQGRSDUWLGiULRHDFHVVRJUDWXLWRDRUiGLRHj
GHSHUGD WHOHYLVmRQDIRUPDGDOHL

9 LPSURELGDGH DGPLQLVWUDWLYDQRVWHUPRVGR †ž eYHGDGD


YHGDGDDXWLOL]DomR
XWLOL]DomRSHORVSDUWLGRVSROtWLFRV
SHORVSDUWLGRVSROtWLFRVGH
DUW†ž RUJDQL]DomRSDUDPLOLWDU
RUJDQL]DomRSDUDPLOLWDU
(VVD SHQD Vy FDEHUi VH R DGPLQLVWUDGRU FRPHWHU
LPSURELGDGHDGPLQLVWUDWLYDRXVHMDDWRLQGLJQRGHVR
QHVWR QD JHVWmR GRV LQWHUHVVHV S~EOLFRV RX VHMD QR
H[HUFtFLRGDDGPLQLVWUDomRS~EOLFD 2UJDQL]DomRGR(VWDGR
$UW $OHL
OHLTXH
TXHDOWHUDU
DOWHUDURSURFHVVR
SURFHVVRHOHLWRUDO
HOHLWRUDOHQWUDUiHP 2UJDQL]DomR3ROtWLFR$GPLQLVWUDWLYD
YLJRU QDGDWD
YLJRUQD GDWDGH
GHVXD
VXDSXEOLFDomR
SXEOLFDomRQmR
QmRVH
VHDSOLFDQ
GRjHOHLomRTXH
GRjHOHLomRTXHRFRUUD
RFRUUDDWp
DWp XP 
XP DQR
DQRGDGDWD
GHVXDYLJrQFLD
2 DUWž GD &RQVWLWXLomR )HGHUDO SURFODPD TXH $
5HS~EOLFD )HGHUDWLYD GR %UDVLO IRUPDGD SHOD XQLmR
LQGLVVRO~YHOGRV(VWDGRV
(VWDGRV
(VWDGRVH0XQLFtSLRV
0XQLFtSLRVHGR
GR'LVWULWR
'LVWULWR)HGHUDO
)HGHUDO
3DUWLGRV3ROtWLFRV FRQVWLWXLVHHP(VWDGR'HPRFUiWLFRGH'LUHLWR

$QDWXUH]D) )HGHUDWLYDGD5HS~EOLFDGR%UDVLOUHYHOD
HGHUDWLYD
$UW eOLYUH
OLYUHDFULDomR
FULDomRIXVmR
IXVmRLQFRUSRUDomR
LQFRUSRUDomRHH[WLQomR VXDIRUPDGR(VWDGRRXVHMDIUDFLRQDGRHPSURYtQFLDV
GH
GH SDU
SDUWLGRV
WLGRV SROtWLFRV
SROtWLFRV UHVJXDUGDGRV
UHVJXDUGDGRV D VREHUDQLD RV(VWDGRV
(VWDGRV
(VWDGRVPHPEURV HPXQLFtSLRVFRPDXWRQRPLDDG
PHPEURV
QDFLRQDO
QDFLRQDORUHJLPHGHPRFUiWLFRRSOXULSDUWLGD PLQLVWUDWLYDHILQDQFHLUD
ULVPR
ULVPRRV
RVGLUHLWRVIXQGDPHQWDLVGD
GLUHLWRVIXQGDPHQWDLV GDSHVVRD
SHVVRDKXPD
QDHREVHUYDGRVRVVHJXLQWHVSUHFHLWRV 7RGRVMXQWRVFRPS}HPXPDXQLGDGHPRQROtWLFD8P
JRYHUQRFHQWUDO GD8QLmR DGPLQLVWUDRVLQWHUHVVHVGD
, FDUiWHUQDFLRQDO
FDUiWHUQDFLRQDO WRWDOLGDGH$UHSDUWLomRGHFRPSHWrQFLDVHQWUHD8QLmRHRV
(VWDGRVPHPEURVFRQVWLWXLRIXOFURGR(VWDGRIHGHUDOFRPR
6LJQLILFDTXHQmRSRGHVHUIXQGDGRXP3DUWLGRSDUD REVHUYD0HVWUH-26e$)2162'$6,/9$ LQ
&XUVRGH
DWXDUH[FOXVLYDPHQWHQXPDGHWHUPLQDGDUHJLmR DOJXQV 'LUHLWR&RQVWLWXFLRQDO3RVLWLYR
HG5HY7ULES 
0XQLFtSLRVRXDOJXQV(VWDGRV 23DUWLGRGHYHUiH[LVWLU RSRVWDPHQWHjLGpLDGHHVWDGRXQLWiULRHPERUDR(VWDGR
HPWRGRR7HUULWyULR1DFLRQDORXQXPQ~PHURPtQLPR IHGHUDOpFRQVLGHUDGRXPDXQLGDGHQDVUHODo}HVLQWHUQDFLR
GH(VWDGRV DWpTXHXPGLDWHQKDIRUoDVSDUD DOFDQoDU QDLV
WRGRRWHUULWyULRQDFLRQDO 
23URI0$12(/*21d$/9(6)(55(,5$),/+2DSyV
,, SURLELomRGHUHFHELPHQWRGHUHFXUVRVILQDQFHLURV REVHUYDU TXH (P WRGR H TXDOTXHU (VWDGR R SRGHU p
GH
GH HQWLGDGH
HQWLGDGH RX
RX JRYHUQR
JRYHUQR HVWUDQJHLURV
HVWUDQJHLURV RX GH UHODWLYDPHQWHFHQWUDOL]DGRDQRWDTXH$GHVFHQWUDOL]DomR
VXERUGLQDomRDHVWHV p LQVWUXPHQWR GH HILFLrQFLD JRYHUQDPHQWDO (P JHUDO D
&kPDUD/HJLVODWLYDGR
 'LVWULWR)HGHUDO 'LU&RQVWLWXFLRQDO 

FHQWUDOL]DomRUHWDUGDDVGHFLV}HVTXHVREUHYrPDGHVWHPSR †ž FULDomRDLQFRUSRUDomR


$FULDomR LQFRUSRUDomRDIXVmRH
IXVmRHRGHVPHP
DWUDVDGDV0DVDGHVFHQWUDOL]DomRpWDPEpPXPDIyUPXOD EUDPHQWR
EUDPHQWRGH
GH0XQLFtSLRV
0XQLFtSLRVIDUVHmR
IDUVHmRSRU
SRUOHL
OHLHVWDGX
GHOLPLWDomRGRSRGHUeJHUDGRUDGHXPVLVWHPDGHIUHLRV DOGHQWURGRSHUtRGRGHWHUPLQDGRSRU
DOGHQWURGRSHUtRGRGHWHUPLQDGRSRUOHL OHLFRP
HFRQWUDSHVRVSURStFLRjOLEHUGDGHGLPLQXLDSUREDELOLGD SOHPHQWDU IHGHUDO
SOHPHQWDU IHGHUDO H GHSHQGHUmR
GHSHQGHUmR GH
GH FRQVXOWD
GH RSUHVVmR GLYLGLQGR R H[HUFtFLR GR SRGHU SRU PXLWRV H SUpYLD
UpYLD PHGLDQWH
PHGLDQWH SOHELVFLWR
SOHELVFLWR jV
jV SRSXODo}HV
SRSXODo}HV GRV
GLIHUHQWHVyUJmRV(WDPEpPSRUDSUR[LPDURVJRYHUQDQWHV 0XQLFtSLRV
0XQLFtSLRV HQYROYLGRV
HQYROYLGRV DSyV GLYXOJDomR GRV
GRVJRYHUQDGRVRTXHIDFLOLWDDLQIOXrQFLDGHVWHVQRSURFHVVR (VWXGRV
(VWXGRVGH
GH9LDELOLGDGH
9LDELOLGDGH0XQLFLSDO
0XQLFLSDODSUHVHQWDGRV
GHWRPDGDGHGHFLV}HV LQ
&XUVR GH 'LUHLWR &RQVWLWXFLRQDO
  HSXEOLFDGRVQDIRUPDGDOHL
HSXEOLFDGRVQDIRUPDGDOHL
HG6DUDLYDSJ 
2EV QR†žDSDUWHGHVWDFDGDLWiOLFDpDGDGDSHOD(PHQGD&RQV
(PHQGD&RQV
WLWXFLRQDO
WLWXFLRQDOQž
Qž2WH[WRDQWHULRUQmRSUHYLDDOLPLWDomRGH

IHGHUDOLVPRFRPRMiYLPRVSUHVVXS}HDSDUWLFLSDomR
2IHGHUDOLVPR
IHGHUDOLVPR SHUtRGR TXH VHUi GHILQLGR HP OHL FRPSOHPHQWDU  H DLQGD
D  GD 8QLmR JHULQGR RV LQWHUHVVHV QDFLRQDLV E 
D E  GH HVWDEHOHFLGDDREULJDomRGHREVHUYkQFLDGD´FRQWLQXLGDGHHD
(VWDGRV0HPEURVDGPLQLVWUDQGRRVGHLQWHUHVVHVGHQWUR XQLGDGHKLVWyULFRFXOWXUDOGRDPELHQWHXUEDQRµ
GRVOLPLWHVGHVHXWHUULWyULRUHJLRQDOH F
F GH0XQLFtSLRV
F
QRV HVWUHLWRV OLPLWHV GH VXD ORFDOLGDGH  WRGRV FRP 2%UDVLOpXPHQRUPHSDtVTXHSDUDVHUDGPLQLVWUDGR
FDSDFLGDGH GH DXWRRUJDQL]DomR H UHVSHFWLYR 3RGHU pGLYLGLGRHPWUrVWLSRVGHJRYHUQRVXPGHFDGDORFDOLGD
&RQVWLWXLQWHSUySULRRXVHMDFRPDXWRQRPLD GH 0XQLFtSLR  XP GH FDGD UHJLmR (VWDGRV  H XP GR
WRGR 8QLmR  +i DLQGD JRYHUQR SUySULR QR 'LVWULWR
GHVFHQWUDOL]DomRGRSRGHUQR(VWDGR%UDVLOHL
(VVDpDGHVFHQWUDOL]DomR )HGHUDO RQGHILFDDFDSLWDOGR3DtV%UDVtOLD 
UR$&RQVWLWXLomREUDVLOHLUDGHQmRVHFRQWHQWDHP
HVWDEHOHFHUD)HGHUDomRGHVFHQWUDOL]DGRRWRGRHVWDEHOHFH 2VFKDPDGRV7HUULWyULRV)HGHUDLVID]HPSDUWHGD8QLm
WDPEpPRPXQLFLSDOLVPRLPSRQGRDGHVFHQWUDOL]DomRGDV RRXVHMDVmRDGPLQLVWUDGRVSHORJRYHUQRFHQWUDO1RVVRV
SDUWHV+iHPQRVVD&RQVWLWXLomRWUrV WUrVRUGHQVHQmRGXDV
WUrV (VWDGRVQHPVmRLQGHSHQGHQWHVQHPLPXWiYHLVSRGHPVHU
FRPR p QRUPDO QR (VWDGR IHGHUDO (P SULPHLUR OXJDU D GLYLGLGRV FRPRDFRQWHFHXUHFHQWHPHQWHFRP*RLiVTXH
RUGHP WRWDO  D 8QLmR  HP VHJXQGR OXJDU RUGHQV VHGLYLGLXIRUPDQGRXPQRYR(VWDGR7RFDQWLQVH*RLiV
UHJLRQDLVRV(VWDGRVHPWHUFHLUROXJDURUGHQVORFDLV ILFDQGR PHQRU  LGHP FRP R 0DWR *URVVR Ki PDLV
 RV 0XQLFtSLRV p R UHJLVWUR GR 3URI0$12(/ *21 WHPSRDWUiV MXQWDGRVXQVDRVRXWURV FRPRDFRQWHFHX
d$/9(6)(55(,5$),/+2 LQ
&XUVRGH'LUHLWR&RQVWLWXFLRQDO
 FRPRDQWLJR(VWDGRGH*XDQDEDUDTXHVHMXQWRXDR5LR
HG6DUDLYDSJ  GH-DQHLUR IRUPDQGRXP~QLFRHVy(VWDGRRXPHVPR
FULDGRVQRYRV FRPRVHPSUHDFRQWHFHFRPRV7HUULWyULRV
$WHQWHVHDTXHQRVVDHVWUXWXUDomRIHGHUDWLYDLQXVLWD TXHVmRWUDQVIRUPDGRVHPQRYRV(VWDGRVH[HPSOR$FUH
GDPHQWHLQFOXLRVPXQLFtSLRV
PXQLFtSLRVGHQWUHVHXVFRPSRQHQWHV
PXQLFtSLRV TXHYLURX(VWDGRGR$FUH5RQG{QLDHWF 
RTXHFRQVWLWXLDUUHPDWDDQRPDOLDjPHGLGDTXHRV
0XQLFtSLRVQmRSDVVDPGHGLYLVmRWHUULWRULDOHDGPLQLVWUD 3DUDTXHLVVRDFRQWHoDVHUiREULJDWyULDDDSURYDomRGD
WLYD GRV (VWDGRVPHPEURV ,QWHLUDPHQWH SURFHGHQWH SRSXODomRGR(VWDGRHPSOHELVFLWR7DPEpPR&RQJUHVVR
DVVLPDFUtWLFDGH-26e$)2162'$6,/9$GHTXH)RL 1DFLRQDO 6HQDGRH&kPDUD GHYHUiDSURYDU
HTXtYRFR R FRQVWLWXLQWH LQFOXLU RV 0XQLFtSLRV FRPR FRP
SRQHQWH GD IHGHUDomR 0XQLFtSLR p GLYLVmR SROtWLFD GR 'DPHVPDIRUPDTXHRV(VWDGRVSRGHPVHUGLYLGLGRV
(VWDGRPHPEUR(DJRUDWHPRVXPDIHGHUDomRGH0XQLFtSL RXMXQWDGRVWDPEpPRV0XQLFtSLRVSRGHUmRGLYLGLUVHRX
RVH(VWDGRVRXXPDIHGHUDomRGH(VWDGRV"IDOWDPRXWURV MXQWDUVHRXVHUHPFULDGRVQRYRV0XQLFtSLRV GLVWULWRV
HOHPHQWRVSDUDDFDUDFWHUL]DomRGHIHGHUDomRGH0XQLFtSLRV EDLUURVTXHVHWRUQDPLQGHSHQGHQWHV 
³ LQ
&XUVRGH'LUHLWR&RQVWLWXFLRQDO3RVLWLYR
HG5HY7ULESJ 
7XGRLVVRSRUpPGHSHQGHUiGDDSURYDomRGDVUHVSHF
$UW$
$ RUJDQL]DomR
RUJDQL]DomR SROtWLFRDGPLQLVWUDWLYDGD5HS~
SROtWLFRDGPLQLVWUDWLYDGD5HS~ WLYDV SRSXODo}HV LQWHUHVVDGDV H D PHGLGD DLQGD GHYHUi
EOLFD)HGHUDWLYDGR%UDVLOFRPSUHHQGH
EOLFD)HGHUDWLYDGR%UDVLOFRPSUHHQGHD8QLmR DWHQGHU D UHTXLVLWRV SUHYLVWRV QD OHL FRPSOHPHQWDU
RV
RV (VWDGRV
(VWDGRV R 'LVWUL
'LVWULWR
WR )HGHUDO
)HGHUDO H RV
RV 0XQLFtSLRV IHGHUDO1mRKiPDLVQHFHVVLGDGHGHDWHQGHUDUHTXLVLWRV
WRGRV
WRGRVDXW{QRPRV
DXW{QRPRVQRV
QRVWH
WHUPRVGHVWD&RQVWLWXLomR
UPRVGHVWD&RQVWLWXLomR HVWDEHOHFLGRVHPOHLFRPSOHPHQWDUHVWDGXDO

†ž %UDVtOLDpD&DSLWDO)HGHUDO 0DVDFULDomRDLQFRUSRUDomRDIXVmRHRGHVPHPEUD


PHQWRVySRGHUmRVHUSURFODPDGRVSRU OHL
OHL HVWDGXDORX
HVWDGXDO
†ž 2V7HUULWyULRV)HGHUDLVLQWHJUDPD8QLmRHVXD VHMDFDEHUij$VVHPEOpLD/HJLVODWLYDGR(VWDGRGHFODUi
FULDomRWUDQVIRUPDomR
FULDomR WUDQVIRUPDomRHP
HP(VWDGR
(VWDGRRX
RXUHLQWHJUD ODV
omR
omRDR
DR(VWDGR
(VWDGRGH
GHRULJHP
RULJHPVHUmR
VHUmRUHJXODGDV
UHJXODGDVHP
HPOHL
FRPSOHPHQWDU $UW eYHGDGR
YHGDGRj8QLmR
8QLmRDRV
DRV(VWDGRV
(VWDGRVDR
DR'LVWULWR)HGH
UDOHDRV0XQLFtSLRV
†ž 2V (VWDGRV SRGHP LQFRUSRUDUVHHQWUHVL
VXEGLYLGLUVH
VXEGLYLGLUVH RX
RX GHVPHPEUDUVHSDUDVHDQH[D
GHVPHPEUDUVHSDUDVHDQH[D , HVWDEHOHFHUFXOWRVUHOLJLRVRVRXLJUHMDVVXEYHQFL
HVWDEHOHFHUFXOWRVUHOLJLRVRVRXLJUHMDVVXEYHQFL
UHP
UHP D RXWURV
RXWURV RX
RX IRUPDUHP
IRUPDUHP QRYRV
QRYRV (VWDGRV
(VWDGRV RX RQiORV
RQiORV HPEDUDoDUOKHV
HPEDUDoDUOKHV R IXQFLRQDP
IXQFLRQDPHQWR
HQWR RX
7HUULWyULRV
7HUULWyULRV )HGHUDLV PHGLDQWH DSURYDomR GD PDQWHU
PDQWHUFRP
FRPHOHV
HOHVRX
RXVHXV
VHXVUHSUHVHQWDQWHV
UHSUHVHQWDQWHVUHODo}HV
SRSXODomR
SRSXODomR GLUHWDPHQWHLQWHUHVVDGDDWUDYpVGH GH
GH GHSHQGrQFLD
GHSHQGrQFLD RX RX DOLDQoD UHVVDOYDGD QD
SOHELVFLWR H GR &RQJUHVVR 1DFLRQDO SRU OHL IRUPD
IRUPDGD
GDOHL
OHLDFRODERUDomR
FRODERUDomRGH
GHLQWHUHVVH
LQWHUHVVHS~EOLFR
FRPSOHPHQWDU
,, UHFXVDUIpDRVGRFXPHQWRVS~EOLFRV
 'LU&RQVWLWXFLRQDO
 &kPDUD/HJLVODWLYDGR'LVWULWR)HGHUDO
,,, FULDU
FULDUGLVWLQo}HV
GLVWLQo}HVHQWUH
HQWUHEUDVLOHLURV
EUDVLOHLURVRX
RXSUHIHUrQFLDV 9 RV
RVUHFXUVRV
UHFXUVRVQDWXUDLV
QDWXUDLVGD
GDSODWDIRUPD
SODWDIRUPDFRQWLQHQWDO
FRQWLQHQWDOH
HQWUHVL GD]RQDHFRQ{PLFDH[FOXVLYD
9, RPDUWHUULWRULDO
2VJRYHUQRV0XQLFLSDO(VWDGXDOGR'LVWULWR)HGHUDOH 9,, RVWHUUHQRVGHPDULQKDHVHXVDFUHVFLGRV
GD8QLmRQmRSRGHUmRWRPDUTXDOTXHUSURYLGrQFLDSDUD 9,,, RVSRWHQFLDLVGHHQHUJLDKLGUiXOLFD
SURWHJHU RX SURLELU TXDOTXHU WLSR GH FXOWR UHOLJLRVR ,; RVUHFXUVRVPLQHUDLVLQFOXVLYHRVGRVXEVROR
$VVLPpSURLELGDDFULDomRGHXPFXOWRRILFLDO2FXOWR ; DV
DVFDYLGDGHV
FDYLGDGHVQDWXUDLV
QDWXUDLVVXEWHUUkQHDV
VXEWHUUkQHDVHRVVtWLRVDU
RVVtWLRV
UHOLJLRVRVHUiVHPSUHOLYUHHSDUWLFXODU TXHROyJLFRVHSUpKLVWyULFRV
;, DV
DVWHUUDVWUDGLFLRQDOPHQWHRFXSDGDVSHORVtQGL
WHUUDVWUDGLFLRQDOPHQWHRFXSDGDVSHORVtQGL
6H VmR WUrV RV WLSRV GH JRYHUQR H DLQGD PDLV XP RV
TXDUWRRGR'LVWULWR)HGHUDO QmRpGLItFLOLPDJLQDUTXH
PXLWRV VHUmR RV GRFXPHQWRV IRUQHFLGRV SHODV PXLWDV †ž eDVVHJXUDGD
DVVHJXUDGDQRV
QRVWHUPRV
WHUPRVGD
GDOHL
OHLDRV
DRV(VWDGRVDR
'LVWULWR )HGHUDO
'LVWULWR )HGHUDO H DRV
DRV 0XQLFtSLRV
0XQLFtSLRV EHP
EHP FRPRD
FRPRD
UHSDUWLo}HVS~EOLFDVWDQWDV3UHIHLWXUDV&kPDUDV0XQLFL
yUJmRV
yUJmRVGD
GDDGPLQLVWUDomR
DGPLQLVWUDomRGLUHWDGD8QLmRSDUWL
SDLV(VWDGRVHWF7RGRGRFXPHQWRHPLWLGRSRUTXDLVTXHU
FLSDomRQRUHVXOWDGRGDH[SORUDomRGHSHWUyOHR
HQWLGDGHVS~EOLFDVVHMDPHODVYLQFXODGDVDR0XQLFtSLR
RXJiVQDWXUDOGH
RXJiVQDWXUDOGHUHFXUVRVKtGULFRVSDUDILQVGH
RXDR(VWDGRRXDR'LVWULWR)HGHUDORXj8QLmRWHUi
JHUDomR
JHUDomR GH
GH HQHUJLD
HQHUJLD HOpWULFD
HOpWULFD H RXWURV UHFXUVRV
VHPSUH Ip S~EOLFD RX VHMD VHUi FRQVLGHUDGR YiOLGR H PLQHUDLV
PLQHUDLV QR
QR UHVSHFWLYR
UHVSHFWLYR WHUULWyULR
WHUULWyULR SODWDIRUPD
GHYHUiVHUDFHLWRFRPRSURYDSRUTXDOTXHURXWURyUJmR FRQWLQHQWDOPDUWHUULWRULDORX]RQDHFRQ{PLFD
FRQWLQHQWDOPDUWHUULWRULDORX]RQDHFRQ{PLFD
S~EOLFRVHMDSHUWHQFHQWHDR0XQLFtSLRRXDR(VWDGRRX H[FOXVLYD
H[FOXVLYD RX
RX FRPSHQVDomR
FRPSHQVDomR ILQDQFHLUD
ILQDQFHLUD SRU
SRU HVVD
DR'LVWULWR)HGHUDORXj8QLmR H[SORUDomR

2V0XQLFtSLRVRXRV(VWDGRVRXR'LVWULWR)HGHUDORX †ž $IDL[D


IDL[DGH
GHDWp
DWpFHQWR
FHQWRHFLQTHQWD
FLQTHQWDTXLO{PHWURV
TXLO{PHWURVGH
D8QLmRQmRSRGHUmRHPTXDOTXHUKLSyWHVHGDUSUHIH ODUJXUD DR
ODUJXUD DR ORQJR
ORQJR GDV
GDV IURQWHLUDV
IURQWHLUDV WHUUHVWUHV
UrQFLDDTXDOTXHUWLSRGHEUDVLOHLUR VHR3UHVLGHQWHpGR GHVLJQDGD
GHVLJQDGDFRPR
FRPRIDL[D
IDL[DGH
GHIURQWHLUD
IURQWHLUDpFRQVLGHUD
0DUDQKmRRXGHTXDOTXHURXWUR(VWDGRQmRSRGHUiGDU GD
GDIXQGDPHQWDO
IXQGDPHQWDOSDUD
SDUDGHIHVD
GHIHVDGR
GRWHUULWyULR
WHUULWyULRQDFLR
RUGHPjVUHSDUWLo}HVS~EOLFDVSDUDVyDGPLWLUHPIXQFLR QDO
QDOHVXD
VXDRFXSDomR
RFXSDomRHXWLOL]DomR
XWLOL]DomRVHUmR
VHUmRUHJXODGDV
QiULR QDVFLGR QR VHX (VWDGR RX GDU PDLV DWHQomR DRV HPOHL
LQWHUHVVHVGHVHXVFRQWHUUkQHRVHWF 

(QILPWRGREUDVLOHLURpLJXDODXPVHXLUPmREUDVLOHLUR &RPSHWrQFLDVGD8QLmR
VHMD GR 2LDSRTXH VHMD GR &KXt GH 6mR 3DXOR RX GR
0DUDQKmRHWF $UW &RPSHWHj8QLmR

, PDQWHU UHODo}HV FRP(VWDGRVHVWUDQJHLURVH


8QLmR SDUWLFLSDUGHRUJDQL]Do}HVLQWHUQDFLRQDLV

,, GHFODUDUDJXHUUDHFHOHEUDUDSD]
$UW6mREHQVGD8QLmR
6mREHQVGD8QLmR
,,, DVVHJXUDUDGHIHVDQDFLRQDO
, RV TXHDWXDOPHQWHOKHSHUWHQFHP HRVTXHOKH
YLHUHPDVHUDWULEXtGRV ,9 SHUPLWLU
SHUPLWLUQRV FDVRVSUHYLVWRV
QRVFDVRV SUHYLVWRVHP
HPOHLFRPSOHPHQ
WDU
WDU TXH
TXH IRUoDV
IRUoDV HVWUDQJHLUDV
HVWUDQJHLUDV WUDQVLWHP
WUDQVLWHP SHOR
WHUULWyULR
WHUULWyULR QDFLRQDO
QDFLRQDO RX
RX QHOH
QHOH SHUPDQHoDP
,, DV
DV WHUUDV
WHUUDV GHYROXWDV
GHYROXWDV LQGLVSHQViYHLVjGHIHVDGDV
LQGLVSHQViYHLVjGHIHVDGDV
WHPSRUDULDPHQWH
IURQWHLUDV
IURQWHLUDVGDV
GDVIRUWLILFDo}HV
IRUWLILFDo}HVHFRQVWUXo}HV
FRQVWUXo}HVPLOLWD
UHV
UHVGDV
GDVYLDV
YLDVIHGHUDLV
IHGHUDLVGH
GHFRPXQLFDomR
FRPXQLFDomRHjSUHVHU
9 GHFUHWDU
GHFUHWDU R HVWDGR
HVWDGR GHVtWLRRHVWDGRGHGHIHVDH
GHVtWLRRHVWDGRGHGHIHVDH
YDomRDPELHQWDOGHILQLGDVHPOHL DLQWHUYHQomRIHGHUDO
,,, RV
RVODJRV
ODJRVULRVHTXDLVTXHUFRUUHQWHV
ULRVHTXDLVTXHUFRUUHQWHVGH GHiJXD
iJXDHP 9, DXWRUL]DU
DXWRUL]DUHILVFDOL]DU
ILVFDOL]DUDSURGXomR
SURGXomRHRFRPpUFLR
FRPpUFLRGH
WHUUHQRV
WHUUHQRV GH
GH VHX
VHX GRPtQLR
GRPtQLR RX
RX TXHEDQKHPPDLV
TXHEDQKHPPDLV PDWHULDOEpOLFR
GH
GH XP
XP (VWDGR
(VWDGR VLUYDP
VLUYDP GH
GH OLPLWHV
OLPLWHV FRP
FRP RXWURV
SDtVHV
SDtVHV RX
RX VH
VH HVWHQGDP
HVWHQGDP D WHUULWyULRHVWUDQJHLUR
WHUULWyULRHVWUDQJHLUR 9,, HPLWLUPRHGD
RX GHOH SURYHQKDP EHP  FRPR RV WHUUHQRV
PDUJLQDLVDDVSUDLDVIOXYLDLV 9,,, DGPLQLVWUDU
DGPLQLVWUDUDV
DVUHVHUYDV
UHVHUYDVFDPELDLV
FDPELDLVGR
GR3DtV
3DtVHILVFDOL
]DU
]DU DV
DV RSHUDo}HV
RSHUDo}HV GHGH QDWXUH]D ILQDQFHLUD
,9 DVLOKDVIOXYLDLVHODFXVWUHVQDV]RQDVOLPtWURIHV HVSHFLDOPHQWH
HVSHFLDOPHQWH DV
DV GHFUpGLWRFkPELRHFDSLWDOL
GHFUpGLWRFkPELRHFDSLWDOL
FRPRXWURVSDtVHVDVSUDLDVPDUtWLPDVDV
FRPRXWURVSDtVHVDVSUDLDVPDUtWLPDVDVLOKDV ]DomR
]DomREHP
EHPFRPR
FRPRDVDVGHVHJXURVHGH
GHVHJXURV GHSUHYLGrQFLD
RFHkQLFDVHDV
RFHkQLFDV DVFRVWHLUDV
FRVWHLUDVH[FOXtGDV GHVWDVDV
H[FOXtGDVGHVWDV DVTXH SULYDGD
FRQWHQKDP
FRQWHQKDP D VHGH
VHGH GH
GH 0XQLFtSLRV
0XQLFtSLRV H[FHWR
H[FHWR DTXHODV
iUHDV DR VHUYLoR
DIHWDGDV DR
iUHDV DIHWDGDV VHUYLoR S~EOLFR
S~EOLFR H D XQLGDGH ,; HODERUDU
HODERUDUHH[HFXWDU
H[HFXWDUSODQRV
SODQRVQDFLRQDLV
QDFLRQDLVHUHJLRQDLV
DPELHQWDOIHGHUDOHDVUHIHULGDVQRDUW,,
DPELHQWDOIHGHUDOHDVUHIHULGDVQRDUW,, GH
GHRUGHQDomR
RUGHQDomRGRWHUULWyULRHGHGHVHQYROYLPHQ
GRWHUULWyULRHGHGHVHQYROYLPHQ
WRHFRQ{PLFRHVRFLDO
2EVUHGDomRDOWHUDGDSHOD(&Qž GH 
&kPDUD/HJLVODWLYDGR
 'LVWULWR)HGHUDO 'LU&RQVWLWXFLRQDO 

; PDQWHU
PDQWHURVHUYLoR
VHUYLoRSRVWDO
SRVWDOHRFRUUHLRDpUHR
FRUUHLRDpUHRQDFLR ;; LQVWLWXLU GLUHWUL]HV SDUDRGHVHQYROYLPHQWR
QDO XUEDQR
XUEDQR LQFOXVLYHKDELWDomRVDQHDPHQWREiVLFR
LQFOXVLYHKDELWDomRVDQHDPHQWREiVLFR
HWUDQVSRUWHVXUEDQRV
;, H[SORUDU
H[SORUDUGLUHWDPHQWH
GLUHWDPHQWHRX
RXPHGLDQWHDXWRUL]DomR
FRQFHVVmR
FRQFHVVmR RX
RX SHUPLVVmR
SHUPLVVmR RV VHUYLoRV GH
RV VHUYLoRV GH WHOHFR ;;, HVWDEHOHFHU
HVWDEHOHFHUSULQFtSLRV
SULQFtSLRVHGLUHWUL]HV
GLUHWUL]HVSDUD
SDUDRVLVWHPD
PXQLFDo}HV
PXQLFDo}HVQRV
QRVWHUPRV
WHUPRVGD
GDOHL
OHLTXH
TXHGLVSRUi
GLVSRUiVREUH QDFLRQDOGHYLDomR
D RUJDQL]DomR GRV VHUYLoRV D FULDomR GH XP
yUJmR
yUJmRUHJXODGRU
UHJXODGRUHRXWURV
RXWURVDVSHFWRV
DVSHFWRVLQVWLWXFLRQDLV
LQVWLWXFLRQDLV ;;,, H[HFXWDU RV VHUYLoRV
H[HFXWDU RV VHUYLoRV GH
GH SROtFLDPDUtWLPDDHUR
SROtFLDPDUtWLPDDHUR
2EV
2EV RWH[WRGHVWHLQFLVR;,IRLPRGLILFDGRSHOD(PHQGD
(PHQGD
(PHQGD&RQVWLWXFLRQDO
&RQVWLWXFLRQDOQž
QžTXH
 SRUWXiULDHGHIURQWHLUDV
PRQRSyOLRGD8QLmRQDH[SORUDomRGRVVHUYLoRVGHWHOHFRPXQLFDo}HV
DFDERXFRPRPRQRSyOLR
PRQRSyOLR
WHOHIRQLDWHOHJUDILDHWUDQVPLVVmRGHGDGRV 2WH[WRDQWHULRUGHVWDFDYDTXHD 2EVUHGDomRGDGDSHOD(PHQGD&RQVWLWXFLRQDOQž 5HIRUPD$GPLQLVWUDWLYD 
FRQFHVVmRVySRGHULDVHUFRQFHGLGDDHPSUHVDVVREFRQWUROHDFLRQiULRHVWDWDO

;,, H[SORUDU
H[SORUDUGLUHWDPHQWH
GLUHWDPHQWHRX
RXPHGLDQWHDXWRUL]DomR ;;,,, H[SORUDU
H[SORUDU RV
RV VHUYLoRV
VHUYLoRV H LQVWDODo}HV
LQVWDODo}HV QXFOHDUHV
QXFOHDUHV GH
FRQFHVVmRRXSHUPLVVmR TXDOTXHUQDWXUH]DHH[HUFHUPRQRSyOLRHVWDWDO
TXDOTXHU QDWXUH]DHH[HUFHUPRQRSyOLRHVWDWDO
VREUH
VREUH D SHVTXLVD
SHVTXLVD D ODYUD
ODYUD R HQULTXHFLPHQWR H
D RVVHUYLoRVGHUDGLRGLIXVmRVRQRUDHGHVRQVH UHSURFHVVDPHQWR
UHSURFHVVDPHQWR D LQGXVWULDOL]DomR
LQGXVWULDOL]DomR H R FRPpU
LPDJHQV2EVHVWDDOtQHDDGRLQF;,,IRLPRGLILFDGDSHOD(PHQGD FLR
FLR GH
GH PLQpULRV
PLQpULRV QXFOHDUHV
QXFOHDUHV H VHXV GHULYDGRV D
&RQVWLWXFLRQDOQž WHQGLGRVRVVHJXLQWHVSULQFtSLRVHFRQGLo}HV
E RVVHUYLoRVHLQVWDODo}HVGHHQHUJLDHOpWULFDH
R DSURYHLWDPHQWR HQHUJpWLFR GRV FXUVRV GH D WRGDDWLYLGDGHQXFOHDUHPWHUULWyULRQDFLRQDO
iJXDHPDUWLFXODomRFRPRV(VWDGRVRQGHVH VRPHQWH VHUi DGPLWLGD SDUD ILQV SDFtILFRV H
VLWXDPRVSRWHQFLDLVKLGURHQHUJpWLFRV PHGLDQWHDSURYDomRGR&RQJUHVVR1DFLRQDO

F D QDYHJDomR DpUHD DHURHVSDFLDO H D E VRE UHJLPH GH SHUPLVVmR VmR DXWRUL]DGDV D


LQIUDHVWUXWXUDDHURSRUWXiULD FRPHUFLDOL]DomRHDXWLOL]DomRGHUDGLRLVyWRSRV
SDUD D SHVTXLVD H XVRV PpGLFRV DJUtFRODV H
G RVVHUYLoRV GHWUDQVSRUWHIHUURYLiULRHDTXD LQGXVWULDLV
2EVDOtQHD´EµFRPUHGDomRGDGDSHOD(&Qž
YLiULR HQWUH SRUWRV EUDVLOHLURV H IURQWHLUDV
QDFLRQDLVRXTXHWUDQVSRQKDPRVOLPLWHVGH F  VRE UHJLPH GH SHUPLVVmR VmR DXWRUL]DGDVD
(VWDGRRX7HUULWyULR SURGXomR FRPHUFLDOL]DomR H XWLOL]DomR GH
UDGLRLVyWRSRVGHPHLDYLGDLJXDORXLQIHULRUD
H RVVHUYLoRVGHWUDQVSRUWHURGRYLiULRLQWHUHVWD GXDVKRUDV
GXDOHLQWHUQDFLRQDOGHSDVVDJHLURV 2EVDOtQHD´FµFRPUHGDomRGDGDSHOD(&Qž

G D UHVSRQVDELOLGDGH FLYLO SRU GDQRV QXFOHDUHV


I 
I RVSRUWRVPDUtWLPRVIOXYLDLVHODFXVWUHV
LQGHSHQGHGDH[LVWrQFLDGHFXOSD
;,,, RUJDQL]DUHPDQWHUR3RGHU-XGLFLiULRR
RUJDQL]DUHPDQWHUR3RGHU-XGLFLiULRR0LQLV 2EVDOtQHD´GµDFUHVFLGDSHOD(&Qž

WpULR
WpULR3~EOLFR
3~EOLFRHD
HD'HIHQVRULD
'HIHQVRULD3~EOLFD
3~EOLFDGR
GR'LVWULWR
;;,9 RUJDQL]DU
RUJDQL]DU PDQWHU
PDQWHU H H[HFXWDU
H[HFXWDU D LQVSHomR
LQVSHomR GR
)HGHUDOHGRV7HUULWyULRV
WUDEDOKR
;,9 RUJDQL]DU
RUJDQL]DU H PDQWHU
PDQWHU D SROtFLD
SROtFLD FLYLO
FLYLO D SROtFLD
PLOLWDU
PLOLWDUHRFRUSR
FRUSRGH
GHERPEHLURV
ERPEHLURVPLOLWDU
PLOLWDUGR
GR'LVWUL ;;9 HVWDEHOHFHU
HVWDEHOHFHUDV
DViUHDV
iUHDVHDV
DVFRQGLo}HV
FRQGLo}HVSDUD
SDUDRH[HUFt
WR)HGHUDOEHP
WR)HGHUDOEHPDVVLPSUHVWDUDVVLVWrQFLDILQDQ FLR GD
FLR GD DWLYLGDGH
DWLYLGDGH GH
GH JDULPSDJHP HP IRUPD
FHLUD
FHLUD DR
DR 'LVWULWR
'LVWULWR )HGHUDO
)HGHUDO SDUD
SDUD D H[HFXomR GH DVVRFLDWLYD
VHUYLoRV
VHUYLoRVS~EOLFRV
S~EOLFRVSRU
SRUPHLR
PHLRGH
GHIXQGR
IXQGRSUySULR
SUySULR 2EV
UHGDomRGDGDSHOD(PHQGD&RQVWLWXFLRQDOQž

;9 RUJDQL]DU H PDQWHU RV VHUYLoRV RILFLDLVGH $3ULYDWLYD&RPSHWrQFLD/HJLVODWLYDGD8QLmR


HVWDWtVWLFD
HVWDWtVWLFD JHRJUDILD
JHRJUDILD JHRORJLD
JHRORJLD H FDUWRJUDILD
FDUWRJUDILD GH
kPELWRQDFLRQDO $UW &RPSHWH
&RPSHWHSULYDWLYDPHQWH
SULYDWLYDPHQWHj8QLmR
8QLmROHJLVODU
OHJLVODUVREUH

;9, H[HUFHU
H[HUFHUDFODVVLILFDomR
FODVVLILFDomRSDUD
SDUDHIHLWR
HIHLWRLQGLFDWLYRGH , GLUHLWRFLYLOFRPHUFLDOSHQDOSURFHVVXDOHOHLWR
GLYHUV}HV
GLYHUV}HV S~EOLFDV
S~EOLFDV H GH
GH SURJUDPDV
SURJUDPDV GH UiGLR H UDODJUiULRPDUtWLPRDHURQiXWLFRHVSDFLDOHGR
WHOHYLVmR WUDEDOKR

;9,, FRQFHGHUDQLVWLD ,, GHVDSURSULDomR

;9,,, SODQHMDU
SODQHMDUHSURPRYHU
SURPRYHUDGHIHVD
GHIHVDSHUPDQHQWH
SHUPDQHQWHFRQWUD ,,, UHTXLVLo}HVFLYLVHPLOLWDUHVHPFDVRGHLPLQHQWH
DV
DVFDODPLGDGHV
FDODPLGDGHVS~EOLFDV
S~EOLFDVHVSHFLDOPHQWH
HVSHFLDOPHQWHDV
DVVHFDV SHULJRHHPWHPSRGHJXHUUD
HDVLQXQGDo}HV
,9 iJXDVHQHUJLDLQIRUPiWLFDWHOHFRPXQLFDo}HVH
;,; LQVWLWXLUVLVWHPDQDFLRQDOGHJHUHQFLDPHQWRGH
LQVWLWXLUVLVWHPDQDFLRQDOGHJHUHQFLDPHQWRGH UDGLRGLIXVmR
UHFXUVRV
HFXUVRV KtGULFRV
KtGULFRV H GHILQLU
GHILQLU FULWpULRV
FULWpULRV GH
GH RXWRUJD
RXWRUJD
GHGLUHLWRVGHVHXXVR 9 VHUYLoRSRVWDO
 'LU&RQVWLWXFLRQDO
 &kPDUD/HJLVODWLYDGR'LVWULWR)HGHUDO
9, VLVWHPDPRQHWiULRHGHPHGLGDVWtWXORVHJDUDQWL ;;9,,, GHIHVDWHUULWRULDOGHIHVDDHURHVSDFLDOGHIHVD
DVGRVPHWDLV PDUtWLPDGHIHVDFLYLOHPRELOL]DomRQDFLRQDO

9,, SROtWLFDGHFUpGLWRFkPELRVHJXURVHWUDQVIHUrQ ;;,; SURSDJDQGDFRPHUFLDO


FLDGHYDORUHV
†~QLFR /HLFRPSOHPHQWDUSRGHUiDXWRUL]DURV(VWD
9,,,FRPpUFLRH[WHULRUHLQWHUHVWDGXDO GRV
GRV DOHJLVODUVREUHTXHVW}HVHVSHFtILFDVGDV
PDWpULDVUHODFLRQDGDVQHVWHDUWLJR
,; GLUHWUL]HVGDSROtWLFDQDFLRQDOGHWUDQVSRUWHV
+i OHLV TXH GLVFLSOLQDP D DWLYLGDGH DSHQDV GH XPD
; UHJLPH GRV SRUWRVQDYHJDomRODFXVWUHIOXYLDO
FRPXQLGDGHORFDORXWUDVTXHRUJDQL]DPDYLGDGHXPD
PDUtWLPDDpUHDHDHURHVSDFLDO
UHJLmRHKiDTXHODVTXHGLVFLSOLQDPDYLGDGRVEUDVLOHLURV
;, WUkQVLWRHWUDQVSRUWH $VOHLVTXHGL]HPUHVSHLWRDRVLQWHUHVVHVGHWRGRVRVEUD
VLOHLURVVHUmRIHLWDVVHPSUHSHOD8QLmR FODURSHORSRGHUOH
JLVODWLYRGD8QLmRTXHpR&RQJUHVVR1DFLRQDO 6HQDGR&kPDUDGRV
;,, MD]LGDV PLQDV RXWURV UHFXUVRV PLQHUDLVH
'HSXWDGRV 
PHWDOXUJLD

;,,, QDFLRQDOLGDGHFLGDGDQLDHQDWXUDOL]DomR 2EVHUYHTXHDVPDWpULDVSUHYLVWDVQHVWHDUWLJRGHYHP


WHUGLVFLSOLQDLGrQWLFDWDQWRQR$PD]RQDV6HUJLSH6mR
;,9 SRSXODo}HVLQGtJHQDV 3DXOR RX 5LR *UDQGH GR 6XO SRU H[HPSOR QmR WHULD
VHQWLGRTXHRTXHIRVVHFULPHQR3LDXtQmRIRVVHFULPHQR
;9 HPLJUDomR H LPLJUDomR HQWUDGDH[WUDGLomRH 3DUDQiRXTXHDSRXSDQoDQR0DUDQKmRUHQGHVVHPDLV
H[SXOVmRGHHVWUDQJHLURV TXHDGH6DQWD&DWDULQDHDVVLPSRUGLDQWH 

;9, RUJDQL]DomR GR VLVWHPD QDFLRQDO GH HPSUHJRH


FRQGLo}HVSDUDRH[HUFtFLRGHSURILVV}HV &RPSHWrQFLD&RQFRUUHQWH
;9,, RUJDQL]DomRMXGLFLiULDGR0LQLVWpULR3~EOLFRHGD $UW eFRPSHWrQFLD
FRPSHWrQFLDFRPXP
FRPXPGD
GD8QLmRGRV(VWDGRVGR
8QLmR
'HIHQVRULD 3~EOLFD GR 'LVWULWR )HGHUDO H GRV 'LVWULWR)HGHUDOHGRV0XQLFtSLRV
7HUULWyULRVEHPFRPRRUJDQL]DomRDGPLQLVWUDWLYD
GHVWHV , ]HODUSHODJXDUGDGD&RQVWLWXLomRGDV/HLVHGDV
LQVWLWXLo}HVGHPRFUiWLFDVHFRQVHUYDURSDWULP{
;9,,, VLVWHPD HVWDWtVWLFR VLVWHPD FDUWRJUiILFR HGH
QLRS~EOLFR
JHRORJLDQDFLRQDLV

;,; VLVWHPDV GH SRXSDQoD FDSWDomR H JDUDQWLDGD ,, FXLGDUGDVD~GHHDVVLVWrQFLDS~EOLFDGDSURWHomR


SRXSDQoDSRSXODU HJDUDQWLDGDVSHVVRDVSRUWDGRUDVGHGHILFLrQFLD

;; VLVWHPDVGHFRQVyUFLRVHVRUWHLRV ,,, SURWHJHURVGRFXPHQWRVDVREUDVHRXWURVEHQV


GHYDORUKLVWyULFRDUWtVWLFRHFXOWXUDORVPRQX
;;, QRUPDVJHUDLVGHRUJDQL]DomRHIHWLYRVPDWHULDO PHQWRVDVSDLVDJHQVQDWXUDLVQRWiYHLVHRVVtWLRV
EpOLFRJDUDQWLDVFRQYRFDomRHPRELOL]DomRGDV DUTXHROyJLFRV
SROtFLDVPLOLWDUHVHFRUSRVGHERPEHLURVPLOLWDUHV
,9 LPSHGLUDHYDVmRDGHVWUXLomRHDGHVFDUDFWHUL]D
;;,, FRPSHWrQFLD GD SROtFLD IHGHUDO H GDVSROtFLDV omR GH REUDV GH DUWH H GH RXWURV EHQV GH YDORU
URGRYLiULDHIHUURYLiULDIHGHUDLV KLVWyULFRDUWtVWLFRHFXOWXUDO

;;,,, VHJXULGDGHVRFLDO 9 SURSRUFLRQDU RV PHLRV GH DFHVVRjFXOWXUDj


HGXFDomRjFLrQFLDjWHFQRORJLDjSHVTXLVDHj
;;,9GLUHWUL]HVHEDVHVGDHGXFDomRQDFLRQDO LQRYDomR UHGDomRGDGDSHOD(PHQGD&RQVWLWXFLRQDOQž
;;9 UHJLVWURVS~EOLFRV 9, SURWHJHURPHLRDPELHQWHHFRPEDWHUDSROXLomR
HPTXDOTXHUGHVXDVIRUPDV
;;9, DWLYLGDGHVQXFOHDUHVGHTXDOTXHUQDWXUH]D

9,, SUHVHUYDUDVIORUHVWDVDIDXQDHDIORUD
;;9,, QRUPDVJHUDLVGHOLFLWDomRHFRQWUDWDomRHPWRGDV
DVPRGDOLGDGHVSDUDDVDGPLQLVWUDo}HVS~EOLFDV
GLUHWDV DXWiUTXLFDV H IXQGDFLRQDLV GD 8QLmR 9,,,IRPHQWDUDSURGXomRDJURSHFXiULDHRUJDQL]DUR
(VWDGRV'LVWULWR)HGHUDOH0XQLFtSLRVREHGHFLGR DEDVWHFLPHQWRDOLPHQWDU
R GLVSRVWR QR DUW  ;;, H SDUD DV HPSUHVDV
S~EOLFDV H VRFLHGDGHV GH HFRQRPLD PLVWD QRV ,; SURPRYHUSURJUDPDVGHFRQVWUXomRGHPRUDGLDV
WHUPRVGRDUW†ƒ,,, H D PHOKRULD GDV FRQGLo}HV KDELWDFLRQDLV H GH
VDQHDPHQWREiVLFR
2EVUHGDomRGDGDSHOD(PHQGD&RQVWLWXFLRQDOQž 5HIRUPD$GPLQLVWUDWLYD 
&kPDUD/HJLVODWLYDGR
 'LVWULWR)HGHUDO 'LU&RQVWLWXFLRQDO 

; FRPEDWHU DV FDXVDV GD SREUH]D HRVIDWRUHVGH ;9, RUJDQL]DomRJDUDQWLDVGLUHLWRVHGHYHUHVGDVSROt


PDUJLQDOL]DomRSURPRYHQGRDLQWHJUDomRVRFLDO FLDVFLYLV
GRVVHWRUHVGHVIDYRUHFLGRV
†ž 1R kPELWR GD OHJLVODomRFRQFRUUHQWHDFRP
;, UHJLVWUDUDFRPSDQKDUHILVFDOL]DUDVFRQFHVV}HV SHWrQFLD
SHWrQFLD GD
GD 8QLmR
8QLmR OLPLWDUVHi
OLPLWDUVHi D HVWD
HVWDEHOHFHU
GHGLUHLWRVGHSHVTXLVDHH[SORUDomRGHUHFXUVRV QRUPDVJHUDLV
KtGULFRVHPLQHUDLVHPVHXVWHUULWyULRV
†ž $ FRPSHWrQFLD GD 8QLmRSDUDOHJLVODUVREUH
;,, HVWDEHOHFHUHLPSODQWDUSROtWLFDGHHGXFDomRSDUD QRUPDV
QRUPDV JHUDLV
JHUDLV QmR FRPSHWrQFLD VXSOH
H[FOXL D FRPSHWrQFLD
QmR H[FOX
DVHJXUDQoDGRWUkQVLWR PHQWDUGRV(VWDGRV

†ž ,QH[LVWLQGR OHL IHGHUDO VREUHQRUPDVJHUDLVRV


†~QLFR /HLV FRPSOHPHQWDUHV IL[DUmR QRUPDVSDUDD
(VWDGRV
(VWDGRV H[HUFHUmR D FRPSHWrQFLD OHJLVODWLYD
FRRSHUDomRHQWUHD8QLmRHRV(VWDGRVR'LVWULWR
SOHQDSDUDDWHQGHUDVXDVSHFXOLDULGDGHV
)HGHUDOHRV0XQLFtSLRVWHQGRHPYLVWDRHTXLOt
EULR GR GHVHQYROYLPHQWR H GR EHPHVWDU HP †ž $VXSHUYHQLrQFLD
VXSHUYHQLrQFLDGH
GHOHL
OHLIHGHUDO
IHGHUDOVREUHDVQRUPDV
kPELWRQDFLRQDO JHUDLV
JHUDLVVXVSHQGH
VXVSHQGHDHILFiFLD
HILFiFLDGD
GDOHL
OHLHVWDGXDO
HVWDGXDOQR
QRTXH
2EVUHGDomRGDGDSHOD(PHQGD&RQVWLWXFLRQDOQž OKHIRUFRQWUiULR
$UW&RPSHWH
&RPSHWH
&RPSHWH j 8QLmR
8QLmR DRV (VWDGRV H DR 'LVWULWR)HGHUDO
OHJLVODUFRQFRUUHQWHPHQWHVREUH
(VWDGRV
, GLUHLWRWULEXWiULRILQDQFHLURSHQLWHQFLiULRHFRQ{
PLFRHXUEDQtVWLFR
$IHGHUDomREUDVLOHLUDpFRPSRVWDGH YLQWHHVHLV
,, RUoDPHQWR (VWDGRV

,,, MXQWDVFRPHUFLDLV
$FUH 0LQDV*HUDLV 5LR*UDQGHGR1RUWH
$ODJRDV 0DWR*URVVRGR6XO 5RQG{QLD
,9 FXVWDVGRVVHUYLoRVIRUHQVHV $PD]RQDV 0DWR*URVVR 5RUDLPD
$PDSi 3DUi 5LR*UDQGHGR6XO
%DKLD 3DUDtED 6DQWD&DWDULQD
9 SURGXomRHFRQVXPR &HDUi 3HUQDPEXFR 6HUJLSH
(VStULWR6DQWR 3LDXt 6mR3DXOR
9, IORUHVWDV FDoD SHVFD IDXQDFRQVHUYDomRGD *RLiV 3DUDQi 7RFDQWLQV
QDWXUH]DGHIHVDGRVRORHGRVUHFXUVRVQDWXUDLV 0DUDQKmR 5LRGH-DQHLUR
SURWHomRGRPHLRDPELHQWHHFRQWUROHGDSROXL PDLVR'LVWULWR)HGHUDO %UDVtOLD
omR
$UW 2V (VWDGRV RUJDQL]DPVH H UHJHPVHSHODV
9,, SURWHomRDRSDWULP{QLRKLVWyULFRFXOWXUDOWXUtVWL &RQVWLWXLo}HV
&RQVWLWXLo}HVHOHLV
OHLVTXH
TXHDGRWDUHP
DGRWDUHPREVHUYDGRV
REVHUYDGRVRV
FRHSDLVDJtVWLFR SULQFtSLRVGHVWD&RQVWLWXLomR

9,,, UHVSRQVDELOLGDGHSRUGDQRDRPHLRDPELHQWHDR †ž 6mR


6mRUHVHUYDGDV
UHVHUYDGDVDRV
DRV(VWDGRV
(VWDGRVDV
DVFRPSHWrQFLDV
FRPSHWrQFLDVTXH
FRQVXPLGRUDEHQVHGLUHLWRVGHYDORUDUWtVWLFR QmROKHVVHMDPYHGDGDVSRUHVWD&RQVWLWXLomR
HVWpWLFRKLVWyULFRWXUtVWLFRHSDLVDJtVWLFR
†ž &DEH DRV (VWDGRV H[SORUDUGLUHWDPHQWHRX
,; HGXFDomR FXOWXUD HQVLQRGHVSRUWRFLrQFLD PHGLDQWH
PHGLDQWH FRQFHVVmR RV VHUYLoRV ORFDLV GH JiV
WHFQRORJLDSHVTXLVDGHVHQYROYLPHQWRHLQRYDomR FDQDOL]DGR
FDQDOL]DGRQD
QDIRUPD
IRUPDGD
GDOHL
OHLYHGDGD
YHGDGDDHGLomR
HGLomRGH
UHGDomRGDGDSHOD(PHQGD&RQVWLWXFLRQDOQž
0HGLGD3URYLVyULDSDUDDVXDUHJXODPHQWDomR
2EV R WH[WR GHVWH † ž IRL PRGLILFDGR SHOD (PHQGD &RQVWLWXFLRQDO Qž  GH
; FULDomRIXQFLRQDPHQWRHSURFHVVRGRMXL]DGRGH  TXHDFDERXFRPRPRQRSyOLRGRV(VWDGRVQDGLVWULEXLomRHVHUYLoRV
SHTXHQDVFDXVDV GHJiVFDQDOL]DGR2WH[WRDQWHULRUGHVWDFDYDDH[FOXVLYLGDGHGRV(VWDGRVSDUDWDO
GLVWULEXLomR

;, SURFHGLPHQWRVHPPDWpULDSURFHVVXDO †ž 2V(VWDGRVSRGHUmRPHGLDQWHOHLFRPSOHPHQWDU


2V(VWDGRVSRGHUmRPHGLDQWHOHLFRPSOHPHQWDU
LQVWLWXLU
LQVWLWXLU UHJL}HV
UHJL}HV PHWURSROLWDQDV DJORPHUDo}HV
;,, SUHYLGrQFLDVRFLDOSURWHomRHGHIHVDGDVD~GH XUEDQDV
XUEDQDVHPLFURUUHJL}HVFRQVWLWXtGDVSRU
HPLFURUUHJL}HVFRQVWLWXtGDVSRUDJUX
SDPHQWRV
SDPHQWRVGH GH0XQLFtSLRVOLPtWURIHVSDUDLQWHJUDU
0XQLFtSLRVOLPtWURIHVSDUDLQWHJUDU
;,,, DVVLVWrQFLDMXUtGLFDHGHIHQVRULDS~EOLFD DRUJDQL]DomRRSODQHMDPHQWRHDH[HFXomRGH
RUJDQL]DomRRSODQHMDPHQWRHDH[HFXomRGH
IXQo}HVS~EOLFDVGHLQWHUHVVHFRPXP
;,9 SURWHomRHLQWHJUDomRVRFLDOGDVSHVVRDVSRUWDGR
-iVDEHPRVTXHKiWUrVQtYHLVGHJRYHUQRRQDFLRQDO
UDVGHGHILFLrQFLD
IHLWR SHOD 8QLmR  R ORFDO IHLWR SHOR 0XQLFtSLR  H R
UHJLRQDO IHLWRSHORV(VWDGRV)HGHUDGRV 
;9 SURWHomRjLQIkQFLDHjMXYHQWXGH
 'LU&RQVWLWXFLRQDO
 &kPDUD/HJLVODWLYDGR'LVWULWR)HGHUDO
$RV(VWDGRV)HGHUDGRVpSHUPLWLGRID]HUWXGRTXHQmR †ž OHL GLVSRUi
$ OHL GLVSRUi VREUHDLQLFLDWLYDSRSXODUQRSUR
VREUHDLQLFLDWLYDSRSXODUQRSUR
HVWHMDSURLELGRSHOD&RQVWLWXLomR7HRULFDPHQWHGHVVDV FHVVROHJLVODWLYRHVWDGXDO
WUrVHQWLGDGHVpR(VWDGRDTXHWHPPDLRUOLEHUGDGHH
LPSRUWkQFLDSDUDDRUJDQL]DomRSROtWLFRDGPLQLVWUDWLYD 2V(VWDGRV0HPEURVWrPVHXSUySULRSRGHUOHJLVODWLYR
SRLV D 8QLmR H RV 0XQLFtSLRV Vy SRGHP ID]HU R TXH D RXVHMDRSRGHUHQFDUUHJDGRGHID]HUDVOHLVHVWDGXDLV
&RQVWLWXLomRH[SUHVVDPHQWHOKHVSHUPLWLU 6HUi FRPSRVWR SRU GHSXWDGRV HVWDGXDLV 4XDQWRV"
1DWXUDOPHQWH GHSHQGH GR WDPDQKR GD SRSXODomR GR
$ FRQVHTrQFLD PDLV LPSRUWDQWH GHVVD DXWRQRPLD (VWDGRTXDQWRPDLVSRSXODomRPDLVGHSXWDGRV3HUPLWH
UHVLGHQRIDWRGHTXHRVSRGHUHVHVWDGXDLVQmRJXDUGDP D&RQVWLWXLomRTXHFDGD(VWDGR0HPEURWHQKDWUrVYH]HV
TXDOTXHUVXMHLomRDRVSRGHUHVIHGHUDLVFRUUHVSRQGHQWHVWDO PDLVGHSXWDGRVHVWDGXDLVGRTXHGHSXWDGRVIHGHUDLV
TXDODDGPLQLVWUDomRS~EOLFDHVWDGXDOQmRVHVXERUGLQD
HPQDGDjIHGHUDO $UW $HOHLomRGR*RYHUQDGRUHGR9LFH*RYHUQDGRU
GH
GH (VWDGR
(VWDGR SDUD
SDUD PDQGDWR
PDQGDWR GHGH TXDWUR
TXDWUR DQRV
$UW,QFOXHPVHHQWUHRVEHQVGRV(VWDGRV
,QFOXHPVHHQWUHRVEHQVGRV(VWDGRV UHDOL]DUVHi
UHDOL]DUVHi QRSULPHLURGRPLQJRGHRXWXEUR
QRSULPHLURGRPLQJRGHRXWXEUR
HP
HP SULPHLUR
SULPHLUR WXUQR H QR ~OWLPR GRPLQJR GH
, DV RXWXEUR
RXWXEUR HPVHJXQGRWXUQRVHKRXYHUGRDQR
DV iJXDV
iJXDV VXSHUILFLDLVRXVXEWHUUkQHDVIOXHQWHV
VXSHUILFLDLVRXVXEWHUUkQHDVIOXHQWHV
DQWHULRU
DQWHULRU DR
DR GR
GR WpUPLQR
WpUPLQR GR
GR PDQGDWR
PDQGDWR GH
GH VHXV
HPHUJHQWHV
HPHUJHQWHV H HP GHSyVLWR UHVVDOYDGDV QHVWH
DQWHFHVVRUHV
DQWHFHVVRUHVHDSRVVH
SRVVHRFRUUHUi
RFRUUHUiHP
HPSULPHLUR
SULPHLURGH
FDVR
FDVRQD
QDIRUPD
IRUPDGD
GDOHL
OHLDV
DVGHFRUUHQWHV
GHFRUUHQWHVGH
GHREUDV
REUDVGD
MDQHLURGR
MDQHLUR GRDQR
DQRVXEVHTHQWH
VXEVHTHQWHREVHUYDQGRTXDQGR
8QLmR
DRPDLVRGLVSRVWRQRDUW
,, DV iUHDV QDV LOKDV RFHkQLFDV HFRVWHLUDVTXH 2EV HVWH DUW WHYH VXD UHGDomR DOWHUDGD SHOD (PHQGD &RQV
HVWLYHUHP
HVWLYHUHPQR
QRVHXGRPtQLRH[FOXtGDVDTXHODVVRE
VHXGRPtQLRH[FOXtGDVDTXHODVVRE WLWXFLRQDOQž2WH[WRDQWHULRUFRQWLQKDRXWURFDOHQGiULR
GRPtQLRGD8QLmR0XQLFtSLRVRXWHUFHLURV SDUDDHOHLomRGR*RYHUQDGRUH9LFHQRVVHJXLQWHVWHUPRV´$
HOHLomR GR *RYHUQDGRU H GR 9LFH*RYHUQDGRU GH (VWDGR SDUD
PDQGDWR GH TXDWUR DQRV UHDOL]DUVHi QRYHQWD GLDV DQWHV GR
,,, DV
DV LOKDV
LOKDV IOXYLDLV
IOXYLDLV HODFXVWUHV QmRSHUWHQFHQWHVj WpUPLQRGRPDQGDWRGHVHXVDQWHFHVVRUHVHDSRVVHRFRUUHUiQR
8QLmR GLDžGHMDQHLURGRDQRVXEVHTHQWHREVHUYDGRTXDQWRDRPDLV
RGLVSRVWRQRDUWµ

,9 DVWHUUDVGHYROXWDV
DVWHUUDVGHYROXWDVQmR
QmRFRPSUHHQGLGDV
FRPSUHHQGLGDVHQWUH
HQWUHDV †ž 3HUGHUi
3HUGHUi R PDQGDWR R *RYHUQDGRUTXHDVVXPLU
GD8QLmR RXWURFDUJR
RXWUR FDUJRRX
RXIXQomR
IXQomRQD
QDDGPLQLVWUDomR
DGPLQLVWUDomRS~EOLFD
GLUHWD
GLUHWDRX
RXLQGLUHWD
LQGLUHWDUHVVDOYDGD
UHVVDOYDGDDSRVVHHPYLUWXGH
SRVVHHP
$UW 2Q~PHUR
Q~PHURGHGH'HSXWDGRV
'HSXWDGRVj$VVHPEOpLD/HJLVODWL GH
GH FRQFXUVR
FRQFXUVR S~EOLFR
S~EOLFR H REVHUYDGRRGLVSRVWRQR
YDFRUUHVSRQGHUiDRWULSORGDUHSUHVHQWDomRGR DUW,,9H9
(VWDGR
(VWDGR QD
QD &kPDUD
&kPDUD GRV
GRV 'HSXWDGRVHDWLQJLGRR
'HSXWDGRVHDWLQJLGRR
Q~PHUR
Q~PHURGH
GHWULQWD
WULQWDHVHLV
VHLVVHUi
VHUiDFUHVFLGR
DFUHVFLGRGH
GHWDQWRV †ž 2V VXEVtGLRV GR *RYHUQDGRUGR9L
TXDQWRVIRUHP
TXDQWRVIRUHPRV
RV'HSXWDGRV
'HSXWDGRV)HGHUDLV
)HGHUDLVDFLPD
DFLPDGH FH*RYHUQDGRU
FH*RYHUQDGRUHGRV
GRV6HFUHWiULRV
6HFUHWiULRVGH
GH(VWDGR
(VWDGRVHUmR
GR]H IL[DGRV
IL[DGRVSRU
SRUOHL
OHLGH
GHLQLFLDWLYD
LQLFLDWLYDGD
GD$VVHPEOpLD
$VVHPEOpLD/HJLV
ODWLYDREVHUYDGRR
ODWLYDREVHUYDGRRTXHGLVS}HPRVDUWV;,
†ž 6HUiGHTXDWURDQRVRPDQGDWRGRV'HSXWDGRV
6HUiGHTXDWURDQRVRPDQGDWRGRV'HSXWDGRV †ž,,,,,H†ž,
†ž,,,,,H†ž,
(VWDGXDLV
(VWDGXDLV DSOLFDQGRVHOKHV DV UHJUDV GHVWD
2EV HVWH † ž IRL DFUHVFHQWDGR SHOD (PHQGD &RQVWLWXFLRQDO Qž 
&RQVWLWXLomR
&RQVWLWXLomRVREUH
VREUHVLVWHPD
VLVWHPDHOHLWRUDO
HOHLWRUDOLQYLRODELOL 5HIRUPD$GPLQLVWUDWLYD 
GDGHLPXQLGDGHV
GDGH LPXQLGDGHVUHPXQHUDomR
UHPXQHUDomRSHUGD
SHUGDGH
GHPDQ
GDWR
GDWR OLFHQoD
OLFHQoD LPSHGLPHQWRVHLQFRUSRUDomRjV
)RUoDV$UPDGDV
&RPSHWrQFLD&RQFRUUHQWHGRV(VWDGRV
&RPSHWrQFLD&RQFRUUHQWH
&RQFRUUHQWHGRV(VWDGRV
H'LVWULWR)HGHUDO
†ž 2VXEVtGLR
VXEVtGLRGRV
GRV'HSXWDGRV
'HSXWDGRV(VWDGXDLV
(VWDGXDLVVHUiVHUiIL[DGR
SRUOHL
SRU OHLGH
GHLQLFLDWLYD
LQLFLDWLYDGD
GD$VVHPEOpLD
$VVHPEOpLD/HJLVODWLYD
/HJLVODWLYDQD
UD]mRGHQRPi[LPRVHWHQWDHFLQFRSRUFHQWR &RPR YLPRV Ki DOJXQV LQWHUHVVHV TXH DR PHVPR
GDTXHOH
GDTXHOHHVWDEHOHFLGR
HVWDEHOHFLGRHP HPHVSpFLH
HVSpFLHSDUD
SDUDRVRV'HSX WHPSRGL]HPUHVSHLWRWDQWRj8QLmRFRPRDRV(VWDGRV
(VWDGRV H
(VWDGRV
WDGRV
WDGRV)HGHUDLV
)HGHUDLVREVHUYDGR
REVHUYDGRRTXH TXHGLVS}HP
GLVS}HPRV RVDUWV 'LVWULWR)HGHUDO FRPRDRV0XQLFtSLRV7DLVDVVXQWRVSRU

 † ž
ž 
 † ž
ž 
 ,,
,, 
 ,,,
,,, H 
 † ž VHUHP GR LQWHUHVVH GH WRGRV HVVHV QtYHLV GH *RYHUQR
,µ
, 8QLmR(VWDGRV'LVWULWR)HGHUDOH0XQLFtSLRV GHYHUmRVHU
2EV UHGDomRGDGDSHOD(PHQGD&RQVWLWXFLRQDOQž 5HIRUPD$GPLQLVWUDWLYD 
WUDWDGRVSRUWRGRVHOHV
2EV GRUDYDQWHGHSXWDGRVHVWDGXDLVQmRPDLVSRGHUmRVHGDUDXPHQWRVSRUVLPSOHV
UHVROXomR DJRUD Ki QHFHVVLGDGH GH ´OHLµ YDOH GL]HU DSURYDomR GD SUySULD 'H IDWR WDQWR D 8QLmR FRPR RV (VWDGRV H 'LVWULWR
$VVHPEOpLDHVDQomRGR*RYHUQDGRUGR(VWDGR&RPLVVRR*RYHUQDGRUSRGHUi )HGHUDO FREUDPWULEXWRV LPSRVWRVWD[DV WrPFDGHLDV
YHWDUDEXVRVGRV'HSXWDGRV
SHQLWHQFLiULDV  VHUYLoRV IRUHQVHV FDUWyULRV H MXVWLoD 
†ž &RPSHWH jV $VVHPEOpLDV/HJLVODWLYDVGLVSRU IORUHVWDV FULDQoDV HWF ³ GDt SRUTXH D &RQVWLWXLomR
DXWRUL]DTXHWRGDVHVVDVHQWLGDGHVIDoDPOHLVSDUDGLVFLSOL
VREUH
VREUH VHX
VHX UHJLPHQWR
UHJLPHQWR LQWHUQR
LQWHUQR SROtFLD
SROtFLD H VHUYLoRV
QDUHVVHVDVVXQWRV
DGPLQLVWUDWLYRV
DGPLQLVWUDWLYRV GH
GH VXD
VXD VHFUHWDULD
VHFUHWDULD H SURYHU
SURYHU RV
UHVSHFWLYRVFDUJRV
&RQILUDQRDUW&)TXDLVVmRDVFRPSHWrQFLDV
&kPDUD/HJLVODWLYDGR
  'LVWULWR)HGHUDO 'LU&RQVWLWXFLRQDO 

$XWRQRPLD)LQDQFHLUD
0XQLFtSLRV
0HUFr GHVVD FRQVDJUDomR FRPR XPD GDV FpOXODV GD
IHGHUDomR EUDVLOHLUD R 0XQLFtSLR JR]D GH DXWRQRPLD
0XQLFtSLR GH SRVLomR SDUHOKD j 8QLmR H DR
*R]D R 0XQLFtSLR DGPLQLVWUDWLYDRXVHMDFDSDFLGDGHSDUDVHDXWRJRYHU
DGPLQLVWUDWLYD
(VWDGR0HPEURQRRUGHQDPHQWRMXUtGLFRFRQVWLWXFLRQDO QDUGHVGHDHOHLomRGHVHX3UHIHLWRHYHUHDGRUHVDWpD
FRP DXWRRUJDQL]DomR H UHVSHFWLYR 3RGHU &RQVWLWXLQWH FDSDFLGDGH GLVSRU VREUH RV DVVXQWRV GH VHX SHFXOLDU
SUySULRRXVHMDFRPDXWRQRPLD LQWHUHVVHQRWDGDPHQWHVXDFDSDFLGDGHSDUDLQVWLWXLURV
WULEXWRVFRPTXHFXVWHDUiVHXVVHUYLoRV
WULEXWRV
(VVD FDSDFLGDGH SDUD VH DXWRRUJDQL]DU ³ DVVLP
HQWHQGLGDDSRVVLELOLGDGHGHGHILQLUVXDVSUySULDVHVWUXWX 'HVWDTXHVHTXHDDXWRQRPLDDGPLQLVWUDWLYDpGLUHWD
UDVFRQVWLWXFLRQDLV³pHVWDEHOHFLGDHPQRVVD&RQVWLWXL PHQWH VXERUGLQDGD j DXWRQRPLD
DXWRQRPLD ILQDQFHLUD
ILQDQFHLUD TXH VH
omR)HGHUDGHFRPR´UHJLPHGHFDUWDVSUySULDVµTXH VXSRUWDQDFDSDFLGDGHGHWULEXWDUHGHDSOLFDUVXDVUHQGDV
EHPDFHQWXDDDXWRQRPLDPXQLFLSDO VHP YLQFXOR GH GHSHQGrQFLD FRP TXDOTXHU HVIHUD GH
SRGHU
&DEH D FDGD 0XQLFtSLR EUDVLOHLUR UHJHUVHi SRU VXD
SUySULDHHVSHFtILFDOHL
OHL
OHLRUJkQLFDTXHDVHUYRWDGDHPGRLV
RUJkQLFD $0XQLFLSDOLGDGH
0XQLFLSDOLGDGHSRLVpUHODFLRQDGDDRVWULEXWRV
0XQLFLSDOLGDGH WULEXWRVTXH
WULEXWRV
WXUQRVREVHUYDGRXPLQWHUYDORPtQLPRGHGH]GLDVHQWUH R0XQLFtSLRDUUHFDGDUi
DVGXDVYRWDo}HVHTXHVyVHUiDSURYDGDVHREWLYHUGRLV
WHUoRVGRVYRWRVGRVPHPEURVGD&kPDUD0XQLFLSDO $GYHUWH +(/< /23(6 0(,5(//(6 TXH ´LQH[SUHVVLYD
VHULDDDXWRQRPLDSROtWLFDHDDXWRQRPLDDGPLQLVWUDWLYD
(FRPRFDUWDFRQVWLWXFLRQDOPXQLFLSDOVHUiSURPXOJD VHPUHFXUVRVSUySULRVTXHJDUDQWLVVHPDUHDOL]DomRGHREUDV
GDSHODSUySULD&kPDUD0XQLFLSDO³VXEPLVVDDSHQDVj H D PDQXWHQomR GH VHUYLoRV S~EOLFRV ORFDLV  6HULD XPD
&RQVWLWXLomRGRUHVSHFWLYR(VWDGRHpFODURj&RQVWLWXL TXLPHUDDWULEXLUVHDXWRJRYHUQRDR0XQLFtSLRVHPOKHGDU
omR )HGHUDO HP HVSHFLDO DRV SULQFtSLRV HVWDWXtGRV QR UHQGDDGHTXDGDjH[HFXomRGRVVHUYLoRVQHFHVViULRVDRVHX
DUWGHVWD SURJUHVVR)HOL]PHQWHROHJLVODGRUFRQVWLWXLQWHGHVGH
FRPSUHHQGHXEHPHVVDUHDOLGDGHHGHILFLrQFLDGRVUHJLPHV
/HPEUD 3,172 )(55(,5$ TXH PHUHFH GLVFXVVmR D DQWHULRUHV TXH HPERUD DSUHJRDQGR D LPSRUWkQFLD GR
QDWXUH]DGDOHLRUJkQLFD´)DODVHIUHTHQWHPHQWHHP&RQV 0XQLFtSLRQDYLGDQDFLRQDOOKHQHJDYDUHFXUVRVLQGLVSHQVi
WLWXLomR PXQLFLSDO H FRQVWLWXLQWH PXQLFLSDO 3RUpP RV YHLVjVXDVXEVLVWrQFLDFRPRHQWLGDGHDXW{QRPDGRWDGDGH
PXQLFtSLRVQmRHVWmRLQYHVWLGRVGHXPSRGHUFRQVWLWXLQWH JRYHUQR SUySULR H GH VHUYLoRV HVSHFLDLV SDUD DWHQGHU jV
QHPWrP&RQVWLWXLo}HVPDVVLPOHLVRUJkQLFDV$OHLRUJkQL QHFHVVLGDGHVGHVXDSRSXODomR
FDpDJRUDSURPXOJDGDSHOD&kPDUD0XQLFLSDO0DVFRPR
DOHLRUJkQLFDPXQLFLSDOpXPDOHLGHYHUiHVWDUVXMHLWDD 1RUHJLPHYLJHQWHRVWULEXWRVGDFRPSHWrQFLDGR0XQLFt
VDQomRHYHWR"(QWHQGLGDFRPR&RQVWLWXLomRPXQLFLSDOQmR SLRHVWmRGLVFULPLQDGRVQD&RQVWLWXLomR)HGHUDOFDEHQGR
HVWDULDVXMHLWDQHPDVDQomRQHPDYHWRµ OKHDLQGDSDUWLFLSDomRHPRXWURVDUUHFDGDGRVSHOD8QLmR
 HSHOR(VWDGRPHPEUR                          

´3HODQRYD&RQVWLWXLomR)HGHUDODOHLRUJkQLFDPXQLFLSDO
QmRVHUiPDLVYRWDGDHGHFUHWDGDSHOD$VVHPEOpLD/HJLVODWL $'LVFLSOLQD&RQVWLWXFLRQDO
YD QHP VDQFLRQDGD SHOR JRYHUQDGRU GR (VWDGR 6HUi
SURPXOJDGD SHOD &kPDUD 0XQLFLSDO LQGHSHQGHQWH GH $UW 20XQLFtSLR
0XQLFtSLRUHJHUVHi
UHJHUVHiSRUOHLRUJkQLFD
SRUOHLRUJkQLFDYRWDGD
VDQomRHYHWRGRSUHIHLWRGHSRLVGHDSURYDGDSRUGRLVWHUoRV HP
HPGRLV
GRLVWXUQRV
WXUQRVFRP
FRPRLQWHUVWtFLR
LQWHUVWtFLRPtQLPR
PtQLPRGH
GHGH]
GRVYHUHDGRUHV GLDV
GLDVHDSURYDGD
DSURYDGDSRU
SRUGRLV
GRLVWHUoRV
WHUoRVGRV
GRVPHPEURV
PHPEURVGD
&kPDUD
&kPDUD 0XQLFLSDO
0XQLFLSDO TXH
TXH SURPXOJDUiDWHQGLGRV
SURPXOJDUiDWHQGLGRV
4XDQGR GHYH VHU YRWDGD D OHL RUJkQLFD PXQLFLSDO" RV
RV SULQFtSLRV
SULQFtSLRV HVWDEHOHFLGRV
HVWDEHOHFLGRV QHVWD
QHVWD &RQVWLWXLomR
3URPXOJDGD D &RQVWLWXLomR GR (VWDGR FDEHUi j &kPDUD QD
QD &RQVWLWXLomR
&RQVWLWXLomR GR
GR UHVSHFWLYR
UHVSHFWLYR (VWDGR H RV
0XQLFLSDO QR SUD]R GH VHLV PHVHV YRWDU D OHL RUJkQLFD VHJXLQWHVSUHFHLWRV
UHVSHFWLYDHPGRLVWXUQRVGHGLVFXVVmRHYRWDomRUHVSHLWDGR
R GLVSRVWR QD &RQVWLWXLomR )HGHUDO H QD &RQVWLWXLomR , HOHLomR
HOHLomRGR
GR3UHIHLWR
3UHIHLWRGR
GR9LFH3UHIHLWR
9LFH3UHIHLWRHGRV9HUHD
(VWDGXDO $WRGDV'LVSRVLo}HV&RQVWLWXFLRQDLV7UDQVLWyULDV GRUHV
GRUHV SDUD
SDUD PDQGDWR
PDQGDWR GH
GH TXDWUR
TXDWUR DQRV
DQRV PHGLDQWH
DUWSDUiJUDIR~QLFR  SOHLWR
SOHLWR GLUHWR
GLUHWR H VLPXOWkQHR
VLPXOWkQHR UHDOL]DGR
UHDOL]DGR HP
HP WRGR
WRGR R
3DtV
$&kPDUD0XQLFLSDOSURPXOJDUiDOHLRUJkQLFDPXQLFL
SDO LQGHSHQGHQWHPHQWH GH VDQomR RX YHWR (QVLQD -RVp ,, HOHLomRGR3UHIHLWRHGR9LFH3UHIHLWRUHDOL]DGD
HOHLomRGR3UHIHLWRHGR9LFH3UHIHLWRUHDOL]DGD
&DUORV&DO*DUFLD /LQKDVPHVWUDVGD&RQVWLWXLomRGH QR
QRSULPHLUR
SULPHLURGRPLQJR
GRPLQJRGHGHRXWXEUR
RXWXEURGR
GRDQR
DQRDQWHULRU
FLWS 9DOHUHVVDOWDUQmRGHSHQGHHODGHVDQomRGR DR
DRWpUPLQR
WpUPLQRGR
GRPDQGDWR
PDQGDWRGRV
GRVTXH
TXHGHYDP
GHYDPVXFHGHU
3UHIHLWR ³ LQ ´&RPHQWiULRV j &RQVWLWXLomR %UDVLOHLUDµ ž YRO DSOLFDGDV
DSOLFDGDV DV
DV UHJUDV
UHJUDV GR
GR DUWLJR
DUWLJR 
 QR
QR FDVR GH
HG6DUDLYDSS  0XQLFtSLRVFRPPDLVGH
0XQLFtSLRVFRPPDLVGHGX]HQWRVPLOHOHLWRUHV
GX]HQWRVPLOHOHLWRUHV
2EV HVWHLQF,,GRDUWWHYHVXDUHGDomRDOWHUDGDSHOD(PHQGD&RQVWLWXFLRQDOQž
 'LU&RQVWLWXFLRQDO &kPDUD/HJLVODWLYDGR'LVWULWR)HGHUDO


,,, SRVVH
SRVVHGR
GR3UHIHLWR
3UHIHLWRHGR
GR9LFH3UHIHLWR
9LFH3UHIHLWRQR
QRGLDžGH P   WULQWDHWUrV 9HUHDGRUHVQRV0XQLFtSLRVGH
MDQHLURGRDQRVXEVHTHQWHDRGDHOHLomR PDLVGH XPPLOKmRHFLQTXHQWDPLO
KDELWDQWHV H GH DWp  XP PLOKmR H
,9 SDUD
SDUDDFRPSRVLomR
FRPSRVLomRGDV
GDV&kPDUDV
&kPDUDV0XQLFLSDLV
0XQLFLSDLVVHUi GX]HQWRVPLO KDELWDQWHV
REVHUYDGRROLPLWHPi[LPRGH
Q   WULQWDHFLQFR 9HUHDGRUHVQRV0XQLFtSLRVGH
2EV UHGDomRGDGDSHOD(&Qž³TXHSRUPHGLGDOLPLQDU PDLVGH XPPLOKmRHGX]HQWRVPLO
GR67)VyWHULDDSOLFDomRSDUDDVHOHLo}HVPXQLFLSDLVQRDQR KDELWDQWHV H GH DWp  XP PLOKmR H
GH


WUH]HQWRVHFLQTXHQWDPLO KDELWDQWHV
D  QRYH  9HUHDGRUHV QRV 0XQLFtSLRV GH DWp
 TXLQ]HPLO KDELWDQWHV R   WULQWDHVHWH 9HUHDGRUHVQRV0XQLFtSLRVGH
 XP PLOKmR H WUH]HQWRV H FLQTXHQWD
PLO KDELWDQWHVHGHDWp XPPLOKmRH
E   RQ]H 9HUHDGRUHVQRV0XQLFtSLRVGHPDLVGH
TXLQKHQWRVPLO KDELWDQWHV
 TXLQ]HPLO KDELWDQWHVHGHDWp
WULQWDPLO KDELWDQWHV
S   WULQWDHQRYH 9HUHDGRUHVQRV0XQLFtSLRVGH
PDLVGH XPPLOKmRHTXLQKHQWRVPLO
F   WUH]H 9HUHDGRUHVQRV0XQLFtSLRVFRPPDLV KDELWDQWHV H GH DWp  XP PLOKmR H
GH WULQWDPLO KDELWDQWHVHGHDWp RLWRFHQWRVPLO KDELWDQWHV
FLQTXHQWDPLO KDELWDQWHV
T   TXDUHQWDHXP 9HUHDGRUHVQRV0XQLFtSLRV
G   TXLQ]H 9HUHDGRUHVQRV0XQLFtSLRVGHPDLV GH PDLV GH  XP PLOKmR H RLWRFHQWRV
GH  FLQTXHQWD PLO  KDELWDQWHV H GH DWp PLO KDELWDQWHVHGHDWp GRLVPLOK}HV
 RLWHQWDPLO KDELWDQWHV HTXDWURFHQWRVPLO KDELWDQWHV

H   GH]HVVHWH  9HUHDGRUHV QRV 0XQLFtSLRVGH U   TXDUHQWDHWUrV 9HUHDGRUHVQRV0XQLFtSLRV


PDLVGH RLWHQWDPLO KDELWDQWHVHGHDWp GHPDLVGH GRLVPLOK}HVHTXDWURFHQ
 FHQWRHYLQWHPLO KDELWDQWHV WRV PLO  KDELWDQWHV H GH DWp  WUrV
PLOK}HV GHKDELWDQWHV
I   GH]HQRYH  9HUHDGRUHV QRV 0XQLFtSLRVGH
PDLVGH FHQWRHYLQWHPLO KDELWDQWHVH V   TXDUHQWDHFLQFR 9HUHDGRUHVQRV0XQLFtSLRV
GHDWp FHQWRVHVVHQWDPLO KDELWDQWHV GHPDLVGH WUrVPLOK}HV GHKDELWDQWHV
HGHDWp TXDWURPLOK}HV GHKDELWDQWHV
J   YLQWH H XP 9HUHDGRUHVQRV0XQLFtSLRVGH
PDLVGH FHQWRHVHVVHQWDPLO KDELWDQWHV W   TXDUHQWDHVHWH 9HUHDGRUHVQRV0XQLFtSLRV
HGHDWp WUH]HQWRVPLO KDELWDQWHV GHPDLVGH TXDWURPLOK}HV GHKDEL
WDQWHV H GH DWp  FLQFR PLOK}HV  GH
K   YLQWHHWUrV 9HUHDGRUHVQRV0XQLFtSLRVGH KDELWDQWHV
PDLVGH WUH]HQWRVPLO KDELWDQWHVHGH
DWp  TXDWURFHQWRV H FLQTXHQWD PLO X   TXDUHQWDHQRYH 9HUHDGRUHVQRV0XQLFtSLRV
KDELWDQWHV GHPDLVGH FLQFRPLOK}HV GHKDELWDQ
WHVHGHDWp VHLVPLOK}HV GHKDELWDQWHV
L   YLQWHHFLQFR 9HUHDGRUHVQRV0XQLFtSLRVGH
Y   FLQTXHQWDHXP 9HUHDGRUHVQRV0XQLFtSLRV
PDLVGH TXDWURFHQWRVHFLQTXHQWDPLO
GHPDLVGH VHLVPLOK}HV GHKDELWDQWHV
KDELWDQWHV H GH DWp  VHLVFHQWRV PLO
HGHDWp VHWHPLOK}HV GHKDELWDQWHV
KDELWDQWHV
Z   FLQTXHQWDHWUrV 9HUHDGRUHVQRV0XQLFtSLRV
M   YLQWHHVHWH 9HUHDGRUHVQRV0XQLFtSLRVGH GHPDLVGH VHWHPLOK}HV GHKDELWDQ
PDLVGH VHLVFHQWRVPLO KDELWDQWHVHGH WHVHGHDWp RLWRPLOK}HV GHKDELWDQ
DWp VHWHFHQWRVFLQTXHQWDPLO KDELWDQ WHVH
WHV
[   FLQTXHQWDHFLQFR 9HUHDGRUHVQRV0XQLFtSL
N   YLQWHHQRYH 9HUHDGRUHVQRV0XQLFtSLRVGH RVGHPDLVGH RLWRPLOK}HV GHKDEL
PDLV GH  VHWHFHQWRV H FLQTXHQWD PLO WDQWHV
KDELWDQWHV H GH DWp  QRYHFHQWRV PLO
KDELWDQWHV $WHQomR 2 Q~PHUR GH YHUHDGRUHV TXH FRPS}HP D
&kPDUD0XQLFLSDOILVFDOL]DPDDVFRQWDVHID
O   WULQWDHXP 9HUHDGRUHVQRV0XQLFtSLRVGH ]HP DV OHLV GR PXQLFtSLR  VHUi GHILQLGR HP IXQomR GR
PDLVGH QRYHFHQWRVPLO KDELWDQWHVHGH Q~PHURGH KDELWDQWHV
KDELWDQWHV QmR p GR Q~PHUR GHHOHLWRUHV FRPR QR
DWp  XP PLOKmR H FLQTXHQWD PLO FULWpULRSDUDRVHJXQGRWXUQR GRPXQLFtSLRPXLWRVKDELWDQWHV
KDELWDQWHV PXLWRV YHUHDGRUHV QR Pi[LPR   SRXFRV KDELWDQWHV
SRXFRVYHUHDGRUHV QRPtQLPR 
&kPDUD/HJLVODWLYDGR
  'LVWULWR)HGHUDO 'LU&RQVWLWXFLRQDO 

2XWURUD FDELD j /HL 2UJkQLFD GR 0XQLFtSLR GHILQLU H HP 0XQLFtSLRV GH WUH]HQWRV PLO H XP D
HQWUHXPPtQLPRHXPPi[LPRRQ~PHURGHYHUHDGRUHV TXLQKHQWRVPLOKDELWDQWHVRVXEVtGLRPi[LPRGRV
GD &kPDUD ORFDO FODUR IL[DYDVH VHPSUH R Q~PHUR 9HUHDGRUHVFRUUHVSRQGHUiDVHVVHQWDSRUFHQWRGR
Pi[LPR $JRUDRQ~PHURpIL[RGHILQLGRHPIXQomRGD VXEVtGLRGRV'HSXWDGRV(VWDGXDLV
SRSXODomR KDELWDQWHV PXQLFLSDO
I  HP0XQLFtSLRVGHPDLVGHTXLQKHQWRVPLOKDEL
9 VXEVtGLRV GR 3UHIHLWR GR 9LFH3UHIHLWRHGRV WDQWHVRVXEVtGLRPi[LPRGRV9HUHDGRUHVFRUUHV
6HFUHWiULRV
6HFUHWiULRV0XQLFLSDLV
0XQLFLSDLVIL[DGRV
IL[DGRVSRU
SRUOHL
OHLGH
GHLQLFLDWL SRQGHUiDVHWHQWDHFLQFRSRUFHQWRGRVXEVtGLR
YD
YD GD
GD &kPDUD
&kPDUD 0XQLFLSDO
0XQLFLSDO REVHUYDGR
REVHUYDGR R TXH
TXH GLV GRV'HSXWDGRV(VWDGXDLV
S}HP
S}HPRV
RVDUWV
DUWV
;,
;,
†ž
ž
,,
,,
,,,
,,,H
†ž, 9,, R WRWDO GD GHVSHVD FRP DUHPXQHUDomRGRV
2EV UHGDomRGDGDSHOD(PHQGD&RQVWLWXFLRQDO
Qž 5HIRUPD$GPLQLVWUDWLYD  9HUHDGRUHV
9HUHDGRUHV QmR
QmR SRGHUi
SRGHUi XOWUDSDVVDU
XOWUDSDVVDU R PRQWDQWH
GH
GH 
 FLQFR
FLQFR SRU
SRU FHQWR 
FHQWR  GD
GD UHFHLWD
UHFHLWD GRPXQLFt
GRPXQLFt
9LQFXODomRj5HFHLWD7ULEXWiULD SLR
SLR

1HQKXP VXEVtGLR ³ QHP GRV DJHQWHV SROtWLFRV GR 9,,, LQYLRODELOLGDGH GRV 9HUHDGRUHV SRU VXDVRSL
([HFXWLYRQHPGRVGR/HJLVODWLYR³SRGHUiVHUIL[DGR QL}HV
QL}HVSDODYUDV
SDODYUDVHYRWRV
YRWRVQR
QRH[HUFtFLR
H[HUFtFLRGR
GRPDQGDWR
YLQFXODGDPHQWHjUHFHLWDWULEXWiULDGR0XQLFtSLRQHPHP HQDFLUFXQVFULomRGR0XQLFtSLR
IUDomRGHVWDQHPHPSHUFHQWXDOGHVWD(VVDSUiWLFDDWp
FRPXPQRSDVVDGRpGHPDQLIHVWDLQFRQVWLWXFLRQDOLGDGH ,; SURLELo}HV
SURLELo}HVHLQFRPSDWLELOLGDGHV
LQFRPSDWLELOLGDGHVQRH[HUFtFLRGD
PHVPRSRUTXHRDUWLJR&)LQF,9SURtEHH[SUHVVD YHUHDQoD
YHUHDQoDVLPLODUHV
VLPLODUHVQR
QRTXH
TXHFRXEHUDRGLVSRVWR
PHQWH´DYLQFXODomRGHUHFHLWDGHLPSRVWRVDyUJmRIXQGR QHVWD
QHVWD&RQVWLWXLomR
&RQVWLWXLomRSDUD
SDUDRV
RVPHPEURV
PHPEURVGR
GR&RQJUHV
RXGHVSHVDµ VR 1DFLRQDO H QD &RQVWLWXLomR GR UHVSHFWLYR
(VWDGR
(VWDGRSDUD
SDUDRV
RVPHPEURV
PHPEURVGD
GD$VVHPEOpLD
$VVHPEOpLD/HJLVOD
6HGHXPODGRUHWLURXVHDLQLFLDWLYDGR3UHIHLWRSDUD WLYD
DXPHQWDUVHXVVXEVtGLRVGHRXWURWDPEpPRV9HUHDGRUHV
QmRPDLVSRGHUmRVHGDUDXPHQWRVSRLVDOHLGHSHQGHUi ; MXOJDPHQWR GR 3UHIHLWR SHUDQWHR7ULEXQDOGH
GH VDQomR GR 3UHIHLWR TXH GLVSRQGR GR SRGHU GHYHWR -XVWLoD
SRGHUiLPSHGLUDEXVRV
;, RUJDQL]DomRGDVIXQo}HVOHJLVODWLYDVHILVFDOL]D
RUJDQL]DomRGDVIXQo}HVOHJLVODWLYDVHILVFDOL]D
e WmR DFHQWXDGD D SUHRFXSDomR GH DFDEDU  FRP  RV GRUDVGD&kPDUD0XQLFLSDO
DEXVRVFRQVWDWDGRVQRVPXLWRVPXQLFtSLRVGR%UDVLO
TXHD&RQJUHVVR1DFLRQDODSURYRXD(PHQGD&RQVWLWXFLR ;,, FRRSHUDomR GDV DVVRFLDo}HV UHSUHVHQWDWLYDVQR
QDO Qž  HVWDEHOHFHQGR QRYRV OLPLWHV SDUD D SODQHMDPHQWRPXQLFLSDO
UHPXQHUDomR VXEVtGLRV  GH VHXV YHUHDGRUHV ³ H SDUD
LVVRGHXQRYDUHGDomRHVWHLQFLVR9,GRDUW ;,,, LQLFLDWLYD
LQLFLDWLYDSRSXODU
SRSXODUGH
GHSURMHWRV
SURMHWRVGH
GHOHL
OHLGHLQWHUHVVH
9, VXEVtGLR
VXEVtGLR GRV
GRV 9HUHDGRUHV VHUi IL[DGRSHODVUHV HVSHFtILFR
HVSHFtILFRGR
GR0XQLFtSLR
0XQLFtSLRGD
GDFLGDGH
FLGDGHRX
RXGH
GHEDLUURV
SHFWLYDV&kPDUDV0XQLFLSDLVHPFDGDOHJLVODWXUD
SHFWLYDV&kPDUDV0XQLFLSDLVHPFDGDOHJLVODWXUD DWUDYpV
DWUDYpV GH
GH PDQLIHVWDomRGHSHORPHQRVFLQFR
PDQLIHVWDomRGHSHORPHQRVFLQFR
SDUD
SDUDDVXEVHTHQWH
VXEVHTHQWHREVHUYDGR
REVHUYDGRRTXH
TXHGLVS}H
GLVS}HHVWD SRUFHQWRGRHOHLWRUDGR
&RQVWLWXLomR
&RQVWLWXLomRREVHUYDGRV
REVHUYDGRVRV
RVFULWpULRV
FULWpULRVHVWDEHOHFL
GRV
GRV QD
QD UHVSHFWLYD
UHVSHFWLYD /HL 2UJkQLFD H RV VHJXLQWHV ;,9 SHUGD
SHUGD GR
GR PDQGDWR
PDQGDWR GR 3UHIHLWR QRV WHUPRVGR
OLPLWHVPi[LPRV DUWSDUiJUDIR~QLFR

D HP0XQLFtSLRVGHDWpGH]PLOKDELWDQWHVRVXEVt $UW$ 2WRWDO


$UW$ WRWDOGD
GDGHVSHVD
GHVSHVDGR
GR3RGHU
3RGHU/HJLVODWLYR
/HJLVODWLYR0XQLFL
GLRPi[LPRGRV9HUHDGRUHVFRUUHVSRQGHUiDYLQWH SDO
DO LQFOXtGRV
LQFOXtGRV RV
RV VXEVtGLRV
VXEVtGLRV GRV
GRV 9HUHDGRUHV
9HUHDGRUHV H
SRUFHQWRGRVXEVtGLRGRV'HSXWDGRV(VWDGXDLV H[FOXtGRV RV JDVWRV FRP LQDWLYRV QmR SRGHUi
H[FOXtGRV
XOWUDSDVVDURVVHJXLQWHVSHUFHQWXDLVUHODWLYRVDR
XOWUDSDVVDURVVHJXLQWHVSHUFHQWXDLVUHODWLYRVDR
E HP0XQLFtSLRVGHGH]PLOHXPDFLQTHQWDPLO VRPDWyULR
VRPDWyULRGD
GDUHFHLWD
UHFHLWDWULEXWiULD
WULEXWiULDHGDV
GDVWUDQVIHUrQ
KDELWDQWHV R VXEVtGLR Pi[LPR GRV 9HUHDGRUHV FLDVSUHYLVWDVQR
FLDVSUHYLVWDVQR†ž
žGR
GRDUW
DUWHQRVDUWV
FRUUHVSRQGHUiDWULQWDSRUFHQWRGRVXEVtGLRGRV H 
 HIHWLYDPHQWH
HIHWLYDPHQWH UHDOL]DGR QR H[HUFtFLR
'HSXWDGRV(VWDGXDLV DQWHULRU
2EVHVWH DUW$ VHXV LQFLVRV H SDUiJUDIRV IRUDP DFUHV
FHQWDGRVSHOD(PHQGD&RQVWLWXFLRQDOQž
F HP0XQLFtSLRVGHFLQTHQWDPLOHXPDFHPPLO
KDELWDQWHV R VXEVtGLR Pi[LPR GRV 9HUHDGRUHV 2EV DSDUWLUGHž-$1
ž-$1SRUIRUoDGRDUWžLQF,,GD(&Qž
ž-$1
FRUUHVSRQGHUiDTXDUHQWDSRUFHQWRGRVXEVtGLR HVVHVSHUFHQWXDLVSDVVDUmRDVHURVVHJXLQWHV
GRV'HSXWDGRV(VWDGXDLV
,  VHWHSRUFHQWR SDUD0XQLFtSLRVFRPSRSXOD
G HP0XQLFtSLRVGHFHPPLOHXPDWUH]HQWRVPLO omRGHDWp FHPPLO KDELWDQWHV
KDELWDQWHV R VXEVtGLR Pi[LPR GRV 9HUHDGRUHV
FRUUHVSRQGHUiDFLQTHQWDSRUFHQWRGRVXEVtGLR ,,  VHLVSRUFHQWR SDUD0XQLFtSLRVFRPSRSXOD
GRV'HSXWDGRV(VWDGXDLV omRHQWUH FHPPLO H WUH]HQWRV
PLO KDELWDQWHV
 'LU&RQVWLWXFLRQDO
 &kPDUD/HJLVODWLYDGR'LVWULWR)HGHUDO
,,,  FLQFRSRUFHQWR SDUD0XQLFtSLRVFRPSRSX 9, PDQWHUFRPDFRRSHUDomRWpFQLFDHILQDQFHLUDGD
ODomRHQWUH WUH]HQWRVPLOHXP H 8QLmRHGR(VWDGRSURJUDPDVGHHGXFDomRLQIDQ
TXLQKHQWRVPLO KDELWDQWHV WLOHGHHQVLQRIXQGDPHQWDO
2EVUHGDomRGDGDSHOD(PHQGD&RQVWLWXFLRQDOQž
,9  TXDWURLQWHLURVHFLQFRGpFLPRVSRUFHQWR
SDUD 0XQLFtSLRV FRP SRSXODomR HQWUH  9,, SUHVWDUFRPDFRRSHUDomRWpFQLFDHILQDQFHLUDGD
TXLQKHQWRVPLOHXP H WUrVPLOK}HV 8QLmR H GR (VWDGR VHUYLoRV GH DWHQGLPHQWR j
GHKDELWDQWHV VD~GHGDSRSXODomR

9  TXDWUR SRU FHQWR SDUD0XQLFtSLRVFRP 9,,, SURPRYHUQRTXHFRXEHUDGHTXDGRRUGHQDPHQWR


SRSXODomRHQWUH WUrVPLOK}HVHXP H WHUULWRULDOPHGLDQWHSODQHMDPHQWRHFRQWUROHGR
 RLWRPLOK}HV GHKDELWDQWHV XVR GR SDUFHODPHQWR H GD RFXSDomR GR VROR
XUEDQR
9,  WUrV LQWHLURV H FLQFRGpFLPRVSRUFHQWR
SDUD0XQLFtSLRVFRPSRSXODomRDFLPDGH ,; SURPRYHU D SURWHomR GRSDWULP{QLR
 RLWRPLOK}HVHXP KDELWDQWHV KLVWyULFRFXOWXUDOORFDOREVHUYDGDDOHJLVODomRHD
DomRILVFDOL]DGRUDIHGHUDOHHVWDGXDO
†ž $&kPDUD
&kPDUD0XQLFLSDO
0XQLFLSDOQmR
QmRJDVWDUi
JDVWDUiPDLVGHVHWHQWD
SRU
SRUFHQWR
FHQWRGH
GHVXD
VXDUHFHLWD
UHFHLWDFRP
FRPIROKD
IROKDGH
GHSDJDPHQ $UW $ ILVFDOL]DomR GR PXQLFtSLR VHUi H[HUFLGDSHOR
WR
WR LQFOXtGR
LQFOXtGR R JDVWR
JDVWR FRP
FRP R VXEVtGLR
VXEVtGLR GH VHXV 3RGHU
3RGHU/HJLVODWLYR0XQLFLSDOPHGLDQWHFRQWUROH
/HJLVODWLYR0XQLFLSDOPHGLDQWHFRQWUROH
9HUHDGRUHV
9HUHDGRUHV H[WHUQR
H[WHUQRHSHORV
SHORVVLVWHPDV
VLVWHPDVGH
GHFRQWUROH
FRQWUROHLQWHUQR
LQWHUQRGR
3RGHU([HFXWLYR0XQLFLSDOQDIRUPDGDOHL
†ž &RQVWLWXL
&RQVWLWXLFULPH
FULPHGHUHVSRQVDELOLGDGHGR3UHIHLWR
0XQLFLSDO †ž 2 FRQWUROH
FRQWUROH H[WHUQR
H[WHUQR GD &kPDUD0XQLFLSDOVHUi
H[HUFLGRFRP
H[HUFLGR FRPRDX[tOLRGRV7ULEXQDLVGH&RQWDV
, HIHWXDU UHSDVVH TXH VXSHUHRVOLPLWHVGHILQLGRV GRV(VWDGRVRXGR0XQLFtSLR
GRV(VWDGRVRXGR0XQLFtSLRRXGRV&RQVHOKRV
QHVWHDUWLJR RX
RX 7ULEXQDLV
7ULEXQDLV GH
GH &RQWDV
&RQWDV GRV
GRV 0XQLFtSLRV
0XQLFtSLRV RQGH
KRXYHU
,, QmRHQYLDURUHSDVVHDWpRGLDYLQWHGHFDGDPrV
RX †ž 2SDUHFHU
SDUHFHUSUpYLR
SUpYLRHPLWLGR
HPLWLGRSHOR
SHORyUJmR
yUJmRFRPSHWHQ
WH
WH VREUH
VREUH DV
DV FRQWDV
FRQWDV TXH
TXH R 3UHIHLWR
3UHIHLWR GHYH
,,, HQYLiORDPHQRUHPUHODomRjSURSRUomRIL[DGD DQXDOPHQWH
DQXDOPHQWH SUHVWDU
SUHVWDU Vy
Vy  GHL[DUi GH SUHYDOHFHU
QD/HL2UoDPHQWiULD SRU
SRU  GHFLVmR GH GRLV WHUoRV GRV PHPEURV GD
†ž &RQVWLWXL
&RQVWLWXLFULPH
FULPHGHUHVSRQVDELOLGDGHGR3UHVLGHQ
GHUHVSRQVDELOLGDGHGR3UHVLGHQ &kPDUD0XQLFLSDO
WH
WH GD
GD &kPDUD
&kPDUD 0XQLFLSDO
0XQLFLSDO R GHVUHVSHLWR DR † ž
GHVWHDUWLJR †ž $V FRQWDV GRV 0XQLFtSLRV ILFDUmRGXUDQWHVHV
2EV††žžHžDFUHVFLGRVSHOD(&Qž VHQWD
VHQWDGLDV
GLDVDQXDOPHQWH
DQXDOPHQWHjGLVSRVLomR
GLVSRVLomRGH
GHTXDOTXHU
FRQWULEXLQWH
FRQWULEXLQWH SDUD
SDUD H[DPH
H[DPH H DSUHFLDomR
DSUHFLDomR R TXDO
SRGHUi
SRGHUi TXHVWLRQDUOKHV
TXHVWLRQDUOKHV D OHJLWLPLGDGH
OHJLWLPLGDGH QRV
&RPSHWrQFLDGRV0XQLFtSLRV WHUPRVGDOHL

†ž e YHGDGD
YHGDGD D FULDomR
FULDomR GH
GH 7ULEXQDLV&RQVHOKRVRX
7ULEXQDLV&RQVHOKRVRX
$UW&RPSHWHDRV0XQLFtSLRV
&RPSHWHDRV0XQLFtSLRV yUJmRVGH&RQWDV0XQLFLSDLV

, OHJLVODUVREUHDVVXQWRVGHLQWHUHVVHORFDO $V DWLYLGDGHV PXQLFLSDLV Vy GL]HP UHVSHLWR jTXHOD


ORFDOLGDGHDRVLQWHUHVVHVGRSRYRGDTXHOHOXJDU1HVVH
,, VXSOHPHQWDUDOHJLVODomRIHGHUDOHHVWDGXDOQRTXH VHQWLGRR0XQLFtSLRGHYHUiSURPRYHUWRGDVDVPHGLGDV
FRXEHU SDUD DWHQGLPHQWR GDV QHFHVVLGDGHV H LQWHUHVVHV GH VHX
SRYR ID]HU VXDV UHVSHFWLYDV OHLV DUUHFDGDU RV WULEXWRV
,,, LQVWLWXLUHDUUHFDGDURVWULEXWRVGHVXDFRPSHWrQ PXQLFLSDLV SDUD SDJDPHQWR GH VXDV GHVSHVDVREUDV
FLDEHPFRPRDSOLFDUVXDVUHQGDVVHPSUHMXt]R RUJDQL]DURVVHUYLoRVS~EOLFRV HVFRODVWUDQVSRUWHVVD~GHHWF
GDREULJDWRULHGDGHGHSUHVWDUFRQWDVHSXEOLFDU QR 0XQLFtSLR HWF &RPR WDPEpP QRV 0XQLFtSLRV p
EDODQFHWHVQRVSUD]RVIL[DGRVHPOHL DUUHFDGDGRHJDVWRGLQKHLURGRSRYRDVFRQWDVHVWDUmR
VXMHLWDVjGHYLGDFRQWDELOLGDGHHVHUmRILVFDOL]DGDVSHOD
,9 FULDURUJDQL]DUHVXSULPLUGLVWULWRVREVHUYDGDD &kPDUD GH 9HUHDGRUHV TXH SRU VXD YH] WDPEpP VHUi
OHJLVODomRHVWDGXDO ILVFDOL]DGD SHOR 7ULEXQDO
7ULEXQDO GH
GH &RQWDV
&RQWDV GR
GR (VWDGR
(VWDGR ³
REVHUYDQGRVHTXHHPDOJXQV0XQLFtSLRVGR%UDVLOH[LVWH
9 RUJDQL]DUHSUHVWDUGLUHWDPHQWHRXVREUHJLPHGH XP7ULEXQDOGH&RQWDV0XQLFLSDOTXHID]HVVDILVFDOL]D
FRQFHVVmRRXSHUPLVVmRRVVHUYLoRVS~EOLFRVGH omR
LQWHUHVVHORFDOLQFOXtGRRGHWUDQVSRUWHFROHWLYR  'HVGH D &RQVWLWXLomR GD 5HS~EOLFD GH  RV
TXHWHPFDUiWHUHVVHQFLDO 0XQLFtSLRVQmRPDLVSRGHUmRFULDUVHXSUySULR7ULEX
&kPDUD/HJLVODWLYDGR
  'LVWULWR)HGHUDO 'LU&RQVWLWXFLRQDO 

QDO RX&RQVHOKRRXyUJmRGHTXDOTXHURXWURQRPH GH &RPSHWrQFLD&RQFRUUHQWH


2DUW&)HVWDEHOHFHD&RPSHWrQFLD
&RPSHWrQFLD&RQFRUUHQWHGR
ILVFDOL]DomR GH FRQWDV RV Mi H[LVWHQWHV FRQWLQXDP VXDV 'LVWULWR)HGHUDO
'LVWULWR)HGHUDO
DWLYLGDGHVPDVQmRPDLVVHUmRFULDGRVQRYRV7ULEXQDLV$
H[LVWrQFLDGHFRQWUROHSRU7ULEXQDLVGH&RQWDVQmRREVWD
2V7HUULWyULRV
TXHRSRYRWDPEpPSRVVDFRQWURODUDVFRQWDVGRV0XQLFt
SLRV SDUD WDQWR DV FRQWDV GHYHP ILFDU j GLVSRVLomR GH
TXDOTXHUFRQWULEXLQWH³WRGDSHVVRDTXHSDJDLPSRVWRV ¢2TXHpXP7HUULWyULR"eXPDUHJLmRWHUULWRULDOTXH
³GXUDQWHGLDVSDUDTXHSRVVDPVHUH[DPLQDGDVH QmRVHQGRXP(VWDGRGHYHVHUDGPLQLVWUDGRSHOD8QLmR
DOJXPD SHVVRD HYHQWXDOPHQWH LQWHUHVVHVH RX WHQKD D 3RUVHUXPDYDVWDUHJLmRVHPHOKDQWHPHQWHDXP(VWDGR
LQLFLDWLYDGHLPSXJQDUVXDYDOLGDGHRXUHJXODULGDGH Ki QRV WHUULWyULRV PXLWRV LQWHUHVVHV FRPXQV jTXHOD
UHJLRQDOLGDGHHTXHGHYHPVHUJHULGRVSDUDDWHQGHUj
UHVSHFWLYDSRSXODomR

'LVWULWR)HGHUDOH7HUULWyULRV 6XFHGH HQWUHWDQWR TXH DWXDOPHQWH QmR Ki PDLV


mR Ki
7HUULWyULRV)HGHUDLV
7HUULWyULRV )HGHUDLVXPDYH]TXHWRGRVIRUDPWUDQVIRU
GHUDLV
PDGRVHP(VWDGRV0HPEURV³RTXHLPSHGHD8QLmRGH
$UW2
2 'LVWULWR )HGHUDO YHGDGD VXD GLYLVmRHP H[HUFLWDU HVVD IDWLD GH VXD FRPSHWrQFLD $FUH $PDSi
0XQLFtSLRVUHJHUVHiSRUOHLRUJkQLFDYRWDGD
0XQLFtSLRVUHJHUVHiSRUOHLRUJkQLFDYRWDGD 5RQG{QLD 5RUDLPD H  %RD 9LVWD ³ WRGRV VmR KRMH
HP
HP GRLV
GRLV WXUQRV
WXUQRV FRP
FRP LQWHUVWtFLR
LQWHUVWtFLR PtQLPR
PtQLPR GH GH] (VWDGRVPHPEURV³HQTXDQWR)HUQDQGRGH1RURQKDR
GLDV
GLDV H DSURYDGD
DSURYDGD SRU
SRU GRLV
GRLV WHUoRV
WHUoRV GD
GD &kPDUD ~OWLPRGHOHVIRLLQFRUSRUDGRDR(VWDGRGH3HUQDPEXFR
/HJLVODWLYD
/HJLVODWLYDTXH
TXHDSURPXOJDUi
SURPXOJDUiDWHQGLGRV
DWHQGLGRVRV
RVSULQ
FtSLRVHVWDEHOHFLGRVQHVWD&RQVWLWXLomR &RQILUDDVGLVSRVLo}HVFRQVWLWXFLRQDLVVREUHRVWHUULWyULRV

†ž $R'LVWULWR
'LVWULWR)HGHUDO
)HGHUDOVmR
VmRDWULEXtGDVDVFRPSHWrQFL
DWULEXtGDVDVFRPSHWrQFL $UW $OHL
OHLGLVSRUi
GLVSRUiVREUH
VREUHDRUJDQL]DomRDGPLQLVWUDWL
DV
DVOHJLVODWLYDV
OHJLVODWLYDV UHVHUYDGDV
UHVHUYDGDVDRV
DRV(VWDGRV
(VWDGRVH0XQLFt YDHMXGLFLiULDGRV7HUULWyULRV
SLRV
†ž 2V
2V7HUULWyULRV
7HUULWyULRVSRGHUmR
SRGHUmRVHU
VHUGLYLGLGRVHP0XQL
†ž $HOHLomR
HOHLomRGR
GR*RYHUQDGRU
*RYHUQDGRUHGR9LFH*RYHUQDGRU FtSLRVDRV
FtSLRV DRVTXDLV
TXDLVVHDSOLFDUiQR
VHDSOLFDUiQRTXH
TXHFRXEHU
FRXEHUR
REVHUYDGDV
REVHUYDGDVDVDVUHJUDV
UHJUDVGR
GRDUW
DUWHGRV
GRV'HSXWDGRV GLVSRVWRQR&DStWXOR,9GHVWH7tWXOR
'LVWULWDLV
'LVWULWDLV FRLQFLGLUi
FRLQFLGLUi FRP
FRP DGRV*RYHUQDGRUHVH
'HSXWDGRV
'HSXWDGRV (VWDGXDLV
(VWDGXDLV SDUD
SDUD PDQGDWR
PDQGDWR GH
GH LJXDO †ž $V FRQWDV GR *RYHUQR GR7HUULWyULRVHUmR
GXUDomR VXEPHWLGDV
VXEPHWLGDVDR
DR&RQJUHVVR
&RQJUHVVR1DFLRQDO
1DFLRQDOFRP
FRPSDUH
FHUSUpYLRGR7ULEXQDOGH&RQWDVGD8QLmR
†ž $RV
$RV'HSXWDGRV
'HSXWDGRV'LVWULWDLV
'LVWULWDLVHj&kPDUD/HJLVODWLYD
DSOLFDVHRGLVSRVWRQRDUW †ž 1RV7HUULWyULRV)HGHUDLVFRP
1RV7HUULWyULRV)HGHUDLVFRPPDLVPDLVGH
GHFHPPLO
KDELWDQWHV DOpP
KDELWDQWHV DOpP GR
GR *RYHUQDGRU
*RYHUQDGRU QRPHDGR
QRPHDGR QD
†ž /HL IHGHUDO GLVSRUi VREUH DXWLOL]DomRSHOR IRUPD
IRUPDGHVWD
GHVWD&RQVWLWXLomRKDYHUiyUJmRVMXGL
&RQVWLWXLomRKDYHUiyUJmRVMXGL
*RYHUQRGR'LVWULWR)HGHUDOGDVSROtFLDV
*RYHUQRGR'LVWULWR)HGHUDOGDVSROtFLDVFLYLO
FLYLOH FLiULRV
FLiULRVGH
GHSULPHLUD
SULPHLUDHVHJXQGD
VHJXQGDLQVWkQFLD
LQVWkQFLDPHP
PLOLWDUHGRFRUSRGHERPEHLURVPLOLWDU EURVGR0LQLVWpULR3~EOLFR
EURVGR0LQLVWpULR3~EOLFRHGHIHQVRUHV
GHIHQVRUHVS~EOL
FRV
FRV IHGHUDLV
IHGHUDLV D OHL GLVSRUi VREUH DV HOHLo}HV
2'LVWULWR)HGHUDOpWUDWDGRQD&RQVWLWXLomRFRPRVH SDUD
SDUD D &kPDUD7HUULWRULDOHVXDFRPSHWrQFLD
&kPDUD7HUULWRULDOHVXDFRPSHWrQFLD
IRVVHDRPHVPRWHPSR D D XP(VWDGRH E E XP0XQLFt GHOLEHUDWLYD
SLR2EVHUYHTXHHOHWHPOHLRUJkQLFDTXDOWRGR0XQLFt
SLR WHP H QmR &RQVWLWXLomR FRPR RV (VWDGRV  WHP
*RYHUQDGRUFRPRRV(VWDGRVWrP HQmR3UHIHLWRFRPR
RV0XQLFtSLRV HWrP'HSXWDGRV'LVWULWDLVQXPD&kPDUD ,QWHUYHQomRQRV(VWDGRVH0XQLFtSLRV
/HJLVODWLYD QRPH PLVWR UHVXOWDQWH GDV &kPDUDV GH
9HUHDGRUHVGRV0XQLFtSLRVHGH$VVHPEOpLDV/HJLVODWLYDV
GRV(VWDGRV  8PGRVSULQFtSLRVPDLVLPSRUWDQWHVGD&RQVWLWXLomR
pDDXWRQRPLDDGPLQLVWUDWLYDGRV(VWDGRV0HPEURVHGRV
7ULEXWDULDPHQWHR'LVWULWR)HGHUDOFREUDRVPHVPtVVL 0XQLFtSLRV H WDPEpP GD 8QLmR p FODUR  &DGD XP
PRVLPSRVWRV TXHSRGHPVHUFREUDGRVSHORV(VWDGRVH DGPLQLVWUD VHXV LQWHUHVVHV H SUREOHPDV GD IRUPD TXH
SHORV0XQLFtSLRV LVVRPHVPRRVWULEXWRVGRVGRLV  PHOKRUFRQYLHUDVHXSRYR

0DLVTXHLVVRR'LVWULWR)HGHUDOWHUiVXDSUySULD3ROtFLD 0DVHVVDUHJUDQmR
QmRpDEVROXWDSRGHUiRFRUUHUTXH
QmR
0LOLWDUSRGHUFRQIHULGRDSHQDVDRV(VWDGRVHSURLELGR QDDGPLQLVWUDomRGHXP0XQLFtSLRVHX3UHIHLWR RXQDGH
DRV0XQLFtSLRV XP(VWDGRVHX*RYHUQDGRU FRPHWDDEXVRVTXHSUHMXGL
TXHPjSRSXODomRRXDRHUiULRS~EOLFRRXDWpSUHMXGL
(QILPR'LVWULWR)HGHUDOpDRPHVPRWHPSRXP(VWD TXHDR3DtVFRPRXPWRGRVHQmRDWRGDXPD5HJLmRRX
GRHXP0XQLFtSLRDWtSLFRV PHVPRDXPRXWUR(VWDGRRXDXPRXWUR0XQLFtSLR
 'LU&RQVWLWXFLRQDO
 &kPDUD/HJLVODWLYDGR'LVWULWR)HGHUDO
¢&RPRUHVROYHURSUREOHPDVHIRVVHDEVROXWDD ,, QmRIRUHPSUHVWDGDVFRQWDVGHYLGDVQDIRUPDGDOHL
DXWRQRPLD "  $OJXpP SUHFLVD DVVXPLU D DGPLQLVWUDomR
DIDVWDQGR R DGPLQLVWUDGRU LUUHVSRQViYHO SDUD SRU ILP ,,, QmR WLYHU VLGR DSOLFDGR R PtQLPR H[LJLGRGD
jTXHOD VLWXDomR GDQRVD ,VVR VH FKDPD LQWHUYHQomR
LQWHUYHQomR UHFHLWD PXQLFLSDO QD PDQXWHQomR H GHVHQYROYL
6HPSUH TXH IRU QHFHVViULD D LQWHUYHQomR
LQWHUYHQomR VH GDUi GD PHQWRGRHQVLQRHQDVDo}HVHVHUYLoRVS~EOLFRV
VHJXLQWHIRUPDR(VWDGRLQWHUYpPQRVVHXV0XQLFtSLRVH GHVD~GH LQFLVRDOWHUDGRSHOD(PHQGD&RQVWLWXFLRQDOQž
D8QLmRLQWHUYpPQR(VWDGR0HPEURHQR'LVWULWR)HGH
UDO ,9 R7ULEXQDOGH-XVWLoDGHUSURYLPHQWRDUHSUHVHQ
WDomRSDUDDVVHJXUDUDREVHUYkQFLDGHSULQFtSLRV
'HVWDTXHVHSRUpPDQDWXUH]DH[FHSFLRQDOtVVLPDGD
LQGLFDGRV QD &RQVWLWXLomR (VWDGXDO RX SDUD
LQWHUYHQomRGDtDH[LJrQFLDGHFXLGDGRVH[WUDRUGLQiULRV
SURYHUDH[HFXomRGHOHLGHRUGHPRXGHGHFLVmR
HUDGLFDLV SDUDWRPDGDGHVVDSRVLomRRVWUrV3RGHUHV MXGLFLDO
SDUWLFLSDUmRGDLQWHUYHQomRSRLV D D R'HFUHWRGH,QWHU
D
YHQomRHDPHGLGDVmRGR([HFXWLYR E E R-XGLFLiULRWHP
E $UW $GHFUHWDomRGDLQWHUYHQomRGHSHQGHUi
$GHFUHWDomRGDLQWHUYHQomRGHSHQGHUi
GHMXOJDUSURFHGHQWHRSHGLGR QDPDLRULDGRVFDVRV H
F R /HJLVODWLYR GHYH DSUHFLiOR ( D ,QWHUYHQomR WHP
F  , QRFDVRGRDUW,9GHVROLFLWDomRGR3RGHU/H
WUDQVLWyULD RX SUHFiULD SDVVDJHLUD  SRLV R
QDWXUH]D WUDQVLWyULD JLVODWLYRRXGR3RGHU([HFXWLYRFRDFWRRXLPSHGL
,QWHUYHQWRUDVVXPLUiD$GPLQLVWUDomR3~EOLFDH[FOXVLYD GRRXGHUHTXLVLomRGR6XSUHPR7ULEXQDO)HGH
PHQWHSDUDVDQHDU
VDQHDURSUREOHPDTXHPRWLYRXDLQWHUYHQ
VDQHDU UDO VH D FRDomR IRU H[HUFLGD FRQWUD R 3RGHU
omR5HVROYLGRRFDVRD$XWRULGDGHVHUiUHFRQGX]LGDDR -XGLFLiULR
SRGHU
,, QR FDVR GH GHVREHGLrQFLD D RUGHPRXGHFLVmR
$UW $ 8QLmR QmR LQWHUYLUi QRV (VWDGRV QHPQR MXGLFLiULD GH UHTXLVLomR GR 6XSUHPR 7ULEXQDO
'LVWULWR)HGHUDOH[FHWRSDUD )HGHUDOGR6XSHULRU7ULEXQDOGH-XVWLoDRXGR
7ULEXQDO6XSHULRU(OHLWRUDO
, PDQWHUDLQWHJULGDGHQDFLRQDO
PDQWHUDLQWHJULGDGHQDFLRQDO
,,, GHSURYLPHQWRSHOR6XSUHPR7ULEXQDO)HGHUDO
,, UHSHOLU
UHSHOLULQYDVmR
LQYDVmRHVWUDQJHLUD
HVWUDQJHLUDRX RXGH
GHXPDXQLGDGH
GHUHSUHVHQWDomRGR3URFXUDGRU*HUDOGD5HS~
GD)HGHUDomRHPRXWUD EOLFD QD KLSyWHVH GR DUW9,, H QR
QR FDVR
FDVR GH
,,, SRU
SRUWHUPR
WHUPRDJUDYH
JUDYHFRPSURPHWLPHQWR
FRPSURPHWLPHQWRGD GDRUGHP UHFXVDjH[HFXomRGHOHLIHGHUDO
UHFXVDjH[HFXomRGHOHLIHGHUDO
S~EOLFD
S~EOLFD
2EV SDUWHILQDOGHVWHLQFLVR,,,DFUHVFLGDSHOD(PHQGD&RQVWLWXFLRQDOQžD
,9 JDUDQWLU R OLYUH H[HUFtFLR GH TXDOTXHUGDV 5HIRUPDGR3RGHU-XGLFLiULR³FRQFHQWUDQGRQDFRPSHWrQFLDGR67)WDPEpP
3RGHUHVQDVXQLGDGHVGD)HGHUDomR
3RGHUHVQDVXQLGDGHVGD)HGHUDomR DKLSyWHVHGH´UHFXVDjH[HFXomRGDOHLIHGHUDOµTXHDQWHVHUDGR67-

9 UHRUJDQL]DU
UHRUJDQL]DUDV
DVILQDQoDV
ILQDQoDVGD
GDXQLGDGH
XQLGDGHGD GD)HGHUD ,9 UHYRJDGRSHOD(PHQGD&RQVWLWXFLRQDOQžD5HIRUPDGR3RGHU-XGLFLiULR

omRTXH
†ž 2 GHFUHWR GH LQWHUYHQomR TXHHVSHFLILFDUiD
D VXVSHQGHURSDJDPHQWRGDGtYLGDIXQGDGDSRU
DPSOLWXGH
DPSOLWXGHRSUD]R
SUD]RHDV
DVFRQGLo}HV
FRQGLo}HVGH
GHH[HFXomR
H[HFXomR
PDLVGHGRLVDQRVFRQVHFXWLYRVVDOYRPRWLYRGH H TXH VH FRXEHU QRPHDUi R LQWHUYHQWRU VHUi
IRUoDPDLRU VXEPHWLGR
VXEPHWLGRjDSUHFLDomRGR&RQJUHVVR
jDSUHFLDomRGR&RQJUHVVR1DFLRQDO
E GHL[DUGHHQWUHJDUDRV0XQLFtSLRVUHFHLWDVWULEX RX
RX GD
GD $VVHPEOpLD
$VVHPEOpLD /HJLVODWLYD
/HJLVODWLYD GR (VWDGR QR
WiULDV IL[DGDV QHVWD &RQVWLWXLomR GHQWUR GRV SUD]RGHYLQWHHTXDWURKRUDV
SUD]RGHYLQWHHTXDWURKRUDV
SUD]RVHVWDEHOHFLGRVHPOHL
9, SURYHU D H[HFXomR GH OHL IHGHUDORUGHPRX †ž 6HQmR
6HQmRHVWLYHUIXQFLRQDQGRR&RQJUHVVR1DFLR
GHFLVmRMXGLFLDO
GHFLVmRMXGLFLDO QDO
QDORXD
RXD$VVHPEOpLD
$VVHPEOpLD/HJLVODWLYD
/HJLVODWLYDIDUVHi
IDUVHiFRQYR
9,, DVVHJXUDU
DVVHJXUDUDREVHUYkQFLD
REVHUYkQFLDGRVGRVVHJXLQWHV
VHJXLQWHVSULQFtSL FDomRH[WUDRUGLQiULD
FDomRH[WUDRUGLQiULDQR
QRPHVPR
PHVPRSUD]R
SUD]RGHYLQWH
RVFRQVWLWXFLRQDLV HTXDWURKRUDV
HTXDWURKRUDV
D IRUPD UHSXEOLFDQD VLVWHPD UHSUHVHQWDWLYR H
UHJLPHGHPRFUiWLFR †ž 1RV
RV FDVRV
FDVRV GR
GR DUW
DUW 9,
9, H 9,,
9,, RX
RX GRDUW,9
E GLUHLWRVGDSHVVRDKXPDQD GLVSHQVDGD
GLVSHQVDGDDDSUHFLDomRSHOR&RQJUHVVR
DDSUHFLDomRSHOR&RQJUHVVR1DFLR
F DXWRQRPLDPXQLFLSDO QDO
QDO RX
RX SHOD $VVHPEOpLD /HJLVODWLYD R GHFUHWR
G SUHVWDomRGHFRQWDVGDDGPLQLVWUDomRS~EOLFD OLPLWDUVHi
OLPLWDUVHi D VXVSHQGHU D H[HFXomR GR DWR
GLUHWDHLQGLUHWD LPSXJQDGR
LPSXJQDGRVH VHHVVD
HVVDPHGLGD
PHGLGDEDVWDU
EDVWDUDRDRUHVWDEHOH
FLPHQWRGDQRUPDOLGDGH
FLPHQWRGDQRUPDOLGDGH
H DSOLFDomRGRPtQLPRH[LJLGRGDUHFHLWDUHVXOWDQ
WHGHLPSRVWRVHVWDGXDLVFRPSUHHQGLGDDSUR
†ž &HVVDGRVRVPRWLYRVGDLQWHUYHQomRDVDXWRUL
&HVVDGRVRVPRWLYRVGDLQWHUYHQomRDVDXWRUL
YHQLHQWHGHWUDQVIHUrQFLDVQDPDQXWHQomRHGH GDGHV
GDGHVDIDVWDGDV
DIDVWDGDVGH
GHVHXV
VHXVFDUJRV
FDUJRVDHVWHV
HVWHVYROWDUmR
VHQYROYLPHQWRGRHQVLQRHQDVDo}HVHVHUYLoRV VDOYRLPSHGLPHQWROHJDO
VDOYRLPSHGLPHQWROHJDO
S~EOLFRVGHVD~GH DOWHUDGDSHOD(PHQGD&RQVWLWXFLRQDOQž
UDQVLWyULD RX SDVVDJHLUD  GD
$QRWHVH D QDWXUH]D WUDQVLWyULD
$UW 2(VWDGR
(VWDGRQmR
QmRLQWHUYLUi
LQWHUYLUiHP
HPVHXV
VHXV0XQLFtSLRVQHP
LQWHUYHQomRR,QWHUYHQWRUDVVXPLUiD$GPLQLVWUDomR3~
D8QLmRQRV0XQLFtSLRV
D8QLmRQRV0XQLFtSLRVORFDOL]DGRV
ORFDOL]DGRVHP
HP7HUULWy EOLFDH[FOXVLYDPHQWHSDUDVDQHDU
VDQHDURSUREOHPDTXHPRWLYRX
VDQHDU
ULR)HGHUDOH[FHWRTXDQGR DLQWHUYHQomR$VVLPWmRORJRUHVROYLGRWDOSUREOHPDD
$XWRULGDGHTXHI{UDDIDVWDGDSRUTXHQmRFRQVHJXLDUHVRO
, GHL[DUGHVHUSDJDVHPPRWLYRGHIRUoDPDLRU YrORVHUiUHFRQGX]LGDDRFDUJRUHDVVXPLQGRVXDVIXQ
SRUGRLVDQRVFRQVHFXWLYRVDGtYLGDIXQGDGD o}HV
&kPDUD/HJLVODWLYDGR
  'LVWULWR)HGHUDO 'LU&RQVWLWXFLRQDO 

GHFRQGXWDVSUySULDVGDGLVFLSOLQDLQWHULRUGD$GPLQLVWUD
$GPLQLVWUDomR3~EOLFD omRGHVGHVXDQDWXUH]DMXUtGLFDVXDILQDOLGDGHLQVWUX
PHQWDomRHSHLDVpWLFDVHOHJDLV

&XPSUHFRQVLGHUDUTXHRVDJHQWHVS~EOLFRVVmRPHURV 2SULQFtSLRGDSXEOLFLGDGH
SXEOLFLGDGHLPSHGLUiTXHD$GPLQLVWUD
SXEOLFLGDGH
LQVWUXPHQWRV GH DomR GD $GPLQLVWUDomR 3~EOLFD H TXH omR 3~EOLFD SUDWLTXH DWRV VHFUHWRV SDUD QD PRLWD
FDGDXPDGDVWUrVHVIHUDVEiVLFDVGD$GPLQLVWUDomR 8QLm SUHMXGLFDURXEHQHILFLDUDTXHPTXHUTXHVHMDDSXEOLFL
R(VWDGR0HPEURH0XQLFtSLRV EHPFRPRVXDVYDULDQ GDGHJDUDQWLUiSHODWUDQVSDUrQFLDGHVHXVDWRVOHYDGRV
WHVLQGLUHWDV DXWDUTXLDVIXQGDo}HVHWF SRGHUiFRQIHULU DRFRQKHFLPHQWRJHUDOXPDILVFDOL]DomR
ILVFDOL]DomR
ILVFDOL]DomRGH
GHWRGD
WRGDFROHWLYL
SRGHUHVSUHUURJDWLYDVHGLUHLWRVDVHXVVHUYLGRUHVPDV GDGHSRVVLELOLWDQGRTXHLQWHUHVVDGRVRXPHVPRFLGDGmRV
GDGH
RV GLUHLWRV RV SRGHUHV DV SUHUURJDWLYDV H RV GHYHUHV SRVVDP LQWHUSRU UHFXUVRV DGPLQLVWUDWLYRV RX SURPRYHU
EiVLFRVHVWmRSUHYLVWRVQRWH[WRFRQVWLWXFLRQDOHGHYHUmR
EiVLFRV Do}HVMXGLFLDLVDGHTXDGDVSDUDFRPEDWHURVDWRVLUUHJXOD
VHUREHGHFLGRVSRUWRGDVDVHVIHUDVGHSRGHU UHVRXLOHJDLV 

$&RQVWLWXLomR)HGHUDO³DRGLVFLSOLQDUD$GPLQLVWUD $ HPHQGD FRQVWLWXFLRQDO Qž  GHQRPLQDGD


omR 3~EOLFD ³ UHVHUYRX D 6HomR ,, DUW  H   ´5HIRUPD $GPLQLVWUDWLYDµ WRUQRX H[SUHVVR PDLV XP
HVSHFLILFDPHQWHSDUDGLVSRUVREUHRV6HUYLGRUHV3~EOLFRV SULQFtSLR D VHU REVHUYDGR SHOD $GPLQLVWUDomR 3~EOLFD
³PDVQHPSRULVVRGHL[RXGHHVWDEHOHFHUUHJUDVJHUDLV %UDVLOHLUDRSULQFtSLRGDHILFLrQFLD
HILFLrQFLD$PiTXLQDDGPLQLV
HILFLrQFLD
LPSRUWDQWHV VREUH RV IXQFLRQiULRV S~EOLFRV QD 6HomR , WUDWLYDpGHPDVLDGDPHQWHOHQWDHEXURFUiWLFDWDLVGHVFD
DUWH  PLQKRV EXURFUiWLFRV KDYHUmR GH VHU HOLPLQDGRV H[DWD
PHQWHSDUDTXHDDGPLQLVWUDomRS~EOLFDDOFDQFHUDSLGH]
$VVLPpLPSRUWDQWHRFRQKHFLPHQWRLQWHJUDOGRVDUW HHILFiFLDQRDWHQGLPHQWRjSRSXODomR$HILFLrQFLDpQD
DGD&DUWD0DJQDSDUDRSOHQDDYDOLDomRGRVSULQ H[SUHVVmRVLQWpWLFDGH&$59$/+26,0$62´GHYHUGHERD
FtSLRVFRQVWLWXFLRQDLVTXHUHJHPDVUHODo}HVGRV6HUYLGR DGPLQLVWUDomRµ LQ´0DQXDO(OHPHQWDUGH'LUHLWR$GPLQLVWUDWLYRµHG
UHV 3~EOLFRV FRP D $GPLQLVWUDomR 3~EOLFD GHVGH VXD 'HS 
DGPLVVmR REULJDWyULD SRU FRQFXUVR HVWiJLR SUREDWyULR
VXDVUHVSRQVDELOLGDGHVVHXVGLUHLWRVHREULJDo}HVHVXD $UW $DGPLQLVWUDomR
DGPLQLVWUDomRS~EOLFD
S~EOLFDGLUHWD
GLUHWDHLQGLUHWD
LQGLUHWDGHTXDO
TXHUGRV3RGHUHVGD8QLmRGRV(VWDGRVGR'LVWULWR)HGHUDOH
ILQDODSRVHQWDGRULD&RQILUDDVHJXLU GRV0XQLFtSLRVREHGHFHUiDRVSULQFtSLRVGHOHJDOLGDGHLPSHVVR
DOLGDGH PRUDOLGDGH SXEOLFLGDGH H HILFLrQFLD H WDPEpP DR
$UW $DGPLQLVWUDomR
DGPLQLVWUDomRS~EOLFD
S~EOLFDGLUHWD
GLUHWDHLQGLUHWD
LQGLUHWDGHTXDO
GH VHJXLQWH
TXHUGRV3RGHUHVGD8QLmRGRV(VWDGRVGR'LVWUL
WR)HGHUDOHGRV0XQLFtSLRVREHGHFHUi
REHGHFHUi
REHGHFHUiDRV
DRVSULQFt , RVFDUJRVHPSUHJRVHIXQo}HVS~EOLFDVVmRDFHV
SLRV
SLRVGHOHJDOLGDGHLPSHVVRDOLGDGHPRUDOLGDGH VtYHLVDRVEUDVLOHLURVTXHSUHHQFKDPRVUHTXLVLWRV
SXEOLFLGDGHHHILFLrQFLDHWDPEpPDRVHJXLQWH
SXEOLFLGDGHHHILFLrQFLD HVWDEHOHFLGRVHPOHLDVVLPFRPRDRVHVWUDQJHLURV
QDIRUPDGDOHL
&RPRYLPRVRSULQFtSLRGDOOHJDOLGDGHVLJQLILFDTXH
HJDOLGDGH
WDLV yUJmRV DWXDUmR VXEPLVVRV j OHLHL GHVGH D FULDomR H ,, DLQYHVWLGXUDHPFDUJRRXHPSUHJRS~EOLFRGHSHQ
H[WLQomRGHFDUJRVFRPRHPWRGDFRQWUDWDomRLQFOXVLYH GHGHDSURYDomRSUpYLDHPFRQFXUVRS~EOLFR GH
GH SHVVRDO &(/62 $1721,2 %$1'(,5$ '( 0(//2 SURYDV RX GH SURYDV H WtWXORV GH DFRUGR FRP D
DQRWDTXHDH[SUHVVmROHJDOLGDGHQmRVHUHVWULQJHDWH[WR QDWXUH]DHDFRPSOH[LGDGHGRFDUJRRXHPSUHJR
GHOHLQRVHQWLGRHVWULWRGDH[SUHVVmRpPDLVDEUDQJHQWH QDIRUPDSUHYLVWDHPOHLUHVVDOYDGDVDVQRPHD
HYLQFXODDDWXDomRGD$GPLQLVWUDomR3~EOLFDjFRQIRUPL o}HVSDUDFDUJRHPFRPLVVmRGHFODUDGRHPOHLGH
GDGH FRP R 'LUHLWR LQ (OHPHQWRV GH 'LUHLWR $GPLQLVWUDWLYR OLYUHQRPHDomRHH[RQHUDomR
S 
 2 SULQFtSLR GD LPSHVVRDOLGDGH
LPSHVVRDOLGDGH
OLGDGH QD H[SUHVVmR GH ,,, RSUD]RGHYDOLGDGHGRFRQFXUVRS~EOLFRVHUiGH
3,172)(55(,5$VLJQLILFDTXHRDWRDGPLQLVWUDWLYRQmR DWpGRLVDQRVSURUURJiYHOXPDYH]SRULJXDOSH
GHYHVHUHGLWDGRQHPHODERUDGRWHQGRSRUREMHWLYREHQHILFLDU UtRGR
D SHVVRD GH DOJXpP DQRWDQGR HP VHJXLGD TXH 1R
GLUHLWRFRPSDUDGRRSULQFtSLRGDLPSHVVRDOLGDGHpFKDPD ,9 GXUDQWHRSUD]RLPSURUURJiYHOSUHYLVWRQRHGLWDO
GR GH SULQFtSLR GD ILQDOLGDGH DGPLQLVWUDWLYD 3DUD &DLR GH FRQYRFDomR DTXHOH DSURYDGR HP FRQFXUVR
7iFLWRRSULQFtSLRGDILQDOLGDGHDGPLQLVWUDWLYDpFRORUiULR S~EOLFR GH SURYDV RX GH SURYDV H WtWXORV VHUi
HVVHQFLDOGRSULQFtSLRGDOHJDOLGDGHSUHWHQGHQGRFRPLVVRTXHWRGD
FRQYRFDGRFRPSULRULGDGHVREUHQRYRVFRQFXUVD
D DWLYLGDGH HVWDWDO VH GLULMD DR HQWHQGLPHQWR GH XP LQWHUHVVH S~EOLFR
GRVSDUDDVVXPLUFDUJRRXHPSUHJRQDFDUUHLUD
TXDOLILFDGRU3URVVHJXHHOH$UHJUDLQYDULiYHOpSRUWDQWRDGHTXH
HPQHQKXPDKLSyWHVHSRGHDDXWRULGDGHVXEVWLWXLURILPQDOHLSRURXWUR
S~EOLFR RX SULYDGR OtFLWR RX LOtFLWR LQ &RPHQWiULRV j &RQVWLWXLomR 9 DVIXQo}HVGHFRQILDQoDH[HUFLGDVH[FOXVLYDPHQWH
%UDVLOHLUDžYROHG6DUDLYDSH  SRU VHUYLGRUHV RFXSDQWHV GH FDUJR HIHWLYR H RV
FDUJRV HP FRPLVVmR D VHUHP SUHHQFKLGRV SRU
2SULQFtSLRGDPRUDOLGDGH
PRUDOLGDGHDGPLQLVWUDWLYDWDPEpP
PRUDOLGDGH VHUYLGRUHV GH FDUUHLUD QRV FDVRV FRQGLo}HV H
FRQKHFLGR SRU SULQFtSLR GDSURELGDGH DGPLQLVWUDWLYD  SHUFHQWXDLVPtQLPRVSUHYLVWRVHPOHLGHVWLQDPVH
REULJDDDWXDomRGD$GPLQLVWUDomR3~EOLFDHPFRQVRQkQ DSHQDVjVDWULEXLo}HVGHGLUHomRFKHILDHDVVHVVR
FLDDVUHJUDVPRUDLVDVVLPHQWHQGLGRRFRQMXQWRGHUHJUDV UDPHQWR
 'LU&RQVWLWXFLRQDO
 &kPDUD/HJLVODWLYDGR'LVWULWR)HGHUDO
9, pJDUDQWLGRDRVHUYLGRUS~EOLFRFLYLORGLUHLWRjOL ;9, p YHGDGD D DFXPXODomR UHPXQHUDGD GHFDUJRV
YUHDVVRFLDomRVLQGLFDO S~EOLFRVH[FHWRTXDQGRKRXYHUFRPSDWLELOLGDGH
GHKRUiULRVREVHUYDGRHPTXDOTXHUFDVRRGLVSRV
9,, RGLUHLWRGHJUHYHVHUiH[HUFLGRQRVWHUPRVHQRV WRQRLQFLVR;,
OLPLWHVGHILQLGRVHPOHLHVSHFtILFD
D DGHGRLVFDUJRVGHSURIHVVRU
9,,, DOHLUHVHUYDUiSHUFHQWXDOGRVFDUJRVHHPSUHJRV
S~EOLFRVSDUDDVSHVVRDVSRUWDGRUDVGHGHILFLrQFLD E DGHXPFDUJRGHSURIHVVRUFRPRXWURWpFQLFR
HGHILQLUiRVFULWpULRVGHVXDDGPLVVmR RXFLHQWtILFR
,; D OHL HVWDEHOHFHUi RV FDVRV GH FRQWUDWDomRSRU F D GH GRLV FDUJRV RX HPSUHJRV SULYDWLYRV GH
WHPSR GHWHUPLQDGR SDUD DWHQGHU D QHFHVVLGDGH SURILVVLRQDLVGHVD~GHFRPSURILVV}HVUHJXOD
WHPSRUiULDGHH[FHSFLRQDOLQWHUHVVHS~EOLFR
PHQWDGDV
; DUHPXQHUDomRGRVVHUYLGRUHVS~EOLFRVHRVXEVt 2EV HVWDDOtQHD´FµIRLDOWHUDGDSHOD(PHQGD&RQVWLWXFLRQDOQƒ
GLRGHTXHWUDWDR†žGRDUWVRPHQWHSRGHUmR
2EV eSUHFLVRSRUpPHVWDUDWHQWRVyFDUJRVRXHPSUHJRVGHSURILVV}HV
VHUIL[DGRVRXDOWHUDGRVSRUOHLHVSHFtILFDREVHUYD UHJXODPHQWDGDV PpGLFRHQIHUPHLUDSDGUmRHWF pTXHSRGHPVHU
GDDLQLFLDWLYDSULYDWLYDHPFDGDFDVRDVVHJXUDGD FXPXODGDVHDWpROLPLWHPi[LPRGHGRLV WUrVHPSUHJRVRXFDUJRV
UHYLVmRJHUDODQXDOVHPSUHQDPHVPDGDWDHVHP QmRVHUiSHUPLWLGR 

GLVWLQomRGHtQGLFHV
;9,, DSURLELomRGHDFXPXODUHVWHQGHVHDHPSUHJRVH
;, D UHPXQHUDomR H R VXEVtGLR GRVRFXSDQWHVGH IXQo}HVHDEUDQJHDXWDUTXLDVIXQGDo}HVHPSUH
FDUJRVIXQo}HVHHPSUHJRVS~EOLFRVGDDGPLQLV VDVS~EOLFDVVRFLHGDGHVGHHFRQRPLDPLVWDVXDV
WUDomRGLUHWDDXWiUTXLFDHIXQGDFLRQDOGRVPHP VXEVLGLiULDVHVRFLHGDGHVFRQWURODGDVGLUHWDRX
EURV GH TXDOTXHU GRV 3RGHUHV GD 8QLmR GRV LQGLUHWDPHQWHSHORSRGHUS~EOLFR
(VWDGRVGR'LVWULWR)HGHUDOHGRV0XQLFtSLRVGRV
GHWHQWRUHV GH PDQGDWR HOHWLYR H GRV GHPDLV ;9,,, DDGPLQLVWUDomRID]HQGiULDHVHXVVHUYLoRVILVFDLV
DJHQWHVSROtWLFRVHRVSURYHQWRVSHQV}HVRXRXWUD WHUmR GHQWUR GH VXDV iUHDV GH FRPSHWrQFLD H
HVSpFLHUHPXQHUDWyULDSHUFHELGRVFXPXODWLYDPHQ MXULVGLomR SUHFHGrQFLD VREUH RV GHPDLV VHWRUHV
WHRXQmRLQFOXtGDVDVYDQWDJHQVSHVVRDLVRXGH DGPLQLVWUDWLYRVQDIRUPDGDOHL
TXDOTXHURXWUDQDWXUH]DQmRSRGHUmRH[FHGHUR
VXEVtGLR PHQVDO HP HVSpFLH GRV 0LQLVWURV GR ;,; VRPHQWHSRUOHLHVSHFtILFDSRGHUiVHUFULDGDDXWDU
6XSUHPR 7ULEXQDO )HGHUDO DSOLFDQGRVH FRPR TXLDHDXWRUL]DGDDLQVWLWXLomRGHHPSUHVDS~EOLFD
OLPLWH QRV 0XQLFtSLRV R VXEVtGLR GR 3UHIHLWR H GHVRFLHGDGHGHHFRQRPLDPLVWDHGHIXQGDomR
QRV (VWDGRV H QR 'LVWULWR )HGHUDO R VXEVtGLR FDEHQGR j OHL FRPSOHPHQWDU QHVWH ~OWLPR FDVR
PHQVDOGR*RYHUQDGRUQRkPELWRGR3RGHU([HFX GHILQLUDViUHDVGHVXDDWXDomR
WLYRRVXEVtGLRGRV'HSXWDGRV(VWDGXDLVH'LVWULWD
LVQRkPELWRGR3RGHU/HJLVODWLYRHRVXEVtGLRGRV ;; GHSHQGHGHDXWRUL]DomROHJLVODWLYDHPFDGDFDVR
'HVHPEDUJDGRUHVGR7ULEXQDOGH-XVWLoDOLPLWDGR DFULDomRGHVXEVLGLiULDVGDVHQWLGDGHVPHQFLRQD
DQRYHQWDLQWHLURVHYLQWHHFLQFRFHQWpVLPRVSRU GDVQRLQFLVRDQWHULRUDVVLPFRPRDSDUWLFLSDomR
FHQWRGRVXEVtGLRPHQVDOHPHVSpFLHGRV0LQLV GHTXDOTXHUGHODVHPHPSUHVDSULYDGD
WURVGR6XSUHPR7ULEXQDO)HGHUDOQRkPELWRGR
3RGHU -XGLFLiULR DSOLFiYHO HVWH OLPLWH DRV PHP ;;, UHVVDOYDGRVRVFDVRVHVSHFLILFDGRVQDOHJLVODomR
EURVGR0LQLVWpULR3~EOLFRDRV3URFXUDGRUHVHDRV DV REUDV VHUYLoRV FRPSUDV H DOLHQDo}HV VHUmR
'HIHQVRUHV3~EOLFRV2EVDDQWHULRUUHGDomRGDGDSHOD(&Qž FRQWUDWDGRVPHGLDQWHSURFHVVRGHOLFLWDomRS~EOL
5HIRUPD $GPLQLVWUDWLYD  PDLV XPD YH] IRL DOWHUDGD DJRUD SHOD (&Qƒ  GH
DFKDPDGD´5HIRUPD3UHYLGrQFLiULDµGR*RYHUQR/XOD37 FDTXHDVVHJXUHLJXDOGDGHGHFRQGLo}HVDWRGRV
;,, RVYHQFLPHQWRVGRVFDUJRVGR3RGHU/HJLVODWLYRH RVFRQFRUUHQWHVFRPFOiXVXODVTXHHVWDEHOHoDP
GR3RGHU-XGLFLiULRQmRSRGHUmRVHUVXSHULRUHVDRV REULJDo}HVGHSDJDPHQWRPDQWLGDVDVFRQGLo}HV
SDJRVSHOR3RGHU([HFXWLYR HIHWLYDV GD SURSRVWD QRV WHUPRV GD OHL R TXDO
VRPHQWHSHUPLWLUiDVH[LJrQFLDVGHTXDOLILFDomR
;,,, p YHGDGDDYLQFXODomRRXHTXLSDUDomRGHTXDLV WpFQLFDHHFRQ{PLFDLQGLVSHQViYHLVjJDUDQWLDGR
TXHU HVSpFLHV UHPXQHUDWyULDV SDUD R HIHLWR GH FXPSULPHQWRGDVREULJDo}HV
UHPXQHUDomRGHSHVVRDOGRVHUYLoRS~EOLFR
;;,, DVDGPLQLVWUDo}HVWULEXWiULDVGD8QLmRGRV(VWD
;,9 RVDFUpVFLPRVSHFXQLiULRVSHUFHELGRVSRUVHUYLGRU GRVGR'LVWULWR)HGHUDOHGRV0XQLFtSLRVDWLYLGD
S~EOLFRQmRVHUmRFRPSXWDGRVQHPDFXPXODGRV GHV HVVHQFLDLV DR IXQFLRQDPHQWR GR (VWDGR
SDUDILQVGHFRQFHVVmRGHDFUpVFLPRVXOWHULRUHV H[HUFLGDVSRUVHUYLGRUHVGHFDUUHLUDVHVSHFtILFDV
WHUmR UHFXUVRV SULRULWiULRV SDUD D UHDOL]DomR GH
;9 R VXEVtGLR H RV YHQFLPHQWRV GRV RFXSDQWHVGH VXDV DWLYLGDGHV H DWXDUmR GH IRUPD LQWHJUDGD
FDUJRVHHPSUHJRVS~EOLFRVVmRLUUHGXWtYHLVUHV LQFOXVLYHFRPRFRPSDUWLOKDPHQWRGHFDGDVWURVH
VDOYDGRRGLVSRVWRQRVLQFLVRV;,H;,9GHVWHDUWLJR GHLQIRUPDo}HVILVFDLVQDIRUPDGDOHLRXFRQYr
HQRVDUWV†ž,,,,,H†ž, QLR2EVHVWHLQFLVR;;,,IRLLQWURGX]LGRSHOD(PHQGD&RQVWLWXFLRQDOQƒ
GHDFKDPDGD5HIRUPD7ULEXWiULDGR*RYHUQR/XOD37
&kPDUD/HJLVODWLYDGR
  'LVWULWR)HGHUDO 'LU&RQVWLWXFLRQDO 

†ž $ SXEOLFLGDGH GRV DWRV SURJUDPDVREUDVVHU ,, RVFRQWUROHVHFULWpULRVGHDYDOLDomRGHGHVHPSH


YLoRVHFDPSDQKDVGRVyUJmRVS~EOLFRVGHYHUiWHU QKRGLUHLWRVREULJDo}HVHUHVSRQVDELOLGDGHGRV
FDUiWHUHGXFDWLYRLQIRUPDWLYRRXGHRULHQWDomR GLULJHQWHV
VRFLDOGHODQmRSRGHQGRFRQVWDUQRPHVVtPERORV
RXLPDJHQVTXHFDUDFWHUL]HPSURPRomRSHVVRDO ,,, DUHPXQHUDomRGRSHVVRDO
GHDXWRULGDGHVRXVHUYLGRUHVS~EOLFRV
†ž 2 GLVSRVWR QR LQFLVR ;, DSOLFDVHjVHPSUHVDV
†ž $QmRREVHUYkQFLDGRGLVSRVWRQRVLQFLVRV,,H,,, S~EOLFDV H jV VRFLHGDGHV GH HFRQRPLD PLVWD H
LPSOLFDUi D QXOLGDGH GR DWR H D SXQLomR GD VXDV VXEVLGLiULDV TXH UHFHEHUHP UHFXUVRV GD
DXWRULGDGHUHVSRQViYHOQRVWHUPRVGDOHL 8QLmR GRV (VWDGRV GR 'LVWULWR )HGHUDO RX GRV
0XQLFtSLRV SDUD SDJDPHQWR GH GHVSHVDV GH
†ž $ OHL GLVFLSOLQDUi DV IRUPDV GH SDUWLFLSDomRGR SHVVRDORXGHFXVWHLRHPJHUDO
XVXiULRQDDGPLQLVWUDomRS~EOLFDGLUHWDHLQGLUH
WDUHJXODQGRHVSHFLDOPHQWH † eYHGDGDDSHUFHSomRVLPXOWkQHDGHSURYHQWRVGH
DSRVHQWDGRULDGHFRUUHQWHVGRDUWRXGRVDUWV
, DVUHFODPDo}HVUHODWLYDVjSUHVWDomRGRVVHUYLoRV HFRPDUHPXQHUDomRGHFDUJRHPSUHJR
S~EOLFRVHPJHUDODVVHJXUDGDVDPDQXWHQomRGH RXIXQomRS~EOLFDUHVVDOYDGRVRVFDUJRVDFXPXOi
VHUYLoRVGHDWHQGLPHQWRDRXVXiULRHDDYDOLDomR YHLVQDIRUPDGHVWD&RQVWLWXLomRRVFDUJRVHOHWL
SHULyGLFD H[WHUQD H LQWHUQD GD TXDOLGDGH GRV YRVHRVFDUJRVHPFRPLVVmRGHFODUDGRVHPOHLGH
VHUYLoRV OLYUHQRPHDomRHH[RQHUDomR

† 1mR VHUmR FRPSXWDGDV SDUDHIHLWRGRVOLPLWHV


,, RDFHVVRGRVXVXiULRVDUHJLVWURVDGPLQLVWUDWLYRV
UHPXQHUDWyULRVGHTXHWUDWDRLQFLVR;,GRFDSXW
HDLQIRUPDo}HVVREUHDWRVGHJRYHUQRREVHUYDGR
GHVWHDUWLJRDVSDUFHODVGHFDUiWHULQGHQL]DWyULR
RGLVSRVWRQRDUWž;H;;;,,,
SUHYLVWDVHPOHL
2EV†DFUHVFLGRSHOD(&QžGH
,,, DGLVFLSOLQDGDUHSUHVHQWDomRFRQWUDRH[HUFtFLR
QHJOLJHQWH RX DEXVLYR GH FDUJR HPSUHJR RX † 3DUD RV ILQV GR GLVSRVWR QR LQFLVR;,GRFDSXW
IXQomRQDDGPLQLVWUDomRS~EOLFD GHVWH DUWLJR ILFD IDFXOWDGR DRV (VWDGRV H DR
'LVWULWR)HGHUDOIL[DUHPVHXkPELWRPHGLDQWH
†ž 2VDWRVGHLPSURELGDGHDGPLQLVWUDWLYDLPSRUWD HPHQGDjVUHVSHFWLYDV&RQVWLWXLo}HVH/HL2UJkQL
UmRDVXVSHQVmRGRVGLUHLWRVSROtWLFRVDSHUGDGD FD FRPR OLPLWH ~QLFR R VXEVtGLR PHQVDO GRV
IXQomRS~EOLFDDLQGLVSRQLELOLGDGHGRVEHQVHR 'HVHPEDUJDGRUHV GR UHVSHFWLYR 7ULEXQDO GH
-XVWLoDOLPLWDGRDQRYHQWDLQWHLURVHYLQWHHFLQFR
UHVVDUFLPHQWR DR HUiULR QD IRUPD H JUDGDomR
FHQWpVLPRV SRU FHQWR GR VXEVtGLR PHQVDO GRV
SUHYLVWDV HP OHL VHP SUHMXt]R GD DomR SHQDO
0LQLVWURV GR 6XSUHPR 7ULEXQDO )HGHUDO QmR VH
FDEtYHO
DSOLFDQGRRGLVSRVWRQHVWHSDUiJUDIRDRVVXEVtGLRV
GRV'HSXWDGRV(VWDGXDLVH'LVWULWDLVHGRV9HUHD
†ž $OHLHVWDEHOHFHUiRVSUD]RVGHSUHVFULomRSDUDLOt
GRUHV
FLWRVSUDWLFDGRVSRUTXDOTXHUDJHQWHVHUYLGRURX 2EV†DFUHVFLGRSHOD(&QžGH
QmRTXHFDXVHPSUHMXt]RVDRHUiULRUHVVDOYDGDV
DVUHVSHFWLYDVDo}HVGHUHVVDUFLPHQWR $UW$R VHUYLGRU S~EOLFR GD DGPLQLVWUDomRGLUHWD
DXWiUTXLFDHIXQGDFLRQDOQRH[HUFtFLRGHPDQGD
†ž $VSHVVRDVMXUtGLFDVGHGLUHLWRS~EOLFRHDVGHGL WRHOHWLYRDSOLFDPVHDVVHJXLQWHVGLVSRVLo}HV
UHLWRSULYDGRSUHVWDGRUDVGHVHUYLoRVS~EOLFRVUHV
SRQGHUmR SHORV GDQRV TXH VHXV DJHQWHV QHVVD , WUDWDQGRVHGHPDQGDWRHOHWLYRIHGHUDOHVWDGXDO
TXDOLGDGH FDXVDUHP D WHUFHLURV DVVHJXUDGR R RXGLVWULWDOILFDUiDIDVWDGRGHVHXFDUJRHPSUHJR
GLUHLWRGHUHJUHVVRFRQWUDRUHVSRQViYHOQRVFDVRV RXIXQomR
GHGRORRXFXOSD
,, LQYHVWLGRQRPDQGDWRGH3UHIHLWRVHUiDIDVWDGR
†ž $OHLGLVSRUiVREUHRVUHTXLVLWRVHDVUHVWULo}HVDR GRFDUJRHPSUHJRRXIXQomRVHQGROKHIDFXOWDGR
RFXSDQWHGHFDUJRRXHPSUHJRGDDGPLQLVWUDomR RSWDUSHODVXDUHPXQHUDomR
GLUHWDHLQGLUHWDTXHSRVVLELOLWHRDFHVVRDLQIRU
,,, LQYHVWLGR QR PDQGDWR GH 9HUHDGRUKDYHQGR
PDo}HVSULYLOHJLDGDV
FRPSDWLELOLGDGHGHKRUiULRVSHUFHEHUiDVYDQWD
JHQV GH VHX FDUJR HPSUHJR RX IXQomR VHP
†ž $DXWRQRPLDJHUHQFLDORUoDPHQWiULDHILQDQFHLUD
SUHMXt]RGDUHPXQHUDomRGRFDUJRHOHWLYRHQmR
GRVyUJmRVHHQWLGDGHVGDDGPLQLVWUDomRGLUHWDH
KDYHQGRFRPSDWLELOLGDGHVHUiDSOLFDGDDQRUPD
LQGLUHWDSRGHUiVHUDPSOLDGDPHGLDQWHFRQWUDWR
GRLQFLVRDQWHULRU
D VHU ILUPDGR HQWUH VHXV DGPLQLVWUDGRUHV H R
SRGHUS~EOLFRTXHWHQKDSRUREMHWRDIL[DomRGH
,9 HPTXDOTXHUFDVRTXHH[LMDRDIDVWDPHQWRSDUDR
PHWDVGHGHVHPSHQKRSDUDRyUJmRRXHQWLGDGH H[HUFtFLR GH PDQGDWR HOHWLYR VHX WHPSR GH
FDEHQGRjOHLGLVSRUVREUH VHUYLoRVHUiFRQWDGRSDUDWRGRVRVHIHLWRVOHJDLV
H[FHWRSDUDSURPRomRSRUPHUHFLPHQWR
, RSUD]RGHGXUDomRGRFRQWUDWR
 'LU&RQVWLWXFLRQDO
 &kPDUD/HJLVODWLYDGR'LVWULWR)HGHUDO
9 SDUDHIHLWRGHEHQHItFLRSUHYLGHQFLiULRQRFDVRGH ,,, DVSHFXOLDULGDGHVGRVFDUJRV
DIDVWDPHQWRRVYDORUHVVHUmRGHWHUPLQDGRVFRPR
VHQRH[HUFtFLRHVWLYHVVH †ž $8QLmRRV(VWDGRVHR'LVWULWR)HGHUDOPDQWHUmR
HVFRODVGHJRYHUQRSDUDDIRUPDomRHRDSHUIHLoR
DPHQWRGRVVHUYLGRUHVS~EOLFRVFRQVWLWXLQGRVHD
SDUWLFLSDomRQRVFXUVRVXPGRVUHTXLVLWRVSDUDD
6HUYLGRUHV3~EOLFRV SURPRomR QD FDUUHLUD IDFXOWDGD SDUD LVVR D
FHOHEUDomR GH FRQYrQLRV RX FRQWUDWRV HQWUH RV
HQWHVIHGHUDGRV
2 0HVWUH -26e $)2162 '$ 6,/9$ HQVLQD TXH RV
DJHQWHV DGPLQLVWUDWLYRV VmR RV VHUYLGRUHV S~EOLFRV †ž $SOLFDVH DRV VHUYLGRUHV RFXSDQWHVGHFDUJR
GHILQLQGRRV FRPR WRGRV DTXHOHV TXH PDQWrP FRPR R S~EOLFRRGLVSRVWRQRDUWž,99,,9,,,,;;,,
3RGHU3~EOLFRUHODomRGHWUDEDOKRQmRHYHQWXDOVREYtQFXOR ;,,, ;9 ;9, ;9,, ;9,,, ;,; ;; ;;,, H ;;;
GHGHSHQGrQFLDFDUDFWHUL]DQGRVHDVVLPSHODSURILVVLRQDOL SRGHQGRDOHLHVWDEHOHFHUUHTXLVLWRVGLIHUHQFLDGRV
GDGH H UHODomR GH VXERUGLQDomR KLHUiUTXLFD LQ
&XUVR GH GHDGPLVVmRTXDQGRDQDWXUH]DGRFDUJRRH[LJLU
'LUHLWR&RQVWLWXFLRQDO3RVLWLYR
HG6DUDLYDS 
2EV $VVLPRIXQFLRQiULRS~EOLFRWHUiGLUHLWRD ,9 VDOiULR
PtQLPR  9,, JDUDQWLDGHVDOiULRPtQLPR 9,,,
$UW$
$ 8QLmR RV (VWDGRV R 'LVWULWR )HGHUDO HRV
GpFLPRWHUFHLURVDOiULR ,; DGLFLRQDOQRWXUQR ;,,
0XQLFtSLRV
0XQLFtSLRVLQVWLWXLUmR
LQVWLWXLUmRQR
QRkPELWR
kPELWRGH
GHVXD
VXDFRPSH
VDOiULRIDPtOLDSDUDVHXVGHSHQGHQWHVH[FOXVLYDPHQ
WrQFLD
WrQFLDUHJLPH
UHJLPHMXUtGLFR
MXUtGLFR~QLFR
~QLFRHSODQRV
SODQRVGH
GHFDUUHLUD WH DR WUDEDOKDGRU ´GH EDL[D UHQGDµ ;,,,  MRUQDGD
SDUD
SDUD RV
RV VHUYLGRUHV
VHUYLGRUHV GD
GD DGPLQLVWUDomR
DGPLQLVWUDomR S~EOLFD GLiULDGHWUDEDOKRGHQRPi[LPRKRUDVHKRUDVVH
GLUHWD
GLUHWDGDV
GDVDXWDUTXLDV
DXWDUTXLDVHGDV
GDVIXQGDo}HV
IXQGDo}HVS~EOLFDV PDQDLV ;9 UHSRXVRVHPDQDOUHPXQHUDGRµ ;9,
KRUDVH[WUDV ;9,, IpULDV ;9,,, OLFHQoDjJHVWDQ
2EV
2EV D(PHQGD&RQVWLWXFLRQDOQž 5HIRUPD$GPLQLVWUDWLYD WH ;,;  OLFHQoDSDWHUQLGDGH  ;;  SURWHomR DR
GHXDVHJXLQWHQRYD
QRYDUHGDomRDHVWHDUWLJR
QRYD ³$UW$8QLmRRV WUDEDOKRGDPXOKHU ;;,, VHJXUDQoDGHWUDEDOKRH
(VWDGRVR'LVWULWR)HGHUDOHRV0XQLFtSLRVLQVWLWXLUmRFRQVHOKRGH ;;; LJXDOGDGHOHJDO
SROtWLFDGHDGPLQLVWUDomRHUHPXQHUDomRGHSHVVRDOLQWHJUDGRSRU
VHUYLGRUHVGHVLJQDGRVSHORVUHVSHFWLYRV3RGHUHV´
†ž 2 PHPEUR GH 3RGHU R GHWHQWRUGHPDQGDWR
PHGLGDOLPLQDU
&RPRYLPRVVXFHGHXHQWUHWDQWRTXHPHGLGD
PHGLGD HOHWLYR RV 0LQLVWURV GH (VWDGR H RV 6HFUHWiULRV
FRQFHGLGDQD$',1
$',1
$',1QžVXVSHQGHXDHILFiFLDGHVVD
Qž (VWDGXDLVH0XQLFLSDLVVHUmRUHPXQHUDGRVH[FOX
QRYDUHGDomRGRFDSXWGRDUW&)DWpMXOJDPHQWRILQDO VLYDPHQWHSRUVXEVtGLRIL[DGRHPSDUFHOD~QLFD
VRE R IXQGDPHQWR GH TXH R WH[WR REWHYH DSHQDV  YHGDGR R DFUpVFLPR GH TXDOTXHU JUDWLILFDomR
YRWRVTXDQGRVHULDPQHFHVViULRVYRWRVHPž7XUQR DGLFLRQDODERQRSUrPLRYHUEDGHUHSUHVHQWDomR
GHYRWDomRQD&kPDUDGRV'HSXWDGRV&RQVHTHQWHPHQ RX RXWUD HVSpFLH UHPXQHUDWyULD REHGHFLGR HP
WH SUHYDOHFH D LPSRVLomR GR ´UHJLPH MXUtGLFR ~QLFRµ TXDOTXHUFDVRRGLVSRVWRQRDUW;H;,
LQFRPSDWtYHOFRPDILJXUDGR´HPSUHJRS~EOLFRµ
†ž /HLGD8QLmRGRV(VWDGRVGR'LVWULWR)HGHUDOH
$SULPHLUDQRYLGDGHVHULDRILPGRUHJLPHMXUtGLFR~QLFR GRV0XQLFtSLRVSRGHUiHVWDEHOHFHUDUHODomRHQWUH
SDUDRVVHUYLGRUHVGDDGPLQLVWUDomRS~EOLFDGLUHWDGDV D PDLRUHDPHQRUUHPXQHUDomRGRVVHUYLGRUHV
DXWDUTXLDVHGDVIXQGDo}HVS~EOLFDV3RUHQTXDQWRFRP S~EOLFRVREHGHFLGRHPTXDOTXHUFDVRRGLVSRVWR
DVXVSHQVmRGHHILFiFLDHVWDEHOHFLGDQDOLPLQDUGD$',1Qž QRDUW;,
 SUHYDOHFH DLQGD D REULJDWRULHGDGH GR UHJLPH
SUHYDOHFH DLQGD
MXUtGLFR~QLFR
MXUtGLFR~QLFR †ž 2V 3RGHUHV ([HFXWLYR /HJLVODWLYRH-XGLFLiULR
SXEOLFDUmRDQXDOPHQWHRVYDORUHVGRVXEVtGLRHGD
$ GHVREULJDomR GH XP ~QLFR UHJLPH MXUtGLFR 5-8 UHPXQHUDomRGRVFDUJRVHHPSUHJRVS~EOLFRV
SHUPLWLULDj8QLmR(VWDGRVPHPEURV'LVWULWR)HGHUDOH
0XQLFtSLRVDGHILQLomRGHQRYDVSROtWLFDVSDUDDFRQWUDWD †ž /HLGD8QLmRGRV(VWDGRVGR'LVWULWR)HGHUDOH
omRGHVHUYLGRUHVLQFOXVLYHSHORUHJLPHFHOHWLVWD GD&/7  GRV0XQLFtSLRVGLVFLSOLQDUiDDSOLFDomRGHUHFXU
HTXLSDUDQGR R IXQFLRQiULR S~EOLFR DR GD DWLYLGDGH VRVRUoDPHQWiULRVSURYHQLHQWHVGDHFRQRPLDFRP
SULYDGDQRTXHFRQFHUQHDVHXVGLUHLWRVHGHYHUHV GHVSHVDV FRUUHQWHV HP FDGD yUJmR DXWDUTXLD H
IXQGDomRSDUDDSOLFDomRQRGHVHQYROYLPHQWRGH
†ž $IL[DomRGRVSDGU}HVGHYHQFLPHQWRHGRVGHPD SURJUDPDVGHTXDOLGDGHHSURGXWLYLGDGHWUHLQD
LVFRPSRQHQWHVGRVLVWHPDUHPXQHUDWyULRREVHU PHQWRHGHVHQYROYLPHQWRPRGHUQL]DomRUHDSD
YDUi UHOKDPHQWRHUDFLRQDOL]DomRGRVHUYLoRS~EOLFR
LQFOXVLYHVREDIRUPDGHDGLFLRQDORXSUrPLRGH
, DQDWXUH]DRJUDXGHUHVSRQVDELOLGDGHHDFRP SURGXWLYLGDGH
SOH[LGDGHGRVFDUJRVFRPSRQHQWHVGHFDGDFDUUHL
UD †ž $UHPXQHUDomRGRVVHUYLGRUHVS~EOLFRVRUJDQL]D
GRVHPFDUUHLUDSRGHUiVHUIL[DGDQRVWHUPRVGR
,, RVUHTXLVLWRVSDUDDLQYHVWLGXUD †ž
 &kPDUD/HJLVODWLYDGR
 'LVWULWR)HGHUDO 'LU&RQVWLWXFLRQDO 

$UW$RV
$RV
$RV VHUYLGRUHV
VHUYLGRUHV WLWXODUHV GH FDUJRV HIHWLYRVGD E VHVVHQWD H FLQFR DQRV GH LGDGH VH KRPHP H
8QLmR
8QLmR GRV
GRV (VWD
(VWDGRV
GRV GR
GR 'LVWULWR
'LVWULWR )HGHUDO
)HGHUDO H GRV VHVVHQWDDQRVGHLGDGHVHPXOKHUFRPSURYHQ
0XQLFtSLRV
0XQLFtSLRV LQFOXtGDV
LQFOXtGDV VXDV
VXDV DXWDUTXLDV
DXWDUTXLDV H IXQGD WRVSURSRUFLRQDLVDRWHPSRGHFRQWULEXLomR
o}HV
o}HV p DVVHJXUDGR
DVVHJXUDGR UHJLPH GH SUHYLGrQFLD GH
FDUiWHU FRQWULEXWLYRHVROLGiULR
FDUiWHUFRQWULEXWLYR VROLGiULRPHGLDQWH
PHGLDQWHFRQWUL †ž 2VSURYHQWRVGHDSRVHQWDGRULDHDVSHQV}HVSRU
EXLomR RFDVLmRGHVXDFRQFHVVmRQmRSRGHUmRH[FHGHUD
EXLomR GR
GR UHVSHFWLYR
UHVSHFWLYR HQWH
HQWH S~EOLFR
S~EOLFR GRV
GRV VHUYLGRUHV
VHUYLGRUHV
UHPXQHUDomR GR UHVSHFWLYR VHUYLGRU QR FDUJR
DWLYRV
DWLYRV H LQDWLYRV
LQDWLYRV H GRV
GRV SHQVLRQLVWDV
SHQVLRQLVWDV REVHUYDGRV
HIHWLYR HP TXH VH GHX D DSRVHQWDGRULD RX TXH
FULWpULRV
FULWpULRVTXH
TXHSUHVHUYHP
SUHVHUYHPRHTXLOtEULR
HTXLOtEULRILQDQFHLUR
ILQDQFHLURH VHUYLXGHUHIHUrQFLDSDUDDFRQFHVVmRGDSHQVmR
DWXDULDO
DWXDULDOHRGLVSRVWR
GLVSRVWRQHVWH
QHVWHDUWLJR
DUWLJR2EVDUHGDomRGHVWHLQFLVR
IRL DOWHUDGD SULPHLUDPHQWH SHOD (& Qž  5HIRUPD GD3UHYLGrQFLD  H DJRUD SHOD
(PHQGD &RQVWLWXFLRQDO Qƒ GH  FKDPDGD GH 5HIRUPD GD 3UHYLGrQFLD GR †ž 3DUDRFiOFXORGRVSURYHQWRVGHDSRVHQWDGRULDSRU
*RYHUQR/XOD
RFDVLmR GD VXD FRQFHVVmR VHUmR FRQVLGHUDGDV DV
$5HIRUPDGD3UHYLGrQFLDLQVWLWXLXRUHJLPHGHSUHYLGrQ UHPXQHUDo}HVXWLOL]DGDVFRPREDVHSDUDDVFRQWULEX
FLD GH FDUiWHU FRQWULEXWLYR SDUD RV VHUYLGRUHV S~EOLFRV Lo}HVGRVHUYLGRUDRVUHJLPHVGHSUHYLGrQFLDGHTXH
WLWXODUHV GH FDUJRV HIHWLYRV GD 8QLmR GRV (VWDGRV GR WUDWDPHVWHDUWLJRHRDUWQDIRUPDGDOHL2EV
DUHGDomRGHVWH†žIRLDOWHUDGDSHOD(PHQGD&RQVWLWXFLRQDOQžD5HIRUPD
'LVWULWR )HGHUDO H GRV 0XQLFtSLRV EHP DVVLP SDUD RV GD3UHYLGrQFLDGR*RYHUQR/XOD37
IXQFLRQiULRVGDVDXWDUTXLDVHIXQGDo}HV IHGHUDLVHVWDGXD
†ž eYHGDGDDDGRomRGHUHTXLVLWRVHFULWpULRVGLIHUHQ
LV GLVWULWDLV RX PXQLFLSDLV  ³ YDOH GL]HU XP VLVWHPD
FLDGRV SDUD D FRQFHVVmR GH DSRVHQWDGRULD DRV
SUHYLGrQFLD EDVHDGR QD FRQWULEXLomR
FRQWULEXLomR SDJDPHQWR  GRV
DEUDQJLGRV SHOR UHJLPH GH TXH WUDWD HVWH DUWLJR
SUySULRV VHUYLGRUHV Vy WHUi GLUHLWR TXHP SDJRX H QD UHVVDOYDGRVQRVWHUPRVGHILQLGRVHPOHLVFRPSOH
SURSRUomRGRTXHSDJRX PHQWDUHVRVFDVRVGHVHUYLGRUHV
2VLVWHPDSUHYLGHQFLiULREDVHDGRQDFRQWULEXLomR³GDt , SRUWDGRUHVGHGHILFLrQFLD
VHX´FDUiWHUFRQWULEXWLYRµ³YLQFXODYDDDSRVHQWDGRULDH
RXWURVEHQHItFLRVDRWHPSRGHFRQWULEXLomRHQmRDR´WHPSR ,, TXHH[HUoDPDWLYLGDGHVGHULVFR
GH VHUYLoRµ $VVLP SH[ R SHUtRGR GH DIDVWDPHQWR GR
VHUYLGRUSH[SDUDIUHTHQWDUFXUVRRXDSHUIHLoRDPHQWR ,,, FXMDV DWLYLGDGHV VHMDP H[HUFLGDV VREFRQGLo}HV
RXGHHVWXGRVQRH[WHULRURXHPRUJDQLVPRLQWHUQDFLRQDO HVSHFLDLVTXHSUHMXGLTXHPDVD~GHRXDLQWHJULGD
RXSRUIRUoDGHPDQGDWRFODVVLVWDVHP{QXVSDUDRVFRIUHV GHItVLFD 2EVUHGDomRGDGDSHOD(&Qž
S~EOLFRV VH QmR UHIOHWLU SDJDPHQWR GD FRQWULEXLomR
†ž 2VUHTXLVLWRVGHLGDGHHGHWHPSRGHFRQWULEXLomR
SUHYLGHQFLiULD LQWHUURPSHQGRVH VHX UHFROKLPHQWR QmR
VHUmR UHGX]LGRV HP FLQFR DQRV HP UHODomR DR
PDLVSRGHUiVHUFRPSXWDGRQHPSDUDHIHLWRGHDSRVHQWD
GLVSRVWRQR†ƒ,,,DSDUDRSURIHVVRUTXHFRP
GRULD QHP SDUD RXWURV EHQHItFLRV TXH H[LMDP WHPSR
SURYHH[FOXVLYDPHQWHWHPSRGHHIHWLYRH[HUFtFLR
PtQLPR GDVIXQo}HVGHPDJLVWpULRQDHGXFDomRLQIDQWLOH
QRHQVLQRIXQGDPHQWDOHPpGLR
†ž 2VVHUYLGRUHVDEUDQJLGRVSHORUHJLPHGHSUHYLGrQ
FLD GH TXH WUDWD HVWH DUWLJR VHUmR DSRVHQWDGRV †ž 5HVVDOYDGDV DV DSRVHQWDGRULDVGHFRUUHQWHVGRV
FDOFXODGRVRVVHXVSURYHQWRVDSDUWLUGRVYDORUHV FDUJRV DFXPXOiYHLV QD IRUPD GD &RQVWLWXLomR p
IL[DGRVQDIRUPDGRV††žH YHGDGDDSHUFHSomRGHPDLVGHXPDDSRVHQWDGRULD
2EV HVWH†IRLLQWURGX]LGRSHOD(PHQGD&RQVWLWXFLRQDOQžHIRLDJRUDDOWHUDGRSHOD j FRQWD GR UHJLPH GH SUHYLGrQFLD SUHYLVWR QHVWH
5HIRUPD GD 3UHYLGrQFLD GR *RYHUQR /XOD37 (& Qƒ  ³ H YLQFXOD D DUWLJR
DSRVHQWDGRULDjVSUHVWDo}HVFRQWULEXWLYDVHIHWLYDPHQWHSDJDVLQFOXVLYHVRERUHJLPH
SULYDGR †ƒ ³GHYLGDPHQWHDWXDOL]DGRV † 
†ž /HLGLVSRUiVREUHDFRQFHVVmRGREHQHItFLRGHSHQ
, SRU LQYDOLGH]SHUPDQHQWHVHQGRRVSURYHQWRV VmRSRUPRUWHTXHVHUiLJXDO 2EVDUHGDomRGHVWH†žIRL
SURSRUFLRQDLVDRWHPSRGHFRQWULEXLomRH[FHWRVH DOWHUDGDSHOD(PHQGD&RQVWLWXFLRQDOQž³D5HIRUPDGD3UHYLGrQFLDGR
*RYHUQR/XOD37
GHFRUUHQWHGHDFLGHQWHHPVHUYLoRPROpVWLDSURILV
VLRQDORXGRHQoDJUDYHFRQWDJLRVDRXLQFXUiYHO , DRYDORUGDWRWDOLGDGHGRVSURYHQWRVGRVHUYLGRU
QDIRUPDGDOHL2EVDUHGDomRGHVWHLQFLVRIRLDOWHUDGDSHOD(&QžH IDOHFLGRDWpROLPLWHPi[LPRHVWDEHOHFLGRSDUDRV
pVXWLOPHQWHDOWHUDGDDJRUDSHOD(&1ƒD5HIRUPDGD3UHYLGrQFLDGR*RYHUQR
/XOD37
EHQHItFLRVGRUHJLPHJHUDOGHSUHYLGrQFLDVRFLDOGH
TXHWUDWDRDUWDFUHVFLGRGHVHWHQWDSRUFHQWR
,, FRPSXOVRULDPHQWH FRPSURYHQWRVSURSRUFLRQDLV GDSDUFHODH[FHGHQWHDHVWHOLPLWHFDVRDSRVHQWDGR
DRWHPSRGHFRQWULEXLomRDRV VHWHQWD DQRVGH jGDWDGRyELWRRX
LGDGHRXDRV VHWHQWDHFLQFR DQRVGHLGDGH
QDIRUPDGHOHLFRPSOHPHQWDU2EVDUHGDomRGHVWHLQFLVRIRL ,, DRYDORUGDWRWDOLGDGHGDUHPXQHUDomRGRVHUYLGRU
DOWHUDGDSHOD(PHQGD&RQVWLWXFLRQDOQž 3(&GD´EHQJDODµ 
QRFDUJRHIHWLYRHPTXHVHGHXRIDOHFLPHQWRDWp
ROLPLWHPi[LPRHVWDEHOHFLGRSDUDRVEHQHItFLRVGR
,,, YROXQWDULDPHQWH GHVGH TXH FXPSULGRWHPSR UHJLPHJHUDOGHSUHYLGrQFLDVRFLDOGHTXHWUDWDR
PtQLPRGHGH]DQRVGHHIHWLYRH[HUFtFLRQRVHUYLoR DUWDFUHVFLGRGHVHWHQWDSRUFHQWRGDSDUFHOD
S~EOLFRHFLQFRDQRVQRFDUJRHIHWLYR HP TXHVH H[FHGHQWHDHVWHOLPLWHFDVRHPDWLYLGDGHQDGDWD
GDUi D DSRVHQWDGRULD REVHUYDGDV DV VHJXLQWHV GRyELWR
FRQGLo}HV
†ž eDVVHJXUDGRRUHDMXVWDPHQWRGRVEHQHItFLRVSDUD
D VHVVHQWDDQRVGHLGDGHHWULQWDHFLQFRGHFRQ SUHVHUYDUOKHV HP FDUiWHU SHUPDQHQWH R YDORU
WULEXLomRVHKRPHPHFLQTHQWDHFLQFRDQRV UHDOFRQIRUPHFULWpULRVHVWDEHOHFLGRVHPOHL2EV
GHLGDGHHWULQWDGHFRQWULEXLomRVHPXOKHU UHGDomRGHVWH†žPRGLILFDGDSHOD(&Qƒ
 'LU&RQVWLWXFLRQDO
 &kPDUD/HJLVODWLYDGR'LVWULWR)HGHUDO
†ž 2 WHPSR GH FRQWULEXLomR IHGHUDOHVWDGXDORX † ,QFLGLUiFRQWULEXLomRVREUHRVSURYHQWRVGHDSR
PXQLFLSDOVHUiFRQWDGRSDUDHIHLWRGHDSRVHQWD VHQWDGRULDVHSHQV}HVFRQFHGLGDVSHORUHJLPHGH
GRULDHRWHPSRGHVHUYLoRFRUUHVSRQGHQWHSDUD TXHWUDWDHVWHDUWLJRTXHVXSHUHPROLPLWHPi[L
HIHLWRGHGLVSRQLELOLGDGH PR HVWDEHOHFLGR SDUD RV EHQHItFLRV GR UHJLPH
JHUDOGHSUHYLGrQFLDVRFLDOGHTXHWUDWDRDUW
† $OHLQmRSRGHUiHVWDEHOHFHUTXDOTXHUIRUPDGH FRP SHUFHQWXDO LJXDO DR HVWDEHOHFLGR SDUD RV
FRQWDJHPGHWHPSRGHFRQWULEXLomRILFWtFLR VHUYLGRUHVWLWXODUHVGHFDUJRVHIHWLYRV2EVUHGDomRGDGD
SHOD(PHQGD&RQVWLWXFLRQDOQž

† $SOLFDVH R OLPLWH IL[DGR QR DUW  ;,jVRPD † 2VHUYLGRUGHTXHWUDWDHVWHDUWLJRTXHWHQKDFRP
WRWDO GRV SURYHQWRV GH LQDWLYLGDGH LQFOXVLYH SOHWDGRDVH[LJrQFLDVSDUDDSRVHQWDGRULDYROXQWi
TXDQGRGHFRUUHQWHVGDDFXPXODomRGHFDUJRVRX ULD HVWDEHOHFLGDV QR † ž ,,, D H TXH RSWH SRU
HPSUHJRVS~EOLFRVEHPFRPRGHRXWUDVDWLYLGD SHUPDQHFHUHPDWLYLGDGHIDUiMXVDXPDERQRGH
GHVVXMHLWDVDFRQWULEXLomRSDUDRUHJLPHJHUDOGH SHUPDQrQFLDHTXLYDOHQWHDRYDORUGDVXDFRQWUL
SUHYLGrQFLDVRFLDOHDRPRQWDQWHUHVXOWDQWHGD EXLomRSUHYLGHQFLiULDDWpFRPSOHWDUDVH[LJrQFLDV
DGLomRGHSURYHQWRVGHLQDWLYLGDGHFRPUHPXQH SDUDDSRVHQWDGRULDFRPSXOVyULDFRQWLGDVQR†ž
UDomRGHFDUJRDFXPXOiYHOQDIRUPDGHVWD&RQVWL ,,2EVUHGDomRGDGDSHOD(PHQGD&RQVWLWXFLRQDOQž
WXLomR FDUJR HP FRPLVVmR GHFODUDGR HP OHL GH
OLYUHQRPHDomRHH[RQHUDomRHGHFDUJRHOHWLYR † )LFDYHGDGDDH[LVWrQFLDGHPDLVGHXPUHJLPH
SUySULRGHSUHYLGrQFLDVRFLDOSDUDRVVHUYLGRUHV
† $OpPGRGLVSRVWRQHVWHDUWLJRRUHJLPHGHSUHYL WLWXODUHV GH FDUJRV HIHWLYRV H GH PDLV GH XPD
GrQFLDGRVVHUYLGRUHVS~EOLFRVWLWXODUHVGHFDUJR XQLGDGH JHVWRUD GR UHVSHFWLYR UHJLPH HP FDGD
HIHWLYRREVHUYDUiQRTXHFRXEHURVUHTXLVLWRVH HQWHHVWDWDOUHVVDOYDGRRGLVSRVWRQRDUW†
ž;2EV†IRLDFUHVFLGRSHOD(&Qƒ
FULWpULRVIL[DGRVSDUDRUHJLPHJHUDOGHSUHYLGrQ
FLDVRFLDO † $ FRQWULEXLomR SUHYLVWD QR †GHVWHDUWLJR
LQFLGLUiDSHQDVVREUHDVSDUFHODVGHSURYHQWRVGH
† 2VHUYLGRURFXSDQWHH[FOXVLYDPHQWHGHFDUJRHP DSRVHQWDGRULDHGHSHQVmRTXHVXSHUHPRGREUR
FRPLVVmRGHFODUDGRHPOHLGHOLYUHQRPHDomRH GROLPLWHPi[LPRHVWDEHOHFLGRSDUDRVEHQHItFLRV
H[RQHUDomREHPFRPRGHRXWURFDUJRWHPSRUiULR GRUHJLPHJHUDOGHSUHYLGrQFLDVRFLDOGHTXHWUDWD
RXGHHPSUHJRS~EOLFRDSOLFDVHRUHJLPHJHUDO RDUWGHVWD&RQVWLWXLomRTXDQGRREHQHILFLi
GHSUHYLGrQFLDVRFLDO ULR QD IRUPD GD OHL IRU SRUWDGRU GH GRHQoD
LQFDSDFLWDQWH2EV†DFUHVFLGRSHOD(&QƒGH
† $8QLmRRV(VWDGRVR'LVWULWR)HGHUDOHRV0XQL
FtSLRVGHVGHTXHLQVWLWXDPUHJLPHGHSUHYLGrQFLD $UW 6mR
6mRHVWiYHLV
HVWiYHLVDSyV
DSyVWUrVDQRVGHHIHWLYRH[HUFtFLR
FRPSOHPHQWDU SDUD VHXV UHVSHFWLYRV VHUYLGRUHV RV
RVVHUYLGRUHV
VHUYLGRUHVQRPHDGRVSDUDFDUJRGHSURYLPHQ
QRPHDGRVSDUDFDUJRGHSURYLPHQ
WLWXODUHVGHFDUJRHIHWLYRSRGHUmRIL[DUSDUDR WRHIHWLYRHPYLUWXGHGHFRQFXUVRS~EOLFR
YDORU GDV DSRVHQWDGRULDV H SHQV}HV D VHUHP
†ž 2VHUYLGRUS~EOLFRHVWiYHOVySHUGHUiRFDUJR
FRQFHGLGDVSHORUHJLPHGHTXHWUDWDHVWHDUWLJR
ROLPLWHPi[LPRHVWDEHOHFLGRSDUDRVEHQHItFLRV
, HP YLUWXGH GH VHQWHQoDMXGLFLDOWUDQVLWDGDHP
GRUHJLPHJHUDOGHSUHYLGrQFLDVRFLDOGHTXHWUDWD MXOJDGR
RDUW ,, PHGLDQWHSURFHVVRDGPLQLVWUDWLYRHPTXHOKHVHMD
DVVHJXUDGDDPSODGHIHVD
† 2 UHJLPH GH SUHYLGrQFLD FRPSOHPHQWDUGHTXH ,,, PHGLDQWHSURFHGLPHQWRGHDYDOLDomRSHULyGLFDGH
WUDWDR†VHUiLQVWLWXtGRSRUOHLGHLQLFLDWLYDGR GHVHPSHQKR QD IRUPD GH OHL FRPSOHPHQWDU
UHVSHFWLYR3RGHU([HFXWLYRREVHUYDGRRGLVSRVWR DVVHJXUDGDDPSODGHIHVD
QRDUWHVHXVSDUiJUDIRVQRTXHFRXEHUSRU
LQWHUPpGLRGHHQWLGDGHVIHFKDGDVGHSUHYLGrQFLD †ž ,QYDOLGDGD SRU VHQWHQoD MXGLFLDO DGHPLVVmRGR
FRPSOHPHQWDUGHQDWXUH]DS~EOLFDTXHRIHUHFH VHUYLGRUHVWiYHOVHUiHOHUHLQWHJUDGRHRHYHQWXDO
UmRDRVUHVSHFWLYRVSDUWLFLSDQWHVSODQRVGHEHQHIt RFXSDQWH GD YDJD VH HVWiYHO UHFRQGX]LGR DR
FLRV VRPHQWH QD PRGDOLGDGH GH FRQWULEXLomR FDUJR GH RULJHP VHP GLUHLWR D LQGHQL]DomR
GHILQLGD 2EVUHGDomRGDGDSHOD(PHQGD&RQVWLWXFLRQDOQž DSURYHLWDGRHPRXWURFDUJRRXSRVWRHPGLVSRQL
ELOLGDGHFRPUHPXQHUDomRSURSRUFLRQDODRWHPSR
† 6RPHQWHPHGLDQWHVXDSUpYLDHH[SUHVVDRSomRR GHVHUYLoR
GLVSRVWRQRV††HSRGHUiVHUDSOLFDGRDR
VHUYLGRUTXHWLYHULQJUHVVDGRQRVHUYLoRS~EOLFR †ž ([WLQWRRFDUJRRXGHFODUDGDDVXDGHVQHFHVVLGD
DWpDGDWDGDSXEOLFDomRGRDWRGHLQVWLWXLomRGR GHRVHUYLGRUHVWiYHOILFDUiHPGLVSRQLELOLGDGH
FRUUHVSRQGHQWH UHJLPH GH SUHYLGrQFLD FRPSOH FRP UHPXQHUDomR SURSRUFLRQDO DR WHPSR GH
VHUYLoR DWp VHX DGHTXDGR DSURYHLWDPHQWR HP
PHQWDU
RXWURFDUJR
† 7RGRV RV YDORUHV GHUHPXQHUDomRFRQVLGHUDGRV
†ž &RPRFRQGLomRSDUDDDTXLVLomRGDHVWDELOLGDGH
SDUDRFiOFXORGREHQHItFLRSUHYLVWRQR†ƒVHUmR pREULJDWyULDDDYDOLDomRHVSHFLDOGHGHVHPSHQKR
GHYLGDPHQWH DWXDOL]DGRV QD IRUPD GD OHL 2EV SRUFRPLVVmRLQVWLWXtGDSDUDHVVDILQDOLGDGH
UHGDomRGDGDSHOD(PHQGD&RQVWLWXFLRQDOQž
&kPDUD/HJLVODWLYDGR
 'LVWULWR)HGHUDO 'LU&RQVWLWXFLRQDO 

GRWH[WRFRQVWLWXFLRQDODGLVFLSOLQDGDSHQVmRGRVPLOLWDUHV
0LOLWDUHVGRV(VWDGRV HVWDGXDLVGRVPLOLWDUHVGLVWULWDLVHWHUULWRULDLV³HQWUH
GR'LVWULWR)HGHUDOHGRV7HUULWyULRV JDQGR SDUD D OHJLVODomR GRV UHVSHFWLYRV (VWDGRV VHX
GLVFLSOLQDPHQWR
$UW2V
2V
2V PHPEURV
PHPEURV GDV
GDV 3ROtFLDV
3ROtFLDV 0LOLWDUHVH&RUSRVGH
0LOLWDUHVH&RUSRVGH
%RPEHLURV
%RPEHLURV 0LOLWDUHV
0LOLWDUHV LQVWLWXLo}HV
LQVWLWXLo}HV RUJDQL]DGDV
FRP
'DV5HJL}HV
FRPEDVH
EDVHQD
QDKLHUDUTXLD
KLHUDUTXLDHGLVFLSOLQDVmRPLOLWD
GLVFLSOLQDVmRPLOLWD
UHV GRV
UHV GRV (VWDGRV
(VWDGRV GR
GR 'LVWULWR
'LVWULWR )HGHUDOHGRV7HU
)HGHUDOHGRV7HU
ULWyULRV 2FRQFHLWRGH´UHJLmRµYLQJRXQRGLUHLWRSROtWLFRWDQWR
2EV 2 FDSXW GHVWH DUWLJR WHP VXDUHGDomRGDGDSHOD(PHQGD
&RQVWLWXFLRQDOQž QRV HVWDGRV GH UHJLPH IHGHUDWLYR DQWLJD 8566 SH[ 
'HILQH HVWH DUWLJR D QDWXUH]D GR UHJLPH MXUtGLFR GRV FRPRQRVXQLWiULRV )UDQoD3RUWXJDO(VSDQKD,WiOLDHWF
3ROLFLDLV0LOLWDUHVHGRV&RUSRVGH%RPEHLURV0LOLWDUHV ³HWHPSRUSURSyVLWRLGHQWLILFDUHUHVSHLWDUDKRPRJHQHL
WRGRVVmRFRQVLGHUDGRVPLOLWDUHVGHVHXV(VWDGRV RXGR GDGHItVLFDHFRQ{PLFDHFXOWXUDOTXHLGHQWLILFDGHWHUPL
') ³QmRVHFRQIXQGLQGRSRUWDQWRFRP RVPLOLWDUHV QDGDViUHDVJHRJUiILFDV$&RQVWLWXLomR)HGHUDOFRQIHUH
IHGHUDLV TXHVmRRVGDVWUrVDUPDV([pUFLWR0DULQKDH YDOLGDGHDRUHFRQKHFLPHQWRGDVUHJL}HVSDUDDWHQGHUDR
$HURQiXWLFD  SURSyVLWR GH VHX GHVHQYROYLPHQWR H VREUHWXGR SDUD
FRPSHQVDUDVGHVLJXDOGDGHVGHVRUWHTXHDVUHJL}HVPDLV
(SRUVHUHPPLOLWDUHVGRV(VWDGRV RXGR') QmRVH SREUHVVHUmRVRFRUULGDVSDUDTXHVHUHGX]DPVXDLQIHULRUL
OKHVDSOLFDRUHJLPHMXUtGLFRGRHVWDWXWRGRVVHUYLGRUHV GDGH RXSRSXODFLRQDORXHFRQ{PLFDFRPHUFLDOLQGXVWUL
S~EOLFRVFLYLVHVXEPHWHUVHmRDRUHJLPHPLOLWDUTXHp DORXDJUtFRODRXFXOWXUDO HPUHODomRjVGHPDLVUHJL}HV
EDVHDGRQDKLHUDUTXLDHQDGLVFLSOLQDVXMHLWRVDREULJD
o}HVSUySULDVGRVPLOLWDUHV $UW 3DUD
3DUD HIHLWRV
HIHLWRV DGPLQLVWUDWLYRV D 8QLmRSRGHUi
DUWLFXODU
DUWLFXODUVXD
VXDDomR
DomRHP
HPXP
XPPHVPR
PHVPRFRPSOH[R
FRPSOH[RJHR
†ž $SOLFDPVH
$SOLFDPVHDRV
DRVPLOLWDUHV
PLOLWDUHVGRV
GRV(VWDGRVGR
(VWDGRVGR'LVWULWR HFRQ{PLFR
HFRQ{PLFR H VRFLDO YLVDQGR D VHX GHVHQYROYL
)HGHUDO
)HGHUDOHGRV
GRV7HUULWyULRV
7HUULWyULRVDOpP
DOpPGR
GRTXH
TXHYLHU
YLHUDVHU PHQWRHjUHGXomRGDVGHVLJXDGDGHVUHJLRQDLV
PHQWRHjUHGXomRGDVGHVLJXDGDGHVUHJLRQDLV
IL[DGRHPOHLDVGLVSRVLo}HVGRDUWLJR
IL[DGRHPOHLDVGLVSRVLo}HVGRD UWLJR
†ž
GR DUWLJR
DUWLJR 
 † ž
ž H GR
GR DUWLJR  †† ž H ž
DUWLJR  ž †ž /HLFRPSOHPHQWDUGLVSRUiVREUH
FDEHQGR
FDEHQGRjOHL
OHLHVWDGXDO
HVWDGXDOHVSHFtILFD
HVSHFtILFDGLVSRU
GLVSRUVREUH
VREUHDV
PDWpULDV
PDWpULDV GR
GR DUWLJR
DUWLJR 
 ž
ž LQFLVR
LQFLVR ;
; VHQGR RV , DV FRQGLo}HV SDUD LQWHJUDomRGHUHJL}HVHP
SDWHQWHV
SDWHQWHVGRV
GRVRILFLDLVFRQIHULGDVSHORVUHVSHFWLYRV
RILFLDLVFRQIHULGDVSHORVUHVSHFWLYRV GHVHQYROYLPHQWR
*RYHUQDGRUHV
,, D FRPSRVLomR GRV RUJDQLVPRVUHJLRQDLVTXH
3DWHQWHVVmRRVFDUJRVRVSRVWRVQRRILFLDODWRSULQFLSLD H[HFXWDUmRQDIRUPDGDOHLRVSODQRVQDFLRQDLV
FRPžWHQHQWHHSRGHDOFDQoDUJHQHUDODWRHDWpPDUHFKDO GH GHVHQYROYLPHQWR HFRQ{PLFR H VRFLDO DSUR
QRFDVRGR([pUFLWR ³PDVQDV3ROtFLDLV0LOLWDUHV HQR YDGRVMXQWDPHQWHFRPHVWHV
&RURQHO
&RUSRVGH%RPEHLURV0LOLWDUHVILQGDHP&RURQHO
&RURQHO
†ž 2VLQFHQWLYRVUHJLRQDLVFRPSUHHQGHUmRDOpPGH
2EV (VWH†žWHPVXDUHGDomRGDGDSHOD(PHQGD&RQVWLWXFLRQDOQž RXWURVQDIRUPDGDOHL

2EV 2 DUW† ž&) VXSUD FLWDGR GLVS}H † ž  2 , LJXDOGDGHGHWDULIDVIUHWHVVHJXURVHRXWURVLWHQV


PLOLWDUDOLVWiYHOpHOHJtYHODWHQGLGDVDVVHJXLQWHV GHFXVWRVHSUHoRVGHUHVSRQVDELOLGDGHGR3RGHU
FRQGLo}HV ,  VH FRQWDU PHQRV GH GH] DQRV GH 3~EOLFR
VHUYLoR GHYHUi DIDVWDUVH GD DWLYLGDGH ,,  VH
FRQWDUPDLVGHGH]DQRVGHVHUYLoRVHUiDJUHJDGR ,, MXURVIDYRUHFLGRVSDUDILQDQFLDPHQWRGHDWLYLGD
SHOD DXWRULGDGH VXSHULRU H VH HOHLWR SDVVDUi GHVSULRULWiULDV
DXWRPDWLFDPHQWHQRDWRGDGLSORPDomRSDUDD
LQDWLYLGDGH ,,, LVHQo}HVUHGXo}HVRXGLIHULPHQWRWHPSRUiULRGH
WULEXWRV IHGHUDLV GHYLGRV SRU SHVVRDV ItVLFDV RX
2EV RDUW†ž&)GLVS}H†ž2WHPSRGHFRQWUL MXUtGLFDV
EXLomRIHGHUDOHVWDGXDORXPXQLFLSDOVHUiFRQWD
GR SDUD HIHLWR GH DSRVHQWDGRULD H R WHPSR GH ,9 SULRULGDGH SDUD R DSURYHLWDPHQWR HFRQ{PLFRH
VHUYLoRFRUUHVSRQGHQWHSDUDHIHLWRGHGLVSRQLELOL VRFLDOGRVULRVHGDVPDVVDVGHiJXDUHSUHVDGDVRX
GDGH UHSUHViYHLVQDVUHJL}HVGHEDL[DUHQGDVXMHLWDVD
VHFDVSHULyGLFDV
†ž $RV
$RV SHQVLRQLVWDV
SHQVLRQLVWDV GRV PLOLWDUHVGRV(VWDGRVGR
'LVWULWR
'LVWULWR)HGHUDO
)HGHUDOHGRV
GRV7HUULWyULRV
7HUULWyULRVDSOLFDVH
DSOLFDVHRTXH †ž 1DViUHDVDTXHVHUHIHUHR†ž,9D8QLmRLQ
IRU
IRU IL[DGR
IL[DGR HP
HP OHL
OHL HVSHF
HVSHFtILFD
tILFD GR
GR UHVSHFWLYR
UHVSHFWLYR HQWH FHQWLYDUiDUHFXSHUDomRGHWHUUDViULGDVHFRRSH
HVWDWDO UDUi FRP RV SHTXHQRV H PpGLRV SURSULHWiULRV
2EVUHGDomRDOWHUDGDSHOD(&Qƒ UXUDLVSDUDRHVWDEHOHFLPHQWRHPVXDVJOHEDVGH
IRQWHVGHiJXDHGHSHTXHQDLUULJDomR
$5HIRUPDGD3UHYLGrQFLDGR*RYHUQR/XOD37H[FOXLX
 'LU&RQVWLWXFLRQDO &kPDUD/HJLVODWLYDGR'LVWULWR)HGHUDO


FRQVXEVWDQFLDPDVUHJUDVGHIXQFLRQDPHQWRVXDRUJDQL]D
2UJDQL]DomRGRV3RGHUHV omR LQ
&XUVR GH 'LUHLWR &RQVWLWXFLRQDO 3RVLWLYR
  HG6DUDLYD
S 
-iDK
KDUPRQLD
DUPRQLD HQWUHRVSRGHUHV
HQWUHRVSRGHUHVFDUDFWHUL]DVHSHOD
WUHRVSRGHUHV
$&RQVWLWXLomRGD5HS~EOLFDMiQRVSULQFtSLRVIXQGD FRQYLYrQFLDUHVSHLWRVDHPTXHP~WXDHUHFLSURFDPHQWH
PHQWDLVHVWDEHOHFH FDGDGRVSRGHUHVREVHUYDDVSUHUURJDWLYDVHIDFXOGDGHV
FRQIHULGDV SHOD RUGHP MXUtGLFD D VL FRPR DRV GHPDLV
$UWž&) 6mR
6mR 3RGHUHV GD 8QLmR LQGHSHQGHQWHVH SRGHUHV]HODQGRSDUDTXHXPQmRLQYDGDDHVIHUDGH
KDUP{QLFRVHQWUHVLR
KDUP{QLFRVHQWUHVLR/HJLVODWLYRR([H FRPSHWrQFLDVHDWULEXLo}HVGRRXWUR
FXWLYRHR-XGLFLiULR

2 3URI0$12(/ *21d$/9(6 )(55(,5$ ),/+2


VLVWHPDVGHFRQIXVmRGHSRGHUHVRQGH
UHJLVWUDTXH+iVLVWHPDVGHFRQIXVmRGHSRGHUHV
VLVWHPDVGHFRQIXVmRGHSRGHUHV
3RGHU/HJLVODWLYR
LQH[LVWHDGLYLVmRGHSRGHUHVSRUH[HPSORDVGLWDGXUDVGH
FRODERUDomR
FRODERUDomR GH SRGHUHV
SRGHUHV RQGH D GLYLVmR H[LVWH PDV RV
1DWULSDUWLomRGRSRGHUSUHFRQL]DGDSRU0RQWHVTXLHX
SRGHUHVQmRVmRLQGHSHQGHQWHVFRPRQRSDUODPHQWDULVPR
DRSRGHU/HJLVODWLYRDWULEXtDVHDWDUHIDGHHVWDEHOHFHUDV
HHQILPVLVWHPDV
VLVWHPDV
VLVWHPDV GH
GH VHSDUDomRGHSRGHUHV
VHSDUDomRGHSRGHUHVFRPRRSUHVL
GHSRGHUHV
UHJUDVGHGLUHLWRJHUDLVHLPSHVVRDLV&ODURTXHKRMHQHP
GHQFLDOLVPR LQ
&XUVR GH 'LUHLWR &RQVWLWXFLRQDO
  WRGDV HVVDVUHJUDVJHUDLVHLPSHVVRDLVVmRHGLWDGDVSRU
HG6DUDLYDSJ  HVVH SRGHU R -XGLFLiULR WDPEpP HGLWD QRUPDV VRE R
QRPHGHSURYLPHQWRR([HFXWLYRSRUVXDYH]HGLWDDV
$WULSDUWLomRGRVSRGHUHVUHVXOWRXGDQRYDFRQFHSomR PHGLGDVSURYLVyULDVRVGHFUHWRVHWF 
GR (VWDGR $QWLJDPHQWH RV UHLV HQFDUQDYDP WRGRV RV
SRGHUHV MXOJDYDP OHJLVODYDP H DGPLQLVWUDYDP   2V 23URI0$12(/*21d$/9(6)(55(,5$),/+2ID]D
SRGHUHVGR(VWDGRQDPRGHUQLGDGHVmRWULSDUWLGRV OLomR VHJXLQWH IRUPDO FRQFHLWXDomR R 3RGHU /HJLVODWLYR QmR
FRQVDJUDGD FRP 0RQWHVTXLHX  DR /HJLVODWLYR FXPSUH SDVVD GR SRGHU GH HGLWDU UHJUDV MXUtGLFDV VHJXQGR XP
ID]HUDVOHLVDR-XGLFLiULRMXOJDURVFDVRVFRQFUHWRVVREUH SURFHVVRIL[DGRQD&RQVWLWXLomRSDUDDHODERUDomRGHOHLV
VXD DSOLFDomR H DR ([HFXWLYR ID]HU FXPSULU WDLV OHLV H LQ
&XUVRGH'LUHLWR&RQVWLWXFLRQDO
HG6DUDLYDS 
WRPDU DV LQLFLDWLYDV SDUD VDWLVID]HU RV LQWHUHVVHV GD
FROHWLYLGDGH $QtYHOQDFLRQDOR3RGHU/HJLVODWLYRpFRPSRVWRSRU
GXDVFDVDVD&kPDUDGRV'HSXWDGRVHR6HQDGR)HGHUDO
-XQWDVHVVDVGXDVFDVDVFRPS}HPR&RQJUHVVR1DFLRQDO
1R %UDVLO FRPR VDEHPRV DGRWRXVH R VLVWHPD GD
$VVLPDVOHLVVmRSURSRVWDVQXPDGDVFDVDVHVHDSURYD
VHSDUDomRGHSRGHUHVDXW{QRPRVHKDUP{QLFRV DUWž&) 
GDVVmRGLVFXWLGDVQDRXWUDFDVD$SURYDGDDOHLQDVGXDV
$WULSDUWLomRGRVSRGHUHVUHVXOWRXGDQRYDFRQFHSomRGR FDVDV VHUi VDQFLRQDGD SHOR 3UHVLGHQWH GD 5HS~EOLFD H
(VWDGR$QWLJDPHQWHRVUHLVHQFDUQDYDPWRGRVRVSRGH SXEOLFDGD3DVVDUiHQWmRDWHUYLJrQFLD
UHV MXOJDYDPOHJLVODYDPHDGPLQLVWUDYDP 
4XDQGRD&RQVWLWXLomRHVWDEHOHFHUTXHXPDOHLGHYHVHU
2VSRGHUHVGR(VWDGRQDPRGHUQLGDGHVmRWULSDUWLGRV IHLWDSHOR&RQJUHVVR1DFLRQDOGHYHUmRUHXQLUVHD&kPDUD
OLomR FRQVDJUDGD FRP 0RQWHVTXLHX  DR /HJLVODWLYR GRV'HSXWDGRVHR6HQDGR)HGHUDOHMXQWRVGLVFXWLUmRH
-XGLFLiULRMXOJDURVFDVRVFRQFUH
FXPSUHID]HUDVOHLVDR-XGLFLiULR
-XGLFLiULR DSURYDUmRRXQmRRSURMHWRGHOHL
WRVVREUHVXDDSOLFDomRHDR( ([HFXWLYRID]HUFXPSULUWDLV
[HFXWLYR
OHLVHWRPDUDVLQLFLDWLYDVSDUDVDWLVID]HURVLQWHUHVVHVGD $DSURYDomRGDVOHLVVHMDSHOD&kPDUDGRV'HSXWDGRV
FROHWLYLGDGH VHMD SHOR 6HQDGR )HGHUDO RX HP UHXQLmR FRQMXQWD GR
&RQJUHVVR 1DFLRQDO GH UHJUD VHUi IHLWD SRU PDLRULD GH
6REUH D LQGHSHQGrQFLD
LQGHSHQGrQFLD GRV SRGHUHV
SRGHUHV R 0HVWUH -26e YRWRVGHVGHTXHSUHVHQWHDPDLRULDDEVROXWDGRVPHP
$)2162'$6,/9$REVHUYDTXHHODVLJQLILFDD TXHD EURVPDLRULDDEVROXWDpPDLVGDPHWDGHGRVPHPEURV
LQYHVWLGXUDHDSHUPDQrQFLDGDVSHVVRDVQXPGRVyUJmRVGR WRGRVH[LVWHQWHVHPDLRULDVLPSOHVpPDLVGDPHWDGHGRV
JRYHUQRQmRGHSHQGHPGDFRQILDQoDQHPGDYRQWDGHGRV SUHVHQWHV
RXWURVE TXHQRH[HUFtFLRGDVDWULEXLo}HVTXHOKHVHMDP
2VGHSXWDGRVVmRHOHLWRVSHORSRYRSDUDXPPDQGDWR
SUySULDVQmRSUHFLVDPRVWLWXODUHVFRQVXOWDURVRXWURVQHP
SDUODPHQWDU GH  DQRV 3RGHUmR VHPSUH VHU UHHOHLWRV
QHFHVVLWDPGHVXDDXWRUL]DomRF TXHQDRUJDQL]DomRGRV ,JXDOPHQWHRV6HQDGRUHVVmRHOHLWRVSHORSRYR0DVVHX
UHVSHFWLYRVVHUYLoRVFDGDXPpOLYUHREVHUYDGDVDSHQDVDV PDQGDWRpSDUDDQRVGR6HQDGRVmRHOHLWRVQXPD
GLVSRVLo}HV FRQVWLWXFLRQDLV H OHJDLV DVVLP p TXH FDEH DR HOHLomR H  DQRV GHSRLV p HVFROKLGR R RXWUR   2V
3UHVLGHQWHGD5HS~EOLFDSURYHUHH[WLQJXLUFDUJRVS~EOLFRV GHSXWDGRV UHSUHVHQWDP R SRYR VmR HOHLWRV SRU UHJL}HV
GD$GPLQLVWUDomRIHGHUDOEHPFRPRH[RQHUDURXGHPLWLU (VWDGRV PDVHPUD]mRGRQ~PHURGHHOHLWRUHVGDTXHOH
VHXVRFXSDQWHVHQTXDQWRpGDFRPSHWrQFLDGR&RQJUHVVR (VWDGR Ki XP OLPLWH PtQLPR H XP Pi[LPR SDUD FDGD
1DFLRQDORXGRV7ULEXQDLVSURYHURVFDUJRVGRVUHVSHFWLYRV (VWDGRSRULVVRTXHVHGL]TXHRHOHLWRU FLGDGmR GH
VHUYLoRVDGPLQLVWUDWLYRVH[RQHUDURXGHPLWLUVHXVRFXSDQ 6mR3DXORYDOHPHQRVTXHRGR$FUHSURSRUFLRQDOPHQWH
WHV jV &kPDUDV GR &RQJUHVVR H DRV 7ULEXQDLV FRPSHWH RVHOHLWRUHVGR$FUHWrPPDLVGHSXWDGRVTXHRVGR5LRGH
HODERUDU RV UHVSHFWLYRV UHJLPHQWRV LQWHUQRV HP TXH VH -DQHLURRXGH6mR3DXOR 
&kPDUD/HJLVODWLYDGR
  'LVWULWR)HGHUDO 'LU&RQVWLWXFLRQDO 

-iRV6HQDGRUHVUHSUHVHQWDPRV(VWDGRVFDGD(VWDGR 2EV DUHGDomRGHVWHDUWLJRIRLGDGDSHOD(PHQGD&RQVWLWXFLRQDOQž 5HIRUPD


$GPLQLVWUDWLYD 
WHPGLUHLWRDWHU6HQDGRUHV SRXFRLPSRUWDVHR(VWDGR
WHPPDLVHOHLWRUHVRXPHQRVPDLVXPDYH]RVSRXFRV , VLVWHPDWULEXWiULRDUUHFDGDomRHGLVWULEXLomRGH
HOHLWRUHVGHXPHVWDGLQKRYDOHPPDLVTXHRVPLOK}HVGH UHQGDV
HOHLWRUHVGHXPJUDQGH(VWDGRFRPRR5LRGH-DQHLURRX
6mR3DXORSRLVHOHJHPRPHVPRQ~PHURGH6HQDGRUHV  ,, SODQRSOXULDQXDOGLUHWUL]HVRUoDPHQWiULDVRUoD
PHQWRDQXDORSHUDo}HVGHFUpGLWRGtYLGDS~EOLFD
&DStWXOR, HHPLVV}HVGHFXUVRIRUoDGR
'R3RGHU/HJLVODWLYR
,,, IL[DomR H PRGLILFDomR GR HIHWLYR GDV)RUoDV
(VWUXWXUD)XQFLRQDPHQWRH$WULEXLo}HV $UPDGDV

6HomR, ,9 SODQRVHSURJUDPDVQDFLRQDLVUHJLRQDLVHVHWRULD


'R&RQJUHVVR1DFLRQDO LVGHGHVHQYROYLPHQWR

$UW 23RGHU/HJLVODWLYRpH[HUFLGRSHOR&RQJUHVVR1D 9 OLPLWHV GR WHUULWyULR QDFLRQDO HVSDoRDpUHRH


FLRQDOTXHVHFRPS}HGD&kPDUDGRV'HSXWDGRV PDUtWLPRHEHQVGRGRPtQLRGD8QLmR
HGR6HQDGR)HGHUDO
9, LQFRUSRUDomRVXEGLYLVmRRXGHVPHPEUDPHQWRGH
†~QLFR &DGDOHJLVODWXUDWHUiDGXUDomRGHTXDWURDQRV iUHDVGH7HUULWyULRVRX(VWDGRVRXYLGDVDVUHVSHF
WLYDV$VVHPEOpLDV/HJLVODWLYDV
$UW $&kPDUDGRV'HSXWDGRVFRPS}HVHGHUHSUHVHQ
WDQWHVGRSRYRHOHLWRVSHORVLVWHPDSURSRUFLRQDO 9,, WUDQVIHUrQFLD WHPSRUiULD GD VHGH GR*RYHUQR
HPFDGD(VWDGRHPFDGD7HUULWyULRHQR'LVWULWR
)HGHUDO
)HGHUDO
9,,,FRQFHVVmRGHDQLVWLD
†ž 2 Q~PHUR WRWDO GH 'HSXWDGRV EHPFRPRD
UHSUHVHQWDomRSRU(VWDGRHSHOR'LVWULWR)HGHUDO
VHUiHVWDEHOHFLGRSRUOHLFRPSOHPHQWDUSURSRUFL ,; RUJDQL]DomRDGPLQLVWUDWLYDMXGLFLiULDGR0LQLVWp
RQDOPHQWHjSRSXODomRSURFHGHQGRVHDRVDMXV ULR3~EOLFRHGD'HIHQVRULD3~EOLFDGD8QLmRHGRV
WHVQHFHVViULRVQRDQRDQWHULRUjVHOHLo}HVSDUD 7HUULWyULRVHRUJDQL]DomRMXGLFLiULDGR0LQLVWpULR
TXH QHQKXPD GDTXHODV XQLGDGHV GD )HGHUDomR 3~EOLFRHGD'HIHQVRULD3~EOLFDGR'LVWULWR)HGH
WHQKDPHQRVGHRLWRRXPDLVGHVHWHQWD'HSXWD UDO
GRV
; FULDomR WUDQVIRUPDomR HH[WLQomRGHFDUJRV
†ž &DGDWHUULWyULRHOHJHUiTXDWUR'HSXWDGRV HPSUHJRV H IXQo}HV S~EOLFDV REVHUYDGR R TXH
HVWDEHOHFHRDUW9,E
$UW2 6HQDGR)HGHUDOFRPS}HVHGHUHSUHVHQWDQWHV
GRV(VWDGRVHGR'LVWULWR)HGHUDOHOHLWRVVHJXQGR ;, FULDomR H H[WLQomR GH 0LQLVWpULRV H yUJmRVGD
RSULQFtSLRPDMRULWiULR DGPLQLVWUDomRS~EOLFD
2EV2VLQFV;H;,WrPVXDUHGDomRGDGDSHOD(PHQGD&RQVWLWXFLRQDOQž
†ž &DGD (VWDGR H R 'LVWULWR )HGHUDOHOHJHUmRWUrV
6HQDGRUHVFRPPDQGDWRGHRLWRDQRV ;,, WHOHFRPXQLFDo}HVHUDGLRGLIXVmR

†ž $ UHSUHVHQWDomR GH FDGD (VWDGR HGR'LVWULWR ;,,, PDWpULDILQDQFHLUDFDPELDOHPRQHWiULDLQVWLWXL


)HGHUDOVHUiUHQRYDGDGHTXDWURHPTXDWURDQRV o}HVILQDQFHLUDVHVXDRSHUDo}HV
DOWHUQDGDPHQWHSRUXPHGRLVWHUoRV
;,9 PRHGDVHXVOLPLWHV GH HPLVVmRHPRQWDQWHGD
†ž &DGD6HQDGRUVHUiHOHLWRFRPGRLVVXSOHQWHV GtYLGDPRELOLiULDIHGHUDO

6HomR,, ;9 IL[DomR GR VXEVtGLR GRV 0LQLVWURV GR6XSUHPR


'DV$WULEXLo}HVGR&RQJUHVVR1DFLRQDO 7ULEXQDO )HGHUDO REVHUYDGR R TXH GLVS}HP RV
DUWV†ž,,,,,H†ž,
$UW6DOYRGLVSRVLomRFRQVWLWXFLRQDOHPFRQWUiULRDV
2EV HVWHLQFLVRDFUHVFLGRSHOD(&Qƒ 5HIRUPD$GPLQLVWUDWLYD IRLPRGLILFDGR
GHOLEHUDo}HVGHFDGD&DVDHGHVXDV&RPLVV}HVVH SHOD5HIRUPDGD3UHYLGrQFLDGR*RYHUQR/XOD37(&Qƒ
UmR WRPDGDV SRU PDLRULD GRV YRWRV SUHVHQWH D
PDLRULDDEVROXWDGHVHXVPHPEURV $UWeGDFRPSHWrQFLDH[FOXVLYDGR&RQJUHVVR1DFLR
QDO
$UW&DEH DR &RQJUHVVR 1DFLRQDO FRP D VDQomRGR
3UHVLGHQWHGD5HS~EOLFDQmRH[LJLGDHVWDSDUDR , UHVROYHUGHILQLWLYDPHQWHVREUHWUDWDGRVDFRUGRV
HVSHFLILFDGRQRVDUWVHGLVSRUVREUH RXDWRVLQWHUQDFLRQDLVTXHDFDUUHWHPHQFDUJRVRX
WRGDV DV PDWpULDV GH FRPSHWrQFLD GD 8QLmR FRPSURPLVVRVJUDYRVRVDRSDWULP{QLRQDFLRQDO
HVSHFLDOPHQWHVREUH
 'LU&RQVWLWXFLRQDO
 &kPDUD/HJLVODWLYDGR'LVWULWR)HGHUDO
,, DXWRUL]DU R 3UHVLGHQWH GD 5HS~EOLFDDGHFODUDU $UW$&kPDUDGRV'HSXWDGRVHR6HQDGR)HGHUDORX
JXHUUD D FHOHEUDU D SD] D SHUPLWLU TXH IRUoDV TXDOTXHU GH VXDV &RPLVV}HV SRGHUmR FRQYRFDU
HVWUDQJHLUDVWUDQVLWHPSHORWHUULWyULRQDFLRQDORX 0LQLVWUR GH (VWDGR RX TXDLVTXHU WLWXODUHV GH
QHOHSHUPDQHoDPWHPSRUDULDPHQWHUHVVDOYDGRV yUJmRVGLUHWDPHQWHVXERUGLQDGRVj3UHVLGrQFLDGD
RVFDVRVSUHYLVWRVHPOHLFRPSOHPHQWDU 5HS~EOLFDSDUDSUHVWDUHPSHVVRDOPHQWHLQIRUPD
o}HV VREUH DVVXQWR SUHYLDPHQWH GHWHUPLQDGR
,,, DXWRUL]DU R 3UHVLGHQWH H R 9LFH3UHVLGHQWHGD LPSRUWDQGR HP FULPH GH UHVSRQVDELOLGDGH D
5HS~EOLFD D VH DXVHQWDUHP GR 3DtV TXDQGR D DXVrQFLDVHPMXVWLILFDomRDGHTXDGD
DXVrQFLDH[FHGHUDTXLQ]HGLDV
2EVRDUWFDSXWWHYHVXDUHGDomRDOWHUDGDSHOD(PHQGD&RQVWLWXFLRQDOGH
5HYLVmRQžGH
,9 DSURYDURHVWDGRGHGHIHVDHDLQWHUYHQomRIHGH
UDODXWRUL]DURHVWDGRGHVtWLRRXVXVSHQGHUTXDO †ž 2V0LQLVWURV GH (VWDGR SRGHUmRFRPSDUHFHUDR
TXHUXPDGHVVDVPHGLGDV 6HQDGR)HGHUDOj&kPDUDGRV'HSXWDGRVRXD
TXDOTXHUGHVXDV&RPLVV}HVSRUVXDLQLFLDWLYDH
9 VXVWDURVDWRVQRUPDWLYRVGR3RGHU([HFXWLYRTXH PHGLDQWHHQWHQGLPHQWRVFRPD0HVDUHVSHFWLYD
H[RUELWHPGRSRGHUUHJXODPHQWDURXGRVOLPLWHV SDUDH[SRUDVVXQWRGHUHOHYkQFLDGHVHX0LQLVWp
GHGHOHJDomROHJLVODWLYD ULR

9, PXGDUWHPSRUDULDPHQWHVXDVHGH †ž $V0HVDVGD&kPDUDGRV'HSXWDGRVHGR6HQDGR


)HGHUDOSRGHUmRHQFDPLQKDUSHGLGRVHVFULWRVGH
9,, IL[DULGrQWLFRVXEVtGLRSDUDRV'HSXWDGRV)HGHUDLV LQIRUPDomRDRV0LQLVWURVGH(VWDGRRXDTXDOTXHU
HRV6HQDGRUHVREVHUYDGRRTXHGLVS}HPRVDUWV GDVSHVVRDVUHIHULGDVQRFDSXWGHVWHDUWLJRLPSRU
;,†ž,,,,,H†ž, WDQGRHPFULPHGHUHVSRQVDELOLGDGHDUHFXVDRX
2EVDUHGDomRGHVWHLQFLVRIRLGDGDSHOD(PHQGD&RQVWLWXFLRQDOQž 5HIRUPD
RQmRDWHQGLPHQWRQRSUD]RGHWULQWDGLDVEHP
$GPLQLVWUDWLYD  FRPRDSUHVWDomRGHLQIRUPDo}HVIDOVDV
9,,,IL[DU RV VXEVtGLRV GR 3UHVLGHQWH HGR 2EVRDUWFDSXWWHYHVXDUHGDomRDOWHUDGDSHOD(PHQGDFRQVWLWXFLRQDO GH
5HYLVmRQž
9LFH3UHVLGHQWHGD5HS~EOLFDHGRV0LQLVWURVGH
(VWDGRREVHUYDGRRTXHGLVS}HPRVDUWV;, 6HomR,,,
†ž,,,,,H†ž, 'D&kPDUDGRV'HSXWDGRV
2EVUHGDomRGHVWHLQFLVRIRLGDGDSHOD(PHQGD&RQVWLWXFLRQDOQž 5HIRUPD
$GPLQLVWUDWLYD 
$UW&RPSHWHSULYDWLYDPHQWHj&kPDUD GRV'HSXWD
,; MXOJDUDQXDOPHQWHDVFRQWDVSUHVWDGDVSHOR3UHVL GRV
GHQWHGD5HS~EOLFDHDSUHFLDURVUHODWyULRVVREUH
DH[HFXomRGRVSODQRVGRJRYHUQR , DXWRUL]DU SRU GRLV WHUoRV GHVHXVPHPEURVD
LQVWDXUDomRGHSURFHVVRFRQWUDR3UHVLGHQWHHR
; ILVFDOL]DUHFRQWURODUGLUHWDPHQWHRXSRUTXDO 9LFH3UHVLGHQWH GD 5HS~EOLFD H RV 0LQLVWURV GH
TXHUGHVXDV&DVDVRVDWRVGR3RGHU([HFXWLYR (VWDGR
LQFOXtGRVRVGDDGPLQLVWUDomRLQGLUHWD
,, SURFHGHU j WRPDGD GH FRQWDV GR3UHVLGHQWHGD
;, ]HODUSHODSUHVHUYDomRGHVXDFRPSHWrQFLDOHJLVOD 5HS~EOLFDTXDQGRQmRDSUHVHQWDGDVDR&RQJUHV
WLYDHPIDFHGDDWULEXLomRQRUPDWLYDGRVRXWURV VR1DFLRQDOGHQWURGHVHVVHQWDGLDVDSyVDDEHUWX
3RGHUHV UDGDVHVVmROHJLVODWLYD

;,, DSUHFLDU RV DWRV GH FRQFHVVmR H UHQRYDomRGH ,,, HODERUDUVHXUHJLPHQWRLQWHUQR


FRQFHVVmRGHHPLVVRUDVGHUiGLRHWHOHYLVmR
,9 GLVSRU VREUH VXD RUJDQL]DomRIXQFLRQDPHQWR
;,,, HVFROKHUGRLVWHUoRVGRVPHPEURVGR7ULEXQDOGH SROtFLD FULDomR WUDQVIRUPDomR RX H[WLQomR GRV
&RQWDVGD8QLmR FDUJRVHPSUHJRVHIXQo}HVGHVHXVVHUYLoRVHD
LQLFLDWLYDGHOHLSDUDIL[DomRGDUHVSHFWLYDUHPX
;,9 DSURYDULQLFLDWLYDVGR3RGHUH[HFXWLYRUHIHUHQWHV QHUDomRREVHUYDGRVRVSDUkPHWURVHVWDEHOHFLGRV
DDWLYLGDGHVQXFOHDUHV QDOHLGHGLUHWUL]HVRUoDPHQWiULDV
2EVUHGDomRGHVWHLQFLVRIRLGDGDSHOD(PHQGD&RQVWLWXFLRQDOQž 5HIRUPD
;9 DXWRUL]DUUHIHUHQGRHFRQYRFDUSOHELVFLWR $GPLQLVWUDWLYD 

;9, DXWRUL]DUHPWHUUDVLQGtJHQDVDH[SORUDomRHR 9 HOHJHUPHPEURVGR&RQVHOKRGD5HS~EOLFDQRV


DSURYHLWDPHQWRGHUHFXUVRVKtGULFRVHDSHVTXLVD WHUPRVGRDUW9,,
HODYUDGHULTXH]DVPLQHUDLV
6HomR,9
;9,, DSURYDUSUHYLDPHQWHDDOLHQDomRRXFRQFHVVmR 'R6HQDGR)HGHUDO
GHWHUUDVS~EOLFDVFRPiUHDVXSHULRUDGRLVPLOH
TXLQKHQWRVKHFWDUHV $UW&RPSHWHSULYDWLYDPHQWHDR6HQDGR)HGHUDO
&kPDUD/HJLVODWLYDGR
  'LVWULWR)HGHUDO 'LU&RQVWLWXFLRQDO 

, SURFHVVDUHMXOJDUR3UHVLGHQWHHR9LFH3UHVLGHQWH ;, DSURYDUSRUPDLRULDDEVROXWDHSRUYRWRVHFUHWR


GD5HS~EOLFDQRVFULPHVGHUHVSRQVDELOLGDGHEHP DH[RQHUDomRGHRItFLRGR3URFXUDGRU*HUDOGD
FRPRRV0LQLVWURVGH(VWDGRHRV&RPDQGDQWHVGD 5HS~EOLFDDQWHVGRWpUPLQRGHVHXPDQGDWR
0DULQKDGR([pUFLWRHGD$HURQiXWLFDQRVFULPHV
GDPHVPDQDWXUH]DFRQH[RVFRPDTXHOHV ;,, HODERUDUVHXUHJLPHQWRLQWHUQR
2EVLQFLVR,PRGLILFDGRSHOD(PHQGD&RQVWLWXFLRQDOQž
;,,, GLVSRU VREUH VXD RUJDQL]DomRIXQFLRQDPHQWR
,, SURFHVVDUHMXOJDURV0LQLVWURVGR6XSUHPR7ULEX SROtFLD FULDomR WUDQVIRUPDomR RX H[WLQomR GRV
QDO)HGHUDORVPHPEURVGR&RQVHOKR1DFLRQDOGH FDUJRVHPSUHJRVHIXQo}HVGHVHXVVHUYLoRVHD
-XVWLoDHGR&RQVHOKR1DFLRQDOGR0LQLVWpULR3~ LQLFLDWLYDGHOHLSDUDIL[DomRGDUHVSHFWLYDUHPX
EOLFRR3URFXUDGRU*HUDOGD5HS~EOLFDHR$GYR QHUDomRREVHUYDGRVRVSDUkPHWURVHVWDEHOHFLGRV
JDGR*HUDOGD8QLmRQRVFULPHVGHUHVSRQVDELOL QDOHLGHGLUHWUL]HVRUoDPHQWiULDV
GDGH
2EVUHGDomRGDGDSHOD(PHQGD&RQVWLWXFLRQDOQž 5HIRUPD$GPLQLVWUDWLYD 
2EV,QFLVR,,PRGLILFDGRSHOD(PHQGD&RQVWLWXFLRQDOQž

,,, DSURYDU SUHYLDPHQWH SRU YRWR VHFUHWRDSyV ;,9 HOHJHUPHPEURVGR&RQVHOKRGD5HS~EOLFDQRV


DUJXLomRS~EOLFDDHVFROKDGH WHUPRVGRDUW9,,

D PDJLVWUDGRVQRVFDVRVHVWDEHOHFLGRVQHVWD&RQV ;9 DYDOLDU SHULRGLFDPHQWH D IXQFLRQDOLGDGHGR


WLWXLomR 6LVWHPD7ULEXWiULR1DFLRQDOHPVXDHVWUXWXUDH
VHXVFRPSRQHQWHVHRGHVHPSHQKRGDVDGPLQLV
E 0LQLVWURVGR7ULEXQDOGH&RQWDVGD8QLmRLQGLFD WUDo}HV WULEXWiULDV GD 8QLmR GRV (VWDGRV H GR
GRVSHOR3UHVLGHQWHGD5HS~EOLFD 'LVWULWR)HGHUDOHGRV0XQLFtSLRV
2EVHVWHLQFLVRIRLDFUHVFLGRSHOD(PHQGD&RQVWLWXFLRQDOQƒ
F *RYHUQDGRUGH7HUULWyULR
†~QLFR 1RVFDVRVSUHYLVWRVQRVLQFLVRV,H,,IXQFLRQDUi
FRPR3UHVLGHQWHRGR6XSUHPR7ULEXQDO)HGHUDO
G SUHVLGHQWHHGLUHWRUHVGREDQFRFHQWUDO
OLPLWDQGRVH D FRQGHQDomR TXH VRPHQWH VHUi
SURIHULGD SRU GRLV WHUoRV GRV YRWRV GR 6HQDGR
H 3URFXUDGRU*HUDOGD5HS~EOLFD
)HGHUDOjSHUGDGRFDUJRFRPLQDELOLWDomRSRU
RLWRDQRVSDUDRH[HUFtFLRGHIXQomRS~EOLFDVHP
I WLWXODUHVGHRXWURVFDUJRVTXHDOHLGHWHUPLQDU
SUHMXt]RGDVGHPDLVVDQo}HVMXGLFLDLVFDEtYHLV
,9 DSURYDU SUHYLDPHQWH SRU YRWR VHFUHWRDSyV
6HomR9
DUJXLomRHPVHVVmRVHFUHWDDHVFROKDGRVFKHIHV
'RV'HSXWDGRVHGRV6HQDGRUHV
GHPLVVmRGLSORPiWLFDGHFDUiWHUSHUPDQHQWH
$UW2V'HSXWDGRVH6HQDGRUHVVmRLQYLROiYHLVFLYLOH
9 DXWRUL]DURSHUDo}HVH[WHUQDVGHQDWXUH]DILQDQFH
SHQDOPHQWH SRU TXDLVTXHU GH VXDV RSLQL}HV
LUDGHLQWHUHVVHGD8QLmRGRV(VWDGRVGR'LVWULWR
SDODYUDVHYRWRV
)HGHUDOGRV7HUULWyULRVHGRV0XQLFtSLRV
†ž 2V'HSXWDGRVH6HQDGRUHVGHVGHDH[SHGLomRGR
9, IL[DU SRUSURSRVWDGR3UHVLGHQWHGD5HS~EOLFD
GLSORPDVHUmRVXEPHWLGRVDMXOJDPHQWRSHUDQWH
OLPLWHVJOREDLVSDUDRPRQWDQWHGDGtYLGDFRQVROL
R6XSUHPR7ULEXQDO)HGHUDO
GDGDGD8QLmRGRV(VWDGRVGR'LVWULWR)HGHUDOH
GRV0XQLFtSLRV
†ž 'HVGHDH[SHGLomRGRGLSORPDRVPHPEURVGR
&RQJUHVVR1DFLRQDOQmRSRGHUmRVHUSUHVRVVDOYR
9,, GLVSRUVREUHOLPLWHVJOREDLVHFRQGLo}HVSDUDDV
HPIODJUDQWHGHFULPHLQDILDQoiYHO1HVVHFDVRRV
RSHUDo}HVGHFUpGLWRH[WHUQRHLQWHUQRGD8QLmR
DXWRV VHUmR UHPHWLGRV GHQWUR GH YLQWH H TXDWUR
GRV(VWDGRVGR'LVWULWR)HGHUDOHGRV0XQLFtSLRV
KRUDVj&DVDUHVSHFWLYDSDUDTXHSHORYRWRGD
GHVXDVDXWDUTXLDVHGHPDLVHQWLGDGHVFRQWUROD
PDLRULDGHVHXVPHPEURVUHVROYDVREUHDSULVmR
GDVSHOR3RGHU3~EOLFRIHGHUDO
†ž 5HFHELGDDGHQ~QFLDFRQWUDR6HQDGRURX'HSXWD
9,,, GLVSRUVREUHOLPLWHVHFRQGLo}HVSDUDDFRQFHVVmR
GR SRU FULPH RFRUULGR DSyV D GLSORPDomR R
GH JDUDQWLD GD 8QLmR HP RSHUDo}HV GH FUpGLWR
6XSUHPR 7ULEXQDO )HGHUDO GDUi FLrQFLD j &DVD
H[WHUQRHLQWHUQR
UHVSHFWLYDTXHSRULQLFLDWLYDGHSDUWLGRSROtWLFR
QHODUHSUHVHQWDGRHSHORYRWRGDPDLRULDGHVHXV
,; HVWDEHOHFHU OLPLWHV JOREDLV H FRQGLo}HV SDUDR
PHPEURV SRGHUi DWp D GHFLVmR ILQDO VXVWDU R
PRQWDQWH GD GtYLGD PRELOLiULD GRV (VWDGRV GR
DQGDPHQWRGDDomR
'LVWULWR)HGHUDOHGRV0XQLFtSLRV
†ž 2 SHGLGR GH VXVWDomR VHUi DSUHFLDGRSHOD&DVD
; VXVSHQGHUDH[HFXomRQRWRGRRXHPSDUWHGHOHL
UHVSHFWLYDQRSUD]RLPSURUURJiYHOGHTXDUHQWDH
GHFODUDGDLQFRQVWLWXFLRQDOSRUGHFLVmRGHILQLWLYD
FLQFRGLDVGRVHXUHFHELPHQWRSHOD0HVD'LUHWRUD
GR6XSUHPR7ULEXQDO)HGHUDO
 'LU&RQVWLWXFLRQDO
 &kPDUD/HJLVODWLYDGR'LVWULWR)HGHUDO
†ž $ VXVWDomR GR SURFHVVR VXVSHQGHDSUHVFULomR ,,, TXHGHL[DUGHFRPSDUHFHUHPFDGDVHVVmROHJLVOD
HQTXDQWRGXUDURPDQGDWR WLYDjWHUoDSDUWHGDVVHVV}HVRUGLQiULDVGD&DVD
DTXHSHUWHQFHUVDOYROLFHQoDRXPLVVmRSRUHVWD
†ž 2V'HSXWDGRVH6HQDGRUHVQmRVHUmRREULJDGRVD DXWRUL]DGD
WHVWHPXQKDUVREUHLQIRUPDo}HVUHFHELGDVRXSUHV
WDGDV HP UD]mR GR H[HUFtFLR GR PDQGDWR QHP ,9 TXHSHUGHURXWLYHUVXVSHQVRVRVGLUHLWRVSROtWLFRV
VREUH DV SHVVRDV TXH OKHV FRQILDUDP RX GHOHV
UHFHEHUDPLQIRUPDo}HV 9 TXDQGRRGHFUHWDUD-XVWLoD(OHLWRUDOQRVFDVRV
SUHYLVWRVQHVWD&RQVWLWXLomR
†ž $LQFRUSRUDomRjV)RUoDV$UPDGDVGH'HSXWDGRV
H 6HQDGRUHV HPERUD PLOLWDUHV H DLQGD TXH HP 9, TXH VRIUHU FRQGHQDomR FULPLQDO HPVHQWHQoD
WHPSRGHJXHUUDGHSHQGHUiGHSUpYLDOLFHQoDGD WUDQVLWDGDHPMXOJDGR
&DVDUHVSHFWLYD
†ž eLQFRPSDWtYHOFRPRGHFRURSDUODPHQWDUDOpP
†ž $VLPXQLGDGHVGH'HSXWDGRVRX6HQDGRUHVVXE GRVFDVRVGHILQLGRVQRUHJLPHQWRLQWHUQRRDEXVR
VLVWLUmRGXUDQWHRHVWDGRGHVtWLRVySRGHQGRVHU GDV SUHUURJDWLYDV DVVHJXUDGDV D PHPEUR GR
VXVSHQVDV PHGLDQWH R YRWR GH GRLV WHUoRV GRV &RQJUHVVR1DFLRQDORXDSHUFHSomRGHYDQWDJHQV
PHPEURV GD &DVD UHVSHFWLYD QRV FDVRV GH DWRV LQGHYLGDV
SUDWLFDGRVIRUDGRUHFLQWRGR&RQJUHVVR1DFLRQDO
TXHVHMDPLQFRPSDWtYHLVFRPDH[HFXomRGDPH †ž 1RVFDVRVGRVLQFLVRV,,,H9,DSHUGDGRPDQGD
GLGD WRVHUiGHFLGLGDSHOD&kPDUDGRV'HSXWDGRVRX
2EV(VWH DUWLJR  WHP VXD UHGDomR GDGD SHOD (PHQGD &RQV SHOR6HQDGR)HGHUDOSRUPDLRULDDEVROXWDPHGL
WLWXFLRQDOQžGH
DQWHSURYRFDomRGDUHVSHFWLYD0HVDRXGHSDUWLGR
$UW2V'HSXWDGRVH6HQDGRUHVQmRSRGHUmR SROtWLFR UHSUHVHQWDGR QR &RQJUHVVR 1DFLRQDO
DVVHJXUDGDDPSODGHIHVD UHGDomRGDGDSHOD(PHQGD&RQVWLWXFL
RQDOQž
, GHVGHDH[SHGLomRGRGLSORPD
†ž 1RV FDVRV SUHYLVWRV QRV LQFLVRV ,,, D 9DSHUGD
D ILUPDURXPDQWHUFRQWUDWRFRPSHVVRDMXUtGLFDGH
VHUiGHFODUDGDSHOD0HVDGD&DVDUHVSHFWLYDGH
GLUHLWR S~EOLFR DXWDUTXLD HPSUHVD S~EOLFD
RItFLR RX PHGLDQWH SURYRFDomR GH TXDOTXHU GH
VRFLHGDGH GH HFRQRPLD PLVWD RX HPSUHVD FRQ
VHXVPHPEURVRXGHSDUWLGRSROtWLFRUHSUHVHQWD
FHVVLRQiULD GH VHUYLoR S~EOLFR VDOYR TXDQGR R
GR QR &RQJUHVVR 1DFLRQDO DVVHJXUDGD DPSOD
FRQWUDWRREHGHFHUDFOiXVXODVXQLIRUPHV
GHIHVD
E DFHLWDU RX H[HUFHU FDUJR IXQomR RX HPSUHJR
†ž $UHQ~QFLDGHSDUODPHQWDUVXEPHWLGRDSURFHVVR
UHPXQHUDGRLQFOXVLYHRVGHTXHVHMDPGHPLVVtYH
TXHYLVHRXSRVVDOHYDUjSHUGDGRPDQGDWRQRV
LVDGQXWXPQDVHQWLGDGHVFRQVWDQWHVGDDOtQHD WHUPRVGHVWHDUWLJRWHUiVHXVHIHLWRVVXVSHQVRV
DQWHULRU DWpDVGHOLEHUDo}HVILQDLVGHTXHWUDWDPRV††ž
Hž 2EV†žDFUHVFHQWDGRSHOD(PHQGD&RQVWLWXFLRQDOQž
,, GHVGHDSRVVH
$UW1mRSHUGHUiRPDQGDWRR'HSXWDGRRX6HQDGRU
D VHU SURSULHWiULRV FRQWURODGRUHV RX GLUHWRUHV GH
HPSUHVDTXHJR]HGHIDYRUGHFRUUHQWHGHFRQWUDWR , LQYHVWLGRQRFDUJRGH0LQLVWURGH(VWDGR*RYHU
FRP SHVVRD MXUtGLFD GH GLUHLWR S~EOLFR RX QHOD QDGRU GH 7HUULWyULR 6HFUHWiULR GH (VWDGR GR
H[HUFHUIXQomRUHPXQHUDGD 'LVWULWR)HGHUDOGH7HUULWyULRGH3UHIHLWXUDGH
&DSLWDORXFKHIHGHPLVVmRGLSORPiWLFDWHPSRUi
E RFXSDUFDUJRRXIXQomRGHTXHVHMDPGHPLVVtYHLV ULD
DGQXWXPQDV HQWLGDGHVUHIHULGDVQRLQFLVR,
D ,, OLFHQFLDGR SHOD UHVSHFWLYD &DVD SRUPRWLYRGH
GRHQoD RX SDUD WUDWDU VHP UHPXQHUDomR GH
F SDWURFLQDUFDXVDHPTXHVHMDLQWHUHVVDGDTXDOTXHU LQWHUHVVH SDUWLFXODU GHVGH TXH QHVWH FDVR R
GDVHQWLGDGHVDTXHVHUHIHUHRLQFLVR,D DIDVWDPHQWRQmRXOWUDSDVVHFHQWRHYLQWHGLDVSRU
VHVVmROHJLVODWLYD
G VHU WLWXODUHV GH PDLV GH XP FDUJR RX PDQGDWR
S~EOLFRHOHWLYR †ž 2VXSOHQWHVHUiFRQYRFDGRQRVFDVRVGHYDJDGH
LQYHVWLGXUDHPIXQo}HVSUHYLVWDVQHVWHDUWLJRRX
$UW3HUGHUiRPDQGDWRR'HSXWDGRRX6HQDGRU GHOLFHQoDVXSHULRUDFHQWRHYLQWHGLDV

, TXHLQIULQJLUTXDOTXHUGDVSURLELo}HVHVWDEHOHFL †ž 2FRUUHQGRYDJDHQmRKDYHQGRVXSOHQWHIDUVHi


GDVQRDUWLJRDQWHULRU HOHLomRSDUDSUHHQFKrODVHIDOWDUHPPDLVGHTXLQ
]HPHVHVSDUDRWpUPLQRGRPDQGDWR
,, FXMR SURFHGLPHQWR IRUGHFODUDGRLQFRPSDWtYHO
FRPRGHFRURSDUODPHQWDU †ž 1DKLSyWHVHGRLQFLVR,R'HSXWDGRRX6HQDGRU
SRGHUiRSWDUSHODUHPXQHUDomRGRPDQGDWR
&kPDUD/HJLVODWLYDGR
 'LVWULWR)HGHUDO 'LU&RQVWLWXFLRQDO 
LQF,, DOWHUDGR SHOD (&Qž  FULDQGR D QHFHVVLGDGH GH DSURYDomRGD
6HomR9, 2EV
FRQYRFDomRH[WUDRUGLQiULDSHODPDLRULDDEVROXWDGHFDGDXPDGDV&DVDVGR&RQ
'DV5HXQL}HV JUHVVR1DFLRQDORSURSyVLWRpVXEPHWHUDFRQYRFDomRDRVUHSUHVHQWDQWHVGRSRYR
QRSDUODPHQWR

$UW2
2 &RQJUHVVR1DFLRQDOUHXQLUVHiDQXDOPHQWH
&RQJUHVVR1DFLRQDOUHXQLUVHiDQXDOPHQWH †ž 1DVHVVmROHJLVODWLYDH[WUDRUGLQiULDR&RQJUHVVR
QD
QD &DSLWDO
&DSLWDO )HGHUDO
)HGHUDO GH
GH  GH
GH IHYHUHLUR
IHYHUHLUR D  GH 1DFLRQDOVRPHQWHGHOLEHUDUiVREUHDPDWpULDSDUD
MXOKRHGHžGHDJRVWRDGHGH]HPEUR DTXDOIRLFRQYRFDGRUHVVDOYDGDDKLSyWHVHGR†
2EV DOWHUDGRSHOD(&QžQRWH[WRDQWHULRURSHUtRGRHUDGHDH ž GHVWH DUWLJR YHGDGR R SDJDPHQWR GH SDUFHOD
GHžDD(PHQGDDWHQGHXDRVUHFODPRVSRSXODUHVGHUHGX]LURUHFHVVR LQGHQL]DWyULDHPUD]mRGDFRQYRFDomR
SDUODPHQWDU
2EV †žDOWHUDGRSHOD(&QžSDUDVDWLVID]HUDLQGLJQDomRSRSXODUFRQWUDRV
†ž $V UHXQL}HV PDUFDGDV SDUD HVVDVGDWDVVHUmR YXOWRVRVVXEVtGLRVSDJRVDRVSDUODPHQWDUHVHPUD]mRGDFRQYRFDomRH[WUDRUGLQiULD
GRUDYDQWHPHVPRTXHWUDEDOKDQGRHPVHVV}HVH[WUDRUGLQiULDVRVGHSXWDGRVH
WUDQVIHULGDVSDUDRSULPHLURGLD~WLOVXEVHTHQWH VHQDGRUHV QDGD UHFHEHUmR Mi TXH ILFRX SURLELGR R SDJDPHQWR GH SDUFHOD
TXDQGR UHFDtUHP HP ViEDGRV GRPLQJRV RX LQGHQL]DWyULDHPUD]mRGDFRQYRFDomR

IHULDGRV †ž +DYHQGRPHGLGDVSURYLVyULDVHPYLJRUQDGDWDGH


FRQYRFDomR H[WUDRUGLQiULD GR &RQJUHVVR 1DFLR
†ž $VHVVmROHJLVODWLYDQmRVHUiLQWHUURPSLGDVHPD QDO VHUmR HODV DXWRPDWLFDPHQWH LQFOXtGDV QD
DSURYDomRGRSURMHWR GHOHLGHGLUHWUL]HVRUoD SDXWDGDFRQYRFDomR
PHQWiULDV
2EV(VWH†žIRLDFUHVFHQWDGRSHOD(PHQGD&RQVWLWXFLRQDOQž

†ž $OpPGHRXWURVFDVRVSUHYLVWRVQHVWD&RQVWLWXLomR
6HomR9,,
D &kPDUD GRV 'HSXWDGRV H R 6HQDGR )HGHUDO
'DV&RPLVV}HV
UHXQLUVHmRHPVHVVmRFRQMXQWDSDUD
$UW2&RQJUHVVR1DFLRQDOHVXDV&DVDVWHUmRFRPLV
, LQDXJXUDUDVHVVmROHJLVODWLYD
V}HVSHUPDQHQWHVHWHPSRUiULDVFRQVWLWXtGDVQD
IRUPDHFRPDVDWULEXLo}HVSUHYLVWDVQRUHVSHFWLYR
,, HODERUDURUHJLPHQWRFRPXPHUHJXODUDFULDomR
UHJLPHQWRRXQRDWRGHTXHUHVXOWDUVXDFULDomR
GHVHUYLoRVFRPXQVjVGXDV&DVDV
†ž 1DFRQVWLWXLomRGDV0HVDVHGHFDGD&RPLVVmRp
,,, UHFHEHURFRPSURPLVVRGR3UHVLGHQWHHGR9LFH
DVVHJXUDGDWDQWRTXDQWRSRVVtYHODUHSUHVHQWD
3UHVLGHQWHGD5HS~EOLFD
omR SURSRUFLRQDO GRV SDUWLGRV RX GRV EORFRV
SDUODPHQWDUHVTXHSDUWLFLSDPGDUHVSHFWLYD&DVD
,9 FRQKHFHUGRYHWRHVREUHHOHGHOLEHUDU
†ž $VFRPLVV}HVHPUD]mRGDPDWpULDGHVXDFRPSH
†ž &DGD XPD GDV &DVDV UHXQLUVHiHPVHVV}HV
WrQFLDFDEH
SUHSDUDWyULDV D SDUWLU GH ž GH IHYHUHLUR QR
SULPHLURDQRGDOHJLVODWXUDSDUDDSRVVHGHVHXV
, GLVFXWLUHYRWDUSURMHWRGHOHLTXHGLVSHQVDUQD
PHPEURV H HOHLomR GDV UHVSHFWLYDV 0HVDV SDUD
IRUPDGRUHJLPHQWRDFRPSHWrQFLDGR3OHQiULR
PDQGDWRGH GRLV DQRVYHGDGDDUHFRQGXomR
VDOYRVHKRXYHUUHFXUVRGHXPGpFLPRGRVPHP
SDUD R PHVPR FDUJR QD HOHLomR LPHGLDWDPHQWH
EURVGD&DVD
VXEVHTHQWH
2EVDOWHUDGRSHOD(&Qž ,, UHDOL]DU DXGLrQFLDV S~EOLFDV FRPHQWLGDGHVGD
VRFLHGDGHFLYLO
†ž $0HVDGR&RQJUHVVR1DFLRQDOVHUiSUHVLGLGDSHOR ,,, FRQYRFDU0LQLVWURVGH(VWDGRSDUDSUHVWDULQIRU
3UHVLGHQWHGR6HQDGR)HGHUDOHRVGHPDLVFDUJRV PDo}HVVREUHDVVXQWRVLQHUHQWHVDVXDVDWULEXL
VHUmRH[HUFLGRVDOWHUQDGDPHQWHSHORVRFXSDQWHV o}HV
GHFDUJRVHTXLYDOHQWHVQD&kPDUDGRV'HSXWDGRV ,9 UHFHEHUSHWLo}HVUHFODPDo}HVUHSUHVHQWDo}HVRX
HQR6HQDGR)HGHUDO TXHL[DVGHTXDOTXHUSHVVRDFRQWUDDWRVRXRPLV
V}HVGDVDXWRULGDGHVRXHQWLGDGHVS~EOLFDV
†ž $FRQYRFDomRH[WUDRUGLQiULDGR&RQJUHVVR1DFLR 9 VROLFLWDUGHSRLPHQWRGHTXDOTXHUDXWRULGDGHRX
QDOIDUVHi FLGDGmR
9, DSUHFLDU SURJUDPDV GH REUDV SODQRVQDFLRQDLV
, SHOR3UHVLGHQWHGR6HQDGR)HGHUDOHPFDVRGH UHJLRQDLVHVHWRULDLVGHGHVHQYROYLPHQWRHVREUH
GHFUHWDomRGHHVWDGRGHGHIHVDRXGHLQWHUYHQomR HOHVHPLWLUSDUHFHU
IHGHUDOGHSHGLGRGHDXWRUL]DomRSDUDDGHFUHWD
omRGHHVWDGRGHVtWLRHSDUDRFRPSURPLVVRHD †ž $V FRPLVV}HV SDUODPHQWDUHV GHLQTXpULWRTXH
SRVVH GR 3UHVLGHQWH H GR 9LFH3UHVLGHQWH GD WHUmRSRGHUHVGHLQYHVWLJDomRSUySULRVGDVDXWRUL
5HS~EOLFD GDGHV MXGLFLDLV DOpP GH RXWURV SUHYLVWRV QRV
UHJLPHQWRV GDV UHVSHFWLYDV &DVDV VHUmR FULDGDV
,, SHOR3UHVLGHQWH GD 5HS~EOLFDSHORV3UHVLGHQWHV SHOD&kPDUDGRV'HSXWDGRVHSHOR6HQDGR)HGH
GD&kPDUDGRV'HSXWDGRVHGR6HQDGR)HGHUDO UDO HP FRQMXQWR RX VHSDUDGDPHQWH PHGLDQWH
RXDUHTXHULPHQWRGDPDLRULDGRVPHPEURVGH UHTXHULPHQWRGHXPWHUoRGHVHXVPHPEURVSDUD
DPEDVDV&DVDVHPFDVRGHXUJrQFLDRXLQWHUHVVH DDSXUDomRGHIDWRGHWHUPLQDGRHSRUSUD]RFHUWR
S~EOLFR UHOHYDQWH HP WRGDV DV KLSyWHVHV GHVWH VHQGRVXDVFRQFOXV}HVVHIRURFDVRHQFDPLQKD
LQFLVR FRP D DSURYDomR GD PDLRULD DEVROXWD GH GDV DR 0LQLVWpULR 3~EOLFR SDUD TXH SURPRYD D
FDGDXPDGDV&DVDVGR&RQJUHVVR1DFLRQDO UHVSRQVDELOLGDGHFLYLORXFULPLQDOGHLQIUDWRUHV
 'LU&RQVWLWXFLRQDO &kPDUD/HJLVODWLYDGR'LVWULWR)HGHUDO


†ž 'XUDQWHRUHFHVVRKDYHUiXPD&RPLVVmRUHSUH , DIRUPDIHGHUDWLYDGH(VWDGR


VHQWDWLYDGR&RQJUHVVR1DFLRQDOHOHLWDSRUVXDV ,, RYRWRGLUHWRVHFUHWRXQLYHUVDOHSHULyGLFR
&DVDV QD ~OWLPD VHVVmR RUGLQiULD GR SHUtRGR ,,, DVHSDUDomRGRV3RGHUHV
OHJLVODWLYRFRPDWULEXLo}HVGHILQLGDVQRUHJLPHQ ,9 RVGLUHLWRVHJDUDQWLDVLQGLYLGXDLV
WRFRPXPFXMDFRPSRVLomRUHSURGX]LUiTXDQWR
SRVVtYHO D SURSRUFLRQDOLGDGH GD UHSUHVHQWDomR †ž $ PDWpULD FRQVWDQWH GH SURSRVWDGHHPHQGD
SDUWLGiULD UHMHLWDGDRXKDYLGDSRUSUHMXGLFDGDQmRSRGHVHU
GHQRYDSURSRVWDQDPHVPDVHVVmROHJLVODWLYD

6XEVHomR,,,
6HomR9,,, 'DV/HLV
3URFHVVR/HJLVODWLYR
$UW$LQLFLDWLYDGDVOHLVFRPSOHPHQWDUHVHRUGLQiULDV
6XEVHomR, FDEHDTXDOTXHUPHPEURRX&RPLVVmRGD&kPDUD
'LVSRVLomR*HUDO
GRV'HSXWDGRVGR6HQDGRIHGHUDORXGR&RQJUHV
VR 1DFLRQDO DR 3UHVLGHQWH GD 5HS~EOLFD DR
6XSUHPR7ULEXQDO)HGHUDODRV7ULEXQDLV6XSHULR
$UW 2SURFHVVROHJLVODWLYRFRPSUHHQGHDHODERUDomR UHV DR 3URFXUDGRU *HUDO GD 5HS~EOLFD H DRV
GH FLGDGmRV QD IRUPD H QRV FDVRV SUHYLVWRV QHVWD
&RQVWLWXLomR
, HPHQGDVj&RQVWLWXLomR
,, OHLVFRPSOHPHQWDUHV †ž 6mRGHLQLFLDWLYD GR 3UHVLGHQWH GD5HS~EOLFDDV
,,, OHLVRUGLQiULDV OHLVTXH
,9 OHLVGHOHJDGDV
9 PHGLGDVSURYLVyULDV , IL[HPRXPRGLILTXHPRVHIHLWRVGDV)RUoDV$UPD
9, GHFUHWRVOHJLVODWLYRV GDV
9,, UHVROXo}HV
,, GLVSRQKDPVREUH
†~QLFR /HLFRPSOHPHQWDUGLVSRUiVREUHDHODERUDomR
UHGDomRDOWHUDomRHFRQVROLGDomRGDVOHLV D FULDomRGHFDUJRVIXQo}HVRXHPSUHJRVS~EOL
FRV QD DGPLQLVWUDomR GLUHWD H DXWiUTXLFD RX
6XEVHomR,, DXPHQWRGHVXDUHPXQHUDomR
'D(PHQGDj&RQVWLWXLomR
E RUJDQL]DomRDGPLQLVWUDWLYDHMXGLFLiULDPDWp
$UW$ &RQVWLWXLomR SRGHUi VHU HPHQGDGDPHGLDQWH ULDWULEXWiULDHRUoDPHQWiULDVHUYLoRVS~EOLFRV
SURSRVWD HSHVVRDOGDDGPLQLVWUDomRGRV7HUULWyULRV

, GHXPWHUoRQRPtQLPRGRVPHPEURVGD&kPDUD F VHUYLGRUHVS~EOLFRVGD8QLmRH7HUULWyULRVVHX


GRV'HSXWDGRVRXGR6HQDGR)HGHUDO UHJLPHMXUtGLFRSURYLPHQWRGHFDUJRVHVWDEL
OLGDGH H DSRVHQWDGRULD 2EV UHGDomR GDGD SHOD (PHQGD
,, GR3UHVLGHQWHGD5HS~EOLFD &RQVWLWXFLRQDOQžGH

G RUJDQL]DomRGR0LQLVWpULR3~EOLFRHGDGHIHQ
,,, GHPDLVGDPHWDGHGDV$VVHPEOpLDV/HJLVODWLYDV
VRULD 3~EOLFD GD 8QLmR EHP FRPR QRUPDV
GDV XQLGDGHV GD )HGHUDomR PDQLIHVWDQGRVH
JHUDLVSDUDDRUJDQL]DomRGR0LQLVWpULR3~EOL
FDGD XPD GHODV SHOD PDLRULD UHODWLYD GH VHXV
FR H GD  'HIHQVRULD 3~EOLFD GRV (VWDGRV GR
PHPEURV
'LVWULWR)HGHUDOHGRV7HUULWyULRV
†ž $ &RQVWLWXLomR QmR SRGHUi VHUHPHQGDGDQD
YLJrQFLD GH  LQWHUYHQomR IHGHUDO GH HVWDGR GH H FULDomRHH[WLQomRGH0LQLVWpULRVHyUJmRVGD
GHIHVDRXGHHVWDGRGHVtWLR DGPLQLVWUDomRS~EOLFDREVHUYDGRRGLVSRVWRQRDUW
9, 2EVUHGDomRGDGDSHOD(PHQGD&RQVWLWXFLRQDOQž

†ž $SURSRVWDVHUiGLVFXWLGDHYRWDGDHPFDGD&DVD
I  PLOLWDUHV GDV )RUoDV $UPDGDV VHXUHJLPH
GR &RQJUHVVR 1DFLRQDO HP GRLV WXUQRV
MXUtGLFR SURYLPHQWR GH FDUJRV SURPRo}HV
FRQVLGHUDQGRVHDSURYDGDVHREWLYHUHPDPERV
HVWDELOLGDGHUHPXQHUDomRUHIRUPDHWUDQVIH
RV WUrV TXLQWRV GRV YRWRV GRV UHVSHFWLYRV PHP
UrQFLDSDUDDUHVHUYD2EVDOtQHDIDFUHVFLGDSHOD(PHQGD
EURV &RQVWLWXFLRQDOQž

†ž $HPHQGDj&RQVWLWXLomRVHUiSURPXOJDGDSHODV †ž $ LQLFLDWLYD SRSXODU SRGH VHUH[HUFLGDSHOD


0HVDV GD  &kPDUD GRV 'HSXWDGRV H GR 6HQDGR DSUHVHQWDomRj&kPDUDGRV'HSXWDGRVGHSURMHWR
)HGHUDOFRPRUHVSHFWLYRQ~PHURGHRUGHP GHOHLVXEVFULWRSRUQRPtQLPRXPSRUFHQWRGR
HOHLWRUDGRQDFLRQDOGLVWULEXtGR SHORPHQRVSRU
†ž 1mR VHUi REMHWR GH GHOLEHUDomR DSURSRVWDGH FLQFR (VWDGRV FRP QmR PHQRV GH WUrV GpFLPRV
HPHQGDWHQGHQWHDDEROLU SRUFHQWRGRVHOHLWRUHVGDFDGDXPGHOHV
&kPDUD/HJLVODWLYDGR
  'LVWULWR)HGHUDO 'LU&RQVWLWXFLRQDO 

$UW (PFDVRGHUHOHYkQFLDHXUJrQFLDR3UHVLGHQWHGD †ž 6HDPHGLGDSURYLVyULDQmRIRUDSUHFLDGDHPDWp


5HS~EOLFD SRGHUi DGRWDU PHGLGDV SURYLVyULDV TXDUHQWDHFLQFRGLDVFRQWDGRVGHVXDSXEOLFDomR
FRP IRUoD GH OHL GHYHQGR VXEPHWrODV GH LPH HQWUDUiHPUHJLPHGHXUJrQFLDVXEVHTHQWHPHQ
GLDWRDR&RQJUHVVR1DFLRQDO WHHPFDGDXPDGDV&DVDVGR&RQJUHVVR1DFLR
QDO ILFDQGR VREUHVWDGDV DWp TXH VH XOWLPH D
†ž eYHGDGDDHGLomRGHPHGLGDVSURYLVyULDVVREUH YRWDomRWRGDVDVGHPDLVGHOLEHUDo}HVOHJLVODWLYDV
PDWpULD GD&DVDHPTXHHVWLYHUWUDPLWDQGR

, UHODWLYDD †ž 3URUURJDUVHiXPD~QLFDYH]SRULJXDOSHUtRGRD


YLJrQFLDGHPHGLGDSURYLVyULDTXHQRSUD]RGH
D QDFLRQDOLGDGH FLGDGDQLD GLUHLWRV SROtWLFRV VHVVHQWD GLDV FRQWDGR GH VXD SXEOLFDomR QmR
SDUWLGRVSROtWLFRVHGLUHLWRHOHLWRUDO WLYHUDVXDYRWDomRHQFHUUDGDQDVGXDV&DVDVGR
&RQJUHVVR1DFLRQDO
E GLUHLWR SHQDO SURFHVVXDO SHQDO H SURFHVVXDO
FLYLO †ž $VPHGLGDVSURYLVyULDVWHUmRVXDYRWDomRLQLFLDGD
QD&kPDUDGRV'HSXWDGRV
F RUJDQL]DomRGR3RGHU-XGLFLiULRHGR0LQLVWp
ULR 3~EOLFR D FDUUHLUD H D JDUDQWLD GH VHXV †ž &DEHUijFRPLVVmRPLVWDGH'HSXWDGRVH6HQDGR
PHPEURV UHVH[DPLQDUDVPHGLGDVSURYLVyULDVHVREUHHODV
HPLWLU SDUHFHU DQWHV GH VHUHP DSUHFLDGDV HP
G SODQRV SOXULDQXDLV GLUHWUL]HV RUoDPHQWiULDV VHVVmRVHSDUDGDSHORSOHQiULRGHFDGDXPDGDV
&DVDVGR&RQJUHVVR1DFLRQDO
RUoDPHQWRHFUpGLWRVDGLFLRQDLVHVXSOHPHQWD
UHVUHVVDOYDGRRSUHYLVWRQRDUW†ž
† eYHGDGDDUHHGLomRQDPHVPDVHVVmROHJLVODWLYD
GHPHGLGDSURYLVyULDTXHWHQKDVLGRUHMHLWDGDRX
,, TXH YLVH D GHWHQomR RX VHTHVWUR GHEHQVGH
TXH WHQKD SHUGLGR VXD HILFiFLD SRU GHFXUVR GH
SRXSDQoDSRSXODURXTXDOTXHURXWURDWLYRILQDQ
SUD]R
FHLUR
† 1mRHGLWDGRRGHFUHWROHJLVODWLYRDTXHVHUHIHUH
,,, UHVHUYDGDDOHLFRPSOHPHQWDU
R†žDWpVHVVHQWDGLDVDSyVDUHMHLomRRXSHUGD
GH HILFiFLD GH PHGLGD SURYLVyULD DV UHODo}HV
,9 Mi GLVFLSOLQDGDHPSURMHWRGHOHLDSURYDGRSHOR MXUtGLFDVFRQVWLWXtGDVHGHFRUUHQWHVGHDWRVSUDWL
&RQJUHVVR1DFLRQDOHSHQGHQWHGHVDQomRRXYHWR FDGRV GXUDQWH VXD YLJrQFLD FRQVHUYDUVHmR SRU
GR3UHVLGHQWHGD5HS~EOLFD HODUHJLGDV

†ž 0HGLGD SURYLVyULD TXH LPSOLTXHLQVWLWXLomRRX † $SURYDGRSURMHWRGHOHLGHFRQYHUVmRDOWHUDQGRR


PDMRUDomR GHLPSRVWRVH[FHWRRVSUHYLVWRVQRV WH[WRRULJLQDOGDPHGLGDSURYLVyULDHVWDPDQWHU
DUWV,,,,99H,,VySURGX]LUiHIHLWRV VHiLQWHJUDOPHQWHHPYLJRUDWpTXHVHMDVDQFLRQD
QR H[HUFtFLR ILQDQFHLUR VHJXLQWH VH KRXYHU VLGR GRRXYHWDGRRSURMHWR
FRQYHUWLGDHPOHLDWpR~OWLPRGLDGDTXHOHHPTXH
2EV 2DUWLJRHVHX†~QLFRIRLPRGLILFDGRSHOD(PHQGD&RQVWLWXFLRQDOQžGH
IRLHGLWDGD TXHWDPEpPDFUHVFHQWRXRVGHPDLVSDUiJUDIRV

†ž $VPHGLGDVSURYLVyULDVUHVVDOYDGRRGLVSRVWRQRV $UW1mRVHUiDGPLWLGRDXPHQWRGDGHVSHVDSUHYLVWD


††HSHUGHUmRHILFiFLDGHVGHDHGLomRVH
, QRVSURMHWRVGHLQLFLDWLYDH[FOXVLYDGR3UHVLGHQWH
QmRIRUHPFRQYHUWLGDVHPOHLQRSUD]RGHVHVVHQ
GD5HS~EOLFDUHVVDOYDGRRGLVSRVWRQRDUW††
WDGLDVSURUURJiYHOQRVWHUPRVGR†žXPDYH]
žHž
SRULJXDOSHUtRGRGHYHQGRR&RQJUHVVR1DFLRQDO
GLVFLSOLQDU SRU GHFUHWR OHJLVODWLYR DV UHODo}HV
,, QRV SURMHWRV VREUH RUJDQL]DomR GRVVHUYLoRV
MXUtGLFDVGHODVGHFRUUHQWHV
DGPLQLVWUDWLYRV GD &kPDUD GRV 'HSXWDGRV GR
6HQDGR )HGHUDO GRV 7ULEXQDLV )HGHUDLV H GR
†ž 2 SUD]R D TXH VH UHIHUH R † žFRQWDUVHiGD
0LQLVWpULR3~EOLFR
SXEOLFDomRGDPHGLGDSURYLVyULDVXVSHQGHQGRVH
GXUDQWH RV SHUtRGRV GH UHFHVVR GR &RQJUHVVR $UW$GLVFXVVmRHYRWDomRGRVSURMHWRVGHOHLGHLQL
1DFLRQDO FLDWLYDGR3UHVLGHQWHGD5HS~EOLFDGRVXSUHPR
7ULEXQDO)HGHUDOHGRV7ULEXQDLV6XSHULRUHVWHUmR
†ž $GHOLEHUDomRGHFDGDXPDGDV&DVDVGR&RQJUHV LQtFLRQD&kPDUDGRV'HSXWDGRV
VR1DFLRQDOVREUHRPpULWRGDVPHGLGDVSURYLVyUL
DVGHSHQGHUiGHMXt]RSUpYLRVREUHRDWHQGLPHQWR †ž 23UHVLGHQWHGD5HS~EOLFDSRGHUiVROLFLWDUXUJrQ
GHVHXVSUHVVXSRVWRVFRQVWLWXFLRQDLV FLDSDUDDSUHFLDomRGHSURMHWRVGHVXDLQLFLDWLYD
 'LU&RQVWLWXFLRQDO
 &kPDUD/HJLVODWLYDGR'LVWULWR)HGHUDO
†ž 6HQRFDVRGR†žD&kPDUDGRV'HSXWDGRVHR $UW$ PDWpULD FRQVWDQWH GH SURMHWR GH OHLUHMHLWDGR
6HQDGR )HGHUDO QmR VH PDQLIHVWDUHP VREUH D VRPHQWHSRGHUiFRQVWLWXLUREMHWRGHQRYRSURMHWR
SURSRVLomR FDGD TXDO VXFHVVLYDPHQWH HP DWp QDPHVPDVHVVmROHJLVODWLYDPHGLDQWHSURSRVWD
TXDUHQWD H FLQFR GLDV VREUHVWDUVHmR WRGDV DV GDPDLRULDDEVROXWDGRVPHPEURVGHTXDOTXHUGDV
GHPDLV GHOLEHUDo}HV OHJLVODWLYDV GD UHVSHFWLYD &DVDVGR&RQJUHVVR1DFLRQDO
&DVDFRPH[FHomRGDVTXHWHQKDPSUD]RFRQVWL
WXFLRQDOGHWHUPLQDGRDWpTXHVHXOWLPHDYRWD $UW $VOHLVGHOHJDGDVVHUmRHODERUDGDVSHOR3UHVLGHQWH
omR GD5HS~EOLFDTXHGHYHUiVROLFLWDUDGHOHJDomRDR
2EV†žFRPUHGDomRGDGDSHOD(PHQGD&RQVWLWXFLRQDOQž
&RQJUHVVR1DFLRQDO
†ž $DSUHFLDomRGDVHPHQGDVGR6HQDGR)HGHUDOSHOD
&kPDUDGRV'HSXWDGRVIDUVHiQRSUD]RGHGH] †ž 1mRVHUmR REMHWR GH GHOHJDomRRVDWRVGHFRP
GLDV REVHUYDGR TXDQWR DR PDLV R GLVSRVWR QR
SHWrQFLDH[FOXVLYDGR&RQJUHVVR1DFLRQDORVGH
SDUiJUDIRDQWHULRU
FRPSHWrQFLDSULYDWLYDGD&kPDUDGRV'HSXWDGRV
†ž 2V SUD]RV GR † ž QmR FRUUHP QRVSHUtRGRVGH RXGR6HQDGR)HGHUDODPDWpULDUHVHUYDGDjOHL
UHFHVVRGR&RQJUHVVR1DFLRQDOQHPVHDSOLFDP FRPSOHPHQWDUQHPDOHJLVODomRVREUH
DRVSURMHWRVGHFyGLJR
, RUJDQL]DomRGR3RGHU-XGLFLiULRHGR0LQLVWpULR
$UW2 SURMHWR GH OHL DSURYDGR SRU XPD &DVDVHUi 3~EOLFRDFDUUHLUDHDJDUDQWLDGHVHXVPHPEURV
UHYLVWRSHODRXWUDHPXPVyWXUQRGHGLVFXVVmRH
YRWDomRHHQYLDGRjVDQomRRXSURPXOJDomRVH ,, QDFLRQDOLGDGH FLGDGDQLD GLUHLWRVLQGLYLGXDLV
D &DVD UHYLVRUD R DSURYDU RX DUTXLYDGR VH R SROtWLFRVHHOHLWRUDLV
UHMHLWDU
,,, SODQRV SOXULDQXDLV GLUHWUL]HV RUoDPHQWiULDVH
†~QLFR 6HQGRRSURMHWRHPHQGDGRYROWDUij&DVDLQLFLD RUoDPHQWRV
GRUD
†ž $ GHOHJDomR DR 3UHVLGHQWH GD 5HS~EOLFDWHUiD
$UW $FDVDQDTXDOWHQKDVLGRFRQFOXtGDDYRWDomRHQ IRUPDGHUHVROXomRGR&RQJUHVVR1DFLRQDOTXH
YLDUiRSURMHWRGHOHLDR3UHVLGHQWHGD5HS~EOLFD HVSHFLILFDUi VHX FRQWH~GR H RV WHUPRV GH VHX
TXHDTXLHVFHQGRRVDQFLRQDUi H[HUFtFLR
†ž 6HR3UHVLGHQWHGD5HS~EOLFDFRQVLGHUDURSURMHWR
†ž 6HDUHVROXomRGHWHUPLQDUDDSUHFLDomRGRSURMHWR
QRWRGRRXHPSDUWHLQFRQVWLWXFLRQDORXFRQWUi
ULR DRV LQWHUHVVH S~EOLFR YHWiORi WRWDO RX SHOR&RQJUHVVR1DFLRQDOHVWHDIDUiHPYRWDomR
SDUFLDOPHQWH QR SUD]R GH TXLQ]H GLDV ~WHLV ~QLFDYHGDGDTXDOTXHUHPHQGD
FRQWDGRVGDGDWDGRUHFHELPHQWRHFRPXQLFDUi
GHQWURGHTXDUHQWDHRLWRKRUDVDR3UHVLGHQWHGR $UW$V OHLV FRPSOHPHQWDUHV VHUmR DSURYDGDVSRU
6HQDGR)HGHUDORVPRWLYRVGRYHWR PDLRULDDEVROXWD

†ž 2YHWRSDUFLDOVRPHQWHDEUDQJHUiWH[WRLQWHJUDO 6HomR,;


GHDUWLJRGHSDUiJUDIRGHLQFLVRRXGHDOtQHD 'D)LVFDOL]DomR
&RQWiELO)LQDQFHLUDH2UoDPHQWiULD
†ž 'HFRUULGRRSUD]RGHTXLQ]HGLDVRVLOrQFLRGR
3UHVLGHQWHGD5HS~EOLFDLPSRUWDUiHPVDQomR
$UW$ILVFDOL]DomRFRQWiELOILQDQFHLUDRUoDPHQWiULD
†ž 2YHWRVHUiDSUHFLDGRHPVHVVmRFRQMXQWDGHQWUR RSHUDFLRQDOHSDWULPRQLDOGD8QLmRHGDVHQWLGD
GH WULQWD GLDV D FRQWDU GH VHX UHFHELPHQWR Vy GHVGDDGPLQLVWUDomRGLUHWDHLQGLUHWDTXDQWRj
SRGHQGRVHUUHMHLWDGRSHORYRWRGDPDLRULDDEVR OHJDOLGDGHOHJLWLPLGDGHHFRQRPLFLGDGHDSOLFD
OXWDGRV'HSXWDGRVH6HQDGRUHV UHGDomRGDGDSHOD(PHQGD omRGDVVXEYHQo}HVHUHQ~QFLDGHUHFHLWDVVHUi
&RQVWLWXFLRQDOQž H[HUFLGDSHOR&RQJUHVVR1DFLRQDOPHGLDQWHFRQ
WUROHH[WHUQRHSHORVLVWHPDGHFRQWUROHLQWHUQR
†ž 6H R YHWR IRU PDQWLGR VHUi RSURMHWRHQYLDGR GHFDGD3RGHU
SDUDSURPXOJDomRDR3UHVLGHQWHGD5HS~EOLFD
†~QLFR 3UHVWDUiFRQWDVTXDOTXHUSHVVRDItVLFDRXMXUtGLFD
†ž (VJRWDGRVHPGHOLEHUDomRRSUD]RHVWDEHOHFLGRQR
† ž R YHWR VHUi FRORFDGR QD RUGHP GR GLD GD S~EOLFDRXSULYDGDTXHXWLOL]HDUUHFDGHJXDUGH
VHVVmRLPHGLDWDVREUHVWDGDVDVGHPDLVSURSRVL JHUHQFLHRXDGPLQLVWUHGLQKHLURVEHQVHYDORUHV
o}HVDWpVXDYRWDomRILQDO †žFRPUHGDomRGDGDSHOD(PHQGD S~EOLFRVRXSHORVTXDLVD8QLmRUHVSRQGDRXTXH
&RQVWLWXFLRQDOQž HPQRPHGHVWDDVVXPDREULJDo}HVGHQDWXUH]D
SHFXQLiULD UHGDomR GDGD SHOD (PHQGD &RQVWLWXFLRQDO Qž  5HIRUPD
†ž 6HDOHLQmRIRUSURPXOJDGDGHQWURGHTXDUHQWDH $GPLQLVWUDWLYD
RLWRKRUDVSHOR3UHVLGHQWHGD5HS~EOLFDQRVFDVRV
GRV††žHžR3UHVLGHQWHGR6HQDGRDSURPXO $UW2FRQWUROHH[WHUQRDFDUJRGR&RQJUHVVR1DFLR
JDUiHVHHVWHQmRRIL]HUHPLJXDOSUD]RFDEHUi QDOVHUiH[HUFLGRFRPRDX[tOLRGR7ULEXQDOGH
DR9LFH3UHVLGHQWHGR6HQDGRID]rOR &RQWDVGD8QLmRDRTXDOFRPSHWH
&kPDUD/HJLVODWLYDGR
  'LVWULWR)HGHUDO 'LU&RQVWLWXFLRQDO 

, DSUHFLDU DV FRQWDVSUHVWDGDVDQXDOPHQWHSHOR ;, UHSUHVHQWDUDR3RGHUFRPSHWHQWHVREUHLUUHJXODUL


3UHVLGHQWHGD5HS~EOLFDPHGLDQWHSDUHFHUSUpYLR GDGHVRXDEXVRVDSXUDGRV
TXH GHYHUi VHU HODERUDGR HP VHVVHQWD GLDV D
FRQWDUGHVHXUHFHELPHQWR †ž 1RFDVRGHFRQWUDWRRDWRGHVXVWDomRVHUiDGRWD
GR GLUHWDPHQWH SHOR FRQJUHVVR 1DFLRQDO TXH
,, MXOJDU DV FRQWDV GRV DGPLQLVWUDGRUHVHGHPDLV VROLFLWDUi GH LPHGLDWR DR 3RGHU ([HFXWLYR DV
UHVSRQViYHLVSRUGLQKHLURVEHQVHYDORUHVS~EOL PHGLGDVFDEtYHLV
FRVGDDGPLQLVWUDomRGLUHWDHLQGLUHWDLQFOXtGDVDV
IXQGDo}HV H VRFLHGDGHV LQVWLWXtGDV H PDQWLGDV †ž 6HR&RQJUHVVR1DFLRQDORXR3RGHUH[HFXWLYRQR
SHOR3RGHU3~EOLFRIHGHUDOHDVFRQWDVGDTXHOHV SUD]RGHQRYHQWDGLDV QmR HIHWLYDUDVPHGLGDV
TXH GHUHP FDXVD D SHUGD H[WUDYLR RX RXWUD SUHYLVWDVQRSDUiJUDIRDQWHULRUR7ULEXQDOGHFLGL
LUUHJXODULGDGHGHTXHUHVXOWHSUHMXt]RDRHUiULR UiDUHVSHLWR
S~EOLFR
†ž $VGHFLV}HVGR7ULEXQDOGHTXHUHVXOWHLPSXWDomR
,,, DSUHFLDUSDUDILQVGH UHJLVWUR DOHJDOLGDGHGRV GH GpELWR RX GH PXOWD WHUmR HILFiFLD GH WtWXOR
DWRVGHDGPLVVmRGHSHVVRDODTXDOTXHUWtWXORQD H[HFXWLYR
DGPLQLVWUDomR GLUHWD H LQGLUHWD LQFOXtGDV DV
IXQGDo}HV LQVWLWXtGDV H PDQWLGDV SHOR 3RGHU †ž 27ULEXQDOHQFDPLQKDUiDR&RQJUHVVR1DFLRQDO
3~EOLFRH[FHWXDGDVDVQRPHDo}HVSDUDFDUJRGH WULPHVWUDOHDQXDOPHQWHUHODWyULRGHVXDVDWLYLGD
SURYLPHQWRHPFRPLVVmREHPFRPRDGDVFRQFHV GHV
V}HV GH DSRVHQWDGRULDV UHIRUPDV H SHQV}HV
UHVVDOYDGDV DV PHOKRULDV SRVWHULRUHV TXH QmR $UW$&RPLVVmRPLVWDSHUPDQHQWHDTXHVHUHIHUHR
DOWHUDPRIXQGDPHQWROHJDOGRDWRFRQFHVVyULR DUW†žGLDQWHGHLQGtFLRVGHGHVSHVDVQmR
DXWRUL]DGDVDLQGDTXHVREDIRUPDGHLQYHVWLPHQ
,9 UHDOL]DU SRU LQLFLDWLYD SUySULD GD &kPDUDGH WRVQmRSURJUDPDGRVRXGHVXEVtGLRVQmRDSUR
'HSXWDGRV GR 6HQDGR )HGHUDO GH &RPLVVmR YDGRVSRGHUiVROLFLWDUjDXWRULGDGHJRYHUQDPHQ
WpFQLFDRXGHLQTXpULWRLQVSHo}HVHDXGLWRULDGH WDOUHVSRQViYHOTXHQRSUD]RGHFLQFRGLDVSUHVWH
QDWXUH]D FRQWiELO ILQDQFHLUD RUoDPHQWiULD RVHVFODUHFLPHQWRVQHFHVViULRV
RSHUDFLRQDOHSDWULPRQLDOQDVXQLGDGHVDGPLQLV
WUDWLYDV GRV 3RGHUHV /HJLVODWLYR ([HFXWLYR H †ž 1mRSUHVWDGRVRVHVFODUHFLPHQWRVRXFRQVLGHUD
-XGLFLiULRHGHPDLVHQWLGDGHVUHIHULGDVQRLQFLVR,, GRVHVWHVLQVXILFLHQWHVD&RPLVVmRVROLFLWDUiDR
7ULEXQDO SURQXQFLDPHQWR FRQFOXVLYR VREUH D
9 ILVFDOL]DU DV FRQWDV QDFLRQDLVGDVHPSUHVDV PDWpULDQRSUD]RGHWULQWDGLDV
VXSUDQDFLRQDLV GH FXMR FDSLWDO VRFLDO D 8QLmR
SDUWLFLSHGHIRUPDGLUHWDRXLQGLUHWDQRVWHUPRV †ž (QWHQGHQGR R 7ULEXQDO LUUHJXODU DGHVSHVDD
GRWUDWDGRFRQVWLWXWLYR &RPLVVmR VH  MXOJDU TXH R JDVWR SRVVD FDXVDU
GDQR LUUHSDUiYHO RX JUDYH OHVmR j HFRQRPLD
9, ILVFDOL]DU D DSOLFDomR GH TXDLVTXHUUHFXUVRV S~EOLFD SURSRUi DR &RQJUHVVR 1DFLRQDO VXD
UHSDVVDGRVSHOD8QLmRPHGLDQWHFRQYrQLRDFRU VXVWDomR
GRDMXVWHRXRXWURVLQVWUXPHQWRVFRQJrQHUHVD
(VWDGRDRGLVWULWR)HGHUDORXD0XQLFtSLR $UW2 7ULEXQDO GH &RQWDV GD 8QLmR LQWHJUDGRSRU
QRYH 0LQLVWURV WHP VHGH QR 'LVWULWR )HGHUDO
9,, SUHVWDUDVLQIRUPDo}HVVROLFLWDGDVSHOR&RQJUHVVR TXDGURSUySULRGHSHVVRDOHMXULVGLomRHPWRGR
1DFLRQDOSRUTXDOTXHUGHVXDVFDVDVRXSRUTXDO WHUULWyULRQDFLRQDOH[HUFHQGRQRTXHFRXEHUDV
TXHUGDVUHVSHFWLYDV&RPLVV}HVVREUHDILVFDOL]D DWULEXLo}HVSUHYLVWDVQRDUW
omRFRQWiELOILQDQFHLUDRUoDPHQWiULDRSHUDFLR
QDOHSDWULPRQLDOHVREUHUHVXOWDGRVGHDXGLWRULDV †ž 2V 0LQLVWURV GR 7ULEXQDO GH &RQWDVGD8QLmR
HLQVSHo}HVUHDOL]DGDV VHUmRQRPHDGRVGHQWUHEUDVLOHLURVTXHVDWLVIDoDP
RVVHJXLQWHVUHTXLVLWRV
9,,, DSOLFDUDRVUHVSRQViYHLVHPFDVRGHLOHJDOLGDGH
GHGHVSHVDRXLUUHJXODULGDGHGHFRQWDVDVVDQ , PDLVGHWULQWDHFLQFRHPHQRVGHVHVVHQWDHFLQFR
o}HV SUHYLVWDV HP OHL TXH HVWDEHOHFHUi HQWUH DQRVGHLGDGH
RXWUDVFRPLQDo}HVPXOWDSURSRUFLRQDODRGDQR
FDXVDGRDRHUiULR ,, LGRQHLGDGHPRUDOHUHSXWDomRLOLEDGD

,; DVVLQDUSUD]RSDUDTXHRyUJmRRXHQWLGDGHDGRWH ,,, QRWyULRVFRQKHFLPHQWRVMXUtGLFRVFRQWiEHLVHFR


DVSURYLGrQFLDVQHFHVViULDVDRH[DWRFXPSULPHQWR Q{PLFRVHILQDQFHLURVRXGHDGPLQLVWUDomRS~EOL
GDOHLVHYHULILFDGDLOHJDOLGDGH FD

; VXVWDUVHQmRDWHQGLGRDH[HFXomRGRDWRLPSXJ ,9 PDLV GH GH] DQRV GH H[HUFtFLR GH IXQomR HGH
QDGR FRPXQLFDQGR D GHFLVmR j &kPDUD GRV HIHWLYDDWLYLGDGHSURILVVLRQDOTXHH[LMDRVFRQKHFL
'HSXWDGRVHDR6HQDGR)HGHUDO PHQWRVPHQFLRQDGRVQRLQFLVRDQWHULRU
 'LU&RQVWLWXFLRQDO
 &kPDUD/HJLVODWLYDGR'LVWULWR)HGHUDO
†ž 2V 0LQLVWURV GR 7ULEXQDO GH &RQWDVGD8QLmR
VHUmRHVFROKLGRV
/HL2UJkQLFDGR'LVWULWR)HGHUDO
, XP WHUoR SHOR 3UHVLGHQWH GD5HS~EOLFDFRP
DSURYDomRGR6HQDGR)HGHUDOVHQGRGRLVDOWHUQD
GDPHQWHGHQWUHDXGLWRUHVHPHPEURVGR0LQLVWp 7Ì78/2,
ULR3~EOLFRMXQWRDR7ULEXQDOLQGLFDGRVHPOLVWD '26)81'$0(1726'$25*$1,=$d®2
WUtSOLFH SHOR 7ULEXQDO VHJXQGR RV FULWpULRV GH '2632'(5(6('2',675,72)('(5$/
DQWLJXLGDGHHPHUHFLPHQWR
$UWž 2 'LVWULWR )HGHUDO QR SOHQR H[HUFtFLR GHVXD
,, GRLVWHUoRVSHOR&RQJUHVVR1DFLRQDO DXWRQRPLD SROtWLFD DGPLQLVWUDWLYD H ILQDQFHLUD
REVHUYDGRU RV SULQFtSLRV FRQVWLWXFLRQDLV UH
†ž 2V 0LQLVWURV GR 7ULEXQDO GH &RQWDVGD8QLmR JHUVHiSRUHVWD/HL2UJkQLFD
WHUmRDVPHVPDVJDUDQWLDVSUHUURJDWLYDVLPSHGL
PHQWRVYHQFLPHQWRVHYDQWDJHQVGRV0LQLVWURV †~QLFR 7RGRRSRGHUHPDQDGRSRYRTXHRH[HUFHSRU
GR 6XSHULRU 7ULEXQDO GH -XVWLoD DSOLFDQGRVH PHLR GH UHSUHVHQWDQWHV HOHLWRV RX GLUHWDPHQWH
OKHVTXDQWRjDSRVHQWDGRULDHSHQVmRDVQRUPDV QRV WHUPRV GD &RQVWLWXLomR )HGHUDO H GHVWD /HL
FRQVWDQWHVGRDUW 2UJkQLFD
2EV D UHGDomR GHVWH † ž IRL DOWHUDGD SHOD (PHQGD &RQVWLWXFLRQDO Qž 
5HIRUPDGD3UHYLGrQFLD 
$UWž 2'LVWULWR)HGHUDOLQWHJUDDXQLmRLQGLVVRO~YHOGD
$UW2V 3RGHUHV /HJLVODWLYR ([HFXWLYR H-XGLFLiULR 5HS~EOLFD)HGHUDWLYDGR%UDVLOHWHPFRPRYDOR
PDQWHUmRGHIRUPDLQWHJUDGDVLVWHPDGHFRQWUR UHVIXQGDPHQWDLV
OHLQWHUQRFRPDILQDOLGDGHGH
, D SUHVHUYDomR GH VXDDXWRQRPLDFRPRXQLGDGH
, DYDOLDU R FXPSULPHQWR GDV PHWDVSUHYLVWDVQR IHGHUDWLYD
SODQRSOXULDQXDODH[HFXo}HVGRVSURJUDPDVGH ,, DSOHQDFLGDGDQLD
JRYHUQRHGRVRUoDPHQWRVGD8QLmR ,,, DGLJQLGDGHGDSHVVRDKXPDQD
,9 RVYDORUHVVRFLDLVGRWUDEDOKRHGDOLYUHLQLFLDWLYD
,, FRPSURYDU D OHJDOLGDGH H DYDOLDU RVUHVXOWDGRV 9 RSOXUDOLVPRSROtWLFR
TXDQWRjHILFiFLDHHILFLrQFLDGDJHVWmRRUoDPHQ
WiULDILQDQFHLUDHSDWULPRQLDOQRVyUJmRVHHQWL †~QLFR 1LQJXpP VHUi GLVFULPLQDGR RXSUHMXGLFDGRHP
GDGHV GD DGPLQLVWUDomR IHGHUDO EHP FRPR GD UD]mRGHQDVFLPHQWRLGDGHHWQLDUDoDFRUVH[R
DSOLFDomRGHUHFXUVRVS~EOLFRVSRUHQWLGDGHVGH FDUDFWHUtVWLFDV JHQpWLFDV HVWDGR FLYLO WUDEDOKR
GLUHLWRSULYDGR UXUDORXXUEDQRUHOLJLmRFRQYLFo}HVSROtWLFDVRX
ILORVyILFDV RULHQWDomR VH[XDO GHILFLrQFLD ItVLFD
,,, H[HUFHURFRQWUROHGDVRSHUDo}HVGHFUpGLWRDYDLV LPXQROyJLFDVHQVRULDORXPHQWDOSRUWHUFXPSUL
HJDUDQWLDVEHPFRPRGRVGLUHLWRVHKDYHUHVGD GR SHQD QHP SRU TXDOTXHU SDUWLFXODULGDGH RX
8QLmR FRQGLomRREVHUYDGDD&RQVWLWXLomR)HGHUDO QRYD
UHGDomRGDGDDR†~QLFRGR$UWžSHODHPHQGDj/2')Qž

,9 DSRLDU R FRQWUROH H[WHUQR QR H[HUFtFLR GHVXD


$UWž 6mRREMHWLYRVSULRULWiULRVGR'LVWULWR)HGHUDO
PLVVmRLQVWLWXFLRQDO
, JDUDQWLUHSURPRYHURVGLUHLWRVKXPDQRVDVVHJX
†ž 2VUHVSRQViYHLVSHORFRQWUROHLQWHUQRDRWRPD
UDGRV QD &RQVWLWXLomR )HGHUDO H QD 'HFODUDomR
UHPFRQKHFLPHQWRGHTXDOTXHULUUHJXODULGDGHRX
8QLYHUVDOGRV'LUHLWRV+XPDQRV
LOHJDOLGDGH GHOD GDUmR FLrQFLD DR 7ULEXQDO GH
&RQWDVGD8QLmRVRESHQDGHUHVSRQVDELOLGDGH
,, DVVHJXUDUDRFLGDGmRRH[HUFtFLRGRVGLUHLWRVGH
VROLGiULD
LQLFLDWLYDTXHOKHFRXEHUHPUHODWLYRVDRFRQWUROH
GD OHJDOLGDGH H OHJLWLPLGDGH GRV DWRV GR 3RGHU
†ž 4XDOTXHUFLGDGmRSDUWLGRSROtWLFRDVVRFLDomRRX
3~EOLFRHGDHILFiFLDGRVVHUYLoRVS~EOLFRV
VLQGLFDWRpSDUWHOHJtWLPDSDUDQDIRUPDGDOHL
GHQXQFLDULUUHJXODULGDGHVRXLOHJDOLGDGHVSHUDQWH
,,, SUHVHUYDURVLQWHUHVVHVJHUDLVHFROHWLYRV
R7ULEXQDOGH&RQWDVGD8QLmR
,9 SURPRYHUREHPGHWRGRV
$UW$VQRUPDVHVWDEHOHFLGDVQHVWDVHomRDSOLFDPVH
QR TXH FRXEHU j RUJDQL]DomR FRPSRVLomR H
9 SURSRUFLRQDU DRV VHXV KDELWDQWHV FRQGLo}HVGH
ILVFDOL]DomRGRV7ULEXQDLVGH&RQWDVGRV(VWDGRV
YLGD FRPSDWtYHLV FRP D GLJQLGDGH KXPDQD D
HGR'LVWULWR)HGHUDOEHPFRPRGR7ULEXQDLVH
MXVWLoDVRFLDOHREHPFRPXP
&RQVHOKRVGH&RQWDVGRV0XQLFtSLRV

†~QLFR $V&RQVWLWXLo}HVHVWDGXDLVGLVSRUmRVREUHRV7UL 9, GDUSULRULGDGHDRDWHQGLPHQWRGDVGHPDQGDVGD


EXQDLVGH&RQWDVUHVSHFWLYRVTXHVHUmRLQWHJUD VRFLHGDGHQDViUHDVGHHGXFDomRVD~GHWUDEDOKR
GRVSRUVHWH&RQVHOKHLURV WUDQVSRUWH VHJXUDQoD S~EOLFD PRUDGLD VDQHD
PHQWREiVLFROD]HUHDVVLVWrQFLDVRFLDO
&kPDUD/HJLVODWLYDGR
 'LVWULWR)HGHUDO 'LU&RQVWLWXFLRQDO 

9,, JDUDQWLUDSUHVWDomRGHDVVLVWrQFLDMXUtGLFDLQWH $UWž 2'LVWULWR)HGHUDOQDH[HFXomRGHVHXSURJUDPD


JUDOHJUDWXLWDDRVTXHFRPSURYDUHPLQVXILFLrQFLD GHGHVHQYROYLPHQWRHFRQ{PLFRVRFLDOEXVFDUiD
GHUHFXUVRV LQWHJUDomRFRPDUHJLmRGRHQWRUQRGR 'LVWULWR
)HGHUDO
9,,, SUHVHUYDUVXDLGHQWLGDGHDGHTXDQGRDVH[LJrQFL
DV GR GHVHQYROYLPHQWR j SUHVHUYDomR GH VXD &$3Ì78/2,,
PHPyULDWUDGLomRHSHFXOLDULGDGHV '$25*$1,=$d®2$'0,1,675$7,9$
'2',675,72)('(5$/
,; YDORUL]DUHGHVHQYROYHUDFXOWXUDORFDOGHPRGR
DFRQWULEXLUSDUDDFXOWXUDEUDVLOHLUD $UW 2'LVWULWR)HGHUDORUJDQL]DVHHP5HODo}HV$GPL
QLVWUDWLYDVFRPYLVWDVjGHVFHQWUDOL]DomRDGPLQLV
; DVVHJXUDUSRUSDUWHGRSRGHUS~EOLFRDSURWHomR WUDWLYDjXWLOL]DomRUDFLRQDOGHUHFXUVRVSDUD R
LQGLYLGXDOL]DGD j YLGD H j LQWHJULGDGH ItVLFD H GHVHQYROYLPHQWRVyFLRHFRQ{PLFRHjPHOKRULDGD
SVLFROyJLFD GDV YtWLPDV H WHVWHPXQKDV GH LQIUD TXDOLGDGHGHYLGD
o}HVSHQDLVHGHVXHVUHVSHFWLYRVIDPLOLDUHV ,QFOXtGR
RLQFLVR;SHOD(PHQGDD/HL2UJkQLFDQžGHGHRXWXEURGH
†ž $ OHL GLVSRUi VREUH D SDUWLFLSDomRSRSXODUQR
;, ]HODUSHORFRQMXQWRXUEDQtVWLFRGH%UDVtOLDWRPED SURFHVVRGHHVFROKDGR$GPLQLVWUDGRU5HJLRQDO
GR VRE D LQVFULomR Qž  GR /LYUR GR 7RPER
+LVWyULFR UHVSHLWDGDV DV GHILQLo}HV H FULWpULRV †ž $ UHPXQHUDomR GRV$GPLQLVWUDGRUHV5HJLRQDLV
FRQVWDQWHVGR'HFUHWRQžGHGHRXWXEUR QmRSRGHUiVHUVXSHULRUjIL[DGDSDUDRV6HFUHWi
GHHGD3RUWDULDQžGHGHRXWXEURGH ULRVGH*RYHUQRGR'LVWULWR)HGHUDO
GRHQWmR,QVWLWXWR%UDVLOHLURGR3DWULP{QLR
&XOWXUDO  ,%3& KRMH ,QVWLWXWR GR 3DWULP{QLR †ž $SURLELomRGHTXHWUDWDRDUW†ƒDSOLFDVHj
+LVWyULFRH$UWtVWLFR1DFLRQDO,3+$1 ,QFOXtGRSHOD QRPHDomRGHDGPLQLVWUDGRUUHJLRQDO
(PHQGDD/HL2UJkQLFDQžGHGHGH]HPEURGHSXEOLFDGDQR'2')GH ILFDDFUHVFHQWDGRR†žDRDUWSHODHPHQGDjOHLRUJkQLFDQžGH
 '2')GH

127$ )LFDVXEVWLWXtGDDH[SUHVVmR´6HFUHWiULRGH*RYHUQRGR'LVWULWR
;,, SURPRYHU SURWHJHU H GHIHQGHU RV GLUHLWRVGD )HGHUDOµ SRU ´6HFUHWiULR GH (VWDGR GR 'LVWULWR )HGHUDOµ
FULDQoDGRDGROHVFHQWHHGRMRYHPDFUHVFHQWDGRRLQFLVR &21)250( (PHQGD ­ /HL 2UJkQLFD Qž  GH  
;,,DR$UWžSHODHPHQGDjOHLRUJkQLFDQžGH'2')GH '2')'(

$UWž eDVVHJXUDGRRH[HUFtFLRGRGLUHLWRGHSHWLomRRX $UW $V$GPLQLVWUDo}HV5HJLRQDLVLQWHJUDPDHVWUXWXUD


UHSUHVHQWDomRLQGHSHQGHQWHPHQWHGHSDJDPHQWR DGPLQLVWUDWLYDGR'LVWULWR)HGHUDO
GH WD[DV RX HPROXPHQWRV RX GH JDUDQWLD GH
LQVWkQFLD $UW&DGD 5HJLmR $GPLQLVWUDWLYD GR 'LVWULWR)HGHUDO
WHUiXP&RQVHOKRGH5HSUHVHQWDQWHV&RPXQLWiUL
$UWž $VREHUDQLDSRSXODUVHUiH[HUFLGDSHORVXIUiJLR RVFRPIXQo}HVFRQVXOWLYDVHILVFDOL]DGRUDVQD
XQLYHUVDOHSHORYRWRGLUHWRHVHFUHWRFRPYDORU IRUPDGDOHL
LJXDOSDUDWRGRVHQRVWHUPRVGDOHLPHGLDQWH
$UW $FULDomRRXH[WLQomRGH5HJL}HV$GPLQLVWUDWLYDV
, SOHELVFLWR RFRUUHUi PHGLDQWH OHL DSURYDGD SHOD PDLRULD
,, UHIHUHQGR DEVROXWDGRV'HSXWDGRV'LVWULWDLV
,,, LQLFLDWLYDSRSXODU
†~QLFR &RPDFULDomRGHQRYDUHJLmRDGPLQLVWUDWLYDILFD
7Ì78/2,, FULDGRDXWRPDWLFDPHQWHFRQVHOKRWXWHODUSDUDD
'$25*$1,=$d®2'2',675,72)('(5$/ UHVSHFWLYDUHJLmR DFUHVFHQWDGRSHODHPHQGDjOHLRUJkQLFDGRGIQž
GH

&$3Ì78/2, &$3Ì78/2,,,
'$6',6326,d¯(6*(5$,6 '$&203(7È1&,$'2',675,72)('(5$/
$UWž %UDVtOLD &DSLWDO GD 5HS~EOLFD )HGHUDWLYDGR $UW $R'LVWULWR)HGHUDOVmRDWULEXtGDVDVFRPSHWrQFLDV
%UDVLOpDVHGHGRJRYHUQRGR'LVWULWR)HGHUDO OHJLVODWLYDVUHVHUYDGDVDRV(VWDGRVH0XQLFtSLRV
FDEHQGROKHH[HUFHUHPVHXWHUULWyULRWRGDVDV
$UWž 6mR VtPERORV GR 'LVWULWR )HGHUDO D EDQGHLUDR FRPSHWrQFLDV TXH QmR OKH VHMDP YHGDGDV SHOD
KLQRHREUDVmR &RQVWLWXLomR)HGHUDO
†~QLFR $OHLSRGHUiHVWDEHOHFHURXWURVVtPERORVHGLVSRU 6HomR,
VREUHVHXXVRQRWHUULWyULRGR'LVWULWR)HGHUDO 'D&RPSHWrQFLD3ULYDWLYD
$UWž 2 WHUULWyULR GR 'LVWULWR )HGHUDO FRPSUHHQGHR $UW&RPSHWHSULYDWLYDPHQWHDR'LVWULWR)HGHUDO
HVSDoRItVLFRJHRJUiILFRTXHVHHQFRQWUDVREVHX
GRPtQLRHMXULVGLomR
 'LU&RQVWLWXFLRQDO
 &kPDUD/HJLVODWLYDGR'LVWULWR)HGHUDO
, RUJDQL]DUVHX*RYHUQRH$GPLQLVWUDomR ;9, UHJXODPHQWDUHILVFDOL]DURFRPpUFLRDPEXODQWH
LQFOXVLYHRGHSDSpLVHGHRXWURVUHVtGXRVUHFLFOi
,, FULDURUJDQL]DURXH[WLQJXLU5HJL}HV$GPLQLVWUDWL YHLV
YDVGHDFRUGRFRPDOHJLVODomRYLJHQWH
;9,, GLVSRUVREUHDOLPSH]DGHORJUDGRXURVS~EOLFRV
,,, LQVWLWXLUHDUUHFDGDUWULEXWRVREVHUYDGDDFRPSH UHPRomRHGHVWLQRGROL[RGRPLFLOLDUHGHRXWURV
WrQFLDFXPXODWLYDGR'LVWULWR)HGHUDO UHVtGXRV

,9 IL[DUILVFDOL]DUHFREUDUWDULIDVHSUHoRVS~EOLFRV ;9,,, GLVSRUVREUHVHUYLoRVIXQHUiULRVHDGPLQLVWUDomR


GHVXDFRPSHWrQFLD GRVFHPLWpULRV

9 GLVSRUVREUHDDGPLQLVWUDomRXWLOL]DomRDTXLVLomR ;,; GLVSRU VREUH DSUHHQVmR GHSyVLWR H GHVWLQRGH


HDOLHQDomRGRVEHQVS~EOLFRV DQLPDLVHPHUFDGRULDVDSUHHQGLGDVHPGHFRUUrQ
FLDGHWUDQVJUHVVmRGDOHJLVODomRORFDO
9, RUJDQL]DUHSUHVWDUGLUHWDPHQWHRXVREUHJLPHGH
FRQFHVVmRRXSHUPLVVmRRVVHUYLoRVGHLQWHUHVVH ;; GLVFLSOLQDUHILVFDOL]DUQRkPELWRGHVXDFRPSH
ORFDOLQFOXtGRRGHWUDQVSRUWHFROHWLYRTXHWHP WrQFLD FRPSHWLo}HV HVSRUWLYDV HVSHWiFXORV
FDUiWHUHVVHQFLDO GLYHUV}HV S~EOLFDV H HYHQWRV GH QDWXUH]D VHPH
OKDQWHUHDOL]DGRVHPORFDLVGHDFHVVRS~EOLFR
9,, PDQWHUFRPDFRRSHUDomRWpFQLFDHILQDQFHLUDGD
8QLmRSURJUDPDVGHHGXFDomRSULRULWDULDPHQWH ;;, GLVSRU VREUH D XWLOL]DomR GH YLDV HORJUDGRXURV
GHHQVLQRIXQGDPHQWDOHSUpHVFRODU S~EOLFRV

9,,,FHOHEUDUHILUPDUDMXVWHVFRQVyUFLRVFRQYrQLRV ;;,, GLVFLSOLQDU R WUkQVLWR ORFDO VLQDOL]DQGR DVYLDV


DFRUGRVHGHFLV}HVDGPLQLVWUDWLYDVFRPD8QLmR XUEDQDVHHVWUDGDVGR'LVWULWR)HGHUDO
(VWDGRVH0XQLFtSLRVSDUDH[HFXomRGHVXDVOHLV
HVHUYLoRV ;;,,, H[HUFHU LQVSHomR H ILVFDOL]DomR VDQLWiULDGH
SRVWXUD DPELHQWDO WULEXWiULD GH VHJXUDQoD
,; HODERUDUHH[HFXWDURSODQRSOXULDQXDODVGLUHWUL S~EOLFDHGRWUDEDOKRUHODWLYDPHQWHDRIXQFLRQD
]HVRUoDPHQWiULDVHRRUoDPHQWRDQXDO PHQWRGHHVWDEHOHFLPHQWRFRPHUFLDOLQGXVWULDO
SUHVWDGRUGHVHUYLoRVHVLPLODUQRkPELWRGHVXD
; HODERUDUHH[HFXWDUR3ODQR'LUHWRUGH2UGHQD FRPSHWrQFLDUHVSHLWDGDDOHJLVODomRIHGHUDO
PHQWR 7HUULWRULDO D /HL GH 8VR H 2FXSDomR GR
6ROR H 3ODQRV GH 'HVHQYROYLPHQWR /RFDO SDUD ;;,9 DGTXLULUEHQVLQFOXVLYHSRUPHLRGHGHVDSURSULD
SURPRYHU DGHTXDGR RUGHQDPHQWR WHUULWRULDO omRSRUQHFHVVLGDGHXWLOLGDGHS~EOLFDRXLQWHUHV
LQWHJUDGR DRV YDORUHV DPELHQWDLV PHGLDQWH VHVRFLDOQRVWHUPRVGDOHJLVODomRHPYLJRU
SODQHMDPHQWRHFRQWUROHGRXVRSDUFHODPHQWRH
RFXSDomRGRVRORXUEDQR ;;9 OLFHQFLDUDFRQVWUXomRGHTXDOTXHUREUD
QRYDUHGDomRGDGDDRLQFLVR;GR$UWSHOD(PHQGD­/HL2UJkQLFDQžGH
'2')
;;9, LQWHUGLWDUHGLILFDo}HVHPUXtQDHPFRQGLo}HVGH
;, DXWRUL]DUFRQFHGHURXSHUPLWLUEHPFRPRUHJX LQVDOXEULGDGHHDVTXHDSUHVHQWHPDVLUUHJXODULGD
ODUOLFHQFLDUHILVFDOL]DURVVHUYLoRVGHYHtFXORVGH GHVSUHYLVWDVQDOHJLVODomRHVSHFtILFDEHPFRPR
DOXJXpLV ID]HUGHPROLUFRQVWUXo}HVTXHDPHDFHPDVHJX
UDQoDLQGLYLGXDORXFROHWLYD
;,, GLVSRUVREUHFULDomRWUDQVIRUPDomRHH[WLQomRGH
FDUJRVHPSUHJRVHIXQo}HVS~EOLFDV ;;9,, GLVSRU VREUH SXEOLFLGDGH H[WHUQD HPHVSHFLDO
VREUHH[LELomRGHFDUWD]HVDQ~QFLRVHTXDLVTXHU
;,,, GLVSRU VREUH D RUJDQL]DomR GR TXDGUR GHVHXV RXWURVPHLRVGHSXEOLFLGDGHRXSURSDJDQGDHP
VHUYLGRUHVLQVWLWXLomRGHSODQRVGHFDUUHLUDQD ORJUDGRXURVS~EOLFRVHPORFDLVGHDFHVVRS~EOLFR
DGPLQLVWUDomR GLUHWD DXWDUTXLDV H IXQGDo}HV RXGHVWHVYLVtYHLV
S~EOLFDV GR 'LVWULWR )HGHUDO UHPXQHUDomR H
UHJLPHMXUtGLFR~QLFRGRVVHUYLGRUHV 6HomR,,
'D&RPSHWrQFLD&RPXP
;,9 H[HUFHURSRGHUGHSROtFLDDGPLQLVWUDWLYD
$UWe FRPSHWrQFLD GR 'LVWULWR )HGHUDO HPFRPXP
;9 OLFHQFLDU HVWDEHOHFLPHQWR LQGXVWULDOFRPHUFLDO FRPD8QLmR
SUHVWDGRUGHVHUYLoRVHVLPLODURXFDVVDURDOYDUi
GHOLFHQoDGRVTXHVHWRUQDUHPGDQRVRVDRPHLR , ]HODUSHODJXDUGDGD&RQVWLWXLomR)HGHUDOGHVWD
DPELHQWHjVD~GHDREHPHVWDUGDSRSXODomRRX /HL2UJkQLFDGDVOHLVHGDVLQVWLWXLo}HVGHPRFUiWL
TXHLQIULQJLUHPGLVSRVLWLYRVOHJDLV FDV
&kPDUD/HJLVODWLYDGR
 'LVWULWR)HGHUDO 'LU&RQVWLWXFLRQDO 

,, FRQVHUYDURSDWULP{QLRS~EOLFR 9,, SURWHomRGRSDWULP{QLRKLVWyULFRFXOWXUDODUWtVWL


FRSDLVDJtVWLFRHWXUtVWLFR
,,, SURWHJHUGRFXPHQWRVHRXWURVEHQVGHYDORUKLV
WyULFR H FXOWXUDO PRQXPHQWRV SDLVDJHQV QDWX 9,,, UHVSRQVDELOLGDGHSRUGDQRVDRPHLRDPELHQWHDR
UDLVQRWiYHLVHVtWLRVDUTXHROyJLFRVEHPFRPRLP FRQVXPLGRUHDEHQVHGLUHLWRVGHYDORUDUWtVWLFR
SHGLUVXDHYDVmRGHVWUXLomRHGHVFDUDFWHUL]DomR HVWpWLFRKLVWyULFRHVSHOHROyJLFRWXUtVWLFRHSDLVD
JtVWLFR
,9 SURWHJHURPHLRDPELHQWHHFRPEDWHUDSROXLomR
HPTXDOTXHUGHVXDVIRUPDV ,; HGXFDomRFXOWXUDHQVLQRHGHVSRUWR

9 SUHVHUYDUDIDXQDDIORUDHRFHUUDGR ; SUHYLGrQFLDVRFLDOSURWHomRHGHIHVDGDVD~GH

9, SURSRUFLRQDU RV PHLRV GH DFHVVR j FXOWXUDj ;, GHIHQVRULD S~EOLFD H DVVLVWrQFLD MXUtGLFDQRV
HGXFDomRHjFLrQFLD WHUPRV GD OHJLVODomR HP YLJRU QRYD UHGDomR GDGD SHOD
HPHQGDjOHLRUJkQLFDGRGIQž

9,, SUHVWDUVHUYLoRVGHDVVLVWrQFLDjVD~GHGDSRSXOD
omRHGHSURWHomRHJDUDQWLDDSHVVRDVSRUWDGRUDV ;,, SURWHomR H LQWHJUDomR VRFLDO GDV SHVVRDVFRP
GHGHILFLrQFLDFRPDFRRSHUDomRWpFQLFDHILQDQ GHILFLrQFLD QRYDUHGDomRGDGDSHODHPHQGDjOHLRUJkQLFDGRGIQž
FHLUDGD8QLmR ;,,, SURWHomRjLQIkQFLDHjMXYHQWXGH

9,,, FRPEDWHUDVFDXVDVGDSREUH]DDVXEQXWULomRHRV ;,9 PDQXWHQomRGDRUGHPHVHJXUDQoDLQWHUQDV


IDWRUHVGHPDUJLQDOL]DomRSURPRYHQGRDLQWHJUD
omRVRFLDOGRVVHJPHQWRVGHVIDYRUHFLGRV ;9 SURFHGLPHQWRVHPPDWpULDSURFHVVXDO

,; IRPHQWDUDSURGXomRDJURSHFXiULDHRUJDQL]DUR ;9, RUJDQL]DomR JDUDQWLDV GLUHLWRV H GHYHUHVGD


DEDVWHFLPHQWRDOLPHQWDU SROtFLDFLYLO

; SURPRYHUSURJUDPDVGHFRQVWUXomRGHPRUDGLDV †ž 2'LVWULWR)HGHUDOQRH[HUFtFLRGHVXDFRPSHWrQ


H D PHOKRULD GDV FRQGLo}HV KDELWDFLRQDLV H GH FLD VXSOHPHQWDU REVHUYDUi DV QRUPDV JHUDLV
VDQHDPHQWREiVLFR HVWDEHOHFLGDVSHOD8QLmR

;, UHJLVWUDUDFRPSDQKDUHILVFDOL]DUDVFRQFHVV}HV †ž ,QH[LVWLQGR OHL IHGHUDO VREUH QRUPDVJHUDLVR


GHGLUHLWRVGHSHVTXLVDHH[SORUDomRGHUHFXUVRV 'LVWULWR)HGHUDOH[HUFHUiFRPSHWrQFLDOHJLVODWLYD
KtGULFRVHPLQHUDLVHPVHXWHUULWyULR SOHQDSDUDDWHQGHUVXDVSHFXOLDULGDGHV

;,, HVWDEHOHFHUHLPSODQWDUSROtWLFDSDUDDVHJXUDQoD †ž $ VXSHUYHQLrQFLD GH OHL IHGHUDOVREUHQRUPDV


GRWUkQVLWR JHUDLVVXVSHQGHDHILFiFLDGHOHLORFDOQRTXHOKH
IRUFRQWUiULR
†~QLFR /HLFRPSOHPHQWDUGHYHIL[DUQRUPDSDUDDFRRSH
UDomRHQWUHD8QLmRHR'LVWULWR)HGHUDOWHQGRHP &$3Ì78/2,9
YLVWDRHTXLOtEULRGRGHVHQYROYLPHQWRHREHPHVW '$69('$d¯(6
DU QR kPELWR GR WHUULWyULR GR 'LVWULWR )HGHUDO
DFUHVFHQWDGRSHODHPHQGDjOHLRUJkQLFDGRGIQžGH
$UWeYHGDGRDR'LVWULWR)HGHUDO
6HomR,,,
, HVWDEHOHFHUFXOWRVUHOLJLRVRVRXLJUHMDVVXEYHQFLR
'D&RPSHWrQFLD&RQFRUUHQWH
QiORV HPEDUDoDUOKHV R IXQFLRQDPHQWR RX
PDQWHUFRPHOHVRXVHXVUHSUHVHQWDQWHVUHODo}HV
$UW&RPSHWHDR'LVWULWR)HGHUDOFRQFRUUHQWHPHQWH
GHGHSHQGrQFLDRXDOLDQoDUHVVDOYDGDQDIRUPD
FRPD8QLmROHJLVODUVREUH
GDOHLDFRODERUDomRGHLQWHUHVVHS~EOLFR
, GLUHLWRWULEXWiULRILQDQFHLURSHQLWHQFLiULRHFRQ{
,, UHFXVDUIpDRVGRFXPHQWRVS~EOLFRV
PLFRHXUEDQtVWLFR
,,, VXEYHQFLRQDURXDX[LOLDUGHTXDOTXHUPRGRFRP
,, RUoDPHQWR
UHFXUVRV S~EOLFRV TXHU SHOD LPSUHQVD UiGLR
WHOHYLVmR VHUYLoR GH DOWRIDODQWH RX TXDOTXHU
,,, MXQWDFRPHUFLDO
RXWURPHLRGHFRPXQLFDomRSURSDJDQGDSROtWL
,9 FXVWDVGHVHUYLoRVIRUHQVHV FRSDUWLGiULDRXFRPILQVHVWUDQKRVjDGPLQLVWUD
omRS~EOLFD
9 SURGXomRHFRQVXPR
,9 GRDUEHQVLPyYHLVGHVHXSDWULP{QLRRXFRQVWLWXLU
9, FHUUDGR FDoD SHVFD IDXQD FRQVHUYDomRGD VREUHHOHV{QXVUHDOEHPFRPRFRQFHGHULVHQo}HV
QDWXUH]DGHIHVDGRVRORHGRVUHFXUVRVQDWXUDLV ILVFDLV RX UHPLVV}HV GH GtYLGDV VHP H[SUHVVD
SURWHomRGRPHLRDPELHQWHHFRQWUROHGDSROXL DXWRUL]DomRGD&kPDUD/HJLVODWLYDVRESHQDGH
omR QXOLGDGHGRDWR
 'LU&RQVWLWXFLRQDO
 &kPDUD/HJLVODWLYDGR'LVWULWR)HGHUDO
&$3Ì78/29 ,; D UHYLVmR JHUDO GH UHPXQHUDomR GRVVHUYLGRUHV
'$$'0,1,675$d®23Ô%/,&$ S~EOLFRVIDUVHiVHPSUHQDPHVPDGDWD

6HomR, ; SDUDILQVGRGLVSRVWRQR$UW;,GD&RQVWLWXL


'LVSRVLo}HV*HUDLV omRGD5HS~EOLFD)HGHUDWLYDGR%UDVLOILFDHVWDEH
OHFLGRTXHDUHPXQHUDomRHRVXEVtGLRGRVRFX
$UW $DGPLQLVWUDomRS~EOLFDGLUHWDLQGLUHWDRXIXQGD SDQWHVGHFDUJRVIXQo}HV HHPSUHJRVS~EOLFRV
FLRQDOGHTXDOTXHUGRV3RGHUHVGR'LVWULWR)HGH GRV PHPEURV GH TXDOTXHU GRV 3RGHUHV H GRV
UDO REHGHFHUi DRV SULQFtSLRV GH OHJDOLGDGH GHPDLVDJHQWHVSROtWLFRVGR'LVWULWR)HGHUDOEHP
LPSHVVRDOLGDGHPRUDOLGDGHSXEOLFLGDGHWUDQVSD FRPRRVSURYHQWRVGHDSRVHQWDGRULDVHSHQV}HV
UrQFLDGDVFRQWDVS~EOLFDVUD]RDELOLGDGHPRWLYD QmR SRGHUmR H[FHGHU R VXEVtGLR PHQVDO HP
omRHLQWHUHVVHS~EOLFRHWDPEpPDRVHJXLQWH HVSpFLH GRV 'HVHPEDUJDGRUHV GR 7ULEXQDO GH
QRYDUHGDomRGDGDSHODHPHQGDjOHLRUJkQLFDQž
-XVWLoDGR'LVWULWR)HGHUDOH7HUULWyULRVQDIRUPD
GD OHL QmR VH DSOLFDQGR R GLVSRVWR QHVWH LQFLVR
, RVFDUJRVRVHPSUHJRVHDVIXQo}HVS~EOLFDVVmR DRVVXEVtGLRVGRV'HSXWDGRV'LVWULWDLV QRYDUHGDomR
DFHVVtYHLV DRV EUDVLOHLURV TXH SUHHQFKDP RV GDGDDRLQFLVR;GR$UWSHOD(PHQGD­/HL2UJkQLFDQž
UHTXLVLWRV HVWDEHOHFLGRV HP OHL DVVLP FRPR DRV
127$ )LFDVXEVWLWXtGDDH[SUHVVmR´6HFUHWiULRGH*RYHUQRGR'LVWULWR
HVWUDQJHLURVQDIRUPDGDOHJLVODomR QRYDUHGDomRGDGD
SHODHPHQGDjOHLRUJkQLFDGRGIQž )HGHUDOµ SRU ´6HFUHWiULR GH (VWDGR GR 'LVWULWR )HGHUDOµ
&21)250((PHQGD­/HL2UJkQLFDQžGH'2')
'(
,, D LQYHVWLGXUD HP FDUJR RX HPSUHJRS~EOLFR
GHSHQGHGHDSURYDomRSUpYLDHPFRQFXUVRS~EOL ;, RVYHQFLPHQWRVGRVFDUJRVGR3RGHU/HJLVODWLYR
FR GH SURYDV RX GH SURYDV H WtWXORV GH DFRUGR QmRSRGHUmRVHUVXSHULRUHVDRVSDJRVSHOR3RGHU
FRP D QDWXUH]D H D FRPSOH[LGDGH GR FDUJR RX ([HFXWLYRQRWDYLGHOHLQžGH'2')'(TXH
HPSUHJRQDIRUPDSUHYLVWDHPOHLUHVVDOYDGDVDV IL[DWHWRGHUHPXQHUDomRQRÇ0%,72'2',675,72)('(5$/
QRPHDo}HV SDUD FDUJR HP FRPLVVmR GHFODUDGR
HPOHLGHOLYUHQRPHDomRHH[RQHUDomR QRYDUHGDomR ;,, pYHGDGDDYLQFXODomRRXHTXLSDUDomRGHTXDLV
GDGDSHODHPHQGDjOHLRUJkQLFDGRGIQž
TXHU HVSpFLHV UHPXQHUDWyULDV SDUD R HIHLWR GH
UHPXQHUDomRGHSHVVRDOGRVHUYLoRS~EOLFR QRYD
,,, RSUD]RGHYDOLGDGHGRFRQFXUVRS~EOLFRVHUiGH UHGDomRGDGDSHODHPHQGDjOHLRUJkQLFDGRGIQž
DWp GRLV DQRV SURUURJiYHO XPD YH] SRU LJXDO
SHUtRGR ;,,, RVDFUpVFLPRVSHFXQLiULRVSHUFHELGRVSRUVHUYLGRU
S~EOLFR QmR VmR FRPSXWDGRV QHP DFXPXODGRV
,9 GXUDQWHRSUD]RLPSURUURJiYHOSUHYLVWRQRHGLWDO SDUDILQVGHFRQFHVVmRGHDFUpVFLPRVXOWHULRUHV
QRYDUHGDomRGDGDSHODHPHQGDjOHLRUJkQLFDGRGIQž
GHFRQYRFDomRRDSURYDGRHPFRQFXUVRS~EOLFR
GHSURYDVRXGHSURYDVHWtWXORVVHUiFRQYRFDGR
FRP SULRULGDGH VREUH QRYRV FRQFXUVDGRV SDUD ;,9 R VXEVtGLR H RV YHQFLPHQWRV GRV RFXSDQWHVGH
DVVXPLUFDUJRRXHPSUHJRQDFDUUHLUD FDUJRV H HPSUHJRV S~EOLFRV VmR LUUHGXWtYHLV
UHVVDOYDGR R GLVSRVWR QRYD UHGDomR GDGD SHOD HPHQGD j OHL
RUJkQLFDGRGIQž
9 DVIXQo}HVGHFRQILDQoDH[HUFLGDVH[FOXVLYDPHQWH
SRUVHUYLGRUHVRFXSDQWHVGHFDUJRHIHWLYRHSHOR D QRVLQFLVRV;H;,,,GHVWHDUWLJRHQRDUW9
PHQRVFLQTHQWDSRUFHQWRGRVFDUJRVHPFRPLV E QRVDUWV†ž,,,,,H†ž,
VmRDVHUHPSUHHQFKLGRVSRUVHUYLGRUHVGHFDUUHL GD&RQVWLWXLomR)HGHUDO
UDQRVFDVRVHFRQGLo}HVSUHYLVWRVHPOHLGHVWL
QDPVHDSHQDVjVDWULEXLo}HVGHGLUHomRFKHILDH ;9 p YHGDGD D DFXPXODomR UHPXQHUDGD GHFDUJRV
DVVHVVRUDPHQWR S~EOLFRVH[FHWRTXDQGRKRXYHUFRPSDWLELOLGDGH
QRYDUHGDomRGDGDDRLQFLVR9GRDUWLJRSHODHPHQGDjOHLRUJkQFLDQžGH
'2')GH GHKRUiULRV
127$ 9,'(/(,1ž'('2')'(48(
D DGHGRLVFDUJRVGHSURIHVVRU
5(*8/$0(17$(67(,1&,629'(67($57,*2
E DGHXPFDUJRGHSURIHVVRUFRPRXWURWpFQLFRRX
9, 5HYRJDGRSHODHPHQGDjOHLRUJkQLFDGRGIQž FLHQWtILFR
F DGHGRLVFDUJRVRXHPSUHJRVSULYDWLYRVGHSURILV
9,, DOHLUHVHUYDUiSHUFHQWXDOGHFDUJRVHHPSUHJRV VLRQDLVGHVD~GHFRPSURILVV}HVUHJXODPHQWDGDV
QRYDUHGDomRGDGDjDOtQHD´FµGRLQFLVR[YGR$UWSHODHPHQGDjOHLRUJkQLFD
S~EOLFRVSDUDSRUWDGRUHVGHGHILFLrQFLDJDUDQWLQ Qž'2')GH
GRDVDGDSWDo}HVQHFHVViULDVDVXDSDUWLFLSDomR
HPFRQFXUVRVS~EOLFRVEHPFRPRGHILQLUiFULWpUL ;9, DSURLELomRGHDFXPXODUHVWHQGHVHDHPSUHJRVH
RVGHVXDDGPLVVmR IXQo}HVHDEUDQJHDXWDUTXLDVIXQGDo}HVHPSUH
VDVS~EOLFDVVRFLHGDGHVGHHFRQRPLDPLVWDVXDV
9,,,D OHL HVWDEHOHFHUi RV FDVRV GH FRQWUDWDomRGH VXEVLGLiULDVHVRFLHGDGHVFRQWURODGDVGLUHWDRX
SHVVRDO SRU WHPSR GHWHUPLQDGR SDUD DWHQGHU D LQGLUHWDPHQWHSHOR3RGHU3~EOLFR QRYDUHGDomRGDGDSHOD
HPHQGDjOHLRUJkQLFDGRGIQž
QHFHVVLGDGHWHPSRUiULDGHH[FHSFLRQDOLQWHUHVVH
S~EOLFR
 &kPDUD/HJLVODWLYDGR
 'LVWULWR)HGHUDO 'LU&RQVWLWXFLRQDO 

;9,, DDGPLQLVWUDomRID]HQGiULDHVHXVDJHQWHVILVFDLV ,9 GLUHWRUHV GH HPSUHVDV S~EOLFDV VRFLHGDGHVGH


DRV TXDLV FRPSHWH H[HUFHU SULYDWLYDPHQWH D HFRQRPLDPLVWDDXWDUTXLDVHIXQGDo}HV QRYDUHGDomR
ILVFDOL]DomRGHWULEXWRVGR'LVWULWR)HGHUDOWHUmR GDGDSHODHPHQGDjOHLRUJkQLFDGRGIQž

HPVXDViUHDVGHFRPSHWrQFLDHMXULVGLomRSUHFH
GrQFLDVREUHRVGHPDLVVHWRUHVDGPLQLVWUDWLYRVQD 9 $GPLQLVWUDGRUHV5HJLRQDLV
IRUPDGDOHL
9, 3URFXUDGRU*HUDOGR'LVWULWR)HGHUDO
;9,,, VRPHQWHSRUOHLHVSHFtILFDSRGHVHU QRYDUHGDomRGDGD
SHODHPHQGDjOHLRUJkQLFDGRGIQž 9,, &RQVHOKHLURV GR 7ULEXQDO GH &RQWDV GR'LVWULWR
)HGHUDO
D FULDGD DXWDUTXLD H DXWRUL]DGD D LQVWLWXLomR GH
HPSUHVDS~EOLFDGHVRFLHGDGHGHHFRQRPLDPLVWD 9,,,'HSXWDGRV'LVWULWDLV
HGHIXQGDomRFDEHQGRDOHLFRPSOHPHQWDUQHVWH
~OWLPRFDVRGHILQLUDViUHDVGHVXDDWXDomR ,; 'HIHQVRU3~EOLFR*HUDOGR'LVWULWR)HGHUDO DFUHVFRGS
SHODHPHQGDjOHLRUJkQLFDGRGIQž

E WUDQVIRUPDGD IXQGLGD FLQGLGD LQFRUSRUDGD


SULYDWL]DGD RX H[WLQWD HQWLGDGH GH TXH WUDWD D †ž 3DUDHIHLWRGROLPLWHUHPXQHUDWyULRGHTXHWUDWD
DOtQHDD RLQFLVR;,QmRVHUmRFRPSXWDGDVDVSDUFHODVGH
FDUiWHULQGHQL]DWyULRSUHYLVWDVHPOHL DFUHVFHQWDGRR†
SHOD(PHQGD­/HL2UJkQLFDQžGH
;,; GHSHQGHGHDXWRUL]DomROHJLVODWLYDHPFDGDFDVR
DFULDomRGHVXEVLGLiULDVGDVHQWLGDGHVPHQFLRQD
†ž $SOLFDVHRGLVSRVWRQRLQFLVR;DWRGDVDVHPSUH
GDVQRLQFLVRDQWHULRUDVVLPFRPRDSDUWLFLSDomR
VDVS~EOLFDVHjVVRFLHGDGHVGHHFRQRPLDPLVWD
GHTXDOTXHUGHODVHPHPSUHVDSULYDGD
GLVWULWDLV H VXDV VXEVLGLiULDV UHGDomR GDGD SHOD (PHQGD
&RQVWLWXFLRQDOQž
;; UHVVDOYDGD D OHJLVODomR IHGHUDO DSOLFiYHODR
VHUYLGRU S~EOLFR GR 'LVWULWR )HGHUDO p SURLELGR †ž 'RSHUFHQWXDOGHILQLGRQRLQFLVR 9GHVWHDUWLJR
VXEVWLWXLU VRE TXDOTXHU SUHWH[WR WUDEDOKDGRUHV H[FOXHPVHRVFDUJRVHPFRPLVVmRGRVJDELQHWHV
GHHPSUHVDVSULYDGDVHPJUHYH SDUODPHQWDUHVHOLGHUDQoDVSDUWLGiULDVGD&kPDUD
/HJLVODWLYDGR'LVWULWR)HGHUDO DFUHVFHQWDGRRSHODHPHQGD
;;, WRGRDJHQWHS~EOLFRTXDOTXHUTXHVHMDVXDFDWH jOHLRUJkQFLDQž

JRULDRXDQDWXUH]DGRFDUJRHPSUHJRIXQomRp
REULJDWyULRDGHFODUDUVHXVEHQVQDSRVVHH[RQH †ž 3DUD D SULYDWL]DomR RX H[WLQomRGHHPSUHVD
UDomRRXDSRVHQWDGRULD S~EOLFDRXVRFLHGDGHGHHFRQRPLDPLVWDDTXHVH
UHIHUHRLQFLVR;9,,,GHVWHDUWLJRDOHLHVSHFtILFD
;;,, OHLGLVSRUiVREUHFDUJRVTXHH[LMDPH[DPHSVLFR GHSHQGHUi GH DSURYDomR SRU GRLV WHUoRV GRV
WpFQLFRSDUDLQJUHVVRHDFRPSDQKDPHQWRSVLFROy PHPEURVGD&kPDUD/HJLVODWLYD DFUHVFHQWDGRR†žSHOD
HPHQGDjOHLRUJkQLFDQžGH'2')GH
JLFRSDUDSURJUHVVmRIXQFLRQDO
, $SULYDWL]DomRGHHPSUHVDS~EOLFDRXVRFLHGDGH
;;,,, DRV LQWHJUDQWHV GD FDUUHLUD GH )LVFDOL]DomRH
GH HFRQRPLD PLVWD GH TXH WUDWD R LQFLVR 9;,,,
,QVSHomRpJDUDQWLGDDLQGHSHQGrQFLDIXQFLRQDO
QR H[HUFtFLR GH VXDV DWULEXLo}HV H[LJLGR QtYHO GHVWHDUWLJRFRQGLFLRQDGDjDXWRUL]DomROHJLVODWL
VXSHULRUGHHVFRODULGDGHSDUDLQJUHVVRQDFDUUHL YDQRVWHUPRVGHVWHSDUiJUDIRGHSHQGHGHPDQL
UD ,QFOXtGRRLQFLVR;;,,,SHOD(PHQGDD/HL2UJkQLFDQžGHGHGH]HPEUR IHVWDomRIDYRUiYHOGDSRSXODomRVREDIRUPDGH
GHSXEOLFDGDQR'2')GH  UHIHUHQGR DFUHVFHQWDGRVRVLQFLVRV,H,,DR†žGRDUWSHODHPHQGD
jOHLRUJkQLFDQžGH

†ž eGLUHLWRGRDJHQWHS~EOLFRHQWUHRXWURVRDFHVVR
j SURILVVLRQDOL]DomR H DR WUHLQDPHQWR FRPR ,, D OHL TXH DXWRUL]DU D SULYDWL]DomRPHGLDQWH
HVWtPXORjSURGXWLYLGDGHHjHILFLrQFLD DOLHQDomRGHDo}HVGHHPSUHVDS~EOLFDHVRFLHGD
GHGHHFRQRPLDPLVWDHVWDEHOHFHUiDH[LJrQFLDGH
†ž $OHLHVWDEHOHFHUiDSXQLomRGRVHUYLGRUS~EOLFR FXPSULPHQWRSHORDGTXLUHQWHGHPHWDVGHTXDOL
TXHGHVFXPSULURVSUHFHLWRVHVWDEHOHFLGRVQHVWH GDGH GR VHUYLoR GH DWHQGLPHQWR DRV REMHWLYRV
DUWLJR VRFLDLVLQVSLUDGRUHVGDFRQVWLWXLomRGDHQWLGDGH

†ž 6mRREULJDGRVDID]HUGHFODUDomRS~EOLFDDQXDOGH †ž eSURLELGDDGHVLJQDomRSDUDIXQomRGHFRQILDQoD


VHXVEHQVVHPSUHMXt]RGRGLVSRVWRQR$UWRV RXDQRPHDomRSDUDHPSUHJRRXFDUJRHPFRPLV
VHJXLQWHVDJHQWHVS~EOLFRV VmRLQFOXtGRVRVGHQDWXUH]DHVSHFLDOGHSHVVRD
TXHWHQKDSUDWLFDGRDWRWLSLILFDGRFRPRFDXVDGH
, *RYHUQDGRU LQHOHJLELOLGDGH SUHYLVWD QD OHJLVODomR HOHLWRUDO
DFUHVFHQWDGRSHODHPHQGDjOHLRUJkQLFDQžGH
,, 9LFH*RYHUQDGRU
,,, 6HFUHWiULRVGH*RYHUQR ,QFOXtGRR†ž(PHQGDD/HL2UJkQLFD
QžGHGHPDUoRGHSXEOLFDGDQR'2')GH †ž )LFDYHGDGDDQRPHDomRGHF{QMXJHFRPSDQKHLUR
127$ )LFD VXEVWLWXtGD D H[SUHVVmR ´6HFUHWiULR GH *RYHUQRGR'LVWULWR)HGHUDOµSRU
RXSDUHQWHHPOLQKDUHWDFRODWHUDORXSRUDILQLGD
´6HFUHWiULRGH(VWDGRGR'LVWULWR)HGHUDOµ&21)250((PHQGD­/HL2UJkQLFDQž GHDWpRWHUFHLURJUDXLQFOXVLYHGDDXWRULGDGH
GH
QRPHDQWHRXGHVHUYLGRUGDPHVPDSHVVRDMXUtGL
 'LU&RQVWLWXFLRQDO
 &kPDUD/HJLVODWLYDGR'LVWULWR)HGHUDO
FD LQYHVWLGR HP FDUJR GH GLUHomR FKHILD RX $UW 2VDWRVGDDGPLQLVWUDomRS~EOLFDGHTXDOTXHUGRV
DVVHVVRUDPHQWR SDUD R H[HUFtFLR GH FDUJR HP 3RGHUHVGR'LVWULWR)HGHUDODOpPGHREHGHFHUDRV
FRPLVVmR RX GH FRQILDQoD RX DLQGD GH IXQomR SULQFtSLRVFRQVWLWXFLRQDLVDSOLFDGRVjDGPLQLVWUD
JUDWLILFDGD QD DGPLQLVWUDomR S~EOLFD GLUHWD H omRS~EOLFDGHYHPREVHUYDUWDPEpPRVHJXLQWH
LQGLUHWD HP TXDOTXHU GRV 3RGHUHV GR 'LVWULWR
)HGHUDO FRPSUHHQGLGR QD YHGDomR R DMXVWH , RVDWRVDGPLQLVWUDWLYRVVmRS~EOLFRVVDOYRTXDQGR
PHGLDQWHGHVLJQDo}HVUHFtSURFDV DFUHVFHQWDGRVRV††žH D OHL QR LQWHUHVVH GD DGPLQLVWUDomR LPSXVHU
DR$UWSHODHPHQGDjOHLRUJkQLFDQžGH'2')GH
VLJLOR 1RWD  YLGH /HL Qž  GH   '2')  TXH
UHJXODPHQWDHVWHLQFLVRLGR$UW
† $YHGDomRGHTXHWUDWDR†žQmRVHDSOLFDDRV
RFXSDQWHV GH FDUJR HIHWLYR GD FDUUHLUD HP FXMD ,, DDGPLQLVWUDomRpREULJDGDDIRUQHFHUFHUWLGmRRX
HVWUXWXUDHVWHMDRFDUJRHPFRPLVVmRRXDIXQomR FySLDDXWHQWLFDGDGHDWRVFRQWUDWRVHFRQYrQLRV
JUDWLILFDGDRFXSDGD DGPLQLVWUDWLYRVDTXDOTXHULQWHUHVVDGRQRSUD]R
† $DSXUDomRGRSHUFHQWXDOGHTXHWUDWDRLQFLVR9 Pi[LPRGHWULQWDGLDVVRESHQDGHUHVSRQVDELOL
p IHLWD HP UHODomR DR VRPDWyULR GRV FDUJRV HP GDGHGHDXWRULGDGHFRPSHWHQWHRXVHUYLGRUTXH
FRPLVVmR SURYLGRV QD DGPLQLVWUDomR GLUHWD QHJDURXUHWDUGDUDH[SHGLomR1RWDYLGH/HLQžGH
'2')TXHUHJXODPHQWDHVWHLQFLVRLLGR$UW
DXWiUTXLFDHIXQGDFLRQDOGHFDGD3RGHU
,,, pJDUDQWLGDDJUDWXLGDGHGDH[SHGLomRGDSULPHL
† $OHLGHYHGLVSRUVREUHRVUHTXLVLWRVHDVUHVWULo}HV UDYLDGDFpGXODGHLGHQWLGDGHSHVVRDO 1RYD5HGDomR
DRRFXSDQWHGHFDUJRRXHPSUHJRGDDGPLQLVWUD (PHQGDD/HL2UJkQLFDQžGHGHVHWHPEURGH

omR GLUHWD H LQGLUHWD TXH SRVVLELOLWH R DFHVVR D


LQIRUPDo}HVSULYLOHJLDGDV ,9 QRSURFHVVRDGPLQLVWUDWLYRTXDOTXHUTXHVHMDR
REMHWR RX SURFHGLPHQWR REVHUYDUVHmR HQWUH
† $DXWRQRPLDJHUHQFLDORUoDPHQWiULDHILQDQFHLUD RXWURVUHTXLVLWRVGHYDOLGDGHRFRQWUDGLWyULRD
GRVyUJmRVHHQWLGDGHVGDDGPLQLVWUDomRS~EOLFD DPSODGHIHVDHRGHVSDFKRRXGHFLVmRPRWLYDGRV
SRGH VHU DPSOLDGD PHGLDQWH FRQWUDWR D VHU
ILUPDGR HQWUH VHXV DGPLQLVWUDGRUHV H R 3RGHU 9 DSXEOLFLGDGHGRVDWRVSURJUDPDVREUDVVHUYLoRV
3~EOLFRTXHWHQKDSRUREMHWRDIL[DomRGHPHWDV HDVFDPSDQKDVGRVyUJmRVHHQWLGDGHVGDDGPL
GH GHVHPSHQKR SDUD R yUJmR RX D HQWLGDGH QLVWUDomRS~EOLFDDLQGDTXHQmRFXVWHDGDGLUHWD
FDEHQGRjOHLGLVSRUVREUH PHQWHSHORHUiULRREHGHFHUiDRVHJXLQWH

, SUD]RGHGXUDomRGRFRQWUDWR  WHUFDUiWHUHGXFDWLYRLQIRUPDWLYRRXGHRULHQWD


,, FRQWUROHVHFULWpULRVGHDYDOLDomRGHGHVHPSHQKR omR VRFLDO GHOD QmR SRGHQGR FRQVWDU VtPERORV
GLUHLWRVREULJDo}HVHUHVSRQVDELOLGDGHGRVGLUL H[SUHVV}HVQRPHVRXLPDJHQVTXHFDUDFWHUL]HP
JHQWHV SURPRomRSHVVRDOGH DXWRULGDGHVRXVHUYLGRUHV
,,, UHPXQHUDomRGRSHVVRDO S~EOLFRV

† eYHGDGDDSHUFHSomRVLPXOWkQHDGHSURYHQWRVGH  VHUVXVSHQVDQRYHQWDGLDVDQWHVGDVHOHLo}HVUHV


DSRVHQWDGRULDGHFRUUHQWHVGRDUWRXGRVDUWV VDOYDGDVDTXHODVHVVHQFLDLVDRLQWHUHVVHS~EOLFR
HGD&RQVWLWXLomR)HGHUDOFRPDUHPXQH
UDomRRXVXEVtGLRGHFDUJRHPSUHJRRXIXQomR †ž 2V3RGHUHVGR'LVWULWR)HGHUDOFRPEDVHQRSODQR
S~EOLFD UHVVDOYDGRV RV FDUJRV DFXPXOiYHLV QD DQXDOGHSXEOLFLGDGHILFDPREULJDGRVDSXEOLFDU
IRUPDGHVWD/HL2UJkQLFDRVFDUJRVHOHWLYRVHRV QRVVHXVyUJmRVRILFLDLVTXDGURVGHPRQVWUDWLYRV
FDUJRVHPFRPLVVmRGHFODUDGRVHPOHLGHOLYUH GHGHVSHVDVUHDOL]DGDVFRPSXEOLFLGDGHHSURSD
QRPHDomRHH[RQHUDomR JDQGDFRQIRUPHGLVSXVHUDOHL

$UW$V SHVVRDV MXUtGLFDV GH GLUHLWR S~EOLFR H DVGH †ž 2V3RGHUHVGR'LVWULWR)HGHUDOPDQGDUmRSXEOL


GLUHLWRSULYDGRSUHVWDGRUDVGHVHUYLoRVS~EOLFRV FDUWULPHVWUDOPHQWHQR'LiULR2ILFLDOGHPRQVWUD
UHVSRQGHUmRSHORVGDQRVTXHVHXVDJHQWHVQHVWD WLYR GDV GHVSHVDV UHDOL]DGDV FRP SURSDJDQGD H
TXDOLGDGH FDXVDUHP D WHUFHLURV DVVHJXUDGR R SXEOLFLGDGHGHWRGRVRVVHXVyUJmRVLQFOXVLYHRV
GLUHLWRGHUHJUHVVRFRQWUDRUHVSRQViYHOQRVFDVRV GD DGPLQLVWUDomR LQGLUHWD HPSUHVDV S~EOLFDV
GHGRORRXFXOSD VRFLHGDGHVGHHFRQRPLDPLVWDHIXQGDo}HVPDQWL
GDVSHOR3RGHU3~EOLFRFRPDGLVFULPLQDomRGR
$UWe YHGDGR GLVFULPLQDU RX SUHMXGLFDUTXDOTXHU EHQHILFLiULRYDORUHILQDOLGDGHFRQIRUPHGLVSXVHU
SHVVRDSHORIDWRGHKDYHUOLWLJDGRRXHVWDUOLWLJDQ DOHL
GRFRQWUDRVyUJmRVS~EOLFRVGR'LVWULWR)HGHUDO
QDVHVIHUDVDGPLQLVWUDWLYDRXMXGLFLDO †ž 2V3RGHUHVGR'LVWULWR)HGHUDOPDQGDUmRSXEOL
FDUPHQVDOPHQWHQRVUHVSHFWLYRVVtWLRVRILFLDLV
†~QLFR $VSHVVRDVItVLFDVRXMXUtGLFDVTXHVHFRQVLGHUD QDLQWHUQHWGHPRQVWUDWLYRGHWRGDVDVGHVSHVDV
UHP SUHMXGLFDGDV SRGHUmR UHTXHUHU UHYLVmR GRV UHDOL]DGDV SRU WRGRV RV VHXV yUJmRV GH IRUPD
DWRVTXHGHUHPFDXVDDHYHQWXDLVSUHMXt]RV FODUDHFRPSUHHQVtYHODRFLGDGmRLQFOXVLYHRVGD
DGPLQLVWUDomRLQGLUHWDHPSUHVDVS~EOLFDVVRFLH
&kPDUD/HJLVODWLYDGR
 'LVWULWR)HGHUDO 'LU&RQVWLWXFLRQDO 

GDGHVGHHFRQRPLDPLVWDHIXQGDo}HVPDQWLGDV $UW2VDWRVGHLPSURELGDGHDGPLQLVWUDWLYDLPSRUWD
SHOR 3RGHU 3~EOLFR FRP D GLVFULPLQDomR GR UmR VXVSHQVmR GRV GLUHLWRV SROtWLFRV SHUGD GD
EHQHILFLiULRGRYDORUHGDILQDOLGDGHFRQIRUPH IXQomR S~EOLFD LQGLVSRQLELOLGDGH GRV EHQV H
GLVSXVHUDOHL ILFD DFUHVFHQWDGR R † ž DR $UW SHOD HPHQGD j OHL UHVVDUFLPHQWR DR HUiULR QD IRUPD H JUDGDomR
RUJkQLFDQžGH'2')GH
SUHYLVWDV HP OHL VHP SUHMXt]R GD DomR SHQDO
FDEtYHO
†ž $OHLGHYHGLVFLSOLQDUDVIRUPDVGHSDUWLFLSDomRGR
XVXiULRQDDGPLQLVWUDomRS~EOLFDGLUHWDHLQGLUH $UW eYHGDGDDFRQWUDWDomRGHREUDVHVHUYLoRVS~EOL
WDUHJXODQGRHVSHFLDOPHQWH DFUHVFHQWDGRSHODHPHQGDjOHL FRVVHPSUpYLDDSURYDomRGRUHVSHFWLYRSURMHWR
RUJkQLFDGRGIQžGH
VRESHQDGHQXOLGDGHGRDWRGHFRQWUDWDomR
, DVUHFODPDo}HVUHODWLYDVjSUHVWDomRGRVVHUYLoRV
$UW 5HYRJDGRSHODHPHQGDjOHLRUJkQLFDGRGIQž
S~EOLFRVHPJHUDODVVHJXUDGDDPDQXWHQomRGH
VHUYLoRVGHDWHQGLPHQWRDRXVXiULRHDDYDOLDomR
$UW/HLGLVSRUiVREUHSDUWLFLSDomRSRSXODUQDILVFDOL
SHULyGLFD H[WHUQD H LQWHUQD GD TXDOLGDGH GRV
]DomR GD SUHVWDomR GRV VHUYLoRV S~EOLFRV GR
VHUYLoRV
'LVWULWR)HGHUDO
,, RDFHVVRGRVXVXiULRVDUHJLVWURVDGPLQLVWUDWLYRV
6HomR,,,
HLQIRUPDo}HVVREUHDWRVGHJRYHUQRREVHUYDGR 'D$GPLQLVWUDomR7ULEXWiULD
RGLVSRVWRQRDUWž;H;;;,,,GD&RQVWLWXLomR
)HGHUDO $UW ­DGPLQLVWUDomRWULEXWiULDLQFXPEHDVIXQo}HVGH
ODQoDPHQWRILVFDOL]DomRHDUUHFDGDomRGRVWULEX
,,, DUHSUHVHQWDomRFRQWUDRH[HUFtFLRQHJOLJHQWHRX WRVGHFRPSHWrQFLDGR'LVWULWR)HGHUDOHRMXOJD
DEXVLYRGHFDUJRHPSUHJRRXIXQomRQDDGPLQLV PHQWR DGPLQLVWUDWLYR GRV SURFHVVRV ILVFDLV RV
WUDomRS~EOLFD TXDLVVHUmRH[HUFLGRVSULYDWLYDPHQWHSRULQWH
JUDQWHVGDFDUUHLUDGHDXGLWRULDWULEXWiULD
$UW$DGPLQLVWUDomRS~EOLFDpREULJDGDD
†ž 2 MXOJDPHQWR GH SURFHVVRV ILVFDLVHPVHJXQGD
, DWHQGHUDUHTXLVLo}HVMXGLFLDLVQRVSUD]RVIL[DGRV LQVWkQFLDVHUiGHFRPSHWrQFLDGHyUJmRFROHJLDGR
SHODDXWRULGDGHMXGLFLiULD LQWHJUDGRSRUVHUYLGRUHVGDFDUUHLUDGHDXGLWRULD
WULEXWiULDHUHSUHVHQWDQWHVGRVFRQWULEXLQWHV
,, IRUQHFHUDTXDOTXHUFLGDGmRQRSUD]RPi[LPRGH )LFD 5HQXPHUDGRR†~QLFRSDUD†žHDFUHVFHQWDGRR†ž3HODHPHQGD D OHL
RUJkQLFDQžGH'2')
GH]GLDV~WHLVLQGHSHQGHQWHPHQWHGHSDJDPHQWR
GH WD[DV RX HPROXPHQWRV FHUWLGmR GH DWRV
†ž ([FHWXDPVHGDFRPSHWrQFLDSULYDWLYDUHIHULGDQR
FRQWUDWRVGHFLV}HVRXSDUHFHUHVSDUDGHIHVDGH
FDSXWRODQoDPHQWRDILVFDOL]DomRHDDUUHFDGD
VHXV GLUHLWRV H HVFODUHFLPHQWR GH VLWXDo}HV GH
omRGDVWD[DVTXHWHQKDPFRPRIDWR JHUDGRUR
LQWHUHVVHSHVVRDORXFROHWLYR
H[HUFtFLRGRSRGHUGHSROtFLDEHPFRPRRMXOJD
PHQWRGHSURFHVVRVDGPLQLVWUDWLYRVGHFRUUHQWHV
†~QLFR $DXWRULGDGHRXVHUYLGRUTXHQHJDURXUHWDUGDUR GHVVDVIXQo}HVQDIRUPDGD/HL
GLVSRVWRQHVWHDUWLJRLQFRUUHUiHPSHQDGHUHV
SRQVDELOLGDGHH[FHWXDGRVRVFDVRVGHFRPSURYD †ž $DGPLQLVWUDomRWULEXWiULDDWLYLGDGHHVVHQFLDODR
GDLPSRVVLELOLGDGH IXQFLRQDPHQWRGR'LVWULWR)HGHUDOH[HUFLGDSRU
VHUYLGRUHV GD FDUUHLUD DXGLWRULD WULEXWiULD WHP
$UW$GLUHomRVXSHULRUGDVHPSUHVDVS~EOLFDVDXWDU UHFXUVRVSULRULWiULRVSDUDDUHDOL]DomRGHVXDVDWL
TXLDVIXQGDo}HVHVRFLHGDGHVGHHFRQRPLDPLVWD YLGDGHV H DWXD GH IRUPD LQWHJUDGD FRP DV DG
WHUiUHSUHVHQWDQWHVGRVVHUYLGRUHVHVFROKLGRVGR PLQLVWUDo}HVWULEXWiULDVGD8QLmRHVWDGRVHPX
TXDGURIXQFLRQDOSDUDH[HUFHUIXQo}HVGHILQLGDV QLFtSLRVLQFOXVLYHFRPRFRPSDUWLOKDPHQWRGHFD
QDIRUPDGDOHL GDVWURVHGHLQIRUPDo}HVILVFDLVQDIRUPDGDOHL
RXGHFRQYrQLR DFUHVFHQWDGRSHODHPHQGDj/2GRGIQž
6HomR,,
'RV6HUYLoRV3~EOLFRV $UW /HLHVSHFtILFDGLVFLSOLQDUiDRUJDQL]DomRHIXQFLR
QDPHQWRGDDGPLQLVWUDomRWULEXWiULDEHPFRPR
$UW 2VVHUYLoRVS~EOLFRVFRQVWLWXHPGHYHUGR'LVWULWR WUDWDUiGDRUJDQL]DomRHHVWUXWXUDomRGDFDUUHLUD
)HGHUDO H VHUmR SUHVWDGRV VHP GLVWLQomR GH HVSHFtILFDGHDXGLWRULDWULEXWiULD
TXDOTXHU QDWXUH]D HP FRQIRUPLGDGH FRP R
HVWDEHOHFLGR QD &RQVWLWXLomR )HGHUDO QHVWD /HL &$3Ì78/29,
2UJkQLFDHQDVOHLVHUHJXODPHQWRVTXHRUJDQL]HP '266(59,'25(63Ô%/,&26
VXDSUHVWDomR
$UW 2'LVWULWR)HGHUDOLQVWLWXLUiUHJLPHMXUtGLFR~QLFR
$UW 2EVHUYDGDDOHJLVODomRIHGHUDODVREUDVFRPSUDV HSODQRVGHFDUUHLUDSDUDRVVHUYLGRUHVGDDGPL
DOLHQDo}HV H VHUYLoRV GD DGPLQLVWUDomR VHUmR QLVWUDomRS~EOLFDGLUHWDDXWiUTXLFDHIXQGDo}HV
FRQWUDWDGRVPHGLDQWHSURFHVVRGHOLFLWDomRS~EOL S~EOLFDV QRV WHUPRV GR $UW GD &RQVWLWXLomR
FDQRVWHUPRVGDOHL )HGHUDO
 'LU&RQVWLWXFLRQDO
 &kPDUD/HJLVODWLYDGR'LVWULWR)HGHUDO
†ž 1R H[HUFtFLR GD FRPSHWrQFLDHVWDEHOHFLGDQR $UW$OHLDVVHJXUDUiDRVVHUYLGRUHVGDDGPLQLVWUDomR
FDSXWVHUmRRXYLGDVDVHQWLGDGHVUHSUHVHQWDWLYDV GLUHWD LVRQRPLD GH YHQFLPHQWRV SDUD FDUJRV GH
GRVVHUYLGRUHVS~EOLFRVSRUHODDEUDQJLGRV DWULEXLo}HV LJXDLV RX DVVHPHOKDGDV GR PHVPR
3RGHURXHQWUHVHUYLGRUHVGRV3RGHUHV([HFXWLYR
†ž $VHQWLGDGHVLQWHJUDQWHVGDDGPLQLVWUDomRS~EOLFD H/HJLVODWLYRUHVVDOYDGDVDVYDQWDJHQVGHFDUiWHU
LQGLUHWD QmR PHQFLRQDGDV QR FDSXW LQVWLWXLUmR LQGLYLGXDOHDVUHODWLYDVGDQDWXUH]DRXORFDOGH
SODQRVGHFDUUHLUDSDUDRVVHXVVHUYLGRUHVREVHU WUDEDOKR
YDGRRGLVSRVWRQRSDUiJUDIRDQWHULRU
$UW6mRGLUHLWRVGRVVHUYLGRUHVS~EOLFRVVXMHLWRVDR
†ž $IL[DomRGRVSDGU}HVGHYHQFLPHQWRHGRVGHPD UHJLPHMXUtGLFR~QLFRDOpPGRVDVVHJXUDGRVQR†
LV FRPSRQHQWHV GR VLVWHPD UHPXQHUDWyULR GHYH žGR$UWGD&RQVWLWXLomR)HGHUDORVVHJXLQ
REVHUYDU DFUHVFHQWDGRSHODHPHQGDjOHLRUJkQLFDGRGIQž WHV

, JUDWLILFDomRGRWLWXODUTXDQGRHPVXEVWLWXLomRRX
, DQDWXUH]DRJUDXGHUHVSRQVDELOLGDGHDVSHFXOL
GHVLJQDGRSDUDUHVSRQGHUSHORH[SHGLHQWH
DULGDGHVHDFRPSOH[LGDGHGRVFDUJRVFRPSRQHQ
WHVGHFDGDFDUUHLUD
,, GXUDomRGRWUDEDOKRQRUPDOQmRVXSHULRUDRLWR
KRUDVGLiULDVHTXDUHQWDKRUDVVHPDQDLVIDFXOWD
,, RVUHTXLVLWRVSDUDDLQYHVWLGXUD GRDR3RGHU3~EOLFRFRQFHGHUDFRPSHQVDomRGH
KRUiULRVHDUHGXomRGDMRUQDGDQRVWHUPRVGD
†ž 2'LVWULWR)HGHUDOGHYHPDQWHUHVFRODGHJRYHUQR OHL
SDUDIRUPDomRHDSHUIHLoRDPHQWRGRVVHUYLGRUHV
S~EOLFRVFRQVWLWXLQGRVHDSDUWLFLSDomRQRVFXUVRV ,,, SURWHomRHVSHFLDOjVHUYLGRUDJHVWDQWHRXODFWDQWH
XP GRV UHTXLVLWRV SDUD SURPRomR QD FDUUHLUD LQFOXVLYH PHGLDQWH D DGHTXDomR RX PXGDQoD
IDFXOWDGDSDUDLVVRDFHOHEUDomRGHFRQYrQLRVRX WHPSRUiULDGHVXDVIXQo}HVTXDQGRIRUUHFRPHQ
FRQWUDWRVFRPRVGHPDLVHQWHVIHGHUDGRVRXVXDV GiYHODVXDVD~GHRXjGRQDVFLWXURVHPSUHMXt]R
HQWLGDGHV DFUHVFHQWDGRSHODHPHQGDjOHLRUJkQLFDGRGIQž GHVHXVYHQFLPHQWRVHGHPDLVYDQWDJHQV

†ž 2 PHPEUR GH 3RGHU R GHWHQWRUGHPDQGDWR ,9 DWHQGLPHQWRHPFUHFKHHSUpHVFRODDVHXVGHSHQ


HOHWLYRRV6HFUHWiULRVGH(VWDGRRVDGPLQLVWUDGR GHQWHVQRVWHUPRVGDOHL QRYDUHGDomRGDGDSHODHPHQGDjOHL
RUJkQLFDGRGIQž
UHV UHJLRQDLV H RV GHPDLV FDVRV SUHYLVWRV QD
&RQVWLWXLomR)HGHUDOVmRUHPXQHUDGRVH[FOXVLYD
9 YHGDomR GR GHVYLR GH IXQomR UHVVDOYDGDVHP
PHQWH SRU VXEVtGLR IL[DGR HP SDUFHOD ~QLFD
SUHMXt]RGHVHXVYHQFLPHQWRVVDOiULRVHGHPDLV
YHGDGR R DFUpVFLPR GH TXDOTXHU JUDWLILFDomR
YDQWDJHQVGRFDUJRHPSUHJRRXIXQomR
DGLFLRQDODERQRSUrPLRYHUEDGHUHSUHVHQWDomR
RX RXWUD HVSpFLH UHPXQHUDWyULD REHGHFLGR HP D DPXGDQoDGHIXQomRFRQFHGLGDDVHUYLGRUD
TXDOTXHU FDVR R GLVSRVWR QR DUW  ,; H ; JHVWDQWHVREUHFRPHQGDomRPpGLFD
DFUHVFHQWDGRSHODHPHQGDjOHLRUJkQLFDGRGIQž

†ž $ UHPXQHUDomR GRV VHUYLGRUHVS~EOLFRVRUJD E DWUDQVIHUrQFLDFRQFHGLGDTXHWLYHUVXDFDSDFL


QL]DGRVHPFDUUHLUDSRGHVHUIL[DGDQRVWHUPRVGR GDGHGHWUDEDOKRUHGX]LGDHPGHFRUUrQFLDGH
DFLGHQWHRXGRHQoDGHWUDEDOKRSDUDORFDLVRX
†ž DFUHVFHQWDGRSHODHPHQGDjOHLRUJkQLFDGRGIQž
DWLYLGDGHVFRPSDWtYHLVFRPVXDVLWXDomR
†ž /HL FRPSOHPHQWDU SRGH HVWDEHOHFHUDUHODomR
9, UHFHELPHQWRGHYDOHWUDQVSRUWHQRVFDVRVSUHYLV
HQWUHDPDLRUHDPHQRUUHPXQHUDomRGRVVHUYL
WRVHPOHL
GRUHVS~EOLFRVREHGHFLGRHPTXDOTXHUFDVRR
GLVSRVWRQRDUW; DFUHVFHQWDGRSHODHPHQGDjOHLRUJkQLFDGR 9,, SDUWLFLSDomRQDHODERUDomRHDOWHUDomRGRVSODQRV
GIQž
GHFDUUHLUD
†ž 2V3RGHUHV([HFXWLYRH/HJLVODWLYRGHYHPSXEOL
FDUDWpGHMDQHLURGHFDGDDQRRVYDORUHVGR 9,,,SURPRo}HVSRUPHUHFLPHQWRRXDQWLJLGDGHQR
VXEVtGLRHGDUHPXQHUDomRGRVFDUJRVHHPSUHJRV VHUYLoRS~EOLFRQRVWHUPRVGDOHL
S~EOLFRV DFUHVFHQWDGRSHODHPHQGDjOHLRUJkQLFDGRGIQž
,; TXLWDomRGDIROKDGHSDJDPHQWRGRVHUYLGRUDWLYR
H LQDWLYR GD DGPLQLVWUDomR GLUHWD LQGLUHWD H
†ž $ OHL GHYH GLVFLSOLQDU D DSOLFDomRGHUHFXUVRV
IXQGDFLRQDOGR'LVWULWR)HGHUDODWpRTXLQWRGLD
RUoDPHQWiULRV SURYHQLHQWHV GD HFRQRPLD FRP
~WLOGRPrVVXEVHTHQWHVRESHQDGHLQFLGrQFLD
GHVSHVDV FRUUHQWHV HP FDGD yUJmR DXWDUTXLD H
GHDWXDOL]DomRPRQHWiULDREHGHFLGRRGLVSRVWR
IXQGDomRSDUDDSOLFDomRQRGHVHQYROYLPHQWRGH HPOHL
SURJUDPDVGHTXDOLGDGHHSURGXWLYLGDGHWUHLQD
PHQWRHGHVHQYROYLPHQWRPRGHUQL]DomRUHDSD †ž 3DUD D DWXDOL]DomR D TXH VH UHIHUH RLQFLVR,;
UHOKDPHQWRHUDFLRQDOL]DomRGRVHUYLoRS~EOLFR XWLOL]DUVHmRRVtQGLFHVRILFLDLVHDLPSRUWkQFLD
LQFOXVLYHVREDIRUPDGHDGLFLRQDORXSUrPLRGH DSXUDGDVHUiSDJDMXQWDPHQWHFRPDUHPXQHUD
SURGXWLYLGDGH DFUHVFHQWDGRSHODHPHQGDjOHLRUJkQLFDGRGIQž omRGRPrVVXEVHTHQWH
&kPDUD/HJLVODWLYDGR
 'LVWULWR)HGHUDO 'LU&RQVWLWXFLRQDO 

†ž eFRPSXWDGRFRPRH[HUFtFLRHIHWLYRSDUDHIHLWR †ž 2WHPSRGHFRQWULEXLomRSUHVWDGRVRERUHJLPH


GH SURJUHVVmR IXQFLRQDO RX FRQFHVVmR GH OLFHQ GHDSRVHQWDGRULDHVSHFLDOpFRPSXWDGRGDPHVPD
oDSUrPLRHDSRVHQWDGRULDQDVFDUUHLUDVHVSHFtIL IRUPDTXDQGRRVHUYLGRURFXSDURXWURFDUJRGH
FDVGRVHUYLoRS~EOLFRRVWHPSRGHVHUYLoRSUHVWD UHJLPHLGrQWLFRRXSHORFULWpULRGDSURSRUFLRQDOL
GRSRUVHUYLGRUUHTXLVLWDGRDTXDOTXHUGRV3RGH GDGH TXDQGR VH WUDWDU GH UHJLPHV GLYHUVRV QD
UHVGR'LVWULWR)HGHUDO IRUPDGDOHL

$UWeJDUDQWLGRDRVHUYLGRUS~EOLFRRGLUHLWRjOLYUH $UWeDVVHJXUDGDDSDUWLFLSDomRGHVHUYLGRUHVS~EOL
DVVRFLDomR VLQGLFDO REVHUYDGR R GLVSRVWR QR FRV QD JHUrQFLD GH IXQGRV H HQWLGDGHV SDUD RV
$UWžGD&RQVWLWXLomR)HGHUDO
TXDLVFRQWULEXLQDIRUPDGDOHL
†~QLFR $OHLGLVSRUiVREUHOLFHQoDVLQGLFDOSDUDRVGLUL
$UW 6HUiFRQFHGLGDOLFHQoDSDUDDWHQGLPHQWRGHILOKR
JHQWHV GH IHGHUDo}HV H VLQGLFDWRV GH VHUYLGRUHV
S~EOLFRV GXUDQWH R H[HUFtFLR GR PDQGDWR UHV JHQLWRUHF{QMXJHGRHQWHDKRPHPRXPXOKHU
JXDUGDGRV RV GLUHLWRV H YDQWDJHQV LQHUHQWHV j PHGLDQWHFRPSURYDomRSRUDWHVWDGRPpGLFRGD
FDUUHLUDGHFDGDXP UHGHRILFLDOGHVD~GHGR'LVWULWR)HGHUDO

$UW­V HQWLGDGHV UHSUHVHQWDWLYDV GRVVHUYLGRUHV †~QLFR eDVVHJXUDGRDRVHUYLGRUS~EOLFRTXHWHQKDF{QMX


S~EOLFRV GR 'LVWULWR )HGHUDO FDEH D GHIHVD GRV JHRXGHSHQGHQWHFRPGHILFLrQFLDKRUiULRHVSHFL
GLUHLWRV H LQWHUHVVHV FROHWLYRV RX LQGLYLGXDLV GD DOGHVHUYLoRLQGHSHQGHQWHPHQWHGDFRPSHQVD
FDWHJRULD LQFOXVLYH HP TXHVW}HV MXGLFLDLV RX omR GH KRUiULR REHGHFLGR R GLVSRVWR HP OHL
DGPLQLVWUDWLYDVREVHUYDGRRGLVSRVWRQR$UWž DFUHVFHQWDGRSHODHPHQGDjOHLRUJkQLFDGRGIQž

GD&RQVWLWXLomR)HGHUDO
$UW$R VHUYLGRU S~EOLFR GD DGPLQLVWUDomRGLUHWD
$UW ­VHQWLGDGHVGHFDUiWHUVLQGLFDOTXHSUHHQFKDPRV DXWiUTXLFDHIXQGDFLRQDOGR'LVWULWR)HGHUDOILFD
UHTXLVLWRV HVWDEHOHFLGRV HP OHL p DVVHJXUDGR R DVVHJXUDGR
GHVFRQWRHPIROKDGHSDJDPHQWRGDVFRQWULEXL
o}HV GRV DVVRFLDGRV DSURYDGDV HP DVVHPEOpLD , SHUFHELPHQWRGHDGLFLRQDOGHXPSRUFHQWRSRU
JHUDO DQRGHVHUYLoRS~EOLFRHIHWLYRQRVWHUPRVGDOHL

$UW2GLUHLWRGH JUHYH p H[HUFLGRQRVWHUPRVHQRV ,, FRQWDJHPSDUDWRGRVRVHIHLWRVOHJDLVGRSHUtRGR


OLPLWHVGHILQLGRVHPOHLFRPSOHPHQWDU QRYDUHGDomR HP TXH R VHUYLGRU HVWLYHU GH OLFHQoD FRQFHGLGD
GDGDSHODHPHQGDjOHLRUJkQLFDGRGIQž
SRUMXQWDPpGLFDRILFLDO
$UW 6mRHVWiYHLVDSyVWUrVDQRVGHHIHWLYRH[HUFtFLRRV
VHUYLGRUHVQRPHDGRVSDUDFDUJRGHSURYLPHQWR ,,, FRQWDJHPUHFtSURFDSDUDHIHLWRGHDSRVHQWDGRULD
HIHWLYRHPYLUWXGHGHFRQFXUVRS~EOLFR QRYDUHGDomR GR WHPSR GH FRQWULEXLomR QD DGPLQLVWUDomR
GDGDSHODHPHQGDjOHLRUJkQLFDGRGIQž S~EOLFDHQDDWLYLGDGHSULYDGDUXUDOHXUEDQDQD
IRUPDSUHYLVWDQRDUW†žGD&RQVWLWXLomR
†ž 2VHUYLGRUS~EOLFRHVWiYHOVySHUGHUiRFDUJRHP )HGHUDO QRYDUHGDomRGDGDSHODHPHQGDjOHLRUJkQLFDGRGIQž
YLUWXGHGHVHQWHQoDMXGLFLDOWUDQVLWDGDHPMXOJDGR
RXPHGLDQWHSURFHVVRDGPLQLVWUDWLYRHPTXHOKH †~QLFR )LFDP DVVHJXUDGRV RV EHQHItFLRVFRQVWDQWHVGR
VHMDDVVHJXUDGDDPSODGHIHVD
DUW,,,,9H9HGRDUWGHVWD/HL2UJkQLFD
DRVVHUYLGRUHVGDVHPSUHVDVS~EOLFDVHGDVVRFLH
†ž ,QYDOLGDGD SRU VHQWHQoD MXGLFLDO DGHPLVVmRGR
GDGHVGHHFRQRPLDPLVWDGR'LVWULWR)HGHUDO QRYD
VHUYLGRUHVWiYHOVHUiHOHUHLQWHJUDGRFRPWRGRV UHGDomRGDGDSHODHPHQGDjOHLRUJkQLFDGRGIQž
RVGLUHLWRVHYDQWDJHQVGHYLGRVGHVGHDGHPLVVmR
HRHYHQWXDORFXSDQWHGDYDJDVHUiUHFRQGX]LGR &$3Ì78/29,,
DR FDUJR GH RULJHP VHP GLUHLWR D LQGHQL]DomR '266(59,'25(63Ô%/,&260,/,7$5(6
DSURYHLWDGRHPRXWURFDUJRRXSRVWRHPGLVSRQL
$UW 5HYRJDGRSHODHPHQGDjOHLRUJkQLFDGRGIQž
ELOLGDGHUHPXQHUDGD

†ž ([WLQWRRFDUJRRXGHFODUDGDVXDGHVQHFHVVLGDGH &$3Ì78/29,,,


R VHUYLGRU HVWiYHO ILFDUi HP GLVSRQLELOLGDGH '26%(16'2',675,72)('(5$/
UHPXQHUDGDDWpVHXDGHTXDGRDSURYHLWDPHQWRHP
RXWURFDUJR $UW6mREHQVGR'LVWULWR)HGHUDO

$UW $RVHUYLGRUS~EOLFRHIHWLYRQRVWHUPRVGD&RQVWL , RVTXHDWXDOPHQWHOKHSHUWHFHPTXHYLHUDDGTXL


WXLomR)HGHUDOpDVVHJXUDGRUHJLPHSUySULRGH ULURXIRUHPDWULEXtGRV
SUHYLGrQFLDVRFLDO QRYDUHGDomRGDGDSHODHPHQGDjOHLRUJkQLFDGR
GIQž
,, DV iJXDV VXSHUILFLDLV RX VXEWHUUkQHDVIOXHQWHV
†ž 2UHJLPHSUySULRGHSUHYLGrQFLDVRFLDOREVHUYD HPHUJHQWHV H HP GHSyVLWR UHVVDOYDGDV QHVWH
GRVRVFULWpULRVTXHSUHVHUYHPRHTXLOtEULRILQDQ FDVRQDIRUPDGDOHLDVGHFRUUHQWHVGHREUDVGD
FHLURHDWXDULDOpLQVWLWXtGRSRUOHLFRPSOHPHQWDU 8QLmR
 'LU&RQVWLWXFLRQDO
 &kPDUD/HJLVODWLYDGR'LVWULWR)HGHUDO
,,, DUHGHYLiULDGR'LVWULWR)HGHUDOVXDLQIUDHVWUX 7Ì78/2,,,
WXUDHEHQVDFHVVyULRV '$25*$1,=$d®2'2632'(5(6

$UW 2VEHQVGR'LVWULWR)HGHUDOGHFODUDGRVLQVHUYtYHLV &$3Ì78/2,


HP SURFHVVR UHJXODU SRGHUmR VHU DOLHQDGRV '$6',6326,d¯(6*(5$,6
PHGLDQWHOLFLWDomRFDEHQGRGRDomRVRPHQWHQRV
FDVRVTXHOHLHVSHFLILFDU $UW 6mR3RGHUHVGR'LVWULWR)HGHUDOLQGHSHQGHQWHVH
KDUP{QLFRVHQWUHVLR([HFXWLYRHR/HJLVODWLYR
†ž 2VEHQVLPyYHLVGR'LVWULWR)HGHUDOVySRGHPVHU
REMHWR GH DOLHQDomR DIRUDPHQWR FRPRGDWR RX †ž e YHGDGD D GHOHJDomR GH DWULEXLo}HVHQWUHRV
FHVVmRGHXVRPHGLDQWHDXWRUL]DomROHJLVODWLYD 3RGHUHV
QRYD UHGDomR GDGD DR † ž GR $UW SHOD HPHQGD j OHL RUJkQLFD Qž  GH
'2')GH
†ž 2FLGDGmRLQYHVWLGRQDIXQomRGHXPGRV3RGH
†ž 7RGRV RV EHQV GR 'LVWULWR )HGHUDOGHYHUmRVHU UHV QmR SRGHUi H[HUFHU D GH RXWUR VDOYR DV
FDGDVWUDGRVFRPDLGHQWLILFDomRUHVSHFWLYD H[FHo}HVSUHYLVWDVQHVWD/HL2UJkQLFD

$UW2XVRGHEHQVGR 'LVWULWR)HGHUDOSRUWHUFHLURV &$3Ì78/2,,


SRGHUi VHU IHLWR PHGLDQWH FRQFHVVmR DGPLQLV '232'(5/(*,6/$7,92
WUDWLYD GH XVR SHUPLVVmR RX DXWRUL]DomR FRQ
IRUPHRFDVRHRLQWHUHVVHS~EOLFRQDIRUPDGD 6HomR,
OHL 'D&kPDUD/HJLVODWLYD

$UW $DTXLVLomRSRUFRPSUDRXSHUPXWDEHPFRPRD $UW2 3RGHU /HJLVODWLYR p H[HUFLGR SHOD&kPDUD


DOLHQDomR GRV EHQV LPyYHLV GR 'LVWULWR )HGHUDO /HJLVODWLYD FRPSRVWD GH 'HSXWDGRV 'LVWULWDLV
GHSHQGHUmRGHSUpYLDDYDOLDomRHDXWRUL]DomRGD UHSUHVHQWDQWHV GR SRYR HOHLWRV H LQYHVWLGRV QD
&kPDUD/HJLVODWLYDVXERUGLQDGDjFRPSURYDomR IRUPDGDOHJLVODomRIHGHUDO
GDH[LVWrQFLDGHLQWHUHVVHS~EOLFRHjREVHUYkQFLD †~QLFR &DGDOHJLVODWXUDWHUiDGXUDomRGHTXDWURDQRV
GDOHJLVODomRSHUWLQHQWHjOLFLWDomR LQLFLDQGRVHFRPDSRVVHGRVHOHLWRV

$UW2 *RYHUQDGRU HQFDPLQKDUi DQXDOPHQWHj $UW $&kPDUD/HJLVODWLYDGR'LVWULWR)HGHUDOWHPVHGH


&kPDUD /HJLVODWLYD UHODWyULR GR TXDO FRQVWH D HP%UDVtOLD&DSLWDOGD5HS~EOLFD)HGHUDWLYDGR
LGHQWLILFDomRGRVEHQVGR'LVWULWR)HGHUDOREMHWR %UDVLO
GHFRQFHVVmRRXSHUPLVVmRGHXVRQRH[HUFtFLR 
DVVLPFRPRVXDGHVWLQDomRHEHQHILFLiULR †~QLFR 3RGHUiD&kPDUD/HJLVODWLYDUHXQLUVHWHPSRUDULD
PHQWHHPTXDOTXHUORFDOGR'LVWULWR)HGHUDOSRU
†~QLFR 2 GHVFXPSULPHQWR GR GLVSRVWRQHVWHDUWLJR GHOLEHUDomR GD PDLRULD DEVROXWD GH VHXV PHP
LPSRUWDFULPHGHUHVSRQVDELOLGDGH EURVVHPSUHTXHKRXYHUPRWLYRUHOHYDQWHHGH
FRQYHQLrQFLDS~EOLFDRXHPYLUWXGHGHDFRQWHFL
$UW2VEHQVGR'LVWULWR)HGHUDOGHVWLQDUVHmRSULRUL PHQWR TXH LPSRVVLELOLWH VHX IXQFLRQDPHQWR QD
WDULDPHQWHDRXVRS~EOLFRUHVSHLWDGDVDVQRUPDV VHGH
GH SURWHomR DR PHLR DPELHQWH DR SDWULP{QLR
KLVWyULFRFXOWXUDODUTXLWHW{QLFRHSDLVDJtVWLFRH $UW6DOYR GLVSRVLomR HP FRQWUiULR GD&RQVWLWXLomR
JDUDQWLGRRLQWHUHVVHVRFLDO )HGHUDOHGHVWD/HL2UJkQLFDDVGHOLEHUDo}HVGD
&kPDUD /HJLVODWLYD H GH VXDV FRPLVV}HV VHUmR
†ž 2V EHQV S~EOLFRV WRUQDUVHmRLQGLVSRQtYHLVRX WRPDGDVSRUPDLRULDGHYRWRVSUHVHQWHDPDLRULD
GLVSRQtYHLVSRUPHLRGHDIHWDomRRXGHVDIHWDomR DEVROXWDGHVHXVPHPEURVHPYRWDomRRVWHQVLYD
129$5('$d®2'$'$$2$UW3(/$(0(1'$­/(,25*Ç1,&$1ž'(
UHVSHFWLYDPHQWHQRVWHUPRVGDOHL '2')'(

†ž $GHVDIHWDomRSRUOHLHVSHFtILFDVyVHUiDGPLWLGD †~QLFR 4XDQGR R VLJLOR IRU LPSUHVFLQGtYHODRLQWHUHVVH


HPFDVRGHFRPSURYDGRLQWHUHVVHS~EOLFRDSyV S~EOLFR GHYLGDPHQWH MXVWLILFDGR D YRWDomR
DPSODDXGLrQFLDjSRSXODomRLQWHUHVVDGD SRGHUiVHUUHDOL]DGDSRUHVFUXWtQLRVHFUHWRGHVGH
TXHUHTXHULGDSRUSDUWLGRSROtWLFRFRPUHSUHVHQ
†ž 2'LVWULWR)HGHUDOXWLOL]DUiVHXVEHQVGRPLQLDLV WDomR QD &kPDUD /HJLVODWLYD H DSURYDGD HP
FRPRLQVWUXPHQWRSDUDDUHDOL]DomRGHSROtWLFDV YRWDomR RVWHQVLYD SHOD PDLRULD DEVROXWD GRV
GHRFXSDomRRUGHQDGDRWHUULWyULR 'HSXWDGRV'LVWULWDLV

$UW&DEH DR 3RGHU ([HFXWLYR D DGPLQLVWUDomRGRV $UW2 3RGHU /HJLVODWLYR VHUi UHSUHVHQWDGR SRUVHX
EHQV GR 'LVWULWR )HGHUDO UHVVDOYDGR j &kPDUD 3UHVLGHQWH H MXGLFLDOPHQWH SHOD 3URFXUDGRULD
/HJLVODWLYDDGPLQLVWUDUDTXHOHVXWLOL]DGRVHPVHXV *HUDOGD&kPDUD/HJLVODWLYD
1RYD 5HGDomR  (PHQGD D /HL 2UJkQLFD Qž  GH  GH GH]HPEUR GH 
VHUYLoRVHVREVXDJXDUGD SXEOLFDGDQR'2')GH
&kPDUD/HJLVODWLYDGR
 'LVWULWR)HGHUDO 'LU&RQVWLWXFLRQDO 

†ž ([FHWXDVHGRGLVSRVWRQHVWHDUWLJRDGHVDIHWDomR ,,, FULDomR WUDQVIRUPDomR H H[WLQomR GHFDUJRV


SUHYLVWDQR3ODQR'LUHWRU/RFDOHDGHVDIHWDomR HPSUHJRVHIXQo}HVS~EOLFDVIL[DomRGRVYHQFL
TXHVHMDIHLWDSRUOHLHVSHFtILFDPRWLYDGDHVWDSRU PHQWRVRXDXPHQWRGHVXDUHPXQHUDomR
VLWXDomRGHUHOHYDQWHLQWHUHVVHS~EOLFRSUHFHGLGD
GH HVWXGRV WpFQLFRV TXH DYDOLHP R LPSDFWR GD ,9 SODQRV H SURJUDPDV ORFDLV GHGHVHQYROYLPHQWR
DOWHUDomRDSURYDGRVSHORyUJmRWpFQLFRGR'LVWUL HFRQ{PLFRVRFLDO
WR)HGHUDO
129$5('$d®2'$'$$2†ž'2$UW3(/$(0(1'$­/(,25*$1,&$1ž 9 HGXFDomRVD~GHSUHYLGrQFLDKDELWDomRFXOWXUD
'('2')'(
HQVLQRGHVSRUWRHVHJXUDQoDS~EOLFD
, UHSUHVHQWDUD&kPDUD/HJLVODWLYDMXGLFLDOPHQWH
9, DXWRUL]DomRSDUDDOLHQDomRGRVEHQVLPyYHLVGR
'LVWULWR)HGHUDORXFHVVmRGHGLUHLWRVUHDLVDHOHV
,, SURPRYHU D GHIHVD GD &kPDUD UHTXHUHQGRD
UHODWLYRV EHP FRPR UHFHELPHQWR SHOR 'LVWULWR
TXDOTXHUyUJmRHQWLGDGHRXWULEXQDODVPHGLGDV
)HGHUDOGHGRDo}HVFRPHQFDUJRQmRVHFRQVLGH
GH LQWHUHVVH GD -XVWLoD GD $GPLQLVWUDomR H GR
UDQGRFRPRWDLVDVLPSOHVGHVWLQDomRHVSHFtILFD
(UiULR
GREHP
,,, SURPRYHU D XQLIRUPL]DomR GDMXULVSUXGrQFLD 9,, FULDomRHVWUXWXUDomRHDWULEXLo}HVGH6HFUHWDULDV
DGPLQLVWUDWLYD H D FRPSLODomR GD OHJLVODomR GD GR*RYHUQRGR'LVWULWR)HGHUDOHGHPDLVyUJmRVH
&kPDUD/HJLVODWLYDHGR'LVWULWR)HGHUDO HQWLGDGHVGDDGPLQLVWUDomRGLUHWDHLQGLUHWD

,9 SUHVWDU FRQVXOWRULD H DVVHVVRULD MXUtGLFD j0HVD 9,,, XVRGRVRORUXUDOREVHUYDGRRGLVSRVWRQRVDUWV


'LUHWRUDHDRVGHPDLVyUJmRVGDHVWUXWXUDDGPLQ DGD&RQVWLWXLomR)HGHUDO
VLWUDWLYD
,; SODQHMDPHQWR H FRQWUROH GR XVRSDUFHODPHQWR
9 5HYRJDGRRLQFLVR9(PHQGDD/HL2UJkQLFDQžGHGH RFXSDomR GR VROR H PXGDQoD GH GHVWLQDomR GH
PDUoRGHSXEOLFDGDQR'2')GH iUHDVXUEDQDVREVHUYDGRRGLVSRVWRQRVDUWV
HGD&RQVWLWXLomR)HGHUDO
†ž 2LQJUHVVRGDFDUUHLUDGH3URFXUDGRUGD&kPDUD
/HJLVODWLYDIDUVHiPHGLDQWHFRQFXUVRS~EOLFRGH ; FULDomRLQFRUSRUDomRIXVmRHGHVPHPEUDPHQWR
SURYDVHWtWXORV GH5HJL}HV$GPLQLVWUDWLYDV

†ž $&kPDUD/HJLVODWLYDGR'LVWULWR)HGHUDOUHJXOD ;, FRQFHVVmR RX SHUPLVVmR SDUD D H[SORUDomRGH


PHQWDUiDRUJDQL]DomRHRIXQFLRQDPHQWRGDVXD VHUYLoRVS~EOLFRVLQFOXtGRRGHWUDQVSRUWHFROHWL
3URFXUDGRULD*HUDO H GD UHVSHFWLYD FDUUHLUD GH YR
3URFXUDGRUGD&kPDUD/HJLVODWLYD
1RYD5HGDomR(PHQGDD/HL2UJkQLFDQžGHGHPDUoRGHSXEOLFDGD
QR'2')GH ;,, RVHUYLGRUS~EOLFRVHXUHJLPHMXUtGLFRSURYLPHQ
WRGHFDUJRVHVWDELOLGDGHHDSRVHQWDGRULD
†ž $ &kPDUD /HJLVODWLYD GLVSRUi DLQGDVREUHR
IXQFLRQDPHQWRGDVXD3URFXUDGRULD*HUDODWpTXH ;,,, FULDomR WUDQVIRUPDomR IXVmR H H[WLQomRGH
VHMDPSURYLGRVSRUFRQFXUVRS~EOLFRRVUHVSHFWL HQWLGDGHVS~EOLFDVGR'LVWULWR)HGHUDOEHPFRPR
YRVFDUJRVGDTXHOHyUJmR QRUPDVJHUDLVVREUHSULYDWL]DomRGDVHQWLGDGHVGH
,QFOXtGRR†ž(PHQGDD/HL2UJkQLFDQžGHGHPDUoRGHSXEOLFDGD GLUHLWR SULYDGR LQWHJUDQWHV GD DGPLQLVWUDomR
QR'2')GH
LQGLUHWD
6HomR,,
;,9 SUHVWDomRGHJDUDQWLDSHOR'LVWULWR)HGHUDOHP
'DV$WULEXLo}HVGD&kPDUD/HJLVODWLYD
RSHUDomRGHFUpGLWRFRQWUDWDGDSRUVXDVDXWDUTXL
DVIXQGDo}HVHPSUHVDVS~EOLFDVHVRFLHGDGHVGH
$UW&DEH j &kPDUD /HJLVODWLYD FRP D VDQomRGR HFRQRPLDPLVWD
*RYHUQDGRUQmRH[LJLGDHVWDSDUDRHVSHFLILFDGR
QR$UWGHVWD/HL2UJkQLFDGLVSRUVREUHWRGDV ;9 DTXLVLomRDGPLQLVWUDomRDOLHQDomRDUUHQGDPHQ
DVPDWpULDVGHFRPSHWrQFLDGR'LVWULWR)HGHUDO WRHFHVVmRGHEHQVLPyYHLVGR'LVWULWR)HGHUDO
HVSHFLDOPHQWHVREUH
;9, WUDQVIHUrQFLDWHPSRUiULDGDVHGHGR*RYHUQR
, PDWpULD WULEXWiULD REVHUYDGRRGLVSRVWRQRV
DUWVHGD&RQV ;9,, SURWHomRHLQWHJUDomRGHSHVVRDVSRUWDGRUDVGH
WLWXLomR)HGHUDO GHILFLrQFLD

,, SODQRSOXULDQXDOGLUHWUL]HVRUoDPHQWiULDVRUoD ;9,,, SURWHomRDLQIkQFLDMXYHQWXGHHLGRVRV


PHQWRDQXDORSHUDo}HVGHFUpGLWRGtYLGDS~EOLFD
H HPSUpVWLPRV H[WHUQRV D TXDOTXHU WtWXOR D VHU ;,; RUJDQL]DomR GR VLVWHPD ORFDO GH HPSUHJRHP
FRQWUDtGRVSHOR'LVWULWR)HGHUDO FRQVRQkQFLDFRPRVLVWHPDQDFLRQDO
 'LU&RQVWLWXFLRQDO
 &kPDUD/HJLVODWLYDGR'LVWULWR)HGHUDO
$UW&RPSHWH j &kPDUD /HJLVODWLYD DXWRUL]DUQRV ;,9 FRQYRFDU 6HFUHWiULRV GH *RYHUQR GLULJHQWHVH
OLPLWHVHVWDEHOHFLGRVSHOR6HQDGR)HGHUDODFHOH VHUYLGRUHVGDDGPLQLVWUDomRGLUHWDHLQGLUHWDGR
EUDomRGHRSHUDo}HVGHFUpGLWRDUHDOL]DomRGH 'LVWULWR)HGHUDODSUHVWDUSHVVRDOPHQWHLQIRUPD
RSHUDo}HVH[WHUQDVGHQDWXUH]DILQDQFHLUDEHP o}HV VREUH DVVXQWRV SUHYLDPHQWH GHWHUPLQDGRV
FRPR D FRQFHVVmR GH TXDOTXHU JDUDQWLD SHOR LPSRUWDQGRFULPHGHUHVSRQVDELOLGDGHDDXVrQFLD
'LVWULWR)HGHUDORXSRUVXDVDXWDUTXLDV VHPMXVWLILFDWLYDDGHTXDGDRXRQmRDWHQGLPHQWR
QRSUD]RGHWULQWDGLDVEHPFRPRDSUHVWDomRGH
$UW &RPSHWHSULYDWLYDPHQWHj&kPDUD/HJLVODWLYDGR LQIRUPDo}HV IDOVDV QRV WHUPRV GD OHJLVODomR
'LVWULWR)HGHUDO SHUWLQHQWH
1RYD 5HGDomR  (PHQGD D /HL 2UJkQLFD Qž  GH  GH GH]HPEUR GH 
SXEOLFDGDQR'2')GH
127$)LFDVXEVWLWXtGDDH[SUHVVmR´6HFUHWiULRGH*RYHUQRGR'LVWULWR)HGHUDOµSRU
, HOHJHURVPHPEURVGD0HVD'LUHWRUDHFRQVWLWXLU ´6HFUHWiULRGH(VWDGRGR'LVWULWR)HGHUDOµ&21)250((PHQGD­/HL2UJkQLFDQž
VXDVFRPLVV}HV GH'2')'(

,, GLVSRU VREUH VHX UHJLPHQWR LQWHUQR SROtFLDH ;9 MXOJDUDQXDOPHQWHDVFRQWDVSUHVWDGDVSHOR*RYHU


VHUYLoRVDGPLQLVWUDWLYRV QDGRUHDSUHFLDURVUHODWyULRVVREUHDH[HFXomR
GRVSODQRVGRJRYHUQR
,,, HVWDEHOHFHUHPXGDUWHPSRUDULDPHQWHVXDVHGHR
ORFDO GH VXDV UHXQL}HV EHP FRPR R GH VXDV ;9, ILVFDOL]DUHFRQWURODURVDWRVGR3RGHU([HFXWLYR
FRPLVV}HVSHUPDQHQWHV LQFOXtGRVRVGDDGPLQLVWUDomRLQGLUHWD

,9 ]HODUSHODSUHVHUYDomRGHVXDFRPSHWrQFLDOHJLVOD ;9,, HVFROKHUFLQFRHQWUHRVVHWHPHPEURVGR7ULEXQDO


WLYD GH&RQWDVGR'LVWULWR)HGHUDO

;9,,, DSURYDUSUHYLDPHQWHHPYRWDomRRVWHQVLYDDSyV
9 FULDU WUDQVIRUPDU RX H[WLQJXLU FDUJRV GHVHXV
DUJLomRHPVHomRS~EOLFDDHVFROKDGRVWLWXODUHV
VHUYLoRVEHPFRPRSURYrORVHIL[DURXPRGLILFDU
GRFDUJRGHFRQVHOKHLURVGR7ULEXQDOGH&RQWDV
DVUHVSHFWLYDVUHPXQHUDo}HV
GR'LVWULWR)HGHUDOLQGLFDGRVSHOR*RYHUQDGRU
129$ 5('$d®2 '$'$ $2 ,1&,62 ;9,,, '2 $UW 3(/$ (0(1'$ ­ /(,
9, VXVWDURVDWRVQRUPDWLYRVGR3RGHU([HFXWLYRTXH 25*$1,&$1ž'('2')'(

H[RUELWHPGRSRGHUUHJXODPHQWDUFRQILJXUDQGR
FULPHGHUHVSRQVDELOLGDGHVXDUHHGLomR ;,; VXVSHQGHUQRWRGRRXHPSDUWHDH[HFXomRGHOHL
RXDWRQRUPDWLYRGHFODUDGRLOHJDORXLQFRQVWLWXFL
9,, IL[DUSDUDFDGDH[HUFtFLRILQDQFHLURDUHPXQHUD RQDOWDQWRSHOR6XSUHPR7ULEXQDO)HGHUDOTXDQWR
SHOR7ULEXQDOGH-XVWLoDGR'LVWULWR)HGHUDOQDV
omRGR*RYHUQDGRU9LFH*RYHUQDGRU6HFUHWiULRV
VXDVUHVSHFWLYDViUHDVGHFRPSHWrQFLDHPVHQWHQ
GH*RYHUQRGR'LVWULWR)HGHUDOH$GPLQLVWUDGRUHV
oDVWUDQVLWDGDVHPMXOJDGR
5HJLRQDLVREVHUYDGRVRVSULQFtSLRVGD&RQVWLWXL
omR)HGHUDO
127$)LFDVXEVWLWXtGDDH[SUHVVmR´6HFUHWiULRGH*RYHUQRGR'LVWULWR)HGHUDOµSRU ;; DSURYDUSUHYLDPHQWHDLQGLFDomRRXGHVWLWXLomR
´6HFUHWiULRGH(VWDGRGR'LVWULWR)HGHUDOµ&21)250((PHQGD­/HL2UJkQLFDQž
GH'2')'(
GR3URFXUDGRU*HUDOGR'LVWULWR)HGHUDO

9,,, IL[DUDUHPXQHUDomRGRV'HSXWDGRV'LVWULWDLVHP ;;, FRQYRFDUR3URFXUDGRU*HUDOGR'LVWULWR)HGHUDO


FDGDOHJLVODWXUDSDUDDVXEVHTHQWH HR'HIHQVRU3~EOLFR*HUDOGR'LVWULWR)HGHUDOD
SUHVWDULQIRUPDo}HVVREUHDVVXQWRVSUHYLDPHQWH
GHWHUPLQDGRVQRSUD]RGHWULQWDGLDVVXMHLWDQ
,; VROLFLWDULQWHUYHQomRIHGHUDOSDUDJDUDQWLUROLYUH
GRVHHVWHVjVSHQDVGDOHLSRUDXVrQFLDLQMXVWLILFD
H[HUFtFLR GH VXDV DWULEXLo}HV QRV WHUPRV GRV
GD
DUWV,9H,GD&RQVWLWXLomR)HGHUDO 1RYD5HGDomRGDGDDRLQFLVR;;,GR$UWSHOD(PHQGD­/HL2UJkQLFDQžGH
'2')GH

; SURPRYHU SHULRGLFDPHQWH D FRQVROLGDomRGRV


WH[WRVOHJLVODWLYRVFRPDILQDOLGDGHGHWRUQDUVXD ;;,, GHFODUDUDSHUGDGRPDQGDWRGR*RYHUQDGRUHGR
FRQVXOWDDFHVVtYHODRVFLGDGmRV 9LFH*RYHUQDGRU

;;,,, DXWRUL]DU SRU GRLV WHUoRV GRV VHXV PHPEURVD


;, GDU SRVVH DR *RYHUQDGRU H 9LFH*RYHUQDGRUH
LQVWDXUDomRGHSURFHVVRFRQWUDR*RYHUQDGRUR
FRQKHFHUGDUHQ~QFLDGHTXDOTXHUGHOHVGHFODUDU
9LFH*RYHUQDGRUHRV6HFUHWiULRVGH*RYHUQR
YDFkQFLDHSURPRYHUDVUHVSHFWLYDVVXEVWLWXLo}HV 127$)LFDVXEVWLWXtGDDH[SUHVVmR´6HFUHWiULRGH*RYHUQRGR'LVWULWR)HGHUDOµSRU
RXVXFHVV}HVQRVWHUPRVGHVWD/HL2UJkQLFD ´6HFUHWiULRGH(VWDGRGR'LVWULWR)HGHUDOµ&21)250((PHQGD­/HL2UJkQLFDQž
GH'2')'(

;,, DXWRUL]DUR*RYHUQDGRUHR9LFH*RYHUQDGRUDVH ;;,9 SURFHVVDU H MXOJDU R *RYHUQDGRU QRV FULPHVGH


DXVHQWDUHP GR 'LVWULWR )HGHUDO SRU PDLV GH UHVSRQVDELOLGDGHEHPFRPRDGRWDUDVSURYLGrQFL
TXLQ]HGLDV DVSHUWLQHQWHVQRVWHUPRVGDOHJLVODomRIHGHUDO
TXDQWR DR 9LFH*RYHUQDGRU H 6HFUHWiULRV GH
;,,, SURFHGHU j WRPDGD GH FRQWDV GR*RYHUQDGRU *RYHUQR QRV FULPHV GD PHVPD QDWXUH]D RX
TXDQGRQmRDSUHVHQWDGDVQRVSUD]RVHVWDEHOHFL FRQH[RVFRPDTXHOHV
GRV 127$)LFDVXEVWLWXtGDDH[SUHVVmR´6HFUHWiULRGH*RYHUQRGR'LVWULWR)HGHUDOµSRU
´6HFUHWiULRGH(VWDGRGR'LVWULWR)HGHUDOµ&21)250((PHQGD­/HL2UJkQLFDQž
GH'2')'(
&kPDUD/HJLVODWLYDGR
  'LVWULWR)HGHUDO 'LU&RQVWLWXFLRQDO 

;;9 SURFHVVDU H MXOJDU R 3URFXUDGRU*HUDOQRV ;/  FRQFHGHU WtWXOR GH FLGDGmR EHQHPpULWRRX
FULPHVGHUHVSRQVDELOLGDGH KRQRUiULRQRVWHUPRVGRUHJLPHQWRLQWHUQR
6XSULPLGRRLQFLVR;/(PHQGDD/HL2UJkQLFDQžGHGHIHYHUHLURGH
SXEOLFDGDQR'2')GH
;;9, DXWRUL]DU RX DSURYDU FRQYrQLRV DFRUGRVRX 5HQXPHUDGR(PHQGDD/HL2UJkQLFDQžGHGHIHYHUHLURGH
SXEOLFDGDQR'2')GH
FRQWUDWRV GH TXH UHVXOWHP SDUD R 'LVWULWR
)HGHUDO HQFDUJRV QmR SUHYLVWRV QD OHL RUoD
;/,  DXWRUL]DUUHIHUHQGRHFRQYRFDUSOHELVFLWR
PHQWiULD ,QFOXtGRRLQFLVR;/,,(PHQGDD/HL2UJkQLFDQžGHGHGH]HPEURGH
SXEOLFDGDQR'2')GH

;;9,, DSURYDU SUHYLDPHQWH HP YRWDomRRVWHQVLYD


DSyVDUJLomRS~EOLFDDHVFROKDGRVPHPEURV †ž (PVXDIXQomRILVFDOL]DGRUDD&kPDUD/HJLVODWLYD
GRFRQVHOKRGH*RYHUQRLQGLFDGRVSHOR*RYHU REVHUYDUiQRTXHFRXEHURGLVSRVWRQRVDUWV
QDGRU DGD&RQVWLWXLomR)HGHUDO
129$5('$d®2'$'$$2,1&,62;;9,,'2$UW3(/$(0(1'$­/(,
25*$1,&$1ž'('2')'(
†ž 1RFDVRGRLQFLVR;,D0HVD'LUHWRUDGD&kPDUD
;;9,,, DSURYDU SUHYLDPHQWH D DOLHQDomR GHWHUUDV /HJLVODWLYD HQYLDUi GHQ~QFLD HP FLQFR GLDV j
S~EOLFDV FRP iUHD VXSHULRU D YLQWH H FLQFR &RPLVVmR (VSHFLDO FRPSRVWD HP FRQIRUPLGDGH
KHFWDUHVHQRFDVRGHFRQFHVVmRGHXVRFRP FRP R $UW JDUDQWLGD D SURSRUFLRQDOLGDGH
iUHDVXSHULRUDFLQTHQWDKHFWDUHV SDUWLGiULD D TXDO HPLWLUi SDUHFHU QR SUD]R GH
TXLQ]H GLDV VXEPHWHQGRR LPHGLDWDPHQWH DR
;;,; DSUHFLDU H MXOJDU DQXDOPHQWH DV FRQWDVGR 3OHQiULR
7ULEXQDOGH&RQWDVGR'LVWULWR)HGHUDO
†ž $UHPXQHUDomRGRV'HSXWDGRV'LVWULWDLVREHGHFH
;;; UHFHEHU UHQ~QFLD GH 'HSXWDGR 'LVWULWDOH UiDROLPLWHHVWDEHOHFLGRSHOD&RQVWLWXLomR)HGH
GHFODUDUDYDFkQFLDGRFDUJR UDO
LQFOXLGR R † ž SHOD (PHQGD ­ /HL 2UJkQLFD Qž  GH   '2') GH
ž
;;;, GHFODUDU D SHUGD GH PDQGDWR GH'HSXWDGR
'LVWULWDOFRPRSUHYrR$UW†ž †ž 6HPSUHMXt]RGRGLVSRVWRQRLQFLVR;,9GRFDSXW
RV6HFUHWiULRVGH(VWDGRHGLULJHQWHVGDDGPLQLV
;;;,, VROLFLWDUDR*RYHUQDGRULQIRUPDomRVREUHDWRV WUDomRS~EOLFDGLUHWDHLQGLUHWDGR'LVWULWR)HGHUDO
GHVXDFRPSHWrQFLD FRPSDUHFHUmR SHUDQWH D &kPDUD /HJLVODWLYD RX
VXDVFRPLVV}HVSDUDH[SRUDVVXQWRVGHLQWHUHVVH
;;;,,, HQFDPLQKDUSRULQWHUPpGLRGD0HVD'LUHWRUD GHVXDiUHDGHDWULEXLomR
UHTXHULPHQWR GH LQIRUPDomR DRV 6HFUHWiULRV
GH*RYHUQRLPSOLFDQGRFULPHGHUHVSRQVDELOL
, SRU LQLFLDWLYD SUySULD DWp RWpUPLQRGHFDGD
GDGHQRVWHUPRVGDOHJLVODomRSHUWLQHQWHD
VHVVmROHJLVODWLYDPHGLDQWHHQWHQGLPHQWRFRPD
UHFXVD RX R QmR DWHQGLPHQWR QR SUD]R GH
0HVD'LUHWRUDRXDSUHVLGrQFLDGH&RPLVVmR
WULQWD GLDV EHP FRPR R IRUQHFLPHQWR GH
LQIRUPDomRIDOVD
127$)LFDVXEVWLWXtGDDH[SUHVVmR´6HFUHWiULRGH*RYHUQR ,, ILQGDDJHVWmRjIUHQWHGDSDVWD
GR'LVWULWR)HGHUDOµSRU´6HFUHWiULRGH(VWDGRGR'LVWULWR
)HGHUDOµ &21)250( (PHQGD ­ /HL 2UJkQLFD Qž  GH 6HomR,,,
'2')'(
'RV'HSXWDGRV'LVWULWDLV
;;;,9 DSUHFLDU YHWRV REVHUYDGR QR TXH FRXEHUR
$UW2V 'HSXWDGRV 'LVWULWDLV VmR LQYLROiYHLV FLYLOH
GLVSRVWR QRV DUWV H  GD &RQVWLWXLomR
SHQDOPHQWH SRU TXDLVTXHU GH VXDV RSLQL}HV
)HGHUDO
SDODYUDVHYRWRV
129$5('$d®2'$'$$2$UW3(/$(0(1'$­/(,25*$1,&$1ž'(
;;;9 DSURYDUSUHYLDPHQWHDLQGLFDomRGHSUHVLGHQWH '2')'(

GH LQVWLWXLomR ILQDQFHLUDV RILFLDLV GR 'LVWULWR


)HGHUDO †ž 2V 'HSXWDGRV 'LVWULWDLV GHVGH DH[SHGLomRGR
GLSORPDVHUmRVXEPHWLGRVDMXOJDPHQWRSHUDQWH
;;;9, FRQFHGHU OLFHQoD SDUD SURFHVVDU'HSXWDGR R7ULEXQDOGH-XVWLoDGR'LVWULWR)HGHUDOH7HUUL
'LVWULWDO WyULRV

;;;9,, HPHQGDUD/HL2UJkQLFDSURPXOJDUOHLVQRV †ž 'HVGHD H[SHGLomR GR GLSORPDRVPHPEURVGD


FDVRV GH VLOrQFLR GR *RYHUQDGRU H[SHGLU &kPDUD/HJLVODWLYDQmRSRGHUmRVHUSUHVRVVDOYR
GHFUHWRVOHJLVODWLYRVHUHVROXo}HV HPIODJUDQWHGHFULPHLQDILDQoiYHO

;;;9,,, UHJXODPHQWDUDVIRUPDVGHSDUWLFLSDomRSRSX †ž 1R FDVR GH IODJUDQWH GH FULPHLQDILDQoiYHORV


ODUSUHYLVWDVQHVWD/HL2UJkQLFD DXWRV VHUmR UHPHWLGRV GHQWUR GH YLQWH H TXDWUR
KRUDVj&kPDUD/HJLVODWLYDSDUDTXHSHORYRWR
;;;,; LQGLFDUPHPEURVGR&RQVHOKRGH*RYHUQRQRV GD PDLRULD GH VHXV PHPEURV UHVROYD VREUH D
WHUPRVGR$UW9 SULVmR
 'LU&RQVWLWXFLRQDO &kPDUD/HJLVODWLYDGR'LVWULWR)HGHUDO


†ž 5HFHELGDDGHQ~QFLDFRQWUDR'HSXWDGR'LVWULWDO F SDWURFLQDUFDXVDHPTXHVHMDLQWHUHVVDGDTXDOTXHU


SRUFULPHRFRUULGRDSyVDGLSORPDomRR7ULEXQDO GDVHQWLGDGHVDTXHVHUHIHUHRLQFLVR,D
GH-XVWLoDGR'LVWULWR)HGHUDOH7HUULWyULRVGDUi
FLrQFLDj&kPDUD/HJLVODWLYDTXHSRULQLFLDWLYD G VHU WLWXODUHV GH PDLV GH XP FDUJR RX PDQGDWR
GHSDUWLGRSROtWLFRQHODUHSUHVHQWDGRHSHORYRWR S~EOLFRHOHWLYR
GDPDLRULDGHVHXVPHPEURVSRGHUiDWpDGHFL
VmRILQDOVXVWDURDQGDPHQWRGDDomR $UW3HUGHUiRPDQGDWRR'HSXWDGR'LVWULWDO

†ž 2SHGLGRGHVXVWDomRVHUiDSUHFLDGRSHOD&kPDUD , TXHLQIULQJLUTXDOTXHUGDVSURLELo}HVHVWDEHOHFL


/HJLVODWLYDQRSUD]RLPSURUURJiYHOGHTXDUHQWDH GDVQRDUWLJRDQWHULRU
FLQFRGLDVGRVHXUHFHELPHQWRSHOD0HVD'LUHWRUD
,, FXMR SURFHGLPHQWR IRU GHFODUDGRLQFRPSDWtYHO
†ž $ VXVWDomR GR SURFHVVRVXVSHQGHDSUHVFULomR FRPRGHFRURSDUODPHQWDU
HQTXDQWRGXUDURPDQGDWR
,,, TXHGHL[DUGHFRPSDUHFHUHPFDGDVHVVmROHJLVOD
†ž 2V 'HSXWDGRV 'LVWULWDLV QmR VHUmRREULJDGRVD WLYD j WHUoD SDUWH GDV VHVV}HV RUGLQiULDV VDOYR
WHVWHPXQKDU VREUH LQIRUPDo}HV UHFHELGDV RX OLFHQoDRXPLVVmRDXWRUL]DGDSHOD&kPDUD/HJLVOD
WLYD
SUHVWDGDVHPUD]mRGRH[HUFtFLRGRPDQGDWRQHP
VREUH DV SHVVRDV TXH OKHV FRQILDUDP RX GHOHV
,9 TXHSHUGHURXWLYHUVXVSHQVRVRVGLUHLWRVSROtWLFRV
UHFHEHUDPLQIRUPDo}HV
9 TXDQGRRGHFUHWDUD-XVWLoD(OHLWRUDOQRVFDVRV
†ž $LQFRUSRUDomRGH'HSXWDGRV'LVWULWDLVjV)RUoDV
SUHYLVWRVQD&RQVWLWXLomR)HGHUDO
$UPDGDVHPERUDPLOLWDUHVHDLQGDTXHHPWHPSR
GHJXHUUDGHSHQGHUiGHSUpYLDOLFHQoDGD&kPDUD 9, TXH VRIUHU FRQGHQDomR FULPLQDO HPVHQWHQoD
/HJLVODWLYD WUDQVLWDGDHPMXOJDGR
†ž $VLPXQLGDGHVGRV'HSXWDGRV'LVWULWDLVVXEVLVWL 9,, TXHXWLOL]DUVHGRPDQGDWRSDUDDSUiWLFDGHDWRV
UmR GXUDQWH R HVWDGR GH VtWLR Vy SRGHQGR VHU GHFRUUXSomRRXLPSURELGDGHDGPLQLVWUDWLYD
VXVSHQVDV PHGLDQWH R YRWR GH GRLV WHUoRV GRV
PHPEURVGD&kPDUD/HJLVODWLYDQRVFDVRVGHDWRV †ž eLQFRPSDWtYHOFRPRGHFRURSDUODPHQWDUDOpP
SUDWLFDGRV IRUD GR UHFLQWR GD &DVD TXH VHMDP GRVFDVRVGHILQLGRVQRUHJLPHQWRLQWHUQRRDEXVR
LQFRPSDWtYHLVFRPDH[HFXomRGDPHGLGD GDV SUHUURJDWLYDV DVVHJXUDGDV DR 'HSXWDGR
'LVWULWDORXDSHUFHSomRGHYDQWDJHQVLQGHYLGDV
† 3RGHUiR'HSXWDGR'LVWULWDOPHGLDQWHOLFHQoDGD
&kPDUD /HJLVODWLYD GHVHPSHQKDU PLVV}HV GH †ž 1RVFDVRVGRVLQFLVRV,,,H9,DSHUGDGRPDQGD
FDUiWHUGLSORPiWLFRHFXOWXUDO WRVHUiGHFLGLGDSRUPDLRULDDEVROXWDGRVPHP
EURVGD&kPDUD/HJLVODWLYDHPYRWDomRRVWHQVLYD
$UW2V'HSXWDGRV'LVWULWDLVQmRSRGHUmR PHGLDQWH SURYRFDomR GD 0HVD 'LUHWRUD RX GH
SDUWLGRSROtWLFRUHSUHVHQWDGRQD&DVDDVVHJXUDGD
, GHVGHDH[SHGLomRGRGLSORPD DPSODGHIHVD
129$ 5('$d®2 '$'$ $2 3$5É*5$)2 ž '2 $UW 3(/$ (0(1'$ ­ /(,
25*$1,&$1ž'('2')'(
D ILUPDURXPDQWHUFRQWUDWRFRPSHVVRDMXUtGLFDGH
GLUHLWR S~EOLFR DXWDUTXLD HPSUHVD S~EOLFD †ž 1RV FDVRV SUHYLVWRV QRV LQFLVRV ,,, D 9DSHUGD
VRFLHGDGHGHHFRQRPLDPLVWDRXHPSUHVDFRQFHV GHFODUDGDSHOD0HVD'LUHWRUDGHRItFLRRXPHGL
VLRQiULDGHVHUYLoRS~EOLFRVDOYRTXDQGRRFRQ DQWH SURYRFDomR GH TXDOTXHU GRV PHPEURV GD
WUDWRREHGHFHUDFOiXVXODVXQLIRUPHV &kPDUD /HJLVODWLYD RX GH SDUWLGR SROtWLFR QHOD
UHSUHVHQWDGRDVVHJXUDGDDPSODGHIHVD
,QFOXtGR R † ž  (PHQGD D /HL 2UJkQLFD Qž  GH  GH IHYHUHLUR GH 
E DFHLWDU RX H[HUFHU FDUJR IXQomR RX HPSUHJR SXEOLFDGDQR'2')GH
UHPXQHUDGRLQFOXVLYHRVGHTXHVHMDPGHPLVVtYH
LVDGQXWXPQDVHQWLGDGHVFRQVWDQWHVGDDOtQHD †ž $ UHQ~QFLD GH 'HSXWDGR 'LVWULWDOVXEPHWLGRD
DQWHULRU SURFHVVR TXH YLVH RX SRVVD OHYDU j SHUGD GR
PDQGDWRQRVWHUPRVGHVWHDUWLJRWHUiVHXHIHLWRV
,, GHVGHDSRVVH VXVSHQVRVDWpDVGHOLEHUDo}HVILQDLVGHTXHWUDWDP
RV††žHž
D VHU SURSULHWiULRV FRQWURODGRUHV RX GLUHWRUHV GH
HPSUHVDTXHJR]HGHIDYRUGHFRUUHQWHGHFRQWUDWR $UW1mRSHUGHUiRPDQGDWRR'HSXWDGR'LVWULWDO
FRP SHVVRD MXUtGLFD GH GLUHLWR S~EOLFR RX QHOD
H[HUFHUIXQomRUHPXQHUDGD , LQYHVWLGRQDIXQomRGH0LQLVWURGH(VWDGR6HFUH
WiULR([HFXWLYR GH 0LQLVWpULR RX HTXLYDOHQWH
E RFXSDUFDUJRRXIXQomRGHTXHVHMDPGHPLVVtYHLV 6HFUHWiULR GH (VWDGR $GPLQLVWUDGRU 5HJLRQDO
DGQXWXPQDVHQWLGDGHVUHIHULGDVQRLQFLVR,D &KHIH GH 0LVVmR 'LSORPiWLFD 7HPSRUiULD RX
 &kPDUD/HJLVODWLYDGR
 'LVWULWR)HGHUDO 'LU&RQVWLWXFLRQDO 

GLULJHQWHPi[LPRGH$XWDUTXLD)XQGDomR3~EOL $UW $FRQYRFDomRH[WUDRUGLQiULDGD&kPDUD/HJLVODWL


FD $JrQFLD (PSUHVD 3~EOLFD RX 6RFLHGDGH GH YDIDUVHi
(FRQRPLD 0LVWD SHUWHQFHQWHV j $GPLQLVWUDomR
3~EOLFD)HGHUDOH'LVWULWDO , SHOR3UHVLGHQWHQRVFDVRVGH
1RYD5HGDomRGDGDDRLQFLVR,GR$UW(PHQGDD/HL2UJkQLFDQžSXEOLFDGD
QR'2')GH
127$)LFDVXEVWLWXtGDDH[SUHVVmR´6HFUHWiULRGH*RYHUQRGR'LVWULWR)HGHUDOµSRU
´6HFUHWiULRGH(VWDGRGR'LVWULWR)HGHUDOµ&21)250((PHQGD­/HL2UJkQLFDQž
D GHFUHWDomRGHHVWDGRGHVtWLRRXHVWDGRGHGHIHVD
GH'2')'( TXHDWLQMDRWHUULWyULRGR'LVWULWR)HGHUDO
E LQWHUYHQomRQR'LVWULWR)HGHUDO
,, OLFHQFLDGRSHOD&kPDUD/HJLVODWLYDSRUPRWLYRGH F UHFHELPHQWR GRV DXWRV GH SULVmR GH 'HSXWDGR
GRHQoD RX SDUD WUDWDU VHP UHPXQHUDomR GH 'LVWULWDOQDKLSyWHVHGHIODJUDQWHGHFULPHLQDIL
LQWHUHVVH SDUWLFXODU GHVGH TXH QHVWH FDVR R DQoiYHO
DIDVWDPHQWRQmRXOWUDSDVVHFHQWRHYLQWHGLDVSRU G SRVVHGR*RYHUQDGRUH9LFH*RYHUQDGRU
VHVVmROHJLVODWLYD
,, SHOD 0HVD 'LUHWRUD RX D UHTXHULPHQWR GHXP
†ž 2VXSOHQWHVHUiFRQYRFDGRQRVFDVRVGHYDJDGH WHUoR GRV 'HSXWDGRV TXH FRPS}HP D &kPDUD
LQYHVWLGXUDQDVIXQo}HVSUHYLVWDVQHVWHDUWLJRRX /HJLVODWLYDSDUDDSUHFLDomRGHDWRGR*RYHUQDGRU
GHOLFHQoDVXSHULRUDFHQWRHYLQWHGLDV GR'LVWULWR)HGHUDOTXHLPSRUWHFULPHGHUHVSRQ
VDELOLGDGH
†ž 2FRUUHQGRYDJDHQmRKDYHQGRVXSOHQWHIDUVHi
HOHLomR SDUD SUHHQFKrOD VH IDOWDUHP PDLV GH
,,, SHOR*RYHUQDGRUGR'LVWULWR)HGHUDOSHOR3UHVL
TXLQ]HPHVHVSDUDRWpUPLQRGRPDQGDWR
GHQWHGD&kPDUD/HJLVODWLYDRXDUHTXHULPHQWR
GDPDLRULDGRVVHXVPHPEURVHPFDVRGHXUJrQ
†ž 1D KLSyWHVH GR LQFLVR , R'HSXWDGR'LVWULWDO
FLDRXLQWHUHVVHS~EOLFRUHOHYDQWH
SRGHUiRSWDUSHODUHPXQHUDomRGHVHXPDQGDWR

6HomR,9 ,9 SHODFRPLVVmRUHSUHVHQWDWLYDSUHYLVWDQR$UW


'R)XQFLRQDPHQWRGD&kPDUD/HJLVODWLYD †žQDVKLSyWHVHVHVWDEHOHFLGDVQHVWD/HL2UJkQL
FD
6XEVHomR,
'DV5HXQL}HV †~QLFR 1D VHVVmR OHJLVODWLYD H[WUDRUGLQiULDD&kPDUD
/HJLVODWLYD VRPHQWH GHOLEHUDUi VREUH D PDWpULD
$UW $&kPDUD/HJLVODWLYDUHXQLUVHiDQXDOPHQWHHP SDUDDTXDOWLYHUVLGRFRQYRFDGD
VXDVHGHGHžGHIHYHUHLURDGHMXQKRHGHž
GHDJRVWRDGHGH]HPEUR 6XEVHomR,,
'DV&RPLVV}HV
†ž $V UHXQL}HV PDUFDGDV SDUD HVVDVGDWDVVHUmR
WUDQVIHULGDVSDUDRSULPHLURGLD~WLOVXEVHTHQWH $UW $&kPDUD/HJLVODWLYDWHUiFRPLVV}HVSHUPDQHQWHV
TXDQGR UHFDtUHP HP ViEDGRV GRPLQJRV RX H WHPSRUiULDV FRQVWLWXtGDV QD IRUPD H FRP DV
IHULDGRV DWULEXLo}HVSUHYLVWDVQRVHXUHJLPHQWRLQWHUQRRX
QRDWROHJLVODWLYRGHTXHUHVXOWDUVXDFULDomR
†ž $VHVVmROHJLVODWLYDQmRVHUiLQWHUURPSLGDVHPD
DSURYDomRGRSURMHWRGHOHLGHGLUHWUL]HVRUoDPHQ †ž 1DFRPSRVLomRGHFDGDFRPLVVmRpDVVHJXUDGD
WiULDVQHPHQFHUUDGDVHPDDSURYDomRGRSURMHWR WDQWRTXDQWRSRVVtYHODUHSUHVHQWDomRSURSRUFLR
GHOHLGRRUoDPHQWR QDOGRVSDUWLGRVRXGRVEORFRVSDUODPHQWDUHVFRP
SDUWLFLSDomRQD&kPDUD/HJLVODWLYD
$UW$ &kPDUD /HJLVODWLYD HP FDGD OHJLVODWXUDUHX
QLUVHi HP VHVV}HV SUHSDUDWyULDV QR GLD ž GH †ž ­VFRPLVV}HVHPUD]mRGDPDWpULDGHVXDFRPSH
MDQHLURREVHUYDGRRVHJXLQWH WrQFLDFDEH

, QD SULPHLUD VHVVmR OHJLVODWLYDSDUDDSRVVHGRV , DSUHFLDU H HPLWLU SDUHFHUVREUHSURSRVLo}HVQD


'HSXWDGRV'LVWULWDLVHOHLomRHSRVVHGRVPHPEURV IRUPDGRUHJLPHQWRLQWHUQRGD&kPDUD/HJLVODWL
GD0HVD'LUHWRUD YD

,, QD WHUFHLUD VHVVmR OHJLVODWLYD SDUD D SRVVHGRV ,, UHDOL]DUDXGLrQFLDVS~EOLFDVFRPHQWLGDGHVUHSUH


PHPEURVGD0HVD'LUHWRUDHOHLWRVQR~OWLPRGLD VHQWDWLYDVGDVRFLHGDGHFLYLO
~WLOGDSULPHLUDTXLQ]HQDGHGH]HPEURGDVHVVmR
OHJLVODWLYDDQWHULRUYHGDGDDUHFRQGXomRSDUDR ,,, FRQYRFDU6HFUHWiULRVGH*RYHUQRGLULJHQWHVHVHU
PHVPRFDUJR
YLGRUHVGDDGPLQLVWUDomRS~EOLFDGLUHWDHLQGLUHWD
GR'LVWULWR)HGHUDOHR3URFXUDGRU*HUDODSUHVWDU
†~QLFR 1DFRPSRVLomR GD 0HVD 'LUHWRUDpDVVHJXUDGD
LQIRUPDo}HV VREUH DVVXQWRV LQHUHQWHV D VXDV
WDQWR TXDQWR SRVVtYHO D SURSRUFLRQDOLGDGH GD
DWULEXLo}HV
UHSUHVHQWDomRSDUWLGiULDRXGHEORFRVSDUODPHQWD 127$)LFDVXEVWLWXtGDDH[SUHVVmR´6HFUHWiULRGH*RYHUQRGR'LVWULWR)HGHUDOµSRU
UHVFRPSDUWLFLSDomRQD&kPDUD/HJLVODWLYD ´6HFUHWiULRGH(VWDGRGR'LVWULWR)HGHUDOµ&21)250((PHQGD­/HL2UJkQLFDQž
GH '2')'(
 'LU&RQVWLWXFLRQDO
 &kPDUD/HJLVODWLYDGR'LVWULWR)HGHUDO
,9 UHFHEHUSHWLo}HVUHFODPDo}HVUHSUHVHQWDo}HVRX , HPHQGDVj/HL2UJkQLFD
TXHL[DVFRQWUDDWRVRXRPLVV}HVGDVDXWRULGDGHV ,, OHLVFRPSOHPHQWDUHV
RXHQWLGDGHVS~EOLFDV ,,, OHLVRUGLQiULDV
,9 GHFUHWRVOHJLVODWLYRV
9 VROLFLWDUGHSRLPHQWRGHTXDOTXHUDXWRULGDGHRX 9 UHVROXo}HV
FLGDGmR
†~QLFR /HL FRPSOHPHQWDU GLVSRUiVREUHHODERUDomR
9, DSUHFLDUSURJUDPDVGHREUDVSODQRVUHJLRQDLVH UHGDomRDOWHUDomRHFRQVROLGDomRGDVOHLVGR
VHWRULDLVGHGHVHQYROYLPHQWRHVREUHHOHVHPLWLU 'LVWULWR)HGHUDO
SDUHFHU
6XEVHomR,
9,, ILVFDOL]DURVDWRVTXHHQYROYDPJDVWRVGHyUJmRV 'DV(PHQGDVj/HL2UJkQLFD
HHQWLGDGHVGDDGPLQLVWUDomRS~EOLFD
$UW$ /HL 2UJkQLFD SRGHUi VHU HPHQGDGDPHGLDQWH
†ž ­VFRPLVV}HVSDUODPHQWDUHVGHLQTXpULWRDSOLFDVH SURSRVWD
RVHJXLQWH UHGDomRGDGDSHOD(PHQGD&RQVWLWXFLRQDOQž
, GHXPWHUoRQRPtQLPRGRVPHPEURVGD&kPDUD
, VmRFULDGDVPHGLDQWHUHTXHULPHQWR /HJLVODWLYD

D GHXPWHUoRGRVPHPEURVGD&kPDUD/HJLVODWLYD ,, GR*RYHUQDGRUGR'LVWULWR)HGHUDO

E GHLQLFLDWLYDSRSXODUFRPRPtQLPRGHVXEVFULWR ,,, GHFLGDGmRVPHGLDQWHLQLFLDWLYDSRSXODUDVVLQDGD


UHVSUHYLVWRQRDUW QR PtQLPR SRU XP SRU FHQWR GRV HOHLWRUHV GR
'LVWULWR)HGHUDOGLVWULEXtGRVHPSHORPHQRVWUrV
,, GHVWLQDPVHjDSXUDomRGHIDWRGHWHUPLQDGRHSRU ]RQDVHOHLWRUDLVFRPQmRPHQRVGHWUrVGpFLPRV
SUD]RFHUWR SRUFHQWRGRHOHLWRUDGRGHFDGDXPDGHODV
,,, WrPSRGHUHVGHLQYHVWLJDomRSUySULRVGDVDXWRUL †ž $SURSRVWDVHUiGLVFXWLGDHYRWDGDHPGRLVWXU
GDGHVMXGLFLDLVDOpPGHRXWURVSUHYLVWRVHPOHLH QRVFRPLQWHUVWtFLRPtQLPRGHGH]GLDVHFRQVL
QRUHJLPHQWRLQWHUQRGD&kPDUD/HJLVODWLYD GHUDGD DSURYDGD VH REWLYHU HP DPERV R YRWR
IDYRUiYHOGHGRLVWHUoRVGRVPHPEURVGD&kPDUD
,9 RUHTXHULPHQWRDWHQGLGDVDVIRUPDOLGDGHVUHJL /HJLVODWLYD
PHQWDLVLQGHSHQGHGHDSURYDomR
†ž $HPHQGDj/HL2UJkQLFDVHUiSURPXOJDGDSHOD
9 DLQVWDODomRGHFRPLVVmRSDUODPHQWDUGHLQTXpULWR 0HVD 'LUHWRUD GD &kPDUD /HJLVODWLYD FRP R
GH LQLFLDWLYD SRSXODU WHP SUHFHGrQFLD VREUH DV UHVSHFWLYRQ~PHURGHRUGHP
GHPDLVHQmRSRGHVHULQYLDELOL]DGDHPUD]mRGH
IRUPDOLGDGHVUHJLPHQWDLV †ž 1mR VHUi REMHWR GH GHOLEHUDomR DSURSRVWDGH
HPHQGD TXH IHULU SULQFtSLRV GD &RQVWLWXLomR
9, VXDVFRQFOXV}HVVHIRURFDVRGHYHPVHUHQFDPL )HGHUDO
QKDGDV DR 7ULEXQDO GH &RQWDV DR 0LQLVWpULR
3~EOLFR RX j 3URFXUDGRULD*HUDO GR 'LVWULWR †ž $ PDWpULD FRQVWDQWH GH SURSRVWDGHHPHQGD
)HGHUDOSDUDTXHSURPRYDPFRQIRUPHRFDVRD
UHMHLWDGDRXKDYLGDSRUSUHMXGLFDGDQmRSRGHVHU
UHVSRQVDELOLGDGHFLYLOFULPLQDODGPLQLVWUDWLYDRX
REMHWRGHQRYDSURSRVWDQDPHVPDVHVVmROHJLVOD
WULEXWiULDGRLQIUDWRU
WLYD
†ž $RPLVVmRGHLQIRUPDomRjVFRPLVV}HVSDUODPHQ
†ž $ /HL 2UJkQLFD QmR SRGHUi VHUHPHQGDGDQD
WDUHV GH LQTXpULWR LQFOXVLYH DV TXH HQYROYDP
YLJrQFLDGHLQWHUYHQomRIHGHUDOHVWDGRGHGHIHVD
VLJLORRXDSUHVWDomRGHLQIRUPDo}HVIDOVDVFRQVWL
RXHVWDGRGHVtWLR
WXHP FULPH GH UHVSRQVDELOLGDGH QD IRUPD GD
OHJLVODomRSHUWLQHQWH
6XEVHomR,,
'DV/HLV
†ž 'XUDQWHRUHFHVVRKDYHUiXPDFRPLVVmRUHSUH
VHQWDWLYDGD&kPDUD/HJLVODWLYDFRPDWULEXLo}HV
$UW$LQLFLDWLYDGDVOHLVFRPSOHPHQWDUHVHRUGLQiULDV
GHILQLGDVQRUHJLPHQWRLQWHUQRFXMDFRPSRVLomR
UHSURGX]LUiWDQWRTXDQWRSRVVtYHODSURSRUFLRQD FDEHDTXDOTXHUPHPEURRXFRPLVVmRGD&kPDUD
OLGDGH GH UHSUHVHQWDomR SDUWLGiULD HOHLWD QD /HJLVODWLYDDR*RYHUQDGRUGR'LVWULWR)HGHUDOH
~OWLPDVHVVmRRUGLQiULDGHFDVDVHVVmROHJLVODWLYD QRVWHUPRVGR$UW,9DR7ULEXQDOGH&RQWDV
GR'LVWULWR)HGHUDODVVLPFRPRDRVFLGDGmRVQD
6HomR9 IRUPDHQRVFDVRVSUHYLVWRVQHVWD/HL2UJkQLFD
'R3URFHVVR/HJLVODWLYR †ž &RPSHWH SULYDWLYDPHQWH DR*RYHUQDGRUGR
'LVWULWR )HGHUDO D LQLFLDWLYD GDV OHLV TXH GLVSR
$UW2SURFHVVROHJLVODWLYRFRPSUHHQGHDHODERUDomR
QKDPVREUH
GH
&kPDUD/HJLVODWLYDGR
 'LVWULWR)HGHUDO 'LU&RQVWLWXFLRQDO 

, FULDomRGHFDUJRVIXQo}HVRXHPSUHJRVS~EOLFRV †ž 2YHWRSDUFLDOVRPHQWHDEUDQJHUiWH[WRLQWHJUDO


QDDGPLQLVWUDomRGLUHWDDXWiUTXLFDHIXQGDFLRQDO GHDUWLJRSDUiJUDIRLQFLVRRXDOtQHD
RXDXPHQWRGHVXDUHPXQHUDomR
†ž 'HFRUULGRRSUD]RGHTXLQ]HGLDVRVLOrQFLRGR
,, VHUYLGRUHVS~EOLFRVGR'LVWULWR)HGHUDOVHXUHJL *RYHUQDGRULPSRUWDUiVDQomR
PHMXUtGLFRSURYLPHQWRGHFDUJRVHVWDELOLGDGHH
DSRVHQWDGRULD †ž 6HRYHWRQmRIRUPDQWLGRVHUiRSURMHWRHQYLDGR
DR*RYHUQDGRUSDUDSURPXOJDomR
,,, RUJDQL]DomR GD 3URFXUDGRULD*HUDO GR'LVWULWR
)HGHUDO †ž (VJRWDGRVHPGHOLEHUDomRRSUD]RHVWDEHOHFLGR
QR$UW†žGD&RQVWLWXLomR)HGHUDORYHWR
,9 FULDomRHVWUXWXUDomRUHHVWUXWXUDomRGHVPHPEUD VHUiLQFOXtGRQDRUGHPGRGLDGDVHVVmRLPHGLDWD
PHQWRH[WLQomRLQFRUSRUDomRIXVmRHDWULEXLo}HV VREUHVWDGDV DV GHPDLV SURSRVLo}HV DWp D VXD
GDV6HFUHWDULDVGH*RYHUQRÐUJmRVHHQWLGDGHV YRWDomRILQDOVySRGHQGRVHUUHMHLWDGRSHORYRWR
GDDGPLQLVWUDomRS~EOLFD GDPDLRULDDEVROXWDGRV'HSXWDGRVHPYRWDomR
127$)LFDVXEVWLWXtGDDH[SUHVVmR´6HFUHW$5,$6GH*RYHUQRGR'LVWULWR)HGHUDOµ
SRU ´6HFUHWiU,$6 GH (VWDGR GR 'LVWULWR )HGHUDOµ &21)250( (PHQGD ­ /HL
RVWHQVLYD
2UJkQLFDQžGH'2')'( 129$ 5('$d®2 '$'$ $2 3$5É*5$)2 ž '2 $UW 3(/$ (0(1'$ ­ /(,
25*$1,&$1ž'( '2')'(

9 SODQR SOXULDQXDO RUoDPHQWR DQXDO HGLUHWUL]HV †ž 6HDOHLQmRIRUSURPXOJDGDHPTXDUHQWDHRLWR


RUoDPHQWiULDV KRUDVSHOR*RYHUQDGRUQRVFDVRVGRV††žHž
R3UHVLGHQWHGD&kPDUD/HJLVODWLYDDSURPXOJDUi
†ž 1mRVHUiREMHWRGHGHOLEHUDomRSURSRVWDTXHYLVH HVHHVWHQmRRIL]HUHPLJXDOSUD]RFDEHUiDR
D FRQFHGHU JUDWXLGDGH RX VXEVtGLR HP VHUYLoR 9LFH3UHVLGHQWHID]rOR
S~EOLFRSUHVWDGRGHIRUPDLQGLUHWDVHPDFRUUHV
SRQGHQWHLQGLFDomRGDIRQWHGHFXVWHLR †ž $ PDWpULD FRQVWDQWH GH SURMHWROHLUHMHLWDGR
VRPHQWHSRGHUiFRQVWLWXLUREMHWRGHQRYRSURMHWR
$UW1mRVHUiDGPLWLGRDXPHQWRGDGHVSHVDSUHYLVWD QDPHVPDVHVVmROHJLVODWLYDPHGLDQWHSURSRVWD
GD PDLRULD DEVROXWD GRV PHPEURV GD &kPDUD
, QRVSURMHWRVGHLQLFLDWLYDH[FOXVLYDGR*RYHUQDGRU /HJLVODWLYD
GR 'LVWULWR )HGHUDO UHVVDOYDGR R GLVSRVWR QR
$UW††žHžGD&RQVWLWXLomR)HGHUDO †ž &DVRRSURMHWRGHOHLVHMDYHWDGRGXUDQWHRUHFHV
VRGD&kPDUD/HJLVODWLYDR*RYHUQDGRUFRPXQL
,, QRV SURMHWRV VREUH RUJDQL]DomR GRVVHUYLoRV FDUiRYHWRjFRPLVVmRDTXHVHUHIHUHR$UW†
DGPLQLVWUDWLYRVGD&kPDUD/HJLVODWLYD žHGHSHQGHQGRGDXUJrQFLDHGDUHOHYkQFLDGD
PDWpULD SRGHUi FRQYRFDU D &kPDUD /HJLVODWLYD
$UW2*RYHUQDGRUGR'LVWULWR)HGHUDOSRGHVROLFLWDU SDUD VREUH HOH VH PDQLIHVWDU QRV WHUPRV GR
XUJrQFLD SDUD DSUHFLDomR GH SURMHWRV GH VXD $UW,9
LQLFLDWLYD
$UW$V OHLV FRPSOHPHQWDUHV VHUmR DSURYDGDVSRU
†ž 6H QD KLSyWHVH SUHYLVWD QR FDSXWD&kPDUD PDLRULD DEVROXWD GRV 'HSXWDGRV GD &kPDUD
/HJLVODWLYDQmRVHPDQLIHVWDUVREUHDSURSRVLomR /HJLVODWLYD H UHFHEHUmR QXPHUDomR GLVWLQWD GDV
HP DWp TXDUHQWD H FLQFR GLDV HVWD GHYHUi VHU OHLVRUGLQiULDV
LQFOXtGD QD 2UGHP GR 'LD VREUHVWDQGRVH D
GHOLEHUDomRTXDQWRDRVGHPDLVDVVXQWRVSDUDTXH †~QLFR 3DUDRVILQVGHVWHDUWLJRFRQVWLWXLUmROHLVFRPSOH
VHXOWLPHDYRWDomR PHQWDUHVHQWUHRXWUDV
†ž 2VSUD]RVGHTXHWUDWDRSDUiJUDIRDQWHULRUQmR , D OHL GH RUJDQL]DomR GR 7ULEXQDOGH&RQWDVGR
RFRUUHP QRV SHUtRGRV GH UHFHVVR GD &kPDUD 'LVWULWR)HGHUDO
/HJLVODWLYDQHPVHDSOLFDPDSURMHWRVGHFyGLJR
HGHHPHQGDVDHVWD/HL2UJkQLFD ,, RHVWDWXWRGRVVHUYLGRUHVS~EOLFRVFLYLV
$UW$SURYDGRRSURMHWRGHOHLQDIRUPDUHJLPHQWDO ,,, D OHL GH RUJDQL]DomR GD 3URFXUDGRULD*HUDOGR
VHUiHOHHQYLDGRDR*RYHUQDGRUTXHDTXLHVFHQGR 'LVWULWR)HGHUDO
RVDQFLRQDUiHSURPXOJDUi
,9 DOHLGRVLVWHPDWULEXWiULRGR'LVWULWR)HGHUDO
†ž 6HR*RYHUQDGRUGR'LVWULWR)HGHUDOFRQVLGHUDUR
SURMHWRGHOHLQRWRGRRXHPSDUWHLQFRQVWLWXFLR 9 D OHL TXH GLVS}H VREUH DV DWULEXLo}HV GR9L
QDO RX FRQWUiULR DR LQWHUHVVH S~EOLFR YHWiORi FH*RYHUQDGRUGR'LVWULWR)HGHUDO
WRWDO RX SDUFLDOPHQWH H FRPXQLFDUi GHQWUR GH
TXDUHQWD H RLWR KRUDV RV PRWLYRV GR YHWR DR 9, DOHLTXHGLVS}HVREUHDRUJDQL]DomRGRVLVWHPD
3UHVLGHQWHGD&kPDUD/HJLVODWLYD GHHGXFDomRGR'LVWULWR)HGHUDO
 'LU&RQVWLWXFLRQDO
 &kPDUD/HJLVODWLYDGR'LVWULWR)HGHUDO
9,, DOHLGHRUJDQL]DomRGDSUHYLGrQFLDGRVVHUYLGRUHV , DSUHFLDU DV FRQWDV DQXDLVGR*RYHUQDGRUID]HU
S~EOLFRVGR'LVWULWR)HGHUDO VREUH HODV UHODWyULRV DQDOtWLFR H HPLWLU SDUHFHU
SUpYLRQRSUD]RGHVHVVHQWDGLDVFRQWDGRVGRVHX
9,,,DOHLTXHGLVS}HVREUHRSODQRGLUHWRUGHRUGHQD UHFHELPHQWRGD&kPDUD/HJLVODWLYD
PHQWRWHUULWRULDOGR'LVWULWR)HGHUDO
LQFOXtGRVRVLQFLVRVL[[H[LSHOD(PHQGD­/HL2UJkQLFDQž ,, MXOJDVDVFRQWDV
GH'2')

D GRV DGPLQLVWUDGRUHV H GHPDLV UHVSRQViYHLV SRU


,; DOHLTXHGLVS}HVREUHD/HLGH8VRH2FXSDomRGR GLQKHLURVEHQVHYDORUHVGDDGPLQLVWUDomRGLUHWD
6ROR HLQGLUHWDRXTXHHVWHMDPVREVXDUHVSRQVDELOLGD
GH LQFOXtGRV RV GDV IXQGDo}HV H VRFLHGDGHV
; DOHLTXHGLVS}HVREUHR3ODQRGH3UHVHUYDomRGR LQVWLWXtGDV RX PDQWLGDV SHOR 3RGHU 3~EOLFR GR
&RQMXQWR8UEDQtVWLFRGH%UDVtOLD 'LVWULWR)HGHUDOEHPFRPRGDTXHOHVTXHGHUHP
FDXVDDSHUGDH[WUDYLRRXRXWUDLUUHJXODULGDGHGH
;, DOHLTXHGLVS}HVREUHR3ODQRGH'HVHQYROYLPHQ TXHUHVXOWHSUHMXt]RDRHUiULR
WR/RFDO
LQFOXLGRRLQFLVR;L,$R$UWSHOD(PHQGD­/HL2UJkQLFDQž
GH '2')GH E GRVGLULJHQWHVRXOLTXLGDQWHVGHHPSUHVDVLQFRU
SRUDGDVH[WLQWDVOLTXLGDGDVRXVRELQWHUYHQomR
;,, DOHLGHRUJDQL]DomRHIXQFLRQDPHQWRGD'HIHQVR RX TXH GH TXDOTXHU PRGR YHQKDP D LQWHJUDU
ULD3~EOLFDGR'LVWULWR)HGHUDO SURYLVyULD RX GHILQLWLYDPHQWH R SDWULP{QLR GR
'LVWULWR)HGHUDORXGHRXWUDHQWLGDGHGDDGPLQLV
6XEVHomR,,, WUDomRLQGLUHWD
'D,QLFLDWLYD3RSXODU
F GDTXHOHV TXH DVVXPDP REULJDo}HV GH QDWXUH]D
$UW$LQLFLDWLYDSRSXODUSRGHVHUH[HUFLGDSHODDSUH SHFXQLiULD HP QRPH GR 'LVWULWR )HGHUDO RX GH
VHQWDomRj&kPDUD/HJLVODWLYDGHHPHQGDj/HL HQWLGDGHGDDGPLQLVWUDomRLQGLUHWD
2UJkQLFDQDIRUPDGR$UW,,,RXGHSURMHWR
GHOHLGHYLGDPHQWHDUWLFXODGRMXVWLILFDGRHVXEV G GRVGLULJHQWHVGHHQWLGDGHVGRWDGDVGHSHUVRQDOL
FULWRSRUQRPtQLPRXPSRUFHQWRGRHOHLWRUDGR GDGH MXUtGLFD GH GLUHLWR SULYDGR TXH UHFHEDP
GR 'LVWULWR )HGHUDO GLVWULEXtGR SRU WUrV ]RQDV FRQWULEXLo}HVVXEYHQo}HVDX[tOLRVHDILQVDWpR
HOHLWRUDLV DVVHJXUDGD D GHIHVD GR SURMHWR SRU OLPLWHGRSDWULP{QLRWUDQVIHULGR
UHSUHVHQWDQWHVGRVUHVSHFWLYRVDXWRUHVSHUDQWHDV
FRPLVV}HVQDVTXDLVWUDPLWDU ,,, DSUHFLDUSDUD ILQV GH UHJLVWURDOHJDOLGDGHGRV
DWRVGHDGPLVVmRGHSHVVRDODTXDOTXHUWtWXORQD
6HomR9, DGPLQLVWUDomR GLUHWD H LQGLUHWD LQFOXtGDV DV
'D)LVFDOL]DomR&RQWiELOH)LQDQFHLUD IXQGDo}HV LQVWLWXtGDV H PDQWLGDV SHOR 3RGHU
3~EOLFRH[FHWXDGDVDVQRPHDo}HVSDUDFDUJRGH
6XEVHomR, SURYLPHQWRHPFRPLVVmREHPFRPRDGDVFRQFHV
'DV'LVSRVLo}HV*HUDLV V}HV GH DSRVHQWDGRULDV UHIRUPDV H SHQV}HV
UHVVDOYDGDV DV PHOKRULDV SRVWHULRUHV TXH QmR
$UW$ILVFDOL]DomRFRQWiELOILQDQFHLUDRUoDPHQWiULD DOWHUHPRIXQGDPHQWROHJDOGRDWRFRQFHVVyULR
RSHUDFLRQDO H SDWULPRQLDO GR 'LVWULWR )HGHUDO H
GDVHQWLGDGHVGDDGPLQLVWUDomRGLUHWDLQGLUHWDH ,9 DYDOLDUDH[HFXomRGDVPHWDVSUHYLVWDVQRSODQR
GDVIXQGDo}HVLQVWLWXtGDVRXPDQWLGDVSHOR3RGHU SOXULDQXDO QDV GLUHWUL]HV RUoDPHQWiULDV H QR
3~EOLFRTXDQWRjOHJDOLGDGHOHJLWLPLGDGHHFRQR RUoDPHQWRDQXDO
PLFLGDGHDSOLFDomRGHVXEYHQo}HVHUHQ~QFLDGH
UHFHLWDV VHUi H[HUFLGD SHOD &kPDUD /HJLVODWLYD 9 UHDOL]DUSRULQLFLDWLYDSUySULDGD&kPDUD/HJLVOD
PHGLDQWH FRQWUROH H[WHUQR H SHOR VLVWHPD GH WLYDRXGHDOJXPDGHVXDVFRPLVV}HVWpFQLFDVRX
FRQWUROHLQWHUQRGHFDGD3RGHU GHLQTXpULWRLQVSHo}HVHDXGLWRULDVGHQDWXUH]D
FRQWiELOILQDQFHLUDRUoDPHQWiULDRSHUDFLRQDOH
†~QLFR 3UHVWDUiFRQWDVTXDOTXHUSHVVRDItVLFDRXHQWLGDGH SDWULPRQLDO QDV XQLGDGHV DGPLQLVWUDWLYDV GRV
S~EOLFDTXHXWLOL]HDUUHFDGHJXDUGHJHUHQFLHRX 3RGHUHV([HFXWLYRH/HJLVODWLYRGR'LVWULWR)HGH
DGPLQLVWUHGLQKHLURVEHQVHYDORUHVS~EOLFRVRX UDO
SHORVTXDLVR'LVWULWR)HGHUDOUHVSRQGDRXTXHP
HPQRPHGHVWHDVVXPDREULJDo}HVGHQDWXUH]D D GDHVWLPDWLYDODQoDPHQWRDUUHFDGDomRUHFROKL
SHFXQLiULD PHQWRSDUFHODPHQWRHUHQ~QFLDGHUHFHLWDV

$UW2FRQWUROHH[WHUQRDFDUJRGD&kPDUD/HJLVODWL E GRV LQFHQWLYRV WUDQVDo}HV UHPLVV}HV H DQLVWLDV


YD VHUi H[HUFLGR FRP DX[tOLR GR 7ULEXQDO GH ILVFDLVLVHQo}HVVXEVtGLRVEHQHItFLRVHDILQVGH
&RQWDVGR'LVWULWR)HGHUDODRTXDOFRPSHWH QDWXUH]DILQDQFHLUDWULEXWiULDFUHGLWtFLDHRXWUDV
FRQFHGLGDVSHOR'LVWULWR)HGHUDO
&kPDUD/HJLVODWLYDGR
  'LVWULWR)HGHUDO 'LU&RQVWLWXFLRQDO 

F GDVGHVSHVDVGHLQYHVWLPHQWRHFXVWHLRLQFOXVLYH †ž 1RVFDVRVGHLUUHJXODULGDGHRXLOHJDOLGDGHFRQVWD


iFRQWDGHIXQGRHVSHFLDOGHQDWXUH]DFRQWiELORX WDGRVVHPLPSXWDomRGHGpELWRHPTXHR7ULEX
ILQDQFHLUD QDO GH &RQWDV GR 'LVWULWR )HGHUDO GHFLGLU QmR
DSOLFDURGLVSRVWRQRLQFLVR,;GHVWHDUWLJRGHYH
G GDV FRQFHVV}HV FHVV}HV GRDo}HV SHUPLVV}HV H UmRRVUHVSHFWLYRVYRWRVVHUSXEOLFDGRVMXQWDPHQ
FRQWUDWRVGHTXDOTXHUQDWXUH]DDWtWXORRQHURVR WHFRPDDWDGDVHVVmRHPTXHVHGHURMXOJDPHQ
RXJUDWXLWRHGDVVXEYHQo}HVVRFLDLVRXHFRQ{PL WR
FDVGRVDX[tOLRVFRQWULEXLo}HVHGRDo}HV
†ž $V GHFLV}HV GR 7ULEXQDO GH &RQWDVGR'LVWULWR
H GHRXWURVDWRVHSURFHGLPHQWRVGHTXHUHVXOWHP
)HGHUDOGHTXHUHVXOWHPLPSXWDomRGHGpELWRVRX
YDULDo}HVSDWULPRQLDLV
PXOWDWHUiHILFiFLDGHWtWXORH[HFXWLYR
9, ILVFDOL]DU DV DSOLFDo}HV GR 3RGHU 3~EOLFRHP
HPSUHVDVGHFXMRFDSLWDOVRFLDOR'LVWULWR)HGHUDO $UW$&kPDUD/HJLVODWLYDRXDFRPLVVmRFRPSHWHQWH
SDUWLFLSHGHIRUPDGLUHWDRXLQGLUHWDQRVWHUPRV GLDQWHGHLQGtFLRVGHGHVSHVDVQmRDXWRUL]DGDV
GRUHVSHFWLYRDWRFRQVWLWXWLYR DLQGDTXHVREIRUPDGHLQYHVWLPHQWRVQmRSURJUD
PDGRVRXGHLQFHQWLYRVLVHQo}HVDQLVWLDVUHPLV
9,, ILVFDOL]DUDDSOLFDomRGHTXDLVTXHUUHFXUVRVUHSDV V}HVVXEVtGLRVRXEHQHItFLRVGHQDWXUH]DILQDQFHL
VDGRVDR'LVWULWR)HGHUDORXSHOR'LVWULWR)HGHUDO UDWULEXWiULDRXFUHGLWtFLDQmRDSURYDGRVSRGHUi
PHGLDQWH FRQYrQLR DFRUGR DMXVWH RX RXWURV VROLFLWDUjDXWRULGDGHJRYHUQDPHQWDOUHVSRQViYHO
LQVWUXPHQWRVFRQJrQHUHV TXHQRSUD]RGHFLQFRGLDVSUHVWHHVFODUHFLPHQ
WRVQHFHVViULRV
9,,,SUHVWDU DV LQIRUPDo}HV VROLFLWDGDV SHOD&kPDUD
/HJLVODWLYD RX SRU TXDOTXHU GH VXDV FRPLVV}HV †ž 1mRSUHVWDGRVRVHVFODUHFLPHQWRVRXFRQVLGHUDGRV
WpFQLFDV RX GH LQTXpULWR VREUH D ILVFDOL]DomR HVWHV LQVXILFLHQWHV D &kPDUD /HJLVODWLYD RX D
FRQWiELOILQDQFHLUDRUoDPHQWiULDRSHUDFLRQDOH
FRPLVVmR FRPSHWHQWH VROLFLWDUi DR 7ULEXQDO GH
SDWULPRQLDO H VREUH UHVXOWDGRV GH DXGLWRULDV H
&RQWDVSURQXQFLDPHQWRFRQFOXVLYRVREUHDPDWp
LQVSHo}HVUHDOL]DGDV
ULDQRSUD]RGHWULQWDGLDV
,; DSOLFDUDRVUHVSRQViYHLVHPFDVRGHLOHJDOLGDGH
GHGHVSHVDRXLUUHJXODULGDGHGHFRQWDVDVVDQ †ž (QWHQGHQGR R 7ULEXQDO GH &RQWDVLUUHJXODUD
o}HVSUHYLVWDVHPOHLDTXDOHVWDEHOHFHUiHQWUH GHVSHVDDFRPLVVmRFRPSHWHQWHVHMXOJDUTXHR
RXWUDVFRPLQDo}HVPXOWDSURSRUFLRQDODRGDGR JDVWR SRVVD FDXVDU GDQR LUUHSDUiYHO RX JUDYH
FDXVDGRDRHUiULR OHVmR j HFRQRPLD S~EOLFD SURSRUi j &kPDUD
/HJLVODWLYDVXDVXVWDomRVHDLQGDQmRUHDOL]DGR
; DVVLQDUSUD]RTXHRyUJmRRXHQWLGDGHDGRWHDV RXVHXUHHPEROVRGHYLGDPHQWHDWXDOL]DGRPRQH
SURYLGrQFLDVQHFHVViULDVDRH[DWRFXPSULPHQWR WDULDPHQWH FRQVRDQWH UHJUDV YLJHQWHV VH Mi
GDOHLYHULILFDGDDLOHJDOLGDGH HIHWXDGR

;, VXVWDUVHQmRDWHQGLGRDH[HFXomRGRDWRLPSXJ †ž 27ULEXQDOGH&RQWDVGR'LVWULWR)HGHUDODJLUiGH


QDGRFRPXQLFDQGRDGHFLVmRj&kPDUD/HJLVODWL RItFLRRXPHGLDQWHLQLFLDWLYDGD&kPDUD/HJLVODWL
YD YD GR 0LQLVWpULR 3~EOLFR RX GDV DXWRULGDGHV
ILQDQFHLUDVHRUoDPHQWiULDVGR'LVWULWR)HGHUDORX
;,, UHSUHVHQWDUDR3RGHUFRPSHWHQWHVREUHLUUHJXODUL
GRVGHPDLVyUJmRVDX[LOLDUHVVHPSUHTXHKRXYHU
GDGHVRXDEXVRVDSXUDGRV
LQGtFLR GH LUUHJXODULGDGH HP TXDOTXHU GHVSHVD
;,9 DSUHFLDUHDSXUDUGHQ~QFLDVVREUHLUUHJXODULGDGHV LQFOXVLYHQDTXHODGHFRUUHQWHGHFRQWUDWR
HLOHJDOLGDGHVGRVDWRVVXMHLWRVDVHXFRQWUROH
$UW2V3RGHUHV/HJLVODWLYRH([HFXWLYRPDQWHUmRGH
†ž 1RFDVRGHFRQWUDWRRDWRGHVXVWDomRVHUiDGRWD IRUPDLQWHJUDGDVLVWHPDGHFRQWUROHLQWHUQRFRP
GR GLUHWDPHQWH SHOD &kPDUD /HJLVODWLYD TXH DILQDOLGDGHGH
VROLFLWDUi GH LPHGLDWR DR 3RGHU ([HFXWLYR DV
PHGLGDVFDEtYHLV , DYDOLDU R FXPSULPHQWR GDVPHWDVSUHYLVWDVQR
SODQRSOXULDQXDODH[HFXomRGRVSURJUDPDVGH
†ž 6HD&kPDUD/HJLVODWLYDRXR3RGHU([HFXWLYRQR JRYHUQRHGRVRUoDPHQWRVGR'LVWULWR)HGHUDO
SUD]RGHQRYHQWDGLDVQmRHIHWLYDUDV PHGLGDV
SUHYLVWDVQRSDUiJUDIRDQWHULRUR7ULEXQDOGHFLGL ,, FRPSURYDU D OHJDOLGDGH H DYDOLDU RVUHVXOWDGRV
UiGDTXHVWmR TXDQWRjHILFiFLDHHILFLrQFLDGDJHVWmRRUoDPHQ
WiULD ILQDQFHLUD FRQWiELO H SDWULPRQLDO QRV
†ž 2 7ULEXQDO HQFDPLQKDUi j&kPDUD/HJLVODWLYD
yUJmRVHHQWLGDGHVGDDGPLQLVWUDomRGR'LVWULWR
WULPHVWUDOHDQXDOPHQWHUHODWyULRFLUFXQVWDQFLDGR
)HGHUDO H TXDQWR j GD DSOLFDomR GH UHFXUVRV
H GHPRQVWUDWLYR GDV DWLYLGDGHV LQWHUQDV H GH
FRQWUROHH[WHUQRUHDOL]DGDV S~EOLFRVSRUHQWLGDGHVGHGLUHLWRSULYDGR
 'LU&RQVWLWXFLRQDO
 &kPDUD/HJLVODWLYDGR'LVWULWR)HGHUDO
,,, H[HUFHURFRQWUROHVREUHRGHIHULPHQWRGHYDQWD 6XEVHomR,,
JHQV H D IRUPD GH FDOFXODU TXDOTXHU SDUFHOD 'R7ULEXQDOGH&RQWDV
LQWHJUDQWHGDUHPXQHUDomRYHQFLPHQWRRXVDOiULR
GHVHXVPHPEURVRXVHUYLGRUHV $UW 27ULEXQDOGH&RQWDVGR'LVWULWR)HGHUDOLQWHJUD
GRSRUVHWH&RQVHOKHLURVWHPVHGHQDFLGDGHGH
,9 H[HUFHURFRQWUROHGDVRSHUDo}HVGHFUpGLWRDYDLV %UDVtOLD TXDGUR SUySULR GH SHVVRDO H MXULVGLomR
HJDUDQWLDVEHPFRPRRVGRVGLUHLWRVHKDYHUHV HPWRGRRWHUULWyULRGR'LVWULWR)HGHUDOH[HUFHQ
GR'LVWULWR)HGHUDO GR QR TXH FRXEHU DV DWULEXLo}HV SUHYLVWDV QR
$UWGD&RQVWLWXLomR)HGHUDO
9 DYDOLDUDUHODomRGHFXVWRHEHQHItFLRGDVUHQ~QFL
DVGHUHFHLWDVHGRVLQFHQWLYRVUHPLVV}HVSDUFHOD †ž 2V &RQVHOKHLURV GR 7ULEXQDOVHUmRQRPHDGRV
PHQWRV GH GtYLGDVDQLVWLDVLVHQo}HVVXEVtGLRV HQWUH EUDVLOHLURV TXH VDWLVIDoDP RV VHJXLQWHV
EHQHItFLRVHDILQVGHQDWXUH]DILQDQFHLUDWULEXWi UHTXLVLWRV
ULDFUHGLWtFLDHRXWURV
, PDLVGHWULQWDHFLQFRHPHQRVGHVHVVHQWDHFLQFR
9, DSRLDU R FRQWUROH H[WHUQR QR H[HUFtFLR GHVXD DQRVGHLGDGH
PLVVmRLQVWLWXFLRQDO ,, LGRQHLGDGHPRUDOHUHSXWDomRLOLEDGD
,,, QRWiYHLV FRQKHFLPHQWRV MXUtGLFRVFRQWiEHLV
†ž 2VUHVSRQViYHLVSHORFRQWUROHLQWHUQRDRWRPD HFRQ{PLFRV H ILQDQFHLURV RX GH DGPLQLVWUDomR
UHP FRQKHFLPHQWR GH TXDOTXHU LUUHJXODULGDGH S~EOLFD
LOHJDOLGDGHRXRIHQVDDRVSULQFtSLRVGR$UWGD ,9 PDLVGHGH]DQRV GH H[HUFtFLR GHIXQomRRXGH
&RQVWLWXLomR)HGHUDOGHODGDUmRFLrQFLDDR7ULEX HIHWLYDDWLYLGDGHSURILVVLRQDOTXHH[LMDRVFRQKHFL
QDO GH &RQWDV GR 'LVWULWR )HGHUDO VRE SHQD GH PHQWRVPHQFLRQDGRVQRLWHPDQWHULRU
UHVSRQVDELOLGDGHVROLGiULD
†ž 2V&RQVHOKHLURVGR7ULEXQDOGH&RQWDVGR'LVWULWR
†ž $V FRQWDV S~EOLFDV GR 'LVWULWR)HGHUDOILFDUmR )HGHUDOVHUmRHVFROKLGRV
GXUDQWH VHVVHQWD GLDV DQXDOPHQWH HP ORFDO
SUySULR GD &kPDUD /HJLVODWLYD j GLVSRVLomR GH , WUrVSHOR*RYHUQDGRUGR'LVWULWR)HGHUDOFRPD
TXDOTXHUFRQWULEXLQWHSDUDH[DPHHDSUHFLDomRH DSURYDomRGD&kPDUD/HJLVODWLYDVHQGRXPGHOL
VHUmR GLVSRQLELOL]DGDV GH PDQHLUD SHUPDQHQWH YUHHVFROKDHGRLVDOWHUQDGDPHQWHGHQWUHDXGLWR
DWXDOL]DGDV PHQVDOPHQWH QRV VtWLRV RILFLDLV QD UHVHPHPEURVGR0LQLVWpULR3~EOLFRMXQWRDR7UL
LQWHUQHWGR3RGHU/HJLVODWLYRGR3RGHU([HFXWLYR EXQDOLQGLFDGRVHPOLVWDWUtSOLFHSHOR7ULEXQDO
H GR 7ULEXQDO GH &RQWDV GR 'LVWULWR )HGHUDO VHJXQGRRVFULWpULRVGHDQWLJXLGDGHHPHUHFLPHQWR
UHFRPHQGDQGRVHDFULDomRGHVtWLRVHVSHFtILFRVQD
LQWHUQHWSDUDDSXEOLFDomRSHUPDQHQWHGDVFRQWDV ,, TXDWURSHOD&kPDUD/HJLVODWLYD
1RYD5HGDomRGDGDDRVLQFLVRV,H,,GR†žGRDUW(PHQGDD/HL2UJkQLFDQž
S~EOLFDV GH IRUPD FODUD H FRPSUHHQVtYHO DR GHGHMDQHLURGHSXEOLFDGDQR'2')GH
FLGDGmR
QRYDUHGDomRGDGDDR†žGR$UWSHODHPHQGDjOHLRUJkQLFDQžGH
'2')GH †ž 5HYRJDGR†žGRDUW(PHQGDD/HL2UJkQLFDQžGH
GHMDQHLURGHSXEOLFDGDQR'2')GH
†ž 4XDOTXHUFLGDGmRSDUWLGRSROtWLFRDVVRFLDomRRX
HQWLGDGHVLQGLFDOpSDUWHOHJtWLPDSDUDQDIRUPD †ž 2V&RQVHOKHLURVGR7ULEXQDOGH&RQWDVWHUmRDV
GDOHLGHQXQFLDULUUHJXODULGDGHVDR7ULEXQDOGH PHVPDVJDUDQWLDVSUHUURJDWLYDVLPSHGLPHQWRV
&RQWDVRXj&kPDUD/HJLVODWLYD YHQFLPHQWRV H YDQWDJHQV GRV 'HVHPEDUJDGRUHV
GR 7ULEXQDO GH -XVWLoD GR 'LVWULWR )HGHUDO H
†ž $SUHVWDomRGHFRQWDVDQXDOGR*RYHUQDGRUHDV 7HUULWyULRVQDIRUPDGD&RQVWLWXLomR)HGHUDOH
WRPDGDV RX SUHVWDo}HV GH FRQWDV DQXDLV GRV VRPHQWHSRGHUmRDSRVHQWDUVHFRPDVYDQWDJHQV
DGPLQLVWUDGRUHVGRVyUJmRVHHQWLGDGHVGR'LVWUL GRFDUJRTXDQWRRWLYHUHPH[HUFLGRHIHWLYDPHQWH
WR)HGHUDOGHYHUmRVHUDFRPSDQKDGDVGHUHODWyULR SRUPDLVGHFLQFRDQRV
FLUFXQVWDQFLDGR GR yUJmR GH FRQWUROH LQWHUQR
VREUHRUHVXOWDGRGDVDWLYLGDGHVLQGLFDGDVQHVWH †ž 2V&RQVHOKHLURVQDVVXDVIDOWDVHLPSHGLPHQWRV
DUWLJR VHUmRVXEVWLWXtGRVSRU$XGLWRUHVQDIRUPDGDOHL

$UW 27ULEXQDOGH&RQWDVGR'LVWULWR)HGHUDOSUHVWDUi †ž 2$XGLWRUTXDQGRHPVXEVWLWXLomRD&RQVHOKHLUR


FRQWDVDQXDOPHQWHGHVXDH[HFXomRRUoDPHQWiULD WHUiDVPHVPDVJDUDQWLDVSUHUURJDWLYDVHLPSHGL
ILQDQFHLUDHSDWULPRQLDOj&kPDUD/HJLVODWLYDDWp PHQWRV GR WLWXODU H QR H[HUFtFLR GDV GHPDLV
VHVVHQWD GLDV GD GDWD GD DEHUWXUD GD VHVVmR GR DWULEXLo}HVGDMXGLFDWXUDDVGH-XL]GH'LUHLWRGD
DQRVHJXLQWHjTXHOHDTXHVHUHIHULURH[HUFtFLR -XVWLoDGR'LVWULWR)HGHUDOH7HUULWyULRV
ILQDQFHLUR TXDQWR DRV DVSHFWRV GH OHJDOLGDGH
OHJLWLPLGDGH H HFRQRPLFLGDGH REVHUYDGRV RV †ž 2V&RQVHOKHLURVGR7ULEXQDOGH&RQWDVGR'LVWULWR
GHPDLVSUHFHLWRVOHJDLV )HGHUDOIDUmRGHFODUDomRS~EOLFDGHEHQVQRDWR
GDSRVVHHQRWpUPLQRGRH[HUFtFLRGRFDUJR
&kPDUD/HJLVODWLYDGR
 'LVWULWR)HGHUDO 'LU&RQVWLWXFLRQDO 

†ž 2V&RQVHOKHLURVGR7ULEXQDOGH&RQWDVGR'LVWULWR
)HGHUDO QRV FDVRV GH FULPH FRPXP H QRV GH /HLIHGHUDOQž
UHVSRQVDELOLGDGH VHUmR SURFHVVDGRV H MXOJDGRV
RULJLQDULDPHQWHSHOR6XSHULRU7ULEXQDOGH-XVWL TXHUHJXODRDFHVVRDLQIRUPDo}HV
oD HPkPELWRQDFLRQDO

†ž eSURLELGDDQRPHDomRSDUDRFDUJRGH&RQVHOKHL
URGR7ULEXQDOGH&RQWDVGR'LVWULWR)HGHUDOGH 5HJXODRDFHVVRDLQIRUPDo}HVSUHYLVWRQRLQFLVR;;;,,,GRDUW
SHVVRDTXHWHQKDSUDWLFDGRDWRWLSLILFDGRFRPR ž QR LQFLVR ,, GR † ž GR DUW  H QR † ž GR DUW  GD
FDXVD GH LQHOHJLELOLGDGH SUHYLVWD QD OHJLVODomR &RQVWLWXLomR)HGHUDODOWHUDD/HLQžGHGHGH]HPEURGH
HOHLWRUDO UHYRJDD/HLQžGHGHPDLRGHHGLVSRVLWL
ILFDDFUHVFHQWDGRR†žDR$UWSHODHPHQGDjOHLRUJkQLFD YRVGD/HLQžGHGHMDQHLURGHHGiRXWUDVSURYL
QžGH'2')GH GrQFLDV
&$3Ì78/2,
$UW 2V&RQVHOKHLURVGR7ULEXQDOGH&RQWDVGR'LVWULWR ',6326,d¯(6*(5$,6
)HGHUDODLQGDTXHHPGLVSRQLELOLGDGHQmRSRGH
UmRH[HUFHURXWUDIXQomRS~EOLFDQHPTXDOTXHU $UWž (VWD/HLGLVS}HVREUHRVSURFHGLPHQWRVDVHUHPREVHU
SURILVVmRUHPXQHUDGDVDOYRXPDGHPDJLVWpULR YDGRVSHOD8QLmR(VWDGRV'LVWULWR)HGHUDOH0XQLFtSL
QHPUHFHEHUDTXDOTXHUWtWXORRXSUHWH[WRSDUWL RV FRP R ILP GH JDUDQWLU R DFHVVR D LQIRUPDo}HV
FLSDomR QRV SURFHVVRV EHP FRPR GHGLFDUVH j SUHYLVWRQRLQFLVR;;;,,,GRDUWRQRLQFLVR,,GR†ž
DWLYLGDGHSROtWLFRSDUWLGiULDVRESHQDGHSHUGD GRDUWHQR†žGRDUWGD&RQVWLWXLomR)HGH
GRFDUJR UDO

†~QLFR 6XERUGLQDPVHDRUHJLPHGHVWD/HL
$UW eGDFRPSHWrQFLDH[FOXVLYDGR7ULEXQDOGH&RQWDV
GR'LVWULWR)HGHUDO
, RVyUJmRVS~EOLFRVLQWHJUDQWHVGDDGPLQLVWUDomRGLUHWD
GRV3RGHUHV([HFXWLYR/HJLVODWLYRLQFOXLQGRDV&RUWHV
, HODERUDUDSURYDUHDOWHUDUVHXUHJLPHQWRLQWHUQR GH&RQWDVH-XGLFLiULRHGR0LQLVWpULR3~EOLFR

,, RUJDQL]DU VHXV VHUYLoRV DX[LOLDUHV H SURYHURV ,, DV DXWDUTXLDV DV IXQGDo}HV S~EOLFDVDVHPSUHVDV
UHVSHFWLYRVFDUJRVRFXSDGRVDTXHOHVHPFRPLVVmR S~EOLFDVDVVRFLHGDGHVGHHFRQRPLDPLVWDHGHPDLV
SUHIHUHQFLDOPHQWHSRUVHUYLGRUHVGHFDUUHLUDGR HQWLGDGHV FRQWURODGDV GLUHWD RX LQGLUHWDPHQWH SHOD
SUySULRWULEXQDOQRVFDVRVHFRQGLo}HVTXHGHYH 8QLmR(VWDGRV'LVWULWR)HGHUDOH0XQLFtSLRV
UmRVHUSUHYLVWRVHPVXDOHLGHRUJDQL]DomR
$UWž $SOLFDPVHDVGLVSRVLo}HVGHVWD/HLQRTXHFRXEHUjV
,,, FRQFHGHUOLFHQoDIpULDVHRXWURVDIDVWDPHQWRVD HQWLGDGHVSULYDGDVVHPILQVOXFUDWLYRVTXHUHFHEDP
&RQVHOKHLURVH$XGLWRUHV SDUDUHDOL]DomRGHDo}HVGHLQWHUHVVHS~EOLFRUHFXUVRV
S~EOLFRV GLUHWDPHQWH GR RUoDPHQWR RX PHGLDQWH
 ,9 SURSRUj&kPDUD/HJLVODWLYDDFULDomRWUDQVIRU VXEYHQo}HV VRFLDLV FRQWUDWR GH JHVWmR WHUPR GH
SDUFHULDFRQYrQLRVDFRUGRDMXVWHVRXRXWURVLQVWUX
PDomRHH[WLQomRGHFDUJRVHDIL[DomRGRVUHVSHF
PHQWRVFRQJrQHUHV
WLYRVYHQFLPHQWRV
†~QLFR $ SXEOLFLGDGH D TXH HVWmR VXEPHWLGDVDVHQWLGDGHV
9 HODERUDUVXDSURSRVWDRUoDPHQWiULDREVHUYDGRV FLWDGDV QR FDSXW UHIHUHVH j SDUFHOD GRV UHFXUVRV
RV SULQFtSLRV HVWDEHOHFLGRV QD OHL GH GLUHWUL]HV S~EOLFRVUHFHELGRVHjVXDGHVWLQDomRVHPSUHMXt]RGDV
RUoDPHQWiULDV SUHVWDo}HVGHFRQWDVDTXHHVWHMDPOHJDOPHQWHREULJD
GDV
$UW )XQFLRQDUiMXQWRDR7ULEXQDOGH&RQWDVR0LQLVWp
ULR3~EOLFRUHJLGRSHORVSULQFtSLRVLQVWLWXFLRQDLV $UWž 2V SURFHGLPHQWRV SUHYLVWRV QHVWD /HL GHVWLQDPVHD
GH XQLGDGH LQGLYLVLELOLGDGH H LQGHSHQGrQFLD DVVHJXUDURGLUHLWRIXQGDPHQWDOGHDFHVVRjLQIRUPD
IXQFLRQDOFRPDVDWULEXLo}HVGHJXDUGDGDOHLH omRHGHYHPVHUH[HFXWDGRVHPFRQIRUPLGDGHFRPRV
ILVFDOGHVXDH[HFXomR SULQFtSLRVEiVLFRVGDDGPLQLVWUDomRS~EOLFDHFRPDV
ILFDDFUHVFHQWDGRR†~QLFRDR$UWSHODHPHQGDjOHLRUJkQLFDQžGH VHJXLQWHVGLUHWUL]HV
'2')GH

, REVHUYkQFLDGDSXEOLFLGDGHFRPRSUHFHLWRJHUDOHGR
†~QLFR $SURLELomRGHTXHWUDWDR$UW†ƒDSOLFDVH VLJLORFRPRH[FHomR
j QRPHDomR GR 3URFXUDGRU*HUDO GR 0LQLVWpULR
3~EOLFRGH&RQWDVGR'LVWULWR)HGHUDO ,, GLYXOJDomR GH LQIRUPDo}HV GHLQWHUHVVHS~EOLFR
LQGHSHQGHQWHPHQWHGHVROLFLWDo}HV
$UW/HL FRPSOHPHQWDU GR 'LVWULWR )HGHUDOGLVSRUi
VREUHDRUJDQL]DomRHIXQFLRQDPHQWRGR7ULEXQDO ,,, XWLOL]DomRGHPHLRVGHFRPXQLFDomRYLDELOL]DGRVSHOD
WHFQRORJLDGDLQIRUPDomR
GH&RQWDVSRGHQGRGLYLGLORHPFkPDUDVHFULDU
GHOHJDo}HVRXyUJmRVGHVWLQDGRVDDX[LOLiORQR ,9 IRPHQWRDRGHVHQYROYLPHQWRGDFXOWXUDGHWUDQVSDUrQ
H[HUFtFLRGHVXDVIXQo}HVHQDGHVFHQWUDOL]DomR FLDQDDGPLQLVWUDomRS~EOLFD
GRVVHXVWUDEDOKRV
 'LU&RQVWLWXFLRQDO &kPDUD/HJLVODWLYDGR'LVWULWR)HGHUDO


9 GHVHQYROYLPHQWRGRFRQWUROHVRFLDOGDDGPLQLVWUDomR , RULHQWDomRVREUHRVSURFHGLPHQWRVSDUDDFRQVHFXomR


S~EOLFD GH DFHVVR EHP FRPR VREUH R ORFDO RQGH SRGHUi VHU
HQFRQWUDGDRXREWLGDDLQIRUPDomRDOPHMDGD
$UWž 3DUDRVHIHLWRVGHVWD/HLFRQVLGHUDVH
,, LQIRUPDomR FRQWLGD HP UHJLVWURVRXGRFXPHQWRV
, LQIRUPDomRGDGRVSURFHVVDGRVRXQmRTXHSRGHPVHU SURGX]LGRVRXDFXPXODGRVSRUVHXVyUJmRVRXHQWLGD
XWLOL]DGRVSDUDSURGXomRHWUDQVPLVVmRGHFRQKHFLPHQ GHVUHFROKLGRVRXQmRDDUTXLYRVS~EOLFRV
WRFRQWLGRVHPTXDOTXHUPHLRVXSRUWHRXIRUPDWR
,,, LQIRUPDomRSURGX]LGDRXFXVWRGLDGDSRUSHVVRDItVLFD
,, GRFXPHQWR XQLGDGH GH UHJLVWURGHLQIRUPDo}HV RX HQWLGDGHSULYDGDGHFRUUHQWHGHTXDOTXHUYtQFXOR
TXDOTXHUTXHVHMDRVXSRUWHRXIRUPDWR FRPVHXVyUJmRVRXHQWLGDGHVPHVPRTXHHVVHYtQFXOR
MiWHQKDFHVVDGR
,,, LQIRUPDomRVLJLORVDDTXHODVXEPHWLGDWHPSRUDULDPHQ
WH j UHVWULomR GH DFHVVR S~EOLFR HP UD]mR GH VXD ,9 LQIRUPDomRSULPiULDtQWHJUDDXWrQWLFDHDWXDOL]DGD
LPSUHVFLQGLELOLGDGHSDUDDVHJXUDQoDGDVRFLHGDGHH
GR(VWDGR 9 LQIRUPDomRVREUHDWLYLGDGHVH[HUFLGDVSHORVyUJmRVH
HQWLGDGHVLQFOXVLYHDVUHODWLYDVjVXDSROtWLFDRUJDQL
,9 LQIRUPDomR SHVVRDO DTXHOD UHODFLRQDGD jSHVVRD
]DomRHVHUYLoRV
QDWXUDOLGHQWLILFDGDRXLGHQWLILFiYHO
9, LQIRUPDomRSHUWLQHQWHjDGPLQLVWUDomRGRSDWULP{QLR
9 WUDWDPHQWRGDLQIRUPDomRFRQMXQWRGHDo}HVUHIHUHQ
S~EOLFR XWLOL]DomR GH UHFXUVRV S~EOLFRV OLFLWDomR
WHV j SURGXomR UHFHSomR FODVVLILFDomR XWLOL]DomR
FRQWUDWRVDGPLQLVWUDWLYRVH
DFHVVRUHSURGXomRWUDQVSRUWHWUDQVPLVVmRGLVWULEXL
omR DUTXLYDPHQWR DUPD]HQDPHQWR HOLPLQDomR
DYDOLDomRGHVWLQDomRRXFRQWUROHGDLQIRUPDomR 9,, LQIRUPDomRUHODWLYD

9, GLVSRQLELOLGDGHTXDOLGDGHGDLQIRUPDomRTXHSRGHVHU D j LPSOHPHQWDomR DFRPSDQKDPHQWR H UHVXOWDGRV


FRQKHFLGDHXWLOL]DGDSRULQGLYtGXRVHTXLSDPHQWRVRX GRV SURJUDPDV SURMHWRV H Do}HV GRV yUJmRV H
VLVWHPDVDXWRUL]DGRV HQWLGDGHVS~EOLFDVEHPFRPRPHWDVHLQGLFDGRUHV
SURSRVWRV
9,, DXWHQWLFLGDGHTXDOLGDGHGDLQIRUPDomRTXHWHQKDVLGR
SURGX]LGD H[SHGLGD UHFHELGD RX PRGLILFDGD SRU E DRUHVXOWDGRGHLQVSHo}HVDXGLWRULDVSUHVWDo}HVH
GHWHUPLQDGRLQGLYtGXRHTXLSDPHQWRRXVLVWHPD WRPDGDV GH FRQWDV UHDOL]DGDV SHORV yUJmRV GH
FRQWUROHLQWHUQRHH[WHUQRLQFOXLQGRSUHVWDo}HVGH
9,,, LQWHJULGDGHTXDOLGDGHGDLQIRUPDomRQmRPRGLILFDGD FRQWDVUHODWLYDVDH[HUFtFLRVDQWHULRUHV
LQFOXVLYHTXDQWRjRULJHPWUkQVLWRHGHVWLQR
†ž 2DFHVVRjLQIRUPDomRSUHYLVWRQRFDSXWQmRFRPSUH
,; SULPDULHGDGH TXDOLGDGH GD LQIRUPDomR FROHWDGDQD HQGHDVLQIRUPDo}HVUHIHUHQWHVDSURMHWRVGHSHVTXLVD
IRQWHFRPRPi[LPRGHGHWDOKDPHQWRSRVVtYHOVHP H GHVHQYROYLPHQWR FLHQWtILFRV RX WHFQROyJLFRV FXMR
PRGLILFDo}HV VLJLORVHMDLPSUHVFLQGtYHOjVHJXUDQoDGDVRFLHGDGHH
GR(VWDGR
$UWž e GHYHU GR (VWDGR JDUDQWLU R GLUHLWR GH DFHVVRj
LQIRUPDomRTXHVHUiIUDQTXHDGDPHGLDQWHSURFHGL †ž 4XDQGRQmRIRUDXWRUL]DGRDFHVVRLQWHJUDOjLQIRUPD
PHQWRVREMHWLYRVHiJHLVGHIRUPDWUDQVSDUHQWHFODUD omRSRUVHUHODSDUFLDOPHQWHVLJLORVDpDVVHJXUDGRR
HHPOLQJXDJHPGHIiFLOFRPSUHHQVmR DFHVVR j SDUWH QmR VLJLORVD SRU PHLR GH FHUWLGmR
H[WUDWRRXFySLDFRPRFXOWDomRGDSDUWHVREVLJLOR

&$3Ì78/2,, †ž 2GLUHLWRGHDFHVVRDRVGRFXPHQWRVRXjVLQIRUPDo}HV


'2$&(662$,1)250$d¯(6 QHOHVFRQWLGDVXWLOL]DGRVFRPRIXQGDPHQWRGDWRPDGD
('$68$',98/*$d®2 GH GHFLVmR H GR DWR DGPLQLVWUDWLYR VHUi DVVHJXUDGR
FRPDHGLomRGRDWRGHFLVyULRUHVSHFWLYR
$UWž &DEHDRVyUJmRVHHQWLGDGHVGRSRGHUS~EOLFRREVHUYD
GDVDVQRUPDVHSURFHGLPHQWRVHVSHFtILFRVDSOLFiYHLV †ž $QHJDWLYDGHDFHVVRjVLQIRUPDo}HVREMHWRGHSHGLGR
DVVHJXUDUD IRUPXODGRDRVyUJmRVHHQWLGDGHVUHIHULGDVQRDUWR
TXDQGRQmRIXQGDPHQWDGDVXMHLWDUiRUHVSRQViYHOD
, JHVWmRWUDQVSDUHQWHGDLQIRUPDomRSURSLFLDQGRDPSOR PHGLGDVGLVFLSOLQDUHVQRVWHUPRVGRDUWGHVWD/HL
DFHVVRDHODHVXDGLYXOJDomR
†ž ,QIRUPDGR GR H[WUDYLR GDLQIRUPDomRVROLFLWDGD
,, SURWHomRGDLQIRUPDomRJDUDQWLQGRVHVXDGLVSRQLELOL SRGHUiRLQWHUHVVDGRUHTXHUHUjDXWRULGDGHFRPSHWHQ
GDGHDXWHQWLFLGDGHHLQWHJULGDGHH WH D LPHGLDWD DEHUWXUD GH VLQGLFkQFLD SDUD DSXUDU R
GHVDSDUHFLPHQWRGDUHVSHFWLYDGRFXPHQWDomR
,,, SURWHomR GD LQIRUPDomR VLJLORVD H GDLQIRUPDomR
SHVVRDOREVHUYDGDDVXDGLVSRQLELOLGDGHDXWHQWLFLGD †ž 9HULILFDGDDKLSyWHVHSUHYLVWDQR†RGHVWHDUWLJRR
GHLQWHJULGDGHHHYHQWXDOUHVWULomRGHDFHVVR UHVSRQViYHO SHOD JXDUGD GD LQIRUPDomR H[WUDYLDGD
GHYHUiQRSUD]RGH GH] GLDVMXVWLILFDURIDWRH
$UWž 2DFHVVRjLQIRUPDomRGHTXHWUDWDHVWD/HLFRPSUHHQ LQGLFDUWHVWHPXQKDVTXHFRPSURYHPVXDDOHJDomR
GHHQWUHRXWURVRVGLUHLWRVGHREWHU
&kPDUD/HJLVODWLYDGR
  'LVWULWR)HGHUDO 'LU&RQVWLWXFLRQDO 

$UWž e GHYHU GRV yUJmRV H HQWLGDGHV S~EOLFDVSURPRYHU †ž 2V0XQLFtSLRVFRPSRSXODomRGHDWp GH]PLO


LQGHSHQGHQWHPHQWH GH UHTXHULPHQWRV D GLYXOJDomR KDELWDQWHVILFDPGLVSHQVDGRVGDGLYXOJDomRREULJDWyULD
HPORFDOGHIiFLODFHVVRQRkPELWRGHVXDVFRPSHWrQ QDLQWHUQHWDTXHVHUHIHUHR†RPDQWLGDDREULJDWR
FLDVGHLQIRUPDo}HVGHLQWHUHVVHFROHWLYRRXJHUDOSRU ULHGDGHGHGLYXOJDomRHPWHPSRUHDOGHLQIRUPDo}HV
HOHVSURGX]LGDVRXFXVWRGLDGDV UHODWLYDV j H[HFXomR RUoDPHQWiULD H ILQDQFHLUD QRV
FULWpULRVHSUD]RVSUHYLVWRVQRDUW%GD/HL&RPSOH
†ž 1DGLYXOJDomRGDVLQIRUPDo}HVDTXHVHUHIHUHRFDSXW PHQWDUQRGHGHPDLRGH /HLGH5HVSRQ
GHYHUmRFRQVWDUQRPtQLPR VDELOLGDGH)LVFDO 

, UHJLVWURGDVFRPSHWrQFLDVHHVWUXWXUDRUJDQL]DFLRQDO $UWž 2 DFHVVR D LQIRUPDo}HV S~EOLFDV VHUiDVVHJXUDGR


HQGHUHoRV H WHOHIRQHV GDV UHVSHFWLYDV XQLGDGHV H PHGLDQWH
KRUiULRVGHDWHQGLPHQWRDRS~EOLFR
, FULDomR GH VHUYLoR GH LQIRUPDo}HVDRFLGDGmRQRV
,, UHJLVWURV GH TXDLVTXHU UHSDVVHV RXWUDQVIHUrQFLDVGH yUJmRV H HQWLGDGHV GR SRGHU S~EOLFR HP ORFDO FRP
UHFXUVRVILQDQFHLURV FRQGLo}HVDSURSULDGDVSDUD

,,, UHJLVWURVGDVGHVSHVDV D DWHQGHU H RULHQWDU R S~EOLFR TXDQWR DR DFHVVR D


LQIRUPDo}HV
,9 LQIRUPDo}HVFRQFHUQHQWHVDSURFHGLPHQWRVOLFLWDWyULRV
LQFOXVLYHRVUHVSHFWLYRVHGLWDLVHUHVXOWDGRVEHPFRPR E LQIRUPDU VREUH D WUDPLWDomR GH GRFXPHQWRV QDV
DWRGRVRVFRQWUDWRVFHOHEUDGRV VXDVUHVSHFWLYDVXQLGDGHV

9 GDGRVJHUDLVSDUDRDFRPSDQKDPHQWRGHSURJUDPDV F SURWRFROL]DUGRFXPHQWRVHUHTXHULPHQWRVGHDFHVVR


Do}HVSURMHWRVHREUDVGHyUJmRVHHQWLGDGHVH DLQIRUPDo}HVH
9, UHVSRVWDVDSHUJXQWDVPDLVIUHTXHQWHVGDVRFLHGDGH
,, UHDOL]DomRGHDXGLrQFLDVRXFRQVXOWDVS~EOLFDVLQFHQWL
YR j SDUWLFLSDomR SRSXODU RX D RXWUDV IRUPDV GH
†ž 3DUDFXPSULPHQWRGRGLVSRVWRQRFDSXWRVyUJmRVH
GLYXOJDomR
HQWLGDGHVS~EOLFDVGHYHUmRXWLOL]DUWRGRVRVPHLRVH
LQVWUXPHQWRV OHJtWLPRV GH TXH GLVSXVHUHP VHQGR
REULJDWyULD D GLYXOJDomR HP VtWLRV RILFLDLV GD UHGH
&$3Ì78/2,,,
PXQGLDOGHFRPSXWDGRUHV LQWHUQHW 
'2352&(',0(172'($&(662­,1)250$d®2
†ž 2V VtWLRV GH TXH WUDWD R † R GHYHUmR QDIRUPDGH
UHJXODPHQWR DWHQGHU HQWUH RXWURV DRV VHJXLQWHV 6HomR,
UHTXLVLWRV 'R3HGLGRGH$FHVVR

, FRQWHU IHUUDPHQWD GH SHVTXLVDGHFRQWH~GRTXH $UW 4XDOTXHU LQWHUHVVDGR SRGHUi DSUHVHQWDU SHGLGRGH
SHUPLWD R DFHVVR j LQIRUPDomR GH IRUPD REMHWLYD DFHVVRDLQIRUPDo}HVDRVyUJmRVHHQWLGDGHVUHIHULGRV
WUDQVSDUHQWHFODUDHHPOLQJXDJHPGHIiFLOFRPSUHHQ QR DUW R GHVWD /HL SRU TXDOTXHU PHLR OHJtWLPR
VmR GHYHQGRRSHGLGRFRQWHUDLGHQWLILFDomRGRUHTXHUHQWH
HDHVSHFLILFDomRGDLQIRUPDomRUHTXHULGD
,, SRVVLELOLWDU D JUDYDomR GH UHODWyULRVHPGLYHUVRV
IRUPDWRVHOHWU{QLFRVLQFOXVLYHDEHUWRVHQmRSURSULHWi †ž 3DUD R DFHVVR D LQIRUPDo}HV GH LQWHUHVVHS~EOLFRD
ULRVWDLVFRPRSODQLOKDVHWH[WRGHPRGRDIDFLOLWDUD LGHQWLILFDomRGRUHTXHUHQWHQmRSRGHFRQWHUH[LJrQFLDV
DQiOLVHGDVLQIRUPDo}HV TXHLQYLDELOL]HPDVROLFLWDomR

,,, SRVVLELOLWDURDFHVVRDXWRPDWL]DGRSRUVLVWHPDVH[WHU †ž 2VyUJmRVHHQWLGDGHVGRSRGHUS~EOLFRGHYHPYLDELOL


QRVHPIRUPDWRVDEHUWRVHVWUXWXUDGRVHOHJtYHLVSRU ]DU DOWHUQDWLYD GH HQFDPLQKDPHQWR GH SHGLGRV GH
PiTXLQD DFHVVRSRUPHLRGHVHXVVtWLRVRILFLDLVQDLQWHUQHW

,9 GLYXOJDU HP GHWDOKHV RV IRUPDWRV XWLOL]DGRVSDUD †ž 6mRYHGDGDVTXDLVTXHUH[LJrQFLDVUHODWLYDVDRVPRWLYRV


HVWUXWXUDomRGDLQIRUPDomR GHWHUPLQDQWHVGDVROLFLWDomRGHLQIRUPDo}HVGHLQWH
UHVVHS~EOLFR
9 JDUDQWLUDDXWHQWLFLGDGHHDLQWHJULGDGHGDVLQIRUPD
o}HVGLVSRQtYHLVSDUDDFHVVR $UW 2 yUJmR RX HQWLGDGH S~EOLFD GHYHUi DXWRUL]DURX
FRQFHGHURDFHVVRLPHGLDWRjLQIRUPDomRGLVSRQtYHO
9, PDQWHU DWXDOL]DGDV DV LQIRUPDo}HV GLVSRQtYHLVSDUD
DFHVVR †ž 1mR VHQGR SRVVtYHO FRQFHGHU R DFHVVRLPHGLDWRQD
IRUPD GLVSRVWD QR FDSXW R yUJmR RX HQWLGDGH TXH
9,, LQGLFDUORFDOHLQVWUXo}HVTXHSHUPLWDPDRLQWHUHVVDGR UHFHEHURSHGLGRGHYHUiHPSUD]RQmRVXSHULRUD
FRPXQLFDUVHSRUYLDHOHWU{QLFDRXWHOHI{QLFDFRPR YLQWH GLDV
yUJmRRXHQWLGDGHGHWHQWRUDGRVtWLRH
, FRPXQLFDU D GDWD ORFDO H PRGR SDUDVHUHDOL]DUD
9,,, DGRWDUDVPHGLGDVQHFHVViULDVSDUDJDUDQWLUDDFHVVLEL FRQVXOWDHIHWXDUDUHSURGXomRRXREWHUDFHUWLGmR
OLGDGHGHFRQWH~GRSDUDSHVVRDVFRPGHILFLrQFLDQRV
WHUPRVGRDUWGD/HLQRGHGHGH]HPEUR ,, LQGLFDUDVUD]}HVGHIDWRRXGHGLUHLWRGDUHFXVDWRWDO
GHHGRDUWRGD&RQYHQomRVREUHRV'LUHLWRV RXSDUFLDOGRDFHVVRSUHWHQGLGRRX
GDV 3HVVRDV FRP 'HILFLrQFLD DSURYDGD SHOR 'HFUHWR
/HJLVODWLYRQRGHGHMXOKRGH
 'LU&RQVWLWXFLRQDO
&kPDUD/HJLVODWLYDGR'LVWULWR)HGHUDO
,,, FRPXQLFDUTXHQmRSRVVXLDLQIRUPDomRLQGLFDUVHIRU 6HomR,,
GR VHX FRQKHFLPHQWR R yUJmR RX D HQWLGDGH TXH D 'RV5HFXUVRV
GHWpPRXDLQGDUHPHWHURUHTXHULPHQWRDHVVHyUJmR
RXHQWLGDGHFLHQWLILFDQGRRLQWHUHVVDGRGDUHPHVVDGH $UW 1RFDVRGHLQGHIHULPHQWRGHDFHVVRDLQIRUPDo}HVRX
VHXSHGLGRGHLQIRUPDomR jVUD]}HVGDQHJDWLYDGRDFHVVRSRGHUiRLQWHUHVVDGR
LQWHUSRUUHFXUVRFRQWUDDGHFLVmRQRSUD]RGH GH]
†ž 2SUD]RUHIHULGRQR†RSRGHUiVHUSURUURJDGRSRU GLDVDFRQWDUGDVXDFLrQFLD
PDLV GH] GLDVPHGLDQWHMXVWLILFDWLYDH[SUHVVDGD
TXDOVHUiFLHQWLILFDGRRUHTXHUHQWH †~QLFR 2UHFXUVRVHUiGLULJLGRjDXWRULGDGHKLHUDUTXLFDPHQWH
VXSHULRU j TXH H[DURX D GHFLVmR LPSXJQDGD TXH
†ž 6HPSUHMXt]RGDVHJXUDQoDHGDSURWHomRGDVLQIRUPD GHYHUiVHPDQLIHVWDUQRSUD]RGH FLQFR GLDV
o}HVHGRFXPSULPHQWRGDOHJLVODomRDSOLFiYHORyUJmR
$UW 1HJDGRRDFHVVRDLQIRUPDomRSHORVyUJmRVRXHQWLGD
RXHQWLGDGHSRGHUiRIHUHFHUPHLRVSDUDTXHRSUySULR
GHVGR3RGHU([HFXWLYR)HGHUDORUHTXHUHQWHSRGHUi
UHTXHUHQWH SRVVD SHVTXLVDU D LQIRUPDomR GH TXH
UHFRUUHUj&RQWURODGRULD*HUDOGD8QLmRTXHGHOLEHUD
QHFHVVLWDU
UiQRSUD]RGH FLQFR GLDVVH
†ž 4XDQGRQmRIRUDXWRUL]DGRRDFHVVRSRUVHWUDWDUGH , RDFHVVRjLQIRUPDomRQmRFODVVLILFDGDFRPRVLJLORVD
LQIRUPDomRWRWDORXSDUFLDOPHQWHVLJLORVDRUHTXHUHQWH IRUQHJDGR
GHYHUiVHULQIRUPDGRVREUHDSRVVLELOLGDGHGHUHFXUVR
SUD]RV H FRQGLo}HV SDUD VXD LQWHUSRVLomR GHYHQGR ,, DGHFLVmRGHQHJDWLYDGHDFHVVRjLQIRUPDomRWRWDORX
DLQGDVHUOKHLQGLFDGDDDXWRULGDGHFRPSHWHQWHSDUD SDUFLDOPHQWHFODVVLILFDGDFRPRVLJLORVDQmRLQGLFDUD
VXDDSUHFLDomR DXWRULGDGH FODVVLILFDGRUD RX D KLHUDUTXLFDPHQWH
VXSHULRUDTXHPSRVVDVHUGLULJLGRSHGLGRGHDFHVVRRX
†ž $ LQIRUPDomR DUPD]HQDGD HP IRUPDWRGLJLWDOVHUi GHVFODVVLILFDomR
IRUQHFLGDQHVVHIRUPDWRFDVRKDMDDQXrQFLDGRUHTXH
UHQWH ,,, RV SURFHGLPHQWRV GH FODVVLILFDomR GHLQIRUPDomR
VLJLORVD HVWDEHOHFLGRV QHVWD /HL QmR WLYHUHP VLGR
†ž &DVR D LQIRUPDomR VROLFLWDGD HVWHMDGLVSRQtYHODR REVHUYDGRVH
S~EOLFRHPIRUPDWRLPSUHVVRHOHWU{QLFRRXHPTXDO
TXHURXWURPHLRGHDFHVVRXQLYHUVDOVHUmRLQIRUPDGRV ,9 HVWLYHUHP VHQGR GHVFXPSULGRV SUD]RV RXRXWURV
DRUHTXHUHQWHSRUHVFULWRROXJDUHDIRUPDSHODTXDO SURFHGLPHQWRVSUHYLVWRVQHVWD/HL
VH SRGHUi FRQVXOWDU REWHU RX UHSURGX]LU D UHIHULGD
LQIRUPDomRSURFHGLPHQWRHVVHTXHGHVRQHUDUiRyUJmR †ž 2 UHFXUVR SUHYLVWR QHVWH DUWLJR VRPHQWHSRGHUiVHU
RXHQWLGDGHS~EOLFDGDREULJDomRGHVHXIRUQHFLPHQWR GLULJLGR j &RQWURODGRULD*HUDO GD 8QLmR GHSRLV GH
GLUHWRVDOYRVH R UHTXHUHQWHGHFODUDUQmRGLVSRUGH VXEPHWLGRjDSUHFLDomRGHSHORPHQRVXPDDXWRULGDGH
PHLRVSDUDUHDOL]DUSRUVLPHVPRWDLVSURFHGLPHQWRV KLHUDUTXLFDPHQWHVXSHULRUjTXHODTXHH[DURXDGHFLVmR
LPSXJQDGDTXHGHOLEHUDUiQRSUD]RGH FLQFR GLDV
$UW 2 VHUYLoR GH EXVFD H IRUQHFLPHQWR GD LQIRUPDomRp
†ž 9HULILFDGD D SURFHGrQFLD GDV UD]}HV GRUHFXUVRD
JUDWXLWRVDOYRQDVKLSyWHVHVGHUHSURGXomRGHGRFX
&RQWURODGRULD*HUDOGD8QLmRGHWHUPLQDUiDRyUJmRRX
PHQWRV SHOR yUJmR RX HQWLGDGH S~EOLFD FRQVXOWDGD
HQWLGDGHTXHDGRWHDVSURYLGrQFLDVQHFHVViULDVSDUD
VLWXDomRHPTXHSRGHUiVHUFREUDGRH[FOXVLYDPHQWHR
GDUFXPSULPHQWRDRGLVSRVWRQHVWD/HL
YDORUQHFHVViULRDRUHVVDUFLPHQWRGRFXVWRGRVVHUYLoRV
HGRVPDWHULDLVXWLOL]DGRV †ž 1HJDGR R DFHVVR jLQIRUPDomRSHOD
&RQWURODGRULD*HUDOGD 8QLmRSRGHUiVHULQWHUSRVWR
†~QLFR (VWDUiLVHQWRGHUHVVDUFLURVFXVWRVSUHYLVWRVQRFDSXW UHFXUVRj&RPLVVmR0LVWDGH5HDYDOLDomRGH,QIRUPD
WRGRDTXHOHFXMDVLWXDomRHFRQ{PLFDQmROKHSHUPLWD o}HVDTXHVHUHIHUHRDUW
ID]rORVHPSUHMXt]RGRVXVWHQWRSUySULRRXGDIDPtOLD
GHFODUDGDQRVWHUPRVGD/HLQžGHGHDJRVWR $UW 1RFDVRGHLQGHIHULPHQWRGHSHGLGRGHGHVFODVVLILFDomR
GH GHLQIRUPDomRSURWRFRODGRHPyUJmRGDDGPLQLVWUDomR
S~EOLFD IHGHUDO SRGHUi R UHTXHUHQWH UHFRUUHU DR
$UW 4XDQGRVHWUDWDUGHDFHVVRjLQIRUPDomRFRQWLGDHP 0LQLVWURGH(VWDGRGDiUHDVHPSUHMXt]RGDVFRPSH
GRFXPHQWR FXMD PDQLSXODomR SRVVD SUHMXGLFDU VXD WrQFLDVGD&RPLVVmR0LVWDGH5HDYDOLDomRGH,QIRUPD
LQWHJULGDGHGHYHUiVHURIHUHFLGDDFRQVXOWDGHFySLD o}HVSUHYLVWDVQRDUWHGRGLVSRVWRQRDUW
FRPFHUWLILFDomRGHTXHHVWDFRQIHUHFRPRRULJLQDO
†ž 2 UHFXUVR SUHYLVWR QHVWH DUWLJR VRPHQWHSRGHUiVHU
GLULJLGRjVDXWRULGDGHVPHQFLRQDGDVGHSRLVGHVXEPH
†~QLFR 1DLPSRVVLELOLGDGHGHREWHQomRGHFySLDVRLQWHUHVVD WLGR j DSUHFLDomR GH SHOR PHQRV XPD DXWRULGDGH
GRSRGHUiVROLFLWDUTXHDVXDVH[SHQVDVHVREVXSHUYL KLHUDUTXLFDPHQWHVXSHULRUjDXWRULGDGHTXHH[DURXD
VmR GH VHUYLGRU S~EOLFR D UHSURGXomR VHMD IHLWD SRU GHFLVmRLPSXJQDGDHQRFDVRGDV)RUoDV$UPDGDVDR
RXWURPHLRTXHQmRSRQKDHPULVFRDFRQVHUYDomRGR UHVSHFWLYR&RPDQGR
GRFXPHQWRRULJLQDO
†ž ,QGHIHULGRRUHFXUVRSUHYLVWRQRFDSXWTXHWHQKDFRPR
$UW eGLUHLWRGRUHTXHUHQWHREWHURLQWHLURWHRUGHGHFLVmR REMHWR D GHVFODVVLILFDomR GH LQIRUPDomR VHFUHWD RX
GHQHJDWLYDGHDFHVVRSRUFHUWLGmRRXFySLD XOWUDVVHFUHWD FDEHUi UHFXUVR j &RPLVVmR 0LVWD GH
5HDYDOLDomRGH,QIRUPDo}HVSUHYLVWDQRDUW
&kPDUD/HJLVODWLYDGR
  'LVWULWR)HGHUDO 'LU&RQVWLWXFLRQDO 

$UW 2VSURFHGLPHQWRVGHUHYLVmRGHGHFLV}HVGHQHJDWyULDV 9 SUHMXGLFDU RX FDXVDU ULVFR D SODQRVRXRSHUDo}HV


SURIHULGDVQRUHFXUVRSUHYLVWRQRDUWHGHUHYLVmR HVWUDWpJLFRVGDV)RUoDV$UPDGDV
GHFODVVLILFDomRGHGRFXPHQWRVVLJLORVRVVHUmRREMHWR
GHUHJXODPHQWDomRSUySULDGRV3RGHUHV/HJLVODWLYRH 9, SUHMXGLFDU RX FDXVDU ULVFR D SURMHWRV GH SHVTXLVDH
-XGLFLiULRHGR0LQLVWpULR3~EOLFRHPVHXVUHVSHFWLYRV GHVHQYROYLPHQWRFLHQWtILFRRXWHFQROyJLFRDVVLPFRPR
kPELWRVDVVHJXUDGRDRVROLFLWDQWHHPTXDOTXHUFDVR D VLVWHPDV EHQV LQVWDODo}HV RX iUHDV GH LQWHUHVVH
RGLUHLWRGHVHULQIRUPDGRVREUHRDQGDPHQWRGHVHX HVWUDWpJLFRQDFLRQDO
SHGLGR
9,, S{UHP ULVFR DVHJXUDQoDGHLQVWLWXLo}HVRXGHDOWDV
$UW 9(7$'2 
DXWRULGDGHVQDFLRQDLVRXHVWUDQJHLUDVHVHXVIDPLOLD
†ž 9(7$'2  UHVRX

†ž 2VyUJmRVGR3RGHU-XGLFLiULRHGR0LQLVWpULR3~EOLFR 9,,, FRPSURPHWHUDWLYLGDGHVGHLQWHOLJrQFLDEHPFRPRGH


LQIRUPDUmR DR &RQVHOKR 1DFLRQDO GH -XVWLoD H DR LQYHVWLJDomRRXILVFDOL]DomRHPDQGDPHQWRUHODFLRQD
&RQVHOKR1DFLRQDOGR0LQLVWpULR3~EOLFRUHVSHFWLYD GDVFRPDSUHYHQomRRXUHSUHVVmRGHLQIUDo}HV
PHQWHDVGHFLV}HVTXHHPJUDXGHUHFXUVRQHJDUHP
DFHVVRDLQIRUPDo}HVGHLQWHUHVVHS~EOLFR
$UW $LQIRUPDomRHPSRGHUGRVyUJmRVHHQWLGDGHVS~EOL
FDVREVHUYDGRRVHXWHRUHHPUD]mRGHVXDLPSUHVFLQ
$UW $SOLFDVH VXEVLGLDULDPHQWH QR TXH FRXEHU D /HLQR
GLELOLGDGH j VHJXUDQoD GD VRFLHGDGH RX GR (VWDGR
GHGHMDQHLURGHDRSURFHGLPHQWRGH
SRGHUiVHUFODVVLILFDGDFRPRXOWUDVVHFUHWDVHFUHWDRX
TXHWUDWDHVWH&DStWXOR
UHVHUYDGD
&$3Ì78/2,9
†ž 2VSUD]RVPi[LPRVGHUHVWULomRGHDFHVVRjLQIRUPD
'$65(675,d¯(6'($&(662 omRFRQIRUPHDFODVVLILFDomRSUHYLVWDQRFDSXWYLJR
­,1)250$d®2 UDPDSDUWLUGDGDWDGHVXDSURGXomRHVmRRVVHJXLQ
WHV
6HomR,
'LVSRVLo}HV*HUDLV , XOWUDVVHFUHWD YLQWHHFLQFR DQRV
,, VHFUHWD TXLQ]H DQRVH
$UW 1mRSRGHUiVHUQHJDGRDFHVVRjLQIRUPDomRQHFHVViULD ,,, UHVHUYDGD FLQFR DQRV
jWXWHODMXGLFLDORXDGPLQLVWUDWLYDGHGLUHLWRVIXQGD
PHQWDLV
†ž $V LQIRUPDo}HV TXH SXGHUHP FRORFDU HPULVFRD
VHJXUDQoDGR3UHVLGHQWHH9LFH3UHVLGHQWHGD5HS~EOL
†~QLFR $V LQIRUPDo}HV RX GRFXPHQWRV TXHYHUVHPVREUH
FDHUHVSHFWLYRVF{QMXJHVHILOKRV DV VHUmRFODVVLILFD
FRQGXWDVTXHLPSOLTXHPYLRODomRGRVGLUHLWRVKXPD
GDVFRPRUHVHUYDGDVHILFDUmRVREVLJLORDWpRWpUPLQR
QRV SUDWLFDGD SRU DJHQWHV S~EOLFRV RX D PDQGR GH
GRPDQGDWRHPH[HUFtFLRRXGR~OWLPRPDQGDWRHP
DXWRULGDGHVS~EOLFDVQmRSRGHUmRVHUREMHWRGHUHVWUL
omRGHDFHVVR FDVRGHUHHOHLomR

$UW 2 GLVSRVWR QHVWD /HL QmR H[FOXL DV GHPDLVKLSyWHVHV †ž $OWHUQDWLYDPHQWHDRVSUD]RVSUHYLVWRVQR†RSRGHUi
OHJDLVGHVLJLORHGHVHJUHGRGHMXVWLoDQHPDVKLSyWH VHU HVWDEHOHFLGD FRPR WHUPR ILQDO GH UHVWULomR GH
VHV GH VHJUHGR LQGXVWULDO GHFRUUHQWHV GD H[SORUDomR DFHVVRDRFRUUrQFLDGHGHWHUPLQDGRHYHQWRGHVGHTXH
GLUHWD GH DWLYLGDGH HFRQ{PLFD SHOR (VWDGR RX SRU HVWHRFRUUDDQWHVGRWUDQVFXUVRGRSUD]RPi[LPRGH
SHVVRDItVLFDRXHQWLGDGHSULYDGDTXHWHQKDTXDOTXHU FODVVLILFDomR
YtQFXORFRPRSRGHUS~EOLFR
†ž 7UDQVFRUULGRRSUD]RGHFODVVLILFDomRRXFRQVXPDGRR
6HomR,, HYHQWR TXH GHILQD R VHX WHUPR ILQDO D LQIRUPDomR
'D&ODVVLILFDomRGD,QIRUPDomR WRUQDUVHiDXWRPDWLFDPHQWHGHDFHVVRS~EOLFR
TXDQWRDR*UDXH3UD]RVGH6LJLOR
†ž 3DUD D FODVVLILFDomR GD LQIRUPDomRHPGHWHUPLQDGR
$UW 6mR FRQVLGHUDGDV LPSUHVFLQGtYHLV j VHJXUDQoDGD JUDXGHVLJLORGHYHUiVHUREVHUYDGRRLQWHUHVVHS~EOLFR
VRFLHGDGH RX GR (VWDGR H SRUWDQWR SDVVtYHLV GH GDLQIRUPDomRHXWLOL]DGRRFULWpULRPHQRVUHVWULWLYR
FODVVLILFDomRDVLQIRUPDo}HVFXMDGLYXOJDomRRXDFHVVR SRVVtYHOFRQVLGHUDGRV
LUUHVWULWRSRVVDP
, DJUDYLGDGHGRULVFRRXGDQRjVHJXUDQoDGDVRFLHGDGH
, S{U HP ULVFR D GHIHVD H D VREHUDQLDQDFLRQDLVRXD HGR(VWDGRH
LQWHJULGDGHGRWHUULWyULRQDFLRQDO
,, RSUD]RPi[LPRGHUHVWULomRGHDFHVVRRXRHYHQWRTXH
,, SUHMXGLFDURXS{UHPULVFRDFRQGXomRGHQHJRFLDo}HV GHILQDVHXWHUPRILQDO
RX DV UHODo}HV LQWHUQDFLRQDLV GR 3DtV RX DV TXH WH
QKDPVLGRIRUQHFLGDVHPFDUiWHUVLJLORVRSRURXWURV
(VWDGRVHRUJDQLVPRVLQWHUQDFLRQDLV 6HomR,,,
'D3URWHomRHGR&RQWUROHGH,QIRUPDo}HV6LJLORVDV
,,, S{UHPULVFRDYLGDDVHJXUDQoDRXDVD~GHGDSRSXOD
omR $UW eGHYHUGR(VWDGRFRQWURODURDFHVVRHDGLYXOJDomRGH
LQIRUPDo}HV VLJLORVDV SURGX]LGDV SRU VHXV yUJmRV H
,9 RIHUHFHUHOHYDGRULVFRjHVWDELOLGDGHILQDQFHLUDHFRQ{ HQWLGDGHVDVVHJXUDQGRDVXDSURWHomR
PLFDRXPRQHWiULDGR3DtV
 'LU&RQVWLWXFLRQDO
&kPDUD/HJLVODWLYDGR'LVWULWR)HGHUDO
†ž 2DFHVVRDGLYXOJDomRHRWUDWDPHQWRGHLQIRUPDomR †ž $FODVVLILFDomRGHLQIRUPDomRQRJUDXGHVLJLORXOWUDV
FODVVLILFDGD FRPR VLJLORVD ILFDUmR UHVWULWRV D SHVVRDV VHFUHWRSHODVDXWRULGDGHVSUHYLVWDVQDVDOtQHDV´GµH´Hµ
TXH WHQKDP QHFHVVLGDGH GH FRQKHFrOD H TXH VHMDP GR LQFLVR , GHYHUi VHU UDWLILFDGD SHORV UHVSHFWLYRV
GHYLGDPHQWHFUHGHQFLDGDVQDIRUPDGRUHJXODPHQWR 0LQLVWURVGH(VWDGRQRSUD]RSUHYLVWRHPUHJXODPHQ
VHP SUHMXt]R GDV DWULEXLo}HV GRV DJHQWHV S~EOLFRV WR
DXWRUL]DGRVSRUOHL
†ž $DXWRULGDGHRXRXWURDJHQWHS~EOLFRTXHFODVVLILFDU
†ž 2DFHVVRjLQIRUPDomRFODVVLILFDGDFRPRVLJLORVDFULDD LQIRUPDomRFRPRXOWUDVVHFUHWDGHYHUiHQFDPLQKDUD
REULJDomRSDUDDTXHOHTXHDREWHYHGHUHVJXDUGDUR GHFLVmR GH TXH WUDWD R DUW  j &RPLVVmR 0LVWD GH
VLJLOR 5HDYDOLDomRGH,QIRUPDo}HVDTXHVHUHIHUHRDUW
QRSUD]RSUHYLVWRHPUHJXODPHQWR
†ž 5HJXODPHQWRGLVSRUiVREUHSURFHGLPHQWRVHPHGLGDV
D VHUHP DGRWDGRV SDUD R WUDWDPHQWR GH LQIRUPDomR $UW $ FODVVLILFDomR GH LQIRUPDomR HP TXDOTXHU JUDXGH
VLJLORVDGHPRGRDSURWHJrODFRQWUDSHUGDDOWHUDomR VLJLORGHYHUiVHUIRUPDOL]DGDHPGHFLVmRTXHFRQWHUi
LQGHYLGDDFHVVRWUDQVPLVVmRHGLYXOJDomRQmRDXWRUL QRPtQLPRRVVHJXLQWHVHOHPHQWRV
]DGRV
, DVVXQWRVREUHRTXDOYHUVDDLQIRUPDomR
$UW $V DXWRULGDGHV S~EOLFDV DGRWDUmR DVSURYLGrQFLDV
QHFHVViULDV SDUD TXH R SHVVRDO D HODV VXERUGLQDGR ,, IXQGDPHQWR GD FODVVLILFDomR REVHUYDGRVRVFULWpULRV
KLHUDUTXLFDPHQWH FRQKHoD DV QRUPDV H REVHUYH DV HVWDEHOHFLGRVQRDUW
PHGLGDVHSURFHGLPHQWRVGHVHJXUDQoDSDUDWUDWDPHQ
WRGHLQIRUPDo}HVVLJLORVDV ,,, LQGLFDomRGRSUD]RGHVLJLORFRQWDGRHPDQRVPHVHV
RX GLDV RX GR HYHQWR TXH GHILQD R VHX WHUPR ILQDO
†~QLFR $SHVVRDItVLFDRXHQWLGDGHSULYDGDTXHHPUD]mRGH FRQIRUPHOLPLWHVSUHYLVWRVQRDUWH
TXDOTXHU YtQFXOR FRP R SRGHU S~EOLFR H[HFXWDU
DWLYLGDGHV GH WUDWDPHQWR GH LQIRUPDo}HV VLJLORVDV ,9 LGHQWLILFDomRGDDXWRULGDGHTXHDFODVVLILFRX
DGRWDUi DV SURYLGrQFLDV QHFHVViULDV SDUD TXH VHXV
HPSUHJDGRVSUHSRVWRVRXUHSUHVHQWDQWHVREVHUYHPDV †~QLFR $GHFLVmRUHIHULGDQRFDSXWVHUiPDQWLGDQRPHVPR
PHGLGDVHSURFHGLPHQWRVGHVHJXUDQoDGDVLQIRUPD JUDXGHVLJLORGDLQIRUPDomRFODVVLILFDGD
o}HVUHVXOWDQWHVGDDSOLFDomRGHVWD/HL
$UW $ FODVVLILFDomR GDV LQIRUPDo}HV VHUi UHDYDOLDGDSHOD
6HomR,9 DXWRULGDGHFODVVLILFDGRUDRXSRUDXWRULGDGHKLHUDUTXL
'RV3URFHGLPHQWRVGH&ODVVLILFDomR FDPHQWHVXSHULRUPHGLDQWHSURYRFDomRRXGHRItFLR
5HFODVVLILFDomRH'HVFODVVLILFDomR QRVWHUPRVHSUD]RVSUHYLVWRVHPUHJXODPHQWRFRP
YLVWDVjVXDGHVFODVVLILFDomRRXjUHGXomRGRSUD]RGH
VLJLORREVHUYDGRRGLVSRVWRQRDUW
$UW $FODVVLILFDomRGRVLJLORGHLQIRUPDo}HVQRkPELWRGD
DGPLQLVWUDomRS~EOLFDIHGHUDOpGHFRPSHWrQFLD
†ž 2UHJXODPHQWRDTXHVHUHIHUHRFDSXWGHYHUiFRQVLGH
UDUDVSHFXOLDULGDGHVGDVLQIRUPDo}HVSURGX]LGDVQR
, QRJUDXGHXOWUDVVHFUHWRGDVVHJXLQWHVDXWRULGDGHV
H[WHULRUSRUDXWRULGDGHVRXDJHQWHVS~EOLFRV
D 3UHVLGHQWHGD5HS~EOLFD †ž 1D UHDYDOLDomR D TXH VH UHIHUH R FDSXWGHYHUmRVHU
E 9LFH3UHVLGHQWHGD5HS~EOLFD H[DPLQDGDVDSHUPDQrQFLDGRVPRWLYRVGRVLJLORHD
F 0LQLVWURVGH(VWDGRHDXWRULGDGHVFRPDVPHVPDV SRVVLELOLGDGH GH GDQRV GHFRUUHQWHV GR DFHVVR RX GD
SUHUURJDWLYDV GLYXOJDomRGDLQIRUPDomR
G &RPDQGDQWHVGD0DULQKDGR([pUFLWRHGD$HUR
QiXWLFDH †ž 1DKLSyWHVHGHUHGXomRGRSUD]RGHVLJLORGDLQIRUPD
H &KHIHV GH 0LVV}HV 'LSORPiWLFDV H &RQVXODUHV omRRQRYRSUD]RGHUHVWULomRPDQWHUiFRPRWHUPR
SHUPDQHQWHVQRH[WHULRU LQLFLDODGDWDGDVXDSURGXomR

,, QRJUDXGHVHFUHWRGDVDXWRULGDGHVUHIHULGDVQRLQFLVR $UW $ DXWRULGDGH Pi[LPD GH FDGD yUJmR RXHQWLGDGH


,GRVWLWXODUHVGHDXWDUTXLDVIXQGDo}HVRXHPSUHVDV SXEOLFDUiDQXDOPHQWHHPVtWLRjGLVSRVLomRQDLQWHU
S~EOLFDVHVRFLHGDGHVGHHFRQRPLDPLVWDH QHWHGHVWLQDGRjYHLFXODomRGHGDGRVHLQIRUPDo}HV
DGPLQLVWUDWLYDVQRVWHUPRVGHUHJXODPHQWR
,,, QRJUDXGHUHVHUYDGRGDV DXWRULGDGHVUHIHULGDVQRV
LQFLVRV,H,,HGDVTXHH[HUoDPIXQo}HV GH GLUHomR , UROGDVLQIRUPDo}HVTXHWHQKDPVLGRGHVFODVVLILFDGDV
FRPDQGRRXFKHILDQtYHO'$6RXVXSHULRUGR QRV~OWLPRV GR]H PHVHV
*UXSR'LUHomR H $VVHVVRUDPHQWR 6XSHULRUHV RX GH
KLHUDUTXLDHTXLYDOHQWHGHDFRUGRFRPUHJXODPHQWDomR ,, UROGHGRFXPHQWRVFODVVLILFDGRVHPFDGDJUDXGHVLJLOR
HVSHFtILFD GH FDGD yUJmR RX HQWLGDGH REVHUYDGR R FRPLGHQWLILFDomRSDUDUHIHUrQFLDIXWXUD
GLVSRVWRQHVWD/HL
,,, UHODWyULRHVWDWtVWLFRFRQWHQGRDTXDQWLGDGHGHSHGLGRV
GHLQIRUPDomRUHFHELGRVDWHQGLGRVHLQGHIHULGRVEHP
†ž $ FRPSHWrQFLD SUHYLVWD QRV LQFLVRV , H ,, QRTXHVH
FRPRLQIRUPDo}HVJHQpULFDVVREUHRVVROLFLWDQWHV
UHIHUH j FODVVLILFDomR FRPR XOWUDVVHFUHWD H VHFUHWD
SRGHUi VHU GHOHJDGD SHOD DXWRULGDGH UHVSRQViYHO D
†ž 2V yUJmRV H HQWLGDGHV GHYHUmR PDQWHUH[HPSODUGD
DJHQWH S~EOLFR LQFOXVLYH HP PLVVmR QR H[WHULRU SXEOLFDomRSUHYLVWDQRFDSXWSDUDFRQVXOWDS~EOLFDHP
YHGDGDDVXEGHOHJDomR VXDVVHGHV
&kPDUD/HJLVODWLYDGR
  'LVWULWR)HGHUDO 'LU&RQVWLWXFLRQDO 

†ž 2VyUJmRVHHQWLGDGHVPDQWHUmRH[WUDWRFRPDOLVWDGH ,, XWLOL]DULQGHYLGDPHQWHEHPFRPRVXEWUDLUGHVWUXLU


LQIRUPDo}HVFODVVLILFDGDVDFRPSDQKDGDVGDGDWDGR LQXWLOL]DU GHVILJXUDU DOWHUDU RX RFXOWDU WRWDO RX
JUDXGHVLJLORHGRVIXQGDPHQWRVGDFODVVLILFDomR SDUFLDOPHQWH LQIRUPDomR TXH VH HQFRQWUH VRE VXD
JXDUGD RX D TXH WHQKD DFHVVR RX FRQKHFLPHQWR HP
6HomR9 UD]mRGRH[HUFtFLRGDVDWULEXLo}HVGHFDUJRHPSUHJR
'DV,QIRUPDo}HV3HVVRDLV RXIXQomRS~EOLFD

$UW 2WUDWDPHQWRGDVLQIRUPDo}HVSHVVRDLVGHYHVHUIHLWR ,,, DJLUFRPGRORRXPiIpQDDQiOLVHGDVVROLFLWDo}HVGH


GH IRUPD WUDQVSDUHQWH H FRP UHVSHLWR j LQWLPLGDGH DFHVVRjLQIRUPDomR
YLGDSULYDGDKRQUDHLPDJHPGDVSHVVRDVEHPFRPR
jVOLEHUGDGHVHJDUDQWLDVLQGLYLGXDLV ,9 GLYXOJDU RX SHUPLWLU D GLYXOJDomR RX DFHVVDURX
SHUPLWLU DFHVVR LQGHYLGR j LQIRUPDomR VLJLORVD RX
†ž $VLQIRUPDo}HVSHVVRDLVDTXHVHUHIHUHHVWHDUWLJR LQIRUPDomRSHVVRDO
UHODWLYDVjLQWLPLGDGHYLGDSULYDGDKRQUDHLPDJHP
9 LPSRUVLJLORjLQIRUPDomRSDUDREWHUSURYHLWRSHVVRDO
RXGHWHUFHLURRXSDUDILQVGHRFXOWDomRGHDWRLOHJDO
, WHUmR VHX DFHVVRUHVWULWRLQGHSHQGHQWHPHQWHGH
FRPHWLGRSRUVLRXSRURXWUHP
FODVVLILFDomR GH VLJLOR H SHOR SUD]R Pi[LPR GH 
FHP  DQRV D FRQWDU GD VXD GDWD GH SURGXomR D 9, RFXOWDUGDUHYLVmRGHDXWRULGDGHVXSHULRUFRPSHWHQWH
DJHQWHVS~EOLFRVOHJDOPHQWHDXWRUL]DGRVHjSHVVRDD LQIRUPDomRVLJLORVDSDUDEHQHILFLDUDVLRXDRXWUHP
TXHHODVVHUHIHULUHPH RXHPSUHMXt]RGHWHUFHLURVH
,, SRGHUmRWHUDXWRUL]DGDVXDGLYXOJDomRRXDFHVVRSRU 9,, GHVWUXLURXVXEWUDLUSRUTXDOTXHUPHLRGRFXPHQWRV
WHUFHLURV GLDQWH GH SUHYLVmR OHJDO RX FRQVHQWLPHQWR FRQFHUQHQWHVDSRVVtYHLVYLRODo}HVGHGLUHLWRVKXPDQRV
H[SUHVVRGDSHVVRDDTXHHODVVHUHIHULUHP SRUSDUWHGHDJHQWHVGR(VWDGR

†ž $TXHOHTXHREWLYHUDFHVVRjVLQIRUPDo}HVGHTXHWUDWD †ž $WHQGLGRRSULQFtSLRGRFRQWUDGLWyULRGDDPSODGHIHVD


HVWHDUWLJRVHUiUHVSRQVDELOL]DGRSRUVHXXVRLQGHYLGR HGRGHYLGRSURFHVVROHJDODVFRQGXWDVGHVFULWDVQR
FDSXWVHUmRFRQVLGHUDGDV
†ž 2FRQVHQWLPHQWRUHIHULGRQRLQFLVR,,GR†RQmRVHUi
H[LJLGRTXDQGRDVLQIRUPDo}HVIRUHPQHFHVViULDV , SDUD ILQV GRV UHJXODPHQWRVGLVFLSOLQDUHVGDV)RUoDV
$UPDGDV WUDQVJUHVV}HV PLOLWDUHV PpGLDV RX JUDYHV
, j SUHYHQomR H GLDJQyVWLFR PpGLFRTXDQGRDSHVVRD VHJXQGRRVFULWpULRVQHOHVHVWDEHOHFLGRVGHVGHTXHQmR
HVWLYHUItVLFDRXOHJDOPHQWHLQFDSD]HSDUDXWLOL]DomR WLSLILFDGDVHPOHLFRPRFULPHRXFRQWUDYHQomRSHQDO
~QLFDHH[FOXVLYDPHQWHSDUDRWUDWDPHQWRPpGLFR RX

,, jUHDOL]DomRGHHVWDWtVWLFDVHSHVTXLVDVFLHQWtILFDVGH ,, SDUDILQVGRGLVSRVWRQD/HLQRGHGHGH]HP


HYLGHQWHLQWHUHVVHS~EOLFRRXJHUDOSUHYLVWRVHPOHL EURGHHVXDVDOWHUDo}HVLQIUDo}HVDGPLQLVWUDWL
VHQGR YHGDGD D LGHQWLILFDomR GD SHVVRD D TXH DV YDV TXH GHYHUmR VHU DSHQDGDV QR PtQLPR FRP
LQIRUPDo}HVVHUHIHULUHP VXVSHQVmRVHJXQGRRVFULWpULRVQHODHVWDEHOHFLGRV
†ž 3HODVFRQGXWDVGHVFULWDVQRFDSXWSRGHUiRPLOLWDURX
,,, DRFXPSULPHQWRGHRUGHPMXGLFLDO DJHQWHS~EOLFRUHVSRQGHUWDPEpPSRULPSURELGDGH
DGPLQLVWUDWLYD FRQIRUPH R GLVSRVWR QDV /HLV QRV
GHGHDEULOGHHGHGHMXQKR
,9 jGHIHVDGHGLUHLWRVKXPDQRVRX
GH
9 jSURWHomRGRLQWHUHVVHS~EOLFRHJHUDOSUHSRQGHUDQWH
$UW $SHVVRDItVLFDRXHQWLGDGHSULYDGDTXHGHWLYHULQIRU
PDo}HVHPYLUWXGHGHYtQFXORGHTXDOTXHUQDWXUH]D
†ž $ UHVWULomR GH DFHVVR j LQIRUPDomR UHODWLYDjYLGD FRPRSRGHUS~EOLFRHGHL[DUGHREVHUYDURGLVSRVWR
SULYDGD KRQUD H LPDJHP GH SHVVRD QmR SRGHUi VHU QHVWD/HLHVWDUiVXMHLWDjVVHJXLQWHVVDQo}HV
LQYRFDGD FRP R LQWXLWR GH SUHMXGLFDU SURFHVVR GH
DSXUDomR GH LUUHJXODULGDGHV HP TXH R WLWXODU GDV , DGYHUWrQFLD
LQIRUPDo}HV HVWLYHU HQYROYLGR EHP FRPR HP Do}HV
YROWDGDV SDUD D UHFXSHUDomR GH IDWRV KLVWyULFRV GH ,, PXOWD
PDLRUUHOHYkQFLD
,,, UHVFLVmRGRYtQFXORFRPRSRGHUS~EOLFR
†ž 5HJXODPHQWR GLVSRUi VREUH RVSURFHGLPHQWRVSDUD
WUDWDPHQWRGHLQIRUPDomRSHVVRDO ,9 VXVSHQVmR WHPSRUiULD GH SDUWLFLSDU HP OLFLWDomRH
LPSHGLPHQWRGHFRQWUDWDUFRPDDGPLQLVWUDomRS~EOLFD
&$3Ì78/29 SRUSUD]RQmRVXSHULRUD GRLV DQRVH
'$65(63216$%,/,'$'(6
9 GHFODUDomR GH LQLGRQHLGDGH SDUD OLFLWDU RXFRQWUDWDU
$UW &RQVWLWXHPFRQGXWDVLOtFLWDVTXHHQVHMDPUHVSRQVDELOL FRPDDGPLQLVWUDomRS~EOLFDDWpTXHVHMDSURPRYLGD
GDGHGRDJHQWHS~EOLFRRXPLOLWDU DUHDELOLWDomRSHUDQWHDSUySULDDXWRULGDGHTXHDSOLFRX
DSHQDOLGDGH
, UHFXVDUVHDIRUQHFHULQIRUPDomRUHTXHULGDQRVWHUPRV
GHVWD/HLUHWDUGDUGHOLEHUDGDPHQWHRVHXIRUQHFLPHQ †ž $VVDQo}HVSUHYLVWDVQRVLQFLVRV,,,,H,9SRGHUmRVHU
DSOLFDGDVMXQWDPHQWHFRPDGRLQFLVR,,DVVHJXUDGRR
WRRXIRUQHFrODLQWHQFLRQDOPHQWHGHIRUPDLQFRUUHWD
GLUHLWRGHGHIHVDGRLQWHUHVVDGRQRUHVSHFWLYRSURFHV
LQFRPSOHWDRXLPSUHFLVD
VRQRSUD]RGH GH] GLDV
 'LU&RQVWLWXFLRQDO &kPDUD/HJLVODWLYDGR'LVWULWR)HGHUDO


†ž $ UHDELOLWDomR UHIHULGD QR LQFLVR 9VHUiDXWRUL]DGD $UW 2 WUDWDPHQWR GH LQIRUPDomR VLJLORVD UHVXOWDQWHGH
VRPHQWHTXDQGRRLQWHUHVVDGRHIHWLYDURUHVVDUFLPHQWR WUDWDGRVDFRUGRVRX DWRVLQWHUQDFLRQDLVDWHQGHUijV
DRyUJmRRXHQWLGDGHGRVSUHMXt]RVUHVXOWDQWHVHDSyV QRUPDVHUHFRPHQGDo}HVFRQVWDQWHVGHVVHVLQVWUXPHQ
GHFRUULGR R SUD]R GD VDQomR DSOLFDGD FRP EDVH QR WRV
LQFLVR,9
$UW e LQVWLWXtGR QR kPELWR GR *DELQHWH GH6HJXUDQoD
†ž $DSOLFDomRGDVDQomRSUHYLVWDQRLQFLVR9pGHFRPSH ,QVWLWXFLRQDOGD3UHVLGrQFLDGD5HS~EOLFDR1~FOHRGH
WrQFLD H[FOXVLYD GD DXWRULGDGH Pi[LPD GR yUJmR RX 6HJXUDQoD H &UHGHQFLDPHQWR 16&  TXH WHP SRU
HQWLGDGHS~EOLFDIDFXOWDGDDGHIHVDGRLQWHUHVVDGRQR REMHWLYRV
UHVSHFWLYR SURFHVVR QR SUD]R GH  GH]  GLDV GD
DEHUWXUDGHYLVWD , SURPRYHU H SURSRU DUHJXODPHQWDomRGRFUHGHQFLD
PHQWR GH VHJXUDQoD GH SHVVRDV ItVLFDV HPSUHVDV
$UW 2VyUJmRVHHQWLGDGHVS~EOLFDVUHVSRQGHPGLUHWDPHQWH yUJmRV H HQWLGDGHV SDUD WUDWDPHQWR GH LQIRUPDo}HV
SHORV GDQRV FDXVDGRV HP GHFRUUrQFLD GD GLYXOJDomR VLJLORVDVH
QmRDXWRUL]DGDRXXWLOL]DomRLQGHYLGDGHLQIRUPDo}HV
VLJLORVDVRXLQIRUPDo}HVSHVVRDLVFDEHQGRDDSXUDomR ,, JDUDQWLUDVHJXUDQoDGHLQIRUPDo}HVVLJLORVDVLQFOXVLYH
GH UHVSRQVDELOLGDGH IXQFLRQDO QRV FDVRV GH GROR RX DTXHODVSURYHQLHQWHVGHSDtVHVRXRUJDQL]Do}HVLQWHU
FXOSDDVVHJXUDGRRUHVSHFWLYRGLUHLWRGHUHJUHVVR QDFLRQDLV FRP RV TXDLV D 5HS~EOLFD )HGHUDWLYD GR
%UDVLO WHQKD ILUPDGR WUDWDGR DFRUGR FRQWUDWR RX
†~QLFR 2 GLVSRVWR QHVWH DUWLJR DSOLFDVH j SHVVRDItVLFDRX TXDOTXHU RXWUR DWR LQWHUQDFLRQDO VHP SUHMXt]R GDV
HQWLGDGHSULYDGDTXHHPYLUWXGHGHYtQFXORGHTXDO DWULEXLo}HVGR0LQLVWpULRGDV5HODo}HV([WHULRUHVHGRV
TXHUQDWXUH]DFRPyUJmRVRXHQWLGDGHVWHQKDDFHVVR GHPDLVyUJmRVFRPSHWHQWHV
DLQIRUPDomRVLJLORVDRXSHVVRDOHDVXEPHWDDWUDWD
PHQWRLQGHYLGR †~QLFR 5HJXODPHQWRGLVSRUiVREUHDFRPSRVLomRRUJDQL]DomR
HIXQFLRQDPHQWRGR16&
&$3Ì78/29,
',6326,d¯(6),1$,6(75$16,7Ð5,$6
$UW $SOLFDVH QR TXH FRXEHU D /HL QR  GH GH
$UW 9(7$'2  QRYHPEUR GH  HP UHODomR j LQIRUPDomR GH
SHVVRD ItVLFD RX MXUtGLFD FRQVWDQWH GH UHJLVWUR RX
†ž e LQVWLWXtGD D &RPLVVmR 0LVWD GH5HDYDOLDomRGH EDQFR GH GDGRV GH HQWLGDGHV JRYHUQDPHQWDLV RX GH
,QIRUPDo}HVTXHGHFLGLUiQRkPELWRGDDGPLQLVWUDomR FDUiWHUS~EOLFR
S~EOLFDIHGHUDOVREUHRWUDWDPHQWRHDFODVVLILFDomRGH
LQIRUPDo}HVVLJLORVDVHWHUiFRPSHWrQFLDSDUD $UW 2V yUJmRV H HQWLGDGHV S~EOLFDV GHYHUmR SURFHGHUj
UHDYDOLDomRGDVLQIRUPDo}HVFODVVLILFDGDVFRPRXOWUDV
, UHTXLVLWDU GD DXWRULGDGHTXHFODVVLILFDULQIRUPDomR VHFUHWDVHVHFUHWDVQRSUD]RPi[LPRGH GRLV DQRV
FRPRXOWUDVVHFUHWDHVHFUHWDHVFODUHFLPHQWRRXFRQWH~ FRQWDGRGRWHUPRLQLFLDOGHYLJrQFLDGHVWD/HL
GRSDUFLDORXLQWHJUDOGDLQIRUPDomR
†ž $ UHVWULomR GH DFHVVR D LQIRUPDo}HV HPUD]mRGD
,, UHYHUDFODVVLILFDomRGHLQIRUPDo}HVXOWUDVVHFUHWDVRX UHDYDOLDomR SUHYLVWD QR FDSXW GHYHUi REVHUYDU RV
VHFUHWDVGHRItFLRRXPHGLDQWHSURYRFDomRGHSHVVRD SUD]RVHFRQGLo}HVSUHYLVWRVQHVWD/HL
LQWHUHVVDGDREVHUYDGRRGLVSRVWRQRDUWRHGHPDLV
GLVSRVLWLYRVGHVWD/HLH †ž 1RkPELWRGDDGPLQLVWUDomRS~EOLFDIHGHUDODUHDYDOL
DomRSUHYLVWDQRFDSXWSRGHUiVHUUHYLVWDDTXDOTXHU
,,, SURUURJDURSUD]RGHVLJLORGHLQIRUPDomRFODVVLILFDGD WHPSRSHOD&RPLVVmR0LVWDGH5HDYDOLDomRGH,QIRU
FRPR XOWUDVVHFUHWD VHPSUH SRU SUD]R GHWHUPLQDGR PDo}HVREVHUYDGRVRVWHUPRVGHVWD/HL
HQTXDQWRRVHXDFHVVRRXGLYXOJDomRSXGHURFDVLRQDU
DPHDoDH[WHUQDjVREHUDQLDQDFLRQDORXjLQWHJULGDGH †ž (QTXDQWR QmR WUDQVFRUULGR R SUD]RGHUHDYDOLDomR
GRWHUULWyULRQDFLRQDORXJUDYHULVFRjVUHODo}HVLQWHU SUHYLVWR QR FDSXW VHUi PDQWLGD D FODVVLILFDomR GD
QDFLRQDLVGR3DtVREVHUYDGRRSUD]RSUHYLVWRQR†R LQIRUPDomRQRVWHUPRVGDOHJLVODomRSUHFHGHQWH
GRDUW
†ž $VLQIRUPDo}HVFODVVLILFDGDVFRPRVHFUHWDVHXOWUDVVH
†ž 2SUD]RUHIHULGRQRLQFLVR,,,pOLPLWDGRDXPD~QLFD FUHWDVQmRUHDYDOLDGDVQRSUD]RSUHYLVWRQRFDSXWVHUmR
UHQRYDomR FRQVLGHUDGDVDXWRPDWLFDPHQWHGHDFHVVRS~EOLFR

†ž $UHYLVmRGHRItFLRDTXHVHUHIHUHRLQFLVR,,GR†R $UW 1RSUD]RGH VHVVHQWD GLDVDFRQWDUGDYLJrQFLD


GHYHUiRFRUUHUQRPi[LPRDFDGD TXDWUR DQRV GHVWD/HLRGLULJHQWHPi[LPRGHFDGDyUJmRRXHQWLGD
DSyVDUHDYDOLDomRSUHYLVWDQRDUWTXDQGRVHWUDWDU GHGDDGPLQLVWUDomRS~EOLFDIHGHUDOGLUHWDHLQGLUHWD
GHGRFXPHQWRVXOWUDVVHFUHWRVRXVHFUHWRV GHVLJQDUiDXWRULGDGHTXHOKHVHMDGLUHWDPHQWHVXERUGL
QDGDSDUDQRkPELWRGRUHVSHFWLYRyUJmRRXHQWLGDGH
†ž $QmRGHOLEHUDomRVREUHDUHYLVmRSHOD&RPLVVmR0LVWD H[HUFHUDVVHJXLQWHVDWULEXLo}HV
GH5HDYDOLDomRGH,QIRUPDo}HVQRVSUD]RVSUHYLVWRVQR
† R LPSOLFDUi D GHVFODVVLILFDomR DXWRPiWLFD GDV , DVVHJXUDU R FXPSULPHQWR GDVQRUPDVUHODWLYDVDR
LQIRUPDo}HV DFHVVR D LQIRUPDomR GH IRUPD HILFLHQWH H DGHTXDGD
DRVREMHWLYRVGHVWD/HL
†ž 5HJXODPHQWRGLVSRUiVREUHDFRPSRVLomRRUJDQL]DomR
HIXQFLRQDPHQWRGD&RPLVVmR0LVWDGH5HDYDOLDomRGH ,, PRQLWRUDU D LPSOHPHQWDomR GR GLVSRVWR QHVWD/HLH
,QIRUPDo}HVREVHUYDGRRPDQGDWRGH GRLV DQRV DSUHVHQWDU UHODWyULRV SHULyGLFRV VREUH R VHX FXPSUL
SDUDVHXVLQWHJUDQWHVHGHPDLVGLVSRVLo}HVGHVWD/HL PHQWR
 &kPDUD/HJLVODWLYDGR
 'LVWULWR)HGHUDO 'LU&RQVWLWXFLRQDO 

,,, UHFRPHQGDUDVPHGLGDVLQGLVSHQViYHLVjLPSOHPHQWD
omRHDRDSHUIHLoRDPHQWRGDVQRUPDVHSURFHGLPHQWRV
QHFHVViULRVDRFRUUHWRFXPSULPHQWRGRGLVSRVWRQHVWD /HLGLVWULWDOQž
/HLH
TXHUHJXODRDFHVVRDLQIRUPDo}HVQR'LVWULWR)HGHUDO
,9 RULHQWDUDVUHVSHFWLYDVXQLGDGHVQRTXHVHUHIHUHDR
FXPSULPHQWRGRGLVSRVWRQHVWD/HLHVHXVUHJXODPHQ
WRV 3XEOLFDGDQR'2')QžGH3DJVD
'HFUHWRQžGH'2')GH5HJXODPHQWR
'HFUHWRQžGH'2')GH5HJXODPHQWDRDUW
$UW 23RGHU([HFXWLYR)HGHUDOGHVLJQDUiyUJmRGDDGPL /HLQžGH'2')GH3DJ$OWHUDomR
QLVWUDomRS~EOLFDIHGHUDOUHVSRQViYHO
5HJXOD R DFHVVR D LQIRUPDo}HV QR 'LVWULWR )HGHUDO
, SHODSURPRomRGHFDPSDQKDGHDEUDQJrQFLDQDFLRQDO SUHYLVWRQRDUWž;;;,,,QRDUW†ž,,HQRDUW
GHIRPHQWRjFXOWXUDGDWUDQVSDUrQFLDQDDGPLQLVWUD
†žGD&RQVWLWXLomR)HGHUDOHQRVWHUPRVGRDUWGD/HL
omRS~EOLFDHFRQVFLHQWL]DomRGRGLUHLWRIXQGDPHQWDO
GHDFHVVRjLQIRUPDomR
IHGHUDOQžGHGHQRYHPEURGHHGiRXWUDV
SURYLGrQFLDV
,, SHORWUHLQDPHQWRGHDJHQWHVS~EOLFRVQRTXHVHUHIHUH
DRGHVHQYROYLPHQWRGHSUiWLFDVUHODFLRQDGDVjWUDQVSD &DStWXOR,
UrQFLDQDDGPLQLVWUDomRS~EOLFD '$6',6326,d¯(635(/,0,1$5(6

,,, SHORPRQLWRUDPHQWRGDDSOLFDomRGDOHLQRkPELWRGD $UWž (VWD/HLGLVS}HVREUHRVSURFHGLPHQWRVDVHUHPREVHU


DGPLQLVWUDomRS~EOLFDIHGHUDOFRQFHQWUDQGRHFRQVROL YDGRVSHOR'LVWULWR)HGHUDOYLVDQGRDJDUDQWLURDFHVVR
GDQGRDSXEOLFDomRGHLQIRUPDo}HVHVWDWtVWLFDVUHODFLR DLQIRUPDo}HVSUHYLVWRQRDUWž;;;,,,QRDUW†
QDGDVQRDUW ž,,HQRDUW†žGD&RQVWLWXLomR)HGHUDOQR
DUW,H,,GD/HL2UJkQLFDGR'LVWULWR)HGHUDOHHP
,9 SHOR HQFDPLQKDPHQWR DR &RQJUHVVR 1DFLRQDOGH FRQIRUPLGDGHFRPD/HLIHGHUDOQžGHGH
UHODWyULRDQXDOFRPLQIRUPDo}HVDWLQHQWHVjLPSOHPHQ QRYHPEURGH
WDomRGHVWD/HL
†~QLFR 6XERUGLQDPVHDRUHJLPHGHVWD/HL
$UW 23RGHU([HFXWLYRUHJXODPHQWDUiRGLVSRVWRQHVWD/HL
QRSUD]RGH FHQWRHRLWHQWD GLDVDFRQWDUGDGDWD , RVyUJmRVS~EOLFRVLQWHJUDQWHVGDDGPLQLVWUDomRGLUHWD
GHVXDSXEOLFDomR GRV 3RGHUHV ([HFXWLYR H /HJLVODWLYR LQFOXLQGR R
7ULEXQDOGH&RQWDVGR'LVWULWR)HGHUDO
$UW 2 LQFLVR 9, GR DUW  GD /HL QR  GH GH
GH]HPEUR GH  SDVVD D YLJRUDU FRP D VHJXLQWH ,, DV DXWDUTXLDV DV IXQGDo}HV S~EOLFDV DVHPSUHVDV
UHGDomR S~EOLFDVDVVRFLHGDGHVGHHFRQRPLDPLVWDHDVGHPDLV
HQWLGDGHV FRQWURODGDV GLUHWD RX LQGLUHWDPHQWH SHOR
´$UW 
'LVWULWR)HGHUDO


9, OHYDUDVLUUHJXODULGDGHVGHTXHWLYHUFLrQFLDHPUD]mRGRFDUJRDRFRQKHFLPHQWRGD $UWž $SOLFDPVHDVGLVSRVLo}HVGHVWD/HLQRTXHFRXEHUjV


DXWRULGDGH VXSHULRU RX TXDQGR KRXYHU VXVSHLWD GH HQYROYLPHQWR GHVWD DR
FRQKHFLPHQWRGHRXWUDDXWRULGDGHFRPSHWHQWHSDUDDSXUDomR HQWLGDGHVSULYDGDVVHPILQVOXFUDWLYRVTXHUHFHEDP
µ 15 
SDUDUHDOL]DomRGHDo}HVGHLQWHUHVVHS~EOLFRUHFXUVRV
S~EOLFRV GLUHWDPHQWH GR RUoDPHQWR RX PHGLDQWH
$UW 2&DStWXOR,9GR7tWXOR,9GD/HLQRGH VXEYHQo}HV VRFLDLV FRQWUDWR GH JHVWmR WHUPR GH
SDVVDDYLJRUDUDFUHVFLGRGRVHJXLQWHDUW$ SDUFHULDFRQYrQLRVDFRUGRVDMXVWHVRXRXWURVLQVWUX
PHQWRVFRQJrQHUHV
´$UW$ 1HQKXPVHUYLGRUSRGHUiVHUUHVSRQVDELOL]DGRFLYLOSHQDORXDGPLQLVWUDWL
YDPHQWH SRU GDU FLrQFLD j DXWRULGDGH VXSHULRU RX TXDQGR KRXYHU
VXVSHLWD GH HQYROYLPHQWR GHVWD D RXWUD DXWRULGDGH FRPSHWHQWH SDUD
DSXUDomRGHLQIRUPDomRFRQFHUQHQWHjSUiWLFDGHFULPHVRXLPSURELGDGH
†~QLFR $ SXEOLFLGDGH D TXH HVWmR VXEPHWLGDVDVHQWLGDGHV
GHTXHWHQKDFRQKHFLPHQWRDLQGDTXHHPGHFRUUrQFLDGRH[HUFtFLRGH FLWDGDV QR FDSXW UHIHUHVH j SDUFHOD GRV UHFXUVRV
FDUJRHPSUHJRRXIXQomRS~EOLFDµ
S~EOLFRVUHFHELGRVjVXDGHVWLQDomRHjFRQWUDSDUWLGD
$UW &DEHDRV(VWDGRVDR'LVWULWR)HGHUDOHDRV0XQLFtSLRV VHPSUHMXt]RGDVSUHVWDo}HVGHFRQWDVDTXHHVWHMDP
HP OHJLVODomR SUySULD REHGHFLGDV DV QRUPDV JHUDLV OHJDOPHQWHREULJDGDV
HVWDEHOHFLGDV QHVWD /HL GHILQLU UHJUDV HVSHFtILFDV
HVSHFLDOPHQWH TXDQWR DR GLVSRVWR QR DUW R H QD $UWž 2V SURFHGLPHQWRV SUHYLVWRV QHVWD /HL GHVWLQDPVHD
6HomR,,GR&DStWXOR,,, DVVHJXUDURGLUHLWRIXQGDPHQWDOGHDFHVVRjLQIRUPD
omRHGHYHPVHUH[HFXWDGRVHPFRQIRUPLGDGHFRPRV
$UW 5HYRJDPVH SULQFtSLRVEiVLFRVGDDGPLQLVWUDomRS~EOLFDHFRPDV
VHJXLQWHVGLUHWUL]HV
, D/HLQRGHGHPDLRGHH
, REVHUYkQFLDGDSXEOLFLGDGHFRPRSUHFHLWRJHUDOHGR
,, RVDUWVDGD/HLQRGHGHMDQHLURGH VLJLORFRPRH[FHomR

,, GLYXOJDomRGHLQIRUPDo}HVGHLQWHUHVVHS~EOLFRLQGH
$UW (VWD/HLHQWUDHPYLJRU FHQWRHRLWHQWD GLDVDSyV SHQGHQWHPHQWHGHVROLFLWDo}HV
DGDWDGHVXDSXEOLFDomR
,,, XWLOL]DomRGHPHLRVGHFRPXQLFDomRYLDELOL]DGRVSHOD
WHFQRORJLDGDLQIRUPDomR
 'LU&RQVWLWXFLRQDO &kPDUD/HJLVODWLYDGR'LVWULWR)HGHUDO


,9 IRPHQWRDRGHVHQYROYLPHQWRGDFXOWXUDGHWUDQVSDUrQ $UWž 2DFHVVRjLQIRUPDomRGHTXHWUDWDHVWD/HLFRPSUHHQ


FLDQDDGPLQLVWUDomRS~EOLFD GHHQWUHRXWURVRGLUHLWRGHREWHU

9 GHVHQYROYLPHQWRGRFRQWUROHVRFLDOGDDGPLQLVWUDomR , RULHQWDomRVREUHRVSURFHGLPHQWRVSDUDDFRQVHFXomR


S~EOLFD GH DFHVVR EHP FRPR VREUH R ORFDO RQGH SRGH VHU
HQFRQWUDGDRXREWLGDDLQIRUPDomRDOPHMDGD
$UWž 3DUDRVHIHLWRVGHVWD/HLHGDVGHPDLVGLVSRVLo}HVGD
OHJLVODomRGLVWULWDOVHPFRQFHLWRSUySULRFRQVLGHUDVH ,, LQIRUPDomR FRQWLGD HP UHJLVWURV RXGRFXPHQWRV
SURGX]LGRVRXDFXPXODGRVSRUyUJmRVRXHQWLGDGHV
, LQIRUPDomRGDGRVSURFHVVDGRVRXQmRTXHSRGHPVHU UHFROKLGRVRXQmRDDUTXLYRVS~EOLFRV
XWLOL]DGRVSDUDDSURGXomRHDWUDQVPLVVmRGHFRQKHFL
PHQWRFRQWLGRVHPTXDOTXHUPHLRVXSRUWHRXIRUPD ,,, LQIRUPDomRSURGX]LGDRXFXVWRGLDGDSRUSHVVRDItVLFD
WR RXMXUtGLFDHPYLUWXGHGHTXDOTXHUYtQFXORFRPyUJmRV
RX HQWLGDGHV S~EOLFDV PHVPR TXH HVVH YtQFXOR Mi
,, GRFXPHQWR XQLGDGH GH UHJLVWUR GHLQIRUPDo}HV WHQKDFHVVDGR
TXDOTXHUTXHVHMDRVXSRUWHRXRIRUPDWR
,9 LQIRUPDomRSULPiULDtQWHJUDDXWrQWLFDHDWXDOL]DGD
,,, LQIRUPDomRVLJLORVDDTXHODVXEPHWLGDWHPSRUDULDPHQ
WH j UHVWULomR GH DFHVVR S~EOLFR HP UD]mR GH VXD 9 LQIRUPDomRVREUHDWLYLGDGHVH[HUFLGDVSRUyUJmRVRX
HQWLGDGHVLQFOXVLYHDVUHODWLYDVjVXDSROtWLFDjVXD
LPSUHVFLQGLELOLGDGHSDUDDVHJXUDQoDGDVRFLHGDGHH
RUJDQL]DomRHDRVVHXVVHUYLoRV
GR(VWDGR
9, LQIRUPDomRSHUWLQHQWHDDGPLQLVWUDomRGRSDWULP{QLR
,9 LQIRUPDomR SHVVRDO DTXHOD UHODFLRQDGD jSHVVRD
S~EOLFR XWLOL]DomR GH UHFXUVRV S~EOLFRV OLFLWDo}HV
QDWXUDOLGHQWLILFDGDRXLGHQWLILFiYHO
FRQWUDWRV DGPLQLVWUDWLYRV FRQYrQLRV H LQVWUXPHQWRV
FRQJrQHUHV
9 WUDWDPHQWRGDLQIRUPDomRFRQMXQWRGHDo}HVUHIHUHQ
WHVjSURGXomRjUHFHSomRjFODVVLILFDomRjXWLOL]DomR 9,, LQIRUPDomRUHODWLYD
DRDFHVVRjUHSURGXomRDRWUDQVSRUWHjWUDQVPLVVmR
jGLVWULEXLomRDRDUTXLYDPHQWRDRDUPD]HQDPHQWR D j LPSOHPHQWDomR DR DFRPSDQKDPHQWR H DRV
jHOLPLQDomRjDYDOLDomRjGHVWLQDomRRXDRFRQWUROH UHVXOWDGRV GH SURJUDPDV SURMHWRV H Do}HV GRV
GDLQIRUPDomR yUJmRV H GDV HQWLGDGHV S~EOLFDV EHP FRPR jV
PHWDVHDRVLQGLFDGRUHVSURSRVWRV
9, GLVSRQLELOLGDGHTXDOLGDGHGDLQIRUPDomRTXHSRGHVHU
FRQKHFLGDHXWLOL]DGDSRULQGLYtGXRVHTXLSDPHQWRVRX E DRUHVXOWDGRGHLQVSHo}HVDXGLWRULDVSUHVWDo}HVH
VLVWHPDVDXWRUL]DGRV WRPDGDV GH FRQWDV UHDOL]DGDV SHORV yUJmRV GH
FRQWUROHLQWHUQRHH[WHUQRLQFOXLQGRSUHVWDo}HVGH
9,, DXWHQWLFLGDGHTXDOLGDGHGDLQIRUPDomRTXHWHQKDVLGR FRQWDVUHODWLYDVDH[HUFtFLRVDQWHULRUHV
SURGX]LGD H[SHGLGD UHFHELGD RX PRGLILFDGD SRU
GHWHUPLQDGRLQGLYtGXRHTXLSDPHQWRRXVLVWHPD †ž 2DFHVVRjLQIRUPDomRSUHYLVWRQRFDSXWQmRFRPSUH
HQGHDVLQIRUPDo}HVUHIHUHQWHVDSURMHWRVGHSHVTXLVD
9,,, LQWHJULGDGHTXDOLGDGHGDLQIRUPDomRQmRPRGLILFDGD H GHVHQYROYLPHQWR FLHQWtILFRV RX WHFQROyJLFRV FXMR
LQFOXVLYHTXDQWRjRULJHPDRWUkQVLWRHDRGHVWLQR VLJLORVHMDLPSUHVFLQGtYHOjVHJXUDQoDGDVRFLHGDGHGR
(VWDGRRXGR'LVWULWR)HGHUDO
,; SULPDULHGDGH TXDOLGDGH GD LQIRUPDomR FROHWDGDQD
IRQWHFRPRPi[LPRGHGHWDOKDPHQWRSRVVtYHOVHP †ž TXDQGRQmRIRUDXWRUL]DGRDFHVVRLQWHJUDOjLQIRUPD
PRGLILFDo}HV omR SDUFLDOPHQWH VLJLORVD p DVVHJXUDGR R DFHVVR j
SDUWH QmR VLJLORVD SRU PHLR GH FHUWLGmR H[WUDWR RX
$UWž e GHYHU GR (VWDGR JDUDQWLU R GLUHLWR GH DFHVVRj FySLDFRPRFXOWDomRGDSDUWHVREVLJLOR
LQIRUPDomRDVHUIUDQTXHDGRPHGLDQWHSURFHGLPHQWRV
REMHWLYRVHiJHLVGHIRUPDWUDQVSDUHQWHHFODUDHHP †ž 2GLUHLWRGHDFHVVRDRVGRFXPHQWRVRXjVLQIRUPDo}HV
OLQJXDJHPGHIiFLOFRPSUHHQVmR QHOHVFRQWLGDVXWLOL]DGRVFRPRIXQGDPHQWRGDWRPDGD
GHGHFLVmRHGRDWRDGPLQLVWUDWLYRpDVVHJXUDGRFRPD
&DStWXOR,, HGLomRGRDWRGHFLVyULRUHVSHFWLYR
'2$&(662$,1)250$d¯(6('$68$',98/*$d®2
†ž $QHJDWLYDGHDFHVVRjVLQIRUPDo}HVREMHWRGHSHGLGR
$UWž &DEH DRV yUJmRV H jV HQWLGDGHV GR 3RGHU3~EOLFR IRUPXODGRDRVyUJmRVHjVHQWLGDGHVUHIHULGDVQRDUW
'LVWULWDO REVHUYDGDV DV QRUPDV H RV SURFHGLPHQWRV žTXDQGRQmRIXQGDPHQWDGDVXMHLWDRUHVSRQViYHOD
HVSHFtILFRVDSOLFiYHLVDVVHJXUDU PHGLGDVGLVFLSOLQDUHVQRVWHUPRVGRDUW

, D JHVWmR WUDQVSDUHQWH GD LQIRUPDomRSURSLFLDQGR †ž ,QIRUPDGRGRH[WUDYLRGDLQIRUPDomRVROLFLWDGDSRGH


DPSORDFHVVRDHODHVXDGLYXOJDomR R LQWHUHVVDGR UHTXHUHU j DXWRULGDGH FRPSHWHQWH D
LPHGLDWDDEHUWXUDGHVLQGLFkQFLDSDUDDSXUDURGHVDSD
,, DSURWHomRGDLQIRUPDomRJDUDQWLQGRVXDGLVSRQLELOL UHFLPHQWRGDUHVSHFWLYDGRFXPHQWDomR
GDGHDXWHQWLFLGDGHHLQWHJULGDGH
†ž 9HULILFDGDDKLSyWHVHSUHYLVWDQR†žRUHVSRQViYHO
SHODJXDUGDGDLQIRUPDomRH[WUDYLDGDGHYHQRSUD]R
,,, D SURWHomR GD LQIRUPDomR VLJLORVD H GDLQIRUPDomR
GHGH]GLDVMXVWLILFDURIDWRHLQGLFDUWHVWHPXQKDVTXH
SHVVRDOREVHUYDGDDVXDGLVSRQLELOLGDGHDXWHQWLFLGD
FRPSURYHPVXDDOHJDomR
GHLQWHJULGDGHHHYHQWXDOUHVWULomRGHDFHVVR
 &kPDUD/HJLVODWLYDGR
 'LVWULWR)HGHUDO 'LU&RQVWLWXFLRQDO 

$UWž 3DUDDLPSOHPHQWDomRGHVWD/HLRVyUJmRVHDVHQWLGD $UWž 3DUDFXPSULPHQWRGRGLVSRVWRQRDUWžRVyUJmRVH


GHVGR'LVWULWR)HGHUDOGHYHPSURPRYHULQGHSHQGHQ DVHQWLGDGHVS~EOLFDVGHYHPXWLOL]DUDGLYXOJDomRHP
WHPHQWHGHUHTXHULPHQWRVDGLYXOJDomRQRkPELWRGH VtWLRV RILFLDLV QD 5HGH 0XQGLDO GH &RPSXWDGRUHV 
VXDVFRPSHWrQFLDVGHLQIRUPDo}HVGHLQWHUHVVHFROHWL LQWHUQHW
YRRXJHUDOSRUHOHVSURGX]LGDVRXFXVWRGLDGDV
†ž 2V VtWLRV GH TXH WUDWD R FDSXW GHYHPDWHQGHUQR
†~QLFR 1DGLYXOJDomRGDVLQIRUPDo}HVDTXHVHUHIHUHRFDSXW PtQLPRDRVVHJXLQWHVUHTXLVLWRV
GHYHPFRQVWDUQRPtQLPR
, FRQWHU IHUUDPHQWD GH SHVTXLVD GH FRQWH~GRTXH
, UHJLVWURGDVFRPSHWrQFLDVHGDHVWUXWXUDRUJDQL]DFLR SHUPLWD R DFHVVR j LQIRUPDomR GH IRUPD REMHWLYD
QDOHQGHUHoRVWHOHIRQHVHFRUUHLRHOHWU{QLFRLQVWLWXFLR WUDQVSDUHQWHFODUDHHPOLQJXDJHPGHIiFLOFRPSUHHQ
QDOGDVUHVSHFWLYDVXQLGDGHVHKRUiULRVGHDWHQGLPHQWR VmR
DRS~EOLFR
,, SRVVLELOLWDU D JUDYDomR GH UHODWyULRV HPGLYHUVRV
,, UHJLVWUR GH TXDLVTXHU UHSDVVHV RX WUDQVIHUrQFLDVGH IRUPDWRVHOHWU{QLFRVLQFOXVLYHDEHUWRVHQmRSURSULHWi
UHFXUVRVILQDQFHLURV ULRVGHPRGRDIDFLOLWDUDDQiOLVHGDVLQIRUPDo}HV

,,, UHJLVWURGDVGHVSHVDV ,,, SRVVLELOLWDURDFHVVRDXWRPDWL]DGRSRUVLVWHPDVH[WHU


QRVHPIRUPDWRVDEHUWRVHVWUXWXUDGRVHOHJtYHLVSRU
PiTXLQD
,9 UHVXOWDGRV GH LQVSHo}HV H DXGLWRULDV SUHVWDo}HVGH
FRQWDVHWRPDGDVGHFRQWDVHVSHFLDLVUHDOL]DGDVSHORV
,9 GLYXOJDU HP GHWDOKHV RV IRUPDWRV XWLOL]DGRVSDUD
yUJmRVGHFRQWUROHLQWHUQRHH[WHUQRLQFOXLQGRSUHVWD
HVWUXWXUDomRGDLQIRUPDomR
omRGHFRQWDVUHODWLYDVDH[HUFtFLRVDQWHULRUHV
9 JDUDQWLUDDXWHQWLFLGDGHHDLQWHJULGDGHGDVLQIRUPD
9 LQIRUPDo}HVFRQFHUQHQWHVDSURFHGLPHQWRVOLFLWDWyULRV
o}HVGLVSRQtYHLVSDUDDFHVVR
LQFOXVLYH DRV UHVSHFWLYRV HGLWDLV H UHVXOWDGRV EHP
FRPRDWRGRVRVFRQWUDWRVFHOHEUDGRV 9, PDQWHU DWXDOL]DGDV DV LQIRUPDo}HV GLVSRQtYHLVSDUD
DFHVVR
9, GDGRVJHUDLVSDUDRDFRPSDQKDPHQWRGHSURJUDPDV
Do}HV SURMHWRV H REUDV FRP LQIRUPDo}HV VREUH VXD 9,, LQGLFDUORFDOHLQVWUXo}HVTXHSHUPLWDPDRLQWHUHVVDGR
H[HFXomRPHWDVHLQGLFDGRUHVHPOLQJXDJHPGHIiFLO FRPXQLFDUVHSRUYLDHOHWU{QLFDRXWHOHI{QLFDFRPR
FRPSUHHQVmR yUJmRRXDHQWLGDGHGHWHQWRUDGRVtWLR

9,, UHVSRVWDV D SHUJXQWDV PDLV IUHTXHQWHV IHLWDVSHOD 9,,, DGRWDUDVPHGLGDVQHFHVViULDVSDUDJDUDQWLUDDFHVVLEL


VRFLHGDGH OLGDGHGHFRQWH~GRSDUDSHVVRDVFRPGHILFLrQFLDQRV
WHUPRVGRDUWGD/HLIHGHUDOQžGHGH
9,,, GDGRVHH[HFXomRGHSURJUDPDVGHGHVHQYROYLPHQWR GH]HPEURGHHGRDUWžGD&RQYHQomRVREUHRV
VRFLDOHKDELWDFLRQDO 'LUHLWRV GDV 3HVVRDV FRP 'HILFLrQFLD DSURYDGD SHOR
'HFUHWR/HJLVODWLYRQžGHGHMXOKRGH
,; FULWpULRVGHDORFDomRHGHXVRGRVUHFXUVRVGHFRUUHQWHV
GHIXQGRVS~EOLFRV ,; FRQWHURVVHJXLQWHVLQVWUXPHQWRVGHDFHVVRjVLQIRUPD
o}HVDUTXLYtVWLFDVGRyUJmRRXGDHQWLGDGH
; FRQWUDWRVGHJHVWmRILUPDGRVFRPHQWLGDGHVTXDOLILFD
GDVFRPRRUJDQL]Do}HVVRFLDLV D &yGLJRGH&ODVVLILFDomRGH'RFXPHQWRVGH$UTXLYR
GDVDWLYLGDGHVPHLRHGDVDWLYLGDGHVILP
;, LQIRUPDo}HVVREUHFRQWUROHHILVFDOL]DomRGHUHFXUVRV
S~EOLFRVGHVWLQDGRVDRUJDQL]Do}HVQmRJRYHUQDPHQWD E 7DEHOD GH 7HPSRUDOLGDGH H 'HVWLQDomR GH 'RFX
LV PHQWRVGDVDWLYLGDGHVPHLRHGDVDWLYLGDGHVILP

;,, 9(7$'2  F 9RFDEXOiULR &RQWURODGR GH WHUPRV UHODWLYRV DRV


GRFXPHQWRVGHDUTXLYRGDVDWLYLGDGHVPHLRHGDV
;,,, YDORUHVHFULWpULRVGHWUDQVIHUrQFLDGHUHFXUVRVILQDQFH DWLYLGDGHVILP
LURVjVXQLGDGHVHVFRODUHVHjVGLUHWRULDVUHJLRQDLVGH
HQVLQRSRUPHLRGHVXDVUHVSHFWLYDVXQLGDGHVH[HFXWR †ž $ HVWUXWXUD H R FRQMXQWR GH LQIRUPDo}HVS~EOLFDVD
UDV VHUHP GLVSRQLELOL]DGDV QR VtWLR GRV yUJmRV H GDV
HQWLGDGHV GHYHP REVHUYDU R PRGHOR SDGURQL]DGR
;,9 UHODomRGHUHFODPDo}HVFRQWUDIRUQHFHGRUHVGHSURGX GHILQLGRSHORVyUJmRVFRPSHWHQWHVGR'LVWULWR)HGHUDO
WRVHGHVHUYLoRV
$UW 2VyUJmRVHDVHQWLGDGHVGR3RGHU3~EOLFRGHYHPFULDU
;9 UHODWyULRV FRP DYDOLDo}HV H GDGRV GD H[HFXomR HGD VHUYLoR GH LQIRUPDo}HV DR FLGDGmR HP ORFDO FRP
XWLOL]DomRGDVJUDWXLGDGHVFRQFHGLGDVSHOR6LVWHPDGH FRQGLo}HVDSURSULDGDVSDUD
7UDQVSRUWH 3~EOLFR &ROHWLYR GR 'LVWULWR )HGHUDO jV
SHVVRDVFRPGHILFLrQFLDHDVHXVDFRPSDQKDQWHV , DWHQGHUHRULHQWDURS~EOLFRTXDQWRDRDFHVVRDLQIRU
PDo}HV
;9, UHODWyULRVFRPDYDOLDomRHGDGRVGDH[HFXomRGR3DVVH
/LYUH(VWXGDQWLO DFUHVFHQWDGRRLQFLVR[YLLDRSDUDJUDIR~QLFRGRDUWžSHOD ,, SURWRFROL]DUGRFXPHQWRVHUHTXHULPHQWRVGHDFHVVRD
OHLQžGHGRGIGH LQIRUPDo}HV

;9,,UHODomR GRV FDUJRV HP FRPLVVmR H GHSURYLPHQWR ,,, LQIRUPDUVREUHDWUDPLWDomRGHGRFXPHQWRVQDVVXDV


HIHWLYRRFXSDGRVHYDJRVHPFDGDyUJmRRXHQWLGDGH UHVSHFWLYDVXQLGDGHV
 'LU&RQVWLWXFLRQDO &kPDUD/HJLVODWLYDGR'LVWULWR)HGHUDO


$UW &DEH DRV yUJmRV H jV HQWLGDGHV GDDGPLQLVWUDomR †ž 2SUD]RUHIHULGRQR†žSRGHVHUSURUURJDGRSRUPDLV


S~EOLFDUHDOL]DUGHQWURGHVXDViUHDVGHFRPSHWrQFLD GH]GLDVPHGLDQWHMXVWLILFDWLYDH[SUHVVDGHTXHVHUi
DXGLrQFLDV RX FRQVXOWDV S~EOLFDV LQFHQWLYDQGR D FLHQWLILFDGRRUHTXHUHQWH
SDUWLFLSDomRSRSXODU
†ž 6HPSUHMXt]RGDVHJXUDQoDHGDSURWHomRGDVLQIRUPD
$UW 26HUYLoRGH,QIRUPDo}HVDR&LGDGmR6,&QR3RGHU o}HVHGRFXPSULPHQWRGDOHJLVODomRDSOLFiYHORyUJmR
([HFXWLYRIXQFLRQDQDVRXYLGRULDVGHFDGDyUJmR RXDHQWLGDGHSRGHRIHUHFHUPHLRVSDUDTXHRSUySULR
UHTXHUHQWH SRVVD SHVTXLVDU D LQIRUPDomR GH TXH
†ž 2VUHFXUVRVKXPDQRVWHFQROyJLFRVORJtVWLFRVHRUoD QHFHVVLWDU
PHQWiULRVSDUDDLPSODQWDomRGRV6HUYLoRVGH,QIRUPD
o}HVDR&LGDGmRVmRGLVSRQLELOL]DGRVSHORVUHVSHFWLYRV
†ž 4XDQGRQmRIRUDXWRUL]DGRRDFHVVRSRUVHWUDWDUGH
yUJmRVHHQWLGDGHV
LQIRUPDomRWRWDORXSDUFLDOPHQWHVLJLORVDRUHTXHUHQWH
†ž )LFDD2XYLGRULD*HUDOGR'LVWULWR)HGHUDOQR3RGHU GHYHVHULQIRUPDGRVREUHDSRVVLELOLGDGHGHUHFXUVRRV
([HFXWLYRUHVSRQViYHOSRURULHQWDURIXQFLRQDPHQWR SUD]RVHDVFRQGLo}HVSDUDVXDLQWHUSRVLomRGHYHQGR
GRV6HUYLoRVGH,QIRUPDo}HVDR&LGDGmRLQFOXLQGRD DLQGDVHUOKHLQGLFDGDDDXWRULGDGHFRPSHWHQWHSDUD
HODERUDomRGHIOX[RLQWHUQRSDUDUHFHSomRHWUDWDPHQ VXDDSUHFLDomR
WRGRVSHGLGRVEHPFRPRRWUHLQDPHQWRGHVHUYLGRUHV
†ž $LQIRUPDomRDUPD]HQDGDHPIRUPDWRGLJLWDOSRGHVHU
$UW 23RGHU([HFXWLYRGLVSRQLELOL]DUiDRVFLGDGmRVFHUWL IRUQHFLGDQHVVHIRUPDWRFDVRKDMDDQXrQFLDGRUHTXH
G}HVUHIHUHQWHVjDGPLQLVWUDomRS~EOLFDHPVHXVtWLR UHQWH
RILFLDOVHPTXDOTXHUFXVWR
†ž &DVR D LQIRUPDomR VROLFLWDGD HVWHMDGLVSRQtYHODR
&DStWXOR,,, S~EOLFRHPIRUPDWRLPSUHVVRHOHWU{QLFRRXHPTXDO
'2352&(',0(172'($&(662­,1)250$d®2 TXHURXWURPHLRGHDFHVVRXQLYHUVDOGHYHPVHULQIRU
PDGRVDRUHTXHUHQWHROXJDUHDIRUPDSHODTXDOVH
6HomR, SRGHFRQVXOWDUREWHURXUHSURGX]LUDUHIHULGDLQIRUPD
'R3HGLGRGH$FHVVR omR SURFHGLPHQWR HVVH TXH GHVRQHUD R yUJmR RX D
HQWLGDGH S~EOLFD GD REULJDomR GH VHX IRUQHFLPHQWR
$UW 4XDOTXHU LQWHUHVVDGR SRGH DSUHVHQWDU SHGLGRGH
GLUHWR VDOYRVHRUHTXHUHQWHGHFODUDUQmRGLVSRUGH
DFHVVRDLQIRUPDo}HVDRVyUJmRVHjVHQWLGDGHVUHIHUL
GRVQRDUWžGHVWD/HLSRUTXDOTXHUPHLROHJtWLPR PHLRVSDUDUHDOL]DUWDLVSURFHGLPHQWRV
GHYHQGRRSHGLGRFRQWHUDLGHQWLILFDomRGRUHTXHUHQWH
HDHVSHFLILFDomRGDLQIRUPDomRUHTXHULGD $UW 2VHUYLoRGHEXVFDHRIRUQHFLPHQWRGDLQIRUPDomRVmR
JUDWXLWRVVDOYRQDVKLSyWHVHVGHUHSURGXomRGHGRFX
†ž $ LGHQWLILFDomR GH TXH WUDWD R FDSXW p IHLWDFRPD PHQWRVSHORyUJmRRXSHODHQWLGDGHGR3RGHU3~EOLFR
LQGLFDomRGRQRPHFRPSOHWRGRQ~PHURGHTXDOTXHU FRQVXOWDGRVLWXDomRHPTXHGHYHVHUFREUDGRH[FOXVL
GRFXPHQWRRILFLDOHGDLQIRUPDomRGHFRQWDWRVHQGR YDPHQWHRYDORUQHFHVViULRDRUHVVDUFLPHQWRGRFXVWR
IDFXOWDGD D LQFOXVmR GH HQGHUHoR HOHWU{QLFR SDUD R GRVVHUYLoRVHGRVPDWHULDLVXWLOL]DGRV
UHFHELPHQWRGDVLQIRUPDo}HVVROLFLWDGDV
†~QLFR (VWi LVHQWR GH UHVVDUFLU RV FXVWRV SUHYLVWRVQRFDSXW
†ž 1R FDVR GH R UHTXHUHQWH VHU PHQRU GH LGDGHHQmR WRGRDTXHOHFXMDVLWXDomRHFRQ{PLFDQmROKHSHUPLWD
SRVVXLU GRFXPHQWR RILFLDO GHYH VHU LQIRUPDGR R ID]rORVHPSUHMXt]RGRVXVWHQWRSUySULRRXGDIDPtOLD
Q~PHURGRGRFXPHQWRGRVSDLVRXGRVUHVSRQViYHLV GHFODUDGDQRVWHUPRVGD/HLIHGHUDOQžGHGH
DJRVWRGH
†ž 2V yUJmRV H DV HQWLGDGHV GR 3RGHU3~EOLFRGHYHP
YLDELOL]DUDOWHUQDWLYDGHHQFDPLQKDPHQWRGHSHGLGRV $UW 4XDQGRVHWUDWDUGHDFHVVRjLQIRUPDomRFRQWLGDHP
GHDFHVVRSRUPHLRGHVHXVVtWLRVRILFLDLVQDLQWHUQHW GRFXPHQWR FXMD PDQLSXODomR SRVVD SUHMXGLFDU VXD
LQWHJULGDGH GHYH VHU RIHUHFLGD D FRQVXOWD GH FySLD
†ž 6mRYHGDGDVTXDLVTXHUH[LJrQFLDVUHODWLYDVDRVPRWLYRV
FRPFHUWLILFDomRGHTXHHVWDFRQIHUHFRPRRULJLQDO
GHWHUPLQDQWHVGDVROLFLWDomRGHLQIRUPDo}HVGHLQWH
UHVVHS~EOLFR
†~QLFR 1DLPSRVVLELOLGDGHGHREWHQomRGHFySLDVRLQWHUHVVD
$UW 2yUJmRRXDHQWLGDGHGR3RGHU3~EOLFRGHYHDXWRUL]DU GRSRGHVROLFLWDUTXHjVVXDVH[SHQVDVHVREVXSHUYL
RXFRQFHGHURDFHVVRLPHGLDWRjLQIRUPDomRGLVSRQt VmR GH VHUYLGRU S~EOLFR D UHSURGXomR VHMD IHLWD SRU
YHO RXWURPHLRTXHQmRSRQKDHPULVFRDFRQVHUYDomRGR
GRFXPHQWRRULJLQDO
†ž 1mR VHQGR SRVVtYHO FRQFHGHU R DFHVVRLPHGLDWRj
LQIRUPDomRQDIRUPDGLVSRVWDQRFDSXWRyUJmRRXD $UW eGLUHLWRGRUHTXHUHQWHREWHURLQWHLURWHRUGHGHFLVmR
HQWLGDGH TXH UHFHEHU R SHGLGR GHYH HP SUD]R QmR GHQHJDWLYDGHDFHVVRSRUFHUWLGmRRXFySLD
VXSHULRUDYLQWHGLDV
6HomR,,
, FRPXQLFDUDGDWDRORFDOHRPRGRSDUDVHUHDOL]DUD 'RV5HFXUVRV
FRQVXOWDHIHWXDUDUHSURGXomRRXREWHUDFHUWLGmR
$UW 1RFDVRGHLQGHIHULPHQWRGHDFHVVRjVLQIRUPDo}HVRX
,, LQGLFDUDVUD]}HVGHIDWRRXGHGLUHLWRGDUHFXVDWRWDO jV UD]}HVGDQHJDWLYDGRDFHVVRSRGHRLQWHUHVVDGR
RXSDUFLDOGRDFHVVRSUHWHQGLGR LQWHUSRUUHFXUVRFRQWUDDGHFLVmRQRSUD]RGHGH]GLDV
DFRQWDUGDVXDFLrQFLD
,,, FRPXQLFDU TXH QmR SRVVXL D LQIRUPDomR VROLFLWDGDH
LQGLFDU VH IRU GR VHX FRQKHFLPHQWR R yUJmR RX D
HQWLGDGHTXHDGHWpPRXDLQGDUHPHWHURUHTXHUL †~QLFR 2UHFXUVRGHYHVHUGLULJLGRjDXWRULGDGHKLHUDUTXLFD
PHQWR D HVVH yUJmR RX HQWLGDGH FLHQWLILFDQGR R PHQWHVXSHULRUjTXHH[DURXDGHFLVmRLPSXJQDGDTXH
LQWHUHVVDGRGDUHPHVVDGHVHXSHGLGRGHLQIRUPDomR GHYHVHPDQLIHVWDUQRSUD]RGHFLQFRGLDV
&kPDUD/HJLVODWLYDGR
  'LVWULWR)HGHUDO 'LU&RQVWLWXFLRQDO 

$UW 1HJDGR R DFHVVR j LQIRUPDomR R UHTXHUHQWHSRGH 6HomR,,


UHFRUUHU j 6HFUHWDULD GH (VWDGR GH 7UDQVSDUrQFLD H 'D&ODVVLILFDomRGD,QIRUPDomRTXDQWRDR*UDX
&RQWUROHTXHGHYHGHOLEHUDUQRSUD]RGHFLQFRGLDV HGRV3UD]RVGH6LJLOR
VH
$UW 6mR FRQVLGHUDGDV LPSUHVFLQGtYHLV j VHJXUDQoDGD
, RDFHVVRjLQIRUPDomRQmRFODVVLILFDGDFRPRVLJLORVD VRFLHGDGH RX GR (VWDGR H SRUWDQWR SDVVtYHLV GH
IRUQHJDGR FODVVLILFDomRDVLQIRUPDo}HVFXMDGLYXOJDomRRXDFHVVR
LUUHVWULWRSRVVDP
,, DGHFLVmRGHQHJDWLYDGHDFHVVRjLQIRUPDomRWRWDORX
SDUFLDOPHQWHFODVVLILFDGDFRPRVLJLORVDQmRLQGLFDUD , S{U HP ULVFR D GHIHVD H D VREHUDQLD QDFLRQDLV RXD
DXWRULGDGH FODVVLILFDGRUD RX D KLHUDUTXLFDPHQWH LQWHJULGDGHGRWHUULWyULRQDFLRQDO
VXSHULRUDTXHPSRVVDVHUGLULJLGRSHGLGRGHDFHVVRRX
GHVFODVVLILFDomR ,, SUHMXGLFDURXS{UHPULVFRDFRQGXomRGHQHJRFLDo}HV
RXDVUHODo}HVLQWHUQDFLRQDLVGR3DtVRXDVLQIRUPD
,,, RV SURFHGLPHQWRV GH FODVVLILFDomR GHLQIRUPDomR o}HVTXHWHQKDPVLGRIRUQHFLGDVHPFDUiWHUVLJLORVR
VLJLORVD HVWDEHOHFLGRV QHVWD /HL QmR WLYHUHP VLGR SRURXWURV(VWDGRVHRUJDQLVPRVLQWHUQDFLRQDLV
REVHUYDGRV
,,, S{UHPULVFRDYLGDDVHJXUDQoDRXDVD~GHGDSRSXOD
,9 HVWLYHUHP VHQGR GHVFXPSULGRV SUD]RV RXRXWURV omR
SURFHGLPHQWRVSUHYLVWRVQHVWD/HL
,9 RIHUHFHUHOHYDGRULVFRjHVWDELOLGDGHILQDQFHLUDHFRQ{
†ž 2 UHFXUVR SUHYLVWR QHVWH DUWLJR VRPHQWHSRGHVHU PLFDRXPRQHWiULDGR3DtV
GLULJLGR j 6HFUHWDULD GH (VWDGR GH 7UDQVSDUrQFLD H
&RQWUROH GHSRLV GH VXEPHWLGR j DSUHFLDomR GH SHOR 9 SUHMXGLFDU RX FDXVDU ULVFR D SODQRV RXRSHUDo}HV
PHQRV XPD DXWRULGDGH KLHUDUTXLFDPHQWH VXSHULRU HVWUDWpJLFRVGDV)RUoDV$UPDGDV
jTXHODTXHH[DURXDGHFLVmRLPSXJQDGD
9, SUHMXGLFDURXFDXVDUULVFRDSURMHWRVGHSHVTXLVDHGH
†ž 9(7$'2  GHVHQYROYLPHQWRFLHQWtILFRRXWHFQROyJLFRDVVLPFRPR
D VLVWHPDV EHQV LQVWDODo}HV RX iUHDV GH LQWHUHVVH
$UW 1HJDGR R SHGLGR GH GHVFODVVLILFDomR GHLQIRUPDomR HVWUDWpJLFRQDFLRQDO
SURWRFRODGRHPyUJmRRXHQWLGDGHSRGHRUHTXHUHQWH
UHFRUUHUDR6HFUHWiULRGH(VWDGRGDiUHD 9,, S{UHPULVFRDVHJXUDQoD GHLQVWLWXLo}HVRXGHDOWDV
DXWRULGDGHVGLVWULWDLVQDFLRQDLVRXHVWUDQJHLUDVHGH
†~QLFR 2 UHFXUVR SUHYLVWR QHVWH DUWLJR VRPHQWHSRGHVHU VHXVIDPLOLDUHV
GLULJLGRjDXWRULGDGHPHQFLRQDGDGHSRLVGHVXEPHWLGR
jDSUHFLDomRGHSHORPHQRVXPDDXWRULGDGHKLHUDUTXL 9,,, FRPSURPHWHUDWLYLGDGHVGHLQWHOLJrQFLDEHPFRPRGH
FDPHQWHVXSHULRUjDXWRULGDGHTXHH[DURXDGHFLVmR LQYHVWLJDomRRXILVFDOL]DomRHPDQGDPHQWRUHODFLRQD
LPSXJQDGD GDVFRPDSUHYHQomRRXDUHSUHVVmRGHLQIUDo}HV

$UW 2VSURFHGLPHQWRVGHUHYLVmRGHGHFLV}HVGHQHJDWyULDV $UW $ LQIRUPDomR HP SRGHU GRV yUJmRV H GDVHQWLGDGHV


SURIHULGDV QRV UHFXUVRV SUHYLVWRV QHVWD VHomR H GH VXMHLWRVDHVWD/HLREVHUYDGRRVHXWHRUHHPUD]mRGH
UHYLVmR GH FODVVLILFDomR GH GRFXPHQWRV VLJLORVRV VmR VXDLPSUHVFLQGLELOLGDGHjVHJXUDQoDGDVRFLHGDGHRX
REMHWRGHUHJXODPHQWDomRSUySULDSHOD&kPDUD/HJLVOD GR'LVWULWR)HGHUDOSRGHVHUFODVVLILFDGDFRPRXOWUDV
WLYDGR'LVWULWR)HGHUDOHSHOR7ULEXQDOGH&RQWDVGR VHFUHWDVHFUHWDRXUHVHUYDGD
'LVWULWR)HGHUDOHPVHXVUHVSHFWLYRVkPELWRVDVVHJX
UDGRDRVROLFLWDQWHHPTXDOTXHUFDVRRGLUHLWRGHVHU †ž 2VSUD]RVPi[LPRVGHUHVWULomRGHDFHVVRjLQIRUPDomR
LQIRUPDGRVREUHRDQGDPHQWRGHVHXSHGLGR YLJRUDP D SDUWLU GD GDWD GH VXD SURGXomR H VmR RV
VHJXLQWHV
&DStWXOR,9
'$65(675,d¯(6'($&(662­,1)250$d®2 , XOWUDVVHFUHWDYLQWHHFLQFRDQRV
,, VHFUHWDTXLQ]HDQRV
6HomR, ,,, UHVHUYDGDFLQFRDQRV
'DV'LVSRVLo}HV*HUDLV
†ž $VLQIRUPDo}HVTXHSRVVDPFRORFDUHPULVFRDVHJXUDQ
$UW 1mRSRGHVHUQHJDGRDFHVVRjLQIRUPDomRQHFHVViULDj oDGR3UHVLGHQWHHGR9LFH3UHVLGHQWHGD5HS~EOLFDGR
WXWHODMXGLFLDORXDGPLQLVWUDWLYDGHGLUHLWRVIXQGDPHQ *RYHUQDGRU H GR 9LFH*RYHUQDGRU GRV UHVSHFWLYRV
WDLV F{QMXJHV RX GHVFHQGHQWHV VmR FODVVLILFDGDV FRPR
UHVHUYDGDVHILFDPVREVLJLORDWpRWpUPLQRGRPDQGDWR
†~QLFR $VLQIRUPDo}HVRXRVGRFXPHQWRVTXHYHUVHPVREUH HP H[HUFtFLR RX GR ~OWLPR PDQGDWR HP FDVR GH
FRQGXWDVTXHLPSOLTXHPYLRODomRGRVGLUHLWRVKXPD UHHOHLomR
QRV SUDWLFDGD SRU DJHQWHV S~EOLFRV RX D PDQGR GH
DXWRULGDGHVS~EOLFDVQmRSRGHPVHUREMHWRGHUHVWULomR †ž $OWHUQDWLYDPHQWHDRVSUD]RVSUHYLVWRVQR†žSRGH
GHDFHVVR VHU HVWDEHOHFLGD FRPR WHUPR ILQDO GH UHVWULomR GH
DFHVVRDRFRUUrQFLDGHGHWHUPLQDGRHYHQWRGHVGHTXH
$UW 2 GLVSRVWR QHVWD /HL QmR H[FOXL DV GHPDLVKLSyWHVHV HVWHRFRUUDDQWHVGRWUDQVFXUVRGRSUD]RPi[LPRGH
OHJDLVGHVLJLORHGHVHJUHGRGHMXVWLoDQHPDVKLSyWH FODVVLILFDomR
VHVGHVHJUHGRHPSUHVDULDOGHFRUUHQWHVGDH[SORUDomR
GLUHWDGHDWLYLGDGHHFRQ{PLFDSHOR'LVWULWR)HGHUDORX †ž 7UDQVFRUULGRRSUD]RGHFODVVLILFDomRRXFRQVXPDGRR
SRU SHVVRD ItVLFD RX MXUtGLFD TXH WHQKD TXDOTXHU HYHQWR TXH GHILQD R VHX WHUPR ILQDO D LQIRUPDomR
YtQFXORFRPR3RGHU3~EOLFR WRUQDVHGHDFHVVRS~EOLFR
 'LU&RQVWLWXFLRQDO &kPDUD/HJLVODWLYDGR'LVWULWR)HGHUDO


†ž 3DUD D FODVVLILFDomR GD LQIRUPDomRHPGHWHUPLQDGR D GDVDXWRULGDGHVUHIHULGDVQRVLQFLVRV,H,,


JUDXGHVLJLORGHYHVHUREVHUYDGRRLQWHUHVVHS~EOLFR
GDLQIRUPDomRHXWLOL]DGRRFULWpULRPHQRVUHVWULWLYR E GDVDXWRULGDGHVTXHH[HUoDPIXQo}HVGHVXEVHFUH
SRVVtYHOFRQVLGHUDGRV WiULRRXGHKLHUDUTXLDHTXLYDOHQWH

, D JUDYLGDGH GR ULVFR RX GR GDQR j VHJXUDQoDGD †~QLFR $ FRPSHWrQFLD SUHYLVWD QRV LQFLVRV , H ,,SRGHVHU
VRFLHGDGHGR(VWDGRHGR'LVWULWR)HGHUDO GHOHJDGDSHODDXWRULGDGHUHVSRQViYHODDJHQWHS~EOLFR
YHGDGDDVXEGHOHJDomR
,, RSUD]RPi[LPRGHUHVWULomRGHDFHVVRRXRHYHQWRTXH
GHILQDVHXWHUPRILQDO $UW $ FODVVLILFDomR GH LQIRUPDomR HP TXDOTXHU JUDXGH
VLJLORGHYHVHUIRUPDOL]DGDHPGHFLVmRTXHFRQWHQKD
6HomR,,, QRPtQLPRRVVHJXLQWHVHOHPHQWRV
'D3URWHomRHGR&RQWUROHGH,QIRUPDo}HV6LJLORVDV
, DVVXQWRVREUHRTXDOYHUVDDLQIRUPDomR
$UW eGHYHUGR'LVWULWR)HGHUDOFRQWURODURDFHVVRDLQIRU
PDo}HVVLJLORVDVSURGX]LGDVSRUVHXVyUJmRVHSRUVXDV ,, IXQGDPHQWR GD FODVVLILFDomR REVHUYDGRV RVFULWpULRV
HQWLGDGHVDVVHJXUDQGRDVXDSURWHomR HVWDEHOHFLGRVQRDUW

†ž 2 DFHVVR H R WUDWDPHQWR GHLQIRUPDomRFODVVLILFDGD ,,, LQGLFDomRGRSUD]RGHVLJLORFRQWDGRHPDQRVPHVHV


FRPR VLJLORVD ILFDP UHVWULWRV D SHVVRDV TXH WHQKDP RX GLDV RX GR HYHQWR TXH GHILQD R VHX WHUPR ILQDO
QHFHVVLGDGHGHFRQKHFrODHTXHVHMDPGHYLGDPHQWH FRQIRUPHOLPLWHVSUHYLVWRVQRDUW
FUHGHQFLDGDVQDIRUPDGRUHJXODPHQWRVHPSUHMXt]R
GDVDWULEXLo}HVGRVDJHQWHVS~EOLFRVDXWRUL]DGRVSRU ,9 LGHQWLILFDomRGDDXWRULGDGHTXHDFODVVLILFRX
OHL
†~QLFR $ GHFLVmR UHIHULGD QR FDSXW GHYH VHUPDQWLGDQR
†ž 2DFHVVRjLQIRUPDomRFODVVLILFDGDFRPRVLJLORVDFULDD PHVPRJUDXGHVLJLORGDLQIRUPDomRFODVVLILFDGD
REULJDomRSDUDDTXHOHTXHDREWHYHGHUHVJXDUGDUR
VLJLOR $UW 9(7$'2 

†ž &DEH DR UHJXODPHQWR GLVSRU VREUHSURFHGLPHQWRVH $UW $DXWRULGDGHPi[LPDGHFDGDyUJmRRXHQWLGDGHGHYH


PHGLGDV D VHUHP DGRWDGRV SDUD R WUDWDPHQWR GH SXEOLFDU DQXDOPHQWH HP VHX VtWLR RILFLDO QD 5HGH
LQIRUPDomR VLJLORVD GH PRGR D SURWHJrOD FRQWUD 0XQGLDO GH &RPSXWDGRUHV RV VHJXLQWHV GDGRV H
SHUGD DOWHUDomR LQGHYLGD DFHVVR WUDQVPLVVmR H LQIRUPDo}HV DGPLQLVWUDWLYDV QRV WHUPRV GR UHJXOD
GLYXOJDomRQmRDXWRUL]DGRV PHQWR

$UW $VDXWRULGDGHVS~EOLFDVGHYHPDGRWDUDVSURYLGrQFLDV , UROGDVLQIRUPDo}HVTXHWHQKDPVLGRGHVFODVVLILFDGDV


QHFHVViULDV SDUD TXH R SHVVRDO D HODV VXERUGLQDGR QRV~OWLPRVGR]HPHVHV
KLHUDUTXLFDPHQWH FRQKHoD DV QRUPDV H REVHUYH DV
PHGLGDV H RV SURFHGLPHQWRV GH VHJXUDQoD SDUD R ,, UROGHGRFXPHQWRVFODVVLILFDGRVHPFDGDJUDXGHVLJLOR
WUDWDPHQWRGHLQIRUPDo}HVVLJLORVDV FRPLGHQWLILFDomRSDUDUHIHUrQFLDIXWXUD

†~QLFR $SHVVRDItVLFDRXMXUtGLFDTXHHPUD]mRGHTXDOTXHU ,,, UHODWyULRHVWDWtVWLFRFRQWHQGRDTXDQWLGDGHGHSHGLGRV


YtQFXORFRPR3RGHU3~EOLFRH[HFXWDUDWLYLGDGHVGH GHLQIRUPDomRUHFHELGRVDWHQGLGRVHLQGHIHULGRVEHP
WUDWDPHQWR GH LQIRUPDo}HV VLJLORVDV GHYH DGRWDU DV FRPRLQIRUPDo}HVJHQpULFDVVREUHRVVROLFLWDQWHV
SURYLGrQFLDV QHFHVViULDV SDUD TXH VHXV HPSUHJDGRV
SUHSRVWRVRXUHSUHVHQWDQWHVREVHUYHPDVPHGLGDVHRV †ž 2VyUJmRVHDVHQWLGDGHVGHYHPPDQWHUH[HPSODUGD
SURFHGLPHQWRVGHVHJXUDQoDGDVLQIRUPDo}HVUHVXOWDQ SXEOLFDomRSUHYLVWDQRFDSXWSDUDFRQVXOWDS~EOLFDHP
WHVGDDSOLFDomRGHVWD/HL VXDVVHGHV

6HomR,9 †ž 2VyUJmRVHDVHQWLGDGHVGHYHPPDQWHUH[WUDWRFRPD


'RV3URFHGLPHQWRVGH&ODVVLILFDomR5HFODVVLILFDomR OLVWD GH LQIRUPDo}HV FODVVLILFDGDV DFRPSDQKDGDV GD
H'HVFODVVLILFDomR GDWDGRJUDXGHVLJLORHGRVIXQGDPHQWRVGDFODVVLILFD
omR
$UW $ FODVVLILFDomR GR VLJLOR GH LQIRUPDo}HV QR3RGHU
([HFXWLYRpGHFRPSHWrQFLD 6HomR9
'DV,QIRUPDo}HV3HVVRDLV
, QRJUDXXOWUDVVHFUHWR
$UW 2WUDWDPHQWRGDVLQIRUPDo}HVSHVVRDLVGHYHVHUIHLWR
D GR*RYHUQDGRU GHIRUPDWUDQVSDUHQWHHFRPUHVSHLWRjLQWLPLGDGHj
E GR9LFH*RYHUQDGRU YLGDSULYDGDjKRQUDHjLPDJHPGDVSHVVRDVEHP
F GH6HFUHWiULRGH(VWDGRRXDXWRULGDGHHTXLYDOHQWH FRPRjVOLEHUGDGHVHjVJDUDQWLDVLQGLYLGXDLV

,, QRJUDXGHVHFUHWR †ž ­V LQIRUPDo}HV SHVVRDLV GH TXH WUDWDHVWHDUWLJR


DSOLFDVHRVHJXLQWH
D GDVDXWRULGDGHVUHIHULGDVQRLQFLVR,
E GRV WLWXODUHV GH DXWDUTXLD IXQGDomR HPSUHVD , VHXDFHVVRpUHVWULWRLQGHSHQGHQWHPHQWHGHFODVVLILFD
S~EOLFDRXVRFLHGDGHGHHFRQRPLDPLVWD omRGHVLJLORHSHORSUD]RGHFHPDQRVDFRQWDUGDVXD
GDWD GH SURGXomR D DJHQWHV S~EOLFRV OHJDOPHQWH
,,, QRJUDXGHUHVHUYDGR DXWRUL]DGRVHjSHVVRDDTXHHODVVHUHILUDP
&kPDUD/HJLVODWLYDGR
  'LVWULWR)HGHUDO 'LU&RQVWLWXFLRQDO 

,, SRGHVHUDXWRUL]DGDDVXDGLYXOJDomRRXRDFHVVRSRU 9, RFXOWDUGDUHYLVmRGHDXWRULGDGHVXSHULRUFRPSHWHQWH


WHUFHLURV HP SUD]R LQIHULRU DR GR LQFLVR , PHGLDQWH LQIRUPDomRVLJLORVDSDUDEHQHILFLDUDVLRXDRXWUHP
SUHYLVmROHJDORXFRQVHQWLPHQWRH[SUHVVRGDSHVVRDD RXHPSUHMXt]RGHWHUFHLURV
TXHHODVVHUHILUDP
9,, GHVWUXLURXVXEWUDLUSRUTXDOTXHUPHLRGRFXPHQWRV
†ž $TXHOHTXHREWLYHUDFHVVRjVLQIRUPDo}HVGHTXHWUDWD FRQFHUQHQWHVDSRVVtYHLVYLRODo}HVGHGLUHLWRVKXPDQRV
HVWHDUWLJRUHVSRQGHUiSRUVHXXVRLQGHYLGR SRUSDUWHGHDJHQWHVGR(VWDGR

†ž 2 FRQVHQWLPHQWR UHIHULGR QR † ž ,, QmRpH[LJLGR †~QLFR 9(7$'2 


TXDQGRDVLQIRUPDo}HVIRUHPQHFHVViULDV
$UW $ SHVVRD ItVLFD RX D HQWLGDGH SULYDGD TXHGHWLYHU
, jSUHYHQomRHDRGLDJQyVWLFRPpGLFRTXDQGRDSHVVRD LQIRUPDo}HVHPYLUWXGHGHYtQFXORGHTXDOTXHUQDWXUH
HVWLYHUItVLFDRXOHJDOPHQWHLQFDSD]HSDUDXWLOL]DomR ]DFRPR3RGHU3~EOLFRHGHL[DUGHREVHUYDURGLVSRVWR
GHWUDWDPHQWRPpGLFR QHVWD/HLHVWDUiVXMHLWDjVVHJXLQWHVVDQo}HV

,, jUHDOL]DomRGHHVWDWtVWLFDVHSHVTXLVDVFLHQWtILFDVGH , DGYHUWrQFLD


HYLGHQWHLQWHUHVVHS~EOLFRRXJHUDOSUHYLVWRVHPOHL
VHQGR YHGDGD D LGHQWLILFDomR GD SHVVRD D TXH DV ,, PXOWD
LQIRUPDo}HVVHUHIHUHP
,,, UHVFLVmRGRYtQFXORFRPR3RGHU3~EOLFR
,,, DRFXPSULPHQWRGHRUGHPMXGLFLDO
,9 VXVSHQVmR WHPSRUiULD GR GLUHLWR GH SDUWLFLSDUGH
OLFLWDomRHLPSHGLPHQWRGHFRQWUDWDUFRPDDGPLQLVWUD
,9 jGHIHVDGHGLUHLWRVKXPDQRV
omRS~EOLFDSRUSUD]RQmRVXSHULRUDGRLVDQRV
9 jSURWHomRGRLQWHUHVVHS~EOLFRHJHUDOSUHSRQGHUDQWH
9 GHFODUDomR GH LQLGRQHLGDGH SDUD OLFLWDU RXFRQWUDWDU
FRPDDGPLQLVWUDomRS~EOLFDDWpTXHVHMDSURPRYLGD
†ž $ UHVWULomR GH DFHVVR j LQIRUPDomR UHODWLYDjYLGD DUHDELOLWDomRSHUDQWHDSUySULDDXWRULGDGHTXHDSOLFRX
SULYDGDjKRQUDHjLPDJHPGHSHVVRDQmRSRGHVHU DSHQDOLGDGH
LQYRFDGD FRP R LQWXLWR GH SUHMXGLFDU SURFHVVR GH
DSXUDomR GH LUUHJXODULGDGHV HP TXH R WLWXODU GDV †ž $VVDQo}HVSUHYLVWDVQRVLQFLVRV,,,,H,9SRGHPVHU
LQIRUPDo}HV HVWLYHU HQYROYLGR EHP FRPR HP Do}HV DSOLFDGDVMXQWDPHQWHFRPDGRLQFLVR,,DVVHJXUDGRR
YROWDGDV SDUD D UHFXSHUDomR GH IDWRV KLVWyULFRV GH GLUHLWRGHGHIHVDGRLQWHUHVVDGRQRUHVSHFWLYRSURFHV
PDLRUUHOHYkQFLD VRQRSUD]RGHGH]GLDV

†ž &DEHDRUHJXODPHQWRGLVSRUVREUHRVSURFHGLPHQWRV †ž $ UHDELOLWDomR UHIHULGD QR LQFLVR 9VHUiDXWRUL]DGD


SDUDRWUDWDPHQWRGHLQIRUPDomRSHVVRDO VRPHQWHTXDQGRRLQWHUHVVDGRHIHWLYDURUHVVDUFLPHQWR
DRyUJmRRXjHQWLGDGHGRVSUHMXt]RVUHVXOWDQWHVHDSyV
$UW e SURLELGD D XWLOL]DomR GH DSDUHOKRV TXHSHUPLWDP GHFRUULGR R SUD]R GD VDQomR DSOLFDGD FRP EDVH QR
UHDOL]DUHVFXWDVWHOHI{QLFDVVDOYRHPFDVRVDXWRUL]DGRV LQFLVR,9
SHODMXVWLoD
†ž $DSOLFDomRGDVDQomRSUHYLVWDQRLQFLVR9pGHFRPSH
&DStWXOR9 WrQFLDH[FOXVLYDGDDXWRULGDGHPi[LPDGRyUJmRRXGD
'$65(63216$%,/,'$'(6 HQWLGDGHS~EOLFDIDFXOWDGDDGHIHVDGRLQWHUHVVDGRQR
UHVSHFWLYRSURFHVVRQRSUD]RGHGH]GLDVGDDEHUWXUD
$UW &RQVWLWXHPFRQGXWDVLOtFLWDVTXHHQVHMDPUHVSRQVDELOL GHYLVWD
GDGHGRDJHQWHS~EOLFRRXPLOLWDU
$UW 9(7$'2 
, UHFXVDUVHDIRUQHFHULQIRUPDomRUHTXHULGDQRVWHUPRV
GHVWD/HLUHWDUGDUGHOLEHUDGDPHQWHRVHXIRUQHFLPHQ &DStWXOR9,
WRRXIRUQHFrODLQWHQFLRQDOPHQWHGHIRUPDLQFRUUHWD '$),6&$/,=$d®2/(*,6/$7,9$
LQFRPSOHWDRXLPSUHFLVD
$UW 2V 5HTXHULPHQWRV GH ,QIRUPDomR DSURYDGRVSHOR
,, XWLOL]DULQGHYLGDPHQWHEHPFRPRVXEWUDLUGHVWUXLU 3RGHU/HJLVODWLYRGHYHPVHUUHVSRQGLGRVSHODVDXWRUL
LQXWLOL]DU GHVILJXUDU DOWHUDU RX RFXOWDU WRWDO RX GDGHV GLVWULWDLV UHVSRQViYHLV QR SUD]R Pi[LPR GH
SDUFLDOPHQWH LQIRUPDomR TXH VH HQFRQWUH VRE VXD WULQWDGLDVVRESHQDGHFULPHGHUHVSRQVDELOLGDGHGD
JXDUGD RX D TXH WHQKD DFHVVR RX FRQKHFLPHQWR HP VHJXLQWHIRUPD
UD]mRGRH[HUFtFLRGDVDWULEXLo}HVGHFDUJRHPSUHJR
RXIXQomRS~EOLFD , DVSiJLQDVGRVGRFXPHQWRVHQFDPLQKDGRVGHYHPVHU
QXPHUDGDV
,,, DJLUFRPGRORRXPiIpQDDQiOLVHGDVVROLFLWDo}HVGH
DFHVVRjLQIRUPDomR ,, RVGRFXPHQWRVHQFDPLQKDGRVGHYHPHVWDUOHJtYHLV

,,, DV UHVSRVWDV GHYHP FRQWHU LQIRUPDo}HV SUHFLVDVH


,9 GLYXOJDU RX SHUPLWLU D GLYXOJDomR RX DFHVVDURX
TXDQGRQHFHVViULRVHUHPUHVSDOGDGDVFRPUHODWyULRV
SHUPLWLU DFHVVR LQGHYLGR j LQIRUPDomR VLJLORVD RX j
WDEHODV TXDGURV LQIRUPDWLYRV H GHPDLV GRFXPHQWRV
LQIRUPDomRSHVVRDO
DIHWRVDRVTXHVWLRQDPHQWRV
9 LPSRUVLJLORjLQIRUPDomRSDUDREWHUSURYHLWRSDUDVL
$UW $V,QGLFDo}HVDSURYDGDVSHOR3RGHU/HJLVODWLYRGHYHP
RXSDUDWHUFHLURRXSDUDILQVGHRFXOWDomRGHDWRLOHJDO
VHUUHVSRQGLGDVSHODVDXWRULGDGHVGLVWULWDLVUHVSRQViYH
FRPHWLGRSRUVLRXSRURXWUHP LVQRSUD]RPi[LPRGHWULQWDGLDV
 'LU&RQVWLWXFLRQDO &kPDUD/HJLVODWLYDGR'LVWULWR)HGHUDO


$UW $VDXGLWRULDVLQVWDXUDGDVSHOD6HFUHWDULDGH(VWDGRGH ,9 RULHQWDU DV UHVSHFWLYDV XQLGDGHV VXERUGLQDGDVDRV


7UDQVSDUrQFLDH&RQWUROHGR'LVWULWR)HGHUDOGHYHPVHU yUJmRVRXjVHQWLGDGHVQRTXHVHUHIHUHDRFXPSULPHQ
HQFDPLQKDGDVj&kPDUD/HJLVODWLYDGR'LVWULWR)HGH WRGRGLVSRVWRQHVWD/HLHHPVHXVUHJXODPHQWRV
UDOWULPHVWUDOPHQWHFRQWHQGRRVVHJXLQWHVGDGRV
†~QLFR 2VyUJmRVGR6LVWHPDGH$UTXLYRVGR'LVWULWR)HGHUDO
, QRPHGRVHUYLGRUGDHPSUHVDRXGRWHUFHLURDXGLWDGR 6,$5')GHDFRUGRFRPDVXDHVWUXWXUDRUJkQLFDHDV
,, H[WUDWRGRSURFHVVRFRQWHQGRRREMHWRGDDXGLWRULD VXDV FRPSHWrQFLDV GHYHP GLVSRQLELOL]DU WRGDV DV
,,, IDVHGDWUDPLWDomR LQIRUPDo}HVDUTXLYtVWLFDVHRVXSRUWHWpFQLFRQHFHVViUL
RVDRHIHWLYRFXPSULPHQWRGHVWD/HL
&DStWXOR9,,
'$6',6326,d¯(6),1$,6(75$16,7Ð5,$6 $UW )LFD D 6HFUHWDULD GH 7UDQVSDUrQFLD H &RQWUROHGR
'LVWULWR)HGHUDOUHVSRQViYHOQR3RGHU([HFXWLYR
$UW 2 WUDWDPHQWR GH LQIRUPDomR VLJLORVD UHVXOWDQWHGH
WUDWDGRV DFRUGRV RX DWRV LQWHUQDFLRQDLV DWHQGH jV , SHODSURPRomRGHIRPHQWRjFXOWXUDGDWUDQVSDUrQFLD
QRUPDVHjVUHFRPHQGDo}HVFRQVWDQWHVGHVVHVLQVWUX QDDGPLQLVWUDomRS~EOLFDHjFRQVFLHQWL]DomRGRGLUHLWR
PHQWRV IXQGDPHQWDOGHDFHVVRjLQIRUPDomR
$UW eLQVWLWXtGRQD&DVD0LOLWDUR1~FOHRGH6HJXUDQoDH ,, SHORWUHLQDPHQWRGHDJHQWHVS~EOLFRVQRTXHVHUHIHUH
&UHGHQFLDPHQWR16&TXHWHPSRUREMHWLYRV DRGHVHQYROYLPHQWRGHSUiWLFDVUHODFLRQDGDVjWUDQVSD
UrQFLDQDDGPLQLVWUDomRS~EOLFD
, SURPRYHU H SURSRU D UHJXODPHQWDomR GRFUHGHQFLD
PHQWR GH VHJXUDQoD GH SHVVRDV ItVLFDV HPSUHVDV
,,, SHOD SDGURQL]DomR GRV SURFHGLPHQWRV QHFHVViULRVj
yUJmRV H HQWLGDGHV SDUD WUDWDPHQWR GH LQIRUPDo}HV
DSOLFDomRGHVWD/HL
VLJLORVDV
,9 SHORPRQLWRUDPHQWRGDDSOLFDomRGHVWD/HLQR3RGHU
,, JDUDQWLUDVHJXUDQoDGHLQIRUPDo}HVVLJLORVDV
([HFXWLYRFRQFHQWUDQGRHFRQVROLGDQGRDSXEOLFDomR
†~QLFR &DEH DR UHJXODPHQWR GLVSRU VREUH DFRPSRVLomRD GHLQIRUPDo}HVHVWDWtVWLFDVUHODFLRQDGDVQRDUW
RUJDQL]DomRHRIXQFLRQDPHQWRGR16&
9 SHORHQFDPLQKDPHQWRj&kPDUD/HJLVODWLYDGR'LVWULWR
$UW $SOLFDVHQRTXHFRXEHUD/HLIHGHUDOQžGH )HGHUDOGHUHODWyULRDQXDOFRPLQIRUPDo}HVDWLQHQWHV
GH QRYHPEUR GH  HP UHODomR j LQIRUPDomR GH jLPSOHPHQWDomRGHVWD/HL
SHVVRD ItVLFD RX MXUtGLFD FRQVWDQWH GH UHJLVWUR RX
EDQFR GH GDGRV GH yUJmRV RX GH HQWLGDGHV RX GH $UW $ /HL Qž  GH  GH DEULO GH  TXHGLVS}H
FDUiWHUS~EOLFR VREUHDSURWHomRGRVGRFXPHQWRVGHDUTXLYRVS~EOLFRV
SDVVDDYLJRUDUFRPDVVHJXLQWHVDOWHUDo}HV
$UW 2VyUJmRVHDVHQWLGDGHVS~EOLFDVGHYHPSURFHGHUj $UWž
UHDYDOLDomRGDVLQIRUPDo}HVFODVVLILFDGDVFRPRXOWUDV †ž &RQVLGHUDVHJHVWmRGHGRFXPHQWRVFRPEDVHQRDUWžGD/HLIHGHUDOQžGHGH
VHFUHWDV H VHFUHWDV QR SUD]R Pi[LPR GH GRLV DQRV MDQHLURGHRFRQMXQWRGHSURFHGLPHQWRVHRSHUDo}HVWpFQLFDVUHIHUHQWHVjSURGXomR
jWUDPLWDomRDRXVRjDYDOLDomRHDRDUTXLYDPHQWRGRVGRFXPHQWRVHPIDVHFRUUHQWHH
FRQWDGRGRWHUPRLQLFLDOGHYLJrQFLDGHVWD/HL LQWHUPHGLiULD LQGHSHQGHQWHPHQWH GR VXSRUWH YLVDQGR D VXD HOLPLQDomR RX R VHX
UHFROKLPHQWRSDUDJXDUGDSHUPDQHQWH

†ž $ UHVWULomR GH DFHVVR D LQIRUPDo}HV HPUD]mRGD $UWž 2'LVWULWR)HGHUDOUHDOL]DUiDJHVWmRGHGRFXPHQWRVGHDUTXLYRGHVHXVyUJmRVHGHVXDV


HQWLGDGHVYLVDQGR
UHDYDOLDomRSUHYLVWDQRFDSXWGHYHREVHUYDURVSUD]RV
HDVFRQGLo}HVSUHYLVWRVQHVWD/HL 

,9 j JDUDQWLD GH DFHVVR DRV GRFXPHQWRV GH DUTXLYR H jV LQIRUPDo}HV QHOHVFRQWLGDV
UHVJXDUGDGRVRVDVSHFWRVGRVLJLOROHJDO
†ž (QTXDQWR QmR WUDQVFRUULGR R SUD]RGHUHDYDOLDomR
SUHYLVWRQRFDSXWGHYHVHUPDQWLGDDFODVVLILFDomRGD
LQIRUPDomRQRVWHUPRVGDOHJLVODomRSUHFHGHQWH $UW 23RGHU([HFXWLYRUHJXODPHQWDUiRGLVSRVWRQHVWD/HL
QRSUD]RGHFHQWRHYLQWHGLDVDFRQWDUGDGDWDGHVXD
†ž $VLQIRUPDo}HVFODVVLILFDGDVFRPRVHFUHWDVRXXOWUDVVH SXEOLFDomR
FUHWDVQmRUHDYDOLDGDVQRSUD]RSUHYLVWRQRFDSXWVmR
FRQVLGHUDGDVGHDFHVVRS~EOLFR $UW (VWD/HLHQWUDHPYLJRUFHQWRHYLQWHGLDVDSyVDGDWD
GHVXDSXEOLFDomR
$UW 1RSUD]RGHVHVVHQWDGLDVDFRQWDUGDYLJrQFLDGHVWD
/HLRGLULJHQWHPi[LPRGHFDGDyUJmRRXHQWLGDGHGR $UW 5HYRJDPVHDVGLVSRVLo}HVHPFRQWUiULRHPHVSHFLDO
3RGHU3~EOLFRGHYHGHVLJQDUDXWRULGDGHTXHOKHVHMD
GLUHWDPHQWH VXERUGLQDGD SDUD H[HUFHU DV VHJXLQWHV , D/HLQžGHGHGH]HPEURGH
DWULEXLo}HV ,, RDUWžGD/HLQžGHGHMDQHLURGH
,,, RDUWžGD/HLQžGHGHDEULOGH
, DVVHJXUDU R FXPSULPHQWR GDV QRUPDV UHODWLYDVDR ,9 RDUWžGD/HLQžGHGHMDQHLURGH
DFHVVR j LQIRUPDomR GH IRUPD HILFLHQWH H DGHTXDGD 9 RDUWž;,;GD/HLQžGHGHVHWHPEURGH
DRVREMHWLYRVGHVWD/HL 
9, R DUW ž † ž GD /HL Qž  GH  GH MDQHLURGH
,, PRQLWRUDU D LPSOHPHQWDomR GR GLVSRVWR QHVWD /HLH 
DSUHVHQWDU UHODWyULRV SHULyGLFRV VREUH R VHX FXPSUL 9,, RDUWž9,GD/HLQžGHGHRXWXEURGH
PHQWR 9,,, RDUWGD/HLQžGHGHMDQHLURGH
,; RDUWGD/HLQžGHGHMXOKRGH
,,, UHFRPHQGDUDVPHGLGDVLQGLVSHQViYHLVjLPSOHPHQWD ; R DUW ž SDUiJUDIR ~QLFR GD /HL Qž  GH GH
omRHDRDSHUIHLoRDPHQWRGDVQRUPDVHGRVSURFHGL IHYHUHLURGH
PHQWRVQHFHVViULRVDRFRUUHWRFXPSULPHQWRGRGLVSRV
WRQHVWD/HL
&kPDUD/HJLVODWLYDGR
 'LVWULWR)HGHUDO 'LU&RQVWLWXFLRQDO 

†ž 1DJUDGDomRGDRUGHPMXUtGLFDDOHLFRPSOHPHQWDUVH
VLWXDHQWUHD/HL2UJkQLFDHDVOHLVRUGLQiULDV
/HL&RPSOHPHQWDUGLVWULWDOQƒ
†ž $OHLRUGLQiULDWHUiVHXQRPHDEUHYLDGRSDUDOHL

3XEOLFDGDQR'2')GHGH6HWHPEURGH $UWž 9(7$'2


$OWHUDomR/HL&RPSOHPHQWDUQžGHGRGIGH &DStWXOR,,
'$(/$%25$d®2
5HJXODPHQWDRDUWGD/HL2UJkQLFDGLVSRQGRVREUH
DHODERUDomRUHGDomRDOWHUDomRHFRQVROLGDomRGDVOHLVGR 6HomR,
'LVWULWR)HGHUDO 'DV'LVSRVLo}HV3UHOLPLQDUHV

$UWž $HODERUDomRGDVOHLVREHGHFHUiDRSURFHVVROHJLVODWLYR
SUHYLVWRQD/HL2UJkQLFDQHVWD/HL&RPSOHPHQWDUHQR
&DStWXOR,
5HJLPHQWR,QWHUQRGD&kPDUD/HJLVODWLYDOHYDQGRVH
'$6',6326,d¯(635(/,0,1$5(6 HPFRQWD

$UWž $HODERUDomRUHGDomRDOWHUDomRHFRQVROLGDomR , DQHFHVVLGDGHVRFLDOHRLGHiULRGHMXVWLoD


GDVOHLVGR'LVWULWR)HGHUDOVXMHLWDVDRSURFHVVR
OHJLVODWLYRUHJHPVHSRUHVWD/HL&RPSOHPHQWDU ,, RV SULQFtSLRV MXUtGLFRV FRQVDJUDGRV SHORVGLYHUVRV
QRV WHUPRV GR TXH GLVS}H R DUW  SDUiJUDIR UDPRVGR'LUHLWR
~QLFRGD/HL2UJkQLFD
,,, D OHJLVODomR H[LVWHQWH REHGHFHQGRVH FRQIRUPHD
HVSpFLHGHOHL
$UWž 2SURFHVVROHJLVODWLYRpRFRQMXQWRGHDWRVSUHRU
GHQDGRVYLVDQGRjIRUPDomRGDVOHLVDWUDYpVGD D j &RQVWLWXLomR )HGHUDO H j /HL 2UJkQLFD H VXDV
FRODERUDomRHQWUHRV3RGHUHVGR'LVWULWR)HGHUDO HPHQGDV

$UWž 5HFHEHDGHQRPLQDomRGHSURFHGLPHQWROHJLVODWL E jVOHLVFRPSOHPHQWDUHVHRUGLQiULDVGD8QLmRTXH


YRRPRGRGHUHDOL]DURVDWRVGRSURFHVVROHJLVOD GLVSRQKDPVREUHQRUPDVJHUDLVSDUDVHUHPREHGH
WLYR FLGDVSHORV(VWDGRV'LVWULWR)HGHUDOH0XQLFtSLRV

†~QLFR 2SURFHGLPHQWROHJLVODWLYRTXHSRGHVHURUGLQi F jVOHLVFRPSOHPHQWDUHVGR'LVWULWR)HGHUDO


ULRVXPiULRRXHVSHFLDOVHUi GLVFLSOLQDGRSHOR
G jV OHLV RUGLQiULDV GR 'LVWULWR )HGHUDO TXH FRQWH
5HJLPHQWR,QWHUQRGD&kPDUD/HJLVODWLYD QKDPQRUPDVJHUDLV

$UWž 3DUD HIHLWRV GHVWD /HL &RPSOHPHQWDU OHLV pR ,9 RKLVWyULFRGDVOHLVRXGHVHXVGLVSRVLWLYRVTXHYHUVHP
JrQHURGHTXHVmRHVSpFLHV VREUHRDVVXQWRDERUGDGRQDQRYDOHL

, DHPHQGDj/HL2UJkQLFD 9 DWUDQVLomRGRUHJLPHMXUtGLFRGDOHLYHOKDSDUDRGDOHL


,, DOHLFRPSOHPHQWDU QRYD
,,, DOHLRUGLQiULD
,9 RGHFUHWROHJLVODWLYR $UWž 2 SURFHVVR GH HODERUDomR GDV OHLV FRPSUHHQGHDV
HWDSDVVHJXLQWHV
9 DUHVROXomR
, LQLFLDWLYD
†ž 1RkPELWROHJLVODWLYRGR'LVWULWR)HGHUDOFRQVL ,, HPHQGDV
GHUDVH ,,, GLVFXVVmR
,9 GHOLEHUDomR
, HPHQGD j /HL 2UJkQLFD D OHL TXHGHWHUPLQH 9 VDQomRRXYHWR
DOWHUDomRHPGLVSRVLWLYRGD/HL2UJkQLFD 9, SURPXOJDomR
9,, SXEOLFDomR
,, OHLFRPSOHPHQWDUDOHLTXHGLVFLSOLQHPDWpULDTXH 6HomR,,
'D,QLFLDWLYD
D/HL2UJkQLFDGHWHUPLQHFRPRVHXREMHWR
$UWž $LQLFLDWLYDpDSURSRVWDGHFULDomRGHGLUHLWRQRYRH
,,, OHL RUGLQiULD D OHL TXH GLVFLSOLQH DVPDWpULDV FRPHODVHLQLFLDRSURFHVVROHJLVODWLYR
OHJLVODWLYDV GD FRPSHWrQFLD GR 'LVWULWR )HGHUDO
TXHQmRHVWHMDPSUHYLVWDVQRVLQFLVRVDQWHULRUHV †~QLFR $ HPHQGD j /HL 2UJkQLFD VHUi LQLFLDGD QDIRUPDGH
SURSRVWDHDVGHPDLVOHLVGHTXHWUDWDRDUWžGHVWD
,9 GHFUHWR OHJLVODWLYR D OHL TXH FRP HVWHQRPH /HL&RPSOHPHQWDUVHUmRLQLFLDGDVQDIRUPDGHSURMHWR
GLVFLSOLQHFRPHIHLWRH[WHUQRPDWpULDGDFRPSH
WrQFLDSULYDWLYDGD&kPDUD/HJLVODWLYD $UWž $LQLFLDWLYDSRGHVHUFRPXPRXSULYDWLYD

†ž $LQLFLDWLYDFRPXPpDTXHSRGHVHUH[HUFLGD
9 UHVROXomR D OHL TXH FRP HVWH QRPHGLVFLSOLQH
FRP HIHLWR LQWHUQR PDWpULD GD FRPSHWrQFLD , SHOR*RYHUQDGRU
SULYDWLYDGD&kPDUD/HJLVODWLYD ,, SRUTXDOTXHUPHPEURRXÐUJmRGD&kPDUD/HJLVODWLYD
,,, SHORVFLGDGmRVQDIRUPDSUHYLVWDQD/HL2UJkQLFD
 'LU&RQVWLWXFLRQDO
 &kPDUD/HJLVODWLYDGR'LVWULWR)HGHUDO
†ž $LQLFLDWLYDSULYDWLYDpDTXHVHUHVHUYDDXP3RGHURX $UW $LQLFLDWLYDGHSURSRUHPHQGDFRPSHWHDRVPHPEURV
D ÐUJmR GRV 3RGHUHV 3~EOLFRV R GLUHLWR H[FOXVLYR GH RXyUJmRVGD&kPDUD/HJLVODWLYDQDIRUPDTXHGLVSX
LQLFLDURSURFHVVROHJLVODWLYR VHUVHX5HJLPHQWR,QWHUQR

†ž $&kPDUD/HJLVODWLYDSRGHUiVHUSURYRFDGDDPDQLIHV †~QLFR 2 *RYHUQDGRU SRGH VROLFLWDU j &kPDUD/HJLVODWLYDD


WDUVH VREUH PDWpULD GH VXD FRPSHWrQFLD SULYDWLYD DOWHUDomR GH SURSRVLomR GH VXD LQLFLDWLYD PHGLDQWH
PHGLDQWHVROLFLWDomR DSUHVHQWDomR GR WH[WR D VHU GHOLEHUDGR DQWHV GD
DSUHFLDomRSHODVFRPLVV}HV DFUHVFHQWDGRRSDUiJUDIR~QLFRDRDUW
, GR*RYHUQDGRU SHODOHLFRPSOHPHQWDUQžGH

,, GR7ULEXQDOGH&RQWDVGR'LVWULWR)HGHUDO $UW $ HPHQGD VHUi YLQFXODGD j SURSRVLomR SULQFLSDOH


REHGHFHUijVQRUPDVFRQWLGDVQHVWD/HL&RPSOHPHQ
,,, GR6XSUHPR7ULEXQDO)HGHUDOGR6XSHULRU7ULEXQDOGH WDUEHPFRPRDRTXHGLVSXVHUR5HJLPHQWR,QWHUQRGD
-XVWLoDRXGR7ULEXQDOGH-XVWLoDGR'LVWULWR)HGHUDOH &kPDUD/HJLVODWLYD
7HUULWyULRV
†~QLFR 6HUiUHSURGX]LGRLQWHJUDOPHQWHGLVSRVLWLYRREMHWRGH
$UW 2EVHUYDGRRGLVSRVWRQRDUWGD/HL2UJkQLFDRV HPHQGD
SURMHWRVGHLQLFLDWLYDSULYDWLYDSRGHPVHUHPHQGDGRV
SHOD&kPDUD/HJLVODWLYD , PRGLILFDWLYD
,, VXEVWLWXWLYD
$UW eYHGDGRRXVRGHSURMHWRDXWRUL]DWLYRSDUDVXSULUD ,,, DJOXWLQDWLYD
LQLFLDWLYD SULYDWLYD GH RXWUR 3RGHU RX GH yUJmR GRV ,9 GHUHGDomR
3RGHUHV3~EOLFRVGR'LVWULWR)HGHUDO
6HomR,9
†ž e DLQGD YHGDGR R XVR GH SURMHWRDXWRUL]DWLYRSDUD 'D'LVFXVVmR
PDWpULDV TXH GHSHQGDP GH GHFLVmR GDV DXWRULGDGHV
DGPLQLVWUDWLYDVGR'LVWULWR)HGHUDORXGHVXDVHPSUH $UW 'LVFXVVmRpDHWDSDGRSURFHVVROHJLVODWLYRGHVWLQDGD
VDVS~EOLFDVHVRFLHGDGHVGHHFRQRPLDPLVWD DRGHEDWHGDVPDWpULDVREMHWRGHHODERUDomRGHOHL

†ž 1mR VHQGR D LQLFLDWLYD SULYDWLYD H[HUFLGDQRSUD]R $UW 2LQtFLRGDGLVFXVVmRGHSHQGHGHTXRUXPHVWDEHOHFLGR


IL[DGRHPOHLD&kPDUD/HJLVODWLYDVROLFLWDUiLQIRUPD QR5HJLPHQWR,QWHUQRGD&kPDUD/HJLVODWLYD
o}HVjDXWRULGDGHFRPSHWHQWHLQFOXVLYHDR*RYHUQD
GRUQRVWHUPRVGRTXHGLVS}HRDUW;;;,,GD/HL †~QLFR 5HFHEH D GHQRPLQDomR GH TXRUXP DH[LJrQFLDGH
2UJkQLFD Q~PHUR PtQLPR GH 'HSXWDGRV 'LVWULWDLV SUHVHQWHV j
VHVVmRGD&kPDUD/HJLVODWLYDSDUDGLVFXVVmRRXGHOLEH
$UW $OpPGRVFDVRVSUHYLVWRVQD/HL2UJkQLFDR5HJLPHQWR UDomRGDVPDWpULDVREMHWRGHOHL
,QWHUQR GD &kPDUD /HJLVODWLYD SRGHUi H[LJLU GRV
'HSXWDGRV'LVWULWDLVQ~PHURPtQLPRGHVXEVFULWRUHV
$UW $IRUPDGHGLVFXVVmRVHUiGLVFLSOLQDGDQR5HJLPHQWR
SDUD LQLFLDU R SURFHVVR OHJLVODWLYR GH GHWHUPLQDGDV
,QWHUQRGD&kPDUD/HJLVODWLYD
PDWpULDV
6HomR9
†~QLFR 5HFHEHDGHQRPLQDomRGHLQLFLDWLYDTXDOLILFDGDDTXH
'DV'HOLEHUDo}HV
H[LJH Q~PHUR PtQLPR GH VXEVFULWRUHV SDUD LQLFLDU R
SURFHVVROHJLVODWLYR
$UW 'HOLEHUDomRpDHWDSDGRSURFHVVROHJLVODWLYRSHODTXDO
$UW 6DOYRQRFDVRSUHYLVWRQRDUW†žGD/HL2UJkQL D &kPDUD /HJLVODWLYD GHFLGH SULYDWLYDPHQWH VREUH D
FD DV SURSRVWDV GH HPHQGD j /HL 2UJkQLFD RX RV FRQYHQLrQFLDRSRUWXQLGDGHHFRQWH~GRGDVSURSRVWDV
SURMHWRV QmR VHUmR UHDSUHVHQWDGRV QD PHVPD VHVVmR GHHPHQGDj/HL2UJkQLFDRXSURMHWRVOHYDGRVjVXD
OHJLVODWLYDHPTXHKDMDPVLGRUHMHLWDGRVRXWLGRVSRU FRQVLGHUDomR
SUHMXGLFDGRV
†~QLFR $VGHOLEHUDo}HVREHGHFHUmRj/HL2UJkQLFDHDR5HJL
†~QLFR 1DVPDWpULDVGHLQLFLDWLYDSULYDWLYDGR*RYHUQDGRUD PHQWR,QWHUQRGD&kPDUD/HJLVODWLYD
UHDSUHVHQWDomR GH SURMHWR UHMHLWDGR GHSHQGH GH
DFHLWDomRSUpYLDGDPDLRULDDEVROXWDGRVPHPEURVGD $UW 3DUDHIHLWRVGHGHOLEHUDomRFRQVLGHUDVH
&kPDUD/HJLVODWLYD  DFUHVFHQWDGR R SDUiJUDIR ~QLFR DR DUW  SHOD OHL
FRPSOHPHQWDUQžGH , PDLRULDTXDOLILFDGDDPDQLIHVWDomRGHGRLVWHUoRVGRV
PHPEURVTXHFRPS}HPD&kPDUD/HJLVODWLYD
6HomR,,,
'DV(PHQGDV ,, PDLRULD DEVROXWDDPDQLIHVWDomRRXSUHVHQoDGHQR
PtQLPRPHWDGHPDLVXPGRVPHPEURVTXHFRPS}HP
$UW (PHQGDpDSURSRVLomRTXHWHPSRUILQDOLGDGHDOWHUDU D&kPDUD/HJLVODWLYD
SURSRVWDGHHPHQGDj/HL2UJkQLFDRXSURMHWR
,,, PDLRULDVLPSOHVDPDQLIHVWDomRSRUPDLRULDGHYRWRV
†~QLFR $HPHQGDSRGHVHU SUHVHQWH D PDLRULD DEVROXWD GRV PHPEURV TXH FRP
S}HPD&kPDUD/HJLVODWLYD
, VXSUHVVLYD
,, DJOXWLQDWLYD
†ž 3DUDVHUDSURYDGRGHSHQGHGDPDQLIHVWDomRIDYRUiYHO
,,, VXEVWLWXWLYD
,9 PRGLILFDWLYD
, GDPDLRULDTXDOLILFDGD
9 DGLWLYD
9, GHUHGDomR
D SURSRVWDGHHPHQGDj/HL2UJkQLFD
 &kPDUD/HJLVODWLYDGR
 'LVWULWR)HGHUDO 'LU&RQVWLWXFLRQDO 

E SURMHWR GH OHL TXH HQYROYD PDWpULD WULEXWiULD RX 6HomR9


SUHYLGHQFLiULDGR'LVWULWR)HGHUDOVREUH 'D6DQomRHGR9HWR

 LVHQomR 6XEVHomR,
 DQLVWLD 'DV'LVSRVLo}HV&RPXQV
 UHPLVVmR
 EHQHItFLRVHLQFHQWLYRVILVFDLV $UW 6DQomRHYHWRVmRDWRVSULYDWLYRVGR*RYHUQDGRU

F SURMHWRGHGHFUHWROHJLVODWLYRTXHDXWRUL]HR3RGHU †ž $SHQDV RV SURMHWRV GH OHL FRPSOHPHQWDU HGHOHL


-XGLFLiULRDSURFHVVDUHMXOJDUQRVFULPHVFRPXQV RUGLQiULDHVWmRVXMHLWRVDYHWRRXVDQomR

 R*RYHUQDGRUHR9LFH*RYHUQDGRU †ž $VDQomRHRYHWRXPDYH]DSUHVHQWDGRVVmRLUUHWUDWi


 RV6HFUHWiULRVGH*RYHUQR YHLV

G SURMHWRGHGHFUHWROHJLVODWLYRTXHFRQGHQHR*RYHU †ž 2GLVSRVWRQHVWDHQDVVXEVHo}HVVHJXLQWHVDSOLFDVH


QDGRURXR3URFXUDGRU*HUDOGR'LVWULWR)HGHUDOSRU LQFOXVLYH D SURMHWRV GH OHL GH LQLFLDWLYD SULYDWLYD GR
FULPHGHUHVSRQVDELOLGDGH 7ULEXQDOGH&RQWDVGR'LVWULWR)HGHUDOHGR3URFXUDGRU
*HUDO GR 0LQLVWpULR 3~EOLFR MXQWR DR 7ULEXQDO GH
H SURMHWR GH GHFUHWR OHJLVODWLYR TXH VXVSHQGD DV &RQWDVGR'LVWULWR)HGHUDO
LPXQLGDGHVSDUODPHQWDUHVGRV'HSXWDGRV'LVWULWD
LV $UW e GH TXLQ]H GLDV ~WHLV FRQWDGRV GR UHFHELPHQWRR
SUD]RSDUDTXHRSURMHWRVHMDVDQFLRQDGRRXYHWDGR
,, GDPDLRULDDEVROXWD
6XEVHomR,,
D SURMHWRGHOHLFRPSOHPHQWDU 'R9HWR

E SURMHWRGHOHLTXHFULHRXH[WLQJD5HJLmR$GPLQLV $UW 9HWRpRDWRSHORTXDOR*RYHUQDGRUQHJDVDQomRQR


WUDWLYD WRGR RX HP SDUWH D SURMHWR DSURYDGR SHOD &kPDUD
/HJLVODWLYD
F SURMHWRGHGHFUHWROHJLVODWLYRTXHDXWRUL]HDLQVWDX
UDomRGHSURFHVVRFULPLQDOFRQWUD'HSXWDGR'LVWUL †ž 2YHWRVHUiVHPSUHH[SUHVVRHPRWLYDGR
WDO
†ž 2*RYHUQDGRUH[SOLFLWDUiDVUD]}HVGHRUGHPMXUtGLFD
G SURMHWR GH GHFUHWR OHJLVODWLYR TXH GHWHUPLQH D RX FRQWUiULDV DR LQWHUHVVH S~EOLFR TXH PRWLYDUDP R
SHUGDGRPDQGDWRGH'HSXWDGR'LVWULWDOQRVFDVRV YHWR
SUHYLVWRVQRDUW,,,H9,GD/HL2UJkQLFD
$UW 2 YHWR SDUFLDO DEUDQJHUi VRPHQWH WH[WR LQWHJUDOGH
H SURMHWRGHUHVROXomRTXHDXWRUL]Hj&kPDUD/HJLVOD DUWLJRSDUiJUDIRLQFLVRDOtQHDRXQ~PHUR
WLYDUHXQLUVHIRUDGHVXDVHGHQRVWHUPRVGRDUW
$UW eGHTXDUHQWDHRLWRKRUDVRSUD]RSDUDTXHRYHWRH
SDUiJUDIR~QLFRGD/HL2UJkQLFD
VXDVUD]}HVVHMDPHQFDPLQKDGRVj&kPDUD/HJLVODWLYD
I UHMHLomRGRYHWR
$UW 2SUD]RGRDUWLJRDQWHULRUFRPHoDDIOXLUGDGDWDHGD
KRUDGDDVVLQDWXUDGRYHWRHVXDVUD]}HV
,,, GDPDLRULDVLPSOHVDPDWpULDQmRFRPSUHHQGLGDQRV
LQFLVRVDQWHULRUHVTXHVHMDREMHWRGH
†ž 1DIDOWDGHLQGLFDomRGDKRUDSUHVXPHVHTXHRYHWR
IRLRSRVWRjVGH]RLWRKRUDV
D OHLRUGLQiULD
E GHFUHWROHJLVODWLYR
†ž 1DIDOWDGHLQGLFDomRGHGDWDSUHVXPHVHTXHRYHWR
F UHVROXomR IRLRSRVWRQR~OWLPRGLD~WLOGRSUD]RSUHYLVWRQRDUW
GHVWD/HL&RPSOHPHQWDURXQR~OWLPRGLD~WLOGR
†ž 2 5HJLPHQWR ,QWHUQR GD &kPDUD/HJLVODWLYDSRGHUi PrVVHIDOWDUDSHQDVRGLD
SUHYHURXWURVFDVRVGHSURMHWRVGHGHFUHWROHJLVODWLYR
RXGHUHVROXomRTXHGHSHQGDPGDPDLRULDTXDOLILFDGD $UW 2YHWRRSRVWRIRUDGRSUD]RRXQmRFRPXQLFDGRGHQWUR
RXGDPDLRULDDEVROXWDSDUDVHUHPDSURYDGRV GRSUD]RGHVWD/HL&RPSOHPHQWDUpWLGRSRULQH[LVWHQ
WH
†ž 3DUDGHOLEHUDUVREUHPDWpULDTXHH[LMDPDLRULDTXDOLIL
FDGDH[LJHVHDSUHVHQoDGHSHORPHQRVGRLVWHUoRV $UW $&kPDUD/HJLVODWLYDPDQLIHVWDUVHiVREUHRYHWRQRV
GRVPHPEURVGD&kPDUD/HJLVODWLYD DFUHVFHQWDGRR†žDRDUW SUD]RV H IRUPD SUHYLVWRV QD /HL 2UJkQLFD H QR VHX
SHODOHLFRPSOHPHQWDUQžGHGRGIGH
5HJLPHQWR,QWHUQR
$UW 1HQKXPSURMHWRVHUiDSURYDGRVHRQ~PHURGHDEVWHQ
o}HV RX YRWRV HP EUDQFR IRU LJXDO RX VXSHULRU DR $UW $&kPDUD/HJLVODWLYDSRGHUiUHMHLWDUWRWDORXSDUFLDO
Q~PHURGHYRWRVIDYRUiYHLV PHQWHRYHWRWRWDO

†ž 2SURMHWRTXHVHHQFRQWUDUQDVLWXDomRGHVFULWDQHVWH $UW $PDQXWHQomRGRYHWRSHOD&kPDUD/HJLVODWLYDDLQGD


DUWLJR VHUi UHLQFOXtGR QD RUGHP GR GLD SDUD QRYD TXHRYHWRVHMDSDUFLDOHTXLYDOHjUHMHLomRGHSURMHWR
GHOLEHUDomR SDUDHIHLWRVGRDUWGHVWD/HL&RPSOHPHQWDU

†ž 3HUVLVWLQGRDVLWXDomRGHVFULWDQHVWHDUWLJRVHUiWLGR $UW 2WH[WRYHWDGRVHUiVXSULPLGRGDOHLILFDQGRDXQLGD


SRU SUHMXGLFDGR SDUD HIHLWRV GR DUW  GHVWD /HL GHGHDUWLFXODomRFRUUHVSRQGHQWHFRPVXDQXPHUDomR
&RPSOHPHQWDU RULJLQDOVHJXLGDGDH[SUHVVmRYHWDGRHQWUHSDUrQWH
VHV
 'LU&RQVWLWXFLRQDO &kPDUD/HJLVODWLYDGR'LVWULWR)HGHUDO


†~QLFR eYHGDGDDUHXWLOL]DomRGDQXPHUDomRGHGLVSRVLWLYR ,9 GR 9LFH3UHVLGHQWH GD &kPDUD /HJLVODWLYD VH QmRR
YHWDGR VDOYR QR FDVR GR DUW  SDUiJUDIR ~QLFR IL]HURVHX3UHVLGHQWHQRVFDVRVLQGLFDGRVQRLQFLVR
GHVWD/HL&RPSOHPHQWDU DQWHULRU

6XEVHomR,,, †~QLFR 'HSRLV GH R SURMHWR GH OHL FRPSOHPHQWDU RXGHOHL


'D6DQomR RUGLQiULDWHUVLGRDSURYDGRSHOD&kPDUD/HJLVODWLYDD
HPHQWDQmRVHVXMHLWDDYHWRSDUFLDOHTXDOTXHUTXH
$UW 6DQomR p R DWR SHOR TXDO R *RYHUQDGRUH[WHULRUL]D VHMDDDXWRULGDGHSURPXOJDGRUDQmRVHDOWHUDFRPD
H[SUHVVDRXWDFLWDPHQWHVXDDTXLHVFrQFLDDRSURMHWR SURPXOJDomR VHQGR YHGDGR XWLOL]DU D H[SUHVVmR
GHOHLFRPSOHPHQWDURXGHOHLRUGLQiULDDSURYDGRSHOD SURPXOJDomRQHJDGD
&kPDUD/HJLVODWLYD
6HomR9,,
†ž 6DQomRH[SUHVVDpDTXHRFRUUHTXDQGRR*RYHUQDGRU 'D3XEOLFDomR
PDQLIHVWDSRUHVFULWRVXDDTXLHVFrQFLD
$UW $SXEOLFDomRFRQGLomRGHYLJrQFLDHHILFiFLDGDOHLp
†ž 6DQomRWiFLWDpDTXHRFRUUHSRUGHFXUVRGHSUD]RHP DHWDSDGRSURFHVVROHJLVODWLYRSHODTXDOVHGiFLrQFLD
YLUWXGHGHVLOrQFLRGR*RYHUQDGRUQRSUD]RGRDUW GDSURPXOJDomRGDVOHLVDRVVHXVGHVWLQDWiULRVWRUQDQ
GHVWD/HL&RPSOHPHQWDU GRREULJDWyULDVXDH[HFXomR

$UW $VDQomRQmRVXSUHYtFLRVGHLQLFLDWLYDQHPGHRXWUDV †ž 7RGDSXEOLFDomRVHUiHVFULWD


HWDSDVDTXHRVSURMHWRVGHOHLFRPSOHPHQWDURXGHOHL
RUGLQiULDHVWmRVXMHLWRV †ž $V OHLV VHUmR SXEOLFDGDV LPHGLDWDPHQWHDSyVVXD
SURPXOJDomR
$UW $ VDQomR VHUi DSRVWD LQFOXVLYH DRV SURMHWRV GHOHL
FRPSOHPHQWDURXGHOHLRUGLQiULDTXHUHFHEHUHPYHWR †ž $OHLVySURGX]HIHLWRGHSRLVGHSXEOLFDGDHDSDUWLUGD
SDUFLDO GDWDLQGLFDGDQDFOiXVXODGHYLJrQFLD
6HomR9,
'D3URPXOJDomR $UW 'HSHQGHPGHSXEOLFDomRQR'LiULR2ILFLDOGR'LVWULWR
)HGHUDO
$UW 3URPXOJDomR p D HWDSD GR SURFHVVR OHJLVODWLYRTXH
DWHVWDDH[LVWrQFLDGDOHLUHFRQKHFHRVIDWRVHDWRVTXH , DVHPHQGDVj/HL2UJkQLFD
D JHUDUDP LQGLFD VXD YDOLGDGH H D WRUQD DSWD D VHU ,, DVOHLVFRPSOHPHQWDUHV
H[HFXWDGD ,,, DVOHLVRUGLQiULDV
,9 RVGHFUHWRVOHJLVODWLYRV
$UW $VOHLVFRPSOHPHQWDUHVHRUGLQiULDVVHUmRSURPXOJD
GDVHPTXDUHQWDHRLWRKRUDVFRQWDGDV $UW $VUHVROXo}HVGHSHQGHPGHSXEOLFDomRQR'LiULRGD
&kPDUD/HJLVODWLYD
, GDGDWDGDVDQomRH[SUHVVD
$UW 6HDOHLIRUUHSXEOLFDGDSRUWHUVDtGRFRPLQFRUUHomR
,, GR~OWLPRGLD~WLOQRFDVRGHVDQomRWiFLWD FRQVLGHUDVHFRPRGDWDGHSXEOLFDomRDTXHRFRUUHU
SRU~OWLPRUHVSHLWDQGRVHRGLVSRVWRQRDUWGHVWD
,,, GDGDWDGRUHFHELPHQWRGRSURMHWRTXHFRQWHQKDYHWR /HL&RPSOHPHQWDU
UHMHLWDGRSHOD&kPDUD/HJLVODWLYD
6HomR9,,,
,9 GD ~OWLPD KRUD GR SUD]R SDUD SURPXOJDomRTXDQGR 'DV$XWRUL]Do}HV
KRXYHUVLOrQFLRGR*RYHUQDGRURXVXFHVVLYDPHQWHGR
3UHVLGHQWHHGR9LFH3UHVLGHQWHGD&kPDUD/HJLVODWLYD $UW $VDXWRUL]Do}HVOHJLVODWLYDVFRQVWLWXHPVHHPOLFHQoDV
QRVFDVRVSUHYLVWRVQRDUW†žGD/HL2UJkQLFD GR3RGHU/HJLVODWLYRGHFRUUHQWHVGHFDVRVSUHYLVWRVHP
OHL
$UW $ HPHQGD j /HL 2UJkQLFD VHUi SURPXOJDGD HPDWp
TXLQ]H GLDV ~WHLV FRQWDGRV GD SXEOLFDomR GH VXD †~QLFR $VDXWRUL]Do}HVOHJLVODWLYDVGHFRUUHPGHFDVRVSUHYLV
UHGDomRILQDOQR'LiULRGD&kPDUD/HJLVODWLYD WRVHPOHL

$UW 2SUD]RSDUDSURPXOJDomRGHGHFUHWROHJLVODWLYRHGH $UW $ DXWRUL]DomR OHJLVODWLYD VHUi GDGD SRU OHL RXSRU
UHVROXomRVHUiIL[DGRQR5HJLPHQWR,QWHUQRGD&kPDUD GHFUHWROHJLVODWLYRHGHSHQGHGHSHGLGRRXSURSRVWDGR
/HJLVODWLYD H QmR SRGHUi VHU VXSHULRU DR SUD]R GR ÐUJmRRXDXWRULGDGHLQWHUHVVDGD
DUWLJRDQWHULRU
$UW 1D DXWRUL]DomR OHJLVODWLYD VHUi HVSHFLILFDGDVXD
$UW $SURPXOJDomRGDVOHLVpDWULEXLomR DEUDQJrQFLDHIL[DGDVDVFRQGLo}HVHPTXHGHYDVHU
FXPSULGD
, GD 0HVD 'LUHWRUD GD &kPDUD /HJLVODWLYD SDUDDV
HPHQGDVj/HL2UJkQLFD &DStWXOR,,,
'$5('$d®2
,, GR*RYHUQDGRUSDUDDVOHLVFRPSOHPHQWDUHVHRUGLQi
ULDV 6HomR,
'DV'LVSRVLo}HV3UHOLPLQDUHV
,,, GR3UHVLGHQWHGD&kPDUD/HJLVODWLYDSDUDRVGHFUHWRV
OHJLVODWLYRVHUHVROXo}HVHVHR*RYHUQDGRUQmRRIL]HU $UW $UHGDomRGDVOHLVREHGHFHUijQRUPDFXOWDGDOtQJXD
SDUDDVOHLVFRPSOHPHQWDUHVHRUGLQiULDV SRUWXJXHVDDGRWDGDQR%UDVLOHVSHFLDOPHQWH
 &kPDUD/HJLVODWLYDGR
 'LVWULWR)HGHUDO 'LU&RQVWLWXFLRQDO 

, DRVDFRUGRVRUWRJUiILFRVHPYLJRU E XVDUXPPHVPRYRFiEXORRXH[SUHVVmRVHPSUHFRP


XPVyVHQWLGR
,, DR9RFDEXOiULR2UWRJUiILFRGD/tQJXD3RUWXJXHVDGD F XVDURVYRFiEXORVHH[SUHVV}HVTXHVHMDPFRPXQV
$FDGHPLD%UDVLOHLUDGH/HWUDV jVGLIHUHQWHVFDPDGDVVRFLDLV
G SDGURQL]DUDOLQJXDJHP
,,, jWHUPLQRORJLDGD1RPHQFODWXUD*UDPDWLFDO%UDVLOHLUD
9,,, HYLWDUVHmR
,9 DRVSDGU}HVOLQJtVWLFRVUHFRPHQGDGRVSHORVILOyORJRV
FRQFHLWXDGRV D RVQHRORJLVPRV
E DVFRQVWUXo}HVVLQWiWLFDVTXHSRVVDPJHUDUGXSOLFL
9 jGHQRPLQDomRRILFLDOGHyUJmRHQGHUHoRRXORJUDGRX GDGHGHVHQWLGR
URS~EOLFR F RHPSUHJRGHYRFiEXORRXH[SUHVVmRTXHFRQILJXUH
GXSORVHQWLGRQRWH[WR
†ž +DYHQGR GLYHUJrQFLD HQWUH DV QRUPDV RXHQWUHDV G DVIUDVHVORQJDV
JUDILDVGRVYRFiEXORVVHUiDGRWDGDDQRUPDRXJUDILD H RHPSUHJRGHVLJODVDEUHYLDWXUDVHVLQDLVTXHQmR
PDLVSRSXODU VHMDPSUySULRVGDVUHJUDVGHDUWLFXODomRGDVOHLV

†ž eIDFXOWDGRHTXLSDUDUQRWH[WRGDVOHLVDGHQRPLQD ,; HYLWDUVHi GDU GHILQLomR GH H[SUHVVmR RXYRFiEXOR


omRRILFLDOGHTXHWUDWDRLQFLVR9GHVWHDUWLJRjGHQR GLYHUVDGDTXHMiFRQVWDUGHRXWUDOHL
PLQDomRSRSXODUTXDQGRHVWDIRUPDLVFRQKHFLGDGR
; DVGDWDVGHGRFXPHQWRVVmRH[SUHVVDVHPGLDPrVH
TXHDTXHOD
DQR DSHQDV QD SULPHLUD UHIHUrQFLD QDV VHJXLQWHV
6HomR,,
DSHQDVSHORDQR DFUHVFHQWDGRRLQFLVR[DRDUWSHODOHLFRPSOHPHQWDUQž
'DV1RUPDV(VSHFtILFDVGH5HGDomR GHGRGIGH

6XEVHomR, †ž 2EVHUYDGRRGLVSRVWRQRLQFLVR9,,,HGHVWHDUWLJRVy


'RV3ULQFtSLRV pSHUPLWLGRRXVRGHVLJODDEUHYLDWXUDRXVLQDOFRQVD
JUDGR SHOR XVR H DSyV D H[SOLFLWDomR QD SULPHLUD
$UW $VOHLVVHUmRUHGLJLGDVFRPSUHFLVmRFODUH]DFRHVmRH UHIHUrQFLDGDTXLORTXHH[SUHVVD
FRQFLVmROHYDQGRVHHPFRQWDRVSULQFtSLRVVHJXLQWHV
†ž $GHILQLomROHJDOTXHVHIL]HUQHFHVViULDQRWH[WRGDOHL
, R YRFDEXOiULR MXUtGLFR FRQVDJUDGR SHOR 'LUHLWRGHYH VHUiUHGLJLGDGHPRGR
SUHYDOHFHUVREUHRYRFDEXOiULRFRPXP
, D JXDUGDU FRHUrQFLD FRP DV GHPDLV GHILQLo}HVMi
,, pYHGDGRRXVRGHH[SUHVV}HVGDVOtQJXDVHVWUDQJHLUDV H[LVWHQWHV
LQFOXVLYHGRODWLPVDOYRDVFRQVDJUDGDVSHODGRXWULQD ,, DSURSLFLDUHTXLOtEULRHQWUHRFRQWH~GRHDIRUPD
MXUtGLFDTXHQmRSXGHUHPVHUWUDGX]LGDVVHPSUHMXt]R ,,, DDVVHJXUDUDFRUUHWDH[SUHVVmRGDVLGpLDV
GHVHQWLGR
6XEVHomR,,
,,, p YHGDGR R XVR GH YRFiEXORV H[SUHVV}HV RXIUDVHV 'DV5HPLVV}HV
H[HPSOLILFDWLYDV HVFODUHFHGRUDV MXVWLILFDWLYDV RX
H[SOLFDWLYDV $UW 5HPLVVmRpDUHIHUrQFLDH[SUHVVDDOHLRXDGLVSRVLWLYR
GHOHL
,9 RVQ~PHURVTXHLQGLTXHPTXDQWLGDGHIUDomRSHUFHQ
WDJHP PHGLGD RX YDORU TXDQGR HPSUHJDGRV QDV †~QLFR 5HFHEH D GHQRPLQDomR GH UHPLVVmR LQWHUQD DTXHp
IUDVHV VmR H[SUHVVRV SRU DOJDULVPRV DUiELFRV RX IHLWDDGLVSRVLWLYRGDPHVPDOHLHPTXHDUHPLVVmRp
IHLWDHUHPLVVmRH[WHUQDDTXHpIHLWDDRXWUDOHLRXD
FRQIRUPHDWUDGLomRSRUDOJDULVPRVURPDQRVYHGDGD
GLVSRVLWLYRGHRXWUDOHL
DUHSURGXomRSRUH[WHQVRHQWUHSDUrQWHVLV QRYD UHGDomR
GDGDDRLQFLVRLYGRDUWSHODOHLFRPSOHPHQWDUQžGH
$UW $ UHPLVVmR D GLVSRVLWLYR GH OHL LQLFLDVH SHORDUWLJR
9 VDOYRVHDOHLIRUGHQDWXUH]DHPLQHQWHPHQWHWpFQLFD TXHTXDQGRVHJXLGRGRUHVSHFWLYRQ~PHURpLQGLFDGR
GDUVHi SUHIHUrQFLD DRV YRFiEXORV FRPXQV TXDQGR SHOD DEUHYLDWXUD DUW RX DUWV FRQIRUPH VH XVH D
HVWHVSXGHUHPH[SUHVVDUFRPSUHFLVmRRVYRFiEXORVGH UHPLVVmRSDUDXPRXPDLVDUWLJRV
QDWXUH]DWpFQLFD
†ž $RQ~PHURGRDUWLJRHLVRODGDVSRUYtUJXODVHIRUR
9, SUHIHULUVHi FDVR VHJXHPVH DV XQLGDGHV GH DUWLFXODomR FRPSOH
PHQWDUHV GR DUWLJR GHYHQGR D PDLV DEUDQJHQWH
D DIRUPDGRSOXUDOjGRVLQJXODU SUHFHGHUDPHQRVDEUDQJHQWH
E DDILUPDomRjQHJDomR
F DGHWHUPLQDomRGRVXMHLWRjVXDLQGHWHUPLQDomR †ž 1D UHPLVVmR D PDLV GH XP SDUiJUDIR VHUiXVDGRR
G D RUGHP GLUHWD GRV WHUPRV GD RUDomR j RUGHP VtPEROR††SDUiJUDIRGREUDGR
LQYHUVD
H D IRUPD YHUEDO QR SUHVHQWH j IRUPD QR IXWXUR $UW &DGDOHLFRPSOHPHQWDUHPVXDHPHQWDIDUiUHPLVVmR
DFUHVFHQWDGDDDOtQHDHDRLQFLVRYLGRDUWSHODOHLFRPSOHPHQWDUQžGH DR GLVSRVLWLYR GD /HL 2UJkQLFD TXH HVWLYHU VHQGR
  GLVFLSOLQDGR
6XEVHomR,,,
9,, EXVFDUVHi WDQWR QR WH[WR GD PHVPD OHL TXDQWRGH 'DV,QFRUSRUDo}HVSRU5HPLVVmR
XPDOHLSDUDRXWUD
$UW ,QFRUSRUDomR SRU UHPLVVmR p R UHFXUVR SHOR TXDOVH
D H[SUHVVDU D PHVPD LGpLD VHPSUH FRP R PHVPR PDQGDDSOLFDUDXPD OHLRTXHHVWiGLVFLSOLQDGRHP
YRFiEXORRXH[SUHVVmR RXWUD
 'LU&RQVWLWXFLRQDO &kPDUD/HJLVODWLYDGR'LVWULWR)HGHUDO


$UW %XVFDUVHi UHGLJLU D LQFRUSRUDomR SRU UHPLVVmRGH $UW &DGDXPDGDVHVSpFLHVGHOHLWUDWDGDVQRDUWžGHVWD


PRGR TXH VHMD SRVVtYHO GHSUHHQGHU SHOD VLPSOHV /HL&RPSOHPHQWDUWHUiQXPHUDomRSUySULD
OHLWXUDRVHQWLGRGRWH[WRLQFRUSRUDGR
†ž $V OHLV VmR QXPHUDGDV FRP DOJDULVPRVDUiELFRVQD
$UW eYHGDGDDLQFRUSRUDomRSRUUHPLVVmR RUGHP FUHVFHQWH GH VXD SXEOLFDomR H HP VHTrQFLD
LQLQWHUUXSWD
, VHDOHLRXGLVSRVLWLYRGHOHLLQFRUSRUDGRQmRVHDGDS
WDUULJRURVDPHQWHDRTXHGLVFLSOLQDDOHLLQFRUSRUDGR †ž $QXPHUDomRGDVOHLVWHUiFRPRLQtFLRDGDWD
UD
, GDSURPXOJDomRGD/HL2UJkQLFDSDUDVXDVHPHQGDV
,, VHDOHLRXGLVSRVLWLYRGHOHLLQFRUSRUDGRIRUGHKLHUDU
TXLDLQIHULRUDRGDOHLLQFRUSRUDGRUD ,, GDSURPXOJDomRGD/HL2UJkQLFDGHSDUDDVOHLV
FRPSOHPHQWDUHV
,,, GHOHLRXGLVSRVLWLYRGHOHLTXHQmRGHSHQGDGHVDQomR
HPOHLTXHGHODGHSHQGD ,,, GDSURPXOJDomRGD&RQVWLWXLomRGHSDUDDVOHLV
RUGLQiULDV
,9 GH QRUPD RX GLVSRVLWLYR GH QRUPD TXH QmRHVWHMD
VXMHLWRDRSURFHVVROHJLVODWLYRGD&RQVWLWXLomR)HGHUDO ,9 GD LQVWDODomR GD SULPHLUD OHJLVODWXUD GD&kPDUD
RXGD/HL2UJkQLFD /HJLVODWLYDSDUDRVGHFUHWRVOHJLVODWLYRVHUHVROXo}HV

†ž 6HUmRQXPHUDGRV
†~QLFR $VYHGDo}HVGHVWHDUWLJRQmRVHDSOLFDPjVUHPLVV}HV
IHLWDVGHPRGRJHQpULFR
, SHOD&kPDUD/HJLVODWLYD
$UW 1DLQFRUSRUDomRSRUUHPLVVmRGHFODUDUVHiH[SUHVVD
D DVHPHQGDVj/HL2UJkQLFD
PHQWHVHWDPEpPILFDLQFRUSRUDGDDOWHUDomRSRVWHULRU
E RVGHFUHWRVOHJLVODWLYRV
F DVUHVROXo}HV
†~QLFR 6DOYRHPFDVRGHQRYDUHGDomRGRGLVSRVLWLYRLQFRUSR
UDGR SRU UHPLVVmR p YHGDGR LQFRUSRUDU DOWHUDomR ,, SHOR3RGHU([HFXWLYR
SRVWHULRUjGDWDGHSXEOLFDomRGDOHLLQFRUSRUDGRUD
D DVOHLVFRPSOHPHQWDUHV
6HomR,,, E DVOHLVRUGLQiULDV
'D(VWUXWXUDGDV/HLV
†ž 2GLVSRVWRQRLQFLVR,,GRSDUiJUDIRDQWHULRUDSOLFDVH
6XEVHomR, DLQGD TXDQGR D SURPXOJDomR WHQKD VLGR IHLWD SHOR
'DV'LVSRVLo}HV3UHOLPLQDUHV 3UHVLGHQWH GD &kPDUD /HJLVODWLYD RX SRU VHX 9L
FH3UHVLGHQWH
$UW $HVWUXWXUDGDVOHLVFRPS}HVHGH
$UW /RJRDSyVRQ~PHURGDOHLVHUiFRORFDGDDGDWDTXH
, SUHkPEXOR WHUiGLDPrVHDQR
,, WH[WR
,,, IHFKR $UW (PHQWDpDSDUWHGRWtWXORTXHSHUPLWHLGHQWLILFDUDOHL
SHODVtQWHVHGHVHXFRQWH~GRRXILQDOLGDGH
†~QLFR 2WH[WRFRQWpPDVGLVSRVLo}HVQRUPDWLYDVGDVOHLV
†ž $ HPHQWD VHUi LQLFLDGD SRU XP YHUERQDWHUFHLUD
6XEVHomR,, SHVVRDGRVLQJXODUGRSUHVHQWHGRLQGLFDWLYRHVLQWHWL
'R3UHkPEXOR ]DUiRFRQWH~GRRXDILQDOLGDGHGDOHL

$UW 3UHkPEXOR p D SDUWH LQLFLDO GD OHL TXH SHUPLWHVXD †ž $HPHQWDVHUiJUDIDGDHPQHJULWRRXQDIDOWDGHVWH
LGHQWLILFDomR SRU PHLR GH FDUDFWHUHV TXH D UHDOFHP H VHX WH[WR
VLWXDUVHiHQWUHRFHQWURHDPDUJHPGLUHLWDGRSDSHO
$UW 2SUHkPEXORFRQWpP
†ž 1DUHGDomRGDHPHQWDVHUiREVHUYDGRRGLVSRVWRQRV
, RWtWXORTXHFRPSUHHQGHDHStJUDIHHDHPHQWD DUWVHGHVWD/HL&RPSOHPHQWDU

,, DIyUPXODGHSURPXOJDomRTXHFRPSUHHQGH $UW $IyUPXODGHSURPXOJDomRVHUiFRORFDGDORJRDSyVD


HPHQWDHDOLQKDGDFRPRWH[WRGDOHL
D DDXWRULD
E RIXQGDPHQWROHJDOGDDXWRULGDGH †~QLFR $ IyUPXOD GH SURPXOJDomR VHUi LQVHULGD QDOHLSRU
F DRUGHPGHH[HFXomR TXHPDSURPXOJDU

$UW $HStJUDIHJUDIDGDHPFDUDFWHUHVPDL~VFXORVpDSDUWH $UW $IyUPXODGHSURPXOJDomRFRQWpP


GRWtWXOR
, DGHQRPLQDomRGRÐUJmRRXGRFDUJRGDDXWRULGDGH
TXHSURPXOJDUDOHL
, TXHTXDOLILFDDOHLGHQRPLQDQGRDSHODVXDHVSpFLH
,, D GHVLJQDomRGHTXHDOHLIRLDSURYDGDSHOD&kPDUD
,, TXHGLVWLQJXHDOHLGHRXWUDVGDPHVPDHVSpFLHSHOD
/HJLVODWLYD
QXPHUDomR
,,, DLQGLFDomRTXDQGRIRURFDVRGHTXHDDXWRULGDGH
,,, TXHVLWXDDOHLQRWHPSRSHODVXDGDWD TXHGHYHULDSURPXOJDUDOHLQmRRIH]QRSUD]ROHJDO
&kPDUD/HJLVODWLYDGR
  'LVWULWR)HGHUDO 'LU&RQVWLWXFLRQDO 

,9 R IXQGDPHQWR OHJDO SDUD R ÐUJmR RXDXWRULGDGH †ž 2VHQWLGRRUDFLRQDOGRSDUiJUDIRSRGHVHUFRPSOHPHQ


SURPXOJDUDOHL WDGRSRULQFLVRV

9 DRUGHPGHH[HFXomR †ž $SOLFDVHjUHGDomRGRSDUiJUDIRRGLVSRVWRQR†žGR


DUWLJRDQWHULRU
†ž 2 IXQGDPHQWR OHJDO SDUD R ÐUJmRRXDXWRULGDGH
SURPXOJDUDOHLGHFRUUHGD/HL2UJkQLFD $UW ,QFLVRpDXQLGDGHGHDUWLFXODomR

†ž $RUGHPGHH[HFXomRVHUiH[SUHVVDSHODIRUPDFRQVD , TXH FRPSOHPHQWD R VHQWLGR RUDFLRQDO GR FDSXWGH


JUDGDSHORXVRSDUDFDGDHVSpFLHGHOHL DUWLJRRXGRSDUiJUDIR

$UW e IDFXOWDGR XVDU DQWHV GD RUGHP GH H[HFXomRD ,, TXHH[SOLFLWDQRUPDVFRQWLGDVHPSULQFtSLRRXWHUPR
MXVWLILFDomRGRVDWRVTXHOHYDUDPjSURPXOJDomRGDOHL GRFDSXWGHDUWLJRRXGRSDUiJUDIR
VREDIRUPDGHFRQVLGHUDQGR
†ž &RPRXQLGDGHFRPSOHPHQWDUGHDUWLFXODomRRVHQWLGR
6XEVHomR,,, GRLQFLVRpVHPSUHGHSHQGHQWHGRVHQWLGRGRFDSXWGH
'D$UWLFXODomR DUWLJRRXGRSDUiJUDIR
$UW 2WH[WRGDVOHLVVHUiDUWLFXODGR †ž 1mRKDYHUiLQFLVR~QLFR
$UW $XQLGDGHEiVLFDGHDUWLFXODomRpRDUWLJRRSDUiJUDIR
†ž 1D UHGDomR GR LQFLVR VHUmR REVHUYDGDVDVQRUPDV
RLQFLVRDDOtQHDHRQ~PHURVmRXQLGDGHVFRPSOHPHQ
VHJXLQWHV
WDUHV
, VHUi QXPHUDGR HP DOJDULVPR URPDQR VHJXLGRGH
†ž &DGD XQLGDGH GH DUWLFXODomR REHGHFHUiDQRUPDV
WUDYHVVmR
SUySULDVHVWDWXtGDVQHVWDVXEVHomR

†ž $V XQLGDGHV FRPSOHPHQWDUHV GHDUWLFXODomRQmR ,, VHXWH[WRSULQFLSLDSRUOHWUDLQLFLDOPLQ~VFXOD


VXEVLVWHPVHPDVTXHSRUHODVVmRFRPSOHPHQWDGDV
,,, XPLQFLVRVHSDUDVHGRRXWURSRUSRQWRHYtUJXOD
†ž 5HFHEHDGHQRPLQDomRGHGLVSRVLWLYRDQRUPDFRQWLGD
HPFDGDXQLGDGHGHDUWLFXODomR ,9 WHUPLQDSRUGRLVSRQWRVVHIRUGHVGREUDGRHPDOtQHDV

$UW 2DUWLJRFRQWHUiDSHQDVXPDUHJUDHVHUiH[SUHVVRSRU 9 R~OWLPRLQFLVRGHFDGDVpULHWHUPLQDSRUSRQWR


XPD ~QLFD IUDVH FXMR VHQWLGR RUDFLRQDO SRGHUi VHU
FRPSOHPHQWDGRRXH[SOLFLWDGRSRULQFLVRV 9, SDUDFDGDFDSXWGHDUWLJRRXSDUiJUDIRLQLFLDVHQRYD
QXPHUDomRGHLQFLVRV
†ž 'HSRLVGHSDUiJUDIRRFDSXWGRDUWLJRQmRSRGHUiVHU
GHVPHPEUDGRHPLQFLVRV †ž 2VHQWLGRRUDFLRQDOGRLQFLVRSRGHVHUFRPSOHPHQWDGR
SRUDOtQHD
†ž 2 DUWLJR VHUi LQGLFDGRDWUDYpVGDDEUHYLDWXUD$UW
VHJXLGDGHQXPHUDomRRUGLQDODWpRQRQRHFDUGLQDO †ž eYHGDGRXVDUDOtQHDQROXJDUGHLQFLVR
GHSRLVGHVWH
$UW $DOtQHDpDXQLGDGHGHDUWLFXODomRTXHFRPSOHPHQWD
†ž (QWUHDQXPHUDomRHPDOJDULVPRRUGLQDOHRWH[WRQmR RVHQWLGRRUDFLRQDOGRLQFLVR
VHUiFRORFDGRQHQKXPVLQDOGHSRLVGDQXPHUDomRHP
DOJDULVPRFDUGLQDOVHUiFRORFDGRXPSRQWR †ž &RPRXQLGDGHFRPSOHPHQWDUGHDUWLFXODomRRVHQWLGR
GDDOtQHDpVHPSUHGHSHQGHQWHGRLQFLVR
†ž $QXPHUDomRDTXHVHUHIHUHR†žGHVWHDUWLJRVHUi
IHLWDHPRUGHPFUHVFHQWHHLQLQWHUUXSWDSDUDFDGDOHL †ž 1mRKDYHUiDOtQHD~QLFD

†ž 2WH[WRGRDUWLJRSULQFLSLDSRUOHWUDLQLFLDOPDL~VFXOD †ž 1D UHGDomR GD DOtQHD VHUmR REVHUYDGDVDVQRUPDV


H WHUPLQD SRU SRQWR VDOYR VH IRU GHVGREUDGR HP VHJXLQWHV
LQFLVRVTXDQGRWHUPLQDUiSRUGRLVSRQWRV
, VHUiLQGLFDGDSRUOHWUDPLQ~VFXODVHJXLGDGRVLQDO 
$UW 2SDUiJUDIRpDXQLGDGHFRPSOHPHQWDUGHDUWLFXODomR
TXHH[SUHVVDRVSRUPHQRUHVQHFHVViULRVjDSUHHQVmR ,, VHXWH[WRSULQFLSLDSRUOHWUDLQLFLDOPLQ~VFXOD
GRVHQWLGRGRDUWLJRRXDVFLUFXQVWkQFLDVTXHDPSOLHP
RXUHVWULQMDPVXDLQWHQomR ,,, XPDDOtQHDVHSDUDVHGDRXWUDSRUSRQWRHYtUJXOD
†ž &RPR XQLGDGH GHSHQGHQWH GR FDSXW GRDUWLJRR ,9 WHUPLQDSRUGRLVSRQWRVVHIRUGHVGREUDGDHPQ~PH
SDUiJUDIRQmRVXEVLVWHVHPHOH URV
†ž &DGDFRQMXQWRGHSDUiJUDIRVWHPQXPHUDomRSUySULD
9 D ~OWLPD DOtQHD GH FDGD VpULH WHUPLQDSRU
GHQWURGRDUWLJRDTXHSHUWHQFHP
SRQWRHYtUJXODVHGHSRLVGHODKRXYHUQRYRLQFLVRH
SRUSRQWRVHQmRKRXYHU
†ž +DYHQGRDSHQDVXPSDUiJUDIRVHUiHOHGHVLJQDGRSHOD
H[SUHVVmR3DUiJUDIR~QLFRVHJXLGDGHSRQWRKDYHQ
9, SDUDFDGDLQFLVRLQLFLDVHQRYDVpULHGHDOtQHDV
GRPDLVGHXPVHUmRHOHVLQGLFDGRVSHORVtPEROR†
VHJXLGRGHQ~PHURRUGLQDODWpRQRQRHFDUGLQDOGDt
HPGLDQWH †ž 2VHQWLGRRUDFLRQDOGDDOtQHDSRGHVHUFRPSOHPHQWDGR
SRUQ~PHUR
 'LU&RQVWLWXFLRQDO &kPDUD/HJLVODWLYDGR'LVWULWR)HGHUDO


$UW 2Q~PHURpDXQLGDGHGHDUWLFXODomRTXHFRPSOHPHQ , FDGD XQLGDGH GH DJUXSDPHQWR GH DUWLJR FRQWHUiD
WDRVHQWLGRRUDFLRQDOGDDOtQHD GHQRPLQDomR

†ž &RPRXQLGDGHFRPSOHPHQWDUGHDUWLFXODomRRQ~PHUR D GR WLSR GH XQLGDGH GH DJUXSDPHQWR VHJXLGD GH


pVHPSUHGHSHQGHQWHGDDOtQHD DOJDULVPRURPDQR

†ž 1mRKDYHUiQ~PHUR~QLFR E GRDVVXQWRTXHQHODpGLVFLSOLQDGR

†ž 1DUHGDomRGRQ~PHURVHUmRREVHUYDGDVDVQRUPDV ,, DGHQRPLQDomRGRDVVXQWRWUDWDGDHPFDGDXQLGDGHGH


VHJXLQWHV DJUXSDPHQWR VHUi LQLFLDGD SHOD SUHSRVLomR 'H
FRPELQDGDFRPRDUWLJRGHILQLGRDSURSULDGR
, VHUiLQGLFDGRSRUDOJDULVPRDUiELFRVHJXLGRGRVLQDO
  ,,, VHPSUH TXH SRVVtYHO DV XQLGDGHV GHDJUXSDPHQWR
VHUmRJUDIDGDVHPQHJULWR
,, VHXWH[WRSULQFLSLDSRUOHWUDLQLFLDOPLQ~VFXOD
,9 RFDStWXORRWtWXORHROLYURVHUmRJUDIDGRVFRPFDUDF
,,, XPQ~PHURVHSDUDVHGRRXWURSRUSRQWRHYtUJXOD WHUHVPDL~VFXORV

,9 R ~OWLPR Q~PHUR GH FDGD VpULH WHUPLQDSRU 9 D VHomR H D VXEVHomR VHUmR JUDIDGDV FRPFDUDFWHUHV
PLQ~VFXORVVDOYRDOHWUDLQLFLDOGRSULPHLURYRFiEXOR
SRQWRHYtUJXODVHGHSRLVGHOHKRXYHUQRYDDOtQHDRX
HDGRVTXHQmRVHMDPPHUDVSDUWtFXODVGHOLJDomRTXH
LQFLVRHSRUSRQWRVHQmRKRXYHU
WHUmRDOHWUDLQLFLDOPDL~VFXOD
9 SDUDFDGDDOtQHDLQLFLDVHQRYDVpULHGHQ~PHURV
†~QLFR +DYHQGR SRVVLELOLGDGH RV WtWXORV HVXEVHo}HVVHUmR
JUDIDGRVHPLWiOLFRHRVOLYURVHDVSDUWHVFRPFDUDF
$UW 2VDUWLJRVGDVGLVSRVLo}HVWUDQVLWyULDVVHUmRQXPHUD
WHUHVHVSHFLDLV
GRVHPVHTrQFLDDRVDUWLJRVGDVGLVSRVLo}HVSHUPD
QHQWHV $UW 1HQKXPDOHLWHUiPDLVGRTXHGXDVSDUWHV $UWLJRPDQWLGR
SHOD&/')DSyVYHWRGR*RYHUQDGRUGR')GRGIGH
†~QLFR 2 GLVSRVWR QHVWH DUWLJR QmR p REULJDWyULRSDUDRV
FyGLJRV †ž $V SDUWHV VHUmR GHVLJQDGDV H[FOXVLYDPHQWHGHSDUWH
6XEVHomR,9 JHUDOHSDUWHHVSHFLDOVHPPHQFLRQDURDVVXQWRQHODV
'RV$JUXSDPHQWRVGH$UWLJRV WUDWDGR 3DUiJUDIRPDQWLGRSHOD&/')DSyVYHWRGR *RYHUQDGRUGR')GRGIGH


$UW 2VDUWLJRVGHXPDOHLSRGHPUHXQLUVHHPXQLGDGHVGH †ž $ SDUWH HVSHFLDO VHPSUH DEULUi SiJLQDQRYD 3DUiJUDIR


DJUXSDPHQWR PDQWLGRSHOD&/')DSyVYHWRGR*RYHUQDGRUGR')GRGIGH

†~QLFR 3DUDFRQVHFXomRGRGLVSRVWRQHVWHDUWLJRVHUiFRQVLGH $UW eYHGDGRUHGLJLUOHLFXMRVDUWLJRVHVWHMDPUHXQLGRVHP


UDGD XQLGDGHVGHDJUXSDPHQWRVHPQXPHUDomR

, D H[WHQVmR GD OHL H D FRPSDUWLPHQWDomR GRDVVXQWR 6XEVHomR9


QHODDERUGDGR 'DV1RUPDVGH6LVWHPDWL]DomR

,, DGHQRPLQDomRGRDVVXQWRHPFDGDXQLGDGHGHDJUX $UW $OHLVHUiHVWUXWXUDGDGHPRGRTXHVHXVGLVSRVLWLYRV


SDPHQWR JXDUGHPFRHUrQFLDHKDUPRQLDHQWUHVLHVHMDLQVHULGD
DGHTXDGDPHQWHQRVLVWHPDMXUtGLFR
,,, DDILQLGDGHHQWUHRVDVVXQWRVGRVDUWLJRVDJUXSDGRV
†~QLFR 5HFHEH D GHQRPLQDomR GH VLVWHPDWL]DomRLQWHUQDD
,9 DVLVWHPDWL]DomRDGRWDGDQDOHL FRHUrQFLD H KDUPRQLD TXH RV GLVSRVLWLYRV GHYDP WHU
HQWUHVLHVLVWHPDWL]DomRH[WHUQDDDGHTXDGDLQVHUomR
$UW 2DJUXSDPHQWRGHDUWLJRVWHUiSRUEDVHRFDStWXOR GDOHLQRVLVWHPDMXUtGLFR

$UW 2 FDStWXOR SRGH GLYLGLUVH HP VHo}HV H HVWDVHP $UW 3DUD D VLVWHPDWL]DomR H[WHUQD VHUmR REVHUYDGRVRV
VXEVHo}HV SULQFtSLRVVHJXLQWHV

, DOHLWHUiVHXREMHWRHkPELWRGHDSOLFDomRLQGLFDGRV
†~QLFR 1mRKDYHUiVHomR~QLFDQHPVXEVHomR~QLFD
HPVHXDUWLJRSULPHLUR
$UW 3RGHUmRVHUDJUXSDGRV
,, QHQKXPDOHLFRQWHUiPDWpULDHVWUDQKDDVHXREMHWRRX
TXHDHVWHQmRHVWHMDYLQFXODGRSRUDILQLGDGHSHUWL
, RVFDStWXORVHPWtWXORV
QrQFLDRXFRQH[mR
,, RVWtWXORVHPOLYURV
,,, RVOLYURVHPSDUWHV ,,, RPHVPRDVVXQWRQmRSRGHUiVHUGLVFLSOLQDGRSRUPDLV
GHXPDOHLVDOYR
†~QLFR $VXQLGDGHVGHDJUXSDPHQWRFRQVWDQWHVGHVWHDUWLJRVy
SRGHUmR VHU ~QLFDV TXDQGR D OHL IRU GLYLGLGD HP D VHOHLSRVWHULRUDOWHUDUOHLDQWHULRU
XQLGDGHVGHDJUXSDPHQWRVPDLVDEUDQJHQWHVGRTXHD E QRFDVRGHOHLJHUDOHOHLHVSHFLDO
FRQVLGHUDGD
,9 EXVFDUVHiGLVFLSOLQDURPDLVHVSHFLILFDPHQWHSRVVtYHO
$UW 3DUDGHQRPLQDUDVXQLGDGHVGHDJUXSDPHQWRVVHUmR DVGLYHUVDVLPSOLFDo}HVGHFRUUHQWHVGDPDWpULDGLVFLSOL
REVHUYDGDVDVQRUPDVVHJXLQWHV QDGDSHODOHL
&kPDUD/HJLVODWLYDGR
  'LVWULWR)HGHUDO 'LU&RQVWLWXFLRQDO 

†ž 6HPSUHTXHGXDVRXPDLVOHLVYHUVDUHPVREUHRPHVPR †ž 5HFHEH D GHQRPLQDomR GH FOiXVXOD GHYLJrQFLDR


DVVXQWRGHYHUmRVHUREVHUYDGDVDVQRUPDVGR&DStWXOR GLVSRVLWLYRTXHGLVFLSOLQDDGDWDGHHQWUDGDHPYLJRU
9GHVWD/HL&RPSOHPHQWDU GDOHL ILFDUHQXPHUDGRRSDUiJUDIR~QLFRSDUD†žHDFUHVFHQWDGRR†žDRDUW
SHODOHLFRPSOHPHQWDUQžGHGRGIGH

†ž 2V DVVXQWRV GH FDUiWHU SHUPDQHQWH QmRSRGHPVHU


WUDWDGRVQDVOHLVGHFDUiWHUWHPSRUiULR †ž 1mR KDYHQGR FOiXVXOD GH YLJrQFLD D OHLFRPHoDD
YLJRUDU HP WRGR R 'LVWULWR )HGHUDO  GLDV DSyV VXD
$UW 3DUD D VLVWHPDWL]DomR LQWHUQD VHUmR REVHUYDGRVRV SXEOLFDomR
SULQFtSLRVVHJXLQWHV
$UW 1DHVWLSXODomRGDFOiXVXODGHYLJrQFLDVHUmROHYDGRV
, GHYHPVHUGLVFLSOLQDGRV HPFRQWD

D VHSDUDGDPHQWHLQVWLWXWRVGLYHUVRV , XUJrQFLD


,, FRPSOH[LGDGHGHVXDVQRUPDV
E HP GLVSRVLWLYRV TXH HVWHMDP SUy[LPRV XQV GRV ,,, DOWHUDo}HVTXHSURYRFDUiQRVLVWHPDMXUtGLFR
RXWURVPDWpULDVDILQVRXFRQH[DV ,9 SUD]RQHFHVViULRSDUDTXHRVGHVWLQDWiULRVVHDGDSWHP
DVXDVH[LJrQFLDV
F VHJXQGRRUGHPFURQROyJLFDRVSURFHGLPHQWRV
†~QLFR 6RPHQWHHQWUDHPYLJRU
,, RVSULQFtSLRVGHYHPSUHFHGHUVXDUHJXODPHQWDomR
, QRYHQWDGLDVGHSRLVGHSXEOLFDGDDOHLTXHLQVWLWXLURX
,,, QDV XQLGDGHV GH DJUXSDPHQWRV GH DUWLJRGHYHP DXPHQWDUFRQWULEXLomRVRFLDOGRVVHUYLGRUHVS~EOLFRV
VHPSUHYLUDQWHV GR'LVWULWR)HGHUDO

D GDVGLVSRVLo}HVWUDQVLWyULDVDVSHUPDQHQWHV ,, QRH[HUFtFLRILQDQFHLURVHJXLQWHDRGDSXEOLFDomRDOHL


E GDVGLVSRVLo}HVDFHVVyULDVDVSULQFLSDLV TXHLQVWLWXLURXDXPHQWDUWULEXWRV
F GDVGLVSRVLo}HVHVSHFLDLVDVSUHOLPLQDUHV
G GDVGLVSRVLo}HVSDUWLFXODUHVDVFRPXQV ,,, QRSULPHLURGLDGRH[HUFtFLRILQDQFHLURSDUDRTXDOIRL
HODERUDGDDOHLRUoDPHQWiULD
†ž ,QVWLWXWRVGLYHUVRVHPDWpULDVDILQVRXFRQH[DVGHYHP
VHUWUDWDGRVVHJXQGRVXDSRVVtYHORUGHPGHRFRUUrQFLD ,9 QR SULPHLURGLDGRDQRVXEVHTHQWHDRGDSRVVHGR
*RYHUQDGRUHOHLWRDOHLTXHDSURYDURSODQRSOXULDQX
†ž $VGLVSRVLo}HVFRPXQVGHLQVWLWXWRVGLYHUVRVGHYHPVHU DO
WUDWDGDVHPFRQMXQWR
$UW 2 SUD]R GDGR SRU OHL SDUD LQtFLR GH VXD YLJrQFLDp
†ž $V GLVSRVLo}HV TXH LQWURGX]DP XPDVVXQWRVHUmR FRQWtQXRHVyVHLQWHUURPSHRXVHVXVSHQGHHPYLUWXGH
WUDWDGDVSUHOLPLQDUPHQWH GHOHLSRVWHULRURXGHGHFLVmRMXGLFLDO

†ž 3DUD TXH D QRUPD HVSHFtILFD GH XPLQVWLWXWRVHMD †ž 1RF{PSXWRGRSUD]RLQFOXLVHRGLDGDSXEOLFDomR


DSOLFDGDDRXWURTXHOKHVHMDDILPRXFRQH[RpQHFHV
ViULRID]HUUHPLVVmRH[SUHVVD †ž 2 GLD GH LQtFLR GD YLJrQFLD GD OHL QmRVHSURUURJD
DLQGDTXHFDLDHPGLDGHViEDGRGRPLQJRRXIHULDGR
$UW 2V DUWLJRV TXH FRQWHQKDP QRUPDV GH FDUiWHUJHUDO
SRGHP VHU DJUXSDGRV HP XQLGDGH GHQRPLQDGD GH $UW 3DUDHIHLWRGHYLJrQFLDFRQVLGHUDVHOHLQRYDDSDUWH
GLVSRVLo}HVJHUDLV YHWDGDFXMRYHWRWHQKDVLGRUHMHLWDGR

†ž 9rPQRLQtFLRGDOHLRXDQWHVGHRXWUDVXQLGDGHVGH $UW +DYHQGRQRYDSXEOLFDomRFRPRSURSyVLWRGHUHWLILFDU


DJUXSDPHQWR DV GLVSRVLo}HV JHUDLV GH FXMDV QRUPDV WH[WRGHOHLVHUiREVHUYDGRRVHJXLQWH
GHSHQGDPRXWUDVGHGLVFLSOLQDPHQWRVHVSHFtILFRV
, SDUDHIHLWRGHYLJrQFLDFRQVLGHUDVHOHLQRYDDUHWLILFD
†ž 9rPQRILQDOGDOHLRXGHSRLVGHRXWUDVXQLGDGHVGH omRGHWH[WRGDOHLTXHMiHVWHMDHPYLJRU
DJUXSDPHQWRV DV GLVSRVLo}HV JHUDLV FXMDV QRUPDV
VHMDPGLUHWDPHQWHGHSHQGHQWHVGHRXWUDVGHGLVFLSOL ,, QmR HVWDQGR DLQGD HP YLJRU D OHL R SUD]R GHVXD
QDPHQWRVHVSHFtILFRV YLJrQFLDUHFRPHoDDIOXLUGD~OWLPDSXEOLFDomR

6XEVHomR9, †~QLFR )LFDPUHVJXDUGDGRV RV GLUHLWRV DGTXLULGRVGHERDIp


'D9LJrQFLDGDV/HLV GXUDQWHDYLJrQFLDGRVGLVSRVLWLYRVTXHWHQKDPVLGR
UHWLILFDGRV
$UW $OHLFRPHoDDYLJRUDUHPWRGRRWHUULWyULRGR'LVWULWR
)HGHUDOQDGDWDSRUHODLQGLFDGDHVRPHQWHSHUGHVXD $UW $ OHL SRGHUi FRPHoDU D SURGX]LU HIHLWRV HPGDWD
YLJrQFLDWRWDORXSDUFLDOPHQWH GLYHUVDjGRLQtFLRGHVXDYLJrQFLD

, SHODUHYRJDomR †~QLFR eYHGDGRRHIHLWRUHWURDWLYRVDOYRVHDOHLYHUVDU

,, SRUWHUH[SLUDGRRSUD]RSDUDRTXDOIRLHODERUDGD , VREUH DXPHQWR RX UHDMXVWH D TXDOTXHU WtWXORGD


UHPXQHUDomRGHDXWRULGDGHRXVHUYLGRUHVS~EOLFRVGR
,,, SHODVXSHUYHQLrQFLDGHOHLIHGHUDOVREUHQRUPDVJHUDLV 'LVWULWR)HGHUDO
QR kPELWR GD OHJLVODomR FRQFRUUHQWH QRV WHUPRV GR ,, VREUHRUoDPHQWRDQXDO
TXHGLVS}HRDUW†žGD&RQVWLWXLomR)HGHUDO ,,, VREUHRGLVSRVWRQR†žGRDUWLJRDQWHULRU
 'LU&RQVWLWXFLRQDO &kPDUD/HJLVODWLYDGR'LVWULWR)HGHUDO


$UW $OHLRXSDUWHGHODTXHWURX[HUDGHWHUPLQDomRGHVHU $UW$ QRYD UHGDomR GDGD D GLVSRVLWLYR GH OHL UHYRJDD
UHJXODPHQWDGDIL[DUiRSUD]RSDUDTXHVHFXPSUDWDO UHGDomRDQWHULRU
GHWHUPLQDomR
$UW 'iVHUHYRJDomRWiFLWDTXDQGRDQRUPDGHXPDOHLTXH
†~QLFR 1mRVHQGRIHLWDDUHJXODPHQWDomRQRSUD]RIL[DGRD QmR IRL H[SUHVVDPHQWH UHYRJDGD VHMD MXULGLFDPHQWH
&kPDUD/HJLVODWLYDVROLFLWDUiLQIRUPDomRDR*RYHUQD LQFRPSDWtYHOFRPQRUPDGHOHLQRYD
GRUQRVWHUPRVGRDUW;;;,,GD/HL2UJkQLFD
†ž $UHYRJDomRWiFLWDREHGHFHUijVUHJUDVGHKHUPHQrXWL
$UW $ OHL TXH FRQFHGD LVHQomR RX EHQHItFLR ILVFDOVHUi FDREVHUYDGRRVHJXLQWH
HODERUDGDFRPSUD]RFHUWRGHYLJrQFLD
, OHL SRVWHULRU UHYRJD D DQWHULRU QDTXLOR TXH OKHIRU
†~QLFR 1HQKXPD LVHQomR RX EHQHItFLR ILVFDOVHUiFRQFHGLGR FRQWUiULR
FRPSUD]RTXHXOWUDSDVVHDYLJrQFLDGDOHLTXHDSURYDU
RSODQRSOXULDQXDO ,, ILFD UHYRJDGD D OHL FXMD PDWpULD VHMDLQWHJUDOPHQWH
GLVFLSOLQDGDSRUOHLSRVWHULRU
$UW $DOWHUDomRHPWH[WRGHOHLFRQVLGHUDVHOHLQRYD
†ž 2GLVSRVWRQRLQFLVR,GRSDUiJUDIRDQWHULRUDSOLFDVH
$UW $OHLUHYRJDGDWRWDORXSDUFLDOPHQWHQmRVHUHVWDXUD LQFOXVLYH TXDQGR KRXYHU LQFRPSDWLELOLGDGH MXUtGLFD
SRUWHUSHUGLGRDYLJrQFLDDOHLUHYRJDGRUD HQWUHOHLTXHHVWDEHOHoDQRUPDVJHUDLVHOHLTXHHVWDEH
OHoDQRUPDVHVSHFLDLV
†~QLFR 2 GLVSRVWR QHVWH DUWLJR QmR VH DSOLFD j OHLTXHIRU
HODERUDGDFRPRSURSyVLWRGHUHVWDEHOHFHUDYLJrQFLD $UWeYHGDGDDUHXWLOL]DomRGDQXPHUDomRGHGLVSRVLWLYR
GHOHLUHYRJDGD UHYRJDGR VDOYR QRV FDVRV SUHYLVWRV QRV DUWV 
SDUiJUDIR ~QLFR H  SDUiJUDIR ~QLFR GHVWD /HL
6XEVHomR9,, &RPSOHPHQWDU
'D5HYRJDomRGDV/HLV
6XEVHomR9,,,
$UW 5HYRJDomR p D GHWHUPLQDomR H[SUHVVD RXWiFLWD 'R)HFKR
FRQWLGD HP OHL TXH PDQGD FHVVDU D YLJrQFLD GH OHL
DQWHULRU $UW$SyV R ~OWLPR DUWLJR VHUmR FRQVLJQDGRV R ORFDO HD
GDWDGDDVVLQDWXUDGDDXWRULGDGHTXHSURPXOJDUDOHL
†ž $UHYRJDomRTXHWHUiGLVSRVLWLYRSUySULRFKDPDGRGH
FOiXVXODUHYRJDWyULDFRQVWDUiGR~OWLPRDUWLJRGDOHL $UW1DVHPHQGDVj/HL2UJkQLFDQDVOHLVFRPSOHPHQWDUHV
HQDVOHLVRUGLQiULDVVHUiIHLWDUHIHUrQFLDDRDQRHP
†ž eGLVSHQVDGDDFOiXVXODUHYRJDWyULDGDOHLFXMDPDWpULD TXHVHHVWLYHUHPUHODomRj3URFODPDomRGD5HS~EOLFD
QmRWHQKDVLGRGLVFLSOLQDGDDQWHULRUPHQWH HjLQDXJXUDomRGH%UDVtOLDFRPR&DSLWDOGR%UDVLO

$UW 'iVHDUHYRJDomRH[SUHVVDTXDQGRDOHLQRYDLGHQWLIL $UW/RJRDSyVDGDWDDOHLGHYHUiVHUDVVLQDGDSRUTXHPD


FDDOHLDQWHULRUDWLQJLGDWRWDORXSDUFLDOPHQWHSHOD SURPXOJDU
UHYRJDomR
$UW$V OHLV FRPSOHPHQWDUHV H RUGLQiULDV TXDQGR IRUR
†ž $UHYRJDomRH[SUHVVDREHGHFHUiDRVHJXLQWH FDVR GHYHUmR VHU UHIHUHQGDGDV SHORV 6HFUHWiULRV GH
*RYHUQRFXMDSDVWDVHDFKHUHODFLRQDGDFRPDPDWpULD
, XPD OHL Vy SRGH VHU UHYRJDGD SRU RXWUD GDPHVPD OHJLVODGDQRVWHUPRVGRTXHGLVS}HRDUWSDUi
HVSpFLHRXGHJUDXVXSHULRU JUDIR~QLFR,,GD/HL2UJkQLFD

,, VyGHYHVHUUHYRJDGDDOHLRXTXDOTXHUGHVHXVGLVSRVL †~QLFR 2GLVSRVWRQHVWHDUWLJRDSOLFDVHDLQGDTXHDOHLQmR


WLYRV TXDQGR KRXYHU FRPSOHWD LQFRPSDWLELOLGDGH WHQKDVLGRSURPXOJDGDSHOR*RYHUQDGRU
MXUtGLFDHQWUHDOHLQRYDHDOHLDQWHULRU
&DStWXOR,9
,,, GHYH VHU HYLWDGD D UHYRJDomR HQWUH OHLV TXHYHUVHP '$6$/7(5$d¯(6
VREUHPDWpULDVGLYHUVDV
6HomR,
,9 OHLTXHHVWDEHOHoDQRUPDVGHFDUiWHUJHUDOQmRGHYH 'DV'LVSRVLo}HV&RPXQV
UHYRJDUOHLTXHHVWDEHOHoDQRUPDVGHFDUiWHUHVSHFLDO
QHPHVWDGHYHUHYRJDUDTXHOD $UW$OWHUDomRpDPRGLILFDomRGHGLVSRVLWLYRGHOHL

9 VyVHUHYRJDWH[WRLQWHJUDOGHDUWLJRSDUiJUDIRLQFLVR †~QLFR $DOWHUDomRRFRUUHSRU


DOtQHDRXQ~PHUR
, VXSUHVVmR
†ž $UHYRJDomRGDXQLGDGHGHDUWLFXODomRFRPSOHPHQWDGD ,, DFUpVFLPR
DWLQJHDVXQLGDGHVGHDUWLFXODomRTXHDFRPSOHPHQ ,,, QRYDUHGDomR
WDP
$UW$VDOWHUDo}HVWrPSRUILQDOLGDGH
†ž eYHGDGDDUHYRJDomRGHGLVSRVLWLYRGHOHLVHDUHYRJD
omRDFDUUHWDUSUHMXt]RDRVGLVSRVLWLYRVUHPDQHVFHQWHV , H[SXUJDUGRVLVWHPDMXUtGLFRGLVSRVLWLYRTXHVHWRUQRX
LQFRQYHQLHQWHRXLQRSRUWXQR
$UW $UHYRJDomRH[SUHVVDGHGLVSRVLWLYRLQFRUSRUDGRSRU
UHPLVVmRVyDWLQJHDOHLDTXHVHUHIHULU ,, FRPSOHPHQWDUODFXQDVGHL[DGDVSHODOHLDQWHULRU
&kPDUD/HJLVODWLYDGR
  'LVWULWR)HGHUDO 'LU&RQVWLWXFLRQDO 

,,, FRUULJLUGLVWRUo}HVQRVLVWHPDMXUtGLFR ,,, GH DXPHQWDU RX GLPLQXLU TXDQWLWDWLYRV IL[DGRVSRU


H[SUHVV}HVQXPpULFDV
,9 DSULPRUDUDOHLH[LVWHQWHHDGHTXiODjVQRYDVH[LJrQFL
DVGDVRFLHGDGH ,9 GHDWHQGHUDRGLVSRVWRQRDUWGHVWD/HL&RPSOH
PHQWDU
†~QLFR $VDOWHUDo}HVGHYHPJXDUGDUFRHUrQFLDFRPRVGLVSRVL
WLYRVQmRDOWHUDGRVEHPFRPRFRPDVLVWHPDWL]DomR $UW$SOLFDVHDHVWDVHomRRHVWDWXtGRQRVDUWVD
TXHDOHLDOWHUDGDDGRWRX GHVWD/HL&RPSOHPHQWDU

$UW$ OHL FXMD ILQDOLGDGH SULQFLSDO IRU D GH DOWHUDURXWUD &DStWXOR9


LQFOXLUiHPVXDHPHQWDDHPHQWDGDOHLDOWHUDGD '$&2162/,'$d®2'$6/(,6

$UW$OHLDOWHUDGRUDREHGHFHUijV QRUPDV GHDUWLFXODomR 6HomR,


HVWDWXtGDVSRUHVWD/HL&RPSOHPHQWDUHLQGLFDUiHP 'DV'LVSRVLo}HV3UHOLPLQDUHV
VHXVGLVSRVLWLYRVDDOWHUDomRRFRUULGD
$UW$FRQVROLGDomRGDVOHLVWHPSRUILQDOLGDGHWRUQDUVXD
$UW6HPSUHTXHIRUFRQVLGHUiYHODDOWHUDomRGDOHLDQWHUL FRQVXOWD DFHVVtYHO DRV FLGDGmRV QRV WHUPRV GR TXH
RUVHUiHODERUDGDOHLQRYDGLVFLSOLQDQGRLQWHJUDOPHQWH GLVS}HRDUW;GD/HL2UJkQLFD
DPDWpULDDQWHULRUPHQWHWUDWDGD
$UW$FRQVROLGDomRGDVOHLVVHUiIHLWD
6HomR,,
'DV6XSUHVV}HV , SHODLQVHUomRQRWH[WRGDOHLGDVDOWHUDo}HVRFRUULGDV

$UW$VXSUHVVmRGHGLVSRVLWLYRGHOHLRFRUUHFRPDUHYRJD ,, SHODFRPSLODomRQXPVyWH[WRHGHPRGRVLVWHPiWLFR


omR GHWRGDVDVOHLVHVSDUVDVVREUHDPHVPDPDWpULD

†~QLFR 3DUD VXSULPLU GLVSRVLWLYR GH OHLREHGHFHUVHiDR ,,, SHODFRQVDJUDomRGHVLJQLILFDGRRXFRQFHLWRDWULEXtGR


GLVSRVWRQDVXEVHomR9,,GRFDStWXORDQWHULRU DGHWHUPLQDGRWHUPR

6HomR,,, †ž &DGDHVSpFLHGHOHLWHUiFRQVROLGDomRSUySULD


'RV$FUpVFLPRV
†ž 4XDQGRGDFRQVROLGDomRVHUmRHOLPLQDGDVDVLPSURSUL
$UW6ypSHUPLWLGRRDFUpVFLPRGHWH[WRLQWHJUDOGHSDUi HGDGHVHYLGHQWHVGHOLQJXDJHPEHPFRPRDVLPSUHFL
JUDIRLQFLVRDOtQHDRXQ~PHUR V}HV WHUPLQROyJLFDV H DWXDOL]DGD D GHQRPLQDomR GH
yUJmRORJUDGRXURRXFDUJRS~EOLFRTXHVRIUHUDOWHUD
$UWe YHGDGD D UHQXPHUDomR GH DUWLJRV HP YLUWXGHGH omR
DOWHUDomR
$UW 2GLVSRVWRQHVWHFDStWXORQmRpREULJDWyULRSDUDDVOHLV
†ž 2GLVSRVWRQHVWHDUWLJRQmRVHDSOLFDjUHQXPHUDomR
GRVDUWLJRVTXHFRQWHQKDPDFOiXVXODGHYLJrQFLDHD , TXHDSURYHPRXDOWHUHPRRUoDPHQWRDQXDO
FOiXVXODUHYRJDWyULD
,, TXH DSURYHP R SODQR SOXULDQXDO RX DVGLUHWUL]HV
†ž $PDWpULDDVHUGLVFLSOLQDGDSRUDUWLJRDTXHQmRVH RUoDPHQWiULDV
DSOLFDURGLVSRVWRQRSDUiJUDIRDQWHULRUVHUiWUDWDGDQD
SUySULDOHLDOWHUDGRUD ,,, TXHWHQKDPFRPRREMHWRDSHQDVRUHDMXVWHVDODULDOGH
DXWRULGDGHVRXVHUYLGRUHVS~EOLFRVGR'LVWULWR)HGHUDO
$UW $OHLLQGLFDUiRQ~PHURGRDUWLJRTXHFRQWpPGLVSRVLWL
YRDFUHVFLGRHVHQGRRFDVRRPRGRGHUHQXPHUDomR 6HomR,,
GRVMiH[LVWHQWHV 'D&RQVROLGDomRSRU,QVHUomR

$UW2GLVSRVLWLYRDFUHVFLGRVHUiGHVWDFDGRQRWH[WRGDOHL $UW$ OHL DOWHUDGD VHUi UHSXEOLFDGD FRP DVDOWHUDo}HV


DOWHUDGRUDGRGLVSRVLWLYRTXHGHWHUPLQDURDFUpVFLPR LQVHULGDVHPVHXWH[WR
HYLUiHQWUHDVSDV
†ž $ FRQVROLGDomR D TXH VH UHIHUH HVWHDUWLJRRFRUUHUi
†~QLFR 6HUmR DEHUWDV QRYDV DVSDV SDUDFDGDGLVSRVLWLYR DQWHV GR HQFHUUDPHQWR GH FDGD VHVVmR OHJLVODWLYD H
DFUHVFLGRHRIHFKDPHQWRVyVHGDUiQR~OWLPRGHOHV DEUDQJHUiDVOHLVDOWHUDGDVQRSHUtRGR

$UW$OHLTXHPDQGDUDFUHVFHUGLVSRVLWLYRVHUiVHPSUHGD †ž $FRQVROLGDomRSRULQVHUomRpGDFRPSHWrQFLDSULYDWLYD


PHVPDHVSpFLHGDTXHWLYHUGLVSRVLWLYRDFUHVFLGR GD&kPDUD/HJLVODWLYDQRVWHUPRVGRTXHGLVS}HRDUW
;GD/HL2UJkQLFD
6HomR,9
'D1RYD5HGDomR †ž 6HUiGHWHUPLQDGD

$UW'iVH QRYD UHGDomR D WH[WR GH GLVSRVLWLYRTXDQGR , SRUGHFUHWROHJLVODWLYRDFRQVROLGDomR


KRXYHUQHFHVVLGDGH
D GD/HL2UJkQLFDFRPVXDVHPHQGDV
, GHVXSULPLURXDFUHVFHUYRFiEXORRXH[SUHVVmR E GDVOHLVFRPSOHPHQWDUHVFRPVXDVDOWHUDo}HV
F GDVOHLVRUGLQiULDVFRPVXDVDOWHUDo}HV
,, GHDPSOLDURXUHVWULQJLUDDEUDQJrQFLDGDQRUPD G GRVGHFUHWRVOHJLVODWLYRVFRPVXDVDOWHUDo}HV
 'LU&RQVWLWXFLRQDO &kPDUD/HJLVODWLYDGR'LVWULWR)HGHUDO


,, SRUUHVROXomRDFRQVROLGDomRGDVUHVROXo}HVTXHIRUHP $UW $FRQVROLGDomRGDVOHLVSRUFRPSLODomRFRQVLGHUDVHOHL


DOWHUDGDV QRYDSDUDWRGRVRVHIHLWRVOHJDLV

$UW3DUDDSXEOLFDomRGDVOHLVFRQVROLGDGDVVHUmRREVHUYD $UW$RVGLVSRVLWLYRVLQFRUSRUDGRVSRUUHPLVVmRDSOLFDVH
GDVDVQRUPDVVHJXLQWHV QRTXHFRXEHURGLVSRVWRQHVWDVHomR

, QmRKDYHUiPRGLILFDomRQDQXPHUDomRGDWDHYLJrQ 6HomR,9


FLD 'D&RQVDJUDomRGH6LJQLILFDGRV&RQFHLWRVH)RUPDV

,, VyVHUmRLQVHULGDVDVDOWHUDo}HVDSURYDGDVDWpDGDWD $UW$&kPDUD/HJLVODWLYDRUJDQL]DUi


DQWHULRUjGDGHWHUPLQDomRGHFRQVROLGDUDVOHLV
, JORVViULRGHH[SUHVV}HVHWHUPRVMXUtGLFRVXVDGRVQDV
,,, DR ODGRGRGLVSRVLWLYRDOWHUDGRRXORJRDEDL[RGHOH OHLVGR'LVWULWR)HGHUDO
VHUiLQGLFDGR
,, PDQXDOGHUHGDomRGDVOHLV
D RWLSRGHDOWHUDomRRFRUULGD
E RQ~PHURHDGDWDGDOHLDOWHUDGRUD †ž 2JORVViULRDTXHVHUHIHUHHVWHDUWLJRVHUiDWXDOL]DGR
F R GLVSRVLWLYR GD OHL DOWHUDGRUD TXH GHWHUPLQRX D DRPHQRVXPDYH]HPFDGDOHJLVODWXUD
DOWHUDomR
†ž 2PDQXDOGHUHGDomRGDVOHLVWHUiSRUEDVHRVSDGU}HV
†~QLFR $SXEOLFDomRGDVOHLVFRQVROLGDGDVREHGHFHUiQRTXH XWLOL]DGRVQD&RQVWLWXLomR)HGHUDOQD/HL2UJkQLFDH
FRXEHUjVQRUPDVHVWDWXtGDVQR&DStWXOR,,6HomR9,, QHVWD/HL&RPSOHPHQWDU
GHVWD/HL&RPSOHPHQWDUHVHUiHIHWXDGDDWpR~OWLPR
GLD~WLOGRDQRFLYLO †ž 1R PDQXDO GH UHGDomR GDV OHLV VHUmRLQFOXtGRVRV
SULQFLSDLVFDVRVGH
$UW3HUPDQHFHPFRPVXDQXPHUDomRRULJLQDORVGLVSRVLWL
YRVVXSULPLGRVHPYLUWXGHGH , RUWRIRQLD
,, RUWRJUDILD
, YHWR ,,, DFHQWXDomRJUiILFD
,9 IOH[mRYRFDEXODU
,, UHYRJDomR 9 UHJrQFLD
9, FRQFRUGkQFLD
,,, VXVSHQVmR SRU LQFRQVWLWXFLRQDOLGDGH RXLOHJDOLGDGH 9,, FRORFDomRGRVWHUPRVQDRUDomR
QRVWHUPRVGRDUW;,;GD/HL2UJkQLFD 9,,, SRQWXDomR
,; HVWLOtVWLFDGDVOHLV
†ž 6HUiFRQVLJQDGDDRILQDOGHFDGDGLVSRVLWLYRVXSULPLGR
DVXDVLWXDomRID]HQGRVHUHIHUrQFLDjOHLGHWHUPLQD &DStWXOR9,
GRUDGDVXSUHVVmR '$6',6326,d¯(675$16,7Ð5,$6
$UW 9(7$'2
†ž 1RFDVRGRLQFLVR,,,GHVWHDUWLJRVHUiIHLWDUHIHUrQFLD
DRGHFUHWROHJLVODWLYRRXUHVROXomRTXHGHWHUPLQDUD $UW'HQWURGHFHQWRHYLQWHGLDVGDSXEOLFDomRGHVWD/HL
VXVSHQVmR &RPSOHPHQWDU D &kPDUD /HJLVODWLYD FRQVROLGDUi DV
OHLVMiH[LVWHQWHV
$UW5HVSHLWDGDV DV QRUPDV GHVWD VHomR D OHLDOWHUDGRUD
SRGHUiGHWHUPLQDUTXHVHSXEOLTXH HPDQH[RDOHL †~QLFR 4XDQGRIRUHPFRQVROLGDGDVDVOHLVVHUmRDGDSWDGDVjV
DOWHUDGDFRPDLQVHUomRGDVDOWHUDo}HV QRUPDV GH DUWLFXODomR H GH DJUXSDPHQWR GH DUWLJRV
SUHYLVWDVQHVWD/HL&RPSOHPHQWDU
6HomR,,,
'D&RQVROLGDomRSRU&RPSLODomR $UWeGHFHQWRHRLWHQWDGLDVRSUD]RSDUDTXHD&kPDUD
/HJLVODWLYDHODERUHRPDQXDOGHUHGDomRHRJORVViULR
$UW$FRQVROLGDomRSRUFRPSLODomRRFRUUHUiSHODUHXQLmR SUHYLVWRVQRDUWGHVWD/HL&RPSOHPHQWDU
HPXPVyWH[WRHGHPRGRVLVWHPiWLFRGHWRGDVDVOHLV
HVSDUVDVVREUHDPHVPDPDWpULD $UW (VWD/HL&RPSOHPHQWDUHQWUDHPYLJRUQDGDWDGHVXDSXEOLFDomR

†~QLFR 2VGLVSRVLWLYRVYHWDGRVUHYRJDGRVRXVXVSHQVRVVHUmR
VXSULPLGRVRXUHDSURYHLWDGRV

$UW$ FRQVROLGDomRGHTXHWUDWDHVWDVHomRVHUiIHLWDQD 5HJLPHQWR,QWHUQR


~OWLPDVHomROHJLVODWLYDGHFDGDOHJLVODWXUD GD
&kPDUD/HJLVODWLYDGR'LVWULWR)HGHUDO
$UW$FRQVROLGDomRGDVOHLVSRUFRPSLODomRVHUiIHLWDSRU
OHLGDPHVPDHVSpFLHGDVFRQVROLGDGDV 5HVROXomRQžFRQVROLGDGDSHOD5HVROXomRQƒ

$UW6HUi PDQWLGD WDQWR TXDQWR SRVVtYHO ILGHOLGDGHDR


WH[WRGDVOHLVFRQVROLGDGDV &DUR&DQGLGDWRYRFrHQFRQWUDUiRFRQWH~GRFRPSOHWRGR
5HJLPHQWR,QWHUQRGD&kPDUD/HJLVODWLYDGR'LVWULWR)HGHUDOQR
†~QLFR +DYHQGRGLYHUJrQFLDHQWUHDVOHLVQRPRGRGHGLVSRU VLWHGD(GLWRUD6ROXomRFXMRHQGHUHoRQDLQWHUQHWp
R PHVPR DVVXQWR GHFLGLUVHi SHOR TXH IRU PDLV
DGHTXDGRjFRQVHFXomRGRVREMHWLYRVGDOHL ZZZHGLWRUDVROXFDRFRPEU
7
7ULEXQDOGH-XVWLoD
  3(UQDPEXFR 'LU&RQVWLWXFLRQDO 

7(67(6 

 $5HS~EOLFD)HGHUDWLYDGR%UDVLOpIRUPDGD  6H
6HXPD
XPDSHVVRD
SHVVRDYLHU
YLHUDVHU
VHUREULJDGD
REULJDGDDID]HU
ID]HU RX
RXSDUDU
SDUDUGH
GHID
D SHORV(VWDGRV0HPEURV ]HU DOJXPDFRLVDTXHQmRHVWiSUHYLVWDHPOHLKDYHUi
E SHOR'LVWULWR)HGHUDOHVHXV7HUULWyULRV D FULPHGHFRQVWUDQJLPHQWRLOHJDO
F SHODXQLmRLQGLVVRO~YHOGRV(VWDGRVH0XQLFtSLRVHGR E FRQWUDYHQomRSHQDO
'LVWULWR)HGHUDO F LQIUDomRGLVFLSOLQDU
G SHOD XQLmR GLVVRO~YHO GRV (VWDGRV H 0XQLFtSLRV H GR G WRGDVDVDOWHUQDWLYDVHVWmRFRUUHWDV
'LVWULWR)HGHUDO
 eSUHFHLWRFRQVWLWXFLRQDOTXHDOHLQmRSUHMXGLFDUi
 2H[HUFtFLR
H[HUFtFLRGHPRFUiWLFR
GHPRFUiWLFRTXH
TXHGHYH
GHYHSURWHJHU
SURWHJHUDOLEHUGDGH
OLEHUGDGHGH D GLUHLWRDGTXLULGR
FRQYLFomR
FRQYLFomR H[HUFLGD
H[HUFLGD DWUDYpV
DWUDYpV GH
GH SDUWLGRV
SDUWLGRV SROtWLFRVQmRVH
SROtWLFRVQmRVH E DWRMXUtGLFRSHUIHLWR
SRGHQGR OLPLWDU
SRGHQGR OLPLWDU R Q~PHUR GH SDUWLGRV SROtWLFRV p XP F FRLVDMXOJDGD
SULQFtSLRFRQVWLWXFLRQDOGHQRPLQDGR G WRGDVDVDOWHUQDWLYDVHVWmRFRUUHWDV
D SOXUDOLVPRSROtWLFR
E FLGDGDQLD  2FULPHGHWRUWXUDpFRQVLGHUDGR
F VREHUDQLD D LQDILDQoiYHOHLQVXVFHWtYHOGHLQGXOWR
G OLYUHLQLFLDWLYD E LQVXVFHWtYHOGHJUDoDRXDQLVWLD
F LQDILDQoiYHOHLQVXVFHWtYHOGHJUDoDRXDQLVWLD
 2V3RGHUHV/HJLVODWLYR([HFXWLYRH-XGLFLiULRVmR G DILDQoiYHOPDVLQVXVFHWtYHOGHJUDoDHLQGXOWR
D LQGHSHQGHQWHV
E KDUP{QLFRVHQWUHVL  $
$SHQD
SHQDVHUi
VHUiFXPSULGD
FXPSULGDGH
GHDFRUGR
DFRUGRFRP
FRPDQDWXUH]D
QDWXUH]DGR
GRGHOLWR
F LQGHSHQGHQWHVHKDUP{QLFRVHQWUHVL DLGDGHHRVH[RGRDSHQDGRHP
G QGD D HVWDEHOHFLPHQWRIHFKDGR
E FDVDVGHDOEHUJXHQRWXUQR
 $HUUDGLFDomRGDSREUH]DHDPDUJLQDOL]DomRHUHGX]LUDV F HVWDEHOHFLPHQWRVGLVWLQWRV
GHVLJXDOGDGHVVRFLDLVHUHJLRQDLVFRQVWLWXL G TXDOTXHUHVWDEHOHFLPHQWRGHVGHTXHKDMDYDJD
D GHYHUVRFLDO
E REMHWLYRIXQGDPHQWDOGD5HS~EOLFD)HGHUDWLYDGR%UDVLO  2SUHVRWHPRVHXUHVSHLWRjLQWHJULGDGHItVLFDHPRUDO
2SUHVRWHPRVHXUHVSHLWRjLQWHJULGDGHItVLFDHPRUDO
F REMHWLYRGHWRGRVRVSRYRV D DVVHJXUDGRVSHOR'LUHLWRVGR&LGDGmR5HFOXVR
G QGD E DVVHJXUDGRVSHOD&RQVWLWXLomR)HGHUDO
F DVVHJXUDGRVSHOD,JUHMD%DWLVWD
 eWDPEpP
WDPEpPREMHWLYR
REMHWLYRIXQGDPHQWDO
IXQGDPHQWDOGR
GR%UDVLO
%UDVLOSURPRYHU
SURPRYHUREHP G DVVHJXUDGRVSHOR9DWLFDQR
HVWDUGHWRGRVVHPSUHFRQFHLWRGH
D RULJHPLGDGHHVH[R  1LQJXpPVHUiFRQVLGHUDGRFXOSDGRDWp
1LQJXpPVHUiFRQVLGHUDGRFXOSDGRDWp
E UDoDRULJHPVH[RLGDGHFRU D TXHFRQIHVVHRFULPH
F VH[RFRUHUDoD E RWUkQVLWRHPMXOJDGRGHVHQWHQoDSHQDOFRQGHQDWyULD
G FRURULJHPHUDoDLGDGH F DFRQGHQDomRHPSULPHLUDLQVWkQFLD
G DFRQGHQDomRHPVHJXQGDLQVWkQFLD
 6mRWDPEpPFRQVLGHUDGRVSULQFtSLRVLQWHUQDFLRQDLV
D FRQFHVVmRGHDVLORSROtWLFRHUHS~GLRDRWHUURULVPR  2SUHVRVHUiLQIRUPDGRGHVHXVGLUHLWRVHQWUHRVTXDLV
2SUHVRVHUiLQIRUPDGRGHVHXVGLUHLWRVHQWUHRVTXDLV
E GHIHVDGDSD]HLJXDOGDGHHQWUHRV(VWDGRV D SURLELUYLVLWD
F VROXomR SDFtILFD GRV FRQIOLWRV H FRRSHUDomR HQWUH RV E SRGHUIXPDU
SRYRVSDUDRSURJUHVVRGDKXPDQLGDGH F SHUPDQHFHUFDODGR
G WRGDVDVDOWHUQDWLYDVHVWmRFRUUHWDV G SRGHUFRQIHVVDURFULPH

 $
$ DUWž,,&)
 &
& DUWž&)  '
' DUWž;;;9,&) 
 $
$ DUWž9&)  &
& DUWž;/,,,&)
 &
& DUWž&)  &
& DUWž;/9,,,&)
 %
% DUWž,,,&)  %
% DUWž;/,;&)
 %
% DUWž,9&)  %
% DUWž/9,,&)
 '
' DUWž&)  &
& DUWž/;,,,&)
 'LU&RQVWLWXFLRQDO 7ULEXQDOGH-XVWLoD3(UQDPEXFR


 6HUiFRQFHGLGRRSHGLGRGH+DEHDV&RUSXV
6HUiFRQFHGLGRRSHGLGRGH+DEHDV&RUSXV  e
eFRQVLGHUDGR
FRQVLGHUDGRXP
XPGLUHLWR
GLUHLWRGR
GRWUDEDOKDGRU
WUDEDOKDGRURUHFHELPHQWR
UHFHELPHQWRGH
D VHPSUH TXH DOJXpP VRIUHU RX VH DFKDU DPHDoDGR GH VDOiULR
VDOiULR PtQLPR
PtQLPR IL[DGR HP OHL QDFLRQDOPHQWH XQLILFDGR
VRIUHUYLROrQFLD FDSD]
FDSD]GH DWHQGHUDVXDV
GHDWHQGHU VXDVQHFHVVLGDGHV
QHFHVVLGDGHVYLWDLV
YLWDLVEiVLFDV
EiVLFDVHjV
jVGH
E VHPSUH TXH DOJXpP VRIUHU RX VH DFKDU DPHDoDGR GR VXDIDPtOLDFRPR
VRIUHUFRDomRHPVXDOLEHUGDGHGHORFRPRomR D PRUDGLDHDOLPHQWDomR
F TXDQGRKRXYHULOHJDOLGDGHRXDEXVRGHSRGHU E HGXFDomRVD~GHHOD]HU
G WRGDVDVDOWHUQDWLYDVHVWmRFRUUHWDV F YHVWXiULRKLJLHQHWUDQVSRUWHHSUHYLGrQFLDVRFLDO
G WRGDVDVDOWHUQDWLYDVHVWmRFRUUHWDV
 3DUD
3DUD
3DUD SURWHJHU
SURWHJHU GLUHLWR
GLUHLWR OtTXLGR
OtTXLGR H FHUWR
FHUWR QmR
QmR DPSDUDGR
DPSDUDGR SRU
KDEHDVFRUSXV
KDEHDVFRUSXV RX
RX KDEHDVGDWD
KDEHDVGDWD  TXDQGR
TXDQGR R UHVSRQViYHO  6DOYR
6DOYR
6DOYRQHJRFLDomR
QHJRFLDomRFROHWLYD
FROHWLYDDV
DVMRUQDGDV
MRUQDGDVGH
GHVHLVKRUDVVHUmR
VHLVKRUDV
SHODLOHJDOLGDGHRXDEXVRGHSRGHUIRUDXWRULGDGHS~EOLFD
SHODLOHJDOLGDGHRXDEXVRGHSRGHUIRUDXWRULGDGHS~EOLFD
DSOLFDGDVDRWUDEDOKRUHDOL]DGR
RXDJHQWHGHSHVVRDMXUtGLFDQRH[HUFtFLRGHDWULEXLo}HV
RXDJHQWHGHSHVVRDMXUtGLFDQRH[HUFtFLRGHDWULEXLo}HV
D GLXUQDPHQWH
GR3RGHU3~EOLFRFRQFHGHUVHi
E QRWXUQDPHQWH
D PDQGDGRGHVHJXUDQoD
E PDQGDGRGHLQMXQomR F HPWXUQRVLQLQWHUUXSWRVGHUHYH]DPHQWR
F PDQGDWRGHVHJXUDQoD G SRUWDUHID
G PDQGDWRGHLQMXQomR
 $
$&RQVWLWXLomR
&RQVWLWXLomR)HGHUDO
)HGHUDOJDUDQWH
JDUDQWHXP
XPDGLFLRQDO
DGLFLRQDOGH
GHUHPXQHUD
 3DUD
3DUDDVVHJXUDURFRQKHFLPHQWRGHLQIRUPDo}HVUHODWLYDV
3DUDDVVHJXUDURFRQKHFLPHQWRGHLQIRUPDo}HVUHODWLYDV omRSDUD
jSHVVRD
SHVVRDGRLPSHWUDQWHFRQVWDQWHVGHUHJLVWURVRXEDQFRV
GRLPSHWUDQWHFRQVWDQWHVGHUHJLVWURVRXEDQFRV D DVDWLYLGDGHVSHQRVDV
GH
GH GDGRV GH HQWLGDGHV JRYHUQDPHQWDLV RX GH FDUiWHU E DWLYLGDGHVLQVDOXEUHV
S~EOLFRFRQFHGHUVHi F DWLYLGDGHVSHULJRVDV
D PDQGDWRGHVHJXUDQoD G WRGDVDVDOWHUQDWLYDVHVWmRFRUUHWDV
E PDQGDGRGHVHJXUDQoDFROHWLYR
F KDEHDVGDWD  $
$LQWHUIHUrQFLD
LQWHUIHUrQFLDHDLQWHUYHQomR
LQWHUYHQomRQD
QDRUJDQL]DomR
RUJDQL]DomRVLQGLFDO
VLQGLFDOp
G PDQGDGRGHLQMXQomR D REULJDWyULDDR(VWDGR
E YHGDGDDR3RGHU3~EOLFR
 6mR
6mR
6mR  JUDWXLWRV SDUD RV  UHFRQKHFLGDPHQWH SREUHV QD F DXWRUL]DGDDR3RGHU3~EOLFR
IRUPDGDOHL G IDFXOWDWLYDDR(VWDGR
D RUHJLVWURFLYLOGHQDVFLPHQWR
E DFHUWLGmRGHyELWR  &DGD0XQLFtSLRWHUiGLUHLWRDFULDomRGH
&DGD0XQLFtSLRWHUiGLUHLWRDFULDomRGH
F RUHJLVWURFLYLOGHQDVFLPHQWRHDFHUWLGmRGHyELWR D XPDRUJDQL]DomRVLQGLFDOSRUFDWHJRULDSURILVVLRQDORX
G WRGDVDVFHUWLG}HVIHLWDVHP&DUWyULR&LYLO HFRQ{PLFD
E XPD RUJDQL]DomR VLQGLFDO GH FDWHJRULD SURILVVLRQDO H
 6HJXQGR
6HJXQGR
6HJXQGRD&RQVWLWXLomR
&RQVWLWXLomR)HGHUDO WRGRVQR
)HGHUDODWRGRV QRkPELWR
kPELWRMXGLFLDO RXWUDGHFDWHJRULDHFRQ{PLFD
H DGPLQLVWUDWL
DGPLQLVWUDWLYR
YR VmR
VmR DVVHJXUDGRV
DVVHJXUDGRV D UD]RiYHO
UD]RiYHO GXUDomR
GXUDomR GR
F GXDVRUJDQL]Do}HVVLQGLFDLVSDUDFDGDFDWHJRULDSURILVVL
SURFHVVRHRVPHLRVTXHJDUDQWDP
RQDORXHFRQ{PLFD
D  DFHOHULGDGHGHVXDWUDPLWDomR
G TXDQWDVRUJDQL]Do}HVTXLVHUHPMiTXHR3RGHU3~EOLFR
E  DSURFHGrQFLDGDDomR
QmRSRGHLQWHUIHULUQDVXDFRQVWLWXLomR
F  DLPSURFHGrQFLDGDDomR
G  WRGDVDVDOWHUQDWLYDVHVWmRFRUUHWDV
 $GHIHVDGRV
$GHIHVDGRV
$GHIHVDGRVGLUHLWRV
GLUHLWRVHLQWHUHVVHVFROHWLYRVRXLQGLYLGXDLV
 6HJXQGR
6HJXQGR &RQVWLWXLomR)HGHUDO
QRVVDD&RQVWLWXLomR
6HJXQGRQRVVD )HGHUDODVD~GH
VD~GHpFRQVLGH GD
GD FDWHJRULDLQFOXVLYHHPTXHVW}HVMXGLFLDLVRXDGPLQLV
FDWHJRULDLQFOXVLYHHPTXHVW}HVMXGLFLDLVRXDGPLQLV
UDGD WUDWLYDVFDEHDR
D GHYHUVRFLDO D 3RGHU-XGLFLiULR
E GLUHLWRVRFLDO E TXDOTXHUSHVVRD
F REULJDomRVRFLDO F 6LQGLFDWR
G JDUDQWLDLQGLYLGXDO G 3RGHU3~EOLFRHPJHUDO

 2WUDEDOKRSDUDD&RQVWLWXLomR)HGHUDOpFRQVLGHUDGR
2WUDEDOKRSDUDD&RQVWLWXLomR)HGHUDOpFRQVLGHUDGR  4XDQGR
4XDQGR
4XDQGRVH
VHWUDWDU
WUDWDUGH
GHFDWHJRULD
FDWHJRULDSURILVVLRQDO
SURILVVLRQDODFRQWULEXLomR
FRQWULEXLomR
D GHYHUVRFLDO VLQGLFDOVHUi
E GLUHLWRVRFLDO D GHVFRQWDGDVHPHVWUDOPHQWH
F REULJDomRVRFLDO E SDJDQR%DQFRGR%UDVLO
G JDUDQWLDLQGLYLGXDO F UHVJDWDGDSHORHPSUHJDGRTXDQGRGHVSHGLGR
G GHVFRQWDGDHPIROKDGHSDJDPHQWR
 6mR
6mR
6mR GLUHLWRV
GLUHLWRV GRV
GRV WUDEDOKDGRU
WUDEDOKDGRU HV
HV XUEDQRV
XUEDQRV H UXUDLV DOpP GH
UXUDLV DOpP
RXWURV TXH
RXWURV TXH YLVHP j PHOKRULD GH VXD FRQGLomR VRFLDO D  5H]D
5H]D
5H]DD&RQVWLWXLomR
&RQVWLWXLomR)HGHUDO
)HGHUDOTXH
TXHDSDUWLFLSDomR
SDUWLFLSDomRGRV
GRVVLQGLFD
UHODomRGHHPSUHJRSURWHJLGDFRQWUD WRVQDVQHJRFLDo}HVFROHWLYDVGHWUDEDOKRVHUi
D GHVSHGLGDDUELWUiULDRXVHPMXVWDFDXVD D REULJDWyULD
E RKRUiULRIL[RGHWUDEDOKR E IDFXOWDWLYD
F RGLUHLWRGHUHFHELPHQWRGHKRUDVH[WUDV F SHUPLVVLYD
G QGD G WRGDVDVDOWHUQDWLYDVHVWmRFRUUHWDV

 '
' DUWž/;9,,,&)  '
' DUWž,9&)
 $
$ DUWž/;,;&)  &
& DUWž;,9&)
 &
& DUWž/;;,,D&)  '
' DUWž;;,,,&)
 &
& DUWž/;;9,&)  %
% DUWž,&)
 $
$ DUWž/;;9,,,&)  $
$ DUWž,,&)
 %
% DUWž&)  &
& DUWž,,,&)
 %
% DUWž&)  '
' DUWž,9&)
 $
$ DUWž,&)
$  $
$ DUWž9,&)
7
7ULEXQDOGH-XVWLoD
  3(UQDPEXFR 'LU&RQVWLWXFLRQDO 
 )D]HURXSDUWLFLSDUGH*UHYHp
)D]HURXSDUWLFLSDUGH*UHYHp G 2VTXHQDIRUPDGDOHLDGTXLUDPQDFLRQDOLGDGHEUDVLOH
D SURLELGRDRWUDEDOKDGRU LUDH[LJLGDVDRVRULJLQiULRVGHSDtVHVGHOtQJXDSRUWX
E GLUHLWRDVVHJXUDGRDRWUDEDOKDGRU JXHVDDSHQDVUHVLGrQFLDSRUXPDQRLQLQWHUUXSWR
F IDFXOWDWLYRDRHPSUHJDGRU
G GHYHUGDFDWHJRULDFRPRXPWRGR  -DPHV
-DPHV
-DPHV H VXD
VXD HVSRVD
HVSRVD 0DU\
0DU\ VmR
VmR  DPHULFDQRV TXH HVWmR
PRUDQGR
PRUDQGR QR
QR %UDVLO
%UDVLO Ki
Ki GRLV
GRLV DQRV
DQRV DEULUDP
DEULUDP XPD
XPD ORMD GH
 $
$ &RQVWLWXLomRSDUDDVVHJXUDUDRUGHPS~EOLFDGHILQLUi EULQTXHGRV
EULQTXHGRV LPSRUWDGRV HP 6mR 3DXOR 1R PrV TXH YHP
HPOHL 0DU\
0DU\ GDUi
GDUi D OX]DTXLQR%UDVLODXPILOKRGRFDVDO(VWH
OX]DTXLQR%UDVLODXPILOKRGRFDVDO(VWH
D RV VHUYLoRV RX DWLYLGDGHV HVVHQFLDLV H GLVSRUi VREUH R ILOKRVHUiFRQVLGHUDGR
DWHQGLPHQWRGDVQHFHVVLGDGHVLQDGLiYHLVGDFRPXQLGDGH D EUDVLOHLURQDWXUDOL]DGR
E VREUH R DWHQGLPHQWR GDV QHFHVVLGDGHV LQDGLiYHLV GD E HVWUDQJHLUR
FRPXQLGDGH F EUDVLOHLURQDWR
F RVVHUYLoRVRXDWLYLGDGHVHVVHQFLDLVGDFRPXQLGDGH G HVWUDQJHLURQDWXUDOL]DGR
G RVVHUYLoRVRXDWLYLGDGHVQmRHVVrQFLDVHGLVSRUiVREUHR
DWHQGLPHQWRGDVQHFHVVLGDGHVVHFXQGiULDVGDFRPXQLGDGH  8P
8P
8PFDVDO
FDVDOGH
GHDOHPmHV
DOHPmHVIRL
IRLHQYLDGR %UDVLOSDUD
DR%UDVLO
HQYLDGRDR SDUDWUDEDOKDU
WUDEDOKDU
QR &RQVXODGR $OHPmR Ki PDLV GH FLQFR DQRV  GXUDQWH
 2V
2V
2V PpGLFRV
PpGLFRV H IXQFLRQiULRV
IXQFLRQiULRV GD 87, GR KRVSLWDO GDV HVWH
HVWH SHUtRGR
SHUtRGR WLYHUDP
WLYHUDP GRLV
GRLV ILOKRVTXHDTXLQR%UDVLOVmR
ILOKRVTXHDTXLQR%UDVLOVmR
FOtQLFDV
FOtQLFDV TXH
TXH UHVROYHUDP
UHVROYHUDP HQWUDU HP JUHYH SOHLWHDQGR FRQVLGHUDGRV
D EUDVLOHLURVQDWXUDOL]DGR
PDLRUHVVDOiULRVGHL[DQGRIHFKDGDVXDXQLGDGHVHUmR
E HVWUDQJHLURV
D DGYHUWLGRVDGPLQLVWUDWLYDPHQWH
F EUDVLOHLURVQDWR
E SURFXUDGRVSDUDGDUH[SOLFDo}HVVREUHRIDWR
G HVWUDQJHLURVQDWXUDOL]DGR
F UHVSRQVDELOL]DGRV SHORV SUHMXt]RV FDXVDGRV DOpP GH
UHVSRQGHUHPFULPLQDOPHQWHSRUVHXVDWRV
 2V
2V
2VEUDVLOHLURV
EUDVLOHLURV3HGUR
3HGURHVXD
VXD0XOKHU
0XOKHU6tOYLD TXHVH
6tOYLDTXH VHHQFRQWUDYD
G VXVSHQVRVGHVXDVDWLYLGDGHVHVHXVYHQFLPHQWRVGXUDQWH
QRPrVGHJHVWDomRIRUDPHQYLDGRVj,WiOLDSDUDXPD
DJUHYHVHUmRFRQILVFDGRV
PLVVmR
PLVVmR GLSORPiWLFD
GLSORPiWLFD EUDVLOHL
EUDVLOHLUD
UD DSyV
DSyV XPD
XPD VHPDQD
VHPDQD 6tOYLD
HQWURX
HQWURX HP
HP WUDEDOKR
WUDEDOKR GH GHX j OX]
SDUWR H GHX
GH SDUWR OX] D XPD
XPD PHQLQD
 &RPSOHWHDVODFXQDV
&RPSOHWHDVODFXQDV (VVDFULDQoDVHUi
´eBBBBBBBBBBBBBD
´eBBBBBBBBBBBBBDSDUWLFLSDomR
SDUWLFLSDomRGRV
GRVWUDEDOKDGRUHVHHP D EUDVLOHLUDQDWXUDOL]DGD
SUHJDGRUHV
SUHJDGRUHVQRV
QRVFROHJLDGRV
FROHJLDGRVGRV
GRVyUJmRV
yUJmRVS~EOLFRV
S~EOLFRVHP
HPTXH
TXHVHXV E ,WDOLDQD
BBBBBBBBBBBBBBBBSURILVVLRQDLV
BBBBBBBBBBBBBBBBSURILVVLRQDLVRXSUHYLGHQFLiULRVVHMDP F EUDVLOHLUDQDWD
REMHWRGHGLVFXVVmRHBBBBBBBBBBBBB´ G ,WDOLDQDQDWXUDOL]DGD
D GHILQLGDGLUHLWRVFRQVXOWD
E DVVHJXUDGDLQWHUHVVHVVROXomR  %LDQFD
%LDQFD
%LDQFDPRUD
PRUDQD
QD,WiOLDRQGHQDVFHXpILOKDGH
,WiOLDRQGHQDVFHXpILOKDGHSDL
SDLLWDOLDQR
F SURLELGDGHYHUHVH[DPH HPmH
PmHEUDVLOHLUD
EUDVLOHLUDVHJXQGR QRVVD&RQVWLWXLomR
VHJXQGRQRVVD &RQVWLWXLomRQR
QR%UDVLO
%UDVLOHOD
G DVVHJXUDGDLQWHUHVVHVGHOLEHUDomR pFRQVLGHUDGD
D EUDVLOHLUDQDWXUDOL]DGD
 1RV
1RV
1RV FROHJLDGRV
FROHJLDGRV GRV
GRV yUJmRV S~EOLFRV HP TXH LQWHUHVVHV E HVWUDQJHLUD
SURILVVLRQDLV
SURILVVLRQDLVVHMDP
VHMDPREMHWR
REMHWRGH
GHGLVFXVVmR
GLVFXVVmRHGHOLEHUDomR
GHOLEHUDomRVHUi F EUDVLOHLUDQDWD
DVVHJXUDGD G HVWUDQJHLUDQDWXUDOL]DGD
D DSDUWLFLSDomRH[FOXVLYDGR*RYHUQR
E DSDUWLFLSDomRGRVWUDEDOKDGRUHVHHPSUHJDGRUHV  2V
2V
2V GRFXPHQWRVS~EOLFRVHSULYDGRVVHUmRHODERUDGRVQR
GRFXPHQWRVS~EOLFRVHSULYDGRVVHUmRHODERUDGRVQR
F DUHQHJRFLDomRVLQGLFDO LGLRPDRILFLDOGD5HS~EOLFD)HGHUDWLYDGR%UDVLOTXHp
G DSDUWLFLSDomRH[FOXVLYDGRVHPSUHJDGRV D DOtQJXDSRUWXJXHVD
E DOtQJXDEUDVLOHLUD
 $
$&RQVWLWXLomR
&RQVWLWXLomRDVVHJXUD
DVVHJXUDDHOHLomR
HOHLomRGH
GHXP
XPUHSUHVHQWDQWH
UHSUHVHQWDQWHGH F DOtQJXDQDFLRQDO
HPSUHJDGRV
HPSUHJDGRV  FRP D ILQDOLGDGH H[FOXVLYD GH SURPRYHU R G TXDOTXHU LGLRPD TXH VHMD HQWHQGLGR SRU TXHP XVH R
HQWHQGLPHQWR
HQWHQGLPHQWRGLUHWR
GLUHWRFRP
FRPRV
RVHPSUHJDGRUHV
HPSUHJDGRUHVQDV
QDVHPSUHVDV GRFXPHQWR
D FRPPDLVGHGX]HQWRVHPSUHJDGRV
E FRPPHQRVGHGX]HQWRVHPSUHJDGRV  (P
(P
(P WRGDV
WRGDV DV
DV UHSDUWLo}HV
UHSDUWLo}HV S~EOLFDV
S~EOLFDV p KDVWHDGD D %DQGHLUD
F FRPPDLVGHFHPHPSUHJDGRV %UDVLOHLUDSRUTXHHODpFRQVLGHUD
G FRPPDLVGHFLQTXHQWDHPSUHJDGRV D RUJXOKRQDFLRQDO
E LGHQWLGDGHGRSRYR
 6mREUDVLOHLURVQDWRV
6mREUDVLOHLURVQDWRV F VtPERORGD5HS~EOLFD)HGHUDWLYDGR%UDVLO
D 2VHVWUDQJHLURVGHTXDOTXHUQDFLRQDOLGDGHUHVLGHQWHVQD G PDUFRQDFLRQDO
5HS~EOLFD )HGHUDWLYD GR %UDVLO Ki PDLV GH  DQRV
LQLQWHUUXSWRV H VHP FRQGHQDomR SHQDO GHVGH TXH  (P
(P VROHQLGDGHRQGH
(PVROHQLGDGH VHHQFRQWUD
RQGHVH HQFRQWUDR3UHVLGHQWH
3UHVLGHQWHGD
GD5HS~EOL
UHTXHLUDPDQDFLRQDOLGDGHEUDVLOHLUD FD
FDFRVWXPDVH
FRVWXPDVHKDVWHDU
KDVWHDUDEDQGHLUD
EDQGHLUDQDFLRQDO
QDFLRQDOHFDQWDU
FDQWDURKLQR
E 2VQDVFLGRVQRHVWUDQJHLURFXMRVDYyVVHMDPEUDVLOHLURV QDFLRQDOTXHVmRFRQVLGHUDGRV
F 2VQDVFLGRVQD5HS~EOLFD)HGHUDWLYDGR%UDVLODLQGDTXH D RUJXOKRQDFLRQDO
GH SDLV HVWUDQJHLURV GHVGH TXH HVWHV QmR HVWHMDP D E LGHQWLGDGHGRSRYR
VHUYLoRGHVHXSDtV F PDUFRQDFLRQDO
G VtPERORGD5HS~EOLFD)HGHUDWLYDGR%UDVLO

 %
% DUWž&)  &
& DUW,D&)
 $
$ DUWž†ž&)  %
% DUW,D&)
 &
& DUWž†ž&)  &
& DUW,E&)
 '
' DUW&)  %
% DUW,E&)
 %
% DUW&)  $
$ DUW&)
 $
$ DUW&)  &
& DUW†ž&)
 &
& DUW,D&)  '
' DUW†ž&)
 'LU&RQVWLWXFLRQDO 7ULEXQDOGH-XVWLoD3(UQDPEXFR


 $VDUPDVHR
$VDUPDVHR
$VDUPDVHRVHOR
VHORQDFLRQDOVmRFRQVLGHUDGRVVtPERORVGD  $LQYHVWLGXUDHPFDUJRRXHPSUHJRS~EOLFRGHSHQGHGH
$LQYHVWLGXUDHPFDUJRRXHPSUHJRS~EOLFRGHSHQGHGH
5HS~EOLFD)HGHUDWLYDGR%UDVLOFRQVDJUDGRVSHOD DSURYDomR
DSURYDomR SUpYLD
SUpYLD HP
HP FRQFXUVR
FRQFXUVR S~EOLFR
S~EOLFR GH SURYDV RX GH
GH SURYDV
D DXWRULGDGHS~EOLFD SURYDV
SURYDVHWtWXORV
WtWXORVGH
GHDFRUGR
DFRUGRFRP
FRPDQDWXUH]D
QDWXUH]DHDFRPSOH[LGD
E &RQVWLWXLomR)HGHUDO GH
GHGR
GRFDUJR
FDUJRRX HPSUHJRQD
RXHPSUHJR QDIRUPD
IRUPDSUHYLVWD
SUHYLVWDHP
HPOHL
OHLUHVVDO
F DXWRULGDGHMXGLFLiULD YDGDV
YDGDV
G &kPDUD)HGHUDO D DVQRPHDo}HVSDUDFDUJRHIHWLYR
E DVQRPHDo}HVSDUDFDUJRHPFRPLVVmRGHFODUDGRHPOHL
 $
$ &RQVWLWXLomR
&RQVWLWXLomR WDPEpP
WDPEpP DVVHJXUD
DVVHJXUD DRV (VWDGRV 'LVWULWR GHOLYUHQRPHDomRHH[RQHUDomR
)HGHUDOHDRV0XQLFtSLRVSRVVXtUHP F DVQRPHDo}HVSDUDFDUJRGHFRQILDQoD
D DUPDVPLOLWDUHV G DVQRPHDo}HVIHLWDVSHOR3UHVLGHQWHGD5HS~EOLFD
E VHORVS~EOLFRV
F VtPERORVSUySULRV  eYHGDGDDDFXPXODomRUHPXQHUDGDGHFDUJRVS~EOLFRV
eYHGDGDDDFXPXODomRUHPXQHUDGDGHFDUJRVS~EOLFRV
G UHVHUYDVQDFLRQDLV H[FHWRTXDQGRKRXYHUFRPSDWLELOLGDGHGHKRUiULRV
D  DGHGRLVFDUJRVGHSURIHVVRU
 $
$ &RQVWLWXLomR
&RQVWLWXLomR WDPEpP
WDPEpP DVVHJXUD
DVVHJXUD DRV
DRV (VWDGRV
(VWDGRV 'LVWULWR E  D GH XP FDUJR GH SURIHVVRU FRP RXWUR WpFQLFRRX
)HGHUDOHDRV0XQLFtSLRVSRVVXtUHP FLHQWtILFR
D DUPDVPLOLWDUHV F  DGHGRLVFDUJRVRXHPSUHJRVSULYDWLYRVGHSURILVVLRQDLV
E VHORVS~EOLFRV GHVD~GHFRPSURILVV}HVUHJXODPHQWDGDV
F VtPERORVSUySULRV G WRGDVDVDOWHUQDWLYDVHVWmRFRUUHWDV
G UHVHUYDVQDFLRQDLV
 1R
1R
1R FDVR
FDVR GH
GH DIDVWDPHQWR
DIDVWDPHQWR RV
RV YDORUHV
YDORUHV VHUmR GHWHUPLQDGRV
 $
$ TXDOLGDGH
TXDOLGDGH GH
GH HOHLWRU
HOHLWRU SRVWD
SRVWD DR
DR DOFDQFH
DOFDQFH GHWRGRFLGDGmR
GHWRGRFLGDGmR FRPRVHQRH[HUFtFLRHVWLYHVVHRVHUYLGRUSDUDHIHLWRGH
FLYLOPHQWHFDSD]pFRQVLGHUDGR D EHQHItFLRVDODULDO
D VREHUDQLD E EHQHItFLRSUHYLGHQFLiULR
E LJXDOGDGH F SURPRomRSRUPHUHFLPHQWR
F UHIHUHQGR G SURPRomRSRUWHPSRGHVHUYLoR
G VXIUiJLRXQLYHUVDO H QGD

 $
$ 5HFXVD
5HFXVD GH
GH Ip
Ip S~EOLFD
S~EOLFD HP
HP GRFXPHQWRV
GRFXPHQWRV S~EOLFRV p  2
2 PHPEUR
PHPEUR GH
GH 3RGHU
3RGHU R GHWHQWRU
GHWHQWRU GH
GH PDQGDWR
PDQGDWR HOHWLYR
HOHWLYR RV
YHGDGR 0LQLVWURV
0LQLVWURVGH
GH(VWDGR
(VWDGRHRV
RV6HFUHWiULRV
6HFUHWiULRV(VWDGXDLVH0XQLFLSDLV
(VWDGXDLVH0XQLFLSDLV
D DR'LVWULWR)HGHUDO VHUmRUHPXQHUDGRVH[FOXVLYDPHQWHSRU
E j8QLmRHDRV(VWDGRV D VXEVtGLRIL[DGRHPGXDVSDUFHODVYHGDGRRDFUpVFLPRGH
F DRV0XQLFtSLRV TXDOTXHUJUDWLILFDomR
G DWRGRVDFLPDFLWDGRV E VXEVtGLRIL[DGRHPXPDSDUFHODSHUPLWLGRRDFUpVFLPR
GH TXDOTXHU JUDWLILFDomR DERQR SUrPLR YHUED GH
 3DU
3DUD
3DUD D &RQVWLWXLomR )HGHUDO RV 7HUUHQRV GH PDULQKD RV UHSUHVHQWDomRRXRXWUDHVSpFLHUHPXQHUDWyULD
UHFXUVRVPLQHUDLVHRPDUWHUULWRULDOVmRFRQVLGHUDGRV F VXEVtGLRIL[DGRHPSDUFHOD~QLFDYHGDGRRDFUpVFLPRGH
D EHQVGXUiYHLV TXDOTXHUJUDWLILFDomRDGLFLRQDODERQRSUrPLRYHUEDGH
E EHQVGD8QLmR UHSUHVHQWDomRRXRXWUDHVSpFLHUHPXQHUDWyULD
F EHQVLQGLYLVtYHLVGD8QLmR G QGD
G EHQVGR(VWDGRRQGHVHHQFRQWUDP
 6HJXQGR
6HJXQGR
6HJXQGR DV
DV QRYDV
QRYDV UHJUDV
UHJUDV SDUD
SDUD VHUYLGRUHV
VHUYLGRUHV S~EOLFRV
S~EOLFRV TXH
 ([HFXWDURVVHUYLoRVGHSROtFLDPDUtWLPDDHURSRUWXiULDH
([HFXWDURVVHUYLoRVGHSROtFLDPDUtWLPDDHURSRUWXiULDH RFXSDUmR
RFXSDUmR FDUJRV
FDUJRV HIHWLYRV
HIHWLYRV DSyV
DSyV D SXEOLFDomR
SXEOLFDomR GD
GD HPHQGD
GHIURQWHLUDVFRPSHWH FRQVWLWXFLRQDO
FRQVWLWXFLRQDORV
RVSURIHVVRUHV
SURIHVVRUHVWHUmR
WHUmRRWHPSR
WHPSRGH
GHFRQWULEXL
D DR(VWDGR omRHDVH[LJrQFLDVGHLGDGHUHGX]LGDVHP
E D8QLmR D  DQRV HP UHODomR DRV GHPDLV VHUYLGRUHV SDUD RV
F DR'LVWULWR)HGHUDO SURIHVVRUHV TXH FRPSURYHP H[FOXVLYDPHQWH WHPSR GH
F DRV0XQLFtSLRV HIHWLYRH[HUFtFLRGDVIXQo}HVGHPDJLVWpULRQDHGXFDomR
LQIDQWLOHGRHQVLQRIXQGDPHQWDOHPpGLR
 /HJLVODU
/HJLVODU
/HJLVODUVREUH
VREUHQRUPDV
QRUPDVJHUDLV
JHUDLVGH
GHOLFLWDomR
OLFLWDomRHFRQWUDWDomR
FRQWUDWDomRHP E DQRVHPUHODomRDRVGHPDLVVHUYLGRUHVSDUDRVSURIHV
WRGDV
WRGDV DV
DV PRGDOLGDGHV
PRGDOLGDGHV SDUD
SDUD DV
DV DGPLQLVWUDo}HV
DGPLQLVWUDo}HV S~EOLFDV VRUHVTXHFRPSURYHPH[FOXVLYDPHQWHWHPSRGHHIHWLYR
GLUHWDV
GLUHWDV DXWiUTXLFDV
DXWiUTXLFDV H IXQGDFLRQDLV
IXQGDFLRQDLV GD
GD 8QLmR
8QLmR (VWDGRV
(VWDGRV H[HUFtFLRGDVIXQo}HVGHPDJLVWpULRQDHGXFDomRLQIDQWLO
'LVWULWR)HGHUDOH0XQLFtSLRVFRPSHWH HGRHQVLQRIXQGDPHQWDOHPpGLR
D D8QLmR F DQRVHPUHODomRDRVGHPDLVVHUYLGRUHVSDUDRVSURIHV
E DRV(VWDGRV VRUHVTXHFRPSURYHPH[FOXVLYDPHQWHWHPSRGHHIHWLYR
F DR'LVWULWR)HGHUDO H[HUFtFLRGDVIXQo}HVGHPDJLVWpULRQDHGXFDomRLQIDQWLO
G WRGDVDVDOWHUQDWLYDVHVWmRFRUUHWDV HGRHQVLQRIXQGDPHQWDOHPpGLR
G DQRVHPUHODomRDRVGHPDLVVHUYLGRUHVSDUDRVSURIHV
VRUHVTXHFRPSURYHPH[FOXVLYDPHQWHWHPSRGHHIHWLYR
H[HUFtFLRGDVIXQo}HVGHPDJLVWpULRQDHGXFDomRLQIDQWLO
HGRHQVLQRIXQGDPHQWDOHPpGLR
 %
% DUW†ž&)
 &
& DUW†ž&)
 &
& DUW†ž&)
 '
' DUW&)  %
% DUW,,&)
 '
' DUW,,&)  '
' DUW;9,&)
 %
% DUW&)  %
% DUW9&)
 %
% DUW;;,,&)  &
& DUW†ž&)
 $
$ DUW;;9,,&)  '
' DUW†ž&)
7
7ULEXQDOGH-XVWLoD
 3(UQDPEXFR 'LU&RQVWLWXFLRQDO 
 $R
$R
$R VHUYLGRU
VHUYLGRU RFXSDQWH
RFXSDQWH H[FOXVLYDPHQW
H[FOXVLYDPHQWH
H GH
GH FDUJR
FDUJR HP F  DVVHJXUDUDRFLGDGmRRH[HUFtFLRGRVGLUHLWRVGHLQLFLDWLYD
FRPLVVmR
FRPLVVmRGHFODUDGR
GHFODUDGRHP
HPOHL
OHLGH
GHOLYUH
OLYUHQRPHDomR
QRPHDomRHH[RQHUDomR TXHOKHFRXEHUHPUHODWLYRVDRFRQWUROHGDOHJDOLGDGHH
EHP FRPR
EHP FRPR GH
GH RXWUR FDUJR WHPSRUiULR RX GH HPSUHJR OHJLWLPLGDGHGRVDWRVGR3RGHU3~EOLFRHGDHILFiFLDGRV
S~EOLFRDSOLFDVHR VHUYLoRVS~EOLFRV
D UHJLPHJHUDOGHSUHYLGrQFLDVRFLDO G  WRGDVDVDOWHUQDWLYDVHVWmRFRUUHWDV
E UHJLPH~QLFRGHSUHYLGrQFLDVRFLDO
F UHJLPHJOREDOGHSUHYLGrQFLDVRFLDO  6HJXQGR
6HJXQGR
6HJXQGR D /HL
/HL 2UJkQLFD
2UJkQLFD GR
GR 'LVWULWR
'LVWULWR )HGHUDO
)HGHUDO HODERUDU
HODERUDU H
G UHJLPHLQGLYLGXDOGHSUHYLGrQFLDSULYDGD H[HFXWDU
H[HFXWDUR3ODQR
3ODQR'LUHWRU
'LUHWRUGH
GH2UGHQDPHQWR
2UGHQDPHQWR7HUULWRULDO
7HUULWRULDOD/HL
GH 8VRH2FXSDomRGR6ROR
GH8VRH2FXSDomRGR 6RORH3ODQRV
3ODQRVGH
GH'HVHQYROYLPHQWR
 6mR
6mR
6mR HVWiYHLV
HVWiYHLV RV
RV VHUYLGRUHV
VHUYLGRUHV QRPHDGRV
QRPHDGRV HPYLUWXGHGHFRQ
HPYLUWXGHGHFRQ /RFDO
/RFDO SDUD
SDUD SURPRYHU
SURPRYHU DGHTXDGR
DGHTXDGR RUGHQDPHQWRWHUULWRULDO
RUGHQDPHQWRWHUULWRULDO
FXUVRS~EOLFRDSyV LQWHJUDGR
LQWHJUDGRDRV
DRVYDORUHV
YDORUHVDPELHQWDLV
DPELHQWDLVPHGLDQWH
PHGLDQWHSODQHMDPHQWR
D  XPDQR H FRQWUROH
FRQWUROH GR
GR XVR
XVR SDUFHODPHQ
SDUFHODPHQ WR
WR H RFXSDomR
RFXSDomR GR
GR VROR
E  PHVHV XUEDQRFRPSHWH
F  GRLVDQRVGHHIHWLYRH[HUFtFLR D  SULYDWLYDPHQWHDR'LVWULWR)HGHUDO
G  WUrVDQRVGHHIHWLYRH[HUFtFLR E  SULYDWLYDPHQWHDR3UHVLGHQWHGD5HS~EOLFD
F  SULYDWLYDPHQWHDR0LQLVWURGH(VWDGRGR'LVWULWR)HGHUDO
 26HUYLGRUHVWiYHOVySHUGHUiRFDUJRHPYLUWXGHGH
26HUYLGRUHVWiYHOVySHUGHUiRFDUJRHPYLUWXGHGH G  WRGDVDVDOWHUQDWLYDVHVWmRFRUUHWDV
D  VHQWHQoDMXGLFLDOWUDQVLWDGDHPMXOJDGR
E  SURFHVVRDGPLQLVWUDWLYRTXHOKHDVVHJXUHDPSODGHIHVD  6HJXQGR
6HJXQGR
6HJXQGR D /HL
/HL 2UJkQLFD GR 'LVWULWR )HGHUDO R SUD]R GH
F  SURFHGLPHQWR GH DYDOLDomR SHULyGLFD GHGHVHPSHQKR
YDOLGDGHGRFRQFXUVRS~EOLFRVHUiGH
DVVHJXUDGDDDPSODGHIHVD
D  DWpDQRVSURUURJiYHOXPDYH]SRULJXDOSHUtRGR
G  WRGDVDVDOWHUQDWLYDVHVWmRFRUUHWDV
E  DWpDQRVSURUURJiYHOXPDYH]SRULJXDOSHUtRGR
F  DWpDQRSURUURJiYHOXPDYH]SRULJXDOSHUtRGR
 2
2 6HQDGR
6HQDGR )HGHUDO
)HGHUDO FRPS}HVH
FRPS}HVH GH
GH UHSUHVHQWDQWHV
UHSUHVHQWDQWHV GRV
G  WRGDVDVDOWHUQDWLYDVHVWmRFRUUHWDV
(VWDGRV
(VWDGRVHGR
GR'LVWULWR
'LVWULWR)HGHUDO
)HGHUDOHOHLWRV
HOHLWRVVHJXQGR
VHJXQGRRSULQFtSLR
SULQFtSLR
D  PLVWR
 6HJXQGR
6HJXQGR
6HJXQGRD/HL/HL2UJkQLFDGR'LVWULWR)HGHUDOjDGPLQLVWUD
E  SURSRUFLRQDO
F  PDMRULWiULR omR
omRWULEXWiULDLQFXPEHDVIXQo}HVGHODQoDPHQWRILVFDOL]D
G  WRGDVDVDOWHUQDWLYDVHVWmRFRUUHWDV omR
omRHDUUHFDGDomR
DUUHFDGDomRGRV
GRVWULEXWRV
WULEXWRVGH
GHFRPSHWrQFLD
FRPSHWrQFLDGR
GR'LVWULWR
)HGHUDO
)HGHUDO H R MXOJDPHQWR
MXOJDPHQWR DGPLQLVWUDWLYR GRV SU RFHVVRV
DGPLQLVWUDWLYR GRV
 &DGD
&DGD
&DGD(VWDGR
(VWDGRHR'LVWULWR
'LVWULWR)HGHUDOHOHJHUmRWUrV
)HGHUDOHOHJHUmRWUrV6HQDGRUHV ILVFDLV
ILVFDLV RV TXDLV VHUmR H[HUFLGRV SULYDWLYDPHQWH SRU
FRPPDQGDWR
FRPPDQGDWR LQWHJUDQWHVGDFDUUHLUDGH
D  TXDWURDQRV D  DXGLWRULDWULEXWiULD
E  VHLVDQRV E  ILVFDOWULEXWiULR
F  RLWRDQRV F  DGPLQLVWUDGRUWULEXWiULR
G  WRGDVDVDOWHUQDWLYDVHVWmRFRUUHWDV G  WRGDVDVDOWHUQDWLYDVHVWmRFRUUHWDV

 &RPSHWHSULYDWLYDPHQWHDR6HQDGR)HGHUDO
&RPSHWHSULYDWLYDPHQWHDR6HQDGR)HGHUDO  6HJXQGR
6HJXQGR
6HJXQGR D /HL
/HL 2UJkQLFD
2UJkQLFD GR
GR 'LVWULWR)HGHUDOVmREHQVGR
'LVWULWR)HGHUDOVmREHQVGR
D  SURFHVVDU H MXOJDU RV 0LQLVWURV GR 6XSUHPR7ULEXQDO 'LVWULWR)HGHUDO
)HGHUDORVPHPEURVGR&RQVHOKR1DFLRQDOGH-XVWLoDH D  RVTXHDWXDOPHQWHOKHSHUWHQFHPTXHYLHUDDGTXLULURX
GR &RQVHOKR 1DFLRQDO GR 0LQLVWpULR 3~EOLFR R IRUHPDWULEXtGRV
3URFXUDGRU*HUDOGD5HS~EOLFDHR$GYRJDGR*HUDOGD E  DViJXDVVXSHUILFLDLVRXVXEWHUUkQHDVIOXHQWHVHPHUJHQ
8QLmRQRVFULPHVGHUHVSRQVDELOLGDGH WHVHHPGHSyVLWRUHVVDOYDGDVQHVWHFDVRQDIRUPDGD
E  SURFHVVDUHMXOJDUR3UHVLGHQWHHR9LFH3UHVLGHQWHGD OHLDVGHFRUUHQWHVGHREUDVGD8QLmR
5HS~EOLFDQRVFULPHVGHUHVSRQVDELOLGDGHEHPFRPRRV F  DUHGHYLiULDGR'LVWULWR)HGHUDO VXDLQIUDHVWUXWXUDH
0LQLVWURVGH(VWDGRHRV&RPDQGDQWHVGD0DULQKDGR EHQVDFHVVyULRV
([pUFLWRHGD$HURQiXWLFDQRVFULPHVGDPHVPDQDWXUH]D G  WRGDVDVDOWHUQDWLYDVHVWmRFRUUHWDV
FRQH[RVFRPDTXHOHV
F  DSURYDUSUHYLDPHQWHSRUYRWRVHFUHWRDSyVDUJXLomRHP  6HJXQGR
6HJXQGR
6HJXQGRD/HL
/HL2UJkQLFD
2UJkQLFDGR
GR'LVWULWR
'LVWULWR)HGHUDO
)HGHUDOFDGD
FDGDOHJLVODWX
VHVVmRVHFUHWDDHVFROKDGRVFKHIHVGHPLVVmRGLSORPiWL UDWHUiDGXUDomRGH
FDGHFDUiWHUSHUPDQHQWH D  DWpRLWRDQRVLQLFLDQGRVHFRPDSRVVHGRVHOHLWRV
G  WRGDVDVDOWHUQDWLYDVHVWmRFRUUHWDV E  VHLVDQRVLQLFLDQGRVHFRPDSRVVHGRVHOHLWRV
F  TXDWURDQRVLQLFLDQGRVHFRPDSRVVHGRVHOHLWRV
 6HJXQGR
6HJXQGR
6HJXQGRD/HL
/HL2UJkQLFD
2UJkQLFDGR
GR'LVWULWR)HGHUDOVmR
'LVWULWR)HGHUDOVmRREMHWLYRV G  WRGDVDVDOWHUQDWLYDVHVWmRFRUUHWDV
SULRULWiULRVGR'LVWULWR)HGHUDO
D  SURPRYHUSURWHJHUHGHIHQGHURVGLUHLWRVGDFULDQoDGR  6HJXQGR
6HJXQGR
6HJXQGR D /HL
/HL 2UJkQLFD
2UJkQLFD GR
GR 'LVWULWR
'LVWULWR )HGHUDO GHVGH D
DGROHVFHQWHHGRMRYHP H[SHGLomR
H[SHGLomRGR
GRGLSORPD
GLSORPDRV
RVPHPEURV
PHPEURVGD
GD&kPDUD
&kPDUD/HJLVODWLYD
E  DVVHJXUDU SRU SDUWH GR SRGHU S~EOLFR DSURWHomR QmRSRGHUmRVHUSUHVRVVDOYR
LQGLYLGXDOL]DGDjYLGDHjLQWHJULGDGHItVLFDHSVLFROyJLFD D  HPIODJUDQWHGHFULPHLQDILDQoiYHO
GDVYtWLPDVHWHVWHPXQKDVGHLQIUDo}HVSHQDLVHGHVXHV E  SRUFRQWUDYHQomRSHQDO
UHVSHFWLYRVIDPLOLDUHV F  HPIODJUDQWHGHFULPHDILDQoiYHO
G  WRGDVDVDOWHUQDWLYDVHVWmRFRUUHWDV

 $
$ DUW†&)
 '
' DUW&)  $
$ DUW/2')
 '
' DUW†ž,,,,,,&)  %
% DUW,,,/2')
 &
& DUW&)  $
$ DUW/2')
 &
& DUW†ž&)  '
' DUW/2')
 '
' DUW&)  &
& DUW†~/2')
 '
' DUWž/2')  $
$ DUW†ž/2')
 'LU&RQVWLWXFLRQDO
7ULEXQDOGH-XVWLoD3(UQDPEXFR
 6HJXQGR
6HJXQGR
6HJXQGR D /HL
/HL 2UJkQLFD GR 'LVWULWR )HGHUDO FRPSHWH D 3DUD R DFHVVR D LQIRUPDo}HV GH LQWHUHVVH S~EOLFR D
SULYDWLYDPHQWH
SULYDWLYDPHQWH DRDR *RYHUQDGRU
*RYHUQDGRU GR 'LVWULWR )HGHUDO D LGHQWLILFDomRGRUHTXHUHQWHQmRSRGHFRQWHUH[LJrQFLDV
LQLFLDWLYDGDVOHLVTXHGLVSRQKDPVREUH TXHLQYLDELOL]HPDVROLFLWDomR
D  FULDomR GH FDUJRV IXQo}HV RX HPSUHJRV S~EOLFRVQD E 2yUJmRRXHQWLGDGHS~EOLFDGHYHUiDXWRUL]DURXFRQFH
DGPLQLVWUDomR GLUHWD DXWiUTXLFD H IXQGDFLRQDO RX GHURDFHVVRLPHGLDWRjLQIRUPDomRGLVSRQtYHO
DXPHQWRGHVXDUHPXQHUDomR F 2 VHUYLoR GH EXVFD H IRUQHFLPHQWR GD LQIRUPDomR p
E  VHUYLGRUHV S~EOLFRV GR 'LVWULWR )HGHUDO VHXUHJLPH JUDWXLWRVDOYRQDVKLSyWHVHVGHUHSURGXomRGHGRFXPHQ
MXUtGLFRSURYLPHQWRGHFDUJRVHVWDELOLGDGHHDSRVHQWD WRVSHORyUJmRRXHQWLGDGHS~EOLFDFRQVXOWDGDVLWXDomR
GRULD HP TXH SRGHUi VHU FREUDGR H[FOXVLYDPHQWH R YDORU
F  RUJDQL]DomRGD3URFXUDGRULD*HUDOGR'LVWULWR)HGHUDO QHFHVViULRDRUHVVDUFLPHQWRGRFXVWRGRVVHUYLoRVHGRV
FULDomRHVWUXWXUDomRUHHVWUXWXUDomRGHVPHPEUDPHQWR PDWHULDLVXWLOL]DGRV
H[WLQomRLQFRUSRUDomRIXVmRHDWULEXLo}HVGDV6HFUHWDUL G 4XDQGR VH WUDWDU GH DFHVVR j LQIRUPDomR FRQWLGD HP
DV GH *RYHUQR ÐUJmRV H HQWLGDGHV GD DGPLQLVWUDomR GRFXPHQWR FXMD PDQLSXODomR SRVVD SUHMXGLFDU VXD
S~EOLFDSODQRSOXULDQXDORUoDPHQWRDQXDOHGLUHWUL]HV LQWHJULGDGHRSHGLGRGHYHUiQHJDGR
RUoDPHQWiULDV
G  WRGDVDVDOWHUQDWLYDVHVWmRFRUUHWDV  $
$DXWRULGDGH
DXWRULGDGHPi[LPD
Pi[LPDGH GHFDGD
FDGDyUJmR
yUJmRRX
RXHQWLGDGH
HQWLGDGHSXEOLFDUi
DQXDOPHQWH
DQXDOPHQWHHPHPVtWLR
VtWLRjGLVSRVLomR
GLVSRVLomRQD
QDLQWHUQHW
LQWHUQHWHGHVWLQDGR
 6HJXQGR
6HJXQGR
6HJXQGRD/HL
/HL2UJkQLFD
2UJkQLFDGR
GR'LVWULWR
'LVWULWR)HGHUDORV&RQVHOKHLURV
)HGHUDORV&RQVHOKHLURV jYHLFXODomR
YHLFXODomRGHGDGRVHLQIRUPDo}HVDGPLQLVWUDWLYDVQRV
GR7ULEXQDO WHUPRVGHUHJXODPHQWR
WHUPRVGHUHJXODPHQWR
GR 7ULEXQDOGH
GH&RQWDV
&RQWDVWHUmR
WHUmRDV
DVPHVPDV
PHVPDVJDUDQWLDV
JDUDQWLDVSUHUUR
D  UROGDVLQIRUPDo}HVTXHWHQKDPVLGRGHVFODVVLILFDGDVQRV
JDWLYDV
JDWLYDV LPSHGLPHQWRV
LPSHGLPHQWRV YHQFLPHQWRV
YHQFLPHQWRV H YDQWDJHQV
YDQWDJHQV GRV
~OWLPRV GR]H PHVHV
'HVHPEDUJDGRUHV
'HVHPEDUJDGRUHV GR GR 7ULEXQDO
7ULEXQDO GH
GH -XVWLoD
-XVWLoD GR 'LVWULWR
E  UROGHGRFXPHQWRVFODVVLILFDGRVHPFDGDJUDXGHVLJLOR
)HGHUDO
)HGHUDOH7HUULWyULRV
7HUULWyULRVQD
QDIRUPD
IRUPDGD
GD&RQVWLWXLomR
&RQVWLWXLomR)HGHUDO
)HGHUDOH
FRPLGHQWLILFDomRSDUDUHIHUrQFLDIXWXUD
VRPHQWH
VRPHQWHSRGHUmR
SRGHUmRDSRVHQWDUVH
DSRVHQWDUVHFRP
FRPDV
DVYDQWDJHQV
YDQWDJHQVGR
GRFDUJR
F  UHODWyULRHVWDWtVWLFRFRQWHQGRDTXDQWLGDGHGHSHGLGRVGH
TXDQWRRWLYHUHPH[HUFLGRHIHWLYDPHQWHSRUPDLVGH
LQIRUPDomR UHFHELGRV DWHQGLGRV H LQGHIHULGRV EHP
D  DQRV
FRPRLQIRUPDo}HVJHQpULFDVVREUHRVVROLFLWDQWHV
E  DQRV
G  WRGDVDVDOWHUQDWLYDVHVWmRFRUUHWDV
F  DQRV 
G  WRGDVDVDOWHUQDWLYDVHVWmRFRUUHWDV  4XDQGR
4XDQGR
4XDQGR QmR
QmR IRUDXWRUL]DGRDFHVVRLQWHJUDOjLQIRUPDomR
SRUVHUHODSDUFLDOPHQWHVLJLORVDpDVVHJXUDGRRDFHVVR
 2
2 DFHVVR
DFHVVR j LQIRUPDomR
LQIRUPDomR FRPSUHHQGH
FRPSUHHQGH HQWUH RXWURV RV D  j SDUWH QmR VLJLORVD SRU PHLR GH FHUWLGmR H[WUDWRRX
GLUHLWRVGHREWHU FySLDFRPRFXOWDomRGDSDUWHVREVLJLOR
D  RULHQWDomRVREUHRVSURFHGLPHQWRVSDUDDFRQVHFXomRGH E  RUROGHGRFXPHQWRVFODVVLILFDGRVFRPRVLJLORVRVFRP
DFHVVREHPFRPRVREUHRORFDORQGHSRGHUiVHUHQFRQ LGHQWLILFDomRSDUDUHIHUrQFLDIXWXUD
WUDGDRXREWLGDDLQIRUPDomRDOPHMDGD F  DRUHODWyULRHVWDWtVWLFRFRQWHQGRDTXDQWLGDGHGHSHGLGRV
E  LQIRUPDomRFRQWLGDHPUHJLVWURVRXGRFXPHQWRVSURGX GHLQIRUPDomRUHFHELGRV
]LGRV RX DFXPXODGRV SRU VHXV yUJmRV RX HQWLGDGHV G  WRGDVDVDOWHUQDWLYDVHVWmRFRUUHWDV
UHFROKLGRVRXQmRDDUTXLYRVS~EOLFRV
F  LQIRUPDomRSURGX]LGDRXFXVWRGLDGDSRUSHVVRDItVLFDRX  e
e GHYHU
GHYHU GRV
GRV yUJmRV
yUJmRV H HQWLGDGHV
HQWLGDGHV S~EOLFDV
S~EOLFDV SURPRYHU D
HQWLGDGH SULYDGD GHFRUUHQWH GH TXDOTXHU YtQFXOR FRP GLYXOJDomR
GLYXOJDomR HP
HP ORFDO
ORFDO GH IiFLO DFHVVR QR kPELWR GH VXDV
VHXV yUJmRV RX HQWLGDGHV PHVPR TXH HVVH YtQFXOR Mi FRPSHWrQFLDV
FRPSHWrQFLDV GH
GH LQIRUPDo}HV
LQIRUPDo}HV GH GH LQWHUHVVH
LQWHUHVVH FROHWLYR
FROHWLYR RX
WHQKDFHVVDGR JHUDOSRUHOHVSURGX]LGDVRXFXVWRGLDGDV
G  WRGDVDVDOWHUQDWLYDVHVWmRFRUUHWDV D  LQGHSHQGHQWHGHUHFHELPHQWR
E  DSyVUHTXHULPHQWRFDEtYHO
 &DEH
&DEH
&DEHDRV
DRVyUJmRV
yUJmRVHHQWLGDGHV
HQWLGDGHVGR
GRSRGHU
SRGHUS~EOLFR
S~EOLFRREVHUYDGDV F  LQGHSHQGHQWHPHQWHGHUHTXHULPHQWRV
DV
DVQRUPDV
QRUPDVHSURFHGLPHQWRV
SURFHGLPHQWRVHVSHFtILFRV
HVSHFtILFRVDSOLFiYHLV
DSOLFiYHLVDVVHJX G  WRGDVDVDOWHUQDWLYDVHVWmRFRUUHWDV
UDUDSURWHomRGDLQIRUPDomRJDUDQWLQGRVHVXD
D  GLVSRQLELOLGDGH  1mR
1mR
1mR SRGHUiVHUQHJDGRDFHVVRjLQIRUPDomRQHFHVViULDj
SRGHUiVHUQHJDGRDFHVVRjLQIRUPDomRQHFHVViULDj
E DXWHQWLFLGDGH WXWHODMXGLFLDORXDGPLQLVWUDWLYDGHGLUHLWRV
F  LQWHJULGDGH D  SDWULPRQLDLV
G  WRGDVDVDOWHUQDWLYDVHVWmRFRUUHWDV E  IXQGDPHQWDLV
F  JHQpULFRV
 6HJXQGR
6HJXQGR
6HJXQGRD/HL
/HLQž
Qž
VHUi
VHUiDVVHJXUDGR
DVVHJXUDGRPHGLDQWH G  WRGDVDVDOWHUQDWLYDVHVWmRFRUUHWDV
UHDOL]DomR GH
UHDOL]DomR GH DXGLrQFLDV
DXGLrQFLDV RX
RX FRQVXOWDV
FRQVXOWDV S~EOLFDV
S~EOLFDV LQFHQWLYR
jSDUWLFLSDomRSRSXODURXDRXWUDVIRUPDVGHGLYXOJDomR
jSDUWLFLSDomRSRSXODURXDRXWUDVIRUPDVGHGLYXOJDomR  $
$ SHVVRD
SHVVRD ItVLFDRXHQWLGDGHSULYDGDTXHGHWLYHULQIRUPD
ItVLFDRXHQWLGDGHSULYDGDTXHGHWLYHULQIRUPD
D  RDFHVVRDLQIRUPDo}HVS~EOLFDV o}HV
o}HV HP
HP YLUWXGH
YLUWXGH GH YtQFXORGHTXDOTXHUQDWXUH]DFRPR
E DDXWHQWLFLGDGHGDVLQIRUPDo}HV SRGHU
SRGHU S~EOLFR H GHL[DU GH REVHUYDU R GLVSRVWR QD /HLQž
F  LQWHJULGDGHGRVDFXVDGRV HVWDUiVXMHLWDDVHJXLQWHVDQomR
G  WRGDVDVDOWHUQDWLYDVHVWmRFRUUHWDV D  DGYHUWrQFLD
E  PXOWD
 (P
(P
(PUHODomR
UHODomRDR
DRDFHVVR
DFHVVRjLQIRUPDomRDVVLQDOHD
jLQIRUPDomRDVVLQDOHDDOWHUQDWLYD F  UHVFLVmRGRYtQFXORFRPRSRGHUS~EOLFR
,QFRUUHWD G  WRGDVDVDOWHUQDWLYDVHVWmRFRUUHWDV

 '
' DUW†ž/2')
 &
& DUW†ž/2')  '
' DUW/Qž
 '
' DUWž/Qž  $
$ DUWž†ž/HLQž
$
 '
' DUWž/HLQž
'  &
& DUW/HLQž
&
 $
$ DUWž,,/HLQž
$  %
% DUW/HLQž
%
 '
' DUW/Qž  '
' DUW/HLQž
7
7ULEXQDOGH-XVWLoD
  3(UQDPEXFR 'LU&RQVWLWXFLRQDO 
 2V
2V
2V yUJmRV
yUJmRV H HQWLGDGHV
HQWLGDGHV S~ EOLFDV
EOLFDV UHVSRQGHP
UHVSRQGHP GLUHWDPHQWH  $
$FODVVLILFDomR
FODVVLILFDomRGR
GRVLJLOR
VLJLORGH
GHLQIRUPDo}HV
LQIRUPDo}HVQR
QR3RGHU
3RGHU([HFXWL
SHORVGDQRVFDXVDGRVHPGHFRUUrQFLDGDGLYXOJDomRQmR YRpGHFRPSHWrQFLDQRJUDXXOWUDVVHFUHWRGR
DXWRUL]DGD
DXWRUL]DGDRX
RXXWLOL]DomR
XWLOL]DomRLQGHYLGD
LQGHYLGDGH
GHLQIRUPDo}HV
LQIRUPDo}HVVLJLORVDV D GR3UHVLGHQWHGD5HS~EOLFD
RX
RXLQIRUPDo}HV
LQIRUPDo}HVSHVVRDLV
SHVVRDLVFDEHQGR
FDEHQGRDDSXUDomR
DSXUDomRGH
GHUHVSRQVD E GR*RYHUQDGRU
ELOLGDGHIXQFLRQDOQRVFDVRVGH F GRV WLWXODUHV GH DXWDUTXLD IXQGDomR HPSUHVD
D  GRORFRPGLUHLWRGHUHJUHVVR S~EOLFDRXVRFLHGDGHGHHFRQRPLDPLVWD
E  FXOSDVHPGLUHLWRGHUHJUHVVR G 3URFXUDGRU*HUDOGD5HS~EOLFD
F  GRORRXFXOSDDVVHJXUDGRRUHVSHFWLYRGLUHLWRGHUHJUHV
VR  1RkPELWROHJLVODWLYRGR'LVWULWR)HGHUDOFRQVLGHUDVH
1RkPELWROHJLVODWLYRGR'LVWULWR)HGHUDOFRQVLGHUDVH
G  WRGDVDVDOWHUQDWLYDVHVWmRFRUUHWDV D UHVROXomRDOHLTXHGHWHUPLQHDOWHUDomRHPGLVSRVLWLYRGD
/HL2UJkQLFD
 3DUD
3DUD
3DUD RV
RV HIHLWRV
HIHLWRV GD
GD /HL
/HL GLVWULWDO
GLVWULWDO Qž
Qž  FRQVLGH E OHL RUGLQiULD D OHL TXH GLVFLSOLQH PDWpULD TXH D /HL
UDVH 2UJkQLFDGHWHUPLQHFRPRVHXREMHWR
D GRFXPHQWRGDGRVSURFHVVDGRVRXQmRTXHSRGHPVHU F OHLFRPSOHPHQWDUDOHLTXHGLVFLSOLQHDVPDWpULDVOHJLVODWL
XWLOL]DGRVSDUDDSURGXomRHDWUDQVPLVVmRGHFRQKHFL YDVGDFRPSHWrQFLDGR'LVWULWR)HGHUDOTXHQmRHVWHMDP
PHQWRFRQWLGRVHPTXDOTXHUPHLRVXSRUWHRXIRUPDWR SUHYLVWDVQRVLQFLVRVDQWHULRUHV
E LQIRUPDomRSHVVRDODTXHODVXEPHWLGDWHPSRUDULDPHQWH G GHFUHWROHJLVODWLYRDOHLTXHFRPHVWHQRPHGLVFLSOLQH
jUHVWULomRGHDFHVVRS~EOLFRHPUD]mRGHVXDLPSUHVFLQ FRPHIHLWRH[WHUQRPDWpULDGDFRPSHWrQFLDSULYDWLYDGD
GLELOLGDGHSDUDDVHJXUDQoDGDVRFLHGDGHHGR(VWDGR &kPDUD/HJLVODWLYD
F DXWHQWLFLGDGHTXDOLGDGHGDLQIRUPDomRTXHWHQKDVLGR
SURGX]LGDH[SHGLGDUHFHELGDRXPRGLILFDGDSRUGHWHU 1RSURFHVVRGHHODERUDomRGDVOHLVDLQLFLDWLYDpD
 1RSURFHVVRGHHODERUDomRGDVOHLVD
1RSURFHVVRGHHODERUDomRGDVOHLVDLQLFLDWLYD
LQLFLDWLYDpD
PLQDGRLQGLYtGXRHTXLSDPHQWRRXVLVWHPD D SURSRVLomRTXHWHPSRUILQDOLGDGHDOWHUDUSURSRVWDGH
G SULPDULHGDGHTXDOLGDGHGDLQIRUPDomRQmRPRGLILFDGD HPHQGDj/HL2UJkQLFDRXSURMHWR
LQFOXVLYHTXDQWRjRULJHPDRWUkQVLWRHDRGHVWLQR E SURSRVWDGHFULDomRGHGLUHLWRQRYRHFRPHODVHLQLFLD
RSURFHVVROHJLVODWLYR
 2V
2V
2V yUJmRV
yUJmRV H DV
DV HQWLGDGHV
HQWLGDGHV GR
GR 3RGHU
3RGHU 3~EOLFR GHYHP FULDU F SURSRVLomRTXHWHPSRUILQDOLGDGHDOWHUDUSURSRVWDGH
VHUYLoR
VHUYLoRGH
GHLQIRUPDo}HV
LQIRUPDo}HVDR
DRFLGDGmR
FLGDGmRHP
HPORFDO
ORFDOFRP
FRPFRQGLo}HV HPHQGDj/HL2UJkQLFDRXSURMHWR
DSURSULDGDVSDUD G HWDSD GR SURFHVVR OHJLVODWLYR GHVWLQDGD DR GHEDWH GDV
D DWHQGHUHRULHQWDURS~EOLFRTXDQWRDRDFHVVRDLQIRUPD PDWpULDVREMHWRGHHODERUDomRGHOHL
o}HV
E SURWRFROL]DU GRFXPHQWRV H UHTXHULPHQWRV GH DFHVVR D 1RSURFHVVRGHHODERUDomRGDVOHLVDHPHQGDpD
 1RSURFHVVRGHHODERUDomRGDVOHLVD
1RSURFHVVRGHHODERUDomRGDVOHLVDHPHQGD
HPHQGDpD
LQIRUPDo}HV D SURSRVLomRTXHWHPSRUILQDOLGDGHDOWHUDUSURSRVWDGH
F LQIRUPDU VREUH D WUDPLWDomR GH GRFXPHQWRV QDV VXDV HPHQGDj/HL2UJkQLFDRXSURMHWR
UHVSHFWLYDVXQLGDGHV E SURSRVWDGHFULDomRGHGLUHLWRQRYRHFRPHODVHLQLFLD
G WRGDVDVDOWHUQDWLYDVHVWmRFRUUHWDV RSURFHVVROHJLVODWLYR
F SURSRVLomRTXHWHPSRUILQDOLGDGHDOWHUDUSURSRVWDGH
 1RFDVRGHLQGHIHULPHQWRGHDFHVVR
1RFDVRGHLQGHIHULPHQWRGHDFHVVR
1RFDVRGHLQGHIHULPHQWRGHDFHVVRjVLQIRUPDo}HVRXjV HPHQGDj/HL2UJkQLFDRXSURMHWR
UD]}HV
UD]}HVGD
GDQHJDWLYD
QHJDWLYDGR
GRDFHVVR
DFHVVRSRGH
SRGHRLQWHUHVVDGR
LQWHUHVVDGRLQWHUSRU G HWDSD GR SURFHVVR OHJLVODWLYR GHVWLQDGD DR GHEDWH GDV
UHFXUVRFRQWUDDGHFLVmRQRSUD]RGH PDWpULDVREMHWRGHHODERUDomRGHOHL
D GH]GLDVDFRQWDUGDVXDFLrQFLD
E TXLQ]HGLDVDFRQWDUGDVXDFLrQFLD  1R
1R
1RSURFHVVR
SURFHVVRGH
GHHODERUDomR
HODERUDomRGDV OHLVDGHOLEHUDomR
GDVOHLV GHOLEHUDomRpDHWDSD
F YLQWHGLDVDFRQWDUGDVXDFLrQFLD GRSURFHVVROHJLVODWLYR
G WULQWDGLDVDFRQWDUGDVXDFLrQFLD D SHOD TXDO D &kPDUD /HJLVODWLYD GHFLGH SULYDWLYDPHQWH
VREUH D FRQYHQLrQFLD RSRUWXQLGDGH H FRQWH~GR GDV
 4XDQWR
4XDQWR
4XDQWR jV
jV UHVWULo}HV
UHVWULo}HV GH DFHVVR j LQIRUPDomR DVVLQDOH D SURSRVWDVGHHPHQGDj/HL2UJkQLFDRXSURMHWRVOHYDGRV
DOWHUQDWLYDFRUUHWD jVXDFRQVLGHUDomR
D 3RGHVHUQHJDGRDFHVVRjLQIRUPDomRQHFHVViULDjWXWHOD E SHODTXDOVHGiFLrQFLDGDSURPXOJDomRGDVOHLVDRVVHXV
MXGLFLDORXDGPLQLVWUDWLYDGHGLUHLWRVIXQGDPHQWDLV GHVWLQDWiULRVWRUQDQGRREULJDWyULDVXDH[HFXomR
E $V LQIRUPDo}HV RX RV GRFXPHQWRV TXH YHUVHP VREUH F TXHDWHVWDDH[LVWrQFLDGDOHLUHFRQKHFHRVIDWRVHDWRV
FRQGXWDVTXHLPSOLTXHPYLRODomRGRVGLUHLWRVKXPDQRV TXHDJHUDUDPLQGLFDVXDYDOLGDGHHDWRUQDDSWDDVHU
SUDWLFDGDSRUDJHQWHVS~EOLFRVRXDPDQGRGHDXWRULGD H[HFXWDGD
GHVS~EOLFDVSRGHPVHUREMHWRGHUHVWULomRGHDFHVVR G GHVWLQDGDDRGHEDWHGDVPDWpULDVREMHWRGHHODERUDomR
F 6mRFRQVLGHUDGDVLPSUHVFLQGtYHLVjVHJXUDQoDGDVRFLH GHOHL
GDGHRXGR(VWDGRHSRUWDQWRSDVVtYHLVGHFODVVLILFDomR
DV LQIRUPDo}HV FXMD GLYXOJDomR RX DFHVVR LUUHVWULWR  1R
1R
1R SURFHVVR
SURFHVVR GH
GH HODERUDomR GDV OHLV D SURPXOJDomR
HODERUDomR GDV URPXOJDomR p D
SRVVDPS{UHPULVFRDGHIHVDHDVREHUDQLDQDFLRQDLVRX HWDSDGRSURFHVVROHJLVODWLYR
DLQWHJULGDGHGRWHUULWyULRQDFLRQDO D SHOD TXDO D &kPDUD /HJLVODWLYD GHFLGH SULYDWLYDPHQWH
G 2VSUD]RVPi[LPRVGHUHVWULomRGHDFHVVRjLQIRUPDomR VREUH D FRQYHQLrQFLD RSRUWXQLGDGH H FRQWH~GR GDV
YLJRUDP D SDUWLU GD GDWD GH VXD SURGXomR H VmR RV SURSRVWDVGHHPHQGDj/HL2UJkQLFDRXSURMHWRVOHYDGRV
VHJXLQWHV  XOWUDVVHFUHWD YLQWH DQRV  VHFUHWD GH]  jVXDFRQVLGHUDomR
UHVHUYDGDFLQFRDQRV

 %
% DUW/HL'LVWULWDOQž
 &
& DUW/HLQž  '
' DUWž†ž/HL&RPSOHPHQWDUGLVWULWDOQƒ
 &
& DUWž/HL'LVWULWDOQž  %
% DUWž/HL&RPSOHPHQWDUGLVWULWDOQƒ
 '
' DUW/HL'LVWULWDOQž  $
$ DUW/HL&RPSOHPHQWDUGLVWULWDOQƒ
 $
$ DUW/HL'LVWULWDOQž  $
$ DUW/HL&RPSOHPHQWDUGLVWULWDOQƒ
 &
& DUW/HL'LVWULWDOQž  &
& DUW/HL&RPSOHPHQWDUGLVWULWDOQƒ
 'LU&RQVWLWXFLRQDO
 7ULEXQDOGH-XVWLoD3(UQDPEXFR
E SHODTXDOVHGiFLrQFLDGDSURPXOJDomRGDVOHLVDRVVHXV  3URSRVLomR
3URSRVLomR
3URSRVLomRpWRGD PDWpULDVXMHLWD
WRGDPDWpULD VXMHLWDjGHOLEHUDomR
GHOLEHUDomRGD
GD&kPDUD
GHVWLQDWiULRVWRUQDQGRREULJDWyULDVXDH[HFXomR /HJLVODWLYD$SURSRVLomRSDUDVHUDGPLWLGDGHYHUi
F TXHDWHVWDDH[LVWrQFLDGDOHLUHFRQKHFHRVIDWRVHDWRV D WUDWDU GH PDWpULD GD FRPSHWrQFLD GR 'LVWULWR )HGHUDO
TXHDJHUDUDPLQGLFDVXDYDOLGDGHHDWRUQDDSWDDVHU VXMHLWDjGHOLEHUDomRGD&kPDUD/HJLVODWLYD
H[HFXWDGD E REVHUYDUDMXULGLFLGDGHHVXDFRUUHWDLQVHUomRQRRUGHQD
G GHVWLQDGDDRGHEDWHGDVPDWpULDVREMHWRGHHODERUDomR PHQWRMXUtGLFRVHDPDWpULDYLHUDVHUDSURYDGD
GHOHL F JXDUGDUFRHUrQFLDFRPDQRUPDDVHUDOWHUDGDQRFDVR
GHSURMHWRFRPHVVHREMHWLYR
 $
$ UHIHUrQFLD
UHIHUrQFLD H[SUH
H[SUHVVD
VVD D OHL
OHL RX
RX D GLVSRVLWLYR
GLVSRVLWLYR GH
GH OHL G WRGDVDVDOWHUQDWLYDVHVWmRFRUUHWDV
GHQRPLQDVH
D YHWR  'H
'H
'HDFRUGR
DFRUGRFRP
FRPR5HJLPHQWR
5HJLPHQWRGD
GD&kPDUD
&kPDUD/HJLVODWLYD
/HJLVODWLYDGR
GR')
E UHPLVVmR LQGLFDomRpDSURSRVLomR
LQGLFDomRpDSURSRVLomR
F SXEOLFDomR D SRUPHLRGDTXDOD&kPDUD/HJLVODWLYDVXJHUHDRXWUR
G VDQomR 3RGHU D H[HFXomR GH PHGLGDV TXH QmR VH LQFOXDP QD
FRPSHWrQFLDGR/HJLVODWLYR
 6HJXQGR
6HJXQGR
6HJXQGR R 5HJLPHQWR
5HJLPHQWR ,QWHUQRGD&kPDUD/HJLVODWLYDGR
,QWHUQRGD&kPDUD/HJLVODWLYDGR E SRUPHLRGDTXDOD&kPDUD/HJLVODWLYDVHPDQLIHVWDSDUD
'LVWULWR)HGHUDOFDGDOHJLVODWXUDWHUiGXUDomRGH KLSRWHFDUDSRLRRXVROLGDULHGDGHRXSDUDSURWHVWDUVREUH
D FLQFRDQRV GHWHUPLQDGRHYHQWR
E TXDWURDQRV F DSUHVHQWDGDFRPRDFHVVyULDGHRXWUDFRPRREMHWLYRGH
F WUrVDQRV DOWHUDUVXDIRUPDRULJLQDO
G GRLVDQRV G GHVWLQDGDDGLVSRUVREUHPDWpULDVSDUDDVTXDLVVHH[LJH
DVDQomRGR*RYHUQDGRU
 $V
$V
$VYDJDV
YDJDVQD
QD&kPDUD
&kPDUD/HJLVODWLYD
/HJLVODWLYDYHULILFDUVHmR
YHULILFDUVHmRHP
HPYLUWXGH
GH  'H
'H DFRUGR
DFRUGR FRP
FRP R 5HJLPHQWR
5HJLPHQWR GD
GD &kPDUD
&kPDUD /HJLVODWLYDGR
/HJLVODWLYDGR
D IDOHFLPHQWRHUHQ~QFLD ')PRomRpDSURSRVLomR
')PRomR
PRomRpDSURSRVLomR
E UHQ~QFLDHSHUGDGHPDQGDWR D SRUPHLRGDTXDOD&kPDUD/HJLVODWLYDVXJHUHDRXWUR
F SHUGDGHPDQGDWRHIDOHFLPHQWR 3RGHUDH[HFXomRGHPHGLGDVTXHQmRVHLQFOXDPQD
G WRGDVDVDOWHUQDWLYDVHVWmRFRUUHWDV FRPSHWrQFLDGR/HJLVODWLYR
E SRU PHLR GD TXDO D &kPDUD /HJLVODWLYD VH PDQLIHVWD
 6mR
6mR
6mRDWULEXLo}HV
DWULEXLo}HVGR
GR3UHVLGHQWH
3UHVLGHQWHGD
GD&kPDUD
&kPDUD/HJLVODWLYD
/HJLVODWLYDDOpP SDUDKLSRWHFDUDSRLRRXVROLGDULHGDGHRXSDUDSURWHVWDU
GH
GHRXWUDV
RXWUDVH[SUHVVDV
H[SUHVVDVQHVWH
QHVWH5HJLPHQWR
5HJLPHQWRRX
RXTXH
TXHGHFRUUDP
GHFRUUDPGD VREUHGHWHUPLQDGRHYHQWR
QDWXUH]DGHVXDVIXQo}HVRXSUHUURJDWLYDV F DSUHVHQWDGDFRPRDFHVVyULDGHRXWUDFRPRREMHWLYRGH
D FRQYRFDUVHVVmROHJLVODWLYDH[WUDRUGLQiULD DOWHUDUVXDIRUPDRULJLQDO
E DQXQFLDU D 2UGHP GR 'LD H R Q~PHUR GH 'HSXWDGRV G GHVWLQDGD D GLVSRU VREUH PDWpULDV SDUD DV TXDLV VH
'LVWULWDLVSUHVHQWHV H[LJHDVDQomRGR*RYHUQDGRU
F GHWHUPLQDUQmRVHMDUHJLVWUDGRGLVFXUVRRXDSDUWHSHOD
WDTXLJUDILDTXDQGRDQWLUUHJLPHQWDO
G WRGDVDVDOWHUQDWLYDVHVWmRFRUUHWDV

 $VFRPLVV}HVSHUPDQHQWHVWHUmR
$VFRPLVV}HVSHUPDQHQWHVWHUmR
D TXDWURPHPEURVHIHWLYRVFDGDXPD
E FLQFRPHPEURVHIHWLYRVFDGDXPD Caro Candidato,
F WUrVPHPEURVHIHWLYRVFDGDXPD
G GRLVPHPEURVHIHWLYRVFDGDXPD Alguns erros de “digitação” podem ocor-
rer... Pensando nisso, colocamos ao lado de
 $VVHVV}HVGD&kPDUD/HJLVODWLYDVHUmR
$VVHVV}HVGD&kPDUD/HJLVODWLYDVHUmR cada resposta (de Legislação) o texto legal
D VROHQWHVDVTXHSUHFHGHPDLQDXJXUDomRGRVWUDEDOKRV referente à questão. Assim, em caso de
GD&kPDUD/HJLVODWLYDQDSULPHLUDHQDWHUFHLUDVHVV}HV dúvida quanto à resposta, você poderá
OHJLVODWLYDVGHFDGDOHJLVODWXUD conferir na apostila - de acordo com texto
E SUHSDUDWyULDV DV GH TXDLVTXHU VHVV}HV OHJLVODWLYDV legal indicado - qual a resposta correta (o
UHDOL]DGDVjVWHUoDVTXDUWDVHTXLQWDVIHLUDV que vale é o texto da lei - sempre!!!)
F H[WUDRUGLQiULDVDVUHDOL]DGDVHPGLDRXKRUDGLYHUVRV
GRVSUHIL[DGRVSDUDDVRUGLQiULDV
G RUGLQiULDVDVUHDOL]DGDVSDUDFRPHPRUDo}HVRXKRPHQD
JHQVHVSHFLDLV

 %
% DUW/HL&RPSOHPHQWDUGLVWULWDOQƒ
 %
% DUWž5HJLPHQWR,QWHUQRGD&kPDUD/HJLVODWLYD')
 '
' DUW5HJLPHQWR,QWHUQRGD&kPDUD/HJLVODWLYD')
 '
' DUW5HJLPHQWR,QWHUQRGD&kPDUD/HJLVODWLYD')  '
' DUW5HJLPHQWR,QWHUQRGD&kPDUD/HJLVODWLYD')
 %
% DUW5HJLPHQWR,QWHUQRGD&kPDUD/HJLVODWLYD')  $
$ DUW5HJLPHQWR,QWHUQRGD&kPDUD/HJLVODWLYD')
 &
& DUW5HJLPHQWR,QWHUQRGD&kPDUD/HJLVODWLYD')  %
% DUW5HJLPHQWR,QWHUQRGD&kPDUD/HJLVODWLYD')
&kPDUD/HJLVODWLYDGR
  'LVWULWR)HGHUDO 'LU$GPLQLVWUDWLYR 

',5(,72$'0,1,675$7,92
GLUHLWRV H JDUDQWLDV LQGLYLGXDLV VmR EHP GHILQLGRV QXP
&RQFHLWRGH$GPLQLVWUDomR3~EOLFD HVWDWXWRPDLRUTXHH[SULPHRVSRGHUHVHRVGHYHUHVGR
(VWDGREHPFRPRDSURWHomRGRVLQGLYtGXRV

(VWDGRpDRUJDQL]DomRSROtWLFDFRPRILPHVSHFtILFR
2(VWDGR
(VWDGR $$GPLQLVWUDomR3XEOLFDpWRGRR DSDUDWR
DSDUDWR GR
GR (VWDGR
H HVVHQFLDO GH UHJXODPHQWDU JOREDOPHQWH DV UHODo}HV SDUD UHDOL]DU R PLVWHU WDUHID  SDUD R TXDO IRL FULDGR
VRFLDLVHQWUHRVPHPEURVGDSRSXODomRGHVHXWHUULWyULR GHVGHDSURWHomRGDVIURQWHLUDVVHJXUDQoDGDSD]DWpDV
2(VWDGR%UDVLOHLURpGHQRPLQDGR5HS~EOLFD)HGHUDWLYD PtQLPDVQHFHVVLGDGHVFRPXQVGDVSHVVRDVTXHWDPEpP
GR%UDVLOQRPHTXHWUDGX]QRVVDIRUPDGHJRYHUQR D VHFRPSUHHQGHPQDTXHODWDUHID(QILPDDGPLQLVWUDomR
UHS~EOLFD  S~EOLFDpDJHVWmRGRVLQWHUHVVHVGDFROHWLYLGDGHUHDOL]DGD
SHOR (VWDGR TXH FRQFUHWDPHQWH DJH SDUD VDWLVID]HU D
*RYHUQRpRFRQMXQWRGHyUJmRVVXSUHPRVDTXHPD
2*RYHUQR
*RYHUQR FRQVHUYDomRGDHVWUXWXUDVRFLDOGREHPHVWDULQGLYLGXDO
FRQVWLWXLomRLQFXPEHRH[HUFtFLRGRSRGHUGHSROtWLFR GDVSHVVRDVHRSURJUHVVRVRFLDO

-i D $GPLQLVWUDomR
GPLQLVWUDomR 3~EOLFD
3~EOLFD p R FRQMXQWR GH yUJmRV 3RULVVRR3URI+(/</23(60(,5(//(6HQVLQDTXH$
GHSHQGHQWHV VXERUGLQDGRV DR 3RGHU 3ROtWLFR *RYHUQR  $GPLQLVWUDomRpRLQVWUXPHQWDOGHTXHGLVS}HR(VWDGRSDUD
RUJDQL]DGRVPDWHULDOILQDQFHLUDHKXPDQDPHQWHSDUDD SRUHPSUiWLFDDVRSo}HVSROtWLFDVGR*RYHUQR,VWRQmRTXHU
H[HFXomRGDVGHFLV}HVSROtWLFDV GL]HUTXHD$GPLQLVWUDomRQmRWHQKDSRGHUGHGHFLVmR7HP
0DVRWHPVRPHQWHQDiUHDGHVXDVDWULEXLo}HVHQRVOLPLWHV
2 0HVWUH -26e $)2162 '$ 6,/9$ OHFLRQD TXH $ OHJDLVGHVXDFRPSHWrQFLDH[HFXWLYDVySRGHQGRRSLQDUH
RUJDQL]DomRDGPLQLVWUDWLYDQR(VWDGRIHGHUDOpFRPSOH[D GHFLGLUVREUHDVVXQWRVMXUtGLFRVWpFQLFRVILQDQFHLURVRXGH
SRUTXH D IXQomR DGPLQLVWUDWLYD p LQVWLWXFLRQDOPHQWH FRQYHQLrQFLDHRSRUWXQLGDGHDGPLQLVWUDWLYDVVHPTXDOTXHU
LPSXWDGDDGLYHUVDVHQWLGDGHVJRYHUQDPHQWDLVDXW{QRPDV IDFXOGDGHGHRSomRSROtWLFDVREUHDPDWpULD³ LQ'LUHLWR
'LUHLWR

TXH QR FDVR EUDVLOHLUR HVWmR H[SUHVVDPHQWH UHIHULGDV QR $GPLQLVWUDWLYR%UDVLOHLURHG57S


$GPLQLVWUDWLYR%UDVLOHLUR 
SUySULR DUW  GH RQGH GHFRUUH D H[LVWrQFLD GH YiULDV
$GPLQLVWUDo}HV 3~EOLFDV D IHGHUDO GD 8QLmR D GH FDGD
(VWDGR $GPLQLVWUDomRHVWDGXDO DGR'LVWULWR)HGHUDOHD
GH FDGD 0XQLFtSLR $GPLQLVWUDomR PXQLFLSDO RX ORFDO  $WRV$GPLQLVWUDWLYRV
FDGDTXDOVXEPHWLGDDXP3RGHUSROtWLFRSUySULRH[SUHVVR
SRUXPDRUJDQL]DomRJRYHUQDPHQWDODXW{QRPD³ LQ
&XUVR
GH'LUHLWR&RQVWLWXFLRQDO3RVLWLYR
HG6DUDLYDS  7RGDDomRKXPDQDWHPDOJXPSURSyVLWR2VDWRVGH
EHQHPHUrQFLDWrPRREMHWLYRGHVDWLVID]HUDVROLGDULHGDGH
(VWDGRPRGHUQDPHQWHpFULDomRGRKRPHPSDUD
2(VWDGR
(VWDGR KXPDQDHVRFLDO2VDWRVMXUtGLFRVYLVDPjSURGXomRGH
LQVWUXPHQWDU D UHDOL]DomR GDV QHFHVVLGDGHV FRPXQV H DOJXPHIHLWRMXUtGLFR2VDWRVDGPLQLVWUDWLYRVVmRSUDWLFD
LGHQWLGDGHVGHQDFLRQDLV&RPRFULDomRGRVLQGLYtGXRVR GRVSDUDJHULURVLQWHUHVVHVGDDGPLQLVWUDomRS~EOLFD
(VWDGR Ki GH VH UHODFLRQDU FRP HOHV FRP R Pi[LPR
UHVSHLWRjVVXDVJDUDQWLDVHGLUHLWRV1DWXUDOPHQWHHVVHV 2VDWRVDGPLQLVWUDWLYRVVmRDWRVMXUtGLFRVHSRUWDQWR
 'LU$GPLQLVWUDWLYR &kPDUD/HJLVODWLYDGR'LVWULWR)HGHUDO


WrPSRUREMHWLYRLPHGLDWRRXDDTXLVLomRRXRUHVJXDUGR REMHWLYRRDFULDomRRXDPRGLILFDomRRDFRPSURYDomRGH
RX D WUDQVIHUrQFLD RX D PRGLILFDomR RX D H[WLQomR GH VLWXDo}HVMXUtGLFDVUHODWLYDVDSHVVRDVFRLVDVRXDWLYLGDGHV
DOJXPGLUHLWR$IILQDOLGDGH
LQDOLGDGHS~EOLFDpDGLIHUHQoDHVVHQFL
S~EOLFD VXMHLWDVjDomRGR3RGHU3~EOLFR
DOHQWUHRDWRDGPLQLVWUDWLYRHGRVDWRVMXUtGLFRVHPJHUDO
1RVDWRVGLVFULFLRQiULRVRREMHWRpHVFROKtYHOOLYUHPHQ
5e*,6)(51$1'(6'(2/,9(,5$GHSRLVGHSURIXQGD WHSHORDGPLQLVWUDGRUS~EOLFR
DQiOLVHGRDWRDGPLQLVWUDWLYRFRQFHLWXDRFRPRGHFODUD
omRXQLODWHUDOGR(VWDGRRXGHTXHPIDoDVXDVYH]HVQR
H[HUFtFLR GD IXQomR DGPLQLVWUDWLYD TXH SURGX]D HIHLWRV
MXUtGLFRV LQGLYLGXDLV H LPHGLDWRV LQ $WR $GPLQLVWUDWLYR  $WULEXWRV
HG57S 

( R 3URI',2*2 '( ),*8(,5('2 025(,5$ 1(72 7DQWRDYDOLGDGHFRPRDHILFiFLDHDDXWRH[HFXWRULH


DVVLP FRQFHLWXD R DWR DGPLQLVWUDWLYR D PDQLIHVWDomR GDGHSUHPLVVDVGDSHUIHLomRGR$WR$GPLQLVWUDWLYRVmR
XQLODWHUDOGHYRQWDGHGD$GPLQLVWUDomR3~EOLFDTXHWHPSRU DWULEXWRVLPDQHQWHVjVXDSUySULDFRQGLomRFRQVRDQWHVH
REMHWRFRQVWLWXLUGHFODUDUFRQILUPDUDOWHUDURXGHVFRQVWL LQIHUHGRHQVLQDPHQWRGR3URI',2*2'(),*8(,5('2
WXLUXPDUHODomRMXUtGLFDHQWUHHODHRVDGPLQLVWUDGRVRX 025(,5$ 1(72 DR FRQVLGHUDU TXH $ $WR
WR 3HUIHLWR
3HUIHLWR p R
HQWUHVHXVyUJmRVHHQWLGDGHV LQ&XUVRGH'LUHLWR$GPLQLVWUDWL GRWDGR GH HILFiFLD H H[HTLELOLGDGH UH~QH RV HOHPHQWRV
YRHG)RUHQVHS 
QHFHVViULRVjVXDH[HFXomR
QHFHVViULRV LQ&XUVRGH'LUHLWR$GPLQLVWUDWLYR
HG)RUHQVHS 

3RVLWLYDPHQWHVHUiLPSHUIHLWRRDWRDGPLQLVWUDWLYRVH
5HTXLVLWRV OKHIDOWDUDOJXPGRVHOHPHQWRVGHPRGRDOKHH[FOXLUD
SRVVLELOLGDGHGHVHUH[HFXWDGR³MiTXHDSHUIHLomRVyp
DWLQJLQGDTXDQGRRDWRpHILFD]PHQWHH[HTtYHO
( SDUD WHU YDOLGDGH
YDOLGDGH WRGR DWR DGPLQLVWUDWLYR GHYHUi
DWHQGHUDFLQFRUHTXLVLWRVTXHVmRVHXVUHTXLVLWRV
UHTXLVLWRV
UHTXLVLWRVHVVHQ 9DOLGDGH
9DOLGDGH VyWHUiRDWRDGPLQLVWUDWLYRTXHSURYLHUGH
FLDLVFRPSHWrQFLDILQDOLGDGHIRUPDPRWLYRHREMHWR
FLDLV DXWRULGDGH FRPSHWHQWH SDUD SUDWLFiOR H FRQWLYHU RV
UHTXLVLWRVLQGLVSHQViYHLVjVXDHILFiFLD$VVLPSDUDWHU
&RPSHWrQFLD
&RPSHWrQFLD VLJQLILFDTXHRDWRDGPLQLVWUDWLYRVy YDOLGDGHWRGRDWRDGPLQLVWUDWLYRGHYHUiDWHQGHUDFLQFR
YDOLGDGH
SRGHVHUSUDWLFDGRSHORDJHQWHDTXHPDOHLDXWRUL]RX UHTXLVLWRVTXHVmRVHXVUUHTXLVLWRV
HTXLVLWRVHVVHQFLDLVFRPSHWrQFLD
HVVHQFLDLV
FRPHWHX HQWUHJRX  VXD SUiWLFD QD RUJDQL]DomR ILQDOLGDGHIRUPDPRWLYRHREMHWR
DGPLQLVWUDWLYDFDGDyUJmRHFDGDDJHQWHWHPVXDIXQomR
VXDWDUHIDGHILQLGDHPOHL8PQmRSRGHUiXVXUSDUDIXQ $QRWD5(*,6)(51$1'(6'(2/,9(,5$TXH´2DWRp
omRGRRXWUR FDGDPDFDFRQRVHXJDOKR  YiOLGRTXDQGRQHOHVHFRQWrPWRGRVRVUHTXLVLWRVOHJDLV
RX VHMD TXDQGR WRGDV DV FRQGLo}HV GH YDOLGDGH IRUDP
)LQDOLGDGH
)LQDOLGDGH TXH GL]HUTXHWRGRDWRDGPLQLVWUDWLYR DWHQGLGDV RX DLQGD TXDQGR IRL SUDWLFDGR FRQIRUPH R
GHYH WHU 8P SURSyVLWR D DOFDQoDU H QDWXUDOPHQWH XP RUGHQDPHQWRMXUtGLFR(VFODUHFH*DUULGR)DOODTXH´SXHV
LQWHUHVVHS~EOLFR$WRDGPLQLVWUDWLYRTXHQmRWHQKDSRU VL HO DFWR HV YiOLGR DQWHV GH VHU FRPXQLFDGR TXLpUHVH
SURSyVLWRDWHQGHUDDOJXPLQWHUHVVHS~EOLFRVHUiDWRDEX GHFLUFRQHOORTXH\DHVWDSURYLVWRGHWRGRVVXVHOHPHQWRV
VLYRHQmRWHUiYDOLGDGH QHFHVVDULRVµ ³ LQ´$WR$GPLQLVWUDWLYRµ5HYLVWDGRV7ULEXQDLV
S 
)RUPD
)RUPD p D PDQHLUD SHOD TXDO R DWR GHYHVHU
SUDWLFDGR7RGRDWRDGPLQLVWUDWLYRGHYHVHUSUDWLFDGRGH $OLiV R UHIHULGR $XWRU DSRQWD FRPR FRQGLo}HV RX
IRUPD HVFULWD  $ UD]mR p VLPSOHV R DWR DGPLQLVWUDWLYR UHTXLVLWRVGHYDOLGDGHGRDWRDGPLQLVWUDWLYR´D VXMHLWRE
GHYHILFDUVHPSUHUHJLVWUDGRWDQWRSDUDTXHKDMDFRQWUROH FRQWH~GRF ILQDOLGDGHG IRUPDOLGDGHH PRWLYRHI
GRV DWRV GD DGPLQLVWUDomR TXDQWR SURYD GH VXD H[LV FDXVD$IDOWDGDSUHVHQoDGHWRGDVDVFRQGLo}HVPHQFLR
WrQFLD QDGDV WDO FRPR FRQFHELGDV WHUVHi D QXOLGDGH GR DWR
DGPLQLVWUDWLYRRXPHOKRUGL]HQGRH[LVWLUiDOJXPYtFLR
0RWLYR
0RWLYR pDFDXVDDLQVSLUDomRSDUDDSUiWLFDGRDWR DPDFXODUDKLJLGH]GRDWRRXVXUJHGHILFLHQWHPHQWHR
DGPLQLVWUDWLYR+DYHUiVHPSUHXPIDWRTXHGHWHUPLQRX REMHWLYRGRDGPLQLVWUDGRUµ LQREUDFLWDGDS 
DUHDOL]DomRGRDWRDGPLQLVWUDWLYR6HXPSUHIHLWRYDLGH
PLWLU XP IXQFLRQiULR D PRWLYDomR FDXVD  WHUi VLGR D $HILFiFLDH[RUQDRDWR´TXDQGRVHHQFRQWUDHPHVWDGR
SUiWLFDGHXPDLQIUDomRGLVFLSOLQDUJUDYH6HYDLFRQVWUX DWXDOGHSURGXWLYLGDGHGHVHXVHIHLWRVRXVHMDTXDQGR
LUXPDSRQWHDFDXVD PRWLYDomR VHUiDQHFHVVLGDGHGH QDGD LPSHGH D HFORVmR GRV HIHLWR TXH R DWR SRVVXL
WUkQVLWRRXDFHVVRGHGHWHUPLQDGDUHJLmRGRPXQLFtSLR LQWULQVHFDPHQWH1mRPDLVGHSHQGHGHTXDOTXHUPDQLIHV
1RV DWRV DGPLQLVWUDWLYRV GLVFULFLRQiULRV p GLVSHQViYHO WDomRRXGHVROXomRXOWHULRUeLPHGLDWDDGHIODJUDomRGRV
H[SOLFLWDUDPRWLYDomR HIHLWRVTXHRDWRSRVVXLµ

2EMHWR
2EMHWR pRSUySULRFRQWH~GRGRDWRDGPLQLVWUDWLYR $ HILFiFLDGRDWRDGPLQLVWUDWLYRpVXDFDSDFLGDGHGH
HILFiFLD
VXD HVVrQFLD $VVLP WRGR DWR DGPLQLVWUDWLYR WHUi SRU SURGX]LU RV HIHLWRV SDUD RV TXDLV H SHORV TXDLV HOH IRL
&kPDUD/HJLVODWLYDGR
  'LVWULWR)HGHUDO 'LU$GPLQLVWUDWLYR 

SUDWLFDGR,QHILFD]pRDWRTXHQmRSRGHSURGX]LUTXDOTXHU UHYRJDGRHOHFRQWLQXDUiVHQGRXPDWRFRPLPSHUDWLYL
LPSHUDWLYL
HIHLWR RX VHMD p QXOR $VVLP D HILFiFLD
HILFiFLD p GLUHWDPHQWH GDGH
GDGH RX VHMD XP DWR HP TXH D $GPLQLVWUDomR SRGHUi
VXERUGLQDGDjYDOLGDGH
YDOLGDGHGRDWRDGPLQLVWUDWLYR
YDOLGDGH XWLOL]DUVHGDIRUoDSDUDVHXFXPSULPHQWRREULJDWyULR

23URI+(/</23(60(,5(//(6HQVLQDTXH$HILFiFLD HILFiFLD )LQDOPHQWH R DWULEXWR GH DXWRH[HFXWRULHGDGH


DXWRH[HFXWRULHGDGH
pDLGRQHLGDGHTXHVHUHFRQKHFHDRDWRDGPLQLVWUDWLYRSDUD FRQVLVWHHPSRVVLELOLWDUTXHD$GPLQLVWUDomR3~EOLFDIRVVH
SURGX]LU VHXV HIHLWRV HVSHFtILFRV 3UHVVXS}H SRUWDQWR D H[HFXWiORGLUHWDRXLPHGLDWDPHQWHQmRSUHFLVDQGRGH
UHDOL]DomR GH WRGDV DV IDVHV H RSHUDo}HV QHFHVViULDV j RUGHP RX DXWRUL]DomR MXGLFLDO QHP HP FDVRV GH IRUoD
IRUPDomRGRDWRILQDOVHJXQGRRGLUHLWRSRVLWLYRYLJHQWH LQ ItVLFD$VVLPSRUH[HPSORVHXPDSHVVRDFRQVWUyLXPD
'LUHLWR$GPLQLVWUDWLYR%UDVLOHLURHG57S  FDVDFODQGHVWLQDHFRQWUDULDPHQWHjVSRVWXUDVPXQLFLSDLV
VREUHFRQVWUXomRFLYLOD3UHIHLWXUDSRGHUiLQWHUGLWDUD
1HPSRULVVRVHFRQIXQGDDHILFiFLDFRPVXDH[HTLELOL REUDHHODPHVPDSRGHUiPDQGDUGHPROLUDTXHODREUD
GDGHSRLVRDWRSRGHUiVHUHILFD]PDVWHUVXDH[HTLELOL QHPSUHFLVDQGRGHRUGHP PDQGDGR MXGLFLDO
GDGH VXVSHQVD RX SRUTXH VXERUGLQDGR D TXDOTXHU
FRQGLomRVXVSHQVLYDRXDXPWHUPRRXPHVPRjRFRU $GYHUWHRVDXGRVR+(/</23(60(,5(//(6TXH´2V
UrQFLDGHXPDWRFRPSOHPHQWDULQGLVSHQViYHODRLQtFLRGH DXWRUHVPDLVPRGHUQRVQmRVHFDQVDPGHDSRQWDUHVVH
VXDH[HFXomR DWULEXWRQRVDWRVDGPLQLVWUDWLYRVTXHRSRVVXHP(QWUH
WDQWRDVQRVVDV$GPLQLVWUDo}HVVHPRVWUDPWtPLGDVQD
3RUTXHHPDQDPGR3RGHU3~EOLFRRV$WRV$GPLQLVWUD VXDXWLOL]DomRHDQRVVD-XVWLoDQHPVHPSUHDWXDOL]DGD
WLYRVWrPDOJXPDVHVSHFLDLVFDUDFWHUtVWLFDVFRPDVTXDLV FRP R GLUHLWR S~EOLFR HP SURQXQFLDPHQWRV IHOL]PHQWH
HOHVVHLPS}HP$VVLPWRGRDWRDGPLQLVWUDWLYRWHP D UDURV WHP SUHWHQGLGR FRQGLFLRQDU D H[HFXomR GH DWRV
SUHVXQomR
SUHVXQomR GH
GH OHJLWLPLGDGH
OHJLWLPLGDGH E  LPSHUDWLYLGDGH
LPSHUDWLYLGDGH H F WLSLFDPHQWHDXWRH[HFXWyULRVjSUpYLDDSUHFLDomRMXGLFLDO
DXWRH[HFXWRULHGDGH
DXWRH[HFXWRULHGDGH 0DV HP FRQWUDSRVLomR D HVVHV MXOJDGRV HVSRUiGLFRV H
HUU{QHRVILUPDVHFDGDYH]PDLVDMXULVSUXGrQFLDQDERD
GRXWULQDUHFRQKHFHQGRj$GPLQLVWUDomR³HVSHFLDOPHQ
$SUHVXQomR
SUHVXQomR GH
GH OHJLWLPLGDGHVLJQLILFDTXHWRGR
WHTXDQWRDRVDWRVGHSROtFLD³RSRGHUGHH[HFXWDUGLUHWD
DWRDGPLQLVWUDWLYRVyVHUiFRQVLGHUDGRLOHJtWLPRVHKRXYHU
H LPHGLDWDPHQWH VHXV DWRV LPSHUDWLYRV LQGHSHQGHQWH
XPDSURYDFDEDOVHJXUDGHVHULQYiOLGR
PHQWHGHSHGLGRFRPLQDWyULRRXPDQGDGRMXGLFLDO
1RXWUDVSDODYUDVVHQmRKRXYHUQHQKXPDSURYDTXH
(QWUH QyV HVVD GRXWULQD p FRUUHQWH FRPR QRV UHYHOD
SHUPLWDDGHFODUDomRGDLQYDOLGDGHGRDWRDGPLQLVWUDWLYR 6HDEUD)DJXQGHVFRPVXDLPHQVDDXWRULGDGHQRDVVXQWR
HOHGHYHVHPSUHVHUFRQVLGHUDGRSHUIHLWDPHQWHYiOLGRp 7DOSURFHVVRH[HFXWyULRWHPFDELPHQWRTXDQGRDVFLUFXQV
DSUHVXQomRGHOHJLWLPLGDGH WkQFLDVLQGLFDPDQHFHVVLGDGHSUHPHQWHGDREWHQomRGRIDWR
RXFRLVD$WXDSHODXWLOL]DomRSRUSDUWHGRDGPLQLVWUDGRU
1LQJXpPSRGHUiSRUWDQWRUHFXVDUREVHUYDURXFXP GRVFKDPDGRVPHLRVGLUHWRVGHFRHUomRDGPLQLVWUDWLYDGH
SULUXPDWRDGPLQLVWUDWLYRDOHJDQGRVHUHOHLQYiOLGRVy PRGR D WRUQDU SRVVtYHO REWHU SRU FRDomR DEVROXWD D
SRGHUi ID]rOR GHSRLV GH SURYDGD WDO LQYDOLGDGH $ SUySULD SUHVWDomR H[LJLGD GR DGPLQLVWUDGR RX QD VXD
DUJLomR GH QXOLGDGH GH XP DWR DGPLQLVWUDWLYR p {QXV LPSRVVLELOLGDGH RXWUD HTXLYDOHQWHµ ³ LQ &XUVR GH 'LUHLWR
GHYHU GHTXHPDDOHJDU$DGPLQLVWUDomR3~EOLFDQmR $GPLQLVWUDWLYR%UDVLOHLURHG57S 
SUHFLVDUSURYDUVXDYDOLGDGHHODpSUHVXPLGD

$LPSHUDWLYLGDGHpRSRGHUGRDWRDGPLQLVWUDWLYRGH
LPSRUVHXFXPSULPHQWRVXDH[HFXomR7RGRDWRDGPLQLV &ODVVLILFDomR
WUDWLYR TXH FRQVXEVWDQFLDU WLYHU HP VXD HVVrQFLD  XP
SURYLPHQWRRXXPDRUGHPDGPLQLVWUDWLYDVHUiVHPSUH
LPSHUDWLYR RX VHMD REULJDUi D VHX ILHO DWHQGLPHQWR 7RGDVDVFRLVDVSRGHPVHUYLVWDVVREYiULRVDVSHFWRV
VXMHLWDQGRVH RV UHQLWHQWHV WHLPRVRV  UHVLVWHQWHV j &RQIRUPHRkQJXORHPTXHVHROKHSDUDTXDOTXHUFRLVD
H[HFXomR IRUoDGD RX VHMD DR HPSUHJR GH IRUoD SHOD GLIHUHQWHVHUiVXDDQiOLVH8PVHUKXPDQRSRUH[HPSOR
SUySULD $GPLQLVWUDomR S~EOLFD R FDVR GRV DWRV DX SRGHVHUYLVWRVRERDVSHFWRGDHOHJkQFLD HRVFODVVLILFD
WRH[HFXWyULRV RXSHOR-XGLFLiULR QRFDVRGRVDWRVQmR UtDPRV HP FKLTXHV FRPXQV H EUHJDV  RX VRE R DV
DXWRH[HFXWyULRV  SHFWR GD EHOH]D HVWpWLFD H VHULDP FODVVLILFDGRV FRPR
OLQGRVFRPXQVHIHLRVRV RXVRERDVSHFWRGDHVWDWXUD
0DV QHP WRGR DWR DGPLQLVWUDWLYR p GRWDGR GH ItVLFD HRVFODVVLILFDUtDPRVFRPRDOWRVPpGLRVHQRU
LPSHUDWLYLGDGHRVDWRVHQXQFLDWLYRVHRVQHJRFLDLVSRU PDLV RXDWpVREkQJXORVGHYLUWXGHVDEVWUDWDV FRPRD
H[HPSOR QmR SUHFLVDP GH LPSHUDWLYLGDGH SRUTXH VXD LQWHOLJrQFLDRXDERQGDGHRXDVLPSDWLDHWF 
QDWXUH]DQmRH[LJHH[HFXomR$VVLPWRGRDWRDGPLQLVWUD
7DPEpPRVDWRVDGPLQLVWUDWLYRVSRGHPVHUDQDOLVD
WLYRTXHIRUGRWDGRGHLPSHUDWLYLGDGHGHYHVHUFXPSULGR
GRV VRE YiULRV kQJXORV H FRQIRUPH R DVSHFWR HP TXH
REHGHFLGRDWHQGLGR(SRXFRLPSRUWDVHHOHpYiOLGRRX
IRUHP YLVWRV UHFHEHUmR FODVVLILFDo}HV YiULDV 9HMDPRV
QmR(QTXDQWRQmRIRUGHFODUDGD MXGLFLDORXDGPLQLVWUD
HQWmRTXHkQJXORVVmRRVPDLVFRPXQVSDUDDDQiOLVHGRV
WLYDPHQWH  VXD LQYDOLGDGH DQXODomR  RX HOH QmR IRU
DWRVDGPLQLVWUDWLYRV
 'LU$GPLQLVWUDWLYR &kPDUD/HJLVODWLYDGR'LVWULWR)HGHUDO


, 4XDQWRDRVVHXVGHVWLQDWiULRV F $WR


$WR&RPSOH[RVHDSHUIHLoRDSHODPDQLIHVWDomRGD
&RPSOH[R
YRQWDGHGHPDLVGHXPyUJmRGD$GPLQLVWUDomR
D $WRV
$WRV*HUDLV FRPILQDOLGDGHQRUPDWLYDRXRUGLQi
*HUDLV +i R FRQFXUVR GH YRQWDGHV GH yUJmRV GLYHUVRV
ULD RVTXHQmRWrPGHVWLQDWiULRVFHUWRVDEUDQ SDUDDIRUPDomRGHXP~QLFRDWR
JHQGR D WRGRV RV VXMHLWRV TXH VH HQFRQWUHP QD
PHVPD VLWXDomR IiWLFD SUHYLVWD SHORV VHXV SUH 9 4XDQWRjHILFiFLD
FHLWRV6mRVHPHOKDQWHVDRVSUHFHLWRVGDOHLDEVWUD
WRVHLPSHVVRDLV([VUHJXODPHQWRVLQVWUXo}HV D $WR
$WR9iOLGRSUDWLFDGRSRUDXWRULGDGHFRPSHWHQWH
9iOLGR
FLUFXODUHV H FRQWpP WRGRV RV UHTXLVLWRV QHFHVViULRV SDUD
HQWUDUQRPXQGRMXUtGLFR
E $WRV
$WRV ,QGLYLGXDLV
,QGLYLGXDLV RX
RX (VSHFLDLV
(VSHFLDLV  GLULJHPVH D
GHVWLQDWiULRV FHUWRV FULDQGROKHV XPD VLWXDomR E $WR 1XOR
1XOR  QDVFH FRP YtFLR LQVDQiYHO HP VHXV
MXUtGLFDSDUWLFXODU([VGHFUHWRVGHQRPHDomRGH
HOHPHQWRV FRQVWLWXWLYRV RX QR VHX SURFHGLPHQWR
H[RQHDUDomR RXWRUJDV GH OLFHQoD DXWRUL]DomR H
IRUPDWLYReXPDWRLOHJtWLPRHLOHJDOHQmR
QmRSURGX]
QmR
SHUPLVVmR
QHQKXPHIHLWR6XDQXOLGDGHSRGHVHUGHFODUDGD
SHODSUySULD$GPLQLVWUDomRRXSHOR3RGHU-XGLFLi
,, 4XDQWRDRVHXDOFDQFH ULR$QXOLGDGHRSHUDH[WXQFLVWRpUHWURDJHDWp
DRULJHPGRDWR
D $WRV
$WRV ,QWHUQRVSURGX]HPHIHLWRVQRLQWHULRUGDV
,QWHUQRV
UHSDUWLo}HVDGPLQLVWUDWLYDVVmRDWRVGHRSHUDWLYL
F $WR
$WR,QH[LVWHQWHHTXLSDUDVHDRDWRQXOR$SHQDV
,QH[LVWHQWH
GDGHLQWHUQDGDVUHSDUWLo}HVS~EOLFDV([VSRUWDUL
WHPDDSDUrQFLDGHPDQLIHVWDomRUHJXODUGD$GPL
DVLQVWUXo}HVPLQLVWHULDLV
QLVWUDomR
E $WRV
$WRV GH
GH (IHLWRV
(IHLWRV ([WHUQRVVmRWRGRVDTXHOHVTXH
([WHUQRV
DOFDQoDPRVDGPLQLVWUDGRVRVFRQWUDWDQWHVHHP 9, 4XDQWRDRFRQWH~GR
FHUWRVFDVRVRVSUySULRVVHUYLGRUHVSURYHQGRVREUH
VHXVGLUHLWRVREULJDo}HVQHJyFLRVRXFRQGXWDSHUDQ D $WR
$WR &RQVWLWXWLYR
&RQVWLWXWLYR  DTXHOH TXH FULD XPD QRYD
WHD$GPLQLVWUDomR +HO\/RSHV0HLUHOOHVREUDFLWSOO  VLWXDomRMXUtGLFDSDUDRLQGLYtGXRSHUDQWHD$GPL
6yHQWUDPHPYLJRUDSDUWLUGHVXDSXEOLFDomRQR QLVWUDomR
yUJmRRILFLDO23URI+HO\/RSHV0HLUHOOHVWDPEpP
FRQVLGHUDDWRVH[WHUQRVRVTXHSURGX]HPHIHLWRV E $WR
$WR ([WLQWLYR
([WLQWLYR  FRPR GL] R QRPH HOH H[WLQJXH
IRUDGDVUHSDUWLo}HVHPERUDQmRDWLQMDGLUHWDPHQ XPDVLWXDomRMXUtGLFDSUHH[LVWHQWHSRQGRILPD
WHRDGPLQLVWUDGREHPFRPRDTXHOHVTXHRQHUHP XPDUHODomRMXUtGLFDGRLQGLYtGXRSHUDQWHD$GPL
DGHVSHVDRXRSDWULP{QLRS~EOLFR QLVWUDomR

,,, 4XDQWRDRVHXREMHWR F $WR


$WR 'HFODUDWyULRpRTXHGHFODUDXPDVLWXDomR
'HFODUDWyULR
SUHH[LVWHQWH UHFRQKHFHQGR VXD HIHWLYLGDGH H
D $WRV
$WRV GH,PSpULR
GH,PSpULRSUDWLFDGRVSHOD$GPLQLVWUDomR
,PSpULR SRVVLELOLWDQGRRH[HUFtFLRGRVGLUHLWRVGHFRUUHQWHV
QRXVRGHVXDVXSUHPDFLDVREUHRSDUWLFXODUVmR
PDQLIHVWDo}HVXQLODWHUDLVGD$GPLQLVWUDomRQRXVR G $WR$OLHQDWLYRDOLHQDUpWUDQVIHULUSURSULHGDGH
$WR$OLHQDWLYR
GHVHXSRGHUVREHUDQR([HPSORGHFODUDomRH[SUR DRXWUHPORJRWDODWRpDTXHOHSHORTXDOD$GPL
SULDWyULD QLVWUDomRWUDQVIHUHEHQVRXGLUHLWRVGHXPWLWXODU
DRXWURQRUPDOPHQWHRVDWRVDOLHQDWLYRVGHSHQ
E $WRVGH*HVWmR$$GPLQLVWUDomRSUDWLFDVHPR
$WRVGH*HVWmR GHPGHSUpYLDDXWRUL]DomRSRUOHLRH[HFXWLYRQmR
XVRGHVHXSRGHUGHFRHUomRVREUHRVGHVWLQDWiULRV SRGH DOLHQDU VHP XPD OHL DXWRUL]DQGR DTXHOD
HVSHFtILFDWUDQVIHUrQFLDGHSURSULHGDGH
F $WRV
$WRVGH
GH([SHGLHQWHGHVWLQDPVHDGDUDQGDPHQWR
([SHGLHQWH
DRV SDSpLV QR UHFHVVR GDV UHSDUWLo}HV S~EOLFDV
H $WR
$WR0RGLILFDWLYRFRPRGL]RQRPHpDTXHOHTXH
0RGLILFDWLYR
SUHSDUDQGRRVSDUDDGHFLVmRGRPpULWRTXHVHUi
PRGLILFD DOWHUD XPDVLWXDomRSUHH[LVWHQWHSRUpP
SURIHULGDSHODDXWRULGDGHFRPSHWHQWH
SUHVHUYDQGR RV GLUHLWRV RX REULJDo}HV FRQH[DV
H[HPSORWtSLFRpDDOWHUDomRGHKRUiULRVSHUFXU
,9 4XDQWRjIRUPDGRDWR VRVORFDLVGHUHXQL}HVHWF
D $WR
$WR6LPSOHVpRTXHVHDSHUIHLoRDSHODPDQLIHVWD
6LPSOHV
I $WR
$WR $EGLFDWLYR
$EGLFDWLYR  DEGLFDU TXHU GL]HU DEULU PmR
omRGDYRQWDGHGHXP~QLFRyUJmRXQLSHVVRDORX
QHVWHV DWRV Ki D DEGLFDomR GH XP GLUHLWR FRPR
FROHJLDGR
WRGD DEGLFDomR WDLV DWRV VmR LUUHYHUVtYHLV QmR
E $WR
$WR&RPSRVWRVHDSHUIHLoRDSHODPDQLIHVWDomRGH
&RPSRVWR SRGHPVHUUHWUDWDGRVRXPRGLILFDGRVGHRXWUR
YRQWDGH ~QLFD GH XP yUJmR PDV GHSHQGH GD ODGRFRPRDEULUPmRGHXPGLUHLWRTXHpS~EOLFR
YHULILFDomRSDUDVHWRUQDUH[HTXtYHOGDYHULILFDomR QRIXQGRpXPDWRGHDOLHQDomRWDOTXDORVDWRV
SRUSDUWHGHRXWURyUJmRGDUDWLILFDomRSRURXWUD DOLHQDWLYRVWDPEpPRVDWRVDEGLFDWLYRVGHYHPVHU
DXWRULGDGH DXWRUL]DGRVSRUOHLSUHYLDPHQWH
&kPDUD/HJLVODWLYDGR
  'LVWULWR)HGHUDO 'LU$GPLQLVWUDWLYR 

9,, 4XDQWRjH[HTXLELOLGDGH H $WR


$WRGH
GH-XULVGLomRFRQWpPXPDGHFLVmRDGPLQLV
-XULVGLomR
WUDWLYDVREUHDPDWpULDFRQWURYHUWLGD
D $WR
$WR 3HUIHLWRUH~QHWRGRVRVHOHPHQWRVSDUDVXD
3HUIHLWR
RSHUDWLYLGDGHVHQGRDSWRDSURGX]LUVHXVUHJXOD
UHVHIHLWRV
(VSpFLHV
E $WR
$WR ,PSHUIHLWRLQFRPSOHWRQDVXDIRUPDomRRX
,PSHUIHLWR
IDOWDOKHRDWRFRPSOHPHQWDUSDUDWRUQDUVHH[H
TXLYHO D $WRV1RUPDWLYRV

F $WR
$WR3HQGHQWHHPERUDSHUIHLWRQmRSURGX]DLQGD
3HQGHQWH &RPR GL] R SUySULR QRPH QRUPDWLYRV VmR RV DWRV
HIHLWRVSRLVQmRVHYHULILFRXRWHUPRRXFRQGLomR DGPLQLVWUDWLYRVTXHFRQWrPXPFRPDQGRJHUDO QRUPD
UHJUD  GR ([HFXWLYR  6mR HGLWDGRV FRP R REMHWLYR GH
G $WR
$WR &RQVXPDGR
&RQVXPDGR SURGX]LXWRGRVVHXVHIHLWRVp DOFDQoDUDFRUUHWDDSOLFDomRGDOHLSHORVyUJmRVHDJHQWHV
LUUHWUDWiYHORXLPRGLILFiYHO S~EOLFRV6mRWLSLFDPHQWHDWRVQRUPDWLYRVRVGHFUHWRV
RV UHJXODPHQWRV RV UHJLPHQWRV DV UHVROXo}HV H DV
9,,, 4XDQWRj5HWUDWDELOLGDGH GHOLEHUDo}HV

D $WR
$WR ,UUHYRJiYHO
,UUHYRJiYHO  Mi SURGX]LX VHXV HIHLWRV RX 2V GHFUHWRV VmR DWRV DGPLQLVWUDWLYRV HGLWDGRV
JHURXGLUHLWRVXEMHWLYRDREHQHILFLiULRRXDLQGD H[FOXVLYDPHQWHSHORVFKHIHVGRSRGHUH[HFXWLYR 3UHIHLWR
SRUUHVXOWDUGHFRLVDMXOJDGDDGPLQLVWUDWLYD *RYHUQDGRU 3UHV5HS~EOLFD  SDUD SRVVLELOLWDU D ILHO
DSOLFDomRGDOHJLVODomRHPJHUDO
E $WR 5HYRJiYHO
5HYRJiYHO  VmR RV DWRV LQYDOLGDGRV SHOD
$GPLQLVWUDomR H Vy HOD SRU TXHVWmR GH PpULWR 2VUHJXODPHQWRVWHPSRUPLVVmRH[SOLFDUHPLQXGHQFL
DGPLQLVWUDWLYR FRQYHQLrQFLD RSRUWXQLGDGH H DUDOHLWXGRGHQWURGRVOLPLWHVGDSUySULDOHL1mRSRGHP
MXVWLoD $UHYRJDomRRSHUDH[QXQFHRVHIHLWRVMi QHPDFRQWUDULDUQHPLUDOpPGRTXHOKHSHUPLWLXDOHL
SURGX]LGRVSUHYDOHFHUmR 1RUPDOPHQWH VmR SRVWRV HP YLJrQFLD DWUDYpV GH XP
GHFUHWR
F $WR
$WR6XVSHQVtYHOTXDQGRD$GPLQLVWUDomR3~EOLFD
6XVSHQVtYHO
ID]FHVVDURVHIHLWRVGRDWRHPGHWHUPLQDGDVFLU -i RV UHJLPHQWRV VmR DWRV GH DWXDomR LQWHUQD SDUD
FXQVWkQFLDVHSRUFHUWRWHPSR23RGHU-XGLFLiULR RUJDQL]DU R IXQFLRQDPHQWR GH yUJmRV FROHJLDGRV H GH
SRGH WDPEpP VXVSHQGHU OLPLQDUPHQWH XP DWR FRUSRUDo}HV OHJLVODWLYDV (
 GLUHFLRQDGR jTXHOHV TXH
DGPLQLVWUDWLYR GHYHPH[HFXWDURVHUYLoRRXUHDOL]DUDDWLYLGDGHIXQFLRQDO
UHJXODPHQWDGDQmRREULJDQGRDRVSDUWLFXODUHVHPJHUDO
,; 4XDQWRDRPRGRGHH[HFXomR
$VUHVROXo}HVVmRH[SHGLGDVSHODVDOWDVDXWRULGDGHGR
D $WR
$WR$XWR([HFXWyULRDTXHOHVTXHVmRH[HFXWDGRV
$XWR([HFXWyULR 3RGHU([HFXWLYR H[FHWRR&KHIHGR([HFXWLYRTXHGHYH
GLUHWDPHQWHSHODSUySULD$GPLQLVWUDomRLQGHSHQ VHYDOHUGHGHFUHWRV 
GHQWHPHQWHGHDXWRUL]DomRMXGLFLDO
1RUPDOPHQWH DV UHVROXo}HV VmR H[SHGLGDV SHORV
E $WR
$WR1®2
1®2$XWR([HFXWyULRDRFRQWUiULRGRRXWUR
$XWR([HFXWyULR 0LQLVWURV6HFUHWiULRVGH(VWDGRRX6HFUHWiULRV0XQLFLSD
HVWHVySRGHVHUH[HFXWDGRGHSRLVGHXPSURQXQFL LV6mRDWRVLQIHULRUHVDRVGHFUHWRVDRUHJXODPHQWRHDR
DPHQWRMXGLFLDO$FREUDQoDGDGtYLGDILVFDO FUp UHJLPHQWRQmRRVSRGHQGRFRQWUDULDU
GLWRV WULEXWiULRV  GRV GHYHGRUHV UHQLWHQWH SRU
H[HPSORVypSRVVtYHOFRPRDX[tOLRGR-XGLFLi )LQDOPHQWH DV GHOLEHUDo}HV VmR DWRV HGLWDGRV SHORV
ULR yUJmRVFROHJLDGRV6HWLYHUHPSRUREMHWRHVWDEHOHFHUXPD
UHJUD JHUDO VHUmR DWRV QRUPDWLYRV JHUDLV PDV VH FRQ
VXEVWDQFLDUHP XP MXOJDPHQWR VHUmR DWRV QRUPDWLYRV
; 4XDQWR
4XDQWRDR
DRREMHWLYR
REMHWLYRYLVDGR
YLVDGRSHOD
SHOD$GPLQLV LQGLYLGXDLV
WUDomR
WUDomR
E $WRV2UGLQDWyULRV
D $WR
$WR 3ULQFLSDO
3ULQFLSDO  HQFHUUD D YRQWDGH ILQDO GD $G
PLQLVWUDomR 2VDWRVDGPLQLVWUDWLYRVRUGLQDWyULRVWrPSRUSURSyVLWR
GLVFLSOLQDURIXQFLRQDPHQWRGD$GPLQLVWUDomR3~EOLFDH
E $WR
$WR &RPSOHPHQWDU
&RPSOHPHQWDU  DSURYD RX UDWLILFD R DWR DFRQGXWDIXQFLRQDOGRVDJHQWHV6mRDWRVHGLWDGRVSRU
SULQFLSDOWRUQDQGRRH[HTXtYHO &KHIHVSDUDVHUHPFXPSULGRVSHORVVHUYLGRUHVVXEDOWHU
QRVQRH[HUFtFLRGHVXDVIXQo}HV(QTXDGUDPVHQHVVD
F $WR
$WR,QWHUPHGLiULRFRQFRUUHSDUDDIRUPDomRGH
,QWHUPHGLiULR HVSpFLHDVLQVWUXo}HVDVFLUFXODUHVRVDYLVRVDVSRUWDULDV
XPDWRSULQFLSDOHILQDO DVRUGHQVGHVHUYLoRRVRItFLRVHRVGHVSDFKRV

G $WR&RQGLomRDQWHS}HVHDRXWURSDUDSHUPLWLU
$WR&RQGLomR F $WRV1HJRFLDLV
VXDUHDOL]DomR
6mR DWRV DGPLQLVWUDWLYRV TXH HPERUD XQLODWHUDLV
 'LU$GPLQLVWUDWLYR &kPDUD/HJLVODWLYDGR'LVWULWR)HGHUDO


SUDWLFDGRVDSHQDVSHOD$GPLQLVWUDomR3~EOLFD FRQWrP QXOR MDPDLV SRGHUi VHU VDOYR RX VHMD p LQVXVFHWtYHO GH
LQWHUHVVHUHFtSURFRGD$GPLQLVWUDomRHGRSDUWLFXODU FRQYDOLGDomR
HPERUDQmRFKHJXHPDVHUXPFRQWUDWR6mRDWRVDGPL
QLVWUDWLYRV FRQWHQGR XP GHFODUDomR GH YRQWDGH GD 2YLVFHUDOGHIHLWRGRDWRQXORFUHGHQFLDWDQWRR-XGLFLi
$GPLQLVWUDomR DSWD D FRQFUHWL]DU GHWHUPLQDGR QHJyFLR ULRFRPRDSUySULD$GPLQLVWUDomR3~EOLFDjSURFODPDomR
MXUtGLFR RX D GHIHULU FHUWD IDFXOGDGH DR SDUWLFXODU QDV GHVXDLQYDOLGDGHHLQHILFiFLD
FRQGLo}HVLPSRVWDVRXFRQVHQWLGDVSHOR3RGHU3~EOLFR
-i QR DWR DQXOiYHO R GHIHLWR QmR p GH WmR LQWHQVD
1HVVDHVSpFLHHQTXDGUDPVHDOLFHQoDDSHUPLVVmRD JUDYLGDGHHWDQWRTXDQWRSRGHUiVHUDQXODGRWDPEpP
DXWRUL]DomRDDGPLVVmRRYLVWRDDSURYDomRDKRPROR SRGHUiVHUFRQVHUWDGRVXSHUDQGRVHVHXGHIHLWRHFRPR
JDomRDGLVSHQVDHDUHQ~QFLD VDQHDPHQWRSUHVHUYDQGRVHVHXVHIHLWRV$OLiVPHVPR
TXHVHMDDQXODGRDRLQYpVGHVDQDGRRVHIHLWRVGRDWR
G $WRV(QXQFLDWLYRV DQXOiYHO SUHVHUYDUVHmR YiOLGRV DWp R PRPHQWR GD
DQXODomRYDOHGL]HUVXDDQXODomRQmRWHPHIHLWRVUHWURD
1HVWHVDWRVQmRKiXPDQRUPDRXXPDRUGHPQHP WLYRRSHUDQGRVHH[QXQF
XPDUHODomRQHJRFLDOHQWUHR3RGHU3~EOLFRHRSDUWLFXODU
1RVDWRVDGPLQLVWUDWLYRVHQXQFLDWLYRVFRPRGL]RSUySULR 1mRpGHPDVLDGDDOHPEUDQoDGHTXHSDUDR3URI',2
QRPHR3RGHU3~EOLFRDSHQDVDQXQFLDXPIDWRRXXPD *2'(),*8(,5('2025(,5$1(7´$&RQVWLWXLomRGH
RSLQLmR VREUH XP DVVXQWR 6mR DWRV DGPLQLVWUDWLYRV DRSUHYHUQRDUWLJRLQFLVR/;;,,,DDQXODomR
HQXQFLDWLYRVSRUH[HPSORDVFHUWLG}HVRVDWHVWDGRVHRV MXGLFLDO GH ´DWR OHVLYR DR SDWULP{QLR S~EOLFR RX GH
SDUHFHUHVDGPLQLVWUDWLYRV HQWLGDGHGHTXHR(VWDGRSDUWLFLSHDPRUDOLGDGHDGPLQLV
WUDWLYD DR PHLR DPELHQWH H DR SDWULP{QLR KLVWyULFR H
H $WRV3XQLWLYRV FXOWXUDOµGLODWRXXPSRXFRPDLVRFDPSRGDVQXOLGDGHV
VHP FRQWXGR LQWURGX]LU D ILJXUD GD DQXODELOLGDGH QR
1RVDWRVSXQLWLYRVKDYHUiVHPSUHDLPSRVLomRGHXPD 'LUHLWR $GPLQLVWUDWLYR 5HDOPHQWH VH VH WUDWDVVH GD
VDQomR SHQD  LPSRVWD SHOD $GPLQLVWUDomR 3~EOLFD DQXODELOLGDGH D VHQWHQoD MXGLFLDO GH DQXODomR VHULD
jTXHOHVTXHWLYHUHPLQIULQJLGRXPDSURLELomRGDOHLGR FRQVWLWXWLYDHILFDULDPUHVVDOYDGRVRVHIHLWRVSUHWpULWRVGR
UHJXODPHQWRRXGHGLVSRVLomRRUGLQDWyULD DWR H[QXQF RTXHQmRDWHQGHULDjILQDOLGDGHLPSOtFLWD
QDTXHODQRUPDFRQVWLWXFLRQDOTXHpGHYDUUHUGRPXQGR
2REMHWLYRpSXQLUHUHSULPLURVLQIUDWRUHVWDQWRQR MXUtGLFRRVDWRVHQFRQWUDGRVFRPRVPHQFLRQDGRVYtFLRV
SODQR LQWHUQR GLVFLSOLQD IXQFLRQDO  FRPR QR SODQR VHPSHUPLWLUOKHVDVREUHYLYrQFLDGHQHQKXPHIHLWR H[
H[WHUQR PXOWDVSH[ 6mRDWRVDGPLQLVWUDWLYRVWLSLFD WXQF  $VVLP HPERUD D PRUDOLGDGH DGPLQLVWUDWLYD SRU
PHQWH SXQLWLYRV D PXOWD D LQWHUGLomR GH DWLYLGDGH D H[HPSOR SRVVD HQYROYHU D DSUHFLDomR GH HOHPHQWR
GHVWUXLomRGHFRLVDVHRDIDVWDPHQWRGRFDUJRRXIXQomR VXEMHWLYR GR DJHQWH D ILJXUD VDQFLRQDWyULD GH TXH VH
FRJLWDHP'LUHLWR$GPLQLVWUDWLYRpVHPSUHDQXOLGDGHµ³
I $WRVGH&RQKHFLPHQWR LQ&XUVRGH'LUHLWR$GPLQLVWUDWLYR
&XUVRGH'LUHLWR$GPLQLVWUDWLYRHG)RUHQVHS 
&XUVRGH'LUHLWR$GPLQLVWUDWLYR

2VHVWXGLRVRVGH'LUHLWR$GPLQLVWUDWLYRUHFRQKHFHUDP 0DVD'RXWULQDGHXPPRGRJHUDOUHFRQKHFHWDQWRD
HVVDQRYDHVSpFLHGHDWRVVmRRVDWRVUHJLVWUDLVRXVHMD QXOLGDGHDEVROXWDGRDWRDGPLQLVWUDWLYRFRPRVXDDQXODEL
DTXHOHV TXH D DGPLQLVWUDomR S~EOLFD SUiWLFD FRP R OLGDGH QXOLGDGHUHODWLYD 
SURSyVLWRGHSRVVLELOLWDUHPRFRQKHFLPHQWRGHIDWRVRX
DWRV SRU WRGRV LQWHUHVVDGRV 6mR UHJLVWURV IHLWRV SHOD 3DUD&(/62$17Ñ1,2%$1'(,5$'(0(//2SH[
DGPLQLVWUDomRS~EOLFDTXHSRUVHUHPS~EOLFRVSRGHP QXORV D RVDWRVTXHDOHLDVVLPRVGHFODUHH E RV
´6mRQXORV
QXORV
VHUFRQVXOWDGRVDWRGRWHPSRLQFOXVLYHSDUDSURGXomRGH DWRV HP TXH p UDFLRQDOPHQWH LPSRVVtYHO D FRQYDOLGDomR
SURYDGDH[LVWrQFLDGHDWRVHIDWRV SRLV VH R PHVPR FRQWH~GR p GL]HU R PHVPR DWR  IRVVH
QRYDPHQWH SURGX]LGR VHULD UHSURGX]LGD D LQYDOLGDGH
DQWHULRU6LUYDPGHH[HPSORRVDWRVGHFRQWH~GR REMHWR
LOtFLWRRVSUDWLFDGRVFRPGHVYLRGHSRGHURVSUDWLFDGRVFRP
,QYDOLGDomRGR$WRV$GPLQLVWUDWLYR
IDOWDGHPRWLYRYLQFXODGRRVSUDWLFDGRVFRPIDOWDGHFDXVD
6mRDDQXOiYHLV D RVTXHDOHLDVVLPRVGHFODUH E RVTXH
QXOiYHLV
SRGHPVHUUHSUDWLFDGRVVHPYtFLR6LUYDPGHH[HPSORRV
$GLVWLQomRHQWUHDWRQXORHDQXOiYHOpIHLWDHPIXQomR DWRVH[SHGLGRVSRUVXMHLWRLQFRPSHWHQWHRV HGLWDGRVFRP
GDQDWXUH]DGHVHXGHIHLWRHDLQGDHPUD]mRGDSURGX YtFLRGHYRQWDGHRVSURIHULGRVFRPGHIHLWRGHIRUPDOLGDGHµ
omRRXQmRGHDOJXPHIHLWR³TXDOGDSRVVLELOLGDGHGHVXD ³>LQ&XUVRGH'LUHLWR$GPLQLVWUDWLYR0DOKHLURV(GLWRUHVHG
FRQYDOLGDomR S@

2DWRQXORpGHWmRJUDYHLOHJtWLPLGDGHHLOHJDOLGDGHTXH ,QLFLDWLYDGD$GPLQLVWUDomR
QmRLUUDGLDUiHOHQHQKXPHIHLWRGDtGHFRUUHQGRTXHVXD
QmR
DQXODomRVHRSHUDUiH[WXQFLVWRpUHWURDJHDWpDRULJHP &XPSUHUHDOoDUTXHQmRREVWDQWHVHMDGHVXDUHVSRQVD
GR DWR LPSHGLQGR R UHFRQKHFLPHQWR GH KDYHU GHOH ELOLGDGHDSUiWLFDGRDWRGHIHLWXRVRpOHJtWLPDHPRUDOD
UHVXOWDGR TXDOTXHU FRQVHTrQFLD 0DLV TXH LVVR R DWR LQLFLDWLYD GD $GPLQLVWUDomR 3~EOLFD SDUD UHWLUiOR GR
PXQGRMXUtGLFRHOLYUDUVHGHVHXVPDOHItFLRV
&kPDUD/HJLVODWLYDGR
  'LVWULWR)HGHUDO 'LU$GPLQLVWUDWLYR 

 3RQGHUD R 3URI+(/< /23(6 0(,5(//(6 TXH ´$ GDTXHOH DWR  9HMDVH SRUWDQWR TXH R PpULWR GR DWR
$GPLQLVWUDomR 3~EOLFD FRPR LQVWLWXLomR GHVWLQDGD D DGPLQLVWUDWLYRVypYLQFXODGRjYRQWDGHGR$GPLQLVWUD
UHDOL]DURGLUHLWRHDSURSLFLDUREHPFRPXPQmRSRGH GRUVHVHWUDWDUGHDWRGLVFULFLRQiULR
DJLUIRUDGDVQRUPDVMXUtGLFDVHGDPRUDODGPLQLVWUDWLYD
QHPUHOHJDURVILQVVRFLDLVDTXHVXDDomRVHGLULJH6H eTXHQRVDWRVYLQFXODGRVRDGPLQLVWUDGRUQmRYDORUD
SRUHUURFXOSDGRORRXLQWHUHVVHVHVFXVRVGHVHXVDJHQ QHPRPRWLYRQHPRREMHWRGRDWR3UDWLFiORSRUTXHDOHL
WHVDDWLYLGDGHGR3RGHU3~EOLFRVHGHVJDUUDGDOHLVH PDQGDTXHHOHRSUDWLTXHPHVPRTXHHOH DGPLQLVWUDGRU
GLYRUFLDGDPRUDORXVHGHVYLDGREHPFRPXPpGHYHU QmR JRVWH GR DWR 3RU H[HPSOR VH XP 3UHIHLWR GHFLGH
GD$GPLQLVWUDomRLQYDOLGDUHVSRQWDQHDPHQWHRXPHGLDQ FRQVWUXLUXPDSRQWHHOHDYDOLDUiVHDTXHODSRQWHVHUiRX
WHSURYRFDomRRSUySULRDWRFRQWUiULRjVXDILQDOLGDGH QmR ERD SDUD D SRSXODomR EHQHILFLiULD VHUi PXLWR RX
SRULQRSRUWXQRLQFRQYHQLHQWHLPRUDORXLOHJDO6HRQmR SRXFRGLVSHQGLRVRVHXFXVWRSDUDRVFRIUHVGDDGPLQLVWUD
IL]HU D WHPSR SRGHUi R LQWHUHVVDGR UHFRUUHU jV YLDV omR VH QR PRPHQWR Ki FDL[D VXILFLHQWH RX QmR SDUD
MXGLFLiULDV DTXHOHLQYHVWLPHQWRHWF

$EUHPVHDVVLPGXDVRSRUWXQLGDGHVSDUDRFRQWUROH 7RGDV HVVDV DYDOLDo}HV VHUmR IHLWDV H[FOXVLYDPHQWH


GRVDWRVDGPLQLVWUDWLYRVXPDLQWHUQDGDSUySULD$GPL SHOR3UHIHLWR,VVRpRPpULWRGRDWRDGPLQLVWUDWLYRGH
QLVWUDomRRXWUDH[WHUQDGR3RGHU-XGLFLiULR GHFLGLUPDQGDUFRQVWUXLUDSRQWH

$IDFXOGDGHGHLQYDOLGDomRGRVDWRVDGPLQLVWUDWLYRV
SHODSUySULD$GPLQLVWUDomRpEHPPDLVDPSODTXHDTXH ,QGHQL]DomRGRV0DOHItFLRV
GRV
VH FRQFHGH j -XVWLoD FRPXP $ $GPLQLVWUDomR SRGH
$WRV1XORV$QXOiYHLV
GHVID]HUVHXVSUySULRVDWRVSRUFRQVLGHUDo}HVGHPpULWR
H GH LOHJDOLGDGH DR SDVVR TXH R -XGLFLiULR Vy RV SRGH
,QHVFRQGLYHOPHQWHWDQWRGRVDWRVQXORVFRPRGRVDWRV
LQYDOLGDUTXDQGRLOHJDLV'RQGHVHGL]HUTXHD$GPLQLVWUD
DQXOiYHLV SRGHUmR UHVXOWDU PDOHItFLRV QmR Vy j SUySULD
omRFRQWURODRVVHXVSUySULRVDWRVHPWRGDSOHQLWXGHLVWR
$GPLQLVWUDomR FRPR DRV SDUWLFXODUHV HP JHUDO ¢4XLG
pVRERVDVSHFWRVGDRSRUWXQLGDGHFRQYHQLrQFLDMXVWLoD
MXULVTXDQWRDWDLVGDQRV"
FRQWH~GR IRUPD ILQDOLGDGH PRUDOLGDGH H OHJDOLGDGH
HQTXDQWRRFRQWUROHMXGLFLiULRVHUHVWULQJHDRH[DPHGD
'HLQtFLRFXPSUHFRQVLGHUDUGLVWLQWLYDPHQWHDERDIp
OHJDOLGDGHRXVHMDGDFRQIRUPDomRGRDWRFRPRRUGHQD
GRDGPLQLVWUDGRHVXDHYHQWXDOFRQWULEXLomRSDUDRYtFLR
PHQWRMXUtGLFRDTXHD$GPLQLVWUDomRVHVXERUGLQDSDUD
GRDWRIXOPLQDGR³DVSHFWRVTXHUHIOHWLUmRVXDFXOSD ODWR
DVXDSUiWLFDµ ³ LQ'LUHLWR
'LUHLWR
'LUHLWR $GPLQLVWUDWLYR
$GPLQLVWUDWLYR %UDVLOHLURHG57
%UDVLOHLUR
S 
VHQVX HSRGHUmRDWpDIDVWDU VHQmRPLQLPL]DU VHXGLUHLWR
jUHSDUDomRSHODOHVmRGHGLUHLWRVRIULGD
,QLFLDWLYDGH7HUFHLURV
3RQGHUD&(/62$17Ñ1,2%$1'(,5$'(0(//2VHU
$ SHVVRD LQWHUHVVDGD SRGHUi SURPRYHU XPD DomR HYLGHQWH TXH ´D LQYDOLGDomR QmR OKH SRGHULD FDXVDU XP
RUGLQiULDSHGLQGRDR-XGLFLiULRTXHDQDOLVHDYDOLGDGHGR GDQRLQMXVWRHPXLWRPHQRVVHULDWROHUiYHOTXHSURSLFLDV
DWR DGPLQLVWUDWLYR H R GHFODUH QXOR RX VHMD LPSHoD D VHHYHQWXDOPHQWHXPHQULTXHFLPHQWRVHPFDXVDSDUDD
SURGXomRGHVHXVHIHLWRV0DVRVUHPpGLRVPDLVXVDGRV $GPLQLVWUDomR $VVLP WDQWR GHYHP VHU LQGHQL]DGDV DV
SDUD WDO ILP VmR R PDQGDGR GH VHJXUDQoD H D DomR GHVSHVDVGHVWDUWHHIHWXDGDVFRPRDIRUWLRULKmRGHVHU
SRSXODU&RPRSULPHLURRSDUWLFXODU SHVVRDLQWHUHVVD UHVSHLWDGRV HIHLWRV SDWULPRQLDLV SDVVDGRV DWLQHQWHV j
GD DSHQDVFRQVHJXHLPSHGLUTXHRDWRWHQKDYDOLGDGH UHODomRDWLQJLGD6HJXHVHWDPEpPTXHVHRDGPLQLVWUD
FRQWUDHODHDVVLPGHL[DGHREHGHFHUjTXHOHDWR-iFRP GR HVWi D GHVFREHUWR HP UHODomR D SDJDPHQWRV TXH D
DDomRSRSXODUXPDSHVVRDTXHQmRpGLUHWDPHQWHLQ $GPLQLVWUDomRDLQGDQmROKHHIHWXRXPDVTXHFRUUHVSRQ
WHUHVVDGD EXVFD D GHFODUDomR GD QXOLGDGH GR DWR SDUD GLDPDSUHVWDo}HVSRUHOHMiFRQVXPDGDVD$GPLQLVWUD
WRGR PXQGR RX VHMD TXH R DWR VHMD DQXODGR H QmR omRQmRSRGHUiH[LPLUVHGHDFREHUWiODVLQGHQL]DQGRR
SURGX]DQHQKXPHIHLWR SRUHODV

$WHQomR FRPRMiIRLGLWRDQWHVR3RGHU-XGLFLiULRQmR
QmR &RPHIHLWRVHRDWRDGPLQLVWUDWLYRHUDLQYiOLGRLVWR
SRGH
SRGH DQDOLVDURXTXHVWLRQDURPpULWRGRDWR
DQDOLVDU VLJQLILFDTXHD$GPLQLVWUDomRDRSUDWLFiORIHULXDRUGHP
DGPLQLVWUDWLYR  6HX H[DPH VHUi UHVWULWR j MXUtGLFD $VVLP DR LQYDOLGDU R DWR HVWDUi LSVR IDFWR
OHJDOLGDGHGRDWR SURFODPDQGRTXHIRUDRXWURUDGHXPDYLRODomRGDRUGHP
MXUtGLFD6HULDLQtTXRTXHRDJHQWHYLRODGRUGR'LUHLWR
FRQIHVVDQGRVH WDO VH OLYUDVVH GH TXDLVTXHU {QXV TXH
,QYDOLGDomR;0pULWR
,QYDOLGDomR; GHFRUUHULDP GR DWR H ODQoDVVH VREUH DV FRVWDV DOKHLDV
WRGDV DV FRQVHTrQFLDV SDWULPRQLDLV JUDYRVDV TXH GDt
2PpULWRGRDWRDGPLQLVWUDWLYRQmR QmRpUHTXLVLWRSDUDVXD
QmR GHFRUUHULDPORFXSOHWDQGRVHDLQGDjFXVWDGHTXHPQmR
YDOLGDGHRXQmR&RQVLVWHHOHQDYDORUDomRGRVPRWLYRV WHQGRFRQFRUULGRSDUDRYtFLRKDMDSURFHGLGRGHERDIp
H QD HVFROKD GR REMHWR GR DWR FRQIRUPH D YRQWDGH GD $FUHVFHTXHQRWRULDPHQWHRVDWRVDGPLQLVWUDWLYRVJR]DP
$GPLQLVWUDomRTXHRSUDWLFDHVHWLYHUHODVLGRSHUPLWLGD GH SUHVXQomR GH OHJLWLPLGDGH 'RQGH TXHP DWXRX
D GHFLGLU VREUH D FRQYHQLrQFLD RSRUWXQLGDGH H MXVWLoD DUULPDGR QHOHV VDOYR VH HVWDYD GH PiIp YtFLR TXH VH
SRGH SURYDU PDV QmR SUHVVXSRU OLPLQDUPHQWH  WHP R
 'LU$GPLQLVWUDWLYR &kPDUD/HJLVODWLYDGR'LVWULWR)HGHUDO


GLUHLWRGHHVSHUDUTXHWDLVDWRVVHUHYLVWDPGHXPPtQLPR (VWH YtFLR GHVYLR GH ILQDOLGDGH DEXVR GH SRGHU RX


GHVHULHGDGH(VWHPtQLPRFRQVLVWHHPQmRVHUHPFDXVDV DLQGDGHVYLRGHSRGHUUHFHEHGHQRPLQDo}HVDQiORJDVHP
SRWHQFLDLVGHIUDXGHDRSDWULP{QLRGHTXHPQHOHVFRQILRX TXDVH WRGRV RV VLVWHPDV GH 'LUHLWR $GPLQLVWUDWLYR
FRPRGHUHVWRWHULDGHFRQILDU VYLDPHQWR GLSRWHUH GpWRXUQHPHQW GH SRXYRLU DEXVH RI
GLVFUHWLRQGHVYLDFLyQGHSRGHUHWFH[SUHVV}HVTXHLGHQWLIL
$OLiV D VROXomR TXH VH YHP GH DSRQWDU QDGD PDLV FDP R PHVPR LQVWLWXWR GH QRWiYHO DOFDQFH PRUDOL]DGRU
UHSUHVHQWDVHQmRXPDDSOLFDomRFRQFUHWDGRGLVSRVWRQR TXHQRGL]HUGH&UHWHOODFRQVLVWHQXPDIDVWDPHQWRGR
DUW  † ž GD &RQVWLWXLomR QR TXDO R SULQFtSLR GD HVStULWRGDOHL³DEHUUDWLRILQLVOHJLV
UHVSRQVDELOLGDGH GR (VWDGR YLGH &DStWXOR ;9,,  HVWi 
FRQVDJUDGR GH PDQHLUD DPSOD H JHQHURVD GH VRUWH D
DEUDQJHUWDQWRUHVSRQVDELOLGDGHSRUDWRVLOtFLWRVTXDQWR 1XOLGDGHSRUGHIHLWRGHIRUPD
SRUDWRVOtFLWRV FRPRVHULDDFRUUHWDIXOPLQDomRGHDWRV
LQYiOLGRV µ³>LQ&XUVRGH'LUHLWR$GPLQLVWUDWLYR0DOKHLURV(GLWRUHV  YtFLR FRQVLVWLUi QD RPLVVmR RX Pi REVHUYkQFLD GR
HGS@ UHTXLVLWR H[WHUQR HVVHQFLDO SUHYLVWR HP OHL +i TXH
GLVWLQJXLUVHDVVLPDIRUPDHVVHQFLDOGDIRUPDFRPSOH
PHQWDU)RUPDHVVHQFLDOpDTXHODH[LJLGDSDUDDH[WHULRUL
]DomR GRV HOHPHQWRV FRQVWLWXWLYRV GR DWR FRQIHULQGR
9tFLRVGR$WR$GPLQLVWUDWLYR
FHUWH]DGHVXDH[LVWrQFLDHQTXDQWRTXHDFRPSOHPHQWDU
GHVWLQDVHDFHUFiORGHPHOKRUHVFRQGLo}HVGHVHJXUDQoD
6H Ki UHTXLVLWRV LPSUHVFLQGtYHLV j IRUPDomR GR DWR HFODUH]D(PFRQVHTrQFLDDIRUPDHVVHQFLDOpVHPSUH
DGPLQLVWUDWLYRSRUyEYLRTXHRGHIHLWRGHTXDOTXHUGHOHV QHFHVViULDDRDWLQJLPHQWRGRREMHWLYRDRSDVVRTXHXPD
UHIOHWLUiRYLFLDPHQWRGRDWRFRPRXPWRGR$VVLPVHD SUHWHULomR GD IRUPD FRPSOHPHQWDU QmR LPSOLFDUi HP
YDOLGDGHGRDWRDGPLQLVWUDWLYRpFRQGLFLRQDGDjFRPSHWrQ
YDOLGDGH QXOLGDGHVHRDWRKRXYHUDWLQJLGRVHXREMHWLYR
FLDjILQDOLGDGHjIRUPDDRPRWLYRHDRREMHWR³RYtFLR
GHTXDOTXHUGHVVDVSUHPLVVDVFRQGX]LUijQXOLILFDomRGR 1RFDVRGHIRUPDOLGDGHHVVHQFLDOVXDSUHWHULomRJHUD
DWR QXOLGDGH QR FDVR GH IRUPDOLGDGH FRPSOHPHQWDU VXD
  3HUWLQHQWH WUD]HUVH j FRODomR D DQiOLVH GR 3URI SUHWHULomR FRQVLVWLUi HP PHUD LUUHJXODULGDGH (LV XPD
',2*2'(),*8(,5('2025(,5$1(7VREUHFDGDXP DSOLFDomR SUiWLFD GR SULQFtSLR GD ILQDOLGDGH HP 'LUHLWR
YtFLRVGRDWRDGPLQLVWUDWLYR
GHVVHYtFLRV
YtFLRV $GPLQLVWUDWLYR

´1XOLGDGHSRUGHIHLWRGHFRPSHWrQFLD 1XOLGDGHSRUGHIHLWRGHPRWLYR

$ PDQLIHVWDomR GH YRQWDGH GH DJHQWH RX GH yUJmR 4XDQWRDHVWHHOHPHQWRpSUHFLVRGLVWLQJXLUDVKLSyWH


DGPLQLVWUDWLYRVHMDHOHPRQRFUiWLFRRXFROHWLYRVyWHUi VHV HP TXH D $GPLQLVWUDomR HVWi YLQFXODGD D PRWLYRV
YDORU VH WLYHU VLGR SUHYLVWD HP WHVH SHOR OHJLVODGRU SUHYLVWRVHPOHLGDTXHODVHPTXHHODSRGHDYDOLiORVH
6RPHQWHDOHLFULDHDWULEXLFRPSHWrQFLDDGPLQLVWUDWLYD FRQVLGHUiORV GLVFULFLRQDULDPHQWH RX VHMD GHQWUR GH
$SUiWLFDGHXPDWRDGPLQLVWUDWLYRVHPREVHUYkQFLDGD OLPLWHVDEHUWRVSHODSUySULDOHL
UHJUD GH FRPSHWrQFLD SRGHUi LQTXLQiOR GRV VHJXLQWHV
YtFLRVXVXUSDomRDEXVRRXLQYDVmRGHFRPSHWrQFLDRXGH $VVLPVHDOHLHVWDEHOHFHVREUHTXHIXQGDPHQWRVHHP
SRGHUHV IDFHGHTXHFLUFXQVWkQFLDVGHYHUiRXSRGHUiDJLUR3RGHU
3~EOLFRWHUHPRVRPRWLYRYLQFXODGR
$FDSDFLGDGHFLYLOGRDJHQWHEHPFRPRRVYtFLRVGD
YRQWDGHTXHOKHSRVVDPVHUDWULEXtGRVSRURXWURODGR  PRWLYR YLQFXODGR VHUi UD]mR QHFHVViULD SDUD DJLU
QmR WrP UHSHUFXVVmR LPHGLDWD VREUH D YDOLGDGH GR DWR HPERUDSRVVDQmRVHUVXILFLHQWH4XDOTXHURXWURPRWLYR
DGPLQLVWUDWLYRSRLVRDJHQWHQmRpDXWRUGRDWRQHPp TXHQmRRYLQFXODGRDFDUUHWDDQXOLGDGHGRDWRLQFOXVLYH
VXDDYRQWDGHFRQVWLWXWLYDVHQmRTXHDJHSHOR(VWDGRH RLQVXILFLHQWHRLQDGHTXDGRHFRPPDLRUUD]mRRIDOVR
GHOHpDYRQWDGHTXHFRQVWLWXLDUHODomRMXUtGLFD
  3RURXWURODGRVHDOHLDEUHj$GPLQLVWUDomRDDYDOLD
omRGDRSRUWXQLGDGHHGDFRQYHQLrQFLDGHDJLUWHPRVR
1XOLGDGHSRUGHIHLWRGHILQDOLGDGH PRWLYR GLVFULFLRQiULR  PRWLYR GLVFULFLRQiULR p DSHQDV
XPD UD]mR SDUD DJLU QHP QHFHVViULD H PXLWR PHQRV
$DWLYLGDGHDGPLQLVWUDWLYDHVWiFRQGLFLRQDGDjSURV VXILFLHQWH PDV GHYH VHU UD]RiYHO UHODWLYDPHQWH DRV
VHFXomRGHUHVXOWDGRVGHLQWHUHVVHS~EOLFRpRTXH6HDEUD REMHWLYRVSUHWHQGLGRV
)DJXQGHVFKDPDGHLQWHQomROHJDOGRDWRGHIHLWRGH 
OHJDOLGDGH TXDQWR D HVWH HOHPHQWR VHUi SRUWDQWR D
WUDLomR GDTXHOD LQWHQomR OHJDO GiVH TXDQGR R DJHQWH 1XOLGDGHSRUGHIHLWRGHREMHWR
GHVYLDURSRGHUGHYHUGHDJLUGHTXHHVWiLQYHVWLGRSDUD
SURVVHJXLURXWURLQWHUHVVHTXHQmRRS~EOLFRYLVDQGRD 7DQWRTXDQWRTXDOTXHUDWRMXUtGLFRRDWRDGPLQLVWUDWL
XPDILQDOLGDGHGLIHUHQWHGDTXHODTXHHVWDYDtQVLWDHFRPR YRGHPDQGDXPREMHWRSRVVtYHOHOtFLWR7RGDYLDFRPR
GHWHUPLQDQWHHPVXDFRPSHWrQFLD H[DPLQDPRV TXDQWR DR HOHPHQWR DQWHULRU R PRWLYR p
SUHFLVRWHUHPOLQKDGHFRQWDDGLVWLQomRHQWUHRREMHWR
&kPDUD/HJLVODWLYDGR
  'LVWULWR)HGHUDO 'LU$GPLQLVWUDWLYR 

YLQFXODGRTXHpDTXHOHIL[DGRSHODOHLHRREMHWRGLVFULFL WRUQDPLOHJDLVSRUTXHGHOHVQmRVHRULJLQDPGLUHLWRV³RX
RQiULRTXHpDTXHOHTXHSRGHUiHOHJHUD$GPLQLVWUDomR UHYRJiORV SRU PRWLYR GH FRQYHQLrQFLD RX RSRUWXQLGDGH
HQWUHYiULRVHIHLWRVMXUtGLFRVSRVVtYHLVGHQWURGRVOLPLWHV UHVSHLWDGRVRVGLUHLWRVDGTXLULGRVHUHVVDOYDGDHPWRGRVRV
OHJDLV FDVRVDDSUHFLDomRMXGLFLDOµ

6HDOHLGHWHUPLQDTXDOGHYDVHURREMHWRGHXPDWR
DGPLQLVWUDWLYRQmRFDEHj$GPLQLVWUDomRVHQmRDFDWiOD
HVWULWDPHQWH VHP TXDOTXHU DOWHUDomR SRVVtYHO 8PD ([WLQomR
DOWHUDomRTXDOLWDWLYDGHXPREMHWRYLQFXODGRDFDUUHWDD
QXOLGDGH GR DWR SUDWLFDGR 8PD DOWHUDomR TXDQWLWDWLYD $QRWDR3URI',2*2'(),*8(,5('2025(,5$1(72
GHYHUiFRQVLGHUDURVUHVXOWDGRVHIHWLYRVRXSURYiYHLVVH TXH (P SULQFtSLR D UHODomR MXUtGLFD DGPLQLVWUDWLYD p
UHGX]LU R REMHWR QmR HVWDUi QXOR R DWR VH VH SXGHU FRQVWLWXtGD RX SDUD GXUDU LQGHWHUPLQDGDPHQWH RX FRP
FRUULJLORGHPRGRDDWHQGHLSOHQDPHQWHDRGHWHUPLQDGR YLJrQFLDGHWHUPLQDGD6XFHGHTXHFHUWDVFRQVLGHUDo}HVGH
SHODOHLVHDPSOLDURREMHWRHVWDUiSDUFLDOPHQWHQXORQR QDWXUH]DMXUtGLFDRXIiWLFDSRGHUmRDFDUUHWDUXPDGHVFRQV
H[FHVVRGHREMHWRVHVHSXGHUHVFRLPiOR WXGRVHJXQGR WLWXLomRH[WUDRUGLQiULD$VVLPpTXHRUDDGHVFRQVWLWXLomR
DUHJUDXWLOHSHULQXWLOHQRQYLWLDWXU µ³ LQ&XUVR
&XUVR
&XUVRGH
GH'LUHLWR GH DSUHVHQWDUi FRPR DFRQWHFLPHQWR QRUPDO
QRUPDO
UPDO QD YLGD GDV
$GPLQLVWUDWLYRHG)RUHQVHS 
$GPLQLVWUDWLYR
UHODo}HVMXUtGLFDVDGPLQLVWUDWLYDVVHFULDGDVFRPYLJrQFLD
GHWHUPLQDGDRUDFRPRDFRQWHFLPHQWRDQRUPDO
DQRUPDOVHDTXHODV
DQRUPDO
HPYLJrQFLDIRUHPUHWLUDGDVGRPXQGRMXUtGLFRVHMDSRUXP
'HVID]LPHQWRGR$WR$GPLQLVWUDWLYR
DWR GD SUySULD $GPLQLVWUDomR VHMD FRPR VH H[DPLQDUi
RSRUWXQDPHQWH GR 3RGHU -XGLFLiULR LQ &XUVR GH 'LUHLWR
$YLQFXODomRGRDWRDGPLQLVWUDWLYRDRLQWHUHVVHS~EOLFR
$GPLQLVWUDWLYRHG)RUHQVHS 
ID]FRPTXHWRGRVHVWHMDPVHPSUHDWHQWRVjVXDYDOLGDGH
H HILFLrQFLD DOpP GD FRQYHQLrQFLD RSRUWXQLGDGH H
1RXWUDVSDODYUDVKDYHUiDH[WLQomR
H[WLQomRGDUHODomRMXUtGLFD
H[WLQomR
OHJLWLPLGDGH
DGPLQLVWUDWLYDRXVHRDWRDGPLQLVWUDWLYRIRUGHFODURQXOR
SHODSUySULD$GPLQLVWUDomR3~EOLFDRXVHHVWDRUHYRJDU
3DUWLFXODUPHQWH QR TXH UHIHUH j HILFLrQFLD H j
1LVVRFRQVLVWLUiRGHVID]LPHQWR
GHVID]LPHQWRGRDWRDGPLQLVWUDWLYR1D
GHVID]LPHQWR
FRQYHQLrQFLDRSRUWXQLGDGHRSUySULR3RGHU3~EOLFRPDQ
DQXODomRD$GPLQLVWUDomR3~EOLFDGHVIDUi
GHVIDUiRDWRSRUQHOH
GHVIDUi
WHPSHUPDQHQWHYLJLOkQFLDVREUHRDWRDGPLQLVWUDWLYR6H
HQ[HUJDUGHIHLWRGHOHJDOLGDGHHDSHQDVRUHYRJDUiTXDQ
SHUFHEHUTXHRDWRDGPLQLVWUDWLYR QmRPDLVYHPVHQGR
mR
GRHQWHQGHUTXHVHXPpULWRGHYHVHUUHDYDOLDGR
HILFLHQWH RX GHL[RX GH VHU GH FRQYHQLHQWH D SUySULD
DXWRULGDGHDGPLQLVWUDWLYDSRGHUiSRUILPjYLGDGRDWR
DXWRULGDGH
7DPEpPVHRSHUDUiDH[WLQomRGDUHODomRDGPLQLVWUDWL
DGPLQLVWUDWLYR ,VVR VH GDUi FRP D UHYRJDomR GR DWR Mi
YDVHDQXOLGDGHIRUVHQWHQFLDGDSHOR3RGHU-XGLFLiULR
SUDWLFDGR

$UUHYRJDomRpRGHVID]LPHQWRGRDWRDGPLQLVWUDWLYR
HYRJDomR
$&RQYDOLGDomRGR$WR$GPLQLVWUDWLYR
eDVXSUHVVmRGHXPDWRDGPLQLVWUDWLYROHJtWLPRHHILFD]
UHDOL]DGDSHODSUySULD$GPLQLVWUDomR3~EOLFD HVRPHQWH
6LPSOLVWDPHQWHDVDQDWyULDGRDWRDGPLQLVWUDWLYRpVHX
SRUHOD SRUQmRPDLVOKHFRQYLUDVXDH[LVWrQFLD
FRQVHUWRHDSURYHLWDPHQWRRXVHMDDVXSHUDomRGHVHXV
GHIHLWRV SDUD TXH VH DOFDQFH VXD YDOLGDGH HILFiFLD H
1RFRQFHLWRGR3URI',2*2'(),*8(,5('2025(,
H[HTLELOLGDGH
5$1(72$UHYRJDomRpGHVVDUWHXPDWRGHVFRQVWLWXWLYR
PHOKRUGLWRFRQVWLWXWLYRQHJDWLYRHPTXHD$GPLQLVWUDomR
1DWXUDOPHQWHDSHQDVRVDWRVDQXOiYHLVVmRSDVVtYHLVGH
HVyHODUHWLUDDHILFiFLDGHXPDWRDQWHFHGHQWHSRUFRQVLGH
FRQYDOLGDomRMiTXHDLPSUHVWDELOLGDGHDEVROXWDGRDWR
UDo}HVH[FOXVLYDPHQWHDGPLQLVWUDWLYDVQXQFDVMXUtGLFDV3RU
QXORLPSHGHVHXUHDSURYHLWDPHQWR
LVWRpTXHR3RGHU-XGLFLiULRDJLQGRQRH[HUFtFLRGRSRGHU
MXGLFDQWH QmR SRGH UHYRJDU SRLV VXD iUHD GH DWXDomR
23URI+(/</23(60(,5(//(6UHJLVWUDTXH2TXHD
FLUFXQVFUHYHVHDRWHPDMXUtGLFRH[DPLQDQGRHVWULWDPHQWH
GRXWULQDDGPLWHpDFKDPDGDFRQYHUVmRRX VDQDWyULDGH
DQDWyULD
R ELQ{PLR OHJDOLGDGH H LOHJDOLGDGH HP &XUVR GH 'LUHLWR
$GPLQLVWUDWLYRHG)RUHQVHS 
DWR DGPLQLVWUDWLYR LPSUHVWiYHO SDUD XP GHWHUPLQDGR
QHJyFLRMXUtGLFRPDVDSURYHLWiYHOHPRXWURSDUDRTXDO
$WHQomR
$WHQomR WHQWHVHDTXHVRPHQWHDSUySULD$GPLQLVWUD WHPRVQHFHVViULRVUHTXLVLWRVOHJDLV([HPSOLILFDQGRXPD
omR3~EOLFDSRGHUHYRJDURDWRDGPLQLVWUDWLYR OLFHQoDSDUDHGLILFDomRGHILQLWLYDQXODFRPROLFHQoDSRGHUi
2 -XGLFLiULR MDPDLV
DPDLV SRGHUi UHYRJDU XP DWR VHU DFHLWD H YDOLGDGH FRPR DXWRUL]DomR SDUD HGLILFDomR
DGPLQLVWUDWLYR SURYLVyULD&RQYHUWHVHDVVLPRDWRQXORSDUDXPHIHLWR
SDUDRTXDOOKHIDOWDPRVUHTXLVLWRVOHJDLVQXPDWRYiOLGR
,QLFLDWLYD SDUDRXWURHIHLWRHPUHODomRDRTXDODSUHVHQWDRVQHFHVViUL
RVUHTXLVLWRVGHOHJLWLPLGDGH LQ'LUHLWR
'LUHLWR
'LUHLWR$GPLQLVWUDWLYR
$GPLQLVWUDWLYR%UDVLOHL
URHG57S 
UR
$6~PXODQžGR6XSUHPR7ULEXQDO)HGHUDOEHP
GHILQHDRULHQWDomRMXULVSUXGHQFLDO´$$GPLQLVWUDomRSRGH
-i R 3URI',2*2 '( ),*8(,5('2 025(,5$ 1(72
DQXODUVHXVSUySULRVDWRVTXDQGRHLYDGRVGHYtFLRVTXHRV
DQRWD TXH UHFRQKHFHQGR DV GLILFXOGDGHV SUiWLFDV FRPR
 'LU$GPLQLVWUDWLYR &kPDUD/HJLVODWLYDGR'LVWULWR)HGHUDO


VHMDPDLQRSRUWXQLGDGHHRVLQFRQYHQLHQWHVGDDQXODomRGH $GHPDLVKiYtFLRVTXHSRXFRRXTXDVHQDGDDIHWDP
XPDWR~WLOHPWHUPRVGHLQWHUHVVHS~EOLFRHPERUDFRP RLQWHUHVVHILQDOtVWLFRSURFXUDGRSHOR'LUHLWReRFDVRGRV
GHIHLWRGHOHJDOLGDGHROHJLVODGRUSRGHLJXDOPHQWHDEULUj GHIHLWRVGHFRPSHWrQFLDQRVDWRVGHFRQWH~GRYLQFXODGR
$GPLQLVWUDomR D DOWHUQDWLYD GLVFULFLRQiULD GD VDQDWyULD $RSDUWLFXODUpTXDVHLQGLIHUHQWHVHXDXWRUHDRLQWHUHVVH
SDUDVDOYiOR'XDVVmRDVFRQGLo}HVSDUDTXHD$GPLQLVWUD S~EOLFR LPSRUWD SRXFR HVWD DXWRULD SRLV DV UHJUDV GH
omRHPSUHJXHDVDQDWyULD
VDQDWyULDFRPH[FHomRjUHJUDGHGHVID]L
VDQDWyULD FRPSHWrQFLD HVWmR SRVWDV QHVWH FDVR HP UD]mR GH
PHQWRGRVDWRVOHJDOPHQWHGHIHLWXRVRVDH[LVWrQFLDGRDWR REMHWLYRVGHRUJDQL]DomRWpFQLFRDGPLQLVWUDWLYDHQmRHP
YLFLDGRDVHUVDQDGRHDDGPLVVLELOLGDGHGDGLVFULFLRQDULHGD DWHQomRDREHPMXUtGLFRDVHUDWHQGLGR
GHSDUDID]rOR LQ&XUVR
&XUVR
&XUVRGH
GH'LUHLWR
'LUHLWR$GPLQLVWUDWLYRHG)RUHQ
$GPLQLVWUDWLYR
VHS  )LQDOPHQWHYDOHFRQVLGHUDUTXHXPGRVLQWHUHVVHV
IXQGDPHQWDLV GR 'LUHLWR p D HVWDELOLGDGH GDV UHODo}HV
(VVHPHVPRDXWRUFODVVLILFDDVVDQDWyULDV
VDQDWyULDVGDVHJXLQWH
VDQDWyULDV FRQVWLWXtGDVeDSDFLILFDomRGRVYtQFXORVHVWDEHOHFLGRVD
IRUPD ILPGHVHSUHVHUYDUDRUGHP(VWHREMHWLYRLPSRUWDPXLWR
PDLVQR'LUHLWR$GPLQLVWUDWLYRGRTXHQR'LUHLWR3ULYDGR
D YROXQWiULD DWUDYpVGHXPRXWURDWR
YROXQWiULD e TXH RV DWRV DGPLQLVWUDWLYRV WrP UHSHUFXVVmR PDLV
DPSODDOFDQoDQGRLQ~PHURVVXMHLWRVXQVGLUHWDHRXWURV
 5DWLILFDomR XP QRYR DWR FRUULJH R GHIHLWRGR LQGLUHWDPHQWHFRPRREVHUYRX6HDEUD)DJXQGHV,QWHUIH
RXWURGHFODUDQGRRtQWHJURHYiOL UHPFRPDRUGHPHHVWDELOLGDGHGDVUHODo}HVVRFLDLVHP
GRGHVGHDRULJHP HVFDODPXLWRPDLRU

'Dt TXH D SRVVLELOLGDGH GH FRQYDOLGDomR GH FHUWDV


 5HIRUPD XPQRYRDWRFRQVHUWDRGHIHLWXRVR
VLWXDo}HV ³ QRomR DQWDJ{QLFD j GH QXOLGDGH HP VHX
GHOHH[FOXLQGRDLOHJDOLGDGH
VHQWLGRFRUUHQWH³WHPHVSHFLDOUHOHYRQR'LUHLWR$GPL
QLVWUDWLYR
 &RQYHUVmR DSURYHLWDVH DSHQDV D SDUWHYiOLGD
1mR EULJDP FRP R SULQFtSLR GD OHJDOLGDGH DQWHV
GRDWRGHIHLWXRVR FRPRQRH[HP
DWHQGHPOKH R HVStULWR DV VROXo}HV TXH VH LQVSLUHP QD
SOR DFLPD WUD]LGR SHOR 3URI+HO\
WUDQTLOL]DomR GDV UHODo}HV TXH QmR FRPSURPHWHP
/RSHV0HLUHOOHV  LQVXSULYHOPHQWHRLQWHUHVVHS~EOLFRFRQTXDQWRWHQKDP
VLGRSURGX]LGDVGHPDQHLUDLQYiOLGDeTXHDFRQYDOLGDomR
E QmR
QmRYROXQWiULD UHVXOWDQWHGHXPIDWRDSUHVFULomR
YROXQWiULD pXPDIRUPDGHUHFRPSRVLomRGDOHJDOLGDGHIHULGD

  ,QWHUQD SUHVFULomRTXHSURGX]HIHLWRQDHVIHUD 3RUWDQWRQmRpUHSXJQDQWHDR'LUHLWR$GPLQLVWUDWL


DGPLQLVWUDWLYD YRDKLSyWHVHGHFRQYDOHVFLPHQWRGRVDWRVLQYiOLGRV

  ([WHUQD SUHVFULomRTXHSURGX]HIHLWRQDHVIHUD 'HWRGRPRGRQmRKiQHJDUTXHXPDWRSDGHFHQWH


MXGLFLiULD  GHYtFLRpLQYDOLGiYHO$SRVVLELOLGDGHGHFRQYDOLGDomRLUi
FDWHJRUL]iOR QD FODVVH GRV DQXOiYHLV HP RSRVLomR DRV
&(/62 $17Ñ1,2 %$1'(,5$ '( 0(//2 EHP QXORVHDRVLQH[LVWHQWHVPDVDPHQRUJUDYLGDGHGRYLFLR
DQDOLVDRJUDXGHLQWROHUkQFLDHPUHODomRDFDWHJRULDVGH QmRSRGHVLJQLILFDULPXQLGDGHFRQWUDHYHQWXDOIXOPLQD
DWRVLQYiOLGRVLQVLVWLQGRTXH´KiGHVHUFRPSDVVDGRFRP omR(PVXPDTXDOTXHUDWRLQYiOLGRpRXQmRFRQYDOLGi
RWLSRGHLOHJLWLPLGDGH6HHVWDpVXVFHWtYHOGHVHUVDQDGD YHO PDV HQWUH RV QmRFRQYDOLGiYHLV DOJXQV VmR QXORV H
UHFXVDUOKH HP WHVH D SRVVLELOLGDGH GH VXSULPHQWR p RXWURVLQH[LVWHQWHVSRUTXHHVWHV~OWLPRVFRPRDRGLDQWH
UHQHJDUDVDWLVIDomRGHLQWHUHVVHVS~EOLFRVHPP~OWLSORV VHGLUiGLIHUHPGRVQXORVWDQWRSRUTXHVmRLQVXVFHWtYHLV
FDVRV GH SUHVFULomR TXDQWR SRUTXH SHUDQWH HOHV H[LVWH XP
&RPHIHLWRPXLWDVYH]HVQDUHODomRDGPLQLVWUDWLYD ´GLUHLWRGHUHVLVWrQFLDµ
HVWDUmRHPSDXWDVLQWHUHVVHVSULYDGRVSXUDPHQWHSDWULPR
QLDLVGRVLQGLYtGXRVHTXHVmRLQWHUHVVHVGLVSRQtYHLV3RGH 'DGD D LPSRUWkQFLD TXH VH DWULEXL j FRQYDOLGDomR
FRLQFLGLUFRPRLQWHUHVVHS~EOLFROHJDOPHQWHDVVLQDGRR FRPRFULWpULRSDUDGLVWLQJXLUHQWUHGRLVJUDQGHVJUXSRVGH
FRQYDOHVFLPHQWR GH XP DWR H[SHGLGR FRP GHVFRQKH DWRVYLFLDGRV DQXOiYHLVGHXPODGRHQXORVHLQH[LVWHQ
FLPHQWR GH GLUHLWRV GR SDUWLFXODU VH HVWH PDQLIHVWDU WHVGHRXWUR pLPSUHVFLQGtYHOHVFODUHFHURVLJQLILFDGRH
XOWHULRUPHQWH VXD DTXLHVFrQFLD RX VH D $GPLQLVWUDomR DOFDQFHGHVWHLQVWLWXWR´ ³>LQ&XUVRGH'LUHLWR$GPLQLVWUDWLYR
FRUULJLU R YtFLR UHWURDWLYDPHQWH DQWHV GH R LQWHUHVVDGR 0DOKHLURV(GLWRUHVHGS@
UHVLVWLUDRDWRRXLPSXJQDUOKHRYtFLR

$OpP GLVVR FRPR EHP R HQVLQD R 3URI 2VZDOGR (IHLWRVGD$QXODomRHGD5HYRJDomR


$UDQKD%DQGHLUDGH0HOORQmRpGHVSUH]iYHODKLSyWHVH
GHXPDJHQWHDGPLQLVWUDWLYRSUDWLFDUDWRQRH[HUFtFLRGH 4XDQGRXPDWRDGPLQLVWUDWLYRpDQXODGRHOHQmR
FRPSHWrQFLDGLVFULFLRQiULDVREFRDomR YtFLRGHYRQWDGH SURGX]QHQKXPHIHLWRRXVHMDVHUiFRQVLGHUDGRFRPRVH
HXOWHULRUPHQWHFRQFOXLUVHUDTXHODDSURYLGrQFLDDGHTXD HOHQmRWLYHVVHH[LVWLGR)LFDUmRGHVIHLWRVWRGRVRVYtQFX
ORVHQWUHDVSDUWHVHDVFRLVDVILFDUmRFRPRHVWDYDPDQWHV
GDGHVGHDRULJHP3RUTXHQHJDUOKHDSRVVLELOLGDGHGH
GD H[LVWrQFLD GR DWR DQXODGR GL]VH TXH D DQXODomR
UDWLILFDUUHWURDWLYDPHQWHRDWR"
SURGX]HIHLWRVH[WXQF 
&kPDUD/HJLVODWLYDGR
 'LVWULWR)HGHUDO 'LU$GPLQLVWUDWLYR 

$VVLPVHUiSRUTXHDDQXODomRGRDWRDGPLQLVWUDWLYR
Vy SRGH WHU SRU IXQGDPHQWR D VXD LOHJLWLPLGDGH RX &RQWUDWRV$GPLQLVWUDWLYRV
LOHJDOLGDGH  ( D $GPLQLVWUDomR S~EOLFD QmR SRGH GDU
QHQKXPD YDOLGDGH QHP SDUFLDO  D TXDOTXHU DWR TXH
UHSUHVHQWHLQIULQJrQFLDGLUHWDRXLQGLUHWDGDVQRUPDVH
&RPRWRGRFRQWUDWRWDPEpPRFRQWUDWR
FRQWUDWR
FRQWUDWRDGPLQLVWUDWL
SULQFtSLRV OHJDLV TXH VmR VHPSUH LPSUHVFLQGtYHLV QD
YRpXPDMXVWHGHYRQWDGHRXVHMDGXDVSDUWHVHQWUDQGR
YR
DWLYLGDGHGR3RGHU3~EOLFR
HPDFRUGRDVVXPLQGRREULJDo}HVHGLUHLWRV$GLIHUHQoD
HVWiHPTXHQRFRQWUDWR
FRQWUDWR
FRQWUDWRDGPLQLVWUDWLYRD$GPLQLVWUDomR
DGPLQLVWUDWLYR
0DVVHRDWRIRUDSHQDVUHYRJDGRHOHSURGX]HIHLWRV
GHVGHTXHIRLSUDWLFDGRDWpDGDWDGHVXDUHYRJDomR6HXV S~EOLFDpXPDGHVVDVSDUWHV
HIHLWRV FHVVDUmR DSHQDV GHSRLV GH VXD UHYRJDomR D
UHYRJDomRSURGX]HIHLWRVH[QXQF  $VVLPRFFRQWUDWR
RQWUDWR DGPLQLVWUDWLYRpRDMXVWHHPTXHD
DGPLQLVWUDWLYR
$GPLQLVWUDomR3~EOLFDDJLQGRQHVVDTXDOLGDGHHVWDEHOH
$QXODomR;5HYRJDomR FHFRPRXWUDSDUWH SDUWLFXODURXPHVPRRXWUDHQWLGDGH
DGPLQLVWUDWLYD  YLVDQGR j UHDOL]DomR GH REMHWLYRV GR
GR0pULWR
GR0pULWR
0pULWRGR$WR$GPLQLVWUDWLYR LQWHUHVVHS~EOLFRHPFRQGLo}HVHVWDEHOHFLGDVSHODSUySULD
$GPLQLVWUDomR3~EOLFD
2 PpULWR
PpULWR GR DWR DGPLQLVWUDWLYR p GLUHWDPHQWH
UHODFLRQDGRjFRQYHQLrQFLDHRSRUWXQLGDGHGHVXDSUiWLFD
3DUDD3URI0$5,$6</9,$=$1(//$',3,(752RVFRQWUDWRV
EHPDVVLPjVXDHILFLrQFLDFRPRSURYLGrQFLDDGPLQLVWUD
WLYD HVFROKLGD SHOD DGPLQLVWUDomR SDUD DOFDQoDU XPD DGPLQLVWUDWLYRVVmR´RVDMXVWHVTXHD$GPLQLVWUDomR
GHWHUPLQDGDILQDOLGDGH QHVVDTXDOLGDGHFHOHEUDFRPSHVVRDVItVLFDVRXMXUtGLFDV
S~EOLFDVRXSULYDGDVSDUDDFRQVHFXomRGHILQVS~EOLFRV
3DUD R VDXGRVR 3URI+(/< /23(6 0(,5(//(6 2 VHJXQGRUHJLPHMXUtGLFRGHGLUHLWRS~EOLFRµ³ LQ´'LUHLWR
PpULWRDGPLQLVWUDWLYRVHFRQVXVEWDQFLDQDYDORUDomRGRV $GPLQLVWUDWLYRµHG$7/$6S 
PRWLYRVHQDHVFROKDGRREMHWRGRDWRIHLWDVSHOD$GPLQLV
WUDomR LQFXPELGD GH VXD SUiWLFD TXDQGR DXWRUL]DGD D 2DUWž†~QLFRGD/HL
/HL
/HLQž
QžWDPEpPDYDQoD

GHFLGLUVREUHDFRQYHQLHQFLDRSRUWXQLGDGHHMXVWLoDGRDWR R FRQFHLWR GH FRQWUDWR DGPLQLVWUDWLYR H HVWDEHOHFH TXH
DUHDOL]DU³ HP'LUHLWR$GPLQLVWUDWLYR%UDVLOHLURHG57S  3DUDRVILQVGHVWD/HLFRQVLGHUDVHFFRQWUDWRWRGRHTXDO
RQWUDWR
TXHU DMXVWH HQWUH yUJmRV RX HQWLGDGHV GD $GPLQLVWUDomR
1RPHVPRVHQWLGRDSRQGHUDomRGR3URI',2*2'( 3~EOLFDHSDUWLFXODUHVHPTXHKDMDXPDFRUGRGHYRQWDGHV
),*8(,5('2 025(,5$ 1(72 TXH 2 PpULWR p R SDUDDIRUPDomRGHYtQFXORHDHVWLSXODomRGHREULJDo}HV
UHVXOWDGRGRH[HUFtFLRGDGLVFULFLRQDULHGDGH$GLVFULFLRQDUL UHFtSURFDVVHMDTXDOIRUDGHQRPLQDomRXWLOL]DGD
HGDGH p XPD WpFQLFD R PpULWR R VHX SURGXWR 3RU HVVD
UD]mRRH[HUFtFLRYLFLRVRGDGLVFULFLRQDULHGDGHFRPSURPHWH 7DO TXDO RV GHPDLV FRQWUDWRV WDPEpP R FRQWUDWR
RPpULWR HP
&XUVRGH'LUHLWR$GPLQLVWUDWLYRHG)RUHQVH DGPLQLVWUDWLYRpFRQVHQVXDO
DGPLQLVWUDWLYR FRQVHQVXDOFRP
FRQVHQVXDO FRPXWDWLYR
FRPXWDWLYRHR
XWDWLYR RQHURVRRX
QHURVR
S  FRQFOXLQGR TXH 2 PpULWR VH ORFDOL]D GHQWUR GD VHMD UHVXOWD GR DFRUGR GH YRQWDGHV GDV SDUWHV FRQ
HVFDQVmRIXQGDPHQWDOGRDWRDGPLQLVWUDWLYRRPRWLYRHR VHQVXDO HDPEDVDVSDUWHVDVVXPHPGLUHLWRVHREULJDo}HV
REMHWR REHSFLW  UHFtSURFDVHHTXLYDOHQWHV FRPXWDWLYR HDLQGDSUHYHUi
DUHPXQHUDomRGRVFRQWUDWDQWHVQRVWHUPRVFRPELQDGRV
/HLQž RQHURVR 
'D$QXODomR5HYRJDomRH&RQYDOLGDomR
'LIHUHTXHKiGHVHUIRUPDO
IRUPDORXVHMDQHFHVVDULDPHQWH
IRUPDO
$UW $ $GPLQLVWUDomR GHYH DQXODU VHXV SUySULRV DWRVTXDQGR HVFULWR H FRP HVSHFLDLV UHTXLVLWRV D VHUHP REVHUYDGRV
HLYDGRV GH YtFLR GH OHJDOLGDGH H SRGH UHYRJiORV SRU DOpPGLVVRpLQWXLWXSHUVRQ LVWRpVHUiFXPSULGRSHOR
PRWLYR GH FRQYHQLrQFLD RX RSRUWXQLGDGH UHVSHLWDGRV RV SUySULRFRQWUDWDQWHSURLELGDV HPUHJUD DWUDQVIHUrQFLD
GLUHLWRVDGTXLULGRV GD REULJDomR D RXWUHP RX PHVPR D VXEVWLWXLomR GR
H[HFXWRU H[FHSFLRQDOPHQWHpSHUPLWLGR 
$UW 2GLUHLWRGD$GPLQLVWUDomRGHDQXODURVDWRVDGPLQLVWUDWLYRV
GHTXHGHFRUUDPHIHLWRVIDYRUiYHLVSDUDRVGHVWLQDWiULRV
GHFDL HP FLQFR DQRV FRQWDGRV GD GDWD HP TXH IRUDP
SUDWLFDGRVVDOYRFRPSURYDGDPiIp ,QWHUSUHWDomR

†ž 1RFDVRGHHIHLWRVSDWULPRQLDLVFRQWtQXRVRSUD]RGHGHFDGrQFLDFRQWDUVH
iGDSHUFHSomRGRSULPHLURSDJDPHQWR &RQTXDQWR UHJLGR SRU QRUPDV GH GLUHLWR S~EOLFR R
†ž &RQVLGHUDVHH[HUFtFLRGRGLUHLWRGHDQXODUTXDOTXHUPHGLGDGHDXWRULGDGH
FRQWUDWRDGPLQLVWUDWLYRDGPLWHTXHUHJUDVGHKHUPHQrXWL
DGPLQLVWUDWLYDTXHLPSRUWHLPSXJQDomRjYDOLGDGHGRDWR FDFRPXQVVHMDPXWLOL]DGDVHPVXDLQWHUSUHWDomRGHVGH
TXHpFODURQmRDIURQWHPVXDHVVrQFLDTXHpRD DWHQGL
$UW (PGHFLVmRQDTXDOVHHYLGHQFLHQmRDFDUUHWDUHPOHVmRDR PHQWR
PHQWRGR LQWHUHVVHS~EOLFR$VVLPMiRSULPHLURSUHFHLWR
GRLQWHUHVVH S~EOLFR
LQWHUHVVH S~EOLFR QHP SUHMXt]R D WHUFHLURV RV DWRV TXH GHLQWHUSUHWDomRGRVFRQWUDWRVHPJHUDOTXHpDEXVFDGD
DSUHVHQWDUHPGHIHLWRVVDQiYHLVSRGHUmRVHUFRQYDOLGDGRV YRQWDGH LQWHQomR GDVSDUWHVGHYHVHURULHQWDGRSDUDWDO
SHODSUySULD$GPLQLVWUDomR SUHPLVVDEiVLFDDLQWHQomRGDVSDUWHVFRQWUDWDQWHVGHYH
VHUVHPSUHDVDWLVIDomRGDVYRQWDGHVFROHWLYDV

 'LU$GPLQLVWUDWLYR &kPDUD/HJLVODWLYDGR'LVWULWR)HGHUDO


+(/</23(60(,5(//(6TXDQGR-XL]MiGHFLGLUDTXH  )RUPD ³ WDQWR TXDQWR R DWR DGPLQLVWUDWLYR R


QRVFRQWUDWRVDGPLQLVWUDWLYRVH[DWDPHQWHSRUTXHFHOHEUD FRQWUDWR p VHPSUH IRUPDO DV QRUPDV TXH UHJHP DV
GRVHPSUROGDFROHWLYLGDGH´QmRVHSRGHLQWHUSUHWDUVXDV IRUPDOLGDGHVFRQVXEVWDQFLDPVHHPGLSORPDVJHUDOPHQWH
FOiXVXODVFRQWUDHVVDPHVPDFROHWLYLGDGHSDUDVyVHDWHQGHU GHVLJQDGRVFRPR&yGLJRVGH$GPLQLVWUDomR)LQDQFHLUD
DRV GLUHLWRV LQGLYLGXDLV GR SDUWLFXODU FRQWUDWDQWHµ DSXG &yGLJRV GH &RQWDELOLGDGH 3~EOLFD RX &DGHUQRV GH
´'LUHLWR$GPLQLVWUDWLYR%UDVLOHLURHG57S  2EULJDo}HV  LQVWUXPHQWR VHUi R WHUPR RX D HVFULWXUD
S~EOLFDRVWHUPRVVmRODYUDGRV HP OLYURVHVSHFLDLVGDV
2XWUDUHJUDLPSRUWDQWtVVLPDSDUDDLQWHUSUHWDomRGRV UHSDUWLo}HVS~EOLFDVHWrPDPHVPDIpTXHRVDVVHQWDPHQ
FRQWUDWRVDGPLQLVWUDWLYRVpDGDYLQFXODomRGD$GPLQLV WRVQRWDULDLV6mRWDPEpPIRUPDOLGDGHVRVUHJLVWURVQRV
WUDomRDRLQWHUHVVHS~EOLFRUD]mRSRUTXHQmRSRGHUiHOD WULEXQDLVGHFRQWDVRXyUJmRVDQiORJRV
VHUOLEHUDOHSH[GHL[DUGHH[LJLUTXHRRXWURFRQWUDWDQ
WHQmRFXPSUDFRPVXDVREULJDo}HVQmRKiSRLVSRVVLEL  0RWLYR ³ R UHVSDOGR IiFWLFR RX MXUtGLFR UHOHYD
OLGDGHGHUHQ~QFLDGHGLUHLWRVRXLQWHUHVVHVS~EOLFRVHVH DSHQDVSDUDD$GPLQLVWUDomR1RTXHVHUHIHUHjPDQLIHV
KRXYHUFOiXVXODQHVVHVHQWLGRpQXODLQH[LVWHQWHRXQmR WDomR GH YRQWDGH GR DGPLQLVWUDGR FRQWLQXD R PRWLYR
HVFULWD LUUHOHYDQWH

2XWURVVLPFRPRRFRQWUDWRDGPLQLVWUDWLYRpXPDWR 2EMHWR³RFRQWUDWRYLVDVHPSUHjFRQVWLWXLomRGH
DGPLQLVWUDWLYRTXDOHVWHWDPEpPDTXHOHJR]DGHSUHVXQ XPD UHODomR MXUtGLFD 'L]VH REULJDFLRQDO SRUTXH GHOH
omR GH OHJLWLPLGDGH  GDt UHVXOWDQGR TXH VXDV FOiXVXODV VXUJHP REULJDo}HV GH SUHVWDU H FRUUHODWRV GLUHLWRV GH
JR]DPGHLGrQWLFDSUHVXQomRTXHVyVHUiDIDVWDGDSRU H[LJLUDSUHVWDomR3DUDD$GPLQLVWUDomRDREULJDomRVHUi
SURYDHPVHQWLGRFRQWUiULR EDVLFDPHQWHDGHSDJDUXPDTXDQWLDFHUWDRXFHGHURXVR
GHXPEHPS~EOLFRSDUDRDGPLQLVWUDGRSRGHUiVHUGH
7DPEpP D SRVVLELOLGDGH GH DOWHUDomR GDV FOiXVXODV GDUFRLVDFHUWDGHID]HURXDLQGDGHSDJDUTXDQWLDFHUWD
UHJXODPHQWDUHV p UHJUD TXH JXLDUi D LQWHUSUHWDomR GR
FRQWUDWR DGPLQLVWUDWLYR RSomR TXH GHYHUi DWHQGHU DR ,PSRUWDQWH IULVDUVH QR WRFDQWH DR REMHWR TXH DV
LQWHUHVVHS~EOLFRVHPHQWUHWDQWRGHVYLUWXDURHTXLOtEULR SHVVRDVMXUtGLFDVGHGLUHLWRS~EOLFRQmRSUHVWDPFRQWUDWX
ILQDQFHLURGRFRQWUDWRHDUHFLSURFLGDGHHTXLYDOrQFLDGRV DOPHQWH VHUYLoRV QHP ID]HP IRUQHFLPHQWRV QD UHODomR
GLUHLWRVHREULJDo}HVGDVSDUWHVD$GPLQLVWUDomR3~EOLFD FRQWUDWXDO GH 'LUHLWR $GPLQLVWUDWLYR VmR HODV TXH SUR
GHIHQGHRVXSUHPRLQWHUHVVHS~EOLFRPDVGDtQmRUHVXOWD FXUDP QR FDPSR SULYDGR SUHVWDGRUHV GH VHUYLoRV H
UiRVDFULItFLRHFRQ{PLFRILQDQFHLURGRFRQWUDWDQWH IRUQHFHGRUHV FRP RV TXDLV DYHQoDP REULJDQGRVH D
SDJDPHQWRVHPHVSpFLHÔQLFDH[FHomRDFRQFHVVmRGH
 )LQDOPHQWH QRV FRQWUDWRV GH DWULEXLomR HP TXH XVRGHEHPS~EOLFRHPTXHHPYH]GHXPSDJDPHQWR
H[FHSFLRQDOPHQWHKiDWHQGLPHQWRGHDOJXPGRLQWHUHVVH VXDSUHVWDomRpDFHVVmRHJDUDQWLD GRXVRGHXPEHP
SULYDGR D LQWHUSUHWDomR GH VXDV FOiXVXODV GHYH VHU S~EOLFR JHUDOPHQWH FRQWUD XPD SUHVWDomR RQHURVD GR
UHVWULWLYDSDUDTXHDVYDQWDJHQVFRQFHGLGDVDRSDUWLFODU DGPLQLVWUDGR&RPRRDGPLQLVWUDGRQmRGLVS}HGRREMHWR
QmRUHVXOWHPSULYLOpJLRRGLRVRHPGHWULPHQWRGDLVRQR GHQDWXUH]DS~EOLFDQmROKHDVVLVWLUiGLUHLWRGHUHWHQomR
PLD HQWUH RV DGPLQLVWUDGRV H GRV LQWHUHVVHV GH WRGD GH REUD S~EOLFD QHP GLUHLWR GH SDUDOLVDU RV VHUYLoRV
FROHWLYLGDGHVHQmRDWpGRVFRIUHVS~EOLFRV S~EOLFRVTXHHVWHMDH[HFXWDQGRHPUD]mRGHFRQWUDWR

 &DSDFLGDGH ³ HVWH HOHPHQWR VH UHIHUH j SDUWH


FRQWUDWDQWHSULYDGDVHUiHPSULQFtSLRR'LUHLWR3ULYDGR
)RUPDOL]DomR
TXHUHJXODUiDVFRQGLo}HVGHYDOLGDGHGDPDQLIHVWDomRGH
YRQWDGHTXHjTXHODFRPSHWH
&RPRHPWRGDFRQYHUJrQFLDGHPDQLIHVWDomRGHYRQWD
GH WDPEpP R FRQWUDWR DGPLQLVWUDWLYR VH IRUPD
RUPD SHOR (QWUHWDQWRDFDSDFLGDGHFLYLOQmREDVWDSDUDDFRQWUD
FRQVHQVRGHSHVVRDVFRPFDSDFLGDGHSDUDH[WHUQDUHPVXD WDomRFRPDVSHVVRDVMXUtGLFDVGHGLUHLWRS~EOLFR'XDV
YRQWDGHYLQFXODGDDRILPFROLPDGR FRQGLo}HVVXEMHWLYDVXPDQHJDWLYDRXWUDSRVLWLYDSRGH
UmR VHU H[LJLGDV $ FRQGLomR QHJDWLYD VHUi R QmR HVWDU
(VWHV VmR RV HOHPHQWRV FRQVWLWXWLYRV GR FRQWUDWR LPSHGLGRRDGPLQLVWUDGRSRULQLGRQHLGDGHGHFRQWUDWDU
DGPLQLVWUDWLYRVQDOLomRGR3URI',2*2'(),*8(,5('2 FRP D $GPLQLVWUDomR $ FRQGLomR SRVLWLYD VHUi R HVWDU
025(,5$1(72 KDELOLWDGRHPOLFLWDomRS~EOLFDRXSURFHGLPHQWRDQiORJR
>@
´&RPSHWrQFLD³UHIHUHVHDRyUJmRGD$GPLQLVWUD
omRTXHDYLQFXODUiQRFRQWUDWRYDOHQGRDVFRQVLGHUDo}HV 3RGHVHIDODUSRUWDQWRHPFDSDFLGDGHDGPLQLVWUDWLYD
H[SHQGLGDVTXDQGRDHVWXGDPRVFRPRHOHPHQWRGRDWR HPUHODomRDRDGPLQLVWUDGRSDUDLQGLFDUDH[LVWrQFLDGH
DGPLQLVWUDWLYR UHTXLVLWRVVXEMHWLYRVGHPDLRUH[WHQVmRTXHRVGH'LUHLWR
3ULYDGR 'Dt GHFRUUH VHU R FRQWUDWR DGPLQLVWUDWLYR
)LQDOLGDGH³FRQWLQXDHVVHQFLDOQRFRQWUDWRTXH SHUVRQDOtVVLPRUHDOL]DGRLQWXLWXSHUVRQ HYHGDGDHP
VH GHVWLQDUi VHPSUH D VDWLVID]HU XP LQWHUHVVH S~EOLFR FRQVHTrQFLDDVXEURJDomRVXEMHWLYD
HVSHFtILFRFDUDFWHUL]DGRH[SOtFLWDRXLPSOLFLWDPHQWHQD
OHLHQRVHXLQVWUXPHQWR &RQVHQVR³pRHOHPHQWRHVVHQFLDOGHWRGRRFRQWUD
&kPDUD/HJLVODWLYDGR
  'LVWULWR)HGHUDO 'LU$GPLQLVWUDWLYR 

WR2FRQVHQWLPHQWRUHFtSURFRYHPDVHUDFRLQFLGrQFLD
GDVYRQWDGHVGDVSDUWHVUHODWLYDPHQWHDRREMHWRLVWRpR (VSpFLHVGH&RQWUDWRV
HVWDUHPDPEDVGHVHMDQGRDDOWHUDomRQRPXQGRMXUtGLFR
TXH R DWR ELODWHUDO VH GHVWLQD D SURGX]LU ,PSRUWDQWH 23URI',2*2'(),*8(,5('2025(,5$1(72 LQ
DVVHQWDUVHTXHHVWDFRPSRVLomRGHYRQWDGHVID]VXUJLU &XUVR
&XUVR
&XUVRGH
GH'LUHLWR
'LUHLWR$GPLQLVWUDWLYRHG)RUHQVHS
$GPLQLVWUDWLYR GLVFUL
FRPRUHVXOWDQWHXPDQRYDFDWHJRULDDEVWUDWDGHYRQWDGH PLQDWLSRVGHFRQWUDWRV
WLSRVGHFRQWUDWRVDGPLQLVWUDWLYRV
WLSRVGHFRQWUDWRV
DYRQWDGHGRFRQWUDWR(FRPRUHVXOWDQWHQmRVHWUDWDGH
PHURVRPDWyULRTXHGLILUDDSHQDVTXDQWLWDWLYDPHQWHGDV  GH
 GH2EUDV
2EUDVH6HUYLoRVDVHUUHDOL]DGRRXHPUHJLPHGH
6HUYLoRV
YRQWDGHVTXHOKHGHUDPRULJHPPDVVHDSUHVHQWDFRPR HPSUHLWDGDRXSRUDGPLQLVWUDomRFRQWUDWDGD
XPQRYRJrQHURGHODVVHGLVWLQJXLQGRTXDOLWDWLYDPHQWH
'DtVHH[SOLFDDVXSUDRUGLQDomRGDYRQWDGHGRFRQWUDWR   GH
  GH &RPSUD
&RPSUD SDUD DTXLVLomR GH EHQV SRGHQGR VHU
HPUHODomRjVYRQWDGHVLQGLYLGXDLVTXHRFRQVWLWXtUDP FRQWUDWDGRRIRUQHFLPHQWRLQWHJUDORXSDUFHODGRTXHVH
JDQKDQGRFRPRTXHXPDDXWRULGDGHREMHWLYDWDOFRPRD GDUiRXHPHQWUHJDVGHWHUPLQDGDVRXFRQWLQXDGDPHQWH
WHP D OHL GLVWR GHFRUUH R DILUPDUVH TXH R FRQWUDWR
JHUDGRUGHQRUPDDXW{QRPDID]OHLHQWUHDVSDUWHV  GH&RQFHVVmRGH6HUYLoRV3~EOLFRVSDUDDPDQXWHQ
 GH&RQFHVVmRGH6HUYLoRV3~EOLFRV
omRGHVHUYLoRGHXWLOLGDGHS~EOLFD
(VVDUHPHPRUDomRVREUHDQDWXUH]DMXUtGLFDGRFRQWUD
WRDGPLQLVWUDWLYRIRLVXVFLWDGDSDUDDFODUDURSUREOHPDGD   GH
  GH &RQFHVVmR
&RQFHVVmR GH 8VR GH %HQV 3~EOLFRV
3~EOLFRV SDUD TXH
SDUWLFXODUHVXVHPEHQVS~EOLFRV QRUPDOPHQWHLPyYHLV 
DXWROLPLWDomRGD$GPLQLVWUDomRFRPRSDUWHGHUHODomR
XVRHVVHSDUDDWHQGHUDDOJXPLQWHUHVVHGDFROHWLYLGDGH
ELODWHUDO
 GH
 GH6HUYLoRV
6HUYLoRV7pFQLFRV
7pFQLFRV(VSHFLDOL]DGRVWDLVSUHVWDGRUHV
(VSHFLDOL]DGRV
6HpFHUWRTXHTXHPFRQWUDWDVHDXWROLPLWDREULJDQGR
GHVHUYLoRVQmRFRPS}HPRTXDGURRUGLQiULRGHVHUYLGR
VHDRFXPSULPHQWRGHWRGDVDVFOiXVXODVGRFRQWUDWRHVVD UHVS~EOLFRVVHQGRQHFHVViULDDFRQWUDWDomR
UHJUDQmRSRGHDWLQJLUR(VWDGRQRFRQWUDWRDGPLQLVWUDWL
YR QR TXDO GLVWLQWDPHQWH GR TXH RFRUUH QR FRQWUDWR   GH
  GH (PSUpVWLPR
(PSUpVWLPR 3~EO
3~EOLFR
LFR XPD IRUPD SDUD D
LFR
SULYDGRHOHQmRVHGHVSHGHVHXSRGHUGHYHUILQDOtVWLFRGH $GPLQLVWUDomR 3~EOLFD REWHU GLQKHLUR SDUD FXVWHLR GH
VDWLVID]HU RV LQWHUHVVHV S~EOLFRV TXH OKH VmR FRPHWLGRV REUDVHVHUYLoRV
SHOD&RQVWLWXLomRHSHODVOHLV1RFRQWUDWRDGPLQLVWUDWLYR
D $GPLQLVWUDomR 3~EOLFD QmR SRGH LPSRUVH QHQKXPD   GH
  GH *DUDQWLD
*DUDQWLD 3~EOLFD
3~EOLFD
OLFD FRQWUDWRV DFHVVyULRV SDUD
DXWROLPLWDomRDRSOHQRH[HUFtFLRGHVVHSRGHUGHYHUTXHD JDUDQWLD D H[HFXomR RX FXPSULPHQWR GH DOJXP RXWUR
OHLOKHDWULEXLHGRTXDOVyHOD OHL H[FHSFLRQDOPHQWH FRQWUDWR $VVLP R VmR RV FRQWUDWRV GH FDXomR ILDQoD H
SRGHUiH[LPLOD VHJXURJDUDQWLD

$VVLPpTXHDTXHOHWHUWLXPJHQXVFROHWLYRDYRQWDGH   GH
  GH $OLHQDomR
$OLHQDomR DOLHQDU p WUDQVIHULU R GRPtQLR D
FRQVHQVXDORULXQGDGRFRQWUDWRPHVPRVXSUDRUGLQDGD SURSULHGDGHVHMDGHEHQVPyYHLVRXLPyYHLV4XDVHVHP
jVYRQWDGHVTXHDFRQVWLWXtUDPQmRVHVREUHS}HjYRQWD SUHVHUiUHDOL]DGDFRPOLFLWDomRH VHVHWUDWDUGHEHP
GHGDOHLRTXHYDOHGL]HUTXHR(VWDGRQmRILFDUiDGVWULWR LPyYHOVHUiSUHFHGLGDGHDXWRUL]DomROHJLVODWLYD
DR FRQWUDWR HP WXGR R TXH GLJD UHVSHLWR DR LQWHUHVVH
S~EOLFRTXHSUHWHQGDVDWLVID]HUSRUHVVHLQVWUXPHQWR+i   GH
  GH VHUYLoR
VHUYLoR WHPSRUiULR
WHPSRUiULR SUHYLVWD QD FRQVWLWXLomR D
XPDSUHOD]LDD[LROyJLFDGRVILQV³LQWHUHVVHS~EOLFR³ SRVVLELOLGDGHGHD$GPLQLVWUDomRFRQWUDWDUSHVVRDVSDUD
VREUHRVPHLRVXWLOL]DGRV³FRQWUDWDomR DWHQGHU QHFHVVLGDGH WHPSRUiULD GH H[FHSFLRQDO LQWHUHVVH
(LVFRPRGHYHVHUHQWHQGLGRRFRQVHQVRQRFRQWUDWR S~EOLFR 7DLV FRQWUDWDGRV QmR
mR VH VXEPHWHUmR DR UHJLPH
~QLFR (VWDWXWRGR6HUYLGRU&LYLO 
DGPLQLVWUDWLYRHVVHQFLDOSDUDVXDIRUPDomRPDVUHODWLYR
QDVXDH[HFXomR'DtUHVXOWDUHPGRLVLQVWLWXWRVDGLDQWH
  GH
  GH ORFDomRD$GPLQLVWUDomRSUHFLVDGHPyYHLVH
ORFDomR
HVWXGDGRV PDLV GHWDOKDGDPHQWH D LQVWDELOLGDGH GR
LPyYHLV SDUD LQVWUXPHQWDU D SUHVWDomR GH VHUYLoRV
FRQWUDWRDGPLQLVWUDWLYRHDGXDOLGDGHGHWLSRVGHFOiXVXODV S~EOLFRVSDUDLVVRDOXJDDXWRPyYHLVTXHVHUmRXVDGRV
FRQWUDWXDLV

9DOH OHPEUDU DLQGD TXH R FRQVHQVR p GHIHFWLYR QRV &RQWUDWRVGH&RQFHVVmRGH6HUYLoRV3~EOLFRV


HPSUpVWLPRVFRPSXOVyULRVPRGDOLGDGHSDUDWULEXWiULDTXH
SRU LVVR WHP FDUDFWHUtVWLFDV PLVWDV DGPLQLVWUDWLYDV H +iDOJXQVFRQWUDWRVHVSHFLDOPHQWHLPSRUWDQWHVILUPD
WULEXWiULDV SUHYLVWRV QR DUW GD &RQVWLWXLomR SDUD GRVSHOD$GPLQLVWUDomRS~EOLFDVmRRVFRQWUDWRVGHIRUQH
DWHQGHU D GHVSHVDV H[WUDRUGLQiULDV SURYRFDGDV SRU FLPHQWRHRVFRQWUDWRVGHFRQFHVVmR
FDODPLGDGH S~EOLFD JXHUUD H[WHUQD RX VXD LPLQrQFLD H
LQYHVWLPHQWRS~EOLFRGHFDUiWHU XUJHQWHHGHUHOHYDQWH &RQWUDWR GH IRUQHFLPHQWR
IRUQHFLPHQWR FRPR DSRQWD R SUySULR
LQWHUHVVHQDFLRQDOµ ³ LQ&XUVR
&XUVR
&XUVR GH
GH 'LUHLWR
'LUHLWR $GPLQLVWUDWLYR
$GPLQLVWUDWLYR QRPHpDTXHOHSHORTXDOD$GPLQLVWUDomRDGTXLUHEHQV
HG)RUHQVHS  FRLVDVPyYHLVGHTXHWHPQHFHVVLGDGHRXSDUDDUHDOL]D
omRGHREUDVRXSDUDDPDQXWHQomRGHVHXVVHUYLoRV
 'LU$GPLQLVWUDWLYR &kPDUD/HJLVODWLYDGR'LVWULWR)HGHUDO


-iQRFRQWUDWR
FRQWUDWR
FRQWUDWRGH
GHFRQFHVVmRD$GPLQLVWUDomRDRLQYpV
FRQFHVVmR WLFLSDomRGD$GPLQLVWUDomRFRPVXSUHPDFLDGH3RGHUµ³
GHUHDOL]DUHODPHVPDXPGHWHUPLQDGRVHUYLoR RXREUD  GLIHUHQFLDQGRRVGRVFRQWUDWRVSDUWLFXODUHVH[DWDPHQWH
HOD FRQFHGH DR 3DUWLFXODU D UHDOL]DomR GR VHUYLoR RX HP UD]mR GD SHFXOLDULGDGH GDV ´FFOiXVXODV
OiXVXODV H[RUELWDQWHV
H[RUELWDQWHV
REUD  GDQGROKH R GLUHLWR GH H[SORUDU SRU VXD FRQWD H H[SOtFLWDVRXLPSOtFLWDVHPWRGRFRQWUDWRDGPLQLVWUDWLYR
ULVFR VHJDQKDUJDQKRXVHSHUGHUSHUGHX  &OiXVXODV([RUELWDQWHVVmRSRLVDVTXHH[FHGHPGRGLUHLWR
FRPXPSDUDFRQVLJQDUXPDYDQWDJHPRXXPDUHVWULomRj
(VVDFRQFHVVmRpVHPSUHSRUXPSUD]RGHILQLGR HPERUD $GPLQLVWUDomRDRFRQWUDWDQWHDEVROXWDPHQWHYiOLGDQR
UHQRYiYHO HVREFRQGLo}HVSUpHVWDEHOHFLGDVQRFRQWUDWR FRQWUDWRDGPLQLVWUDWLYRGHVGHTXHGHFRUUHQWHGDOHLRXRV
TXDOLGDGHGRVHUYLoRRXPDWHULDOGDREUDHWF  SULQFtSLRV TXH UHJHP D DWLYLGDGH DGPLQLVWUDWLYD SRUTXH
YLVDDHVWDEHOHFHUXPDSUHUURJDWLYDHPIDYRUGHXPDGDV
$VVLPWHPRVWUrVWLSRVGHFRQWUDWRVGHFRQFHVVmR
FRQFHVVmR
FRQFHVVmR SDUWHVSDUDRSHUIHLWRDWHQGLPHQWRGRLQWHUHVVHS~EOLFRTXH
VHVREUHS}HVHPSUHDRVLQWHUHVVHVSDUWLFXODUHVµ³ LQ´'LUHLWR
 &RQFHVVmRGHVHUYLoR3~EOLFR
 $GPLQLVWUDWLYR%UDVLOHLURHG57S 
 FRQFHVVmRGHREUDS~EOLFDH

 FRQFHVVmRGHXVRGHEHPS~EOLFR
 'HWDOKDRVDXGRVR0HVWUHTXH$VFOiXVXODVH[RUELWDQWHV
SRGHPFRQVLJQDUDVPDLVGLYHUVDVSUHUURJDWLYDVQRLQWHUHV
(QVLQDQRV DLQGD R 3URI ',2*2 '( ),*8(,5('2 VH GR VHUYLoR S~EOLFR WDLV FRPR D RFXSDomR GR GRPtQLR
025(,5$1(72TXHDFRQFHVVmRFRPRFRQWUDWRDGPLQLV S~EOLFRRSRGHUH[SURSULDWyULRHDDWULEXLomRGHDUUHFDGDU
WUDWLYRWHPSRUFDUDFWHUtVWLFDV WULEXWRVFRQFHGLGRVDRSDUWLFXODUFRQWUDWDQWHSDUDDFDEDO
H[HFXomRGRFRQWUDWR7RGDYLDDVSULQFLSDLVVmRDVTXHVH
D DLPSHUDWLYLGDGHLPDQHQWHPDLVSRWHQFLDO H[WHULRUL]DP QD SRVVLELOLGDGH GH DOWHUDomR H UHVFLVmR
XQLODWHUDOGRFRQWUDWRQRHTXLOtEULRHFRQ{PLFRHILQDQFHLUR
E DSUHVXQomRGHYHUDFLGDGHGRVSUHVVXSRVWRVIiFWLFRV QDUHYLVmRGHSUHoRVHWDULIDVQDLQRSRQLELOLGDGHGDH[FHomR
HMXUtGLFRVVREUHRVTXDLVIRLHVWDEHOHFLGD GHFRQWUDWRQmRFXPSULQGRQRFRQWUROHGRFRQWUDWRHQD
DSOLFDomRGHSHQDOLGDGHVFRQWUDWXDLVSHOD$GPLQLVWUDomR
F DHILFiFLDQRUPDOGHFRUUHQWHGDVLPSOHVYDOLGDGH 0HUHFHDLQGDUHIHUrQFLDDSRVVLELOLGDGHGHVHDSOLFDUHPDRV
FRQWUDWRVDGPLQLVWUDWLYRVDWHRULDGRIDWRGRSUtQFLSHHD
G DH[HTLELOLGDGHSRVVLELOLGDGHGHH[HFXomRLPHGLD WHRULDGDLPSUHYLVmR LQREUDFLWDGDS 
WD
$VVLP p SHFXOLDULGDGH
SHFXOLDULGDGH GR FRQWUDWR DGPLQLVWUDWLYR D
H DH[HFXWRULHGDGHSUySULDGRVDWRVHPTXHD$GPL IRUoDGD$GPLQLVWUDomR3~EOLFDTXHQHOHVHLPS}HFRP
IRUoD
QLVWUDomRpSDUWH VXSUHPDFLDGHSRGHUQDVFKDPDGDVFOiXVXODVH[RUELWDQ
WHVRXVHMDFOiXVXODVTXHVHULDPSURLELGDVQXPFRQWUDWR
I D ELODWHUDOLGDGH GRV HIHLWRV D FRPXWDWLYLGDGH FRPXP SRUTXH OHRQLQDV PDV GH WRWDO YDOLGDGH QR
H[SUHVVDQDHTXLYDOrQFLDGDVSUHVWDo}HVUHFtSURFDV FRQWUDWRDGPLQLVWUDWLYRSRUTXHDXWRUL]DGDVSRUOHLHYRO
WDGDVSDUDRDWHQGLPHQWRGRLQWHUHVVHS~EOLFR
J DRQHURVLGDGHSRLVQmRSRGHUiVHUJUDWXLWR
2XWUD SHFXOLDULGDGHp
SHFXOLDULGDGH TXHRFRQWUDWRDGPLQLVWUDWLYR
K D LQVWDELOLGDGH WDQWR TXDQWR DR FRQWH~GR GH WDPEpPSRGHVHUD UHVFLQGLGR XQLODWHUDOPHQWH
DOWHUDGRHUHVFLQGLGR
OWHUDGR UHVFLQGLGRXQLODWHUDOPHQWH
LQWHUHVVHS~EOLFRTXDQWRjSUySULDVREUHH[LVWrQFLD SHOD $GPLQLVWUDomR 3~EOLFD HVVH SRGHU GH DOWHUDU H
GDUHODomRFRQWUDWXDO UHVFLQGLU R FRQWUDWR H XP SUHFHLWR GH RUGHP
UGHP S~EOLFD
S~EOLFD
FD
GHFRUUHQWH GR SULQFtSLR GH TXH FDEH j $GPLQLVWUDomR
2 UHJLPH FRQFHVVLRQDO VHUi VHPSUH GH ODVWUR OHJDO RUJDQL]DUHDGPLQLVWUDUDVREUDVHVHUYLoRVS~EOLFRVSDUD
SUHFHGLGR GH OLFLWDomR VDOYR QRV FDVRV HP TXH VHMD PHOKRU DWHQGHU DR LQWHUHVVH S~EOLFR H LJXDOPHQWH GH
GLVSHQVDGD RX LQH[LJtYHO SDXWDGRV SHORV SULQFtSLRV DVVHJXUDUDFRQWLQXLGDGHGRVHUYLoRS~EOLFR VHRFRQWUD
VHWRULDLVHVWXGDGRV WDQWH QmR YHP FXPSULQGR D FRQWHQWR UHVFLQGHVH R
FRQWUDWRJDUDQWLQGRTXHRVHUYLoRS~EOLFRFRQWLQXDUiD
8PFRQWUDWRGHFRQFHVVmRDSUHVHQWDEDVLFDPHQWHDV VHUSUHVWDGRHEHPSUHVWDGR 
VHJXLQWHVFOiXVXODVREULJDo}HVGRFRQFHVVLRQiULRDWXDomR
GR 3RGHU &RQFHGHQWH HQXPHUDomR GRV GLUHLWRV GR 3RU/HLRUHJLPHMXUtGLFRGRVFRQWUDWRVDGPLQLVWUDWLYRV
FRQFHVVLRQiULR HP HVSHFLDO GH VXD UHPXQHUDomR H D FRQIHUHj$GPLQLVWUDomR3~EOLFDDSUHUURJDWLYDGH
UHJXODomR GD H[WLQomR GR YtQFXOR FRP GHYROXomR GR
VHUYLoRFRQFHGLGR³>LQ&XUVRGH'LUHLWR$GPLQLVWUDWLYRHG D PRGLILFiORVXQLODWHUDOPHQWHSDUDPHOKRUDGHTXD
(G)RUHQVHSS@ omRjVILQDOLGDGHVGHLQWHUHVVHS~EOLFRUHVSHLWDGRV
RVGLUHLWRVGRFRQWUDWDGR

E UHVFLQGLORVXQLODWHUDOPHQWHQRVFDVRVHVSHFLILFD
&OiXVXODV1HFHVViULDV
GRVQRLQF,GRDUWGD/HL

+(/< /23(6 0(,5(//(6 DQRWD TXH RV FRQWUDWRV F ILVFDOL]DUOKHVDH[HFXomR


DGPLQLVWUDWLYRV WrP SRU FDUDFWHUtVWLFD HVVHQFLDO D ´SDU
&kPDUD/HJLVODWLYDGR
  'LVWULWR)HGHUDO 'LU$GPLQLVWUDWLYR 

G DSOLFDUVDQo}HVPRWLYDGDVSHODLQH[HFXomRWRWDORX $ /HL
/HL Qž  GHWHUPLQD TXH QRV FRQWUDWRV
Qž 

SDUFLDOGRDMXVWH DGPLQLVWUDWLYRV SDVVDP D VHU REULJDWyULDV
REULJDWyULDV
LJDWyULDV DOJXPDV
FOiXVXODVSDUDHVWDEHOHFHU
H QRV FDVRV GH VHUYLoRV HVVHQFLDLV RFXSDU
SURYLVRULDPHQWH EHQV PyYHLV LPyYHLV SHVVRDO H D RREMHWRHVHXVHOHPHQWRVFDUDFWHUtVWLFRV
VHUYLoRV YLQFXODGRV DR REMHWR GR FRQWUDWR QD
KLSyWHVH GD QHFHVVLGDGH GH DFDXWHODU DSXUDomR E RUHJLPHGHH[HFXomRRXDIRUPDGHIRUQHFLPHQWR
DGPLQLVWUDWLYDGHIDOWDVFRQWUDWXDLVSHORFRQWUDWD
GREHPFRPRQDKLSyWHVHGHUHVFLVmRGRFRQWUDWR F RSUHoRHDVFRQGLo}HVGHSDJDPHQWRRVFULWpULRV
DGPLQLVWUDWLYR GDWDEDVH H SHULRGLFLGDGH GR UHDMXVWDPHQWR GH
SUHoRVRVFULWpULRVGHDWXDOL]DomRPRQHWiULDHQWUH
$WHQomR
$WHQomR D DOWHUDomR XQLODWHUDO Vy p SRVVtYHO QDV FKD D GDWD GR DGLPSOHPHQWR GDV REULJDo}HV H D GR
PDGDVFOiXVXODV
FOiXVXODV
FOiXVXODVUHJXODPHQWDUHV WDPEpPFKD
UHJXODPHQWDUHV HIHWLYRSDJDPHQWR
PDGDVGHFOiXVXODVGHVHUYLoR TXHUGL]HUTXHVyQRTXH
WDQJHDRREMHWRGRFRQWUDWR SH[WDPDQKRGDSRQWHD G RVSUD]RVGHLQtFLRGHHWDSDVGHH[HFXomRGHFRQ
VHUFRQVWUXtGDQžGHPiTXLQDVDVHUHPIRUQHFLGDVHWF  FOXVmRGHHQWUHJDGHREVHUYDomRHGHUHFHELPHQWR
H DR PRGR GH H[HFXomR p TXH SRGH LQFLGLU D DOWHUDomR
GHILQLWLYRFRQIRUPHRFDVR
XQLODWHUDO ,VVR VLJQLILFD TXH DV FOiXVXODV HFRQ{PLFR
ILQDQFHLUDV H PRQHWiULDV GRV FRQWUDWRV DGPLQLVWUDWLYRV
H RFUpGLWRSHORTXDOFRUUHUiDGHVSHVDFRPDLQGLFD
QmR
QmR SRGHUmR VHU DOWHUDGDV VHP SUpYLD FRQFRUGkQFLD GR
omR GD FODVVLILFDomR IXQFLRQDO SURJUDPiWLFD H GD
FRQWUDWDGRRXVHMDD$GPLQLVWUDomR3~EOLFDQmRSRGHUi
FDWHJRULDHFRQ{PLFD
SH[DOWHUDURSUHoRRXFRQGLo}HVGHSDJDPHQWRHWF
I  DV JDUDQWLDV RIHUHFLGDV SDUD DVVHJXUDU VXDSOHQD
1D KLSyWHVH GH PRGLILFDomR XQLODWHUDO HP TXH D
$GPLQLVWUDomRYLVDUDPHOKRUDGHTXDomRjVILQDOLGDGHVGH H[HFXomRTXDQGRH[LJLGDV
LQWHUHVVHS~EOLFRDOpPGHUHVSHLWDURVGLUHLWRVGRFRQWUD
WDGR GHYHUi WDPEpP ]HODU SDUD TXH DV FOiXVXODV J RV GLUHLWRV H DV UHVSRQVDELOLGDGHV GDV SDUWHV DV
HFRQ{PLFRILQDQFHLUDVGRFRQWUDWRVHMDPUHYLVWDVGHVRUWH SHQDOLGDGHVFDEtYHLVHRVYDORUHVGDVPXOWDV
DPDQWHURHTXLOtEULRFRQWUDWXDO
K RVFDVRVGHUHVFLVmR
2XWUD FDUDFWHUtVWLFD EHP SHFXOLDU DR FRQWUDWR
YHGDomR GD H[FHSWLR QRQ DGLPSOHWL
DGPLQLVWUDWLYR p D YHGDomR L  RUHFRQKHFLPHQWRGRVGLUHLWRVGD$GPLQLVWUDomR
FRQWUDFWXV QRV FRQWUDWRV SDUWLFXODUHV XPD SDUWH QmR HPFDVRGHUHVFLVmRDGPLQLVWUDWLYDSUHYLVWDQRDUW
SRGH H[LJLU R FXPSULPHQWR SHOD RXWUD SDUWH VH QmR GD/HL
FXPSULX WDPEpP FRP VXD REULJDomR 0DV QR FRQWUDWR
DGPLQLVWUDWLYRVy D $GPLQLVWUDomRSRGHDUJXLUHPVHX M  DV FRQGLo}HV GH LPSRUWDomR D GDWD H D WD[DGH
IDYRU HVVD H[FHomR HQTXDQWR R SDUWLFXODU QmR
QmR SRGH FkPELRSDUDFRQYHUVmRTXDQGRIRURFDVR
OHYDQWDUHVVDDUJXLomRSDUDSDUDOLVDUXPDREUDRXFHVVDU
DSUHVWDomRGHXPVHUYLoRSUHWH[WDQGRTXHD$GPLQLVWUD O  DYLQFXODomRDRHGLWDOGHOLFLWDomRRXDRWHUPRTXH
omRQmRYHPFXPSULQGRVXDVREULJDo}HVFRQWUDWXDLV DGLVSHQVRXRXDLQH[LJLXDRFRQYLWHHjSURSRVWD
GROLFLWDQWHYHQFHGRU
0DLVXPDYH]RTXHSUHYDOHFHpRSULQFtSLRGDQHFHVVL
GDGHGDFRQWLQXLGDGHGRVHUYLoRS~EOLFRLPSHGLQGRD P D OHJLVODomR DSOLFiYHO j H[HFXomR GR FRQWUDWR
SDUDOLVDomRGRVHUYLoRREUDSHORFRQWUDWDQWHSDUWLFXODU HVSHFLDOPHQWHDRVFDVRVRPLVVRV
6HLVVRRFRUUHURXRSDUWLFXODUFRQWLQXDUDREUDVHUYLoR
RXVXDFRQWLQXDomRVHUiHQWUHJXHDRXWURSDUWLFXODU Q DREULJDomRGRFRQWUDWRGHPDQWHUGXUDQWHWRGDD
H[HFXomRGRFRQWUDWRHPFRPSDWLELOLGDGHFRPDV
REULJDo}HVSRUHOHVDVVXPLGDVWRGDVDVFRQGLo}HV
&OiXVXODV&RQWUDWXDLV2EULJDWyULDV GHKDELOLWDomRHTXDOLILFDomRH[LJLGDVQDOLFLWDomR

&RPRWRGRFRQWUDWRWDPEpPQRVFRQWUDWRVDGPLQLVWUD (PDLVQRVFRQWUDWRVFHOHEUDGRVSHOD$GPLQLVWUDomR
WLYRV DV FOiXVXODV GHYHP VHU UHGLJLGDV SDUD TXH KDMD 3~EOLFDFRPSHVVRDVItVLFDVRXMXUtGLFDVLQFOXVLYHDTXHODV
FODUH]D H SUHFLVmR QDV FRQGLo}HV SDUD VXD H[HFXomR GRPLFLOLDGDVQRHVWUDQJHLURGHYHUiFRQVWDU QHFHVVDULD
GHILQLQGRGHIRUPDLQHTXtYRFDRVGLUHLWRVREULJDo}HVH PHQWHFOiXVXODTXHGHFODUHFRPSHWHQWHRIRURGDVHGHGD
PHQWH
UHVSRQVDELOLGDGHVGDVSDUWHVH[DWDPHQWHFRPRMiGHPDU $GPLQLVWUDomRSDUDGLULPLUTXDOTXHUTXHVWmRFRQWUDWXDO
FDGDQDOLFLWDomRHQDSURSRVWDDTXHVHYLQFXODP
$~QLFDH[FHomR
H[FHomRpTXDQGRVHWUDWDUGHFRQWUDWRUHODWLYR
H[FHomR
1RVFRQWUDWRVDGPLQLVWUDWLYRVGHFRUUHQWHVGHGLVSHQVD j DTXLVLomR GH EHQV H VHUYLoRV UHDOL]DGD SRU XQLGDGHV
RX GH LQH[LJLELOLGDGH GH OLFLWDomR VHUi QHFHVViULR R ILHO DGPLQLVWUDWLYDVFRPVHGHQRH[WHULRU †žGRDUWGD
DWHQGLPHQWR DR DWR TXH RV DXWRUL]RX H GD UHVSHFWLYD /HLQž 
SURSRVWD
 'LU$GPLQLVWUDWLYR &kPDUD/HJLVODWLYDGR'LVWULWR)HGHUDO


*DUDQWLDV&RQWUDWXDLV F FRQWUDWRGHDOXJXHOGHHTXLSDPHQWRVHjXWLOL]DomR
GHSURJUDPDVGHLQIRUPiWLFDSRGHQGRDGXUDomR
4XHPFRQWUDWDTXHUVHPSUHDPtQLPDFHUWH]DGHTXH HVWHQGHUVHSHORSUD]RGHDWp TXDUHQWDHRLWR
DRXWUDSDUWHFXPSULUiFRPVXDREULJDomRFRQWUDWXDO(VVD PHVHVDSyVRLQtFLRGDYLJrQFLDGRFRQWUDWR
JDUDQWLDWDPEpPID]SDUWHGRFRQWUDWR

1RFDVRGRVFRQWUDWRVDGPLQLVWUDWLYRVILFDUiDFULWpULR
GDDXWRULGDGHDGPLQLVWUDWLYDHPFDGDFDVRHGHVGHTXH 3URUURJDomR
SUHYLVWDQRLQVWUXPHQWRFRQYRFDWyULRH[LJLUDSUHVWDomR
SUHVWDomR
GH
GH JDUDQWLDQDVFRQWUDWDo}HVGH
JDUDQWLD REUDVVHUYLoRVHFRP $SURUURJDomR
SURUURJDomRGRFRQWUDWRQDGDPDLVpTXHRSURORQJD
SURUURJDomR
SUDV$SUySULD/HL
/HL
/HLQž
QžHVSHFLILFRXDVPRGDOLGD
 PHQWRGHVXDYLJrQFLD(PERUDWHQKDEXVFDGRDILGHOLGD
GHVGHJDUDQWLDTXHSRGHUmRVHUH[LJLGDVGRVFRQWUDWDQ GHGRVFRQWUDWDQWHVQRFXPSULPHQWRGRVSUD]RVFRQWUDWX
WHV ULJLGH]QDFRQFHVVmRGHSUD]RV
DLVTXDVHTXHDGRWDQGRDULJLGH]
ULJLGH]
D 
D FDXomRHPGLQKHLURHPWtWXORVGHGtYLGD jH[HFXomRGRVFRQWUDWRVD/HL
/HL
/HL Qž
Qž IH]FRQFHV

S~EOLFDRXILGHMXVVyULD V}HV$VVLPSHUPLWLXTXHRVSUD]RVGHLQtFLRGHHWDSDVGH
H[HFXomRRVSUD]RVGHFRQFOXVmRHRVSUD]RVGHHQWUHJD
E 
E ILDQoDEDQFiULD DGPLWHP SURUURJDomR
SURUURJDomR PDV GHYHUmR VHU PDQWLGDV DV
GHPDLVFOiXVXODVGRFRQWUDWRHDVVHJXUDGDDPDQXWHQomR
0DVDFDXomRHPGLQKHLURHPWtWXORVGHGtYLGDS~EOLFD GHVHXHTXLOtEULRHFRQ{PLFRILQDQFHLUR
RX ILGHMXVVyULD TXDQGR H[LJLGDV QmR H[FHGHUmR D 
FLQFRSRUFHQWR GRYDORUGRFRQWUDWRHVHUiOLEHUDGDRX 5HVVDOWDVHTXHHVVDSURUURJDomRQmRpWmRIDFLOPHQWH
UHVWLWXtGDDSyVDH[HFXomRGRFRQWUDWRVHQGRTXHVHIRL GHIHULGD'HSHQGHUiGDRFRUUrQFLDGHPRWLYRVIRUWHVHD
SUHVWDGDHPGLQKHLURVHUiDWXDOL]DGDPRQHWDULDPHQWH OHLDLQGDVXERUGLQDRUHFRQKHFLPHQWRGHVVHVPRWLYRVD
XPSURFHVVRDGPLQLVWUDWLYReLPSRUWDQWHFRQVLGHUDUTXH
$WHQomR 1RVFDVRVGHFRQWUDWRVTXHLPSRUWHPDHQWUH WRGDHTXDOTXHUSURUURJDomRGHSUD]RGHYHUiVHUMXVWLILFD
JD GH EHQV SHOD $GPLQLVWUDomR GRV TXDLV R GD SRU HVFULWR H SUHYLDPHQWH DXWRUL]DGD SHOD PHVPD
FRQWUDWDGR ILFDUi GHSRVLWiULR DR YDORU
YDORU GD DXWRULGDGHTXHRFHOHEURX&RQILUDRVPRWLYRVH[FHSFLRQD
JDUDQWLDGHYHUiVHUDFUHVFLGRRYDORUGHVVHV
JDUDQWLD LVTXHDOHLDGPLWHSDUDDSURUURJDomRGRVSUD]RVFRQWUD
EHQVSDUDTXHD$GPLQLVWUDomR3~EOLFDQ QmR WXDLV
FRUUDULVFRGHVRIUHUTXDOTXHUSUHMXt]R
D DOWHUDomRGRSURMHWRRXHVSHFLILFDo}HVSHOD$GPL
QLVWUDomR
'XUDomR
E VXSHUYHQLrQFLDGDIDWRH[FHSFLRQDORXLPSUHYLVtYHO
HVWUDQKRjYRQWDGHGDVSDUWHVTXHDOWHUHIXQGD
$ /HL
/HL Qž
Qž 
SURFXURXPRUDOL]DUDGXUDomRGR PHQWDOPHQWHDVFRQGLo}HVGHH[HFXomRGRFRQWUD
SUD]RV$SULPHLUD
FRQWUDWRGLVFLSOLQDQGRPHOKRURVVHXSUD]RV
SUD]RV WR
UHJUDpD SURLELomR
SURLELomR GHFRQWUDWRFRPSUD]RGH YLJrQFLD
LQGHWHUPLQDGD DUW†ž $OpPGLVVRDGXUDomR
LQGHWHUPLQDGD GXUDomR
GXUDomR GRV F LQWHUUXSomRGDH[HFXomRGRFRQWUDWRRXGLPLQXLomR
FRQWUDWRV
FRQWUDWRV ILFDUi DGVWULWD j YLJrQFLD
LJrQFLD GRV UHVSHFWLYRV GRULWPRGHWUDEDOKRSRURUGHPHQRLQWHUHVVHGD
FUpGLWRVRUoDPHQWiULRV
FUpGLWRVRUoDPHQWiULRV $GPLQLVWUDomR

&ODURTXHHVVDYLQFXODomRLQYLDELOL]DULDPXLWDVLPSRU G DXPHQWRGDVTXDQWLGDGHVLQLFLDOPHQWHSUHYLVWDVQR
WDQWHV FRQWUDWDo}HV ORQJDV 3RU LVVR D /HL
HL Qž
Qž  FRQWUDWR QRV OLPLWHV SHUPLWLGRV SHOD /HL Qž
HVWDEHOHFHX H[FHo}HV
H[FHo}HV SHUPLWLQGR DOJXPDV FRQWUDWDo}HV 
SRUWHPSRGHVYLQFXODGRGDYLJrQFLDGRFUpGLWRRUoDPHQ
WiULR6mRHVWDVDVH[FHo}HV
H[FHo}HV
H[FHo}HV H LPSHGLPHQWRGHH[HFXomRGRFRQWUDWRSRUIDWRRX
DWRGHWHUFHLURUHFRQKHFLGRSHOD$GPLQLVWUDomRHP
D RV FRQWUDWRV GH SURMHWRV FXMRV SURGXWRV HVWHMDP GRFXPHQWRFRQWHPSRUkQHRjVXDRFRUUrQFLD
FRQWHPSODGRV QDV PHWDV HVWDEHOHFLGDV QR 3ODQR I RPLVVmR QR DWUDVR GH SURYLGrQFLDV D FDUJR GD
3OXULDQXDO RV TXDLV SRGHUmR VHU SURUURJDGRV VH $GPLQLVWUDomR LQFOXVLYH TXDQWR DRV SDJDPHQWRV
KRXYHULQWHUHVVHGD$GPLQLVWUDomRHGHVGHTXHLVVR SUHYLVWRVGHTXHUHVXOWHGLUHWDPHQWHLPSHGLPHQWR
WHQKDVLGRSUHYLVWRQRDWRFRQYRFDWyULR RX UHWDUGDPHQWR QD H[HFXomR GR FRQWUDWR VHP
SUHMXt]RGDVVDQo}HVOHJDLVDSOLFiYHLVDRVUHVSRQVi
E RVFRQWUDWRVGHSUHVWDomRGHVHUYLoRVDVHUHPH[H YHLV
FXWDGRVGHIRUPDFRQWtQXDTXHSRGHUiWHUDVXD
GXUDomRSURUURJDGDSRULJXDLVHVXFHVVLYRVSHUtR 3URUURJDomR$XWRPiWLFD
GRVFRPYLVWDVjREWHQomRGHSUHoRVHFRQGLo}HV
PDLVYDQWDMRVDVSDUDD$GPLQLVWUDomROLPLWDGDD $WHQomRVHPSUHTXHRFRUUHULPSHGLPHQWRRXSDUDOLVD
$WHQomR
VHVVHQWDPHVHV³SRGHQGRVHUSURUURJDGRH[FHS omRRXVXVWDomRGRFRQWUDWRRFURQRJUDPDGHH[HFXomR
FLRQDOPHQWHSRUDWpGR]HPHVHV VHUiSURUURJDGRDXWRPDWLFDPHQWH
SURUURJDGRDXWRPDWLFDPHQWHSRULJXDOWHPSR
SURUURJDGRDXWRPDWLFDPHQWH
&kPDUD/HJLVODWLYDGR
  'LVWULWR)HGHUDO 'LU$GPLQLVWUDWLYR 

5HJLPH-XUtGLFRGRV&RQWUDWRV 3RGHUHVGD$GPLQLVWUDomR

´5HJLPH MXUtGLFRµ VLJQLILFD D QDWXUH]D GDV QRUPDV


MXUtGLFDVTXHGLVFLSOLQDUmRRV´FRQWUDWRVµDGPLQLVWUDWLYRV &RPR EHP REVHUYD +(/< /23(6 0(,5(//(6 SDUD
$IDVWHVH GRV FRQWUDWRV DGPLQLVWUDWLYRV D LQFLGrQFLD GR EHP DWHQGHU DR LQWHUHVVH S~EOLFR TXH VH FRQIHUHP
UHJLPH MXUtGLFR SULYDGR ³ TXH UHJH D UHODomR SULYDGD SRGHUHVj$GPLQLVWUDomRH[SOLFDQGRRVDXGRVR
HVSHFLDLVSRGHUHV
SRGHUHV
6LJQLILFD SRLV TXH RV FRQWUDWRV DGPLQLVWUDWLYRV VH 0HVWUHVTXH7DLVSRGHUHVVmRYHUGDGHLURVLQVWUXPHQWRVGH
UHJLPHMXUtGLFRGHGLUHLWRS~EOLFR
VXEPHWHPDXPUHJLPHMXUtGLFRGHGLUHLWRS~EOLFR
UHJLPHMXUtGLFRGHGLUHLWRS~EOLFR WUDEDOKRDGHTXDGRVjUHDOL]DomRGDVWDUHIDVDGPLQLVWUDWL
YDV 'Dt R VHUHP FRQVLGHUDGRV SRGHUHV
SRGHUHV LQVWUXPHQWDLV
LQVWUXPHQWDLV
PHQWDLV
,QLFLDOPHQWHDSUySULD&RQVWLWXLomR)HGHUDOHQWUHJDj GLYHUVDPHQWH GRV SRGHUHV SROtWLFRV TXH VmR HVWUXWXUDLV H
8QLmRFRPSHWrQFLDSDUDOHJLVODUVREUH´QRUPDV
QRUPDV
QRUPDVJHUDLVGH
JHUDLV RUJkQLFRVSRUTXHFRPS}HPDHVWUXWXUD GR(VWDGRHLQWH
OLFLWDomRHFRQWUDWDomR
FRQWUDWDomR
FRQWUDWDomRHP
HPWRGDV
WRGDVDV
DVPRGDOLGDGHV
PRGDOLGDGHVSDUD
SDUDDV JUDPDRUJDQ]LDomRFRQVWLWXFLRQDO2VSRGHUHV
SRGHUHV
SRGHUHVDGPLQLVWUD
DGPLQLVWUDo}HV
DGPLQLVWUDo}HVS~EOLFDVGLUHWDVDXWiUTXLFDVHIXQGDFLR
S~EOLFDV WLYRVQDVFHPFRPD$GPLQVWUDomRHVHDSUHVHQWDPGLYHUVLIL
WLYRV
QDLV GD 8QLmR (VWDGRV 'LVWULWR )HGHUDO H 0XQLFtSLRV FDGRVVHJXQGRDVH[LJrQFLDVGRVHUYLoRS~EOLFRRLQWHUHVVH
REHGHFLGRRGLVSRVWRQRDUW;;,HSDUDDVHPSUHVDV GDFROHWLYLGDGHHRVREMHWLYRVDTXHVHGLULJHP LQ'LUHLWR
S~EOLFDV H VRFLHGDGHV GH HFRQRPLD PLVWDµ DUW $GPLQLVWUDWLYR%UDVLOHLURHG57SJ 
LQF;;9,, 
(QILP SDUD UHDOL]DU DV DWLYLGDGHV TXH DWHQGDP DR
-iGDVLPSOHVOHLWXUDGHVVHGLVSRVLWLYRFRQIHUHVHTXH LQWHUHVVHFROHWLYRR$GPLQLVWUDGRUSUHFLVDUSRGHUDJLU
D´FRQWUDWDomRSDUDDVDGPLQLVWUDo}HVS~EOLFDVµUHJHUVH 3RULVVRWRGRDJHQWHDGPLQLVWUDWLYRpLQYHVWLGRGHDOJXP
iSRUQRUPDVJHUDLVHVWDWXtGDVHPOHJLVODomRHVSHFtILFD SRGHUTXHGHYHHVWDUYROWDGRHVWULWDPHQWHjUHDOL]DomR
DVHUHGLWDGDSHOD8QLmR GHVVHLQWHUHVVHFROHWLYRIDWRTXHRWRUQDDR PHVPR
WHPSRTXHXPSRGHUGHDJLUWDPEpPXPGHYHUGHDJLU
0DLVTXHLVVRQHVVDOHJLVODomRGHYHUiVHU´REHGHFLGRR
MiTXHWDODomRpDHVVrQFLDHH[FHOrQFLDGR$GPLQLVWUDGRU
GLVSRVWRQRDUWLQF;;,µHPTXHVHGHWHUPLQDTXH´DV
3~EOLFR
REUDVVHUYLoRVFRPSUDVHDOLHQDo}HVVHUmRFFRQWUDWDGRV
PHGLDQWH SURFHVVR GH OLFLWDomR S~EOLFD TXH DVVHJXUH
LJXDOGDGHGHFRQGLo}HVDWRGRVRVFRQFRUUHQWHVFRPFOiX
VXODV TXH HVWDEHOHoDP REULJDo}HV GH SDJDPHQWR PDQ 3RGHU9LQFXODGRH3RGHU'LVFULFLRQiULR
WLGDVDVFRQGLo}HVHIHWLYDVGDSURSRVWDQRVWHUPRVGDOHL
R TXDO VRPHQWH SHUPLWLUi DV H[LJrQFLDV GH TXDOLILFDomR
WpFQLFDHHFRQ{PLFDLQGLVSHQViYHLVjJDUDQWLDGRFXPSUL 23RGHU
3RGHU 3RGHU
3RGHU5HJUD
3RGHUYLQFXODGRWDPEpPFKDPDGRGH3RGHU
YLQFXODGR
PHQWRGDVREULJDo}HVµ GRpDTXHOHTXHDOHLFRQIHUHDRDJHQWHS~EOLFRSDUDD
GR
SUiWLFDGHDWRVGHWHUPLQDQGRRVHOHPHQWRVHRVUHTXLVLWRV
&RQILUDVHSRLVTXHRUHJLPH
UHJLPH
UHJLPH MXUtGLFRGRVFRQWUDWRV
MXUtGLFR QHFHVViULRVjVXDIRUPDOL]DomR
DGPLQLVWUDWLYRVpREULJDWRULDPHQWHS~EOLFRHVWHDGRHP
SDUkPHWURVWUDoDGRVQDSUySULD&RQVWLWXLomR)HGHUDOHQD 3RUHVWHSRGHUD$GPLQLVWUDomRILFDWRWDOPHQWHSUHVD
´OHJLVODomRµIHGHUDOHPTXHD8QLmRHGLWDUiDV´QRUPDV
DRTXHDOHLGHWHUPLQDUHVHRFRUUHUXPGHVYLRGRSUHYLVWR
JHUDLVµDSOLFiYHLVHQXQFLGDGDVQD/HLQž
HPOHLSUDWLFDUiDWRLOHJDOHQXOR2DGPLQLVWUDGRUWHP
PtQLPDFRQGLomRGHOLEHUGDGHSDUDDJLUILFDQGRGHSHQ
5HVVDOWHVHSRUpPTXHDFRPSHWrQFLDGD8QLmRSDUD
GHQWHGRHQXQFLDGRPLQXFLRVRGRGLUHLWRSRVLWLYR$DomR
HGLWDU´QRUPDVJHUDLVµQmRLPSHGHTXH(VWDGRVPHPEURV
DGPLQLVWUDWLYDHQFRQWUDVHGHOLPLWDGDSHODOHL
OHJLVOHPVXSOHPHQWDUPHQWHVREUHRDVVXQWRHVWDEHOHFHQ
GR UHJUDV SUySULDV SDUD D ILUPDWXUD GRV FRQWUDWRV QR
kPELWR GH VXDV UHVSHFWLYDV DGPLQLVWUDo}HV S~EOLFDV ³ 'RVHOHPHQWRVGRDWRDGPLQLVWUDWLYRDFRPSHWrQFLDD
GHVGH TXH p yEYLR QmR FROLGDP WDLV UHJUDV FRP DV ILQDOLGDGHHDIRUPDVmRVHPSUHYLQFXODGDV1DSUiWLFDGH
´QRUPDVJHUDLVµGDOHJLVODomRIHGHUDO XPDDWLYLGDGHYLQFXODGDD$GPLQLVWUDomRGHYHUiDWHUVH
jQRUPDSUHGHWHUPLQDGDRXUHJXODGRUDGHVXDSUiWLFD
'HIDWRVHDFRPSHWrQFLDSULYDWLYDGD8QLmRpUHVWULWD ILFDUiDGVWULWDjOHLHPWRGRVRVUHTXLVLWRVIRUPDGRUHVGR
jHGLomRGH´QRUPDVJHUDLVµ³FRPRRHQXQFLDGHLQtFLR DWR
R DUW LQF;;9,, &)  ³ Ki GH VH FRQVLGHUDU TXH ´$ -i'LVFULFLRQDULHGDGHpDLQWHJUDomR GDYRQWDGHOHJDO
FRPSHWrQFLDGD8QLmRSDUDOHJLVODUVREUHQRUPDVJHUDLV IHLWDSHORDGPLQLVWUDGRUTXHHVFROKHXPFRPSRUWDPHQWR
QmR
QmR H[FOXL
H[FOXL D FRPSHWrQFLDVXSOHPHQWDUGRV(VWDGRV
FRPSHWrQFLDVXSOHPHQWDUGRV(VWDGRVµ³
LDVXSOHPHQWDUGRV(VWDGRV SUHYLDPHQWH YDOLGDGR SHOD QRUPD GHQWUR GRV OLPLWHV GH
FRQVRDQWHRGHVWDFDR†žGRDUW&) OLEHUGDGHUHVXOWDQWHVGDLPSUHFLVmRGDOHLSDUDDWLQJLUD
ILQDOLGDGHS~EOLFDpDGHILQLomRGH5(*,6)(51$1'(6
eRTXHDSRQWDD3URI0$5,$6</9,$=$1(//$',3,(752 '( 2/,9(,5$ HP VXD REUD $WR $GPLQLVWUDWLYR 
´D PDWpULD UHODWLYD D FRQWUDWRV DGPLQLVWUDWLYRV TXH HGLomR6357S
DQWHVHUDGHFRPSHWrQFLDSULYDWLYDGHFDGDHQWHSROtWLFR
UHSDUWHVHDJRUDHQWUHGHXPODGRD8QLmRTXHHVWDEHOH e R H[HUFLGRSHOD$GPLQLVWUDomRDRSUDWLFDURVDWRV
FHQRUPDVJHUDLVHGHRXWUR(VWDGRVH0XQLFtSLRVTXH DGPLQLVWUDWLYRVFRPOLEHUGDGHQDHVFROKDGHVXDFRQYH
HVWDEHOHFHP DV QRUPDV VXSOHPHQWDUHVµ ³ LQ ´'LUHLWR QLrQFLDRSRUWXQLGDGHHFRQWH~GR2DGPLQLVWUDGRUDR
$GPLQLVWUDWLYRµHG$7/$6S  SUDWLFDUXPDWRGLVFULFLRQiULRpOLYUHGHDWXDUGHQWURGRV
 'LU$GPLQLVWUDWLYR &kPDUD/HJLVODWLYDGR'LVWULWR)HGHUDO


SDUkPHWURV IL[DGRV SHOD OHL 0HVPR QR FDVR GH DWR PtQLPDFRQGLomRGHOLEHUGDGHSDUDDJLUILFDQGRGHSHQ
GLVFULFLRQiULRDFRPSHWrQFLDDILQDOLGDGHHDIRUPDVmR GHQWHGRHQXQFLDGRPLQXFLRVRGRGLUHLWRSRVLWLYR$DomR
VHPSUHUHTXLVLWRVYLQFXODGRV DGPLQLVWUDWLYDHQFRQWUDVHGHOLPLWDGDSHODOHL

1mRKiSRVVLELOLGDGHGR3RGHU-XGLFLiULRVXEVWLWXLUD 'RVHOHPHQWRVGRDWRDGPLQLVWUDWLYRDFRPSHWrQFLDD
GLVFULFLRQDULHGDGHGR$GPLQLVWUDGRUSHODGR0DJLVWUDGR ILQDOLGDGHHDIRUPDVmRYLQFXODGDVVHPSUH1DSUiWLFDGH
3RU LVVR TXH GH PRGR HTXtYRFR DILUPDP TXH R 3RGHU XPDDWLYLGDGHYLQFXODGDD$GPLQLVWUDomRGHYHUiDWHUVH
-XGLFLiULRQmRSRGHDSUHFLDUDWRVGLVFULFLRQiULRV jQRUPDSUHGHWHUPLQDGDRXUHJXODGRUDGHVXDSUiWLFD
ILFDUiDGVWULWDjOHLHPWRGRVRVUHTXLVLWRVIRUPDGRUHVGR
2-XGLFLiULRVHPSUHSRGHUiDJLUVyTXHQDDSUHFLDomR DWR
GRDWRGLVFULFLRQiULROLPLWDUVHiDYHULILFDUDOHJDOLGDGH (VVH WDPEpP p R HQVLQDPHQWR GR 3URI',2*2 '(
GRDWRHVHIRUDPREVHUYDGRVRVOLPLWHVGDGLVFULFLRQDULH ),*8(,5('2 025(,5$ 1(72 $WR $WR
$WR YLQFXODGR
YLQFXODGR RX
GDGHSHOR$GPLQLVWUDGRU UHJUDGRpDTXHOHTXHpSUDWLFDGRHPHVWULFDFRQIRUPLGDGH
jV SUHVFULo}HV OHJDLV $ $GPLQLVWUDomR PDQLIHVWD VXD
YRQWDGHQDRSRUWXQLGDGHHSDUDRVHIHLWRVSUHYLVWRVHPOHL
2V/LPLWHVGD3RGHU9LQFXODGR 2DJHQWHQQmRWHPTXDOTXHUPDUJHPGHHVFROKDGHDWXDomR
HGR3RGHU'LVFULFLRQiULR VHMDGHWHPSRRXGHFRQWH~GR LQ&XUVRGH'LUHLWR$GPLQLVWUDWL
YRHG)RUHQVHSJ 
3DUDDSUiWLFDGRDWRDGPLQLVWUDWLYRRDJHQWHSRGHUi
GLVSRU GH PDLRU RX PHQRU OLEHUGDGH GH SRGHURGHU 'LVFULFLRQDULHGDGHpDLQWHJUDomRGDYRQWDGHOHJDO
-iD'LVFULFLRQDULHGDGH
'LVFULFLRQDULHGDGH
VXERUGLQDUVHRXQmRDUHJUDVRXSDUkPHWURV$OJXQVVmR IHLWDSHORDGPLQLVWUDGRUTXHHVFROKHXPFRPSRUWDPHQWR
RXQmRSUDWLFDGRVSHOR$GPLQLVWUDGRUDVHXEHOSUD]HUH SUHYLDPHQWH YDOLGDGR SHOD QRUPD GHQWUR GRV OLPLWHV GH
RXWURV GHYHP VHU SUDWLFDGRV REULJDWRULDPHQWH SHOR OLEHUGDGHUHVXOWDQWHVGDLPSUHFLVmRGDOHLSDUDDWLQJLUD
$GPLQLVWUDGRUQDKRUDGHWHUPLQDGDHGRMHLWRGHWHUPL ILQDOLGDGHS~EOLFDpDGHILQLomRGH5(*,6)(51$1'(6
QDGRSHODSUySULDOHL9HMDPRV '( 2/,9(,5$ HP VXD REUD $WR $GPLQLVWUDWLYR 
HGLomR6357S $VVLPR3RGHU'LVFULFLRQiULR
$WRV9LQFXODGRV
$WRV9LQFXODGRV WDPEpP FRQKHFLGRVFRPRDWRV pRH[HUFLGRSHOD$GPLQLVWUDomRDRSUDWLFDURVDWRVDGPL
UHJUDGRVVmRDTXHOHVFXMRVUHTXLVLWRVHFRQGLo}HVGHVXD QLVWUDWLYRVFRPOLEHUGDGHQDHVFROKDGHVXDFRQYHQLrQFLD
UHDOL]DomRYHPHVWDEHOHFLGRVSHODOHL7RGRVRVHOHPHQWRV RSRUWXQLGDGHHFRQWH~GR
GRDWRDGPLQLVWUDWLYRYrPSUHYLVWRVHPOHLHVHGHOHVVH
DIDVWDURDGPLQLVWUDGRUSUDWLFDUiDWRQXOR$VVLPVHXP 2DGPLQLVWUDGRUDRSUDWLFDUXPDWRGLVFULFLRQiULRp
3UHIHLWR GHFLGH FRQVWUXLU XPD SRQWH HOH GHYHUi
GHYHUi DEULU OLYUH GH DWXDU GHQWUR GRV SDUkPHWURV IL[DGRV SHOD OHL
FRQFRUUrQFLDS~EOLFD 0HVPR QRFDVRGHDWRGLVFULFLRQiULRDFRPSHWrQFLDD
ILQDOLGDGHHDIRUPDVmRVHPSUHUHTXLVLWRVYLQFXODGRV
$WRV
$WRV 'LVFULFLRQiULRV
'LVFULFLRQiULRV  VmR RV TXH D $GPLQLVWUDomR
3~EOLFDSUDWLFDOHYDQGRHPFRQWDPDLVTXHDRSRUWXQLGD 23URI+(/</23(60(,5(//(6HQVLQDTXH$GLVFULFLR
GHRXDFRQYHQLrQFLDGDPHGLGD1RDWRGLVFULFLRQiULRR QDULHGDGHHVWiGRXWULQD)OHLQHUHPSHUPLWLUROHJLVOD
DGPLQLVWUDGRUWHPOLEHUGDGHGHHVFROKDGRFRQWH~GRGR GRU TXH D DXWRULGDGH DGPLQLVWUDWLYD HVFROKD HQWUH DV
DWRGHVHXGHVWLQDWiULRGRPRGRGHVXDUHDOL]DomRGH YiULDVSRVVLELOLGDGHVGHVROXomRDTXHODTXHPHOKRUFRUUHV
VXDRSRUWXQLGDGHHVXDFRQYHQLrQFLD1RH[HPSORDFLPD SRQGD QR FDVR FRQFUHWR DR GHVHMR GD OHL  0DV GHYHUi
DGHFLVmRGR3UHIHLWRGHFRQVWUXLURXQmRXPDSRQWHHP VHPSUHHVWULWDREVHUYkQFLDjOHLSRUTXHDH[RUELWkQFLDGR
VXDFLGDGHHVVDGHFLVmRpWRWDOPHQWHGLVFULFLRQiULDHOH SRGHUGLVFULFLRQiULRFRQVWLWXLDWRLOtFLWRNFRPRWRGDDomR
HVFROKHUi R ORFDO HOH HVFROKHUi TXH WLSR GH SRQWH IDUi YROXQWiULD FDUHQWH GH GLUHLWR  LQ 'LUHLWR $GPLQLVWUDWLYR
%UDVLOHLURHG57SJ 
FRQFUHWRPDGHLUDIHUURHWF HOHGHILQLUiTXDQGRIDUiD
SRQWHHWF
2XWUD DGYHUWrQFLD GR VDXGRVR 0HVWUH p D GH TXH
'LVFULFLRQiULRVVySRGHPVHURVPHLRVHPRGRVGHDGPL
2LQHVTXHFtYHO9,&725181(6/($/HQVLQDYDTXHQmR
QLVWUDUQXQFDRVILQVDDWLQJLU(PWHPDGHILQVDOLomR
KiDWRDGPLQLVWUDWLYRYLQFXODGRPDVDWRDGPLQLVWUDWLYR
pGH%RQQDUGQmRH[LVWHMDPDLVSDUDD$GPLQLVWUDomR
GHFRUUHQWH GR SRGHU YLQFXODGR
YLQFXODGR WDQWR TXDQWR Vy SRU
XPSRGHUGLVFULFLRQiULR3RUTXHQmROKHpQXQFDGHL[DGR
WROHUkQFLD VH SRGHULD FRQVLGHUDU XP DWR GLVFULFLRQiULR
SRGHUGHOLYUHDSUHFLDomRTXDQWRDRILPDDOFDQoDU2ILP
TXDQGR QmR SDVVD HOH GH XP DWR GHFRUUHQWH GR SRGHU
pVHPSUHLPSRVWRSHODVOHLVHUHJXODPHQWRVVHMDH[SOtFLWD
GLVFULFLRQiULRGD$GPLQLVWUDomR LQ3RGHU'LVFULFLRQiULR
GLVFULFLRQiULR VHMDLPSOLFLWDPHQWH ³ LQ 'LUHLWR $GPLQLVWUDWLYR %UDVLOHLUR 
H$omR$UELWUiULDGD$GPLQLVWUDomR5'$ 
HG57S 

$VVLP R DWR DGPLQLVWUDWLYR SUDWLFDGR SRU IRUoD GR 'HRXWURODGRQmR


QmRKiSRVVLELOLGDGHGR3RGHU-XGLFLiULR
QmR
3RGHU 9LQFXODGR p DTXHOH TXH D OHL FRQIHUH DR DJHQWH VXEVWLWXLUDGLVFULFLRQDULHGDGHGR$GPLQLVWUDGRUSHODGR
S~EOLFRSDUDDSUiWLFDGHDWRVGHWHUPLQDQGRRVHOHPHQ 0DJLVWUDGR3RULVVRTXHGHPRGRHTXtYRFRDILUPDPTXH
WRVHRVUHTXLVLWRVQHFHVViULRVjVXDIRUPDOL]DomR R3RGHU-XGLFLiULRQmRSRGHDSUHFLDUDWRVGLVFULFLRQiULRV
2-XGLFLiULRVHPSUHSRGHUiDJLUVyTXHQDDSUHFLDomRGR
3RUHVWHSRGHUD$GPLQLVWUDomRILFDWRWDOPHQWHSUHVD DWRGLVFULFLRQiULROLPLWDUVHiDYHULILFDUDOHJDOLGDGHGR
DRTXHDOHLGHWHUPLQDUHVHRFRUUHUXPGHVYLRGRSUHYLVWR DWRHVHIRUDPREVHUYDGRVRVOLPLWHVGDGLVFULFLRQDULHGDGH
HPOHLSUDWLFDUiDWRLOHJDOHQXOR2DGPLQLVWUDGRUWHP SHOR$GPLQLVWUDGRU
&kPDUD/HJLVODWLYDGR
  'LVWULWR)HGHUDO 'LU$GPLQLVWUDWLYR 

3RGHU+LHUiUTXLFR 3RGHU'LVFLSOLQDU

e R H[HUFLGR SHOR 3RGHU ([HFXWLYR SDUD RUGHQDU 3RU HOH D $GPLQLVWUDomR FRQWUROD R GHVHPSHQKR GDV
FRRUGHQDUFRQWURODUHFRUULJLUDDWXDomRGHVHXVDJHQWHV IXQo}HV DGPLQLVWUDWLYDV H R FRPSRUWDPHQWR LQWHUQR GH
eRTXHGLVS}HR([HFXWLYRSDUDGLVWULEXLUHHVFDORQDUDV VHXV DJHQWHV SXQLQGRRV SHODV IDOWDV DSXUDGDV 0DV
IXQo}HVGHVHXVyUJmRVRUGHQDUHUHYHUDDWXDomRGHVHXV DWHQomRDSXQLomRSHOR3RGHU'LVFLSOLQDUVyDEUDQJHDV
DJHQWHVHVWDEHOHFHQGRDUHODomRGHVXERUGLQDomRHQWUHRV IDOWDVUHODFLRQDGDVFRPRVHUYLoR
VHUYLGRUHV GR VHX TXDGUR GH SHVVRDO FRQIRUPH GHILQH R
VDXGRVR +(/< /23(6 0(,5(//(6 HP VHX FRQVDJUDGR 23RGHU'LVFLSOLQDUWHPVXDRULJHPHUD]mRGHVHUQR
'LUHLWR$GPLQLVWUDWLYR%UDVLOHLURS LQWHUHVVHHQDQHFHVVLGDGHGHDSHUIHLoRDPHQWRSURJUHVVLYRGR
VHUYLoR S~EOLFR HVFUHYH 0$5&(/2 &$(7$12 HP VHX
'R 3RGHU +LHUiUTXLFR GHFRUUH D SRVVLELOLGDGH GH R OLYUR'R3RGHU'LVFLSOLQDU/LVERDSiJ
VXSHULRU GDU RUGHQV ILVFDOL]DU GHOHJDU DYRFDU H
UHYHUDWRVGRVVXEDOWHUQRVHGHVWHVRGHYHUGHREHGLrQ $VSULQFLSDLVSHQDVGLVFLSOLQDUHVVmR
FLD
D UHSUHHQVmR
$'HOHJDomR
'HOHJDomRpDIDFXOGDGHGHTXHGLVS}HD$XWRULGDGH
'HOHJDomR E PXOWD
$GPLQLVWUDWLYDGHFRPHWHUDRXWURDJHQWHDGPLQLVWUDWLYR F VXVSHQVmR
RSRGHUGHYHUGH DJLUQDWXUDOPHQWHVySRGHDXWRUL]DU
DJLU G GHVWLWXLomRGHIXQomR
TXHDOJXpPIDoDSRUHOHDSHVVRDDTXHPDOHLFRQIHULXD H GHPLVVmR
FRPSHWrQFLDSDUDDWXDU I FDVVDomRGHDSRVHQWDGRULDRXGLVSRQLELOLGDGH

+(/</23(60(,5(//(6GHILQHDGHOHJDomRFRPRR $DSXUDomRGHIDOWDGHYHUiVHUSRUPHLRGHSURFHVVR
DWRGH´FRQIHULUDRXWUHPDWULEXLo}HVTXHRULJLQDULDPHQWH DGPLQLVWUDWLYRRXSRUPHLRVVXPiULRVFRQIRUPHDPDLRU
OKHFRPSHWLDP$VGHOHJDo}HVGHQWURGRPHVPR3RGHUVmR RXPHQRUJUDYLGDGHGDLQIUDomRRXGDQDWXUH]DGDSHQD
HPSULQFtSLRDGPLVVtYHLVGHVGHTXHDDXWRULGDGHGHOHJDGD DVHULPSRVWDGDQGRVHRGLUHLWRGHGHIHVDDRDJHQWHHR
HVWHMDHPFRQGLo}HVGHH[HUFrODV2TXHQmRpSRVVtYHQR DSOLFDGRUGHYHUiPRWLYDUDVDQomRLPSRVWD
QRVVRVLVWHPDFRQVWLWXFLRQDOpDGHOHJDomRGHDWULEXLo}HVGH
XP 3RGHU D RXWUR FRPR SRU H[HPSOR GR ([HFXWLYR DR
/HJLVODWLYRRXYLFHYHUVDµ LQ 'LUHLWR $GPLQLVWUDWLYR %UDVLOHLUR
SJ  3RGHU5HJXODPHQWDU

-iD$YRFDomR
$YRFDomRpDIDFXOGDGHGD$XWRULGDGHGHWLUDUD
$YRFDomR
eRH[HUFLGRSHORV&KHIHVGR3RGHU([HFXWLYRQDV
IXQomRGHXPVXERUGLQDGRSDUDTXHHODPHVPDDH[HUFr
WUrV HVIHUDV FRQVWLWXFLRQDLV  3UHVLGHQWH GD 5HS~EOLFD
ODH[FHSFLRQDOPHQWH ´GHL[DTXHHXPHVPRIDoRLVVRµ 6y
*RYHUQDGRUGR(VWDGRH3UHIHLWR0XQLFLSDOTXDQGRD
GHWpP R SRGHU GH DYRFDU TXHP p KLHUDUTXLFDPHQWH
OHLDVHUFXPSULGDUHFODPDDLQWHUIHUrQFLDGD$GPLQLVWUD
VXSHULRU jTXHOH DJHQWH TXH WHULD D FRPSHWrQFLD SDUD
omRFRPRUHTXLVLWRGHVXDDSOLFDomRFRPRREVHUYD&(/62
DWXDU  RX VHMD QmR SRGH DYRFDU XPD $XWRULGDGH GR
$1721,2 %$1'(,5$ '( 0(//2 LQ $WR $GPLQLVWUDWLYR H
PHVPRQtYHODWpSRUTXHLQFRUUHULDHPXVXUSDomRGH 'LUHLWRVGRV$GPLQLVWUDGRUHV6357SJ 
IXQomRS~EOLFDTXHpFULPH
23RGHU5HJXODPHQWDUFRQIHUHDRV&KHIHVGR([HFXWLYR
(PDYRFDQGRDIXQomRD$XWRULGDGHDOLYLDUiDUHVSRQ DSRVVLELOLGDGHGHH[SOLFLWDUDOHLSDUDVXDFRUUHWDH[HFX
VDELOLGDGH GR VXERUGLQDGR FRPSHWHQWH H DVVXPLUi D omRPRXGHH[SHGLUGHFUHWRVDXW{QRPRVVREUHPDWpULDGH
UHVSRQVDELOLGDGH FLYLO DGPLQLVWUDWLYD H FULPLQDO SHOD VXDFRPSHWrQFLDDLQGDQmRGLVFLSOLQDGDSRUOHL +(/</23(6
OLFLWXGHHUHJXODULGDGHGRDWR 0(,5(//(6REUDFLWDGDSiJ 

$YRFDUpFKDPDUDVLIXQo}HVRULJLQDULDPHQWHDWULEXtGDV
´$YRFDU
$YRFDU (QILPpRSRGHUGHEDL[DUUHJXO
UHJXODPHQWRV
UHJXODPHQWRVTXHH[SOLFL
DPHQWRV
DXPVXERUGLQDGR1DGDLPSHGHWDOSUiWLFDTXHSRUpPVy WDPRVPRGRVGHVHH[HFXWDUXPDOHLRXSURYHUVLWXDo}HV
GHYHVHUDGRWDGDSHORVXSHULRUKLHUiUTXLFRTXDQGRKRXYHU DLQGDQmRGLVFLSOLQDGDVHPOHL
PRWLYRV UHOHYDQWHV SDUD WDO VXEVWLWXLomR LVWR SRUTXH D
DYRFDomR GH XP DWR VHPSUH GHVSUHVWLJLD R LQIHULRU H QmR
UDURGHVRUJDQL]DRQRUPDOIXQFLRQDPHQWRGRVHUYLoR3HOD
DYRFDomR VXEVWLWXLVH D FRPSHWrQFLD GR LQIHULRU SHOD GR 3RGHUGH3ROtFLD
VXSHULRU KLHUiUTXLFR FRP WRGDV DV FRQVHTrQFLDV GHVVD
VXEVWLWXLomR QRWDGDPHQWH D GHVORFDomR GR MXt]R RX GD
eRFRQMXQWRGHDWULEXLo}HVFRQFHGLGDVj$GPLQLVWUDomR
LQVWkQFLDSDUDDMXVWiORDRGDDXWRULGDGHDYRFDQWHHPFDVR
SDUDGLVFLSOLQDUHUHVWULQJLUHPIDYRUGRLQWHUHVVHS~EOLFR
GHGHPDQGD$VVLQDOHVHWDPEpPTXHDDYRFDomRGHVRQHUD
DGHTXDGRGLUHLWRVHOLEHUGDGHVLQGLYLGXDLV &DLR7iFLWR3RGHU
RLQIHULRUGHWRGDUHVSRQVDELOLGDGHSHORDWRDYRFDGRSHOR GH3ROtFLDHVHXVOLPLWHVSXEOLFDGRHP5HYLVWDGH'LUHLWR$GPLQLVWUDWL
VXSHULRUµ  HQVLQD +(/< /23(6 0(,5(//(6 HP VHX YRYROSJ 
FRQVDJUDGR'LUHLWR$GPLQLVWUDWLYR%UDVLOHLURS 
 'LU$GPLQLVWUDWLYR &kPDUD/HJLVODWLYDGR'LVWULWR)HGHUDO


eRFRQMXQWRGHSRGHUHVFRHUFLWLYRVH[HUFLGRVSRUDJHQWHV DIHWHPDFROHWLYLGDGHFDGDFLGDGmRFHGHSDUFHODVPtQLPDV
GR (VWDGR VREUH DV DWLYLGDGHV GR FLGDGmR PHGLDQWH D GHVHXVGLUHLWRVjFRPXQLGDGHHR(VWDGROKHUHWULEXLHP
LPSRVLomRGHUHVWULo}HVDWDLVDWLYLGDGHVDILPGHDVVHJXUDU VHJXUDQoD RUGHP KLJLHQH VRVVHJR PRUDOLGDGH H RXWURV
D RUGHP S~EOLFD -RVp &UHWHOOD -~QLRU 'LUHLWR $GPLQLVWUDWLYR EHQHItFLRVS~EOLFRVSURSLFLDGRUHVGRFRQIRUWRLQGLYLGXDOH
)RUHQVH5-S  GREHPHVWDUJHUDO LQ'LUHLWR$GPLQLVWUDWLYR%UDVLOHLURHG57
S 
eDDWXDomRTXHD$GPLQLVWUDomR3~EOLFDH[HUFHVREUH
DV DWLYLGDGHV H RV EHQV HP WXGR R TXH VH UHIHUHP j $WULEXWRV
FROHWLYLGDGH2VDXGRVR+(/</23(60(,5(//(6HQVLQD
YD TXH $WUDYpV GH UHVWULo}HV LPSRVWDV jV DWLYLGDGHV GR 6mRDWULEXWRVGR3RGHUGH3ROtFLD/GLVFULFLRQDULHGDGH
LQGLYtGXR TXH DIHWHP D FROHWLYLGDGH FDGD FLGDGmR FHGH /DXWRH[HFXWRULHGDGH/FRHUFLELOLGDGH
SDUFHODVPtQLPDVGHVHXVGLUHLWRVjFRPXQLGDGHHR(VWDGR
OKHUHWULEXLHPVHJXUDQoDRUGHPKLJLHQHVRVVHJRPRUDOL $GLVFULFLRQDULHGDGHpUHFRQKHFLGDQDOLEHUGDGHGHD
GDGHHRXWURVEHQHItFLRVS~EOLFRVSURSLFLDGRUHVGRFRQIRUWR $GPLQLVWUDomR3~EOLFDHVFROKHUDRSRUWXQLGDGHHDFRQYH
LQGLYLGXDO H GR EHP HVWDU JHUDO LQ 'LUHLWR $GPLQLVWUDWLYR QLrQFLD GH H[HUFLWDU VHX SRGHU GH SROtFLD DSOLFDQGR DV
%UDVLOHLURHG57SJ 
VDQo}HVHYDOHUVHGRVPHLRVQHFHVViULRVjSURWHomRGR
LQWHUHVVHS~EOLFR
)XQGDPHQWR $DXWRH[HFXWRULHGDGHHVWiQDIDFXOGDGHGHD$GPLQLV
WUDomRGHFLGLUHH[HFXWDURVDWRVGH3RGHUGH3ROtFLD
eDLQGDGRHWHUQR+(/</23(60(,5(//(6DHQVLQDQ SRUVLPHVPDVHPQHFHVVLGDGHGRDX[tOLRRXDXWRUL]DomR
oDGHTXH$UD]mRGRSRGHUGHSROtFLDpRLQWHUHVVHVRFLDO GR-XGLFLiULR
HRVHXIXQGDPHQWR
IXQGDPHQWRHVWiQDVXSUHPDFLDJHUDOTXHR(VWDGR
IXQGDPHQWR
H[HUFH HP VHX WHUULWyULR VREUH WRGDV DV SHVVRDVV EHQV H )LQDOPHQWH D FRHUFLELOLGDGH VLJQLILFD R SRGHU GH D
DWLYLGDGHV VXSUHPDFLD TXH VH UHYHOD QRV PDQGDPHQWRV $GPLQLVWUDomR3~EOLFDLPSRUPHGLGDVFRDWLYDVLQFOXVLYH
FRQVWLWXFLRQDLVHQDVQRUPDVGHRUGHPS~EOLFDTXHDFDGD XWLOL]DQGRIRUoDItVLFDVHKRXYHURSRVLomRGRLQIUDWRURX
SDVVR RS}HP FRQGLFLRQDPHQWRV H UHVWULo}HV DRV GLUHLWRV VHKRXYHUQHFHVVLGDGHGHVXSULUDOJXPDRPLVVmR
LQGLYLGXDLVHPIDYRUGDFROHWLYLGDGHLQFXPELQGRDR3RGHU
3~EOLFR R VHX SROLFLDPHQWR DGPLQLVWUDWLYR LQ 'LUHLWR
$GPLQLVWUDWLYR%UDVLOHLURHG57SJ 
0HLRVGH$WXDomR

23RGHUGH3ROtFLDpH[HUFLGRDWUDYpVGHRUGHQVSURLEL
2EMHWRH)LQDOLGDGH
o}HV H QRUPDV OLPLWDGRUDV H VDQFLRQDGRUDV GR FRP
SRUWDPHQWRGRVLQGLYtGXRVTXHFRQYLYHPQDVRFLHGDGH
,QHVFRQGLYHOPHQWHVyVHUHFRQKHFHDOHJLWLPLGDGHGR
*HUDOPHQWHQRXVRGRVEHQVHQRH[HUFtFLRGDVDWLYLGD
SRGHUGHSROtFLDTXDQGRYLVDHOHDSURWHJHURLQWHUHVVH
GHVRFRQWUROHGR3RGHUGH3ROtFLDpPDWHULDOL]DGRSRU
S~EOLFR  DVVLP HQWHQGLGR R VXSHULRU LQWHUHVVH GD
DOYDUiV
FRPXQLGDGHFRPSUHHQGLGRVGHVGHRLQWHUHVVHSDWULPR
QLDOPDWHULDOPRUDODWpRLQWHUHVVHHVSLULWXDOGRSRYR
$OYDUi p R LQVWUXPHQWR GD OLFHQoD RX GD DXWRUL]DomR
$OYDUi
DWXWHODGDVLQVWLWXLo}HVHGRVDQVHLRVQDFLRQDLV
SDUDDSUiWLFDGHDWRUHDOL]DomRGHDWLYLGDGHRXH[HUFtFLR
GHGLUHLWRGHSHQGHQWHGHSROLFLDPHQWRDGPLQLVWUDWLYR +HO\
2 3URI',2*2 '( ),*8(,5('2 025(,5$ 1(72
/RSHV0HLUHOOHVREUDFLWDGDSiJ 
GHVWDFDTXHRSRGHUGHSROtFLDWHPSRUREMHWR REMHWROLPLWDUH
REMHWR
FRQGLFLRQDU R H[HUFtFLR GH GLUHLWRV IXQGDPHQWDLV 2 DOYDUi VH GHILQLWLYR FKDPDVH OLFHQoD
OLFHQoD R DOYDUi
FRPSDWLELOL]DQGRRV FRP LQWHUHVVHV S~EOLFRV OHJDOPHQWH SUHFiULRpDDXWRUL]DomR
DXWRUL]DomR
DXWRUL]DomR
GHILQLGRVHTXHVXDILQDOLGDGH
ILQDOLGDGHpSHUPLWLUXPDFRQYLYrQFLD
ILQDOLGDGH
RUGHLUDHYDOLRVD LQ&XUVRGH'LUHLWR$GPLQLVWUDWLYRHG)RUHQ 6DQo}HV
VHSJ 

23RGHUGH3ROtFLDYHPUHYHVWLGRGHVDQo}HVTXHVmR
([WHQVmRH/LPLWHV
HOHPHQWRVGHFRDomRHLQWLPLGDomR1HQKXPDRUGHPVHULD
LPSHUDWLYDRXVHMDQmRWHULDRSRGHUGHVHID]HUREHGH
23RGHUGH3ROtFLDHQFRQWUDOLPLWHVIXQGDPHQWDLVRV
FHU VH QmR KRXYHVVH D DPHDoD GH XPD FRQVHTrQFLD
GLUHLWRV GRV FLGDGmRV DV SUHUURJDWLYDV LQGLYLGXDLV GRV
QHJDWLYDGHXPFDVWLJR XPDVDQomR 
FLGDGmRVHDVOLEHUGDGHVS~EOLFDV0DVSRURXWURODGRp
R3RGHUGH3ROtFLDTXHHYLWDHFRQWpPRVDEXVRVGRGLUHLWR
$VPDLVFRQKHFLGDVVDQo}HVTXHVHUYHPGHLQVWUXPHQ
LQGLYLGXDO
WR GH FRDomR H LQWLPLGDomR SDUD TXH R SDUWLFXODU VH
VXEPHWD DR SRGHU GH SROtFLD  OHPEUDGDV SRU +(/<
2VDXGRVR+(/</23(60(,5(//(6REVHUYDTXH2V
/23(60(,5(//(6 LQ'LUHLWR$GPLQLVWUDWLYR%UDVLOHLUR
OLPLWHVGRSRGHUGHSROtFLDDGPLQLVWUDWLYDVmRGHPDUFDGRV
 HG57 SJ   VmR DV PXOWDV D LQWHUGLomR GH
SHORLQWHUHVVHVRFLDOHPFRQFLOLDomRFRPRVGLUHLWRVIXQGD
DWLYLGDGHRIHFKDPHQWRGHHVWDEHOHFLPHQWRDGHPROLomR
PHQWDLVGRLQGLYtGXRDVVHJXUDGRVQD&RQVWLWXLomR$WUDYpV
GH FRQVWUXomR R HPEDUJR DGPLQLVWUDWLYR GH REUD D
GH UHVWULo}HV LPSRVWDV jV DWLYLGDGHV GR LQGLYtGXR TXH
&kPDUD/HJLVODWLYDGR
 'LVWULWR)HGHUDO 'LU$GPLQLVWUDWLYR 

GHVWUXLomRGHREMHWRVDLQXWLOL]DomRGHJrQHURVDSURLEL DSDUHQWHVDWLVIDomRGHXPLQWHUHVVHFROHWLYRLPHGLDWRRUD
omR GH IDEULFDomR RX FRPpUFLR GH FHUWRV SURGXWRV D VXWLV HQWUDQKDGDV QR H[FHVVR
H[FHVVR GH SRGHU
SRGHU RX QR GHVYLR
GHVYLR GH
YHGDomRGHORFDOL]DomRGHLQG~VWULDVRXGHFRPpUFLRHP ILQDOLGDGH
ILQDOLGDGH
GHWHUPLQDGDV]RQDVHWF
2([FHVVR

&RQGLo}HVGH9DOLGDGH 2FRUUHUiH[FHVVRGHSRGHU
H[FHVVRGHSRGHUVHPSUHTXHDDXWRULGDGH
H[FHVVRGHSRGHU
IRUDOpPGRTXHVHXGHYHUGHDJLUOKHSHUPLWLULDH[RUEL
2V DWRV SUDWLFDGRV D WtWXOR GH H[HUFtFLR GH SRGHU GH WDQGRDVVLPQRXVRGDVIDFXOGDGHVQHFHVViULDVHFRQVHQ
SROtFLD VmR DWRV DGPLQLVWUDWLYRV FRPR TXDOTXHU RXWUR WLGDVSHODOHL
UD]mRSRUTXHVHXVUHTXLVLWRVGHYDOLGDGHVmRH[DWDPHQWH
RVPHVPRVGHYHPVHUSUDWLFDGRVSHORDJHQWHFRPSHWHQWH 2 VDXGRVR +(/< /23(6 0(,5(//(6 HQVLQD TXH 2
GHYHWHUSRUILQDOLGDGHDWXWHODGHXPLQWHUHVVHS~EOLFRH H[FHVVR GH SRGHU RFRUUH TXDQGR D DXWRULGDGH HPERUD
VHUSUDWLFDGRQDIRUPDSUHYLVWDHPOHL FRPSHWHQWHSDUDSUDWLFDURDWRYDLDOpPGRSHUPLWLGRHVH
H[RUELWDQRXVRGHVXDVIDFXOGDGHVDGPLQLVWUDWLYDV([FHGH
$FUHVFHQWHVHDSHQDV D SURSRUomR
D DQHFHVVLGDGHGHSURSRUomR
D SRUWDQWRDVXDFRPSHWrQFLDOHJDOHFRPLVVRLQYDOLGDRDWR
HQWUHDVDQomRDSOLFDGDjQDWXUH]DHJUDYLGDGHGDLQIUD SRUTXHQLQJXpPSRGHDJLUHPQRPHGD$GPLQLVWUDomRIRUD
omR SDUD TXH QmR UHVXOWH GHVSURSRUFLRQDO VDFULItFLR GH GR TXH D OHL OKH SHUPLWH 2 H[FHVVR GH SRGHU WRUQD R DWR
GLUHLWRRXOLEHUGDGHVHPDFRUUHVSRQGHQWHYDQWDJHPSDUD DUELWUiULRLOtFLWRHQXOR LQ'LUHLWR$GPLQLVWUDWLYR%UDVLOHLUR
R LQWHUHVVH S~EOLFR QmR VH PDWD R FDFKRUUR DSHQDV HG57S 
SRUTXHRGRQRROHYRXjSUDLD  E E OHJDOLGDGHLQGLVSHQ
E DOHJDOLGDGH
OHJDOLGDGH
ViYHOjOHJLWLPLGDGHGRVDWRVGHSROtFLD
2'HVYLRGH)LQDOLGDGH
+(/</23(60(,5(//(6DSURSyVLWRDGYHUWHTXH
$GHVSURSRUFLRQDOLGDGHGRDWRGHSROtFLDRXRVHXH[FHVVR 0HQRVHVW~SLGRSRUpPGHLJXDOLPRUDOLGDGHpR
HTXLYDOHDDEXVRGHSRGHUHFRPRWDOWLSLILFDLOHJDOLGDGH GHVYLR
GHVYLRGH
DEXVRGHSRGHUGHVYLR GHILQDOLGDGHHPTXHDDXWRULGD
ILQDOLGDGH
QXOLILFDGRUDGDVDQomRHTXHQmRVHFRPSUHHQGHRSRGHU GHDWXDQGRQRVOLPLWHVGHVXDFRPSHWrQFLDHPYHUGDGH
GH XWLOL]DU PHLRV LOHJDLV SDUD D VXD FRQVHFXomR HPERUD SUHWH[WDDOFDQoDUXPILPPDVQDSUiWLFDYLVDDSURGX]LU
OtFLWRHOHJDORILPSUHWHQGLGR LQ'LUHLWR$GPLQLVWUDWLYR%UDVLOHL XPUHVXOWDGRGLYHUVRHLOHJDO
URHG57S  DEXVR
DEXVRGH
1DOHLTXHGLVFLSOLQDYDDDomRSRSXODURDEXVR GHSRGHU
SRU
SRU GHVYLR
GHVYLR GH
GH ILQDOLGDGHpUHFRQKHFLGRFRPRDTXHOHHP
ILQDOLGDGH
TXHRDJHQWHSUDWLFDRDWRYLVDQGRDILPGLYHUVRGDTXHOH
SUHYLVWRH[SOtFLWDRXLPSOLFLWDPHQWHQDUHJUDGHFRPSHWrQ
8VRH$EXVRGH3RGHU
FLD(KiDLQGDRDEXVRGHSRGHUSRU SRU
SRU RPLVVmRHPTXH
RPLVVmR
DDXWRULGDGHS~EOLFDFDXVDDOHVmRDRGLUHLWRLQGLYLGXDOGD
&RPR YLPRV R $GPLQLVWUDGRU 3~EOLFR WHP SRGHUHV YtWLPDGHL[DQGRGHH[HFXWDURVHUYLoRS~EOLFRTXHpGH
SDUDID]HULPSRURVLQWHUHVVHVGDDGPLQLVWUDomRS~EOLFD VHXGHYHU
QmRUDURID]HQGRVXFXPELULQWHUHVVHVGHSDUWLFXODUHV2
H[HUFtFLRGHVVHSRGHUGHYHUiREHGHFHUVHPSUHDRVOLPL 1RVVD&RQVWLWXLomR)HGHUDOSURFXUDSURWHJHUQRVFRQWUD
WHV H FRQGLo}HV GD OHL 1mR SRGHUi R $GPLQLVWUDGRU RVDEXVRVGHSRGHUGDQGRQRV D D RPDQGDGRGHVHJX
3~EOLFRSUHWH[WDURH[HUFtFLRGHXPDWDUHIDS~EOLFDSDUD UDQoD EE RKDEHDVFRUSXVRKDEHDVGDWDHR FF GLUHLWR
UHDOL]DU SHUVHJXLo}HV SHVVRDLV RX DSDQLJXDU DPLJRV GHUHSUHVHQWDomRFRPRLQVWUXPHQWRVLPHGLDWRVDIRUD
DPDQWHVHWF p FODUR D SURWHomR GR 3RGHU -XGLFLiULR FRP DV Do}HV
RUGLQiULDV
7RGR DWR GH DXWRULGDGH Ki GH VHU SDXWDGR QD OHL
DILQDGRFRPDPRUDOHYLVDQGRDDWHQGHUH[FOXVLYDPHQWH +DYHQGRDEXVRGHSRGHUHPTXDOTXHUGDVPRGDOLGD
DRLQWHUHVVHS~EOLFR$VVLPRXVRGRSRGHUSHODDXWRULGD GHV H[FHVVRRXGHVYLRGHILQDOLGDGH RDWRDGPLQLVWUDWLYR
GH S~EOLFD p XPD SUHUURJDWLYD GHOD PDV VXEPLVVD jV SDGHFHUiGHQXOLGDGH
QXOLGDGH
QXOLGDGH
QRUPDVOHJDLVjILQDOLGDGHHVSHFtILFDGRDWRREMHWLYDGRH
DRLQWHUHVVHS~EOLFR +(/< /23(6 0(,5(//(6 DQRWD TXH 2 GHVYLR HVYLR GH
ILQDOLGDGH RX GH
ILQDOLGDGH GH SRGHU
SRGHU VH YHULILFD TXDQGR D DXWRULGDGH
XVRGRSRGHUVHUiVHPSUH
2EVHUYDGDVWDLVGLUHWUL]HVRXVR
XVR HPERUDDWXDQGRQRVOLPLWHVGHVXDFRPSHWrQFLDSUDWLFDR
OtFLWRSRUTXHFRQVRDQWHDOHL DWRSRUPRWLYRVRXFRPILQVGLYHUVRVGRVREMHWLYDGRVSHODOHL
RXH[LJLGRVSHORLQWHUHVVHS~EOLFR2GHVYLRGHILQDOLGDGHRX
DEXVR
DEXVRGR
+DYHUiWRGDYLDDEXVR GRSRGHUVHPSUHTXHDDXWRUL
SRGHU GH SRGHU p DVVLP D YLRODomR LGHROyJLFD GD OHL RX SRU
GDGH XOWUDSDVVDU RV OLPLWHV GH VXDV DWULEXLo}HV RX VH RXWUDV SDODYUDV D YLRODomR PRUDO GD OHL FROLPDQGR R
GHVYLDUGDVILQDOLGDGHVDGPLQLVWUDWLYDV5HSXJQDQWHPHQWH DGPLQLVWUDGRUS~EOLFRILQVQmRTXHULGRVSHOROHJLVODGRURX
LOHJDOHHVVHQFLDOPHQWHLPRUDORDEXVRGHSRGHUVHUHYHOD XWLOL]DQGRPRWLYRVHPHLRVLPRUDLVSDUDDSUiWLFDGHXP
GH YiULDV IRUPDV GDV PDLV JURVVHLUDV FRP YLROrQFLD H DWR DGPLQLVWUDWLYR DSDUHQWHPHQWH OHJDO ³ LQ 'LUHLWR
WUXFXOrQFLD FRPXPQDDWLYLGDGHFRQH[DDRSRGHUGHSROtFLD RX $GPLQLVWUDWLYR%UDVLOHLURHG57S 
GLVVLPXODGD HVWHOLRQDWDULDPHQWH DFREHUWDQGRVH QXPD
 'LU$GPLQLVWUDWLYR &kPDUD/HJLVODWLYDGR'LVWULWR)HGHUDO


GHFLQTHQWDSRUFHQWRGRSDWULP{QLRRXGDUHFHLWDDQXDO
/HLQž OLPLWDQGRVHQHVWHVFDVRVDVDQomRSDWULPRQLDOjUHSHU
FXVVmRGRLOtFLWRVREUHDFRQWULEXLomRGRVFRIUHVS~EOLFRV
/HLGH,PSURELGDGH$GPLQLVWUDWLYD
¢0DV DILQDO R TXH FDUDFWHUL]D D LPSURELGDGH" $ OHL
GLVWLQJXH WUrV   HVSpFLHV GH DWRV GH LPSURELGDGH
DGPLQLVWUDWLYD D  RV TXH SURGX]HP HQULTXHFLPHQWR
'HULYDGD GR ODWLP ³ LPSURELWDWH ³ D H[SUHVVmR
LOtFLWR E RVTXHFDXVDPGDQRDRSDWULP{QLRS~EOLFRH
LPSURELGDGHUHYHODDGHVRQHVWLGDGHRPDXFDUiWHUVHQmR
F RVTXHDWHQWDPFRQWUDRVSULQFtSLRVGDDGPLQLVWUDomR
DWp D PDOGDGH H D SHUYHUVLGDGH ³ H TXDOLILFDGD SHOR
S~EOLFD
DGMHWLYR´DGPLQLVWUDWLYDµUHSDVVDGDDLGpLDGDFRQGXWD
GHVRQHVWDGRDJHQWHS~EOLFRTXHDGPLQLVWUDRSDWULP{QLR
2VDWRVGHLPSURELGDGHDGPLQLVWUDWLYDTXHLQVWUXPHQ
HRVLQWHUHVVHVS~EOLFRV
WDPR´HQULTXHFLPHQWRLOtFLWRµVmRGHILQLGRVHGHVFULWRVQR
DUWžGD/HLQžHQTXDQWRRVTXH´FDXVDPGDQR
$DWLYLGDGHDGPLQLVWUDWLYDKiGHVHUHYHVWLUGHFUpGLWR
DRHUiULRµHVWmRSUHYLVWRVQRDUWGDUHIHULGDOH[TXH
H HILFLrQFLD SUHQKH GH pWLFD H OHJDOLGDGH ³ D SDU GH
GHVFUHYH QR DUW DWRV GH LPSURELGDGH DGPLQLVWUDWLYD
GLUHFLRQDGRSDUDDHIHWLYDVDWLVIDomRGRLQWHUHVVHVRFLDO
TXH DWHQWDP FRQWUD RV  ´SULQFtSLRV GD DGPLQLVWUDomR
VHPHQVDQFKDVjPHVTXLQKH]GHVHUYLOLGDGHDRVLQWHUHVVHV
S~EOLFDµ&RQILUD
SULYDGRV RX SDUWLFXODUHV  RSUREULRVR VHUi R DJHQWH
&DStWXOR,
S~EOLFRTXHDSDUWDUVXDFRQGXWDGHVVHVSDUkPHWURV
'LVSRVLo}HV*HUDLV
&RPR EHP REVHUYDP 0$5,12 )$==$*/,1, ),/+2 (
$UWž 2VDWRV
DWRV
DWRVGH
GHLPSURELGDGH SUDWLFDGRVSRU
LPSURELGDGHSUDWLFDGRV SRUTXDOTXHU
287526´UHFODPDVHGHWRGRHTXDOTXHUDJHQWHS~EOLFR
DJHQWH
DJHQWH S~EOLFRVHUYLGRURXQmRFRQWUDDDGPL
S~EOLFR
GHTXDOTXHUQtYHOTXHSRVVXDXPFRQWLQJHQWHPtQLPRGH
QLVWUDomR GLUHWD LQGLUHWD RX IXQGDFLRQDO GH
SUHGLFDGRV OLJDGRV j PRUDOLGDGH S~EOLFD WDLV FRPR D
TXDOTXHUGRV3RGHUHVGD8QLmRGRV(VWDGRVGR
KRQHVWLGDGHDOHDOGDGHHDLPSDUFLDOLGDGH6mRTXDOLGD
'LVWULWR)HGHUDOGRV0XQLFtSLRVGH7HUULWyULRGH
GHV HVVHQFLDLV QDWXUDOPHQWH H[LJtYHLV HP TXDOTXHU
HPSUHVDLQFRUSRUDGDDRSDWULP{QLRS~EOLFRRXGH
VHJPHQWR GD DWLYLGDGH SURILVVLRQDO H FRP PXLWR PDLV
HQWLGDGHSDUDFXMDFULDomRRXFXVWHLRRHUiULRKDMD
UD]mR GDTXHOHV TXH LQWHJUDP RV TXDGURV S~EOLFRV H
FRQFRUULGRRXFRQFRUUDFRPPDLVFLQTHQWDSRU
JHUHQFLDP EHQV GD FROHWLYLGDGH GRV TXDLV QmR SRGHP
FHQWR GR SDWULP{QLR RX GD UHFHLWD DQXDO VHUmR
GLVSRU H SHORV TXDLV GHYHP ]HODU 6H p QDWXUDO TXH D
SXQLGRVQDIRUPDGHVWD/HL
FRQGXWD GRV DJHQWHV S~EOLFRV HVWHMD SHUPDQHQWHPHQWH
VREDILVFDOL]DomRSRSXODUHVWDSRUpPTXDVHVHPSUHp
†~QLFR (VWmRWDPEpPVXMHLWRVjVSHQDOLGDGHVGHVWD/HLRV RV
LQVXILFLHQWHSDUDFRUULJLUDVGLVWRUo}HVSDWURFLQDGDVSRU
DWRVGHLPSURELGDGHSUDWLFDGRVFRQWUDRSDWULP{
DWRV GHLPSURELGDGHSUDWLFDGRVFRQWUDRSDWULP{
FRQGXWDVTXHVHPDFDUUHWDUTXDOTXHUGDQRDR7HVRXURH
QLRGHHQWLGDGHTXHUHFHEDVXEYHQomREHQHItFLR
QLR
VHP HQVHMDU D FRQILJXUDomR GR HQULTXHFLPHQWR LOtFLWR
RXLQFHQWLYRILVFDORXFUHGLWtFLRGHyUJmRS~EOLFR
IHUHPSURIXQGDPHQWHRVSULQFtSLRVpWLFRVHMXUtGLFRVTXH
EHPFRPRGDTXHODVSDUDFXMDFULDomRRXFXVWHLR
SUHVLGHP D $GPLQLVWUDomR 3~EOLFDµ ³ LQ ´,PSURELGDGH
$GPLQLVWUDWLYR$VSHFWRV-XUtGLFRVGD'HIHVDGR3DWULP{QLR3~EOLFRµ RHUiULRKDMDFRQFRUULGRRXFRQFRUUDFRPPHQRV
HG$7/$6S  GH FLQTHQWD SRU FHQWR GR SDWULP{QLR RX GD
UHFHLWDDQXDOOLPLWDQGRVHQHVWHVFDVRVDVDQomR
(LVDtRVIXQGDPHQWRVSDUDDHGLomRGD/HLQž SDWULPRQLDOjUHSHUFXVVmRGRLOtFLWRVREUHDFRQ
YLVDQGRQmRVyjULJRURVDSXQLomRGLVFLSOLQDUHDGPLQLV WULEXLomRGRVFRIUHVS~EOLFRV
WUDWLYD GRV $JHQWHV 3~EOLFRV tPSURERV
PSURERV ³ FRPR j VXD
UHVSRQVDELOL]DomR FULPLQDO H VREUHWXGR FLYLO
FLYLO SDUD D $UWž 5HSXWDVH DJHQWH S~EOLFR SDUD RV HIHLWRVGHVWD
UHFRPSRVLomRGRSDWULP{QLRS~EOLFRFRQWUDDGDQRVLGDGH /HLWRGRDTXHOHTXHH[HUFHDLQGDTXHWUDQVLWRULD
GHVVDVGHVRQHVWLGDGHV PHQWHRXVHPUHPXQHUDomRSRUHOHLomRQRPHD
omRGHVLJQDomRFRQWUDWDomRRXTXDOTXHURXWUD
$WHQWHVHPDLVXPDYH]TXHVHUmRDOFDQoDGRVWRGRVRV IRUPDGHLQYHVWLGXUDRXYtQFXORPDQGDWRFDUJR
DJHQWHVTXHFRPHWHUHPLPSURELGDGHH[HUFtFLRGHPDQGD HPSUHJRRXIXQomRQDVHQWLGDGHVPHQFLRQDGDV
WRFDUJRHPSUHJRRXIXQomRQDDGPLQLVWUDomRS~EOLFD QRDUWLJRDQWHULRU
GLUHWD LQGLUHWD RX IXQGDFLRQDO ³ SRXFR LPSRUWD GH
)HGHUDO(VWDGXDO0XQLFLSDORX'LVWULWDO³RXPHVPRD $UWž $V GLVSRVLo}HV GHVWD /HL VmR DSOLFiYHLV QRTXH
HPSUHVDLQFRUSRUDGDDRSDWULP{QLRS~EOLFRRXGHHQWLGD FRXEHU jTXHOH TXH PHVPR QmR VHQGR DJHQWH
GHSDUDFXMDFULDomRRXFXVWHLRRHUiULRKDMDFRQFRUULGR S~EOLFRLQGX]DRXFRQFRUUDSDUDDSUiWLFDGRDWR
RXFRQFRUUDFRPPDLVGHFLQTHQWDSRUFHQWRGRSDWULP{ GHLPSURELGDGHRXGHOHVHEHQHILFLHVRETXDOTXHU
QLRRXGDUHFHLWDDQXDO IRUPDGLUHWDRXLQGLUHWD

$OLiVDFRQGXWDtPSUREDUHVWDUiFDUDFWHUL]DGDWDPEpP $UWž 2VDJHQWHVS~EOLFRVGHTXDOTXHUQtYHORXKLHUDU


QDLQYHVWLGDFRQWUDRSDWULP{QLRGHHQWLGDGHTXHUHFHED TXLDVmRREULJDGRVDYHODUSHODHVWULWDREVHUYkQFLD
VXEYHQomREHQHItFLRRXLQFHQWLYRILVFDORXFUHGLWtFLRGH GRV SULQFtSLRV GH OHJDOLGDGH LPSHVVRDOLGDGH
yUJmRS~EOLFREHPFRPRGDTXHODVSDUDFXMDFULDomRRX PRUDOLGDGHHSXEOLFLGDGHQRWUDWRGRVDVVXQWRV
FXVWHLRRHUiULRKDMDFRQFRUULGRRXFRQFRUUDFRPPHQRV TXHOKHVmRDIHWRV
&kPDUD/HJLVODWLYDGR
  'LVWULWR)HGHUDO 'LU$GPLQLVWUDWLYR 

$UWž 2FRUUHQGR
2FRUUHQGROHVmR
OHVmRDR
DRSDWULP{QLR
SDWULP{QLRS~EOLFR
S~EOLFRSRU
SRUDomR ,9 XWLOL]DUHPREUDRXVHUYLoRSDUWLFXODUYHtFXORV
RX
RXRPLVVmR
RPLVVmRGRORVD
GRORVDRX
RXFXOSRVD
FXOSRVDGR
GRDJHQWH
DJHQWHRX
RXGH PiTXLQDVHTXLSDPHQWRVRXPDWHULDOGHTXDOTXHU
WHUFHLUR
WHUFHLUR GDUVHi
GDUVHi R LQWHJUDO
LQWHJUDO UHVVDUFLPHQWR
UHVVDUFLPHQWR GR QDWXUH]D GH SURSULHGDGH RX j GLVSRVLomR GH
GDQR
GDQR TXDOTXHU GDV HQWLGDGHV PHQFLRQDGDV QR DUWž
GHVWD /HL EHP FRPR R WUDEDOKR GH VHUYLGRUHV
$UWž 1R FDVR GH HQULTXHFLPHQWR LOtFLWR SHUGHUiR S~EOLFRV HPSUHJDGRV RX WHUFHLURV FRQWUDWDGRV
DJHQWH
DJHQWH S~EOLFR
S~EOLFR RX
RX WHUFHLUREHQHILFLiULRRVEHQV
WHUFHLUREHQHILFLiULRRVEHQV SRUHVVDVHQWLGDGHV
RXYDORUHVDFUHVFLGRVDRVHXSDWULP{QLR
9 UHFHEHUYDQWDJHPHFRQ{PLFDGHTXDOTXHUQDWXUH
$UWž 4XDQGR R DWR GH LPSURELGDGH FDXVDU OHVmRDR ]DGLUHWDRXLQGLUHWDSDUDWROHUDUDH[SORUDomR
SDWULP{QLR S~EOLFR RX HQVHMDU HQULTXHFLPHQWR RX D SUiWLFD GH MRJRV GH D]DU GH OHQRFtQLR GH
LOtFLWRFDEHUiDDXWRULGDGHDGPLQLVWUDWLYDUHVSRQ QDUFRWUiILFR GH FRQWUDEDQGR GH XVXUD RX GH
TXDOTXHU RXWUD DWLYLGDGH LOtFLWD RX DFHLWDU SUR
ViYHO SHOR LQTXpULWR UHSUHVHQWDU DR 0LQLVWpULR
PHVVDGHWDOYDQWDJHP
3~EOLFR SDUD D LQGLVSRQLELOLGDGH GRV EHQV GR
LQGLFLDGR
9, UHFHEHUYDQWDJHPHFRQ{PLFDGHTXDOTXHUQDWXUH
]DGLUHWDRXLQGLUHWDSDUDID]HUGHFODUDomRIDOVD
†~QLFR $LQGLVSRQLELOLGDGHDTXHVHUHIHUHRFDSXWGHVWH VREUHPHGLomRRXDYDOLDomRHPREUDVS~EOLFDVRX
DUWLJR UHFDLUi VREUH RV EHQV TXH DVVHJXUHP R TXDOTXHU RXWUR VHUYLoR RX VREUH TXDQWLGDGH
LQWHJUDOUHVVDUFLPHQWRGRGDQRRXVREUHRDFUpV SHVR PHGLGD TXDOLGDGH RX FDUDFWHUtVWLFD GH
FLPR SDWULPRQLDO UHVXOWDQWH GR HQULTXHFLPHQWR PHUFDGRULDV RX EHQV IRUQHFLGRV D TXDOTXHU GDV
LOtFLWR HQWLGDGHVPHQFLRQDGDVQRDUWžGHVWD/HL

$UWž 2VXFHVVRUGDTXHOHTXHFDXVDUOHVmRDRSDWULP{ 9,, DGTXLULUSDUDVLRXSDUDRXWUHPQRH[HUFtFLRGH


QLR S~EOLFR RX VH HQULTXHFHU LOLFLWDPHQWH HVWi PDQGDWRFDUJRHPSUHJRRXIXQomRS~EOLFDEHQV
VXMHLWR jV FRPLQDo}HV GHVWD /HL DWp R OLPLWH GR GHTXDOTXHUQDWXUH]DFXMRYDORUVHMDGHVSURSRUFL
YDORUGDKHUDQoD RQDO j HYROXomR GR SDWULP{QLR RX j UHQGD GR
DJHQWHS~EOLFR
&DStWXOR,,
'RV$WRVGH,PSURELGDGH$GPLQLVWUDWLYD 9,,, DFHLWDUHPSUHJRFRPLVVmRRXH[HUFHUDWLYLGDGH
GH FRQVXOWRULD RX DVVHVVRUDPHQWR SDUD SHVVRD
6HomR, ItVLFDRXMXUtGLFDTXHWHQKDLQWHUHVVHVXVFHWtYHOGH
'RV$WRVGH,PSURELGDGH$GPLQLVWUDWLYD VHU DWLQJLGR RX DPSDUDGR SRU DomR RX RPLVVmR
TXHLPSRUWDP(QULTXHFLPHQWR,OtFLWR GHFRUUHQWH GDV DWULEXLo}HV GR DJHQWH S~EOLFR
GXUDQWHDDWLYLGDGH
$UWž &RQVWLWXL DWR GH LPSURELGDGHDGPLQLVWUDWLYD
LPSRUWDQGR HQULTXHFLPHQWR LOtFLWR DXIHULU TXDO ,; SHUFHEHUYDQWDJHPHFRQ{PLFDSDUDLQWHUPHGLDUD
TXHUWLSRGHYDQWDJHPSDWULPRQLDOLQGHYLGDHP OLEHUDomRRXDSOLFDomRGHYHUEDS~EOLFDGHTXDO
UD]mR GR H[HUFtFLR GH FDUJR PDQGDWR IXQomR TXHUQDWXUH]D
HPSUHJRRXDWLYLGDGHQDVHQWLGDGHVPHQFLRQDGDV
QRDUWžGHVWD/HLHQRWDGDPHQWH ; UHFHEHUYDQWDJHPHFRQ{PLFDGHTXDOTXHUQDWXUH
]DGLUHWDRXLQGLUHWDSDUDRPLWLUDWRGHRItFLR
SURYLGrQFLDRXGHFODUDomRDTXHHVWHMDREULJDGR
, UHFHEHU SDUD VL RX SDUDRXWUHPGLQKHLUREHP
PyYHO RX LPyYHO RX TXDOTXHU RXWUD YDQWDJHP
;, LQFRUSRUDUSRUTXDOTXHUIRUPDDRVHXSDWULP{
HFRQ{PLFDGLUHWDRXLQGLUHWDDWtWXORGHFRPLV
QLREHQVUHQGDVYHUEDVRXYDORUHVLQWHJUDQWHVGR
VmR SHUFHQWDJHP JUDWLILFDomR RX SUHVHQWH GH DFHUYRSDWULPRQLDOGDVHQWLGDGHVPHQFLRQDGDVQR
TXHP WHQKD LQWHUHVVH GLUHWR RX LQGLUHWR TXH DUWžGHVWD/HL
SRVVD VHU DWLQJLGR RX DPSDUDGR SRU DomR RX
RPLVVmR GHFRUUHQWH GDV DWULEXLo}HV GR DJHQWH ;,, XVDUHPSURYHLWRSUySULREHQVUHQGDVYHUEDVRX
S~EOLFR YDORUHV LQWHJUDQWHV GR DFHUYR SDWULPRQLDO GDV
HQWLGDGHVPHQFLRQDGDVQRDUWžGHVWD/HL
,, SHUFHEHUYDQWDJHPHFRQ{PLFDGLUHWDRXLQGLUHWD
SDUD IDFLOLWDU DTXLVLomR SHUPXWD RX ORFDomR GH 6HomR,,
EHPPyYHORXLPyYHORXDFRQWUDWDomRGHVHUYL  'RV$WRVGH,PSURELGDGH$GPLQLVWUDWLYD
oRVSHODVHQWLGDGHVUHIHULGDVQRDUWžSRUSUHoR TXHFDXVDP3UHMXt]RDR(UiULR
VXSHULRUDRYDORUGHPHUFDGR
$UW &RQVWLWXL
&RQVWLWXLDWR
DWRGH
GHLPSURELGDGH
LPSURELGDGHDGPLQLVWUDWLYDTXH
,,, SHUFHEHUYDQWDJHPHFRQ{PLFDGLUHWDRXLQGLUHWD FDXVD
FDXVDOHVmR
OHVmRDR
DRHUiULR
HUiULRTXDOTXHU
TXDOTXHUDomR
DomRRX
RXRPLVVmR
SDUDIDFLOLWDUDDOLHQDomRSHUPXWDRXORFDomRGH GRORVD
GRORVDRX
RXFXOSRVD
FXOSRVDTXH
TXHHQVHMH
HQVHMHSHUGD
SHUGDSDWULPRQLDO
EHP S~EOLFR RX R IRUQHFLPHQWR GH VHUYLoR SRU GHVYLR
GHVYLRDSURSULDomR
DSURSULDomRPDOEDUDWDPHQWR
PDOEDUDWDPHQWRRXRXGLODSL
HQWHHVWDWDOSRUSUHoRLQIHULRUDRYDORUGHPHUFD GDomR
GDomRGRV
GRVEHQV
EHQVRX
RXKDYHUHV
KDYHUHVGDV
GDVHQWLGDGHV
HQWLGDGHVUHIHUL
GR GDVQRDUWžGHVWD/HLHQRWDGDPHQWH
GDVQRDUWžGHVWD/HLHQRWDGDPHQWH
 'LU$GPLQLVWUDWLYR &kPDUD/HJLVODWLYDGR'LVWULWR)HGHUDO


, IDFLOLWDURXFRQFRUUHUSRUTXDOTXHUIRUPDSDUDD ;,9 FHOHEUDU FRQWUDWR RX RXWUR LQVWUXPHQWRTXH


LQFRUSRUDomRDRSDWULP{QLRSDUWLFXODUGHSHVVRD WHQKDSRUREMHWRDSUHVWDomRGHVHUYLoRVS~EOLFRV
ItVLFD RX MXUtGLFD GH EHQV UHQGDV YHUEDV RX SRU PHLR GD JHVWmR DVVRFLDGD VHP REVHUYDU DV
YDORUHV LQWHJUDQWHV GR DFHUYR SDWULPRQLDO GDV IRUPDOLGDGHVSUHYLVWDVQDOHL ,QFOXtGRSHOD/HLQž
HQWLGDGHVPHQFLRQDGDVQRDUWžGHVWD/HL GH

;9 FHOHEUDUFRQWUDWRGHUDWHLRGHFRQVyUFLRS~EOLFR
,, SHUPLWLU RX FRQFRUUHU SDUD TXHSHVVRDItVLFDRX VHPVXILFLHQWHHSUpYLDGRWDomRRUoDPHQWiULDRX
MXUtGLFD SULYDGD XWLOL]H EHQV UHQGDV YHUEDV RX VHP REVHUYDU DV IRUPDOLGDGHV SUHYLVWDV QD OHL
YDORUHV LQWHJUDQWHV GR DFHUYR SDWULPRQLDO GDV ,QFOXtGRSHOD/HLQžGH
HQWLGDGHVPHQFLRQDGDVQRDUWžGHVWD/HLVHPD
REVHUYkQFLDGDVIRUPDOLGDGHVOHJDLVRXUHJXODPHQ ;9, IDFLOLWDURXFRQFRUUHUSRUTXDOTXHUIRUPDSDUD
WDUHVDSOLFiYHLVjHVSpFLH D LQFRUSRUDomR DR SDWULP{QLR SDUWLFXODU GH
SHVVRDItVLFDRXMXUtGLFDGHEHQVUHQGDVYHUEDV
,,, GRDUjSHVVRDItVLFDRXMXUtGLFDEHPFRPRDRHQWH RXYDORUHVS~EOLFRVWUDQVIHULGRVSHODDGPLQLVWUD
GHVSHUVRQDOL]DGRDLQGDTXHGHILQVHGXFDWLYRVRX omR S~EOLFD D HQWLGDGHV SULYDGDV PHGLDQWH
DVVLVWHQFLDLV EHQV UHQGDV YHUEDV RX YDORUHV GR FHOHEUDomRGHSDUFHULDVVHPDREVHUYkQFLDGDV
SDWULP{QLR GH TXDOTXHU GDV HQWLGDGHV PHQFLR IRUPDOLGDGHVOHJDLVRXUHJXODPHQWDUHVDSOLFiYHLV
QDGDV QR DUWž GHVWD /HL VHP REVHUYkQFLD GDV jHVSpFLH LQFOXtGRSHOD/HLQž
IRUPDOLGDGHVOHJDLVHUHJXODPHQWDUHVDSOLFiYHLVj
HVSpFLH ;9,, SHUPLWLURXFRQFRUUHUSDUDTXHSHVVRDItVLFDRX
MXUtGLFDSULYDGDXWLOL]HEHQVUHQGDVYHUEDVRX
,9 SHUPLWLU RX IDFLOLWDU D DOLHQDomR SHUPXWDRX YDORUHVS~EOLFRVWUDQVIHULGRVSHODDGPLQLVWUDomR
ORFDomR GH EHP LQWHJUDQWH GR SDWULP{QLR GH S~EOLFDDHQWLGDGHSULYDGDPHGLDQWHFHOHEUDomR
TXDOTXHUGDVHQWLGDGHVUHIHULGDVQRDUWžGHVWD GHSDUFHULDVVHPDREVHUYkQFLDGDVIRUPDOLGDGHV
/HLRXDLQGDDSUHVWDomRGHVHUYLoRSRUSDUWHGH OHJDLV RX UHJXODPHQWDUHV DSOLFiYHLV j HVSpFLH
ODVSRUSUHoRLQIHULRUDRGHPHUFDGR LQFOXtGRSHOD/HLQž

9 SHUPLWLU RX IDFLOLWDU D DTXLVLomRSHUPXWDRX ;9,,, FHOHEUDUSDUFHULDVGDDGPLQLVWUDomRS~EOLFDFRP


ORFDomRGHEHPRXVHUYLoRSRUSUHoRVXSHULRUDR HQWLGDGHVSULYDGDVVHPDREVHUYkQFLDGDVIRUPD
GHPHUFDGR OLGDGHV OHJDLV RX UHJXODPHQWDUHV DSOLFiYHLV j
HVSpFLH LQFOXtGRSHOD/HLQž
9, UHDOL]DURSHUDomRILQDQFHLUDVHPREVHUYkQFLDGDV
QRUPDVOHJDLVHUHJXODPHQWDUHVRXDFHLWDUJDUDQWLD ;,; DJLUQHJOLJHQWHPHQWHQDFHOHEUDomRILVFDOL]DomR
LQVXILFLHQWHRXLQLG{QHD HDQiOLVHGDVSUHVWDo}HVGHFRQWDVGHSDUFHULDV
ILUPDGDVSHODDGPLQLVWUDomRS~EOLFDFRPHQWLGD
9,, FRQFHGHUEHQHItFLRDGPLQLVWUDWLYRRXILVFDOVHPD GHVSULYDGDV LQFOXtGRSHOD/HLQžHPRGLILFDGRSHOD/HL
REVHUYkQFLDGDVIRUPDOLGDGHVOHJDLVRXUHJXODPHQ QžGH

WDUHVDSOLFiYHLVjHVSpFLH
;; OLEHUDU UHFXUVRV GH SDUFHULDV ILUPDGDVSHOD
9,,, IUXVWUDU D OLFLWXGH GH SURFHVVR OLFLWDWyULR RXGH DGPLQLVWUDomR S~EOLFD FRP HQWLGDGHV SULYDGDV
SURFHVVRVHOHWLYRSDUDFHOHEUDomRGHSDUFHULDVFRP VHPDHVWULWDREVHUYkQFLDGDVQRUPDVSHUWLQHQWHV
HQWLGDGHV VHP ILQV OXFUDWLYRV RX GLVSHQViORV RXLQIOXLUGHTXDOTXHUIRUPDSDUDDVXDDSOLFDomR
LQGHYLGDPHQWH UHGDomRGDGDSHOD/HLQž LUUHJXODU LQFOXtGR SHOD /HL Qž  H PRGLILFDGR SHOD /HL Qž
GH

,; RUGHQDURXSHUPLWLUDUHDOL]DomRGHGHVSHVDVQmR
;;, OLEHUDU UHFXUVRV GH SDUFHULDV ILUPDGDVSHOD
DXWRUL]DGDVHPOHLRXUHJXODPHQWR
DGPLQLVWUDomR S~EOLFD FRP HQWLGDGHV SULYDGDV
; DJLUQHJOLJHQWHPHQWHQDDUUHFDGDomRGHWULEXWRRX VHPDHVWULWDREVHUYkQFLDGDVQRUPDVSHUWLQHQWHV
UHQGDEHPFRPRQRTXHGL]UHVSHLWRjFRQVHUYD RXLQIOXLUGHTXDOTXHUIRUPDSDUDDVXDDSOLFDomR
omRGRSDWULP{QLRS~EOLFR LUUHJXODU LQFOXtGRSHOD/HLQž

;, OLEHUDUYHUEDS~EOLFDVHPDHVWULWDREVHUYkQFLDGDV 6HomR,,$


,QFOXtGRSHOD/HL&RPSOHPHQWDUQžGH
QRUPDVSHUWLQHQWHVRXLQIOXLUGHTXDOTXHUIRUPD 'RV$WRVGH,PSURELGDGH$GPLQLVWUDWLYD
SDUDDVXDDSOLFDomRLUUHJXODU 'HFRUUHQWHVGH&RQFHVVmRRX$SOLFDomR,QGHYLGD
GH%HQHItFLR)LQDQFHLURRX7ULEXWiULR
;,, SHUPLWLUIDFLOLWDURXFRQFRUUHUSDUDTXHWHUFHLURVH
HQULTXHoDLOLFLWDPHQWH $UW$ &RQVWLWXL DWR GH LPSURELGDGH DGPLQLVWUDWLYD
TXDOTXHUDomRRXRPLVVmRSDUDFRQFHGHUDSOLFDU
;,,, SHUPLWLUTXHVHXWLOL]HHPREUDRXVHUYLoRSDUWLFX RX PDQWHU EHQHItFLR ILQDQFHLUR RX WULEXWiULR
ODUYHtFXORVPiTXLQDVHTXLSDPHQWRVRXPDWHULDO
FRQWUiULRDRTXHGLVS}HPRFDSXWHR†žGRDUW
GHTXDOTXHUQDWXUH]DGHSURSULHGDGHRXjGLVSRVL
ž$GD/HL&RPSOHPHQWDUQžGHGHMXOKR
omR GH TXDOTXHU GDV HQWLGDGHV PHQFLRQDGDV QR
GH ,QFOXtGRSHOD/HL&RPSOHPHQWDUQžGH 
DUWžGHVWD/HLEHPFRPRRWUDEDOKRGHVHUYLGRU
S~EOLFRHPSUHJDGRVRXWHUFHLURVFRQWUDWDGRVSRU 2EV $UWž (VWD/HL&RPSOHPHQWDUHQWUDHPYLJRUQDGDWDGHVXDSXEOLFDomR
HVVDVHQWLGDGHV
 &kPDUD/HJLVODWLYDGR
 'LVWULWR)HGHUDO 'LU$GPLQLVWUDWLYR 

†ž2GLVSRVWRQRFDSXWHQRV††žHžGRDUWR$GD/HL&RPSOH , QDKLSyWHVHGRDUWžSHUGDGRVEHQVRXYDORUHV


PHQWDUQRGHGHMXOKRGHHQRDUW$QRLQFLVR
,9GRDUWHQR†GRDUWWRGRVGD/HLQRGHGH DFUHVFLGRVLOLFLWDPHQWHDRSDWULP{QLRUHVVDUFL
MXQKRGHVRPHQWHSURGX]LUmRHIHLWRVDSyVRGHFXUVRGR PHQWRLQWHJUDOGRGDQRTXDQGRKRXYHUSHUGD
SUD]RUHIHULGRQRDUWžGHVWD/HL&RPSOHPHQWDU GDIXQomRS~EOLFDVXVSHQVmRGRVGLUHLWRVSROtWL

FRVGHRLWRDGH]DQRVSDJDPHQWRGHPXOWDFLYLO
$UWž 2VHQWHVIHGHUDGRVGHYHUmRQRSUD]RGH XP DQRFRQWDGRGD GHDWpWUrVYH]HVRYDORUGRDFUpVFLPRSDWULPRQL
SXEOLFDomRGHVWD/HL&RPSOHPHQWDUUHYRJDURVGLVSRVLWLYRVTXH
FRQWUDULHP R GLVSRVWR QR FDSXW H QR † ž GR DUW ž$ GD /HL DO H SURLELomR GH FRQWUDWDU FRP R 3RGHU RX
&RPSOHPHQWDUQžGHGHMXOKRGH UHFHEHUEHQHItFLRVRXLQFHQWLYRVILVFDLVRXFUHGL
WtFLRV GLUHWD RX LQGLUHWDPHQWH DLQGD TXH SRU
6HomR,,, LQWHUPpGLRGHSHVVRDMXUtGLFDGDTXDOVHMDVyFLR
'RVDWRVGH,PSURELGDGH$GPLQLVWUDWLYD PDMRULWiULRSHORSUD]RGHGH]DQRV
TXHDWHQWDPFRQWUDRV
3ULQFtSLRVGD$GPLQLVWUDomR3~EOLFD ,, QDKLSyWHVHGRDUWUHVVDUFLPHQWRLQWHJUDOGR
GDQR SHUGD GRV EHQV RX YDORUHV DFUHVFLGRV
$UW &RQVWLWXL DWR GH LPSURELGDGHDGPLQLVWUDWLYD
LOLFLWDPHQWH DR SDWULP{QLR VH FRQFRUUHU HVWD
TXH
TXH DWHQWD
DWHQWD FRQWUD
FRQWUD RV
RV SULQFtSLRV
SULQFtSLRV GD DGPLQLV
WUDomR
WUDomR S~EOLFD
S~EOLFD TXDOTXHU
TXDOTXHU DomR
DomR RX
RX RPLVVmR
RPLVVmR TXH FLUFXQVWkQFLDSHUGDGDIXQomRS~EOLFDVXVSHQ
YLROH
YLROHRV
RVGHYHUHV
GHYHUHVGH
GHKRQH
KRQHVWLGDGH
VWLGDGHLPSDUFLDOLGD VmRGRV GLUHLWRVSROtWLFRVGHFLQFRDRLWRDQRV
GH
GH OHJDOLGDGH H OHDOGDGH jV LQVWLWXLo}HV H SDJDPHQWR GH PXOWD FLYLO GH DWp GXDV YH]HV R
QRWDGDPHQWH
QRWDGDPHQWH YDORU GR GDQR H SURLELomR GH FRQWUDWDU FRP
3RGHU3~EOLFRRXUHFHEHUEHQHItFLRVRXLQFHQWL
, SUDWLFDU DWR YLVDQGR ILPSURLELGRHPOHLRX YRVILVFDLVRXFUHGLWtFLRVGLUHWDRXLQGLUHWDPHQ
UHJXODPHQWR RX GLYHUVR GDTXHOH SUHYLVWR QD WHDLQGDTXHSRULQWHUPpGLRGHSHVVRDMXUtGLFD
UHJUDGHFRPSHWrQFLD GD TXDO VHMD VyFLR PDMRULWiULR SHOR SUD]R GH
FLQFRDQRV
,, UHWDUGDURXGHL[DUGHSUDWLFDULQGHYLGDPHQWH
DWRGHRItFLR ,,, QDKLSyWHVHGRDUWUHVVDUFLPHQWRLQWHJUDOGR
GDQRVHKRXYHUSHUGDGDIXQomRS~EOLFDVXV
,,, UHYHODUIDWRRXFLUFXQVWkQFLDGHTXHWHPFLrQFLD
SHQVmRGRVGLUHLWRVSROtWLFRVGHWUrVDFLQFRDQRV
HPUD]mRGDVDWULEXLo}HVHTXHGHYDSHUPDQHFHU
HPVHJUHGR SDJDPHQWR GH PXOWD FLYLO GH DWp FHP YH]HV R
YDORUGDUHPXQHUDomRSHUFHELGDSHORDJHQWHH
,9 QHJDUSXEOLFLGDGHDRVDWRVRILFLDLV SURLELomRGHFRQWUDWDUFRPR3RGHU3~EOLFRRX
UHFHEHU EHQHItFLRV RX LQFHQWLYRV ILVFDLV RX
9 IUXVWUDUDOLFLWXGHGHFRQFXUVRS~EOLFR FUHGLWtFLRV GLUHWD RX LQGLUHWDPHQWH DLQGD TXH
SRULQWHUPpGLRGHSHVVRDMXUtGLFDGDTXDOVHMD
9, GHL[DUGHSUHVWDUFRQWDVTXDQGRHVWHMDREULJDGR VyFLRPDMRULWiULRSHORSUD]RGHWUrVDQRV
DID]rOR
,9 QD KLSyWHVH SUHYLVWD QR DUW $ SHUGDGD
9,, UHYHODURXSHUPLWLUTXHFKHJXHDRFRQKHFLPHQWR IXQomRS~EOLFDVXVSHQVmRGRVGLUHLWRVSROtWLFRV
GHWHUFHLURDQWHVGDUHVSHFWLYDGLYXOJDomRRIL GH FLQFR D RLWR DQRVHPXOWDFLYLOGHDWp
FLDOWHRUGHPHGLGDSROtWLFDRXHFRQ{PLFDFDSD]  WUrV YH]HVRYDORUGREHQHItFLRILQDQFHLURRX
GHDIHWDURSUHoRGHPHUFDGRULDEHPRXVHUYLoR
WULEXWiULRFRQFHGLGR ,QFOXtGRSHOD/HL&RPSOHPHQWDUQžGH

9,,, GHVFXPSULU DV QRUPDV UHODWLYDV jFHOHEUDomR
ILVFDOL]DomRHDSURYDomRGHFRQWDVGHSDUFHULDV 2EV $UWž (VWD/HL&RPSOHPHQWDUHQWUDHPYLJRUQDGDWDGHVXDSXEOLFD
ILUPDGDVSHODDGPLQLVWUDomRS~EOLFDFRPHQWLGD omR
GHVSULYDGDV LQFOXtGRSHOD/HLQžHPYLJRUFRQWDGRV
GLDVDSyVDGDWDGHVXDSXEOLFDomR  †ž 2GLVSRVWRQRFDSXWHQRV††žHžGRDUWR$GD/HL&RPSOH
PHQWDUQRGHGHMXOKRGHHQRDUW$QRLQFLVR
,9GRDUWHQR†GRDUWWRGRVGD/HLQRGHGH
,; GHL[DUGHFXPSULUDH[LJrQFLDGHUHTXLVLWRVGH MXQKRGHVRPHQWHSURGX]LUmRHIHLWRVDSyVRGHFXUVRGR
DFHVVLELOLGDGHSUHYLVWRVQDOHJLVODomR UHGDomR GDGD SUD]RUHIHULGRQRDUWžGHVWD/HL&RPSOHPHQWDU
SHOD/HLQž


&DStWXOR,,, $UWž 2VHQWHVIHGHUDGRVGHYHUmRQRSUD]RGH XP DQRFRQWDGRGD


'DV3HQDV SXEOLFDomRGHVWD/HL&RPSOHPHQWDUUHYRJDURVGLVSRVLWLYRVTXH
FRQWUDULHP R GLVSRVWR QR FDSXW H QR † ž GR DUW ž$ GD /HL
&RPSOHPHQWDUQžGHGHMXOKRGH
$UW ,QGHSHQGHQWHPHQWH
,QGHSHQGHQWHPHQWHGDV GDVVDQo}HV
VDQo}HVSHQDLV
SHQDLVFLYLV
FLYLVH
DGPLQLVWUDWLYDV
DGPLQLVWUDWLYDVSUHYLVWDV
SUHYLVWDVQD
QDOHJLVODomR
OHJLVODomRHVSHFtIL
FD
FDHVWi
HVWiRUHVSRQViYHO
UHVSRQViYHOSHOR
SHORDWR
DWRGH
GHLPSURELGDGH †~QLFR 1DIL[DomRGDVSHQDVSUHYLVWDVQHVWD/HLRMXL]
VXMHLWR
VXMHLWRjV
jVVHJXLQWHV
VHJXLQWHVFRPLQDo}HV
FRPLQDo}HVTXH
TXHSRGHP
SRGHPVHU OHYDUi HP FRQWD D H[WHQVmR GR GDQR FDXVDGR
DSOLFDGDV
DSOLFDGDV LVRODGD
LVRODGD RX
RX FXPXODWLYDPHQWH
FXPXODWLYDPHQWH GH DVVLPFRPRRSURYHLWRSDWULPRQLDOREWLGRSHOR
DFRUGR
DFRUGRFRP JUDYLGDGHGR
FRPDJUDYLGDGH GRIDWR
IDWR 5HGDomRGDGDSHOD/HL DJHQWH
QžGH 
 'LU$GPLQLVWUDWLYR &kPDUD/HJLVODWLYDGR'LVWULWR)HGHUDO


&DStWXOR,9 †ž $WHQGLGRV RV UHTXLVLWRV GDUHSUHVHQWDomRD


'D'HFODUDomRGH%HQV DXWRULGDGHGHWHUPLQDUiDLPHGLDWDDSXUDomRGRV
IDWRVTXHHPVHWUDWDQGRGHVHUYLGRUHVIHGHUDLV
$UW $ SRVVH
SRVVH H R H[HUFtFLR
H[HUFtFLR GH
GH DJHQWHS~EOLFRILFDP
DJHQWHS~EOLFRILFDP VHUiSURFHVVDGDQDIRUPDSUHYLVWDQRVDUWVD
FRQGLFLRQDGRV
FRQGLFLRQDGRV j DSUHVHQWDomR
DSUHVHQWDomR GH
GH GHFODUDomR GD/HLQžGHGHGH]HPEURGH
GRV
GRVEHQV
EHQVHYDORUHV
YDORUHVTXH
TXHFRPS}HP
FRPS}HPRVHX
VHXSDWULP{ HHPVHWUDWDQGRGHVHUYLGRUPLOLWDUGHDFRUGR
QLR
QLRSULYDGR
SULYDGRDILP
ILPGH
GHVHUDUTXLYDGDQR6HUYLoR FRPRUHVSHFWLYRVUHJXODPHQWRVGLVFLSOLQDUHV
GH3HVVRDOFRPSHWHQWH
GH3HVVRDOFRPSHWHQWH
$UW $FRPLVVmR SURFHVVDQWHGDUiFRQKHFLPHQWRDR
0LQLVWpULR3~EOLFRHDR7ULEXQDORX&RQVHOKRGH
†ž $ GHFODUDomRFRPSUHHQGHUiLPyYHLVPyYHLV
&RQWDVGDH[LVWrQFLDGHSURFHGLPHQWRDGPLQLV
VHPRYHQWHVGLQKHLURWtWXORVDo}HVHTXDOTXHU WUDWLYRSDUDDSXUDUDSUiWLFDGHDWRGHLPSUREL
RXWUD HVSpFLH GH EHQV H YDORUHV SDWULPRQLDLV GDGH
ORFDOL]DGRVQR3DtVRXQRH[WHULRUHTXDQGRIRU
RFDVRDEUDQJHUiRVEHQVHYDORUHVSDWULPRQLDLV †~QLFR 20LQLVWpULR3~EOLFRRX7ULEXQDORX&RQVHOKRGH
GRF{QMXJHRXFRPSDQKHLURGRVILOKRVHGHRX &RQWDVSRGHUiDUHTXHULPHQWRGHVLJQDUUHSUH
WUDVSHVVRDVTXHYLYDPVREDGHSHQGrQFLDHFRQ{ VHQWDQWH SDUD DFRPSDQKDU R SURFHGLPHQWR
PLFDGRGHFODUDQWHH[FOXtGRVDSHQDVRVREMHWRV DGPLQLVWUDWLYR
HXWHQVtOLRVGHXVRGRPpVWLFR
$UW +DYHQGRIXQGDGRVLQGtFLRVGHUHVSRQVDELOLGDGH
†ž $GHFODUDomRGHEHQVVHUiDQXDOPHQWHDWXDOL]D
DQXDOPHQWHDWXDOL]D DFRPLVVmRUHSUHVHQWDUiDR0LQLVWpULR3~EOLFRRX
GDHQDGDWDHPTXHRDJHQWHS~EOLFRGHL[DUR
GD j SURFXUDGRULD GR yUJmR SDUD TXH UHTXHLUD DR
H[HUFtFLRGRPDQGDWRFDUJRHPSUHJRRXIXQ MXt]RFRPSHWHQWHDDGHFUHWDomR
GHFUHWDomRGR
GRVHTHVWUR
VHTHVWURGRV
omR EHQV
EHQVGR
GRDJHQWH
DJHQWHRX
RXWHUFHLURTXHWHQKDHQULTXHFL
WHUFHLUR
GRLOLFLWDPHQWHRXFDXVDGRGDQRDRSDWULP{QLR
S~EOLFR
†ž 6HUi
6HUiSXQLGR FRPDSHQD
SXQLGRFRP SHQDGH
GHGHPLVVmR
GHPLVVmRDEHP
EHPGR
VHUYLoRS~EOLFR
VHUYLoR S~EOLFRVHP
VHPSUHMXt]R
SUHMXt]RGH
GHRXWUDV
RXWUDVVDQo}HV
†ž 2SHGLGRGHVHTHVWURVHUiSURFHVVDGRGHDFRUGR
FDEtYHLV
FDEtYHLVRDJHQWH S~EOLFRTXH
DJHQWHS~EOLFR TXHVH
VHUHFXVDU
UHFXVDUDSUHV FRPRGLVSRVWRQRVDUWVHGR&yGLJRGH
WDU
WDUGHFODUDomR
GHFODUDomRGRV
GRVEHQV
EHQVGHQWUR
GHQWURGR
GRSUD]R
SUD]RGHWHU 3URFHVVR&LYLO
PLQDGRRXTXHDSUHVWDUIDOVD
PLQDGRRXTXHDSUHVWDUIDOVD
†ž 4XDQGRIRURFDVRRSHGLGRLQFOXLUiDLQYHVWLJD
†ž 2 GHFODUDQWH D VHXFULWpULRSRGHUiHQWUHJDU omRRH[DPHHREORTXHLRGHEHQVFRQWDVEDQFi
FySLDGDGHFODUDomRDQXDOGHEHQVDSUHVHQWDGD ULDV H DSOLFDo}HV ILQDQFHLUDV PDQWLGDV SHOR
j'HOHJDFLDGD5HFHLWD)HGHUDOQDFRQIRUPLGDGH LQGLFLDGR QR H[WHULRU QRV WHUPRV GD OHL H GRV
GDOHJLVODomRGR,PSRVWRVREUHD5HQGDHSUR WUDWDGRVLQWHUQDFLRQDLV
YHQWRVGHTXDOTXHUQDWXUH]DFRPDVQHFHVViULDV
DWXDOL]Do}HVSDUDVXSULUDH[LJrQFLDFRQWLGDQR $UW $DomRSULQFLSDOTXHWHUiRULWRRUGLQiULRVHUi
FDSXWHQR†žGHVWHDUWLJR SURSRVWDSHOR0LQLVWpULR3~EOLFRRXSHODSHVVRD
MXUtGLFD LQWHUHVVDGD GHQWUR GH WULQWD GLDV GD
HIHWLYDomRGDPHGLGDFDXWHODU
&DStWXOR9
†ž eYHGDGDDWUDQVDomRDFRUGRRXFRQFLOLDomRQDV
3URFHGLPHQWR$GPLQLVWUDWLYRHGR3URFHVVR-XGLFLDO
Do}HVGHTXHWUDWDRFDSXW
4XDOTXHUSHVVRD
$UW 4XDOTXHU SHVVRDSRGHU
SRGHUiUHSUHVHQWDU
iUHSUHVHQWDUjDXWRULGDGH
iUHSUHVHQWDU †ž $)D]HQGD3~EOLFDTXDQGRIRURFDVRSURPRYH
DGPLQLVWUDWLYDFRPSHWHQWHSDUDTXHVHMDLQVWDX Ui DV Do}HV QHFHVViULDV j FRPSOHPHQWDomR GR
UDGDLQYHVWLJDomRGHVWLQDGDDDSXUDUDSUiWLFDGH UHVVDUFLPHQWRGRSDWULP{QLRS~EOLFR
DWRGHLPSURELGDGH
†ž 1R FDVR GH D DomR SULQFLSDO WHUVLGRSURSRVWD
†ž $UHSUHVHQWDomRTXHVHUiHVFULWDRXUHGX]LGDD SHOR0LQLVWpULR3~EOLFRDSOLFDVHQRTXHFRXEHU
WHUPR H DVVLQDGD FRQWHUi D TXDOLILFDomR GR RGLVSRVWRQR†žGR$UWLJRžGD/HLQž
UHSUHVHQWDQWHDVLQIRUPDo}HVVREUHRIDWRHVXD GHGHMXQKRGH2EV†žFRPUHGDomRGDGDSHOD/HL
Qž
DXWRULDHDLQGLFDomRGDVSURYDVGHTXHWHQKD
FRQKHFLPHQWR †ž 20LQLVWpULR3~EOLFRVHQmRLQWHUYLHUQRSURFHV
VRFRPRSDUWHDWXDUiREULJDWRULDPHQWHFRPR
†ž $DXWRULGDGHDGPLQLVWUDWLYDUHMHLWDUiDUHSUHVHQ ILVFDOGDOHLVRESHQDGHQXOLGDGH
WDomRHPGHVSDFKRIXQGDPHQWDGRVHHVWDQmR
FRQWLYHUDVIRUPDOLGDGHVHVWDEHOHFLGDVQR † ž †ž $SURSRVLWXUDGDDomRSUHYHQLUiDMXULVGLomRGR
GHVWHDUWLJR$UHMHLomRQmRLPSHGHDUHSUHVHQWD MXt]RSDUDWRGDVDVDo}HVSRVWHULRUPHQWHLQWHQWD
omRDR0LQLVWpULR3~EOLFRQRVWHUPRVGRDUW GDVTXHSRVVXDPDPHVPDFDXVDGHSHGLURXR
GHVWD/HL PHVPRREMHWR2EV†žDFUHVFLGRSHOD0HGLGD3URYLVyULDQž

&kPDUD/HJLVODWLYDGR
 'LVWULWR)HGHUDO 'LU$GPLQLVWUDWLYR 

†ž $DomRVHUiLQVWUXtGDFRPGRFXPHQWRVRXMXVWLIL &DStWXOR9,


FDomR TXH FRQWHQKDP LQGtFLRV VXILFLHQWHV GD 'DV'LVSRVLo}HV3HQDLV
H[LVWrQFLDGRDWRGHLPSURELGDGHRXFRPUD]}HV
IXQGDPHQWDGDV GD LPSRVVLELOLGDGH GH $UW &RQVWLWXL FULPH
FULPH D UHSUHVHQWDomR SRU DWRGH
DSUHVHQWDomRGHTXDOTXHUGHVVDVSURYDVREVHU LPSURELGDGHFRQWUDDJHQWHS~EOLFRRXWHUFHLUR
YDGDDOHJLVODomRYLJHQWHLQFOXVLYHDVGLVSRVLo}HV EHQHILFLiULRTXDQGRRDXWRUGDGHQ~QFLDRVDEH
LQVFULWDVQRVDUWVDGR&yGLJRGH3URFHVVR LQRFHQWH
&LYLO 3HQDGHWHQomRGHVHLVDGH]PHVHVHPXOWD
3HQD
2EV (VWH † ž IRL DFUHVFHQWDGR SHOD 0HGLGD 3URYLVyULD Qž  GH †~QLFR $OpPGDVDQomRSHQDORGHQXQFLDQWHHVWiVXMHLWR
TXHWDPEpPDFUHVFHQWRXRV††žžžH
DLQGHQL]DURGHQXQFLDGRSHORVGDQRVPDWHULDLV
†ž (VWDQGRDLQLFLDOHPGHYLGDIRUPDRMXL]PDQGD PRUDLVRXjLPDJHPTXHKRXYHUSURYRFDGR
UiDXWXiODHRUGHQDUiDQRWLILFDomRGRUHTXHUL
GRSDUDRIHUHFHUPDQLIHVWDomRSRUHVFULWRTXH $UW $ SHUGD
SHUGD GD
GD IXQomR
IXQomR S~EOLFD H D VXVSHQVmRGRV
SRGHUiVHULQVWUXtGDFRPGRFXPHQWRVHMXVWLILFD GLUHLWRVSROtWLFRVVyVH
GLUHLWRVSROtWLFRVVyVHHIHWLYDPFRPRWUkQVLWR
o}HVGHQWURGRSUD]RGHTXLQ]HGLDV HPMXOJDGRGDVHQWHQoDFRQGHQDWyULD
HPMXOJDGRGDVHQWHQoDFRQGHQDWyULD

†~QLFR $DXWRULGDGHMXGLFLDORXDGPLQLVWUDWLYDFRPSH
†ž 5HFHELGD D PDQLIHVWDomR R MXL] QRSUD]RGH
WHQWH SRGHUi GHWHUPLQDU R DIDVWDPHQWR GR
WULQWDGLDVHPGHFLVmRIXQGDPHQWDGDUHMHLWDUi DJHQWHS~EOLFRGRH[HUFtFLRGRFDUJRHPSUHJR
DDomRVHFRQYHQFLGRGDLQH[LVWrQFLDGRDWRGH RXIXQomRVHPSUHMXt]RGDUHPXQHUDomRTXDQ
LPSURELGDGHGDLPSURFHGrQFLDGDDomRRXGD GR D PHGLGD VH IL]HU QHFHVViULD j LQVWUXomR
LQDGHTXDomRGDYLDHOHLWD SURFHVVXDO

†ž 5HFHELGDDSHWLomRLQLFLDOVHUiRUpXFLWDGRSDUD $UW $ DSOLFDomR GDV VDQo}HV SUHYLVWDV QHVWD/HL


DSUHVHQWDUFRQWHVWDomR LQGHSHQGH
LQGHSHQGH

† 'DGHFLVmRTXHUHFHEHUDSHWLomRLQLFLDOFDEHUi , GD HIHWLYD RFRUUrQFLD GHGDQRDRSDWULP{QLR


DJUDYRGHLQVWUXPHQWR S~EOLFRVDOYRTXDQWRjSHQDGHUHVVDUFLPHQWR
2EV5HGDomRGDGDSHOD/HLQžGH

† (P TXDOTXHU IDVH GR SURFHVVRUHFRQKHFLGDD ,, GDDSURYDomRRXUHMHLomRGDVFRQWDVSHORyUJmR


LQDGHTXDomR GD DomR GH LPSURELGDGH R MXL] GHFRQWUROHLQWHUQRRXSHOR7ULEXQDORX&RQVH
H[WLQJXLUiRSURFHVVRVHPMXOJDPHQWRGRPpULWR OKRGH&RQWDV

† $SOLFDVHDRVGHSRLPHQWRVRXLQTXLULo}HVUHDOL]D $UW 3DUDDSXUDUTXDOTXHULOtFLWRSUHYLVWRQHVWD/HLR


GDVQRVSURFHVVRVUHJLGRVSRUHVWD/HLRGLVSRVWR 0LQLVWpULR3~EOLFRGHRItFLRDUHTXHULPHQWRGH
QRDUWFDSXWH†žGR&yGLJRGH3URFHV DXWRULGDGHDGPLQLVWUDWLYDRXPHGLDQWHUHSUHVHQ
WDomR IRUPXODGD GH DFRUGR FRP R GLVSRVWR QR
VR3HQDO
DUWSRGHUiUHTXLVLWDUDLQVWDXUDomRGHLQTXp
ULWRSROLFLDORXSURFHGLPHQWRDGPLQLVWUDWLYR
† 3DUDRVHIHLWRVGHVWHDUWLJRWDPEpPVHFRQVLGHUD
SHVVRDMXUtGLFDLQWHUHVVDGDRHQWHWULEXWDQWHTXH &DStWXOR9,,
ILJXUDUQRSRORDWLYRGDREULJDomRWULEXWiULDGH  'D3UHVFULomR
TXHWUDWDPR†žGRDUWžHRDUWž$GD/HL
&RPSOHPHQWDUQžGHGHMXOKRGH $UW $VDo}HVGHVWLQDGDVDOHYDUDHIHLWRDVVDQo}HV
,QFOXtGRSHOD/HL&RPSOHPHQWDUQžGH
SUHYLVWDVQHVWD/HLSRGHPVHUSURSRVWDV
2EV $UWž (VWD/HL&RPSOHPHQWDUHQWUDHPYLJRUQDGDWDGHVXDSXEOLFD
omR , DWp FLQFR DQRV DSyV R WpUPLQRGRH[HUFtFLRGH
PDQGDWRGHFDUJRHPFRPLVVmRRXGHIXQomRGH
†ž 2GLVSRVWRQRFDSXWHQRV††žHžGRDUWR$GD/HL&RPSOH FRQILDQoD
PHQWDUQRGHGHMXOKRGHHQRDUW$QRLQFLVR
,9GRDUWHQR†GRDUWWRGRVGD/HLQRGHGH
MXQKRGHVRPHQWHSURGX]LUmRHIHLWRVDSyVRGHFXUVRGR ,, GHQWUR GR SUD]R SUHVFULFLRQDO SUHYLVWRHPOHL
SUD]RUHIHULGRQRDUWžGHVWD/HL&RPSOHPHQWDU HVSHFtILFDSDUDIDOWDVGLVFLSOLQDUHVSXQtYHLVFRP
 GHPLVVmRDEHPGRVHUYLoRS~EOLFRQRVFDVRVGH
H[HUFtFLRGHFDUJRHIHWLYRRXHPSUHJR
$UWž 2VHQWHVIHGHUDGRVGHYHUmRQRSUD]RGH XP DQRFRQWDGRGD
SXEOLFDomRGHVWD/HL&RPSOHPHQWDUUHYRJDURVGLVSRVLWLYRVTXH
FRQWUDULHP R GLVSRVWR QR FDSXW H QR † ž GR DUW ž$ GD /HL ,,, DWpFLQFRDQRVGDGDWDGDDSUHVHQWDomRjDGPL
&RPSOHPHQWDUQžGHGHMXOKRGH QLVWUDomR S~EOLFD GD SUHVWDomR GH FRQWDV ILQDO
SHODVHQWLGDGHVUHIHULGDVQRSDUiJUDIR~QLFRGR
$UW $VHQWHQoDTXHMXOJDUSURFHGHQWHDomRFLYLOGH DUWRGHVWD/HL LQFOXtGRSHOD/HLQžDOWHUDGDSHOD/HL
Qž
UHSDUDomRGRGDQRRXGHFUHWDUDSHUGDGRVEHQV
KDYLGRVLOLFLWDPHQWHGHWHUPLQDUiRSDJDPHQWR &DStWXOR9,,
RX D UHYHUVmR GRV EHQV FRQIRUPH R FDVR HP  'DV'LVSRVLo}HV)LQDLV
IDYRUGDSHVVRDMXUtGLFDSUHMXGLFDGDSHORLOtFLWR
$UW (VWD/HLHQWUDHPYLJRUQDGDWDGHVXDSXEOLFDomR
 'LU$GPLQLVWUDWLYR &kPDUD/HJLVODWLYDGR'LVWULWR)HGHUDO


$UWž $ OLFLWDomR GHVWLQDVH D JDUDQWLU D REVHUYkQFLDGR


SULQFtSLR FRQVWLWXFLRQDO GD LVRQRPLD D VHOHomR GD
/HLQž SURSRVWD PDLV YDQWDMRVD SDUD D DGPLQLVWUDomR H D
SURPRomRGRGHVHQYROYLPHQWRQDFLRQDOVXVWHQWiYHOH
VHUiSURFHVVDGDHMXOJDGDHPHVWULWDFRQIRUPLGDGHFRP
/LFLWDo}HVH&RQWUDWRV$GPLQLVWUDWLYRV RVSULQFtSLRVEiVLFRVGDOHJDOLGDGHGDLPSHVVRDOLGDGH
GD PRUDOLGDGH GD LJXDOGDGH GD SXEOLFLGDGH GD
SURELGDGHDGPLQLVWUDWLYDGDYLQFXODomRDRLQVWUXPHQ
WRFRQYRFDWyULRGRMXOJDPHQWRREMHWLYRHGRVTXHOKHV
$/LFLWDomRpRFRQMXQWRGHSURFHGLPHQWRVDGPLQLVWUD VmRFRUUHODWRV 5HGDomRGDGDSHOD/HLQžGH
WLYRVOHJDOPHQWHHVWDEHOHFLGRVDWUDYpVGRTXDOD$GPL
QLVWUDomR3~EOLFDFULDPHLRVGHYHULILFDUHQWUHLQWHUHVVD †ž eYHGDGRDRVDJHQWHVS~EOLFRV
GRVKDELOLWDGRVTXHPRIHUHFHPHOKRUHVFRQGLo}HVSDUD
DTXLVLomRHDOLHQDomRGHEHQVHVHUYLoRVHUHDOL]DomRGH , DGPLWLUSUHYHULQFOXLURXWROHUDUQRVDWRVGHFRQYRFD
omRFOiXVXODVRXFRQGLo}HVTXHFRPSURPHWDPUHVWULQ
REUDV$VVLPDSyVWDOYHULILFDomRD$GPLQLVWUDomR3~EOLFD
MDP RX IUXVWUHP R VHX FDUiWHU FRPSHWLWLYR LQFOXVLYH
WHUi XP SUHVWDGRU RX IRUQHFHGRU H FRP HOH WHUi TXH QRV FDVRV GH VRFLHGDGHV FRRSHUDWLYDV H HVWDEHOHoDP
ILUPDUXPFRQWUDWR7HPRVHQWmRRTXHFKDPDPRVR´FRQ SUHIHUrQFLDVRXGLVWLQo}HVHPUD]mRGDQDWXUDOLGDGH
WUDWRDGPLQLVWUDWLYRµ GD VHGH RX GRPLFtOLR GRV OLFLWDQWHV RX GH TXDOTXHU
RXWUDFLUFXQVWkQFLDLPSHUWLQHQWHRXLUUHOHYDQWHSDUDR
$VVLPRFRQWUDWRDGPLQLVWUDWLYRpRDMXVWHHPTXHD HVSHFtILFRREMHWRGRFRQWUDWRUHVVDOYDGRRGLVSRVWRQRV
††RDGHVWHDUWLJRHQRDUWRGD/HLQRGH
$GPLQLVWUDomR 3~EOLFD  DJLQGR QHVVD TXDOLGDGH  GHRXWXEURGH 5HGDomRGDGDSHOD/HLQžGH
HVWDEHOHFHFRPRXWUDSDUWH SDUWLFXODURXPHVPRRXWUD
HQWLGDGHDGPLQLVWUDWLYD YLVDQGRjUHDOL]DomRGHREMHWL ,, HVWDEHOHFHU WUDWDPHQWR GLIHUHQFLDGRGHQDWXUH]D
YRVGRLQWHUHVVHS~EOLFRHPFRQGLo}HVHVWDEHOHFLGDVSHOD FRPHUFLDOOHJDOWUDEDOKLVWDSUHYLGHQFLiULDRXTXDOTXHU
SUySULD$GPLQLVWUDomR3~EOLFD RXWUDHQWUHHPSUHVDVEUDVLOHLUDVHHVWUDQJHLUDVLQFOXVL
YH QR TXH VH UHIHUH D PRHGD PRGDOLGDGH H ORFDO GH
SDJDPHQWRVPHVPRTXDQGRHQYROYLGRVILQDQFLDPHQWRV
&RQILUD DJRUD DV GLVSRVLo}HV GD /HL Qž  ³ GH DJrQFLDV LQWHUQDFLRQDLV UHVVDOYDGR R GLVSRVWR QR
VREUHOLFLWDomR DUWžGD/HLQžGHGHRXWXEURGH
&DStWXOR
'DV'LVSRVLo}HV*HUDLV †ž (PLJXDOGDGHGHFRQGLo}HVFRPRFULWpULRGHGHVHPSD
WH VHUi DVVHJXUDGD SUHIHUrQFLD VXFHVVLYDPHQWH DRV
EHQVHVHUYLoRV
6HomR,
'RV3ULQFtSLRV , 5HYRJDGRSHOD/HLQžGH

,, SURGX]LGRVQR3DtV
$UWž (VWD/HLHVWDEHOHFHQRUPDVJHUDLVVREUHOLFLWDo}HV
HFRQWUDWRVDGPLQLVWUDWLYRVSHUWLQHQWHVDREUDV ,,, SURGX]LGRVRXSUHVWDGRVSRUHPSUHVDVEUDVLOHLUDV
VHUYLoRV LQFOXVLYH GH SXEOLFLGDGH FRPSUDV
DOLHQDo}HVHORFDo}HVQRkPELWRGRV3RGHUHVGD ,9 SURGX]LGRVRXSUHVWDGRVSRUHPSUHVDVTXHLQYLVWDPHP
SHVTXLVDHQRGHVHQYROYLPHQWRGHWHFQRORJLDQR3DtV
8QLmR GRV (VWDGRV GR 'LVWULWR )HGHUDO H GRV 2EVLQFLVRLQWURGX]LGRSHOD/HLQž
0XQLFtSLRV 9 SURGX]LGRVRXSUHVWDGRVSRUHPSUHVDVTXHFRPSURYHP
FXPSULPHQWRGHUHVHUYDGHFDUJRVSUHYLVWDHPOHLSDUD
†~QLFR 6XERUGLQDPVH DR UHJLPH GHVWD /HLDOpPGRV SHVVRDFRPGHILFLrQFLDRXSDUDUHDELOLWDGRGD3UHYLGrQ
yUJmRVGDDGPLQLVWUDomRGLUHWDRVIXQGRVHVSH FLD6RFLDOHTXHDWHQGDPjVUHJUDVGH DFHVVLELOLGDGH
FLDLV DV DXWDUTXLDV DV IXQGDo}HV S~EOLFDV DV SUHYLVWDVQDOHJLVODomR2EVLQFLVRLQWURGX]LGRSHOD/HLQž
HPSUHVDV S~EOLFDV DV VRFLHGDGHV GH HFRQRPLD
†ž $OLFLWDomRQmRVHUiVLJLORVDVHQGRS~EOLFRVHDFHVVtYHLV
PLVWDHGHPDLVHQWLGDGHVFRQWURODGDVGLUHWDRX DRS~EOLFRRVDWRVGHVHXSURFHGLPHQWRVDOYRTXDQWR
LQGLUHWDPHQWHSHOD8QLmR(VWDGRV'LVWULWR)HGH DRFRQWH~GRGDVSURSRVWDVDWpDUHVSHFWLYDDEHUWXUD
UDOH0XQLFtSLRV †ž 9HWDGR  ,QFOXtGRSHOD/HLQžGH

†ž  1RV SURFHVVRV GH OLFLWDomRSRGHUiVHUHVWDEHOHFLGD


$UWž $VREUDVVHUYLoRVLQFOXVLYHGHSXEOLFLGDGHFRP PDUJHPGHSUHIHUrQFLDSDUD
SUDVHDOLHQDo}HVFRQFHVV}HVSHUPLVV}HVHORFD
o}HVGD$GPLQLVWUDomR3~EOLFDTXDQGRFRQWUDWD , SURGXWRVPDQXIDWXUDGRVHSDUDVHUYLoRVQDFLRQDLVTXH
GDVFRPWHUFHLURVVHUmRQHFHVVDULDPHQWHSUHFHGL DWHQGDPDQRUPDVWpFQLFDVEUDVLOHLUDVH
GDVGHOLFLWDomRUHVVDOYDGDVDVKLSyWHVHVSUHYLVWDV
,, EHQVHVHUYLoRVSURGX]LGRVRXSUHVWDGRVSRUHPSUHVDV
QHVWD/HL
TXH FRPSURYHP FXPSULPHQWR GH UHVHUYD GH FDUJRV
SUHYLVWD HP OHL SDUD SHVVRD FRP GHILFLrQFLD RX SDUD
†~QLFR 3DUDRVILQVGHVWD/HLFRQVLGHUDVHFRQWUDWRWRGR UHDELOLWDGR GD 3UHYLGrQFLD 6RFLDO H TXH DWHQGDP jV
H TXDOTXHU DMXVWH HQWUH yUJmRV RX HQWLGDGHV GD UHJUDVGHDFHVVLELOLGDGHSUHYLVWDVQDOHJLVODomR2EVUHGDomR
GDGDSHOD/HLQž
$GPLQLVWUDomR3~EOLFDHSDUWLFXODUHVHPTXHKDMD
XP DFRUGR GH YRQWDGHV SDUD D IRUPDomR GH †ž $ PDUJHP GH SUHIHUrQFLD GH TXH WUDWD R †RVHUi
YtQFXORHDHVWLSXODomRGHREULJDo}HVUHFtSURFDV HVWDEHOHFLGDFRPEDVHHPHVWXGRVUHYLVWRVSHULRGLFD
VHMDTXDOIRUDGHQRPLQDomRXWLOL]DGD PHQWHHPSUD]RQmR VXSHULRUD FLQFR DQRVTXH
OHYHPHPFRQVLGHUDomR ,QFOXtGRSHOD/HLQžGH
 &kPDUD/HJLVODWLYDGR
 'LVWULWR)HGHUDO 'LU$GPLQLVWUDWLYR 

, JHUDomRGHHPSUHJRHUHQGD $UWž 7RGRVTXDQWRVSDUWLFLSHPGHOLFLWDomRSURPRYLGDSHORV


,, HIHLWRQDDUUHFDGDomRGHWULEXWRVIHGHUDLVHVWDGXDLVH yUJmRVRXHQWLGDGHVDTXHVHUHIHUHRDUWžWrPGLUHLWR
PXQLFLSDLV S~EOLFR VXEMHWLYR j ILHO REVHUYkQFLD GR SHUWLQHQWH
,,, GHVHQYROYLPHQWRHLQRYDomRWHFQROyJLFDUHDOL]DGRVQR SURFHGLPHQWRHVWDEHOHFLGRQHVWD/HLSRGHQGRTXDOTXHU
3DtV FLGDGmRDFRPSDQKDURVHXGHVHQYROYLPHQWRGHVGHTXH
,9 FXVWRDGLFLRQDOGRVSURGXWRVHVHUYLoRVH QmRLQWHUILUDGHPRGRDSHUWXUEDURXLPSHGLUDUHDOL]D
9 HPVXDVUHYLV}HVDQiOLVHUHWURVSHFWLYDGHUHVXOWDGRV omRGRVWUDEDOKRV

†ž 3DUDRVSURGXWRVPDQXIDWXUDGRVHVHUYLoRVQDFLRQDLV †~QLFR 2SURFHGLPHQWROLFLWDWyULRSUHYLVWRQHVWD/HLFDUDFWHUL


UHVXOWDQWHVGHGHVHQYROYLPHQWRHLQRYDomRWHFQROyJLFD ]D DWR DGPLQLVWUDWLYR IRUPDO VHMD HOH SUDWLFDGR HP
UHDOL]DGRVQR3DtVSRGHUiVHUHVWDEHOHFLGRPDUJHPGH TXDOTXHUHVIHUDGD$GPLQLVWUDomR3~EOLFD
SUHIHUrQFLDDGLFLRQDOjTXHODSUHYLVWDQR†ž ,QFOXtGRSHOD
/HLQžGH $UWž 7RGRVRVYDORUHVSUHoRVHFXVWRVXWLOL]DGRVQDVOLFLWD
o}HVWHUmRFRPRH[SUHVVmRPRQHWiULDDPRHGDFRUUHQWH
†ž $VPDUJHQVGHSUHIHUrQFLDSRUSURGXWRVHUYLoRJUXSR QDFLRQDO UHVVDOYDGR R GLVSRVWR QR DUW GHVWD /HL
GHSURGXWRVRXJUXSRGHVHUYLoRVDTXHVHUHIHUHPRV GHYHQGRFDGDXQLGDGHGD$GPLQLVWUDomRQRSDJDPHQ
†† R H R VHUmR GHILQLGDV SHOR 3RGHU ([HFXWLYR WRGDVREULJDo}HVUHODWLYDVDRIRUQHFLPHQWRGHEHQV
IHGHUDO QmR SRGHQGR D VRPD GHODV XOWUDSDVVDU R ORFDo}HVUHDOL]DomRGHREUDVHSUHVWDomRGHVHUYLoRV
PRQWDQWH GH  YLQWH H FLQFR SRU FHQWR  VREUH R REHGHFHUSDUDFDGDIRQWHGLIHUHQFLDGDGHUHFXUVRVD
SUHoRGRVSURGXWRVPDQXIDWXUDGRVHVHUYLoRVHVWUDQJHL HVWULWDRUGHPFURQROyJLFDGDVGDWDVGHVXDVH[LJLELOLGD
URV ,QFOXtGRSHOD/HLQžGH GHV VDOYR TXDQGR SUHVHQWHV UHOHYDQWHV UD]}HV GH
LQWHUHVVH S~EOLFR H PHGLDQWH SUpYLD MXVWLILFDWLYD GD
DXWRULGDGHFRPSHWHQWHGHYLGDPHQWHSXEOLFDGD
†ž $VGLVSRVLo}HVFRQWLGDVQRV††RHRGHVWHDUWLJRQmR
VHDSOLFDPDRVEHQVHDRVVHUYLoRVFXMDFDSDFLGDGHGH †ž 2V FUpGLWRV D TXH VH UHIHUH HVWH DUWLJRWHUmRVHXV
SURGXomRRXSUHVWDomRQR3DtVVHMDLQIHULRU ,QFOXtGRSHOD/HL YDORUHV FRUULJLGRV SRU FULWpULRV SUHYLVWRV QR DWR FRQ
QžGH
YRFDWyULRHTXHOKHVSUHVHUYHPRYDORU
, jTXDQWLGDGHDVHUDGTXLULGDRXFRQWUDWDGDRX †ž $ FRUUHomR GH TXH WUDWD R SDUiJUDIRDQWHULRUFXMR
SDJDPHQWRVHUiIHLWRMXQWRFRPRSULQFLSDOFRUUHUij
,, DRTXDQWLWDWLYRIL[DGRFRPIXQGDPHQWRQR†RGRDUW FRQWDGDVPHVPDVGRWDo}HVRUoDPHQWiULDVTXHDWHQGH
GHVWD/HLTXDQGRIRURFDVR UDPDRVFUpGLWRVDTXHUHIHUHP 2EV FRP UHGDomR GDGD SHOD /HL


† $PDUJHPGHSUHIHUrQFLDDTXHVHUHIHUHR†RSRGHUi †ž 2EVHUYDGR R GLVSRVWR QR ´FDSXWµRVSDJDPHQWRV


VHUHVWHQGLGDWRWDORXSDUFLDOPHQWHDRVEHQVHVHUYL GHFRUUHQWHVGHGHVSHVDVFXMRVYDORUHVQmRXOWUDSDVVHP
oRVRULJLQiULRVGRV(VWDGRV3DUWHVGR0HUFDGR&RPXP ROLPLWHGHTXHWUDWDRLQFLVR,,GRDUWVHPSUHMXt]R
GR6XO0HUFRVXO ,QFOXtGRSHOD/HLQžGH GR TXH GLVS}HV VHX SDUiJUDIR ~QLFR GHYHUmR VHU
HIHWXDGRVQRSUD]RGHDWp FLQFR GLDV~WHLVFRQWDGRV
† 2V HGLWDLV GH OLFLWDomR SDUD D FRQWUDWDomRGHEHQV
GDDSUHVHQWDomRGDIDWXUD2EV †žDFUHVFLGRSHOD/HLQž
VHUYLoRVHREUDVSRGHUmRPHGLDQWHSUpYLDMXVWLILFDWLYD
GD DXWRULGDGH FRPSHWHQWH H[LJLU TXH R FRQWUDWDGR $UWž$ $VQRUPDVGHOLFLWDo}HVHFRQWUDWRVGHYHPSULYLOHJLDUR
SURPRYDHPIDYRUGHyUJmRRXHQWLGDGHLQWHJUDQWHGD WUDWDPHQWRGLIHUHQFLDGRHIDYRUHFLGRjVPLFURHPSUHVDV
DGPLQLVWUDomRS~EOLFDRXGDTXHOHVSRUHODLQGLFDGRVD HHPSUHVDVGHSHTXHQRSRUWHQDIRUPDGDOHL ,QFOXtGRSHOD
SDUWLUGHSURFHVVRLVRQ{PLFRPHGLGDVGHFRPSHQVDomR /HL&RPSOHPHQWDUQžGH

FRPHUFLDOLQGXVWULDOWHFQROyJLFDRXDFHVVRDFRQGLo}HV
YDQWDMRVDVGHILQDQFLDPHQWRFXPXODWLYDPHQWHRXQmR 6HomR,,
QD IRUPD HVWDEHOHFLGD SHOR 3RGHU ([HFXWLYR IHGHUDO 'DV'HILQLo}HV
,QFOXtGRSHOD/HLQžGH
$UWž 3DUDRVILQVGHVWD/HLFRQVLGHUDVH
† 1DVFRQWUDWDo}HVGHVWLQDGDVjLPSODQWDomRPDQXWHQ
omRHDRDSHUIHLoRDPHQWRGRVVLVWHPDVGHWHFQRORJLDGH , 2EUDWRGDFRQVWUXomRUHIRUPDIDEULFDomRUHFXSHUD
LQIRUPDomRHFRPXQLFDomRFRQVLGHUDGRVHVWUDWpJLFRV omR RX DPSOLDomR UHDOL]DGD SRU H[HFXomR GLUHWD RX
HPDWRGR3RGHU([HFXWLYRIHGHUDODOLFLWDomRSRGHUi LQGLUHWD
VHUUHVWULWDDEHQVHVHUYLoRVFRPWHFQRORJLDGHVHQYROYL
GD QR 3DtV H SURGX]LGRV GH DFRUGR FRP R SURFHVVR ,, 6HUYLoR7RGDDWLYLGDGHGHVWLQDGDDREWHUGHWHUPLQDGD
SURGXWLYREiVLFRGHTXHWUDWDD/HLQžGHGH XWLOLGDGHGHLQWHUHVVHSDUDD$GPLQLVWUDomRWDLVFRPR
MDQHLURGH ,QFOXtGRSHOD/HLQžGH GHPROLomRFRQVHUWRLQVWDODomRPRQWDJHPRSHUDomR
FRQVHUYDomRUHSDUDomRDGDSWDomRPDQXWHQomRWUDQV
† 6HUiGLYXOJDGDQDLQWHUQHWDFDGDH[HUFtFLRILQDQFHLUR SRUWHORFDomRGHEHQVSXEOLFLGDGHVHJXURRXWUDED
DUHODomRGHHPSUHVDVIDYRUHFLGDVHPGHFRUUrQFLDGR OKRVWpFQLFRSURILVVLRQDLV
GLVSRVWRQRV††RRHGHVWHDUWLJRFRP
LQGLFDomR GR YROXPH GH UHFXUVRV GHVWLQDGRV D FDGD ,,, &RPSUD  WRGD DTXLVLomR UHPXQHUDGD GHEHQVSDUD
XPDGHODV ,QFOXtGRSHOD/HLQžGH IRUQHFLPHQWRGHXPDVyYH]RXSDUFHODGDPHQWH

† $V SUHIHUrQFLDV GHILQLGDV QHVWH DUWLJRHQDVGHPDLV ,9 $OLHQDomRWRGDWUDQVIHUrQFLDGHGRPtQLRGHEHQVD


QRUPDV GH OLFLWDomR H FRQWUDWRV GHYHP SULYLOHJLDU R WHUFHLURV
WUDWDPHQWRGLIHUHQFLDGRHIDYRUHFLGRjVPLFURHPSUHVDV
HHPSUHVDVGHSHTXHQRSRUWHQDIRUPDGDOHL ,QFOXtGRSHOD 9 2EUDVVHUYLoRVHFRPSUDVGHJUDQGHYXOWRDTXHODV
/HL&RPSOHPHQWDUQžGH FXMRYDORUHVWLPDGRVHMDVXSHULRUD YLQWHHFLQFR
YH]HVXPELOKmRGHFUX]HLURV YDORUDWXDOL]iYHODSDUWLUGH
† $VSUHIHUrQFLDVGLVSRVWDVQHVWHDUWLJRSUHYDOHFHPVREUH  
DVGHPDLVSUHIHUrQFLDVSUHYLVWDVQDOHJLVODomRTXDQGR
HVWDV IRUHP DSOLFDGDV VREUH SURGXWRV RX VHUYLoRV 9, 6HJXUR*DUDQWLD  R VHJXUR TXH JDUDQWHRILHOFXP
HVWUDQJHLURV ,QFOXtGRSHOD/HL&RPSOHPHQWDUQž SULPHQWRGDVREULJDo}HVDVVXPLGDVSRUHPSUHVDVHP
OLFLWDo}HVHFRQWUDWRV
 'LU$GPLQLVWUDWLYR
 &kPDUD/HJLVODWLYDGR'LVWULWR)HGHUDO
9,, ([HFXomR'LUHWDDTXHpIHLWDSHORVyUJmRVHHQWLGD ;,, $GPLQLVWUDomRyUJmRHQWLGDGHRXXQLGDGHDGPLQLV
GHVGD$GPLQLVWUDomRSHORVSUySULRVPHLRV WUDWLYDSHODTXDOD$GPLQLVWUDomR3~EOLFDRSHUDHDWXD
FRQFUHWDPHQWH
9,,, ([HFXomR,QGLUHWDDTXHRyUJmRRXHQWLGDGHFRQWUD ;,,, ,PSUHQVD 2ILFLDO  YHtFXOR RILFLDO GH GLYXOJDomRGD
WDFRPWHUFHLURVVRETXDOTXHUGRVVHJXLQWHVUHJLPHV $GPLQLVWUDomR3~EOLFDVHQGRSDUDD8QLmRR'LiULR
2EV5HGDomRGDGDSHOD/HL
2ILFLDOGD8QLmRHSDUDRV(VWDGRVR'LVWULWR)HGHUDO
D (PSUHLWD SRU 3UHoR *OREDO  TXDQGR VH FRQWUDWD D H RV 0XQLFtSLRV R TXH IRU GHILQLGR QDV UHVSHFWLYDV
H[HFXomR GD REUD RX GR VHUYLoR SRU SUHoR FHUWR H OHLV UHGDomRGDGDSHOD/HL 
WRWDO ;,9 &RQWUDWDQWH  p R yUJmR RX HQWLGDGH VLJQDWiULDGR
E (PSUHLWDGDSRU3UHoR8QLWiULRTXDQGRVHFRQWUDWDD LQVWUXPHQWRFRQWUDWXDO
H[HFXomR GD REUD RX GR VHUYLoR SRU SUHoR FHUWR GH ;9 &RQWUDWDGRDSHVVRDItVLFDRXMXUtGLFDVLJQDWiULDGH
XQLGDGHVGHWHUPLQDGDV FRQWUDWRFRPD$GPLQLVWUDomR3~EOLFD
F YHWDGR ;9, &RPLVVmRFRPLVVmRSHUPDQHQWHRXHVSHFLDOFULDGD
G 7DUHIDTXDQGRVHDMXVWDPmRGHREUDSDUDSHTXHQRV SHOD$GPLQLVWUDomRFRPDIXQomRGHUHFHEHUH[DPL
WUDEDOKRVSRUSUHoRFHUWRFRPRXVHPIRUQHFLPHQWR QDU H MXOJDU WRGRV RV GRFXPHQWRV H SURFHGLPHQWRV
GHPDWHULDLV
UHODWLYRVjVOLFLWDo}HVHDRFDGDVWUDPHQWRGHOLFLWDQWHV
H (PSUHLWDGD,QWHJUDOTXDQGRVHFRQWUDWDXPHPSUH
;9,, SURGXWRVPDQXIDWXUDGRVQDFLRQDLVSURGXWRVPDQXID
HQGLPHQWR HP VXD LQWHJUDOLGDGH FRPSUHHQGHQGR
WXUDGRVSURGX]LGRVQRWHUULWyULRQDFLRQDOGHDFRUGR
WRGDV DV HWDSDV GDV REUDV VHUYLoRV H LQVWDODo}HV
FRPRSURFHVVRSURGXWLYREiVLFRRXFRPDVUHJUDVGH
QHFHVViULDVVRELQWHLUDUHVSRQVDELOLGDGHGDFRQWUDWD
GDDWpDVXDHQWUHJDDRFRQWUDWDQWHHPFRQGLo}HVGH RULJHP HVWDEHOHFLGDV SHOR 3RGHU ([HFXWLYR IHGHUDO
,QFOXtGRSHOD/HLQžGH
HQWUDGDHPRSHUDomRDWHQGLGRVRVUHTXLVLWRVWpFQLFRV ;9,,, VHUYLoRV QDFLRQDLV  VHUYLoRV SUHVWDGRV QR 3DtVQDV
HOHJDLVSDUDVXDXWLOL]DomRHPFRQGLo}HVGHVHJXUDQ FRQGLo}HVHVWDEHOHFLGDVSHOR3RGHU([HFXWLYRIHGHUDO
oD HVWUXWXUDO H RSHUDFLRQDO H FRP DV FDUDFWHUtVWLFDV ,QFOXtGRSHOD/HLQžGH
DGHTXDGDVjVILQDOLGDGHVSDUDTXHIRLFRQWUDWDGD ;,; VLVWHPDVGHWHFQRORJLDGHLQIRUPDomRHFRPXQLFDomR
HVWUDWpJLFRVEHQVHVHUYLoRVGHWHFQRORJLDGDLQIRU
,; 3URMHWR%iVLFRFRQMXQWRGHHOHPHQWRVQHFHVViULRVH PDomRHFRPXQLFDomRFXMDGHVFRQWLQXLGDGHSURYRTXH
VXILFLHQWHV FRP QtYHO GH SUHFLVmR DGHTXDGR SDUD GDQR VLJQLILFDWLYR j DGPLQLVWUDomR S~EOLFD H TXH
FDUDFWHUL]DUDREUDRXVHUYLoRRXFRPSOH[RGHREUDV HQYROYDP SHOR PHQRV XP GRV VHJXLQWHV UHTXLVLWRV
RX VHUYLoRV REMHWR GD OLFLWDomR HODERUDGR FRP EDVH UHODFLRQDGRVjVLQIRUPDo}HVFUtWLFDVGLVSRQLELOLGDGH
QDVLQGLFDo}HVGRVHVWXGRVWpFQLFRVSUHOLPLQDUHVTXH FRQILDELOLGDGHVHJXUDQoDHFRQILGHQFLDOLGDGH ,QFOXtGR
VH DVVHJXUHP D YLDELOLGDGH WpFQLFD H R DGHTXDGR SHOD/HLQžGH

WUDWDPHQWRGRLPSDFWRDPELHQWDOGRHPSUHHQGLPHQ ;; SURGXWRV SDUD SHVTXLVD H GHVHQYROYLPHQWR EHQV


WRHTXHSRVVLELOLWHDDYDOLDomRGRFXVWRGDREUDHD LQVXPRVVHUYLoRVHREUDVQHFHVViULRVSDUDDWLYLGDGH
GHILQLomR GRV PpWRGRV H GR SUD]R GH H[HFXomR GHSHVTXLVDFLHQWtILFDHWHFQROyJLFDGHVHQYROYLPHQWR
GHYHQGRFRQWHURVVHJXLQWHVHOHPHQWRV GHWHFQRORJLDRXLQRYDomRWHFQROyJLFDGLVFULPLQDGRV
HP SURMHWR GH SHVTXLVD DSURYDGR SHOD LQVWLWXLomR
D GHVHQYROYLPHQWR GD VROXomR HVFROKLGD GH IRUPD D FRQWUDWDQWH ,QFOXtGRSHOD/HLQžGH
IRUQHFHUYLVmRJOREDOGDREUDHLGHQWLILFDUWRGRVRV
VHXVHOHPHQWRVFRQVWLWXWLYRVFRPFODUH]D 6HomR,,,
E VROXo}HVWpFQLFDVJOREDLVHORFDOL]DGDVVXILFLHQWHPHQ 'DV2EUDVH6HUYLoRV
WHGHWDOKDGDVGHIRUPDDPLQLPL]DUDQHFHVVLGDGHGH
UHIRUPXODomR RX GH YDULDQWHV GXUDQWH DV IDVHV GH $UWž $VOLFLWDo}HVSDUDDH[HFXomRGHREUDVHSDUDDSUHVWD
HODERUDomRGRSURMHWRH[HFXWLYRHGHUHDOL]DomRGDV omRGHVHUYLoRVREHGHFHUmRDRGLVSRVWRQHVWHDUWLJRH
REUDVHPRQWDJHP HPSDUWLFXODUjVHJXLQWHVHTrQFLD
F LGHQWLILFDomR GRV WLSRV GH VHUYLoRV D H[HFXWDU H GH
PDWHULDLVHHTXLSDPHQWRVDLQFRUSRUDUjREUDEHP , SURMHWREiVLFR
FRPRVXDVHVSHFLILFDo}HVTXHDVVHJXUHPRVPHOKRUHV ,, SURMHWRH[HFXWLYR
UHVXOWDGRV SDUD R HPSUHHQGLPHQWR VHP IUXVWUDU R ,,, H[HFXomRGDVREUDVHVHUYLoRV
FDUiWHUFRPSHWLWLYRSDUDDVXDH[HFXomR
G LQIRUPDo}HVTXHSRVVLELOLWHPRHVWXGRHDGHGXomRGH †ž  $ H[HFXomR GH FDGD HWDSDVHUiREULJDWRULDPHQWH
PpWRGRVFRQVWUXWLYRVLQVWDODo}HVSURYLVyULDVHFRQGL SUHFHGLGDGDFRQFOXVmRHDSURYDomRSHODDXWRULGDGH
o}HV RUJDQL]DFLRQDLV SDUD D REUD VHP IUXVWUDU R FRPSHWHQWHGRVWUDEDOKRVUHODWLYRVjVHWDSDVDQWHULR
FDUiWHUFRPSHWLWLYRSDUDDVXDH[HFXomR UHVjH[FHomRGRSURMHWRH[HFXWLYRRTXDOSRGHUiVHU
H VXEVtGLRV SDUD PRQWDJHP GR SODQR GH OLFLWDomR H GHVHQYROYLGRFRQFRPLWDQWHPHQWHFRPDH[HFXomRGDV
JHVWmRGDREUDFRPSUHHQGHQGRDVXDSURJUDPDomR REUDVHVHUYLoRVGHVGHTXHWDPEpPDXWRUL]DGRSHOD
DHVWUDWpJLDGHVXSULPHQWRVDVQRUPDVGHILVFDOL]DomR $GPLQLVWUDomR
HRXWURVGDGRVQHFHVViULRVHPFDGDFDVR
I RUoDPHQWRGHWDOKDGRGRFXVWRJOREDOGDREUDIXQGD
†ž  $VREUDVHRVVHUYLoRVVRPHQWHSRGHUmRVHUOLFLWDGRV
PHQWDGRHPTXDQWLWDWLYRVGHVHUYLoRVHIRUQHFLPHQWRV
TXDQGR
SUySULRVDYDOLDGRV

; 3URMHWR([HFXWLYRRFRQMXQWRGRVHOHPHQWRVQHFHVVi ,  KRXYHU SURMHWR EiVLFRDSURYDGRSHODDXWRULGDGH


ULRV H VXILFLHQWHV j H[HFXomR FRPSOHWD GD REUD GH FRPSHWHQWHHGLVSRQtYHOSDUDH[DPHGRVLQWHUHVVDGRV
DFRUGR FRP DV QRUPDV SHUWLQHQWHV GD $VVRFLDomR HPSDUWLFLSDUGRSURFHVVROLFLWDWyULR
%UDVLOHLUDGH1RUPDV7pFQLFDV$%17
,,  H[LVWLURUoDPHQWRGHWDOKDGRHPSODQLOKDVTXHH[SUHV
;, $GPLQLVWUDomR 3~EOLFD  D DGPLQLVWUDomRGLUHWDH VHPDFRPSRVLomRGHWRGRVRVVHXVFXVWRVXQLWiULRV
LQGLUHWDGD8QLmRGRV(VWDGRVGR'LVWULWR)HGHUDOH
GRV 0XQLFtSLRV DEUDQJHQGR LQFOXVLYH DV HQWLGDGHV ,,,  KRXYHUSUHYLVmRGHUHFXUVRVRUoDPHQWiULRVTXHDVVHJX
FRP SHUVRQDOLGDGH MXUtGLFD GH GLUHLWR SULYDGR VRE UHPRSDJDPHQWRGDVREULJDo}HVGHFRUUHQWHVGHREUDV
FRQWUROH GR SRGHU S~EOLFR H GDV IXQGDo}HV SRU HOH RXVHUYLoRVDVHUHPH[HFXWDGDVQRH[HUFtFLRILQDQFHLUR
LQVWLWXtGDVRXPDQWLGDV HPFXUVRGHDFRUGRFRPRUHVSHFWLYRFURQRJUDPD
 &kPDUD/HJLVODWLYDGR
 'LVWULWR)HGHUDO 'LU$GPLQLVWUDWLYR 

,9  R SURGXWR GHOD HVSHUDGR HVWLYHU FRQWHPSODGRQDV †ž  eSHUPLWLGDDSDUWLFLSDomRGRDXWRUGRSURMHWRRXGD


PHWDVHVWDEHOHFLGDVQR3ODQR3OXULDQXDOGHTXHWUDWD HPSUHVD D TXH VH UHIHUH R LQFLVR ,, GHVWH DUWLJR QD
RDUWGD&RQVWLWXLomR)HGHUDOTXDQGRIRURFDVR OLFLWDomR GH REUD RX VHUYLoR RX QD H[HFXomR FRPR
FRQVXOWRU RX WpFQLFR QDV IXQo}HV GH ILVFDOL]DomR
†ž  eYHGDGRLQFOXLUQRREMHWRGDOLFLWDomRDREWHQomRGH VXSHUYLVmRRXJHUHQFLDPHQWRH[FOXVLYDPHQWHDVHUYLoR
UHFXUVRVILQDQFHLURVSDUDVXDH[HFXomRTXDOTXHUTXH GD$GPLQLVWUDomRLQWHUHVVDGD
VHMDDVXDRULJHPH[FHWRQRVFDVRVGHHPSUHHQGLPHQ
WRVH[HFXWDGRVHH[SORUDGRVVRERUHJLPHGHFRQFHV †ž  2 GLVSRVWR QHVWH DUWLJR QmR LPSHGH DOLFLWDomRRX
VmRQRVWHUPRVGDOHJLVODomRHVSHFtILFD FRQWUDWDomRGHREUDRXVHUYLoRTXHLQFOXDDHODERUDomR
GHSURMHWRH[HFXWLYRFRPRHQFDUJRGRFRQWUDWDGRRX
†ž  eYHGDGDDLQGDDLQFOXVmRQRREMHWRGDOLFLWDomRGH SHORSUHoRSUHYLDPHQWHIL[DGRSHOD$GPLQLVWUDomR
IRUQHFLPHQWRGHPDWHULDLVHVHUYLoRVVHPSUHYLVmRGH
TXDQWLGDGHVRXFXMRVTXDQWLWDWLYRVQmRFRUUHVSRQGDP †ž  &RQVLGHUDVHSDUWLFLSDomRLQGLUHWDSDUDILQVGRGLVSRV
jVSUHYLV}HVUHDLVGRSURMHWREiVLFRRXH[HFXWLYR WR QHVWH DUWLJR D H[LVWrQFLD GH TXDOTXHU YtQFXOR GH
QDWXUH]DWpFQLFDFRPHUFLDOHFRQ{PLFDILQDQFHLUDRX
†ž  eYHGDGDDUHDOL]DomRGHOLFLWDomRFXMRREMHWRLQFOXD WUDEDOKLVWDHQWUHRDXWRUGRSURMHWRSHVVRDItVLFDRX
EHQVHVHUYLoRVVHPVLPLODULGDGHRXGHPDUFDVFDUDF MXUtGLFD H R OLFLWDQWH RX UHVSRQViYHO SHORV VHUYLoRV
WHUtVWLFDVHHVSHFLILFDo}HVH[FOXVLYDVVDOYRQRVFDVRV IRUQHFLPHQWRVHREUDVLQFOXLQGRVHRVIRUQHFLPHQWRV
HPTXHIRUWHFQLFDPHQWHMXVWLILFiYHORXDLQGDTXDQGR GHEHQVHVHUYLoRVDHVWHVQHFHVViULRV
RIRUQHFLPHQWRGHWDLVPDWHULDLVHVHUYLoRVIRUIHLWR
VRERUHJLPHGHDGPLQLVWUDomRFRQWUDWDGDSUHYLVWRH †ž  2GLVSRVWRQRSDUiJUDIRDQWHULRUDSOLFDVHDRVPHP
GLVFULPLQDGRQRDWRFRQYRFDWyULR EURVGDFRPLVVmRGHOLFLWDomR

†ž  $ LQIULQJrQFLD GR GLVSRVWR QHVWH DUWLJRLPSOLFDD $UW $VREUDVHVHUYLoRVSRGHUmRVHUH[HFXWDGDVQDVVHJXLQ


QXOLGDGH GRV DWRV RX FRQWUDWRV UHDOL]DGRV H D WHVIRUPDV
2EVFRPUHGDomRGDGDSHOD/HL
UHVSRQVDELOLGDGHGHTXHPOKHVWHQKDGDGRFDXVD , H[HFXomRGLUHWD
,, H[HFXomRLQGLUHWDQRVVHJXLQWHVUHJLPHV
†ž  1mR VHUi DLQGD FRPSXWDGR FRPR YDORU GDREUDRX
VHUYLoR SDUD ILQV GH MXOJDPHQWR GDV SURSRVWDV GH D HPSUHLWDGDSRUSUHoRJOREDO
SUHoRV D DWXDOL]DomR PRQHWiULD GDV REULJDo}HV GH E HPSUHLWDGDSRUSUHoRXQLWiULR
SDJDPHQWR GHVGH D GDWD ILQDO GH FDGD SHUtRGR GH F 9HWDGR 
DIHULomR DWp D GR UHVSHFWLYR SDJDPHQWR TXH VHUi G WDUHID
FDOFXODGD SHORV PHVPRV FULWpULRV HVWDEHOHFLGRV REUL H HPSUHLWDGDLQWHJUDO2EVFRPUHGDomRGDGDSHOD/HL
JDWRULDPHQWHQRDWRFRQYRFDWyULR
†~QLFR 9HWDGR 
†ž  4XDOTXHU FLGDGmR SRGHUi UHTXHUHUj$GPLQLVWUDomR
3~EOLFDRVTXDQWLWDWLYRVGDVREUDVHSUHoRVXQLWiULRVGH , MXVWLILFDGR WHFQLFDPHQWH FRPDGHPRQVWUDomRGH
GHWHUPLQDGDREUDH[HFXWDGD YDQWDJHQVSDUDDDGPLQLVWUDomRHPUHODomRDRVGHPDLV
UHJLPHV
,, RV YDORUHV QmR XOWUDSDVVDUHP RVOLPLWHVPi[LPRV
†ž  2 GLVSRVWR QHVWH DUWLJR DSOLFDVH WDPEpPQRTXH
HVWDEHOHFLGRVSDUDDPRGDOLGDGHGHWRPDGDGHSUHoRV
FRXEHUDRVFDVRVGHGLVSHQVDHGHLQH[LJLELOLGDGHGH
FRQVWDQWHVQRDUWGHVWD/HL
OLFLWDomR
,,, SUHYLDPHQWHDSURYDGRSHODDXWRULGDGHFRPSHWHQWH2EV
,QFLVRVIRUDPDFUHVFHQWDGRVSHOD/HL
$UWž $H[HFXomRGDVREUDVHGRVVHUYLoRVGHYHSURJUDPDU
VH VHPSUH HP VXD WRWDOLGDGH SUHYLVWRV VHXV FXVWRV $UW $VREUDVHVHUYLoRVGHVWLQDGRVDRVPHVPRVILQVWHUmR
DWXDOHILQDOHFRQVLGHUDGRVRVSUD]RVGHVXDH[HFXomR SURMHWRVSDGURQL]DGRVSRUWLSRVFDWHJRULDVRXFODVVHV
H[FHWRTXDQGRRSURMHWRSDGUmRQmRDWHQGHUjVFRQGL
†~QLFR eSURLELGRRUHWDUGDPHQWRLPRWLYDGRGDH[HFXomRGH o}HVSHFXOLDUHVGRORFDORXjVH[LJrQFLDVHVSHFtILFDVGR
REUD RX VHUYLoR RX GH VXDV SDUFHODV VH H[LVWHQWH HPSUHHQGLPHQWR
SUHYLVmRRUoDPHQWiULDSDUDVXDH[HFXomRWRWDOVDOYR
LQVXILFLrQFLD ILQDQFHLUD RX FRPSURYDGR PRWLYR GH $UW 1RVSURMHWRVEiVLFRVHSURMHWRVH[HFXWLYRVGHREUDVH
RUGHPWpFQLFDMXVWLILFDGRVHPGHVSDFKRFLUFXQVWDQFLD VHUYLoRVVHUmRFRQVLGHUDGRVSULQFLSDOPHQWHRVVHJXLQ
GRGDDXWRULGDGHDTXHVHUHIHUHRDUWGHVWD/HL FRP WHVUHTXLVLWRV
UHGDomRGDGDSHOD/HL 
,VHJXUDQoD
$UWž 1mR SRGHUi SDUWLFLSDU GLUHWD RX LQGLUHWDPHQWHGD ,,IXQFLRQDOLGDGHHDGHTXDomRDRLQWHUHVVHS~EOLFR
OLFLWDomR RX GD H[HFXomR GH REUD RX VHUYLoR H GR ,,,HFRQRPLDQDH[HFXomRFRQVHUYDomRHRSHUDomR
IRUQHFLPHQWRGHEHQVDHOHVQHFHVViULRV ,9 SRVVLELOLGDGHGHHPSUHJRGHPmRGHREUDPDWHULDLV
WHFQRORJLDHPDWpULDVSULPDVH[LVWHQWHVQRORFDOSDUD
,  RDXWRUGRSURMHWREiVLFRRXH[HFXWLYRSHVVRDItVLFD H[HFXomRFRQVHUYDomRHRSHUDomR
RXMXUtGLFD 9 IDFLOLGDGHQDH[HFXomRFRQVHUYDomRHRSHUDomRVHP
SUHMXt]RGDGXUDELOLGDGHGDREUDRXGRVHUYLoR
,,  HPSUHVDLVRODGDPHQWHRXHPFRQVyUFLRUHVSRQViYHO 9, DGRomRGDVQRUPDVWpFQLFDVGHVD~GHHGHVHJXUDQoD
SHODHODERUDomRGRSURMHWREiVLFRRXH[HFXWLYRRXGD GRWUDEDOKRDGHTXDGDV
TXDORDXWRUGRSURMHWRVHMDGLULJHQWHJHUHQWHDFLRQLV 9,, LPSDFWRDPELHQWDO FRPUHGDomRGDGDSHOD/HL
WDRXGHWHQWRUGHPDLVGH FLQFRSRUFHQWR GRFDSLWDO
FRPGLUHLWRDYRWRRXFRQWURODGRUUHVSRQViYHOWpFQLFR 6HomR,9
RXVXEFRQWUDWDGR 'RV6HUYLoRV7pFQLFRV3URILVVLRQDLV(VSHFLDOL]DGRV

,,,  VHUYLGRURXGLULJHQWHGHyUJmRRXHQWLGDGHFRQWUDWDQWH $UW 3DUDRVILQVGHVWD/HLFRQVLGHUDPVHVHUYLoRVWpFQLFRV


RXUHVSRQViYHOSHODOLFLWDomR SURILVVLRQDLVHVSHFLDOL]DGRVRVWUDEDOKRVUHODWLYRVD
 'LU$GPLQLVWUDWLYR
 &kPDUD/HJLVODWLYDGR'LVWULWR)HGHUDO
, HVWXGRV WpFQLFRV SODQHMDPHQWR HSURMHWRVEiVLFRVRX , VHOHomRIHLWDPHGLDQWHFRQFRUUrQFLD
H[HFXWLYRV
,, HVWLSXODomRSUpYLDGRVLVWHPDGHFRQWUROHHDWXDOL]DomR
,, SDUHFHUHVSHUtFLDVHDYDOLDo}HVHPJHUDO GRVSUHoRVUHJLVWUDGRV

,,, DVVHVVRULDVRXFRQVXOWRULDVWpFQLFDVHDXGLWRULDVILQDQFHL ,,, YDOLGDGHGRUHJLVWURQmRVXSHULRUDXPDDQR


UDVRXWULEXWiULDV
†ž $H[LVWrQFLDGHSUHoRVUHJLVWUDGRVQmRREULJDD$GPL
,9 ILVFDOL]DomR VXSHUYLVmR RX JHUHQFLDPHQWR GH REUDVRX QLVWUDomRDILUPDUDVFRQWUDWDo}HVTXHGHOHVSRGHUmR
VHUYLoRV DGYLU ILFDQGROKH IDFXOWDGD D XWLOL]DomR GH RXWURV
PHLRV UHVSHLWDGD D OHJLVODomR UHODWLYD jV OLFLWDo}HV
9 SDWURFtQLRRXGHIHVDGHFDXVDVMXGLFLDLVRXDGPLQLVWUDWL VHQGRDVVHJXUDGRDREHQHILFLiULRGRUHJLVWURGHSUHIH
YDV UrQFLDHPLJXDOGDGHGHFRQGLo}HV
9, WUHLQDPHQWRHDSHUIHLoRDPHQWRGHSHVVRDO
†ž 2 VLVWHPD GH FRQWUROH RULJLQDGR QR TXDGURJHUDOGH
9,, UHVWDXUDomRGHREUDVGHDUWHHEHQVGHYDORUKLVWyULFR
FRPUHGDomRGDGDSHOD/HL 
SUHoRVTXDQGRSRVVtYHOGHYHUiVHULQIRUPDWL]DGR

†ž 5HVVDOYDGRVRVFDVRVGH LQH[LJLELOLGDGH GHOLFLWDomRRV †ž 4XDOTXHUFLGDGmRpSDUWHOHJtWLPDSDUDLPSXJQDUSUHoR


FRQWUDWRVSDUDDSUHVWDomRGHVHUYLoRVWpFQLFRVSURILVVLR FRQVWDQWH GR TXDGUR JHUDO HP UD]mR GH
QDLV HVSHFLDOL]DGRV GHYHUmR SUHIHUHQFLDOPHQWH VHU LQFRPSDWLELOLGDGH GHVVH FRP R SUHoR YLJHQWH QR
FHOHEUDGRV PHGLDQWH D UHDOL]DomR GH FRQFXUVR FRP PHUFDGR
HVWLSXODomRSUpYLDGHSUrPLRRXUHPXQHUDomR
†ž 1DVFRPSUDVGHYHUmRVHUREVHUYDGDVDLQGD
†ž $RVVHUYLoRVWpFQLFRSUHYLVWRVQHVWHDUWLJRDSOLFDVHQR
TXHFRXEHURGLVSRVWRQRDUWGHVWD/HL , DHVSHFLILFDomRFRPSOHWDGREHPDVHUDGTXLULGRVHP
LQGLFDomRGHPDUFD
†ž $HPSUHVDGHSUHVWDomRGHVHUYLoRVWpFQLFRVHVSHFLDOL]DGRV
TXHDSUHVHQWHUHODomRGHLQWHJUDQWHVGHVHXFRUSRWpFQLFR ,, D GHILQLomR GDV XQLGDGHV H GDV TXDQWLGDGHVDVHUHP
HPSURFHGLPHQWROLFLWDWyULRRXFRPRHOHPHQWRGHMXVWLIL DGTXLULGDVHPIXQomRGRFRQVXPRHXWLOL]DomRSURYiYH
FDomRGHGLVSHQVDRXLQH[LJLELOLGDGHGHOLFLWDomRILFDUi LV FXMD HVWLPDWLYD VHUi REWLGD VHPSUH TXH SRVVtYHO
REULJDGDDJDUDQWLUTXHRVUHIHULGRVLQWHJUDQWHVUHDOL]HP PHGLDQWHDGHTXDGDVWpFQLFDVTXDQWLWDWLYDVGHHVWLPD
SHVVRDOHGLUHWDPHQWHRVVHUYLoRVREMHWRGRFRQWUDWR omR

6HomR9 ,,, DV FRQGLo}HV GH JXDUGD H DUPD]HQDPHQWR TXHQmR


'DV&RPSUDV SHUPLWDPDGHWHULRUDomRGRPDWHULDO

$UW 1HQKXPDFRPSUDVHUiIHLWDVHPDDGHTXDGDFDUDFWHUL]D †ž 2UHFHELPHQWRGHPDWHULDOGHYDORUVXSHULRUDROLPLWH


HVWDEHOHFLGRQRDUWGHVWD/HLSDUDDPRGDOLGDGHGH
omRGHVHXREMHWRHLQGLFDomRGRVUHFXUVRVRUoDPHQWiULRV
FRQYLWH GHYHUi VHU FRQILDGR D XPD FRPLVVmR GH QR
SDUD VHX SDJDPHQWR VRE SHQD GH QXOLGDGH GR DWR H
PtQLPR WUrV PHPEURV
UHVSRQVDELOLGDGHGHTXHPOKHWLYHUGDGRFDXVD
$UW 6HUi GDGD SXEOLFLGDGH PHQVDOPHQWH HP yUJmRGH
$UW $VFRPSUDVVHPSUHTXHSRVVtYHOGHYHUmR
GLYXOJDomR RILFLDO RX HP TXDGUR GH DYLVRV GH DPSOR
DFHVVR S~EOLFR j UHODomR GH WRGDV DV FRPSUDV IHLWDV
, DWHQGHU DR SULQFtSLR GDSDGURQL]DomRTXHLPSRQKD SHOD $GPLQLVWUDomR GLUHWD RX LQGLUHWD GH PDQHLUD D
FRPSDWLELOLGDGHGHHVSHFLILFDo}HVWpFQLFDVHGHGHVHPSH FODULILFDUDLGHQWLILFDomRGREHPFRPSUDGRVHXSUHoR
QKR REVHUYDGDV TXDQGR IRU R FDVR DV FRQGLo}HV GH XQLWiULRDTXDQWLGDGHDGTXLULGDRQRPHGRYHQGHGRU
PDQXWHQomRDVVLVWrQFLDHJDUDQWLDRIHUHFLGDV HRYDORUWRWDOGDRSHUDomRSRGHQGRVHUDJOXWLQDGDV
SRULWHQVDVFRPSUDVIHLWDVFRPGLVSHQVDHLQH[LJLELOL
,, VHUSURFHVVDGDVDWUDYpVGHVLVWHPDGHUHJLVWURGHSUHoRV GDGHGHOLFLWDomR

,,, VXEPHWHUVH jV FRQGLo}HV GH DTXLVLomR HSDJDPHQWR †~QLFR 2 GLVSRVWR QHVWH DUWLJR QmR VH DSOLFD DRVFDVRVGH
VHPHOKDQWHVjVGRVHWRUSULYDGR GLVSHQVDGHOLFLWDomRSUHYLVWRVQRLQFLVR,;GRDUW

,9 VHUVXEGLYLGLGDVHPWDQWDVSDUFHODVTXDQWDVQHFHVViULDV 6HomR9,


SDUDDSURYHLWDUDVSHFXOLDULGDGHVGRPHUFDGRYLVDQGR 'DV$OLHQDo}HV
HFRQRPLFLGDGH
$UW $DOLHQDomRGHEHQVGD$GPLQLVWUDomR3~EOLFDVXERUGL
9 EDOL]DUVHSHORVSUHoRVSUDWLFDGRVQRkPELWRGRVyUJmRV QDGD j H[LVWrQFLD GH LQWHUHVVH S~EOLFR GHYLGDPHQWH
HHQWLGDGHVGD$GPLQLVWUDomR3~EOLFD MXVWLILFDGRVHUiSUHFHGLGDGHDYDOLDomRHREHGHFHUijV
VHJXLQWHVQRUPDV
†ž 2UHJLVWURGHSUHoRVVHUiSUHFHGLGRGHDPSODSHVTXLVDGH
PHUFDGR , TXDQGRLPyYHLVGHSHQGHUiGHDXWRUL]DomROHJLVODWLYD
SDUD yUJmRV GD DGPLQLVWUDomR GLUHWD H HQWLGDGHV
†ž 2VSUHoRVUHJLVWUDGRVVHUmRSXEOLFDGRVWULPHVWUDOPHQWH DXWiUTXLFDVHIXQGDFLRQDLVHSDUDWRGRVLQFOXVLYHDV
SDUDRULHQWDomRGD$GPLQLVWUDomRQDLPSUHQVDRILFLDO HQWLGDGHVSDUDHVWDWDLVGHSHQGHUiGHDYDOLDomRSUpYLD
HGHOLFLWDomRQDPRGDOLGDGHGHFRQFRUUrQFLDGLVSHQVD
†ž 2VLVWHPDGHUHJLVWURGHSUHoRVVHUiUHJXODPHQWDGRSRU GDHVWDQRVVHJXLQWHVFDVRV
GHFUHWRDWHQGLGDVDVSHFXOLDULGDGHVUHJLRQDLVREVHUYD
GDVDVVHJXLQWHVFRQGLo}HV D GDomRHPSDJDPHQWR
&kPDUD/HJLVODWLYDGR
  'LVWULWR)HGHUDO 'LU$GPLQLVWUDWLYR 

E GRDomRSHUPLWLGDH[FOXVLYDPHQWHSDUDRXWURyUJmRRX †ž $ $GPLQLVWUDomR WDPEpP SRGHUi FRQFHGHUWtWXORGH


HQWLGDGHGDDGPLQLVWUDomRS~EOLFDGHTXDOTXHUHVIHUD SURSULHGDGH RX GH GLUHLWR UHDO GH XVR GH LPyYHLV
GHJRYHUQRUHVVDOYDGRRGLVSRVWRQDVDOtQHDVIKH GLVSHQVDGDOLFLWDomRTXDQGRRXVRGHVWLQDUVH
L 2EVUHGDomRGDGDSHOD/HLQž
, DRXWURyUJmRRXHQWLGDGHGD$GPLQLVWUDomR3~EOLFD
F SHUPXWDSRURXWURLPyYHOTXHDWHQGDDRVUHTXLVLWRV TXDOTXHUTXHVHMDDORFDOL]DomRGRLPyYHO
FRQVWDQWHVGRLQFLVR;GRDUWGHVWD/HL
,, DSHVVRDQDWXUDOTXHQRVWHUPRVGHOHLUHJXODPHQWR
G LQYHVWLGXUD RXDWRQRUPDWLYRGRyUJmRFRPSHWHQWHKDMDLPSOHPHQ
WDGRRVUHTXLVLWRVPtQLPRVGHFXOWXUDRFXSDomRPDQVD
H YHQGD D RXWUR yUJmR RX HQWLGDGH GD $GPLQLVWUDomR HSDFtILFDHH[SORUDomRGLUHWDVREUHiUHDUXUDOREVHUYD
3~EOLFDGHTXDOTXHUHVIHUDGHJRYHUQR GRROLPLWHGHTXHWUDWDR†RGRDUW R GD/HLQR
GHGHMXQKRGH 5HGDomRGDGDSHOD/HLQž
I  DOLHQDomRJUDWXLWDRXRQHURVDDIRUDPHQWRFRQFHVVmR 
GHGLUHLWRUHDOGHXVRORFDomRRXSHUPLVVmRGHXVRGH
EHQV LPyYHLV UHVLGHQFLDLV FRQVWUXtGRV GHVWLQDGRV RX †ž$ $VKLSyWHVHVGRLQFLVR,,GR†RILFDPGLVSHQVDGDVGH
HIHWLYDPHQWH XWLOL]DGRV QR kPELWR GH SURJUDPDV DXWRUL]DomROHJLVODWLYDSRUpPVXEPHWHPVHDRVVHJXLQ
KDELWDFLRQDLVRXGHUHJXODUL]DomRIXQGLiULDGHLQWHUHVVH WHVFRQGLFLRQDPHQWRV
2EVUHGDomRGDGDSHOD/HLQž
VRFLDO GHVHQYROYLGRV SRU yUJmRV RX HQWLGDGHV GD
DGPLQLVWUDomRS~EOLFD 2EVFRPUHGDomRGDGDSHOD/HLQž , DSOLFDomRH[FOXVLYDPHQWHjViUHDVHPTXHDGHWHQomR
SRUSDUWLFXODUVHMDFRPSURYDGDPHQWHDQWHULRUDžGH
J SURFHGLPHQWRVGHOHJLWLPDomRGHSRVVHGHTXHWUDWDR GH]HPEURGH
DUWGD/HLQRGHGHGH]HPEURGH
PHGLDQWHLQLFLDWLYDHGHOLEHUDomRGRVyUJmRVGD$GPL ,, VXEPLVVmR DRV GHPDLV UHTXLVLWRV HLPSHGLPHQWRVGR
QLVWUDomR3~EOLFDHPFXMDFRPSHWrQFLDOHJDOLQFOXDVH UHJLPH OHJDO H DGPLQLVWUDWLYR GD GHVWLQDomR H GD
WDODWULEXLomR 2EVFRPUHGDomRGDGDSHOD/HLQž
UHJXODUL]DomRIXQGLiULDGHWHUUDVS~EOLFDV
K DOLHQDomRJUDWXLWDRXRQHURVDDIRUDPHQWRFRQFHVVmR
GHGLUHLWRUHDOGHXVRORFDomRRXSHUPLVVmRGHXVRGH ,,, YHGDomR GH FRQFHVV}HV SDUD KLSyWHVHV GHH[SORUDomR
EHQV LPyYHLV GH XVR FRPHUFLDO GH kPELWR ORFDO FRP QmRFRQWHPSODGDVQDOHLDJUiULDQDVOHLVGHGHVWLQDomR
iUHD GH DWp  Pò GX]HQWRV H FLQTHQWD PHWURV GHWHUUDVS~EOLFDVRXQDVQRUPDVOHJDLVRXDGPLQLVWUD
TXDGUDGRV  H LQVHULGRV QR kPELWR GH SURJUDPDV GH WLYDVGH]RQHDPHQWRHFROyJLFRHFRQ{PLFRH
UHJXODUL]DomRIXQGLiULDGHLQWHUHVVHVRFLDOGHVHQYROYL
GRVSRUyUJmRVRXHQWLGDGHVGDDGPLQLVWUDomRS~EOLFD ,9 SUHYLVmRGHUHVFLVmRDXWRPiWLFDGDFRQFHVVmRGLVSHQ
2EVDOtQHDDFUHVFHQWDGDSHOD/HLQžGH VDGDQRWLILFDomRHPFDVRGHGHFODUDomRGHXWLOLGDGH
L DOLHQDomRHFRQFHVVmRGHGLUHLWRUHDOGHXVRJUDWXLWD RXQHFHVVLGDGHS~EOLFDRXLQWHUHVVHVRFLDO
RX RQHURVD GH WHUUDV S~EOLFDV UXUDLV GD 8QLmR H GR †ž% $KLSyWHVHGRLQFLVR,,GR†žGHVWHDUWLJR
,QFUDRQGHLQFLGDPRFXSDo}HVDWpROLPLWHGHTXHWUDWD
R†RGRDUWRGD/HLQRGHGHMXQKRGH , VyVHDSOLFDDLPyYHOVLWXDGRHP]RQDUXUDOQmRVXMHLWR
SDUDILQVGHUHJXODUL]DomRIXQGLiULDDWHQGLGRV D YHGDomR LPSHGLPHQWR RX LQFRQYHQLHQWH D VXD
RVUHTXLVLWRVOHJDLVH 5HGDomRGDGDSHOD/HLQž H[SORUDomRPHGLDQWHDWLYLGDGHVDJURSHFXiULDV
,, TXDQGR PyYHLV GHSHQGHUi GH DYDOLDomR SUpYLDHGH
OLFLWDomRGLVSHQVDGDHVWDQRVVHJXLQWHVFDVRV ,, ILFD OLPLWDGD D iUHDV GH DWp TXLQ]HPyGXORVILVFDLV
GHVGH TXH QmR H[FHGD PLO H TXLQKHQWRV KHFWDUHV
D GRDomRSHUPLWLGDH[FOXVLYDPHQWHSDUDILQVHXVRGH YHGDGDDGLVSHQVDGHOLFLWDomRSDUDiUHDVVXSHULRUHVD
LQWHUHVVHVRFLDODSyVDYDOLDomRGHVXDRSRUWXQLGDGHH HVVHOLPLWH
2EVUHGDomRGDGDSHOD/HLQž
FRQYHQLrQFLDVyFLRHFRQ{PLFDUHODWLYDPHQWHjHVFROKD
GHRXWUDIRUPDGHDOLHQDomR ,,, SRGHVHUFXPXODGDFRPRTXDQWLWDWLYRGHiUHDGHFRU
UHQWHGDILJXUDSUHYLVWDQDDOtQHDJGRLQFLVR,GRFDSXW
E SHUPXWD SHUPLWLGD H[FOXVLYDPHQWH HQWUH yUJmRV RX GHVWH DUWLJR DWp R OLPLWH SUHYLVWR QR LQFLVR ,, GHVWH
HQWLGDGHVGD$GPLQLVWUDomR3~EOLFD SDUiJUDIR
2EV††ž$H%DFUHVFLGRVSHOD/HLQž

F YHQGDGHDo}HVTXHSRGHUmRVHUQHJRFLDGDVHPEROVD †ž (QWHQGHVHSRULQYHVWLGXUDSDUDRVILQVGHVWD/HL


REVHUYDGDDOHJLVODomRHVSHFtILFD
, DDOLHQDomRDRVSURSULHWiULRVGHLPyYHLVOLQGHLURVGH
G YHQGDGHWtWXORVQDIRUPDGDOHJLVODomRSHUWLQHQWH iUHDUHPDQHVFHQWHRXUHVXOWDQWHGHREUDS~EOLFDiUHD
HVWD TXH VH WRUQDU LQDSURYHLWiYHO LVRODGDPHQWH SRU
H YHQGD GH EHQV SURGX]LGRV RX FRPHUFLDOL]DGRV SRU SUHoRQXQFDLQIHULRUDRGDDYDOLDomRHGHVGHTXHHVVH
yUJmRV RX HQWLGDGHV GD $GPLQLVWUDomR 3~EOLFD HP QmRXOWUDSDVVHD FLQTHQWDSRUFHQWR GRYDORU
YLUWXGHGHVXDVILQDOLGDGHV FRQVWDQWHGDDOtQHDDGRLQFLVR,,GRDUWGHVWD/HL
I  YHQGDGHPDWHULDLVHHTXLSDPHQWRVSDUDRXWURVyUJmRV ,, DDOLHQDomRDRVOHJtWLPRVSRVVXLGRUHVGLUHWRVRXQD
RXHQWLGDGHVGD$GPLQLVWUDomR3~EOLFDVHPXWLOL]DomR IDOWD GHVWHV DR 3RGHU 3~EOLFR GH LPyYHLV SDUD ILQV
SUHYLVtYHOSRUTXHPGHOHVGLVS}H UHVLGHQFLDLVFRQVWUXtGRVHPQ~FOHRVXUEDQRVDQH[RVD
XVLQDVKLGUHOpWULFDVGHVGHTXHFRQVLGHUDGRVGLVSHQVi
†ž 2VLPyYHLVGRDGRVFRPEDVHQDDOtQHDEGRLQFLVR, YHLVQDIDVHGHRSHUDomRGHVVDVXQLGDGHVHQmRLQWH
GHVWHDUWLJRFHVVDGDVDVUD]}HVTXHMXVWLILFDUHPDVXD JUHPDFDWHJRULDGHEHQVUHYHUVtYHLVDRILQDOGDFRQ
GRDomR UHYHUWHUmR DR SDWULP{QLR GD SHVVRD MXUtGLFD FHVVmR
GRDGRUDYHGDGDDVXDDOLHQDomRSHOREHQHILFLiULR 2EV†žWHPVXDUHGDomRGDGDSHOD/HLQž
 'LU$GPLQLVWUDWLYR
 &kPDUD/HJLVODWLYDGR'LVWULWR)HGHUDO
†ž $ GRDomR FRP HQFDUJR SRGHUi VHU OLFLWDGD HGHVHX ,,, HP MRUQDO GLiULR GH JUDQGH FLUFXODomR QR (VWDGRH
LQVWUXPHQWRFRQVWDUmRREULJDWRULDPHQWHRVHQFDUJRV WDPEpPVHKRXYHUHPMRUQDOGHFLUFXODomRQR0XQLFt
RSUD]RGHVHXFXPSULPHQWRHFOiXVXODGHUHYHUVmR SLRRXQDUHJLmRRQGHVHUiUHDOL]DGDDREUDSUHVWDGR
VRE SHQD GH QXOLGDGH GR DWR VHQGR GLVSHQVDGD D R VHUYLoR IRUQHFLGR DOLHQDGR RX DOXJDGR R EHP
OLFLWDomR QR FDVR GH LQWHUHVVH S~EOLFR GHYLGDPHQWH SRGHQGRDLQGDD$GPLQLVWUDomRFRQIRUPHRYXOWRGD
MXVWLILFDGR OLFLWDomRXWLOL]DUVHGHRXWURVPHLRVGHGLYXOJDomRSDUD
2EVUHGDomRGDSHOD/HL
DPSOLDUDiUHDGHFRPSHWLomR
2EVFRPUHGDomRGDGDSHOD/HL
†ž 1D KLSyWHVH GR SDUiJUDIR DQWHULRU FDVRRGRQDWiULR
QHFHVVLWH RIHUHFHU R LPyYHO HP JDUDQWLD GH ILQDQFLD †ž 2DYLVRSXEOLFDGRFRQWHUiDLQGLFDomRGRORFDOHPTXH
PHQWR D FOiXVXOD GH UHYHUVmR H GHPDLV REULJDo}HV RVLQWHUHVVDGRVSRGHUmROHUHREWHURWH[WRLQWHJUDOGR
VHUmRJDUDQWLGDVSRUKLSRWHFDHPžJUDXHPIDYRUGR HGLWDOHWRGDVDVLQIRUPDo}HVVREUHDOLFLWDomR
GRDGRU
†ž 2SUD]RPtQLPRDWpRUHFHELPHQWRGDVSURSRVWDVRXGD
†ž 3DUD D YHQGD GH EHQV PyYHLV DYDOLDGRVLVRODGDRX UHDOL]DomRGRHYHQWRVHUi
JOREDOPHQWHHPTXDQWLDQmRVXSHULRUDROLPLWHSUHYLV
WRQRDUWLQF,,DOtQHDEGHVWD/HLD$GPLQLVWUD , TXDUHQWDHFLQFRGLDVSDUD
omRSRGHUiSHUPLWLUROHLOmR
2EV††žHžIRUDPLQFOXtGRVSHOD/HL
D FRQFXUVR
†ž YHWDGR  2EVDFUHVFLGRSHOD/HLQž
E FRQFRUUrQFLDTXDQGRRFRQWUDWRDVHUFHOHEUDGR
$UW 1DFRQFRUUrQFLDSDUDDYHQGDGHEHQVLPyYHLVDIDVH FRQWHPSODURUHJLPHGHHPSUHLWDGD LQWHJUDORX
GHKDELOLWDomROLPLWDUVHijFRPSURYDomRGRUHFROKL TXDQGRDOLFLWDomRIRUGRWLSRPHOKRUWpFQLFDRX
PHQWR GH TXDQWLD FRUUHVSRQGHQWH D  FLQFR SRU WpFQLFDHSUHoR 2EVFRPUHGDomRGDGDSHOD/HL

FHQWR GDDYDOLDomR
,, WULQWDGLDVSDUD
†~QLFR 3DUD D YHQGD GH EHQV PyYHLV DYDOLDGRVLVRODGDRX
JOREDOPHQWH HP TXDQWLD QmR VXSHULRU DR OLPLWH D FRQFRUUrQFLD QRV FDVRV QmR HVSHFLILFDGRV QD
SUHYLVWR QR DUW LQFLVR ,, DOtQHD E GHVWD /HL D DOtQHDEGRLQFLVRDQWHULRU
$GPLQLVWUDomRSRGHUiSHUPLWLUROHLOmR
E WRPDGDGHSUHoRVTXDQGRDOLFLWDomRIRUGRWLSR
$UW 2VEHQVLPyYHLVGD$GPLQLVWUDomR3~EOLFDFXMDDTXLVL PHOKRUWpFQLFDRXWpFQLFDHSUHoR2EVFRPUHGDomRGDGD
SHOD/HL
omR KDMD GHULYDGR GH SURFHGLPHQWRV MXGLFLDLV RX GH
GDomRHPSDJDPHQWRSRGHUmRVHUDOLHQDGRVSRUDWRGD ,,, TXLQ]H GLDV SDUD WRPDGD GH SUHoRV QRV FDVRVQmR
DXWRULGDGHFRPSHWHQWHREVHUYDGDVDVVHJXLQWHVUHJUDV HVSHFLILFDGRV QD DOtQHD E GR LQFLVR DQWHULRU RX
OHLOmR 2EVFRPUHGDomRGDGDSHOD/HL

, DYDOLDomRGRVEHQVDOLHQiYHLV ,9 FLQFRGLDV~WHLVSDUDFRQYLWH2EVUHGDomRGDGDSHOD/HL


,, FRPSURYDomRGDQHFHVVLGDGHRXXWLOLGDGHGDDOLHQDomR 9 5HGDomRVXSULPLGDSHOD/HL

,,, DGRomRGRSURFHGLPHQWROLFLWDWyULRVREDPRGDOLGDGH †ž 2VSUD]RVHVWDEHOHFLGRVQRSDUiJUDIRDQWHULRUVHUmR


GHFRQFRUUrQFLDRXOHLOmR 2EVFRPUHGDomRGDGDSHOD/HL FRQWDGRV D SDUWLU GD ~OWLPD SXEOLFDomR GR HGLWDO
UHVXPLGRRXGDH[SHGLomRGRFRQYLWHRXGDHIHWLYD
&DStWXOR,, GLVSRQLELOLGDGHGRHGLWDORXGRFRQYLWHHUHVSHFWLYRV
'D/LFLWDomR DQH[RVSUHYDOHFHQGRDGDWDTXHRFRUUHUPDLVWDUGH
2EVFRPUHGDomRGDGDSHOD/HL

6HomR, †ž 4XDOTXHUPRGLILFDomRQRHGLWDOH[LJHGLYXOJDomRSHOD


'DV0RGDOLGDGHV/LPLWHVH'LVSHQVD PHVPDIRUPDTXHVHGHXRWH[WRRULJLQDOUHDEULQGRVH
R SUD]R LQLFLDOPHQWH HVWDEHOHFLGR H[FHWR TXDQGR
$UW $VOLFLWDo}HVVHUmRHIHWXDGDVQRORFDORQGHVHVLWXDUD LQTHVWLRQDYHOPHQWHDDOWHUDomRQmRDIHWDUDIRUPX
UHSDUWLomRLQWHUHVVDGDVDOYRSRUPRWLYRGDLQWHUHVVH ODomRGDVSURSRVWDV
S~EOLFRGHYLGDPHQWHMXVWLILFDGR
$UW 6mRPRGDOLGDGHVGHOLFLWDomR
†~QLFR 2GLVSRVWRQHVWHDUWLJRQmRLPSHGLUiDKDELOLWDomRGH
LQWHUHVVDGRVUHVLGHQWHVRXVHGLDGRVHPRXWURVORFDLV , FRQFRUUrQFLD
,, WRPDGDGHSUHoRV
$UW 2VDYLVRVFRQWHQGRRVUHVXPRVGRVHGLWDLVGDVFRQFRU ,,, FRQYLWH
UrQFLDVGDVWRPDGDVGHSUHoRVGRVFRQFXUVRVH GRV ,9 FRQFXUVR
OHLO}HV HPERUD UHDOL]DGRV QR ORFDO GD UHSDUWLomR 9 OHLOmR
LQWHUHVVDGDGHYHUmRVHUSXEOLFDGRVFRPDQWHFHGrQFLD
QRPtQLPRSRUXPDYH] 2EVFRPUHGDomRGDGDSHOD/HL †ž &RQFRUUrQFLD p D PRGDOLGDGH GHOLFLWDomRHQWUH
, QR'LiULR2ILFLDOGD8QLmRTXDQGRVHWUDWDUGHOLFLWDomR LQWHUHVVDGRVTXHQDIDVHLQLFLDOGHKDELOLWDomRSUH
IHLWD SRU yUJmR GD $GPLQLVWUDomR 3~EOLFD )HGHUDO H OLPLQDUFRPSURYHPSRVVXLURVUHTXLVLWRVPtQLPRVGH
DLQGDTXDQGRVHWUDWDUGHREUDVILQDQFLDGDVSDUFLDORX TXDOLILFDomRH[LJLGRVQRHGLWDSDUDH[HFXomRGHVHX
WRWDOPHQWH FRP UHFXUVRV IHGHUDLV RX JDUDQWLGRV SRU REMHWR
LQVWLWXLo}HVIHGHUDLV2EVFRPUHGDomRGDGDSHOD/HL
†ž 7RPDGDGHSUHoRVpDPRGDOLGDGHGHOLFLWDomRHQWUH
,, QR 'LiULR 2ILFLDO GR (VWDGR RX GR 'LVWULWR)HGHUDO LQWHUHVVDGRVGHYLGDPHQWHFDGDVWUDGRVRXTXHDWHQGH
TXDQGRVHWUDWDUUHVSHFWLYDPHQWHGHOLFLWDomRIHLWDSRU UHPDWRGDVDVFRQGLo}HVH[LJLGDVSDUDFDGDVWUDPHQWR
yUJmRRXHQWLGDGHGD$GPLQLVWUDomR3~EOLFD(VWDGXDO DWpRWHUFHLURGLDDQWHULRUjGDWDGRUHFHELPHQWRGDV
RX0XQLFLSDORXGR'LVWULWR)HGHUDO2EVFRPUHGDomRGDGDSHOD SURSRVWDVREVHUYDGDDQHFHVViULDTXDOLILFDomR
/HL
&kPDUD/HJLVODWLYDGR
  'LVWULWR)HGHUDO 'LU$GPLQLVWUDWLYR 

†ž &RQYLWHpDPRGDOLGDGHGHOLFLWDomRHQWUHLQWHUHVVDGRV F FRQFRUUrQFLDDFLPDGH5 VHLVFHQWRVH


GRUDPRSHUWLQHQWHDRVHXREMHWRFDGDVWUDGRVRXQmR FLQTHQWDPLOUHDLV 2EVFRPUHGDomRGDGDSHOD/HLQž

HVFROKLGRV H FRQYLGDGRV HP Q~PHUR PtQLPR GH 


†ž $VREUDVVHUYLoRVHFRPSUDVHIHWXDGDVSHOD$GPLQLV
WUrV SHODXQLGDGHDGPLQLVWUDWLYDDTXDOIL[DUiHP
WUDomRVHUmRGLYLGLGDVHPWDQWDVSDUFHODVTXDQWDVVH
ORFDODSURSULDGRFySLDGRLQVWUXPHQWRFRQYRFDWyULR
FRPSURYDUHP WpFQLFD H HFRQRPLFDPHQWH YLiYHLV
HRHVWHQGHUiDRVGHPDLVFDGDVWUDGRVQDFRUUHVSRQ
GHQWHHVSHFLDOLGDGHTXHPDQLIHVWDUHPVHXLQWHUHVVH SURFHGHQGRVH j OLFLWDomR FRP YLVWDV DR PHOKRU
FRPDQWHFHGrQFLDGHDWp YLQWHHTXDWUR KRUDVGD DSURYHLWDPHQWRGRVUHFXUVRVGLVSRQtYHLVQRPHUFDGR
DSUHVHQWDomRGDVSURSRVWDV H j DPSOLDomR GD FRPSHWLWLYLGDGH VHP SHUGD GD
HFRQRPLDGHHVFDOD
†ž &RQFXUVRpDPRGDOLGDGHGHOLFLWDomRHQWUHTXDLVTXHU
LQWHUHVVDGRVSDUDHVFROKDGHWUDEDOKRWpFQLFRFLHQWtIL †ž 1D H[HFXomR GH REUDV H VHUYLoRV H QDVFRPSUDVGH
FRRXDUWtVWLFRPHGLDQWHDLQVWLWXLomRGHSUrPLRVRX EHQVSDUFHODGDVQRVWHUPRVGRSDUiJUDIRDQWHULRUD
UHPXQHUDomR DRV YHQFHGRUHV FRQIRUPH FULWpULRV GH FDGDHWDSDRXFRQMXQWRGHHWDSDVGDREUDVHUYLoRRX
HGLWDOSXEOLFDGRQDLPSUHQVDRILFLDOFRPDQWHFHGrQFLD FRPSUDKiGHFRUUHVSRQGHUOLFLWDomRGLVWLQWDSUHVHU
PtQLPDGH TXDUHQWDHFLQFR GLDV YDGD D PRGDOLGDGH SHUWLQHQWH SDUD D H[HFXomR GR
REMHWRHPOLFLWDomR
†ž /HLOmR p D PRGDOLGDGH GH OLFLWDomRHQWUHTXDLVTXHU
LQWHUHVVDGRVSDUDDYHQGDGHEHQVPyYHLVLQVHUYtYHLV †ž $ FRQFRUUrQFLDpDPRGDOLGDGHGHOLFLWDomRFDEtYHO
SDUD D $GPLQLVWUDomR RX GH SURGXWRV OHJDOPHQWH TXDOTXHUTXHVHMDRYDORUGRREMHWRWDQWRQDFRPSUD
DSUHHQGLGRVRXSHQKRUDGRVRXSDUDDDOLHQDomRGH RXDOLHQDomRGHEHQVLPyYHLVUHVVDOYDGRRGLVSRVWR
EHQV LPyYHLV SUHYLVWD QR DUW D TXHP RIHUHFHU R QRDUWFRPRQDVFRQFHVV}HVGHGLUHLWRUHDOGHXVR
PDLRUODQFHLJXDORXVXSHULRUDRYDORUGDDYDOLDomR H QDV OLFLWDo}HV LQWHUQDFLRQDLV DGPLWLQGRVH QHVWH
~OWLPR FDVR REVHUYDGRV RV OLPLWHV GHVWH DUWLJR D
†ž 1DKLSyWHVHGR†žGHVWHDUWLJRH[LVWLQGRQDSUDoD WRPDGD GH SUHoRV TXDQGR R yUJmR RX HQWLGDGH
PDLVGH WUrV SRVVtYHLVLQWHUHVVDGRVDFDGDQRYR GLVSXVHUGHFDGDVWURLQWHUQDFLRQDOGHIRUQHFHGRUHVRX
FRQYLWHUHDOL]DGRSDUDREMHWRLGrQWLFRRXDVVHPHOKDGR RFRQYLWHTXDQGRQmRKRXYHUIRUQHFHGRUGREHPRX
pREULJDWyULRRFRQYLWHDQRPtQLPRPDLVXPLQWHUHV VHUYLoRQR3DtV
VDGR HQTXDQWR H[LVWLUHP FDGDVWURV QmR FRQYLGDGRV
QDV~OWLPDVOLFLWDo}HV †ž QRV FDVRV HP TXH FRXEHU FRQYLWHD$GPLQLVWUDomR
2EV††žHžFRPUHGDomRGDGDSHOD/HL
SRGHUi XWLOL]DU D WRPDGD GH SUHoRV H HP TXDOTXHU
†ž 4XDQGR SRU OLPLWDo}HV GR PHUFDGRRXPDQLIHVWR FDVRDFRQFRUUrQFLD
GHVLQWHUHVVHGRVFRQYLGDGRVIRULPSRVVtYHODREWHQ
omRGRQ~PHURPtQLPRGHOLFLWDQWHVH[LJLGRVQR†ž †ž e YHGDGD D XWLOL]DomR GD PRGDOLGDGHFRQYLWHRX
GHVWHDUWLJRHVVDVFLUFXQVWkQFLDVGHYHUmRVHUGHYLGD WRPDGDGHSUHoRVFRQIRUPHRFDVRSDUDSDUFHODVGH
PHQWHMXVWLILFDGDVQRSURFHVVRVRESHQDGHUHSHWLomR XPDPHVPDREUDRXVHUYLoRRXDLQGDSDUDREUDVH
GRFRQYLWH VHUYLoRV GD PHVPD QDWXUH]D H QR PHVPR ORFDO TXH
SRVVDPVHUUHDOL]DGDVFRQMXQWDHFRQFRPLWDQWHPHQWH
†ž eYHGDGDDFULDomRGHRXWUDVPRGDOLGDGHVGHOLFLWDomR
VHPSUHTXHRVRPDWyULRGHVHXVYDORUHVFDUDFWHUL]DU
RXDFRPELQDomRGDVUHIHULGDVQHVWHDUWLJR
RFDVRGHWRPDGDGHSUHoRVRXFRQFRUUrQFLDUHVSHFWL
YDPHQWH QRV WHUPRV GHVWH DUWLJR H[FHWR SDUD DV
†ž 1D KLSyWHVH GR † ž GHVWH DUWLJRD$GPLQLVWUDomR
VRPHQWHSRGHUiH[LJLUGROLFLWDQWHQmRFDGDVWUDGRRV SDUFHODVGHQDWXUH]DHVSHFtILFDTXHSRVVDPVHUH[HFX
GRFXPHQWRVSUHYLVWRVQRVDUWVDTXHFRPSUR WDGDV SRU SHVVRDV RX HPSUHVDV GH HVSHFLDOLGDGH
YHPKDELOLWDomRFRPSDWtYHOFRPRREMHWRGDOLFLWDomR GLYHUVDGDTXHODGRH[HFXWRUGDREUDRXVHUYLoR
QRVWHUPRVGRHGLWDO 2EV††žžžžHž³FRPUHGDomRGDGDSHOD/HL
2EV(VWH†IRLLQFOXtGRSHOD/HL
†ž $VRUJDQL]Do}HVLQGXVWULDLVGD$GPLQLVWUDomR)HGHUDO
$UW $V PRGDOLGDGHV GH OLFLWDomR D TXH VH UHIHUHPRV GLUHWDHPIDFHGHVXDVSHFXOLDULGDGHVREHGHFHUmRDRV
LQFLVRV,D,,,GRDUWLJRDQWHULRUVHUmRGHWHUPLQDGDV
OLPLWHVHVWDEHOHFLGRVQRLQFLVR,GHVWHDUWLJRWDPEpP
HP IXQomR GRV VHJXLQWHV OLPLWHV WHQGR HP YLVWD R
SDUDDVVXDVFRPSUDVHVHUYLoRVHPJHUDOGHVGHTXH
YDORUHVWLPDGRGDFRQWUDWDomR
SDUDDDTXLVLomRGHPDWHULDLVDSOLFDGRVH[FOXVLYDPHQ
WH QD PDQXWHQomR UHSDUR RX IDEULFDomR GH PHLRV
, SDUDREUDVHVHUYLoRVGHHQJHQKDULD
RSHUDFLRQDLVEpOLFRVSHUWHQFHQWHVj8QLmR2EV(VWH† IRL
LQFOXtGRSHOD/HL
D FRQYLWH  DWp 5  FHQWR H FLQTHQWD PLO
UHDLV
†ž 1DFRPSUDGHEHQVGHQDWXUH]DGLYLVtYHOHGHVGHTXH
QmR KDMD SUHMXt]R SDUD R FRQMXQWR RX FRPSOH[R p
E WRPDGDGHSUHoRVDWp5 XPPLOKmR
HTXLQKHQWRVPLOUHDLV SHUPLWLGDDFRWDomRGHTXDQWLGDGHLQIHULRUjGHPDQ
GDGDQDOLFLWDomRFRPYLVWDVDDPSOLDomRGDFRPSHWL
F FRQFRUUrQFLDDFLPDGH5 XPPLOKmR WLYLGDGHSRGHQGRRHGLWDOIL[DUTXDQWLWDWLYRPtQLPR
HTXLQKHQWRVPLOUHDLV 2EVFRPUHGDomRGDGDSHOD/HLQž SDUDSUHVHUYDUDHFRQRPLDGHHVFDOD2EV,QFOXtGRSHOD/HLQž

,, SDUD FRPSUDV H VHUYLoRV QmR UHIHULGRV QRLQFLVR
DQWHULRU †ž 1RFDVRGHFRQVyUFLRVS~EOLFRVDSOLFDUVHiRGREUR
GRVYDORUHVPHQFLRQDGRVQRFDSXWGHVWHDUWLJRTXDQ
D FRQYLWHDWp5 RLWHQWDPLOUHDLV GRIRUPDGRSRUDWp WUrV HQWHVGD)HGHUDomRHR
WULSORTXDQGRIRUPDGRSRUPDLRUQ~PHUR2EV,QFOXtGRSHOD
/HLQžGH
E WRPDGDGHSUHoRVDWp5 VHLVFHQWRVH
FLQTHQWDPLOUHDLV $UW eGLVSHQViYHODOLFLWDomR
 'LU$GPLQLVWUDWLYR
 &kPDUD/HJLVODWLYDGR'LVWULWR)HGHUDO
, SDUDREUDVHVHUYLoRVGHHQJHQKDULDGHYDORUDWp ;,, QDV FRPSUDV GH KRUWLIUXWLJUDQMHLURV SmR HRXWURV
GH] SRU FHQWR  GR OLPLWH SUHYLVWR QD DOtQHD D GR JrQHURV SHUHFtYHLV QR WHPSR QHFHVViULR SDUD D
LQFLVR,GRDUWLJRDQWHULRUGHVGHTXHQmRVHUHILUDPD UHDOL]DomRGRVSURFHVVRVOLFLWDWyULRVFRUUHVSRQGHQWHV
SDUFHODVGHXPDPHVPDREUDRXVHUYLoRRXDLQGDSDUD UHDOL]DGDVGLUHWDPHQWHFRPEDVHQRSUHoRGRGLD
REUDVHVHUYLoRVGDPHVPDQDWXUH]DHQRPHVPRORFDO 2EVFRPUHGDomRGDGDSHOD/HL

TXHSRVVDPVHUUHDOL]DGRVFRQMXQWDHFRQFRPLWDQWH ;,,, QD FRQWUDWDomR GH LQVWLWXLomR EUDVLOHLUDLQFXPELGD


PHQWH 2EVFRPUHGDomRGDGDSHOD/HLQž
UHJLPHQWDO RX HVWDWXWDULDPHQWH GD SHVTXLVD GR
,, SDUDRXWURVVHUYLoRVHFRPSUDVGHYDORUDWp GH] HQVLQR RX GR GHVHQYROYLPHQWR LQVWLWXFLRQDO RX
SRUFHQWR GROLPLWHSUHYLVWRQDDOtQHDDGRLQFLVR,, LQVWLWXLomR GHGLFDGD j UHFXSHUDomR VRFLDO GR SUHVR
GRDUWLJRDQWHULRUGHVGHTXHQmRVHUHILUDPDSDUFHODV GHVGH TXH D FRQWUDWDGD GHWHQKD LQTXHVWLRQiYHO
GHXPPHVPRVHUYLoRFRPSUDRXDOLHQDomRGHPDLRU UHSXWDomRpWLFRSURILVVLRQDOHQmRWHQKDILQVOXFUDWtFL
YXOWRTXHSRVVDVHUUHDOL]DGDGHXPDVyYH] RV
2EVUHGDomRGDGDSHOD/HL

,,, QRVFDVRVGHJXHUUDRXJUDYHSHUWXUEDomRGDRUGHP ;,9 SDUDDDTXLVLomRGHEHQVRXVHUYLoRVQRVWHUPRVGH


DFRUGR LQWHUQDFLRQDO HVSHFtILFR DSURYDGR SHOR &RQ
,9 QRVFDVRVGHHPHUJrQFLDRXGHFDODPLGDGHS~EOLFD JUHVVR1DFLRQDOTXDQGRDVFRQGLo}HVRIHUWDGDVIRUHP
TXDQGRFDUDFWHUL]DGDDXUJrQFLDGHDWHQGLPHQWRGH PDQLIHVWDPHQWHYDQWDMRVDVSDUDR3RGHU3~EOLFR
VLWXDomRTXHSRVVDRFDVLRQDUSUHMXt]RRXFRPSURPH 2EVFRPUHGDomRGDGDSHOD/HL

WHU D VHJXUDQoD GH SHVVRDV REUDV VHUYLoRV ;9 SDUD D DTXLVLomR RX UHVWDXUDomR GH REUDV GH DUWHH
HTXLSDPHQWRVHRXWURVEHQVS~EOLFRVRXSDUWLFXODUHV
REMHWRVKLVWyULFRVGHDXWHQWLFLGDGHFHUWLILFDGDGHVGH
H VRPHQWH RV EHQV QHFHVViULRV DR DWHQGLPHQWR GD
TXHFRPSDWtYHLVRXLQHUHQWHVjVILQDOLGDGHVGRyUJmR
VLWXDomRHPHUJHQFLDORXFDODPLWRVDHSDUDDVSDUFHODV
RXHQWLGDGH
GH REUDV H VHUYLoRV TXH SRVVDP VHU FRQFOXtGRV QR
;9, SDUDDLPSUHVVmRGRVGLiULRVRILFLDLVGHIRUPXOiULRV
SUD]RPi[LPRGHGLDVFRQVHFXWLYRVHLQLQWHUUXS
SDGURQL]DGRVGHXVRGD$GPLQLVWUDomRHGHHGLo}HV
WRVFRQWDGRVGDRFRUUrQFLDGDHPHUJrQFLDRXFDODPL
GDGHYHGDGDDSURUURJDomRGRVUHVSHFWLYRVFRQWUDWRV WpFQLFDV RILFLDLV EHP FRPR SDUD D SUHVWDomR GH
VHUYLoRV GH LQIRUPiWLFD D SHVVRD MXUtGLFD GH GLUHLWR
9 TXDQGRQmRDFXGLUHP LQWHUHVVDGRVjOLFLWDomRDQWH S~EOLFRLQWHUQRSRUyUJmRVRXHQWLGDGHVTXHLQWHJUDP
ULRUHHVWDMXVWLILFDGDPHQWHQmRSXGHUVHUUHSHWLGD D$GPLQLVWUDomR3~EOLFDFULDGRVSDUDHVVHILPHVSHFt
VHPSUHMXt]RSDUDD$GPLQLVWUDomRPDQWLGDVQHVWH ILFR
2EV(VWHLQFLVR;9,IRLLQFOXtGRSHOD/HL
FDVRWRGDVDVFRQGLo}HVSUHHVWDEHOHFLGDV
;9,, SDUDDDTXLVLomRGHFRPSRQHQWHVRXSHoDVGHRULJHP
9, TXDQGR D 8QLmR WLYHU TXH LQWHUYLU QRGRPtQLRHFR QDFLRQDORXHVWUDQJHLUDQHFHVViULRVjPDQXWHQomRGH
Q{PLFR SDUD UHJXODU SUHoRV RX QRUPDOL]DU R DEDV HTXLSDPHQWRVGXUDQWHRSHUtRGRGHJDUDQWLDWpFQLFD
WHFLPHQWR MXQWR DR IRUQHFHGRU RULJLQDO GHVVHV HTXLSDPHQWRV
TXDQGRWDOFRQGLomRGHH[FOXVLYLGDGHIRULQGLVSHQVi
9,, TXDQGRDVSURSRVWDVDSUHVHQWDGDVFRQVLJQDUHPSUHoRV YHOSDUDDYLJrQFLDGDJDUDQWLD
PDQLIHVWDPHQWHVXSHULRUHVDRVSUDWLFDGRVQRPHUFDGR 2EV(VWHLQFLVR;9,,IRLLQFOXtGRSHOD/HL

QDFLRQDORXIRUHPLQFRPSDWtYHLVFRPRVIL[DGRVSHORV ;9,,, QDV FRPSUDV RX FRQWUDWDo}HV VH VHUYLoRV SDUDR


yUJmRVRILFLDLVFRPSHWHQWHVFDVRVHPTXHREVHUYDGR DEDVWHFLPHQWR GH QDYLRV HPEDUFDo}HV XQLGDGHV
R†~QLFRGRDUWGHVWD/HLHSHUVLVWLQGRDVLWXDomR DpUHDV RX WURSDV H VHXV PHLRV GH GHVORFDPHQWR
VHUi DGPLWLGD D DGMXGLFDomR GLUHWD GRV EHQV RX TXDQGR HP HVWDGD HYHQWXDO GH FXUWD GXUDomR HP
VHUYLoRV SRU YDORU QmR VXSHULRU DR FRQVWDQWH GR SRUWRVDHURSRUWRVRXORFDOLGDGHVGLIHUHQWHVGHVXDV
UHJLVWURGHSUHoRVRXGRVVHUYLoRV VHGHVSRUPRWLYRVGHPRYLPHQWDomRRSHUDFLRQDORX
GH DGHVWUDPHQWR TXDQGR D H[LJLGDGH GRV SUD]RV
9,,, SDUDDDTXLVLomRSRUSHVVRDMXUtGLFDGHGLUHLWRS~EOLFR OHJDLVSXGHUFRPSURPHWHUDQRUPDOLGDGHHRVSURSy
LQWHUQRGHEHQVSURGX]LGRVRXVHUYLoRVSUHVWDGRVSRU
VLWRVGDVRSHUDo}HVHGHVGHTXHVHXYDORUQmRH[FHGD
yUJmR RX HQWLGDGH TXH LQWHJUH D $GPLQLVWUDomR
DROLPLWHSUHYLVWRQDDOtQHDDGRLQFLVR,,GRDUW
3~EOLFDHTXHWHQKDVLGRFULDGRSDUDHVVHILPHVSHFtIL
GHVWD/HL 2EV(VWHLQFLVR;9,,,IRLLQFOXtGRSHOD/HL
FRHPGDWDDQWHULRUjYLJrQFLDGHVWD/HLGHVGHTXHR
SUHoRFRQWUDWDGRVHMDFRPSDWtYHOFRPRSUDWLFDGRQR ;,; SDUD DV FRPSUDV GH PDWHULDLV GH XVR SHODV)RUoDV
PHUFDGR $UPDGDVFRPH[FHomRGHPDWHULDLVGHXVRSHVVRDOH
DGPLQLVWUDWLYRTXDQGRKRXYHUQHFHVVLGDGHGHPDQWHU
,; TXDQGRKRXYHUSRVVLELOLGDGHGHFRPSURPHWLPHQWRGD D SDGURQL]DomR UHTXHULGD SHOD HVWUXWXUD GH DSRLR
VHJXUDQoD QDFLRQDO QRV FDVRV HVWDEHOHFLGRV HP ORJtVWLFRGRVPHLRVQDYDLVDpUHRVHWHUUHVWUHVPHGL
GHFUHWRGR3UHVLGHQWHGD5HS~EOLFDRXYLGRR&RQVH DQWHSDUHFHUGHFRPLVVmRLQVWLWXtGDSRUGHFUHWR
OKRGH'HIHVD1DFLRQDO
2EV(VWHLQFLVR;9,;IRLLQFOXtGRSHOD/HL

; SDUD D FRPSUD RX ORFDomR GHLPyYHOGHVWLQDGRDR ;; QDFRQWUDWDomRGHDVVRFLDomRGHSRUWDGRUHVGHGHIL


DWHQGLPHQWRGDVILQDOLGDGHVSUHFtSXDVGD$GPLQLVWUD FLrQFLD ItVLFD VHP ILQV OXFUDWLYRV H GH FRPSURYDGD
omR FXMDV QHFHVVLGDGHV GH LQVWDODomR H ORFDOL]DomR LGRQHLGDGHSRUyUJmRVRXHQWLGDGHVGD$GPLQLVWUDomR
FRQGLFLRQHP D VXD HVFROKD GHVGH TXH SUHoR VHMD 3~EOLFDSDUDDSUHVWDomRGHVHUYLoRVRXIRUQHFLPHQWR
FRPSDWtYHOFRPRYDORUGHPHUFDGRVHJXQGRDYDOLD GH PmRGHREUD GHVGH TXH R SUHoR FRQWUDWDGR VHMD
omRSUpYLD 2EVFRPUHGDomRGDGDSHOD/HL FRPSDWtYHOFRPRSUDWLFDGRQRPHUFDGR
;, QDFRQWUDWDomRGHUHPDQHVFHQWHGHREUDVHUYLoRRX 2EV(VWHLQFLVR;;IRLLQFOXtGRSHOD/HL

IRUQHFLPHQWRHPFRQVHTrQFLDGHUHVFLVmRFRQWUDWX
;;, SDUD D DTXLVLomR RX FRQWUDWDomR GH SURGXWRSDUD
DO GHVGH TXH DWHQGLGD D RUGHP GH FODVVLILFDomR GD
SHVTXLVD H GHVHQYROYLPHQWR OLPLWDGD QR FDVR GH
OLFLWDomR DQWHULRU H DFHLWDV DV PHVPDV FRQGLo}HV
REUDV H VHUYLoRV GH HQJHQKDULD D  YLQWH SRU
RIHUHFLGDVSHOROLFLWDQWHYHQFHGRULQFOXVLYHTXDQWRDR
SUHoRGHYLGDPHQWHFRUULJLGR FHQWR GRYDORUGHTXHWUDWDDDOtQHD´EµGRLQFLVR,GR
FDSXWGRDUW 2EVFRPUHGDomRGDGDSHOD/HLQž
 &kPDUD/HJLVODWLYDGR
 'LVWULWR)HGHUDO 'LU$GPLQLVWUDWLYR 

;;,, QD FRQWUDWDomR GR IRUQHFLPHQWR RX VXSULPHQWRGH ;;;,,, QDFRQWUDWDomRGHHQWLGDGHVSULYDGDVVHPILQVOXFUDWL


HQHUJLDHOpWULFDFRPFRQFHVVLRQiULRRXSHUPLVVLRQi YRV SDUD D LPSOHPHQWDomR GH FLVWHUQDV RX RXWUDV
ULRGRVHUYLoRS~EOLFRGHGLVWULEXLomRRXFRPSURGXWRU WHFQRORJLDV VRFLDLV GH DFHVVR j iJXD SDUD FRQVXPR
LQGHSHQGHQWHRXDXWRSURGXWRUVHJXQGRDVQRUPDVGD KXPDQRHSURGXomRGHDOLPHQWRVSDUDEHQHILFLDUDV
OHJLVODomRHVSHFtILFD2EVUHGDomRGDGDSHOD/HLQž IDPtOLDVUXUDLVGHEDL[DUHQGDDWLQJLGDVSHODVHFDRX
IDOWDUHJXODUGHiJXD ,QFOXtGRSHOD/HLQžGH
;;,,, QD FRQWUDWDomR UHDOL]DGD SRU HPSUHVDV S~EOLFDVH
VRFLHGDGHVGHHFRQRPLDPLVWDFRPVXDVVXEVLGLiULDV
;;;,9 SDUDDDTXLVLomRSRUSHVVRDMXUtGLFDGHGLUHLWRS~EOLFR
HFRQWURODGDVGLUHWDRXLQGLUHWDPHQWHSDUDDDTXLVL
omRGHEHQVRXVHUYLoRVGHVGHTXHRSUHoRFRQWUDWDGR LQWHUQRGHLQVXPRVHVWUDWpJLFRVSDUDDVD~GHSURGX]L
VHMDFRPSDWtYHOFRPRSUDWLFDGRQRPHUFDGR GRVRXGLVWULEXtGRVSRUIXQGDomRTXHUHJLPHQWDORX
HVWDWXWDULDPHQWHWHQKDSRUILQDOLGDGHDSRLDUyUJmR
;;,9 SDUDDFHOHEUDomRGHFRQWUDWRVGHSUHVWDomRGHVHUYL GD DGPLQLVWUDomR S~EOLFD GLUHWD VXD DXWDUTXLD RX
oRV FRP DV RUJDQL]Do}HV VRFLDLV TXDOLILFDGDV QR IXQGDomRHPSURMHWRVGHHQVLQRSHVTXLVDH[WHQVmR
kPELWR GDV UHVSHFWLYDV HVIHUDV GH JRYHUQR SDUD GHVHQYROYLPHQWRLQVWLWXFLRQDOFLHQWtILFRHWHFQROyJLFR
DWLYLGDGHVFRQWHPSODGDVQRFRQWUDWRGHJHVWmR HHVWtPXORjLQRYDomRLQFOXVLYHQDJHVWmRDGPLQLVWUD
 LQFLVR;;,,,H;;,9FRPUHGDomRGDGDSHOD/HLQž WLYDHILQDQFHLUDQHFHVViULDjH[HFXomRGHVVHVSURMH
;;9 QDFRQWUDWDomRUHDOL]DGDSRU,QVWLWXLomR&LHQWtILFDH WRVRXHPSDUFHULDVTXHHQYROYDPWUDQVIHUrQFLDGH
7HFQROyJLFD,&7RXSRUDJrQFLDGHIRPHQWRSDUDD WHFQRORJLD GH SURGXWRV HVWUDWpJLFRV SDUD R 6LVWHPD
WUDQVIHUrQFLDGHWHFQRORJLDHSDUDROLFHQFLDPHQWRGH ÔQLFR GH 6D~GH ² 686 QRV WHUPRV GR LQFLVR ;;;,,
GLUHLWRGHXVRRXGHH[SORUDomRGHFULDomRSURWHJLGD GHVWH DUWLJR H TXH WHQKD VLGR FULDGD SDUD HVVH ILP
,QFOXtGRSHOD/HLQž HVSHFtILFRHPGDWDDQWHULRUjYLJrQFLDGHVWD/HLGHVGH
TXHRSUHoRFRQWUDWDGRVHMDFRPSDWtYHOFRPRSUDWLFD
 ;;9, QDFHOHEUDomRGHFRQWUDWRGHSURJUDPDFRPHQWHGD GRQRPHUFDGR LQFOXtGRSHOD/HLQž
)HGHUDomR RX FRP HQWLGDGH GH VXD DGPLQLVWUDomR
LQGLUHWD SDUD D SUHVWDomR GH VHUYLoRV S~EOLFRV GH †ž 2V SHUFHQWXDLV UHIHULGRV QRV LQFLVRV , H,,GRFDSXW
IRUPDDVVRFLDGDQRVWHUPRVGRDXWRUL]DGRHPFRQWUD GHVWHDUWLJRVHUmR YLQWHSRUFHQWR SDUDFRP
WRGHFRQVyUFLRS~EOLFRRXHPFRQYrQLRGHFRRSHUD SUDV REUDV H VHUYLoRV FRQWUDWDGRV SRU FRQVyUFLRV
omR ,QFOXtGRSHOD/HLQž S~EOLFRV VRFLHGDGH GH HFRQRPLD PLVWD HPSUHVD
;;9,, QDFRQWUDWDomRGDFROHWDSURFHVVDPHQWRHFRPHUFLDOL S~EOLFDHSRUDXWDUTXLDRXIXQGDomRTXDOLILFDGDVQD
]DomR GH UHVtGXRV VyOLGRV XUEDQRV UHFLFOiYHLV RX IRUPDGDOHLFRPR$JrQFLDV([HFXWLYDV2EVUHGDomRGDGD
SHOD/HLQž
UHXWLOL]iYHLVHPiUHDVFRPVLVWHPDGHFROHWDVHOHWLYD
GH OL[R HIHWXDGRV SRU DVVRFLDo}HV RX FRRSHUDWLYDV †ž 2 OLPLWH WHPSRUDO GH FULDomR GR yUJmRRXHQWLGDGH
IRUPDGDVH[FOXVLYDPHQWHSRUSHVVRDVItVLFDVGHEDL[D TXHLQWHJUHDDGPLQLVWUDomRS~EOLFDHVWDEHOHFLGRQR
UHQGDUHFRQKHFLGDVSHORSRGHUS~EOLFRFRPRFDWDGR LQFLVR 9,,, GR FDSXW GHVWH DUWLJR QmR VH DSOLFD DRV
UHVGHPDWHULDLVUHFLFOiYHLVFRPRXVRGHHTXLSDPHQ yUJmRVRXHQWLGDGHVTXHSURGX]HPSURGXWRVHVWUDWpJL
WRVFRPSDWtYHLVFRPDVQRUPDVWpFQLFDVDPELHQWDLVH FRVSDUDR686QRkPELWRGD/HLQRGHGH
GHVD~GHS~EOLFD LQFLVRLQWURGX]LGRSHOD/HLQžHPRGLILFDGR VHWHPEUR GH  FRQIRUPH HOHQFDGRV HP DWR GD
SHOD/HLQžGH
GLUHomRQDFLRQDOGR6862EVUHGDomRGDGDSHOD/HLQž
;;9,,, SDUDRIRUQHFLPHQWRGHEHQVHVHUYLoRVSURGX]LGRVRX †ž $ KLSyWHVH GH GLVSHQVD SUHYLVWD QR LQFLVR;;,GR
SUHVWDGRVQR3DtVTXHHQYROYDPFXPXODWLYDPHQWH FDSXWTXDQGRDSOLFDGDDREUDVHVHUYLoRVGHHQJHQKD
DOWD FRPSOH[LGDGH WHFQROyJLFD H GHIHVD QDFLRQDO ULD VHJXLUi SURFHGLPHQWRV HVSHFLDLV LQVWLWXtGRV HP
PHGLDQWHSDUHFHUGHFRPLVVmRHVSHFLDOPHQWHGHVLJQD UHJXODPHQWDomRHVSHFtILFD LQFOXtGRSHOD/HLQž
GDSHODDXWRULGDGHPi[LPDGRyUJmR LQWURGX]LGRSHOD/HLQž
GH
†ž 1mRVHDSOLFDDYHGDomRSUHYLVWDQRLQFLVR,GRFDSXW
;;,; QDDTXLVLomRGHEHQVHFRQWUDWDomRGHVHUYLoRVSDUD GRDUWRjKLSyWHVHSUHYLVWDQRLQFLVR;;,GRFDSXW
DWHQGHUDRVFRQWLQJHQWHVPLOLWDUHVGDV)RUoDV6LQJXOD LQFOXtGRSHOD/HLQž

UHVEUDVLOHLUDVHPSUHJDGDVHPRSHUDo}HVGHSD]QR
H[WHULRUQHFHVVDULDPHQWHMXVWLILFDGDVTXDQWRDRSUHoR $UW eLQH[LJtYHODOLFLWDomRTXDQGRKRXYHULQYLDELOLGDGHGH
HjHVFROKDGRIRUQHFHGRURXH[HFXWDQWHHUDWLILFDGDV FRPSHWLomRHPHVSHFLDO
SHOR&RPDQGDQWHGD)RUoD DFUHVFLGRSHOD/HLQž
, SDUDDDTXLVLomRGHPDWHULDLVHTXLSDPHQWRVRXJrQH
;;; QDFRQWUDWDomRGHLQVWLWXLomRRXRUJDQL]DomRS~EOLFD URVTXHVySRVVDPVHUIRUQHFLGRVSRUSURGXWRUHPSUH
RXSULYDGDFRPRXVHPILQVOXFUDWLYRVSDUDDSUHVWD VD RX UHSUHVHQWDQWH FRPHUFLDO H[FOXVLYR YHGDGD D
omRGHVHUYLoRVGHDVVLVWrQFLDWpFQLFDHH[WHQVmRUXUDO SUHIHUrQFLD GH PDUFD GHYHQGR D FRPSURYDomR GH
QR kPELWR GR 3URJUDPD 1DFLRQDO GH $VVLVWrQFLD H[FOXVLYLGDGHVHUIHLWDDWUDYpVGHDWHVWDGRIRUQHFLGR
7pFQLFDH([WHQVmR5XUDOQD$JULFXOWXUD)DPLOLDUHQD SHORyUJmRGHUHJLVWURGRFRPpUFLRGRORFDOHPTXHVH
5HIRUPD$JUiULDLQVWLWXtGRSRUOHLIHGHUDO LQWURGX]LGRSHOD UHDOL]DULD D OLFLWDomR RX D REUD RX R VHUYLoR SHOR
/HLQž
6LQGLFDWR )HGHUDomR RX FRQIHGHUDomR 3DWURQDO RX
DLQGDSHODVHQWLGDGHVHTXLYDOHQWHV
;;;, QDVFRQWUDWDo}HVYLVDQGRDRFXPSULPHQWRGRGLVSRVWR
QRVDUWVRRRHGD/HLQRGHGH ,, SDUDDFRQWUDWDomRGHVHUYLoRVWpFQLFRVHQXPHUDGRV
GH]HPEURGHREVHUYDGRVRVSULQFtSLRVJHUDLVGH QRDUWGHVWD/HLGHQDWXUH]DVLQJXODUFRPSURILVVL
FRQWUDWDomRGHODFRQVWDQWHV ,QFOXtGRSHOD/HLQžGH RQDLVRXHPSUHVDVGHQRWyULDHVSHFLDOL]DomRYHGDGD
;;;,, QD FRQWUDWDomR HP TXH KRXYHU WUDQVIHUrQFLDGH D LQH[LJLELOLGDGH SDUD VHUYLoRV GH SXEOLFLGDGH H
WHFQRORJLD GH SURGXWRV HVWUDWpJLFRV SDUD R 6LVWHPD GLYXOJDomR
ÔQLFRGH6D~GH686QRkPELWRGD/HLQRGH
GHVHWHPEURGHFRQIRUPHHOHQFDGRVHPDWR ,,, SDUDDFRQWUDWDomRGHSURILVVLRQDOGHTXDOTXHUVHWRU
GDGLUHomRQDFLRQDOGR686LQFOXVLYHSRURFDVLmRGD DUWtVWLFR GLUHWDPHQWH RX DWUDYpV GH HPSUHViULR
DTXLVLomR GHVWHV SURGXWRV GXUDQWH DV HWDSDV GH H[FOXVLYR GHVGH TXH FRQVDJUDGR SHOD FUtWLFD
DEVRUomRWHFQROyJLFD UHGDomRGDGDSHOD/HLQž HVSHFLDOL]DGDRXSHODRSLQLmRS~EOLFD
 'LU$GPLQLVWUDWLYR
 &kPDUD/HJLVODWLYDGR'LVWULWR)HGHUDO
†ž &RQVLGHUDVHGHQRWyULDHVSHFLDOL]DomRRSURILVVLRQDO ,9 LQVFULomRGRDWRFRQVWLWXWLYRQRFDVRGHVRFLHGDGHV
RXHPSUHVDFXMRFRQFHLWRQRFDPSRGHVXDHVSHFLDOL FLYLVDFRPSDQKDGDGHSURYDGHGLUHWRULDHPH[HUFt
GDGH GHFRUUHQWH GH GHVHPSHQKR DQWHULRU HVWXGRV FLR
H[SHULrQFLDVSXEOLFDo}HVRUJDQL]DomRDSDUHOKDPHQ
WRHTXLSHWpFQLFDRXGHRXWURVUHTXLVLWRVUHODFLRQD 9 GHFUHWRGHDXWRUL]DomRHPVHWUDWDQGRGHHPSUHVDRX
GRV FRP VXDV DWLYLGDGHV SHUPLWD LQIHULU TXH R VHX VRFLHGDGH HVWUDQJHLUDHPIXQFLRQDPHQWRQR3DtVH
WUDEDOKR p HVVHQFLDO H LQGLVFXWLYHOPHQWH R PDLV DWR GH UHJLVWUR RX DXWRUL]DomR SDUD IXQFLRQDPHQWR
DGHTXDGRjSOHQDVDWLVIDomRGRREMHWRGRFRQWUDWR H[SHGLGRSHORyUJmRFRPSHWHQWHTXDQGRDDWLYLGDGH
DVVLPRH[LJLU
†ž 1DKLSyWHVHGHVWHDUWLJRHHPTXDOTXHUGRVFDVRVGH
GLVSHQVDVHFRPSURYDGRVXSHUIDWXUDPHQWRUHVSRQ $UW $ GRFXPHQWDomR UHODWLYD j UHJXODULGDGH ILVFDOH
GHP VROLGDULDPHQWH SHOR GDQR FDXVDGR j )D]HQGD WUDEDOKLVWDFRQIRUPHRFDVRFRQVLVWLUiHP 5HGDomRGDGD
SHOD/HLQžGH
3~EOLFDRIRUQHFHGRURXRSUHVWDGRUGHVHUYLoRVHR
DJHQWH S~EOLFR UHVSRQViYHO VHP SUHMXt]R GH RXWUDV , SURYD GH LQVFULomR QR&DGDVWURGH3HVVRDV)tVLFDV
VDQo}HVOHJDLVFDEtYHLV &3) RXQR&DGDVWUR*HUDOGH&RQWULEXLQWHV &*& 

$UW $VGLVSHQVDVSUHYLVWDVQRV††žHžGR$UWHQR ,, SURYD GH LQVFULomR QRFDGDVWURGHFRQWULEXLQWHV


LQFLVR,,,HVHJXLQWHVGR$UWDVVLWXDo}HVGHLQH[LJL HVWDGXDO RX PXQLFLSDO VH KRXYHU UHODWLYR DR GR
ELOLGDGHUHIHULGDVQR$UWQHFHVVDULDPHQWHMXVWLILFD PLFtOLRRXVHGHGROLFLWDQWHSHUWLQHQWHDRVHXUDPRGH
GDVHRUHWDUGDPHQWRSUHYLVWRQRILQDOGRSDUiJUDIR DWLYLGDGHHFRPSDWtYHOFRPRREMHWRFRQWUDWXDO
~QLFRGR$UWžGHVWD/HLGHYHUmRVHUFRPXQLFDGRV
GHQWURGH WUrV GLDVjDXWRULGDGHVXSHULRUSDUD ,,, SURYDGHUHJXODULGDGHSDUDFRPD)D]HQGD)HGHUDO
UDWLILFDomRHSXEOLFDomRQDLPSUHQVDRILFLDOQRSUD]R (VWDGXDOH0XQLFLSDOGRGRPLFtOLRRXVHGHGROLFLWDQ
GH FLQFR GLDVFRPRFRQGLomRSDUDDHILFiFLDGRV WHRXRXWUDHTXLYDOHQWHQDIRUPDGDOHL
DWRV 2EV5HGDomRGDGDSHOD/HLQžGH
,9 SURYDGHUHJXODULGDGHUHODWLYDj6HJXULGDGH6RFLDOH
†~QLFR 2 SURFHVVR GH GLVSHQVD GH LQH[LJLELOLGDGHRXGH DR)XQGRGH*DUDQWLDSRU7HPSRGH6HUYLoR )*76 
UHWDUGDPHQWRSUHYLVWRQHVWHDUWLJRVHUiLQVWUXtGRQR GHPRQVWUDQGRVLWXDomRUHJXODUQRFXPSULPHQWRGRV
TXHFRXEHUFRPRVVHJXLQWHVHOHPHQWRV HQFDUJRVVRFLDLVLQVWLWXtGRVSRUOHL2EVFRPUHGDomRGDGDSHOD
/HLQž

, FDUDFWHUL]DomRGDVLWXDomRHPHUJHQFLDORXFDODPLWRVD 9 SURYDGHLQH[LVWrQFLDGHGpELWRVLQDGLPSOLGRVSHUDQWH


TXHMXVWLILTXHDGLVSHQVDTXDQGRIRURFDVR D -XVWLoD GR 7UDEDOKR PHGLDQWH D DSUHVHQWDomR GH
FHUWLGmR QHJDWLYD QRV WHUPRV GR 7tWXOR 9,,$ GD
,, UD]mRGDHVFROKDGRIRUQHFHGRURXH[HFXWDQWH &RQVROLGDomR GDV /HLV GR 7UDEDOKR DSURYDGD SHOR
'HFUHWR/HLQžGHžGHPDLRGH2EVLQFOXtGR
,,, MXVWLILFDWLYDGRSUHoR SHOD/HLQž

,9 GRFXPHQWRGHDSURYDomRGRVSURMHWRVGHSHVTXLVDDRV $UW $ GRFXPHQWDomR UHODWLYD j TXDOLILFDomRWpFQLFD


TXDLVRVEHQVVHUmRDORFDGRV OLPLWDUVHiD
2EVLQFLVR,9WHPVXDUHGDomRGDGDSHOD/HLQž , UHJLVWUR RX LQVFULomRQDHQWLGDGHSURILVVLRQDOFRP
SHWHQWH
6HomR,,
'D+DELOLWDomR ,, FRPSURYDomRGHDSWLGmRSDUDGHVHPSHQKRGHDWLYLGD
GHSHUWLQHQWHHFRPSDWtYHOHPFDUDFWHUtVWLFDVTXDQWL
$UW 3DUDDKDELOLWDomRQDVOLFLWDo}HVH[LJLUVHiGRVLQWH GDGHVHSUD]RVFRPRREMHWRGDOLFLWDomRHLQGLFDomR
UHVVDGRVH[FOXVLYDPHQWHGRFXPHQWDomRUHODWLYDD GDV LQVWDODo}HV H GR DSDUHOKDPHQWR H GR SHVVRDO
WpFQLFRDGHTXDGRVHGLVSRQtYHLVSDUDDUHDOL]DomRGR
, KDELOLWDomRMXUtGLFD REMHWRGDOLFLWDomREHPFRPRGDTXDOLILFDomRGHFDGD
XPGRVPHPEURVGDHTXLSHWpFQLFDTXHVHUHVSRQVDEL
,, TXDOLILFDomRWpFQLFD OL]DUiSHORVWUDEDOKRV
,,, FRPSURYDomRIRUQHFLGDSHORyUJmROLFLWDQWHGHTXH
,,, TXDOLILFDomRHFRQ{PLFDILQDQFHLUD UHFHEHX RV GRFXPHQWRV H TXDQGR H[LJLGR GH TXH
WRPRXFRQKHFLPHQWRGHWRGDVDVLQIRUPDo}HVHGDV
,9 UHJXODULGDGHILVFDOHWUDEDOKLVWD2EV
EV LQFLVRDOWHUDGRSHOD/HLQž
FRQGLo}HVORFDLVSDUDRFXPSULPHQWRGDVREULJDo}HV

REMHWRGDOLFLWDomR
9 FXPSULPHQWRGRGLVSRVWRQRLQFLVR;;;,,,GRDUWž
GD&RQVWLWXLomR)HGHUDO 2EV(VWHLQFLVRIRLDFUHVFHQWDGRSHOD/HLQž
2EV ,9 SURYDGHDWHQGLPHQWRGHUHTXLVLWRVSUHYLVWRVHPOHL
 HVSHFLDOTXDQGRIRURFDVR
$UW $ GRFXPHQWDomR UHODWLYD j KDELOLWDomRMXUtGLFD †ž $ FRPSURYDomR GH DSWLGmR UHIHULGD QR LQFLVR,,GR
FRQIRUPHRFDVRFRQVLVWLUiHP FDSXW QR FDVR GDV OLFLWDo}HV SHUWLQHQWHV D REUDV H
VHUYLoRV VHUi IHLWD SRU DWHVWDGRV IRUQHFLGRV SRU
, FpGXODGHLGHQWLGDGH SHVVRDVMXUtGLFDVGHGLUHLWRS~EOLFRRXSULYDGRGHYL
GDPHQWH UHJLVWUDGRV QDV HQWLGDGHV SURILVVLRQDLV
,, UHJLVWURFRPHUFLDOQRFDVRGHHPSUHVDLQGLYLGXDO FRPSHWHQWHVOLPLWDGDVDVH[LJrQFLDVD2EVFRPUHGDomRGDGD
SHOD/HLQž

,,, DWRFRQVWLWXWLYRHVWDWXWRRXFRQWUDWRVRFLDOHPYLJRU , FDSDFLWDomR WpFQLFRSURILVVLRQDOFRPSURYDomRGR


GHYLGDPHQWHUHJLVWUDGRHPVHWUDWDQGRGHVRFLHGDGHV OLFLWDQWH GH SRVVXLU HP VHX TXDGUR SHUPDQHQWH QD
FRPHUFLDLV H QR FDVR GH VRFLHGDGHV SRU Do}HV GDWDSUHYLVWDSDUDHQWUHJDGDSURSRVWDSURILVVLRQDOGH
DFRPSDQKDGR GH GRFXPHQWRV GH HOHLomR GH VHXV QtYHOVXSHULRURXRXWURGHYLGDPHQWHUHFRQKHFLGRSHOD
DGPLQLVWUDGRUHV HQWLGDGHFRPSHWHQWHGHWHQWRUGHDWHVWDGRGHUHVSRQ
 &kPDUD/HJLVODWLYDGR
 'LVWULWR)HGHUDO 'LU$GPLQLVWUDWLYR 

VDELOLGDGHWpFQLFDSRUH[HFXomRGHREUDRXVHUYLoRGH ,, FHUWLGmRQHJDWLYDGHIDOrQFLDRXFRQFRUGDWDH[SHGLGD


FDUDFWHUtVWLFDVVHPHOKDQWHVOLPLWDGDVHVWDVH[FOXVLYD SHOR GLVWULEXLGRU GD VHGH GD SHVVRD MXUtGLFD RX GH
PHQWHjVSDUFHODVGHPDLRUUHOHYkQFLDHYDORUVLJQLIL H[HFXomR SDWULPRQLDO H[SHGLGD QR GRPLFtOLR GD
FDWLYRGRREMHWRGDOLFLWDomRYHGDGDVDVH[LJrQFLDVGH SHVVRDItVLFD
TXDQWLGDGHVPtQLPDVRXSUD]RVPi[LPRV2EVFRPUHGDomR
GDGDSHOD/HLQž
,,, JDUDQWLDQDVPHVPDVPRGDOLGDGHVHFULWpULRVSUHYLV
,, 9HWDGR  WRVQRFDSXWH†žGRDUWGHVWD/HLOLPLWDGDD
 XPSRUFHQWR GRYDORUHVWLPDGRGRREMHWRGD
†ž $VSDUFHODVGHPDLRUUHOHYkQFLDWpFQLFDRXGHYDORU FRQWUDomR
VLJQLILFDWLYRPHQFLRQDGDVQRSDUiJUDIRDQWHULRUVHUmR
GHILQLGDVQRLQVWUXPHQWRFRQYRFDWyULR2EVFRPUHGDomRGDGD †ž $H[LJrQFLDGHLQGLFDGRUHVOLPLWDUVHijGHPRQVWUD
SHOD/HLQž
omRGDFDSDFLGDGHILQDQFHLUDGROLFLWDQWHFRPYLVWDV
†ž 6HUi VHPSUH DGPLWLGD D FRPSURYDomRGHDSWLGmR DRVFRPSURPLVVRVTXHWHUiTXHDVVXPLUFDVROKHVHMD
DWUDYpVGHFHUWLG}HVRXDWHVWDGRVGHREUDVRXVHUYLoRV DGMXGLFDGRRFRQWUDWRYHGDGDDH[LJrQFLDGHYDORUHV
VLPLODUHVGHFRPSOH[LGDGHWHFQROyJLFDHRSHUDFLRQDO PtQLPRVGHIDWXUDPHQWRDQWHULRUtQGLFHVGHUHQWDELOL
HTXLYDOHQWHRXVXSHULRU GDGHRXOXFUDWLYLGDGH2EVFRPUHGDomRGDGDSHOD/HLQž
†ž $$GPLQLVWUDomRQDVFRPSUDVSDUDHQWUHJDIXWXUDH
†ž 1DV OLFLWDo}HV SDUD IRUQHFLPHQWR GH EHQVDFRP QDH[HFXomRGHREUDVHVHUYLoRVSRGHUiHVWDEHOHFHU
SURYDomR GH DSWLGmR TXDQGR IRU R FDVR VHUi IHLWD QRLQVWUXPHQWRFRQYRFDWyULRGDOLFLWDomRDH[LJrQFLD
DWUDYpVGHDWHVWDGRVIRUQHFLGRVSRUSHVVRDMXUtGLFDGH GHFDSLWDOPtQLPRRXGHSDWULP{QLROtTXLGRPtQLPR
GLUHLWRS~EOLFRRXSULYDGR RX DLQGD DV JDUDQWLDV SUHYLVWDV QR † ž GR DUW 
GHVWD /HL FRPR GDGR REMHWLYR GH FRPSURYDomR GD
†ž eYHGDGDDH[LJrQFLDGHFRPSURYDomRGHDWLYLGDGHRX TXDOLILFDomRHFRQ{PLFRILQDQFHLUDGRVOLFLWDQWHVHSDUD
GHDSWLGmRFRPOLPLWDo}HVGHWHPSRRXGHpSRFDRX HIHLWRGHJDUDQWLDDRDGLPSOHPHQWRGRFRQWUDWRDVHU
DLQGDHPORFDLVHVSHFtILFRVRXTXDLVTXHURXWUDVQmR XOWHULRUPHQWHFHOHEUDGR
SUHYLVWDV QHVWD /HL TXH LQLEDP D SDUWLFLSDomR QD
OLFLWDomR †ž 2FDSLWDOPtQLPRRXRYDORUGRSDWULP{QLROtTXLGRD
TXHVHUHIHUHRSDUiJUDIRDQWHULRUQmRSRGHUiH[FHGHU
†ž $V H[LJrQFLDV PtQLPDV UHODWLYDV DLQVWDODo}HVGH D GH]SRUFHQWR GRYDORUHVWLPDGRGDFRQWUDWD
FDQWHLURVPiTXLQDVHTXLSDPHQWRVHSHVVRDOWpFQLFR omRGHYHQGRDFRPSURYDomRVHUIHLWDUHODWLYDPHQWHj
HVSHFLDOL]DGRFRQVLGHUDGRVHVVHQFLDLVSDUDRFXPSUL GDWD GD DSUHVHQWDomR GD SURSRVWD QD IRUPD GD OHL
PHQWRGRREMHWRGDOLFLWDomRVHUmRDWHQGLGDVPHGLDQ DGPLWLGDDDWXDOL]DomRSDUDHVWDGDWDGDDSUHVHQWDomR
WHDDSUHVHQWDomRGHUHODomRH[SOtFLWDHGDGHFODUDomR GDSURSRVWDQDIRUPDGDOHLDGPLWLGDDDWXDOL]DomR
IRUPDOGDVXDGLVSRQLELOLGDGHVREDVSHQDVFDEtYHLV SDUDHVWDGDWDDWUDYpVGHtQGLFHVRILFLDLV
YHGDGDDVH[LJrQFLDVGHSURSULHGDGHHGHORFDOL]DomR
SUpYLD †ž 3RGHUiVHUH[LJLGDDLQGDDUHODomRGRVFRPSURPLV
†ž YHWDGR  VRVDVVXPLGRVSHOROLFLWDQWHTXHLPSRUWHPGLPLQXLomR
GDFDSDFLGDGHRSHUDWLYDRXDEVRUomRGHGLVSRQLELOLGD
†ž 1RFDVRGHREUDVVHUYLoRVHFRPSUDVGHJUDQGHYXOWR GHILQDQFHLUDFDOFXODGDHVWDHPIXQomRGRSDWULP{QLR
GHDOWDFRPSOH[LGDGHWpFQLFDSRGHUiD$GPLQLVWUDomR OtTXLGRDWXDOL]DGRHVXDFDSDFLGDGHGHURWDomR
H[LJLUGRVOLFLWDQWHVDPHWRGRORJLDGHH[HFXomRFXMD
DYDOLDomRSDUDHIHLWRGHVXDDFHLWDomRRXQmRDQWHFH †ž $FRPSURYDomRGHERDVLWXDomRILQDQFHLUDGDHPSUHVD
GHUi VHPSUH j DQiOLVH GRV SUHoRV H VHUi HIHWXDGD VHUi IHLWD GH IRUPD REMHWLYD DWUDYpV GR FiOFXOR GH
H[FOXVLYDPHQWHSRUFULWpULRVREMHWLYRV tQGLFHV FRQWiEHLV SUHYLVWRV QR HGLWDO H GHYLGDPHQWH
MXVWLILFDGRV QR SURFHVVR DGPLQLVWUDWLYR GD OLFLWDomR
†ž (QWHQGHVHSRUOLFLWDomRGHDOWDFRPSOH[LGDGHWpFQLFD TXHWHQKDGDGRLQtFLRDRFHUWDPHOLFLWDWyULRYHGDGDD
DTXHODTXHHQYROYDDOWDHVSHFLDOL]DomRFRPRIDWRUGH H[LJrQFLDGHtQGLFHVHYDORUHVQmRXVXDOPHQWHDGRWD
H[WUHPDUHOHYkQFLDSDUDJDUDQWLUDH[HFXomRGRREMHWR GRVSDUDDDYDOLDomRGHVLWXDomRILQDQFHLUDVVXILFLHQWH
DVHUFRQWUDWDGRRXTXHSRVVDFRPSURPHWHUDFRQWL DR FXPSULPHQWR GDV REULJDo}HV GHFRUUHQWHV GD
QXLGDGHGDSUHVWDomRGHVHUYLoRVS~EOLFRVHVVHQFLDLV OLFLWDomR
2EVFRPUHGDomRGDGDSHOD/HLQž
†ž YHWDGR 
† 2VSURILVVLRQDLVLQGLFDGRVSHOROLFLWDQWHSDUDILQVGH
FRPSURYDomR GD FDSDFLWDomR WpFQLFRRSHUDFLRQDO GH $UW 2VGRFXPHQWRVQHFHVViULRVjKDELOLWDomRSRGHUmRVHU
TXH WUDWD R LQFLVR , GR † ž GHVWH DUWLJR GHYHUmR DSUHVHQWDGRVHPRULJLQDOSRUTXDOTXHUSURFHVVRGH
SDUWLFLSDU GD REUD RX VHUYLoR REMHWR GD OLFLWDomR FySLD DXWHQWLFDGD SRU FDUWyULR FRPSHWHQWH RX SRU
DGPLWLQGRVH D VXEVWLWXLomR SRU SURILVVLRQDLV GH VHUYLGRUGDDGPLQLVWUDomRRXSXEOLFDomRHPyUJmRGH
H[SHULrQFLDHTXLYDOHQWHRXVXSHULRUGHVGHTXHDSUR LPSUHVVDRILFLDO2EVFRPUHGDomRGDGDSHOD/HLQž
YDGDSHOD$GPLQLVWUDomR
2EV(VWH†IRLLQFOXtGRSHOD/HL †ž $GRFXPHQWDomRGHTXHWUDWDPRVDUWVDGHVWD
$UW $ GRFXPHQWDomR UHODWLYD j TXDOLILFDomRHFRQ{PLFD /HLSRGHUiVHUGLVSHQVDGDQRWRGRRXHPSDUWHQRV
ILQDQFHLUDOLPLWDUVHi FDVRVGHFRQYLWHFRQFXUVRIRUQHFLPHQWRGHEHQVSDUD
SURQWDHQWUHJDHOHLOmR
, EDODQoR SDWULPRQLDO H GHPRQVWUDo}HVFRQWiEHLVGR
~OWLPRH[HUFtFLRVRFLDOMiH[LJtYHLVHDSUHVHQWDGRVQD †ž 2FHUWLILFDGRGHUHJLVWURFDGDVWUDODTXHVHUHIHUHR†
IRUPDGDOHLTXHFRPSURYHPDERDVLWXDomRILQDQFHL žGRDUWVXEVWLWXLRVGRFXPHQWRVHQXPHUDGRVQRV
UDGDHPSUHVDYHGDGDDVXDVXEVWLWXLomRSRUEDODQFH DUWVDTXDQWRjVLQIRUPDo}HVGLVSRQLELOL]DGDV
WHVRXEDODQoRVSURYLVyULRVSRGHQGRVHUDWXDOL]DGRV HPVLVWHPDLQIRUPDWL]DGRGHFRQVXOWDGLUHWDLQGLFDGR
SRUtQGLFHVRILFLDLVTXDQGRHQFHUUDGRKiPDLVGH QR HGLWDO REULJDQGRVH D SDUWH D GHFODUDU VRE DV
WUrV PHVHVGDGDWDGHDSUHVHQWDomRGDSURSRVWD SHQDOLGDGHVFDEtYHLVDVXSHUYHQLrQFLDGHIDWRLPSHGL
WLYRGDKDELOLWDomR2EVFRPUHGDomRGDGDSHOD/HLQž
 'LU$GPLQLVWUDWLYR
&kPDUD/HJLVODWLYDGR'LVWULWR)HGHUDO
†ž $ GRFXPHQWDomR UHIHULGD QHVWH DUWLJRSRGHUiVHU †ž 1RFRQVyUFLRGHHPSUHVDVEUDVLOHLUDVHHVWUDQJHLUDVD
VXEVWLWXtGDSRUUHJLVWURFDGDVWUDOHPLWLGRSRUyUJmRRX OLGHUDQoDFDEHUiREULJDWRULDPHQWHjHPSUHVDEUDVLOHL
HQWLGDGH S~EOLFD GHVGH TXH SUHYLVWR QR HGLWDO H R UDREVHUYDGRRGLVSRVWRQRLQF,,GHVWHDUWLJR
UHJLVWUR WHQKD VLGR IHLWR HP REHGLrQFLD DR GLVSRVWR
QHVWD/HL †ž 2OLFLWDQWHYHQFHGRUILFDREULJDGRDSURPRYHUDQWHVGD
FHOHEUDomRGRFRQWUDWRDFRQVWLWXLomRHRUHJLVWURGR
†ž $VHPSUHVDVHVWUDQJHLUDVTXHQmRIXQFLRQHPQRSDtV FRQVyUFLR QRV WHUPRV GR FRPSURPLVVR UHIHULGR QR
WDQWR TXDQWR SRVVtYHO DWHQGHUmR QDV OLFLWDo}HV LQFLVR,GHVWHDUWLJR
LQWHUQDFLRQDLVjVH[LJrQFLDVGRVSDUiJUDIRVDQWHULRUHV
PHGLDQWHGRFXPHQWRVHTXLYDOHQWHVDXWHQWLFDGRVSHORV 6HomR,,,
UHVSHFWLYRVFRQVXODGRVHWUD]LGRVSRUWUDGXWRUMXUDPHQ 'RV5HJLVWURV&DGDVWUDLV
WDGR GHYHQGR WHU UHSUHVHQWDomR OHJDO QR %UDVLO FRP
SRGHUHV H[SUHVVRV SDUD UHFHEHU FLWDomR H UHVSRQGHU $UW 3DUDRVILQVGHVWD/HLRVyUJmRVHHQWLGDGHVGD$GPL
DGPLQLVWUDWLYDRXMXGLFLDOPHQWH QLVWUDomR3~EOLFDTXHUHDOL]HPIUHTHQWHPHQWHOLFLWD
o}HV PDQWHUmR UHJLVWURV FDGDVWUDLV SDUD HIHLWR GH
†ž 1mRVHH[LJLUiSDUDKDELOLWDomRGHTXHWUDWDHVWHDUWLJR KDELOLWDomR QD IRUPD UHJXODPHQWDU YiOLGRV SRU QR
SUpYLRUHFROKLPHQWRGHWD[DVRXHPROXPHQWRVVDOYRRV Pi[LPRXPDQR
UHIHUHQWHVDIRUQHFLPHQWRGRHGLWDOTXDQGRVROLFLWDGR
FRPVHXVHOHPHQWRVFRQVWLWXWLYRVOLPLWDGRVDRYDORUGR †ž 2UHJLVWURFDGDVWUDOGHYHUiVHUDPSODPHQWHGLYXOJDGR
FXVWRHIHWLYRGHUHSURGXomRJUiILFDGDGRFXPHQWDomR HGHYHUiHVWDUSHUPDQHQWHPHQWHDEHUWRDRVLQWHUHVVD
IRUQHFLGD GRV REULJDQGRVH D XQLGDGH SRU HOH UHVSRQViYHO D
SURFHGHUQRPtQLPRDQXDOPHQWHDWUDYpVGDLPSUHQVD
†ž 2GLVSRVWRQR†žGHVWHDUWLJRQR†žGRDUWHQR RILFLDOHGHMRUQDOGLiULRDFKDPDPHQWRS~EOLFRSDUD
†žGRDUWQmRVHDSOLFDDVOLFLWDo}HVLQWHUQDFLRQDLV DWXDOL]DomRGRVUHJLVWURVH[LVWHQWHVHSDUDRLQJUHVVRGH
SDUDDTXLVLomRGHEHQVHVHUYLoRVFXMRSDJDPHQWRVHMD QRYRVLQWHUHVVDGRV
IHLWR FRP SURGXWR GH ILQDQFLDPHQWR FRQFHGLGR SRU
RUJDQLVPRILQDQFHLURLQWHUQDFLRQDOGHTXHR%UDVLOIDoD †ž eIDFXOWDGRjVXQLGDGHVDGPLQLVWUDWLYDVXWLOL]DUHPVH
SDUWHRXSRUDJrQFLDHVWUDQJHLUDGHFRRSHUDomRQHP GHUHJLVWURVFDGDVWUDLVGHRXWURVyUJmRVRXHQWLGDGHVGD
QRVFDVRVGHFRQWUDWDomRFRPHPSUHVDHVWUDQJHLUDSDUD $GPLQLVWUDomR3~EOLFD
FRPSUD GH HTXLSDPHQWRV IDEULFDGRV H HQWUHJXHV QR
H[WHULRUGHVGHTXHSDUDHVWHFDVRWHQKDKDYLGRSUpYLD $UW $RUHTXHUHULQVFULomRQRFDGDVWURRXDWXDOL]DomRGHVWH
DXWRUL]DomR GR &KHIH GR 3RGHU ([HFXWLYR QHP QRV DTXDOTXHUWHPSRRLQWHUHVVDGRIRUQHFHUiRVHOHPHQWRV
FDVRV GH DTXLVLomR GH EHQV H VHUYLoRV UHDOL]DGD SRU QHFHVViULRVjVDWLVIDomRGDVH[LJrQFLDVGRDUWGHVWD
XQLGDGHVDGPLQLVWUDWLYDVQRH[WHULRU /HL
†ž $GRFXPHQWDomRGHTXHWUDWDPRVDUWVDHHVWH $UW 2VLQVFULWRVVHUmRFODVVLILFDGRVSRUFDWHJRULDVWHQGRVH
DUWLJRSRGHUiVHUGLVSHQVDGDQRVWHUPRVGHUHJXODPHQ HP YLVWD VXD HVSHFLDOL]DomR VXEGLYLGDV HP JUXSR
WRQRWRGRRXHPSDUWHSDUDDFRQWUDWDomRGHSURGXWR VHJXQGR D TXDOLILFDomR WpFQLFD H HFRQ{PLFD DYDOLDGR
SDUD SHVTXLVD H GHVHQYROYLPHQWR GHVGH TXH SDUD
SHORVHOHPHQWRVFRQVWDQWHVGDGRFXPHQWDomRUHODFLRQD
SURQWDHQWUHJDRXDWpRYDORUSUHYLVWRQDDOtQHD´DµGR
GDQRVDUWVHGHVWD/HL
LQFLVR,,GRFDSXWGRDUW LQFOXtGRSHOD/HLQž
†ž $RV LQVFULWRV VHUi IRUQHFLGRFHUWLILFDGRUHQRYiYHO
$UW 4XDQGRSHUPLWLGDQDOLFLWDomRDSDUWLFLSDomRGHHPSUH
VHPSUHTXHDWXDOL]DUHPRUHJLVWUR
VDVHPFRQVyUFLRREVHUYDUVHmRDVVHJXLQWHVQRUPDV
†ž $DWXDomRGHOLFLWDQWHQRFXPSULPHQWRGHREULJDo}HV
, FRPSURYDomRGRFRPSURPLVVRS~EOLFRRXSDUWLFXODUGH
DVVXPLGDVVHUiDQRWDGDQRUHVSHFWLYRUHJLVWURFDGDVWUDO
FRQVWLWXLomRGHFRQVyUFLRVXEVFULWRSHORVFRQVRUFLDGRV
$UW $ TXDOTXHU WHPSR SRGHUi VHU DOWHUDGR VXVSHQVRRX
,, LQGLFDomRGDHPSUHVDUHVSRQViYHOSHORFRQVyUFLRTXH
GHYHUi DWHQGHU jV FRQGLo}HV GH OLGHUDQoD REULJDWR FDQFHODGR R UHJLVWUR GR LQVFULWR GH TXH GHL[DU GH
ULDPHQWHIL[DGDVQRHGLWDO VDWLVID]HU DV H[LJrQFLDV GR DUW GHVWD /HL RX DV
HVWDEHOHFLGDVSDUDFODVVLILFDomRFDGDVWUDO
,,, DSUHVHQWDomRGRVGRFXPHQWRVH[LJLGRVQRVDUWVD
GHVWD/HLSRUSDUWHGHFDGDFRQVRUFLDGRDGPLWLQGRVH 6HomR,9
SDUD HIHLWR GH TXDOLILFDomR WpFQLFD R VRPDWyULR GRV 'R3URFHGLPHQWRH-XOJDPHQWR
TXDQWLWDWLYRV GH FDGD FRQVRUFLDGR H SDUD HIHLWR GH
TXDOLILFDomR HFRQ{PLFRILQDQFHLUD R VRPDWyULR GRV $UW 2SURFHGLPHQWRGDOLFLWDomRVHUiLQLFLDGRFRPDEHUWXUD
YDORUHV GH FDGD FRQVRUFLDGR QD SURSRUomR GH VXD GH SURFHVVR DGPLQLVWUDWLYR GHYLGDPHQWH DXWXDGR
UHVSHFWLYD SDUWLFLSDomR SRGHQGR D $GPLQLVWUDomR SURWRFRODGRHQXPHUDGRFRQWHQGRDXWRUL]DomRUHVSHF
HVWDEHOHFHU SDUD R FRQVyUFLR XP DFUpVFLPR GH  WLYD D LQGLFDomR VXVFLQWD GH VHX REMHWR H GR UHFXUVR
WULQWD SRU FHQWR  GRV YDORUHV H[LJLGRV SDUD OLFLWDQWH SUySULRSDUDGHVSHVDHDRTXDOVHUmRMXQWDGRVRSRUWX
LQGLYLGXDOLQH[LJtYHOHVWHDFUpVFLPRSDUDRVFRQVyUFLRV QDPHQWH
FRPSRVWRV HP VXD WRWDOLGDGH SRU PLFUR H SHTXHQDV
HPSUHVDVDVVLPGHILQLGDVHP/HL , HGLWDO RX FRQYLWH H UHVSHFWLYRVDQH[RVTXDQGRIRUR
FDVR
,9 LPSHGLPHQWRGHSDUWLFLSDomRGHHPSUHVDFRQVRUFLDGD
QDPHVPDOLFLWDomRDWUDYpVGHPDLVGHXPFRQVyUFLRRX ,, FRPSURYDQWH GDV SXEOLFDo}HV GRHGLWDOUHVXPLGRQD
LVRODGDPHQWH IRUPDGRDUWGHVWD/HLRXGDHQWUHJDGRFRQYLWH

9 UHVSRQVDELOLGDGH VROLGiULD GRVLQWHJUDQWHVSHORVDWRV ,,, DWRGHGHVLJQDomRGDFRPLVVmRGHOLFLWDomRGROHLORHLUR


SUDWLFDGRV HP FRQVyUFLR WDQWR QD IDVH GH OLFLWDomR DGPLQLVWUDWLYR RX RILFLDO RX GR UHVSRQViYHO SHOR
TXDQWRQDGHH[HFXomRGHFRQWUDWR FRQYLWH
 &kPDUD/HJLVODWLYDGR
 'LVWULWR)HGHUDO 'LU$GPLQLVWUDWLYR 

,9 RULJLQDOGDVSURSRVWDVHGRVGRFXPHQWRVTXHDVLQVWUXt 9 VHKiSURMHWRH[HFXWLYRGLVSRQtYHOQDGDWDGDSXEOLFD


UHP omR GR HGLWDO GH OLFLWDomR H R ORFDO RQGH SRVVD VHU
H[DPLQDGRHDGTXLULGR
9 DWDVUHODWyULRVHGHOLEHUDo}HVGDFRPLVVmRMXOJDGRUD
9, FRQGLo}HVSDUDSDUWLFLSDomRQDOLFLWDomRHPFRQIRUPL
9, SDUHFHUHVWpFQLFRVRXMXUtGLFRVHPLWLGRVVREUHDOLFLWD GDGH FRP RV DUWV D  GHVWD /HL HP IRUPD GH
omRGLVSHQVDLQH[LJLELOLGDGH DSUHVHQWDomRGDVSURSRVWDV

9,, DWRV GH DGMXGLFDomR GR REMHWR GD  OLFLWDomR H GDVXD 9,, FULWpULRSDUDMXOJDPHQWRFRPGLVSRVLo}HVFODUDVHSDUk
KRPRORJDomR PHWURVREMHWLYRV

9,,, UHFXUVRVHYHQWXDOPHQWHDSUHVHQWDGRVSHORVOLFLWDQWHVH 9,,, ORFDLV KRUiULRV H FyGLJRV GH DFHVVR GRV PHLRVGH
UHVSHFWLYDVPDQLIHVWDo}HVHGHFLV}HV FRPXQLFDomR j GLVWkQFLD HP TXH VHUmR IRUQHFLGRV
HOHPHQWRV LQIRUPDo}HV H HVFODUHFLPHQWRV UHODWLYRV j
,; GHVSDFKRVGHDQXODomRRXGHUHYRJDomRGDOLFLWDomR OLFLWDomRHjVFRQGLo}HVSDUDDWHQGLPHQWRGDVREULJD
TXDQGRIRURFDVRIXQGDPHQWDGRFLUFXQVWDQFLDGDPHQ o}HVQHFHVViULDVDRFXPSULPHQWRGHVHXREMHWLYR
WH
,; FRQGLo}HVHTXLYDOHQWHVGHSDJDPHQWRHQWUHHPSUHVDV
; WHUPRGHFRQWUDWRRXLQVWUXPHQWRHTXLYDOHQWHFRQIRU EUDVLOHLUDV H HVWUDQJHLUDV QR FDVR GH OLFLWDo}HV
LQWHUQDFLRQDLV
PHRFDVR
; RFULWpULRGHDFHLWDELOLGDGHGRVSUHoRVXQLWiULRHJOREDO
;, RXWURVFRPSURYDQWHVGHSXEOLFDo}HV
FRQIRUPHRFDVRSHUPLWLGDDIL[DomRGHSUHoRVPi[L
PRVHYHGDGRVDIL[DomRGHSUHoRVPtQLPRVFULWpULRV
;,, GHPDLVGRFXPHQWRVUHODWLYRVDOLFLWDomR
HVWDWtVWLFRVRXIDL[DVGHYDULDomRHPUHODomRDSUHoRVGH
UHIHUrQFLDUHVVDOYDGRRGLVSRVWRQRV††žHžGRDUW
†~QLFR $VPLQXWDVGRVHGLWDLVGHOLFLWDomREHPFRPRDVGRV
2EVFRPUHGDomRGDGDSHOD/HLQž
FRQWUDWRV DFRUGRV FRQYrQLRV RX DMXVWHV GHYHP VHU
SUHYLDPHQWHH[DPLQDGDVHFRPSURYDGDVSRUDVVHVVRULD ;, FULWpULR GH UHDMXVWH TXH GHYHUi UHWUDWDUDYDULDomR
MXUtGLFDGD$GPLQLVWUDomR2EVFRPUHGDomRGDGDSHOD/HLQž HIHWLYDGRFXVWRGHUHSURGXomRDGPLWLGDDDGRomRGH
tQGLFHVHVSHFtILFRVRXVHWRULDLVGHVGHDGDWDSUHYLVWD
$UW 6HPSUH TXH R YDORU HVWLPDGR SDUD XPD OLFLWDomRRX
SDUDDSUHVHQWDomRGDSURSRVWDRXGRRUoDPHQWRDTXH
SDUDXPFRQMXQWRGHOLFLWDo}HVVLPXOWkQHDVRXVXFHVVL
HVVDSURSRVWDVHUHIHULUDWpDGDWDGRDGLPSOHPHQWRGH
YDVIRUVXSHULRUD FHP YH]HVROLPLWHSUHYLVWRQR
FDGDSDUFHOD
DUWLQF,DOtQHDFGHVWD/HLRSURFHVVROLFLWDWyULR 2EVFRPUHGDomRGDGDSHOD/HL
VHUi LQLFLDGR REULJDWRULDPHQWH FRP XPD DXGLrQFLD ;,, YHWDGR 
S~EOLFD FRQFHGLGD SHOD DXWRULGDGH UHVSRQViYHO FRP
DQWHFHGrQFLDPtQLPDGH TXLQ]H GLDV~WHLVGDGDWD ;,,,OLPLWHV SDUD SDJDPHQWR GH LQVWDODomR HPRELOL]DomR
SDUDH[HFXomRGHREUDVRXVHUYLoRVTXHVHUmRREULJDWR
SUHYLVWDSDUDSXEOLFDomRGRHGLWDOHGLYXOJDGDFRPD
ULDPHQWHSUHYLVWRVHPVHSDUDGRGDVGHPDLVSDUFHODV
DQWHFHGrQFLD PtQLPD GH  GH]  GLDV ~WHLV GH VXD
HWDSDVRXWDUHIDV
UHDOL]DomRSHORVPHVPRVPHLRVSUHYLVWRVSDUDSXEOLFL
GDGHGDOLFLWDomRjTXDOWHUmRDFHVVRHGLUHLWRDWRGDV
;,9FRQGLo}HVGHSDJDPHQWRSUHYHQGR
LQIRUPDo}HVSHUWLQHQWHVHDVHPDQLIHVWDURVLQWHUHVVD
GRV D SUD]R GH SDJDPHQWR QmR VXSHULRU D WULQWD GLDV
FRQWDGRDSDUWLUGDGDWDILQDOGRSHUtRGRGHDGLP
†~QLFR 3DUD RV ILQV GHVWH DUWLJRFRQVLGHUDPVHOLFLWDo}HV SOHPHQWRGHFDGDSDUFHOD2EVFRPUHGDomRGDGDSHOD/HLQž
VLPXOWkQHDV DTXHODV FRP REMHWRV VLPLODUHV H FRP 
UHDOL]DomR SUHYLVWD SDUD LQWHUYDORV QmR VXSHULRUHV D
E FURQRJUDPDGHGHVHPEROVRPi[LPRSRUSHUtRGR
WULQWD GLDV H OLFLWDo}HV VXFHVVLYDV DTXHODV HP TXH
HPFRQIRUPLGDGHFRPDGLVSRQLELOLGDGHGHUHFXU
WDPEpP FRP REMHWRV VLPLODUHV R HGLWDO VXEVHTHQWH
VRVILQDQFHLURV
WHQKD XPD GDWD DQWHULRU D FHQWR H YLQWH GLDV DSyV R
WpUPLQRGRFRQWUDWRUHVXOWDQWHGDOLFLWDomRDQWHFHGHQWH F FULWpULR GH DWXDOL]DomR ILQDQFHLUD GRV YDORUHV D
2EVFRPUHGDomRGDGDSHOD/HLQž
VHUHP SDJRV GHVGH D GDWD ILQDO GR SHUtRGR GH
$UW 2HGLWDOFRQWHUiQRSUHkPEXORRQ~PHURGHRUGHPHP DGLPSOHPHQWR GH FDGD SDUFHOD DWp D GDWD GR
VpULHDQXDORQRPHGDUHSDUWLomRLQWHUHVVDGDHGHVHX HIHWLYRSDJDPHQWR2EVFRPUHGDomRGDGDSHOD/HLQž
VHWRUDPRGDOLGDGHRUHJLPHGHH[HFXomRHRWLSRGD
OLFLWDomRDPHUomRGHTXHVHUiUHJLGDSRUHVWD/HLR G FRPSHQVDo}HV ILQDQFHLUDV H SHQDOL]Do}HV SRU
ORFDOGLDHKRUDSDUDUHFHELPHQWRGDGRFXPHQWDomRH HYHQWXDLV DWUDVRV H GHVFRQWRV SRU HYHQWXDLV
SURSRVWDEHPFRPRSDUDLQtFLRGDDEHUWXUDGRVHQYHOR DQWHFLSDo}HVGHSDJDPHQWRV
SHVHLQGLFDUiREULJDWRULDPHQWHRVHJXLQWH
H H[LJrQFLDVGHVHJXURVTXDQGRIRURFDVR
, REMHWRGDOLFLWDomRHPGHVFULomRVXVFLQWDHFODUD
;9 LQVWUXo}HV H QRUPDV SDUD RV UHFXUVRV SUHYLVWRVQHVWD
/HL
,, SUD]R H FRQGLo}HV SDUD DVVLQDWXUDGRFRQWUDWRRX
UHWLUDGD GRV LQVWUXPHQWRV FRPR SUHYLVWR QR DUW
;9, FRQGLo}HVGHUHFHELPHQWRQRREMHWRGDOLFLWDomR
GHVWD/HLSDUDH[HFXomRGRFRQWUDWRHSDUDHQWUHJDGR
REMHWRGDOLFLWDomR
†ž 2RULJLQDOGRHGLWDOGHYHUiVHUGDWDGRUXEULFDGRHP
WRGDV DV IROKDV H DVVLQDGR SHOD DXWRULGDGH TXH R
,,, VDQo}HVSDUDRFDVRGHLQDGLPSOHPHQWR H[SHGLUSHUPDQHFHQGRQRSURFHVVRGHOLFLWDomRHGHOH
H[WUDLQGRVH FySLDV LQWHJUDLV RX UHVXPLGDV SDUD VXD
,9 ORFDORQGHSRGHUiVHUH[DPLQDGRHDGTXLULGRRSURMHWR GLYXOJDomRHIRUQHFLPHQWRDRVLQWHUHVVDGRV
EiVLFR
 'LU$GPLQLVWUDWLYR
 &kPDUD/HJLVODWLYDGR'LVWULWR)HGHUDO
†ž &RQVWLWXHP DQH[RV GR HGLWDO GHOHID]HQGRSDUWH †ž 2SDJDPHQWRIHLWRDROLFLWDQWHEUDVLOHLURHYHQWXDOPHQWH
LQWHJUDQWH FRQWUDWDGR HP YLUWXGH GD OLFLWDomR GH TXH WUDWD R
SDUiJUDIRDQWHULRUVHUiHIHWXDGRHPPRHGDEUDVLOHLUDj
, RSURMHWREiVLFRHRH[HFXWLYRFRPWRGDVDVVXDVSDUWHV WD[D GH FkPELR YLJHQWH QR GLD ~WLO LPHGLDWDPHQWH
GHVHQKRVHVSHFLILFDo}HVHRXWURVFRPSOHPHQWRV DQWHULRUDGDWDGRHIHWLYRSDJDPHQWR2EVFRPUHGDomRGDGDSHOD
/HLQž

,, RUoDPHQWR HVWLPDGR HP SODQLOKDV GHTXDQWLWDWLYRVH †ž $VJDUDQWLDVGHSDJDPHQWRDROLFLWDQWHEUDVLOHLURVHUmR


SUHoRVXQLWiULRV 2EVUHGDomRGDGDSHOD/HLQž HTXLYDOHQWHVjTXHODVRIHUHFLGDVDROLFLWDQWHHVWUDQJHLUR
,,, DPLQXWDGRFRQWUDWRDVHUILUPDGRHQWUHDDGPLQLVWUD †ž 3DUD ILQV GH MXOJDPHQWR GD OLFLWDomRDVSURSRVWDV
omRHROLFLWDQWHYHQFHGRU DSUHVHQWDGDV SRU OLFLWDQWHV HVWUDQJHLURV VHUmR DFUHV
FLGDVGRVJUDYDPHVFRQVHTHQWHVGRVPHVPRVWULEXWRV
,9 DV HVSHFLILFDo}HV FRPSOHPHQWDUHV H DV QRUPDVGH TXH RQHUDP H[FOXVLYDPHQWH RV OLFLWDQWHV EUDVLOHLURV
H[HFXomRSHUWLQHQWHVjOLFLWDomR TXDQWRjRSHUDomRILQDOGHYHQGD
†ž 3DUDHIHLWRGHGLVSRVWRQHVWD/HLFRQVLGHUDVHDGLPSOH †ž 3DUD D UHDOL]DomR GH REUDV SUHVWDomR GHVHUYLoRVRX
PHQWRGDREULJDomRFRQWUDWXDODSUHVWDomRGRVHUYLoRD DTXLVLomRGHEHQVFRPUHFXUVRVSURYHQLHQWHVGHILQDQFL
UHDOL]DomR GD REUD D HQWUHJD GR EHP RX GH SDUFHOD DPHQWR RX GRDomR RULXQGRV GH DJrQFLD RILFLDO GH
GHVWHVEHPFRPRTXDOTXHURXWURHYHQWRFRQWUDWXDOD FRRSHUDomRHVWUDQJHLUDRXRUJDQLVPRILQDQFHLURPXOWL
FXMDRFRUUrQFLDHVWHMDYLQFXODGDDHPLVVmRGHGRFXPHQ ODWHUDOGHTXHR%UDVLOVHMDSDUWHSRGHUmRVHUDGPLWLGDV
WRGHFREUDQoD QD UHVSHFWLYD OLFLWDomR DV FRQGLo}HV GHFRUUHQWHV GH
DFRUGRVSURWRFRORVFRQYHQo}HVRXWUDWDGRVLQWHUQDFLR
†ž 1DVFRPSUDVSDUDHQWUHJDLPHGLDWDDVVLPHQWHQGLGDV QDLVDSURYDGDVSHOR&RQJUHVVR1DFLRQDOEHPFRPRDV
DTXHODVFRPSUD]RGHHQWUHJDDWpWULQWDGLDVGDGDWD QRUPDVHSURFHGLPHQWRVGDTXHODVHQWLGDGHVLQFOXVLYH
SUHYLVWDSDUDDDSUHVHQWDomRGDSURSRVWDSRGHUmRVHU TXDQWRDRFULWpULRGHVHOHomRGDSURSRVWDPDLVYDQWDMR
GLVSHQVDGRV VD SDUD D $GPLQLVWUDomR R TXDO SRGHUi FRQWHPSODU
DOpPGRSUHoRRXWURVIDWRUHVGHDYDOLDomRGHVGHTXH
, RGLVSRVWRQRLQFLVR;,GHVWHDUWLJR SRUHODVH[LJLGRVSDUDDREWHQomRGRILQDQFLDPHQWRRX
GDGRDomRHTXHWDPEpPQmRFRQIOLWHPFRPRSULQFtSLR
,, D DWXDOL]DomR ILQDQFHLUD D TXH UHIHUH D DOtQHD FGR GR MXOJDPHQWR REMHWLYR H VHMD REMHWR GH GHVSDFKR
LQFLVR ;,9 GHVWH DUWLJR FRUUHVSRQGHQWH DR SHUtRGR PRWLYDGRGRyUJmRH[HFXWRUGRFRQWUDWRGHVSDFKRHVVH
FRPSUHHQGLGR HQWUH DV GDWDV GR DGLPSOHPHQWR H D UDWLILFDGRSHODDXWRULGDGHLPHGLDWDPHQWHVXSHULRU2EV
SUHYLVWDSDUDRSDJDPHQWRGHVGHTXHQmRVXSHULRUD FRPUHGDomRGDGDSHOD/HLQž

TXLQ]HGLDV
2EV(VWH†HVHXVLQFLVRVIRUDPLQFOXtGRVSHOD/HL †ž $VFRWDo}HVGHWRGRVRVOLFLWDQWHVVHUmRSDUDHQWUHJDQR
PHVPRORFDOGHGHVWLQR
$UW $ $GPLQLVWUDomR QmR SRGH GHVFXPSULU DV QRUPDVH
FRQGLo}HV GR HGLWDO DR TXDO VH DFKD HVWULWDPHQWH $UW $OLFLWDomRVHUiSURFHVVDGDHMXOJDGDFRPREVHUYkQFLD
YLQFXODGD GRVVHJXLQWHVSURFHGLPHQWRV
†ž 4XDOTXHUFLGDGmRpSDUWHOHJtWLPDSDUDLPSXJQDUHGLWDO , DEHUWXUD GRVHQYHORSHVFRQWHQGRDGRFXPHQWDomR
GHOLFLWDomRSRULUUHJXODULGDGHQDDSOLFDomRGHVWD/HL UHODWLYDjKDELOLWDomRGRVFRQFRUUHQWHVHVXDDSUHFLD
GHYHQGRSURWRFRODURSHGLGR DWp FLQFR GLDV~WHLV omR
DQWHV GD GDWD IL[DGD SDUD DEHUWXUD GRV HQYHORSHV GH
KDELOLWDomRGHYHQGRD$GPLQLVWUDomRMXOJDUHUHVSRQ ,, GHYROXomR GRV HQYHORSHVIHFKDGRVDRVFRQFRUUHQWHV
GHU j LPSXJQDomR HP DWp  WUrV  GLDV ~WHLV VHP LQDELOLWDGRVFRQWHQGRDVUHVSHFWLYDVSURSRVWDVGHVGH
SUHMXt]RGDIDFXOGDGHSUHYLVWDQR†žGRDUW TXHQmRWHQKDKDYLGRUHFXUVRRXDSyVVXDGHQHJDomR

†ž 'HFDLUiGRGLUHLWRGHLPSXJQDURVWHUPRVGRHGLWDOGH ,,, DEHUWXUDGRVHQYHORSHVIHFKDGRVFRQWHQGRDVSURSRVWDV


OLFLWDomRSHUDQWHD$GPLQLVWUDomRROLFLWDQWHTXHQmRR GRVFRQFRUUHQWHVKDELOLWDGRVGHVGHTXHWUDQVFRUULGRR
IL]HUDWpRVHJXQGRGLD~WLOTXHDQWHFHGHUDDEHUWXUD SUD]R VHP LQWHUSRVLomR GH UHFXUVR RX WHQKD KDYLGR
GRVHQYHORSHVGHKDELOLWDomRHPFRQFRUUrQFLDDDEHUWX GHVLVWrQFLDH[SUHVVDRXDSyVRMXOJDPHQWRGRVUHFXUVRV
UDGRVHQYHORSHVFRPDVSURSRVWDVHPFRQYLWHWRPDGD LQWHUSRVWRV
GH SUHoRV RX FRQFXUVR RX D UHDOL]DomR GH OHLOmR DV
IDOKDV RX LUUHJXODULGDGHV TXH YLFLDULDP HVVH HGLWDO ,9 YHULILFDomRGDFRQIRUPLGDGHGHFDGDSURSRVWDFRPRV
KLSyWHVH HP TXH WDO FRPXQLFDomR QmR WHUi HIHLWR GH UHTXLVLWRVGRHGLWDOHFRQIRUPHRFDVRFRPRVSUHoRV
UHFXUVR 2EVFRPUHGDomRGDGDSHOD/HLQž FRUUHQWHV QR PHUFDGR RX IL[DGRV SRU yUJmR RILFLDO
FRPSHWHQWHRXDLQGDFRPRVFRQVWDQWHVGRVVLVWHPDGH
†ž $LPSXJQDomRIHLWDWHPSHVWLYDPHQWHSHOROLFLWDQWHQmR UHJLVWURGHSUHoRVRVTXDLVGHYHUmRVHUGHYLGDPHQWH
RLPSHGLUiGHSDUWLFLSDUGRSURFHVVROLFLWDWyULRDWpR UHJLVWUDGRV QD DWD GH MXOJDPHQWR SURPRYHQGRVH D
WUkQVLWRHPMXOJDGRGDGHFLVmRDHODSHUWLQHQWH GHVFODVVLILFDomR GDV SURSRVWDV GHVFRQIRUPHV RX
LQFRPSDWtYHLV
†ž $ LQDELOLWDomR GR OLFLWDQWH LPSRUWD SUHFOXVmRGRVHX
GLUHLWRGHSDUWLFLSDUGDVIDVHVVXEVHTHQWHV 9 MXOJDPHQWRHFODVVLILFDomRGDVSURSRVWDVGHDFRUGRFRP
RFULWpULRGHDYDOLDomRFRQVWDQWHVGRHGLWDO
$UW 1DV FRQFRUUrQFLDV GH kPELWR LQWHUQDFLRQDO RHGLWDO
GHYHUiDMXVWDUVHjVGLUHWUL]HVGDSROtWLFDPRQHWiULDH 9, GHOLEHUDomRGDDXWRULGDGHFRPSHWHQWHTXDQWRjKRPR
GRFRPpUFLRH[WHULRUjVH[LJrQFLDVGRVyUJmRVFRPSH ORJDomRHDGMXGLFDomRGRREMHWRGDOLFLWDomR
WHQWHV
†ž $ DEHUWXUD GRV HQYHORSHV FRQWHQGRDGRFXPHQWDomR
†ž 4XDQGR IRU SHUPLWLGR DR OLFLWDQWHHVWUDQJHLURFRWDU SDUDKDELOLWDomRHDVSURSRVWDVVHUiUHDOL]DGDVHPSUH
SUHoRHPPRHGDHVWUDQJHLUDLJXDOPHQWHRSRGHUiID]HU HP DWR S~EOLFR SUHYLDPHQWH GHVLJQDGR GR TXDO VH
ROLFLWDQWHEUDVLOHLUR ODYUDUiDDWDFLUFXQVWDQFLDGDDVVLQDGDSHORVOLFLWDQWHV
SUHVHQWHVHSHOD&RPLVVmR
 &kPDUD/HJLVODWLYDGR
 'LVWULWR)HGHUDO 'LU$GPLQLVWUDWLYR 

†ž 7RGRV RV GRFXPHQWRV H SURSRVWDVVHUmRUXEULFDGRV ,,, DGHWpFQLFDHSUHoR


SHORVOLFLWDQWHVSUHVHQWHVHSHOD&RPLVVmR ,9 DGHPDLRUODQFHRXRIHUWDQRVFDVRVGHDOLHQDomRGH
EHQVRXFRQFHVVmRGHGLUHLWRUHDOGHXVR
†ž e IDFXOWDGD j &RPLVVmR RX DXWRULGDGHVXSHULRUHP 2EVLQFOXtGRSHOD/HL
TXDOTXHU IDVH GD OLFLWDomR D SURPRomR GH GLOLJrQFLD
GHVWLQDGDDHVFODUHFHURXDFRPSOHPHQWDUDLQVWUXomR †ž 1R FDVR GH HPSDWH HQWUH GXDV RX PDLVSURSRVWDVH
GRSURFHVVRYHGDGDDLQFOXVmRSRVWHULRUGHGRFXPHQWR DSyVREHGHFLGRRGLVSRVWRQR†žGRDUWžGHVWD/HL
RXLQIRUPDomRTXHGHYHULDFRQVWDURULJLQDULDPHQWHGD D FODVVLILFDomR VH IDUi REULJDWRULDPHQWH SRU VRUWHLR
SURSRVWD HP DWR S~EOLFR SDUD R TXDO WRGRV RV OLFLWDQWHV VHUmR
FRQYRFDGRVYHGDGRTXDOTXHURXWURSURFHVVR
†ž 2GLVSRVWRQHVWHDUWLJRDSOLFDVHjFRQFRUUrQFLDHQR
TXHFRXEHUDRFRQFXUVRDROHLOmRjWRPDGDGHSUHoRV †ž 1R FDVR GD OLFLWDomR GR WLSR PHQRU SUHoRHQWUHRV
HDRFRQYLWH2EVFRPUHGDomRGDGDSHOD/HLQž OLFLWDQWHV FRQVLGHUDGRV TXDOLILFDGRV D FODVVLILFDomR VH
IDUiSHODRUGHPFUHVFHQWHGRVSUHoRVSURSRVWRVSUHYD
†ž 8OWUDSDVVDGD D IDVH GH KDELOLWDomRGRVFRQFRUUHQWHV OHFHQGRQRFDVRGHHPSDWHH[FOXVLYDPHQWHRFULWpULR
LQFLVRV,H,, HDEHUWDVDVSURSRVWDV LQFLVR,,, QmR SUHYLVWRQRSDUiJUDIRDQWHULRU 2EV FRP UHGDomR GDGD SHOD /HL Qž

FDEH GHVFODVVLILFiORV SRU PRWLYR UHODFLRQDGR FRP D
KDELOLWDomRVDOYRHPUD]mRGHIDWRVVXSHUYHQLHQWHVRX †ž 3DUDFRQWUDWDomRGHEHQVHVHUYLoRVGHLQIRUPiWLFDD
VyFRQKHFLGRVDSyVRMXOJDPHQWR $GPLQLVWUDomR3~EOLFDREVHUYDUiRGLVSRVWRQRDUWžGD
/HLQžGHGHRXWXEURGHOHYDQGRHP
†ž $SyV D IDVH GH KDELOLWDomR QmR FDEHGHVLVWrQFLDGH FRQWDRVIDWRUHVHVSHFLILFDGRVHPVHX†žHDGRWDQGR
SURSRVWD VDOYR SRU PRWLYR MXVWR GHFRUUHQWH GH IDWR REULJDWRULDPHQWH R WLSR GH OLFLWDomR WpFQLFD H SUHoR
VXSHUYHQLHQWHHDFHLWRSHOD&RPLVVmR SHUPLWLQGR R HPSUHJR GH RXWUR WLSR GH OLFLWDomR QRV
FDVRVLQGLFDGRVHP'HFUHWRGR3RGHU([HFXWLYR
$UW 1R MXOJDPHQWR GDV SURSRVWDV D &RPLVVmR OHYDUiHP
2EVFRPUHGDomRGDGDSHOD/HLQž
FRQVLGHUDomRRVFULWpULRVREMHWLYRVGHILQLGRVQRHGLWDO
RXFRQYLWHRVTXDLVQmRGHYHPFRQWUDULDUDVQRUPDVH †ž eYHGDGDDXWLOL]DomRGHRXWURVWLSRVGHOLFLWDomRQmR
SULQFtSLRVHVWDEHOHFLGRVSRUHVWD/HL SUHYLVWRVQHVWHDUWLJR

†ž eYHGDGDDXWLOL]DomRGHTXDOTXHUHOHPHQWRFULWpULRRX †ž 1DKLSyWHVHSUHYLVWDQRDUW†žVHUmRVHOHFLRQDGDV


IDWRUVLJLORVRVHFUHWRVXEMHWLYRRXUHVHUYDGRTXHSRVVD WDQWDVSURSRVWDVTXDQWDVQHFHVViULDVDWpTXHVHDWLQMD
DLQGDTXHLQGLUHWDPHQWHHOLGLURSULQFtSLRGDLJXDOGDGH DTXDQWLGDGHGHPDQGDGDQDOLFLWDomR2EV†žDFUHVFHQWDGRSHOD
/HLQž
HQWUHRVOLFLWDQWHV
$UW 2V WLSRV GH OLFLWDomR PHOKRU WpFQLFD RX WpFQLFDH
†ž 1mRVHFRQVLGHUDUiTXDOTXHURIHUWDGHYDQWDJHPQmR SUHoRVHUmRXWLOL]DGRVH[FOXVLYDPHQWHSDUDVHUYLoRVGH
SUHYLVWDQRHGLWDORXQRFRQYLWHLQFOXVLYHILQDQFLDPHQ QDWXUH]DSUHGRPLQDQWHPHQWHLQWHOHFWXDOHPHVSHFLDO
WR VXEVLGLDGRV RX D IXQGR SHUGLGR QHP SUHoR RX QDHODERUDomRGHSURMHWRVFiOFXORVILVFDOL]DomRVXSHU
YDQWDJHPEDVHDGDQDVRIHUWDVGRVGHPDLVOLFLWDQWHV YLVmRHJHUHQFLDPHQWRHGHHQJHQKDULDFRQVXOWLYDHP
JHUDOHHPSDUWLFXODUSDUDDHODERUDomRGHHVWXGRV
†ž 1mRVHDGPLWLUiSURSRVWDTXHDSUHVHQWHSUHoRVJOREDO WpFQLFRVSUHOLPLQDUHVHSURMHWRVEiVLFRVHH[HFXWLYRV
RX XQLWiULRV VLPEyOLFRV LUULVyULRV RX GH YDORU ]HUR UHVVDOYDGRRGLVSRVWRQR†žGRDUWLJRDQWHULRU
LQFRPSDWtYHLVFRPRVSUHoRVGRVLQVXPRVHVDOiULRVGH
2EVFRPUHGDomRGDGDSHOD/HLQž
PHUFDGR DFUHVFLGRV GRV UHVSHFWLYRV HQFDUJRV DLQGD
TXHRDWRFRQYRFDWyULRGDOLFLWDomRQmRWHQKDHVWDEHOH †ž 1DVOLFLWDo}HVGRWLSRPHOKRUWpFQLFDVHUiDGRWDGRR
FLGR OLPLWHV PtQLPRV H[FHWR TXDQGR VH UHIHULUHP D VHJXLQWH SURFHGLPHQWR FODUDPHQWH H[SOLFLWDGR QR
PDWHULDLVHLQVWDODo}HVGHSULRULGDGHGHSUySULROLFLWDQ LQVWUXPHQWRFRQYRFDWyULRRTXDOIL[DUiRSUHoRPi[L
WHSDUDRVTXDLVHOHUHQXQFLHDSDUFHODRXjWRWDOLGDGH PRTXHD$GPLQLVWUDomRVHSURS}HDSDJDU
GDUHPXQHUDomR 2EVUHGDomRGDGDSHOD/HLQž
, VHUmR DEHUWRV RV HQYHORSHVFRQWHQGRDVSURSRVWDV
†ž 2GLVSRVWRQRSDUiJUDIRDQWHULRUVHDSOLFDWDPEpPD WpFQLFDV H[FOXVLYDPHQWH GRV OLFLWDQWHV SUHYLDPHQWH
SURSRVWDV TXH LQFOXDP PmRGHREUD HVWUDQJHLUD RX TXDOLILFDGRV H IHLWD HQWmR D DYDOLDomR H FODVVLILFDomR
LPSRUWDo}HVGHTXDOTXHUQDWXUH]D2EVFRPUHGDomRGDGDSHOD/HL GHVWDVSURSRVWDVGHDFRUGRFRPRVFULWpULRVSHUWLQHQWHV
Qž
HDGHTXDGRVDRREMHWROLFLWDGRGHILQLGRVFRPFODUH]DH
$UW 2MXOJDPHQWRGDVSURSRVWDVVHUiREMHWLYRGHYHQGRD REMHWLYLGDGHQRLQVWUXPHQWRFRQYRFDWyULRHTXHFRQVL
&RPLVVmR GH OLFLWDomR RX R UHVSRQViYHO SHOR FRQYLWH GHUHPDFDSDFLWDomRHDH[SHULrQFLDGRSURSRQHQWHD
UHDOL]iORHPFRQIRUPLGDGHFRPRVWLSRVGHOLFLWDomRRV TXDOLGDGHWpFQLFDGDSURSRVWDFRPSUHHQGHQGRPHWR
FULWpULRVSUHYLDPHQWHHVWDEHOHFLGRVQRDWRFRQYRFDWyULR GRORJLDRUJDQL]DomRWHFQRORJLDVHUHFXUVRVPDWHULDLV
HGHDFRUGRFRPRVIDWRUHVH[FOXVLYDPHQWHQHOHUHIHUL DVHUHPXWLOL]DGRVQRVWUDEDOKRVHDTXDOLILFDomRGDV
GRVGHPDQHLUDDSRVVLELOLWDUVXDDIHULomRSHORVOLFLWDQ HTXLSHVWpFQLFDVDVHUHPPRELOL]DGDVSDUDDVXDH[HFX
WHVHSHORVyUJmRVGHFRQWUROH omR

†ž 3DUDRVHIHLWRVGHVWHDUWLJRFRQVWLWXHPWLSRVGHOLFLWD ,, XPDYH]FODVVLILFDGDVDVSURSRVWDVWpFQLFDVSURFHGHU


omRH[FHWRQDPRGDOLGDGHGHFRQFXUVRHOHLOmR2EVFRP VHi j DEHUWXUD GDV SURSRVWDV GH SUHoR GRV OLFLWDQWHV
UHGDomRGDGDSHOD/HLQž
TXHWHQKDPDWLQJLGRDYDORUL]DomRPtQLPDHVWDEHOHFLGD
, D GH PHQRU SUHoR  TXDQGR R FULWpULRGHVHOHomRGD QRLQVWUXPHQWRFRQYRFDWyULRHjQHJRFLDomRGDVFRQGL
SURSRVWDPDLVYDQWDMRVDSDUDD$GPLQLVWUDomRGHWHUPL o}HVSURSRVWDVFRPDSURSRQHQWHPHOKRUFODVVLILFDGD
QDU TXH VHUi YHQFHGRU R OLFLWDQWH TXH DSUHVHQWDU D FRP EDVH QRV RUoDPHQWRV GHWDOKDGRV DSUHVHQWDGRV H
SURSRVWDGHDFRUGRFRPDVHVSHFLILFDo}HVGRHGLWDORX UHVSHFWLYRVSUHoRVXQLWiULRVHWHQGRFRPRUHIHUrQFLDR
FRQYLWHHRIHUWDURPHQRUSUHoR OLPLWHUHSUHVHQWDGRSHODSURSRVWDGHPHQRUSUHoRHQWUH
RVOLFLWDQWHVTXHREWLYHUDPDYDORUL]DomRPtQLPD
,, DGHPHOKRUWpFQLFD
 'LU$GPLQLVWUDWLYR
 &kPDUD/HJLVODWLYDGR'LVWULWR)HGHUDO
,,, QR FDVR GH LPSDVVH QD QHJRFLDomR DQWHULRUSURFHGL D PpGLDDULWPpWLFDGRVYDORUHVGDVSURSRVWDVVXSHULRUHV
PHQWRLGrQWLFRVHUiDGRWDGRVXFHVVLYDPHQWHFRPRV D  FLQTHQWD SRU FHQWR  GR YDORU RUoDGR SHOD
GHPDLVSURSRQHQWHVSHODRUGHPGHFODVVLILFDomRDWpD $GPLQLVWUDomRRX
FRQVHFXomRGHDFRUGRSDUDDFRQWUDWDomR
E YDORURUoDGRSHODDGPLQLVWUDomR
,9 DV SURSRVWDV GH SUHoRV VHUmR GHYROYLGDV LQWDFWDVDRV
OLFLWDQWHVTXHQmRIRUHPSUHOLPLQDUPHQWHKDELOLWDGRV †ž 'RV OLFLWDQWHV FODVVLILFDGRV QD IRUPDGRSDUiJUDIR
RXTXHQmRREWLYHUHPDYDORUL]DomRPtQLPDHVWDEHOHFL DQWHULRUFXMRYDORUJOREDOGDSURSRVWDIRULQIHULRUD
GDSDUDDSURSRVWDWpFQLFD RLWHQWDSRUFHQWR GRPHQRUYDORUDTXHVHUHIHUHPDV
DOtQHDV D H E VHUi H[LJLGD SDUD D DVVLQDWXUD GR
†ž 1DVOLFLWDo}HVGRWLSRWpFQLFDHSUHoRVHUiDGRWDGR FRQWUDWR SUHVWDomR GH JDUDQWLD DGLFLRQDO GHQWUH DV
DGLFLRQDOPHQWH DR LQFLVR , GR SDUiJUDIR DQWHULRU R PRGDOLGDGHV SUHYLVWDV QR † ž GR DUW  LJXDO D
VHJXLQWH SURFHGLPHQWR FODUDPHQWH H[SOLFLWDGR QR GLIHUHQoDHQWUHRYDORUUHVXOWDQWHGRSDUiJUDIRDQWHULRU
LQVWUXPHQWRFRQYRFDWyULR HRYDORUGDFRUUHVSRQGHQWHSURSRVWD
2EV†žDFUHVFLGRSHOD/HLQž

, VHUiIHLWDDDYDOLDomRHDYDORUL]DomRGDVSURSRVWDVGH
†ž 4XDQGRWRGRVRVOLFLWDQWHVIRUHPLQDELOLWDGRVRXWRGDV
SUHoRVGHDFRUGRFRPFULWpULRVREMHWLYRVSUHHVWDEHOHFL DV SURSRVWDV IRUHP GHVFODVVLILFDGDV D $GPLQLVWUDomR
GRVQRLQVWUXPHQWRFRQYRFDWyULR SRGHUi IL[DU DRV OLFLWDQWHV R SUD]R GH RLWR GLDV ~WHLV
SDUD D DSUHVHQWDomR GH QRYD GRFXPHQWDomR RX GH
,, DFODVVLILFDomRGRVSURSRQHQWHVIDUVHiGHDFRUGRFRP RXWUDVSURSRVWDVHVFRLPDGDVGDVFDXVDVUHIHULGDVQHVWH
D PpGLD SRQGHUDGD GDV YDORUL]Do}HV GDV SURSRVWDV DUWLJRIDFXOWDGDQRFDVRGHFRQYLWHDUHGXomRGHVWH
WpFQLFDVHGHSUHoRGHDFRUGRFRPRVSHVRVSUHHVWDEH SUD]RSDUDWUrVGLDV~WHLV
OHFLGRVQRVLQVWUXPHQWRFRQYRFDWyULR
2EV3ULPLWLYR†~QLFRWUDQVIRUPDGRHP†žSHOD/HLQž

†ž ([FHSFLRQDOPHQWHRVWLSRVGHOLFLWDomRSUHYLVWRVQHVWH $UW $DXWRULGDGHFRPSHWHQWHSDUDDDSURYDomRGRSURFHGL


DUWLJRSRGHUmRVHUDGRWDGRVSRUDXWRUL]DomRH[SUHVVD PHQWRVRPHQWHSRGHUiUHYRJDUDOLFLWDomRSRUUD]}HVGH
H PHGLDQWH MXVWLILFDWLYD FLUFXQVWDQFLDGD GD PDLRU LQWHUHVVH S~EOLFR GHFRUUHQWH GH IDWR VXSHUYHQLHQWH
DXWRULGDGHGD$GPLQLVWUDomRSURPRWRUDFRQVWDQWHGR GHYLGDPHQWHFRPSURYDGRSHUWLQHQWHHVXILFLHQWHSDUD
DWRFRQYRFDWyULRSDUDIRUQHFLPHQWRGHEHQVHH[HFXomR MXVWLILFDUWDOFRQGXWDGHYHQGRDQXOiODSRULOHJDOLGDGH
GH REUDV RX SUHVWDomR GH VHUYLoRV GH JUDQGH YXOWR GH RItFLR RX SRU SURYRFDomR GH WHUFHLURV PHGLDQWH
PDMRULWDULDPHQWHGHSHQGHQWHVGHWHFQRORJLDQLWLGDPHQ SDUHFHUHVFULWRHGHYLGDPHQWHIXQGDPHQWDGR
WH VRILVWLFDGD H GH GRPtQLR UHVWULWR DWHVWDGR SRU
DXWRULGDGHVWpFQLFDVGHUHFRQKHFLGDTXDOLILFDomRQRV †ž $DQXODomRGRSURFHGLPHQWROLFLWDWyULRSRUPRWLYRGH
FDVRV HP TXH R REMHWR SUHWHQGLGR DGPLWLU VROXo}HV LOHJDOLGDGHQmRJHUDREULJDomRGHLQGHQL]DUUHVVDOYDGR
DOWHUQDWLYDVHYDULDo}HVGHH[HFXomRFRPUHSHUFXVV}HV RGLVSRVWRQRSDUiJUDIR~QLFRGRDUWGHVWD/HL
VLJQLILFDWLYDV VREUH VXD TXDOLGDGH SURGXWLYLGDGH
UHQGLPHQWRHGXUDELOLGDGHFRQFUHWDPHQWHPHQVXUiYHLV †ž $ QXOLGDGH GR SURFHGLPHQWR OLFLWDWyULR LQGX]jGR
H HVWDV SXGHUHP VHU DGRWDGDV j OLYUH HVFROKD GRV FRQWUDWRUHVVDOYDGRRGLVSRVWRQRSDUiJUDIR~QLFRGR
OLFLWDQWHVQDFRQIRUPLGDGHGRVFULWpULRVREMHWLYDPHQWH DUWGHVWD/HL
IL[DGRVQRDWRFRQYRFDWyULR
†ž 1R FDVR GH GHVID]LPHQWR GR SURFHVVROLFLWDWyULRILFD
$UW 1DV OLFLWDo}HV SDUD D H[HFXomR GH REUDV HVHUYLoRV DVVHJXUDGRRFRQWUDGLWyULRHDDPSODGHIHVD
TXDQGR IRU DGRWDGD D PRGDOLGDGH GH H[HFXomR GH
HPSUHLWDGDSRUSUHoRJOREDOD$GPLQLVWUDomRGHYHUi †ž 2GLVSRVWRQHVWHDUWLJRHHPVHXVSDUiJUDIRVDSOLFDVH
IRUQHFHUREULJDWRULDPHQWHMXQWRFRPRHGLWDOWRGRVRV DRVDWRVGRSURFHGLPHQWRGHGLVSHQVDHGHLQH[LJLELOL
HOHPHQWRV H LQIRUPDo}HV QHFHVViULRV SDUD TXH RV GDGHGHOLFLWDomR
OLFLWDQWHVSRVVDPHODERUDUVXDVSURSRVWDVGHSUHoRVFRP
WRWDOHFRPSOHWRFRQKHFLPHQWRGRREMHWRGDOLFLWDomR $UW $ $GPLQLVWUDomR QmR SRGHUi FHOHEUDU FRQWUDWRFRP
SUHWHULomRGDRUGHPGHFODVVLILFDomRGDVSURSRVWDVRX
$UW 6HUmRGHVFODVVLILFDGDV FRPWHUFHLURVHVWUDQKRVDRSURFHGLPHQWROLFLWDWyULRVRE
SHQDGHQXOLGDGH
, DV SURSRVWDV TXH QmR DWHQGDP jV H[LJrQFLDVGRDWR
FRQYRFDWyULRGDOLFLWDomR $UW $KDELOLWDomRSUHOLPLQDUDLQVFULomRHPUHJLVWURFDGDV
WUDODVXDDOWHUDomRRXFDQFHODPHQWRHDVSURSRVWDV
VHUmRSURFHVVDGDVHMXOJDGDVSRUFRPLVVmRSHUPDQHQWH
,, SURSRVWDVFRPYDORUJOREDOVXSHULRUDROLPLWHHVWDEHOH
RX HVSHFLDO GH QR PtQLPR  WUrV  PHPEURV VHQGR
FLGRRXFRPSUHoRVPDQLIHVWDPHQWHLQH[HTtYHLVDVVLP
SHOR PHQRV  GRLV  GHOHV VHUYLGRUHV TXDOLILFDGRV
FRQVLGHUDGRVDTXHOHVTXHQmRYHQKDPDWHUGHPRQVWUD
SHUWHQFHQWHVDRVTXDGURVSHUPDQHQWHVGRVyUJmRVGDV
GD VXD YLDELOLGDGH DWUDYpV GH GRFXPHQWDomR TXH
$GPLQLVWUDomRUHVSRQViYHLVSHODOLFLWDomR
FRPSURYHTXHRVFXVWRVGRVLQVXPRVVmRFRHUHQWHVFRP
RVGHPHUFDGRHTXHRVFRHILFLHQWHVGHSURGXWLYLGDGH †ž 1RFDVRGHFRQYLWHD&RPLVVmRGHOLFLWDomRH[FHSFLR
VmRFRPSDWtYHLVFRPDH[HFXomRGRREMHWRGRFRQWUDWR QDOPHQWHQDVSHTXHQDVXQLGDGHVDGPLQLVWUDWLYDVHHP
FRQGLo}HVHVWDVQHFHVVDULDPHQWHHVSHFLILFDGDVQRDWR IDFH GD H[LJLGDGH GH SHVVRDO GLVSRQtYHO SRGHUi VHU
FRQYRFDWyULRGDOLFLWDomR 2EVFRPUHGDomRGDGDSHOD/HLQž VXEVWLWXtGD SRU VHUYLGRU IRUPDOPHQWH GHVLJQDGR SHOD
†ž 3DUD RV HIHLWRV GR GLVSRVWR QR LQFLVR ,,GHVWHDUWLJR DXWRULGDGHFRPSHWHQWH
FRQVLGHUDPVHPDQLIHVWDPHQWHLQH[HTtYHLVQRFDVRGH
OLFLWDo}HV GH PHQRU SUHoR SDUD REUDV H VHUYLoRV GH †ž $&RPLVVmRSDUDMXOJDPHQWRGRVSHGLGRVGHLQVFULomR
HQJHQKDULDDVSURSRVWDVFXMRVYDORUHVVHMDPLQIHULRUHV HPUHJLVWURFDGDVWUDOVXDDOWHUDomRRXFDQFHODPHQWR
D  VHWHQWD SRU FHQWR  GR PHQRU GRV VHJXLQWHV VHUiLQWHJUDGDSRUSURILVVLRQDLVOHJDOPHQWHKDELOLWDGRV
YDORUHV QRFDVRGHREUDVVHUYLoRVRXDTXLVLomRGHHTXLSDPHQ
2EV†žHDOtQHDVDFUHVFLGRVSHOD/HLQž WRV
 &kPDUD/HJLVODWLYDGR
 'LVWULWR)HGHUDO 'LU$GPLQLVWUDWLYR 

†ž 2V PHPEURV GDV &RPLVV}HV GHOLFLWDomRUHVSRQGHUmR †ž 2VFRQWUDWRVGHFRUUHQWHVGHGLVSHQVDRXGHLQH[LJLELOL


VROLGDULDPHQWHSRUWRGRVRVDWRVSUDWLFDGRVSHOD&RPLV GDGHGHOLFLWDomRGHYHPDWHQGHUDRVWHUPRVGRDWRTXH
VmR VDOYR VH SRVLomR LQGLYLGXDO GLYHUJHQWH HVWLYHU RVDXWRUL]RXHGDUHVSHFWLYDSURSRVWD
GHYLGDPHQWHIXQGDPHQWDGDHUHJLVWUDGDHPDWDODYUDGD
QDUHXQLmRHPTXHWLYHUVLGRWRPDGDDGHFLVmR $UW 6mR FOiXVXODV QHFHVViULDV HP WRGR FRQWUDWR DVTXH
HVWDEHOHoDP
†ž $LQYHVWLGXUDGRVPHPEURVGDV&RPLVV}HVSHUPDQHQWHV
QmRH[FHGHUiD XP DQRYHGDGDDUHFRQGXomRGD , RREMHWRHVHXVHOHPHQWRVFDUDFWHUtVWLFRV
WRWDOLGDGHGHVHXVPHPEURVSDUDDPHVPDFRPLVVmRQR
SHUtRGRVXEVHTHQWH ,, RUHJLPHGHH[HFXomRRXDIRUPDGHIRUQHFLPHQWR

†ž 1RFDVRGHFRQFXUVRRMXOJDPHQWRVHUiIHLWRSRUXPD ,,, RSUHoRHDVFRQGLo}HVGHSDJDPHQWRRVFULWpULRVGDWD


FRPLVVmRHVSHFLDOLQWHJUDGDSRUSHVVRDVGHUHSXWDomR EDVH H SHULRGLFLGDGH GR UHDMXVWDPHQWR GH SUHoRV RV
LOLEDGD H UHFRQKHFLGR FRQKHFLPHQWR GD PDWpULD HP FULWpULRV GH DWXDOL]DomR PRQHWiULD HQWUH D GDWD GR
H[DPHVHUYLGRUHVS~EOLFRVRXQmR DGLPSOHPHQWRGDVREULJDo}HVHDGRHIHWLYRSDJDPHQ
WR
$UW 2FRQFXUVRDTXHVHUHIHUHR†žGRDUWGHVWD/HL
GHYHVHUSUHFHGLGRGHUHJXODPHQWRSUySULRDVHUREWLGR ,9 RVSUD]RVGHLQtFLRGHHWDSDVGHH[HFXomRGHFRQFOXVmR
SHORVLQWHUHVVDGRVQRORFDOLQGLFDGRQRHGLWDO GHHQWUHJDGHREVHUYDomRHGHUHFHELPHQWRGHILQLWLYR
FRQIRUPHRFDVR
†ž 2UHJXODPHQWRGHYHUiLQGLFDU
9 RFUpGLWRSHORTXDOFRUUHUiDGHVSHVDFRPDLQGLFDomR
GDFODVVLILFDomRIXQFLRQDOSURJUDPiWLFDHGDFDWHJRULD
, DTXDOLILFDomRH[LJLGDGRVSDUWLFLSDQWHV
HFRQ{PLFD
,, DVGLUHWUL]HVHDIRUPDGHDSUHVHQWDomRGRWUDEDOKR
,,, DVFRQGLo}HVGHUHDOL]DomRGRFRQFXUVRHRVSUrPLRVD 9, DV JDUDQWLDV RIHUHFLGDV SDUD DVVHJXUDUVXDSOHQD
VHUHPFRQFHGLGRV H[HFXomRTXDQGRH[LJLGDV
†ž (PVHWUDWDQGRGHSURMHWRRYHQFHGRUGHYHUiDXWRUL]DU 9,, RVGLUHLWRVHDVUHVSRQVDELOLGDGHVGDVSDUWHVDVSHQDOL
D$GPLQLVWUDomRDH[HFXWiORTXDQGRMXOJDUFRQYHQLHQ GDGHVFDEtYHLVHRVYDORUHVGDVPXOWDV
WH
9,,, RVFDVRVGHUHVFLVmR
$UW 2 OHLOmR SRGH VHU VXEPHWLGR D OHLORHLUR RILFLDO RXD
VHUYLGRUGHVLJQDGRSHOD$GPLQLVWUDomRSURFHGHQGRVH ,; R UHFRQKHFLPHQWR GRV GLUHLWRV GD$GPLQLVWUDomRHP
QDIRUPDGDOHJLVODomRSHUWLQHQWH FDVRGHUHVFLVmRDGPLQLVWUDWLYDSUHYLVWDQRDUWGHVWD
/HL
†ž 7RGREHPDVHUOHLORDGRVHUiSUHYLDPHQWHDYDOLDGRSHOD
$GPLQLVWUDomRSDUDIL[DomRGRSUHoRPtQLPRGHDUUH ; DVFRQGLo}HVGHLPSRUWDomRDGDWDHDWD[DGHFkPELR
PDWDomR SDUDFRQYHUVmRTXDQGRIRURFDVR

†ž 2VEHQVDUUHPDWDGRVVHUmRSDJRVjYLVWDRXQRSHUFHQ ;, DYLQFXODomRDRHGLWDOGHOLFLWDomRRXDRWHUPRTXHD


WXDOHVWDEHOHFLGRQRHGLWDOQmRLQIHULRUD FLQFRSRU GLVSHQVRX RX D LQH[LJLX DR FRQYLWH H j SURSRVWD GR
FHQWR HDSyVDDVVLQDWXUDGDUHVSHFWLYDDWDODYUDGDQR OLFLWDQWHYHQFHGRU
ORFDOGROHLOmRLPHGLDWDPHQWHHQWUHJXHVDRDUUHPDWDQ
WH R TXDO VH REULJDUi DR SDJDPHQWR GR UHVWDQWH QR ;,, DOHJLVODomRDSOLFiYHOjH[HFXomRGRFRQWUDWRHVSHFLDO
SUD]RHVWLSXODGRQRHGLWDOGHFRQYRFDomRVRESHQDGH PHQWHDRVFDVRVRPLVVRV
SHUGHUHPIDYRUGD$GPLQLVWUDomRRYDORUMiUHFROKLGR
;,,, D REULJDomR GR FRQWUDWR GH PDQWHU GXUDQWH WRGDD
†ž 1RVOHLO}HVLQWHUQDFLRQDLVRSDJDPHQWRGDSDUFHODj H[HFXomR GR FRQWUDWR HP FRPSDWLELOLGDGH FRP DV
YLVWDSRGHUiVHUHIHLWRHPDWpYLQWHHTXDWURKRUDV2EV REULJDo}HVSRUHOHVDVVXPLGDVWRGDVDVFRQGLo}HVGH
FRPUHGDomRGDGDSHOD/HLQž KDELOLWDomRHTXDOLILFDomRH[LJLGDVQDOLFLWDomR
†ž 2HGLWDOGHOHLOmRGHYHVHUDPSODPHQWHGLYXOJDGRSULQ †ž YHWDGR 
FLSDOPHQWHQRPXQLFtSLRHPTXHVHUHDOL]DUi2EV(VWH†IRL
LQFOXtGRSHOD/HL
†ž 1RV FRQWUDWRV FHOHEUDGRV SHOD$GPLQLVWUDomR3~EOLFD
FRP SHVVRDV ItVLFDV RX MXUtGLFDV LQFOXVLYH DTXHODV
&DStWXOR,,, GRPLFLOLDGDVQRHVWUDQJHLURGHYHUiFRQVWDUQHFHVVDULD
'RV&RQWUDWRV PHQWHFOiXVXODTXHGHFODUHFRPSHWHQWHRIRURGDVHGH
GD $GPLQLVWUDomR SDUD GHULPLU TXDOTXHU TXHVWmR
6HomR, FRQWUDWXDOVDOYRRGLVSRVWRQR†žGRDUWGHVWD/HL
'LVSRVLo}HV3UHOLPLQDUHV
†ž 1RDWRGDOLTXLGDomRGDGHVSHVD RV VHUYLoRVGHFRQ
$UW 2V FRQWUDWRV DGPLQLVWUDWLYRV GH TXH WUDWD HVWD/HL WDELOLGDGH FRPXQLFDUmR DRV yUJmRV LQFXPELGRV GD
UHJXODPVH SHODV VXDV FOiXVXODV H SHORV SUHFHLWRV GH DUUHFDGDomRHILVFDOL]DomRGHWULEXWRVGD8QLmR(VWDGR
GLUHLWR S~EOLFR DSOLFDQGRVHOKHV VXSOHWLYDPHQWH RV RX 0XQLFtSLR DV FDUDFWHUtVWLFDV H RV YDORUHV SDJRV
SULQFtSLRVGDWHRULDJHUDOGRVFRQWUDWRVHDVGLVSRVLo}HV VHJXQGRRGLVSRVWRQRDUWGD/HLQžGHGH
GHGLUHLWRSULYDGR PDUoRGH

†ž 2VFRQWUDWRVGHYHPHVWDEHOHFHUFRPFODUH]DHSUHFLVmR $UW $FULWpULRGDDXWRULGDGHFRPSHWHQWHHPFDGDFDVRH


DVFRQGLo}HVSDUDVXDH[HFXomRH[SUHVVDVHPFOiXVXODV GHVGH TXH SUHYLVWD QR LQVWUXPHQWR FRQYRFDWyULR
TXHGHILQDPRVGLUHLWRVREULJDo}HVHUHVSRQVDELOLGDGHV SRGHUiVHUH[LJLGDSUHVWDomRGHJDUDQWLDQDVFRQWUDWD
GDVSDUWHVHPFRQIRUPLGDGHFRPRVWHUPRVGDOLFLWDomR o}HVGHREUDVVHUYLoRVHFRPSUDV
HGDSURSRVWDDTXHVHYLQFXODP
 'LU$GPLQLVWUDWLYR
 &kPDUD/HJLVODWLYDGR'LVWULWR)HGHUDO
†ž &DEHUi DR FRQWUDWDGR RSWDU SRU XPDGDVVHJXLQWHV ,, VXSHUYHQLrQFLD GD IDWR H[FHSFLRQDORXLPSUHYLVtYHO
PRGDOLGDGHVGHJDUDQWLD HVWUDQKRjYRQWDGHGDVSDUWHVTXHDOWHUHIXQGDPHQWDO
PHQWHDVFRQGLo}HVGHH[HFXomRGRFRQWUDWR
, FDXomRHPGLQKHLURRXWtWXORVGHGtYLGDS~EOLFDGHYHQ
GR HVWHV WHU VLGR HPLWLGRV VRE D IRUPD HVFULWXUDO ,,, LQWHUUXSomRGDH[HFXomRGRFRQWUDWRRXGLPLQXLomRGR
PHGLDQWHUHJLVWURHPVLVWHPDFHQWUDOL]DGRGHOLTXLGD ULWPRGHWUDEDOKRSRURUGHPHQRLQWHUHVVHGD$GPLQLV
omR H GH FXVWyGLD DXWRUL]DGR SHOR %DQFR &HQWUDO GR WUDomR
%UDVLO H DYDOLDGRV SHORV VHXV YDORUHV HFRQ{PLFRV
FRQIRUPHGHILQLGRSHOR0LQLVWpULRGD)D]HQGD ,9 DXPHQWR GDV TXDQWLGDGHV LQLFLDOPHQWH SUHYLVWDVQR
2EVUHGDomRGDGDSHOD/HLQž FRQWUDWRQRVOLPLWHVSHUPLWLGRVSRUHVWD/HL
,, VHJXURJDUDQWLD
9 LPSHGLPHQWRGHH[HFXomRGRFRQWUDWRSRUIDWRRXDWR
,,, ILDQoDEDQFiULD GHWHUFHLURUHFRQKHFLGRSHOD$GPLQLVWUDomRHPGRFX
PHQWRFRQWHPSRUkQHRjVXDRFRUUrQFLD
†ž $ JDUDQWLD D TXH VH UHIHUH R FDSXW GHVWHDUWLJRQmR
H[FHGHUiDFLQFRSRUFHQWRGRYDORUGRFRQWUDWRHWHUi
9, RPLVVmRQRDWUDVRGHSURYLGrQFLDVDFDUJRGD$GPLQLV
VHX YDORU DWXDOL]DGR QDV PHVPDV FRQGLo}HV GDTXHOH
WUDomRLQFOXVLYHTXDQWRDRVSDJDPHQWRVSUHYLVWRVGH
UHVVDOYDGRRSUHYLVWRQR†žGHVWHDUWLJR
TXHUHVXOWHGLUHWDPHQWHLPSHGLPHQWRRXUHWDUGDPHQ
†ž 3DUDREUDVVHUYLoRVHIRUQHFLPHQWRVGHJUDQGHYXOWR WRQDH[HFXomRGRFRQWUDWRVHPSUHMXt]RGDVVDQo}HV
HQYROYHQGRDOWDFRPSOH[LGDGHWpFQLFDHULVFRVILQDQFHL OHJDLVDSOLFiYHLVDRVUHVSRQViYHLV
URV FRQVLGHUiYHLV GHPRQVWUDGRV DWUDYpV GH SDUHFHU
WHFQLFDPHQWHDSURYDGRSHODDXWRULGDGHFRPSHWHQWHR †ž 7RGDSURUURJDomRGHSUD]RGHYHUiVHUMXVWLILFDGDSRU
OLPLWHGHJDUDQWLDSUHYLVWRQRSDUiJUDIRDQWHULRUSRGHUi HVFULWR H SUHYLDPHQWH DXWRUL]DGD SHOD DXWRULGDGH
VHUHOHYDGRSDUDDWpGH]SRUFHQWRGRYDORUGRFRQWUDWR FRPSHWHQWHSDUDFHOHEUDURFRQWUDWR

†ž $JDUDQWLDSUHVWDGDSHORFRQWUDWDGRVHUiOLEHUDGDRX †ž eYHGDGRRFRQWUDWRFRPSUD]RGHYLJrQFLDLQGHWHUPL


UHVWLWXtGDDSyVDH[HFXomRGRFRQWUDWRHTXDQGRHP QDGR
GLQKHLURDWXDOL]DGDPRQHWDULDPHQWH
2EV††žžžHžFRPUHGDomRGDGDSHOD/HLQž †ž (P FDUiWHU H[FHSFLRQDO GHYLGDPHQWHMXVWLILFDGRH
PHGLDQWHDXWRUL]DomRVXSHULRURSUD]RGHTXHWUDWDR
†ž 1RV FDVRV GH FRQWUDWRV TXH LPSRUWHP QDHQWUHJDGH LQFLVR,,GR´FDSXWµGHVWHDUWLJRSRGHUiVHUSURUURJDGR
EHQVSHOD$GPLQLVWUDomRGRVTXDLVRFRQWUDWDGRILFDUi
HPDWpGR]HPHVHV
GHSRVLWiULRDRYDORUGDJDUDQWLDGHYHUiVHUDFUHVFLGRR 2EV(VWH†žIRLLQFOXtGRSHOD/HLQž
YDORUGHVVHVEHQV
$UW 2UHJLPHMXUtGLFRGRVFRQWUDWRVDGPLQLVWUDWLYRVLQVWLWXt
$UW $ GXUDomR GRV FRQWUDWRV UHJLGRV SRU HVWD /HLILFDUi GRVSRUHVWD/HLFRQIHUHj$GPLQLVWUDomRHPUHODomRD
DGVWULWDjYLJrQFLDGRVUHVSHFWLYRVFUpGLWRVRUoDPHQWiUL HOHVDSUHUURJDWLYDGH
RVH[FHWRTXDQWRDRVUHODWLYRV
, PRGLILFiORVXQLODWHUDOPHQWHSDUDPHOKRUDGHTXDomR
, DRVSURMHWRVFXMRVSURGXWRVHVWHMDPFRQWHPSODGRVQDV jV ILQDOLGDGHV GH LQWHUHVVH S~EOLFR UHVSHLWDGRV RV
PHWDV HVWDEHOHFLGDV QR 3ODQR 3OXULDQXDO RV TXDLV GLUHLWRVGRFRQWUDWDGR
SRGHUmRVHUSURUURJDGRVVHKRXYHULQWHUHVVHGD$GPL
QLVWUDomRHGHVGHTXHLVVRWHQKDVLGRSUHYLVWRQRDWR ,, UHVFLQGLORVXQLODWHUDOPHQWHQRVFDVRVHVSHFLILFDGRVQR
FRQYRFDWyULR LQFLVR,GRDUWGHVWD/HL

,, jSUHVWDomRGHVHUYLoRVDVHUHPH[HFXWDGRVGHIRUPD ,,, ILVFDOL]DUOKHVDH[HFXomR


FRQWtQXD TXH SRGHUmR WHU D VXD GXUDomR SURUURJDGD
SRULJXDLVHVXFHVVLYRVSHUtRGRVFRPYLVWDVjREWHQomR ,9 DSOLFDU VDQo}HV PRWLYDGDV SHOD LQH[HFXomRWRWDORX
GHSUHoRVHFRQGLo}HVPDLVYDQWDMRVDVSDUDD$GPLQLV SDUFLDOGRDMXVWH
WUDomROLPLWDGDDVHVVHQWDPHVHV 2EV UHGDomR GDGD SHOD /HL Qž


,,, YHWDGR 
9 QRVFDVRVGHVHUYLoRVHVVHQFLDLVRFXSDUSURYLVRULDPHQ
WHEHQVPyYHLVLPyYHLVSHVVRDOHVHUYLoRVYLQFXODGRV
,9 DRDOXJXHOGHHTXLSDPHQWRVHjXWLOL]DomRGHSURJUD DRREMHWRGRFRQWUDWRQDKLSyWHVHGDQHFHVVLGDGHGH
PDV GH LQIRUPiWLFD SRGHQGR D GXUDomR HVWHQGHUVH DFDXWHODUDSXUDomRDGPLQLVWUDWLYDGHIDOWDVFRQWUDWXDLV
SHORSUD]RGHDWp TXDUHQWDHRLWR PHVHVDSyVRLQtFLR SHORFRQWUDWDGREHPFRPRQDKLSyWHVHGHUHVFLVmRGR
GDYLJrQFLDGRFRQWUDWR FRQWUDWRDGPLQLVWUDWLYR
9 jVKLSyWHVHVSUHYLVWDVQRVLQFLVRV,;;,;;;9,,,H;;;, †ž $V FOiXVXODV HFRQ{PLFRILQDQFHLUDV HPRQHWiULDVGRV
GRDUWFXMRVFRQWUDWRVSRGHUmRWHUYLJrQFLDSRUDWp
FRQWUDWRV DGPLQLVWUDWLYRV QmR SRGHUmR VHU DOWHUDGDV
 FHQWR H YLQWH  PHVHV FDVR KDMD LQWHUHVVH GD
VHPSUpYLDFRQFRUGkQFLDGRFRQWUDWDGR
DGPLQLVWUDomR ,QFOXtGRSHOD/HLQžGH

†ž 2VSUD]RVGHLQtFLRGHHWDSDVGHH[HFXomRGHFRQFOXVmR †ž 1D KLSyWHVH GR LQFLVR , GHVWH DUWLJRDVFOiXVXODV


HGHHQWUHJDDGPLWHPSURUURJDomRPDQWLGDVDVGHPDLV HFRQ{PLFRILQDQFHLUDVGRFRQWUDWRGHYHUmRVHUUHYLVWDV
FOiXVXODVGRFRQWUDWRHDVVHJXUDGDDPDQXWHQomRGH SDUDTXHVHPDQWHQKDRHTXLOtEULRFRQWUDWXDO
VHXHTXLOtEULRHFRQ{PLFRILQDQFHLURGHVGHTXHRFRUUD
DOJXP GRV VHJXLQWHV PRWLYRV GHYLGDPHQWH DXWXDGRV $UW $ GHFODUDomR GH QXOLGDGH GR FRQWUDWRDGPLQLVWUDWLYR
HPSURFHVVR RSHUD UHWURDWLYDPHQWH LPSHGLQGR RV HIHLWRV MXUtGLFRV
TXH HOH RUGLQDULDPHQWH GHYHULD SURGX]LU DOpP GH
, DOWHUDomRGRSURMHWRRXHVSHFLILFDo}HVSHOD$GPLQLVWUD GHVFRQVWLWXLURVMiSURGX]LGRV
omR
 &kPDUD/HJLVODWLYDGR
 'LVWULWR)HGHUDO 'LU$GPLQLVWUDWLYR 

†~QLFR $QXOLGDGHQmRH[RQHUDD$GPLQLVWUDomRGRGHYHUGH †ž e GLVSHQViYHO R WHUPR GH FRQWUDWR HIDFXOWDGDD


LQGHQL]DURFRQWUDWDGRSHORTXHHVWHKRXYHUH[HFXWDGR VXEVWLWXLomRSUHYLVWDQHVWHDUWLJRDFULWpULRGD$GPLQLV
DWpDGDWDHPTXHHODIRUGHFODUDGDHSRURXWURVSUHMXt WUDomRHLQGHSHQGHQWHPHQWHGHVHXYDORUQRVFDVRVGH
]RVUHJXODUPHQWHFRPSURYDGRVFRQWDQWRTXHQmROKH FRPSUD FRP HQWUHJD LPHGLDWD H LQWHJUDO GRV EHQV
VHMDLPSXWiYHOSURPRYHQGRVHDUHVSRQVDELOLGDGHGH DGTXLULGRVGRVTXDLVQmRUHVXOWHPREULJDo}HVIXWXUDV
TXHPOKHGHXFDXVD LQFOXVLYHDVVLVWrQFLDWpFQLFD

6HomR,, $UW e SHUPLWLGR D TXDOTXHU OLFLWDQWH R FRQKHFLPHQWRGRV


'D)RUPDOL]DomRGRV&RQWUDWRV WHUPRVGRFRQWUDWRHGRUHVSHFWLYRSURFHVVROLFLWDWyULR
HDTXDOTXHULQWHUHVVDGRDREWHQomRGHFySLDDXWHQWL
$UW 2V FRQWUDWRV H VHXV DGLWDPHQWRV VHUmR ODYUDGRVQDV FDGDPHGLDQWHRSDJDPHQWRGRVHPROXPHQWRVGHYLGRV
UHSDUWLo}HV LQWHUHVVDGDV DV TXDLV PDQWHUmR DUTXLYR
FURQROyJLFRGRVVHXVDXWyJUDIRVHUHJLVWURVLVWHPiWLFR $UW $$GPLQLVWUDomRFRQYRFDUiUHJXODUPHQWHRLQWHUHVVDGR
GRVHXH[WUDWRVDOYRRVUHODWLYRVDGLUHLWRVUHDLVVREUH SDUDDVVLQDURWHUPRGHFRQWUDWRDFHLWDURXUHWLUDUR
LPyYHLVTXHVHIRUPDOL]DPSRULQVWUXPHQWRODYUDGRHP LQVWUXPHQWRHTXLYDOHQWHGHQWURGRSUD]RHFRQGLo}HV
HVWDEHOHFLGRVVRESHQDGHGHFDLURGLUHLWRjFRQWUDWD
FDUWyULRGHQRWDVGHWXGRMXQWDQGRVHFySLDQRSURFHV
omRVHPSUHMXt]RGDVVDQo}HVSUHYLVWDVQRDUWGHVWD
VRTXHOKHGHXRULJHP
/HL
†~QLFR e QXOR H GH QHQKXP HIHLWR R FRQWUDWR YHUEDOFRPD
$GPLQLVWUDomRVDOYRRGHSHTXHQDVFRPSUDVGHSURQWR †ž 2SUD]RGHFRQYRFDomRSRGHUiVHUSURUURJDGRXPDYH]
SDJDPHQWR DVVLP HQWHQGLGDV DTXHODV GH YDORU QmR SRULJXDOSHUtRGRTXDQGRVROLFLWDGRSHODSDUWHGXUDQWH
VXSHULRUD FLQFRSRUFHQWR GROLPLWHHVWDEHOHFLGR RVHXWUDQVFXUVRHGHVGHTXHRFRUUDPRWLYRMXVWLILFDGR
QR DUW LQFLVR ,, DOtQHD D GHVWD /HL IHLWDV HP DFHLWRSHOD$GPLQLVWUDomR
UHJLPHGHDGLDQWDPHQWR
†ž eIDFXOWDGRj$GPLQLVWUDomRTXDQGRRFRQYRFDGRQmR
DVVLQDURWHUPRGHFRQWUDWRRXQmRDFHLWDURXUHWLUDUR
$UW 7RGRFRQWUDWRGHYHPHQFLRQDURVQRPHVGDVSDUWHVHRV
LQVWUXPHQWRHTXLYDOHQWHQRSUD]RHFRQGLo}HVHVWDEHOH
GHVHXVUHSUHVHQWDQWHVDILQDOLGDGHRDWRTXHDXWRUL FLGRVFRQYRFDURVOLFLWDQWHVUHPDQHVFHQWHVQDRUGHP
]RXDVXDODYUDWXUDRQ~PHURGRSURFHVVRGDOLFLWDomR GH FODVVLILFDomR SDUD ID]rOR HP LJXDO SUD]R H QDV
GD GLVSHQVD RX GD LQH[LJLELOLGDGH D VXMHLomR GRV PHVPDVFRQGLo}HVSURSRVWDVSHORSULPHLURFODVVLILFDGR
FRQWUDWDQWHVjVQRUPDVGHVWD/HLHjVFOiXVXODVFRQWUD LQFOXVLYHTXDQWRDRVSUHoRVDWXDOL]DGRVGHFRQIRUPLGD
WXDLV GH FRP R DWR FRQYRFDWyULR RX UHYRJDU D OLFLWDomR
LQGHSHQGHQWHPHQWH GD FRPLQDomR SUHYLVWD QR DUW
†~QLFR $SXEOLFDomRUHVXPLGDGRLQVWUXPHQWRGHFRQWUDWRRX GHVWD/HL
GHVHXVDGLWDPHQWRVQDLPSUHQVDRILFLDOTXHpFRQGLomR
LQGLVSHQViYHOSDUDVXDHILFiFLDVHUiSURYLGHQFLDGDSHOD †ž 'HFRUULGRV VHVVHQWD GLDVGDGDWDGDHQWUHJDGDV
$GPLQLVWUDomRDWpRTXLQWRGLD~WLOGRPrVVHJXLQWHDR SURSRVWDVVHPFRQYRFDomRSDUDDFRQWUDWDomRILFDPRV
GHVXDDVVLQDWXUDSDUDRFRUUHUQRSUD]RGHYLQWHGLDV OLFLWDQWHVOLEHUDGRVGRVFRPSURPLVVRVDVVXPLGRV
GDTXHODGDWDTXDOTXHUTXHVHMDRVHXYDORUDLQGDTXH
VHP{QXVUHVVDOYDGRRGLVSRVWRSHORDUWGHVWD/HL 6HomR,,,
/HL  'D$OWHUDomRGRV&RQWUDWRV
2EVUHGDomRGDGDSHOD/HLQž

$UW 2LQVWUXPHQWRGHFRQWUDWRpREULJDWyULRQRVFDVRVGH $UW 2VFRQWUDWRVUHJLGRVSRUHVWD/HLSRGHUmRVHUDOWHUDGRV


FRQFRUUrQFLD H GH WRPDGD GH SUHoRV EHP FRPR QDV FRPDVGHYLGDVMXVWLILFDWLYDVQRVVHJXLQWHVFDVRV
GLVSHQVDV H LQH[LJLELOLGDGHV FXMRV SUHoRV HVWHMDP
FRPSUHHQGLGRVQRVOLPLWHVGHVWDVGXDVPRGDOLGDGHVGH , XQLODWHUDOPHQWHSHOD$GPLQLVWUDomR
OLFLWDomRHIDFXOWDWLYRQRVGHPDLVHPTXHD$GPLQLVWUD
omRSXGHUVXEVWLWXtORSRURXWURVLQVWUXPHQWRVKiEHLV D TXDQGR KRXYHU PRGLILFDomR GR SURMHWR RX GDV
WDLVFRPRFDUWDFRQWUDWRQRWDGHHPSHQKRGHGHVSHVD HVSHFLILFDo}HVSDUDPHOKRUDGHTXDomRWpFQLFDDRV
DXWRUL]DomR GH FRPSUD RX RUGHP GH H[HFXomR GH VHXVREMHWLYRV
VHUYLoR
E TXDQGR QHFHVViULD D PRGLILFDomR GR YDORU FRQ
†ž $PLQXWDGRIXWXURFRQWUDWRLQWHJUDUiVHPSUHRHGLWDO WUDWXDOHPGHFRUUrQFLDGHDFUpVFLPRRXGLPLQXLomR
RXDWRFRQYRFDWyULRGDOLFLWDomR TXDQWLWDWLYDGHVHXREMHWRQRVOLPLWHVSHUPLWLGRV
SRUHVWD/HL
†ž (P FDUWDFRQWUDWR QRWD GH HPSHQKRGHGHVSHVD
DXWRUL]DomRGHFRPSUDRUGHPGHH[HFXomRGHVHUYLoR ,, SRUDFRUGRGDVSDUWHV
RXRXWURVLQVWUXPHQWRVKiEHLVDSOLFDVHQRTXHFRXEHU
D TXDQGRFRQYHQLHQWHDVXEVWLWXLomRGDJDUDQWLDGD
RGLVSRVWRQRDUWGHVWD/HL
H[HFXomR
2EVUHGDomRGDGDSHOD/HLQž

†ž $SOLFDVHRGLVSRVWRQRVDUWVHDGHVWD/HLH E TXDQGR QHFHVViULD D PRGLILFDomR GR UHJLPH GH


GHPDLVQRUPDVJHUDLVQRTXHFRXEHU H[HFXomRGDREUDRXVHUYLoREHPFRPRGRPRGR
GHIRUQHFLPHQWRHPIDFHGHYHULILFDomRWpFQLFDGD
, DRVFRQWUDWRVGHVHJXURGHILQDQFLDPHQWRGHORFDomR LQDSOLFDELOLGDGHGRVWHUPRVFRQWUDWXDLVRULJLQiULRV
HP TXH R 3RGHU 3~EOLFR VHMD ORFDWiULR H DRV GHPDLV
F TXDQGR QHFHVViULD D PRGLILFDomR GD IRUPD GH
FXMR FRQWH~GR VHMD UHJLGR SUHGRPLQDQWHPHQWH SRU
SDJDPHQWR SRU LPSRVLomR GH FLUFXQVWkQFLDV
QRUPDGHGLUHLWRSULYDGR
VXSHUYHQLHQWHVPDQWLGRRYDORULQLFLDODWXDOL]DGR
YHGDGDDDQWHFLSDomRGRSDJDPHQWRFRPUHODomR
,, DRVFRQWUDWRVHPTXHD$GPLQLVWUDomRIRUSDUWHFRPR DRFURQRJUDPDILQDQFHLURIL[DGRVHPDFRUUHVSRQ
XVXiULDGHVHUYLoRS~EOLFR GHQWHFRQWUDSUHVWDomRGHIRUQHFLPHQWRGHEHQVRX
H[HFXomRGHREUDRXVHUYLoR
 'LU$GPLQLVWUDWLYR
 &kPDUD/HJLVODWLYDGR'LVWULWR)HGHUDO
G SDUDUHVWDEHOHFHUDUHODomRTXHDVSDUWHVSDFWXD $UW$ $V HPSUHVDV HQTXDGUDGDV QR LQFLVR 9 GR † R H QR
UDPLQLFLDOPHQWHHQWUHRVHQFDUJRVGRFRQWUDWDGR LQFLVR,,GR†RGRDUWRGHVWD/HLGHYHUmRFXPSULU
H D UHWULEXLomR GD $GPLQLVWUDomR SDUD D MXVWD GXUDQWH WRGR R SHUtRGR GH H[HFXomR GR FRQWUDWR D
UHPXQHUDomR GD REUD VHUYLoR RX IRUQHFLPHQWR UHVHUYD GH FDUJRV SUHYLVWD HP OHL SDUD SHVVRD FRP
REMHWLYDQGR D PDQXWHQomR GR HTXLOtEULR GHILFLrQFLDRXSDUDUHDELOLWDGRGD3UHYLGrQFLD6RFLDO
HFRQ{PLFRILQDQFHLUR LQLFLDO GR FRQWUDWR QD EHP FRPR DV UHJUDV GH DFHVVLELOLGDGH SUHYLVWDV QD
KLSyWHVH GH VREUHYLUHP IDWRV LPSUHYLVtYHLV RX OHJLVODomR2EVDUWLJRHVHXSDUiJUDIR~QLFRLQWURGX]LGRVSHOD/HLQž
SUHYLVtYHLVSRUpPGHFRQVHTrQFLDVLQFDOFXOiYHLV
UHWDUGDGRUHV RX LPSHGLWLYRV GD H[HFXomR GR †~QLFR &DEH j DGPLQLVWUDomR ILVFDOL]DU RFXPSULPHQWRGRV
DMXVWDGRRXDLQGDHPFDGDGHIRUoDPDLRUFDVR UHTXLVLWRVGHDFHVVLELOLGDGHQRVVHUYLoRVHQRVDPELHQ
IRUWXLWR RX IDWR GR SULQFtSH FRQILJXUDQGR iOHD WHVGHWUDEDOKR
HFRQ{PLFD H[WUDRUGLQiULD H H[WUDFRQWUDWXDO 2EV
UHGDomRGDGDSHOD/HLQž
$UW $ H[HFXomR GR FRQWUDWR GHYHUi VHU DFRPSDQKDGDH
†ž 2 FRQWUDWDGR ILFD REULJDGR D DFHLWDUQDVPHVPDV ILVFDOL]DGD SRU XP UHSUHVHQWDQWH GD $GPLQLVWUDomR
FRQGLo}HVFRQWUDWXDLVRVDFUpVFLPRVRXVXSUHVV}HVTXH HVSHFLDOPHQWHGHVLJQDGRSHUPLWLGDDFRQWUDWDomRGH
VH IL]HUHP QDV REUDV VHUYLoRV RX FRPSUDV DWp  WHUFHLURV SDUD DVVLVWLOR H VXEVLGLiOR GH LQIRUPDo}HV
YLQWHHFLQFRSRUFHQWR GRYDORULQLFLDODWXDOL]DGRGR SHUWLQHQWHVDHVVDDWULEXLomR
FRQWUDWRHQRFDVRSDUWLFXODUGHUHIRUPDGHHGLItFLRRX
GHHTXLSDPHQWRDWpROLPLWHGH FLQTHQWDSRU †ž 2UHSUHVHQWDQWHGD$GPLQLVWUDomRDQRWDUiHPUHJLVWUR
FHQWR SDUDRVVHXVDFUpVFLPRV SUySULRWRGDVDVRFRUUrQFLDVUHODFLRQDGDVFRPDH[HFX
omRGRFRQWUDWRGHWHUPLQDQGRRTXHIRUQHFHVViULRj
†ž 1HQKXP DFUpVFLPR RX VXSUHVVmR SRGHUiH[FHGHURV UHJXODUL]DomRGDVIDOWDVRXGHIHLWRVREVHUYDGRV
OLPLWHVHVWDEHOHFLGRVQRSDUiJUDIRDQWHULRUVDOYR 2EV
UHGDomRGDGDSHOD/HLQž
†ž $V GHFLV}HV H SURYLGrQFLDV TXH XOWUDSDVVHPDFRP
, YHWDGR  SHWrQFLDGRUHSUHVHQWDQWHGHYHUmRVHUVROLFLWDGDVDVHXV
,, DVVXSUHVV}HVUHVXOWDQWHVGHDFRUGRFHOHEUDGRHQWUHRV VXSHULRUHVHPWHPSRKiELOSDUDDDGRomRGDVPHGLGDV
FRQWUDWDQWHV FRQYHQLHQWHV
†ž 6HQRFRQWUDWRQmRKRXYHUHPVLGRFRQWHPSODGRVSUHoRV $UW 2 FRQWUDWDGR GHYHUi PDQWHU SUHSRVWR DFHLWRSHOD
XQLWiULRV SDUD REUDV RX VHUYLoRV HVVHV VHUmR IL[DGRV $GPLQLVWUDomR QR ORFDO GD REUD RX VHUYLoR SDUD
PHGLDQWHDFRUGRHQWUHDVSDUWHVUHVSHLWDGRVRVOLPLWHV UHSUHVHQWiORQDH[HFXomRGRFRQWUDWR
HVWDEHOHFLGRVQR†žGHVWHDUWLJR
$UW 2FRQWUDWDGRpREULJDGRDUHSDUDUFRUULJLUUHPRYHU
†ž 1RFDVRGHVXSUHVVmRGHREUDVEHQVRXVHUYLoRVVHR UHFRQVWUXLURXVXEVWLWXLUjVVXDVH[SHQVDVQRWRWDORX
FRQWUDWDGRMiKRXYHUDGTXLULGRRVPDWHULDLVHSRVWRQR HPSDUWHRREMHWRGRFRQWUDWRHPTXHVHYHULILFDUHP
ORFDO GRV WUDEDOKRV HVWHV GHYHUmR VHU SDJRV SHOD YtFLRVGHIHLWRVRXLQFRUUHo}HVUHVXOWDQWHVGDH[HFXomR
$GPLQLVWUDomRSHORVFXVWRVGHDTXLVLomRUHJXODUPHQWH RXGHPDWHULDLVHPSUHJDGRV
FRPSURYDGRV H PRQHWDULDPHQWH FRUULJLGRV SRGHQGR
FDEHU LQGHQL]DomR SRU RXWURV GDQRV HYHQWXDOPHQWH
$UW 2 FRQWUDWDGR p UHVSRQViYHO SHORV GDQRVFDXVDGRV
GHFRUUHQWHV GD VXSUHVVmR GHVGH TXH UHJXODUPHQWH
GLUHWDPHQWHj$GPLQLVWUDomRRXDWHUFHLURVGHFRUUHQWHV
FRPSURYDGRV
GH VXD FXOSD RX GROR QD H[HFXomR GR FRQWUDWR QmR
H[FOXLQGRRXUHGX]LQGRHVVDUHVSRQVDELOLGDGHDILVFDOL
†ž 4XDLVTXHUWULEXWRVRXHQFDUJRVOHJDLVFULDGRVDOWHUDGRV
]DomRRXRDFRPSDQKDPHQWRSHORyUJmRLQWHUHVVDGR
RXH[WLQWRVEHPFRPRDVXSHUYHQLrQFLDGHGLVSRVLo}HV
OHJDLVTXDQGRRFRUULGDVDSyVDGDWDGDDSUHVHQWDomR
$UW 2FRQWUDWDGRpUHVSRQViYHOSHORVHQFDUJRVWUDEDOKLVWDV
GD SURSRVWD GH FRPSURYDGD UHSHUFXVVmR QRV SUHoRV
SUHYLGHQFLiULRV ILVFDLV H FRPHUFLDLV UHVXOWDQWHV GD
FRQWUDWDGRVLPSOLFDUmRDUHYLVmRGHVWHVSDUDPDLVRX
H[HFXomRGRFRQWUDWR
SDUDPHQRVFRQIRUPHRFDVR

†ž (P KDYHQGR DOWHUDomR XQLODWHUDO GRFRQWUDWRTXH †ž $ LQDGLPSOrQFLD GR FRQWUDWDGR FRPUHIHUrQFLDDRV
DXPHQWHRVHQFDUJRVGRFRQWUDWDGRD$GPLQLVWUDomR HQFDUJRVWUDEDOKLVWDVILVFDLVHFRPHUFLDLVQmRWUDQVIHUH
GHYHUi UHVWDEHOHFHU SRU DGLWDPHQWR R HTXLOtEULR j $GPLQLVWUDomR 3~EOLFD D UHVSRQVDELOLGDGH SRU VHX
HFRQ{PLFRILQDQFHLURLQLFLDO SDJDPHQWRQHPSRGHUiRQHUDURREMHWRGRFRQWUDWRRX
UHVWULQJLUDUHJXODUL]DomRHRXVRGDVREUDVHHGLILFD
†ž YHWDGR 
o}HVLQFOXVLYHSHUDQWHR5HJLVWURGH,PyYHLV
†ž $ YDULDomR GR YDORU FRQWUDWXDO SDUD ID]HUIDFHDR 2EVFRPUHGDomRGDGDSHOD/HLQž
UHDMXVWH GH SUHoRV SUHYLVWR QR SUySULR FRQWUDWR DV
DWXDOL]Do}HVFRPSHQVDo}HVRXSHQDOL]Do}HVILQDQFHLUDV †ž $$GPLQLVWUDomR3~EOLFDUHVSRQGHVROLGDULDPHQWHFRP
GHFRUUHQWHVGDVFRQGLo}HVGHSDJDPHQWRQHOHSUHYLVWDV RFRQWUDWDGRSHORVHQFDUJRVSUHYLGHQFLiULRVUHVXOWDQWHV
EHP FRPR R HPSHQKR GH GRWDo}HV RUoDPHQWiULDV GDH[HFXomRGRFRQWUDWRQRVWHUPRVGRDUWGD/HL
VXSOHPHQWDUHVDWpROLPLWHGRVHXYDORUFRUULJLGRQmR QžGHGHMXOKRGH
2EVFRPUHGDomRGDGDSHOD/HLQž
FDUDFWHUL]DPDOWHUDomRGRPHVPRSRGHQGRVHUUHJLVWUD
GRVSRUVLPSOHVDSRVWLODGLVSHQVDQGRDFHOHEUDomRGH $UW 2FRQWUDWDGRQDH[HFXomRGRFRQWUDWRVHPSUHMXt]R
DGLWDPHQWR GDV UHVSRQVDELOLGDGHV FRQWUDWXDLV H OHJDLV SRGHUi
6HomR,9 VXEFRQWUDWDUSDUWHVGDREUDVHUYLoRRXIRUQHFLPHQWR
'D([HFXomRGRV&RQWUDWRV DWpROLPLWHDGPLWLGRHPFDGDFDVRSHOD$GPLQLVWUD
omR
$UW 2FRQWUDWRGHYHUiVHUH[HFXWDGRILHOPHQWHSHODVSDUWHV
GH DFRUGR FRP DV FOiXVXODV DYHQoDGDV H DV QRUPDV $UW ([HFXWDGRRFRQWUDWRRVHXREMHWRVHUiUHFHELGR
GHVWD/HLUHVSRQGHQGRFDGDXPDSHODVFRQVHTrQFLDV
GHVXDLQH[HFXomRWRWDORXSDUFLDO , HPVHWUDWDQGRGHREUDVHVHUYLoRV
&kPDUD/HJLVODWLYDGR
  'LVWULWR)HGHUDO 'LU$GPLQLVWUDWLYR 

D SURYLVRULDPHQWH SHOR UHVSRQViYHO SRU VH 6HomR9


DFRPSDQKDPHQWRHILVFDOL]DomRPHGLDQWHWHUPR 'D,QH[HFXomRHGD5HVFLVmRGRV&RQWUDWRV
FLUFXQVWDQFLDGRDVVLQDGRSHODVSDUWHVHPDWp
TXLQ]H GLDVGDFRPXQLFDomRHVFULWDGRFRQWUDWD $UW $LQH[HFXomRWRWDORXSDUFLDOGRFRQWUDWRHQVHMDDVXD
GR UHVFLVmRFRPDVFRQVHTrQFLDVFRQWUDWXDLVHDVSUHYLVWDV
HPOHLRXUHJXODPHQWR
E GHILQLWLYDPHQWHSRUVHUYLGRURXFRPLVVmRGHVWLQD
GDSHODDXWRULGDGHFRPSHWHQWHPHGLDQWHWHUPR $UW &RQVWLWXHPPRWLYRSDUDUHVFLVmRGRFRQWUDWR
FLUFXQVWDQFLDGR DVVLQDGR SHODV SDUWHV DSyV R
GHFXUVR GR SUD]R GH REVHUYDomR RX YLVWRULD TXH , RQmRFXPSULPHQWRGHFOiXVXODVFRQWUDWXDLVHVSHFLILFD
FRPSURYH D DGHTXDomR GR REMHWR DRV WHUPRV o}HVSURMHWRVRXSUD]RV
FRQWUDWXDLVREVHUYDGRRGLVSRVWRQRDUWGHVWD
/HL ,, R FXPSULPHQWR LUUHJXODUGHFOiXVXODVFRQWUDWXDLV
HVSHFLILFDo}HVSURMHWRVHSUD]RV
,, HP VH WUDWDQGR GH FRPSUDV RX GH ORFDomRGH
HTXLSDPHQWRV ,,, DOHQWLGmRGRVHXFXPSULPHQWROHYDQGRD$GPLQLVWUD
omR D FRPSURYDU D LPSRVVLELOLGDGH GD FRQFOXVmR GD
D SURYLVRULDPHQWHSDUDHIHLWRGHSRVWHULRUYHULILFD REUDGRVHUYLoRRXGRIRUQHFLPHQWRQRVSUD]RVHVWLSX
omRGDFRQIRUPLGDGHGRPDWHULDOFRPDHVSHFLILFD ODGRV
omR
,9 R DWUDVR LQMXVWLILFDGR QR LQtFLR GD REUDVHUYLoRRX
E GHILQLWLYDPHQWHDSyVDYHULILFDomRGDTXDOLGDGHH IRUQHFLPHQWR
TXDQWLGDGHGRPDWHULDOHFRQVHTHQWHDFHLWDomR
9 DSDUDOLVDomRGDREUDGRVHUYLoRRXGRIRUQHFLPHQWR
†ž 1RV FDVRV GH DTXLVLomR GH HTXLSDPHQWRVGHJUDQGH VHPMXVWDFDXVDHSUpYLDFRPXQLFDomRj$GPLQLVWUDomR
YXOWR R UHFHELPHQWR IDUVHi PHGLDQWH WHUPR
FLUFXQVWDQFLDGRHQRVGHPDLVPHGLDQWHUHFLER 9, D VXEFRQWUDWDomR WRWDO RX SDUFLDO GR VHXREMHWRD
DVVRFLDomRGRFRQWUDWRFRPRXWUHPDFHVVmRRXWUDQVIH
†ž 2 UHFHELPHQWR SURYLVyULR RX GHILQLWLYR QmRH[FOXLD UrQFLD WRWDO RX SDUFLDO EHP FRPR D IXVmR FLVmR RX
UHVSRQVDELOLGDGHFLYLOSHODVROLGH]HVHJXUDQoDGDREUD LQFRUSRUDomRQmRDGPLWLGDVQRHGLWDOHQRFRQWUDWR
RX GR VHUYLoR QHP pWLFRSURILVVLRQDO SHOD SHUIHLWD
H[HFXomRGRFRQWUDWRGHQWURGRVOLPLWHVHVWDEHOHFLGRV 9,, R GHVDWHQGLPHQWR GDV GHWHUPLQDo}HV UHJXODUHVGD
SHODOHLRXSHORFRQWUDWR DXWRULGDGHGHVLJQDGDSDUDDFRPSDQKDUHILVFDOL]DUD
VXDH[HFXomRDVVLPFRPRDVGHVHXVVXSHULRUHV
†ž 2SUD]RDTXHVHUHIHUHDDOtQHDEGRLQFLVR,GHVWH
DUWLJR QmR SRGHUi VHU VXSHULRU D  QRYHQWD  GLDV 9,,, 2FRPHWLPHQWRUHLWHUDGRGDVIDOWDVQDVXDH[HFXomR
VDOYRHPFDVRVH[FHSFLRQDLVGHYLGDPHQWHMXVWLILFDGRV DQRWDGDVQDIRUPDGR†žGRDUWGHVWDOHL
HSUHYLVWRVQRHGLWDO
,; DGHFUHWDomRGHIDOrQFLDRXDLQVWDXUDomRGHLQVROYrQFLD
†ž 1DKLSyWHVHGHWHUPRFLUFXQVWDQFLDGRRXDYHULILFDomR FLYLO
DTXHVHUHIHUHHVWHDUWLJRQmRVHUHPUHVSHFWLYDPHQWH
ODYUDGR RX SURFHGLGD GHQWUR GRV SUD]RV IL[DGRV ; DGLVVROXomRGDVRFLHGDGHRXRIDOHFLPHQWRGRFRQWUDWD
UHSXWDUVHmRFRPRUHDOL]DGRVGHVGHTXHFRPXQLFDGRV GR
j $GPLQLVWUDomR QRV  TXLQ]H  GLDV DQWHULRUHV j
H[HFXomRGRVPHVPRV ;, DDOWHUDomRVRFLDORXDPRGLILFDomRGDILQDOLGDGHRXGD
HVWUXWXUD GD HPSUHVD TXH SUHMXGLTXH D H[HFXomR GR
$UW 3RGHUi VHU GLVSHQVDGR R UHFHELPHQWR SURYLVyULRQRV FRQWUDWR
VHJXLQWHVFDVRV
;,, UD]}HVGHLQWHUHVVHS~EOLFRGHDOWDUHOHYkQFLDHDPSOR
, JrQHURVSHUHFtYHLVHDOLPHQWDomRSUHSDUDGD FRQKHFLPHQWRMXVWLILFDGDVHGHWHUPLQDGDVSHODPi[LPD
DXWRULGDGH GD HVIHUD DGPLQLVWUDWLYD D TXH HVWi
,, VHUYLoRVSURILVVLRQDLV VXERUGLQDGR R FRQWUDWDQWH H H[DUDGDV QR SURFHVVR
DGPLQLVWUDWLYRDTXHVHUHIHUHRFRQWUDWR
,,, REUDVHVHUYLoRVGHYDORUDWpRSUHYLVWRQRDUWLQFLVR
,,DOtQHDDGHVWD/HLGHVGHTXHQmRVHFRPSRQKDP ;,,, D VXVSHQVmR SRU SDUWH GD $GPLQLVWUDomR GHREUDV
GH DSDUHOKRV HTXLSDPHQWRV H LQVWDODo}HV VXMHLWRV j VHUYLoRVRXFRPSUDVDFDUUHWDQGRPRGLILFDomRGRYDORU
YHULILFDomRGHIXQFLRQDPHQWRHSURGXWLYLGDGH LQLFLDOGRFRQWUDWRDOpPGROLPLWHSHUPLWLGRQR†žGR
DUWGHVWDOHL
†~QLFR 1RV FDVRV GHVWH DUWLJR R UHFHELPHQWRVHUiIHLWR
PHGLDQWHUHFLER ;,9 D VXVSHQVmR GH VXD H[HFXomR SRU RUGHP HVFULWDGD
$GPLQLVWUDomRSRUSUD]RVXSHULRUD FHQWRHYLQWH
$UW 6DOYRGLVSRVLo}HVHPFRQWUiULRFRQVWDQWHVGRHGLWDOGR GLDV VDOYR HP FDVR GH FDODPLGDGH S~EOLFD JUDYH
FRQYLWHRXGHDWRQRUPDWLYRRVHQVDLRVWHVWHVHGHPDLV SHUWXUEDomRGDRUGHPLQWHUQDRXJXHUUDRXDLQGDSRU
SURYDVH[LJLGRVSRUQRUPDVWpFQLFDVRILFLDLVSDUDDERD UHSHWLGDV VXVSHQV}HV TXH WRWDOL]HP R PHVPR SUD]R
H[HFXomR GRREMHWRGRFRQWUDWRFRUUHPSRUFRQWDGR LQGHSHQGHQWHPHQWH GR SDJDPHQWR REULJDWyULR GH
FRQWUDWDGR LQGHQL]Do}HV SHODV VXFHVVLYDV H FRQWUDWXDOPHQWH
LPSUHYLVWDV GHVPRELOL]Do}HV H PRELOL]Do}HV H RXWUDV
$UW $$GPLQLVWUDomRUHMHLWDUiQRWRGRRXHPSDUWHREUD SUHYLVWDV DVVHJXUDGR DR FRQWUDWDGR QHVVHV FDVRV R
VHUYLoRRXIRUQHFLPHQWRH[HFXWDGRHPGHVDFRUGRFRP GLUHLWR GH RSWDU SHOD VXVSHQVmR GR FXPSULPHQWR GDV
RFRQWUDWR REULJDo}HV DVVXPLGDV DWp TXH VHMD QRUPDOL]DGD D
VLWXDomR
 'LU$GPLQLVWUDWLYR
 &kPDUD/HJLVODWLYDGR'LVWULWR)HGHUDO
;9 RDWUDVRVXSHULRUD QRYHQWD GLDVGRVSDJDPHQWRV ,9 UHWHQomR GRV FUpGLWRV GHFRUUHQWHV GR FRQWUDWR DWpR
GHYLGRV SHOD $GPLQLVWUDomR GHFRUUHQWHV GH REUDV OLPLWHGRVSUHMXt]RVFDXVDGRVj$GPLQLVWUDomR
VHUYLoRVRXIRUQHFLPHQWRRXSDUFHODVGHVWHVMiUHFHEL
GRVRXH[HFXWDGRVVDOYRHPFDVRGHFDODPLGDGHS~EOL †ž $ DSOLFDomR GDV PHGLGDV SUHYLVWDV QRV LQFLVRV ,H,,
FD JUDYH SHUWXUEDomR GD RUGHP LQWHUQD RX JXHUUD GHVWH DUWLJR ILFD D FULWpULR GD $GPLQLVWUDomR TXH
DVVHJXUDGR DR FRQWUDWDGR R GLUHLWR GH RSWDU SHOD SRGHUi GDU FRQWLQXLGDGH j REUD RX DR VHUYLoR SRU
VXVSHQVmRGRFXPSULPHQWRGHVXDVREULJDo}HVDWpTXH H[HFXomRGLUHWDRXLQGLUHWD
VHMDQRUPDOL]DGDDVLWXDomR
†ž eSHUPLWLGRj$GPLQLVWUDomRQRFDVRGHFRQFRUGDWDGR
;9, DQmROLEHUDomRSRUSDUWHGD$GPLQLVWUDomRGHiUHD FRQWUDWRPDQWHURFRQWUDWRSRGHQGRDVVXPLURFRQWUR
ORFDO RX REMHWR SDUD H[HFXomR GH REUD VHUYLoR RX OHGHGHWHUPLQDGDVDWLYLGDGHVGHVHUYLoRVHVVHQFLDLV
IRUQHFLPHQWR QRV SUD]RV FRQWUDWXDLV EHP FRPR GDV
IRQWHVGHPDWHULDLVQDWXUDLVHVSHFLILFDGDVQRSURMHWR
†ž 1DKLSyWHVHGRLQFLVR,,GHVWHDUWLJRRDWRGHYHUiVHU
SUHFHGLGR GH DXWRUL]DomR H[SUHVVD GR 0LQLVWUR GH
;9,, DRFRUUrQFLDGHFDVRIRUWXLWRRXGHIRUoDPDLRUUHJXODU
PHQWHFRPSURYDGDLPSHGLWLYDGDH[HFXomRGRFRQWUD (VWDGRFRPSHWHQWHRX6HFUHWiULR(VWDGXDORX0XQLFL
WR SDOFRQIRUPHRFDVR

;9,,, GHVFXPSULPHQWRGRGLVSRVWRQRLQFLVR9GRDUWVHP †ž $UHVFLVmRGHTXHWUDWDRLQFLVR,9GRDUWLJRDQWHULRU


SUHMXt]RGDVVDQo}HVSHQDLVFDEtYHLV SHUPLWH j $GPLQLVWUDomR D VHX FULWpULR DSOLFDU D
PHGLGDSUHYLVWDQRLQFLVR,GHVWHDUWLJR
2EV(VWHLQFLVRIRLDFUHVFHQWDGRSHOD/HLQž

†~QLFR 2V FDVRV GH UHVFLVmR FRQWUDWXDOVHUmRIRUPDOPHQWH &DStWXOR,9


PRWLYDGRV QRV DXWRV GR SURFHVVR DVVHJXUDGR R 'DVVDQo}HVDGPLQLVWUDWLYDVHGDWXWHODMXGLFLDO
FRQWUDGLWyULRHDDPSODGHIHVD
6HomR,
$UW $UHVFLVmRGRFRQWUDWRSRGHUiVHU 'LVSRVLo}HV*HUDLV
, GHWHUPLQDGDSRUDWRXQLODWHUDOHHVFULWRGD$GPLQLVWUD $UW $ UHFXVD LQMXVWLILFDGD GR DGMXGLFDWiULR HP DVVLQDUR
omRQRVFDRVHQXPHUDGRVQRVLQFLVRV,D;,,H;9,,GR FRQWUDWRDFHLWDURXUHWLUDURLQVWUXPHQWRHTXLYDOHQWH
DUWLJRDQWHULRU GHQWUR GR SUD]R HVWDEHOHFLGR SHOD $GPLQLVWUDomR
,, DPLJiYHOSRUDFRUGRHQWUHDVSDUWHVUHGX]LGDDWHUPR
FDUDFWHUL]D R GHVFXPSULPHQWR WRWDO GD REULJDomR
QRSURFHVVRGDOLFLWDomRGHVGHTXHKDMDFRQYHQLrQFLD
DVVXPLGD VXMHLWDQGRR jV SHQDOLGDGHV OHJDOPHQWH
SDUDD$GPLQLVWUDomR
HVWDEHOHFLGDV
,,, MXGLFLDOQRVWHUPRVGDOHJLVODomR
,9 9HWDGR
†~QLFR 2 GLVSRVWR QHVWH DUWLJR QmR VH DSOLFD DRVOLFLWDQWHV
†ž $ UHVFLVmR DGPLQLVWUDWLYD RX DPLJiYHOGHYHUiVHU FRQYRFDGRVQRVWHUPRVGRDUW†žGHVWD/HLTXH
SUHFHGLGD GH DXWRUL]DomR HVFULWD H IXQGDPHQWDGD GD QmR DFHLWDUHP D FRQWUDWDomR QDV PHVPDV FRQGLo}HV
DXWRULGDGHFRPSHWHQWH SURSRVWDVSHORSULPHLURDGMXGLFDWiULRLQFOXVLYHTXDQWR
DRSUD]RHSUHoR
†ž 4XDQGRDUHVFLVmRRFRUUHUFRPEDVHQRVLQFLVRV;,,D
;9,,GRDUWLJRDQWHULRUVHPTXHKDMDFXOSDGRFRQWUD $UW 2V DJHQWHV DGPLQLVWUDWLYRV TXH SUDWLFDUHP DWRVHP
WDGRVHUiHVWHUHVVDUFLGRGRVSUHMXt]RVUHJXODUPHQWH GHVDFRUGRFRPRVSUHFHLWRVGHVWD/HLRXYLVDQGRDIUXV
FRPSURYDGRVTXHKRXYHUVRIULGRWHQGRDLQGDGLUHLWRD WUDU RV REMHWLYRV GD OLFLWDomR VXMHLWDPVH jV VDQo}HV
SUHYLVWDVQHVWD/HLHQRVUHJXODPHQWRVSUySULRVVHP
, GHYROXomRGHJDUDQWLD SUHMXt]RGDVUHVSRQVDELOLGDGHVFLYLOHFULPLQDOTXHVHX
,, SDJDPHQWRVGHYLGRVSHOD H[HFXomRGRFRQWUDWRDWpD DWRHQVHMDU
GDWDGDUHVFLVmR
,,, SDJDPHQWRGRFXVWRGDGHVPRELOL]DomR $UW 2VFULPHVGHILQLGRVQHVWD/HLDLQGDTXHVLPSOHVPHQWH
WHQWDGRVVXMHLWDPRVVHXVDXWRUHVTXDQGRVHUYLGRUHV
††žHž YHWDGRV S~EOLFRVDOpPGDVVDQo}HVSHQDLVjSHUGDGRFDUJR
HPSUHJRIXQomRRXPDQGDWRHOHWLYR
†ž 2FRUUHQGR LPSHGLPHQWR SDUDOLVDomR RXVXVWDomRGR
FRQWUDWRRFURQRJUDPDGHH[HFXomRVHUiSURUURJDGR $UW &RQVLGHUDVH VHUYLGRU S~EOLFR SDUD RV ILQV GHVWD/HL
DXWRPDWLFDPHQWHSRULJXDOWHPSR DTXHOHTXHH[HUFHPHVPRTXHWUDQVLWRULDPHQWHRXVHP
UHPXQHUDomRFDUJRIXQomRRXHPSUHJRS~EOLFR
$UW $ UHVFLVmR GH TXH WUDWD R LQFLVR , GR DUWLJRDQWHULRU
DFDUUHWDDVVHJXLQWHVFRQVHTrQFLDVVHPSUHMXt]RGDV
VDQo}HVSUHYLVWDVHPOHL †ž (TXLSDUDVHDVHUYLGRUS~EOLFRSDUDRVILQVGHVWD/HL
TXHPH[HUFH FDUJRHPSUHJRRXIXQomRHPHQWLGDGH
, DVVXQomRLPHGLDWDGRREMHWRGRFRQWUDWRQRHVWDGRH SDUDHVWDWDO DVVLPFRQVLGHUDGDVDOpPGDVIXQGDo}HV
ORFDOHPTXHVHHQFRQWUDUSRUDWRSUySULRGD$GPLQLV HPSUHVDVS~EOLFDVHVRFLHGDGHVGHHFRQRPLDPLVWDDV
WUDomR GHPDLVHQWLGDGHVVREFRQWUROHGLUHWRRXLQGLUHWRGR
3RGHU3~EOLFR
,, RFXSDomRHXWLOL]DomRGRORFDOLQVWDODo}HVHTXLSDPHQ
WRV PDWHULDO H SHVVRDO HPSUHJDGRV QD H[HFXomR GR †ž $SHQDLPSRVWDVHUiDFUHVFLGDGDWHUoDSDUWHTXDQGRRV
FRQWUDWRQHFHVViULRVjVXDFRQWLQXLGDGHQDIRUPDGR DXWRUHVGRVFULPHVSUHYLVWRVQHVWD/HLIRUHPRFXSDQWHV
LQFLVR9GRDUWGHVWDOHL GHFDUJRHPFRPLVVmRRXGHIXQomRGHFRQILDQoDHP
yUJmR GD $GPLQLVWUDomR GLUHWD DXWDUTXLD HPSUHVD
,,, H[HFXomRGDJDUDQWLDFRQWUDWXDOSDUDUHVVDUFLPHQWRGD S~EOLFD VRFLHGDGH GH HFRQRPLD PLVWD IXQGDomR
$GPLQLVWUDomRHGRVYDORUHVGDVPXOWDVHLQGHQL]Do}HV S~EOLFDRXRXWUDHQWLGDGHFRQWURODGDGLUHWDRXLQGLUH
DHODGHYLGRV WDPHQWHSHOR3RGHU3~EOLFR
&kPDUD/HJLVODWLYDGR
 'LVWULWR)HGHUDO 'LU$GPLQLVWUDWLYR 

$UW $V LQIUDo}HV SHQDLV SUHYLVWDV QHVWD /HL SHUWLQHPjV , WHQKDPVRIULGRFRQGHQDomRGHILQLWLYDSRUSUDWLFDUHP
OLFLWDo}HVHDRVFRQWUDWRVFHOHEUDGRVSHOD8QLmR'LVWULWR SRU PHLRV GRORVRV IUDXGH ILVFDO QR UHFROKLPHQWR GH
)HGHUDO 0XQLFtSLRV H UHVSHFWLYDPHQWH DXWDUTXLDV TXDLVTXHUWULEXWRV
HPSUHVDV S~EOLFDV VRFLHGDGHV GH HFRQRPLD PLVWD
IXQGDo}HVS~EOLFDVHTXDLVTXHURXWUDVHQWLGDGHVVRE ,, WHQKDP SUDWLFDGR DWRV LOtFLWRV YLVDQGR DIUXVWUDURV
VHXFRQWUROHGLUHWRRXLQGLUHWR REMHWLYRVGDOLFLWDomR

6HomR,, ,,, GHPRQVWUDUHPQmRSRVVXLULGRQHLGDGHSDUDFRQWUDWDU


'DV6DQo}HV$GPLQLVWUDWLYDV FRPD$GPLQLVWUDomRHPYLUWXGHGHDWRVLOtFLWRVSUDWLFD
GRV
$UW 2DWUDVRLQMXVWLILFDGRQDH[HFXomRGRFRQWUDWRVXMHLWDUi 6HomR,,,
R FRQWUDWDGR j PXOWD GH PRUD QD IRUPD SUHYLVWD QR 'RV&ULPHVHGDV3HQDV
LQVWUXPHQWRFRQYRFDWyULRRXQRFRQWUDWR
$UW 'LVSHQVDU RX LQH[LJLU OLFLWDomR IRUD GDVKLSyWHVHV
†ž $ PXOWD D TXH DOXGH HVWH DUWLJR QmR LPSHGHTXHD SUHYLVWDVHPOHLRXGHL[DUGHREVHUYDUDVIRUPDOLGDGHV
$GPLQLVWUDomR UHVFLQGD XQLODWHUDOPHQWH R FRQWUDWR H SHUWLQHQWHVjGLVSHQVDRXjLQH[LJLELOLGDGH
DSOLTXHDVRXWUDVVDQo}HVSUHYLVWDVQHVWD/HL
3HQD GHWHQomRGH WUrV D FLQFR DQRVHPXOWD
†ž $PXOWDDSOLFDGDDSyVUHJXODUSURFHVVRDGPLQLVWUDWLYR
VHUiGHVFRQWDGDGDJDUDQWLDGRUHVSHFWLYRFRQWUDWDGR
†~QLFR 1D PHVPD SHQD LQFRUUH DTXHOH TXHWHQGR
FRPSURYDGDPHQWHFRQFRUULGRSDUDDFRQVXPDomRGD
†ž 6HDPXOWDIRUGHYDORUVXSHULRUDRYDORUGDJDUDQWLD
LOHJDOLGDGHEHQHILFLRXVHGDGLVSHQVDRXLQH[LJLELOLGD
SUHVWDGDDOpPGDSHUGDGHVWDUHVSRQGHUiRFRQWUDWDGR
GHLOHJDOSDUDFHOHEUDUFRQWUDWRFRPR3RGHU3~EOLFR
SHOD VXD GLIHUHQoD D TXDO VHUi GHVFRQWDGD GRV
SDJDPHQWRVHYHQWXDOPHQWHGHYLGRVSHOD$GPLQLVWUD
omRRXDLQGDTXDQGRIRURFDVRFREUDGDMXGLFLDOPHQWH $UW )UXVWUDURXIUDXGDUPHGLDQWHDMXVWHFRPELQDomRRX
TXDOTXHURXWURH[SHGLHQWHRFDUiWHUFRPSHWLWLYRGR
$UW 3HODLQH[HFXomRWRWDORXSDUFLDOGRFRQWUDWRD$GPLQLV SURFHGLPHQWROLFLWDWyULRFRPRLQWXLWRGHREWHUSDUD
WUDomR SRGHUi JDUDQWLGD D SUpYLD GHIHVD DSOLFDU DR VLRXSDUDRXWUHPYDQWDJHPGHFRUUHQWHGDDGMXGLFD
FRQWUDWDGRDVVHJXLQWHVVDQo}HV omRGRREMHWRGDOLFLWDomR

, DGYHUWrQFLD 3HQD GHWHQomR GH  GRLV  D  TXDWUR  DQRVH


PXOWD
,, PXOWDQDIRUPDSUHYLVWDQRLQVWUXPHQWRFRQYRFDWyULR
RXQRFRQWUDWR $UW 3DWURFLQDUGLUHWDRXLQGLUHWDPHQWHLQWHUHVVHSULYDGR
SHUDQWHD$GPLQLVWUDomRGDQGRFDXVDjLQVWDXUDomR
,,, VXVSHQVmR WHPSRUiULD GH SDUWLFLSDomR HPOLFLWDomRH GHOLFLWDomRRXjFHOHEUDomRGHFRQWUDWRFXMDLQYDOLGD
LPSHGLPHQWRGHFRQWUDWDUD$GPLQLVWUDomRSRUSUD]R omRYLHUDVHUGHFUHWDGDSHOR3RGHU-XGLFLiULR
QmRVXSHULRUD GRLV DQRV
3HQD GHWHQomRGH VHLV PHVHVD GRLV DQRVH
,9 GHFODUDomR GH LQLGRQHLGDGH SDUD OLFLWDURXFRQWUDWDU PXOWD
FRPD$GPLQLVWUDomR3~EOLFDHQTXDQWRSHUGXUDUHPRV
PRWLYRV GHWHUPLQDQWHV GD SXQLomR RX DWp TXH VHMD $UW $GPLWLUSRVVLELOLWDURXGDUFDXVDDTXDOTXHUPRGLILFD
SURPRYLGDDUHDELOLWDomRSHUDQWHDSUySULDDXWRULGDGH omRRXYDQWDJHPLQFOXVLYHSURUURJDomRFRQWUDWXDOHP
TXHDSOLFRXDSHQDOLGDGHTXHVHUiFRQFHGLGDVHPSUH IDYRUGRDGMXGLFDWiULRGXUDQWHDH[HFXomRGRVFRQWUD
TXH R FRQWUDWDGR UHVVDUFLU D $GPLQLVWUDomR SHORV WRVFHOHEUDGRVFRPR3RGHU3~EOLFRVHPDXWRUL]DomR
SUHMXt]RV UHVXOWDQWHV H DSyV GHFRUULGR R SUD]R GD HPOHLQRDWRFRQYRFDWyULRGDOLFLWDomRRXQRVUHVSHF
VDQomRDSOLFDGDFRPEDVHQRLQFLVRDQWHULRU WLYRVLQVWUXPHQWRVFRQWUDWXDLVRXDLQGDSDJDUIDWXUD
FRPSUHWHULomRGDRUGHPFURQROyJLFDGHVXDH[LJLELOL
†ž 6HDPXOWDDSOLFDGDIRUVXSHULRUDRYDORUGDJDUDQWLD GDGHREVHUYDGRRGLVSRVWRQRDUWGHVWD/HL
SUHVWDGDDOpPGDSHUGDGHVWDUHVSRQGHUiRFRQWUDWDGR 2EVFRPUHGDomRGDGDSHOD/HLQž
SHODVXDGLIHUHQoDTXHVHUiGHVFRQWDGDGRVSDJDPHQ
WRV HYHQWXDOPHQWH GHYLGRV SHOD $GPLQLVWUDomR RX 3HQD GHWHQomR GH  GRLV  D  TXDWUR  DQRVH
FREUDGDMXGLFLDOPHQWH PXOWD

†ž $VVDQo}HVSUHYLVWDVQRVLQFLVRV,,,,H,9GHVWHDUWLJR †~QLFR ,QFLGH QD PHVPD SHQD R FRQWUDWDGRTXHWHQGR


SRGHUmRVHUDSOLFDGDVMXQWDPHQWHFRPDGRLQFLVR,, FRPSURYDGDPHQWHFRQFRUULGRSDUDDFRQVXPDomRGD
IDFXOWDGDDGHIHVDSUpYLDGRLQWHUHVVDGRQRUHVSHFWLYR LOHJDOLGDGHREWpPYDQWDJHPLQGHYLGDRXVHEHQHILFLD
SURFHVVRQRSUD]RGH FLQFR GLDV~WHLV LQMXVWDPHQWH GDV PRGLILFDo}HV RX SURUURJDo}HV
FRQWUDWXDLV
†ž $ VDQomR HVWDEHOHFLGD QR LQFLVR ,9 GHVWH DUWLJRpGH
FRPSHWrQFLD H[FOXVLYD GR 0LQLVWUR GH (VWDGR GR $UW ,PSHGLUSHUWXUEDURXIUDXGDUDUHDOL]DomRGHTXDOTXHU
6HFUHWiULR (VWDGXDO RX 0XQLFLSDO FRQIRUPH R FDVR DWRGHSURFHGLPHQWROLFLWDWyULR
IDFXOWDGDDGHIHVDGRLQWHUHVVDGRQRUHVSHFWLYRSURFHV
VR QR SUD]R GH  GH]  GLDV GD DEHUWXUD GH YLVWD 3HQD GHWHQomRGH VHLV PHVHVD GRLV DQRVH
SRGHQGRDUHDELOLWDomRVHUUHTXHULGDDSyV GRLV DQRV PXOWD
GHVXDDSOLFDomR
$UW 'HYDVVDURVLJLORGHSURSRVWDDSUHVHQWDGDHPSURFHGL
$UW $V VDQo}HV SUHYLVWDV QRV LQFLVRV ,,, H ,9 GRDUWLJR PHQWROLFLWDWyULRRXSURSRUFLRQDUDWHUFHLURRHQVHMR
DQWHULRUSRGHUmRWDPEpPVHUDSOLFDGDVjVHPSUHVDVRX GHGHYDVViOR
DRVSURILVVLRQDLVTXHHPUD]mRGRVFRQWUDWRVUHJLGRV
SRUHVWD/HL 3HQD GHWHQomRGH GRLV D WUrV DQRVHPXOWD
 'LU$GPLQLVWUDWLYR &kPDUD/HJLVODWLYDGR'LVWULWR)HGHUDO


$UW $IDVWDU RX SURFXUDU DIDVWDU OLFLWDQWH SRU PHLRGH †~QLFR 4XDQGRDFRPXQLFDomRIRUYHUEDOPDQGDUiDDXWRUL
YLROrQFLD JUDYHDPHDoDIUDXGHRXRIHUHFLPHQWRGH GDGHUHGX]LODDWHUPRDVVLQDGRSHORDSUHVHQWDQWH
YDQWDJHPGHTXDOTXHUWLSR HSRUGXDVWHVWHPXQKDV

3HQD GHWHQomRGH GRLV D TXDWUR DQRVHPXOWDDOpP $UW 4XDQGRHPDXWRVRXGRFXPHQWRVGHTXHFRQKHFH


GDSHQDFRUUHVSRQGHQWHjYLROrQFLD UHPRVPDJLVWUDGRVRVPHPEURVGRV7ULEXQDLVRX
&RQVHOKRV GH &RQWDV RX RV WLWXODUHV GRV yUJmRV
†~QLFR ,QFRUUHQDPHVPDSHQDTXHPVHDEVWpPRXGHVLVWHGH LQWHJUDQWHV GR VLVWHPD GH FRQWUROH LQWHUQR GH
OLFLWDUHPUD]mRGDYDQWDJHPRIHUHFLGD TXDOTXHUGRV3RGHUHVYHULILFDUHPDH[LVWrQFLDGRV
FULPHVGHILQLGRVQHVWD/HLUHPHWHUmRDR0LQLVWpULR
$UW )UDXGDU HP SUHMXt]R GD )D]HQGD 3~EOLFDOLFLWDomR 3~EOLFR DV FySLDV H RV GRFXPHQWRV QHFHVViULRV DR
LQVWDXUDGDSDUDDTXLVLomRRXYHQGDGHEHQVRXPHUFD RIHUHFLPHQWRGDGHQ~QFLD
GRULDVRXFRQWUDWRGHODGHFRUUHQWH
$UW 6HUi DGPLWLGD DomR SHQDO SULYDGD VXEVLGLiULDGD
, HOHYDQGRDUELWUDULDPHQWHRVSUHoRV S~EOLFD VH HVWD QmR IRU DMXL]DGD QR SUD]R OHJDO
DSOLFDQGRVHQRTXHFRXEHURGLVSRVWRQRVDUWV
,, YHQGHQGR FRPR YHUGDGHLUDRXSHUIHLWDPHUFDGRULD HGR&yGLJRGH3URFHVVR3HQDO
IDOVLILFDGDRXGHWHULRUDGD
$UW 5HFHELGDDGHQ~QFLDHFLWDGRRUpXWHUiHVWHRSUD]R
,,, HQWUHJDQGRXPDPHUFDGRULDSRURXWUD GH GH] GLDVSDUDDSUHVHQWDomRGHGHIHVDHVFULWD
FRQWDGR GD GDWD GR VHX LQWHUURJDWyULR SRGHQGR
,9 DOWHUDQGR VXEVWkQFLD TXDOLGDGH RXTXDQWLGDGHGD MXQWDUGRFXPHQWRVDUURODUDVWHVWHPXQKDVTXHWLYHU
PHUFDGRULDIRUQHFLGD HP Q~PHUR QmR VXSHULRU D  FLQFR  H LQGLFDU DV
GHPDLVSURYDVTXHSUHWHQGDSURGX]LU
9 WRUQDQGR SRU TXDOTXHUPRGRLQMXVWDPHQWHPDLV
RQHURVDDSURSRVWDRXjH[HFXomRGRFRQWUDWR $UW 2XYLGDVDVWHVWHPXQKDVGDDFXVDomRHGDGHIHVDH
SUDWLFDGDV DV GLOLJrQFLDV LQVWUXWyULDV GHIHULGDV RX
3HQDGHWHQomRGH WUrV D VHLV DQRVHPXOWD RUGHQDGDV SHOR MXL] DEULUVHi VXFHVVLYDPHQWH R
SUD]RGH FLQFR GLDVDFDGDSDUWHSDUDDOHJDo}HV
$UW $GPLWLUjOLFLWDomRRXFHOHEUDUFRQWUDWRFRPHPSUHVD ILQDLV
RXSURILVVLRQDOGHFODUDGRLQLG{QHR
$UW 'HFRUULGRHVVHSUD]RHFRQFOXVRVRVDXWRVGHQWURGH
3HQD GHWHQomRGH VHLV PHVHVD GRLV DQRVHPXOWD  YLQWHHTXDWUR KRUDVWHUiRMXL] GH] GLDV
SDUDSURIHULUDVHQWHQoD
†~QLFR ,QFLGH QD PHVPDSHQDDTXHOHTXHGHFODUDGRLQLG{
QHRYHQKDDOLFLWDURXFRQWUDWDUFRPD$GPLQLVWUDomR $UW 'DVHQWHQoDFDEHDSHODomRLQWHUSRQtYHOQRSUD]RGH
 FLQFR GLDV
$UW 2EVWDULPSHGLURXGLILFXOWDULQMXVWDPHQWHDLQVFULomR
GH TXDOTXHU LQWHUHVVDGR QRV UHJLVWURV FDGDVWUDLV RX $UW 1RSURFHVVDPHQWRHMXOJDPHQWRGDVLQIUDo}HVSHQDLV
SURPRYHU LQGHYLGDPHQWH D DOWHUDomR VXVSHQVmR RX GHILQLGDV QHVWD /HL DVVLP FRPR RV UHFXUVRV H QDV
FDQFHODPHQWRGHUHJLVWURGRLQVFULWR H[HFXo}HV TXH OKHV GLJDP UHVSHLWR DSOLFDUVHmR
VXEVLGLDULDPHQWHR&yGLJRGH3URFHVVR3HQDOHD/HL
3HQD GHWHQomRGH VHLV PHVHVD GRLV DQRVHPXOWD GH([HFXomR3HQDO

$UW $SHQDGHPXOWDFRPLQDGDQRVDUWVDGHVWD/HL
FRQVLVWHQRSDJDPHQWRGHTXDQWLDIL[DGDQDVHQWHQoD &DStWXOR9
HFDOFXODGDHPtQGLFHVSHUFHQWXDLVFXMDEDVHFRUUHV 'RV5HFXUVRV$GPLQLVWUDWLYRV
SRQGHUiDRYDORUGDYDQWDJHPHIHWLYDPHQWHREWLGDRX
SRWHQFLDOPHQWHDXIHUtYHOSHORDJHQWH $UW 'RVDWRVGD$GPLQLVWUDomRGHFRUUHQWHVGDDSOLFDomR
GHVWD/HLFDEHP
†ž 2VtQGLFHVDTXHVHUHIHUHHVWHDUWLJRQmRSRGHUmRVHU
LQIHULRUHVD GRLVSRUFHQWR QHPVXSHULRUHVD , UHFXUVRQRSUD]RGH FLQFR GLDV~WHLVDFRQWDUGD
FLQFR SRU FHQWR  GR YDORU GR FRQWUDWR OLFLWDGR RX LQWLPDomRGRDWRRXGDODYUDWXUDGDDWDQRVFDVRV
FHOHEUDGRFRPGLVSHQVDRXLQH[LJLELOLGDGHGHOLFLWDomR GH

†ž 2SURGXWRGDDUUHFDGDomRGDPXOWDUHYHUWHUiFRQIRU D KDELOLWDomRRXLQDELOLWDomRGROLFLWDQWH


PHRFDVRj)D]HQGD)HGHUDO'LVWULWDO(VWDGXDORX E MXOJDPHQWRGDVSURSRVWDV
0XQLFLSDO F DQXODomRRXUHYRJDomRGDOLFLWDomR
6HomR,9 G LQGHIHULPHQWRGRSHGLGRGHLQVFULomRHPUHJLV
'R3URFHVVRHGR3URFHGLPHQWR-XGLFLDO WURFDGDVWUDOVXDDOWHUDomRRXFDQFHODPHQWR
H UHVFLVmRGRFRQWUDWRDTXHVHUHIHUHRLQFLVR,
$UW 2V FULPHV GHILQLGRV QHVWD /HL VmR GH DomRSHQDO GR DUW GHVWD /HL 2EV
2EV FRP UHGDomR GDGD
S~EOLFD LQFRQGLFLRQDGD FDEHQGR DR 0LQLVWpULR SHOD/HLQž
3~EOLFRSURPRYrOD I DSOLFDomRGDVSHQDVGHDGYHUWrQFLDVXVSHQVmR
WHPSRUiULDRXGHPXOWD
$UW 4XDOTXHU SHVVRD SRGHUi SURYRFDU SDUD RVHIHLWRV
GHVWD /HL D LQLFLDWLYD GR 0LQLVWpULR 3~EOLFR ,, UHSUHVHQWDomRQRSUD]RGH FLQFR  GLDV~WHLVGD
IRUQHFHQGROKHSRUHVFULWRLQIRUPDo}HVVREUHRIDWR LQWLPDomR GD GHFLVmR UHODFLRQDGD FRP R REMHWR GD
HVXDDXWRULDEHPFRPRDVFLUFXQVWkQFLDVHPTXHVH OLFLWDomR RXGRFRQWUDWRGHTXHQmRFDLEDUHFXUVR
GHXDRFRUUrQFLD KLHUiUTXLFR
&kPDUD/HJLVODWLYDGR
  'LVWULWR)HGHUDO 'LU$GPLQLVWUDWLYR 

,,, SHGLGRGHUHFRQVLGHUDomRGHGHFLVmRGH0LQLVWURGH †ž 2VFRQVyUFLRVS~EOLFRVSRGHUmRUHDOL]DUOLFLWDomRGD


(VWDGRRX6HFUHWiULR(VWDGXDORX0XQLFLSDOFRQIRU TXDO QRV WHUPRV GR HGLWDO GHFRUUDP FRQWUDWRV
PHRFDVRQDKLSyWHVHGR†žGRDUWGHVWD/HLQR DGPLQLVWUDWLYRVFHOHEUDGRVSRUyUJmRVRXHQWLGDGHV
SUD]RGH GH] GLDV~WHLVGDLQWLPDomRGRDWR GRVHQWHVGD)HGHUDomRFRQVRUFLDGRV2EV,QFOXtGRSHOD/HLQž
GH

†ž $LQWLPDomRGRVDWRVUHIHULGRVQRLQFLVR,DOtQHDVD †ž eIDFXOWDGRjHQWLGDGHLQWHUHVVDGDRDFRPSDQKDPHQ


E F H H GHVWH DUWLJR H[FOXtGRV RV UHODWLYRV D WRGDOLFLWDomRHGDH[HFXomRGRFRQWUDWR
DGYHUWrQFLD H PXOWD GH PRUD H QR LQFLVR ,,, VHUi
IHLWDPHGLDQWHSXEOLFDomRQDLPSUHQVDRILFLDOVDOYR 2EV DQWLJRSDUiJUDIR~QLFRUHQXPHUDGRSDUD†žHPUD]mRGDLQFOXVmRGR†ž
SHOD/HLQžGH
SDUD RV FDVRV SUHYLVWRV QDV DOtQHDV D H E VH
SUHVHQWHVRVSUHSRVWRVGRVOLFLWDQWHVQRDWRHPTXH $UW 2FRQWUROHGDVGHVSHVDVGHFRUUHQWHVGRVFRQWUDWRVH
IRLDGRWDGDDGHFLVmRTXDQGRSRGHUiVHUIHLWDSRU GHPDLVLQVWUXPHQWRVUHJLGRVSRUHVWD/HLVHUiIHLWR
FRPXQLFDomRGLUHWDDRVLQWHUHVVDGRVHODYUDGDDDWD SHOR 7ULEXQDO GH &RQWDV FRPSHWHQWH QD IRUPD GD
OHJLVODomRSHUWLQHQWHILFDQGRRVyUJmRVLQWHUHVVDGRV
†ž 2UHFXUVRSUHYLVWR QDV DOtQHDVDHEGRLQFLVR, GD$GPLQLVWUDomRUHVSRQViYHLVSHODGHPRQVWUDomRGD
GHVWHDUWLJRWHUiHIHLWRVXVSHQVLYRSRGHQGRDDXWRUL OHJDOLGDGHHUHJXODULGDGHGDGHVSHVDHH[HFXomRQRV
GDGHFRPSHWHQWHPRWLYDGDPHQWHHSUHVHQWHVUD]}HV WHUPRVGD&RQVWLWXLomRHVHPSUHMXt]RGRVLVWHPDGH
GHLQWHUHVVHS~EOLFRDWULEXLUDRUHFXUVRLQWHUSRVWR FRQWUROHLQWHUQRQHODSUHYLVWR
HILFiFLDVXVSHQVLYDDRVGHPDLVUHFXUVRV
†ž 4XDOTXHU OLFLWDQWH FRQWUDWDGR RX SHVVRDItVLFDRX
†ž ,QWHUSRVWR R UHFXUVR VHUi FRPXQLFDGRDRVGHPDLV MXUtGLFDSRGHUiUHSUHVHQWDUDR7ULEXQDOGH&RQWDVRX
OLFLWDQWHV TXH SRGHUmR LPSXJQiOR QR SUD]R GH  DRVyUJmRVLQWHJUDQWHVGRVLVWHPDGHFRQWUROHLQWHUQR
FLQFR GLDV~WHLV FRQWUDLUUHJXODULGDGHVQDDSOLFDomRGHVWD/HLSDUDRV
ILQVGRGLVSRVWRQHVWHDUWLJR
†ž 2 UHFXUVR VHUi GLULJLGR j DXWRULGDGHVXSHULRUSRU
LQWHUPpGLRGDTXHSUDWLFRXRDWRUHFRUULGRDTXDO †ž 2V 7ULEXQDLV GH &RQWDV H RV yUJmRVLQWHJUDQWHVGR
SRGHUi UHFRQVLGHUDU VXD GHFLVmR QR SUD]R GH  VLVWHPD GH FRQWUROH LQWHUQR SRGHUmR VROLFLWDU SDUD
FLQFR  GLDV ~WHLV RX QHVVH PHVPR SUD]R ID]rOR H[DPHDWpRGLD~WLOLPHGLDWDPHQWHDQWHULRUjGDWD
VXELUGHYLGDPHQWHLQIRUPDGRGHYHQGRQHVWHFDVR GH UHFHELPHQWR GDV SURSRVWDV FySLD GH HGLWDO GH
DGHFLVmRVHUSURIHULGDGHQWURGRSUD]RGH FLQFR OLFLWDomR Mi SXEOLFDGR REULJDQGRVH RV yUJmRV RX
GLDV~WHLVFRQWDGRGRUHFHELPHQWRGRUHFXUVRVRE
HQWLGDGHVGD$GPLQLVWUDomRLQWHUHVVDGDjDGRomRGH
SHQDGHUHVSRQVDELOLGDGH
PHGLGDVFRUUHWLYDVSHUWLQHQWHVTXHHPIXQomRGHVVH
H[DPHOKHVIRUHPGHWHUPLQDGDV
†ž 1HQKXPSUD]RGHUHFXUVRUHSUHVHQWDomRRXSHGLGR
GHUHFRQVLGHUDomRVHLQLFLDRXFRUUHVHPTXHRVDXWRV
GRSURFHVVRHVWHMDPFRPYLVWDIUDQTXHDGDDRLQWHUHV $UW 2VLVWHPDLQVWLWXtGRQHVWD/HLQmRLPSHGHDSUpTXD
VDGR OLILFDomRGHOLFLWDQWHVQDVFRQFRUUrQFLDVDVHUSURFHGL
GD VHPSUH TXH R REMHWR GD OLFLWDomR UHFRPHQGH D
†ž (PVHWUDWDQGRGHOLFLWDo}HVHIHWXDGDVQDPRGDOLGD DQiOLVHPDLVGHWLGDGDTXDOLILFDomRWpFQLFDGRVLQWH
GH GH FDUWD FRQYLWH RV SUD]RV HVWDEHOHFLGRV QRV UHVVDGRV
LQFLVRV,H,,HQR†žGHVWHDUWLJRVHUmRGHGRLVGLDV
~WHLV †ž $DGRomRGRSURFHGLPHQWRGHSUpTXDOLILFDomRVHUi
&DStWXOR9, IHLWDPHGLDQWHSURSRVWDGDDXWRULGDGHFRPSHWHQWH
'LVSRVLo}HV)LQDLVH7UDQVLWyULDV DSURYDGDSHODLPHGLDWDPHQWHVXSHULRU

$UW 1D FRQWDJHP GRV SUD]RV HVWDEHOHFLGRV QHVWD/HL †ž 1D SUpTXDOLILFDomR VHUmR REVHUYDGDVDVH[LJrQFLDV
H[FOXLUVHiRGLDGRLQtFLRHLQFOXLUVHiRGRYHQFL GHVWD/HLUHODWLYDVjFRQFRUUrQFLDjFRQYRFDomRGRV
PHQWR H FRQVLGHUDUVHmR RV GLDV FRQVHFXWLYRV H[ LQWHUHVVDGRVDRSURFHGLPHQWRHjDQiOLVHGDGRFX
FHWRTXDQGRIRUH[SOLFLWDPHQWHGLVSRVWRHPFRQWUiULR PHQWDomR

†~QLFR 6y VH LQLFLDP H YHQFHP RV SUD]RVUHIHULGRVQHVWH $UW 2VyUJmRVGD$GPLQLVWUDomRSRGHUmRH[SHGLUQRUPDV


DUWLJRHPGLDGHH[SHGLHQWHQRyUJmRRXQDHQWLGDGH UHODWLYDV DRV SURFHGLPHQWRV RSHUDFLRQDLV D VHUHP
REVHUYDGRVQDH[HFXomRGDVOLFLWDo}HVQRkPELWRGH
$UW $$GPLQLVWUDomRVySRGHUiFRQWUDWDUSDJDUSUHPLDU VXDFRPSHWrQFLDREVHUYDGDVDVGLVSRVLo}HVGHVWD/HL
RX UHFHEHU SURMHWR RX VHUYLoR WpFQLFR HVSHFLDOL]DGR
GHVGHTXHRDXWRUFHGDRVGLUHLWRVSDWULPRQLDLVDHOH †~QLFR $VQRUPDVDTXHVHUHIHUHHVWHDUWLJRDSyVDSURYDomR
UHODWLYRVHD$GPLQLVWUDomRSRVVDXWLOL]iORGHDFRUGR GDDXWRULGDGHFRPSHWHQWHGHYHUmRVHUSXEOLFDGDVQD
FRP R SUHYLVWR QR UHJXODPHQWR GH FRQFXUVR RX QR LPSUHQVDRILFLDO
DMXVWHSDUDVXDHODERUDomR
$UW $SOLFDPVHDVGLVSRVLo}HVGHVWD/HLQRTXHFRXEHU
†~QLFR 4XDQGR R SURMHWR UHIHULUVH D REUDLPDWHULDOGH DRVFRQYrQLRVDFRUGRVDMXVWHVHRXWURVLQVWUXPHQWRV
FDUiWHU WHFQROyJLFR LQVXVFHWtYHO GH SULYLOpJLR D FRQJrQHUHV FHOHEUDGRV SRU yUJmRV H HQWLGDGHV GD
FHVVmRGRVGLUHLWRVLQFOXLUiRIRUQHFLPHQWRGHWRGRV $GPLQLVWUDomR
RV GDGRV GRFXPHQWRV H HOHPHQWRV GH LQIRUPDomR
SHUWLQHQWHV j WHFQRORJLD GH FRQFHSomR GHVHQYROYL †ž $ FHOHEUDomR GH FRQYrQLR DFRUGR RXDMXVWHSHORV
PHQWRIL[DomRHPVXSRUWHItVLFRGHTXDOTXHUQDWXUH
yUJmRVRXHQWLGDGHVGD$GPLQLVWUDomR3~EOLFDGHSHQ
]DHDSOLFDomRGDREUD
GH GH SUpYLD DSURYDomR LQWHUHVVDGD R TXDO GHYHUi
FRQWHUQRPtQLPRDVVHJXLQWHVLQIRUPDo}HV
$UW 4XDQGR R REMHWR GR FRQWUDWR LQWHUHVVDU D PDLVGH
XPDHQWLGDGHS~EOLFDFDEHUiDRyUJmRFRQWUDWDQWH
SHUDQWHD HQWLGDGHLQWHUHVVDGDUHVSRQGHUSHODVXD , LGHQWLILFDomRGRREMHWRDVHUH[HFXWDGR
ERDH[HFXomRILVFDOL]DomRHSDJDPHQWR ,, PHWDVDVHUHPDWLQJLGDV
,,, HWDSDVRXIDVHVGHH[HFXomR
 'LU$GPLQLVWUDWLYR &kPDUD/HJLVODWLYDGR'LVWULWR)HGHUDO


,9 SODQRGHDSOLFDomRGRVUHFXUVRVILQDQFHLURV $UW 2V(VWDGRVR'LVWULWR)HGHUDORV0XQLFtSLRVHDVHQ


9 FURQRJUDPDGHGHVHPEROVR WLGDGHV GD DGPLQLVWUDomR LQGLUHWD GHYHUmR DGDSWDU
9, SUHYLVmRGHLQtFLRHILPGDH[HFXomRGRREMHWREHP VXDVQRUPDVVREUHOLFLWDo}HVHFRQWUDWRVDRGLVSRVWR
DVVLPGDFRQFOXVmRGDVHWDSDVRXIDVHVSURJUDPDGDV QHVWD/HL
9,, VHRDMXVWHFRPSUHHQGHUREUDRXVHUYLoRGHHQJHQKD
ULD FRPSURYDomR GH TXH RV UHFXUVRV SUySULRV SDUD $UW $VVRFLHGDGHVGHHFRQRPLDPLVWDHPSUHVDVHIXQGD
FRPSOHPHQWDUDH[HFXomRGRREMHWRHVWmRGHYLGDPHQ o}HVS~EOLFDVHGHPDLVHQWLGDGHVFRQWURODGDVGLUHWD
WHDVVHJXUDGRVVDOYRVHRFXVWRWRWDOGRHPSUHHQGL RXLQGLUHWDPHQWHSHOD8QLmRHSHODVHQWLGDGHVUHIHUL
PHQWRUHFDLUVREUHDHQWLGDGHRXyUJmRGHVFHQWUDOL]DGRU GDVQRDUWLJRDQWHULRUHGLWDUmRUHJXODPHQWRVSUySULRV
GHYLGDPHQWHSXEOLFDGRVILFDQGRVXMHLWDVjVGLVSRVL
†ž $VVLQDGRRFRQYrQLRDHQWLGDGHRXyUJmRUHSDVVDGRU o}HVGHVWD/HL
GDUiFLrQFLDGRPHVPRj$VVHPEOpLD/HJLVODWLYDRXj
&kPDUD0XQLFLSDOUHVSHFWLYD †~QLFR 2V UHJXODPHQWRV D TXH VH UHIHUH HVWHDUWLJRQR
kPELWR GD $GPLQLVWUDomR 3~EOLFD DSyV DSURYDGRV
†ž $V SDUFHODV GR FRQYrQLR VHUmR OLEHUDGDVHPHVWULWD SHOD DXWRULGDGH GH QtYHO VXSHULRU D TXH HVWLYHUHP
FRQIRUPLGDGHFRPRSODQRGHDSOLFDomRDSURYDGRH[ YLQFXODGRV RV UHVSHFWLYRV yUJmRV VRFLHGDGHV H HQ
FHWRQRVFDVRVDVHJXLUHPTXHDVPHVPDVILFDUmRUH WLGDGHVGHYHUmRVHUSXEOLFDGRVQDLPSUHQVDRILFLDO
WLGDVDWpRVDQHDPHQWRGDVLPSURSULHGDGHVRFRUUHQ
WHV $UW 2VYDORUHVIL[DGRVSRUHVWD/HLSRGHUmRVHUDQXDOPHQ
, TXDQGR QmR WLYHU KDYLGR FRPSURYDomRGDERDH WHUHYLVWRVSHOR3RGHU([HFXWLYR)HGHUDOTXHRVIDUi
UHJXODUDSOLFDomRGDSDUFHODDQWHULRUPHQWHUHFHELGD SXEOLFDUQR'LiULR2ILFLDOGD8QLmRREVHUYDQGRFRPR
QDIRUPDGDOHJLVODomRDSOLFiYHOLQFOXVLYHPHGLDQWH OLPLWHVXSHULRUDYDULDomRJHUDOGRVSUHoRVGRPHUFD
SURFHGLPHQWRVGHILVFDOL]DomRORFDOUHDOL]DGRVSHULRGL GRQRSHUtRGR
FDPHQWHSHODHQWLGDGHRXyUJmRGHVFHQWUDOL]DGRUGRV
UHFXUVRV RX SHOR yUJmR FRPSHWHQWH GR VLVWHPD GH $UW 2GLVSRVWRQHVWD/HLQmRVHDSOLFDjVOLFLWDo}HVLQVWDX
FRQWUROHLQWHUQRGD$GPLQLVWUDomR3~EOLFD UDGDVHDRVFRQWUDWRVDVVLQDGRVDQWHULRUPHQWHjVXD
YLJrQFLDUHVVDOYDGRRGLVSRVWRQRDUWQRV††ž
,, TXDQGRYHULILFDGRGHVYLRGHILQDOLGDGHQDDSOLFDomR žHžGRDUWQRLQFLVR;9GRDUWEHPDVVLP
GRVUHFXUVRVDWUDVRVQmRMXVWLILFDGRVQRFXPSULPHQWR RGLVSRVWRQRFDSXWGRDUWžFRPUHODomRDRSDJD
GDVHWDSDVRXIDVHVSURJUDPDGDVSUiWLFDVDWHQWDWyUL PHQWRGDVREULJDo}HVQDRUGHPFURQROyJLFDSRGHQGR
DV DRV SULQFtSLRV IXQGDPHQWDLV GH $GPLQLVWUDomR HVWDVHUREVHUYDGDQRSUD]RGHQRYHQWDGLDVFRQWDGRV
3~EOLFDQDVFRQWUDWDo}HVHGHPDLVDWRVSUDWLFDGRVQD GDYLJrQFLDGHVWD/HLVHSDUDGDPHQWHSDUDDVREULJD
H[HFXomR GR FRQYrQLR RX R LQDGLPSOHPHQWR GR o}HVUHODWLYDVDRVFRQWUDWRVUHJLGRVSRUOHJLVODomRDQWHULRU
H[HFXWRU FRP UHODomR D RXWUDV FOiXVXODV FRQYHQLDLV j/HLQžGHGHMXQKRGH
EiVLFDV
†~QLFR 2V FRQWUDWRV UHODWLYRV D LPyYHLV GRSDWULP{QLRGD
,,, TXDQGR R H[HFXWRU GHL[DU GH DGRWDU DVPHGLGDV 8QLmR FRQWLQXDP D UHJHUVH SHODV GLVSRVLo}HV GR
VDQHDGRUDVDSRQWDGDVSHORSDUWtFLSHUHSDVVDGRUGRV 'HFUHWROHLQžGHGHVHWHPEURGHFRP
UHFXUVRVRXSRULQWHJUDQWHVGRUHVSHFWLYRVLVWHPDGH VXDVDOWHUDo}HVHRVUHODWLYRVDRSHUDo}HVGHFUpGLWR
FRQWUROHLQWHUQR LQWHUQRRXH[WHUQRFHOHEUDGRVSHOD8QLmRRXDFRQFHV
VmR GH JDUDQWLD GR 7HVRXUR 1DFLRQDO FRQWLQXDP
†ž 2VVDOGRVGHFRQYrQLRHQTXDQWRQmRXWLOL]DGRVVHUmR UHJLGRVSHODOHJLVODomRSHUWLQHQWHDSOLFDQGRVHHVWD
REULJDWRULDPHQWHDSOLFDGRVHPFDGHUQHWDVGHSRXSDQ /HLQRTXHFRXEHU
oDGHLQVWLWXLomRILQDQFHLUDRILFLDOVHDSUHYLVmRGHVHX
XVRIRULJXDORXVXSHULRUDXPPrVRXHPIXQGRGH $UW 1DVFRQFHVV}HVGHOLQKDVDpUHDVREVHUYDUVHiSURFH
DSOLFDomRILQDQFHLUDGHFXUWRSUD]RRXRSHUDomRGH GLPHQWROLFLWDWyULRHVSHFtILFRDVHUHVWDEHOHFLGRQR
PHUFDGRDEHUWRODVWUHDGDHPWtWXORVGDGtYLGDS~EOL &yGLJR%UDVLOHLURGH$HURQiXWLFD
FDTXDQGRDXWLOL]DomRGRVPHVPRVYHULILFDUVHHP
SUD]RVPHQRUHVTXHXPPrV $UW (PVXDVOLFLWDo}HVHFRQWUDWDo}HVDGPLQLVWUDWLYDVDV
UHSDUWLo}HVVHGLDGDVQRH[WHULRUREVHUYDUmRDVSHFXOL
†ž $VUHFHLWDVILQDQFHLUDVDXIHULGDVQDIRUPDGRSDUiJUD DULGDGHVORFDLVHRVSULQFtSLRVEiVLFRVGHVWD/HLQD
IR DQWHULRU VHUmR REULJDWRULDPHQWH FRPSXWDGDV D IRUPDGHUHJXODPHQWDomRHVSHFtILFD
FUpGLWRGRFRQYrQLRHDSOLFDGDVH[FOXVLYDPHQWHQR
REMHWR GH VXD ILQDOLGDGH GHYHQGR FRQVWDU GH GH $UW $SOLFDPVHjVOLFLWDo}HVHDRVFRQWUDWRVSDUDSHUPLV
PRQVWUDWLYRHVSHFtILFRTXHLQWHJUDUiDVSUHVWDo}HVGH VmRRXFRQFHVVmRGHVHUYLoRVS~EOLFRVRVGLVSRVLWLYRV
FRQWDVGRDMXVWH GHVWD/HLTXHQmRFRQIOLWHPFRPDOHJLVODomRHVSHFtIL
FDVREUHRDVVXQWR
†ž 4XDQGRGDFRQFOXVmRGHQ~QFLDUHVFLVmRRXH[WLQomR
GRFRQYrQLRDFRUGRRXDMXVWHRVVDOGRVILQDQFHLURV †~QLFR $VH[LJrQFLDVFRQWLGDVQRVLQFLVRV,,D,9GR†žGR
UHPDQHVFHQWHVLQFOXVLYHRVSURYHQLHQWHVGDVUHFHLWDV DUWžVHUmRGLVSHQVDGDVQDVOLFLWDo}HVSDUDFRQFHVVmR
REWLGDV GDV DSOLFDo}HV ILQDQFHLUDV UHDOL]DGDV VHUmR GHVHUYLoRVFRPH[HFXomRSUpYLDGHREUDVHPTXHQmR
GHYROYLGRV j HQWLGDGH RX yUJmR UHSDVVDGRU GRV IRUDPSUHYLVWRVGHVHPEROVRSRUSDUWHGD$GPLQLVWUD
UHFXUVRVQRSUD]RLPSURUURJiYHOGH WULQWD GLDV omR3~EOLFDFRQFHGHQWH
GR HYHQWR VRE SHQD GD LPHGLDWD LQVWDXUDomR GH
WRPDGDGHFRQWDVHVSHFLDOGRUHVSRQViYHOSURYLGHQFL $UW (VWDOHLHQWUDHPYLJRUQDGDWDGHVXDSXEOLFDomR
DGDSHODDXWRULGDGHFRPSHWHQWHGRyUJmRRXHQWLGDGH
WLWXODUGRVUHFXUVRV $UW 5HYRJDPVHDVGLVSRVLo}HVHPFRQWUiULRHVSHFLDOPHQ
WHRV'HFUHWRVOHLVQžVGHGHQRYHPEURGH
$UW $V REUDV VHUYLoRV FRPSUDV H DOLHQDo}HVUHDOL]DGRV GHGHMXOKRGHGHGH
SHORVyUJmRVGRV3RGHUHV/HJLVODWLYRH-XGLFLiULRHGR VHWHPEURGHD/HLQžGHGHVHWHPEUR
7ULEXQDOGH&RQWDVUHJHPVHSHODVQRUPDVGHVWD/HL GHHRDUWGD/HLQžGH
QRTXHFRXEHUQDVWUrVHVIHUDVDGPLQLVWUDWLYDV
 6HFUHWDULDGH(VWDGRGH$ GPLQLVWUDomR3~EOLFD'LVWULWR)HGHUDO /HJLVODomR 

†ž 3HORPHQRVFLQTXHQWDSRUFHQWRGRVFDUJRVHPFRPLV
/HJLVODomRGRVHUYLGRUS~EOLFRGR'LVWULWR)HGHUDO VmRGHYHPVHUSURYLGRVSRUVHUYLGRUS~EOLFRGHFDUUHL
UDQRVFDVRVHFRQGLo}HVSUHYLVWRVHPOHL
/HL&RPSOHPHQWDUGLVWULWDOQž
†ž eSURLELGDDGHVLJQDomRSDUDIXQomRGHFRQILDQoDRXD
QRPHDomR SDUD FDUJR HP FRPLVVmR LQFOXtGRV RV GH
QDWXUH]DHVSHFLDOGHSHVVRDTXHWHQKDSUDWLFDGRDWR
3XEOLFDGDQR'2')QžGH3iJVD
WLSLILFDGR FRPR FDXVD GH LQHOHJLELOLGDGH SUHYLVWD QD
$OWHUDo}HV
/HL&RPSOHPHQWDUQžGHGRGIGH3DJ
OHJLVODomR HOHLWRUDO REVHUYDGR R PHVPR SUD]R GH
/HL&RPSOHPHQWDUQžGHGRGIGH3DJ6XSOHPHQWR$ LQFRPSDWLELOLGDGHGHVVDOHJLVODomR
/HL&RPSOHPHQWDUQžGHGRGIGH3iJ
/HL&RPSOHPHQWDUQžGHGRGIGH3iJ
$UWž $VIXQo}HVGHFRQILDQoDSULYDWLYDVGHVHUYLGRUHIHWLYR
'LVS}HVREUHRUHJLPHMXUtGLFRVGRVVHUYLGRUHVS~EOLFRVFLYLVGR GHVWLQDPVHH[FOXVLYDPHQWHjVDWULEXLo}HVGHGLUHomR
'LVWULWR)HGHUDOGDVDXWDUTXLDVHGDVIXQGDo}HVS~EOLFDVGLVWULWDLV FKHILDHDVVHVVRUDPHQWR

7tWXOR, $UWž 6mR UHTXLVLWRV EiVLFRV SDUD LQYHVWLGXUD HPFDUJR


&DStWXORÔQLFR S~EOLFR
'$6',6326,d¯(635(/,0,1$5(6
, DQDFLRQDOLGDGHEUDVLOHLUD
$UWž (VWD/HL&RPSOHPHQWDULQVWLWXLRUHJLPHMXUtGLFRGRV ,, RJR]RGRVGLUHLWRVSROtWLFRV
VHUYLGRUHV S~EOLFRV FLYLV GD DGPLQLVWUDomR GLUHWD ,,,DTXLWDomRFRPDVREULJDo}HVPLOLWDUHVHHOHLWRUDLV
DXWiUTXLFDHIXQGDFLRQDOHGRVyUJmRV UHODWLYDPHQWH ,9 R QtYHO GH HVFRODULGDGH H[LJLGR SDUD R H[HUFtFLRGR
DXW{QRPRVGR'LVWULWR)HGHUDO FDUJR
9 DLGDGHPtQLPDGHGH]RLWRDQRV
$UWž 3DUD RV HIHLWRV GHVWD /HL &RPSOHPHQWDUVHUYLGRU 9, DDSWLGmRItVLFDHPHQWDO
S~EOLFR p D SHVVRD OHJDOPHQWH LQYHVWLGD HP FDUJR
S~EOLFR †ž $ OHL SRGH HVWDEHOHFHU UHTXLVLWRV HVSHFtILFRVSDUDD
LQYHVWLGXUDHPFDUJRVS~EOLFRV
$UWž &DUJRS~EOLFRpRFRQMXQWRGHDWULEXLo}HVHUHVSRQVDEL
OLGDGHVSUHYLVWDVQDHVWUXWXUDRUJDQL]DFLRQDOHFRPHWL †ž 2SURYLPHQWRGHFDUJRS~EOLFR SRUHVWUDQJHLURGHYH
GDVDXPVHUYLGRUS~EOLFR REVHUYDURGLVSRVWRHP/HLIHGHUDO

†~QLFR 2VFDUJRVS~EOLFRVVmRFULDGRVSRUOHLFRPGHQRPLQD †ž 2VUHTXLVLWRVSDUDLQYHVWLGXUDHPFDUJRS~EOLFRGHYHP


omR SUySULD H VXEVtGLR RX YHQFLPHQWRV SDJRV SHORV VHUFRPSURYDGRVSRURFDVLmRGDSRVVH
FRIUHVS~EOLFRVSDUDSURYLPHQWRHPFDUiWHUHIHWLYRRX
HPFRPLVVmR $UWž 6mRIRUPDVGHSURYLPHQWRGHFDUJRS~EOLFR

7Ì78/2,, , QRPHDomR


'26&$5*263Ô%/,&26('$6)81d¯(6'(&21),$1d$ ,, UHYHUVmR
,,, DSURYHLWDPHQWR
&DStWXOR, ,9 UHLQWHJUDomR
'23529,0(172 9 UHFRQGXomR

6HomR, $UWž eYHGDGRHGLWDUDWRVGHQRPHDomRSRVVHRXH[HUFtFLR


'DV'LVSRVLo}HV*HUDLV FRPHIHLWRUHWURDWLYR

$UWž $LQYHVWLGXUDHPFDUJRGHSURYLPHQWRHIHWLYRGHSHQGH $UW 2DWRGHSURYLPHQWRGHFDUJRS~EOLFRFRPSHWHDR


GHSUpYLDDSURYDomRHPFRQFXUVRS~EOLFR
, *RYHUQDGRUQR3RGHU([HFXWLYR
$UWž 2VFDUJRVHPFRPLVVmRGHVWLQDGRVH[FOXVLYDPHQWHjV ,, 3UHVLGHQWHGD&kPDUD/HJLVODWLYD
DWULEXLo}HVGHGLUHomRFKHILDHDVVHVVRUDPHQWRVmRGH ,,, 3UHVLGHQWHGR7ULEXQDOGH&RQWDV
OLYUHQRPHDomRHH[RQHUDomRSHODDXWRULGDGHFRPSH
WHQWH 6HomR,,
'R&RQFXUVR3~EOLFR
†ž 3DUD RV ILQV GHVWD /HL&RPSOHPHQWDUFRQVLGHUDVH
FDUJRHPFRPLVVmR $UW $VQRUPDVJHUDLVVREUHFRQFXUVRS~EOLFRVmRDVIL[DGDV
HPOHLHVSHFtILFD
, GHGLUHomRDTXHOHFXMRGHVHPSHQKRHQYROYDDWULEXL
o}HVGDDGPLQLVWUDomRVXSHULRU †ž 9(7$'2 

,, GH FKHILD DTXHOH FXMR GHVHPSHQKR HQYROYDUHODomR †ž 2FRQFXUVRS~EOLFRpGHSURYDVRXGHSURYDVHWtWXORV


GLUHWDHLPHGLDWDGHVXERUGLQDomR FRQIRUPHGLVSXVHUDOHLGRUHVSHFWLYRSODQRGHFDUUHLUD

,,, GHDVVHVVRUDPHQWRDTXHOHFXMDVDWULEXLo}HVVHMDPSDUD $UW 2HGLWDOGHFRQFXUVRS~EOLFRWHPGHUHVHUYDUYLQWHSRU


DX[LOLDU FHQWR GDV YDJDV SDUD VHUHP SUHHQFKLGDV SRU SHVVRD
FRPGHILFLrQFLDGHVSUH]DGDDSDUWHGHFLPDO
D RVGHWHQWRUHVGHPDQGDWRHOHWLYR
E RVRFXSDQWHVGHFDUJRVYLWDOtFLRV †ž $YDJDQmRSUHHQFKLGDQDIRUPDGRFDSXWUHYHUWHVH
F RVRFXSDQWHVGHFDUJRVGHGLUHomRRXGHFKHILD SDUDSURYLPHQWRGRVGHPDLVFDQGLGDWRV
  /HJLVODomR 6HFUHWDULDGH(VWDGRGH$GPLQLVWUDomR3~EOLFD'LVWULWR)HGHUDO


†ž $GHILFLrQFLDHDFRPSDWLELOLGDGHSDUDDVDWULEXLo}HVGR , GHVHUYLGRURFXSDQWHGHFDUJRGHSURYLPHQWRHIHWLYR


FDUJRVmRYHULILFDGDVDQWHVGDSRVVHJDUDQWLGRUHFXUVR LQFOXtGRVRVDSRVHQWDGRVGHVGHTXHVHMDREVHUYDGD
HP FDVR GH GHFLVmR GHQHJDWyULD FRP VXVSHQVmR GD
FRQWDJHPGRSUD]RSDUDDSRVVH D DFRPSDWLELOLGDGHGRJUDXGHHVFRODULGDGHGRFDUJR
HIHWLYRFRPRFDUJRHPFRPLVVmRRXDIXQomRGH
†ž 1mR HVWmR DEUDQJLGDV SHORV EHQHItFLRV GHVWHDUWLJRD FRQILDQoD
SHVVRDFRPGHILFLrQFLDDSWDSDUDWUDEDOKDUQRUPDOPHQ
WHHDLQDSWDSDUDTXDOTXHUWUDEDOKR E DFRPSDWLELOLGDGHHDFRPSOH[LGDGHGDVDWULEXLo}HV
GR FDUJR HIHWLYR FRP R FDUJR HP FRPLVVmR RX D
$UW 2FRQFXUVRS~EOLFRWHPYDOLGDGHGHDWpGRLVDQRVD IXQomRGHFRQILDQoD
TXDO SRGH VHU SURUURJDGD XPD ~QLFD YH] SRU LJXDO
SHUtRGRQDIRUPDGRHGLWDO ,, UHDOL]DGD DQWHV GR LQtFLR GR YtQFXOR IDPLOLDU HQWUHR
DJHQWHS~EOLFRHRQRPHDGRRXGHVLJQDGR
†ž 1RSHUtRGRGHYDOLGDGHGRFRQFXUVRS~EOLFRRFDQGLGD
WRDSURYDGRGHYHVHUQRPHDGRFRPSULRULGDGHVREUH ,,, GHSHVVRDMiHPH[HUFtFLRQRPHVPRyUJmRDXWDUTXLD
QRYRVFRQFXUVDGRVSDUDDVVXPLUFDUJRQDFDUUHLUD RXIXQGDomRDQWHVGRLQtFLRGRYtQFXORIDPLOLDUFRPR
DJHQWHS~EOLFRSDUDFDUJRIXQomRRXHPSUHJRGHQtYHO
†ž 2FDQGLGDWRDSURYDGRHPFRQFXUVRS~EOLFRQRSUD]R KLHUiUTXLFRLJXDORXPDLVEDL[RTXHRDQWHULRUPHQWH
GHFLQFRGLDVFRQWDGRVGDSXEOLFDomRGRDWRGHQRPHD RFXSDGR
omRSRGHVROLFLWDUVHXUHSRVLFLRQDPHQWRSDUDRILQDOGD
†ž (PTXDOTXHUFDVRpYHGDGDDPDQXWHQomRGHIDPLOLDU
OLVWDGHFODVVLILFDomR
RFXSDQWHGHFDUJRHPFRPLVVmRRXIXQomRGHFRQILDQoD
VREVXERUGLQDomRKLHUiUTXLFDPHGLDWDRXLPHGLDWD
6HomR,,,
'D1RPHDomR
6HomR,9
'D3RVVHHGR([HUFtFLR
$UW $QRPHDomRID]VHHPFDUJR
$UW $SRVVHRFRUUHFRPDDVVLQDWXUDGRUHVSHFWLYRWHUPR
, GHSURYLPHQWRHIHWLYR GRTXDOGHYHPFRQVWDUDVDWULEXLo}HVRVGLUHLWRVHRV
,, HPFRPLVVmR GHYHUHVLQHUHQWHVDRFDUJRRFXSDGR
†ž $QRPHDomRSDUDFDUJRHIHWLYRGHYHREVHUYDUDRUGHP †ž $SRVVHGHYHRFRUUHUQRSUD]RGHWULQWDGLDVFRQWDGRV
GH FODVVLILFDomR H R SUD]R GH YDOLGDGH GR FRQFXUVR GDSXEOLFDomRGRDWRGHQRPHDomR
S~EOLFR
†ž 2SUD]RGHTXHWUDWDR†žSRGHVHUSURUURJDGRSDUD
†ž 2FDQGLGDWRDSURYDGRQRQ~PHURGHYDJDVSUHYLVWDVQR WHULQtFLRDSyVRWpUPLQRGDVOLFHQoDVRXGRVDIDVWDPHQ
HGLWDOGRFRQFXUVRWHPGLUHLWR j QRPHDomRQRFDUJR WRVVHJXLQWHV
SDUDRTXDOFRQFRUUHX
, OLFHQoDPpGLFDRXRGRQWROyJLFD
$UW 2 VHUYLGRU RFXSDQWHGHFDUJRHPFRPLVVmRSRGHVHU ,, OLFHQoDPDWHUQLGDGH
QRPHDGRSDUDWHUH[HUFtFLRLQWHULQDPHQWHHPRXWUR ,,, OLFHQoDSDWHUQLGDGH
FDUJRHPFRPLVVmRKLSyWHVHHPTXHGHYH ,9 OLFHQoDSDUDRVHUYLoRPLOLWDU

, DFXPXODUDVDWULEXLo}HVGHDPERVRVFDUJRV †ž $SRVVHSRGHRFRUUHUPHGLDQWHSURFXUDomRFRPSRGH


UHVHVSHFtILFRV
,, RSWDUSHODUHPXQHUDomRGHXPGHOHVGXUDQWHRSHUtRGR
GDLQWHULQLGDGH †ž 6yKiSRVVHQRVFDVRVGHSURYLPHQWRSRUQRPHDomR

$UW eYHGDGDDQRPHDomRSDUDFDUJRHPFRPLVVmRRXD †ž 'HYHVHUWRUQDGRVHPHIHLWRRDWRGHQRPHDomRVHD


GHVLJQDomRSDUDIXQomRGHFRQILDQoDGRF{QMXJHGH SRVVHQmRRFRUUHUQRSUD]RSUHYLVWRQHVWHDUWLJR
FRPSDQKHLURRXGHSDUHQWHSRUFRQVDQJXLQLGDGHDWp
RWHUFHLURJUDXRXSRUDILQLGDGH $UW 3RURFDVLmRGDSRVVHpH[LJLGRGRQRPHDGRDSUHVHQ
WDU
, GR*RYHUQDGRUHGR9LFH*RYHUQDGRUQDDGPLQLVWUDomR
S~EOLFD GLUHWD DXWiUTXLFD RX IXQGDFLRQDO GR 3RGHU , RVFRPSURYDQWHVGHVDWLVIDomRGRVUHTXLVLWRVSUHYLVWRV
([HFXWLYR QRDUWžHQDVQRUPDVHVSHFtILFDVSDUDDLQYHVWLGXUD
QRFDUJR
,, GH'HSXWDGR'LVWULWDOQD&kPDUD/HJLVODWLYD
,, GHFODUDomR
,,, GH&RQVHOKHLUR$XGLWRURX3URFXUDGRUGR0LQLVWpULR
3~EOLFRQR7ULEXQDOGH&RQWDV D GHEHQVHYDORUHVTXHFRQVWLWXHPVHXSDWULP{QLR

,9 9(7$'2  E VREUH DFXPXODomR RX QmR GH FDUJR RX HPSUHJR
S~EOLFREHPFRPRGHSURYHQWRVGDDSRVHQWDGRULD
GHUHJLPHSUySULRGHSUHYLGrQFLDVRFLDO
†ž $VYHGDo}HVGHVWHDUWLJRDSOLFDPVH
F VREUHDH[LVWrQFLDRXQmRGHLPSHGLPHQWRSDUDR
, DRVFDVRVGHUHFLSURFLGDGHGHQRPHDomRRXGHVLJQDomR
H[HUFtFLRGHFDUJRS~EOLFR
,, jVUHODo}HVKRPRDIHWLYDV
†ž eQXORRDWRGHSRVVHUHDOL]DGRVHPDDSUHVHQWDomRGRV
†ž 1mRVHLQFOXLQDVYHGDo}HVGHVWHDUWLJRDQRPHDomRRX GRFXPHQWRVDTXHVHUHIHUHHVWHDUWLJR
DGHVLJQDomR
6HFUHWDULDGH(VWDGRGH$ GPLQLVWUDomR3~EOLFD'LVWULWR)HGHUDO
 /HJLVODomR 

†ž $ DSWLGmR ItVLFD H PHQWDO p YHULILFDGDHPLQVSHomR $UW 1DKLSyWHVHGHDFXPXODomROtFLWDGHFDUJRVRHVWiJLR


PpGLFDRILFLDO SUREDWyULRpFXPSULGRHPUHODomRDFDGDFDUJRHPFXMR
H[HUFtFLRHVWHMDRVHUYLGRUYHGDGRRDSURYHLWDPHQWRGH
†ž $GHFODUDomRSUHYLVWDQRLQFLVR,,DGHYHVHUIHLWDHP SUD]RRXSRQWXDomR
IRUPXOiULRIRUQHFLGRSHORVHWRUGHSHVVRDOGDUHSDUWL
omR H GHOH GHYH FRQVWDU FDPSR SDUD LQIRUPDU EHQV $UW 2 VHUYLGRU SRGH GHVLVWLU GR HVWiJLR SUREDWyULR HVHU
YDORUHV GtYLGDV H {QXV UHDLV H[LJLGRV QD GHFODUDomR UHFRQGX]LGRDRFDUJRGHSURYLPHQWRHIHWLYRDQWHULRU
DQXDO GR LPSRVWR GH UHQGD GD SHVVRD ItVLFD FRP DV PHQWHRFXSDGRQRTXDOMiSRVVXtDHVWDELOLGDGHREVHU
VHJXLQWHVHVSHFLILFDo}HV YDGRRGLVSRVWRQRDUW

, DGHVFULomRGREHPFRPVXDORFDOL]DomRHVSHFLILFDo}HV †~QLFR 1mRSRGHGHVLVWLUGRHVWiJLRSUREDWyULRRVHUYLGRUTXH


JHUDLVGDWDHYDORUGDDTXLVLomRQRPHGRYHQGHGRUH UHVSRQGHDSURFHVVRGLVFLSOLQDU
YDORUGDVEHQIHLWRULDVVHKRXYHU
$UW eYHGDGRjDGPLQLVWUDomRS~EOLFDFRQFHGHUOLFHQoDQmR
,, DVGtYLGDVHR{QXVUHDOVREUHRVEHQVFRPVXDVHVSHFL UHPXQHUDGDRXDXWRUL]DUDIDVWDPHQWRVHPUHPXQHUD
ILFDo}HVJHUDLVYDORUHSUD]RSDUDTXLWDomREHPFRPR omRDRVHUYLGRUHPHVWiJLRSUREDWyULR
RQRPHGRFUHGRU
†ž ([FHWXDVHGRGLVSRVWRQHVWHDUWLJRRDIDVWDPHQWRSDUD
,,, DIRQWHGHUHQGDGRV~OWLPRVGR]HPHVHVFRPDHVSHFL R VHUYLoR PLOLWDU RX SDUD R H[HUFtFLR GH PDQGDWR
ILFDomRGRYDORUDXIHULGRQRSHUtRGR HOHWLYR

$UW ([HUFtFLR p R HIHWLYR GHVHPSHQKR GDV DWULEXLo}HVGR †ž $YHGDomRGHTXHWUDWDHVWHDUWLJRDSOLFDVHDRJR]RGH


FDUJRS~EOLFR OLFHQoDSUrPLRSRUDVVLGXLGDGH

†ž 2VHUYLGRUQmRSRGHHQWUDUHPH[HUFtFLR $UW 2VHUYLGRUHPHVWiJLRSUREDWyULRSRGH

, VH RFXSDU FDUJR LQDFXPXOiYHO VHP FRPSURYDUD , H[HUFHU TXDOTXHU FDUJR HP FRPLVVmR RX IXQomRGH
H[RQHUDomRRXDYDFkQFLDGHTXHWUDWDRDUW FRQILDQoDQRyUJmRDXWDUTXLDRXIXQGDomRGHORWDomR

,, VHRFXSDUFDUJRDFXPXOiYHOVHPFRPSURYDUDFRPSDWL ,, VHUFHGLGRDRXWURyUJmRRXHQWLGDGHSDUDRFXSDUFDUJR


ELOLGDGHGHKRUiULRV GHQDWXUH]DHVSHFLDORXGHHTXLYDOHQWHQtYHOKLHUiUTXL
FR
,,, VHUHFHEHUSURYHQWRVGHDSRVHQWDGRULDLQDFXPXOiYHLV
FRPDUHPXQHUDomRRXVXEVtGLRGRFDUJRHIHWLYRVHP $UW )LFDVXVSHQVDDFRQWDJHPGRWHPSRGHHVWiJLRSURED
FRPSURYDUDRSomRSRUXPDGDVIRUPDVGHSDJDPHQWR WyULRTXDQGRRFRUUHU

†ž eGHFLQFRGLDV~WHLVRSUD]RSDUDRVHUYLGRUHQWUDUHP , RDIDVWDPHQWRGHTXHWUDWDPRVDUWV,,H


H[HUFtFLRFRQWDGRGDSRVVH
,, OLFHQoDUHPXQHUDGDSRUPRWLYRGHGRHQoDHPSHVVRD
†ž &RPSHWHDRWLWXODUGDXQLGDGHDGPLQLVWUDWLYDRQGHIRU GDIDPtOLDGRVHUYLGRU
ORWDGRRVHUYLGRUGDUOKHH[HUFtFLR
$UW 'XUDQWHRHVWiJLRSUREDWyULRVmRDYDOLDGDVDDSWLGmR
†ž &RPRH[HUFtFLRLQLFLDVHDFRQWDJHPGRWHPSRHIHWLYR DFDSDFLGDGHHDHILFLrQFLDGRVHUYLGRUSDUDRGHVHPSH
GHVHUYLoR QKRGRFDUJRFRPDREVHUYkQFLDGRVIDWRUHV

†ž 2VHUYLGRUTXHQmRHQWUDUHPH[HUFtFLRQRSUD]RGR† , DVVLGXLGDGH


žGHYHVHUH[RQHUDGR ,, SRQWXDOLGDGH
,,, GLVFLSOLQD
$UW $RHQWUDUHPH[HUFtFLRRVHUYLGRUWHPGHDSUHVHQWDUDR ,9 FDSDFLGDGHGHLQLFLDWLYD
yUJmR FRPSHWHQWH RV GRFXPHQWRV QHFHVViULRV DRV 9 SURGXWLYLGDGH
DVVHQWDPHQWRVLQGLYLGXDLV 9, UHVSRQVDELOLGDGH

†~QLFR 2 LQtFLR D VXVSHQVmR D LQWHUUXSomR H RUHLQtFLRGR †ž 2 3RGHU ([HFXWLYR H RV yUJmRV GR3RGHU/HJLVODWLYR
H[HUFtFLRVmRUHJLVWUDGRVQRVDVVHQWDPHQWRVLQGLYLGXDLV GHYHPUHJXODPHQWDUHPVHXVUHVSHFWLYRVkPELWRVGH
GRVHUYLGRU DWXDomR RV SURFHGLPHQWRV GH DYDOLDomR GR HVWiJLR
SUREDWyULRREVHUYDGRQRPtQLPRRVHJXLQWH
$UW 2 H[HUFtFLR GH IXQomR GH FRQILDQoD LQLFLDVH FRPD
SXEOLFDomR GR DWR GH GHVLJQDomR VDOYR TXDQGR R , DWpRWULJpVLPRPrVGRHVWiJLRSUREDWyULRDDYDOLDomR
VHUYLGRUHVWLYHUHPOLFHQoDRXDIDVWDGRSRUTXDOTXHU p IHLWD VHPHVWUDOPHQWH FRP SRQWXDomR SRU QRWDV
PRWLYROHJDOKLSyWHVHHPTXHRH[HUFtFLRVHLQLFLDQR QXPpULFDVGH]HURDGH]
SULPHLURGLD~WLODSyVRWpUPLQRGRLPSHGLPHQWRTXH
QmRSRGHH[FHGHUDWULQWDGLDVGDSXEOLFDomR ,, DV DYDOLDo}HV GH TXH WUDWD R LQFLVR , VmR IHLWDVSHOD
FKHILD LPHGLDWD GR VHUYLGRU HP ILFKD SUHYLDPHQWH
6HomR9 SUHSDUDGDHGDTXDOFRQVWHSHORPHQRVRVHJXLQWH
'R(VWiJLR3UREDWyULR
D DVSULQFLSDLVDWULEXLo}HVWDUHIDVHURWLQDVDVHUHP
$UW $RHQWUDUHPH[HUFtFLRRVHUYLGRUQRPHDGRSDUDFDUJR GHVHPSHQKDGDV SHOR VHUYLGRU QR VHPHVWUH GH
GHSURYLPHQWRHIHWLYRILFDVXMHLWRDRHVWiJLRSUREDWyULR DYDOLDomR
SHORSUD]RGHWUrVDQRV
E RVHOHPHQWRVHRVIDWRUHVSUHYLVWRVQHVWHDUWLJR
  /HJLVODomR 6HFUHWDULDGH(VWDGRGH$GPLQLVWUDomR3~EOLFD'LVWULWR)HGHUDO


F RFLHQWHGRVHUYLGRUDYDOLDGR 6HomR9,
'D(VWDELOLGDGH
†ž (PWRGDVDVDYDOLDo}HVpDVVHJXUDGRDRDYDOLDGR
$UW 2 VHUYLGRU RFXSDQWH GH FDUJR GH SURYLPHQWRHIHWLYR
, RDPSORDFHVVRDRVFULWpULRVGHDYDOLDomR UHJXODUPHQWHDSURYDGRQRHVWiJLRSUREDWyULRDGTXLUH
HVWDELOLGDGHQRVHUYLoRS~EOLFRDRFRPSOHWDUWUrVDQRV
,, RFRQKHFLPHQWRGRVPRWLYRVGDVQRWDVTXHOKHIRUDP GHHIHWLYRH[HUFtFLR
DWULEXtGDV
$UW 2 VHUYLGRU HVWiYHO Vy SHUGH R FDUJR QDVKLSyWHVHV
,,, RFRQWUDGLWyULRHDDPSODGHIHVDQRVWHUPRVGHVWD/HL SUHYLVWDVQD&RQVWLWXLomR)HGHUDO
&RPSOHPHQWDU
6HomR9,,
†ž $VDYDOLDo}HVGHYHPVHUPRQLWRUDGDVSHODFRPLVVmRGH 'D5HYHUVmR
TXHWUDWDRDUW
$UW 5HYHUVmRpRUHWRUQRjDWLYLGDGHGHVHUYLGRUDSRVHQWD
$UW $DYDOLDomRHVSHFLDOSUHYLVWDQD&RQVWLWXLomR)HGHUDO GR
FRPRFRQGLomRSDUDDTXLVLomRGDHVWDELOLGDGHGHYHVHU
IHLWDSRUFRPLVVmRTXDWURPHVHVDQWHVGHWHUPLQDUR , SRULQYDOLGH]TXDQGRSRUMXQWDPpGLFDRILFLDOILFDU
HVWiJLRSUREDWyULR FRPSURYDGDDVXDUHDELOLWDomR

†ž $FRPLVVmRGHTXHWUDWDHVWHDUWLJRpFRPSRVWDSRUWUrV ,, TXDQGRFRQVWDWDGDDGPLQLVWUDWLYDRXMXGLFLDOPHQWHD


VHUYLGRUHV HVWiYHLV GR PHVPR FDUJR RX GH FDUJR GH LQVXEVLVWrQFLD GRV IXQGDPHQWRV GH FRQFHVVmR GD
HVFRODULGDGHVXSHULRUGDPHVPDFDUUHLUDGRDYDOLDGR DSRVHQWDGRULD

†ž 1mRVHQGRSRVVtYHODDSOLFDomRGRGLVSRVWRQR†žD ,,, YROXQWDULDPHQWHGHVGHTXHFXPXODWLYDPHQWH


FRPSRVLomRGDFRPLVVmRGHYHVHUGHILQLGDFRQIRUPHR
FDVR D KDMDPDQLIHVWRLQWHUHVVHGDDGPLQLVWUDomRH[SUHVVR
HPHGLWDOTXHIL[HRVFULWpULRVGHUHYHUVmRYROXQWi
, SHOR3UHVLGHQWHGD&kPDUD/HJLVODWLYD ULDDRVLQWHUHVVDGRVTXHHVWHMDPHPLJXDOVLWXDomR

,, SHOR3UHVLGHQWHGR7ULEXQDOGH&RQWDV E WHQKDPGHFRUULGRPHQRVGHFLQFRDQRVGDGDWDGH


DSRVHQWDGRULD
,,, SHOR 6HFUHWiULR GH (VWDGR D TXH R DYDOLDGRHVWHMD
VXERUGLQDGR LQFOXtGRV RV VHUYLGRUHV GH DXWDUTXLD F KDMDFDUJRYDJR
IXQGDomRHGHPDLVyUJmRVYLQFXODGRV
†ž eGHTXLQ]HGLDV~WHLVRSUD]RSDUDRVHUYLGRUUHWRUQDU
†ž 3DUD SURFHGHU j DYDOLDomR HVSHFLDO DFRPLVVmRGHYH DRH[HUFtFLRGRFDUJRFRQWDGRVGDGDWDHPTXHWRPRX
REVHUYDURVVHJXLQWHVSURFHGLPHQWRV FLrQFLDGDUHYHUVmR

, DGRWDUFRPRVXEVtGLRVSDUDVXDGHFLVmRDVDYDOLDo}HV †ž 1mRSRGHUHYHUWHURDSRVHQWDGRTXHWHQKDFRPSOHWDGR


IHLWDVQDIRUPDGRDUWLQFOXtGRVHYHQWXDLVSHGLGRV VHWHQWDDQRV
GH UHFRQVLGHUDomR UHFXUVRV H GHFLV}HV VREUH HOHV
SURIHULGDV $UW $UHYHUVmRGHYHVHUIHLWDQRPHVPRFDUJRRXQRFDUJR
UHVXOWDQWHGHVXDWUDQVIRUPDomR
,, RXYLU VHSDUDGDPHQWH R DYDOLDGRU H HP VHJXLGDR
DYDOLDGR †~QLFR 1DVKLSyWHVHVGRDUW,H,,HQFRQWUDQGRVHSURYLGR
RFDUJRRVHUYLGRUGHYHH[HUFHUVXDVDWULEXLo}HVFRPR
,,, UHDOL]DU D SHGLGR RX GH RItFLR DV GLOLJrQFLDVTXH H[FHGHQWHDWpDRFRUUrQFLDGHYDJD
HYHQWXDOPHQWHHPHUJLUHPGDV RLWLYDVGHTXHWUDWDR
LQFLVR,, 6HomR9,,,
'D5HLQWHJUDomR
,9 DSURYDURXUHSURYDURVHUYLGRUQRHVWiJLRSUREDWyULR
SRUGHFLVmRIXQGDPHQWDGD $UW $UHLQWHJUDomRpDUHLQYHVWLGXUDGRVHUYLGRUQRFDUJR
DQWHULRUPHQWHRFXSDGRRXQRFDUJRUHVXOWDQWHGHVXD
†ž &RQWUDDUHSURYDomRQRHVWiJLRSUREDWyULRFDEHSHGLGR WUDQVIRUPDomRTXDQGRLQYDOLGDGDDVXDGHPLVVmRSRU
GHUHFRQVLGHUDomRRXUHFXUVRDVHUHPSURFHVVDGRVQD GHFLVmRDGPLQLVWUDWLYDRXMXGLFLDOFRPRUHVWDEHOHFL
IRUPDGHVWD/HL&RPSOHPHQWDU PHQWRGRVGLUHLWRVTXHGHL[RXGHDXIHULUQRSHUtRGR
HPTXHHVWHYHGHPLWLGR
$UW $VDXWRULGDGHVGHTXHWUDWDRDUW†žVmRFRPSH
WHQWHVSDUD †ž 1DKLSyWHVHGHRFDUJRWHUVLGRH[WLQWRRVHUYLGRUILFD
HPGLVSRQLELOLGDGHREVHUYDGRRGLVSRVWRQRVDUWV
, MXOJDUHP~QLFDH~OWLPDLQVWkQFLDTXDOTXHUUHFXUVR H
LQWHUSRVWRQDIRUPDGRDUW
†ž (QFRQWUDQGRVHSURYLGRRFDUJRRVHXHYHQWXDORFX
,, KRPRORJDURUHVXOWDGRGDDYDOLDomRHVSHFLDOIHLWDSHOD SDQWHGHYHVHUUHFRQGX]LGRDRFDUJRGHRULJHPVHP
FRPLVVmRHFRPRFRQVHTXrQFLDHIHWLYDURVHUYLGRUQR GLUHLWRDLQGHQL]DomRRXDSURYHLWDGRHPRXWURFDUJR
FDUJRTXDQGRHOHIRUDSURYDGRQRHVWiJLRSUREDWyULR RXDLQGDSRVWRHPGLVSRQLELOLGDGH

$UW 2VHUYLGRUUHSURYDGRQRHVWiJLRSUREDWyULRGHYHVHU †ž eGHFLQFRGLDV~WHLVRSUD]RSDUDRVHUYLGRUUHWRUQDU


FRQIRUPHRFDVRH[RQHUDGRRXUHFRQGX]LGRDRFDUJR DRH[HUFtFLRGRFDUJRFRQWDGRVGDGDWDHPTXHWRPRX
GHRULJHP FLrQFLDGRDWRGHUHLQWHJUDomR
6HFUHWDULDGH(VWDGRGH$ GPLQLVWUDomR3~EOLFD'LVWULWR)HGHUDO
 /HJLVODomR 

6HomR,; †ž $ UHPRomR GH RItFLR GHVWLQDVHH[FOXVLYDPHQWHD


'D5HFRQGXomR DWHQGHUDQHFHVVLGDGHGHVHUYLoRVTXHQmRFRPSRUWHR
FRQFXUVRGHUHPRomR
$UW $UHFRQGXomRpRUHWRUQRGRVHUYLGRUHVWiYHODRFDUJR
DQWHULRUPHQWHRFXSDGRREVHUYDGRRGLVSRVWRQRDUW $UW e OtFLWDDSHUPXWDHQWUHVHUYLGRUHVGRPHVPRFDUJR
†žHGHFRUUHGH PHGLDQWHDXWRUL]DomRSUpYLDGDVUHVSHFWLYDVFKHILDV

, UHSURYDomRHPHVWiJLRSUREDWyULR 6HomR,,


,, GHVLVWrQFLDGHHVWiJLRSUREDWyULR 'D5HGLVWULEXLomR
,,, UHLQWHJUDomRGRDQWHULRURFXSDQWH
$UW 5HGLVWULEXLomRpRGHVORFDPHQWRGRFDUJRRFXSDGRRX
†ž (QFRQWUDQGRVHSURYLGRRFDUJRGHRULJHPRVHUYLGRU YDJR SDUD RXWUR yUJmR DXWDUTXLD RX IXQGDomR GR
WHPGHVHUDSURYHLWDGRHPRXWURFDUJRREVHUYDGRR PHVPR3RGHU
GLVSRVWRQRDUW
†ž $UHGLVWULEXLomRGiVH
†ž 2VHUYLGRUWHPGHUHWRUQDUDRH[HUFtFLRGRFDUJRDWpR
GLDVHJXLQWHDRGDFLrQFLDGRDWRGHUHFRQGXomR , SDUDFDUJRGHXPDPHVPDFDUUHLUDQRFDVRGHUHRUJD
QL]DomR RX DMXVWDPHQWR GH TXDGUR GH SHVVRDO jV
6HomR; QHFHVVLGDGHVGRVHUYLoR
'D'LVSRQLELOLGDGHHGR$SURYHLWDPHQWR
,, QRFDVRGHH[WLQomRRXFULDomRGHyUJmRDXWDUTXLDRX
IXQGDomR
$UW 2VHUYLGRU VySRGHVHUSRVWRHPGLVSRQLELOLGDGHQRV
FDVRVSUHYLVWRVQD&RQVWLWXLomR)HGHUDO
†ž 1DV KLSyWHVHV GR † ž ,, GHYHP VHUREVHUYDGRVR
†~QLFR $UHPXQHUDomRGRVHUYLGRUSRVWRHPGLVSRQLELOLGDGH LQWHUHVVHGDDGPLQLVWUDomRS~EOLFDDYLQFXODomRHQWUH
SURSRUFLRQDODRWHPSRGHVHUYLoRQmRSRGHVHULQIHULRU RVJUDXVGHFRPSOH[LGDGHHUHVSRQVDELOLGDGHGRFDUJR
D XP WHUoR GR TXH SHUFHELD QR PrV DQWHULRU DR GD D FRUUHODomR GDV DWULEXLo}HV D HTXLYDOrQFLD HQWUH RV
GLVSRQLELOLGDGH YHQFLPHQWRVRXVXEVtGLRHDSUpYLDDSUHFLDomRGRyUJmR
FHQWUDOGHSHVVRDO
$UW 2UHWRUQRjDWLYLGDGHGHVHUYLGRUHPGLVSRQLELOLGDGHp
&DStWXOR,,,
IHLWRPHGLDQWHDSURYHLWDPHQWR
'$68%67,78,d®2
, QRPHVPRFDUJR
$UW 2RFXSDQWHGHFDUJRRXIXQomRGHGLUHomRRXFKHILD
WHPVXEVWLWXWRLQGLFDGRQRUHJLPHQWRLQWHUQRRXQR
,, HPFDUJRUHVXOWDQWHGDWUDQVIRUPDomRGRFDUJRDQWHUL FDVRGHRPLVVmRSUHYLDPHQWHGHVLJQDGRSHODDXWRULGD
RUPHQWHRFXSDGR GHFRPSHWHQWH
,,, HP RXWUR FDUJR REVHUYDGD D FRPSDWLELOLGDGHGH †ž 2VXEVWLWXWRGHYHDVVXPLUDXWRPDWLFDPHQWHRH[HUFtFLR
DWULEXLo}HVHYHQFLPHQWRVRXVXEVtGLRGRFDUJRDQWHUL GRFDUJRRXIXQomRGHGLUHomRRXFKHILD
RUPHQWHRFXSDGR
, HPOLFHQoDVDIDVWDPHQWRVIpULDVHGHPDLVDXVrQFLDVRX
$UW eREULJDWyULR R LPHGLDWRDSURYHLWDPHQWRGHVHUYLGRU LPSHGLPHQWRVOHJDLVRXUHJXODPHQWDUHVGRWLWXODU
HPGLVSRQLELOLGDGHDVVLPTXHKRXYHUYDJDHPyUJmR
DXWDUTXLDRXIXQGDomR ,, HPFDVRGHYDFkQFLDGRFDUJR

†ž e GH WULQWD GLDV R SUD]R SDUD R VHUYLGRUUHWRUQDUDR †ž 2VXEVWLWXWRID]MXVDRVYHQFLPHQWRVRXVXEVtGLRSHOR


H[HUFtFLRFRQWDGRVGDGDWDHPTXHWRPRXFLrQFLDGR H[HUFtFLR GR FDUJR GH GLUHomR RX FKHILD SDJRV QD
DSURYHLWDPHQWR SURSRUomRGRVGLDVGHHIHWLYDVXEVWLWXLomR

†ž 'HYH VHU WRUQDGR VHP HIHLWR R DSURYHLWDPHQWRHVHU $UW 2GLVSRVWRQRDUWDSOLFDVHDRVWLWXODUHVGHXQLGD


FDVVDGDDGLVSRQLELOLGDGHVHRVHUYLGRUQmRUHWRUQDUDR GHVDGPLQLVWUDWLYDVRUJDQL]DGDVHPQtYHOGHDVVHVVRULD
H[HUFtFLRQRSUD]RGR†žVDOYRVHSRUGRHQoDFRPSUR
YDGDSRUMXQWDPpGLFDRILFLDO &DStWXOR,9
'$$&808/$d®2
&DStWXOR,,
'265(0$1(-$0(1726 $UW eSURLELGDDDFXPXODomRUHPXQHUDGDGHFDUJRVS~EOL
FRVH[FHWRTXDQGRKRXYHUFRPSDWLELOLGDGHGHKRUiUL
6HomR, RVSDUD
'D5HPRomR
, GRLVFDUJRVGHSURIHVVRU
$UW 5HPRomRpRGHVORFDPHQWRGDORWDomRGRVHUYLGRUQR ,, XPFDUJRGHSURIHVVRUFRPRXWURWpFQLFRRXFLHQWtILFR
PHVPR yUJmR DXWDUTXLD RX IXQGDomR H QD PHVPD ,,, GRLVFDUJRVRXHPSUHJRVSULYDWLYRVGHSURILVVLRQDLVGH
FDUUHLUDGHXPDORFDOLGDGHSDUDRXWUD VD~GHFRPSURILVV}HVUHJXODPHQWDGDV

†ž $UHPRomRpIHLWDDSHGLGRGHVHUYLGRUTXHSUHHQFKDDV †ž 3UHVXPHVHFRPRFDUJRGHQDWXUH]DWpFQLFDRXFLHQWtIL


FRQGLo}HV IL[DGDV QR HGLWDO GR FRQFXUVR DEHUWR SDUD FD SDUD RV ILQV GR LQFLVR ,, TXDOTXHU FDUJR S~EOLFR
HVVDILQDOLGDGH SDUD R TXDO VH H[LMD HGXFDomR VXSHULRU RX HGXFDomR
SURILVVLRQDO PLQLVWUDGD QD IRUPD H QDV FRQGLo}HV
†ž 2VLQGLFDWRUHVSHFWLYRWHPGHVHURXYLGRHPWRGDVDV SUHYLVWDV QD /HL GH 'LUHWUL]HV H %DVHV GD (GXFDomR
HWDSDVGRFRQFXUVRGHUHPRomR 1DFLRQDO
  /HJLVODomR 6HFUHWDULDGH(VWDGRGH$GPLQLVWUDomR3~EOLFD'LVWULWR)HGHUDO


†ž $SURLELomRGHDFXPXODUHVWHQGHVH †ž eYHGDGDDUHPXQHUDomRSHODSDUWLFLSDomRHPPDLVGH


XPFRQVHOKR
, DHPSUHJRVHIXQo}HVHDEUDQJHDXWDUTXLDVIXQGDo}HV
HPSUHVDVS~EOLFDVVRFLHGDGHVGHHFRQRPLDPLVWDVXDV †ž eSHUPLWLGDREVHUYDGRRGLVSRVWRQR†žDSDUWLFLSD
VXEVLGLiULDVHVRFLHGDGHVFRQWURODGDVGLUHWDRXLQGLUH omRUHPXQHUDGDGHVHUYLGRUHPFRQVHOKRGHDGPLQLV
WDPHQWHSHORSRGHUS~EOLFR WUDomRRXFRQVHOKRILVFDOGHHPSUHVDS~EOLFDRXVRFLH
GDGH GH HFRQRPLD PLVWD HP TXH R 'LVWULWR )HGHUDO
,, DRV SURYHQWRV GH DSRVHQWDGRULD SDJRV SRUUHJLPH GHWHQKD GLUHWD RX LQGLUHWDPHQWH SDUWLFLSDomR QR
SUySULR GH SUHYLGrQFLD VRFLDO GR 'LVWULWR )HGHUDO GD FDSLWDOVRFLDO
8QLmRGH(VWDGRRX0XQLFtSLRUHVVDOYDGRVRVSURYHQ &DStWXOR9
WRV GHFRUUHQWHV GH FDUJR DFXPXOiYHO QD IRUPD GHVWH '$9$&Ç1&,$
DUWLJR
$UW $YDFkQFLDGRFDUJRS~EOLFRGHFRUUHGH
†ž 2VHUYLGRUTXHDFXPXODUOLFLWDPHQWHFDUJRS~EOLFRILFD
REULJDGRDFRPSURYDUDQXDOPHQWHDFRPSDWLELOLGDGH , H[RQHUDomR
GHKRUiULRV ,, GHPLVVmR
,,, GHVWLWXLomRGHFDUJRHPFRPLVVmR
$UW 5HVVDOYDGRVRVFDVRVGHLQWHULQLGDGHHVXEVWLWXLomRR ,9 DSRVHQWDGRULD
VHUYLGRUQmRSRGH 9 IDOHFLPHQWR
9, SHUGDGRFDUJRQRVGHPDLVFDVRVSUHYLVWRVQD&RQVWLWX
LomR)HGHUDO
, H[HUFHUPDLVGHXPFDUJRHPFRPLVVmRRXIXQomRGH
FRQILDQoD
$UW $H[RQHUDomRGHFDUJRGHSURYLPHQWRHIHWLYRGiVHD
SHGLGRGRVHUYLGRURXGHRItFLR
,, DFXPXODUFDUJRHPFRPLVVmRFRPIXQomRGHFRQILDQoD
†~QLFR $H[RQHUDomRGHRItFLRGiVHH[FOXVLYDPHQWHTXDQGR
$UW 9HULILFDGDDTXDOTXHUWHPSRDDFXPXODomRLOHJDOGH RVHUYLGRU
FDUJRV HPSUHJRV IXQo}HV S~EOLFDV RX SURYHQWRV GH
DSRVHQWDGRULD R VHUYLGRU GHYH VHU QRWLILFDGR SDUD , IRUUHSURYDGRQRHVWiJLRSUREDWyULR
DSUHVHQWDURSomRQRSUD]RLPSURUURJiYHOGHGH]GLDV ,, WHQGRWRPDGRSRVVHQmRHQWUDUHPH[HUFtFLRQRSUD]R
FRQWDGRVGDGDWDGDFLrQFLDGDQRWLILFDomR HVWDEHOHFLGR

†ž (PGHFRUUrQFLDGDRSomRRVHUYLGRUGHYHVHUH[RQHUD $UW $H[RQHUDomRGHFDUJRHPFRPLVVmRGiVH


GR GR FDUJR HPSUHJR RX IXQomR SRU TXH QmR PDLV
WHQKDLQWHUHVVH , DFULWpULRGDDXWRULGDGHFRPSHWHQWH
,, DSHGLGRGRVHUYLGRU
†ž &RPDRSomRSHODUHQ~QFLDDRVSURYHQWRVGHDSRVHQWD
GRULDRVHXSDJDPHQWRFHVVDLPHGLDWDPHQWH $UW $VHUYLGRUDJHVWDQWHTXHRFXSHFDUJRHPFRPLVVmRVHP
YtQFXOR FRP R VHUYLoR S~EOLFR QmR SRGH VHP MXVWD
†ž 6HRVHUYLGRUQmRIL]HUDRSomRQRSUD]RGHVWHDUWLJR FDXVDVHUH[RQHUDGDGHRItFLRGHVGHDFRQILUPDomRGD
RVHWRUGHSHVVRDOGDUHSDUWLomRGHYHVROLFLWDUjDXWRUL JUDYLGH]DWpFLQFRPHVHVDSyVRSDUWRVDOYRPHGLDQWH
GDGHFRPSHWHQWHDLQVWDXUDomRGHSURFHVVRGLVFLSOLQDU LQGHQL]DomRSDJDQDIRUPDGRUHJXODPHQWR
SDUDDSXUDomRHUHJXODUL]DomRLPHGLDWD
†~QLFR 'HYH VHU WRUQDGR VHP HIHLWR R DWRGHH[RQHUDomR
†ž ,QVWDXUDGRRSURFHVVRGLVFLSOLQDUVHRVHUYLGRUDWpR TXDQGRFRQVWDWDGRTXHDVHUYLGRUDHVWDYDJHVWDQWHH
~OWLPRGLDGHSUD]RSDUDGHIHVDHVFULWDIL]HUDRSomR QmRIRLLQGHQL]DGD
GHTXHWUDWDHVWHDUWLJRRSURFHVVRGHYHVHUDUTXLYDGR
$UW $R WRPDU SRVVH HP RXWUR FDUJR LQDFXPXOiYHOGH
VHPMXOJDPHQWRGRPpULWR
TXDOTXHU yUJmR DXWDUTXLD RX IXQGDomR GR 'LVWULWR
)HGHUDO R VHUYLGRU HVWiYHO SRGH SHGLU D YDFkQFLD GR
†ž 2GLVSRVWRQR†žQmRVHDSOLFDVHKRXYHUGHFODUDomR
FDUJRHIHWLYRSRUHOHRFXSDGRREVHUYDQGRVHRVHJXLQ
IDOVDIHLWDSHORVHUYLGRUVREUHDFXPXODomRGHFDUJRV
WH
†ž &DUDFWHUL]DGD QR SURFHVVR GLVFLSOLQDUDDFXPXODomR , GXUDQWHRSUD]RGHTXHWUDWDRDUWRVHUYLGRUSRGH
LOHJDODDGPLQLVWUDomRS~EOLFDGHYHREVHUYDURVHJXLQ UHWRUQDU DR FDUJR DQWHULRUPHQWH RFXSDGR QRV FDVRV
WH SUHYLVWRVQRDUW
, UHFRQKHFLGD D ERDIp H[RQHUDU R VHUYLGRU GRFDUJR ,, RFDUJRSDUDRTXDOVHSHGLXYDFkQFLDSRGHVHUSURYLGR
YLQFXODGR DR yUJmR DXWDUTXLD RX IXQGDomR RQGH R SHODDGPLQLVWUDomRS~EOLFD
SURFHVVRIRLLQVWDXUDGR
7tWXOR,,,
,, SURYDGDDPiIpDSOLFDUDVDQomRGHGHPLVVmRGHVWLWX '$6&$55(,5$6('25(*,0(('$-251$'$'(75$%$/+2
LomRRXFDVVDomRGHDSRVHQWDGRULDRXGLVSRQLELOLGDGH
HP UHODomR DRV FDUJRV RX HPSUHJRV HP UHJLPH GH &DStWXOR,
DFXPXODomR LOHJDO KLSyWHVH HP TXH RV yUJmRV RX '$6&$55(,5$6
HQWLGDGHVGHYLQFXODomRGHYHPVHUFRPXQLFDGRV
6HomR,
$UW eYHGDGDDSDUWLFLSDomRGHVHUYLGRUVDOYRQDFRQGLomR 'DV'LVSRVLo}HV*HUDLV
GH6HFUHWiULRGH(VWDGRDLQGDTXHVXSOHQWHHPPDLV
GHXPFRQVHOKRFRPLVVmRFRPLWryUJmRGHGHOLEHUD $UW 2VFDUJRVGHSURYLPHQWRHIHWLYRVmRRUJDQL]DGRVHP
omRFROHWLYDRXDVVHPHOKDGRQDDGPLQLVWUDomRGLUHWD FDUUHLUDFULDGDSRUOHLTXHGHYHIL[DU
DXWiUTXLFDRXIXQGDFLRQDOGR'LVWULWR)HGHUDO
 6HFUHWDULDGH(VWDGRGH$ GPLQLVWUDomR3~EOLFD'LVWULWR)HGHUDO /HJLVODomR 

, D GHQRPLQDomR R TXDQWLWDWLYR H DV DWULEXLo}HVGRV †~QLFR 1RVFDVRVGHULVFRGHFRPSURPHWLPHQWRGDRUGHPHGD


FDUJRV VD~GHS~EOLFDVR*RYHUQDGRUSRGHDXWRUL]DUH[FHSFLR
QDOPHQWH D H[WUDSRODomR GRV OLPLWHV SUHYLVWRV QHVWH
,, RVUHTXLVLWRVSDUDLQYHVWLGXUDQRFDUJRHGHVHQYROYL DUWLJRSDUDRVVHUYLGRUHVTXHDWXHPGLUHWDPHQWHQDV
PHQWRQDFDUUHLUD iUHDVHQYROYLGDV

,,, DHVWUXWXUDGDFDUUHLUDFRPDIL[DomRGRVYHQFLPHQWRV $UW 3RGHVHUFRQFHGLGRKRUiULRHVSHFLDODRVHUYLGRU QRYD


UHGDomR GDGD DR DUW  SHOD OHL FRPSOHPHQWDU Qž  GH   GRGI GH
RXGRVXEVtGLR 

,9 RVFULWpULRVGHFDSDFLWDomR , FRPGHILFLrQFLDRXFRPGRHQoDIDOFLIRUPH

9 RUHJLPHHDMRUQDGDGHWUDEDOKR ,, TXHWHQKDF{QMXJHRXGHSHQGHQWHFRPGHILFLrQFLDRX


FRPGRHQoDIDOFLIRUPH
†~QLFR $VDOWHUDo}HVGHUHTXLVLWRVSDUDSURYLPHQWRGHFDUJR
S~EOLFRGHFDUUHLUDDSOLFDPVHH[FOXVLYDPHQWHjTXHOHV ,,, PDWULFXODGRHPFXUVRGDHGXFDomREiVLFDHGDHGXFD
VHUYLGRUHV FXMR LQJUHVVR VH GHU DSyV HODV WHUHP VLGR omRVXSHULRUTXDQGRFRPSURYDGDDLQFRPSDWLELOLGDGH
SXEOLFDGDV HQWUHRKRUiULRHVFRODUHRGDXQLGDGHDGPLQLVWUDWLYD
VHPSUHMXt]RGRH[HUFtFLRGRFDUJR
6HomR,,
'D3URPRomR ,9 QDKLSyWHVHGRDUW†ž
QRWDYLGHGHFUHWRQžGHGRGIGHTXHUHJXODPHQWDHVWHDUWLJR

$UW 6DOYR GLVSRVLomR OHJDO HP FRQWUiULR D SURPRomR pD †ž 1DV KLSyWHVHV GRV LQFLVRV , H ,, RKRUiULRHVSHFLDO
PRYLPHQWDomRGHVHUYLGRUGR~OWLPRSDGUmRGHXPD FRQVLVWHQDUHGXomRGHDWpGDMRUQDGDGHWUDEDOKR
FODVVHSDUDRSULPHLURSDGUmRGDFODVVHLPHGLDWDPHQWH HVXDQHFHVVLGDGHGHYHVHUDWHVWDGDSRUMXQWDPpGLFD
VXSHULRU RILFLDO

†ž $SURPRomRGiVHSRUPHUHFLPHQWRRXSRUDQWLJXLGD †ž 1RVFDVRVGRVLQFLVRV,,,H,9pH[LJLGDGRVHUYLGRUD


GHQDIRUPDGRSODQRGHFDUUHLUDGH FDGDFDWHJRULD FRPSHQVDomRGHKRUiULRQDXQLGDGHDGPLQLVWUDWLYDGH
IXQFLRQDO PRGR D FXPSULU LQWHJUDOPHQWH R UHJLPH VHPDQDO GH
WUDEDOKR
†ž $SURPRomRQmRLQWHUURPSHRWHPSRGHH[HUFtFLRQR
FDUJR †ž 2VHUYLGRUHVWXGDQWHGHYHFRPSURYDUPHQVDOPHQWHD
VXDIUHTXrQFLDHVFRODU

&DStWXOR,, $UW 6HPSUHMXt]RGDUHPXQHUDomRRXVXEVtGLRRVHUYLGRU


'25(*,0(('$-251$'$'(75$%$/+2 SRGH DXVHQWDUVH GR VHUYLoR PHGLDQWH FRPXQLFDomR
SUpYLDjFKHILDLPHGLDWD
$UW 6DOYRGLVSRVLomROHJDOHPFRQWUiULRRVHUYLGRUHIHWLYR
ILFD VXMHLWR DR UHJLPH GH WUDEDOKR GH WULQWD KRUDV , SRUXPGLDSDUD
VHPDQDLV
D GRDUVDQJXH
†ž 1R LQWHUHVVH GD DGPLQLVWUDomR S~EOLFDHPHGLDQWH E UHDOL]DUXPDYH]SRUDQRH[DPHVPpGLFRVSUHYHQ
DQXrQFLDGRVHUYLGRURUHJLPHGHWUDEDOKRSRGHVHU WLYRVRXSHULyGLFRVYROWDGRVDRFRQWUROHGHFkQFHU
DPSOLDGRSDUDTXDUHQWDKRUDVVHPDQDLVREVHUYDGDD GHSUyVWDWDGHPDPDRXGRFRORGH~WHUR
SURSRUFLRQDOLGDGHVDODULDO
,, SRU DWp GRLV GLDV SDUD VH DOLVWDU FRPR HOHLWRURX
†ž eYHGDGRDSOLFDUDRUHJLPHGHWUDEDOKRLQWHUSUHWDomR UHTXHUHUWUDQVIHUrQFLDGRGRPLFtOLRHOHLWRUDO
SRUDQDORJLDH[WHQVmRRXVHPHOKDQoDGHDWULEXLo}HV
,,, SRURLWRGLDVFRQVHFXWLYRVLQFOXtGRRGLDGDRFRUUrQFLD
†ž $MRUQDGDGHWUDEDOKRHPVLVWHPDGHHVFDODGHUHYH]D HPUD]mRGH
PHQWR GHYH VHU GHILQLGD HP OHL RX UHJXODPHQWR
REVHUYDQGRRUHJLVWURHPIROKDGHSRQWRGRKRUiULRGH D FDVDPHQWR
HQWUDGDHGHVDtGD E IDOHFLPHQWR GR F{QMXJH FRPSDQKHLUR SDUFHLUR
KRPRDIHWLYRSDLPmHSDGUDVWRPDGUDVWDILOKR
$UW 2 VHUYLGRU RFXSDQWH GH FDUJR HP FRPLVVmR RXQR LUPmRHQWHDGRRXPHQRUVREJXDUGDRXWXWHOD
H[HUFtFLRGHIXQomRGHFRQILDQoDWHPUHJLPHGHWUDED
OKRGHTXDUHQWDKRUDVVHPDQDLVFRPLQWHJUDOGHGLFD $UW (PFDVRGHIDOWDDRVHUYLoRDWUDVRDXVrQFLDRXVDtGD
omRDRVHUYLoR DQWHFLSDGD GHVGH TXH GHYLGDPHQWH MXVWLILFDGRV p
IDFXOWDGRjFKHILDLPHGLDWDDWHQGHQGRDUHTXHULPHQWR
$UW 1RVHUYLoRQRWXUQRDKRUDpFRQVLGHUDGDFRPRWHQGR GRLQWHUHVVDGRDXWRUL]DUDFRPSHQVDomRGHKRUiULRD
FLQTXHQWDHGRLVPLQXWRVHWULQWDVHJXQGRV VHU UHDOL]DGD DWp R ILQDO GR PrV VXEVHTXHQWH DR GD
RFRUUrQFLD
†~QLFR &RQVLGHUDVHQRWXUQRRVHUYLoRSUHVWDGRHQWUHDVYLQWH
HGXDVKRUDVGHXPGLDHDVFLQFRKRUDVGRGLDVHJXLQ †ž 2DWUDVRDDXVrQFLDMXVWLILFDGDRXDVDtGDDQWHFLSDGD
WH VmRFRPSXWDGRVSRUPLQXWRVDVHUHPFRQYHUWLGRVHP
KRUDGHQWURGHFDGDPrV
$UW 3DUDDWHQGHUDVLWXDo}HVH[FHSFLRQDLVHWHPSRUiULDVGR
VHUYLoR D MRUQDGD GH WUDEDOKR SRGH VHU DPSOLDGD D †ž $SXUDGRRWHPSRQDIRUPDGR†žVmRGHVSUH]DGRVRV
WtWXORGHVHUYLoRH[WUDRUGLQiULRHPDWpGXDVKRUDV UHVtGXRVLQIHULRUHVDVHVVHQWDPLQXWRV
  /HJLVODomR 6HFUHWDULDGH(VWDGRGH$GPLQLVWUDomR3~EOLFD'LVWULWR)HGHUDO


†ž 7RGDFRPSHQVDomRGHKRUiULRGHYHVHUUHJLVWUDGDSHOD ,9 RDERQRGHSHUPDQrQFLD


FKHILDLPHGLDWDMXQWRDRVHWRUGHSHVVRDOGDUHSDUWLomR
9 RDGLFLRQDOSRUVHUYLoRH[WUDRUGLQiULR
$UW $VIDOWDVLQMXVWLILFDGDVDRVHUYLoRFRQILJXUDP
9, RDGLFLRQDOQRWXUQR
, DEDQGRQR GR FDUJRVHRFRUUHUHPSRUPDLVGHWULQWD
GLDVFRQVHFXWLYRV 9,, DVYDQWDJHQVGHFDUiWHULQGHQL]DWyULR

,, LQDVVLGXLGDGH KDELWXDO VH RFRUUHUHP SRU PDLVGH 9,,, DUHPXQHUDomRRXVXEVtGLR


VHVVHQWDGLDVLQWHUSRODGDPHQWHQRSHUtRGRGHGR]H
PHVHV D SHORH[HUFtFLRGHFDUJRHPFRPLVVmRRXGHIXQomR
GHFRQILDQoDGHTXHWUDWDRDUW
$UW 6DOYRQDKLSyWHVHGHOLFHQoDRXDIDVWDPHQWRSUHYLVWD
QRDUW†žFRQVLGHUDVHIDOWDLQMXVWLILFDGDHVSHFL E GHFRUUHQWHGHVXEVWLWXLo}HV
DOPHQWHDTXHGHFRUUDGH
$UW $UHPXQHUDomRpFRQVWLWXtGDGHSDUFHODVHFRPSUHHQ
, QmR UHWRUQR DR H[HUFtFLR QR SUD]R IL[DGR QHVWD/HL GH
&RPSOHPHQWDU HP FDVR GH UHYHUVmR UHLQWHJUDomR
UHFRQGXomRRXDSURYHLWDPHQWR , RVYHQFLPHQWRVTXHVHFRPS}HP

,, QmR DSUHVHQWDomR LPHGLDWD SDUD H[HUFtFLR QRyUJmR D GRYHQFLPHQWREiVLFR


DXWDUTXLDRXIXQGDomRHPFDVRGHUHPRomRRXUHGLV E GDVYDQWDJHQVSHUPDQHQWHVUHODWLYDVDRFDUJR
WULEXLomR
,, DVYDQWDJHQVUHODWLYDVjVSHFXOLDULGDGHVGHWUDEDOKR
,,, LQWHUVWtFLRHQWUH
,,, DVYDQWDJHQVSHVVRDLV
D RDIDVWDPHQWRGRyUJmRDXWDUTXLDRXIXQGDomRGH
RULJHPHRH[HUFtFLRQRyUJmRRXHQWLGDGHSDUDR ,9 DVYDQWDJHQVGHQDWXUH]DSHULyGLFDRXHYHQWXDO
TXDORVHUYLGRUIRLFHGLGRRXFRORFDGRjGLVSRVLomR
9 DVYDQWDJHQVGHFDUiWHULQGHQL]DWyULR
E RWpUPLQRGDFHVVmRRXGDGLVSRVLomRGHTXHWUDWD
DDOtQHDDHRUHLQtFLRGRH[HUFtFLRQRyUJmRDXWDU $UW 2VYHQFLPHQWRVRXRVXEVtGLRVmRLUUHGXWtYHLV
TXLDRXIXQGDomRGHRULJHP
$UW $UHPXQHUDomRRXRVXEVtGLRGRVRFXSDQWHVGHFDUJRV
7tWXOR,9 HIXQo}HVS~EOLFRVGDDGPLQLVWUDomRGLUHWDDXWiUTXLFD
'26',5(,726 H IXQGDFLRQDO LQFOXtGRV RV FDUJRV SUHHQFKLGRV SRU
PDQGDWRHOHWLYRHRVSURYHQWRVDVSHQV}HVRXRXWUD
&DStWXOR, HVSpFLHUHPXQHUDWyULDSHUFHELGRVFXPXODWLYDPHQWHRX
'26,67(0$5(081(5$7Ð5,2 QmR LQFOXtGDV DV YDQWDJHQV SHVVRDLV RX GH TXDOTXHU
RXWUDQDWXUH]DQmRSRGHPH[FHGHURVXEVtGLRPHQVDO
6HomR, HP HVSpFLH GRV 'HVHPEDUJDGRUHV GR 7ULEXQDO GH
'RV&RQFHLWRV*HUDLV -XVWLoDGR'LVWULWR)HGHUDOH7HUULWyULRV

$UW $UHWULEXLomRSHFXQLiULDSHORH[HUFtFLRGHFDUJRS~EOLFR †ž 2YDORUGRWHWRGHUHPXQHUDomRRXVXEVtGLRGHYHVHU


pIL[DGDHPOHLVREDIRUPDGHVXEVtGLRRXUHPXQHUD SXEOLFDGR QR 'LiULR 2ILFLDO GR 'LVWULWR )HGHUDO SHOR
omRPHQVDO 3RGHU([HFXWLYRVHPSUHTXHVHDOWHUDURVXEVtGLRGRV
'HVHPEDUJDGRUHV GR 7ULEXQDO GH -XVWLoD GR 'LVWULWR
†ž 2YDORUGLiULRGDUHPXQHUDomRRXVXEVtGLRREWpPVH )HGHUDOH7HUULWyULRV
GLYLGLQGRVHRYDORUGDUHWULEXLomRSHFXQLiULDPHQVDO
SRUWULQWD †ž ([FOXHPVHGRYDORUGRWHWRGHUHPXQHUDomRRGpFLPR
WHUFHLURVDOiULRRDGLDQWDPHQWRGHIpULDVRDGLFLRQDO
†ž 2YDORUKRUiULRGDUHPXQHUDomRRXVXEVtGLRREWpPVH GHIpULDVRDX[tOLRQDWDOLGDGHRDX[tOLRSUpHVFRODUHDV
GLYLGLQGRVH D UHWULEXLomR SHFXQLiULD PHQVDO SHOR YDQWDJHQVGHFDUiWHULQGHQL]DWyULR
TXtQWXSORGDFDUJDKRUiULDVHPDQDO
6HomR,,
†ž 1DUHWULEXLomRSHFXQLiULDPHQVDOGHTXHWUDWDPRV†† 'R9HQFLPHQWR%iVLFRHGR6XEVtGLR
žHžQmRVHLQFOXHP
$UW 2YHQFLPHQWREiVLFRpIL[DGRSRUSDGUmRQDWDEHODGH
, DVYDQWDJHQVGHQDWXUH]DSHULyGLFDRXHYHQWXDODVGH UHPXQHUDomRGDFDUUHLUD
FDUiWHULQGHQL]DWyULRRDGLFLRQDOQRWXUQRHRDGLFLRQDO
SRUVHUYLoRH[WUDRUGLQiULR $UW 1DIL[DomRGRVXEVtGLRRXGRVSDGU}HVGRYHQFLPHQWR
EiVLFRHGDVGHPDLVSDUFHODVGRVLVWHPDUHPXQHUDWyULR
,, RVDFUpVFLPRVGHTXHWUDWDRDUW,D9,, GHYHPVHUREVHUYDGRV

$UW 2VXEVtGLRpFRQVWLWXtGRGHSDUFHOD~QLFDHDHOHSRGH , DQDWXUH]DRJUDXGHUHVSRQVDELOLGDGHHDFRPSOH[LGD


VHUDFUHVFLGRH[FOXVLYDPHQWH GHGRVFDUJRVFRPSRQHQWHVGHFDGDFDUUHLUD

, RGpFLPRWHUFHLURVDOiULR ,, RVUHTXLVLWRVSDUDLQYHVWLGXUD


,, RDGLFLRQDOGHIpULDV
,,, RDX[tOLRQDWDOLGDGH ,,, DVSHFXOLDULGDGHVGRVFDUJRV
6HFUHWDULDGH(VWDGRGH$ GPLQLVWUDomR3~EOLFD'LVWULWR)HGHUDO
 /HJLVODomR 

$UW 2VXEVtGLRRXRYHQFLPHQWREiVLFRLQLFLDOGDFDUUHLUD 6XEVHomR,,


QmRSRGHVHULQIHULRUDRVDOiULRPtQLPR 'RV$GLFLRQDLVGH,QVDOXEULGDGHHGH3HULFXORVLGDGH

†ž 2YDORUGRVXEVtGLRRXGRYHQFLPHQWREiVLFRGHYHVHU $UW 2VHUYLGRUTXHWUDEDOKDFRPKDELWXDOLGDGHHPORFDLV


FRPSOHPHQWDGR VHPSUH TXH ILFDU DEDL[R GR VDOi LQVDOXEUHVRXHPFRQWDWRSHUPDQHQWHFRPVXEVWkQFLDV
ULRPtQLPR Wy[LFDVUDGLRDWLYDVRXFRPULVFRGHYLGDID]MXVDXP
DGLFLRQDOGHLQVDOXEULGDGHRXGHSHULFXORVLGDGH
†ž 6REUHRYDORUGDFRPSOHPHQWDomRGHTXHWUDWDR†ž
GHYHPLQFLGLUDVSDUFHODVGDUHPXQHUDomRTXHLQFLGHP †ž 2VHUYLGRUTXHIL]HUMXVDRVDGLFLRQDLVGHLQVDOXEULGDGH
VREUHRYHQFLPHQWREiVLFR HGHSHULFXORVLGDGHWHPGHRSWDUSRUXPGHOHV

6HomR,,, †ž 2GLUHLWRDRDGLFLRQDOGHLQVDOXEULGDGHRXSHULFXORVLGD


'DV9DQWDJHQV GHFHVVDFRPDHOLPLQDomRGDVFRQGLo}HVRXGRVULVFRV
TXHGHUDPFDXVDDVXDFRQFHVVmR
$UW $OpP GR YHQFLPHQWR EiVLFR SRGHP VHU SDJDVDR
VHUYLGRU FRPR YDQWDJHQV DV VHJXLQWHV SDUFHODV $UW 'HYH KDYHU SHUPDQHQWH FRQWUROH GD DWLYLGDGHGH
UHPXQHUDWyULDV VHUYLGRUHVHPRSHUDo}HVRXORFDLVFRQVLGHUDGRVLQVDOX
EUHVRXSHULJRVRV
, JUDWLILFDo}HV
,, DGLFLRQDLV †~QLFR $ VHUYLGRUD JHVWDQWH RX ODFWDQWH HQTXDQWRGXUDUD
,,, DERQRV JHVWDomRHDODFWDomRGHYHH[HUFHUVXDVDWLYLGDGHVHP
,9 LQGHQL]Do}HV ORFDOVDOXEUHHHPVHUYLoRQmRSHULJRVR
†ž $V JUDWLILFDo}HV H RV DGLFLRQDLVLQFRUSRUDPVHDR $UW 1D FRQFHVVmR GRV DGLFLRQDLV GH LQVDOXEULGDGH RXGH
YHQFLPHQWR QRVFDVRVHQDVFRQGLo}HVLQGLFDGRVHP SHULFXORVLGDGH GHYHP VHU REVHUYDGDV DV VLWXDo}HV
OHL HVWDEHOHFLGDVHPOHJLVODomRHVSHFtILFD
†ž $VLQGHQL]Do}HVQmRVHLQFRUSRUDPDRYHQFLPHQWRRX $UW 2VORFDLVGHWUDEDOKRHRVVHUYLGRUHVTXHRSHUDPFRP
SURYHQWRSDUDTXDOTXHUHIHLWR
UDLRV;RXVXEVWkQFLDVUDGLRDWLYDVGHYHPVHUPDQWLGRV
VRE FRQWUROH SHUPDQHQWH GH PRGR TXH DV GRVHV GH
$UW $V YDQWDJHQV SHFXQLiULDV QmR VmR FRPSXWDGDVQHP
UDGLDomR LRQL]DQWH QmR XOWUDSDVVHP R QtYHO Pi[LPR
DFXPXODGDV SDUD HIHLWR GH FRQFHVVmR GH TXDOTXHU
SUHYLVWRQDOHJLVODomRSUySULD
RXWURDFUpVFLPRSHFXQLiULRXOWHULRU
†~QLFR 2V VHUYLGRUHV D TXH VH UHIHUH HVWH DUWLJRGHYHPVHU
6HomR,9 VXEPHWLGRVDH[DPHVPpGLFRVDFDGDVHLVPHVHV
'DV9DQWDJHQV3HUPDQHQWHV5HODWLYDVDR&DUJR
$UW 2 DGLFLRQDO GH LQVDOXEULGDGH RX GH SHULFXORVLGDGHp
$UW $VYDQWDJHQVSHUPDQHQWHVUHODWLYDVDRFDUJRFULDGDV
GHYLGRQRVWHUPRVGDVQRUPDVOHJDLVHUHJXODPHQWDUHV
SRUOHLFRPSUHHQGHPDVJUDWLILFDo}HVHRVDGLFLRQDLV
SHUWLQHQWHVDRVWUDEDOKDGRUHVHPJHUDOREVHUYDGRVRV
YLQFXODGRVDRVFDUJRVGHFDUUHLUDRXDRVHXH[HUFtFLR
SHUFHQWXDLVVHJXLQWHVLQFLGHQWHVVREUHRYHQFLPHQWR
EiVLFR
6HomR9
'DV9DQWDJHQV5HODWLYDVjV3HFXOLDULGDGHVGH7UDEDOKR
, FLQFRGH]RXYLQWHSRUFHQWRQRFDVRGHLQVDOXEULGDGH
6XEVHomR, QRVJUDXVPtQLPRPpGLRRXPi[LPRUHVSHFWLYDPHQWH
'D*UDWLILFDomRGH)XQomRGH&RQILDQoDHGRV9HQFLPHQWRV
GH&DUJRHP&RPLVVmR ,, GH]SRUFHQWRQRFDVRGHSHULFXORVLGDGH

$UW 6HP SUHMXt]R GD UHPXQHUDomR RX VXEVtGLR GRFDUJR †ž 2DGLFLRQDOGHLUUDGLDomRLRQL]DQWHGHYHVHUFRQFHGLGR
HIHWLYRRVHUYLGRUID]MXV QRVSHUFHQWXDLVGHFLQFRGH]RXYLQWHSRUFHQWRQD
IRUPDGRUHJXODPHQWR
, DRYDORULQWHJUDOGDIXQomRGHFRQILDQoDSDUDDTXDOIRL
GHVLJQDGR †ž $JUDWLILFDomRSRUWUDEDOKRVFRPUDLRV;RXVXEVWkQFLDV
UDGLRDWLYDVpFRQFHGLGDQRSHUFHQWXDOGHGH]SRUFHQWR
,, D RLWHQWD SRU FHQWR GRV YHQFLPHQWRV RX VXEVtGLRGR
FDUJRHPFRPLVVmRSRUHOHH[HUFLGRVDOYRGLVSRVLomR 6XEVHomR,,,
OHJDOHPFRQWUiULR 'R$GLFLRQDOSRU6HUYLoR([WUDRUGLQiULR

†ž $V IpULDV R DGLFLRQDO GH IpULDV H RGpFLPRWHUFHLUR $UW 2VHUYLoRH[WUDRUGLQiULRpUHPXQHUDGRFRPDFUpVFLPR


VDOiULR VmR SDJRV SURSRUFLRQDOPHQWH DRV PHVHV GH GHFLQTXHQWDSRUFHQWRHPUHODomRDRYDORUGDUHPXQH
HIHWLYR H[HUFtFLR GR VHUYLGRU HIHWLYR QR FDUJR HP UDomRRXVXEVtGLRGDKRUDQRUPDOGHWUDEDOKR
FRPLVVmRRXIXQomRGHFRQILDQoD
6XEVHomR,9
†ž 2 VHUYLGRU HIHWLYR SRGH RSWDU SHOR YDORULQWHJUDOGR 'R$GLFLRQDO1RWXUQR
FDUJRHPFRPLVVmRKLSyWHVHHPTXHQmRSRGHSHUFHEHU
RVXEVtGLRRXDUHPXQHUDomRGRFDUJRHIHWLYR $UW 2VHUYLoRQRWXUQRDTXHVHUHIHUHRDUWpUHPXQHUD
GRFRPDFUpVFLPRGHYLQWHHFLQFRSRUFHQWRVREUHR
$UW 2GLVSRVWRQRDUWDSOLFDVHDRVHUYLGRURXHPSUHJD YDORUGDUHPXQHUDomRRXVXEVtGLRGDKRUDWUDEDOKDGD
GR UHTXLVLWDGR GH TXDOTXHU yUJmR RX HQWLGDGH GRV
3RGHUHV GR 'LVWULWR )HGHUDO GD 8QLmR GH (VWDGR RX †~QLFR 2DGLFLRQDOQRWXUQRLQFLGHVREUHRDGLFLRQDOGHVHUYLoR
0XQLFtSLR H[WUDRUGLQiULR
  /HJLVODomR 6HFUHWDULDGH(VWDGRGH$GPLQLVWUDomR3~EOLFD'LVWULWR)HGHUDO


6HomR9, †ž $IUDomRVXSHULRUDTXDWRU]HGLDVpFRQVLGHUDGDFRPR


'DV9DQWDJHQV3HVVRDLV PrVLQWHJUDO

6XEVHomR, †ž 2GpFLPRWHUFHLURVDOiULRpGHYLGRVREUHDSDUFHODGD


'DV'LVSRVLo}HV*HUDLV UHWULEXLomR SHFXQLiULD SHUFHELGD SRU VHUYLGRU HIHWLYR
SHOR H[HUFtFLR GH IXQomR GH FRQILDQoD RX FDUJR HP
$UW &RQVLGHUDPVH SHVVRDLV DV SDUFHODV GDUHPXQHUDomR FRPLVVmRREVHUYDGDDSURSRUFLRQDOLGDGHGHTXHWUDWD
TXHGHSHQGDPGDVLWXDomRLQGLYLGXDOGHFDGDVHUYLGRU HVWHDUWLJRHRDUW†ž
SHUDQWHDDGPLQLVWUDomRS~EOLFD
$UW 2GpFLPRWHUFHLURVDOiULRpSDJR
$UW $V YDQWDJHQV SHVVRDLV XPD YH] DGTXLULGDVLQFRUSR
UDPVHjUHPXQHUDomR , QRPrVGHDQLYHUViULRGRVHUYLGRURFXSDQWHGHFDUJR
GH SURYLPHQWR HIHWLYR LQFOXtGR R UHTXLVLWDGR GD
6XEVHomR,, DGPLQLVWUDomR GLUHWD DXWiUTXLFD RX IXQGDFLRQDO GH
'R$GLFLRQDOSRU7HPSRGH6HUYLoR TXDOTXHU 3RGHU GR 'LVWULWR )HGHUDO GD 8QLmR GH
(VWDGRRX0XQLFtSLR
$UW 2DGLFLRQDOSRUWHPSRGHVHUYLoRpGHYLGRjUD]mRGH
XPSRUFHQWRVREUHRYHQFLPHQWREiVLFRGRFDUJRGH ,, DWpRGLDYLQWHGRPrVGHGH]HPEURGHFDGDDQRSDUD
SURYLPHQWRHIHWLYRSRUDQRGHHIHWLYRVHUYLoR RVVHUYLGRUHVQmRFRQWHPSODGRVQRLQFLVR,

†~QLFR 2DGLFLRQDOGHWHPSRGHVHUYLoRpGHYLGRDSDUWLUGR †ž 1R PrV GH GH]HPEUR R VHUYLGRU HIHWLYR ID]MXVD
PrVHPTXHRVHUYLGRUFRPSOHWDURDQXrQLR HYHQWXDLVGLIHUHQoDVHQWUHRYDORUSDJRFRPRGpFLPR
WHUFHLURVDOiULRHDUHPXQHUDomRGHYLGDQHVVHPrV
6XEVHomR,,,
'R$GLFLRQDOGH4XDOLILFDomR †ž 2 3RGHU ([HFXWLYR H RV yUJmRV GR3RGHU/HJLVODWLYR
SRGHPDOWHUDUDGDWDGHSDJDPHQWRGRGpFLPRWHUFHLUR
$UW 2DGLFLRQDOGHTXDOLILFDomRLQVWLWXtGRSRUOHLHVSHFtILFD VDOiULRGHVGHTXHHOHVHMDHIHWLYDGRDWpRGLDYLQWHGH
GHVWLQDVHDUHPXQHUDUDPHOKRULDQDFDSDFLWDomRSDUD GH]HPEURGHFDGDDQR
RH[HUFtFLRGRFDUJRHIHWLYR
$UW $R VHUYLGRU GHPLWLGR H[RQHUDGR RX TXH HQWUHHP
†~QLFR 2VFRQWH~GRVGRVFXUVRVGHTXDOLILFDomRGHYHPJXDUGDU OLFHQoDVHPUHPXQHUDomRpGHYLGRRGpFLPRWHUFHLUR
SHUWLQrQFLDFRPDVDWULEXLo}HVGRFDUJRHIHWLYRRXGD VDOiULR SURSRUFLRQDOPHQWH DRV PHVHV GH H[HUFtFLR
XQLGDGHGHORWDomRHH[HUFtFLR FDOFXODGRVREUHRVXEVtGLRRXDUHPXQHUDomRGRPrV
HPTXHRFRUUHURHYHQWR
6XEVHomR,9
'DV9DQWDJHQV3HVVRDLV1RPLQDOPHQWH,GHQWLILFiYHLV †~QLFR 6H R VHUYLGRU UHDVVXPLU R FDUJR RGpFLPRWHUFHLUR
VDOiULRGHYHVHUSDJRSURSRUFLRQDOPHQWHDRVPHVHVGH
$UW $VYDQWDJHQVSHVVRDLVQRPLQDOPHQWHLGHQWLILFiYHLVVmR H[HUFtFLRDSyVDUHDVVXQomR
GHILQLGDVHPOHLRXUHFRQKHFLGDVHPGHFLVmRMXGLFLDO
†~QLFR 9(7$'2 
$UW 2GpFLPRWHUFHLURVDOiULRQmRSRGH
6HomR9,,
'DV9DQWDJHQV3HULyGLFDV , VHUFRQVLGHUDGRSDUDFiOFXORGHTXDOTXHURXWUDYDQWD
JHP
6XEVHomR, ,, VHUVXSHULRUDRYDORUGRWHWRGHUHPXQHUDomRDTXHR
'R$GLFLRQDOGH)pULDV VHUYLGRUHVWiVXEPHWLGR

$UW ,QGHSHQGHQWHPHQWHGHVROLFLWDomRpSDJRDRVHUYLGRU 6HomR9,,,


SRURFDVLmRGDVIpULDVXPDGLFLRQDOFRUUHVSRQGHQWHD 'DV9DQWDJHQV(YHQWXDLV
XPWHUoRGDUHPXQHUDomRRXVXEVtGLRGRPrVHPTXH
DVIpULDVIRUHPLQLFLDGDV 6XEVHomR,
'R$X[tOLR1DWDOLGDGH
†ž 1RFDVRGHRVHUYLGRUHIHWLYRH[HUFHUIXQomRGHFRQIL
DQoDRXFDUJRHPFRPLVVmRDUHVSHFWLYDYDQWDJHPp $UW 2DX[tOLRQDWDOLGDGHpGHYLGRjVHUYLGRUDHIHWLYDSRU
FRQVLGHUDGDQRFiOFXORGRDGLFLRQDOGHTXHWUDWDHVWH PRWLYRGHQDVFLPHQWRGHILOKRHPTXDQWLDHTXLYDOHQWH
DUWLJRREVHUYDGDDSURSRUFLRQDOLGDGHGHTXHWUDWDR DRPHQRUYHQFLPHQWREiVLFRGRVHUYLoRS~EOLFRGLVWUL
DUW†ž WDOLQFOXVLYHQRFDVRGHQDWLPRUWR

†ž 2 DGLFLRQDO GH IpULDV LQFLGH VREUH R YDORUGRDERQR †ž 1DKLSyWHVHGHSDUWRP~OWLSORRYDORUGHYHVHUDFUHVFL


SHFXQLiULR GRGHFLQTXHQWDSRUFHQWRSRUQDVFLWXUR

†ž $EDVHSDUDRFiOFXORGRDGLFLRQDOGHIpULDVQmRSRGH †ž 2 DX[tOLRQDWDOLGDGH GHYH VHU SDJR DRF{QMXJHRX


VHUVXSHULRUDRWHWRGHUHPXQHUDomRRXVXEVtGLRVDOYR FRPSDQKHLUR VHUYLGRU S~EOLFR TXDQGR D SDUWXULHQWH
HPUHODomRDRDERQRSHFXQLiULR QmRIRUVHUYLGRUDS~EOLFDGLVWULWDO

6XEVHomR,, †ž 2GLVSRVWRQHVWHDUWLJRDSOLFDVHjVVLWXDo}HVGHDGR


'R'pFLPR7HUFHLUR6DOiULR omR
6XEVHomR,,
$UW 2GpFLPRWHUFHLURVDOiULRREVHUYDGRRGLVSRVWRQRDUW 'R$X[tOLR)XQHUDO
†žFRUUHVSRQGHjUHWULEXLomRSHFXQLiULDGRPrV
HPTXHpGHYLGRjUD]mRGHXPGR]HDYRVSRUPrVGH $UW 2DX[tOLRIXQHUDOpGHYLGRjIDPtOLDGRVHUYLGRUHIHWLYR
H[HUFtFLRQRVGR]HPHVHVDQWHULRUHV IDOHFLGRHPDWLYLGDGHRXDSRVHQWDGRHPYDORUHTXLYD
OHQWHDXPPrVGDUHPXQHUDomRVXEVtGLRRXSURYHQWR
6HFUHWDULDGH(VWDGRGH$ GPLQLVWUDomR3~EOLFD'LVWULWR)HGHUDO
 /HJLVODomR 

†ž 1R FDVR GH DFXPXODomR OHJDO GH FDUJRVRDX[t D GRLVLQWHLURVHGRLVGpFLPRVSRUFHQWRHPVHWUDWDQ


OLRIXQHUDOpSDJRVRPHQWHHPUD]mRGRFDUJRGHPDLRU GR GH DWLYLGDGHV SUHYLVWDV QRV LQFLVRV , H ,, GR
UHPXQHUDomRRXVXEVtGLR FDSXW

†ž 2DX[tOLRIXQHUDOGHYHVHUSDJRQRSUD]RGHTXDUHQWD E XPLQWHLURHGRLVGpFLPRVSRUFHQWRHPVHWUDWDQGR


HRLWRKRUDVSRUPHLRGHSURFHGLPHQWRVXPDUtVVLPR GHDWLYLGDGHSUHYLVWDQRVLQFLVRV,,,H,9GRFDSXW
jSHVVRDGDIDPtOLDTXHKRXYHUFXVWHDGRRIXQHUDO
†ž $ JUDWLILFDomR SRU HQFDUJR GH FXUVRRXFRQFXUVR
†ž 1R FDVR GH VHUYLGRU DSRVHQWDGR RDX[tOLRIXQHUDOp VRPHQWHSRGHVHUSDJDVHDVDWLYLGDGHVUHIHULGDVQRV
SDJRSHORUHJLPHSUySULRGHSUHYLGrQFLDVRFLDOPHGL LQFLVRV GR FDSXW IRUHP H[HUFLGDV VHP SUHMXt]R GDV
DQWHUHVVDUFLPHQWRGRVYDORUHVSHOR7HVRXURGR'LVWULWR DWULEXLo}HV GR FDUJR GH TXH R VHUYLGRU IRU WLWXODU
)HGHUDO GHYHQGR LPSOLFDU FRPSHQVDomR GH KRUiULR TXDQGR
GHVHPSHQKDGDV GXUDQWH D MRUQDGD GH WUDEDOKR QD
$UW 2 WHUFHLUR TXH FXVWHDU R IXQHUDO WHP GLUHLWR GHVHU IRUPDGRDUW†ž
LQGHQL]DGRQmRSRGHQGRDLQGHQL]DomRVXSHUDURYDORU
GHXPPrVGDUHPXQHUDomRVXEVtGLRRXSURYHQWR †ž $JUDWLILFDomRSRUHQFDUJRGHFXUVRRXFRQFXUVRQmRVH
LQFRUSRUDjUHPXQHUDomRGRVHUYLGRUSDUDTXDOTXHU
$UW (PFDVRGHIDOHFLPHQWRGHVHUYLGRUHPVHUYLoRIRUDGR HIHLWRHQmRSRGHVHUXWLOL]DGDFRPREDVHSDUDFiOFXOR
ORFDOGHWUDEDOKRLQFOXVLYHQRH[WHULRUDVGHVSHVDVGH GHTXDOTXHURXWUDYDQWDJHPQHPSDUDILQVGHFiOFXOR
WUDQVSRUWH GR FRUSR FRUUHP j FRQWD GH UHFXUVRV GR GRVSURYHQWRVGHDSRVHQWDGRULDRXGDVSHQV}HV
'LVWULWR)HGHUDOGDDXWDUTXLDRXGDIXQGDomRS~EOLFD
6HomR,;
6XEVHomR,,, 'DV9DQWDJHQVGH&DUiWHU,QGHQL]DWyULR
'D*UDWLILFDomRSRU(QFDUJRGH&XUVRRX&RQFXUVR
6XEVHomR,
$UW $ JUDWLILFDomR SRU HQFDUJR GH FXUVR RX FRQFXUVRp 'DV'LVSRVLo}HV*HUDLV
GHYLGDDRVHUYLGRUHVWiYHOTXHHPFDUiWHUHYHQWXDO
$UW 7HPFDUiWHULQGHQL]DWyULRRYDORUGDVSDUFHODVUHODWL
, DWXDUFRPRLQVWUXWRUHPFXUVRGHIRUPDomRGHGHVHQ YDVD
YROYLPHQWRRXGHWUHLQDPHQWRUHJXODUPHQWHLQVWLWXtGR
QRWDYLGHGHFUHWRQžGHGRGIGHTXHUHJXODPHQWDHVWHLQFLVR
QRV3RGHUHV([HFXWLYRRX/HJLVODWLYR LGHVWHDUW

,, SDUWLFLSDUGHEDQFDH[DPLQDGRUDRXGHFRPLVVmRGH , GLiULDHSDVVDJHPSDUDYLDJHP


FRQFXUVRSDUD ,, WUDQVSRUWH
,,, DOLPHQWDomR
D H[DPHVRUDLV ,9 FUHFKHRXHVFROD
E DQiOLVHGHFXUUtFXOR 9 IDUGDPHQWR
F FRUUHomRGHSURYDVGLVFXUVLYDV 9, FRQYHUVmRGHIpULDVRXGHSDUWHGHODVHPSHF~QLD
G HODERUDomRGHTXHVW}HVGHSURYDV 9,, DERQRGHSHUPDQrQFLD
H MXOJDPHQWRGHUHFXUVRVLQWHUSRVWRVSRUFDQGLGDWRV 9,,, FUpGLWRV GHFRUUHQWHV GH GHPLVVmR H[RQHUDomRH
DSRVHQWDGRULDRXUHODWLYRVDIpULDVDGLFLRQDOGHIpULDV
,,, SDUWLFLSDUGDORJtVWLFDGHSUHSDUDomRHGHUHDOL]DomRGH RXFRQYHUVmRGHOLFHQoDSUrPLRHPSHF~QLD
FRQFXUVR S~EOLFR HQYROYHQGR DWLYLGDGHV GH SODQHMD
PHQWRFRRUGHQDomRVXSHUYLVmRH[HFXomRHDYDOLDomR $UW 2VYDORUHVGDVLQGHQL]Do}HVDVVLPFRPRDVFRQGLo}HV
GH UHVXOWDGR TXDQGR WDLV DWLYLGDGHV QmR HVWLYHUHP SDUD D VXD FRQFHVVmR VmR HVWDEHOHFLGRV HP OHL RX
LQFOXtGDVHQWUHDVVXDVDWULEXLo}HVSHUPDQHQWHV UHJXODPHQWR REVHUYDGDV DV GLVSRVLo}HV GRV DUWLJRV
VHJXLQWHV
,9 SDUWLFLSDUGDDSOLFDomRGHSURYDVGHFRQFXUVRS~EOLFR
ILVFDOL]iODRXDYDOLiODEHPFRPRVXSHUYLVLRQDUHVVDV $UW 2YDORUGDVLQGHQL]Do}HVQmRSRGHVHU
DWLYLGDGHV
, LQFRUSRUDGRjUHPXQHUDomRRXDRVXEVtGLR
†ž 2VFULWpULRVGHFRQFHVVmRHRVOLPLWHVGDJUDWLILFDomR
SDUDDVDWLYLGDGHVGHTXHWUDWDHVWHDUWLJRVmRIL[DGRV ,, FRPSXWDGRQDEDVHGHFiOFXORSDUDILQVGHLQFLGrQFLD
HPUHJXODPHQWRREVHUYDGRVRVVHJXLQWHVSDUkPHWURV GHLPSRVWRGHUHQGDRXGHFRQWULEXLomRSDUDDSUHYL
GrQFLDVRFLDOUHVVDOYDGDVDVGLVSRVLo}HVHPFRQWUiULR
, RYDORUGDJUDWLILFDomRGHYHVHUFDOFXODGRHPKRUDV QDOHJLVODomRIHGHUDO
REVHUYDGDVDQDWXUH]DHDFRPSOH[LGDGHGDDWLYLGDGH
H[HUFLGD ,,, FRPSXWDGRSDUDFiOFXORGHTXDOTXHURXWUDYDQWDJHP
SHFXQLiULD
,, RSHUtRGRGHWUDEDOKRQDVDWLYLGDGHVGHTXHWUDWDHVWH
DUWLJRQmRSRGHH[FHGHUDFHQWRHYLQWHKRUDVDQXDLV 6XEVHomR,,
RX TXDQGR GHYLGDPHQWH MXVWLILFDGR H SUHYLDPHQWH 'D'LiULDHGD3DVVDJHP
DXWRUL]DGRSHODDXWRULGDGHPi[LPDGRyUJmRDXWDU
TXLDRXIXQGDomRDGX]HQWDVHTXDUHQWDKRUDVDQXDLV QRWDYLGHGHFUHWRQžGHGRGIGHTXHUHJXODPHQWDRDUW

,,, RYDORUPi[LPRGDKRUDWUDEDOKDGDFRUUHVSRQGHDRV $UW 2VHUYLGRUTXHDVHUYLoRVHDIDVWDUGR'LVWULWR)HGHUDO


VHJXLQWHV SHUFHQWXDLV LQFLGHQWHV VREUH R PDLRU HPFDUiWHUHYHQWXDORXWUDQVLWyULRID]MXVDSDVVDJHP
YHQFLPHQWR EiVLFR GD WDEHOD GH UHPXQHUDomR RX HGLiULDSDUDFREULUDVGHVSHVDVGHSRXVDGDDOLPHQWD
VXEVtGLRGRVHUYLGRU omRHORFRPRomRXUEDQD
  /HJLVODomR
 6HFUHWDULDGH(VWDGRGH$GPLQLVWUDomR3~EOLFD'LVWULWR)HGHUDO
†ž $GLiULDpFRQFHGLGD SRU GLD GHDIDVWDPHQWRVHQGR D DFXPXODomROtFLWDGHFDUJRVS~EOLFRV
GHYLGDSHODPHWDGHTXDQGRRGHVORFDPHQWRQmRH[LJLU
SHUQRLWH E VHUYLGRU TXH H[HUoD VXDV DWULEXLo}HV HP PDLV GH
XPDXQLGDGHDGPLQLVWUDWLYDGRyUJmRRXHQWLGDGH
†ž 1RVFDVRVHPTXHRDIDVWDPHQWRGR'LVWULWR)HGHUDO D TXH HVWHMD YLQFXODGR DTXL FRPSUHHQGLGRV RV
FRQVWLWXLUH[LJrQFLDSHUPDQHQWHGRFDUJRRVHUYLGRU HVWDEHOHFLPHQWRV S~EOLFRV GH HQVLQR H VD~GH GR
QmRID]MXVDGLiULD 'LVWULWR)HGHUDO
QRWDYLGHGHFUHWRQžGHGRGIGHTXHUHJXODPHQWDRDUW
†ž eIDFXOWDGRDRVHUYLGRURSWDUSHODSHUFHSomRGRDX[tOLR
$UW 2VHUYLGRUTXHUHFHEHUGLiULDRXSDVVDJHPHQmRVH UHIHUHQWHDRGHVORFDPHQWR
DIDVWDUGR'LVWULWR)HGHUDOSRUTXDOTXHUPRWLYRILFD
REULJDGR D UHVWLWXtODV LQWHJUDOPHQWH QR SUD]R GH , GDUHSDUWLomRS~EOLFDSDUDRXWURORFDOGHWUDEDOKRRX
VHWHQWDHGXDVKRUDVFRQWDGDVGDGDWDHPTXHGHYHULD YLFHYHUVD
WHUYLDMDGR
,, GR WUDEDOKR SDUD LQVWLWXLomR GH HQVLQR RQGHHVWHMD
†~QLFR 1DKLSyWHVHGHRVHUYLGRUUHWRUQDUjVHGHHPSUD]R UHJXODPHQWHPDWULFXODGRRXYLFHYHUVD
PHQRUGRTXHRSUHYLVWRSDUDRVHXDIDVWDPHQWRWHP
GHUHVWLWXLUQRSUD]RSUHYLVWRQHVWHDUWLJRDVGLiULDV $UW 2YDORUPHQVDOGRDX[tOLRWUDQVSRUWHFRUUHVSRQGHDR
UHFHELGDVHPH[FHVVR PRQWDQWH GDV GHVSHVDV UHDOL]DGDV FRP WUDQVSRUWH
FROHWLYRQRVWHUPRVGRDUWVXEWUDtGRRPRQWDQWH
6XEVHomR,,, GHVHLVSRUFHQWRLQFLGHQWHH[FOXVLYDPHQWHVREUH
'D,QGHQL]DomRGH7UDQVSRUWH
, VXEVtGLRRXYHQFLPHQWREiVLFRGRFDUJRHIHWLYRRFXSD
$UW 2VHUYLGRUTXHUHDOL]DGHVSHVDVFRPDXWLOL]DomRGH GRSHORVHUYLGRU
PHLRSUySULRGHORFRPRomRSDUDDH[HFXomRGHVHUYL
oRV H[WHUQRV SRU IRUoD GDV DWULEXLo}HV SUySULDV GR
,, UHWULEXLomRSHFXQLiULDGHFDUJRHPFRPLVVmRTXDQGR
FDUJRID]MXVjLQGHQL]DomRGHWUDQVSRUWHQDIRUPDGR
VHWUDWDUGHVHUYLGRUQmRGHWHQWRUGHFDUJRHIHWLYR
UHJXODPHQWR
6XEVHomR,9
'R$X[tOLR7UDQVSRUWH $UW 2SDJDPHQWRGRDX[tOLRWUDQVSRUWHHPSHF~QLDRXHP
YDOHWUDQVSRUWHGHYHVHUHIHWXDGRQRPrVDQWHULRUDR
$UW $RVHUYLGRUpGHYLGRDX[tOLRWUDQVSRUWHDVHUSDJRHP GDXWLOL]DomRGHWUDQVSRUWHFROHWLYRVDOYRQDVVHJXLQ
SHF~QLDRXHPYDOHWUDQVSRUWHGHVWLQDGRDRFXVWHLR WHVKLSyWHVHVTXDQGRSRGHVHUIHLWRDWpRPrVLPHGLD
SDUFLDOGDVGHVSHVDVUHDOL]DGDVFRPWUDQVSRUWHFROHWL WDPHQWHVXEVHTXHQWH
YR LQFOXVLYH LQWHUHVWDGXDO QR LQtFLR H QR ILP GD
MRUQDGDGHWUDEDOKRUHODFLRQDGDVFRPRGHVORFDPHQWR , HIHWLYR H[HUFtFLR QR FDUJR HP UD]mR GHSULPHLUD
GDUHVLGrQFLDSDUDRWUDEDOKRHYLFHYHUVD LQYHVWLGXUD RX UHLQtFLR GR H[HUFtFLR GHFRUUHQWH GH
OLFHQoDRXDIDVWDPHQWRSUHYLVWRVHPOHL
†ž 2 DX[tOLRWUDQVSRUWH QmR SRGH VHUFRPSXWDGRSDUD
FiOFXORGHTXDOTXHURXWUDYDQWDJHPSHFXQLiULD ,, PRGLILFDomRQRYDORUGDWDULIDGRWUDQVSRUWHFROHWLYR
QR HQGHUHoR UHVLGHQFLDO QR ORFDO GH WUDEDOKR QR
†ž 2DX[tOLRWUDQVSRUWHQmRpGHYLGR WUDMHWR RX QR PHLR GH WUDQVSRUWH XWLOL]DGR TXDQGR
SDVVDDVHUGHYLGDDFRPSOHPHQWDomRFRUUHVSRQGHQWH
, TXDQGRRyUJmRDXWDUTXLDRXIXQGDomRSURSRUFLRQDU
SRUPHLRVSUySULRVRXSRUPHLRGHWHUFHLURVFRQWUDWD ,,, PXGDQoDGHH[HUFtFLRILQDQFHLUR
GRV R WUDQVSRUWH GR VHUYLGRU SDUD R WUDEDOKR H YL
FHYHUVD †~QLFR $SOLFDVH R GLVSRVWR QR DUW  † ž QRFDVRGH
SDJDPHQWRLQGHYLGRGRDX[tOLRWUDQVSRUWH
,, GXUDQWHDVIpULDVOLFHQoDVDIDVWDPHQWRVRXDXVrQFLDV
DRVHUYLoRH[FHWRQRVFDVRVGH $UW $FRQFHVVmRGRDX[tOLRWUDQVSRUWHILFDFRQGLFLRQDGDj
DSUHVHQWDomR GH GHFODUDomR ILUPDGD SHOR SUySULR
D FHVVmR GR VHUYLGRU SDUD yUJmR GD DGPLQLVWUDomR VHUYLGRU GH TXH UHDOL]D GHVSHVDV FRP WUDQVSRUWH
GLUHWD DXWiUTXLFD RX IXQGDFLRQDO GR 'LVWULWR FROHWLYRQRVWHUPRVGRDUW
)HGHUDOFXMR{QXVGDUHPXQHUDomRUHFDLDVREUHR
yUJmRFHGHQWH †ž 2VHUYLGRUGHYHPDQWHUDWXDOL]DGRVRVGDGRVFDGDVWUD
LV TXH IXQGDPHQWDP D FRQFHVVmR GR DX[t
E SDUWLFLSDomRHPSURJUDPDGHWUHLQDPHQWRUHJXODU OLRWUDQVSRUWH
PHQWHLQVWLWXtGR
†ž 6HPSUHMXt]RGDILVFDOL]DomRGDDGPLQLVWUDomRS~EOLFD
F SDUWLFLSDomRHPM~ULHRXWURVVHUYLoRVREULJDWyULRV HGHHYHQWXDOUHVSRQVDELOLGDGHDGPLQLVWUDWLYDFLYLORX
SRUOHL
SHQDO SUHVXPHPVH YHUGDGHLUDV DV LQIRUPDo}HV
FRQVWDQWHVGDGHFODUDomRSUHVWDGDSHORVHUYLGRU
,,, TXDQGRDGHVSHVDPHQVDOFRPWUDQVSRUWHFROHWLYRIRU
LJXDORXLQIHULRUDRYDORUUHVXOWDQWHGDDSOLFDomRGR
SHUFHQWXDOGHTXHWUDWDRDUW 6XEVHomR9
'R$X[tOLR$OLPHQWDomR
,9 FXPXODWLYDPHQWHFRPRXWUREHQHItFLRRXYDQWDJHPGH
QDWXUH]D LJXDO RX VHPHOKDQWH RX FRP YDQWDJHP $UW e GHYLGR DR VHUYLGRU PHQVDOPHQWH RDX[t
SHVVRDORULJLQiULDGHTXDOTXHUIRUPDGHLQGHQL]DomR OLRDOLPHQWDomRFRPRYDORUIL[DGRQDIRUPDGDOHL
RXDX[tOLRSDJRVRERPHVPRWtWXORRXLGrQWLFRIXQGD
PHQWRVDOYRQRVFDVRVGH $UW 2DX[tOLRDOLPHQWDomRVXMHLWDVHDRVVHJXLQWHVFULWpUL
RV
6HFUHWDULDGH(VWDGRGH$ GPLQLVWUDomR3~EOLFD'LVWULWR)HGHUDO
 /HJLVODomR 

, RSDJDPHQWRpIHLWRHPSHF~QLDVHPFRQWUDSDUWLGD †ž $ FRQVLJQDomR HP IROKD GH SDJDPHQWRQmRWUD]


QHQKXPD UHVSRQVDELOLGDGH SDUD D DGPLQLVWUDomR
,, QmR SRGH VHU DFXPXODGR FRP RXWUR EHQHItFLRGD S~EOLFDVDOYRDGHUHSDVVDUDRWHUFHLURRYDORUGHVFRQ
PHVPDHVSpFLHDLQGDTXHSDJRLQQDWXUD WDGRGRVHUYLGRU

,,, GHSHQGHGHUHTXHULPHQWRGRVHUYLGRULQWHUHVVDGRQR $UW 2 VXEVtGLR D UHPXQHUDomR RX TXDOTXHU GHVXDV


TXDOGHFODUHQmRUHFHEHURPHVPREHQHItFLRHPRXWUR SDUFHODV WHP QDWXUH]D DOLPHQWDU H QmR p REMHWR GH
yUJmRRXHQWLGDGH DUUHVWR VHTXHVWUR RX SHQKRUD H[FHWR QRV FDVRV GH
SUHVWDomRGHDOLPHQWRVUHVXOWDQWHVGHGHFLVmRMXGLFLDO
,9 R VHX YDORU GHYH VHU DWXDOL]DGR DQXDOPHQWHSHOR
PHVPRtQGLFHTXHDWXDOL]DURVYDORUHVH[SUHVVRVHP †~QLFR 2 FUpGLWR HP FRQWD EDQFiULD QmRGHVFDUDFWHUL]DD
PRHGDFRUUHQWHQDOHJLVODomRGR'LVWULWR)HGHUDO QDWXUH]DMXUtGLFDGRVXEVtGLRRXUHPXQHUDomR

9 QmRpGHYLGRDRVHUYLGRUHPFDVRGH $UW $TXLWDomRGDIROKDGHSDJDPHQWRpIHLWDDWpRTXLQWR


GLD~WLOGRPrVVXEVHTXHQWH
D OLFHQoDRXDIDVWDPHQWRVHPUHPXQHUDomR
E OLFHQoDSRUPRWLYRGHGRHQoDHPSHVVRDGDIDPtOLD †~QLFR 1RFDVRGHHUURGHVIDYRUiYHODRVHUYLGRUQRSURFHVVD
F DIDVWDPHQWRSDUDHVWXGRRXPLVVmRQRH[WHULRU PHQWRGDIROKDGHSDJDPHQWRDTXLWDomRGRGpELWR
G VXVSHQVmRHPYLUWXGHGHSHQDGLVFLSOLQDU GHYHVHUIHLWDQRSUD]RGHDWpVHWHQWDHGXDVKRUDV
H IDOWDLQMXVWLILFDGDHQmRFRPSHQVDGD FRQWDGRVGDGDWDGHTXHWUDWDHVWHDUWLJR
†~QLFR $SOLFDVH R GLVSRVWR QR DUW  † ž DRFDVRGH
$UW $V UHSRVLo}HV H LQGHQL]Do}HV DR HUiULR GHYHPVHU
SDJDPHQWRLQGHYLGRGRDX[tOLRDOLPHQWDomR
FRPXQLFDGDVDRVHUYLGRUSDUDSDJDPHQWRQRSUD]RGH
6XEVHomR9, DWpGH]GLDVSRGHQGRDVHXSHGLGRVHUGHVFRQWDGDV
'R$ERQR3HFXQLiULR GDUHPXQHUDomRRXVXEVtGLR

$UW $FRQYHUVmRGHXPWHUoRGDVIpULDVHPDERQRSHFXQLi †ž 2GHVFRQWRGHYHVHUIHLWR


ULRGHSHQGHGHDXWRUL]DomRGR*RYHUQDGRUGR3UHVL
GHQWH GD &kPDUD /HJLVODWLYD RX GR 3UHVLGHQWH GR , HP SDUFHOD ~QLFD VH GH YDORU LJXDO RX LQIHULRUj
7ULEXQDOGH&RQWDV GpFLPDSDUWHGDUHPXQHUDomRRXVXEVtGLR

†ž 6REUHRYDORUGRDERQRSHFXQLiULRLQFLGHRDGLFLRQDO ,, HPSDUFHODVPHQVDLVLJXDLVjGpFLPDSDUWHGRVXEVtGLR


GHIpULDV RXUHPXQHUDomRGHYHQGRRUHVtGXRFRQVWLWXLUVHFRPR
~OWLPDSDUFHOD
†ž $EDVHSDUDRFiOFXORGRDERQRSHFXQLiULRQmRSRGH
VHUVXSHULRUDRWHWRGHUHPXQHUDomRRXVXEVtGLR †ž 1RFDVRGHHUURQRSURFHVVDPHQWRGDIROKDGHSDJD
PHQWR R YDORU LQGHYLGDPHQWH UHFHELGR GHYH VHU
6XEVHomR9,, GHYROYLGRSHORVHUYLGRUHPSDUFHOD~QLFDQRSUD]RGH
'R$ERQRGH3HUPDQrQFLD VHWHQWD H GXDV KRUDV FRQWDGRV GD GDWD HP TXH R
VHUYLGRUIRLFRPXQLFDGR
$UW 2 VHUYLGRU TXH SHUPDQHFHU HP DWLYLGDGH DSyVWHU
FRPSOHWDGRDVH[LJrQFLDVSDUDDSRVHQWDGRULDYROXQWi $UW 2SDJDPHQWRHIHWXDGRSHODDGPLQLVWUDomRS~EOLFDHP
ULDID]MXVDXPDERQRGHSHUPDQrQFLDHTXLYDOHQWHDR GHVDFRUGRFRPDOHJLVODomRQmRDSURYHLWDDRVHUYLGRU
YDORUGDVXDFRQWULEXLomRSUHYLGHQFLiULDQDIRUPDH EHQHILFLDGRDLQGDTXHHOHQmRWHQKDGDGRFDXVDDR
QDVFRQGLo}HVSUHYLVWDVQD&RQVWLWXLomR)HGHUDO HUUR
6HomR; †~QLFR e YHGDGR H[LJLU UHSRVLomR GH YDORU HPYLUWXGHGH
'DV'LVSRVLo}HV*HUDLV DSOLFDomRUHWURDWLYDGHQRYDLQWHUSUHWDomRGDQRUPD
GHUHJrQFLD
$UW 6HQmRIRUIHLWDDFRPSHQVDomRGHKRUiULRGHTXHWUDWD
RDUWRVHUYLGRUSHUGH $UW (PFDVRGHGHPLVVmRH[RQHUDomRDSRVHQWDGRULDRX
TXDOTXHUOLFHQoDRXDIDVWDPHQWRVHPUHPXQHUDomRR
, DUHPXQHUDomRRXVXEVtGLRGRVGLDVHPTXHIDOWDUDR VHUYLGRUWHPGLUHLWRGHUHFHEHURVFUpGLWRVDTXHID]
VHUYLoRVHPPRWLYRMXVWLILFDGR
MXVDWpDGDWDGRHYHQWR
,, DSDUFHODGDUHPXQHUDomRRXVXEVtGLRGLiULRSURSRUFL
†ž 2GLVSRVWRQHVWHDUWLJRDSOLFDVHLQFOXVLYHDRVFDVRV
RQDO DRV DWUDVRV DXVrQFLDV LQMXVWLILFDGDV H VDtGDV
GHGLVSHQVDGDIXQomRGHFRQILDQoDRXH[RQHUDomRGH
DQWHFLSDGDV
FDUJRHPFRPLVVmRTXDQGR
$UW 6DOYR SRU LPSRVLomR OHJDO RX PDQGDGRMXGLFLDO
QHQKXPGHVFRQWRSRGHLQFLGLUVREUHDUHPXQHUDomR , VHJXLGDVGHQRYDGLVSHQVDRXQRPHDomR
RXVXEVtGLR
,, VHWUDWDUGHVHUYLGRUHIHWLYRKLSyWHVHHPTXHID]MXVj
†ž 0HGLDQWH DXWRUL]DomR GR VHUYLGRU H DFULWpULRGD SHUFHSomR GRV FUpGLWRV GDt GHFRUUHQWHV LQFOXVLYH R
DGPLQLVWUDomR S~EOLFD SRGH KDYHU FRQVLJQDomR HP GpFLPR WHUFHLUR VDOiULR H DV IpULDV QD SURSRUomR
IROKDGHSDJDPHQWRDIDYRUGHWHUFHLURVFRPUHSRVL SUHYLVWDQHVWD/HL&RPSOHPHQWDU
omRGHFXVWRVQDIRUPDGHILQLGDHPUHJXODPHQWR
†ž 1DVKLSyWHVHVGHVWHDUWLJRKDYHQGRGpELWRGRVHUYLGRU
†ž $VRPDGDVFRQVLJQDo}HVGHTXHWUDWDR†žQmRSRGH FRPRHUiULRWHPHOHGHVHUGHGX]LGRLQWHJUDOPHQWH
H[FHGHUDWULQWDSRUFHQWRGDUHPXQHUDomRRXVXEVtGLR GRVFUpGLWRVTXHWHQKDRXYHQKDDWHUHPYLUWXGHGR
GRVHUYLGRU FDUJRRFXSDGR
  /HJLVODomR 6HFUHWDULDGH(VWDGRGH$GPLQLVWUDomR3~EOLFD'LVWULWR)HGHUDO


†ž 6HQGRLQVXILFLHQWHVRVFUpGLWRVRGpELWRQmRGHGX]LGR ,, RDERQRSHFXQLiULRVHGHIHULGR


WHPGHVHUTXLWDGRQRSUD]RGHVHVVHQWDGLDV
,,, RDGLDQWDPHQWRGHSDUFHODFRUUHVSRQGHQWHDTXDUHQWD
†ž 2GpELWRQmRTXLWDGRQDIRUPDGRV††žHžGHYHVHU SRUFHQWRGRYDORUOtTXLGRGRVXEVtGLRRXUHPXQHUD
GHVFRQWDGRGHTXDOTXHUYDORUTXHRGHYHGRUWHQKDRX omRGHVGHTXHUHTXHULGR
YHQKD D WHU FRPR FUpGLWR MXQWR DR 'LVWULWR )HGHUDO
LQFOXVLYHUHPXQHUDomRRXVXEVtGLRGHTXDOTXHUFDUJR †~QLFR 2DGLDQWDPHQWRGHTXHWUDWDRLQFLVR,,,pGHVFRQWDGR
S~EOLFRIXQomRGHFRQILDQoDSURYHQWRVGHDSRVHQWD GR VXEVtGLR RX UHPXQHUDomR GR VHUYLGRU HP TXDWUR
GRULDRXSHQVmRREVHUYDGRRGLVSRVWRQRDUW SDUFHODVPHQVDLVHVXFHVVLYDVGHLGrQWLFRYDORU

†ž $QmRTXLWDomRGRGpELWRQRSUD]RSUHYLVWRLPSOLFDVXD $UW 2VHUYLGRUTXHRSHUDGLUHWDHSHUPDQHQWHPHQWHFRP


LQVFULomRQDGtYLGDDWLYD UDLRV;RXVXEVWkQFLDVUDGLRDWLYDVWHPGHJR]DUYLQWH
GLDVFRQVHFXWLYRVGHIpULDVSRUVHPHVWUHGHDWLYLGDGH
†ž 2V FUpGLWRV D TXH R H[VHUYLGRU ID] MXVGHYHPVHU SURILVVLRQDOSURLELGDHPTXDOTXHUKLSyWHVHDDFXPXOD
TXLWDGRVQRSUD]RGHDWpVHVVHQWDGLDVVDOYRQRVFDVRV omR
GHLQVXILFLrQFLDGHGRWDomRRUoDPHQWiULDREVHUYDGR
RUHJXODPHQWR †~QLFR 2VHUYLGRUUHIHULGRQHVWHDUWLJRQmRID]MXVDRDERQR
SHFXQLiULR
$UW (PFDVRGHIDOHFLPHQWRGRVHUYLGRUHDSyVDDSXUDomR
GRVYDORUHVHGRVSURFHGLPHQWRVGHTXHWUDWDRDUW $UW $VIpULDVVRPHQWHSRGHPVHUVXVSHQVDVSRUPRWLYRGH
RVDOGRUHPDQHVFHQWHGHYHVHU FDODPLGDGHS~EOLFDFRPRomRLQWHUQDFRQYRFDomRSDUD
M~ULVHUYLoRPLOLWDURXHOHLWRUDORXSRUQHFHVVLGDGHGR
, SDJRDRVEHQHILFLiULRVGDSHQVmRHQDIDOWDGHVWHVDRV VHUYLoR
VXFHVVRUHVMXGLFLDOPHQWHKDELOLWDGRV †~QLFR $VXVSHQVmRGDVIpULDVGHSHQGHGH
,, FREUDGRQDIRUPDGDOHLFLYLOVHQHJDWLYR , SRUWDULDGR6HFUHWiULRGH(VWDGRRXDXWRULGDGHHTXLYD
OHQWHQR3RGHU([HFXWLYR
$UW 2 GpELWRGRVHUYLGRUFRPRHUiULRRXRFUpGLWRTXH
YHQKD D VHU UHFRQKHFLGR DGPLQLVWUDWLYD RX MXGLFLDO ,, DWRGR3UHVLGHQWHGD&kPDUD/HJLVODWLYDRXGR7ULEX
PHQWHGHYH QDOGH&RQWDVQRVUHVSHFWLYRVyUJmRV
, VHU DWXDOL]DGR SHOR PHVPR tQGLFH TXH DWXDOL]DURV $UW (PFDVRGHGHPLVVmRGHVWLWXLomRGHFDUJRHPFRPLV
YDORUHVH[SUHVVRVHPPRHGDFRUUHQWHQDOHJLVODomRGR VmR H[RQHUDomR RX DSRVHQWDGRULD DV IpULDV QmR
'LVWULWR)HGHUDO JR]DGDVVmRLQGHQL]DGDVSHORYDORUGDUHPXQHUDomR
RXVXEVtGLRGHYLGRQRPrVGDRFRUUrQFLDGRHYHQWR
,, VRIUHUFRPSHQVDomRGHPRUDQDIRUPDGDOHJLVODomR DFUHVFLGRGRDGLFLRQDOGHIpULDV
YLJHQWH
†ž 2 SHUtRGR GH IpULDV LQFRPSOHWR pLQGHQL]DGRQD
$UW eSURLELGDDSUHVWDomRGHVHUYLoRVJUDWXLWRVVDOYRRV SURSRUomRGHXPGR]HDYRVSRUPrVGHHIHWLYRH[HUFt
FDVRVSUHYLVWRVHPOHL FLR
&DStWXOR,, †ž 3DUDRVHIHLWRVGR†žDIUDomRVXSHULRUDTXDWRU]H
'$6)e5,$6 GLDVpFRQVLGHUDGDFRPRPrVLQWHJUDO
$UW $FDGDSHUtRGRGHGR]HPHVHVGHH[HUFtFLRRVHUYLGRU &DStWXOR,,,
ID]MXVDWULQWDGLDVGHIpULDV '$6/,&(1d$6
†ž 3DUD R SULPHLUR SHUtRGR DTXLVLWLYR GHIpULDVVmR 6HomR,
H[LJLGRVGR]HPHVHVGHHIHWLYRH[HUFtFLR 'DV'LVSRVLo}HV*HUDLV
†ž 2GLVSRVWRQR†žQmRVHDSOLFDDRVFDVRVGHIpULDV $UW $OpPGRDERQRGHSRQWRRVHUYLGRUID]MXVDOLFHQoD
FROHWLYDV KLSyWHVH HP TXH DV SULPHLUDV IpULDV VmR
SURSRUFLRQDLVDRHIHWLYRH[HUFtFLR , SRUPRWLYRGHDIDVWDPHQWRGRF{QMXJHRXFRPSDQKHL
UR
†ž e YHGDGR OHYDU j FRQWD GH IpULDV TXDOTXHUIDOWDDR ,, SRUPRWLYRGHGRHQoDHPSHVVRDGDIDPtOLD
VHUYLoR ,,, SDUDRVHUYLoRPLOLWDU
,9 SDUDDWLYLGDGHSROtWLFD
†ž $VIpULDVSRGHPVHUDFXPXODGDVSRUDWpGRLVSHUtRGRV 9 SUrPLRSRUDVVLGXLGDGH
QR FDVR GH QHFHVVLGDGH GR VHUYLoR UHVVDOYDGDV DV 9, SDUDWUDWDUGHLQWHUHVVHVSDUWLFXODUHV
KLSyWHVHVSUHYLVWDVHPOHJLVODomRHVSHFtILFD 9,, SDUDGHVHPSHQKRGHPDQGDWRFODVVLVWD
9,,, SDWHUQLGDGH
†ž 0HGLDQWHUHTXHULPHQWRGRVHUYLGRUHQRLQWHUHVVHGD ,; PDWHUQLGDGH
DGPLQLVWUDomRS~EOLFDDVIpULDVSRGHPVHUSDUFHODGDV
HPDWpWUrVSHUtRGRVQHQKXPGHOHVLQIHULRUDGH]GLDV ; PpGLFDRXRGRQWROyJLFD
$UW $WpGRLVGLDVDQWHVGHDVIpULDVVHUHPLQLFLDGDVGHYHP †~QLFR $ FRQFHVVmR GD OLFHQoDPDWHUQLGDGHVXMHLWDVHjV
VHUSDJRVDRVHUYLGRU QRUPDV GR UHJLPH GH SUHYLGrQFLD VRFLDO D TXH D
VHUYLGRUDVHHQFRQWUDILOLDGD
, RDGLFLRQDOGHIpULDV
6HFUHWDULDGH(VWDGRGH$ GPLQLVWUDomR3~EOLFD'LVWULWR)HGHUDO /HJLVODomR 

$UW $OLFHQoDFRQFHGLGDGHQWURGHVHVVHQWDGLDVGRWpUPL 6HomR,9


QR GH RXWUD GD PHVPD HVSpFLH p FRQVLGHUDGD FRPR 'D/LFHQoDSDUDR6HUYLoR0LOLWDU
SURUURJDomR
$UW $RVHUYLGRUFRQYRFDGRSDUDRVHUYLoRPLOLWDUpFRQFH
$UW $RWpUPLQRGDVOLFHQoDVSUHYLVWDVQRDUW,,D;R GLGD OLFHQoD QD IRUPD H QDV FRQGLo}HV SUHYLVWDV QD
VHUYLGRUWHPRGLUHLWRGHUHWRUQDUjPHVPDORWDomR OHJLVODomRHVSHFtILFD
FRPDPHVPDMRUQDGDGHWUDEDOKRGHDQWHVGRLQtFLR
GDOLFHQoDGHVGHTXHXPDRXRXWUDQmRWHQKDVRIULGR †~QLFR &RQFOXtGRRVHUYLoRPLOLWDURVHUYLGRUWHPDWpWULQWD
DOWHUDomRQRUPDWLYD GLDVVHPUHPXQHUDomRSDUDUHDVVXPLURH[HUFtFLRGR
FDUJR
6HomR,, 6HomR9
'D/LFHQoDSRU0RWLYRGH$IDVWDPHQWRGR&{QMXJH 'D/LFHQoDSDUD$WLYLGDGH3ROtWLFD
RX&RPSDQKHLUR
$UW 2VHUYLGRUWHPGLUHLWRDOLFHQoDSDUDDWLYLGDGHSROtWLFD
$UW 3RGH VHU FRQFHGLGD OLFHQoD DR VHUYLGRU HVWiYHOSDUD QRVSHUtRGRVFRPSUHHQGLGRVHQWUH
DFRPSDQKDUF{QMXJHRXFRPSDQKHLURTXHIRUGHVORFD
GRSDUD , DGDWDGHVXDHVFROKDHPFRQYHQomRSDUWLGiULDFRPR
FDQGLGDWRDFDUJRHOHWLYRHDYpVSHUDGRUHJLVWURGD
, WUDEDOKDUHPORFDOLGDGHVLWXDGDIRUDGD5HJLmR,QWH FDQGLGDWXUDSHUDQWHD-XVWLoD(OHLWRUDO
JUDGD GH 'HVHQYROYLPHQWR (FRQ{PLFR GR 'LVWULWR
)HGHUDOH(QWRUQR5,'( ,, RUHJLVWURGDFDQGLGDWXUDSHUDQWHD-XVWLoD(OHLWRUDOH
DWpGH]GLDVDSyVDGDWDGDHOHLomRSDUDDTXDOFRQFRU
,, H[HUFHUPDQGDWRHOHWLYRHP(VWDGRRX0XQLFtSLRQmR UH
FRPSUHHQGLGRQD5,'(
†ž 1RFDVRGRLQFLVR,DOLFHQoDpVHPUHPXQHUDomRRX
†ž $OLFHQoDpSRUSUD]RGHDWpFLQFRDQRVHVHPUHPXQH VXEVtGLRQRFDVRGRLQFLVR,,pFRPUHPXQHUDomRRX
UDomRRXVXEVtGLR VXEVtGLR

†ž $PDQXWHQomRGRYtQFXORFRQMXJDOGHYHVHUFRPSURYD †ž 1HJDGRRUHJLVWURRXKDYHQGRGHVLVWrQFLDGDFDQGLGD


GDDQXDOPHQWHVRESHQDGHFDQFHODPHQWRGDOLFHQoD WXUDRVHUYLGRUWHPGHUHDVVXPLURFDUJRLPHGLDWD
PHQWH
†ž 9(7$'2 
†ž 2VHUYLGRUFDQGLGDWRDFDUJRHOHWLYRTXHH[HUoDFDUJR
6HomR,,, HP FRPLVVmR RX IXQomR GH FRQILDQoD GHOH GHYH VHU
'D/LFHQoDSRU0RWLYRGH'RHQoDHP3HVVRDGD)DPtOLD H[RQHUDGR RX GLVSHQVDGR REVHUYDGRV RV SUD]RV GD
OHJLVODomRHOHLWRUDO
$UW 3RGHVHUFRQFHGLGDOLFHQoDDRVHUYLGRUSRUPRWLYRGH
GRHQoD GR F{QMXJH RX FRPSDQKHLUR SDGUDVWR RX $UW 2 VHUYLGRU HIHWLYR TXH SUHWHQGD VHU FDQGLGDWRGHYH
PDGUDVWDDVFHQGHQWHGHVFHQGHQWHHQWHDGRHFRODWH ILFDU DIDVWDGR GH VXDV DWULEXLo}HV KDELWXDLV TXDQGR
UDO FRQVDQJXtQHR RX DILP DWp R VHJXQGR JUDX FLYLO DVVLPRH[LJLUDOHJLVODomRHOHLWRUDO
PHGLDQWHFRPSURYDomRSRUMXQWDPpGLFDRILFLDO QRYD
UHGDomRGDGDDRFDSXWGRDUWLJRSHOD/(,&203/(0(17$51ž'( †ž $R VHUYLGRU DIDVWDGR QD IRUPD GHVWHDUWLJRVHP
SUHMXt]R GD UHPXQHUDomR RX VXEVtGLR GHYHP VHU
†ž $OLFHQoDVRPHQWHSRGHVHUGHIHULGDVHDDVVLVWrQFLD FRPHWLGDVDWULEXLo}HVFRPSDWtYHLVFRPVHXFDUJRHD
GLUHWDGRVHUYLGRUIRULQGLVSHQViYHOHQmRSXGHUVHU OHJLVODomRHOHLWRUDO
SUHVWDGDVLPXOWDQHDPHQWHFRPRH[HUFtFLRGRFDUJR
†ž 2DIDVWDPHQWRGHTXHWUDWDR†žHQFHUUDVHQDGDWD
†ž $OLFHQoDpFRQFHGLGDVHPSUHMXt]RGDUHPXQHUDomRRX GDFRQYHQomRSDUWLGiULDDSOLFDQGRVHDSDUWLUGDtR
VXEVtGLRGRFDUJRHIHWLYR GLVSRVWRQRDUW,H,,

†ž 1HQKXPSHUtRGRGHOLFHQoDSRGHVHUVXSHULRUDWULQWD 6HomR9,


GLDVHRVRPDWyULRGRVSHUtRGRVQmRSRGHXOWUDSDVVDU 'D/LFHQoD3UrPLRSRU$VVLGXLGDGH
FHQWRHRLWHQWDGLDVSRUDQRLQLFLDQGRVHDFRQWDJHP
FRPDSULPHLUDOLFHQoD $UW $SyV FDGD TXLQTXrQLR LQLQWHUUXSWR GH H[HUFtFLRR
VHUYLGRUHIHWLYRID]MXVDWUrVPHVHVGHOLFHQoDSUrPLR
†ž &RPSURYDGDSRUMXQWDPpGLFDRILFLDODQHFHVVLGDGHGH SRU DVVLGXLGDGH VHP SUHMXt]R GD UHPXQHUDomR RX
OLFHQoDSRUSHUtRGRVXSHULRUDFHQWRHRLWHQWDGLDVD VXEVtGLRGRFDUJRHIHWLYR
OLFHQoDpVHPUHPXQHUDomRRXVXEVtGLRREVHUYDGRR
SUD]RLQLFLDOSUHYLVWRQR†ž $UW $FRQWDJHPGRSUD]RSDUDDTXLVLomRGDOLFHQoDSUrPLR
pLQWHUURPSLGDTXDQGRRVHUYLGRUGXUDQWHRSHUtRGR
$UW eYHGDGRRH[HUFtFLRGHDWLYLGDGHUHPXQHUDGDGXUDQWH DTXLVLWLYR
RXVXIUXWRGDOLFHQoDSUHYLVWDQRDUW
, VRIUHUVDQomRGLVFLSOLQDUGHVXVSHQVmR
†~QLFR 6mRFRQVLGHUDGRVFRPRIDOWDVLQMXVWLILFDGDVDRVHUYLoR
SDUDWRGRVRVHIHLWRVOHJDLVRVGLDVHPTXHIRUFRQVWD ,, OLFHQFLDUVHRXDIDVWDUVHGRFDUJRVHPUHPXQHUDomR
WDGRHPSURFHVVRGLVFLSOLQDURH[HUFtFLRGHDWLYLGDGH
UHPXQHUDGD GXUDQWH D OLFHQoD SUHYLVWD QR DUW  †~QLFR $VIDOWDVLQMXVWLILFDGDVDRVHUYLoRUHWDUGDPDFRQFHVVmR
DLQGDTXHDOLFHQoDVHWHQKDGDGRVHPUHPXQHUDomR GDOLFHQoDSUHYLVWDQHVWHDUWLJRQDSURSRUomRGHXP
RXVXEVtGLR PrVSDUDFDGDIDOWD
  /HJLVODomR 6HFUHWDULDGH(VWDGRGH$GPLQLVWUDomR3~EOLFD'LVWULWR)HGHUDO


$UW 2 Q~PHUR GH VHUYLGRUHV HP JR]R VLPXOWkQHRGH ,, FDGDVLQGLFDWRWHPGLUHLWRjOLFHQoDGH


OLFHQoDSUrPLRQmRSRGHVHUVXSHULRUDXPWHUoRGD
ORWDomRGDUHVSHFWLYDXQLGDGHDGPLQLVWUDWLYDGRyUJmR D GRLV GLULJHQWHV GHVGH TXH WHQKD QR PtQLPR
DXWDUTXLDRXIXQGDomR WUH]HQWRVVHUYLGRUHVILOLDGRV

$UW 2V SHUtRGRV GH OLFHQoDSUrPLR DGTXLULGRV HQmR E XPGLULJHQWHSDUDFDGDJUXSRGHGRLVPLOVHUYLGR


JR]DGRVVmRFRQYHUWLGRVHPSHF~QLDTXDQGRRVHUYL UHVILOLDGRVDOpPGRVGLULJHQWHVSUHYLVWRVQDDOtQHD
GRUIRUDSRVHQWDGR DDWpROLPLWHGHGH]GLULJHQWHV

†~QLFR (PFDVRGHIDOHFLPHQWRGRVHUYLGRUDFRQYHUVmRHP †~QLFR 3DUD FDGD GRLV GLULJHQWHV VLQGLFDLVOLFHQFLDGRVQD


SHF~QLDGHTXHWUDWDHVWHDUWLJRpSDJDDRVEHQHILFLiUL IRUPDGHVWHDUWLJRREVHUYDGRRUHJXODPHQWRSRGHVHU
RV GD SHQVmR RX QmR RV KDYHQGR DRV VXFHVVRUHV OLFHQFLDGRPDLVXPGHYHQGRRVLQGLFDWRUHVVDUFLUDR
MXGLFLDOPHQWHKDELOLWDGRV yUJmR RX HQWLGDGH R YDORU WRWDO GHVSHQGLGR FRP
UHPXQHUDomR RX VXEVtGLR DFUHVFLGR GRV HQFDUJRV
VRFLDLV H SURYLV}HV SDUD IpULDV DGLFLRQDO GH IpULDV
$UW )LFD DVVHJXUDGR jV VHUYLGRUDV S~EOLFDV R GLUHLWRGH
GpFLPRWHUFHLURVDOiULRHFRQYHUVmRGHOLFHQoDSUrPLR
LQLFLDU D IUXLomR GH OLFHQoDSUrPLR SRU DVVLGXLGDGH
HPSHF~QLD
ORJRDSyVRWpUPLQRGDOLFHQoDPDWHUQLGDGH

†~QLFR 2 GLUHLWR DVVHJXUDGR QHVWH DUWLJR DSOLFDVHjOLFHQ $UW 3DUDRGHVHPSHQKRGHPDQGDWRHPFHQWUDOVLQGLFDO


FRQIHGHUDomR RX IHGHUDomR SRGH VHU OLFHQFLDGR XP
oDSUrPLRSRUDVVLGXLGDGHFXMRSHUtRGRGHDTXLVLomR
VHUYLGRUSDUDFDGDJUXSRGHYLQWHHFLQFRPLODVVRFLD
IRU FRPSOHWDGR DWp GH] GLDV DQWHV GR WpUPLQR GD
GRVSRULQVWLWXLomR
OLFHQoDPDWHUQLGDGH
†ž 2JUXSRGHVHUYLGRUHVUHIHULGRQRFDSXWpDIHULGRSHOR
6HomR9,, Q~PHURGHVHUYLGRUHVDVVRFLDGRVDRVVLQGLFDWRVILOLDGRV
'D/LFHQoDSDUD7UDWDUGH,QWHUHVVHV3DUWLFXODUHV DFDGDLQVWLWXLomRGHTXHWUDWDHVWHDUWLJR
$UW $FULWpULRGDDGPLQLVWUDomRS~EOLFDSRGHVHUFRQFHGL †ž 2VHUYLGRUGHYHVHUHOHLWRGLULJHQWHSHODFDWHJRULD
GDDRVHUYLGRUHVWiYHOOLFHQoDSDUDWUDWDUGHDVVXQWRV
SDUWLFXODUHVSHORSUD]RGHDWpWUrVDQRVFRQVHFXWLYRV $UW $OLFHQoDWHPGXUDomRLJXDOjGRPDQGDWRSRGHQGR
VHPUHPXQHUDomRGHVGHTXH VHUSURUURJDGDQRFDVRGHUHHOHLomR

, QmRSRVVXDGpELWRFRPRHUiULRUHODFLRQDGRFRPVXD $UW 2VHUYLGRULQYHVWLGRHPPDQGDWRFODVVLVWDGXUDQWHR


VLWXDomRIXQFLRQDO PDQGDWRHDWpXPDQRDSyVRVHXWpUPLQRQmRSRGH
VHUUHPRYLGRRXUHGLVWULEXtGRGHRItFLRSDUDXQLGDGH
,, QmRVHHQFRQWUHUHVSRQGHQGRDSURFHVVRGLVFLSOLQDU DGPLQLVWUDWLYDGLYHUVDGDTXHODGHRQGHVHDIDVWRXSDUD
H[HUFHURPDQGDWR
†ž $OLFHQoDSRGHVHULQWHUURPSLGDDTXDOTXHUWHPSRD
SHGLGRGRVHUYLGRURXDFULWpULRGDDGPLQLVWUDomR 6HomR,;
'D/LFHQoD3DWHUQLGDGH
†ž 2VHUYLGRUQmRSRGHH[HUFHUFDUJRRXHPSUHJRS~EOLFR
QRWD YLGH GHFUHWR Qž  GH   GRGI GH  TXH LQVWLWXL R SURJUDPD GH
LQDFXPXOiYHO GXUDQWH D OLFHQoD GH TXH WUDWD HVWH SURUURJDomRGDOLFHQoDSDWHUQLGDGHSDUDRVVHUYLGRUHVUHJLGRVSRUHVWDOHLFRPSOHPHQWDU
DUWLJR
$UW 3HORQDVFLPHQWRRXDGRomRGHILOKRVRVHUYLGRUWHP
†ž $OLFHQoDSRGHVHUSURUURJDGDSRULJXDOSHUtRGRXPD GLUHLWRDOLFHQoDSDWHUQLGDGHGHVHWHGLDVFRQVHFXWLYRV
~QLFDYH] LQFOXtGRRGLDGDRFRUUrQFLD

6HomR9,,, 6HomR;
'D/LFHQoDSDUDR'HVHPSHQKRGH0DQGDWR&ODVVLVWD 'R$ERQRGH3RQWR

$UW )LFDDVVHJXUDGRDRVHUYLGRUHVWiYHORGLUHLWRDOLFHQoD $UW 2 VHUYLGRU TXH QmR WLYHU IDOWD LQMXVWLILFDGD QRDQR
SDUDRGHVHPSHQKRGHPDQGDWRHPFHQWUDOVLQGLFDO DQWHULRUID]MXVDRDERQRGHSRQWRGHFLQFRGLDV
FRQIHGHUDomRIHGHUDomRRXVLQGLFDWRUHSUHVHQWDWLYRV
GH VHUYLGRUHV GR 'LVWULWR )HGHUDO UHJXODUPHQWH †ž 3DUDDTXLVLomRGRGLUHLWRDRDERQRGHSRQWRpQHFHV
ViULRTXHRVHUYLGRUWHQKDHVWDGRHPHIHWLYRH[HUFtFLR
UHJLVWUDGRVQRyUJmRFRPSHWHQWH
GHžGHMDQHLURDGHGH]HPEURGRDQRDTXLVLWLYR
†ž $ OLFHQoD SUHYLVWD QHVWH DUWLJR pFRQVLGHUDGDFRPR
†ž 2GLUHLWRDRJR]RGRDERQRGHSRQWRH[WLQJXHVHHP
HIHWLYRH[HUFtFLR GHGH]HPEURGRDQRVHJXLQWHDRGRDQRDTXLVLWLYR
†ž $UHPXQHUDomRRXVXEVtGLRGRVHUYLGRUOLFHQFLDGRQD †ž 2JR]RGRDERQRGHSRQWRSRGHVHUHPGLDVLQWHUFDOD
IRUPDGHVWHDUWLJRHRVHQFDUJRVVRFLDLVGHFRUUHQWHV GRV
VmR SDJRV SHOR yUJmR RX HQWLGDGH GH ORWDomR GR
VHUYLGRU †ž 2Q~PHURGHVHUYLGRUHVHPJR]RGHDERQRGHSRQWR
QmR SRGH VHU VXSHULRU D XP TXLQWR GD ORWDomR GD
$UW $OLFHQoDGHVHUYLGRUSDUDVLQGLFDWRUHSUHVHQWDWLYRGH UHVSHFWLYDXQLGDGHDGPLQLVWUDWLYDGRyUJmRDXWDUTXLD
FDWHJRULDGHVHUYLGRUHVFLYLVGR'LVWULWR)HGHUDOpIHLWD RXIXQGDomR
GDIRUPDVHJXLQWH
†ž 2FRUUHQGRDLQYHVWLGXUDDSyVžGHMDQHLURGRSHUtRGR
, R VHUYLGRU WHP GH VHU HOHLWR GLULJHQWH VLQGLFDOSHOD DTXLVLWLYR R VHUYLGRU ID] MXV D XP GLD GH DERQR GH
FDWHJRULD SRQWRSRUELPHVWUHGHHIHWLYRH[HUFtFLRDWpROLPLWHGH
FLQFRGLDV
6HFUHWDULDGH(VWDGRGH$ GPLQLVWUDomR3~EOLFD'LVWULWR)HGHUDO
 /HJLVODomR 

&DStWXOR,9 †~QLFR 7HUPLQDGDDFHVVmRRVHUYLGRUWHPGHDSUHVHQWDUVH


'26$)$67$0(1726 DRyUJmRDXWDUTXLDRXIXQGDomRGHRULJHPDWpRGLD
VHJXLQWHDRGDH[RQHUDomRRXGDUHYRJDomRLQGHSHQ
6HomR, GHQWHPHQWHGHFRPXQLFDomRHQWUHRFHVVLRQiULRHR
'R$IDVWDPHQWRSDUD6HUYLUHP2XWURÐUJmRRX(QWLGDGH FHGHQWH

6XEVHomR, $UW 2{QXVGDFHVVmRpGRyUJmRRXHQWLGDGHFHVVLRQiULD


'R([HUFtFLRHP2XWUR&DUJR
†~QLFR ([FHWXDVHGRGLVSRVWRQHVWHDUWLJRSDVVDQGRR{QXV
$UW 'HVGHTXHQmRKDMDSUHMXt]RSDUDRVHUYLoRRVHUYLGRU SDUDRyUJmRDXWDUTXLDRXIXQGDomRFHGHQWHDFHVVmR
HIHWLYRSRGHVHUFHGLGRDRXWURyUJmRRXHQWLGDGHGRV SDUDH[HUFtFLRGHFDUJR
3RGHUHVGR'LVWULWR)HGHUDOGD8QLmRGRV(VWDGRVRX
GRV0XQLFtSLRVSDUDRH[HUFtFLRGH , SUHYLVWRQRDUW,,D9H†ž

, HPSUHJRRXFDUJRHPFRPLVVmRRXIXQomRGHFRQILDQ ,, HPFRPLVVmRGDDGPLQLVWUDomRGLUHWDDXWiUTXLFDRX


oDFXMDUHPXQHUDomRRXVXEVtGLRVHMDVXSHULRUD IXQGDFLRQDO GH TXDOTXHU GRV 3RGHUHV GR 'LVWULWR
)HGHUDO
D XPGpFLPRGRVXEVtGLRGH6HFUHWiULRGH(VWDGRQR
FDVRGR'LVWULWR)HGHUDO $UW 1DFHVVmRFRP{QXVSDUDRFHVVLRQiULRVmRUHVVDUFLGRV
DR yUJmR FHGHQWH RV YDORUHV GD UHPXQHUDomR RX
E XPTXLQWRGRVXEVtGLRGH6HFUHWiULRGH(VWDGRQRV VXEVtGLRDFUHVFLGRVGRVHQFDUJRVVRFLDLVHGDVSURYL
GHPDLVFDVRV V}HVSDUDIpULDVDGLFLRQDOGHIpULDVGpFLPRWHUFHLUR
VDOiULRHOLFHQoDSUrPLRSRUDVVLGXLGDGH
,, FDUJRV LQWHJUDQWHV GD *RYHUQDGRULD RX9L
FH*RYHUQDGRULDGR'LVWULWR)HGHUDORXGD&DVD&LYLO †ž 2 yUJmR RX HQWLGDGH FHGHQWH WHP GHDSUHVHQWDUDR
HGR*DELQHWHGH6HJXUDQoD,QVWLWXFLRQDOGD3UHVLGrQ FHVVLRQiULR PHQVDOPHQWH D IDWXUD FRP RV YDORUHV
FLDGD5HS~EOLFD GLVFULPLQDGRVSRUSDUFHODVUHPXQHUDWyULDVHQFDUJRV
VRFLDLVHSURYLV}HV
,,, FDUJRHPFRPLVVmRRXIXQomRGHFRQILDQoDHPJDELQH
WH GH 'HSXWDGR )HGHUDO RX 6HQDGRU GD 5HS~EOLFD †ž +DYHQGRDWUDVRVVXSHULRUHVDVHVVHQWDGLDVQRUHVVDUFL
LQWHJUDQWHGDEDQFDGDGR'LVWULWR)HGHUDO PHQWR D FHVVmR WHP GH VHU UHYRJDGD GHYHQGR R
VHUYLGRU UHDSUHVHQWDUVH DR VHX yUJmR DXWDUTXLD RX
,9 FDUJRHPFRPLVVmRRXIXQomRGHFRQILDQoDGH6HFUHWi IXQGDomRGHRULJHP
ULR0XQLFLSDOQRV0XQLFtSLRVTXHFRQVWLWXHPD5,'( †ž )LFDDXWRUL]DGDDFRPSHQVDomRGHYDORUHVTXDQGRR
'LVWULWR)HGHUDOIRUFHGHQWHHFHVVLRQiULRGHVHUYLGR
9 FDUJRHPFRPLVVmRRXIXQomRGHFRQILDQoDQDViUHDV UHV
FRUUHODWDVGD8QLmRGHVHUYLGRUHVGDViUHDVGHVD~GH
HGXFDomRRXVHJXUDQoDS~EOLFD $UW 2VHUYLGRUTXDQGRQRH[HUFtFLRGHFDUJRHPFRPLVVmR
RXIXQomRGHFRQILDQoDILFDDIDVWDGRGDVDWULEXLo}HV
†ž ­FHVVmRGHVHUYLGRUGR3RGHU([HFXWLYRSDUDyUJmRGR GRVHXFDUJRGHSURYLPHQWRHIHWLYR
3RGHU/HJLVODWLYRDSOLFDVHRVHJXLQWH
†ž 2 GLVSRVWR QHVWH DUWLJR DSOLFDVH DRVHUYLGRUTXH
, QRFDVRGD&kPDUD/HJLVODWLYDSRGHPVHUFHGLGRVDWp DFXPXODUOLFLWDPHQWHGRLVFDUJRVHIHWLYRV
FLQFRVHUYLGRUHVSRU*DELQHWH3DUODPHQWDU
†ž 1R FDVR GR † ž D UHPXQHUDomR GRVHJXQGRFDUJR
,, QRFDVRGR&RQJUHVVR1DFLRQDOSRGHPVHUFHGLGRVDWp HIHWLYR GHSHQGH GD FRQWUDSUHVWDomR GH VHUYLoR H GD
GRLVVHUYLGRUHVSRUJDELQHWHGH'HSXWDGR)HGHUDORX FRPSDWLELOLGDGHGHKRUiULRFRPRFDUJRHPFRPLVVmR
6HQDGRUGD5HS~EOLFDHOHLWRSHOR'LVWULWR)HGHUDO RXIXQomRGHFRQILDQoD

†ž $FHVVmRGHVHUYLGRUpDXWRUL]DGDSHOR †ž $FRQWUDSUHVWDomRGHVHUYLoRHDFRPSDWLELOLGDGHGH


KRUiULRFRPRFDUJRHPFRPLVVmRRXIXQomRGHFRQIL
, *RYHUQDGRUQR3RGHU([HFXWLYR DQoDGHTXHWUDWDR†žGHYHPVHUGHFODUDGDVSHODV
,, 3UHVLGHQWHGD&kPDUD/HJLVODWLYD DXWRULGDGHVPi[LPDVGRVyUJmRVRXHQWLGDGHVHQYROYL
,,, 3UHVLGHQWHGR7ULEXQDOGH&RQWDV GRV

†ž (PFDUiWHUH[FHSFLRQDOSRGHVHUDXWRUL]DGDFHVVmRH †ž ,QGHSHQGHQWHPHQWHGDFRQWUDSUHVWDomRGRVHUYLoRVH


UHTXLVLomRIRUDGDVKLSyWHVHVSUHYLVWDVQHVWHDUWLJRH DVRPDGDVKRUDVGHWUDEDOKRGRVFDUJRVHPUHJLPHGH
QRDUW DFXPXODomR QmR VXSHUDU TXDUHQWD H TXDWUR KRUDV
VHPDQDLVRVHUYLGRUDIDVWDGRQDIRUPDGHVWH DUWLJR
†ž 2VHUYLGRUWHPJDUDQWLGRVWRGRVRVGLUHLWRVUHIHUHQWHV ID] MXV j UHPXQHUDomR RX VXEVtGLR GRV GRLV FDUJRV
DR H[HUFtFLR GR FDUJR HIHWLYR GXUDQWH R SHUtRGR HP HIHWLYRVVDOYRQRFDVRGDRSomRGHTXHWUDWDRDUW
TXHHVWLYHUFHGLGR †ž
6XEVHomR,,
$UW $FHVVmRWHUPLQDFRPD 'R([HUFtFLRHP2XWURÐUJmR

, H[RQHUDomRGRFDUJRSDUDRTXDORVHUYLGRUIRLFHGLGR $UW 2VHUYLGRUHVWiYHOVHPSUHMXt]RGDUHPXQHUDomRRX


VDOYRVHKRXYHUQRYDQRPHDomRQDPHVPDGDWD VXEVtGLR H GRV GHPDLV GLUHLWRV UHODWLYRV DR FDUJR
HIHWLYRSRGHVHUFRORFDGRjGLVSRVLomRGHRXWURyUJmR
,, UHYRJDomRSHODDXWRULGDGHFHGHQWH RXHQWLGDGHSDUDRH[HUFtFLRGHDWULEXLo}HVHVSHFtILFDV
QRVVHJXLQWHVFDVRV
  /HJLVODomR
 6HFUHWDULDGH(VWDGRGH$GPLQLVWUDomR3~EOLFD'LVWULWR)HGHUDO
, LQWHUHVVHGRVHUYLoR , HVWXGR RX PLVVmR RILFLDO FRP D UHPXQHUDomRRX
,, GHILFLrQFLDGHSHVVRDOHPyUJmRDXWDUTXLDRXIXQGD VXEVtGLRGRFDUJRHIHWLYR
omRVHPTXDGURSUySULRGHVHUYLGRUHVGHFDUUHLUD
,,, UHTXLVLomRGD3UHVLGrQFLDGD5HS~EOLFD ,, VHUYLoRVHPUHPXQHUDomRHPRUJDQLVPRLQWHUQDFLRQDO
,9 UHTXLVLomRGR7ULEXQDO6XSHULRU(OHLWRUDORXGR7ULEX GHTXHR%UDVLOSDUWLFLSHRXFRPRTXDOFRRSHUH
QDO5HJLRQDO(OHLWRUDOGR'LVWULWR)HGHUDO
†ž $DXVrQFLDQmRSRGHH[FHGHUDTXDWURDQRVQHPSRGH
†ž 2LQWHUHVVHGRVHUYLoRFDUDFWHUL]DVHTXDQGRRUHPD VHUFRQFHGLGD QRYDOLFHQoDDQWHVGHGHFRUULGRLJXDO
QHMDPHQWRGHSHVVRDOVHGHVWLQDD SHUtRGR

, ORWDUSHVVRDOGHyUJmRRXXQLGDGHRUJkQLFDUHHVWUXWX †ž (P FDVR GH H[RQHUDomRGHPLVVmRDSRVHQWDGRULD


UDGRRXFRPH[FHVVRGHSHVVRDO YROXQWiULDOLFHQoDSDUDWUDWDUGHLQWHUHVVHSDUWLFXODU
RXYDFkQFLDHPUD]mRGHSRVVHHPRXWURFDUJRLQDFX
,, SURPRYHU R DMXVWDPHQWRGHSHVVRDOjVQHFHVVLGDGHV PXOiYHODQWHVGHGHFRUULGRSHUtRGRLJXDODRGRDIDVWD
GRVVHUYLoRVSDUDJDUDQWLURGHVHPSHQKRGDVDWLYLGD PHQWRRVHUYLGRUEHQHILFLDGRSHORGLVSRVWRQRLQFLVR
GHVGRyUJmRFHVVLRQiULR , WHP GH UHVVDUFLU SURSRUFLRQDOPHQWH D GHVSHVD
LQFOXtGDDUHPXQHUDomRRXRVXEVtGLRHRVHQFDUJRV
,,, YLDELOL]DU D H[HFXomR GH SURMHWRV RX Do}HV FRPILP VRFLDLVKDYLGDFRPVHXDIDVWDPHQWRHGXUDQWHHOH
GHWHUPLQDGRHSUD]RFHUWR
6HomR,9
†ž 1RFDVRGRVLQFLVRV,H,,GRFDSXWRDIDVWDPHQWRGR 'R$IDVWDPHQWRSDUD3DUWLFLSDUGH&RPSHWLomR'HVSRUWLYD
FDUJRHIHWLYRUHVWULQJHVHDRkPELWRGRPHVPR3RGHU
HVySRGHVHUSDUDILPGHWHUPLQDGRHDSUD]RFHUWR $UW 0HGLDQWHDXWRUL]DomRGR*RYHUQDGRUGR3UHVLGHQWH
GD&kPDUD/HJLVODWLYDRXGR3UHVLGHQWHGR7ULEXQDOGH
†ž (PFDUiWHUH[FHSFLRQDOSRGHVHUDXWRUL]DGDDGLVSRVL &RQWDVSRGHVHUDXWRUL]DGRRDIDVWDPHQWRUHPXQHUD
omRIRUDGDVKLSyWHVHVSUHYLVWDVQHVWHDUWLJRSUHFHGLGD GRGRVHUYLGRUHVWiYHO
GHDXWRUL]DomRSRUDXWRULGDGHFRPSHWHQWHQRVPROGHV
GRDUW†ž DFUHVFHQWDGRR†žDRDUWSHODOHLFRPSOHPHQWDUQž , SDUDSDUWLFLSDUGHFRPSHWLomRGHVSRUWLYDQDFLRQDOSDUD
GHGRGIGH
DTXDOWHQKDVLGRSUHYLDPHQWHVHOHFLRQDGR
6HomR,, ,, TXDQGR FRQYRFDGR SDUD LQWHJUDU UHSUHVHQWDomRGHV
'R$IDVWDPHQWRSDUD([HUFtFLRGH0DQGDWR(OHWLYR SRUWLYDQDFLRQDOQR3DtVRXQRH[WHULRU
$UW $R VHUYLGRU HIHWLYR LQYHVWLGR HP PDQGDWRHOHWLYR †~QLFR 2DIDVWDPHQWRGHTXHWUDWDHVWHDUWLJRpSHORSUD]RGD
DSOLFDPVHDVVHJXLQWHVGLVSRVLo}HV FRPSHWLomRHJHUDFRPR~QLFDGHVSHVDSDUDRyUJmR
DXWDUTXLDRXIXQGDomRDSUHYLVWDQRFDSXW
, WUDWDQGRVHGHPDQGDWRIHGHUDOHVWDGXDORXGLVWULWDO
ILFDDIDVWDGRGRFDUJR 6HomR9
'R$IDVWDPHQWRSDUD3DUWLFLSDUGH3URJUDPD
,, LQYHVWLGR QR PDQGDWR GH SUHIHLWR ILFD DIDVWDGRGR GH3yV*UDGXDomR6WULFWR6HQVX
FDUJRVHQGROKHIDFXOWDGRRSWDUSHODUHPXQHUDomRGR
FDUJRHIHWLYR $UW 2VHUYLGRUHVWiYHOSRGHQRLQWHUHVVHGDDGPLQLVWUDomR
S~EOLFDHGHVGHTXHDSDUWLFLSDomRQmRSRVVDRFRUUHU
,,, LQYHVWLGRQRPDQGDWRGHYHUHDGRU VLPXOWDQHDPHQWHFRPRH[HUFtFLRGRFDUJRRXPHGLDQ
WHFRPSHQVDomRGHKRUiULRDIDVWDUVHGRH[HUFtFLRGR
D KDYHQGR FRPSDWLELOLGDGH GH KRUiULR SHUFHEH DV FDUJR HIHWLYR FRP D UHVSHFWLYD UHPXQHUDomR RX
YDQWDJHQVGHVHXFDUJRVHPSUHMXt]RGDUHPXQHUD VXEVtGLRSDUDSDUWLFLSDUGHSURJUDPDGHSyVJUDGXDo
omRGRFDUJRHOHWLYR mRVWULFWRVHQVXHPLQVWLWXLomRGHHQVLQRVXSHULRUQR
3DtVRXQRH[WHULRU
E QmRKDYHQGRFRPSDWLELOLGDGHGHKRUiULRpDIDVWDGR
GRFDUJRVHQGROKHIDFXOWDGRRSWDUSHODUHPXQHUD †ž 2WLWXODUGRyUJmRDXWDUTXLDRXIXQGDomRGHYHGHILQLU
omRGRFDUJRHIHWLYR RVSURJUDPDVGHFDSDFLWDomRHRVFULWpULRVSDUDSDUWLFL
SDomRHPSURJUDPDVGHSyVJUDGXDomRGHTXHWUDWD
†ž 2VHUYLGRUGHTXHWUDWDHVWHDUWLJRGXUDQWHRPDQGDWR HVWH DUWLJR FRP RX VHP DIDVWDPHQWR GR VHUYLGRU
HDWpXPDQRDSyVRVHXWpUPLQRQmRSRGHVHUUHPRYL REVHUYDGRRUHJXODPHQWR
GRRXUHGLVWULEXtGRGHRItFLRSDUDXQLGDGHDGPLQLVWUD
WLYDGLYHUVDGDTXHODGHRQGHVHDIDVWRXSDUDH[HUFHUR †ž 2 DIDVWDPHQWR SDUD UHDOL]DomR GHSURJUDPDVGH
PDQGDWR PHVWUDGRGRXWRUDGRRXSyVGRXWRUDGRVRPHQWHSRGH
VHUFRQFHGLGRDRVHUYLGRUHVWiYHOTXHHVWHMDHPHIHWLYR
†ž 2VHUYLGRUWHPJDUDQWLGRVWRGRVRVGLUHLWRVUHIHUHQWHV H[HUFtFLRQRUHVSHFWLYRyUJmRDXWDUTXLDRXIXQGDomR
DR H[HUFtFLR GR FDUJR HIHWLYR GXUDQWH R SHUtRGR HP KiSHORPHQRV
TXHHVWLYHUHPFDUJRHOHWLYR
, WUrVDQRVFRQVHFXWLYRVSDUDPHVWUDGR
6HomR,,,
'R$IDVWDPHQWRSDUD(VWXGRRX0LVVmRQR([WHULRU ,, TXDWUR DQRV FRQVHFXWLYRV SDUD GRXWRUDGRRX
SyVGRXWRUDGR
$UW 0HGLDQWHDXWRUL]DomRGR*RYHUQDGRUGR3UHVLGHQWH
GD&kPDUD/HJLVODWLYDRXGR3UHVLGHQWHGR7ULEXQDOGH †ž eYHGDGRDXWRUL]DUQRYRDIDVWDPHQWR
&RQWDVRVHUYLGRUHVWiYHOSRGHDXVHQWDUVHGR'LVWULWR
)HGHUDORXGR3DtVSDUD , SDUDFXUVRGRPHVPRQtYHO
6HFUHWDULDGH(VWDGRGH$ GPLQLVWUDomR3~EOLFD'LVWULWR)HGHUDO
 /HJLVODomR 

,, DQWHVGHGHFRUULGRSUD]RLJXDODRGHDIDVWDPHQWRMi †ž $FRQWDJHPGRWHPSRGHVHUYLoRpIHLWDHPGLDVTXH


FRQFHGLGR VmRFRQYHUWLGRVHPDQRVFRQVLGHUDGRRDQRFRPRGH
WUH]HQWRVHVHVVHQWDHFLQFRGLDV
†ž 2 VHUYLGRU EHQHILFLDGR SHORVDIDVWDPHQWRVSUHYLVWRV
QRV††žžHžWHPGH †ž eYHGDGRSURFHGHU

, DSUHVHQWDU R WtWXOR RX JUDX REWLGR FRP R FXUVRTXH , DRDUUHGRQGDPHQWRGHGLDVIDOWDQWHVSDUDFRPSOHPHQ
MXVWLILFRXVHXDIDVWDPHQWR WDUSHUtRGRUHVVDOYDGRVRVFDVRVSUHYLVWRVQHVWD/HL
&RPSOHPHQWDU
,, FRPSDUWLOKDUFRPRVGHPDLVVHUYLGRUHVGHVHXyUJmR
DXWDUTXLDRXIXQGDomRRVFRQKHFLPHQWRVDGTXLULGRV ,, DTXDOTXHUIRUPDGHFRQWDJHP GHWHPSRGHVHUYLoR
QRFXUVR ILFWtFLR

,,, SHUPDQHFHU QR HIHWLYR H[HUFtFLR GH VXDVDWULEXLo}HV ,,, jFRQWDJHPFXPXODWLYDGHWHPSRGHVHUYLoRSUHVWDGR


DSyVRVHXUHWRUQRSRUXPSHUtRGRLJXDODRGRDIDVWD FRQFRPLWDQWHPHQWH
PHQWRFRQFHGLGR
D HPGLIHUHQWHVFDUJRVGRVHUYLoRS~EOLFR
†ž 2VHUYLGRUEHQHILFLDGRSHORGLVSRVWRQHVWHDUWLJRWHP
GHUHVVDUFLUDGHVSHVDKDYLGDFRPVHXDIDVWDPHQWR E HP FDUJR GR VHUYLoR S~EOLFR H HP HPSUHJR QD
LQFOXtGRVDUHPXQHUDomRRXRVXEVtGLRHRVHQFDUJRV DGPLQLVWUDomRLQGLUHWDRXQDLQLFLDWLYDSULYDGD
VRFLDLVGDIRUPDVHJXLQWH
,9 jFRQWDJHPGRWHPSRGHVHUYLoRMiFRPSXWDGR
, SURSRUFLRQDO HP FDVR GH H[RQHUDomRGHPLVVmR
DSRVHQWDGRULD YROXQWiULD OLFHQoD SDUD WUDWDU GH D HPyUJmRRXHQWLGDGHHPTXHRVHUYLGRUDFXPXOH
LQWHUHVVHSDUWLFXODURXYDFkQFLDHPUD]mRGHSRVVHHP FDUJRS~EOLFR
RXWURFDUJRLQDFXPXOiYHODQWHVGHGHFRUULGRSHUtRGR
LJXDODRGRDIDVWDPHQWR E SDUD FRQFHVVmR GH DSRVHQWDGRULD HP TXDOTXHU
UHJLPHGHSUHYLGrQFLDVRFLDOSHORTXDORVHUYLGRU
,, LQWHJUDOHPFDVRGHQmRREWHQomRGRWtWXORRXJUDX UHFHEDSURYHQWRV
TXH MXVWLILFRX VHX DIDVWDPHQWR VDOYR QD KLSyWHVH
FRPSURYDGDGHIRUoDPDLRURXGHFDVRIRUWXLWR $UW 6DOYRGLVSRVLomROHJDOHPFRQWUiULRQmRVmRFRQWDGRV
FRPRWHPSRGHVHUYLoR
6HomR9,
'R$IDVWDPHQWRSDUD)UHTXrQFLDHP&XUVRGH)RUPDomR , DIDOWDLQMXVWLILFDGDDRVHUYLoRHDQmRFRPSHQVDGDQD
IRUPDGHVWD/HL&RPSOHPHQWDU
$UW 2 VHUYLGRU SRGH DIDVWDUVH GR FDUJR RFXSDGRSDUD
SDUWLFLSDUGHFXUVRGHIRUPDomRSUHYLVWRFRPRHWDSD ,, RSHUtRGRHPTXHRVHUYLGRUHVWLYHU
GHFRQFXUVRS~EOLFRGHVGHTXHKDMD
D OLFHQFLDGRRXDIDVWDGRVHPUHPXQHUDomR
, H[SUHVVDSUHYLVmRGRFXUVRQRHGLWDOGRFRQFXUVR E FXPSULQGRVDQomRGLVFLSOLQDUGHVXVSHQVmR

,, LQFRPSDWLELOLGDGHHQWUHRVKRUiULRVGDVDXODVHRVGD ,,, RSHUtRGRGHFRUULGRHQWUH


UHSDUWLomR
D D H[RQHUDomR H R H[HUFtFLR HP RXWUR FDUJR GH
†ž +DYHQGRLQFRPSDWLELOLGDGHHQWUHRVKRUiULRVGDVDXODV SURYLPHQWRHIHWLYR
HRVGDUHSDUWLomRRVHUYLGRUILFDDIDVWDGR
E DFRQFHVVmRGHDSRVHQWDGRULDYROXQWiULDHDUHYHU
, FRPUHPXQHUDomRRXVXEVtGLRQRVFDVRVGHFXUVRGH VmR
IRUPDomRSDUDFDUJRHIHWLYRGHyUJmRDXWDUTXLD RX
IXQGDomR GRV 3RGHUHV /HJLVODWLYR RX ([HFXWLYR GR F DGDWDGHSXEOLFDomRGRDWRGHUHYHUVmRUHLQWHJUD
'LVWULWR)HGHUDO omRUHFRQGXomRRXDSURYHLWDPHQWRHRUHWRUQRDR
H[HUFtFLRGRFDUJR
,, VHP UHPXQHUDomR QRV FDVRV GH FXUVR GHIRUPDomR
SDUDFDUJRQmRFRQWHPSODGRQRLQFLVR,GHVWHSDUiJUD $UW 6mRFRQVLGHUDGRVFRPRHIHWLYRH[HUFtFLR
IR
, DVIpULDV
†ž 2VHUYLGRUSRGHRSWDUSRUHYHQWXDODMXGDILQDQFHLUD
SDJDHPUD]mRGRFXUVRGHIRUPDomRYHGDGDDSHUFHS ,, DVDXVrQFLDVSUHYLVWDVQRDUW
omRGDUHPXQHUDomRSUHYLVWDQR†ž,
,,, DOLFHQoD
&DStWXOR9
'27(032'(6(59,d2('27(032'(&2175,%8,d®2 D PDWHUQLGDGHRXSDWHUQLGDGH
E PpGLFDRXRGRQWROyJLFD
6HomR, F SUrPLRSRUDVVLGXLGDGH
'R7HPSRGH6HUYLoR G SDUDRVHUYLoRPLOLWDUREULJDWyULR

$UW 6DOYRGLVSRVLomROHJDOHPFRQWUiULRp FRQWDGRSDUD ,9 RDERQRGHSRQWR


WRGRVRVHIHLWRVRWHPSRGHVHUYLoRS~EOLFRUHPXQHUD
GR SUHVWDGR D yUJmR DXWDUTXLD RX IXQGDomR GRV 9 RDIDVWDPHQWRSDUD
3RGHUHV([HFXWLYRH/HJLVODWLYRGR'LVWULWR)HGHUDO
  /HJLVODomR 6HFUHWDULDGH(VWDGRGH$GPLQLVWUDomR3~EOLFD'LVWULWR)HGHUDO


D H[HUFtFLRHPRXWURyUJmRRXHQWLGDGHLQFOXVLYHHP †ž 2 GLUHLWR GH SHWLomR FRPSUHHQGH DDSUHVHQWDomRGH


FDUJR HP FRPLVVmR RX IXQomR GH FRQILDQoD GH UHTXHULPHQWRSHGLGRGHUHFRQVLGHUDomRUHFXUVRRX
TXDOTXHU GRV 3RGHUHV GR 'LVWULWR )HGHUDO 8QLmR TXDOTXHU RXWUD PDQLIHVWDomR QHFHVViULD j GHIHVD GH
(VWDGRRX0XQLFtSLR GLUHLWRRXLQWHUHVVHOHJtWLPRRXjDPSODGHIHVDHDR
FRQWUDGLWyULRGRSUySULRVHUYLGRURXGHSHVVRDGDVXD
E HVWXGRRXPLVVmRQRH[WHULRUFRPUHPXQHUDomR IDPtOLD

F SDUWLFLSDomRHPFRPSHWLomRGHVSRUWLYD †ž 3DUDRH[HUFtFLRGRGLUHLWRGHSHWLomRpDVVHJXUDGD

G SDUWLFLSDomRHPSURJUDPDGHWUHLQDPHQWRUHJXODU , YLVWDGRSURFHVVRRXGRGRFXPHQWRQDUHSDUWLomRDR


PHQWHLQVWLWXtGRRXHPSURJUDPDGHSyVJUDGXDomR VHUYLGRURXDSURFXUDGRUSRUHOHFRQVWLWXtGR
VWULFWRVHQVX
,, FySLDGHGRFXPHQWRRXGHSHoDSURFHVVXDOREVHUYDGDV
H 9(7$'2  DVQRUPDVGDTXHOHVFODVVLILFDGRVFRPJUDXGHVLJLOR

9, RUHYRJDGRSHOD/HL&RPSOHPHQWDUQžGH †ž $FySLDGHGRFXPHQWRRXGHSHoDSURFHVVXDOSRGHVHU


IRUQHFLGDHPPHLRHOHWU{QLFR
9,, RSHUtRGRHQWUHDGHPLVVmRHDGDWDGHSXEOLFDomRGR
DWRGHUHLQWHJUDomR $UW 2 UHTXHULPHQWR R SHGLGR GH UHFRQVLGHUDomR RXR
UHFXUVR p GLULJLGR j DXWRULGDGH FRPSHWHQWH SDUD
9,,, DSDUWLFLSDomRHPWULEXQDOGRM~ULRXRXWURVVHUYLoRV GHFLGLOR
REULJDWyULRVSRUOHL
†~QLFR $DXWRULGDGHFRPSHWHQWHGHVGHTXHIXQGDPHQWHVXD
†~QLFR $OLFHQoDSDUDRGHVHPSHQKRGHPDQGDWRFODVVLVWDRX GHFLVmRSRGHGDUHIHLWRVXVSHQVLYRDRUHFXUVR
RDIDVWDPHQWRSDUDH[HUFHUPDQGDWRHOHWLYRIHGHUDO
HVWDGXDOGLVWULWDORXPXQLFLSDOVmRFRQVLGHUDGRVFRPR $UW &DEH SHGLGR GH UHFRQVLGHUDomR j DXWRULGDGHTXH
HIHWLYRH[HUFtFLR KRXYHUH[SHGLGRRDWRRXSURIHULGRDSULPHLUDGHFLVmR
QmRSRGHQGRVHUUHQRYDGR
$UW &RQWDVHSDUDHIHLWRGHGLVSRQLELOLGDGH
$UW &DEHUHFXUVR
, RWHPSRGHVHUYLoRSUHVWDGRD0XQLFtSLR(VWDGRRX
8QLmR LQFOXVLYH R SUHVWDGR DR 7ULEXQDO GH -XVWLoD , GR LQGHIHULPHQWR GR UHTXHULPHQWR GHVGH TXHQmR
0LQLVWpULR3~EOLFRRX'HIHQVRULD3~EOLFDGR'LVWULWR WHQKDVLGRLQWHUSRVWRSHGLGRGHUHFRQVLGHUDomR
)HGHUDOH7HUULWyULRV
,, GDGHFLVmRVREUHSHGLGRGHUHFRQVLGHUDomRRXGHRXWUR
,, RWHPSRGHVHUYLoRHPDWLYLGDGHSULYDGDYLQFXODGDDR UHFXUVRLQWHUSRVWR
UHJLPHJHUDOGHSUHYLGrQFLDVRFLDOLQFOXVLYHRSUHVWD
GR j HPSUHVD S~EOLFD RX j VRFLHGDGH GH HFRQRPLD †~QLFR 2 UHFXUVR p GLULJLGR jDXWRULGDGHLPHGLDWDPHQWH
PLVWDGHTXDOTXHUHQWHGDIHGHUDomR VXSHULRU j TXH WLYHU H[SHGLGR R DWR RX SURIHULGR D
GHFLVmRHVXFHVVLYDPHQWHHPHVFDODDVFHQGHQWHjV
,,, DOLFHQoDUHPXQHUDGDSRUPRWLYRGHGRHQoDHPSHVVRD GHPDLVDXWRULGDGHV
GDIDPtOLDGRVHUYLGRU
$UW 2SUD]RSDUDLQWHUSRVLomRGHSHGLGRGHUHFRQVLGHUDomR
,9 DOLFHQoDUHPXQHUDGDSDUDDWLYLGDGHSROtWLFD RXGHUHFXUVRpGHWULQWDGLDVFRQWDGRVGDSXEOLFDomR
RXGDFLrQFLDSHORLQWHUHVVDGRGDGHFLVmRLPSXJQDGD
9 RWHPSRGHPDQGDWRHOHWLYRIHGHUDOHVWDGXDOPXQLFL
SDORXGLVWULWDODQWHULRUDRLQJUHVVRQRVHUYLoRS~EOLFR $UW 2 UHTXHULPHQWR R SHGLGR GH UHFRQVLGHUDomR RXR
GR'LVWULWR)HGHUDO UHFXUVR GH TXH WUDWDP RV DUWV  D  GHYH VHU
GHVSDFKDGRQRSUD]RGHFLQFRGLDVHGHFLGLGRGHQWUR
9, RDIDVWDPHQWRSDUDIUHTXrQFLDHPFXUVRGHIRUPDomR GHWULQWDGLDVFRQWDGRVGDGDWDGHVHXSURWRFROR
TXDQGRUHPXQHUDGR
$UW (PFDVRGHSURYLPHQWRGRSHGLGRGHUHFRQVLGHUDomR
6HomR,, RXGRUHFXUVRRVHIHLWRVGDGHFLVmRUHWURDJHPjGDWD
'R7HPSRGH&RQWULEXLomR GRDWRLPSXJQDGR

$UW )D]VHQDIRUPDGDOHJLVODomRSUHYLGHQFLiULDDFRQWD $UW 2GLUHLWRGHUHTXHUHUSUHVFUHYH


JHPGRWHPSR
, HP FLQFR DQRV TXDQWR DRV DWRV GH GHPLVVmRGH
, GHFRQWULEXLomR FDVVDomRGHDSRVHQWDGRULDRXGLVSRQLELOLGDGHRXGH
,, QRVHUYLoRS~EOLFR GHVWLWXLomRGRFDUJRHPFRPLVVmR
,,, GHVHUYLoRQRFDUJRHIHWLYR
,9 GHVHUYLoRQDFDUUHLUD ,, HP FLQFR DQRV TXDQWR DR LQWHUHVVH SDWULPRQLDORX
FUpGLWRVUHVXOWDQWHVGDVUHODo}HVGHWUDEDOKR
&DStWXOR9,
'2',5(,72'(3(7,d®2 ,,, HPFHQWRHYLQWHGLDVQRVGHPDLVFDVRVVDOYRGLVSRVL
omROHJDOHPFRQWUiULR
$UW eDVVHJXUDGRDRVHUYLGRURGLUHLWRGHSHWLomRMXQWRDRV
yUJmRVS~EOLFRVRQGHH[HUFHVXDVDWULEXLo}HVRXMXQWR †~QLFR 2SUD]RGHSUHVFULomRpFRQWDGRGDGDWD
jTXHOHVHPTXHWHQKDLQWHUHVVHIXQFLRQDO
, GDSXEOLFDomRGRDWRLPSXJQDGR
6HFUHWDULDGH(VWDGRGH$ GPLQLVWUDomR3~EOLFD'LVWULWR)HGHUDO
 /HJLVODomR 

,, GD FLrQFLD SHOR LQWHUHVVDGR TXDQGR R DWR QmRIRU ; JXDUGDUVLJLORVREUHDVVXQWRGDUHSDUWLomR
SXEOLFDGR
;, VHUOHDOjVLQVWLWXLo}HVDTXHVHUYLU
,,, GRWUkQVLWRHPMXOJDGRGDGHFLVmRMXGLFLDO
;,, VHUDVVtGXRHSRQWXDODRVHUYLoR
$UW 2 SHGLGR GH UHFRQVLGHUDomR H R UHFXUVRTXDQGR
FDEtYHLVLQWHUURPSHPDSUHVFULomR ;,,, PDQWHUFRQGXWDFRPSDWtYHOFRPDPRUDOLGDGHDGPL
QLVWUDWLYD
$UW $ SUHVFULomR p GH RUGHP S~EOLFD QmR SRGHQGRVHU
UHOHYDGDSHODDGPLQLVWUDomRS~EOLFD ;,9 GHFODUDUVHVXVSHLWRRXLPSHGLGRQDVKLSyWHVHVSUHYLV
WDVHPOHLRXUHJXODPHQWR
$UW $ DGPLQLVWUDomR S~EOLFD GHYH UHYHU VHXV DWRVD
TXDOTXHU WHPSR TXDQGR HLYDGRV GH YtFLRV TXH RV ;9 WUDWDUDVSHVVRDVFRPFLYLOLGDGH
WRUQHPLOHJDLVDVVHJXUDGRRFRQWUDGLWyULRHDDPSOD
GHIHVD ;9, DWHQGHUFRPSUHVWH]D
†ž 2VDWRVTXHDSUHVHQWDUHPGHIHLWRVVDQiYHLVSRGHPVHU D R S~EOLFR HP JHUDO SUHVWDQGR DV LQIRUPDo}HV
FRQYDOLGDGRV SHOD SUySULD DGPLQLVWUDomR S~EOLFD UHTXHULGDVUHVVDOYDGDVDVSURWHJLGDVSRUVLJLOR
GHVGHTXHQmRDFDUUHWHPOHVmRDRLQWHUHVVHS~EOLFR
QHPSUHMXt]RDWHUFHLURV
E RVUHTXHULPHQWRVGHH[SHGLomRGHFHUWLG}HVSDUD
GHIHVDGHGLUHLWRRXHVFODUHFLPHQWRGHVLWXDo}HVGH
†ž 2 GLUHLWR GH D DGPLQLVWUDomR S~EOLFD DQXODURVDWRV
LQWHUHVVHSHVVRDO
DGPLQLVWUDWLYRV GH TXH GHFRUUDP HIHLWRV IDYRUiYHLV
SDUDRVHUYLGRUGHFDLHPFLQFRDQRVFRQWDGRVGDGDWD
F DV UHTXLVLo}HV SDUD D GHIHVD GD DGPLQLVWUDomR
HPTXHIRUDPSUDWLFDGRVVDOYRHPFDVRGHFRPSURYD
S~EOLFD
GDPiIp
7tWXOR9,
†ž 1RFDVRGHHIHLWRVSDWULPRQLDLVFRQWtQXRVRSUD]RGH '25(*,0(',6&,3/,1$5
GHFDGrQFLD p FRQWDGR GD SHUFHSomR GR SULPHLUR
SDJDPHQWR &DStWXOR,
'$65(63216$%,/,'$'(6
†ž 1R FDVR GH DWR VXMHLWR D UHJLVWUR SHOR7ULEXQDOGH
&RQWDVGR'LVWULWR)HGHUDORSUD]RGHTXHWUDWDR†ž $UW 2VHUYLGRUUHVSRQGHSHQDOFLYLOHDGPLQLVWUDWLYDPHQWH
FRPHoD D VHU FRQWDGR GD GDWD HP TXH R SURFHVVR SHORH[HUFtFLRLUUHJXODUGHVXDVDWULEXLo}HV
UHVSHFWLYROKHIRLHQFDPLQKDGR
†ž $V VDQo}HV FLYLV SHQDLV HDGPLQLVWUDWLYDVSRGHP
$UW 6mR IDWDLV H LPSURUURJiYHLV RV SUD]RVHVWDEHOHFLGRV FXPXODUVHVHQGRLQGHSHQGHQWHVHQWUHVL
QHVWH&DStWXORVDOYRSRUPRWLYRGHIRUoDPDLRU
†ž $ UHVSRQVDELOLGDGH DGPLQLVWUDWLYD GRVHUYLGRUp
7tWXOR9 DIDVWDGD QR FDVR GH DEVROYLomR SHQDO TXH QHJXH D
&DStWXORÔQLFR H[LVWrQFLDGRIDWRRXVXDDXWRULDFRPGHFLVmRWUDQVLWD
'26'(9(5(6 GDHPMXOJDGR

$UW 6mRGHYHUHVGRVHUYLGRU †ž $UHVSRQVDELOLGDGHDGPLQLVWUDWLYDSHUDQWHDDGPLQLV


WUDomRS~EOLFDQmRH[FOXLDFRPSHWrQFLDGR7ULEXQDO
, H[HUFHUFRP]HORHGHGLFDomRVXDVDWULEXLo}HV GH&RQWDVSUHYLVWDQD/HL2UJkQLFDGR'LVWULWR)HGHUDO

,, PDQWHUVHDWXDOL]DGRQRVFRQKHFLPHQWRVH[LJLGRVSDUD $UW $UHVSRQVDELOLGDGHSHQDODEUDQJHFULPHVHFRQWUDYHQ


RH[HUFtFLRGHVXDVDWULEXLo}HV o}HVLPSXWDGRVDRVHUYLGRUQHVVDTXDOLGDGH

,,, DJLUFRPSHUtFLDSUXGrQFLDHGLOLJrQFLDQRH[HUFtFLR $UW $ UHVSRQVDELOLGDGH FLYLO GHFRUUH GH DWR RPLVVLYRRX


GHVXDVDWULEXLo}HV FRPLVVLYRGRORVRRXFXOSRVRTXHUHVXOWHHPSUHMXt]R
DRHUiULRRXDWHUFHLUR
,9 DWXDOL]DUTXDQGRVROLFLWDGRVHXVGDGRVFDGDVWUDLV
†ž $ LQGHQL]DomR GH SUHMXt]R GRORVDPHQWHFDXVDGRDR
9 REVHUYDUDVQRUPDVOHJDLVHUHJXODPHQWDUHVQRH[HUFt HUiULRVRPHQWHSRGHVHUOLTXLGDGDQDIRUPDSUHYLVWD
FLRGHVXDVDWULEXLo}HV QR DUW HVHJXLQWHVQDIDOWDGHRXWURVEHQVTXH
DVVHJXUHPDH[HFXomRGRGpELWRSHODYLDMXGLFLDO
9, FXPSULUDVRUGHQVVXSHULRUHVH[FHWRTXDQGRPDQLIHV
WDPHQWHLOHJDLV †ž 7UDWDQGRVHGHGDQRFDXVDGRDWHUFHLURVUHVSRQGHR
VHUYLGRUSHUDQWHD)D]HQGD3~EOLFDHPDomRUHJUHVVL
9,, OHYDU DR FRQKHFLPHQWR GD DXWRULGDGH VXSHULRUDV YD
IDOKDV YXOQHUDELOLGDGHV H DV LUUHJXODULGDGHV GH TXH
WLYHUFLrQFLDHPUD]mRGRFDUJRS~EOLFRRXIXQomRGH
†ž $REULJDomRGHUHSDUDURGDQRHVWHQGHVHDRVVXFHVVR
FRQILDQoD
UHVHFRQWUDHOHVWHPGHVHUH[HFXWDGDQDIRUPDGD
OHLFLYLO
9,,, UHSUHVHQWDUFRQWUDLOHJDOLGDGHRPLVVmRRXDEXVRGH
SRGHU
$UW $ UHVSRQVDELOLGDGH SHUDQWH R 7ULEXQDO GH&RQWDV
GHFRUUH GH DWRV VXMHLWRV DR FRQWUROH H[WHUQR QRV
,; ]HODUSHODHFRQRPLDGRPDWHULDOHSHODFRQVHUYDomRGR
SDWULP{QLRS~EOLFR WHUPRVGD/HL2UJkQLFDGR'LVWULWR)HGHUDO
  /HJLVODomR 6HFUHWDULDGH(VWDGRGH$GPLQLVWUDomR3~EOLFD'LVWULWR)HGHUDO


$UW $SHUGDGRFDUJRS~EOLFRRXDFDVVDomRGHDSRVHQWDGR , GHVFXPSULUGHYHUIXQFLRQDORXGHFLV}HVDGPLQLVWUDWL


ULD GHWHUPLQDGD HP GHFLVmR MXGLFLDO WUDQVLWDGD HP YDVHPDQDGDVGRVyUJmRVFRPSHWHQWHV
MXOJDGRGLVSHQVDDLQVWDXUDomRGHSURFHVVRGLVFLSOLQDU
HGHYHVHUGHFODUDGDSHODDXWRULGDGHFRPSHWHQWHSDUD ,, UHWLUDU VHP SUpYLD DQXrQFLD GD FKHILDLPHGLDWD
ID]HUDQRPHDomR TXDOTXHUGRFXPHQWRRXREMHWRGDUHSDUWLomR

$UW $UHVSRQVDELOLGDGHDGPLQLVWUDWLYDDSXUDGDQDIRUPD ,,, GHL[DUGHSUDWLFDUDWRQHFHVViULRjDSXUDomRGHLQIUD


GHVWD/HL&RPSOHPHQWDUUHVXOWDGHLQIUDomRGLVFLSOL omRGLVFLSOLQDUUHWDUGDULQGHYLGDPHQWHDVXDSUiWLFD
QDUFRPHWLGDSRUVHUYLGRUQRH[HUFtFLRGHVXDVDWULEXL RXGDUFDXVDjSUHVFULomRHPSURFHVVRGLVFLSOLQDU
o}HVHPUD]mRGHODVRXFRPHODVLQFRPSDWtYHLV
,9 UHFXVDUVHTXDQGRVROLFLWDGRSRUDXWRULGDGHFRPSH
†ž $UHVSRQVDELOLGDGHDGPLQLVWUDWLYDGRVHUYLGRUREVHU WHQWHDSUHVWDULQIRUPDomRGHTXHWHQKDFRQKHFLPHQ
YDGRRSUD]RSUHVFULFLRQDOSHUPDQHFHHPUHODomRDRV WRHPUD]mRGRH[HUFtFLRGHVXDVDWULEXLo}HV
DWRVSUDWLFDGRVQRH[HUFtFLRGRFDUJR
9 UHFXVDUVHLQMXVWLILFDGDPHQWHDLQWHJUDUFRPLVVmRRX
, DSyVDH[RQHUDomR JUXSRGHWUDEDOKRRXGHL[DUGHDWHQGHUGHVLJQDomR
SDUD FRPSRU FRPLVVmR JUXSR GH WUDEDOKR RX SDUD
,, DSyVDDSRVHQWDGRULD DWXDUFRPRSHULWRRXDVVLVWHQWHWpFQLFRHPSURFHVVR
DGPLQLVWUDWLYRRXMXGLFLDO
,,, DSyV D YDFkQFLD HP UD]mR GH SRVVH HP RXWURFDUJR
LQDFXPXOiYHO 9, UHFXVDUIpDGRFXPHQWRS~EOLFR

,9 GXUDQWHDVOLFHQoDVDIDVWDPHQWRVHGHPDLVDXVrQFLDV 9,, QHJDUVH D SDUWLFLSDU GH SURJUDPD GHWUHLQDPHQWR


SUHYLVWRVQHVWD/HL&RPSOHPHQWDU H[LJLGR GH WRGRV RV VHUYLGRUHV GD PHVPD VLWXDomR
IXQFLRQDO
†ž $DSOLFDomRGDVDQomRFRPLQDGDjLQIUDomRGLVFLSOLQDU
GHFRUUH GD UHVSRQVDELOLGDGH DGPLQLVWUDWLYD VHP 9,,, QmR FRPSDUHFHU TXDQGR FRQYRFDGR D LQVSHomRRX
SUHMXt]R SHUtFLDPpGLFD

, GHHYHQWXDODomRFLYLORXSHQDO ,; RSRU UHVLVWrQFLD LQMXVWLILFDGD RX UHWDUGDUUHLWHUDGD


PHQWHHVHPMXVWDFDXVD
,, GRUHVVDUFLPHQWRDRHUiULRGRVYDORUHVFRUUHVSRQGHQ
WHVDRVGDQRVHDRVSUHMXt]RVFDXVDGRVjDGPLQLVWUDomR D RDQGDPHQWRGHGRFXPHQWRSURFHVVRRXH[HFXomR
S~EOLFD GHVHUYLoR

,,, GD GHYROXomR DR HUiULR GR EHP RX GR YDORUS~EOLFR E DSUiWLFDGHDWRVSUHYLVWRVHPVXDVDWULEXLo}HV
GHVYLDGRQDVPHVPDVFRQGLo}HVHPTXHVHHQFRQWUD
YDPTXDQGRGDRFRUUrQFLDGRIDWRFRPDFRQVHTXHQWH ; FRPHWHUDVHUYLGRUDWULEXLo}HVHVWUDQKDVDRFDUJRTXH
LQGHQL]DomRSURSRUFLRQDOjGHSUHFLDomR RFXSD H[FHWR HP VLWXDo}HV GH HPHUJrQFLD H HP
FDUiWHUWUDQVLWyULR
&DStWXOR,,
'$6,1)5$d¯(6',6&,3/,1$5(6 ;, PDQWHUVREVXDFKHILDLPHGLDWDHPFDUJRHPFRPLVVmR
RXIXQomRGHFRQILDQoDRF{QMXJHRFRPSDQKHLURRX
6HomR, SDUHQWHSRUFRQVDQJXLQLGDGHDWpRWHUFHLURJUDXRX
'DV'LVSRVLo}HV*HUDLV SRUDILQLGDGH

$UW $ LQIUDomR GLVFLSOLQDU GHFRUUH GH DWR RPLVVLYRRX ;,, SURPRYHU PDQLIHVWDomR GH DSUHoR RX GHVDSUHoRQR
FRPLVVLYR SUDWLFDGR FRP GROR RX FXOSD H VXMHLWD R UHFLQWRGDUHSDUWLomR
VHUYLGRUjVVDQo}HVSUHYLVWDVQHVWD/HL&RPSOHPHQWDU
;,,, SHUWXUEDUVHPMXVWDFDXVDDRUGHPHDVHUHQLGDGHQR
$UW $VLQIUDo}HVGLVFLSOLQDUHVFODVVLILFDPVHSDUDHIHLWRV UHFLQWRGDUHSDUWLomR
GHFRPLQDomRGDVDQomRHPOHYHVPpGLDVHJUDYHV
;,9 DFHVVDU DUPD]HQDU RX WUDQVIHULULQWHQFLRQDOPHQWH
†~QLFR $VLQIUDo}HVPpGLDVHDVLQIUDo}HVJUDYHVVmRVXEFODVVL FRPUHFXUVRVHOHWU{QLFRVGDDGPLQLVWUDomRS~EOLFDRX
ILFDGDVHPJUXSRVQDIRUPDGHVWD/HL&RPSOHPHQWDU SRVWRV j VXD GLVSRVLomR LQIRUPDo}HV GH FRQWH~GR
SRUQRJUiILFRRXHUyWLFRRXTXHLQFHQWLYHPDYLROrQFLD
$UW 3DUD HIHLWRV GHVWD /HL &RPSOHPHQWDUFRQVLGHUDVH RXDGLVFULPLQDomRHPTXDOTXHUGHVXDVIRUPDV
UHLQFLGrQFLDRFRPHWLPHQWRGHQRYDLQIUDomRGLVFLSOL
QDUGRPHVPRJUXSRRXFODVVHGHLQIUDomRGLVFLSOLQDU ;9 XVDULQGHYLGDPHQWHDLGHQWLILFDomRIXQFLRQDORXRXWUR
DQWHULRUPHQWH FRPHWLGD DLQGD TXH XPD H RXWUD GRFXPHQWR TXH R YLQFXOH FRP R FDUJR S~EOLFR RX
SRVVXDPFDUDFWHUtVWLFDVIiWLFDVGLYHUVDV IXQomRGHFRQILDQoDHPLOHJtWLPREHQHItFLRSUySULRRX
GHWHUFHLUR
†~QLFR (QWHQGHVH SRU LQIUDomRGLVFLSOLQDUDQWHULRUPHQWH 6HomR,,,
FRPHWLGDDTXHODMiSXQLGDQDIRUPDGHVWD/HL&RPSOH 'DV,QIUDo}HV0pGLDV
PHQWDU
6HomR,, $UW 6mRLQIUDo}HVPpGLDVGRJUXSR,
'DV,QIUDo}HV/HYHV
, FRPHWHUDSHVVRDHVWUDQKDjUHSDUWLomRIRUDGRVFDVRV
$UW 6mRLQIUDo}HVOHYHV SUHYLVWRVHPOHLRGHVHPSHQKRGHDWULEXLomRTXHVHMD
GHVXDUHVSRQVDELOLGDGHRXGHVHXVXERUGLQDGR
6HFUHWDULDGH(VWDGRGH$ GPLQLVWUDomR3~EOLFD'LVWULWR)HGHUDO
 /HJLVODomR 

,, DXVHQWDUVH GR VHUYLoR FRP IUHTXrQFLD GXUDQWHR D DEDQGRQRGHFDUJR


H[SHGLHQWHHVHPSUpYLDDXWRUL]DomRGDFKHILDLPHGLD E LQDVVLGXLGDGHKDELWXDO
WD
,, DFXPXODU LOHJDOPHQWH FDUJRV HPSUHJRVIXQo}HV
,,, H[HUFHUDWLYLGDGHSULYDGDLQFRPSDWtYHOFRPRKRUiULR S~EOLFDVRXSURYHQWRVGHDSRVHQWDGRULDVDOYRVHIRU
GRVHUYLoR IHLWDDRSomRQDIRUPDGHVWD/HL&RPSOHPHQWDU

,9 SUDWLFDUDWRLQFRPSDWtYHOFRPDPRUDOLGDGHDGPLQLV ,,, SURFHGHUGHIRUPDGHVLGLRVDLQFRUUHQGRUHSHWLGDPHQ


WUDWLYD WHHPGHVFXPSULPHQWRGHYiULRVGHYHUHVHDWULEXLo}HV
IXQFLRQDLV
9 SUDWLFDURFRPpUFLRRXDXVXUDQDUHSDUWLomR
,9 DFRPHWHUVHGHLQFRQWLQrQFLDS~EOLFDRXWHUFRQGXWD
9, GLVFULPLQDUTXDOTXHUSHVVRDQRUHFLQWRGDUHSDUWLomR HVFDQGDORVD QD UHSDUWLomR TXH SHUWXUEH D RUGHP R
FRP D ILQDOLGDGH GH H[S{OD D VLWXDomR KXPLOKDQWH DQGDPHQWRGRVWUDEDOKRVRXFDXVHGDQRjLPDJHPGD
YH[DWyULDDQJXVWLDQWHRXFRQVWUDQJHGRUDHPUHODomR DGPLQLVWUDomRS~EOLFD
DQDVFLPHQWRLGDGHHWQLDUDoDFRUVH[RHVWDGRFLYLO
WUDEDOKRUXUDORXXUEDQRUHOLJLmRFRQYLFo}HVSROtWLFDV 9 FRPHWHULQVXERUGLQDomRJUDYHHPVHUYLoRVXEYHUWHQGR
RX ILORVyILFDV RULHQWDomR VH[XDO GHILFLrQFLD ItVLFD DRUGHPKLHUiUTXLFDGHIRUPDRVWHQVLYD
LPXQROyJLFD VHQVRULDO RX PHQWDO SRU WHU FXPSULGR
SHQDRXSRUTXDOTXHUSDUWLFXODULGDGHRXFRQGLomR
9, GLVSHQVDUOLFLWDomRSDUDFRQWUDWDUSHVVRDMXUtGLFDTXH
WHQKDFRPRSURSULHWiULRVyFLRRXDGPLQLVWUDGRU
$UW 6mRLQIUDo}HVPpGLDVGRJUXSR,,
D SHVVRDGHVXDIDPtOLDRXRXWURSDUHQWHSRUFRQVDQ
, RIHQGHU ILVLFDPHQWH D RXWUHP HP VHUYLoR VDOYRHP
JXLQLGDGHDWpRWHUFHLURJUDXRXSRUDILQLGDGH
UHVSRVWD D LQMXVWD DJUHVVmR RX HP OHJtWLPD GHIHVD
SUySULDRXGHRXWUHP
E SHVVRDGDIDPtOLDGHVXDFKHILDPHGLDWDRXLPHGLD
,, SUDWLFDUDWRGHDVVpGLRVH[XDORXPRUDO WDRXRXWURSDUHQWHGHODSRUFRQVDQJXLQLGDGHDWp
RWHUFHLURJUDXRXSRUDILQLGDGH
,,, FRDJLURXDOLFLDUVXERUGLQDGRQRVHQWLGRGHILOLDUVHD
DVVRFLDomR VLQGLFDWR SDUWLGR SROtWLFR RX TXDOTXHU 9,, GLVSHQVDU OLFLWDomR SDUD FRQWUDWDU SHVVRD ItVLFDGH
RXWUDHVSpFLHGHDJUHPLDomR IDPtOLDRXSDUHQWHPHQFLRQDGRQRLQFLVR9,DHE

,9 H[HUFHUDWLYLGDGHSULYDGDLQFRPSDWtYHOFRPRH[HUFtFLR 9,,, DFHLWDU FRPLVVmR HPSUHJR RX SHQVmR GHHVWDGR


GRFDUJRS~EOLFRRXGDIXQomRGHFRQILDQoD HVWUDQJHLUR

9 XVDU UHFXUVRV FRPSXWDFLRQDLV GDDGPLQLVWUDomR ,; H[HUFHURFRPpUFLRH[FHWRQDTXDOLGDGHGHDFLRQLVWD


S~EOLFDSDUDLQWHQFLRQDOPHQWH FRWLVWDRXFRPDQGLWiULR

D YLRODUVLVWHPDVRXH[HUFHURXWUDVDWLYLGDGHVSUHMXGL ; SDUWLFLSDUGHJHUrQFLDRXDGPLQLVWUDomRGHVRFLHGDGH


FLDLVDVLWHVS~EOLFRVRXSULYDGRV RXHPSUHVDSULYDGDSHUVRQLILFDGDRXQmRSHUVRQLILFD
GDVDOYR
E GLVVHPLQDUYtUXVFDYDORVGHWUyLDVS\ZDUHHRXWURV
PDOHVSUDJDVHSURJUDPDVLQGHVHMiYHLV D QRVFDVRVSUHYLVWRVQHVWD/HL&RPSOHPHQWDU

F GLVSRQLELOL]DUHPVLWHVGRVHUYLoRS~EOLFRSURSD E QRVSHUtRGRVGHOLFHQoDRXDIDVWDPHQWRGRFDUJR
JDQGDRXSXEOLFLGDGHGHFRQWH~GRSULYDGRLQIRU VHPUHPXQHUDomRGHVGHTXHQmRKDMDSURLELomRHP
PDo}HVHRXWURVFRQWH~GRVLQFRPSDWtYHLVFRPRV VHQWLGRFRQWUiULRQHPLQFRPSDWLELOLGDGH
IXQGDPHQWRV H RV SULQFtSLRV GD DGPLQLVWUDomR
S~EOLFD F HPLQVWLWXLo}HVRXHQWLGDGHVEHQHILFHQWHVILODQWUy
SLFDV GH FDUiWHU VRFLDO H KXPDQLWiULR H VHP ILQV
G UHSDVVDUGDGRVFDGDVWUDLVHLQIRUPDo}HVGHVHUYLGR OXFUDWLYRVTXDQGRFRPSDWtYHLVFRP DMRUQDGDGH
UHV S~EOLFRV RX GD UHSDUWLomR SDUD WHUFHLURV VHP WUDEDOKR
DXWRUL]DomR
†~QLFR $UHDVVXQomRGDVDWULEXLo}HVGHSRLVGHFRQVXPDGRR
9, SHUPLWLURXIDFLOLWDURDFHVVRGHSHVVRDQmRDXWRUL]DGD DEDQGRQR GH FDUJR QmR DIDVWD D UHVSRQVDELOLGDGH
PHGLDQWHDWULEXLomRIRUQHFLPHQWRRXHPSUpVWLPRGH DGPLQLVWUDWLYD QHP FDUDFWHUL]D SHUGmR WiFLWR GD
VHQKDRXTXDOTXHURXWURPHLR DGPLQLVWUDomRS~EOLFDUHVVDOYDGDDSUHVFULomR

D DUHFXUVRVFRPSXWDFLRQDLVVLVWHPDVGHLQIRUPDo}HV $UW 6mRLQIUDo}HVJUDYHVGRJUXSR,,


RXEDQFRGHGDGRVGDDGPLQLVWUDomRS~EOLFD
, SUDWLFDUGRORVDPHQWHDWRGHILQLGRHPOHLFRPR
E DORFDLVGHDFHVVRUHVWULWR
D FULPHFRQWUDDDGPLQLVWUDomRS~EOLFD
6HomR,9 E LPSURELGDGHDGPLQLVWUDWLYD
'DV,QIUDo}HV*UDYHV
,, XVDU FRQKHFLPHQWRV H LQIRUPDo}HV DGTXLULGRVQR
$UW 6mRLQIUDo}HVJUDYHVGRJUXSR, H[HUFtFLR GH VXDV DWULEXLo}HV SDUD YLRODU RX WRUQDU
YXOQHUiYHO D VHJXUDQoD RV VLVWHPDV GH LQIRUPiWLFD
, LQFRUUHUQDKLSyWHVHGH VLWHV RX TXDOTXHU RXWUD URWLQD RX HTXLSDPHQWR GD
UHSDUWLomR
  /HJLVODomR 6HFUHWDULDGH(VWDGRGH$GPLQLVWUDomR3~EOLFD'LVWULWR)HGHUDO


,,, H[LJLUVROLFLWDUUHFHEHURXDFHLWDUSURSLQDJUDWLILFD 9,, RIDWRGHRVHUYLGRUWHU


omRFRPLVVmRSUHVHQWHRXDXIHULUYDQWDJHPLQGHYLGD
GHTXDOTXHUHVSpFLHHVRETXDOTXHUSUHWH[WR D FRPHWLGR D LQIUDomR GLVFLSOLQDU VRE FRDomR D TXH
SRGLD UHVLVWLU RX HP FXPSULPHQWR D RUGHP GH
,9 YDOHUVHGRFDUJRSDUDREWHUSURYHLWRLQGHYLGRSDUDVL DXWRULGDGHVXSHULRURXVREDLQIOXrQFLDGHYLROHQWD
RX SDUD RXWUHP HP GHWULPHQWR GD GLJQLGDGH GD HPRomR SURYRFDGD SRU DWR LQMXVWR SURYLQGR GH
IXQomRS~EOLFD WHUFHLUR

9 XWLOL]DUVHGHGRFXPHQWRVDELGDPHQWHIDOVRSDUDSURYD E FRPHWLGRDLQIUDomRGLVFLSOLQDUQDGHIHVDDLQGDTXH


GH IDWR RX FLUFXQVWkQFLD TXH FULH GLUHLWR RX H[WLQJD SXWDWLYDRXFRPH[FHVVRPRGHUDGRGHSUHUURJDWLYD
REULJDomRSHUDQWHDDGPLQLVWUDomRS~EOLFDGLVWULWDO IXQFLRQDO

†~QLFR 3DUDHIHLWRVGRLQFLVR,,,QmRVHFRQVLGHUDSUHVHQWHR F SURFXUDGR SRU VXD HVSRQWkQHD YRQWDGH H FRP


EULQGHGHILQLGRQDOHJLVODomR HILFLrQFLDORJRDSyVDLQIUDomRGLVFLSOLQDUHYLWDURX
PLQRUDUDVVXDVFRQVHTXrQFLDV
&DStWXOR,,,
'$66$1d¯(6',6&,3/,1$5(6 G UHSDUDGR R GDQR FDXVDGR SRU VXD HVSRQWkQHD
YRQWDGHHDQWHVGRMXOJDPHQWR
$UW 6mRVDQo}HVGLVFLSOLQDUHV
$UW 6mRFLUFXQVWkQFLDVDJUDYDQWHV
, DGYHUWrQFLD
,, VXVSHQVmR , DSUiWLFDGHDWRTXHFRQFRUUDJUDYHHREMHWLYDPHQWH
,,, GHPLVVmR SDUDRGHVSUHVWtJLRGRyUJmRDXWDUTXLDRXIXQGDomR
,9 FDVVDomRGHDSRVHQWDGRULDRXGHGLVSRQLELOLGDGH RXGDFDWHJRULDIXQFLRQDOGRVHUYLGRU
9 GHVWLWXLomRGRFDUJRHPFRPLVVmR
,, RFRQFXUVRGHSHVVRDV
†~QLFR $V VDQo}HV GLVFLSOLQDUHV VmR DSOLFDGDVjVLQIUDo}HV
GLVFLSOLQDUHVWLSLILFDGDVHPOHL ,,, RFRPHWLPHQWRGDLQIUDomRGLVFLSOLQDUHPSUHMXt]RGH
FULDQoD DGROHVFHQWH LGRVR SHVVRD FRP GHILFLrQFLD
$UW 1D DSOLFDomR GDV VDQo}HV GLVFLSOLQDUHV GHYHPVHU SHVVRD LQFDSD] GH VH GHIHQGHU RX SHVVRD VRE VHXV
FRQVLGHUDGRV FXLGDGRVSRUIRUoDGHVXDVDWULEXLo}HV
, D QDWXUH]D H D JUDYLGDGH GD LQIUDomRGLVFLSOLQDU ,9 RFRPHWLPHQWRGDLQIUDomRGLVFLSOLQDUFRPYLROrQFLD
FRPHWLGD RXJUDYHDPHDoDTXDQGRQmRHOHPHQWDUHVGDLQIUD
omR
,, RVGDQRVFDXVDGRVSDUDRVHUYLoRS~EOLFR
9 VHURVHUYLGRUTXHP
,,, RkQLPRHDLQWHQomRGRVHUYLGRU
D SURPRYH RX RUJDQL]D D FRRSHUDomR RX GLULJH D
,9 DVFLUFXQVWkQFLDVDWHQXDQWHVHDJUDYDQWHV DWLYLGDGHGRVGHPDLVFRDXWRUHV
9 D FXOSDELOLGDGH H RV DQWHFHGHQWHV IXQFLRQDLVGR E LQVWLJD VXERUGLQDGR RX OKH RUGHQD D SUiWLFD GD
VHUYLGRU LQIUDomRGLVFLSOLQDU
†ž $LQIUDomRGLVFLSOLQDUGHPHQRUJUDYLGDGHpDEVRUYLGD F LQVWLJDRXWURVHUYLGRUSURS}HRXVROLFLWDDSUiWLFD
SHODGHPDLRUJUDYLGDGH GDLQIUDomRGLVFLSOLQDU
†ž 1HQKXPDVDQomRGLVFLSOLQDUSRGHVHUDSOLFDGD $UW $DGYHUWrQFLDpDVDQomRSRULQIUDomRGLVFLSOLQDUOHYH
SRUPHLRGDTXDOVHUHSURYDSRUHVFULWRDFRQGXWDGR
, VHPSUHYLVmROHJDO VHUYLGRU
,, VHPDSXUDomRHPUHJXODUSURFHVVRGLVFLSOLQDUSUHYLVWR
QHVWD/HL&RPSOHPHQWDU †~QLFR 1ROXJDUGDDGYHUWrQFLDSRGHVHUDSOLFDGDPRWLYDGD
PHQWHDVXVSHQVmRDWpWULQWDGLDVVHDVFLUFXQVWkQFLDV
$UW 6mRFLUFXQVWkQFLDVDWHQXDQWHV DVVLPRMXVWLILFDUHP

, DXVrQFLDGHSXQLomRDQWHULRU $UW $VXVSHQVmRpDVDQomRSRULQIUDomRGLVFLSOLQDUPpGLD


SHODTXDOVHLPS}HDRVHUYLGRURDIDVWDPHQWRFRPSXO
,, SUHVWDomR GH ERQV VHUYLoRV j DGPLQLVWUDomRS~EOLFD VyULR GR H[HUFtFLR GR FDUJR HIHWLYR FRP SHUGD GD
GLVWULWDO UHPXQHUDomR RX VXEVtGLR GRV GLDV HP TXH HVWLYHU
DIDVWDGR
,,, GHVFRQKHFLPHQWRMXVWLILFiYHOGHQRUPDDGPLQLVWUDWLYD
†ž $VXVSHQVmRQmRSRGHVHU
,9 PRWLYRGHUHOHYDQWHYDORUVRFLDORXPRUDO
, VXSHULRUDWULQWDGLDVQRFDVRGHLQIUDomRGLVFLSOLQDU
9 HVWDGRItVLFRSVLFROyJLFRPHQWDORXHPRFLRQDODEDOD PpGLDGRJUXSR,
GRTXHLQIOXHQFLHRXVHMDGHFLVLYRSDUDDSUiWLFDGD
LQIUDomRGLVFLSOLQDU ,, VXSHULRUDQRYHQWDGLDVQRFDVRGHLQIUDomRGLVFLSOLQDU
PpGLDGRJUXSR,,
9, FRH[LVWrQFLDGHFDXVDVUHODWLYDVjFDUrQFLDGHFRQGL
o}HVGHPDWHULDORXSHVVRDOQDUHSDUWLomR †ž $SOLFDVHDVXVSHQVmRGHDWp
6HFUHWDULDGH(VWDGRGH$ GPLQLVWUDomR3~EOLFD'LVWULWR)HGHUDO
 /HJLVODomR 

, WULQWDGLDVTXDQGRRVHUYLGRULQFRUUHUHPUHLQFLGrQFLD †~QLFR $FDVVDomRGHDSRVHQWDGRULDpDSOLFDGDSRULQIUDomR


SRULQIUDomRGLVFLSOLQDUOHYH GLVFLSOLQDUSXQtYHOFRPGHPLVVmR

,, QRYHQWDGLDVTXDQGRRVHUYLGRULQFRUUHUHPUHLQFL $UW $FDVVDomRGHGLVSRQLELOLGDGHpDVDQomRSRULQIUDomR


GrQFLDSRULQIUDomRGLVFLSOLQDPpGLDGRJUXSR, GLVFLSOLQDU TXH KRXYHU VLGR FRPHWLGD HP DWLYLGDGH
SHODTXDOVHLPS}HDSHUGDGRFDUJRS~EOLFRRFXSDGR
†ž 4XDQGRKRXYHUFRQYHQLrQFLDSDUDRVHUYLoRDSHQDOL HGRVGLUHLWRVGHFRUUHQWHVGDGLVSRQLELOLGDGHSRGHQGR
GDGH GH VXVSHQVmR SRGH VHU FRQYHUWLGD HP PXOWD VHUFRPLQDGDFRPRLPSHGLPHQWRGHQRYDLQYHVWLGXUD
REVHUYDGRRVHJXLQWH HPFDUJRS~EOLFR

, DPXOWDpGHFLQTXHQWDSRUFHQWRGRYDORUGLiULRGD †~QLFR $FDVVDomRGHGLVSRQLELOLGDGHpDSOLFDGDSRULQIUDomR


UHPXQHUDomRRXVXEVtGLRSRUGLDGHVXVSHQVmR GLVFLSOLQDUSXQtYHOFRPGHPLVVmRHQDKLSyWHVHGRDUW
†ž
,, R VHUYLGRU ILFD REULJDGR D FXPSULU LQWHJUDOPHQWHD
MRUQDGDGHWUDEDOKRDTXHHVWiVXEPHWLGR $UW $ GHVWLWXLomR GR FDUJR HP FRPLVVmR p D VDQomRSRU
LQIUDomR GLVFLSOLQDU PpGLD RX JUDYH SHOD TXDO VH
†ž e DSOLFDGD PXOWD DR VHUYLGRU LQDWLYRTXHKRXYHU LPS}HDRVHUYLGRUVHPYtQFXORHIHWLYRFRPR'LVWULWR
SUDWLFDGRQDDWLYLGDGHLQIUDomRGLVFLSOLQDUSXQtYHOFRP )HGHUDODSHUGDGRFDUJRHPFRPLVVmRSRUHOHRFXSD
VXVSHQVmR GR SRGHQGR VHU FRPLQDGD FRP R LPSHGLPHQWR GH
QRYDLQYHVWLGXUDHPRXWURFDUJRHIHWLYRRXHPFRPLV
†ž $ PXOWD GH TXH WUDWD R † ž FRUUHVSRQGHDRYDORU VmR
GLiULR GRV SURYHQWRV GH DSRVHQWDGRULD SRU GLD GH
VXVSHQVmRFDEtYHO †~QLFR 6H R VHUYLGRU Mi WLYHU VLGR H[RQHUDGRTXDQGRGD
DSOLFDomRGDVDQomRSUHYLVWDQHVWHDUWLJRDH[RQHUD
$UW $DGYHUWrQFLDHDVXVSHQVmRWrPVHXVUHJLVWURVFDQFH omRpFRQYHUWLGDHPGHVWLWXLomRGRFDUJRHPFRPLVVmR
ODGRVDSyVRGHFXUVRGHWUrVHFLQFRDQRVGHHIHWLYR
H[HUFtFLRUHVSHFWLYDPHQWHVHRVHUYLGRUQmRKRXYHU $UW $GHPLVVmRDFDVVDomRGHDSRVHQWDGRULDRXGLVSRQLEL
QHVVH SHUtRGR SUDWLFDGR QRYD LQIUDomR GLVFLSOLQDU OLGDGHRXDGHVWLWXLomRGHFDUJRHPFRPLVVmRPRWLYD
LJXDORXGLYHUVDGDDQWHULRUPHQWHFRPHWLGD GDSRULQIUDomRGLVFLSOLQDUJUDYHGRJUXSR,,LPSOLFD
DLQFRPSDWLELOL]DomRSDUDQRYDLQYHVWLGXUDHPFDUJR
†ž 2FDQFHODPHQWRGDVDQomRGLVFLSOLQDUQmRVXUWHHIHLWRV S~EOLFR GR 'LVWULWR )HGHUDO SHOR SUD]R GH GH] DQRV
UHWURDWLYRV H p UHJLVWUDGR HP FHUWLGmR IRUPDO QRV VHP SUHMXt]R GH DomR FtYHO RX SHQDO H GDV GHPDLV
DVVHQWDPHQWRVIXQFLRQDLVGRVHUYLGRU PHGLGDVDGPLQLVWUDWLYDV

†ž &HVVDPRVHIHLWRVGDDGYHUWrQFLDRXGDVXVSHQVmRVH $UW $SXQLELOLGDGHpH[WLQWDSHOD


OHL SRVWHULRU GHL[DU GH FRQVLGHUDU FRPR LQIUDomR
GLVFLSOLQDURIDWRTXHDVPRWLYRX , PRUWHGRVHUYLGRU
,, SUHVFULomR
†ž $VDQomRGLVFLSOLQDUFDQFHODGDQRVWHUPRVGHVWHDUWLJR
QmRSRGHVHUFRQVLGHUDGDSDUDHIHLWRVGHUHLQFLGrQFLD $UW $DomRGLVFLSOLQDUSUHVFUHYHHP

$UW $ GHPLVVmR p D VDQomR SHODV LQIUDo}HVGLVFLSOLQDUHV , FLQFRDQRVTXDQWRjGHPLVVmRGHVWLWXLomRGHFDUJR


JUDYHVSHODTXDOVHLPS}HDRVHUYLGRUHIHWLYRDSHUGD HPFRPLVVmRRXFDVVDomRGHDSRVHQWDGRULDRXGLVSRQL
GRFDUJRS~EOLFRSRUHOHRFXSDGRSRGHQGRVHUFRPL ELOLGDGH
QDGD FRP R LPSHGLPHQWR GH QRYD LQYHVWLGXUD HP
FDUJRS~EOLFR ,, GRLVDQRVTXDQWRjVXVSHQVmR
,,, XPDQRTXDQWRjDGYHUWrQFLD
†ž $GHPLVVmRGHTXHWUDWDHVWHDUWLJRWDPEpPVHDSOLFD
QRFDVRGH †ž 2 SUD]R GH SUHVFULomR FRPHoD D FRUUHUGDSULPHLUD
GDWDHPTXHRIDWRRXDWRVHWRUQRXFRQKHFLGRSHOD
, LQIUDomR GLVFLSOLQDU JUDYH TXDQGR FRPHWLGDSRU FKHILD GD UHSDUWLomR RQGH HOH RFRUUHX SHOD FKHILD
VHUYLGRUHIHWLYRQRH[HUFtFLRGHFDUJRHPFRPLVVmRRX PHGLDWDRXLPHGLDWDGRVHUYLGRURXSHODDXWRULGDGH
IXQomRGHFRQILDQoDGR3RGHU([HFXWLYRRX/HJLVODWLYR FRPSHWHQWH SDUD LQVWDXUDU VLQGLFkQFLD RX SURFHVVR
GR'LVWULWR)HGHUDO GLVFLSOLQDU

,, UHLQFLGrQFLDHPLQIUDomRGLVFLSOLQDUPpGLDGRJUXSR,, †ž $ LQVWDXUDomR GH SURFHVVR GLVFLSOLQDULQWHUURPSHD


SUHVFULomRXPD~QLFDYH]
†ž 6H R VHUYLGRU Mi WLYHU VLGR H[RQHUDGRTXDQGRGD
DSOLFDomRGDVDQomRSUHYLVWDQHVWHDUWLJRDH[RQHUD †ž ,QWHUURPSLGDDSUHVFULomRVXDFRQWDJHPpUHLQLFLDGD
omRpFRQYHUWLGDHPGHPLVVmR GHSRLV GH HVJRWDGRV RV SUD]RV SDUD FRQFOXVmR GR
SURFHVVRGLVFLSOLQDUSUHYLVWRVQHVWD/HL&RPSOHPHQ
†ž 7DPEpP VH FRQYHUWH HP GHPLVVmR DYDFkQFLDHP WDULQFOXtGRVRVSUD]RVGHSURUURJDomRVHKRXYHU
GHFRUUrQFLD GH SRVVH HP RXWUR FDUJR LQDFXPXOiYHO
RFRUULGDDQWHVGDDSOLFDomRGDVDQomRSUHYLVWDQHVWH †ž 2 SUD]R GH SUHVFULomR ILFD VXVSHQVRHQTXDQWRD
DUWLJR LQVWDXUDomRRXDWUDPLWDomRGRSURFHVVRGLVFLSOLQDURX
DDSOLFDomRGHVDQomRGLVFLSOLQDUHVWLYHUREVWDGDSRU
$UW $FDVVDomRGHDSRVHQWDGRULDpDVDQomRSRULQIUDomR GHWHUPLQDomRMXGLFLDO
GLVFLSOLQDUTXHKRXYHUVLGRFRPHWLGDSHORVHUYLGRUHP
DWLYLGDGH SHOD TXDO VH LPS}H D SHUGD GR GLUHLWR j †ž 2VSUD]RVGHSUHVFULomRSUHYLVWRVQDOHLSHQDOKDYHQ
DSRVHQWDGRULDSRGHQGRVHUFRPLQDGDFRPRLPSHGL GR DomR SHQDO HP FXUVR DSOLFDPVH jV LQIUDo}HV
PHQWRGHQRYDLQYHVWLGXUDHPFDUJRS~EOLFR GLVFLSOLQDUHVFDSLWXODGDVWDPEpPFRPRFULPH
  /HJLVODomR 6HFUHWDULDGH(VWDGRGH$GPLQLVWUDomR3~EOLFD'LVWULWR)HGHUDO


$UW 1mR p SXQLGR R VHUYLGRU TXH DR WHPSR GDLQIUDomR †ž $ UHSUHVHQWDomR VREUH LQIUDomRGLVFLSOLQDUFRPHWLGD
GLVFLSOLQDU HUD LQWHLUDPHQWH LQFDSD] GH HQWHQGHU R SRUVHUYLGRUGHYHVHUIRUPXODGDSRUHVFULWRHFRQWHUD
FDUiWHULOtFLWRGRIDWRRXGHGHWHUPLQDUVHGHDFRUGR LGHQWLILFDomRHRHQGHUHoRGRGHQXQFLDQWH
FRPHVVHHQWHQGLPHQWRGHYLGRD
†ž 1R FDVR GH GHQ~QFLDV DQ{QLPDVDDGPLQLVWUDomR
, LQVDQLGDGH PHQWDO GHYLGDPHQWH FRPSURYDGDSRU S~EOLFD SRGH LQLFLDU UHVHUYDGDPHQWH LQYHVWLJDo}HV
ODXGRGHMXQWDPpGLFDRILFLDO SDUDFROHWDGHRXWURVPHLRVGHSURYDQHFHVViULRVSDUD
DLQVWDXUDomRGHVLQGLFkQFLDRXSURFHVVRGLVFLSOLQDU
,, HPEULDJXH]FRPSOHWDSURYHQLHQWHGHFDVRIRUWXLWRRX
IRUoDPDLRU †ž (PFDVRGHLQIUDomRGLVFLSOLQDUQRWLFLDGDSHODLPSUHQ
VDQDVUHGHVVRFLDLVRXHPFRUUHVSRQGrQFLDVHVFULWDV
†~QLFR $SXQLELOLGDGHQmRVHH[FOXLSHODHPEULDJXH]YROXQWi DDXWRULGDGHFRPSHWHQWHDQWHVGHLQVWDXUDUVLQGLFkQ
ULDRXFXOSRVDSRUiOFRROHQWRUSHFHQWHRXVXEVWkQFLD FLDRXSURFHVVRGLVFLSOLQDUGHYHYHULILFDUVHKiLQGtFL
GHHIHLWRVDQiORJRV RVPtQLPRVGHVXDRFRUUrQFLD
†ž 1DKLSyWHVHGR†žQRFDVRGHQmRFRPSURYDomRGRV
$UW )LFD LVHQWR GH VDQomR GLVFLSOLQDU R VHUYLGRUFXMD IDWRVDDXWRULGDGHFRPSHWHQWHGHYHVHSURQXQFLDUSRU
FRQGXWDIXQFLRQDOFODVVLILFDGDFRPRHUURGHSURFHGL HVFULWRVREUHRPRWLYRGRDUTXLYDPHQWRGDYHULILFDomR
PHQWRVHMDFDUDFWHUL]DGDFXPXODWLYDPHQWHSHOD
†ž 6H KRXYHU LQGtFLRV VXILFLHQWHV TXDQWR j DXWRULDHj
, DXVrQFLDGHGROR PDWHULDOLGDGH GD LQIUDomR GLVFLSOLQDU D DXWRULGDGH
,, HYHQWXDOLGDGHGRHUUR DGPLQLVWUDWLYDSRGHLQVWDXUDULPHGLDWDPHQWHRSURFHV
,,, RIHQVDtQILPDDRVEHQVMXUtGLFRVWXWHODGRV VRGLVFLSOLQDUGLVSHQVDGDDLQVWDXUDomRGHVLQGLFkQFLD
,9 SUHMXt]RPRUDOLUUHOHYDQWH
9 UHSDUDomRGHHYHQWXDOSUHMXt]RPDWHULDODQWHVGHVH $UW 1mRpREMHWRGHDSXUDomRHPVLQGLFkQFLDRXSURFHVVR
LQVWDXUDUVLQGLFkQFLDRXSURFHVVRGLVFLSOLQDU GLVFLSOLQDURIDWRTXH

, QmR FRQILJXUHLQIUDomRGLVFLSOLQDUSUHYLVWDQHVWD/HL


7tWXOR9,, &RPSOHPHQWDURXHPOHJLVODomRHVSHFtILFD
'26352&(6626'($385$d®2'(,1)5$d®2',6&,3/,1$5
,, MiWHQKDVLGRREMHWRGHMXOJDPHQWRSHOR3RGHU-XGLFLi
&DStWXOR, ULR HP VHQWHQoD SHQDO WUDQVLWDGD HP MXOJDGR TXH
'$6',6326,d¯(6*(5$,6 UHFRQKHFHX D LQH[LVWrQFLD GR IDWR RX D QHJDWLYD GD
DXWRULDVDOYRVHH[LVWHQWHLQIUDomRGLVFLSOLQDUUHVLGXDO
6HomR,
'DV'LVSRVLo}HV&RPXQV †ž 2VHUYLGRUQmRUHVSRQGH

$UW 'LDQWHGHLQGtFLRVGHLQIUDomRGLVFLSOLQDURXGLDQWHGH , SRUDWRSUDWLFDGRFRPIXQGDPHQWRHPOHLRXUHJXOD


UHSUHVHQWDomRDDXWRULGDGHDGPLQLVWUDWLYDFRPSHWHQ PHQWR SRVWHULRUPHQWH FRQVLGHUDGR LQFRQVWLWXFLRQDO
WH GHYH GHWHUPLQDU D LQVWDXUDomR GH VLQGLFkQFLD RX SHOR3RGHU-XGLFLiULR
SURFHVVR GLVFLSOLQDU SDUD DSXUDU RV IDWRV H VH IRU R
FDVRDSOLFDUDVDQomRGLVFLSOLQDU ,, TXDQGRDSXQLELOLGDGHHVWLYHUH[WLQWD

†ž 6mRFRPSHWHQWHVSDUDLQVWDXUDUVLQGLFkQFLDRXSURFHV †ž 'HYHVHUDUTXLYDGDHYHQWXDOGHQ~QFLDRXUHSUHVHQWD


VRGLVFLSOLQDUDVDXWRULGDGHVGHILQLGDVQRDUWHP omRTXHVHUHILUDDTXDOTXHUGDV KLSyWHVHVSUHYLVWDV
UHODomR jV LQIUDo}HV GLVFLSOLQDUHV RFRUULGDV HP VHXV QHVWHDUWLJR
UHVSHFWLYRVyUJmRVDXWDUTXLDVRXIXQGDo}HVLQGHSHQ
GHQWHPHQWHGDVDQomRFRPLQDGD 6HomR,,
'D6LQGLFkQFLD
†ž $FRPSHWrQFLDSDUDLQVWDXUDUSURFHVVRGLVFLSOLQDUSDUD
DSXUDU LQIUDomR FRPHWLGD SRU VHUYLGRU HIHWLYR QR $UW $VLQGLFkQFLDpRSURFHGLPHQWRLQYHVWLJDWLYRGHVWLQDGR
H[HUFtFLRGHFDUJRHPFRPLVVmRRXIXQomRGHFRQILDQ D
oDGRTXDOIRLH[RQHUDGRRXGLVSHQVDGRpGDDXWRULGD
GHGRyUJmRDXWDUTXLDRXIXQGDomRRQGHDLQIUDomR , LGHQWLILFDU D DXWRULD GH LQIUDomR GLVFLSOLQDUTXDQGR
GLVFLSOLQDUIRLFRPHWLGD GHVFRQKHFLGD

†ž 3RUVROLFLWDomRRXGHWHUPLQDomRGDDXWRULGDGHFRPSH ,, DSXUDUDPDWHULDOLGDGHGHLQIUDomRGLVFLSOLQDUVREUHD


WHQWHDDSXUDomRGDLQIUDomRGLVFLSOLQDUSRGHVHUIHLWD TXDOKDMD DSHQDVLQGtFLRVRXTXHWHQKDVLGRDSHQDV
SHORyUJmRFHQWUDOGRVLVWHPDGHFRUUHLomRSUHVHUYDGD QRWLFLDGD
DFRPSHWrQFLDSDUDRMXOJDPHQWR
†ž 2DWRGHLQVWDXUDomRGDVLQGLFkQFLDGHYHVHUSXEOLFDGR
†ž 2VFRQIOLWRVHQWUHVHUYLGRUHVSRGHPVHUWUDWDGRVHP QR'LiULR2ILFLDOGR'LVWULWR)HGHUDO
PHVDGHFRPLVVmRGHPHGLDomRDVHUGLVFLSOLQDGDHP
OHLHVSHFtILFD †ž 2SUD]RSDUDFRQFOXVmRGDVLQGLFkQFLDpGHDWpWULQWD
GLDV SURUURJiYHO SRU LJXDO SHUtRGR D FULWpULR GD
$UW $LQIUDomRGLVFLSOLQDUFRPHWLGDSRUVHUYLGRUpDSXUDGD DXWRULGDGHFRPSHWHQWH
PHGLDQWH
$UW 'DVLQGLFkQFLDSRGHUHVXOWDU
, VLQGLFkQFLD
,, SURFHVVRGLVFLSOLQDU , RDUTXLYDPHQWRGRSURFHVVR
6HFUHWDULDGH(VWDGRGH$ GPLQLVWUDomR3~EOLFD'LVWULWR)HGHUDO
 /HJLVODomR 

,, LQVWDXUDomRGHSURFHVVRGLVFLSOLQDU †ž 2V DWRV GR SURFHVVR GLVFLSOLQDU QmRGHSHQGHPGH


IRUPDGHWHUPLQDGDVHQmRTXDQGRDOHLH[SUHVVDPHQWH
,,, DSOLFDomRGHVDQomRGHDGYHUWrQFLDRXVXVSHQVmRGH R H[LJLU UHSXWDQGRVH YiOLGRV RV TXH UHDOL]DGRV GH
DWpWULQWDGLDV RXWURPRGRSUHHQFKDPVXDILQDOLGDGHHVVHQFLDO

†ž &RQVWDWDGRQDVLQGLFkQFLDTXHDLQIUDomRFODVVLILFDVH †ž eSHUPLWLGD


FRPROHYHRXPpGLDGRJUXSR,DFRPLVVmRGHVLQGL
FkQFLDGHYHFLWDURVHUYLGRUDFXVDGRSDUDDFRPSDQKDU , DQRWLILFDomRRXDLQWLPDomRGRVHUYLGRUDFXVDGRRX
RSURVVHJXLPHQWRGDDSXUDomRQRVPHVPRVDXWRV LQGLFLDGRRXGHVHXSURFXUDGRUHPDXGLrQFLD

†ž $SOLFDPVHDSDUWLUGRDWRSURFHVVXDOGHTXHWUDWDR† ,, DFRPXQLFDomRYLDSRVWDOHQWUHDFRPLVVmRSURFHVVDQ


WHHRVHUYLGRUDFXVDGRRXLQGLFLDGR
ž DV QRUPDV GR SURFHVVR GLVFLSOLQDU LQFOXtGDV DV
JDUDQWLDV DR FRQWUDGLWyULR H j DPSOD GHIHVD H DV
,,, D XWLOL]DomR GH PHLR HOHWU{QLFR VH FRQILUPDGRR
QRUPDVUHODWLYDVjFRPLVVmRSURFHVVDQWH
UHFHELPHQWRSHORGHVWLQDWiULRRXPHGLDQWHFHUWLILFDomR
GLJLWDOSDUD
6HomR,,,
'D6LQGLFkQFLD3DWULPRQLDO D DHQWUHJDGHSHWLomRjFRPLVVmRSURFHVVDQWHVDOYR
DGHIHVDHVFULWDSUHYLVWDQRDUWGHVGHTXHR
$UW 'LDQWHGHIXQGDGRVLQGtFLRVGHHQULTXHFLPHQWRLOtFLWR PHLR XWLOL]DGR SHOR UHPHWHQWH VHMD SUHYLDPHQWH
GHVHUYLGRURXGHHYROXomRSDWULPRQLDOLQFRPSDWtYHO FDGDVWUDGRQDFRPLVVmRSURFHVVDQWH
FRP D UHPXQHUDomR RX VXEVtGLR SRU HOH SHUFHELGR
SRGH VHU GHWHUPLQDGD D LQVWDXUDomR GH VLQGLFkQFLD E DQRWLILFDomRRXDLQWLPDomRVREUHDWRVGRSURFHVVR
SDWULPRQLDO GLVFLSOLQDUVDOYRRVSUHYLVWRVQRVDUWVH
GHVGHTXHRPHLRHOHWU{QLFRWHQKDVLGRSUHYLDPHQ
†ž 6mR FRPSHWHQWHV SDUD GHWHUPLQDU DLQVWDXUDomRGH WHFDGDVWUDGRSHORVHUYLGRUDFXVDGRRXLQGLFLDGRQD
VLQGLFkQFLDSDWULPRQLDO FRPLVVmRSURFHVVDQWH

, R3UHVLGHQWHGD&kPDUD/HJLVODWLYDRXGR7ULEXQDOGH †ž 6HDFRPLVVmRQRWLILFDURXLQWLPDURVHUYLGRUSRUPHLR


&RQWDVQRVUHVSHFWLYRVyUJmRV HOHWU{QLFR GHYH VHPSUH TXH SRVVtYHO DYLViOR SRU
PHLRWHOHI{QLFRGHTXHDFRPXQLFDomRIRLHQYLDGD
,, R*RYHUQDGRURXRWLWXODUGRyUJmRFHQWUDOGHVLVWHPD
GHFRUUHLomRQR3RGHU([HFXWLYR †ž 2XVRGRVPHLRVSHUPLWLGRVQRV††žHžGHYHVHU
FHUWLILFDGRQRVDXWRVMXQWDQGRVHFySLDGDVFRUUHVSRQ
†ž $VLQGLFkQFLDSDWULPRQLDOFRQVWLWXLVHGHSURFHGLPHQWR GrQFLDVUHFHELGDVRXHQYLDGDV
VLJLORVRFRPFDUiWHUH[FOXVLYDPHQWHLQYHVWLJDWLYR
†ž 1mRpFDXVDGHQXOLGDGHGRDWRSURFHVVXDODDXVrQFLD
†ž 2SURFHGLPHQWRGHVLQGLFkQFLDSDWULPRQLDOpFRQGX]L
, GRVHUYLGRUDFXVDGRRXGHVHXSURFXUDGRUQDRLWLYDGH
GRSRUFRPLVVmRFRPSRVWDSRUWUrVVHUYLGRUHVHVWiYHLV
WHVWHPXQKDTXDQGRRVHUYLGRUWHQKDVLGRSUHYLDPHQ
WHQRWLILFDGR
†ž 2SUD]RSDUDFRQFOXVmRGRSURFHGLPHQWRGHVLQGLFkQ
FLDSDWULPRQLDOpGHWULQWDGLDVSURUURJiYHOSRULJXDO ,, GRSURFXUDGRUQRLQWHUURJDWyULRGRVHUYLGRUDFXVDGR
SHUtRGR
$UW 2V DXWRV GR SURFHVVR GLVFLSOLQDU DV UHXQL}HVGD
†ž &RQFOXtGRVRVWUDEDOKRVGDVLQGLFkQFLDSDWULPRQLDOD FRPLVVmRHRVDWRVSURFHVVXDLVWrPFDUiWHUUHVHUYDGR
FRPLVVmRUHVSRQViYHOSRUVXDFRQGXomRGHYHHODERUDU
UHODWyULR VREUH RV IDWRV DSXUDGRV FRQFOXLQGR SHOR †ž 2V DXWRV GR SURFHVVR GLVFLSOLQDU QmRSRGHPVHU
DUTXLYDPHQWRRXSHODLQVWDXUDomRGHSURFHVVRGLVFLSOL UHWLUDGRVGDUHSDUWLomRRQGHVHHQFRQWUDP
QDU
†ž eOtFLWRRIRUQHFLPHQWRGHFySLDGHSHoDVGRVDXWRVDR
6HomR,9 VHUYLGRURXDRVHXSURFXUDGRUREVHUYDGRRGLVSRVWR
'R3URFHVVR'LVFLSOLQDU QRDUW††žHž

$UW 2 SURFHVVR GLVFLSOLQDU p R LQVWUXPHQWR GHVWLQDGRD $UW 6DOYRTXDQGRDXWRUL]DGRSHODDXWRULGDGHLQVWDXUDGR


DSXUDU UHVSRQVDELOLGDGH GR VHUYLGRU SRU LQIUDomR UD p YHGDGR GHIHULU DR VHUYLGRU DFXVDGR GHVGH D
GLVFLSOLQDU LQVWDXUDomRGRSURFHVVRGLVFLSOLQDUDWpDFRQFOXVmRGR
SUD]RSDUDGHIHVDHVFULWD
†~QLFR 2SUD]RSDUDDFRQFOXVmRGRSURFHVVRGLVFLSOLQDUpGH
DWpVHVVHQWDGLDVSURUURJiYHOSRULJXDOSHUtRGR , JR]RGHIpULDV
,, OLFHQoDRXDIDVWDPHQWRYROXQWiULRV
$UW 2VDXWRVGDVLQGLFkQFLDVHKRXYHUVmRDSHQVDGRVDRV ,,, H[RQHUDomRDSHGLGR
GR SURFHVVR GLVFLSOLQDU FRPR SHoD LQIRUPDWLYD GD ,9 DSRVHQWDGRULDYROXQWiULD
LQVWUXomR
&DStWXOR,,
$UW 2 SURFHVVR GLVFLSOLQDU REHGHFH DRV SULQFtSLRVGD '2$)$67$0(17235(9(17,92
OHJDOLGDGHPRUDOLGDGHLPSHVVRDOLGDGHSXEOLFLGDGH
$UW &RPRPHGLGDFDXWHODUHDILPGHTXHRVHUYLGRUQmR
HILFLrQFLD LQWHUHVVH S~EOLFR FRQWUDGLWyULR DPSOD
YHQKDDLQIOXLUQDDSXUDomRGDLQIUDomRGLVFLSOLQDUD
GHIHVDSURSRUFLRQDOLGDGHUD]RDELOLGDGHPRWLYDomR
DXWRULGDGHLQVWDXUDGRUDGRSURFHVVRGLVFLSOLQDUSRGH
VHJXUDQoD MXUtGLFD LQIRUPDOLVPR PRGHUDGR MXVWLoD GHWHUPLQDURVHXDIDVWDPHQWRGRH[HUFtFLRGRFDUJR
YHUGDGHPDWHULDOHLQGLVSRQLELOLGDGH SHOR SUD]R GH DWp VHVVHQWD GLDV VHP SUHMXt]R GD
UHPXQHUDomR
  /HJLVODomR 6HFUHWDULDGH(VWDGRGH$GPLQLVWUDomR3~EOLFD'LVWULWR)HGHUDO


†ž 2DIDVWDPHQWRSUHYHQWLYRSRGH ,; WHUDFHVVRjVSHoDVGRVDXWRVREVHUYDGDVDVUHJUDVGH


VLJLOR
, VHUSURUURJDGRSRULJXDOSUD]RILQGRRTXDOFHVVDPRV
VHXV HIHLWRV DLQGD TXH QmR FRQFOXtGR R SURFHVVR ; DSUHVHQWDU SHGLGR GH UHFRQVLGHUDomR UHFXUVRRX
GLVFLSOLQDU UHYLVmRGRMXOJDPHQWR

,, FHVVDUSRUGHWHUPLQDomRGDDXWRULGDGHFRPSHWHQWH †ž $DUJXLomRGHTXHWUDWDRLQFLVR,GRFDSXWGHYHVHU


UHVROYLGD
†ž 6DOYRPRWLYRGHFDVRIRUWXLWRRXIRUoDPDLRURVHUYL
GRU DIDVWDGR QmR SRGH FRPSDUHFHU j UHSDUWLomR GH , SHODDXWRULGDGHLPHGLDWDPHQWHVXSHULRUQRFDVRGR
RQGH IRL DIDVWDGR H[FHWR TXDQWR DXWRUL]DGR SHOD LQFLVR,DRXSHORVXEVWLWXWROHJDOVHH[DXULGDDYLD
DXWRULGDGHFRPSHWHQWHRXSHODFRPLVVmRSURFHVVDQWH KLHUiUTXLFD

$UW (PVXEVWLWXLomRDRDIDVWDPHQWRSUHYHQWLYRDDXWRUL ,, SHODDXWRULGDGHTXHLQVWDXURXRSURFHVVRGLVFLSOLQDU


GDGHLQVWDXUDGRUDSRGHQRSUD]RGRDUWGHWHU QRFDVRGRLQFLVR,E
PLQDU TXH R VHUYLGRU WHQKD H[HUFtFLR SURYLVyULR HP
RXWUDXQLGDGHDGPLQLVWUDWLYDGRPHVPRyUJmRDXWDU †ž eGRVHUYLGRUDFXVDGRRFXVWRGHSHUtFLDVRXH[DPHV
TXLDRXIXQGDomRGHVXDORWDomR SRUHOHUHTXHULGRVVHQmRKRXYHUWpFQLFRKDELOLWDGR
QRVTXDGURVGDDGPLQLVWUDomRS~EOLFDGLVWULWDO
&DStWXOR,,,
'$$03/$'()(6$('2&2175$',7Ð5,2 $UW 4XDQGR KRXYHU G~YLGD VREUH D VDQLGDGH PHQWDOGR
VHUYLGRUDFXVDGRDFRPLVVmRSURFHVVDQWHGHYHSURSRU
$UW 1R SURFHVVR GLVFLSOLQDU p VHPSUH DVVHJXUDGRDR j DXWRULGDGH FRPSHWHQWH TXH HOH VHMD VXEPHWLGR D
VHUYLGRUDFXVDGRRGLUHLWRDRFRQWUDGLWyULRHjDPSOD H[DPHSRUMXQWDPpGLFDRILFLDOGDTXDOSDUWLFLSHSHOR
GHIHVD PHQRVXPPpGLFRSVLTXLDWUD

$UW 2VHUYLGRUDFXVDGRGHYHVHU †~QLFR 2LQFLGHQWHGHVDQLGDGHPHQWDOGHYHVHUSURFHVVDGR


HP DXWRV DSDUWDGRV H DSHQVR DR SURFHVVR SULQFLSDO
, FLWDGR VREUH D LQVWDXUDomR GH SURFHVVRGLVFLSOLQDU DSyVDH[SHGLomRGRODXGRSHULFLDO
FRQWUDVXDSHVVRD
$UW (VWDQGRSUHVRRVHUYLGRUDFXVDGRDSOLFDVHRVHJXLQWH
,, LQWLPDGRRXQRWLILFDGRGRVDWRVSURFHVVXDLV
, DFLWDomRLQLFLDOHDLQWLPDomRSDUDGHIHVDHVFULWDVmR
,,, LQWLPDGRSHVVRDOPHQWHSDUDDSUHVHQWDomRGHGHIHVD SURPRYLGDVRQGHHOHHVWLYHUUHFROKLGR
HVFULWDQDIRUPDGRDUW
,, RDFRPSDQKDPHQWRGRSURFHVVRGLVFLSOLQDUpSURPRYL
,9 LQWLPDGR GD GHFLVmR SURIHULGD HP VLQGLFkQFLDRX GRSRUSURFXUDGRUSRUHOHGHVLJQDGRRXQDDXVrQFLD
SURFHVVRGLVFLSOLQDUVHPVXVSHQVmRGRVHIHLWRVGHFRU SRUGHIHQVRUGDWLYR
UHQWHV GD SXEOLFDomR QR 'LiULR 2ILFLDO GR 'LVWULWR
)HGHUDO ,,, RLQWHUURJDWyULRpUHDOL]DGRHPORFDODSURSULDGRQD
IRUPDSUHYLDPHQWHDFRUGDGDFRPDDXWRULGDGHFRPSH
†~QLFR $LQWLPDomRGHTXHWUDWDRLQFLVR,,GHYHVHUIHLWDFRP WHQWH
DQWHFHGrQFLDPtQLPDGHWUrVGLDVGDGDWDGHFRPSDUH &DStWXOR,9
FLPHQWR '$&20,66®2352&(66$17(

$UW $RVHUYLGRUDFXVDGRpIDFXOWDGR $UW $ VLQGLFkQFLD RX R SURFHVVR GLVFLSOLQDU pFRQGX]LGR


SRUFRPLVVmRSURFHVVDQWHGHFDUiWHUSHUPDQHQWHRX
, DUJXLUDLQFRPSHWrQFLDRLPSHGLPHQWRRXDVXVSHL HVSHFLDO
omR
†ž $ FRPLVVmR p FRPSRVWD GH WUrVVHUYLGRUHVHVWiYHLV
D GDDXWRULGDGHLQVWDXUDGRUDRXMXOJDGRUDGDVLQGL GHVLJQDGRVSHODDXWRULGDGHFRPSHWHQWH
FkQFLDRXSURFHVVRGLVFLSOLQDU
†ž 2VPHPEURVGDFRPLVVmRSURFHVVDQWHVmRHVFROKLGRV
E GHTXDOTXHUPHPEURGDFRPLVVmRSURFHVVDQWH SHOD DXWRULGDGH FRPSHWHQWH HQWUH RV RFXSDQWHV GH
FDUJR SDUD R TXDO VH H[LMD HVFRODULGDGH LJXDO RX
,, FRQVWLWXLUSURFXUDGRU VXSHULRUjGRVHUYLGRUDFXVDGR

,,, DFRPSDQKDUGHSRLPHQWRGHWHVWHPXQKDSHVVRDOPHQ †ž 1RVFDVRVGHFDUUHLUDRUJDQL]DGDHPQtYHOKLHUiUTXLFR


WHRXSRUVHXSURFXUDGRU RV PHPEURV GD FRPLVVmR GHYHP VHU RFXSDQWHV GH
FDUJRHIHWLYRVXSHULRURXGRPHVPRQtYHOGRVHUYLGRU
,9 DUURODUWHVWHPXQKD DFXVDGR

9 UHLQTXLULUWHVWHPXQKDSRULQWHUPpGLRGRSUHVLGHQWH †ž &RPSHWHDRSUHVLGHQWHGDFRPLVVmRPDQWHUDRUGHP


GDFRPLVVmRSURFHVVDQWH HDVHJXUDQoDGDVDXGLrQFLDVSRGHQGRUHTXLVLWDUIRUoD
SROLFLDOVHQHFHVViULD
9, FRQWUDGLWDUWHVWHPXQKD
†ž $&RPLVVmRWHPFRPRVHFUHWiULRVHUYLGRUGHVLJQDGR
9,, SURGX]LUSURYDVHFRQWUDSURYDV SHOR VHX SUHVLGHQWH SRGHQGR D LQGLFDomR UHFDLU HP
XPGHVHXVPHPEURV
9,,, IRUPXODUTXHVLWRVQRFDVRGHSURYDSHULFLDO
6HFUHWDULDGH(VWDGRGH$ GPLQLVWUDomR3~EOLFD'LVWULWR)HGHUDO
 /HJLVODomR 

†ž $FRPLVVmRSURFHVVDQWHTXDQGRSHUPDQHQWHGHYHVHU †~QLFR 2SUHVLGHQWHGDFRPLVVmRGHVLQGLFkQFLDRXGHSURFHV


UHQRYDGD QR PtQLPR D FDGD GRLV DQRV YHGDGR DR VRGLVFLSOLQDUSRGHUHTXLVLWDUDSRLRLQFOXVLYHSROLFLDO
PHVPRPHPEURVHUYLUSRUPDLVGHTXDWURDQRVFRQVH GRVyUJmRVGDDGPLQLVWUDomRS~EOLFDSDUDUHDOL]DomR
FXWLYRV GHGLOLJrQFLDVHJXUDQoDRXORFRPRomRDWpRORFDOGH
FROHWDGHSURYDRXGHUHDOL]DomRGHDWRSURFHVVXDO
†ž 1DVOLFHQoDVDIDVWDPHQWRVIpULDVHGHPDLVDXVrQFLDV
GH PHPEUR GD FRPLVVmR SURFHVVDQWH D DXWRULGDGH $UW $V UHXQL}HV GD FRPLVVmR SURFHVVDQWH WrP GHVHU
FRPSHWHQWHSRGHGHVLJQDUVXEVWLWXWRHYHQWXDO UHJLVWUDGDV HP DWD GD TXDO GHYH FRQVWDU R GHWDOKD
PHQWRGDVGHOLEHUDo}HVDGRWDGDV
†ž 2ORFDOHRVUHFXUVRVPDWHULDLVSDUDRIXQFLRQDPHQWR
GRV WUDEDOKRV GD FRPLVVmR SURFHVVDQWH GHYHP VHU $UW 6HPSUHTXHQHFHVViULRDFRPLVVmRSURFHVVDQWHGHYH
IRUQHFLGRVSHODDXWRULGDGHLQVWDXUDGRUDGDVLQGLFkQFLD GHGLFDUWHPSRLQWHJUDODRVVHXVWUDEDOKRVILFDQGRVHXV
RXGRSURFHVVRGLVFLSOLQDU PHPEURVGLVSHQVDGRVGRVWUDEDOKRVQDUHSDUWLomRGH
RULJHPDWpDHQWUHJDGRUHODWyULRILQDO
†ž 3RGHPSDUWLFLSDUFRPRPHPEURVGDFRPLVVmRSURFHV
VDQWH VHUYLGRUHV LQWHJUDQWHV GH RXWURV yUJmRV GD $UW 6mRDVVHJXUDGDVSDVVDJHQVHGLiULDVDRVPHPEURVGD
DGPLQLVWUDomRS~EOLFDGLVWLQWRVGDTXHOHRQGHRFRUUH FRPLVVmR H DR VHUYLGRU DFXVDGR QRV FDVRV GH DWRV
UDPDVLQIUDo}HVGLVFLSOLQDUHVVHFRQYHQLHQWHSDUDR SURFHVVXDLV VHUHP SUDWLFDGRV IRUD GR WHUULWyULR GD
LQWHUHVVHS~EOLFR 5,'(

† $FRPLVVmRIXQFLRQDFRPDSUHVHQoDGHWRGRVRVVHXV &DStWXOR9


PHPEURV '$6)$6(6352&(668$,6

$UW 2VHUYLGRUQmRSRGHSDUWLFLSDUGHFRPLVVmRSURFHVVDQ 6HomR,


WHTXDQGRRVHUYLGRUDFXVDGRIRUSHVVRDGHVXDIDPt 'DV'LVSRVLo}HV*HUDLV
OLDVHXSDGUDVWRPDGUDVWDHQWHDGRRXSDUHQWHQD
IRUPDGDOHLFLYLO $UW 2 SURFHVVR GLVFLSOLQDU GHVHQYROYHVH QDVVHJXLQWHV
IDVHV
†ž 7DPEpPQmRSRGHSDUWLFLSDUGHFRPLVVmRSURFHVVDQWH
RVHUYLGRUTXH , LQVWDXUDomR
,, LQVWUXomR
, VHMD DPLJR tQWLPR RX LQLPLJR FDSLWDO FUHGRURX ,,, GHIHVD
GHYHGRUWXWRURXFXUDGRUGRVHUYLGRUDFXVDGR ,9 UHODWyULR
9 MXOJDPHQWR
,, VHMDWHVWHPXQKDRXSHULWRQRSURFHVVRGLVFLSOLQDU 6HomR,,
'D,QVWDXUDomR
,,, WHQKDVLGRDXWRUGHUHSUHVHQWDomRREMHWRGDDSXUDomR
$UW 2 SURFHVVR GLVFLSOLQDU p LQVWDXUDGR SHODDXWRULGDGH
,9 WHQKDDWXDGRHPVLQGLFkQFLDDXGLWRULDRXLQYHVWLJD FRPSHWHQWH
omR GD TXDO UHVXOWRX D VLQGLFkQFLD RX R SURFHVVR
GLVFLSOLQDU $UW 3DUD D LQVWDXUDomR GH SURFHVVR GLVFLSOLQDUGHYH
FRQVWDUGRVDXWRV
9 DWXH RX WHQKD DWXDGR FRPR SURFXUDGRU GRVHUYLGRU
DFXVDGR , DLQGLFDomRGDDXWRULDFRPQRPHPDWUtFXODHFDUJR
GRVHUYLGRU
9, WHQKD LQWHUHVVH HP GHFLVmR DGPLQLVWUDWLYD DVHU
WRPDGDSHORVHUYLGRUDFXVDGR ,, DPDWHULDOLGDGHGDLQIUDomRGLVFLSOLQDU

9,, WHQKDLQWHUHVVHQRDVVXQWRTXHUHVXOWRXQDLQVWDXUDomR †~QLFR $LQVWDXUDomRGHSURFHVVRGLVFLSOLQDUGHSHQGHGHDWR


GDVLQGLFkQFLDRXGRSURFHVVRGLVFLSOLQDU SXEOLFDGRQR'LiULR2ILFLDOGR'LVWULWR)HGHUDOGRTXDO
FRQVWH
9,,, HVWHMDOLWLJDQGRMXGLFLDORXDGPLQLVWUDWLYDPHQWHFRP
RVHUYLGRUVLQGLFDGRDFXVDGRRXLQGLFLDGRRXFRPR , DFRPLVVmRSURFHVVDQWH
UHVSHFWLYRF{QMXJHRXFRPSDQKHLUR
,, R Q~PHUR GR SURFHVVR TXH FRQWpP DVLQIRUPDo}HV
,; UHVSRQGDDVLQGLFkQFLDRXSURFHVVRGLVFLSOLQDU SUHYLVWDVQRFDSXW,H,,

; WHQKDVLGRSXQLGRSRUTXDOTXHULQIUDomRGLVFLSOLQDU $UW ,QVWDXUDGRRSURFHVVRGLVFLSOLQDURVHUYLGRUDFXVDGR


UHVVDOYDGRRGLVSRVWRQRDUW GHYHVHUFLWDGRSDUDVHTXLVHUDFRPSDQKDURSURFHVVR
SHVVRDOPHQWHRXSRULQWHUPpGLRGHSURFXUDGRU
;, VHMD F{QMXJH FRPSDQKHLUR SDGUDVWRPDGUDVWD
HQWHDGR RX SDUHQWH QD IRUPD GD OHL FLYLO GH RXWUR †ž $FLWDomRGHYHVHUDFRPSDQKDGDGHFySLDHOHWU{QLFD
PHPEURGDPHVPDFRPLVVmRSURFHVVDQWH RXHPSDSHOGDVSHoDVSURFHVVXDLVSUHYLVWDVQRDUW
HFRQWHUQ~PHURGRWHOHIRQHPHLRHOHWU{QLFRSDUD
$UW $ FRPLVVmR SURFHVVDQWH H[HUFH VXDV DWLYLGDGHVFRP FRPXQLFDomR HQGHUHoR KRUiULR H GLDV GH IXQFLRQD
LQGHSHQGrQFLDHLPSDUFLDOLGDGHDVVHJXUDGRRDFHVVR PHQWRGDFRPLVVmRSURFHVVDQWH
QDVUHSDUWLo}HVS~EOLFDVDLQIRUPDo}HVGRFXPHQWRV
H DXGLrQFLDV QHFHVViULRV j HOXFLGDomR GR IDWR HP †ž 2 VHUYLGRU DFXVDGR TXH PXGDU GHUHVLGrQFLDILFD
DSXUDomR REULJDGRDFRPXQLFDUjFRPLVVmRSURFHVVDQWHROXJDU
RQGHSRGHVHUHQFRQWUDGR
  /HJLVODomR 6HFUHWDULDGH(VWDGRGH$GPLQLVWUDomR3~EOLFD'LVWULWR)HGHUDO


†ž (VWDQGRRVHUYLGRUDFXVDGRHPORFDOLQFHUWRRXQmR , GHFDUiWHUVLJLORVRUHTXHULGRVSHODFRPLVVmRSURFHVVDQ


VDELGRDFLWDomRGHTXHWUDWDHVWHDUWLJRpIHLWDSRU WHRXDHODHQWUHJXHVSHORVHUYLGRUDFXVDGRRXLQGLFLD
HGLWDOSXEOLFDGRQR'LiULR2ILFLDOGR'LVWULWR)HGHUDO GR
HHPMRUQDOGHJUDQGHFLUFXODomRQR'LVWULWR)HGHUDO
,, VREUHDVLWXDomRHFRQ{PLFDILQDQFHLUDRXSDWULPRQLDO
†ž 6HQRSUD]RGHTXLQ]HGLDVFRQWDGRVGDSXEOLFDomRGH GRVHUYLGRUDFXVDGRRXLQGLFLDGR
TXHWUDWDR†žRVHUYLGRUDFXVDGRQmRVHDSUHVHQWDU
j FRPLVVmR SURFHVVDQWH D DXWRULGDGH LQVWDXUDGRUD ,,, VREUH DV IRQWHV GH UHQGD GR VHUYLGRU DFXVDGRRX
GHYH GHVLJQDU GHIHQVRU GDWLYR SDUD DFRPSDQKDU R LQGLFLDGR
SURFHVVRGLVFLSOLQDUHQTXDQWRRVHUYLGRUDFXVDGRQmR
VHDSUHVHQWDU ,9 VREUHRVUHODFLRQDPHQWRVSHVVRDLVGRVHUYLGRUDFXVDGR
6HomR,,, RXLQGLFLDGR
'D,QVWUXomR
†ž 2V GRFXPHQWRV GH TXH WUDWD R † ž VmRGHDFHVVR
UHVWULWR
$UW 1D IDVH GD LQVWUXomR D FRPLVVmR SURFHVVDQWHGHYH
SURPRYHUWRPDGDGHGHSRLPHQWRVDFDUHDo}HVLQYHVWL
, DRVPHPEURVGDFRPLVVmRSURFHVVDQWH
JDo}HVHGLOLJrQFLDVFDEtYHLVREMHWLYDQGRDFROHWDGH ,, DRVHUYLGRUDFXVDGRRXDRVHXSURFXUDGRU
SURYD UHFRUUHQGR TXDQGR QHFHVViULR D WpFQLFRV H ,,, DRVDJHQWHVS~EOLFRVTXHGHYDPDWXDUQRSURFHVVR
SHULWRVGHPRGRDSHUPLWLUDFRPSOHWDHOXFLGDomRGRV
IDWRV †ž 2V GRFXPHQWRV HP LGLRPD HVWUDQJHLURWUD]LGRVDRV
DXWRVSHODFRPLVVmRSURFHVVDQWHGHYHPVHUWUDGX]LGRV
$UW 3DUD D SURGXomR GH SURYDV D FRPLVVmRSURFHVVDQWH SDUD D OtQJXD SRUWXJXHVD GLVSHQVDGD D WUDGXomR
SRGHGHRItFLRRXDUHTXHULPHQWRGRVHUYLGRUDFXVD MXUDPHQWDGD VH QmR KRXYHU FRQWURYpUVLD UHOHYDQWH
GR SDUDRMXOJDPHQWRGDLQIUDomRGLVFLSOLQDU

, WRPDUGHSRLPHQWRVGHWHVWHPXQKDV $UW $V WHVWHPXQKDV VmR LQWLPDGDV D GHSRUPHGLDQWH


PDQGDGR H[SHGLGR SHOR SUHVLGHQWH GD FRPLVVmR
,, ID]HUDFDUHDo}HV SURFHVVDQWHGHYHQGRDVHJXQGDYLDFRPRFLHQWHGR
LQWHUHVVDGRVHUDQH[DGDDRVDXWRV
,,, FROKHUSURYDVGRFXPHQWDLV
†ž 6HDWHVWHPXQKDIRUVHUYLGRUS~EOLFRDH[SHGLomRGR
,9 FROKHUSURYDVHPSUHVWDGDVGHSURFHVVRVDGPLQLVWUDWL PDQGDGRGHYHVHUFRPXQLFDGDDRFKHIHGDUHSDUWLomR
YRVRXMXGLFLDLV RQGHWHPH[HUFtFLRFRPDLQGLFDomRGRGLDHGDKRUD
PDUFDGRVSDUDLQTXLULomR
9 SURFHGHU j UHFRQVWLWXLomR VLPXODGD GRV IDWRVGHVGH
TXHQmRRIHQGDDPRUDORXRVERQVFRVWXPHV †ž $ DXVrQFLD LQMXVWLILFDGD GH VHUYLGRUS~EOLFRGHYLGD
PHQWHLQWLPDGRFRPRWHVWHPXQKDGHYHVHUFRPXQLFD
9, VROLFLWDUSRULQWHUPpGLRGDDXWRULGDGHFRPSHWHQWH GDjDXWRULGDGHFRPSHWHQWHSDUDDSXUDomRGHUHVSRQ
VDELOLGDGH
D UHDOL]DomRGHEXVFDVHDSUHHQV}HV
$UW 2GHSRLPHQWRGHWHVWHPXQKDpIHLWRRUDOPHQWHVRE
FRPSURPLVVRHUHGX]LGRDWHUPRQmRVHQGROtFLWRj
E LQIRUPDo}HVj)D]HQGD3~EOLFDQDIRUPDDXWRUL]D
WHVWHPXQKDWUD]rORSRUHVFULWR
GDQDOHJLVODomR
†ž $VWHVWHPXQKDVVmRLQTXLULGDVVHSDUDGDPHQWH
F TXHEUDGRVLJLOREDQFiULRRXWHOHI{QLFR
†ž 1DKLSyWHVHGHGHSRLPHQWRVFRQWUDGLWyULRVRXTXHVH
G DFHVVR DRV UHODWyULRV GH XVR IHLWR SHOR VHUYLGRU LQILUPHP SRGHVH SURFHGHU j DFDUHDomR HQWUH RV
DFXVDGRHPVLVWHPDLQIRUPDWL]DGRRXDDWRVTXHHOH GHSRHQWHV
WHQKDSUDWLFDGR
†ž 2VHUYLGRUDFXVDGRVHXSURFXUDGRURXDPERVSRGHP
H H[DPHGHVDQLGDGHPHQWDOGRVHUYLGRUDFXVDGRRX DVVLVWLUjLQTXLULomRGDVWHVWHPXQKDVVHQGROKHV
LQGLFLDGR
, YHGDGRLQWHUIHULUQDVSHUJXQWDVHQDVUHVSRVWDV
9,, GHWHUPLQDUDUHDOL]DomRGHSHUtFLDV ,, IDFXOWDGRUHLQTXLULODVSRULQWHUPpGLRGRSUHVLGHQWH
GDFRPLVVmRSURFHVVDQWH
9,,, SURFHGHUDRLQWHUURJDWyULRGRVHUYLGRUDFXVDGR
$UW &RQFOXtGDDLQTXLULomRGDVWHVWHPXQKDVHDFROHWDGDV
†ž 2SUHVLGHQWHGDFRPLVVmRSURFHVVDQWHSRUGHVSDFKR GHPDLVSURYDVDFRPLVVmRSURFHVVDQWHGHYHSURPRYHU
IXQGDPHQWDGRSRGHLQGHIHULU R LQWHUURJDWyULR GR VHUYLGRU DFXVDGR REVHUYDGRV RV
SURFHGLPHQWRVSUHYLVWRVQRVDUWVH
, SHGLGRV FRQVLGHUDGRV LPSHUWLQHQWHVPHUDPHQWH
SURWHODWyULRVRXGHQHQKXPLQWHUHVVHSDUDRHVFODUHFL †ž 1RFDVRGHPDLVGHXPVHUYLGRUDFXVDGRRLQWHUURJD
PHQWRGRVIDWRV WyULRpIHLWRHPVHSDUDGRHKDYHQGRGLYHUJrQFLDHQWUH
VXDVGHFODUDo}HVVREUHIDWRVRXFLUFXQVWkQFLDVSRGH
,, SHGLGRGHSURYDSHULFLDOTXDQGRDFRPSURYDomRGR VHUSURPRYLGDDDFDUHDomRHQWUHHOHV
IDWRLQGHSHQGHUGHFRQKHFLPHQWRHVSHFLDO
†ž 2 QmR FRPSDUHFLPHQWR GR VHUYLGRUDFXVDGRDR
†ž 6mR FODVVLILFDGRV FRPRFRQILGHQFLDLVLGHQWLILFDGRV LQWHUURJDWyULRRXDVXDUHFXVDHPVHULQWHUURJDGRQmR
SHOD FRPLVVmR SURFHVVDQWH H DXWXDGRV HP DXWRV REVWDRSURVVHJXLPHQWRGRSURFHVVRQHPpFDXVDGH
DSDUWDGRVRVGRFXPHQWRV QXOLGDGH
6HFUHWDULDGH(VWDGRGH$ GPLQLVWUDomR3~EOLFD'LVWULWR)HGHUDO
 /HJLVODomR 

†ž 2 SURFXUDGRU GR VHUYLGRU DFXVDGR SRGHDVVLVWLUDR †ž 2HGLWDOGHFLWDomRGHYHVHUSXEOLFDGRQR'LiULR2ILFLDO


LQWHUURJDWyULRVHQGROKHYHGDGRLQWHUIHULUQDVSHUJXQ GR'LVWULWR)HGHUDOHHPMRUQDOGHJUDQGHFLUFXODomR
WDVHQDVUHVSRVWDVIDFXOWDQGRVHOKHSRUpPSURSRU QR'LVWULWR)HGHUDO
SHUJXQWDVSRULQWHUPpGLRGRSUHVLGHQWHGDFRPLVVmR
SURFHVVDQWHDSyVDLQTXLULomRRILFLDO †ž 1D KLSyWHVH GHVWH DUWLJR R SUD]R SDUD GHIHVDpGH
TXLQ]HGLDVFRQWDGRVGD~OWLPDSXEOLFDomRGRHGLWDO
$UW (QFHUUDGDDLQVWUXomRHWLSLILFDGDDLQIUDomRGLVFLSOL
QDUGHYHVHUIRUPXODGDDLQGLFLDomRGRVHUYLGRUFRP $UW &RQVLGHUDVHUHYHORVHUYLGRULQGLFLDGRTXHUHJXODU
DHVSHFLILFDomRGRVIDWRVDHOHLPSXWDGRVHGDVUHVSHF PHQWHLQWLPDGRQmRDSUHVHQWDUGHIHVDQRSUD]ROHJDO
WLYDVSURYDV
†ž $UHYHOLDGHYHVHUGHFODUDGDHPWHUPRVXEVFULWRSHORV
LQWHJUDQWHV GD FRPLVVmR SURFHVVDQWH QRV DXWRV GR
†ž 1mRFDEHDLQGLFLDomRGRVHUYLGRUVHFRPDVSURYDV
SURFHVVRGLVFLSOLQDU
FROKLGDVILFDUFRPSURYDGRTXH
†ž 3DUDGHIHQGHURVHUYLGRUUHYHODDXWRULGDGHLQVWDXUD
, QmRKRXYHDLQIUDomRGLVFLSOLQDU GRUDGRSURFHVVRGHYHGHVLJQDUXPVHUYLGRUHVWiYHO
,, RVHUYLGRUDFXVDGRQmRIRLRDXWRUGDLQIUDomRGLVFLSOL FRPR GHIHQVRU GDWLYR RFXSDQWH GH FDUJR GH QtYHO
QDU LJXDORXVXSHULRUDRGRVHUYLGRULQGLFLDGRSUHIHUHQFL
,,, DSXQLELOLGDGHHVWHMDH[WLQWD DOPHQWHFRPIRUPDomRHP'LUHLWR

†ž 2FRUUHQGRDKLSyWHVHGR†žDFRPLVVmRSURFHVVDQWH $UW 2SUD]RSDUDDSUHVHQWDUGHIHVDHVFULWDpGHGH]GLDV


GHYHHODERUDURVHXUHODWyULRFRQFOXLQGRSHORDUTXLYD
PHQWRGRVDXWRV †ž +DYHQGRGRLVRXPDLVVHUYLGRUHVLQGLFLDGRVRSUD]Rp
FRPXPHGHYLQWHGLDV
6HomR,9
'D'HIHVD †ž 2 SUD]R GH GHIHVD SRGH VHU SURUURJDGRSHORGREUR
SDUDGLOLJrQFLDVUHSXWDGDVLQGLVSHQViYHLV
$UW 2 VHUYLGRU XPD YH] LQGLFLDGR GHYH VHULQWLPDGR
SHVVRDOPHQWHSRUPDQGDGRH[SHGLGRSHORSUHVLGHQWH $UW &XPSULGDVHYHQWXDLVGLOLJrQFLDVUHTXHULGDVQDGHIHVD
GD FRPLVVmR SURFHVVDQWH SDUD DSUHVHQWDU GHIHVD HVFULWDDFRPLVVmRSURFHVVDQWHGHYHGHFODUDUHQFHUUD
HVFULWDQRSUD]RGRDUW GDVDVIDVHVGHLQVWUXomRHGHIHVD

†~QLFR $FRPLVVmRSRGHDOWHUDUDLQGLFLDomRIRUPDOL]DGDRX
†ž $FLWDomRGHTXHWUDWDRDUW†žQmRH[FOXLR
SURSRU D DEVROYLomR GR VHUYLGRU DFXVDGR HP IXQomR
FXPSULPHQWRGRGLVSRVWRQHVWHDUWLJR
GRVIDWRVKDYLGRVGDVGLOLJrQFLDVUHDOL]DGDV
†ž 1R FDVR GH UHFXVD GR VHUYLGRU LQGLFLDGR HPDSRUR 6HomR9
FLHQWH QD FySLD GD LQWLPDomR R SUD]R SDUD GHIHVD 'R5HODWyULR
FRQWDVH GD GDWD GHFODUDGD HP WHUPR SUySULR SHOR
PHPEURRXVHFUHWiULRGDFRPLVVmRSURFHVVDQWHTXHIH] $UW &RQFOXtGD D LQVWUXomR H DSUHVHQWDGD D GHIHVDD
DLQWLPDomRFRPDDVVLQDWXUDGHGXDVWHVWHPXQKDV FRPLVVmRSURFHVVDQWHGHYHHODERUDUUHODWyULRFLUFXQV
WDQFLDGRGRTXDOFRQVWHP
$UW 4XDQGRSRUGXDVYH]HVRPHPEURRXRVHFUHWiULRGD
FRPLVVmR SURFHVVDQWH KRXYHU SURFXUDGR R VHUYLGRU , DVLQIRUPDo}HVVREUHDLQVWDXUDomRGRSURFHVVR
LQGLFLDGRHPVHXGRPLFtOLRUHVLGrQFLDRXUHSDUWLomR
GHH[HUFtFLRVHPRHQFRQWUDUGHYHKDYHQGRVXVSHLWD ,, RUHVXPRGDVSHoDVSULQFLSDLVGRVDXWRVFRPHVSHFLIL
GHRFXOWDomRLQWLPDUDTXDOTXHUSHVVRDGDIDPtOLDRX FDomRREMHWLYDGRVIDWRVDSXUDGRVGDVSURYDVFROKLGDV
HPVXDIDOWDDTXDOTXHUYL]LQKRTXHYROWDUiHPGLDH HGRVIXQGDPHQWRVMXUtGLFRVGHVXDFRQYLFomR
KRUDGHVLJQDGRVDILPGHHIHWXDUDLQWLPDomR
,,, DFRQFOXVmRVREUHDLQRFrQFLDRXUHVSRQVDELOLGDGHGR
†ž 1RGLDHKRUDGHVLJQDGRVRPHPEURRXRVHFUHWiULR VHUYLGRULQGLFLDGRFRPDLQGLFDomRGRGLVSRVLWLYROHJDO
GDFRPLVVmRSURFHVVDQWHGHYHFRPSDUHFHUDRGRPLFtOLR RXUHJXODPHQWDULQIULQJLGREHPFRPRDVFLUFXQVWkQFL
RXjUHVLGrQFLDGRVHUYLGRULQGLFLDGRDILPGH LQWL DVDJUDYDQWHVRXDWHQXDQWHV
PiOR
,9 DLQGLFDomRGDVDQomRDVHUDSOLFDGDHGRGLVSRVLWLYR
†ž 6HRVHUYLGRULQGLFLDGRQmRHVWLYHUSUHVHQWHRPHPEUR GHVWD/HL&RPSOHPHQWDUHPTXHHODVHHQFRQWUD
RXRVHFUHWiULRGDFRPLVVmRSURFHVVDQWHGHYH
$UW $ FRPLVVmR SURFHVVDQWH GHYH UHPHWHU jDXWRULGDGH
LQVWDXUDGRUDRVDXWRVGRSURFHVVRGLVFLSOLQDUFRPR
, LQIRUPDUVHGDVUD]}HVGDDXVrQFLDHGDUSRUIHLWDD
UHVSHFWLYRUHODWyULR
FLWDomRODYUDQGRGHWXGRDUHVSHFWLYDFHUWLGmR
$UW 1D KLSyWHVH GH R UHODWyULR FRQFOXLU TXH DLQIUDomR
,, GHL[DUFySLDGRPDQGDGRGHLQWLPDomRFRPSHVVRDGD GLVFLSOLQDU DSUHVHQWD LQGtFLRV GH LQIUDomR SHQDO D
IDPtOLDGRVHUYLGRULQGLFLDGRRXFRPTXDOTXHUYL]LQKR DXWRULGDGH FRPSHWHQWH GHYH HQFDPLQKDU FySLD GRV
FRQIRUPHRFDVRGHFODUDQGROKHRQRPH DXWRVDR0LQLVWpULR3~EOLFR

$UW -XQWR j LQWLPDomR SDUD DSUHVHQWDU D GHIHVDHVFULWD 6HomR9,


GHYH VHU DSUHVHQWDGD DR VHUYLGRU DFXVDGR FySLD GD 'R-XOJDPHQWR
LQGLFLDomR
$UW 6DOYRGLVSRVLomROHJDOHPFRQWUiULRRMXOJDPHQWRGR
$UW 2VHUYLGRULQGLFLDGRTXHVHHQFRQWUDUHPOXJDULQFHUWR SURFHVVRGLVFLSOLQDUHDDSOLFDomRGDVDQomRGLVFLSOL
HQmRVDELGRGHYHVHULQWLPDGRSRUHGLWDOSDUDDSUH QDU REVHUYDGD D VXERUGLQDomR KLHUiUTXLFD RX D
VHQWDUGHIHVD YLQFXODomRGRVHUYLGRUVmRGDFRPSHWrQFLD
  /HJLVODomR 6HFUHWDULDGH(VWDGRGH$GPLQLVWUDomR3~EOLFD'LVWULWR)HGHUDO


, QR3RGHU/HJLVODWLYRGR3UHVLGHQWHGD&kPDUD/HJLVOD †ž 2MXOJDPHQWRIRUDGRSUD]ROHJDOQmRLPSOLFDQXOLGDGH


WLYDRXGR7ULEXQDOGH&RQWDV GRSURFHVVRREVHUYDGDDSUHVFULomR

,, QR3RGHU([HFXWLYR †ž $DXWRULGDGHTXHGHUFDXVDjSUHVFULomRGHTXHWUDWD


R DUW  SRGH VHU UHVSRQVDELOL]DGD QD IRUPD GR
D GR *RYHUQDGRU TXDQGR VH WUDWDU GH GHPLVVmR &DStWXOR,GR7tWXOR9,
GHVWLWXLomRGHFDUJRHPFRPLVVmRRXFDVVDomRGH
DSRVHQWDGRULDRXGLVSRQLELOLGDGH $UW $DXWRULGDGHMXOJDGRUDGHYHGHFLGLUPRWLYDGDPHQWH
FRQIRUPHDVSURYDVGRVDXWRV
E GH6HFUHWiULRGH(VWDGRRXDXWRULGDGHHTXLYDOHQWH
TXDQGRVHWUDWDUGHVXVSHQVmRVXSHULRUDWULQWDGLDV †ž $DXWRULGDGHMXOJDGRUDSRGHFRQYHUWHURMXOJDPHQWR
RXUHVVDOYDGRRGLVSRVWRQDDOtQHDDGDVGHPDLV
HP GLOLJrQFLD SDUD UHSHWLomR GH DWRV SURFHVVXDLV RX
VDQo}HVDVHUYLGRUTXHDHOHHVWHMDLPHGLDWDPHQWH
FROHWD GH QRYDV SURYDV FDVR VHMD QHFHVViULR SDUD D
VXERUGLQDGR
HOXFLGDomRFRPSOHWDGRVIDWRV
F GH DGPLQLVWUDGRU UHJLRQDO GLULJHQWH GH yUJmR
UHODWLYDPHQWH DXW{QRPR VXEVHFUHWiULR GLUHWRU †ž (PFDVRGHGLYHUJrQFLDFRPDVFRQFOXV}HVGRUHODWyULR
UHJLRQDORXDXWRULGDGHHTXLYDOHQWHDTXHRVHUYLGRU GDFRPLVVmRSURFHVVDQWHDDXWRULGDGHMXOJDGRUDSRGH
HVWHMD PHGLDWD RX LPHGLDWDPHQWH VXERUGLQDGR DJUDYDUDVDQomRGLVFLSOLQDUSURSRVWDDEUDQGiODRX
TXDQGRVHWUDWDUGHVDQomRQmRFRPSUHHQGLGDQDV LVHQWDURVHUYLGRUGHUHVSRQVDELOLGDGH
DOtQHDVDHE
†ž $DXWRULGDGHFRPSHWHQWHSDUDDSOLFDUDVDQomRGLVFL
†ž 1R FDVR GH VHUYLGRU GH DXWDUTXLD RXIXQGDomRGR SOLQDUPDLVJUDYHpWDPEpPFRPSHWHQWHSDUDDSOLFDU
3RGHU([HFXWLYRRMXOJDPHQWRGRSURFHVVRGLVFLSOLQDU VDQomRGLVFLSOLQDUPDLVEUDQGDRXLVHQWDURVHUYLGRUGH
HDDSOLFDomRGDVDQomRGLVFLSOLQDUVmRGDFRPSHWrQ UHVSRQVDELOLGDGHQDVKLSyWHVHVSUHYLVWDVQR†ž
FLD
†ž 6HGLVFRUGDUGDSURSRVWDGHDEVROYLomRRXGDLQRFrQ
, GR*RYHUQDGRUTXDQGRVHWUDWDUGHGHPLVVmRGHVWLWX FLDGRVHUYLGRUDFXVDGRQmRDQWHULRUPHQWHLQGLFLDGR
LomRGHFDUJRHPFRPLVVmRRXFDVVDomRGHDSRVHQWDGR DDXWRULGDGHMXOJDGRUDGHYHGHVLJQDUQRYDFRPLVVmR
ULDRXGLVSRQLELOLGDGH SURFHVVDQWH SDUD HODERUDU D LQGLFLDomR H SUDWLFDU RV
GHPDLVDWRVSURFHVVXDLVSRVWHULRUHV
,, GRUHVSHFWLYRGLULJHQWHPi[LPRTXDQWRVHWUDWDUGH
VDQomRGLVFLSOLQDUQmRFRPSUHHQGLGDQRLQFLVR,GHVWH †ž 9HULILFDGDDH[LVWrQFLDGHYtFLRLQVDQiYHODDXWRULGDGH
SDUiJUDIR MXOJDGRUDGHYHGHFODUDUDQXOLGDGHWRWDORXSDUFLDOGR
SURFHVVRGLVFLSOLQDUHRUGHQDUFRQIRUPHRFDVR
†ž 1R FDVR GH VHUYLGRU GH FRQVHOKR RX RXWURyUJmRGH
GHOLEHUDomRFROHWLYDLQVWLWXtGRQR3RGHU([HFXWLYRR
, DUHDOL]DomRGHGLOLJrQFLD
MXOJDPHQWR GR SURFHVVR GLVFLSOLQDU H D DSOLFDomR GD
,, DUHDEHUWXUDGDLQVWUXomRSURFHVVXDO
VDQomRGLVFLSOLQDUVmRGDFRPSHWrQFLD
,,, D FRQVWLWXLomR GH RXWUD FRPLVVmR SURFHVVDQWHSDUD
, GR*RYHUQDGRUTXDQGRVHWUDWDUGHGHPLVVmRGHVWLWX LQVWDXUDomRGHQRYRSURFHVVR
LomRGHFDUJRHPFRPLVVmRRXFDVVDomRGHDSRVHQWDGR
ULDRXGLVSRQLELOLGDGH †ž 2VDWRVQmRFRQWDPLQDGRVSHORYtFLRGHYHPVHUUHD
SURYHLWDGRV
,, GH6HFUHWiULRGH(VWDGRRXDXWRULGDGHHTXLYDOHQWHD
FXMD6HFUHWDULDGH(VWDGRRFRQVHOKRRXRyUJmRHVWHMD †ž 1HQKXP DWR p GHFODUDGR QXOR VH GDQXOLGDGHQmR
YLQFXODGRTXDQGRVHWUDWDUGHVXVSHQVmR UHVXOWDU SUHMXt]R SDUD D DSXUDomR GRV IDWRV SDUD D
GHIHVDRXSDUDDFRQFOXVmRGRSURFHVVR
,,, GRUHVSHFWLYRSUHVLGHQWHTXDQGRVHWUDWDUGHDGYHU
WrQFLD †ž 2YtFLRDTXHRVHUYLGRUDFXVDGRRXLQGLFLDGRWHQKD
GDGRFDXVDQmRREVWDRMXOJDPHQWRGRSURFHVVR
†ž $ FRPSHWrQFLD SDUD MXOJDU RSURFHVVRGLVFLSOLQDU
UHJXODVHSHODVXERUGLQDomRKLHUiUTXLFDH[LVWHQWHQD $UW 2DWRGHMXOJDPHQWRGRSURFHVVRGLVFLSOLQDUGHYH
GDWDGRMXOJDPHQWR
, PHQFLRQDUVHPSUHRIXQGDPHQWROHJDOSDUDLPSRVLomR
†ž 'D GHFLVmR TXH DSOLFDU VDQomR GHDGYHUWrQFLDRX GDSHQDOLGDGH
VXVSHQVmRFDEHUHFXUVRKLHUiUTXLFRQDIRUPDGRDUW
YHGDGRRDJUDYDPHQWRGDVDQomR ,, LQGLFDUDFDXVDGDVDQomRGLVFLSOLQDU
$UW 1RSUD]RGHYLQWHGLDVFRQWDGRVGRUHFHELPHQWRGRV ,,, VHUSXEOLFDGRQR'LiULR2ILFLDOGR'LVWULWR)HGHUDO
DXWRVGRSURFHVVRGLVFLSOLQDUDDXWRULGDGHFRPSHWHQWH
GHYHSURIHULUVXDGHFLVmR
&DStWXOR9,
'$5(9,6®2'2352&(662
†ž 6HDVDQomRDVHUDSOLFDGDH[FHGHUDDOoDGDGDDXWRUL
GDGHLQVWDXUDGRUDGRSURFHVVRGLVFLSOLQDUHVWHGHYH
VHU HQFDPLQKDGR j DXWRULGDGH FRPSHWHQWH SDUD $UW 2 SURFHVVR GLVFLSOLQDU SRGH VHU UHYLVWR DTXDOTXHU
GHFLGLUQRPHVPRSUD]RGHVWHDUWLJR WHPSRDSHGLGRRXGHRItFLRTXDQGRIRUHPDGX]LGRV
IDWRV QRYRV RX FLUFXQVWkQFLDV QmR DSUHFLDGDV QR
†ž +DYHQGRPDLVGHXPVHUYLGRULQGLFLDGRHGLYHUVLGDGH SURFHVVRRULJLQiULRVXVFHWtYHLVGHMXVWLILFDUDLQRFrQFLD
GHVDQo}HVSURSRVWDVQRUHODWyULRGDFRPLVVmRSURFHV GR VHUYLGRU SXQLGR RX D LQDGHTXDomR GD VDQomR
VDQWHRMXOJDPHQWRHDDSOLFDomRGDVVDQo}HVFDEHj GLVFLSOLQDUDSOLFDGDREVHUYDGRRGLVSRVWRQRDUW
DXWRULGDGH FRPSHWHQWH SDUD D LPSRVLomR GD VDQomR ,,
PDLVJUDYH
6HFUHWDULDGH(VWDGRGH$ GPLQLVWUDomR3~EOLFD'LVWULWR)HGHUDO
 /HJLVODomR 

†ž (PFDVRGHIDOHFLPHQWRDXVrQFLDRXGHVDSDUHFLPHQWR 7tWXOR9,,,


GRVHUYLGRUTXDOTXHUSHVVRDGDIDPtOLDSRGHUHTXHUHU '$6(*85,'$'(62&,$/'26(59,'25
DUHYLVmRGRSURFHVVR
&DStWXOR,
†ž 1RFDVRGHLQFDSDFLGDGHPHQWDOGRVHUYLGRUDUHYLVmR '$6',6326,d¯(6*(5$,6
SRGHVHUUHTXHULGDSHORUHVSHFWLYRFXUDGRU
$UW $ VHJXULGDGH VRFLDO GR VHUYLGRU S~EOLFRGLVWULWDO
†ž $VLPSOHVDOHJDomRGHLQMXVWLoDGDVDQomRGLVFLSOLQDU FRPSUHHQGHXPFRQMXQWRLQWHJUDGRGHDo}HVGHVWLQD
DSOLFDGDQmRFRQVWLWXLIXQGDPHQWRSDUDDUHYLVmR GDVDDVVHJXUDUGLUHLWRVUHODWLYRVjVD~GHjSUHYLGrQ
FLDHjDVVLVWrQFLDVRFLDO
†ž 1mRpDGPLWLGRSHGLGRGHUHYLVmRTXDQGRDSHUGDGR
FDUJRS~EOLFRRXDFDVVDomRGHDSRVHQWDGRULDGHFRUUHU $UW $ SUHYLGrQFLD VRFLDO GHVWLQDVH H[FOXVLYDPHQWHDRV
GHGHFLVmRMXGLFLDO VHUYLGRUHVRFXSDQWHVGHFDUJRGHSURYLPHQWRHIHWLYR
QD IRUPD SUHYLVWD QD &RQVWLWXLomR )HGHUDO H HP OHL
$UW 1R SURFHVVR UHYLVLRQDO R {QXV GD SURYD FDEHDR FRPSOHPHQWDUHVSHFtILFD
UHTXHUHQWH
$UW $ DVVLVWrQFLD VRFLDO GHYH VHU SUHVWDGD QD IRUPDGD
†~QLFR 1DSHWLomRLQLFLDORUHTXHUHQWHGHYHSHGLUGLDHKRUD OHJLVODomRHVSHFtILFDHVHJXQGRRVSURJUDPDVSDWURFL
SDUDSURGXomRGHSURYDVHLQTXLULomRGDVWHVWHPXQKDV QDGRVSHORyUJmRDXWDUTXLDRXIXQGDomR
TXHDUURODU
&DStWXOR,,
$UW 2 UHTXHULPHQWR GH UHYLVmR GR SURFHVVR GHYHVHU '$$66,67È1&,$­6$Ô'(
GLULJLGRFRQIRUPHRFDVRjDXWRULGDGHDGPLQLVWUDWLYD
TXHMXOJRXRULJLQDULDPHQWHRSURFHVVRGLVFLSOLQDU 6HomR,
'DV'LVSRVLo}HV*HUDLV
†ž $XWRUL]DGDDUHYLVmRRSHGLGRGHYHVHUHQFDPLQKDGR
DRGLULJHQWHGRyUJmRDXWDUTXLDRXIXQGDomRRQGHVH $UW $DVVLVWrQFLDjVD~GHGRVHUYLGRUDWLYRRXLQDWLYRGH
RULJLQRX R SURFHVVR GLVFLSOLQDU SDUD SURYLGHQFLDU D VHXF{QMXJHFRPSDQKHLURGHSHQGHQWHVHGRSHQVLR
FRQVWLWXLomRGHFRPLVVmRUHYLVRUDREVHUYDGDVQRTXH QLVWD FRPSUHHQGH D DVVLVWrQFLD PpGLFD KRVSLWDODU
FRXEHUDVGLVSRVLo}HVGRVDUWVD RGRQWROyJLFDSVLFROyJLFDHIDUPDFrXWLFDHpSUHVWDGD

†ž 1mRSRGHLQWHJUDUDFRPLVVmRUHYLVRUDRVHUYLGRUTXH , SHOR6LVWHPDÔQLFRGH6D~GH


WHQKDDWXDGRQDVLQGLFkQFLDRXQRSURFHVVRGLVFLSOLQDU
FXMRMXOJDPHQWRVHSUHWHQGDUHYLVDU ,, GLUHWDPHQWHSHORVHUYLoRGHVD~GHGRyUJmRDXWDUTXLD
RXIXQGDomRDTXHRVHUYLGRUHVWLYHUYLQFXODGR
$UW $UHYLVmRFRUUHHPDSHQVRDRSURFHVVRRULJLQiULR
,,, SHODUHGHSULYDGDGHVD~GHPHGLDQWHFUHGHQFLDPHQWR
$UW $FRPLVVmRUHYLVRUDWHPRSUD]RGHVHVVHQWDGLDVSDUD SRUFRQYrQLRQDIRUPDHVWDEHOHFLGDHPOHLRXUHJXOD
DFRQFOXVmRGRVWUDEDOKRV PHQWR

$UW $SOLFDPVHDRVWUDEDOKRVGDFRPLVVmRUHYLVRUDQRTXH ,9 QDIRUPDGHDX[tOLRPHGLDQWHUHVVDUFLPHQWRSDUFLDO


FRXEHUDVQRUPDVHSURFHGLPHQWRVGR&DStWXOR9 GRYDORUGHVSHQGLGRFRPSODQRVRXVHJXURVSULYDGRV
GH DVVLVWrQFLD j VD~GH QD IRUPD HVWDEHOHFLGD HP
$UW $FRPSHWrQFLDSDUDMXOJDPHQWRGRSHGLGRGHUHYLVmR UHJXODPHQWR
pGDDXWRULGDGHDGPLQLVWUDWLYDTXHDSOLFRXRULJLQDULD
PHQWHDVDQomRGLVFLSOLQDU $UW 2 VHUYLGRU GHYH VHU VXEPHWLGR D H[DPHVPpGLFRV
SHULyGLFRVJUDWXLWRVQRVWHUPRVHFRQGLo}HVGHILQLGRV
†~QLFR 2SUD]RSDUDMXOJDPHQWRpGHYLQWHGLDVFRQWDGRVGR HPUHJXODPHQWR
UHFHELPHQWRGRVDXWRVGRSURFHVVRGLVFLSOLQDUGXUDQWH
RTXDODDXWRULGDGHMXOJDGRUDSRGHGHWHUPLQDUGLOL 6HomR,,
JrQFLDV 'D/LFHQoD0pGLFDHGD/LFHQoD2GRQWROyJLFD
$UW -XOJDGD SURFHGHQWH D UHYLVmR VHUi GHFODUDGDVHP $UW 3RGH VHU FRQFHGLGD OLFHQoD PpGLFD RXRGRQWROyJLFD
HIHLWRDSHQDOLGDGHDSOLFDGD SDUDRVHUYLGRUWUDWDUGDSUySULDVD~GHVHPSUHMXt]R
GDUHPXQHUDomRRXGRVXEVtGLR QRYDUHGDomRGDGDDRDUWSHOD
†ž 6H D FRQFOXVmR VREUH R SHGLGR GH UHYLVmRIRUSHOD /HL&RPSOHPHQWDUQžGH
LQRFrQFLDGRVHUYLGRUSXQLGRGHYHVHUGHFODUDGDVHP
HIHLWRDVDQomRGLVFLSOLQDUDSOLFDGDUHVWDEHOHFHQGRVH †ž $SyVPHVHVFRQVHFXWLYRVGHOLFHQoDSDUDWUDWDPHQ
WRGRV RV GLUHLWRV GR VHUYLGRU H[FHWR HP UHODomR j WR GH VD~GH RX  PHVHV FXPXODWLYRV DR ORQJR GR
GHVWLWXLomR GH FDUJR HP FRPLVVmR TXH GHYH VHU WHPSR GH VHUYLoR SUHVWDGR DR 'LVWULWR )HGHUDO HP
FRQYHUWLGDHPH[RQHUDomR FDUJRHIHWLYRHPUD]mRGDPHVPDGRHQoDRVHUYLGRU
GHYHVHUVXEPHWLGRjSHUtFLDPpGLFDTXHRSLQDUiSHOD
†ž 6H D FRQFOXVmR VREUH R SHGLGR GH UHYLVmRIRUSHOD SRVVLELOLGDGHGHUHWRUQRDRVHUYLoRSHODUHDGDSWDomR
LQDGHTXDomRGDVDQomRGLVFLSOLQDUDSOLFDGDGHYHVH RXSHODDSRVHQWDGRULDSRULQYDOLGH]
SURFHGHUjQRYDDGHTXDomRUHVWDEHOHFHQGRVHWRGRV
RVGLUHLWRVGRVHUYLGRUQDTXLORTXHDVDQomRGLVFLSOLQDU †ž &DVRRVHUYLGRUVHMDUHDGDSWDGRDSyVRSHUtRGRPHQFL
DSOLFDGDWHQKDH[FHGLGR RQDGRQR†žHYROWHDVHDIDVWDUHPUD]mRGDPHVPD
GRHQoDGHYHWHUVHXTXDGURGHVD~GHDQDOLVDGRSRU
$UW 'DUHYLVmRGRSURFHVVRQmRSRGHUHVXOWDUDJUDYDPHQWR -XQWD0pGLFD2ILFLDO
GHVDQomRGLVFLSOLQDU
  /HJLVODomR 6HFUHWDULDGH(VWDGRGH$GPLQLVWUDomR3~EOLFD'LVWULWR)HGHUDO


†ž 1RFDVRGHVHUYLGRUVHPYtQFXORHIHWLYRFRPR'LVWULWR , SUrPLR SHOD DSUHVHQWDomR GH LGHLDV LQYHQWRVRX


)HGHUDOVXDVDXWDUTXLDVRXIXQGDo}HVDSOLFDPVHj WUDEDOKRVTXHIDYRUHoDPRDXPHQWRGHSURGXWLYLGDGH
OLFHQoDPpGLFDRXRGRQWROyJLFDDVQRUPDVGRUHJLPH HDUHGXomRGRVFXVWRVRSHUDFLRQDLV
JHUDOGHSUHYLGrQFLDVRFLDO
,, FRQFHVVmRGHPHGDOKDGLSORPDGHKRQUDDRPpULWR
$UW $OLFHQoDGHTXHWUDWDRDUWGHSHQGHGHLQVSHomR FRQGHFRUDomRHHORJLR
IHLWD SRU PpGLFR RX FLUXUJLmRGHQWLVWD GR VHWRU GH
DVVLVWrQFLDjVD~GH $UW $RV SUD]RV SUHYLVWRV QHVWD /HL &RPSOHPHQWDUVDOYR
GLVSRVLomROHJDOHPFRQWUiULRDSOLFDVHRVHJXLQWH
†ž 6HQHFHVViULRDLQVSHomRGHTXHWUDWDHVWHDUWLJRSRGH
VHUUHDOL]DGDRQGHRVHUYLGRUVHHQFRQWUDU , VXDFRQWDJHPpIHLWDHPGLDVFRUULGRVH[FOXLQGRVHR
GLDGRFRPHoRHLQFOXLQGRVHRGRYHQFLPHQWRILFDQGR
†ž 2DWHVWDGRGHPpGLFRRXGHFLUXUJLmRGHQWLVWDSDUWLFX SURUURJDGRSDUDRSULPHLURGLD~WLOVHJXLQWHRFRPHoR
ODUVySURGX]HIHLWRVGHSRLVGHKRPRORJDGRSHORVHWRU RXRYHQFLPHQWRGRSUD]RTXHFDLUHPGLD
GHDVVLVWrQFLDjVD~GHGRUHVSHFWLYRyUJmRDXWDUTXLD
RXIXQGDomR D VHPH[SHGLHQWH
E GHSRQWRIDFXOWDWLYR
†ž 1R FDVR GH DWHVWDGR GH FRPSDUHFLPHQWRDVHUYLoRV F HPTXHDUHSDUWLomRILFRXIHFKDGD
PpGLFRVRGRQWROyJLFRVRXODERUDWRULDLVDDXVrQFLDDR G FXMR H[SHGLHQWH IRL HQFHUUDGR DQWHV GR KRUiULR
KDELWXDO
VHUYLoR UHVWULQJHVH DR WXUQR HP TXH R VHUYLGRU IRL
DWHQGLGR
,, SHODLQWHUUXSomRH[WLQJXHVHDFRQWDJHPGRSUD]RMi
IHLWDHUHLQLFLDVHQRYDFRQWDJHPDSDUWLUGDGDWDHP
†ž 2DWHVWDGRRXRODXGRGDMXQWDPpGLFDQmRSRGHVH
TXHRSUD]RIRLLQWHUURPSLGR
UHIHULUDRQRPHRXQDWXUH]DGDGRHQoDVDOYRTXDQGR
VHWUDWDUGHOHV}HVSURGX]LGDVSRUDFLGHQWHHPVHUYLoR
,,, GXUDQWHDVXVSHQVmRDFRQWDJHPGRSUD]RILFDSDUDOL
GRHQoDSURILVVLRQDORXTXDOTXHUGDVGRHQoDVHVSHFLIL VDGDGHYHQGRVHUUHWRPDGDGHRQGHSDURXQDGDWDHP
FDGDVQDOHJLVODomRGRUHJLPHSUySULRGHSUHYLGrQFLD TXHFHVVDUDFDXVDVXVSHQVLYD
GRVVHUYLGRUHVS~EOLFRVGR'LVWULWR)HGHUDO
†ž 6DOYR GLVSRVLomR OHJDO HP FRQWUiULR RVSUD]RVVmR
†ž 2DWHVWDGRPpGLFRGHDWpWUrVGLDVGXUDQWHRELPHVWUH FRQWtQXRV QmR VH LQWHUURPSHP QmR VH VXVSHQGHP
GRDQRFLYLOSRGHVHUUHFHELGRSHODFKHILDLPHGLDWD QHPVHSURUURJDP
VHPDKRPRORJDomRGRVHUYLoRGHVD~GH
†ž 2V SUD]RV IL[DGRV HP PHVHV RX DQRVFRQWDPVHGH
$UW 2VHUYLGRUTXHDSUHVHQWDULQGtFLRVGHOHV}HVRUJkQLFDV GDWDDGDWD
RXIXQFLRQDLVGHYHVHUVXEPHWLGRjLQVSHomRPpGLFD
†ž 6HQRPrVGRYHQFLPHQWRQmRKRXYHURGLDHTXLYDOHQ
†~QLFR $ DGPLQLVWUDomR S~EOLFD GHYH DGRWDUSURJUDPDVGH WHDRGRFRPHoRGRSUD]RWHPVHFRPRWHUPRR~OWLPR
SUHYHQomRDPROpVWLDSURILVVLRQDO GLDGRPrV
$UW 2 VHUYLGRU DFLGHQWDGR HP VHUYLoR TXH QHFHVVLWHGH $UW (P UD]mR GH QDFLRQDOLGDGH QDWXUDOLGDGHFRQGLomR
WUDWDPHQWRHVSHFLDOL]DGRSRGHVHUWUDWDGRHPLQVWLWXL VRFLDOItVLFDLPXQROyJLFDVHQVRULDORXPHQWDOQDVFL
omRSULYDGDjVH[SHQVDVGR'LVWULWR)HGHUDO PHQWRLGDGHHVFRODULGDGHHVWDGRFLYLOHWQLDUDoD
FRU VH[R RULHQWDomR VH[XDO FRQYLFomR UHOLJLRVD
†~QLFR 2WUDWDPHQWRUHIHULGRQHVWHDUWLJRFRQVWLWXLPHGLGDGH SROtWLFD RX ILORVyILFD GH WHU FXPSULGR SHQD RX GH
H[FHomR H VRPHQWH p DGPLVVtYHO TXDQGR LQH[LVWLUHP TXDOTXHUSDUWLFXODULGDGHRXFRQGLomRRVHUYLGRUQmR
PHLRVHUHFXUVRVDGHTXDGRVHPLQVWLWXLomRS~EOLFD SRGH
6HomR,,, , VHUSULYDGRGHTXDOTXHUGHVHXVGLUHLWRV
'D5HDGDSWDomR ,, VHU SUHMXGLFDGR HP VHXV GLUHLWRV RX HP VXDYLGD
IXQFLRQDO
$UW $RVHUYLGRUHIHWLYRTXHVRIUHUUHGXomRGDFDSDFLGDGH ,,, VRIUHUGLVFULPLQDomRHPVXDYLGDIXQFLRQDORXSHVVRDO
ODERUDOFRPSURYDGDHPLQVSHomRPpGLFDGHYHPVHU ,9 H[LPLUVHGRFXPSULPHQWRGHVHXVGHYHUHV
SURSRUFLRQDGDVDWLYLGDGHVFRPSDWtYHLVFRPDOLPLWDomR
VRIULGDUHVSHLWDGDDKDELOLWDomRH[LJLGDQRFRQFXUVR $UW $RVHUYLGRUS~EOLFRFLYLOVmRDVVHJXUDGRVQRVWHUPRV
S~EOLFR GD &RQVWLWXLomR )HGHUDO R GLUHLWR j OLYUH DVVRFLDomR
VLQGLFDO H RV VHJXLQWHV GLUHLWRV HQWUH RXWURV GHOD
†~QLFR 2 VHUYLGRU UHDGDSWDGR QmR VRIUH SUHMXt]RHPVXD GHFRUUHQWHV
UHPXQHUDomRRXVXEVtGLR
, UHSUHVHQWDomRSHORVLQGLFDWRLQFOXVLYHFRPRVXEVWLWXWR
7tWXOR,; SURFHVVXDO
&DStWXORÔQLFR
'$6',6326,d¯(6),1$,6(75$16,7Ð5,$6 ,, GHVFRQWRHPIROKDVHP{QXVSDUDDHQWLGDGHVLQGLFDO
DTXHIRUILOLDGRGRYDORUGDVPHQVDOLGDGHVHFRQWULEX
$UW 2GLDGRVHUYLGRUS~EOLFRpFRPHPRUDGRHPYLQWHH Lo}HVGHILQLGDVHPDVVHPEOHLDJHUDOGDFDWHJRULD
RLWRGHRXWXEUR
$UW 3DUDHIHLWRVGHVWD/HL&RPSOHPHQWDUFRQVLGHUDPVH
$UW 3RGHPVHULQVWLWXtGRVRVVHJXLQWHVLQFHQWLYRVIXQFLRQD GDIDPtOLDGRVHUYLGRURF{QMXJHRXRFRPSDQKHLURRV
LVDOpPGDTXHOHVMiSUHYLVWRVQRVUHVSHFWLYRVSODQRVGH ILOKRVHQDIRUPDGDOHJLVODomRIHGHUDOVREUHLPSRVWR
FDUUHLUD GHUHQGDGDSHVVRDItVLFDRVTXHIRUHPVHXVGHSHQ
GHQWHVHFRQ{PLFRV
 6HFUHWDULDGH(VWDGRGH$ GPLQLVWUDomR3~EOLFD'LVWULWR)HGHUDO /HJLVODomR 
†ž 2VHUYLGRUDSRVHQWDGRFRPSURYHQWRSURSRUFLRQDODRWHPSRGHFRQWULEXLomRVHDFRPHWLGR
GHTXDOTXHUGDVPROpVWLDVHVSHFLILFDGDVQR†žGHYHSDVVDUDSHUFHEHUSURYHQWRLQWHJUDO
†ž 2VHUYLGRUSRGHUHTXHUHURUHJLVWURHPVHXVDVVHQWD FDOFXODGRFRPEDVHQRIXQGDPHQWROHJDOGHFRQFHVVmRGDDSRVHQWDGRULD

PHQWRVIXQFLRQDLVGHTXDOTXHUSHVVRDGHVXDIDPtOLD † $GRHQoDOHVmRRXGHILFLrQFLDGHTXHRVHUYLGRUS~EOLFRHUDSRUWDGRUDRLQJUHVVDUQRFDUJR


S~EOLFRQmROKHFRQIHUHRGLUHLWRjDSRVHQWDGRULDSRULQYDOLGH]VDOYRTXDQGRVREUHYLHU
LQFDSDFLGDGHSRUPRWLYRGHSURJUHVVmRRXDJUDYDPHQWRGDVFDXVDVGHGHILFLrQFLD
†ž $GHSHQGrQFLDHFRQ{PLFDGHYHVHUFRPSURYDGDSRU $UW 2VHJXUDGRHPJR]RGHDX[tOLRGRHQoDLQVXVFHSWtYHOGHUHDGDSWDomRGHYHVHUDSRVHQWDGR
RFDVLmR GR SHGLGR H D VXD FRPSURYDomR GHYH VHU SRULQYDOLGH]

UHQRYDGDDQXDOPHQWHQDIRUPDGRUHJXODPHQWR $UW 

†ž $SHQVmRGHYHVHUFRQFHGLGDDRGHSHQGHQWHTXHVHKDELOLWDU
†ž (TXLSDUDPVHjFRQGLomRGHFRPSDQKHLUDRXFRPSD †ž $FRQFHVVmRGDSHQVmRQmRSRGHVHUSURWHODGDSHODIDOWDGHKDELOLWDomRGHRXWURSRVVtYHO
QKHLUR RV SDUFHLURV KRPRDIHWLYRV TXH PDQWHQKDP GHSHQGHQWH

UHODFLRQDPHQWRFLYLOSHUPDQHQWHGHVGHTXHGHYLGD †ž 2 F{QMXJH DXVHQWH QmR H[FOXL GR GLUHLWR j SHQVmR SRU PRUWH R FRPSDQKHLURRXD
PHQWHFRPSURYDGR FRPSDQKHLUD

†ž $KDELOLWDomRSRVWHULRUTXHLPSRUWHLQFOXVmRRXH[FOXVmRGHGHSHQGHQWHVySURGX]HIHLWRV
DFRQWDUGDGDWDGDKDELOLWDomR
$UW $VRULHQWDo}HVQRUPDWLYDVSDUDDXQLIRUPL]DomRGRV
$UW $VSHQV}HVGLVWLQJXHPVHTXDQWRjQDWXUH]DHPYLWDOtFLDVHWHPSRUiULDV
SURFHGLPHQWRVGHDSOLFDomRGHVWD/HL&RPSOHPHQWDU
VmR IRUPXODGDV QR 3RGHU ([HFXWLYR SHOR yUJmR †ž $SHQVmRYLWDOtFLDpFRPSRVWDGHFRWDRXFRWDVSHUPDQHQWHVTXHVRPHQWHVHH[WLQJXHP
RXUHYHUWHPFRPDPRUWHGRSHQVLRQLVWD
FHQWUDOGRVLVWHPDGH
†ž $SHQVmRWHPSRUiULDpFRPSRVWDGHFRWDRXFRWDVTXHSRGHPVHH[WLQJXLURXUHYHUWHUSRU
PRWLYRGHPRUWHFHVVDomRGHLQYDOLGH]RXPDLRULGDGHGRSHQVLRQLVWD
, FRUUHLomR VREUH TXHVW}HV DWLQHQWHV DR UHJLPHj $UW$6mREHQHILFLiULRVGDSHQVmR
VDQomR H DR SURFHVVR GLVFLSOLQDU VHP SUHMXt]R GDV , YLWDOtFLD
FRPSHWrQFLDVGHFRUUHJHGRULDVHVSHFtILFDV
D RF{QMXJH
E D SHVVRD VHSDUDGD MXGLFLDOPHQWH GLYRUFLDGD RX FXMD XQLmR HVWiYHO IRL OHJDOPHQWH
,, SHVVRDO VREUH DV TXHVW}HV QmR FRPSUHHQGLGDVQR F
GLVVROYLGDFRPSHUFHSomRGHSHQVmRDOLPHQWtFLD
RFRPSDQKHLURRXFRPSDQKHLUDTXHFRPSURYHXQLmRHVWiYHO
LQFLVR, G DPmHRXRSDLFRPSHUFHSomRGHSHQVmRDOLPHQWtFLD

,, WHPSRUiULD
$UW $VGLVSRVLo}HVGHVWD/HL&RPSOHPHQWDUQmRDOWHUDPD D R ILOKR RX R HQWHDGR DWp FRPSOHWDU YLQWH H XP DQRV GH LGDGH RX VH LQYiOLGRV
MRUQDGDGHWUDEDOKRYLJHQWHQDGDWDGHVXDSXEOLFDomR E
HQTXDQWRGXUDUDLQYDOLGH]
RPHQRUVREWXWHOD
QmR H[WLQJXHP GLUHLWRV DGTXLULGRV QHP GLUHLWRV RX F RLUPmRQmRHPDQFLSDGRDWpFRPSOHWDUYLQWHHXPDQRVGHLGDGHRXVHLQYiOLGR
HQTXDQWRGXUDUDLQYDOLGH]TXHSHUFHEDSHQVmRDOLPHQWtFLD
GHYHUHVSUHYLVWRVHPOHLHVSHFLDO
†~QLFR eYHGDGDDFRQFHVVmRGHSHQVmRYLWDOtFLD

$UW $Wp TXH OHL HVSHFtILFD IL[H R YDORU GRDX[t , DREHQHILFLiULRLQGLFDGRQRLQFLVR,FVHKRXYHUEHQHILFLiULRLQGLFDGRQRLQFLVR,D
,, DPDLVGHXPFRPSDQKHLURRXFRPSDQKHLUD
OLRDOLPHQWDomRSUHYLVWRQRDUWILFDPPDQWLGRV
$UW%2YDORUGDSHQVmRFDOFXODGRQDIRUPDGRDUWGHYHVHUUDWHDGRHQWUHRVKDELOLWDGRVGH
RVYDORUHVSDJRVQDIRUPDGDOHJLVODomRYLJHQWHDWpD PRGRDLQGLYLGXDOL]DUDFRWDDTXHFDGDEHQHILFLiULRID]MXV
GDWDGHSXEOLFDomRGHVWD/HL&RPSOHPHQWDU †ž 1mRKDYHQGRGHSHQGHQWHVSUHYLVWRVQRDUW$,ERXGRXQRDUW$,,FGHYHVH
REVHUYDUQRFiOFXORGDFRWDGHFDGDSHQVLRQLVWDRVHJXLQWH

$UW )LFDPDQWLGRFRPRVUHVSHFWLYRVHIHLWRVRWHPSRGH , KDYHQGRDSHQDVXPSHQVLRQLVWDKDELOLWDGRRYDORUGDFRWDFRUUHVSRQGHDRYDORUGD


SHQVmR
VHUYLoRUHJXODUPHQWHDYHUEDGRQDIRUPDGDOHJLVODomR ,, RFRUUHQGRKDELOLWDomRjVSHQV}HVYLWDOtFLDHWHPSRUiULD PHWDGHGRYDORUFDEHDRV
DQWHULRUjSXEOLFDomRGHVWD/HL&RPSOHPHQWDU KDELOLWDGRVjSHQVmRYLWDOtFLDDRXWUDPHWDGHDRVKDELOLWDGRVjSHQVmRWHPSRUiULD

†ž +DYHQGRGHSHQGHQWHVSUHYLVWRVQRDUW$,ERXGRXQRDUW$,,FDSOLFDVHR
VHJXLQWH
$UW )LFDP PDQWLGDV DWp VXD DGHTXDomR jVGLVSRVLo}HV
, D FRWD GHVVHV GHSHQGHQWHV p FDOFXODGD GH PRGR SURSRUFLRQDO DR YDORU GDSHQVmR
GHVWD/HL&RPSOHPHQWDUDVQRUPDVUHJXODPHQWDUHV DOLPHQWtFLDSHUFHELGDWHQGRFRPREDVHSDUDFiOFXORRYDORUWRWDOGDSHQVmR
H[SHGLGDV FRP EDVH QD OHJLVODomR DQWHULRU H[FHWR ,, DFRWDGRVGHPDLVGHSHQGHQWHVVHKRXYHUGHYHVHUFDOFXODGDQDIRUPDGR†žWHQGR
FRPREDVHSDUDFiOFXORRVDOGRGRYDORUGDSHQVmRTXHUHPDQHVFHUDSyVGHGX]LUD
QDTXLORTXHFRQIOLWDUHPFRPHVWD/HL&RPSOHPHQWDU FRWDGHTXHWUDWDRLQFLVR,GHVWHSDUiJUDIR

†ž 2YDORUDSXUDGRQDIRUPDGR†ž,ILFDOLPLWDGRSHODFRWDGHYLGDDFDGDEHQHILFLiULRGD
$UW 2GpFLPRWHUFHLURVDOiULRSUHYLVWRQHVWD/HL&RPSOH SHQVmRYLWDOtFLDRXGDSHQVmRWHPSRUiULD

PHQWDUVXEVWLWXLDJUDWLILFDomRQDWDOtFLDSUHYLVWDQD $UW& $ FRWD GR SHQVLRQLVWD TXH SHUGHX HVVD TXDOLGDGH UHYHUWHVH H[FOXVLYDPHQWH SDUD VHX
DVFHQGHQWHGHVFHQGHQWHRXLUPmRTXHWDPEpPVHMDSHQVLRQLVWDGRPHVPRLQVWLWXLGRUGHSHQVmR
/HLQžGHGHGH]HPEURGH
$UW'5HVVDOYDGRRGLUHLWRGHRSomRpYHGDGDDSHUFHSomRFXPXODWLYDGHPDLVGHGXDVSHQV}HV
SDJDVSRUUHJLPHSUySULRGHSUHYLGrQFLDVRFLDO
$UW $VUHPLVV}HVIHLWDVQDOHJLVODomRGLVWULWDODGLVSRVLWLYR
GD/HLIHGHUDOQžGHGHGH]HPEURGH $UW 9(7$'2 
RX D GLVSRVLWLYRV GDV OHLV UHYRJDGDV SRU HVWD /HL
$UW (VWD/HL&RPSOHPHQWDUHQWUDHPYLJRUQRGLDžGHMDQHLURGH
&RPSOHPHQWDU FRQVLGHUDPVH IHLWDV jV GLVSRVLo}HV
FRUUHVSRQGHQWHVGHVWD/HL&RPSOHPHQWDU $UW )LFDP UHYRJDGDV DV GLVSRVLo}HV HP FRQWUiULR GHL[DQGR GHVHU
DSOLFDGDV QR 'LVWULWR )HGHUDO D /HL IHGHUDO Qž  GH  GH
GH]HPEURGHHD/HLIHGHUDOQžGHGHDEULOGH
$UW $/HL&RPSOHPHQWDUQžGHGHMXQKRGH
SDVVDDYLJRUDUFRPDVVHJXLQWHVDOWHUDo}HV $UW 6DOYR DV GLVSRVLo}HV DSOLFiYHLV DRV HPSUHJDGRV GDVHPSUHVDV
S~EOLFDV RX VRFLHGDGHV GH HFRQRPLD PLVWD ILFDP H[SUHVVDPHQWH
$UWž 
UHYRJDGRV
†ž $SOLFDPVHVXEVLGLDULDPHQWHjVGLVSRVLo}HVGHVWD/HL&RPSOHPHQWDUDVQRUPDVGRUHJLPH
MXUtGLFRGRVVHUYLGRUHVS~EOLFRVFLYLVGR'LVWULWR)HGHUDO , DUWžGD/HLQžGHGHVHWHPEURGH
$UW 
,, DUWGD/HLQžGHGHQRYHPEURGH
,,, DUWžGD/HLGHGHGH]HPEURGH
†ž $GHSHQGrQFLDHFRQ{PLFDGRF{QMXJHHGRVILOKRVLQGLFDGRVQRLQFLVR,9pSUHVXPLGDH ,9 DUWGD/HLGHGHGH]HPEURGH
DGDVSHVVRDVLQGLFDGDVQRVLQFLVRV,D,,,GHYHVHUFRPSURYDGD 9 DUWGD/HLGHGHGH]HPEURGH
†ž $ H[LVWrQFLD GH GHSHQGHQWH LQGLFDGR QR LQFLVR ,9 H[FOXL GR GLUHLWR DREHQHItFLRRV 9, DUWžGD/HLQžGHGHDJRVWRGH
LQGLFDGRVQRVLQFLVRV,D,,, 9,, DUWžGD/HLQžGHGHRXWXEURGH
9,,, DUWVHGD/HLQžGHGHDJRVWRGH

,; DUWVžHžGD/HLQžGHGHGH]HPEURGH
$UW $ DSRVHQWDGRULD SRU LQYDOLGH] p GHYLGD DR VHJXUDGR TXH HVWDQGR RX QmR HP JR]RGH ; DUWžGD/HLQžGHGHGH]HPEURGH
DX[tOLRGRHQoDIRUFRQVLGHUDGRLQFDSD]GHUHDGDSWDomRSDUDRH[HUFtFLRGDVDWULEXLo}HV ;, DUWžGD/HLQžGHGHRXWXEURGH
GRFDUJRGHIRUPDFRPSDWtYHOFRPDOLPLWDomRTXHWHQKDVRIULGRHGHYHVHUSDJDFRP
EDVHQDOHJLVODomRYLJHQWHDSDUWLUGDGDWDGDSXEOLFDomRGRUHVSHFWLYRDWRHHQTXDQWRR
;,, DUWVžHžGD/HLQžGHGHMXOKRGH
VHUYLGRUSHUPDQHFHUQHVVDFRQGLomR ;,,, DUWVžžžžHžGD/HLQžGHGHMDQHLURGH
;,9 DUWžGD/HLQžGHGHIHYHUHLURGH
 ;9 DUWžGD/HLQžGHGHMXOKRGH
  /HJLVODomR
 6HFUHWDULDGH(VWDGRGH$GPLQLVWUDomR3~EOLFD'LVWULWR)HGHUDO
;9, /HLQžGHGHMXOKRGH †ž 1mR LQWHJUDP R 5336') RVVHUYLGRUHVRFXSDQWHV
;9,, /HLQžGHGHVHWHPEURGH
H[FOXVLYDPHQWH GH FDUJRV HP FRPLVVmR GHFODUDGRV
;9,,, /HLQžGHGHGH]HPEURGH
;,; /HLQžGHGHMDQHLURGH HPOHLGHOLYUHQRPHDomRHH[RQHUDomREHPFRPRGH
;; /HLQžGHGHMXQKRGH RXWURVFDUJRVWHPSRUiULRVRXGHHPSUHJRVS~EOLFRV
;;, /HLQžGHGHQRYHPEURGH
;;,, /HLQžGHGHVHWHPEURGH
†ž 2V PLOLWDUHV H RV SROLFLDLV FLYLV GR'LVWULWR)HGHUDO
;;,,, /HLQžGHGH]HPEURGH
;;,9 /HLQžGHGHMDQHLURGH SHODVSHFXOLDULGDGHVGLVSRVWDVQD&RQVWLWXLomR)HGHUDO
;;9 /HLQžGHGHMXOKRGH H QD /HL )HGHUDO Qž  GH  GH GH]HPEUR GH
;;9, /HLQžGHGHMXOKRGH TXHLQVWLWXLR)XQGR&RQVWLWXFLRQDOGR'LVWULWR
;;9,, /HLQžGHGHGH]HPEURGH
)HGHUDOWHUmRUHJXODPHQWDomRQR5HJLPH3UySULRGH
;;9,,, /HLQžGHGHMDQHLURGH
;;,; /HLQžGHGHPDLRGH 3UHYLGrQFLD6RFLDOGR'LVWULWR)HGHUDOGHILQLGDHPOHL
;;; /HLQžGHGHMXOKRGH FRPSOHPHQWDUHVSHFtILFD
;;;, /HLQžGHGHQRYHPEURGH
;;;,, /HLQžGHGHQRYHPEURGH †ž $SOLFDPVHVXEVLGLDULDPHQWHjVGLVSRVLo}HVGHVWD/HL
;;;,,, /HLQžGHGHGH]HPEURGH
;;;,9 /HLQžGHGHMDQHLURGH &RPSOHPHQWDU DV QRUPDV GR UHJLPH MXUtGLFR GRV
;;;9 /HLQžGHGHRXWXEURGH VHUYLGRUHV S~EOLFRV FLYLV GR 'LVWULWR )HGHUDO  ILFD
;;;9, /HLQžGHGHQRYHPEURGH DFUHVFHQWDGRR†žDRDUWžSHODOHLFRPSOHPHQWDUQžGH
;;;9,, /HLQžGHGHGH]HPEURGH
;;;9,,, /HLQžGHGHRXWXEURGH $UWž )LFDYHGDGDQRVWHUPRVGHVWD/HL&RPSOHPHQWDUHGR
;;;,; /HLQžGHGHMDQHLURGH DUWLJR†GD&RQVWLWXLomR)HGHUDODH[LVWrQFLD
;/ /HLQžGHGHMDQHLURGH
;/, /HLQžGHGHMDQHLURGH GHPDLVGHXPUHJLPHSUySULRGHSUHYLGrQFLDVRFLDOH
;/,, /HLQžGHGHDEULOGH GHPDLVGHXPDXQLGDGHJHVWRUDGRUHJLPHSUySULRQR
;/,,, /HLQžGHGHDEULOGH kPELWRGR'LVWULWR)HGHUDO
;/,9 /HLQžGHGHPDLRGH
;/9 /HLQžGHGHPDLRGH
;/9, /HLQžGHGHMXQKRGH $UWž )LFDLQVWLWXtGRRyUJmRJHVWRU~QLFRGR5HJLPH3UySULR
;/9,, /HLQžGHGHGH]HPEURGH GH3UHYLGrQFLD6RFLDOGR'LVWULWR)HGHUDOVREDGHQR
;/9,,, /HLQžGHGHMDQHLURGH PLQDomRGH,QVWLWXWRGH3UHYLGrQFLDGRV6HUYLGRUHVGR
;/,; /HLQžGHGHMXOKRGH 'LVWULWR )HGHUDO  ,SUHY') DXWDUTXLD HP UHJLPH
/ /HLQžGHGHGH]HPEURGH
/, /HLQžGHGHMDQHLURGH HVSHFLDOFRPSHUVRQDOLGDGHMXUtGLFDGHGLUHLWRS~EOLFR
/,, /HLQžGHGHDEULOGH GRWDGD GH DXWRQRPLD DGPLQLVWUDWLYD ILQDQFHLUD H
/,,, /HLQžGHGHDJRVWRGH SDWULPRQLDO YLQFXODGD j 6HFUHWDULD GH (VWDGR GH
/,9 /HLQžGHGHQRYHPEURGH 3ODQHMDPHQWRH*HVWmRGR'LVWULWR)HGHUDOHPVXEVWL
/9 /HLQžGHGHPDLRGH
/9, /HLQžGHGHMXOKRGH WXLomR DR ,QVWLWXWR GH 3UHYLGrQFLD H $VVLVWrQFLD GRV
/9,, /HLQžGHžGHMXQKRGH 6HUYLGRUHVGR'LVWULWR)HGHUDOGHQRPLQDGR,SDVIHQRV
WHUPRVGD/HLQžGHGHPDLRGHHGRDUW
 GR$WRGDV'LVSRVLo}HV7UDQVLWyULDVGD/HL2UJkQLFD
GR'LVWULWR)HGHUDO
/HJLVODomRGRVHUYLGRUS~EOLFRGR'LVWULWR)HGHUDO
$UWž 2 ,SUHY') WHP FRPR DWULEXLomR SULQFLSDO FDSWDUH
/HL&RPSOHPHQWDUGLVWULWDOQž FDSLWDOL]DU RV UHFXUVRV QHFHVViULRV j JDUDQWLD GH
SDJDPHQWR GRV EHQHItFLRV SUHYLGHQFLiULRV DWXDLV H
IXWXURVGRVVHJXUDGRVHGHSHQGHQWHVGHTXHWUDWDHVWD
/HL&RPSOHPHQWDUSRUPHLRGHXPDJHVWmRSDUWLFLSD
3XEOLFDomR'2')QžGHž3iJVD
5HJXODPHQWDGDSHOR'HFUHWRQžGHGRGIGH WLYDWUDQVSDUHQWHHILFLHQWHHHILFD]GRWDGDGHFUHGLEL
$OWHUDo}HV
/HL&RPSOHPHQWDUQžGHGRGIGH
OLGDGHHH[FHOrQFLDQRDWHQGLPHQWR
/HL&RPSOHPHQWDUQžGHGRGIGH
/HL&RPSOHPHQWDUQžGHGRGIGH
/HL&RPSOHPHQWDUQžGHGRGIGH †ž 3DUDRVILQVSUHYLVWRVQRFDSXWLQFXPEHPDR,SUHY')
/HL&RPSOHPHQWDUQžGHGRGIGH3DJ6XSOHPHQWR$
RJHUHQFLDPHQWRHDRSHUDFLRQDOL]DomRGR5336')
LQFOXLQGRDDUUHFDGDomRHDJHVWmRGHUHFXUVRVILQDQ
5HRUJDQL]DHXQLILFDR5HJLPH3UySULRGH3UHYLGrQFLD6RFLDOGR FHLURVHSUHYLGHQFLiULRVDFRQFHVVmRRSDJDPHQWRH
'LVWULWR)HGHUDO5336')HGiRXWUDVSURYLGrQFLDV D PDQXWHQomR GRV EHQHItFLRV SUHYLGHQFLiULRV RUD
UHRUJDQL]DGRVHXQLILFDGRVSRUHVWD/HL&RPSOHPHQWDU
7tWXORÔQLFR GHYLGRVDRVVHJXUDGRVHVHXVGHSHQGHQWHV
'25(*,0(35Ð35,2'(35(9,'È1&,$62&,$/
'2',675,72)('(5$/ †ž 2 'LVWULWR )HGHUDO FRQVWLWXLVH HPJDUDQWLGRUGDV
REULJDo}HVGR,SUHY')UHVSRQGHQGRVXEVLGLDULDPHQ
&DStWXOR, WHSHORFXVWHLRGRVEHQHItFLRVSUHYLGHQFLiULRVGHYLGRV
'RÐUJmR*HVWRUGR5HJLPH3UySULRGH3UHYLGrQFLD6RFLDO DRVVHXVVHJXUDGRVHGHSHQGHQWHVFREULQGRTXDOTXHU
GR'LVWULWR)HGHUDOHGRV2EMHWLYRV LQVXILFLrQFLDILQDQFHLUDGR5HJLPH3UySULRGH3UHYL
GrQFLD6RFLDOGR'LVWULWR)HGHUDO
$UWž 2 5HJLPH 3UySULR GH 3UHYLGrQFLD 6RFLDO GR'LVWULWR
)HGHUDO5336')GHTXHWUDWDRDUWGD&RQVWLWXL $UWž 2 ,SUHY') QD FRQVHFXomR GH VXDVILQDOLGDGHV
omR)HGHUDOILFDUHRUJDQL]DGRHXQLILFDGRQRVWHUPRV DWHQGHUiREULJDWRULDPHQWHDRVVHJXLQWHVSULQFtSLRV
GHVWD /HL &RPSOHPHQWDU VHQGR REULJDWRULDPHQWH
ILOLDGRVWRGRVRVVHUYLGRUHVWLWXODUHVGHFDUJRVHIHWLYRV , SURYLPHQWRGHUHJLPHGHSUHYLGrQFLDVRFLDOGHFDUiWHU
DWLYRVHLQDWLYRVHRVSHQVLRQLVWDVGR3RGHU([HFXWLYR FRQWULEXWLYRHVROLGiULRDRVVHJXUDGRVHGHSHQGHQWHV
HGR3RGHU/HJLVODWLYRGR'LVWULWR)HGHUDOLQFOXtGRVR
7ULEXQDOGH&RQWDVGR'LVWULWR)HGHUDODVDXWDUTXLDV ,, FDUiWHU GHPRFUiWLFR H HILFLHQWH GH JHVWmR FRPD
HDVIXQGDo}HVQDTXDOLGDGHGHVHJXUDGRVEHPFRPR SDUWLFLSDomR GH UHSUHVHQWDQWHV GR 3RGHU 3~EOLFR GR
VHXVUHVSHFWLYRVGHSHQGHQWHV 'LVWULWR)HGHUDOGRVVHJXUDGRVHGHSHQGHQWHV
6HFUHWDULDGH(VWDGRGH$ GPLQLVWUDomR3~EOLFD'LVWULWR)HGHUDO
 /HJLVODomR 

,,, WUDQVSDUrQFLDQDJHVWmRGHVHXVUHFXUVRVILQDQFHLURVH †ž 1DKLSyWHVHGHDFXPXODomRGHFDUJRUHPXQHUDGRR


SUHYLGHQFLiULRV VHUYLGRU PHQFLRQDGR QHVWH DUWLJR VHUi VHJXUDGR
REULJDWyULRHPUHODomRDFDGDXPGRVFDUJRVRFXSD
,9 JHVWmRDGPLQLVWUDWLYRILQDQFHLUDDXW{QRPDHPUHODomR GRV
DR'LVWULWR)HGHUDO
†ž 2VHJXUDGRLQDWLYRYLQFXODGRDR5336')TXHH[HUoD
9 FXVWHLRGDSUHYLGrQFLDVRFLDOPHGLDQWHFRQWULEXLo}HV RXYHQKDDH[HUFHUFDUJRHPFRPLVVmRFDUJRWHPSRUi
GRVyUJmRVHGRVVHUYLGRUHVDWLYRVHLQDWLYRVHSHQVLR ULRRXHPSUHJRS~EOLFRYLQFXODVHREULJDWRULDPHQWH
QLVWDVGHTXHWUDWDRDUWžGHVWD/HL&RPSOHPHQWDU DR5HJLPH*HUDOGH3UHYLGrQFLD6RFLDO 5*36
VHJXQGRFULWpULRVVRFLDOPHQWHMXVWRVHDWXDULDOPHQWH
FRPSDWtYHLV †ž 2VHJXUDGRGR5336')PDQWpPDVXDILOLDomRDHVVH
UHJLPHGXUDQWHRDIDVWDPHQWRGRFDUJRHIHWLYRSDUDR
9, SUHVHUYDomRGRHTXLOtEULRILQDQFHLURHDWXDULDO H[HUFtFLRGHPDQGDWRHOHWLYR

†ž 2 VHJXUDGR TXH H[HUoDFRQFRPLWDQWHPHQWHFDUJR


9,, SURLELomR GD FULDomR PDMRUDomR RX H[WHQVmRGH
HIHWLYR H PDQGDWR HOHWLYR GH 9HUHDGRU ILOLDVH DR
TXDLVTXHUEHQHItFLRVRXVHUYLoRVVHPDFRUUHVSRQGHQ
5336')SHORFDUJRHIHWLYRHDR5*36SHORPDQGDWR
WHIRQWHGHFXVWHLRWRWDO HOHWLYR
$UWž 25336')JHULGRSHOR,SUHY')YLVDGDUFREHUWXUD $UW $SHUGDGDFRQGLomRGHVHJXUDGRGR5336')RFRUUH
DRVHYHQWRVDTXHHVWmRVXMHLWRVRVVHXVEHQHILFLiULRV UiQDVKLSyWHVHVGHPRUWHH[RQHUDomRRXGHPLVVmR
HFRPSUHHQGHXPFRQMXQWRGHEHQHItFLRVTXHDWHQGHP
jVVHJXLQWHVILQDOLGDGHV 6HomR,,
'RV'HSHQGHQWHV
, JDUDQWLUPHLRVGHVXEVLVWrQFLDQRVHYHQWRVGHLQYDOL
GH]DFLGHQWHHPVHUYLoRLGDGHDYDQoDGDUHFOXVmRH $UW 6mREHQHILFLiULRVGR5336')QDFRQGLomRGHGHSHQ
PRUWH GHQWHGRVHJXUDGR

,, SURWHomRjIDPtOLD , 9(7$'2 

,, RVSDLV
&DStWXOR,, ,,, R LUPmR QmRHPDQFLSDGR GH TXDOTXHUFRQGLomR
'RV%HQHILFLiULRV PHQRUGHYLQWHHXPDQRVRXLQYiOLGR
$UWž 6mRILOLDGRVDR5336')QDTXDOLGDGHGHEHQHILFLiUL ,9 RF{QMXJHDFRPSDQKHLUDRFRPSDQKHLURHRILOKRQmR
RVRVVHJXUDGRVHVHXVGHSHQGHQWHVGHILQLGRVQRDUW HPDQFLSDGRGHTXDOTXHUFRQGLomRPHQRUGHYLQWHH
žQRDUWHQRDUW XPDQRVRXLQYiOLGR )LFDDFUHVFHQWDGRRLQFLVR ,9 DRDUWSHOD/HL
&RPSOHPHQWDUQžGH

$UWž 3HUPDQHFH ILOLDGR DR 5336') QD TXDOLGDGHGH


VHJXUDGR R VHUYLGRU WLWXODU GH FDUJR HIHWLYR TXH †ž $ GHSHQGrQFLD HFRQ{PLFD GR F{QMXJH HGRVILOKRV
HVWLYHU LQGLFDGRVQRLQFLVR,9pSUHVXPLGDHDGDVSHVVRDV
LQGLFDGDVQRVLQFLVRV,D,,,GHYHVHUFRPSURYDGD
QRYDVUHGDo}HVGDGDVDRV††žHžSHODOHLFRPSOHPHQWDUQžGHGRGIGH
, FHGLGRDyUJmRRXHQWLGDGHGD$GPLQLVWUDomR'LUHWDH 

,QGLUHWDGHRXWUR(QWHIHGHUDWLYRFRPRXVHP{QXV
SDUDR'LVWULWR)HGHUDO †ž $ H[LVWrQFLD GH GHSHQGHQWH LQGLFDGR QRLQFLVR,9
H[FOXLGRGLUHLWRDREHQHItFLRRVLQGLFDGRVQRVLQFLVRV
,, DIDVWDGRRXOLFHQFLDGRLQFOXVLYHSDUDRH[HUFtFLRGH ,D,,,
PDQGDWR FODVVLVWD GHVGH TXH REVHUYDGRV RV SUD]RV
†ž &RQVLGHUDVHFRPSDQKHLUDRXFRPSDQKHLURDSHVVRD
SUHYLVWRVHPOHLHGHVGHTXHRWHPSRGHOLFHQFLDPHQWR
TXHVHPVHUFDVDGDFRPSURYHXQLmRHVWiYHOFRPR
VHMDFRQVLGHUDGRFRPRGHHIHWLYRH[HUFtFLRQRFDUJR
VHJXUDGRRXVHJXUDGD
,,, OLFHQFLDGRSDUDWUDWDUGHLQWHUHVVHVSDUWLFXODUHV †ž (TXLSDUDPVHjFRQGLomRGHFRPSDQKHLUDRXFRPSD
QKHLURGHTXHWUDWDRLQFLVR,GHVWHDUWLJRRVSDUFHLURV
,9 GXUDQWHRDIDVWDPHQWRSDUDRH[HUFtFLRGHPDQGDWR KRPRDIHWLYRV TXH PDQWHQKDP UHODFLRQDPHQWR FLYLO
HOHWLYR SHUPDQHQWH GHVGH TXH GHYLGDPHQWH FRPSURYDGR
DSOLFDQGRVHSDUDFRQILJXUDomRGHVWHQRTXHFRXEHU
9 GXUDQWHRDIDVWDPHQWRGRSDtVSRUFHVVmRRXOLFHQoD RV SUHFHLWRV OHJDLV LQFLGHQWHV VREUH D XQLmR HVWiYHO
UHPXQHUDGD HQWUHSDUFHLURVGHVH[RVGLIHUHQWHV )LFDPDFUHVFHQWDGRVRV††
žHžDRDUWSHOD/HL&RPSOHPHQWDUQžGHGRGIGH

$UWž 2VHUYLGRUHIHWLYRUHTXLVLWDGRGD8QLmRGH(VWDGRRX
GH0XQLFtSLRSHUPDQHFHILOLDGRDRUHJLPHSUHYLGHQ †ž $RVVHUYLGRUHVS~EOLFRVGR'LVWULWR)HGHUDOWLWXODUHV
FLiULRGHRULJHP GHFDUJRHIHWLYRILFDDVVHJXUDGRRGLUHLWRGHDYHUED
omRMXQWRjDXWRULGDGHFRPSHWHQWHSDUDILQVSUHYLGHQ
6HomR, FLiULRVGDFRQGLomRGHSDUFHLURVKRPRDIHWLYRV
'RV6HJXUDGRV
$UW (TXLSDUDPVHDRVILOKRVQDVFRQGLo}HVGRLQFLVR,GR
DUW  PHGLDQWH GHFODUDomR HVFULWD GR VHJXUDGR H
$UW 6mRREULJDWRULDPHQWHILOLDGRVDR5336')QDFRQGL
GHVGHTXHFRPSURYDGDDGHSHQGrQFLDHFRQ{PLFDR
omRGHVHJXUDGRVRVVHUYLGRUHVGHTXHWUDWDRDUWž
HQWHDGR H R PHQRU TXH HVWHMD VRE VXD WXWHOD H QmR
GHVWD/HL&RPSOHPHQWDUDLQGDTXHHPGLVSRQLELOLGD SRVVXD EHQV VXILFLHQWHV SDUD R SUySULR VXVWHQWR H
GH HGXFDomR
  /HJLVODomR
 6HFUHWDULDGH(VWDGRGH$GPLQLVWUDomR3~EOLFD'LVWULWR)HGHUDO
†~QLFR 2PHQRUVRE WXWHOD VRPHQWH SRGHUiVHUHTXLSDUDGR †~QLFR 2VHJXUDGRSRGHUHQXQFLDUDTXDOTXHUGRVEHQHItFLRV
DRV ILOKRV GR VHJXUDGR PHGLDQWH DSUHVHQWDomR GH SUHYLVWRVQHVWHDUWLJR ILFDDFUHVFHQWDGRRSDUiJUDIR~QLFRDRDUWSHOD
/HL&RPSOHPHQWDUQžGHGRGIGH
WHUPRGHWXWHOD

$UW $ SHUGD GH FRQGLomR GR GHSHQGHQWH RFRUUHUiQDV , TXDQWRDRVHJXUDGR


VHJXLQWHVKLSyWHVHV
D DSRVHQWDGRULDFRPSXOVyULDSRULQYDOLGH]SHUPDQHQ
, TXDQWRDRF{QMXJH WH

D SHODVHSDUDomRMXGLFLDORXGLYyUFLRHQTXDQWRQmR E DSRVHQWDGRULDFRPSXOVyULDSRULGDGH
OKHIRUDVVHJXUDGDDSUHVWDomRGHDOLPHQWRV QRYD
UHGDomRGDGDjDOtQHDDGRLQFLVR,GRDUWSHOD/HL&RPSOHPHQWDUQžGH F DSRVHQWDGRULD YROXQWiULD SRU LGDGH H WHPSR GH
GRGIGH
FRQWULEXLomR
E SHODDQXODomRGRFDVDPHQWR G DSRVHQWDGRULDYROXQWiULDSRULGDGH
,, TXDQWRDRFRPSDQKHLURRXFRPSDQKHLUDSHODFHVVD H DSRVHQWDGRULDHVSHFLDOGRSURIHVVRU
omRGDXQLmRHVWiYHOFRPRVHJXUDGR
I DSRVHQWDGRULDHVSHFLDOQRVFDVRVSUHYLVWRVHPOHL
,,, TXDQWRDRILOKRHHTXLSDUDGRVHDRLUPmRGHTXDOTXHU FRPSOHPHQWDUIHGHUDOQRVWHUPRVGRDUW†ž
FRQGLomR DR FRPSOHWDUHP  YLQWH H XP  DQRV RX GD&RQVWLWXLomR)HGHUDO
SHODHPDQFLSDomRVDOYRVHLQYiOLGRV
J UHYRJDGDSHOD/HL&RPSOHPHQWDUQžGH
,9 SHODFHVVDomRGDLQYDOLGH]GRVILOKRVHTXLSDUDGRVRX
LUPmRVPDLRUHVGH YLQWHHXP DQRV K OLFHQoDPDWHUQLGDGH QRYDUHGDomRGDGDDRDUWLQFLVR,DOtQHD
´KµSHOD/HL&RPSOHPHQWDUQž'2')GH

9 SHODFHVVDomRGDGHSHQGrQFLDHFRQ{PLFD L VDOiULRIDPtOLD


9, SHODDFXPXODomRLOtFLWDGHSHQVmR ,, TXDQWRDRVGHSHQGHQWHVGRVVHJXUDGRV
9,, SHORIDOHFLPHQWRRXSHODSHUGDGHTXDOTXHUXPDGDV D SHQVmRSRUPRUWH
FRQGLo}HVTXHOKHJDUDQWDPRGLUHLWRDREHQHItFLR E DX[tOLRUHFOXVmR
6HomR,,, 6HomR,
'DV,QVFULo}HV 'D$SRVHQWDGRULD&RPSXOVyULDSRU,QYDOLGH]3HUPDQHQWH
$UW $LQVFULomRGRVHJXUDGRpDXWRPiWLFDHRFRUUHTXDQGR $UW $DSRVHQWDGRULDSRULQYDOLGH]pGHYLGDDRVHJXUDGR
GDLQYHVWLGXUDQRFDUJRHIHWLYRPHGLDQWHFDGDVWURQR TXH IRU FRQVLGHUDGR LQFDSD] GH UHDGDSWDomR SDUD R
5336') H[HUFtFLRGDVDWULEXLo}HVGRFDUJRGHIRUPDFRPSDWt
YHOFRPDOLPLWDomRTXHWHQKDVRIULGRHGHYHVHUSDJD
$UW ,QFXPEHDRVHJXUDGRDLQVFULomRGHVHXVGHSHQGHQWHV FRP EDVH QD OHJLVODomR YLJHQWH D SDUWLU GD GDWD GD
RVTXDLVSRGHUmRSURPRYrODFDVRHOHIDOHoDVHPWrOD SXEOLFDomR GR UHVSHFWLYR DWR H HQTXDQWR R VHUYLGRU
HIHWLYDGR SHUPDQHFHUQHVVDFRQGLomR QRYDUHGDomRGDGDDRFDSXWGRDUW
SHOD/HL&RPSOHPHQWDUQžGHGRGIGH
†ž $LQVFULomRGHGHSHQGHQWHLQYiOLGRUHTXHUVHPSUHD
FRPSURYDomR GHVVD FRQGLomR SRU LQVSHomR PpGLFD †ž 2V SURYHQWRV GD DSRVHQWDGRULD SRULQYDOLGH]VHUmR
FRQIRUPHSUHYLVWRQHVWD/HL&RPSOHPHQWDU SURSRUFLRQDLV DR WHPSR GH FRQWULEXLomR H[FHWR VH
GHFRUUHQWHVGHDFLGHQWHHPVHUYLoRPROpVWLDSURILVVLR
†ž $V LQIRUPDo}HV UHIHUHQWHV DRVGHSHQGHQWHVGHYHUmR QDORXGRHQoDJUDYHFRQWDJLRVDRXLQFXUiYHOKLSyWH
VHUFRPSURYDGDVGRFXPHQWDOPHQWH VHVHPTXHRVSURYHQWRVVHUmRLQWHJUDLVREVHUYDGR
TXDQWRDRVHXFiOFXORRGLVSRVWRQRDUW
†ž $SHUGDGDFRQGLomRGHVHJXUDGRLPSOLFDRDXWRPiWLFR
FDQFHODPHQWRGDLQVFULomR †ž $FLGHQWHHPVHUYLoRpDTXHOHRFRUULGRQRH[HUFtFLRGR
FDUJRTXHVHUHODFLRQHGLUHWDRXLQGLUHWDPHQWHFRP
†ž $LQVFULomRGHGHSHQGHQWHRFRUULGDDSyV WULQWD DV DWULEXLo}HV GHVWH SURYRFDQGR OHVmR FRUSRUDO RX
GLDVGRIDOHFLPHQWRGRVHJXUDGRVRPHQWHSURGX]LUi SHUWXUEDomRIXQFLRQDOTXHFDXVHDSHUGDRXUHGXomR
HIHLWRVDSDUWLUGDGDWDGRSURWRFRORGRUHTXHULPHQWR SHUPDQHQWH RX WHPSRUiULD GD FDSDFLGDGH SDUD R
WUDEDOKR
†ž 2 VHJXUDGR GHYHUi LQIRUPDU D PRGLILFDomRGRVHX
JUXSR GH GHSHQGHQWHV SRU LQFOXVmR H[FOXVmR RX †ž (TXLSDUDPVHDRDFLGHQWHHPVHUYLoRSDUDRVHIHLWRV
DOWHUDomRRTXHVySURGX]LUiHIHLWRDSDUWLUGDGDWDGH GHVWD/HL&RPSOHPHQWDU
HQWUDGDGRUHVSHFWLYRUHTXHULPHQWRVHKRPRORJDGR
, RDFLGHQWHOLJDGRDRVHUYLoRTXHHPERUDQmRWHQKD
VLGRDFDXVD~QLFDKDMDFRQWULEXtGRGLUHWDPHQWHSDUD
&DStWXOR,,, DUHGXomRRXSHUGDGDVXDFDSDFLGDGHSDUDRWUDEDOKR
'R3ODQRGH%HQHItFLRV RXSURGX]LGROHVmRTXHH[LMDDWHQomRPpGLFDSDUDD
VXDUHFXSHUDomR
$UW 2 5336') JHULGR SHOR ,SUHY') DVVHJXUDDRV
EHQHILFLiULRV TXH SUHHQFKDP RV UHTXLVLWRV OHJDLV RV ,, RDFLGHQWHVRIULGRSHORVHJXUDGRQRORFDOHQRKRUiULR
VHJXLQWHVEHQHItFLRV GRWUDEDOKRHPFRQVHTrQFLDGH
6HFUHWDULDGH(VWDGRGH$ GPLQLVWUDomR3~EOLFD'LVWULWR)HGHUDO
 /HJLVODomR 

D DWRGHDJUHVVmRVDERWDJHPRXWHUURULVPRSUDWLFDGR †ž 2DSRVHQWDGRTXHYROWDUDH[HUFHUDWLYLGDGHODERUDO


SRUWHUFHLURRXFRPSDQKHLURGHVHUYLoR WHUiDDSRVHQWDGRULDSRULQYDOLGH]SHUPDQHQWHFHVVD
GDDSDUWLUGDGDWDGRUHWRUQRLQFOXVLYHHPFDVRGH
E RIHQVDItVLFDLQWHQFLRQDOLQFOXVLYHGHWHUFHLURSRU H[HUFtFLRGHFDUJRHOHWLYR
PRWLYRGHGLVSXWDUHODFLRQDGDDRVHUYLoR
†ž 2VHUYLGRUDSRVHQWDGRFRPSURYHQWRSURSRUFLRQDODR
F DWRGHLPSUXGrQFLDGHQHJOLJrQFLDRXGHLPSHUtFLD WHPSRGHFRQWULEXLomRVHDFRPHWLGRGHTXDOTXHUGDV
GHWHUFHLURRXGHFRPSDQKHLURGHVHUYLoR PROpVWLDVHVSHFLILFDGDVQR†žGHYHSDVVDUDSHUFHEHU
SURYHQWRLQWHJUDOFDOFXODGRFRPEDVHQRIXQGDPHQWR
G DWRGHSHVVRDSULYDGDGRXVRGDUD]mR OHJDOGHFRQFHVVmRGDDSRVHQWDGRULD

H GHVDEDPHQWRLQXQGDomRLQFrQGLRHRXWURVFDVRV † $ GRHQoD OHVmR RX GHILFLrQFLD GH TXHRVHUYLGRU


IRUWXLWRVRXGHFRUUHQWHVGHIRUoDPDLRU S~EOLFRHUDSRUWDGRUDRLQJUHVVDUQRFDUJRS~EOLFRQmR
OKH FRQIHUH R GLUHLWR j DSRVHQWDGRULD SRU LQYDOLGH]
,,, DGRHQoDSURYHQLHQWHGHFRQWDPLQDomRDFLGHQWDOGR VDOYR TXDQGR VREUHYLHU LQFDSDFLGDGH SRU PRWLYR GH
VHJXUDGRQRH[HUFtFLRGRFDUJR SURJUHVVmRRXDJUDYDPHQWRGDVFDXVDVGHGHILFLrQFLD
ILFDPDFUHVFHQWDGRVRV††žHDRDUWSHODOHLFRPSOHPHQWDUQžGH
GRGIGH
,9 RDFLGHQWHVRIULGRSHORVHJXUDGRDLQGDTXHIRUDGR
ORFDOHKRUiULRGHVHUYLoR 6HomR,,
'D$SRVHQWDGRULD&RPSXOVyULDSRU,GDGH
D QDH[HFXomRGHRUGHPRXQDUHDOL]DomRGHVHUYLoR
UHODFLRQDGRDRFDUJR $UW 2 VHJXUDGR KRPHP RX PXOKHU VHUiDSRVHQWDGR
FRPSXOVRULDPHQWHQROLPLWHGHLGDGHHVWDEHOHFLGRQD
E QD SUHVWDomR HVSRQWkQHD GH TXDOTXHU VHUYLoR DR &RQVWLWXLomR)HGHUDOFRPSURYHQWRVSURSRUFLRQDLVDR
'LVWULWR)HGHUDOSDUDOKHHYLWDUSUHMXt]RRXSURSRU WHPSRGHFRQWULEXLomRFDOFXODGRVQDIRUPDHVWDEHOHFL
FLRQDUSURYHLWR GDQRDUWQmRSRGHQGRVHULQIHULRUHVDRYDORUGR
VDOiULRPtQLPR
F HPYLDJHPDVHUYLoRLQFOXVLYHSDUDHVWXGRTXDQGR
ILQDQFLDGD SHOR 'LVWULWR )HGHUDO GHQWUR GH VHXV †~QLFR $DSRVHQWDGRULDVHUiGHFODUDGDSRUDWRGDDXWRULGDGH
SODQRV SDUD PHOKRU FDSDFLWDomR GD PmRGHREUD FRPSHWHQWH FRP YLJrQFLD D SDUWLU GR GLD LPHGLDWR
LQGHSHQGHQWHPHQWHGRPHLRGHORFRPRomRXWLOL]D jTXHOH HP TXH R VHUYLGRU DWLQJLU D LGDGHOLPLWH GH
GRLQFOXVLYHYHtFXORGHSURSULHGDGHGRVHJXUDGR SHUPDQrQFLDQRVHUYLoR

G QRSHUFXUVRGDUHVLGrQFLDSDUDRORFDOGHWUDEDOKR 6HomR,,,
RXGHVWHSDUDDTXHODTXDOTXHUTXHVHMDRPHLRGH 'D$SRVHQWDGRULD9ROXQWiULDSRU,GDGH
ORFRPRomR LQFOXVLYH YHtFXOR GH SURSULHGDGH GR H7HPSRGH&RQWULEXLomR
VHJXUDGR
$UW 2 VHJXUDGR IDUi MXV j DSRVHQWDGRULD YROXQWiULDSRU
†ž 1RV SHUtRGRV GHVWLQDGRV D UHIHLomR RXGHVFDQVRRX LGDGHHWHPSRGHFRQWULEXLomRFRPSURYHQWRVFDOFXOD
SRU RFDVLmR GD VDWLVIDomR GH RXWUDV QHFHVVLGDGHV GRVQDIRUPDGRDUWGHVGHTXHSUHHQFKDFXPXOD
ILVLROyJLFDV QR ORFDO GR WUDEDOKR RX GXUDQWH HVWH R WLYDPHQWHRVVHJXLQWHVUHTXLVLWRV
VHUYLGRUpFRQVLGHUDGRQRH[HUFtFLRGRFDUJR
, WHPSR PtQLPR GH GH] DQRV GH HIHWLYR H[HUFtFLRQR
†ž 3DUDHIHLWRGHFRQFHVVmRGHDSRVHQWDGRULDFRPSXOVyULD VHUYLoRS~EOLFRIHGHUDOHVWDGXDOGLVWULWDORXPXQLFL
SRU LQYDOLGH] SHUPDQHQWH FRP SURYHQWRV LQWHJUDLV SDO
FRQVLGHUDPVHPROpVWLDSURILVVLRQDORXGRHQoDVJUDYHV
FRQWDJLRVDVRXLQFXUiYHLVDTXHVHUHIHUHRSDUiJUDIR ,, WHPSRPtQLPRGHFLQFRDQRVGHHIHWLYRH[HUFtFLRQR
SULPHLURDVVHJXLQWHVWXEHUFXORVHDWLYDKDQVHQtDVH FDUJRHPTXHVHGDUiDDSRVHQWDGRULD
OHXFHPLDSrQILJRIROLiFHRDOLHQDomRPHQWDOQHRSOD
VLDPDOLJQDFHJXHLUDSRVWHULRUDRLQJUHVVRQRVHUYLoR ,,, VHVVHQWDDQRVGHLGDGHHWULQWDHFLQFRDQRVGHWHPSR
S~EOLFRSDUDOLVLDLUUHYHUVtYHOHLQFDSDFLWDQWHFDUGLR GHFRQWULEXLomRVHKRPHPHFLQTHQWDHFLQFRDQRV
SDWLD JUDYH GRHQoD GH 3DUNLQVRQ HVSRQGLORDUWURVH GHLGDGHHWULQWDDQRVGHWHPSRGHFRQWULEXLomRVH
DQTXLORVDQWH QHIURSDWLD JUDYH HVWDGR DYDQoDGR GD PXOKHU
GRHQoD GH 3DJHW RVWHtWH GHIRUPDQWH  VtQGURPH GD 6HomR,9
GHILFLrQFLDLPXQROyJLFDDGTXLULGD$LGVQHXURSDWLD 'D$SRVHQWDGRULD9ROXQWiULDSRU,GDGH
JUDYHHVFOHURVHP~OWLSODFRQWDPLQDomRSRUUDGLDomR
FRPEDVHHPFRQFOXVmRGDPHGLFLQDHVSHFLDOL]DGDH $UW 2VHJXUDGRIDUiMXVjDSRVHQWDGRULDSRULGDGHFRP
KHSDWRSDWLD DSOLFDQGRVH DLQGD QR TXH FRXEHU RV SURYHQWRV SURSRUFLRQDLV DR WHPSR GH FRQWULEXLomR
FULWpULRVHVWDEHOHFLGRVSHOR5HJLPH*HUDOGH3UHYLGrQ FDOFXODGRVQDIRUPDHVWDEHOHFLGDQRDUWGHVGHTXH
FLD6RFLDO SUHHQFKDFXPXODWLYDPHQWHRVVHJXLQWHVUHTXLVLWRV

†ž $FRQFHVVmRGHDSRVHQWDGRULDSRULQYDOLGH]GHSHQGHUi , WHPSR PtQLPR GH GH] DQRV GH HIHWLYR H[HUFtFLRQR
GDYHULILFDomRGDFRQGLomRGHLQFDSDFLGDGHPHGLDQWH VHUYLoRS~EOLFRIHGHUDOHVWDGXDOGLVWULWDORXPXQLFL
H[DPHPpGLFRSHULFLDOGRyUJmRFRPSHWHQWH SDO

†ž 2 SDJDPHQWR GR EHQHItFLR GHDSRVHQWDGRULDSRU ,, WHPSRPtQLPRGHFLQFRDQRVGHHIHWLYRH[HUFtFLRQR


LQYDOLGH] GHFRUUHQWH GH GRHQoD PHQWDO VHUi IHLWR FDUJRHPTXHVHGDUiDDSRVHQWDGRULD
VRPHQWH DR FXUDGRU GR VHJXUDGR FRQGLFLRQDGR j
DSUHVHQWDomRGRWHUPRGHFXUDWHODDLQGDTXHSURYLVy ,,, VHVVHQWDHFLQFRDQRVGHLGDGHVHKRPHPHVHVVHQWD
ULR DQRVGHLGDGHVHPXOKHU
  /HJLVODomR
 6HFUHWDULDGH(VWDGRGH$GPLQLVWUDomR3~EOLFD'LVWULWR)HGHUDO
6HomR9 6HomR9,,,
'D$SRVHQWDGRULD(VSHFLDOGR3URIHVVRU 'R6DOiULR)DPtOLD

$UW 2SURIHVVRUTXHFRPSURYHH[FOXVLYDPHQWHWHPSRGH $UW 6HUi FRQFHGLGR R VDOiULRIDPtOLD PHQVDOPHQWHSRU


HIHWLYRH[HUFtFLRQDVIXQo}HVGHPDJLVWpULRQDHGXFD ILOKR RX HTXLSDUDGR PHQRU GH  FDWRU]H  DQRV GH
omRLQIDQWLOHQRHQVLQRIXQGDPHQWDOHPpGLRTXDQGR LGDGHRXLQYiOLGRDRVHJXUDGRTXHWHQKDUHPXQHUD
GDDSRVHQWDGRULDSUHYLVWDQRDUWWHUiRVUHTXLVLWRV omRRXVXEVtGLRLJXDORXLQIHULRUDRYDORUHVWDEHOHFLGR
GH LGDGH H GH WHPSR GH FRQWULEXLomR UHGX]LGRV HP
QRDUWGD(PHQGD&RQVWLWXFLRQDOQžGH
FLQFRDQRV

†~QLFR 6mRFRQVLGHUDGDVIXQo}HVGHPDJLVWpULRDVH[HUFLGDV †ž 2 VDOiULRIDPtOLD WHUi R PHVPR YDORU HUHDMXVWHGR


SRUSURIHVVRUHVHHVSHFLDOLVWDVHPHGXFDomRQRGHVHP PHVPREHQHItFLRSDJRSHOR5*36
SHQKRGHDWLYLGDGHVHGXFDWLYDVTXDQGRH[HUFLGDVHP
HVWDEHOHFLPHQWR GH HGXFDomR EiVLFD IRUPDGD SHOD †ž $RILOKRRXHTXLSDUDGRPHQRUGH FDWRU]H DQRVRX
HGXFDomR LQIDQWLO HQVLQR IXQGDPHQWDO H PpGLR HP DR LQYiOLGR FRUUHVSRQGHUi XPD FRWD GR VDOi
VHXV GLYHUVRV QtYHLV H PRGDOLGDGHV LQFOXtGDV DV ULRIDPtOLD UHVSHLWDGR R YDORU OLPLWH GHVWH DUWLJR
H[HUFLGDVSRUSURIHVVRUHVHHVSHFLDOLVWDVHPHGXFDomR FRQGLFLRQDGDjDSUHVHQWDomRGDFHUWLGmRGHQDVFLPHQ
UHDGDSWDGRVEHPFRPRDVGHILQLGDVQD/HL)HGHUDOQž WRGRILOKRRXGDGRFXPHQWDomRUHODWLYDDRHTXLSDUDGR
GHGHPDLRGH RXDRLQYiOLGR
6HomR9,
'R$X[tOLR'RHQoD †ž 2SDJDPHQWRGRVDOiULRIDPtOLDVHUiFRQGLFLRQDGRj
DSUHVHQWDomRDQXDOGHDWHVWDGRGHYDFLQDomRREULJD
$UW UHYRJDGRRDUWSHOD/HL&RPSOHPHQWDUQžGH WyULDDWpVHLVDQRVGHLGDGHHjFRPSURYDomRVHPHV
$UW UHYRJDGRRDUWSHOD/HL&RPSOHPHQWDUQžGH
WUDOGHIUHTrQFLDjHVFRODGRILOKRRXHTXLSDUDGRD
6HomR9,, SDUWLUGRVVHLVDQRVGHLGDGH
'D/LFHQoD0DWHUQLGDGH
QRYDUHGDomRGDGDjVHomRYLLGRFDStWXORLLLSHODOHLFRPSOHPHQWDUQžGHGRGIGH †ž 6HRVHJXUDGRQmRDSUHVHQWDURDWHVWDGRGHYDFLQDomR

REULJDWyULDHDFRPSURYDomRGHIUHTrQFLDHVFRODUGR
$UW $VHJXUDGDJHVWDQWHID]MXVjOLFHQoDPDWHUQLGDGHSHOR ILOKRRXHTXLSDUDGRQDVGDWDVGHILQLGDVSHOR,SUHY')
SUD]RGH FHQWRHRLWHQWD GLDVVHPSUHMXt]RGD REHQHItFLRGRVDOiULRIDPtOLDVHUiVXVSHQVRDWpTXHD
UHPXQHUDomRDFRQWDUGRGLDGRSDUWR GRFXPHQWDomRVHMDDSUHVHQWDGD

†ž 2EHQHItFLRGHTXHWUDWDRFDSXWSRGHUiVHUDQWHFLSDGR †ž 1mRpGHYLGRVDOiULRIDPtOLDQRSHUtRGRHQWUHDVXV


HPDWp YLQWHHRLWR GLDVGRSDUWRSRUSUHVFULomR SHQVmRGREHQHItFLRPRWLYDGDSHODIDOWDGHFRPSURYD
PpGLFD omRGDIUHTrQFLDHVFRODUHDVXDUHDWLYDomRVDOYRVH
SURYDGDDIUHTrQFLDHVFRODUUHJXODUQRSHUtRGR
†ž 1R FDVR GH QDWLPRUWR RX GH QDVFLPHQWRFRPYLGD
VHJXLGRGHyELWRDVHJXUDGDUHDVVXPLUiVXDVIXQo}HV †ž $FRPSURYDomRGHIUHTrQFLDHVFRODUVHUiIHLWDPHGL
GHFRUULGRV WULQWD GLDVGRHYHQWRFDVRVHMDMXOJDGD DQWHDSUHVHQWDomRGHGRFXPHQWRHPLWLGRSHODHVFROD
DSWD QDIRUPDGHOHJLVODomRSUySULDHPQRPHGRDOXQRHP
TXH FRQVWH R UHJLVWUR GH IUHTrQFLD UHJXODU RX GH
†ž 1R FDVR GH DERUWR DWHVWDGR SRU PpGLFRRILFLDOD
DWHVWDGRGRHVWDEHOHFLPHQWRGHHQVLQRFRPSURYDQGR
VHJXUDGDWHUiGLUHLWRD WULQWD GLDVGREHQHItFLRGH
TXHWUDWDHVWHDUWLJR DUHJXODULGDGHGDPDWUtFXODHDIUHTrQFLDHVFRODUGR
DOXQR
$UW $VHJXUDGDTXHDGRWDURXREWLYHUJXDUGDMXGLFLDOSDUD
ILQV GH DGRomR IDUi MXV j OLFHQoDPDWHUQLGDGH SHORV †ž 2VDOiULRIDPtOLDQmRVHUiSDJRTXDQGRGRDIDVWDPHQ
VHJXLQWHVSHUtRGRV WRSRUTXDOTXHUPRWLYRGRVHJXUDGR

,  FHQWRHRLWHQWD GLDVVHDFULDQoDWLYHUPHQRVGH †ž 2 VDOiULRIDPtOLD QmR VH LQFRUSRUDUi DRVXEVtGLRj
 XP DQRGHLGDGH UHPXQHUDomRRXDREHQHItFLRSDUDTXDOTXHUHIHLWR

,,  QRYHQWD GLDVVHDFULDQoDWLYHUHQWUH XP H †ž 1RV FDVRV GH DFXPXODomR OHJDO GH FDUJRVRVDOi
TXDWUR DQRVGHLGDGH ULRIDPtOLDVHUiSDJRVRPHQWHHPUHODomRDXPGHOHV

,,,  WULQWD GLDVVHDFULDQoDWLYHUGH TXDWUR D † 7HQGRKDYLGRGLYyUFLRVHSDUDomRMXGLFLDORXGHIDWR


RLWR DQRVGHLGDGH
GRVSDLVRXHPFDVRGHDEDQGRQROHJDOPHQWHFDUDFWH
†~QLFR 2 EHQHItFLR GH TXH WUDWD HVWH DUWLJRVHUiGHIHULGR UL]DGR RX SHUGD GR SRGHU IDPLOLDU R VDOiULRIDPtOLD
VRPHQWHPHGLDQWHDSUHVHQWDomRGHWHUPRMXGLFLDOGH SDVVDUi D VHU SDJR GLUHWDPHQWH jTXHOH D FXMR FDUJR
JXDUGDjDGRWDQWHRXJXDUGLm ILFDURVXVWHQWRGRPHQRURXDRXWUDSHVVRDVHKRXYHU
GHWHUPLQDomRMXGLFLDOQHVVHVHQWLGR
$UW$ $VHUYLGRUDFRPLVVLRQDGDVHPYtQFXORHIHWLYRFRPD
$GPLQLVWUDomRWDPEpPID]MXVDRVEHQHItFLRVSUHYLVWRV † 3DUD HIHLWR GH FRQFHVVmR H PDQXWHQomRGRVDOi
QRVDUWVHGHVWD/HL&RPSOHPHQWDU ULRIDPtOLDRVHJXUDGRGHYHILUPDUWHUPRGHUHVSRQVD
ELOLGDGH QR TXDO VH FRPSURPHWD D FRPXQLFDU DR
†~QLFR 1RVFDVRVGRVEHQHItFLRVSUHYLVWRVQRDUWHQRDUW ,SUHY')TXDOTXHUIDWRRXFLUFXQVWkQFLDTXHGHWHUPL
,DVGHVSHVDVUHODWLYDVDRV~OWLPRV VHVVHQWD
QHDSHUGDGRGLUHLWRDREHQHItFLRILFDQGRVXMHLWRHP
GLDV FRUUHUmR j FRQWD GRV UHFXUVRV GR WHVRXUR GR
FDVRGRQmRFXPSULPHQWRjVVDQo}HVSHQDLV
'LVWULWR)HGHUDO
6HFUHWDULDGH(VWDGRGH$ GPLQLVWUDomR3~EOLFD'LVWULWR)HGHUDO
 /HJLVODomR 

† $IDOWDGHFRPXQLFDomRRSRUWXQDGHIDWRTXHLPSOLTXH †ž 2F{QMXJHDXVHQWHQmRH[FOXLGRGLUHLWRjSHQVmRSRU


FHVVDomRGRVDOiULRIDPtOLDEHPFRPRDSUiWLFDSHOR PRUWHRFRPSDQKHLURRXDFRPSDQKHLUD
VHJXUDGRGHIUDXGHGHTXDOTXHUQDWXUH]DSDUDRVHX
UHFHELPHQWR DXWRUL]D R ,SUHY') D GHVFRQWDU GRV †ž $ KDELOLWDomR SRVWHULRU TXH LPSRUWHLQFOXVmRRX
SDJDPHQWRV GH FRWDV GHYLGDV FRP UHODomR D RXWURV H[FOXVmRGHGHSHQGHQWHVySURGX]HIHLWRVDFRQWDUGD
ILOKRVRXQDIDOWDGHODVGRSUySULRVDOiULRGRVHJXUD GDWDGDKDELOLWDomR ILFDPDFUHVFHQWDGRVRV††žžžHžDRDUWSHOD
GRRXGDUHQGDPHQVDOGRVHXEHQHItFLRRYDORUGDV OHLFRPSOHPHQWDUQžGHGRGIGH

FRWDV LQGHYLGDPHQWH UHFHELGDV VHP SUHMXt]R GDV


VDQo}HVSHQDLVFDEtYHLV $UW $V SHQV}HV GLVWLQJXHPVH TXDQWR j QDWXUH]DHP
YLWDOtFLDV H WHPSRUiULDV  QRYD UHGDomR GDGD DR DUW  SHOD OHL
FRPSOHPHQWDUQžGHGRGIGH
$UW 2GLUHLWRDRVDOiULRIDPtOLDFHVVDDXWRPDWLFDPHQWH
†ž $SHQVmRYLWDOtFLDpFRPSRVWDGHFRWDRXFRWDVSHUPD
, SRU PRUWH GR ILOKR RX HTXLSDUDGR D FRQWDU GRPrV
QHQWHVTXHVRPHQWHVHH[WLQJXHPRXUHYHUWHPFRPD
VHJXLQWHDRGRyELWR
PRUWHGRSHQVLRQLVWD
,, TXDQGRRILOKRRXHTXLSDUDGRFRPSOHWDU FDWRU]H
†ž $SHQVmRWHPSRUiULDpFRPSRVWDGHFRWDRXFRWDVTXH
DQRV GH LGDGH VDOYR VH LQYiOLGR D FRQWDU GR PrV
VHJXLQWHDRGDGDWDGRDQLYHUViULR SRGHPVHH[WLQJXLURXUHYHUWHUSRUPRWLYRGHPRUWH
FHVVDomRGHLQYDOLGH]RXPDLRULGDGHGRSHQVLRQLVWD
,,, SHODUHFXSHUDomRGDFDSDFLGDGHGRILOKRRXHTXLSDUD
GRLQYiOLGRDFRQWDUGRPrVVHJXLQWHDRGDFHVVDomR $UW$ 6mREHQHILFLiULRVGDSHQVmR  ILFDP DFUHVFHQWDGRV RV DUWLJRV D
EFHGSHODOHLFRPSOHPHQWDUQžGHGRGIGH
GDLQFDSDFLGDGH
, YLWDOtFLD
,9 SHODSHUGDGDFRQGLomRGHVHJXUDGR
D RF{QMXJH
6HomR,;
'D3HQVmRSRU0RUWH E DSHVVRDVHSDUDGDMXGLFLDOPHQWHGLYRUFLDGDRXFXMD
XQLmRHVWiYHOIRLOHJDOPHQWHGLVVROYLGDFRPSHUFHS
$UW $SHQVmRSRUPRUWHFRQIHULGDDRFRQMXQWRGRVGHSHQ omRGHSHQVmRDOLPHQWtFLD
GHQWHVGRVHJXUDGRIDOHFLGRDSDUWLUGHGHIHYHUHLUR
GHGDWDGHSXEOLFDomRGD0HGLGD3URYLVyULDQž
F R FRPSDQKHLUR RX FRPSDQKHLUD TXH FRPSURYH
TXHRULJLQRXD/HL)HGHUDOQžGHGH
XQLmRHVWiYHO
MXQKRGHFRUUHVSRQGHUi
G DPmHRXRSDLFRPSHUFHSomRGHSHQVmRDOLPHQWt
, jWRWDOLGDGHGRVSURYHQWRVSHUFHELGRVSHORDSRVHQWDGR
QD GDWD DQWHULRU j GR yELWR DWp R OLPLWH Pi[LPR FLD
HVWDEHOHFLGRSDUDRVEHQHItFLRVGR5*36DFUHVFLGDGH
VHWHQWDSRUFHQWRGDSDUFHODH[FHGHQWHDHVVHOLPLWH ,, WHPSRUiULD

,, j WRWDOLGDGH GD UHPXQHUDomR GR VHUYLGRU QRFDUJR D RILOKRRXRHQWHDGRDWpFRPSOHWDUYLQWHHXPDQRV


HIHWLYRQDGDWDDQWHULRUjGRyELWRDWpROLPLWHPi[L GHLGDGHRXVHLQYiOLGRVHQTXDQWRGXUDUDLQYDOL
PRHVWDEHOHFLGRSDUDRVEHQHItFLRVGR5*36DFUHVFLGD GH]
GH VHWHQWD SRU FHQWR GD SDUFHOD H[FHGHQWH D HVVH
OLPLWH VH R IDOHFLPHQWR RFRUUHU TXDQGR R VHUYLGRU E RPHQRUVREWXWHOD
DLQGDHVWLYHUHPDWLYLGDGH
F RLUPmRQmRHPDQFLSDGRDWpFRPSOHWDUYLQWHHXP
†ž 1DKLSyWHVHGHFiOFXORGHSHQVmRRULXQGDGHIDOHFL DQRVGHLGDGHRXVHLQYiOLGR HQTXDQWRGXUDUD
PHQWRGRVHUYLGRUQDDWLYLGDGHpYHGDGDDLQFOXVmR LQYDOLGH]TXHSHUFHEDSHQVmRDOLPHQWtFLD
GHSDUFHODVUHPXQHUDWyULDVSDJDVHPGHFRUUrQFLDGH
ORFDOGHWUDEDOKRGHIXQomRGHFRQILDQoDGHFDUJRHP †~QLFR eYHGDGDDFRQFHVVmRGHSHQVmRYLWDOtFLD
FRPLVVmRRXGRDERQRGHSHUPDQrQFLDGHTXHWUDWDR
DUWEHPFRPRDSUHYLVmRGHLQFRUSRUDomRGHWDLV , DR EHQHILFLiULR LQGLFDGR QR LQFLVR , F VHKRXYHU
SDUFHODVGLUHWDPHQWHQRYDORUGDSHQVmRRXQDUHPX EHQHILFLiULRLQGLFDGRQRLQFLVR,D
QHUDomRDSHQDVSDUDHIHLWRGHFRQFHVVmRGREHQHItFLR
DLQGDTXHPHGLDQWHUHJUDVHVSHFtILFDV ,, DPDLVGHXPFRPSDQKHLURRXFRPSDQKHLUD

†ž 2 GLUHLWR j SHQVmR p GHYLGR D FRQWDU GDGDWDGR $UW% 2YDORUGDSHQVmRFDOFXODGRQDIRUPDGRDUWGHYH


IDOHFLPHQWRGRVHJXUDGRGDGDWDGDGHFLVmRMXGLFLDO VHUUDWHDGRHQWUHRVKDELOLWDGRVGHPRGRDLQGLYLGXDOL
QR FDVR GH GHFODUDomR GH DXVrQFLD RX GD GDWD GD ]DUDFRWDDTXHFDGDEHQHILFLiULRID]MXV
RFRUUrQFLD GR GHVDSDUHFLPHQWR GR VHJXUDGR SRU
PRWLYRGHDFLGHQWHGHVDVWUHRXFDWiVWURIHPHGLDQWH †ž 1mRKDYHQGRGHSHQGHQWHVSUHYLVWRVQRDUW$,E
SURYDLG{QHDVHQGRREHQHItFLRFRQFHGLGRFRPEDVH RX G RX QR DUW $ ,, F GHYHVH REVHUYDU QR
QDOHJLVODomRYLJHQWHQHVVDGDWDYHGDGRQRYRFiOFXOR FiOFXORGDFRWDGHFDGDSHQVLRQLVWDRVHJXLQWH
HP UD]mR GR UHDMXVWDPHQWR GR OLPLWH Pi[LPR GRV
EHQHItFLRVGR5*36 , KDYHQGRDSHQDVXPSHQVLRQLVWDKDELOLWDGRRYDORUGD
FRWDFRUUHVSRQGHDRYDORUGDSHQVmR
†ž $SHQVmRGHYHVHUFRQFHGLGD DR GHSHQGHQWHTXHVH
KDELOLWDU ,, RFRUUHQGRKDELOLWDomRjVSHQV}HVYLWDOtFLDHWHPSRUi
ULD PHWDGH GR YDORU FDEH DRV KDELOLWDGRV j SHQVmR
†ž $ FRQFHVVmRGDSHQVmRQmRSRGHVHUSURWHODGDSHOD YLWDOtFLD D RXWUD PHWDGH DRV KDELOLWDGRV j SHQVmR
IDOWDGHKDELOLWDomRGHRXWURSRVVtYHOGHSHQGHQWH WHPSRUiULD
  /HJLVODomR 6HFUHWDULDGH(VWDGRGH$GPLQLVWUDomR3~EOLFD'LVWULWR)HGHUDO


†ž +DYHQGRGHSHQGHQWHVSUHYLVWRVQRDUW$,ERXG †ž 3DUDDFRQFHVVmRGHVVHEHQHItFLRDOpPGDGRFXPHQWD


RXQRDUW$,,FDSOLFDVHRVHJXLQWH omRTXHFRPSURYHDFRQGLomRGHVHJXUDGRHGHGHSHQ
GHQWHVVHUiH[LJLGDDDSUHVHQWDomRGDFHUWLGmRHPLWL
, D FRWD GHVVHV GHSHQGHQWHV p FDOFXODGD GHPRGR GDSHODDXWRULGDGHFRPSHWHQWHVREUHRHIHWLYRUHFROKL
SURSRUFLRQDODRYDORUGDSHQVmRDOLPHQWtFLDSHUFHELGD PHQWRGRVHJXUDGRjSULVmRRXUHVSHFWLYRUHJLPHGH
WHQGRFRPREDVHSDUDFiOFXORRYDORUWRWDOGDSHQVmR FXPSULPHQWRGHSHQDVHQGRWDOSURFHGLPHQWRUHQRYD
GRWULPHVWUDOPHQWH
,, DFRWDGRVGHPDLVGHSHQGHQWHVVHKRXYHUGHYHVHU
FDOFXODGD QD IRUPD GR † ž WHQGR FRPR EDVH SDUD †ž +DYHQGR PDLV GH XP GHSHQGHQWH R YDORUGRDX[t
FiOFXOR R VDOGR GR YDORU GD SHQVmR TXH UHPDQHVFHU OLRUHFOXVmRVHUiUDWHDGRGDPHVPDIRUPDHVWDEHOHFLGD
DSyV GHGX]LU D FRWD GH TXH WUDWD R LQFLVR , GHVWH SDUDDSHQVmRSRUPRUWH
SDUiJUDIR
†ž 2DX[tOLRUHFOXVmRVHUiGHYLGRDFRQWDUGDGDWDHPTXH
†ž 2YDORUDSXUDGRQDIRUPDGR†ž,ILFDOLPLWDGRSHOD RVHJXUDGRGHL[DUGHSHUFHEHUTXDOTXHUUHPXQHUDomR
FRWDGHYLGDDFDGDEHQHILFLiULRGDSHQVmRYLWDOtFLDRX GRVFRIUHVS~EOLFRVDSyVVHQWHQoDSHQDOFRQGHQDWyULD
GDSHQVmRWHPSRUiULD WUDQVLWDGDHPMXOJDGR

$UW& $ FRWD GR SHQVLRQLVWD TXH SHUGHX HVVD TXDOLGDGH †ž )DOHFHQGR R VHJXUDGR GHWHQWR RX UHFOXVRGHQWURGR
UHYHUWHVH H[FOXVLYDPHQWH SDUD VHX DVFHQGHQWH SUD]R HVWDEHOHFLGR QR † ž R DX[tOLR UHFOXVmR TXH
GHVFHQGHQWHRXLUPmRTXHWDPEpPVHMDSHQVLRQLVWDGR HVWLYHUVHQGRSDJRDRVVHXVGHSHQGHQWHVVHUiFRQYHUWL
PHVPRLQVWLWXLGRUGHSHQVmR GRDXWRPDWLFDPHQWHHPSHQVmRSRUPRUWH

$UW' 5HVVDOYDGRRGLUHLWRGHRSomRpYHGDGDDSHUFHSomR †ž 1D KLSyWHVH GH IXJD GR VHJXUDGR REHQHItFLRVHUi
FXPXODWLYDGHPDLVGHGXDVSHQV}HVSDJDVSRUUHJLPH VXVSHQVR QDGD VHQGR GHYLGR DRV VHXV GHSHQGHQWHV
SUySULRGHSUHYLGrQFLDVRFLDO HQTXDQWRHVWLYHURVHJXUDGRHYDGLGRHSHORSHUtRGRGD
IXJDVHQGRREHQHItFLRUHVWDEHOHFLGRDSDUWLUGDGDWD
$UW 6HUiFRQFHGLGDSHQVmRSURYLVyULDSRUPRUWHTXDQGRR GDUHFDSWXUDRXGDUHDSUHVHQWDomRjSULVmR
IDOHFLPHQWRGRVHJXUDGRIRUSUHVXPLGR
†ž &DVRRVHJXUDGRYHQKDDVHUUHVVDUFLGRFRPRSDJD
†ž $SHQVmRGHTXHWUDWDRFDSXWGHVWHDUWLJRGHL[DUiGH PHQWRGDUHPXQHUDomRFRUUHVSRQGHQWHDRSHUtRGRHP
VHU WHPSRUiULD GHFRUULGRV  FLQFR  DQRV GH VXD TXHHVWHYHSUHVRHVHXVGHSHQGHQWHVWHQKDPUHFHELGR
YLJrQFLD UHVVDOYDGR R HYHQWXDO UHDSDUHFLPHQWR GR DX[tOLRUHFOXVmRRYDORUFRUUHVSRQGHQWHDRSHUtRGRGH
VHUYLGRUKLSyWHVHHPTXHREHQHItFLRVHUiDXWRPDWLFD JR]RGREHQHItFLRGHYHUiVHUUHWLGRSHORyUJmRSDJDGRU
PHQWHFDQFHODGRILFDQGRREHQHILFLiULRGHVREULJDGR D TXH R VHJXUDGR HVWLYHU YLQFXODGR H UHVWLWXtGR DR
GDUHSRVLomRGRVYDORUHVUHFHELGRVVDOYRPiIp ,SUHY')DSOLFDQGRVHRVMXURVHtQGLFHVGHFRUUHomR
LQFLGHQWHVQRUHVVDUFLPHQWRGDUHPXQHUDomR
†ž 2EHQHILFLiULRGDSHQVmRSURYLVyULDGHYHUiDQXDOPHQ
WHGHFODUDUTXHRVHJXUDGRSHUPDQHFHGHVDSDUHFLGR †ž 6HKRXYHUH[HUFtFLRGHDWLYLGDGHGXUDQWHRSHUtRGRGH
ILFDQGR REULJDGR D FRPXQLFDU DR ,SUHY') R VHX IXJDHOHVHUiFRQVLGHUDGRSDUDDSHUGDGDTXDOLGDGH
UHDSDUHFLPHQWRVRESHQDGHVHUUHVSRQVDELOL]DGRFLYLO GHVHJXUDGR
HFULPLQDOPHQWH
†ž $SOLFDPVH DR DX[tOLRUHFOXVmR QR TXHFRXEHUDV
$UW $ SHQVmR SRGHUi VHU UHTXHULGD D TXDOTXHUWHPSR QRUPDVUHIHUHQWHVjSHQVmRSRUPRUWH
SUHVFUHYHQGRWmRVRPHQWHDVSUHVWDo}HVH[LJtYHLVKi
PDLVGH FLQFR DQRV † 2EHQHItFLRFRQFHGLGRDWpGHGH]HPEURGH
VHUiPDQWLGRQDPHVPDIRUPDHPTXHIRLFRQFHGLGR
†~QLFR &RQFHGLGDDSHQVmRSRUPRUWHTXDOTXHUSURYDSRVWH LQGHSHQGHQWHPHQWH GR YDORU GD UHPXQHUDomR GR
ULRU RX KDELOLWDomR WDUGLD TXH LPSOLTXH H[FOXVmR GH VHUYLGRU
EHQHILFLiULRRXUHGXomRGHSHQVmRVySURGX]LUiHIHLWRV 6HomR;,
DSDUWLUGDGDWDHPTXHIRURIHUHFLGDDVVHJXUDGRDRV 'R$ERQR$QXDO
EHQHILFLiULRVRGLUHLWRjSUpYLDFLrQFLDjDPSODGHIHVD
HDRFRQWUDGLWyULR $UW 2DERQRDQXDOpGHYLGRjTXHOHTXHGXUDQWHRDQR
WHQKD UHFHELGR SURYHQWRV GH DSRVHQWDGRULD SHQVmR
$UW 1mRIDUiMXVjSHQVmRRGHSHQGHQWHFRQGHQDGRSHOD SRU PRUWH DX[tOLRUHFOXVmR RX VDOiULRPDWHUQLGDGH
SUiWLFD GH FULPH GRORVR GH TXH WHQKD UHVXOWDGR D SDJRVSHOR,SUHY') QRYDUHGDomR GDGDDRFDSXWGRDUWSHOD/HL
PRUWHGRVHJXUDGR &RPSOHPHQWDUQžGH

6HomR; †~QLFR 2DERQRGHTXHWUDWDRFDSXWVHUiSURSRUFLRQDOHP


'R$X[tOLR5HFOXVmR FDGDDQRDRQ~PHURGHPHVHVGHEHQHItFLRSDJRSHOR
,SUHY')HPTXHFDGDPrVFRUUHVSRQGHUiDXPGR]H
$UW 2DX[tOLRUHFOXVmRVHUiFRQFHGLGRPHGLDQWHUHTXHUL DYRVHWHUiSRUEDVHRYDORUGREHQHItFLRGRPrVGH
PHQWR DR FRQMXQWR GH GHSHQGHQWHV KDELOLWDGRV GR GH]HPEURH[FHWRTXDQGRREHQHItFLRVHHQFHUUDUDQWHV
VHJXUDGRGHWHQWRRXUHFOXVRTXHWHQKDUHPXQHUDomR GHVVHPrVTXDQGRRYDORUVHUiRGRPrVGDFHVVDomR
RXVXEVtGLRLJXDORXLQIHULRUDRYDORUHVWDEHOHFLGRQR
DUWGD(PHQGD&RQVWLWXFLRQDOQžGH 6HomR;,,
'DV'LVSRVLo}HV*HUDLVVREUH%HQHItFLRV
†ž 2YDORUGRDX[tOLRUHFOXVmRFRUUHVSRQGHUijUHPXQHUD
omRGRVHJXUDGROLPLWDGRDRYDORUHVWDEHOHFLGRQRDUW $UW 1DKLSyWHVHGHRFDUJRHPTXHVHGHUDDSRVHQWDGRULD
GD (PHQGD&RQVWLWXFLRQDOQžTXHVHUi QmRHVWDULQVHULGRHPSODQRGHFDUUHLUDRVUHTXLVLWRV
FRUULJLGRSHORVPHVPRVtQGLFHVDSOLFDGRVDRVEHQHItFL SUHYLVWRVQRDUW,9HQRDUW,,,GHYHUmRVHU
RVGR5*36 FXPSULGRVQR~OWLPRFDUJRHIHWLYR
6HFUHWDULDGH(VWDGRGH$ GPLQLVWUDomR3~EOLFD'LVWULWR)HGHUDO
 /HJLVODomR 

$UW 2 WHPSR GH FDUUHLUD H[LJLGR SDUD FRQFHVVmRGRV , WLYHU FLQTHQWD H WUrV DQRV GH LGDGH VH KRPHPH
EHQHItFLRV SUHYLVWRV QRV DUWV  H  GHYHUi VHU TXDUHQWDHRLWRDQRVGHLGDGHVHPXOKHU
FXPSULGR QR PHVPR (QWH IHGHUDWLYR H QR PHVPR
3RGHU ,, WLYHUFLQFRDQRVGHHIHWLYRH[HUFtFLRQRFDUJRHPTXH
VHGHUDDSRVHQWDGRULD
$UW 3DUDHIHLWRGRFXPSULPHQWRGRVUHTXLVLWRVGHFRQFHV
VmRGDVDSRVHQWDGRULDVSUHYLVWDVQRVDUWV ,,, FRQWDU WHPSR GH FRQWULEXLomR LJXDO QR PtQLPRj
HRWHPSRGHHIHWLYRH[HUFtFLRQRFDUJRHPTXH VRPDGH
VHGDUiDDSRVHQWDGRULDGHYHUiVHUFXPSULGRQRFDUJR
HIHWLYRHPTXHRVHUYLGRUHVWLYHUHPH[HUFtFLRQDGDWD D WULQWD H FLQFR DQRV VH KRPHP H WULQWD DQRV VH
LPHGLDWDPHQWHDQWHULRUjGDFRQFHVVmRGREHQHItFLR PXOKHU

$UW $FRQFHVVmRGHEHQHItFLRVSUHYLGHQFLiULRVSHOR5336 E XPSHUtRGRDGLFLRQDOGHFRQWULEXLomRHTXLYDOHQWH


')LQGHSHQGHGHFDUrQFLDUHVVDOYDGDDREVHUYkQFLDGH DYLQWHSRUFHQWRGRWHPSRTXHQDGDWDGHSXEOLFD
FXPSULPHQWRGRVSUD]RVPtQLPRVSUHYLVWRVQRVDUWV omR GD (PHQGD &RQVWLWXFLRQDO Qƒ  GH  GH
HSDUDFRQFHVVmRGHDSRVHQWDGRULD GH]HPEURGHIDOWDULDSDUDDWLQJLUROLPLWHGH
WHPSRFRQVWDQWHGDDOtQHD´DµGHVWHLQFLVR
$UW 6mRYHGDGRV
†ž 2 VHUYLGRU GH TXH WUDWD HVWH DUWLJR TXHFXPSULUDV
, D FRQFHVVmR GH SURYHQWRV HP YDORU LQIHULRU DRVDOi H[LJrQFLDVSDUDDSRVHQWDGRULDQDIRUPDGRFDSXWWHUi
ULRPtQLPRQDFLRQDO RVVHXVSURYHQWRVGHLQDWLYLGDGHUHGX]LGRVSDUDFDGD
,, R F{PSXWR GH WHPSR GH FRQWULEXLomR ILFWtFLR SDUDR DQR DQWHFLSDGR HP UHODomR DRV OLPLWHV GH LGDGH
FiOFXORGHEHQHItFLRSUHYLGHQFLiULR HVWDEHOHFLGRVSHORDUWHSHORDUWQDVHJXLQWH
,,, DFRQFHVVmRGHDSRVHQWDGRULDHVSHFLDOQRVWHUPRVGR SURSRUomR
DUW  † ž GD &RQVWLWXLomR )HGHUDO DWp TXH OHLV
FRPSOHPHQWDUHVIHGHUDLVGLVFLSOLQHPDPDWpULD
, WUrVLQWHLURV H FLQFRGpFLPRVSRUFHQWRSDUDDTXHOH
,9 DSHUFHSomRGHPDLVGHXPDDSRVHQWDGRULDjFRQWDGR
TXH FRPSOHWDU DV H[LJrQFLDV SDUD DSRVHQWDGRULD QD
UHJLPH SUySULR D VHUYLGRU S~EOLFR WLWXODU GH FDUJR
IRUPDGRFDSXWDWpGHGH]HPEURGHLQGHSHQ
HIHWLYRUHVVDOYDGDVDVGHFRUUHQWHVGRVFDUJRVDFXPX
GHQWHPHQWHGHDFRQFHVVmRGDDSRVHQWDGRULDRFRUUHU
OiYHLVSUHYLVWRVQD&RQVWLWXLomR)HGHUDO
HPGDWDSRVWHULRUjTXHOD
9 DSHUFHSomRVLPXOWkQHDGHSURYHQWRVGHDSRVHQWDGRULD
GHFRUUHQWH GH UHJLPH SUySULR GH VHUYLGRU WLWXODU GH
,, FLQFRSRUFHQWRSDUDDTXHOHTXHFRPSOHWDUDVH[LJrQ
FDUJRHIHWLYRFRPDUHPXQHUDomRGHFDUJRHPSUHJR
RXIXQomRS~EOLFDUHVVDOYDGRVRVFDUJRVDFXPXOiYHLV FLDVSDUDDSRVHQWDGRULDQDIRUPDGRFDSXWDSDUWLUGH
SUHYLVWRVQD&RQVWLWXLomR)HGHUDORVFDUJRVHOHWLYRVH žGHMDQHLURGH
RV FDUJRV HP FRPLVVmR GHFODUDGRV HP OHL GH OLYUH
QRPHDomRHH[RQHUDomR †ž 2Q~PHURGHDQRVDQWHFLSDGRVSDUDFiOFXORGDUHGX
omRGHTXHWUDWDR†žVHUiYHULILFDGRQRPRPHQWRGD
†ž 1mRVHFRQVLGHUDILFWtFLRRWHPSRGHILQLGRHPOHLFRPR FRQFHVVmRGREHQHItFLR
WHPSR GH FRQWULEXLomR SDUD ILQV GH FRQFHVVmR GH
DSRVHQWDGRULD TXDQGR WHQKD KDYLGR SRU SDUWH GR †ž 2VSHUFHQWXDLVGHUHGXomRGHTXHWUDWDR†ž,H,,
VHUYLGRUDSUHVWDomRGHVHUYLoRRXDFRUUHVSRQGHQWH VHUmR DSOLFDGRV VREUH R YDORU GR EHQHItFLR LQLFLDO
FRQWULEXLomR FDOFXODGRSHODPpGLDGDVFRQWULEXLo}HVVHJXQGRRDUW
YHULILFDQGRVHSUHYLDPHQWHDREVHUYkQFLDDROLPLWH
†ž $ YHGDomR SUHYLVWD QR LQFLVR 9 QmR VHDSOLFDDRV GDUHPXQHUDomRGRVHUYLGRUQRFDUJRHIHWLYRSUHYLVWR
PHPEURVGH3RGHUDRVLQDWLYRVHDRVVHUYLGRUHVTXH QR†žGRPHVPRDUWLJR
DWp  GH GH]HPEUR GH  WHQKDP LQJUHVVDGR
QRYDPHQWHQRVHUYLoRS~EOLFRSRUFRQFXUVRS~EOLFRGH †ž $SOLFDVHDRPHPEURGR7ULEXQDOGH&RQWDVRGLVSRVWR
SURYDVRXGHSURYDVHWtWXORVHSHODVGHPDLVIRUPDV QHVWHDUWLJR
SUHYLVWDVQD&RQVWLWXLomR)HGHUDOVHQGROKHVSURLELGD
D SHUFHSomR GH PDLV GH XPD DSRVHQWDGRULD SHOR †ž 1DDSOLFDomRGRGLVSRVWRQR†žRPHPEURGR7ULEX
UHJLPH SUySULR H[FHWR VH GHFRUUHQWHV GH FDUJRV QDO GH &RQWDV VH KRPHP WHUi R WHPSR GH VHUYLoR
DFXPXOiYHLVSUHYLVWRVQD&RQVWLWXLomR)HGHUDO H[HUFLGRDWpGHGH]HPEURGHFRQWDGRFRP
DFUpVFLPR GH GH]HVVHWH SRU FHQWR REVHUYDQGRVH R
†ž 2VHUYLGRULQDWLYRSDUDVHULQYHVWLGRHPFDUJRS~EOLFR GLVSRVWRQRV††žžHž
HIHWLYR QmR DFXPXOiYHO FRP DTXHOH TXH JHURX D
DSRVHQWDGRULDGHYHUiUHQXQFLDUDRVSURYHQWRVGHOD †ž 2VHJXUDGRSURIHVVRUTXHDWpDGDWDGHSXEOLFDomRGD
(PHQGD&RQVWLWXFLRQDOQƒGHGHGH]HPEURGH
$UW &RQFHGLGD D DSRVHQWDGRULD RX D SHQVmR VHUi RDWR  WHQKD LQJUHVVDGR UHJXODUPHQWH HP FDUJR
SXEOLFDGRHHQFDPLQKDGRSHOR,SUHY')DR7ULEXQDO HIHWLYR GH PDJLVWpULR QD 8QLmR (VWDGRV 'LVWULWR
GH&RQWDVGR'LVWULWR)HGHUDOSDUDKRPRORJDomR )HGHUDO RX 0XQLFtSLRV LQFOXtGDV VXDV DXWDUTXLDV H
IXQGDo}HVHTXHRSWHSRUDSRVHQWDUVHQDIRUPDGR
&DStWXOR,9 GLVSRVWRQRFDSXWWHUiRWHPSRGHVHUYLoRH[HUFLGRDWp
'DV5HJUDVGH7UDQVLomRSDUD&RQFHVVmRGH$SRVHQWDGRULD DSXEOLFDomRGDTXHOD(PHQGDFRQWDGRFRPDFUpVFLPR
GH GH]HVVHWH SRU FHQWR VH KRPHP H GH YLQWH SRU
$UW $R VHJXUDGR GR 5336') TXH WLYHU LQJUHVVDGRSRU FHQWRVHPXOKHUGHVGHTXHVHDSRVHQWHH[FOXVLYDPHQ
FRQFXUVRS~EOLFRGHSURYDVRXGHSURYDVHWtWXORVHP WH FRP WHPSR GH HIHWLYR H[HUFtFLR QDV IXQo}HV GH
FDUJRS~EOLFRHIHWLYRQDDGPLQLVWUDomRS~EOLFDGLUHWD PDJLVWpULRREVHUYDGRRGLVSRVWRQRV††žžHž
DXWiUTXLFD H IXQGDFLRQDO GD 8QLmR GRV (VWDGRV GR
'LVWULWR)HGHUDORXGRV0XQLFtSLRVDWpGHGH]HP †ž $V DSRVHQWDGRULDV FRQFHGLGDV FRQIRUPHHVWHDUWLJR
EURGHpIDFXOWDGRDSRVHQWDUVHFRPSURYHQWRV VHUmRUHDMXVWDGDVSDUDPDQWHURYDORUUHDOGHDFRUGR
FDOFXODGRVGHDFRUGRFRPRDUWTXDQGRRVHUYLGRU FRPRGLVSRVWRQRDUW
FXPXODWLYDPHQWH
  /HJLVODomR 6HFUHWDULDGH(VWDGRGH$GPLQLVWUDomR3~EOLFD'LVWULWR)HGHUDO


$UW 5HVVDOYDGRRGLUHLWRGHRSomRjDSRVHQWDGRULDSHODV †ž 2UHFHELPHQWRGRDERQRGHSHUPDQrQFLDSHORVHUYLGRU


QRUPDVHVWDEHOHFLGDVQRVDUWVRXRVHJXUD TXH FXPSULX WRGRV RV UHTXLVLWRV SDUD REWHQomR GD
GRGR5336')TXHWLYHULQJUHVVDGRQRVHUYLoRS~EOLFR DSRVHQWDGRULDYROXQWiULDFRPSURYHQWRVLQWHJUDLVRX
QDDGPLQLVWUDomRS~EOLFDGLUHWDDXWiUTXLFDHIXQGDFL SURSRUFLRQDLVHPTXDOTXHUGDVKLSyWHVHVSUHYLVWDVQRV
RQDOGD8QLmRGRV(VWDGRVGR'LVWULWR)HGHUDOHGRV DUWVHFRQIRUPHSUHYLVWRQRFDSXWH†
0XQLFtSLRV DWp  GH GH]HPEUR GH  SRGHUi žQmRFRQVWLWXLLPSHGLPHQWRjFRQFHVVmRGREHQHIt
DSRVHQWDUVHFRPSURYHQWRVLQWHJUDLVTXHFRUUHVSRQ FLR GH DFRUGR FRP RXWUD UHJUD YLJHQWH LQFOXVLYH DV
GHUmR j WRWDOLGDGH GD UHPXQHUDomR GR VHUYLGRU QR SUHYLVWDVQRVDUWVHGHVGHTXHFXPSULGRVRV
FDUJRHIHWLYRHPTXHVHGHUDDSRVHQWDGRULDTXDQGR UHTXLVLWRVSUHYLVWRVSDUDHVVDVKLSyWHVHV
REVHUYDGDVDVUHGXo}HVGHFRUUHQWHVGHLGDGHHWHPSR
GHFRQWULEXLomRFRQWLGDVQRDUWFXPXODWLYDPHQWH †ž 2YDORUGRDERQRGHSHUPDQrQFLDVHUiHTXLYDOHQWHDR
YLHUDSUHHQFKHUDVVHJXLQWHVFRQGLo}HV YDORU GD FRQWULEXLomR HIHWLYDPHQWH GHVFRQWDGD GR
VHUYLGRURXUHFROKLGDSRUHVWHUHODWLYDPHQWHDFDGD
, VHVVHQWDDQRVGHLGDGHVHKRPHPHFLQTHQWDHFLQFR FRPSHWrQFLD
DQRVGHLGDGHVHPXOKHU
†ž 2SDJDPHQWRGRDERQRGHSHUPDQrQFLDpGHUHVSRQVD
,, WULQWDHFLQFRDQRVGHFRQWULEXLomRVHKRPHPHWULQWD ELOLGDGHGR7HVRXURGR'LVWULWR)HGHUDOHVHUiGHYLGR
DQRVGHFRQWULEXLomRVHPXOKHU DSDUWLUGRFXPSULPHQWRGRVUHTXLVLWRVSDUDREWHQomR
GR EHQHItFLR FRQIRUPH GLVSRVWR QR FDSXW H † ž
,,, YLQWH DQRV GH HIHWLYR H[HUFtFLR QR VHUYLoRS~EOLFR PHGLDQWHRSomRSHODSHUPDQrQFLDHPDWLYLGDGH
IHGHUDOHVWDGXDOGLVWULWDORXPXQLFLSDO
&DStWXOR9,
,9 GH]DQRVGHFDUUHLUDHFLQFRDQRVGHHIHWLYRH[HUFtFLR 'DV5HJUDVGH&iOFXORGRV3URYHQWRVH5HDMXVWHGRV%HQHItFLRV
QRFDUJRHPTXHVHGHUDDSRVHQWDGRULD
6HomR,
$UW 5HVVDOYDGRRGLUHLWRGHRSomRjDSRVHQWDGRULDSHODV 'DV5HJUDVGR&iOFXORGRV3URYHQWRVGH$SRVHQWDGRULD
QRUPDV HVWDEHOHFLGDV SHOR DUW    H  R
VHUYLGRUTXHWHQKDLQJUHVVDGRQRVHUYLoRS~EOLFRGD $UW 1RFiOFXORGRVSURYHQWRVGDVDSRVHQWDGRULDVUHIHULGDV
8QLmRGRV(VWDGRVGR'LVWULWR)HGHUDOHGRV0XQLFtSL QRVDUWVHVHUiFRQVLGHUDGDD
RVLQFOXtGDVVXDVDXWDUTXLDVHIXQGDo}HVDWpGH PpGLDDULWPpWLFDVLPSOHVGDVPDLRUHVUHPXQHUDo}HV
GH]HPEURGHSRGHUiDSRVHQWDUVHFRPSURYHQ RXVXEVtGLRVXWLOL]DGRVFRPREDVHSDUDDVFRQWULEXL
WRVLQWHJUDLVTXHFRUUHVSRQGHUmRj~OWLPDUHPXQHUD o}HV GR VHUYLGRU DRV UHJLPHV GH SUHYLGrQFLD D TXH
omRGRVHUYLGRUQRFDUJRHIHWLYRGHVGHTXHSUHHQFKD HVWHYHYLQFXODGRFRUUHVSRQGHQWHVDRLWHQWDSRUFHQWR
FXPXODWLYDPHQWHDVVHJXLQWHVFRQGLo}HV GHWRGRRSHUtRGRFRQWULEXWLYRGHVGHRPrVGHFRPSH
WrQFLD GH MXOKR GH  RX GHVGH D GR LQtFLR GD
, WULQWDHFLQFRDQRVGHFRQWULEXLomRVHKRPHPHWULQWD FRQWULEXLomRVHSRVWHULRUjTXHODFRPSHWrQFLD
DQRVGHFRQWULEXLomRVHPXOKHU
†ž 3DUDRVHIHLWRVGRGLVSRVWRQRFDSXWVHUmRXWLOL]DGRV
,, YLQWH H FLQFR DQRV GH HIHWLYR H[HUFtFLR QRVHUYLoR RV YDORUHV GDV UHPXQHUDo}HV TXH FRQVWLWXtUDP EDVH
S~EOLFR IHGHUDO HVWDGXDO GLVWULWDO RX PXQLFLSDO SDUD DV FRQWULEXLo}HV GR VHUYLGRU DRV UHJLPHV GH
TXLQ]HDQRVGHFDUUHLUDHFLQFRDQRVQRFDUJRHPTXH SUHYLGrQFLD LQGHSHQGHQWHPHQWH GR SHUFHQWXDO GD
VHGHUDDSRVHQWDGRULD DOtTXRWDHVWDEHOHFLGDRXGHWHUHPVLGRHODVGHVWLQDGDV
SDUDRFXVWHLRGHDSHQDVSDUWHGRVEHQHItFLRVSUHYL
,,, LGDGH PtQLPD UHVXOWDQWH GD UHGXomRUHODWLYDPHQWH GHQFLiULRV
DRVOLPLWHVGHLGDGHGHILQLGRVQRDUW,,,GHXP
DQR GH LGDGH SDUD FDGD DQR GH FRQWULEXLomR TXH †ž $VUHPXQHUDo}HVRXVXEVtGLRVFRQVLGHUDGRVQRFiOFXOR
H[FHGHUDFRQGLomRSUHYLVWDQRLQFLVR,GHVWHDUWLJR GR YDORU LQLFLDO GRV SURYHQWRV WHUmR RV VHXV YDORUHV
DWXDOL]DGRV PrV D PrV GH DFRUGR FRP D YDULDomR
LQWHJUDO GR tQGLFH IL[DGR SDUD D DWXDOL]DomR GDV
&DStWXOR9 UHPXQHUDo}HVGHFRQWULEXLomRFRQVLGHUDGDVQRFiOFXOR
'R$ERQRGH3HUPDQrQFLD GRV EHQHItFLRV GR 5*36 FRQIRUPH DWR FRPSHWHQWH
HGLWDGRSHULRGLFDPHQWHSHOR0LQLVWpULRGD3UHYLGrQFLD
$UW 2VHJXUDGRDWLYRTXHWHQKDFRPSOHWDGRDVH[LJrQFLDV 6RFLDO
SDUDDSRVHQWDGRULDYROXQWiULDFRQIRUPHHVWDEHOHFLGR
QRVDUWVHHTXHRSWHSRUSHUPDQHFHUHP †ž 1DVFRPSHWrQFLDVDSDUWLUGHMXOKRGHHPTXH
DWLYLGDGHIDUiMXVDXPDERQRGHSHUPDQrQFLDHTXLYD QmRWHQKDKDYLGRFRQWULEXLomRGRVHUYLGRUYLQFXODGR
OHQWHDRYDORUGDVXDFRQWULEXLomRSUHYLGHQFLiULDDWp DUHJLPHSUySULRDEDVHGHFiOFXORGRVSURYHQWRVVHUi
FRPSOHWDUDVH[LJrQFLDVSDUDDSRVHQWDGRULDFRPSXOVy DUHPXQHUDomRGRVHUYLGRUQRFDUJRHIHWLYRLQFOXVLYH
ULDFRQWLGDVQRDUW QRVSHUtRGRVHPTXHKRXYHLVHQomRGHFRQWULEXLomRRX
DIDVWDPHQWRGRFDUJRGHVGHTXHRUHVSHFWLYRDIDVWD
†ž 2DERQRSUHYLVWRQRFDSXWVHUiFRQFHGLGRQDVPHV PHQWRVHMDFRQVLGHUDGRFRPRGHHIHWLYRH[HUFtFLR
PDVFRQGLo}HVDRVHUYLGRUTXHDWpGHGH]HPEUR
GHGDWDGHSXEOLFDomRGD(PHQGD&RQVWLWXFLRQDO †ž 1DDXVrQFLDGHFRQWULEXLomRGRVHUYLGRUQmRWLWXODUGH
QžWHQKDFXPSULGRWRGRVRVUHTXLVLWRVSDUDREWHQ FDUJRHIHWLYRYLQFXODGRDUHJLPHSUySULRDWpGH]HP
omR GD DSRVHQWDGRULD YROXQWiULD FRP SURYHQWRV EURGHVHUiFRQVLGHUDGDDVXDUHPXQHUDomRQR
LQWHJUDLVRXSURSRUFLRQDLVFRPEDVHQRVFULWpULRVGD FDUJRRFXSDGRQRSHUtRGRFRUUHVSRQGHQWH
OHJLVODomR HQWmR YLJHQWH FRPR SUHYLVWR QR DUW 
GHVGHTXHFRQWHFRPQRPtQLPRYLQWHHFLQFRDQRV †ž $VUHPXQHUDo}HVFRQVLGHUDGDVQRFiOFXORGDPpGLD
GHFRQWULEXLomRVHPXOKHURXWULQWDDQRVVHKRPHP GHSRLVGHDWXDOL]DGDVQDIRUPDGR†žQmRSRGHUmR
VHU
6HFUHWDULDGH(VWDGRGH$ GPLQLVWUDomR3~EOLFD'LVWULWR)HGHUDO
 /HJLVODomR 

, LQIHULRUHVDRYDORUGRVDOiULRPtQLPR †ž $RVHUYLGRUTXHWHQKDSHORPHQRVFLQFRDQRVQRFDUJR


HGH]DQRVGHVHUYLoRS~EOLFR QR 'LVWULWR)HGHUDOD
,, VXSHULRUHV DR OLPLWH Pi[LPR GDUHPXQHUD DSRVHQWDGRULD FRP SURYHQWRV SURSRUFLRQDLV VHUi GH
omRGHFRQWULEXLomR TXDQWR DRV PHVHV HP TXH R  TXDUHQWDSRUFHQWR GRVYDORUHVFRUUHVSRQGHQWHV
VHUYLGRUHVWHYHYLQFXODGRDR5*36 DRTXHVHULDDDSRVHQWDGRULDFRPSURYHQWRVLQWHJUDLV
PDLV GRLVSRUFHQWR GHVWHJUXSRGHGR]HFRQWUL
†ž $VPDLRUHVUHPXQHUDo}HVGHTXHWUDWDRFDSXWVHUmR EXLo}HVQmRSRGHQGRXOWUDSDVVDURYDORUGDUHPXQH
GHILQLGDVGHSRLVGDDSOLFDomRGRVIDWRUHVGHDWXDOL]D UDomR QR FDUJR HIHWLYR  )LFD DFUHVFLGR R † ž DR DUW  SHOD OHL
FRPSOHPHQWDUQžGHGRGIGH
omRHGDREVHUYkQFLDPrVDPrVGRVOLPLWHVHVWDEHOH
FLGRVQR†ž 6HomR,,
'RV'RFXPHQWRV&RPSUREDWyULRVGD&RQWULEXLomR
†ž 1DGHWHUPLQDomRGRQ~PHURGHFRPSHWrQFLDVFRUUHV
SRQGHQWHV D RLWHQWD SRU FHQWR GH WRGR R SHUtRGR $UW 2V YDORUHV GDV UHPXQHUDo}HV D VHUHP XWLOL]DGDVQR
FRQWULEXWLYR GH TXH WUDWD R FDSXW GHVSUH]DUVHi D FiOFXORGHTXHWUDWDRDUWEHPFRPRRWHPSRGH
SDUWHGHFLPDO FRQWULEXLomR FRUUHVSRQGHQWH VHUmR FRPSURYDGRV
PHGLDQWHGRFXPHQWRIRUQHFLGRSHORVyUJmRVHHQWLGD
†ž 6H D SDUWLU GH MXOKR GH  KRXYHUODFXQDVQR GHVJHVWRUDVGRVUHJLPHVGHSUHYLGrQFLDDRVTXDLVR
SHUtRGRFRQWULEXWLYRGRVHJXUDGRSRUQmRYLQFXODomR VHUYLGRU HVWHYH YLQFXODGR RX QD IDOWD GDTXHOH SRU
D UHJLPH SUHYLGHQFLiULR HP UD]mR GH DXVrQFLD GH RXWURGRFXPHQWRS~EOLFRVHQGRSDVVtYHLVGHFRQILUPD
SUHVWDomRGHVHUYLoRRXGHFRQWULEXLomRHVVHSHUtRGR omRDVLQIRUPDo}HVIRUQHFLGDV
VHUiGHVSUH]DGRGRFiOFXORGHTXHWUDWDHVWHDUWLJR
†ž 2YDORULQLFLDOGRSURYHQWRFDOFXODGRGHDFRUGRFRP †ž 2V GRFXPHQWRV GH FRPSURYDomR GRVYDORUHVGDV
R FDSXW SRU RFDVLmR GH VXD FRQFHVVmR QmR SRGHUi UHPXQHUDo}HVGHTXHWUDWDRFDSXWEHPFRPRRVGH
H[FHGHU D UHPXQHUDomR GR UHVSHFWLYR VHUYLGRU QR FHUWLILFDomR GH WHPSR GH FRQWULEXLomR TXH IRUDP
FDUJRHIHWLYRHPTXHVHGHXDDSRVHQWDGRULDVHQGR HPLWLGRV SHORV GLYHUVRV yUJmRV GD DGPLQLVWUDomR
YHGDGDDLQFOXVmRGHSDUFHODVWHPSRUiULDVFRQIRUPH UHODWLYRV D VHUYLGRU YLQFXODGR D 5336') DSyV D
DUW SXEOLFDomR GD 0HGLGD 3URYLVyULD Qž  GH  GH
IHYHUHLUR GH  TXH RULJLQRX D /HL )HGHUDO Qž
$UW eYHGDGDDLQFOXVmRQRVEHQHItFLRVGHDSRVHQWDGRULD  WHUmR YDOLGDGH DSyV KRPRORJDomR GD
HSHQVmRSDUDHIHLWRGHSHUFHSomRGHOHVGHSDUFHODV XQLGDGHJHVWRUDGRUHJLPH
UHPXQHUDWyULDV SDJDV HP GHFRUUrQFLD GH ORFDO GH
WUDEDOKRGHIXQomRGHFRQILDQoDGHFDUJRHPFRPLV †ž &RQWLQXDPYiOLGDVDVFHUWLG}HVGHWHPSRGHVHUYLoRH
VmRRXGRDERQRGHSHUPDQrQFLDGHTXHWUDWDRDUW GHFRQWULEXLomRHPLWLGDVSHORVyUJmRVGDDGPLQLVWUD
 omR S~EOLFD GD 8QLmR (VWDGRV 'LVWULWR )HGHUDO RX
0XQLFtSLRVVXDVDXWDUTXLDVIXQGDo}HVRXXQLGDGHV
†ž &RPSUHHQGHVH QD YHGDomR GR FDSXW DSUHYLVmRGH JHVWRUDV GRV UHJLPHV GH SUHYLGrQFLD VRFLDO UHODWLYD
LQFRUSRUDomR GDV SDUFHODV WHPSRUiULDV GLUHWDPHQWH PHQWHDRWHPSRGHVHUYLoRHGHFRQWULEXLomRSDUDR
QRVEHQHItFLRVRXQDUHPXQHUDomRDSHQDVSDUDHIHLWR UHVSHFWLYR UHJLPH HP GDWD DQWHULRU j SXEOLFDomR GD
GHFRQFHVVmRGHEHQHItFLRVDLQGDTXHPHGLDQWHUHJUDV 0HGLGD3URYLVyULDQžGHGHIHYHUHLURGH
HVSHFtILFDVLQGHSHQGHQWHPHQWHGHWHUKDYLGRLQFLGrQ
FLDGHFRQWULEXLomRVREUHWDLVSDUFHODV $UW 2 ,SUHY') IRUQHFHUi JUDWXLWDPHQWH DRVHUYLGRU
GHWHQWRUH[FOXVLYDPHQWHGHFDUJRGHOLYUHQRPHDomR
†ž 1mR VH LQFOXHP QD YHGDomR SUHYLVWD QRFDSXWDV HH[RQHUDomRHDRVHUYLGRUWLWXODUGHFDUJRHPSUHJR
SDUFHODV TXH WLYHUHP LQWHJUDGR D UHPXQHUD RXIXQomRDPSDUDGRSHOR5*36GRFXPHQWRFRPSUR
omRGHFRQWULEXLomRGRVHUYLGRUTXHVHDSRVHQWDUFRP EDWyULRGHYtQFXORIXQFLRQDOSDUDILQVGHFRQFHVVmRGH
EHQHItFLRVRXSDUDHPLVVmRGD&HUWLGmRGH7HPSRGH
SURYHQWRVFDOFXODGRVSHODPpGLDDULWPpWLFDFRQIRUPH
&RQWULEXLomRSHOR5*36VHPSUHMXt]RGDDSUHVHQWDomR
DUWUHVSHLWDQGRVHHPTXDOTXHUKLSyWHVHROLPLWH
GD*XLDGH5HFROKLPHQWRGR)XQGRGH*DUDQWLDSRU
GH UHPXQHUDomR GR UHVSHFWLYR VHUYLGRU QR FDUJR
7HPSRGH6HUYLoRH,QIRUPDo}HVj3UHYLGrQFLD6RFLDO
HIHWLYRHPTXHVHGHXDDSRVHQWDGRULD
*),3
$UW 3DUDRFiOFXORGRYDORULQLFLDOGRVSURYHQWRVSURSRUFLR
6HomR,,,
QDLVDRWHPSRGHFRQWULEXLomRVHUiXWLOL]DGDIUDomR
'R5HDMXVWDPHQWRGRV%HQHItFLRV
FXMRQXPHUDGRUVHUiRWRWDOGHVVHWHPSRHRGHQRPL
QDGRURWHPSRQHFHVViULRjUHVSHFWLYDDSRVHQWDGRULD $UW 2VEHQHItFLRVGHDSRVHQWDGRULDHSHQVmRGHTXHWUDWDP
YROXQWiULD FRP SURYHQWRV LQWHJUDLV FRQIRUPH R DUW RVDUWVHVHUmRUHDMXVWDGRV
,,,QmRVHDSOLFDQGRDUHGXomRQRWHPSRGHLGDGH SDUDSUHVHUYDUOKHVHPFDUiWHUSHUPDQHQWHRYDORU
H FRQWULEXLomR GH TXH WUDWD R DUW  UHODWLYD DR UHDO QD PHVPD GDWD HP TXH VH GHU R UHDMXVWH GRV
SURIHVVRU EHQHItFLRV GR 5*36 GH DFRUGR FRP D YDULDomR GR
tQGLFHGHILQLGRHPOHLSHOR'LVWULWR)HGHUDODSOLFDGR
†ž $ IUDomR GH TXH WUDWD RFDSXWVHUiDSOLFDGDVREUHR GHIRUPDSURSRUFLRQDOHQWUHDGDWDGDFRQFHVVmRHD
YDORU LQLFLDO GR SURYHQWR FDOFXODGR SHOD PpGLD GDV GRSULPHLURUHDMXVWDPHQWR
FRQWULEXLo}HVFRQIRUPHDUWREVHUYDQGRVHSUHYLD
PHQWHDDSOLFDomRGROLPLWHGHUHPXQHUDomRGRFDUJR †~QLFR 2VEHQHItFLRVVHUmRFRUULJLGRV SHORVPHVPRVtQGLFHV
HIHWLYRGHTXHWUDWD†žGRPHVPRDUWLJR DSOLFDGRVDRVEHQHItFLRVGR5*36QRFDVRGHDXVrQFLD
GH tQGLFH RILFLDO GR 'LVWULWR )HGHUDO TXH GHILQD R
†ž 2VSHUtRGRVGHWHPSRXWLOL]DGRV QRFiOFXORSUHYLVWR UHDMXVWDPHQWRTXHSUHVHUYHHPFDUiWHUSHUPDQHQWHR
QHVWHDUWLJRVHUmRFRQVLGHUDGRVHPQ~PHURGHGLDV YDORUUHDOGRVEHQHItFLRV
  /HJLVODomR 6HFUHWDULDGH(VWDGRGH$GPLQLVWUDomR3~EOLFD'LVWULWR)HGHUDO


$UW 2VEHQHItFLRVDEUDQJLGRVSHORGLVSRVWRQRVDUWV 9,, GRDo}HVOHJDGRVHUHQGDVH[WUDRUGLQiULDVRXHYHQWXD


HDVSHQV}HVGHULYDGDVGRVSURYHQWRVGHVHUYLGRUHV LV
IDOHFLGRVTXHWHQKDPVHDSRVHQWDGRHPFRQIRUPLGDGH
FRP R DUW  H RV EHQHItFLRV HP IUXLomR HP  GH 9,,, RSURGXWRGDDOLHQDomRGHVHXVEHQV
GH]HPEURGHVHUmRUHYLVWRVQDPHVPDSURSRUomR
HQDPHVPDGDWDVHPSUHTXHVHPRGLILFDUDUHPXQH ,; RV FUpGLWRV GH QDWXUH]D SUHYLGHQFLiULD GHYLGRVDRV
UDomRGRVVHUYLGRUHVHPDWLYLGDGHLQFOXVLYHTXDQGR yUJmRVGDDGPLQLVWUDomRGLUHWDHLQGLUHWDGRV3RGHUHV
HPGHFRUUrQFLDGDWUDQVIRUPDomRRXUHFODVVLILFDomRGR GR'LVWULWR)HGHUDO([HFXWLYRH/HJLVODWLYRLQFOXtGRR
FDUJRRXIXQomRHPTXHVHGHXDDSRVHQWDGRULDQD
7ULEXQDOGH&RQWDV
IRUPDGDOHJLVODomRDSOLFDGD

†~QLFR eYHGDGDDH[WHQVmRFRPUHFXUVRVSUHYLGHQFLiULRVGR ; RVFUpGLWRVGHYLGRVDRUHJLPHSUySULRGHSUHYLGrQFLD


UHDMXVWDPHQWRSDULWiULRGHTXHWUDWD HVWHDUWLJRDRV UHODWLYDPHQWH DRV VHUYLGRUHV S~EOLFRV GR 'LVWULWR
EHQHItFLRVDEUDQJLGRVSHORGLVSRVWRQRDUWDLQGD )HGHUDODWtWXORGHFRPSHQVDomRILQDQFHLUDHQWUHRV
TXHDWtWXORGHDQWHFLSDomRGRUHDMXVWHDQXDORXGH UHJLPHVSUHYLGHQFLiULRVGHTXHWUDWDD/HL)HGHUDOQž
UHFRPSRVLomRGHSHUGDVVDODULDLVDQWHULRUHVjFRQFHV 
VmRGREHQHItFLR
;, FUpGLWRVWULEXWiULRVHQmRWULEXWiULRVTXHYHQKDPDVHU
&DStWXOR9,, RX Mi HVWHMDP LQVFULWRV HP GtYLGD DWLYD GR 'LVWULWR
'R'LUHLWR$GTXLULGR )HGHUDOGHVXDVDXWDUTXLDVHIXQGDo}HVRXUHFXUVRV
DGYLQGRVGDUHVSHFWLYDOLTXLGDomR
$UW eDVVHJXUDGDDFRQFHVVmRGHDSRVHQWDGRULDHSHQVmR
DTXDOTXHUWHPSRDRVVHJXUDGRVHVHXVGHSHQGHQWHV ;,, DVSDUWLFLSDo}HVVRFLHWiULDVGHSURSULHGDGHGR'LVWULWR
TXHDWpGHGH]HPEURGHWHQKDPFXPSULGR
)HGHUDOGHVXDVDXWDUTXLDVHIXQGDo}HVEHPFRPRGH
RVUHTXLVLWRVSDUDDREWHQomRGHVVHVEHQHItFLRVFRP
HPSUHVDV S~EOLFDV H VRFLHGDGHV GH HFRQRPLD PLVWD
EDVHQRVFULWpULRVGDOHJLVODomRHQWmRYLJHQWHREVHUYD
GRRGLVSRVWRQRDUW;,GD&RQVWLWXLomR)HGHUDO HVWDGXDLV PHGLDQWH SUpYLD DXWRUL]DomR OHJLVODWLYD
HVSHFtILFD
†ž 2V SURYHQWRV GD DSRVHQWDGRULD D VHUFRQFHGLGDDRV
VHJXUDGRVUHIHULGRVQRFDSXWHPWHUPRVLQWHJUDLVRX ;,,, UHFHEtYHLVGLUHLWRVGHFUpGLWRGLUHLWRVDWtWXORSDUWLFL
SURSRUFLRQDLVDRWHPSRGHFRQWULEXLomRMiH[HUFLGRDWp SDo}HVHPIXQGRVGHTXHVHMDWLWXODUR'LVWULWR)HGH
GHGH]HPEURGHEHPFRPRDV SHQV}HVGH UDO
VHXVGHSHQGHQWHVVHUmRFDOFXODGRVGHDFRUGRFRPD
OHJLVODomRHPYLJRUjpSRFDHPTXHIRUDPDWHQGLGDVDV ;,9 EHQVGRPLQLFDLVGHSURSULHGDGHVGR'LVWULWR)HGHUDO
SUHVFULo}HVQHODHVWDEHOHFLGDVSDUDDFRQFHVVmRGHVVHV IXQGDo}HVHDXWDUTXLDVWUDQVIHULGDVQDIRUPDGHVWD
EHQHItFLRVRXQDVFRQGLo}HVGDOHJLVODomRYLJHQWH /HL&RPSOHPHQWDU

†ž 4XDQGR R EHQHItFLR IRU FDOFXODGR GH DFRUGRFRPD †ž 2V&KHIHVGRV3RGHUHVGR'LVWULWR)HGHUDO([HFXWLYR


OHJLVODomRHPYLJRUjpSRFDGDDTXLVLomRGRGLUHLWR H/HJLVODWLYRLQFOXtGRVR7ULEXQDOGH&RQWDVDXWDUTXL
VHUi XWLOL]DGD D UHPXQHUDomR GR VHUYLGRU QR FDUJR DV H IXQGDo}HV ILFDP DXWRUL]DGRV D WUDQVIHULU DR
HIHWLYRQRPRPHQWRGDFRQFHVVmRGDDSRVHQWDGRULDH SDWULP{QLR GR ,SUHY') EHQV GLUHLWRV H DWLYRV GH
HPFDVRGHSURYHQWRVSURSRUFLRQDLVFRQVLGHUDUVHiR
TXDOTXHUQDWXUH]DREVHUYDGRVRVFULWpULRVHSDUkPH
WHPSRGHFRQWULEXLomRFXPSULGRDWpGHGH]HPEUR
WURVOHJDLVDILPGHFDSLWDOL]DURUHJLPHGHSUHYLGrQ
GH
&DStWXOR9,,, FLDJHULGRSRUDTXHODDXWDUTXLDEHPFRPRDVVHJXUDU
'R&XVWHLRGR5HJLPH3UySULRGH3UHYLGrQFLD RSDJDPHQWRGHVHXVFRPSURPLVVRV
GR'LVWULWR)HGHUDO
†ž 2&KHIHGR3RGHU([HFXWLYRSURSRUiTXDQGRQHFHVVi
$UW 25336')GHTXHWUDWDHVWD/HL&RPSOHPHQWDUVHUi ULDDDEHUWXUDGHFUpGLWRVRUoDPHQWiULRVDGLFLRQDLV
FXVWHDGRPHGLDQWHRVVHJXLQWHVUHFXUVRV YLVDQGRDVVHJXUDUDR,SUHY')DORFDomRGHUHFXUVRV
RUoDPHQWiULRV GHVWLQDGRV j FREHUWXUD GH HYHQWXDLV
, FRQWULEXLomR SUHYLGHQFLiULD GR HQWH S~EOLFR'LVWULWR LQVXILFLrQFLDV RUoDPHQWiULDV H ILQDQFHLUDV SDUD D
)HGHUDO JDUDQWLDGRSDJDPHQWRGDVDSRVHQWDGRULDVSHQV}HVH
RXWURVEHQHItFLRVSUHYLGHQFLiULRVGHYLGRV
,, FRQWULEXLomRSUHYLGHQFLiULDGRVVHJXUDGRVDWLYRV
$UW )LFD R 3RGHU ([HFXWLYR DXWRUL]DGR D LQFRUSRUDUDR
,,, FRQWULEXLomRSUHYLGHQFLiULDGRVVHJXUDGRVDSRVHQWDGRV SDWULP{QLRGR,SUHY')RVVHJXLQWHVDWLYRV
HGRVSHQVLRQLVWDV
, RVEHQVLPyYHLVGRPLQLFDLVGHWLWXODULGDGHGR'LVWULWR
,9 RV DWLYRV H UHQGLPHQWRV DGYLQGRV GD H[SORUDomRGR
)HGHUDO
SDWULP{QLRLPRELOLiULRGR,SUHY')

9 RVUHQGLPHQWRVGRSDWULP{QLRGR,SUHY')WDLVFRPR ,, RVEHQVLPyYHLVGRPLQLFDLVGHWLWXODULGDGHGHDXWDU


RVREWLGRVFRPDSOLFDo}HVILQDQFHLUDVRXFRPRUHFHEL TXLDVHIXQGDo}HVS~EOLFDV
PHQWRGHFRQWUDSDUWLGDSHORXVRGHVHXVEHQV
†ž 2ÐUJmRFRPSHWHQWHTXHWUDWDGR3DWULP{QLR,PRELOLi
9, DVGRWDo}HVRUoDPHQWiULDVGHVWLQDGDVDRSDJDPHQWR ULRGR*RYHUQRGR'LVWULWR)HGHUDO*')SURFHGHUiDR
GHSHVVRDOLQDWLYRSHQV}HVHRXWURVEHQHItFLRVSUHYL LQYHQWiULR GRV EHQV HQTXDGUDGRV QRV LQFLVRV , H ,,
GHQFLiULRVGHYLGRVSHODDGPLQLVWUDomRGLUHWDHLQGLUHWD GHVWH DUWLJR GHYHQGR D FDGD  QRYHQWD  GLDV D
GRV3RGHUHV([HFXWLYRH/HJLVODWLYRGR'LVWULWR)HGH FRQWDUGDSXEOLFDomRGHVWD/HL&RPSOHPHQWDUSURPR
UDO LQFOXtGR R 7ULEXQDO GH &RQWDV FXMRV VHUYLGRUHV YHUDSXEOLFDomRGRVEHQVLQYHQWDULDGRVQRSHUtRGR
VHMDPVHJXUDGRVRXEHQHILFLiULRV
6HFUHWDULDGH(VWDGRGH$ GPLQLVWUDomR3~EOLFD'LVWULWR)HGHUDO
 /HJLVODomR 

†ž &XPSULGD D IRUPDOLGDGH SUHYLVWD QR FDSXWR3RGHU $UW $FRQWULEXLomRSUHYLGHQFLiULDGRVVHJXUDGRVLQDWLYRV


([HFXWLYRSURPRYHUiDLQFRUSRUDomRGRVDOXGLGRVEHQV HGRVSHQVLRQLVWDVGHTXHWUDWDRDUW,,,VHUiGH
LPyYHLV DR ,SUHY') TXH VH HIHWLYDUi SRU PHLR GH  RQ]HSRUFHQWR FRQIRUPH/HL&RPSOHPHQWDU
WHUPR DGPLQLVWUDWLYR HODERUDGR VHJXQGR PLQXWD 'LVWULWDO Qž  LQFLGHQWH VREUH D SDUFHOD GR
SDGUmRDSURYDGDSHOD3URFXUDGRULD*HUDOGR'LVWULWR SURYHQWRTXHVXSHUHRYDORUGROLPLWHPi[LPRHVWDEH
)HGHUDO OHFLGRSDUDRVEHQHItFLRVGR5*36

†ž 2VLPyYHLVSUySULRVGR'LVWULWR)HGHUDOFRPVLWXDomR †ž 4XDQGRREHQHILFLiULRGDDSRVHQWDGRULDRXGDSHQVmR


GRPLQLDODLQGDQmRWLWXODUL]DGDSHUDQWHR5HJLVWURGH IRUSRUWDGRUGHGRHQoDLQFDSDFLWDQWHDFRQWULEXLomR
,PyYHLV FRPSHWHQWH VHUmR REMHWR GH SURFHVVR GH GHTXHWUDWDRFDSXWLQFLGLUiDSHQDVVREUHDSDUFHODGH
UHJXODUL]DomR SHOR yUJmR FRPSHWHQWH GR 'LVWULWR SURYHQWRTXHVXSHUHRGREURGROLPLWHPi[LPRHVWDEH
)HGHUDOFRPRQHFHVViULRVXSRUWHMXUtGLFRGD3URFXUD OHFLGRSDUDRVEHQHItFLRVGR5*36
GRULD*HUDO GR 'LVWULWR )HGHUDO SDVVDQGRVH HP
VHJXLGDVXDWLWXODULGDGHSDUDR,SUHY')QRVWHUPRV †ž $FRQWULEXLomRFDOFXODGDVREUHREHQHItFLRGHSHQVmR
GRSDUiJUDIRDQWHULRU SRUPRUWHWHUiFRPREDVHGHFiOFXORRYDORUWRWDOGHVVH
EHQHItFLR LQGHSHQGHQWHPHQWH GR Q~PHUR GH FRWDV
†ž $JHVWmRLPRELOLiULDGR,SUHY')LQGHSHQGHGHDXWRUL VHQGRRYDORUGDFRQWULEXLomRUDWHDGRHQWUHRVSHQVLR
]DomR GR *RYHUQDGRU GR 'LVWULWR )HGHUDO H GHYHUi QLVWDVQDSURSRUomRGHFDGDFRWDSDUWH
REVHUYDURVYDORUHVSUDWLFDGRVSHORPHUFDGRLPRELOLi
ULRVHQGRYHGDGDDDOLHQDomRRXDXWLOL]DomRGRVEHQV $UW (QWHQGHVHFRPRUHPXQHUDomRGHFRQWULEXLomRRYDORU
LPyYHLVDWtWXORJUDWXLWR FRQVWLWXtGRSHORYHQFLPHQWRGRFDUJRHIHWLYRDFUHVFL
GRGDVYDQWDJHQVSHFXQLiULDVSHUPDQHQWHVHVWDEHOHFL
$UW 2V UHFXUVRV SUHYLGHQFLiULRV YLQFXODGRV DR5336') GDV HP OHL GRV DGLFLRQDLV GH FDUiWHU LQGLYLGXDO RX
VHUmR DSOLFDGRV QDV FRQGLo}HV GH PHUFDGR FRP RXWUDVYDQWDJHQVH[FOXtGDV
REVHUYkQFLDGHUHJUDVGHVHJXUDQoDVROYrQFLDOLTXL
GH] UHQWDELOLGDGH SURWHomR H SUXGrQFLD ILQDQFHLUD , DVGLiULDVSDUDYLDJHQV
FRQIRUPHGLUHWUL]HVSUHYLVWDVHPQRUPDHVSHFtILFDGR ,, DDMXGDGHFXVWRHPUD]mRGHPXGDQoDGHVHGH
&RQVHOKR0RQHWiULR1DFLRQDO ,,, DLQGHQL]DomRGHWUDQVSRUWH
,9 RVDOiULRIDPtOLD
$UW )LFDSURLELGDDWUDQVIHUrQFLDGHEHQVGLUHLWRVHDWLYRV 9 RDX[tOLRDOLPHQWDomR
GHTXDOTXHUQDWXUH]DGR,SUHY')DTXDOTXHURXWUR 9, RDX[tOLRFUHFKH
yUJmRGDDGPLQLVWUDomRS~EOLFDEHPFRPRDDOLHQDomR 9,, DVSDUFHODVUHPXQHUDWyULDVSDJDVHPGHFRUUrQFLDGH
RXFRQVWLWXLomRGH{QXVUHDLVVREUHTXDOTXHUEHPGR ORFDOGHWUDEDOKR
VHXSDWULP{QLRDWtWXORJUDWXLWRDRVPHVPRVyUJmRV 9,,, D SDUFHOD SHUFHELGD HP GHFRUUrQFLD GR H[HUFtFLRGH
FDUJRHPFRPLVVmRRXGHIXQomRGHFRQILDQoD
$UW $VUHFHLWDVGHTXHWUDWDRDUWGHVWD/HL&RPSOH ,; RDERQRGHSHUPDQrQFLDGHTXHWUDWDRDUWGHVWD
PHQWDUVHUmRXWLOL]DGDVVRPHQWHSDUDSDJDPHQWRVGRV /HL&RPSOHPHQWDU
EHQHItFLRV SUHYLGHQFLiULRV YHGDGD D XWLOL]DomR SDUD ; RDGLFLRQDOGHIpULDV
ILQVDVVLVWHQFLDLVHGHVD~GHEHPFRPRSDUDFRQFHVVmR ;, RXWUDV SDUFHODV FXMR FDUiWHU LQGHQL]DWyULRHVWHMD
GH YHUEDV LQGHQL]DWyULDV DLQGD TXH SRU DFLGHQWH GH GHILQLGRHPOHL
VHUYLoR
†ž 2 VHJXUDGR DWLYR SRGHUi RSWDU SHODLQFOXVmRQD
6HomR, UHPXQHUDomRGHFRQWULEXLomRGHSDUFHODVUHPXQHUDWy
'R&DUiWHU&RQWULEXWLYR ULDVSHUFHELGDVHPGHFRUUrQFLDGHORFDOGHWUDEDOKRGR
H[HUFtFLR GH FDUJR HP FRPLVVmR RX GH IXQomR GH
$UW $ FRQWULEXLomR SUHYLGHQFLiULD SDWURQDO GR'LVWULWR FRQILDQoD SDUD HIHLWR GH FiOFXOR GR EHQHItFLR D VHU
)HGHUDOGHTXHWUDWDRDUW,VHUiGH FRQFHGLGRFRPIXQGDPHQWRQRVDUWV
HUHVSHLWDGDHPTXDOTXHUKLSyWHVHDOLPLWDomR
, SDUDR)XQGR)LQDQFHLURGH3UHYLGrQFLD6HJXULGDGH HVWDEHOHFLGDQRDUW†ž
6RFLDOGHTXHWUDWDRDUW†žGHVWD/HL&RPSOH
PHQWDUGHQRPtQLPRRHTXLYDOHQWHjDOtTXRWDGH †ž eYHGDGDDLQFOXVmRQRVEHQHItFLRVGHDSRVHQWDGRULDH
FRQWULEXLomRGRVVHJXUDGRVDWLYRVHGHQRPi[LPRR SHQVmR SDUD HIHLWR GH SHUFHSomR GHOHV GH SDUFHODV
GREUR SDUD RV TXH WHQKDP LQJUHVVDGR QR VHUYLoR UHPXQHUDWyULDV SDJDV HP GHFRUUrQFLD GH ORFDO GH
S~EOLFRDWpGHGH]HPEURGH WUDEDOKRGHIXQomRGHFRQILDQoDGHFDUJRHPFRPLV
VmRRXGRDERQRGHSHUPDQrQFLDGHTXHWUDWDRDUW
,, SDUD R )XQGR 3UHYLGHQFLiULR GR 'LVWULWR)HGHUDO 
UHIHULGRQRDUW†žGHVWD/HL&RPSOHPHQWDUR
GREUR GD FRQWULEXLomR GRV VHUYLGRUHV DWLYRV TXH $UW $VFRQWULEXLo}HVGHQDWXUH]DSDWURQDOEHPFRPRDV
WHQKDPLQJUHVVDGRQRVHUYLoRS~EOLFRDSDUWLUGHžGH FRQWULEXLo}HV GRV VHJXUDGRV DWLYRV LQDWLYRV H GRV
MDQHLURGH SHQVLRQLVWDVSUHYLVWDVQRDUW,,,H,,,REHGHFHUmR
DR3ODQRGH&XVWHLRHVHUmRUHSDVVDGDVDR,SUHY')
†~QLFR $V DOtTXRWDV GH FRQWULEXLomR SUHYLVWDVQHVWHDUWLJR SHOR7HVRXURGR'LVWULWR)HGHUDO
VHUmRREMHWRGHUHDYDOLDomRDWXDULDODQXDOHGHYHUmR
FRQVWDUGD/HLGH'LUHWUL]HV2UoDPHQWiULD/'2 †~QLFR 2UHSDVVHGDVFRQWULEXLo}HVGHILQLGDVQRFDSXWRFRUUH
UiHPDWp FLQFR GLDVFRQWDGRVGDGDWDGHSDJDPHQ
$UW $FRQWULEXLomRSUHYLGHQFLiULDGRVVHJXUDGRVDWLYRVGH WRGR~OWLPRJUXSRTXHFRPS}HDVIROKDVGHSDJDPHQ
TXHWUDWDRDUW,,VHUiGH RQ]HSRUFHQWR  WRVUHIHUHQWHVDRVVXEVtGLRVjUHPXQHUDomRjJUDWLIL
FRQIRUPH /HL &RPSOHPHQWDU 'LVWULWDO Qž  FDomRQDWDOtFLDHjGHFLVmRMXGLFLDORXDGPLQLVWUDWLYD
LQFLGHQWH VREUH D UHPXQHUDomRGHFRQWULEXLomR
FRQIRUPHRGLVSRVWRQRDUW
  /HJLVODomR 6HFUHWDULDGH(VWDGRGH$GPLQLVWUDomR3~EOLFD'LVWULWR)HGHUDO


$UW $ JUDWLILFDomR QDWDOtFLD VHUi FRQVLGHUDGD SDUDILQV †ž $ LQREVHUYkQFLD SRU  WUrV  PHVHVFRQVHFXWLYRVGR
FRQWULEXWLYRV VHSDUDGDPHQWH GD UHPXQHUD UHFROKLPHQWR SUHYLGHQFLiULR RFDVLRQDUi D VXVSHQVmR
omRGHFRQWULEXLomRUHODWLYDDRPrVHPTXHIRUSDJR GRVGLUHLWRVSUHYLGHQFLiULRVGRVHJXUDGRHVHXVGHSHQ
GHQWHVVyUHDYHQGRHOHVRGLUHLWRDRVEHQHItFLRVDSyV
$UW 3DUDRVHJXUDGRHPUHJLPHGHDFXPXODomRUHPXQHUD TXLWDomRGRWRWDOGRGpELWRGDVFRQWULEXLo}HVSUHYLGHQ
GDGHFDUJRVFRQVLGHUDUVHiSDUDILQVGR5336') FLiULDVTXHSRGHVHUIHLWDSRUPHLRGHSDUFHODPHQWR
RVRPDWyULRGDUHPXQHUDomRGHFRQWULEXLomRUHIHUHQWH FRQIRUPHFULWpULRGLVSRVWRSHOD'LUHWRULD([HFXWLYDGR
DFDGDFDUJR ,SUHY') PHGLDQWH GHVFRQWRV LQFLGHQWHV VREUH RV
SURYHQWRVGHDSRVHQWDGRULDRXEHQHItFLRVGHSHQVmR
$UW 1DFHVVmRGHVHUYLGRUHVSDUDRXWURHQWHIHGHUDWLYRHP SRUPRUWH
TXHRSDJDPHQWRGDUHPXQHUDomRVHMDFRP{QXVGR
yUJmRRXGDHQWLGDGHFHVVLRQiULDVHUiGHUHVSRQVDELOL $UW 2UHFROKLPHQWRGDVFRQWULEXLo}HVGRVVHJXUDGRVDWLYRV
GDGHGHVWD pGHUHVSRQVDELOLGDGHGRyUJmRRXHQWLGDGHHPTXHR
VHUYLGRUHVWLYHUHPH[HUFtFLRQRVVHJXLQWHVFDVRV
, RGHVFRQWRGDFRQWULEXLomRGHYLGDSHORVHUYLGRU
,, DFRQWULEXLomRGHYLGDSHORHQWHGHRULJHP , FHGLGRSDUDRXWURyUJmRRXHQWLGDGHGD$GPLQLVWUDomR
GLUHWDRXLQGLUHWDGD8QLmRGRV(VWDGRVGR'LVWULWR
†ž &DEHUiDRFHVVLRQiULRHIHWXDURUHSDVVHGDVFRQWULEXL )HGHUDORXGRV0XQLFtSLRV
o}HVFRUUHVSRQGHQWHVDRHQWHIHGHUDWLYRHDRVHUYLGRU
jXQLGDGHJHVWRUDGR5336GRHQWHIHGHUDWLYRFHGHQWH ,, LQYHVWLGR HP PDQGDWR HOHWLYR IHGHUDOHVWDGXDO
GLVWULWDORXPXQLFLSDOQRVWHUPRVGRDUWGD&RQVWL
†ž &DVRRFHVVLRQiULRQmRHIHWXHRUHSDVVHGDVFRQWULEXL WXLomRGD5HS~EOLFDGHVGHTXHRDIDVWDPHQWRGRFDUJR
o}HVjXQLGDGHJHVWRUDQRSUD]ROHJDOFDEHUiDRHQWH VHGrFRPSUHMXt]RGDUHPXQHUDomRRXVXEVtGLR
IHGHUDWLYRFHGHQWHHIHWXiOREXVFDQGRRUHHPEROVRGH
WDLVYDORUHVMXQWRDRFHVVLRQiULR $UW 2 7HVRXUR GR 'LVWULWR )HGHUDO p UHVSRQViYHOSHOD
FREHUWXUD GH HYHQWXDLV LQVXILFLrQFLDV ILQDQFHLUDV
†ž 2WHUPRRXDWRGHFHVVmRGRVHUYLGRUFRP{QXVSDUD PHQVDLV GR 5336') GHFRUUHQWHV GR SDJDPHQWR GH
RFHVVLRQiULRGHYHUiSUHYHUDUHVSRQVDELOLGDGHGHVWH EHQHItFLRVSUHYLGHQFLiULRVHREVHUYDUiDSURSRUFLRQDOL
SHORGHVFRQWRUHFROKLPHQWRHUHSDVVHGDVFRQWULEXL GDGH GDV GHVSHVDV HQWUH RV 3RGHUHV ([HFXWLYR H
o}HV SUHYLGHQFLiULDV DR UHJLPH GH RULJHP UHODWLYD /HJLVODWLYRGR'LVWULWR)HGHUDOLQFOXtGRVR7ULEXQDOGH
PHQWHjSDUWHSDWURQDOHjSDUWHGRVHJXUDGRFRQIRU &RQWDVDXWDUTXLDVHIXQGDo}HV
PHYDORUHVLQIRUPDGRVPHQVDOPHQWHSHORFHGHQWH
$UW $V FRQWULEXLo}HV SUHYLGHQFLiULDV H GHPDLVGpELWRV
$UW 1D FHVVmR GH VHUYLGRUHV SDUD RXWUR HQWHIHGHUDWLYR SUHYLGHQFLiULRVQmRUHFROKLGRVDWpRSUD]RHVWDEHOHFL
VHP{QXVSDUDRFHVVLRQiULRFRQWLQXDUiVREDUHVSRQ GRQRDUWSDUiJUDIR~QLFRGDSUHVHQWH/HL&RP
VDELOLGDGH GR FHGHQWH R GHVFRQWR H R UHSDVVH GDV SOHPHQWDU GHYHUmR VHU DWXDOL]DGRV PRQHWDULDPHQWH
FRQWULEXLo}HVjXQLGDGHJHVWRUDGR5336 SHORV PHVPRV tQGLFHV SUDWLFDGRV HP UHODomR DRV
GpELWRVSDUDFRPR5*36HVRIUHUmRDLQFLGrQFLDGH
$UW 1DVKLSyWHVHVGHFHVVmROLFHQFLDPHQWRRXDIDVWDPHQWR PXOWDGH GRLVSRUFHQWR DRPrVDOpPGRVMXURV
GHVHUYLGRUGHTXHWUDWDRDUWžRFiOFXORGDFRQWUL GHPRUDGH GH]FHQWpVLPRVSRUFHQWR SRUGLD
EXLomR VHUi IHLWR GH DFRUGR FRP D UHPXQHUDomR GR GHDWUDVR
FDUJRHIHWLYRGHTXHRVHUYLGRUpWLWXODU 6HomR,,
'R3ODQRGH&XVWHLR
†~QLFR 1mR LQFLGLUmR FRQWULEXLo}HV SDUD R 5336GRHQWH
FHGHQWH RX GR HQWH FHVVLRQiULR QHP SDUD R 5*36 $UW 25336')VHUiILQDQFLDGRPHGLDQWHRUHJLPHILQDQ
VREUHDVSDUFHODVUHPXQHUDWyULDVFRPSOHPHQWDUHVQmR FHLURGHUHSDUWLomRVLPSOHVGHUHVHUYDVPDWHPiWLFDVH
FRPSRQHQWHVGDUHPXQHUDomRGRFDUJRHIHWLYRSDJDV UHJLPH FDSLWDOL]DGR FRP D JHVWmR GH XP IXQGR GH
SHOR HQWH FHVVLRQiULR DR VHUYLGRU FHGLGR H[FHWR QD QDWXUH]DILQDQFHLUDHXPIXQGRGHQDWXUH]DSUHYLGHQ
KLSyWHVH HP TXH KRXYHU D RSomR SHOD FRQWULEXLomR FLiULDSDUDFREHUWXUDGHEHQHItFLRVSUHYLGHQFLiULRV
IDFXOWDWLYDDR5336GRHQWHFHGHQWHQDIRUPDSUHYLVWD
HPVXDOHJLVODomRFRQIRUPHDUW †ž )LFD LQVWLWXtGR R )XQGR )LQDQFHLUR GH3UHYLGrQFLD
6(*85,'$'(62&,$/ FRPDVHJXLQWHGHVWLQDomRH
$UW 2VHUYLGRUDIDVWDGRRXOLFHQFLDGRWHPSRUDULDPHQWHGR FDUDFWHUtVWLFDV
H[HUFtFLRGRFDUJRHIHWLYRVHPUHFHELPHQWRGHUHPX
QHUDomRGRHQWHIHGHUDWLYRLQFOXVLYHRVDIDVWDGRVSDUD , GHVWLQDGRDRSDJDPHQWRGHEHQHItFLRVSUHYLGHQFLiULRV
RH[HUFtFLRGHPDQGDWRHOHWLYRHPRXWURHQWHIHGHUDWL DRV VHJXUDGRV TXH WHQKDP LQJUHVVDGR QR VHUYLoR
YRVRPHQWHFRQWDUiRUHVSHFWLYRWHPSRGHDIDVWDPHQ S~EOLFRDWpGHGH]HPEURGHEHPFRPRDRV
WR RX OLFHQFLDPHQWR SDUD ILQV GH DSRVHQWDGRULD TXHMiUHFHELDPEHQHItFLRVQHVVDGDWDHRVUHVSHFWLYRV
PHGLDQWH R UHFROKLPHQWR PHQVDO GDV FRQWULEXLo}HV GHSHQGHQWHV
SUHYLGHQFLiULDVUHODWLYDVjSDUWHSDWURQDOHjSDUWHGR
VHJXUDGR ,, EDVHDGRQRUHJLPHGHUHSDUWLomRVLPSOHVHPTXHWRGD
D DUUHFDGDomR p XWLOL]DGD SDUD R SDJDPHQWR GRV
†ž 2VHJXUDGRHPDWLYLGDGHTXHVHHQFRQWUHHPJR]RGH EHQHItFLRVHPPDQXWHQomRQRPHVPRH[HUFtFLR
OLFHQoDVHPYHQFLPHQWRVVHP{QXVjDGPLQLVWUDomR
S~EOLFDGR'LVWULWR)HGHUDOSDUDILQVGHDVVHJXUDUR ,,, ILQDQFLDGR SHODV FRQWULEXLo}HV SUHYLGHQFLiULDVGRV
FXVWHLR GH VHX EHQHItFLR IXWXUR GHYHUi HIHWXDU R VHUYLGRUHVDWLYRVLQDWLYRVHSHQVLRQLVWDVFRQWULEXLomR
UHFROKLPHQWRPHQVDODVHUFDOFXODGRFRPEDVHQDVXD SDWURQDOSRUDSRUWHVILQDQFHLURVGR'LVWULWR)HGHUDO
UHPXQHUDomR EHP FRPR GHPDLV YDQWDJHQV GH ILQV SRUUHFXUVRVGDDOLHQDomRGHEHQVSRURXWURVUHFXUVRV
SUHYLGHQFLiULRVGLUHWDPHQWHDR,SUHY')RXPHGLDQWH H GLUHLWRV TXH OKH IRUHP GHVWLQDGRV H LQFRUSRUDGRV
GHSyVLWREDQFiULR GHVGHTXHDFHLWRVSHOR&RQVHOKRGH$GPLQLVWUDomRGR
6HFUHWDULDGH(VWDGRGH$ GPLQLVWUDomR3~EOLFD'LVWULWR)HGHUDO
 /HJLVODomR 

,SUHY')SHORSURGXWRGHDSOLFDo}HVILQDQFHLUDVHGH †~QLFR $VGLVSRQLELOLGDGHVGHFDL[DGR5336')GHYHUmRVHU


LQYHVWLPHQWRVSHORVYDORUHVGHFRUUHQWHVGD&RPSHQ VHPSUHGHSRVLWDGDVHPDQWLGDVHPFRQWDVEDQFiULDV
VDomR3UHYLGHQFLiULDHQWUHUHJLPHVHSHODFREHUWXUDGH HPQRPHGR,SUHY')VHSDUDGDVGDVGHPDLVGLVSRQL
HYHQWXDLVLQVXILFLrQFLDVILQDQFHLUDVPHQVDLVGR5336 ELOLGDGHVGR7HVRXURGR'LVWULWR)HGHUDO
')SHOR7HVRXURGR'LVWULWR)HGHUDO
6HomR,9
†ž )LFD LQVWLWXtGR R )XQGR 3UHYLGHQFLiULRGR'LVWULWR 'D'HVSHVDHGD&RQWDELOLGDGH
)HGHUDO')35(9FRPDVHJXLQWHGHVWLQDomRHFDUDF
WHUtVWLFDV $UW 2,SUHY')REVHUYDUiQRUPDVHSULQFtSLRVGD$GPLQLV
WUDomRH)LQDQoDV3~EOLFDVIL[DGRVSHOD8QLmRHSHOR
, GHVWLQDGR DRV VHUYLGRUHV TXH WHQKDP LQJUHVVDGRQR 'LVWULWR )HGHUDO SULQFLSDOPHQWH D /HL )HGHUDO Qž
VHUYLoRS~EOLFRDSDUWLUGHžGHMDQHLURGHHDRV  D /HL )HGHUDO Qž  H D /HL
VHXVGHSHQGHQWHV &RPSOHPHQWDUQž/HLGH5HVSRQVDELOLGDGH
)LVFDOFRPVXDVDOWHUDo}HVHPRGLILFDo}HV
,, EDVHDGRQRVLVWHPDGHFDSLWDOL]DomRTXHLPSOLTXHD
IRUPDomR GH UHVHUYDV DV TXDLV VHUmR GHYLGDPHQWH $UW 2 ,SUHY') PDQWHUi UHJLVWUR LQGLYLGXDOL]DGRGRV
DSOLFDGDVQDVFRQGLo}HVGHPHUFDGRFRPREVHUYkQFLD VHJXUDGRVGRUHJLPHSUySULRTXHFRQWHUiDVVHJXLQWHV
GHUHJUDVGHVHJXUDQoDVROYrQFLDOLTXLGH]UHQWDELOL LQIRUPDo}HV
GDGH SURWHomR H SUXGrQFLD ILQDQFHLUD FRQIRUPH
GLUHWUL]HVSUHYLVWDVHPQRUPDHVSHFtILFDGR&RQVHOKR , QRPHHGHPDLVGDGRVSHVVRDLVLQFOXVLYHGRVGHSHQ
0RQHWiULR1DFLRQDOHOHJLVODomRDSOLFiYHOHGHVWLQDGR GHQWHV
DDVVHJXUDURFXVWHLRGRVEHQHItFLRVSUHYLGHQFLiULRV ,, PDWUtFXODHRXWURVGDGRVIXQFLRQDLV
,,, UHPXQHUDomRGHFRQWULEXLomRPrVDPrV
,,, IRUPDGRSRUFRQWULEXLo}HVSUHYLGHQFLiULDVGRVVHUYLGR ,9 YDORUHVPHQVDLVGDFRQWULEXLomRGRVHJXUDGR
UHVGR'LVWULWR)HGHUDOHSHODFRQWULEXLomRSDWURQDO 9 YDORUHVPHQVDLVGDFRQWULEXLomRGRHQWHIHGHUDWLYR
DUUHFDGDGDV DR ORQJR GR SHUtRGR ODERUDWLYR SDUD
DVVHJXUDU R FXVWHLR GRV EHQHItFLRV SUHYLGHQFLiULRV †ž $RVVHJXUDGRVVHUmRGLVSRQLELOL]DGDVDVLQIRUPDo}HV
VHQGR GH UHVSRQVDELOLGDGH GR 7HVRXUR GR 'LVWULWR FRQVWDQWHVGHVHXUHJLVWURLQGLYLGXDOL]DGRPHGLDQWH
)HGHUDODFREHUWXUDGHHYHQWXDLVLQVXILFLrQFLDVILQDQ H[WUDWRDQXDOUHODWLYDVDRH[HUFtFLRILQDQFHLURDQWHUL
FHLUDV RU
$UW 2V EHQHItFLRV GR 3ODQR &DSLWDOL]DGR SRGHUmRVHU †ž 2VYDORUHVFRQVWDQWHVGRUHJLVWURFDGDVWUDOLQGLYLGXDOL
ILQDQFLDGRVSRU5HSDUWLomRFRP&DSLWDLVGH&REHUWXUD ]DGRVHUmRFRQVROLGDGRVSDUDILQVFRQWiEHLV
5HSDUWLomR6LPSOHVRX&DSLWDOL]DomRFRQIRUPHRWLSR
GHSUHVWDomRGHILQLGRSHOR,SUHY')DQXDOPHQWHSRU $UW &RPSHWHDR,SUHY')UHDOL]DUDVVHJXLQWHVGHVSHVDV
RFDVLmRGDUHDYDOLDomRDWXDULDOGHIRUPDDJDUDQWLUR
HTXLOtEULRILQDQFHLURHDWXDULDO , GH EHQHItFLRV SUHYLGHQFLiULRV SUHYLVWRV QHVWD/HL
&RPSOHPHQWDUHHPFRQIRUPLGDGHFRPDOHJLVODomR
$UW $ 'LUHWRULD ([HFXWLYD GR ,SUHY') GHYHUi UHYHUR
IHGHUDO
SODQRGHFXVWHLRDQXDOPHQWHFRPEDVHHPDYDOLDo}HV
DWXDULDLVDVHUHPUHDOL]DGDVVRPHQWHSRUHPSUHVDGR
,, GHSHVVRDOSUySULRGR,SUHY')FRPVHXVUHVSHFWLYRV
UDPR RX SURILVVLRQDO UHJXODUPHQWH FDGDVWUDGR QR
HQFDUJRV
,QVWLWXWR%UDVLOHLURGH$WXiULDFRQWHQGRQHFHVVDULD
PHQWH
,,, GHPDWHULDOSHUPDQHQWHHGHFRQVXPRFRPRWRGRVRV
LQVXPRVQHFHVViULRVjPDQXWHQomRGR5336')
, RUHJLPHILQDQFHLURXWLOL]DGR
,9 GHPDQXWHQomRHGHDSHUIHLoRDPHQWRGRVLQVWUXPHQ
,, GLVFULPLQDomRGHFRPSURPLVVRVGHQDWXUH]DSUHYLGHQ
WRVGHJHVWmRGR5336')
FLiULDGHPRQVWUDGRVDWXDULDOPHQWH
9 FRPLQYHVWLPHQWRVHPFRQIRUPLGDGHFRPDVQRUPDVH
,,, WRWDOGHUHVHUYDVFDVRH[LVWHQWHV
UHJXODPHQWRVYLJHQWHVSDUDDDSOLFDomRGRVUHFXUVRV
,9 HVWLPDWLYDGHGHVSHVDVGHFDUiWHUDGPLQLVWUDWLYRHGH SUHYLGHQFLiULRV
SHVVRDO
9, FRPVHJXURGHEHQV SHUPDQHQWHVSDUDSURWHomRGR
9 HVWLPDWLYDGHDSRUWHVH[WUDRUGLQiULRVQHFHVViULRVDR SDWULP{QLRGR5336')DSOLFDGDVVXEVLGLDULDPHQWHDV
FXPSULPHQWRGHVXDVREULJDo}HVEHPFRPRjFRQVWL UHJUDVHQRUPDVYLJHQWHV
WXLomRGHUHVHUYDVSDUDFXVWHLRGHEHQHItFLRVIXWXURV
9,, FRP RXWURV HQFDUJRV HYHQWXDLV YLQFXODGRV jVVXDV
6HomR,,, ILQDOLGDGHVHVVHQFLDLV
'D6HSDUDomRGDV&RQWDVGR5HJLPH3UySULR
GH3UHYLGrQFLD6RFLDOGR'LVWULWR)HGHUDO $UW 2 SDJDPHQWR GRV EHQHItFLRV SUHYLGHQFLiULRVGRV
VHJXUDGRV GH FDGD 3RGHU RX yUJmR VXERUGLQDGRV DR
$UW 2,SUHY')SDUDSHUPLWLUSOHQRFRQWUROHILQDQFHLURH 5336')GHTXHWUDWDHVWD/HL&RPSOHPHQWDUVHUi
FRQWiELOGHVXDVUHFHLWDVLPSODQWDUiJUDGXDOPHQWH UHDOL]DGRQDPHVPDGDWDHPTXHRFRUUHURSDJDPHQWR
GRVVHJXUDGRVVHUYLGRUHVDWLYRVDHOHVYLQFXODGRV
, FRQWUROHGLVWLQWRGHFRQWDVEDQFiULDVHFRQWDELOLGDGH
GR3ODQR $UW eYHGDGRRSDJDPHQWRGHEHQHItFLRVSUHYLGHQFLiULRV
GHTXHWUDWDHVWD/HL&RPSOHPHQWDUPHGLDQWHFRQYr
,, UHJLVWURV LQGLYLGXDOL]DGRV GDV FRQWULEXLo}HVSRU QLRFRQVyUFLRRXRXWUDIRUPDGHDVVRFLDomRGR'LVWULWR
VHJXUDGRHGR3ODQR )HGHUDOFRPD8QLmR(VWDGRVRX0XQLFtSLRV
  /HJLVODomR 6HFUHWDULDGH(VWDGRGH$GPLQLVWUDomR3~EOLFD'LVWULWR)HGHUDO


$UW $ SDUWLU GD FRPSHWrQFLD GH MDQHLUR GH VHUi 9,,, SURYLPHQWRGHVLVWHPDS~EOLFRHVROLGiULRGHSUHYLGrQ
XWLOL]DGRREULJDWRULDPHQWHR3ODQRGH&RQWDVDSURYD FLDVRFLDO
GRSHOR0LQLVWpULRGD3UHYLGrQFLD6RFLDO
$UW 2,SUHY')DXWDUTXLDFRPVHGHHIRURQD&DSLWDOGD
6HomR9 5HS~EOLFDJR]DHPWRGDDVXDSOHQLWXGHQRTXHVH
'D$YDOLDomR$WXDULDO UHIHUHD VHXV EHQVVHUYLoRVHDo}HVGRVSULYLOpJLRV
LQFOXVLYHGHQDWXUH]DSURFHVVXDOHWULEXWiULDHLPXQL
$UW 2,SUHY')GHYHUiSURPRYHUDYDOLDomRDWXDULDOSDUD GDGHVJDUDQWLGRVDRVyUJmRVGRVHQWHVS~EOLFRVIHGHUD
DGHWHUPLQDomRGHWD[DGHFXVWHLRSDUDDWUDQVIRUPD WLYRV
omRGHFDSLWDLVFXPXODWLYRVHPYDORUHVGHEHQHItFLRH
SDUDDGHWHUPLQDomRGHUHVHUYDVPDWHPiWLFDVHQWUH $UW 2,SUHY')FRQWDUiFRPRVVHJXLQWHVyUJmRVQDVXD
RXWUDV QD IRUPD HVWDEHOHFLGD QD OHJLVODomR IHGHUDO HVWUXWXUD
DSOLFiYHO
, &RQVHOKRGH$GPLQLVWUDomR
,, &RQVHOKR)LVFDO
$UW $VDOtTXRWDVGHFRQWULEXLomRSUHYLVWDVQHVWD/HL&RP
,,, 'LUHWRULD([HFXWLYD
SOHPHQWDUGHYHUmRVHUUHYLVWDVFRPEDVHQDDYDOLDomR
DWXDULDO GR SODQR DQXDO GH FXVWHLR SRU RFDVLmR GR $UW 2&RQVHOKRGH$GPLQLVWUDomRGR,SUHY')VHUiFRP
HQFHUUDPHQWRGREDODQoRDQXDOGR5336') SRVWR SRU  TXDWRU]H  PHPEURV QRPHDGRV SHOR
*RYHUQDGRUGR'LVWULWR)HGHUDODVDEHU
†~QLFR &RQVWDWDGDDH[LVWrQFLDGHGpILFLWWpFQLFRDWXDULDOR
,SUHY')FRPXQLFDUiDR&KHIHGR3RGHU([HFXWLYRD
, R6HFUHWiULRGH(VWDGRGH3ODQHMDPHQWRH*HVWmR
TXHPFDEHUiDLQLFLDWLYDGHUHPHWHUDR3RGHU/HJLVODWL
YRSURMHWRGHOHLSURSRQGRDOWHUDomRGDVDOtTXRWDVGH ,, R6HFUHWiULRGH(VWDGRGH*RYHUQR
FRQWULEXLomRjH[FHomRGDVDOtTXRWDVGHFRQWULEXLomR
HVWDEHOHFLGDV SDUD RV VHUYLGRUHV DWLYRV LQDWLYRV H ,,, R6HFUHWiULRGH(VWDGRGH)D]HQGD
SHQVLRQLVWDV TXH Vy SRGHUmR VHU PDMRUDGDV SDUD
DFRPSDQKDU D DOtTXRWD GH FRQWULEXLomR PtQLPD ,9 26HFUHWiULR$GMXQWRGH*RYHUQRGR'LVWULWR)HGHUDO
SUDWLFDGDSHOD8QLmRDRVVHXVVHUYLGRUHVWLWXODUHVGH QRYDUHGDomRGDGDDRLQFLVRLYGRDUWSHODOHLFRPSOHPHQWDUQžGH

FDUJRVHIHWLYRV
9  XP  UHSUHVHQWDQWH GD &kPDUD /HJLVODWLYDGR
&DStWXOR,; 'LVWULWR)HGHUDO
'D*HVWmRH(VWUXWXUD$GPLQLVWUDWLYD
9,  XP  UHSUHVHQWDQWH GR 7ULEXQDO GH &RQWDVGR
$UW 2,SUHY')GHYHUiREVHUYDUQDVXDDWXDomRRVVHJXLQ 'LVWULWR)HGHUDO
WHVSDUkPHWURVDOpPGRVSULQFtSLRVEiVLFRVUHJHQWHV
GDDWLYLGDGHS~EOLFD 9,,  VHWH UHSUHVHQWDQWHVGRVVHJXUDGRVSDUWLFLSDQWHV
RXEHQHILFLiULRVLQGLFDGRVSHODVHQWLGDGHVUHSUHVHQWD
WLYDVGRVVHUYLGRUHVDWLYRVLQDWLYRVRXSHQVLRQLVWDVGR
, JHVWmRILQDQFHLUDHDGPLQLVWUDWLYDGHVFHQWUDOL]DGDHP
'LVWULWR)HGHUDODVVHJXUDGDSHORPHQRVXPDLQGLFDomR
UHODomRDR(VWDGRGHYHQGRSDUDWDQWRRSHUDUFRP
DHQWLGDGHVUHSUHVHQWDWLYDVGRVVHUYLGRUHVGR3RGHU
FRQWDVSUySULDVGLVWLQWDVGDVGR7HVRXURGR'LVWULWR /HJLVODWLYR 1RYDUHGDomRGDGDDRLQFLVR9,,GRDUWSHODOHLFRPSOHPHQWDU
)HGHUDO QžGHGRGIGH

,, SOHQRDFHVVRGDVLQIRUPDo}HVUHIHUHQWHVjVXDJHVWmR 9,,, R'LUHWRU3UHVLGHQWHGR,SUHY')


DRV VHJXUDGRV H GHSHQGHQWHV H D SDUWLFLSDomR GH
UHSUHVHQWDQWHV GRV VHUYLGRUHV S~EOLFRV WLWXODUHV GH †ž 23UHVLGHQWHGR&RQVHOKRGH$GPLQLVWUDomRVHUiHOHLWR
FDUJRVHIHWLYRVDWLYRVHLQDWLYRVQRVFROHJLDGRVHP SHORVVHXVSDUHV
TXHVHXVLQWHUHVVHVVHMDPREMHWRGHGLVFXVVmRHGHOLEH
UDomR †ž $VUHXQL}HVGR&RQVHOKRVHLQVWDODUmRFRPDSUHVHQoD
GDPDLRULDDEVROXWDGHVHXVPHPEURV
,,, SUHVHUYDomRGRHTXLOtEULRILQDQFHLURHDWXDULDO
†ž 2&RQVHOKRGHOLEHUDUiSRUPDLRULDVLPSOHVGHYRWRV
,9 FXVWHLR H[FOXVLYR GD SUHYLGrQFLD VRFLDO GHFDUiWHU FDEHQGR DR 3UHVLGHQWH GR &RQVHOKR HP FDVR GH
FRQWULEXWLYR H VROLGiULR PHGLDQWH FRQWULEXLo}HV HPSDWH QDV GHOLEHUDo}HV DOpP GR VHX R YRWR GH
TXDOLGDGH
YHUWLGDVSHORVyUJmRVGHTXHWUDWDRDUWžGHVWD/HL
&RPSOHPHQWDUGRVVHXVVHUYLGRUHVWLWXODUHVGHFDUJRV
†ž &DGD PHPEUR GR &RQVHOKR SRVVXLUiXPVXSOHQWH
HIHWLYRV DWLYRV H LQDWLYRV LQFOXtGRV RV SHQVLRQLVWDV
GHVLJQDGR QD IRUPD GHVWH DUWLJR H QRPHDGR SHOR
DOpPGRVUHFXUVRVREWLGRVSHODJHVWmRGHUHFXUVRVH *RYHUQDGRUGR'LVWULWR)HGHUDO
DWLYRVGHVWLQDGRVDRVHXSDWULP{QLR
$UW 2&RQVHOKR)LVFDOVHUiFRPSRVWRSRU WUrV PHP
9 YHGDomRGDFULDomRPDMRUDomRRXH[WHQVmRGHTXDLV EURVHIHWLYRVH WUrV PHPEURVVXSOHQWHVVHQGR
TXHU EHQHItFLRV VHP D LQGLFDomR GH VXD IRQWH GH GRLV  HVFROKLGRV HQWUH VHJXUDGRV RX EHQHILFLiULRV
FXVWHLRWRWDO LQGLFDGRVSHODVUHVSHFWLYDVHQWLGDGHVUHSUHVHQWDWLYDV
GH FODVVH H  XP  LQGLFDGR SHOR *RYHUQDGRU GR
9, UHDOL]DomRGHHVFULWXUDomRFRQWiELOGLVWLQWDGR7HVRXUR 'LVWULWR)HGHUDO
GR'LVWULWR)HGHUDOLQFOXVLYHGHUXEULFDVGHVWDFDGDV
QRV RUoDPHQWRV SDUD SDJDPHQWRV GRV EHQHItFLRV †~QLFR 2V PHPEURV D TXH VH UHIHUH R FDSXWGHYHUmRWHU
SUHYLGHQFLiULRV IRUPDomRVXSHULRUHPDGPLQLVWUDomRFLrQFLDVFRQWiEH
LVHFRQ{PLFDVRXDWXDULDLV
9,, PDQXWHQomRGHUHJLVWURLQGLYLGXDOGRVVHJXUDGRV
$UW &RPSHWHDR&RQVHOKRGH$GPLQLVWUDomRGR,SUHY')
6HFUHWDULDGH(VWDGRGH$ GPLQLVWUDomR3~EOLFD'LVWULWR)HGHUDO
 /HJLVODomR 

, UHXQLUVHRUGLQDULDPHQWHXPDYH]HPFDGDWULPHVWUH †ž $'LUHWRULDGH3UHYLGrQFLDVHUiRFXSDGDSRUVHJXUDGR


FLYLOSRUFRQYRFDomRGHVHX3UHVLGHQWHHH[WUDRUGLQD RXEHQHILFLiULRHVFROKLGRSHOR*RYHUQDGRUGR'LVWULWR
ULDPHQWH SRU FRQYRFDomR GH VHX 3UHVLGHQWH RX GD )HGHUDOGHQWUHRVLQGLFDGRVSHODVHQWLGDGHVUHSUHVHQ
PDLRULDGHVHXVPHPEURV WDWLYDVGRVVHUYLGRUHVHPOLVWDVr[WXSOD

,, IL[DU DV GLUHWUL]HV JHUDLV GH JHVWmR LQYHVWLPHQWRH †ž 2VPHPEURVLQGLFDGRVSHODVHQWLGDGHVUHSUHVHQWDWLYDV
DORFDomRGRVUHFXUVRV GRVVHUYLGRUHVGHYHUmRDWHQGHURVVHJXLQWHVUHTXLVLWRV

,,, H[HUFHUDVXSHUYLVmRGDVRSHUDo}HVGR,SUHY') , WHUFRPSURYDGDH[SHULrQFLDQRH[HUFtFLRGHDWLYLGDGH


QD iUHD SUHYLGHQFLiULD ILQDQFHLUD DGPLQLVWUDWLYD
,9 H[DPLQDU H DSURYDU DQXDOPHQWH VXDDYDOLDomR FRQWiELOMXUtGLFDGHILVFDOL]DomRRXGHDXGLWRULD
DWXDULDOHRSODQRGHFXVWHLR
,, QmR WHU VRIULGR FRQGHQDomR FULPLQDO WUDQVLWDGDHP
9 DXWRUL]DUDFHOHEUDomRGHFRQWUDWRVDFRUGRVHFRQYrQL MXOJDGRHPFULPHGHUHVSRQVDELOLGDGHFULPHFRQWUDD
RVTXHLPSRUWHPQDFRQVWLWXLomRGH{QXVUHDLVVREUH DGPLQLVWUDomR S~EOLFD RX HP LOtFLWR GH LPSURELGDGH
RVEHQVGR,SUHY') DGPLQLVWUDWLYD

9, HODERUDUHPRGLILFDURVHX5HJLPHQWR,QWHUQR $UW &RPS}HPDHVWUXWXUDRUJDQL]DFLRQDOGR,SUHY')RV


&DUJRVGH1DWXUH]D(VSHFLDOHRV&DUJRVHP&RPLVVmR
9,, UHFHEHUGHQ~QFLDFRQWUDDWRVGD'LUHWRULDGR,SUHY FRQVWDQWHVQR$QH[RÔQLFRGHVWD/HL&RPSOHPHQWDU
FULDGRVVHPDXPHQWRGHGHVSHVDPHGLDQWHWUDQVIRU
9,,, GHWHUPLQDUDVXVWDomRGHDWRVGD'LUHWRULDGR,SUHY PDomR GH FDUJRV GR EDQFR GH FDUJRV H IXQo}HV GR
TXH VHMDP OHVLYRV DR SULQFtSLR GH HFRQRPLFLGDGH H *RYHUQRGR'LVWULWR)HGHUDOGHTXHWUDWDRDUWž†
HILFiFLDRXRFRQWUDULHP žGR'HFUHWRQžGHžGHMDQHLURGH

$UW &RPSHWHDR&RQVHOKR)LVFDOGR,SUHY') †~QLFR 23RGHU([HFXWLYRIDUiSXEOLFDUQR'LiULR2ILFLDOGR


'LVWULWR)HGHUDOQRSUD]RGHWULQWDGLDVFRQWDGRVGD
, UHXQLUVHRUGLQDULDPHQWHXPDYH]HPFDGDWULPHVWUH SXEOLFDomR GHVWD /HL &RPSOHPHQWDU D UHODomR FRP
FLYLOSRUFRQYRFDomRGHVHX3UHVLGHQWH VtPERORVHYDORUHVGRVFDUJRVH[WLQWRV

,, H[DPLQDUDVFRQWDVDSXUDGDVQRVEDODQFHWHVHHPLWLU $UW 2SDWURFtQLRMXGLFLDOGR,SUHY')VHUiH[HUFLGRSHOD


SDUHFHUVREUHHODV 3URFXUDGRULD*HUDOGR'LVWULWR)HGHUDO

,,, GDUSDUHFHUVREUHREDODQoRDQXDOFRQWDVHDWRVGD $UW 2V FUpGLWRV GR ,SUHY') FRQVWLWXHP GtYLGDDWLYD


'LUHWRULD([HFXWLYDEHPFRPRVREUHRFXPSULPHQWR FRQVLGHUDGD OtTXLGD H FHUWD TXDQGR GHYLGDPHQWH
GRSODQRGHFXVWHLRHDFRHUrQFLDGRVUHVXOWDGRVGD LQVFULWDHPOLYURSUySULRFRPREVHUYkQFLDGRVUHTXLVL
DYDOLDomRDWXDULDOLQFOXVLYHHPUHODomRjVKLSyWHVHV WRVH[LJLGRVQDOHJLVODomRDGRWDGDSHOR'LVWULWR)HGHUDO
SDUDRPHVPRILP
,9 H[DPLQDUDTXDOTXHUWHPSROLYURVHGRFXPHQWRVGR
,SUHY') &DStWXOR;
'DV'LVSRVLo}HV)LQDLV
9 ODYUDUHPOLYURGHDWDVHSDUHFHUHVRVUHVXOWDGRVGRV
H[DPHVDTXHVHSURFHGHX $UW 2VPHPEURVGR&RQVHOKR$GPLQLVWUDWLYRGD'LUHWRULD
([HFXWLYD H GR &RQVHOKR )LVFDO VHUmR VROLGiULRV QDV
9, UHODWDUDR&RQVHOKRGH$GPLQLVWUDomRDVLUUHJXODULGD UHVSRQVDELOLGDGHVHUHVSRQGHUmRFLYLOHFULPLQDOPHQWH
GHV HYHQWXDOPHQWH DSXUDGDV VXJHULQGR PHGLGDV LQFOXVLYHFRPVHXSDWULP{QLRSHVVRDOSRUTXDOTXHUDWR
VDQHDGRUDV OHVLYR j DGPLQLVWUDomR S~EOLFD H DR SDWULP{QLR GR
UHJLPH SUySULR GH SUHYLGrQFLD GR 'LVWULWR )HGHUDO
9,, VROLFLWDUPRWLYDGDPHQWHDR&RQVHOKRGH$GPLQLVWUD REVHUYDQGRVHDLQGDDVQRUPDVGHJHVWmRILVFDOHDV
omR D FRQWUDWDomR GH DVVHVVRUDPHQWR GH WpFQLFR RX SHQDOLGDGHVSUHYLVWDVQD/HL&RPSOHPHQWDU)HGHUDOQž
HPSUHVD HVSHFLDOL]DGD VHP SUHMXt]R GR FRQWUROH GH /HLGH5HVSRQVDELOLGDGH)LVFDO
FRQWDVH[WHUQR
†~QLFR 6HUmRRVGLULJHQWHVDOXGLGRVQRFDSXWUHVSRQVDELOL]D
†~QLFR $VGHOLEHUDo}HVGR&RQVHOKR)LVFDOVHUmRWRPDGDVSRU GRV SHVVRDOPHQWH WDPEpP SHOD LQREVHUYkQFLD GDV
PDLRULDGHYRWRV QRUPDVSDUDHPLVVmRGR&HUWLILFDGRGH5HJXODULGDGH
3UHYLGHQFLiULD  &53 SHOR 0LQLVWpULR GD 3UHYLGrQFLD
$UW 2PDQGDWRGRVPHPEURVGR&RQVHOKRGH$GPLQLVWUD 6RFLDOFDVRFRPSURYDGDRFRUUrQFLDGHLPSUXGrQFLDRX
omRHGR&RQVHOKR)LVFDOVHUiGH WUrV DQRVSHUPLWL QHJOLJrQFLDQRWUDWRGDTXHVWmR
GDXPDUHFRQGXomR
$UW 2,SUHY')GHYHUiLGHQWLILFDUHFRQVROLGDUWULPHVWUDO
†~QLFR 1RDWRGDSRVVHHQRWpUPLQRGRPDQGDWRRVPHP PHQWHHPGHPRQVWUDWLYRVILQDQFHLURVHRUoDPHQWiUL
EURVGRV&RQVHOKRVGH$GPLQLVWUDomRH)LVFDOGHYHUmR RV WRGDV DV GHVSHVDV IL[DV H YDULiYHLV FRP SHVVRDO
ID]HUGHFODUDomRGHVHXVEHQVDTXDOVHUiWUDQVFULWD LQDWLYRHSHQVLRQLVWDEHPFRPRFRPHQFDUJRVLQFL
HP OLYUR SUySULR H SXEOLFDGR QR 'LiULR 2ILFLDO GR GHQWHVVREUHRVSURYHQWRVHSHQV}HVSDJRVHWDPEpP
'LVWULWR)HGHUDO WRGRRGHPRQVWUDWLYRSHUWLQHQWHjVXDiUHDGHDWXDomR
H[LJLGDSHOD/HL&RPSOHPHQWDU)HGHUDOQž
$UW $'LUHWRULD([HFXWLYDGR,SUHY')VHUiFRPSRVWDSRU /HLGH5HVSRQVDELOLGDGH)LVFDO
 FLQFR  'LUHWRUHV QRPHDGRV SHOR *RYHUQDGRU GR
'LVWULWR )HGHUDO VHQGR XP 'LUHWRU3UHVLGHQWH XP $UW 1HQKXPEHQHItFLRJOREDOGHDSRVHQWDGRULDHSHQVmR
'LUHWRU9LFH3UHVLGHQWHXP'LUHWRU3UHYLGHQFLiULRXP SRU PRUWH SRGHUi WHU YDORU EUXWR LQIHULRU DR VDOiULR
'LUHWRU -XUtGLFR H XP 'LUHWRU $GPLQLVWUDWL PtQLPRHVWDEHOHFLGRSDUDRVVHUYLGRUHVHVWDWXWiULRVGR
YR)LQDQFHLUR 'LVWULWR)HGHUDO
  /HJLVODomR 6HFUHWDULDGH(VWDGRGH$GPLQLVWUDomR3~EOLFD'LVWULWR)HGHUDO


$UW 2 UHFHELPHQWR LQGHYLGR GH EHQHItFLRV HP UD]mRGH †ž 2TXDGURGHSHVVRDOLQLFLDOGR,SUHY')VHUiIRUPDGR


GROR IUDXGH RX PiIp LPSOLFDUi GHYROXomR WRWDO GR SRU VHUYLGRUHV S~EOLFRV GR 4XDGUR GH 3HVVRDO GR
YDORUDXIHULGRTXHGHYHFDVRQmRKDMDDFRUGRDPLJi 'LVWULWR )HGHUDO PHGLDQWH UHTXLVLomR GH VHX 'LUH
YHO VHU LQVFULWR HP GtYLGD SDUD FREUDQoD MXGLFLDO WRU3UHVLGHQWHDR*RYHUQDGRUGR'LVWULWR)HGHUDO
FDEtYHO UHVSHLWDGRV RV GLUHLWRV j DPSOD GHIHVD H DR
FRQWUDGLWyULR †ž $FHVVmRGHVHUYLGRUHVGHTXHWUDWDR†žVHGDUiFRP
{QXV SDUD D RULJHP ILFDQGR DVVHJXUDGRV WRGRV RV
$UW )LFD DVVHJXUDGD D FRQWDJHP UHFtSURFD GR WHPSRGH GLUHLWRVHYDQWDJHQVGRVHUYLGRULQFOXVLYHRVLVWHPD
FRQWULEXLomR HQWUH R 5336') H RXWURV UHJLPHV UHPXQHUDWyULR GH RULJHP DWp TXH VH SURFHGD D VXD
SUHYLGHQFLiULRVKLSyWHVHHPTXHRVGLIHUHQWHVVLVWH VXEVWLWXLomR TXDQGR GD LPSODQWDomR GR 4XDGUR
PDVGHSUHYLGrQFLDVRFLDOVHFRPSHQVDUmRILQDQFHLUD 3HUPDQHQWHGH3HVVRDOGR,SUHY')
PHQWHSDUDHIHLWRGHDSRVHQWDGRULDYHGDGDDFRQWD
JHPGHWHPSRFRQFRPLWDQWH †ž $FRQVWLWXLomRGR4XDGUR3HUPDQHQWHGH3HVVRDOGR
,SUHY') VHUi REMHWR GH OHL HVSHFtILFD H R ,SUHY')
†~QLFR $FRQWDJHPUHFtSURFDGHTXHWUDWDRFDSXWGHYHUiVHU
DSUHVHQWDUiHPSUD]RQmRVXSHULRUD WUH]HQWRV
IHLWDPHGLDQWHDSUHVHQWDomRGH&HUWLGmRGH7HPSRGH
H VHVVHQWD  GLDV D FRQWDU GD SXEOLFDomR GHVWD /HL
&RQWULEXLomR IRUQHFLGD SHOR VHWRU FRPSHWHQWH GR
UHJLPHGHSUHYLGrQFLDGHRULJHPGRWHPSR SURSRVWDSDUDDUHDOL]DomRGHFRQFXUVRS~EOLFR

$UW $ &HUWLGmR GH &RQWDJHP 5HFtSURFD GH 7HPSRGH $UW $ 6HFUHWDULD GH (VWDGR GH 3ODQHMDPHQWR H*HVWmR
&RQWULEXLomRSRGHUiVHUUHTXHULGDSHORVHJXUDGRGR SUHVWDUiDR,SUHY')DWpDDSURYDomRGHVHXRUoDPHQ
5336')DTXDOTXHUWHPSRSDUDILQVGHFRPSURYDomR WRRDSRLRDGPLQLVWUDWLYRORJtVWLFRHILQDQFHLURTXHVH
GH WHPSR GH FRQWULEXLomR MXQWR D TXDOTXHU UHJLPH IL]HUQHFHVViULR
SUHYLGHQFLiULRGLVWLQWRGRSUHYLVWRQHVWD/HL&RPSOH
PHQWDU $UW 2 3RGHU ([HFXWLYR HQFDPLQKDUi HP DWp WULQWDGLDV
DSyVDSXEOLFDomRGHVWD/HL&RPSOHPHQWDUj&kPDUD
†ž $FHUWLGmRDTXHVHUHIHUHRFDSXWTXDQGRSDUDILQVGH /HJLVODWLYDGR'LVWULWR)HGHUDOSURSRVWDSDUDDEHUWXUD
DSRVHQWDGRULD HP RXWUR UHJLPH SUHYLGHQFLiULR VHUi GHFUpGLWRHVSHFLDOFRPDILQDOLGDGHGHGRWDURUoDPHQ
KRPRORJDGDH[FOXVLYDPHQWHSHOR,SUHY') WDULDPHQWHR,SUHY')

†ž 2,SUHY')GLVFLSOLQDUiRVSURFHGLPHQWRVUHODWLYRVj $UW $VGHVSHVDVGHFRUUHQWHVGDDSOLFDomRGHVWD/HL&RP


HPLVVmRGDFHUWLGmRGHTXHWUDWDRFDSXW SOHPHQWDUFRUUHUmRjFRQWDGRVUHFXUVRVFRQVWDQWHVGR
RUoDPHQWRGR'LVWULWR)HGHUDO
$UW ([FHWXDGDDKLSyWHVHGHUHFROKLPHQWRLQGHYLGRQmR
KDYHUi UHVWLWXLomR GH FRQWULEXLo}HV SUHYLGHQFLiULDV $UW 2 *RYHUQDGRU GR 'LVWULWR )HGHUDO HQFDPLQKDUij
SUHYLVWDVHPOHL &kPDUD/HJLVODWLYDGR'LVWULWR)HGHUDOSURMHWRGHOHL
FRPSOHPHQWDUSDUDLQVWLWXLURUHJLPHGHSUHYLGrQFLD
$UW )LFDYHGDGDDDGRomRGHUHTXLVLWRVHFULWpULRVGLIHUHQ FRPSOHPHQWDUGR'LVWULWR)HGHUDO
FLDGRVSDUDDFRQFHVVmRGHDSRVHQWDGRULDDRVDEUDQJL
GRVSHORUHJLPHGHTXHWUDWDHVWD/HL&RPSOHPHQWDU $UW 2VPHPEURVUHSUHVHQWDQWHVGRVVHJXUDGRVHEHQHILFLi
UHVVDOYDGRV RV FDVRV SUHYLVWRV QR DUW  † ž GD ULRV QR &RQVHOKR GH $GPLQLVWUDomR H QR &RQVHOKR
&RQVWLWXLomR)HGHUDOFRPDUHGDomRGDGDSHOD(PHQ )LVFDOGHYHUmRVHULQGLFDGRVQRSUD]RPi[LPRGH
GD&RQVWLWXFLRQDOQžQRVWHUPRVGHILQLGRV FHQWRHRLWHQWD GLDVFRQWDGRVGDSXEOLFDomRGHVWD/HL
HPOHLFRPSOHPHQWDUIHGHUDO &RPSOHPHQWDU

$UW 1mR VHUi FRPSXWDGR SDUD ILQV GH DSRVHQWDGRULDR †ž 2*RYHUQDGRUGR'LVWULWR)HGHUDOLQGLFDUiRVPHPEURV
WHPSRGHFRQWULEXLomRTXHWLYHUVHUYLGRGHEDVHSDUD GR&RQVHOKRGH$GPLQLVWUDomRHGR&RQVHOKR )LVFDO
DSRVHQWDGRULDHPRXWURUHJLPHGHSUHYLGrQFLDVRFLDO FLWDGRVQRFDSXWFDVRDVHQWLGDGHVGHFODVVHQmRRV
LQGLTXHPQRSUD]RHVWDEHOHFLGR
$UW 2H[HUFtFLRILQDQFHLURFRLQFLGLUiFRPRDQRFLYLOHD
FRQWDELOLGDGH REHGHFHUi QR TXH FRXEHU jV QRUPDV †ž 2FRPSDUHFLPHQWRjVUHXQL}HVGR&RQVHOKRGH$GPL
JHUDLV S~EOLFDV GD DGPLQLVWUDomR ILQDQFHLUD H SUHYL QLVWUDomRHjVGR&RQVHOKR)LVFDOHPKRUiULRFRLQFL
GHQFLiULD GHQWH DR GD MRUQDGD GH WUDEDOKR VHUi FRQVLGHUDGD
FRPR H[HUFtFLR GR FDUJR RX GR HPSUHJR S~EOLFR
$UW 2VRUoDPHQWRVDSURJUDPDomRILQDQFHLUDHRVEDODQ ILFDQGR YHGDGD D LPSXWDomR GH IDOWD DR VHUYLoR GRV
oRV GR ,SUHY') REHGHFHUmR DRV SDGU}HV H QRUPDV
UHVSHFWLYRVFRQVHOKHLURV
LQVWLWXtGRVSRUOHJLVODomRHVSHFtILFDDMXVWDGRVjVVXDV
SHFXOLDULGDGHV
†ž (QWUHRVVHWHPHPEURVGR&RQVHOKRGH$GPLQLVWUDomR
†~QLFR -XQWDPHQWHFRPREDODQoRJHUDODFDGDDQRGHYHUi GHTXHWUDWDRFDSXW TXDWUR FXPSULUmRPDQGDWR
D 'LUHWRULD ([HFXWLYD UHDOL]DU REULJDWRULDPHQWH D GH WUrV DQRVH WUrV GH GRLV DQRV
DYDOLDomRDWXDULDOGR5336')
†ž 1DVVXFHVV}HVGRVPHPEURVGR&RQVHOKRGH$GPLQLV
$UW 2VEHQHItFLRVFRQFHGLGRVQmRHOHQFDGRVQDSUHVHQWH WUDomRFLWDGRVQRSDUiJUDIRDQWHULRURPDQGDWRVHUi
/HL &RPSOHPHQWDU SHUPDQHFHUmR FXVWHDGRV FRP GH WUrV DQRV
UHFXUVRV GR 7HVRXUR GR 'LVWULWR )HGHUDO D WtWXOR GH
EHQHItFLRSDWURQDO $UW (VWD/HL&RPSOHPHQWDUHQWUDHPYLJRUQDGDWDGHVXD
SXEOLFDomR
$UW $V DWULEXLo}HV GRV 'LUHWRUHV H GHPDLV &DUJRVGH
1DWXUH]D(VSHFLDOH&DUJRVHP&RPLVVmRVHUmRHVWDEH $UW 5HYRJDPVHDVGLVSRVLo}HVHPFRQWUiULR
OHFLGDVHPGHFUHWRUHJXODPHQWDGRU
6HFUHWDULDGH(VWDGRGH$ GPLQLVWUDomR3~EOLFD'LVWULWR)HGHUDO
 /HJLVODomR 

$1(;2Ô1,&2
(6758785$25*$1,=$&,21$/'2,SUHY')
&$5*26'(1$785(=$(63(&,$/((0&20,66®2 /HLIHGHUDOQž
$UWGD/HL&RPSOHPHQWDUQžGHGHMXQKRGH

&$5*2 6Ì0%2/2 48$17,'$'(


',5(72535(6,'(17( &1(  &DStWXOR,
',5(7259,&(35(6,'(17( &1(  'DV'LVSRVLo}HV*HUDLV
',5(725 &1( 
$66(6625(63(&,$/ &1( 
&+()('(',9,6®2 &1(  $UWž (VWD /HL HVWDEHOHFH QRUPDV EiVLFDV VREUH RSURFHVVR
&+()('(1Ô&/(2-85Ì',&2 &1(  DGPLQLVWUDWLYR QR kPELWR GD $GPLQLVWUDomR )HGHUDO
$66(6625(63(&,$/ &1(  GLUHWDHLQGLUHWDYLVDQGRHPHVSHFLDOjSURWHomRGRV
*(5(17( ')* 
$66(6625-85Ì',&2 ')$  GLUHLWRVGRVDGPLQLVWUDGRVHDRPHOKRUFXPSULPHQWR
289,'25 ')$  GRVILQVGD$GPLQLVWUDomR
$66(6625 ')$ 
$66(6625 ')$  †ž 2VSUHFHLWRVGHVWD/HLWDPEpPVHDSOLFDPDRVyUJmRV
$66,67(17( ')$ 
GRV3RGHUHV/HJLVODWLYRH-XGLFLiULRGD8QLmRTXDQGR
&$5*26Ì0%2/248$17,'$'( QRGHVHPSHQKRGHIXQomRDGPLQLVWUDWLYD

,  35(6,'È1&,$  'LUHWRU 3UHVLGHQWH &1(  'LUHWRU 9LFH3UHVLGHQWH †ž 3DUDRVILQVGHVWD/HLFRQVLGHUDPVH
&1(  $VVHVVRU (VSHFLDO &1(  $VVHVVRU (VSHFLDO GH $XGLWRULD
&1($VVHVVRU(VSHFLDOGH,QYHVWLPHQWRV&1($VVHVVRU(VSHFLDO
GH&RPXQLFDomR&1($VVHVVRU(VSHFLDO&1($VVHVVRU')$  , yUJmRDXQLGDGHGHDWXDomRLQWHJUDQWHGDHVWUXWXUDGD
$VVHVVRU')$$VVLVWHQWH')$ $GPLQLVWUDomRGLUHWDHGDHVWUXWXUDGD$GPLQLVWUDomR
LQGLUHWD
,, ',5(725,$-85Ì',&$'LUHWRU-XUtGLFR&1($VVHVVRU')$
$VVLVWHQWH')$,, 1~FOHR-XUtGLFR3UHYLGHQFLiULR&KHIHGR1~FOHR
-XUtGLFR 3UHYLGHQFLiULR &1(  $VVHVVRU -XUtGLFR ')$   ,, ,, HQWLGDGHDXQLGDGHGHDWXDomRGRWDGDGHSHUVRQDOLGD
1~FOHR-XUtGLFR,QVWLWXFLRQDO&KHIHGR1~FOHR-XUtGLFR,QVWLWXFLRQDO&1( GHMXUtGLFD
$VVHVVRU-XUtGLFR')$

,,,  ',5(725,$ '( 35(9,'È1&,$  'LUHWRU GH 3UHYLGrQFLD &1(  ,,, DXWRULGDGH  R VHUYLGRU RX DJHQWH S~EOLFR GRWDGRGH
$VVLVWHQWH')$,,, 2XYLGRULD$VVHVVRU&KHIHGD2XYLGRULD')$ SRGHUGHGHFLVmR
,,, 'LYLVmRGH%HQHItFLRV&KHIHGD'LYLVmRGH%HQHItFLRV&1(
$VVHVVRU')$,,, *HUrQFLDGH,QDWLYRVH3HQVLRQLVWDV*HUHQWHGH $UWž $$GPLQLVWUDomR3~EOLFDREHGHFHUiGHQWUHRXWURVDRV
,QDWLYRVH3HQVLRQLVWDV')*$VVHVVRU')$,,, *HUrQFLDGH
$WHQGLPHQWRH&DGDVWUR*HUHQWHGH$WHQGLPHQWRH&DGDVWUR')* SULQFtSLRVGDOHJDOLGDGHILQDOLGDGHPRWLYDomRUD]RDEL
$VVHVVRU ')$  ,,,  'LYLVmR GH &RPSHQVDomR H $WXiULD² &KHIH GD OLGDGHSURSRUFLRQDOLGDGHPRUDOLGDGHDPSODGHIHVD
'LYLVmRGH&RPSHQVDomRH$WXiULD&1($VVHVVRU')$,,, FRQWUDGLWyULR VHJXUDQoD MXUtGLFD LQWHUHVVH S~EOLFR H
*HUrQFLDGH&RPSHQVDomR3UHYLGHQFLiULD*HUHQWHGH&RPSHQVDomR3UHYLGHQ HILFLrQFLD
FLiULD')*$VVHVVRU')$,,, *HUrQFLDGH$FRPSDQKDPHQWR
$WXDULDOH3ODQHMDPHQWR*HUHQWHGH$FRPSDQKDPHQWR$WXDULDOH3ODQHMDPHQ
WR')*$VVHVVRU')$ †~QLFR 1RVSURFHVVRVDGPLQLVWUDWLYRVVHUmRREVHUYDGRVHQWUH
RXWURVRVFULWpULRVGH
,9  ',5(725,$ '( ),1$1d$6 ( $'0,1,675$d®2  'LUHWRU GH )LQDQoDV H
$GPLQLVWUDomR&1($VVLVWHQWH')$,9 'LYLVmRGH)LQDQoDV
&KHIH GD 'LYLVmR GH )LQDQoDV &1(  $VVHVVRU ')$  ,9 , DWXDomRFRQIRUPHDOHLHR'LUHLWR
*HUrQFLDGH&RQWDELOLGDGH*HUHQWHGH&RQWDELOLGDGH')*$VVLVWHQWH
')$,9 *HUrQFLDGH)LQDQoDV*HUHQWHGH)LQDQoDV')* ,, DWHQGLPHQWR D ILQV GH LQWHUHVVH JHUDOYHGDGDD
$VVLVWHQWH')$,9 *HUrQFLDGH2UoDPHQWRH3ODQHMDPHQWR*HUHQWH UHQ~QFLDWRWDORXSDUFLDOGHSRGHUHVRXFRPSHWrQFL
GH 2UoDPHQWR H 3ODQHMDPHQWR ')*  $VVLVWHQWH ')$  ,9
'LYLVmRGH*HVWmR$GPLQLVWUDWLYD&KHIHGD'LYLVmRGH*HVWmR$GPLQLVWUDWLYD
DVVDOYRDXWRUL]DomRHPOHL
&1(  $VVHVVRU ')$  ,9  *HUrQFLD GH $SRLR 2SHUDFLRQDO H
5HFXUVRV +XPDQRV  *HUHQWH GH $SRLR 2SHUDFLRQDO H 5HFXUVRV +XPDQRV ,,, REMHWLYLGDGH QR DWHQGLPHQWR GR LQWHUHVVHS~EOLFR
')*$VVLVWHQWH')$H,9 *HUrQFLDGH,QIRUPiWLFD*HUHQWH YHGDGDDSURPRomRSHVVRDOGHDJHQWHVRXDXWRULGD
GH,QIRUPiWLFD')*
GHV

 ,9 DWXDomRVHJXQGRSDGU}HVpWLFRVGHSURELGDGHGHFRUR
HERDIp
/HJLVODomRGRVHUYLGRUS~EOLFRGR'LVWULWR)HGHUDO
9 GLYXOJDomRRILFLDOGRVDWRVDGPLQLVWUDWLYRVUHVVDOYD
GDVDVKLSyWHVHVGHVLJLORSUHYLVWDVQD&RQVWLWXLomR
/HLIHGHUDOQž
UHFHSFLRQDGDSHOD/HLGLVWULWDOQž
9, DGHTXDomRHQWUHPHLRVHILQVYHGDGDDLPSRVLomRGH
REULJDo}HVUHVWULo}HVHVDQo}HVHPPHGLGDVXSHULRU
jTXHODVHVWULWDPHQWHQHFHVViULDVDRDWHQGLPHQWRGR
/(,1ž'('('(=(0%52'( LQWHUHVVHS~EOLFR

5HFHSFLRQDD/HL)HGHUDOQžGHGHMDQHLURGH 9,, LQGLFDomRGRVSUHVVXSRVWRVGHIDWRHGHGLUHLWRTXH


GHWHUPLQDUHPDGHFLVmR
$UWž $SOLFDPVH DRV DWRV H SURFHVVRV DGPLQLVWUDWLYRVQR
kPELWRGD$GPLQLVWUDomRGLUHWDHLQGLUHWDGR'LVWULWR 9,,, REVHUYkQFLD GDV IRUPDOLGDGHV HVVHQFLDLV jJDUDQWLD
)HGHUDOQRTXHFRXEHUDVGLVSRVLo}HVGD/HL)HGHUDO GRVGLUHLWRVGRVDGPLQLVWUDGRV
QžGHGHMDQHLURGH
,; DGRomRGHIRUPDVVLPSOHVVXILFLHQWHVSDUDSURSLFLDU
$UWž (VWD/HLHQWUDHPYLJRUQDGDWDGHVXDSXEOLFDomR
DGHTXDGRJUDXGHFHUWH]DVHJXUDQoDHUHVSHLWRDRV
$UWž 5HYRJDPVHDVGLVSRVLo}HVHPFRQWUiULR GLUHLWRVGRVDGPLQLVWUDGRV
  /HJLVODomR 6HFUHWDULDGH(VWDGRGH$GPLQLVWUDomR3~EOLFD'LVWULWR)HGHUDO


; JDUDQWLDGRVGLUHLWRVjFRPXQLFDomRjDSUHVHQWDomRGH †~QLFR e YHGDGD j $GPLQLVWUDomR D UHFXVDLPRWLYDGDGH


DOHJDo}HVILQDLVjSURGXomRGHSURYDVHjLQWHUSRVLomR UHFHELPHQWR GH GRFXPHQWRV GHYHQGR R VHUYLGRU
GH  UHFXUVRV QRV SURFHVVRV GH TXH SRVVDP UHVXOWDU RULHQWDURLQWHUHVVDGRTXDQWRDRVXSULPHQWRGHHYHQ
VDQo}HVHQDVVLWXDo}HVGHOLWtJLR WXDLVIDOKDV

;, SURLELomRGHFREUDQoDGHGHVSHVDVSURFHVVXDLVUHVVDO $UWž 2VyUJmRVHHQWLGDGHVDGPLQLVWUDWLYDVGHYHUmRHODERUDU


YDGDVDVSUHYLVWDVHPOHL PRGHORV RX IRUPXOiULRV SDGURQL]DGRV SDUD DVVXQWRV
TXHLPSRUWHPSUHWHQV}HVHTXLYDOHQWHV
;,, LPSXOVmR GH RItFLR GR SURFHVVR DGPLQLVWUDWLYRVHP
SUHMXt]RGDDWXDomRGRVLQWHUHVVDGRV $UWž 4XDQGRRVSHGLGRVGHXPDSOXUDOLGDGHGHLQWHUHVVDGRV
WLYHUHPFRQWH~GRHIXQGDPHQWRVLGrQWLFRVSRGHUmRVHU
;,,, LQWHUSUHWDomRGDQRUPDDGPLQLVWUDWLYDGDIRUPDTXH IRUPXODGRVHPXP~QLFRUHTXHULPHQWRVDOYRSUHFHLWR
PHOKRUJDUDQWDRDWHQGLPHQWRGRILPS~EOLFRDTXHVH OHJDOHPFRQWUiULR
GLULJHYHGDGDDSOLFDomRUHWURDWLYDGHQRYDLQWHUSUHWD
omR &DStWXOR9
'RV,QWHUHVVDGRV
&DStWXOR,,
'RV'LUHLWRVGRV$GPLQLVWUDGRV $UWž 6mROHJLWLPDGRVFRPRLQWHUHVVDGRVQRSURFHVVRDGPL
QLVWUDWLYR
$UWž 2 DGPLQLVWUDGR WHP RV VHJXLQWHV GLUHLWRV SHUDQWHD  , SHVVRDVItVLFDVRXMXUtGLFDVTXHRLQLFLHPFRPRWLWXODUHV
$GPLQLVWUDomRVHPSUHMXt]RGHRXWURVTXHOKHVHMDP GHGLUHLWRVRXLQWHUHVVHVLQGLYLGXDLVRXQRH[HUFtFLRGR
DVVHJXUDGRV GLUHLWRGHUHSUHVHQWDomR

, VHUWUDWDGRFRPUHVSHLWRSHODVDXWRULGDGHVHVHUYLGR ,, DTXHOHV TXH VHP WHUHP LQLFLDGR R SURFHVVRWrP
UHVTXHGHYHUmRIDFLOLWDURH[HUFtFLRGHVHXVGLUHLWRVH GLUHLWRV RX LQWHUHVVHV TXH SRVVDP VHU DIHWDGRV SHOD
RFXPSULPHQWRGHVXDVREULJDo}HV GHFLVmRDVHUDGRWDGD

,, WHUFLrQFLDGDWUDPLWDomRGRVSURFHVVRVDGPLQLVWUDWLYRV ,,, DV RUJDQL]Do}HV H DVVRFLDo}HV UHSUHVHQWDWLYDVQR


HPTXHWHQKDDFRQGLomRGHLQWHUHVVDGRWHUYLVWDGRV WRFDQWHDGLUHLWRVHLQWHUHVVHVFROHWLYRV
DXWRV  REWHU FySLDV GH GRFXPHQWRV QHOHV FRQWLGRV H 
FRQKHFHUDVGHFLV}HVSURIHULGDV ,9 DVSHVVRDVRXDVDVVRFLDo}HV OHJDOPHQWHFRQVWLWXtGDV
TXDQWRDGLUHLWRVRXLQWHUHVVHVGLIXVRV
,,, IRUPXODUDOHJDo}HVHDSUHVHQWDUGRFXPHQWRVDQWHVGD
GHFLVmR RV TXDLV VHUmR REMHWR GH FRQVLGHUDomR SHOR $UW 6mRFDSD]HVSDUDILQVGHSURFHVVRDGPLQLVWUDWLYRRV
yUJmRFRPSHWHQWH PDLRUHVGHGH]RLWRDQRVUHVVDOYDGDSUHYLVmRHVSHFLDO
HPDWRQRUPDWLYRSUySULR
,9 ID]HUVHDVVLVWLUIDFXOWDWLYDPHQWHSRUDGYRJDGRVDOYR
TXDQGRREULJDWyULDDUHSUHVHQWDomRSRUIRUoDGHOHL &DStWXOR9,
'D&RPSHWrQFLD
&DStWXOR,9
'RV'HYHUHVGRV$GPLQLVWUDGRV $UW $FRPSHWrQFLDpLUUHQXQFLiYHOHVHH[HUFHSHORVyUJmRV
DGPLQLVWUDWLYRVDTXHIRLDWULEXtGDFRPRSUySULDVDOYR
$UWž 6mR
6mR GHYHUHV
GHYHUHV GR
GR DGPLQLVWUDGRSHUDQWHD$GPLQLVWUD
DGPLQLVWUDGRSHUDQWHD$GPLQLVWUD RVFDVRVGHGHOHJDomRHDYRFDomROHJDOPHQWHDGPLWL
omR
omRVHP
VHPSUHMXt]R
SUHMXt]RGH
GHRXWURV
RXWURVSUHYLVWRVHPDWR
SUHYLVWRVHP DWRQRUPDWL GRV
YR
YR
$UW 8PyUJmRDGPLQLVWUDWLYRHVHXWLWXODUSRGHUmRVHQmR
, H[SRURVIDWRVFRQIRUPHDYHUGDGH KRXYHU LPSHGLPHQWR OHJDO GHOHJDU SDUWH GD VXD
,, SURFHGHUFRPOHDOGDGHXUEDQLGDGHHERDIp FRPSHWrQFLD D RXWURV yUJmRV RX WLWXODUHV DLQGD TXH
,,, QmRDJLUGHPRGRWHPHUiULR HVWHV QmR OKH VHMDP KLHUDUTXLFDPHQWH VXERUGLQDGRV
,9 SUHVWDU DV LQIRUPDo}HV TXH OKH IRUHP VROLFLWDGDVH TXDQGRIRUFRQYHQLHQWHHPUD]mRGHFLUFXQVWkQFLDVGH
FRODERUDUSDUDRHVFODUHFLPHQWRGRVIDWRV tQGROHWpFQLFDVRFLDOHFRQ{PLFDMXUtGLFDRXWHUULWRULDO

&DStWXOR,9 †~QLFR 2GLVSRVWRQRFDSXWGHVWHDUWLJRDSOLFDVHjGHOHJDomR


'R,QtFLRGR3URFHVVR
'R,QtFLRGR3URFHVVR GHFRPSHWrQFLDGRVyUJmRVFROHJLDGRVDRVUHVSHFWLYRV
SUHVLGHQWHV
$UWž 2SURFHVVRDGPLQLVWUDWLYRSRGHLQLFLDUVHGHRItFLRRX
DSHGLGRGHLQWHUHVVDGR $UW 1mRSRGHPVHUREMHWRGHGHOHJDomR

$UWž 2UHTXHULPHQWRLQLFLDOGRLQWHUHVVDGRVDOYRFDVRVHP , DHGLomRGHDWRVGHFDUiWHUQRUPDWLYR
TXHIRUDGPLWLGDVROLFLWDomRRUDOGHYHVHUIRUPXODGR ,, DGHFLVmRGHUHFXUVRVDGPLQLVWUDWLYRV
SRUHVFULWRHFRQWHURVVHJXLQWHVGDGRV  ,,, DV PDWpULDV GH FRPSHWrQFLD H[FOXVLYDGRyUJmRRX
DXWRULGDGH
, yUJmRRXDXWRULGDGHDGPLQLVWUDWLYDDTXHVHGLULJH
,, LGHQWLILFDomRGRLQWHUHVVDGRRXGHTXHPRUHSUHVHQWH $UW 2DWRGHGHOHJDomRHVXDUHYRJDomRGHYHUmRVHUSXEOL
 ,,, GRPLFtOLRGRUHTXHUHQWHRXORFDOSDUDUHFHELPHQWRGH FDGRVQRPHLRRILFLDO
FRPXQLFDo}HV 
 ,9 IRUPXODomR GR SHGLGRFRPH[SRVLomRGRVIDWRVHGH †ž 2DWRGHGHOHJDomRHVSHFLILFDUiDVPDWpULDVHSRGHUHV
VHXVIXQGDPHQWRV WUDQVIHULGRV RV OLPLWHV GD DWXDomR GR GHOHJDGR D
 9 GDWDHDVVLQDWXUDGRUHTXHUHQWHRXGHVHXUHSUHVHQWDQ GXUDomRHRVREMHWLYRVGDGHOHJDomRHRUHFXUVRFDEtYHO
WH SRGHQGR FRQWHU UHVVDOYD GH H[HUFtFLR GD DWULEXLomR
 GHOHJDGD
6HFUHWDULDGH(VWDGRGH$ GPLQLVWUDomR3~EOLFD'LVWULWR)HGHUDO
 /HJLVODomR 

†ž 2DWRGHGHOHJDomRpUHYRJiYHODTXDOTXHUWHPSRSHOD †ž 2SURFHVVRGHYHUiWHUVXDVSiJLQDVQXPHUDGDVVHTXHQ


DXWRULGDGHGHOHJDQWH FLDOPHQWHHUXEULFDGDV

†ž $VGHFLV}HVDGRWDGDVSRUGHOHJDomRGHYHPPHQFLRQDU $UW 2VDWRVGRSURFHVVRGHYHPUHDOL]DUVHHPGLDV~WHLVQR
H[SOLFLWDPHQWH HVWD TXDOLGDGH H FRQVLGHUDUVHmR KRUiULRQRUPDOGHIXQFLRQDPHQWRGDUHSDUWLomRQDTXDO
HGLWDGDVSHORGHOHJDGR WUDPLWDURSURFHVVR

$UW 6HUiSHUPLWLGDHPFDUiWHUH[FHSFLRQDOHSRUPRWLYRV †~QLFR 6HUmRFRQFOXtGRVGHSRLVGRKRUiULRQRUPDORVDWRVMi


UHOHYDQWHVGHYLGDPHQWHMXVWLILFDGRVDDYRFDomRWHPSR LQLFLDGRVFXMRDGLDPHQWRSUHMXGLTXHRFXUVRUHJXODUGR
UiULD GH FRPSHWrQFLD DWULEXtGD D yUJmR KLHUDUTXLFD SURFHGLPHQWR RX FDXVH GDQR DR LQWHUHVVDGR RX j
PHQWHLQIHULRU $GPLQLVWUDomR

$UW 2V yUJmRV H HQWLGDGHV DGPLQLVWUDWLYDVGLYXOJDUmR $UW ,QH[LVWLQGRGLVSRVLomRHVSHFtILFDRVDWRVGRyUJmRRX


SXEOLFDPHQWHRVORFDLVGDVUHVSHFWLYDVVHGHVHTXDQGR DXWRULGDGHUHVSRQViYHOSHORSURFHVVRHGRVDGPLQLVWUD
FRQYHQLHQWH D XQLGDGH IXQGDFLRQDO FRPSHWHQWH HP GRVTXHGHOHSDUWLFLSHPGHYHPVHUSUDWLFDGRVQRSUD]R
PDWpULDGHLQWHUHVVHHVSHFLDO GHFLQFRGLDVVDOYRPRWLYRGHIRUoDPDLRU

$UW ,QH[LVWLQGR FRPSHWrQFLD OHJDO HVSHFtILFD RSURFHVVR †~QLFR 2SUD]RSUHYLVWRQHVWHDUWLJRSRGHVHUGLODWDGRDWpR


DGPLQLVWUDWLYRGHYHUiVHULQLFLDGRSHUDQWHDDXWRULGDGH GREURPHGLDQWHFRPSURYDGDMXVWLILFDomR
GHPHQRUJUDXKLHUiUTXLFRSDUDGHFLGLU
$UW 2VDWRVGRSURFHVVRGHYHPUHDOL]DUVHSUHIHUHQFLDOPHQ
&DStWXOR9,, WHQDVHGHGRyUJmRFLHQWLILFDQGRVHRLQWHUHVVDGRVH
'RV,PSHGLPHQWRVHGD6XVSHLomR RXWURIRURORFDOGHUHDOL]DomR

$UW e LPSHGLGR GH DWXDU HP SURFHVVR DGPLQLVWUDWLYRR &DStWXOR,;


VHUYLGRURXDXWRULGDGHTXH 'D&RPXQLFDomRGR$WRV

, WHQKDLQWHUHVVHGLUHWRRXLQGLUHWRQDPDWpULD $UW 2yUJmRFRPSHWHQWHSHUDQWHRTXDOWUDPLWDRSURFHVVR


 DGPLQLVWUDWLYRGHWHUPLQDUiDLQWLPDomRGRLQWHUHVVDGR
,, WHQKDSDUWLFLSDGRRXYHQKDDSDUWLFLSDUFRPRSHULWR SDUDFLrQFLDGHGHFLVmRRXDHIHWLYDomRGHGLOLJrQFLDV
WHVWHPXQKDRXUHSUHVHQWDQWHRXVHWDLVVLWXDo}HVRFRU
UHP TXDQWR DR F{QMXJH FRPSDQKHLUR RX SDUHQWH H †ž $LQWLPDomRGHYHUiFRQWHU
DILQVDWpRWHUFHLURJUDX
 , LGHQWLILFDomRGRLQWLPDGRHQRPHGRyUJmRRXHQWLGDGH
,,, HVWHMDOLWLJDQGRMXGLFLDORXDGPLQLVWUDWLYDPHQWHFRPR DGPLQLVWUDWLYD
LQWHUHVVDGRRXUHVSHFWLYRF{QMXJHRXFRPSDQKHLUR  ,, ILQDOLGDGHGDLQWLPDomR
,,, GDWDKRUDHORFDOHPTXHGHYHFRPSDUHFHU
$UW $DXWRULGDGHRXVHUYLGRUTXHLQFRUUHUHPLPSHGLPHQWR ,9 VHRLQWLPDGRGHYHFRPSDUHFHUSHVVRDOPHQWHRXID]HU
GHYH FRPXQLFDU R IDWR j DXWRULGDGH FRPSHWHQWH VHUHSUHVHQWDU
DEVWHQGRVHGHDWXDU 9 LQIRUPDomRGDFRQWLQXLGDGHGRSURFHVVRLQGHSHQGHQWH
PHQWHGRVHXFRPSDUHFLPHQWR
†~QLFR $RPLVVmRGRGHYHUGHFRPXQLFDURLPSHGLPHQWRFRQV 9, LQGLFDomRGRVIDWRVHIXQGDPHQWRVOHJDLVSHUWLQHQWHV
WLWXLIDOWDJUDYHSDUDHIHLWRVGLVFLSOLQDUHV 
†ž $LQWLPDomRREVHUYDUiDDQWHFHGrQFLDPtQLPDGHWUrV
$UW 3RGHVHUDUJXLGDDVXVSHLomRGHDXWRULGDGHRXVHUYLGRU GLDV~WHLVTXDQWRjGDWDGHFRPSDUHFLPHQWR
TXHWHQKDDPL]DGHtQWLPDRXLQLPL]DGHQRWyULDFRP
DOJXPGRVLQWHUHVVDGRVRXFRPRVUHVSHFWLYRVF{QMXJHV †ž $LQWLPDomRSRGHVHUHIHWXDGDSRUFLrQFLDQRSURFHVVR
FRPSDQKHLURVSDUHQWHVHDILQVDWpRWHUFHLURJUDX SRUYLDSRVWDOFRPDYLVRGHUHFHELPHQWRSRUWHOHJUDPD
RX RXWUR PHLR TXH DVVHJXUH D FHUWH]D GD FLrQFLD GR
$UW 2LQGHIHULPHQWRGHDOHJDomRGHVXVSHLomRSRGHUiVHU LQWHUHVVDGR
REMHWRGHUHFXUVRVHPHIHLWRVXVSHQVLYR
†ž 1RFDVRGHLQWHUHVVDGRVLQGHWHUPLQDGRVGHVFRQKHFLGRV
&DStWXOR9,,, RX FRP GRPLFtOLR LQGHILQLGR D LQWLPDomR GHYH VHU
'D)RUPD7HPSRH/XJDUGRV$WRVGR3URFHVVR HIHWXDGDSRUPHLRGHSXEOLFDomRRILFLDO

$UW 2VDWRVGRSURFHVVRDGPLQLVWUDWLYRQmRGHSHQGHPGH †ž $VLQWLPDo}HVVHUmRQXODVTXDQGRIHLWDVVHPREVHUYkQ


IRUPDGHWHUPLQDGDVHQmRTXDQGRDOHLH[SUHVVDPHQWH FLDGDVSUHVFULo}HVOHJDLVPDVRFRPSDUHFLPHQWRGR
DH[LJLU DGPLQLVWUDGRVXSUHVXDIDOWDRXLUUHJXODULGDGH

†ž 2VDWRVGRSURFHVVRGHYHPVHUSURGX]LGRVSRUHVFULWR $UW 2 GHVDWHQGLPHQWRGDLQWLPDomRQmRLPSRUWDRUHFR
HPYHUQiFXORFRPDGDWDHRORFDOGHVXDUHDOL]DomRH QKHFLPHQWR GD YHUGDGH GRV IDWRV QHP D UHQ~QFLD D
DDVVLQDWXUDGDDXWRULGDGHUHVSRQViYHO GLUHLWRSHORDGPLQLVWUDGR

†~QLFR 1RSURVVHJXLPHQWRGRSURFHVVRVHUiJDUDQWLGRGLUHLWR
†ž 6DOYR LPSRVLomR OHJDO RUHFRQKHFLPHQWRGHILUPD
VRPHQWHVHUiH[LJLGRTXDQGRKRXYHUG~YLGDGHDXWHQWL GHDPSODGHIHVDDRLQWHUHVVDGR
FLGDGH
 $UW 'HYHPVHUREMHWRGHLQWLPDomRRVDWRVGRSURFHVVRTXH
†ž $ DXWHQWLFLGDGH GH GRFXPHQWRVH[LJLGRVHPFySLD UHVXOWHPSDUDRLQWHUHVVDGRHPLPSRVLomRGHGHYHUHV
SRGHUiVHUIHLWDSHORyUJmRDGPLQLVWUDWLYR {QXV VDQo}HV RX UHVWULomR DR H[HUFtFLR GH GLUHLWRV H
 DWLYLGDGHVHRVDWRVGHRXWUDQDWXUH]DGHVHXLQWHUHVVH
  /HJLVODomR 6HFUHWDULDGH(VWDGRGH$GPLQLVWUDomR3~EOLFD'LVWULWR)HGHUDO


&DStWXOR; $UW 2 LQWHUHVVDGR SRGHUi QD IDVH LQVWUXWyULD H DQWHVGD


'D,QVWUXomR WRPDGD GD GHFLVmR MXQWDU GRFXPHQWRV H SDUHFHUHV
UHTXHUHU GLOLJrQFLDV H SHUtFLDV EHP FRPR DGX]LU
$UW $V DWLYLGDGHV GH LQVWUXomR GHVWLQDGDV D DYHULJXDUH DOHJDo}HVUHIHUHQWHVjPDWpULDREMHWRGRSURFHVVR
FRPSURYDURVGDGRVQHFHVViULRVjWRPDGDGHGHFLVmR
UHDOL]DPVHGHRItFLRRXPHGLDQWHLPSXOVmRGRyUJmR †ž 2VHOHPHQWRVSUREDWyULRVGHYHUmRVHUFRQVLGHUDGRVQD
UHVSRQViYHOSHORSURFHVVRVHPSUHMXt]RGRGLUHLWRGRV PRWLYDomRGRUHODWyULRHGDGHFLVmR
LQWHUHVVDGRVGHSURSRUDWXDo}HVSUREDWyULDV 
†ž 6RPHQWH SRGHUmR VHUUHFXVDGDVPHGLDQWHGHFLVmR
†ž 2yUJmRFRPSHWHQWHSDUDDLQVWUXomRIDUiFRQVWDUGRV IXQGDPHQWDGDDVSURYDVSURSRVWDVSHORVLQWHUHVVDGRV
DXWRVRVGDGRVQHFHVViULRVjGHFLVmRGRSURFHVVR TXDQGRVHMDPLOtFLWDVLPSHUWLQHQWHVGHVQHFHVViULDVRX
SURWHODWyULDV
†ž 2VDWRVGHLQVWUXomRTXHH[LMDPDDWXDomRGRVLQWHUHV
VDGRVGHYHPUHDOL]DUVHGRPRGRPHQRVRQHURVRSDUD $UW 4XDQGRIRUQHFHVViULDDSUHVWDomRGHLQIRUPDo}HVRX
HVWHV DDSUHVHQWDomRGHSURYDVSHORVLQWHUHVVDGRVRXWHUFHL
URV VHUmR H[SHGLGDV LQWLPDo}HV SDUD HVVH ILP
$UW 6mRLQDGPLVVtYHLVQRSURFHVVRDGPLQLVWUDWLYRDVSURYDV PHQFLRQDQGRVH GDWD SUD]R IRUPD H FRQGLo}HV GH
REWLGDVSRUPHLRVLOtFLWRV DWHQGLPHQWR

$UW 4XDQGR D PDWpULD GR SURFHVVR HQYROYHU DVVXQWRGH †~QLFR 1mR VHQGR DWHQGLGD D LQWLPDomRSRGHUiRyUJmR
LQWHUHVVHJHUDORyUJmRFRPSHWHQWHSRGHUiPHGLDQWH FRPSHWHQWHVHHQWHQGHUUHOHYDQWHDPDWpULDVXSULUGH
GHVSDFKRPRWLYDGRDEULUSHUtRGRGHFRQVXOWDS~EOLFD RItFLRDRPLVVmRQmRVHH[LPLQGRGHSURIHULUDGHFLVmR
SDUD PDQLIHVWDomR GH WHUFHLURV DQWHV GD GHFLVmR GR
SHGLGRVHQmRKRXYHUSUHMXt]RSDUDDSDUWHLQWHUHVVD $UW 4XDQGRGDGRVDWXDo}HVRXGRFXPHQWRVVROLFLWDGRVDR
GD LQWHUHVVDGRIRUHPQHFHVViULRVjDSUHFLDomRGHSHGLGR
IRUPXODGR R QmR DWHQGLPHQWR QR SUD]R IL[DGR SHOD
†ž $DEHUWXUDGDFRQVXOWDS~EOLFDVHUiREMHWRGHGLYXOJD $GPLQLVWUDomRSDUDDUHVSHFWLYDDSUHVHQWDomRLPSOLFDUi
omRSHORVPHLRVRILFLDLVDILPGHTXHSHVVRDVItVLFDVRX DUTXLYDPHQWRGRSURFHVVR
MXUtGLFDVSRVVDPH[DPLQDURVDXWRVIL[DQGRVHSUD]R
SDUDRIHUHFLPHQWRGHDOHJDo}HVHVFULWDV $UW 2VLQWHUHVVDGRVVHUmRLQWLPDGRVGHSURYDRXGLOLJrQFLD
RUGHQDGDFRPDQWHFHGrQFLDPtQLPDGHWUrVGLDV~WHLV
†ž 2FRPSDUHFLPHQWRjFRQVXOWDS~EOLFDQmRFRQIHUHSRU PHQFLRQDQGRVHGDWDKRUDHORFDOGHUHDOL]DomR
VLDFRQGLomRGHLQWHUHVVDGRGRSURFHVVRPDVFRQIHUH
R GLUHLWR GH REWHU GD $GPLQLVWUDomR UHVSRVWD IXQGD $UW 4XDQGRGHYHVHUREULJDWRULDPHQWH RXYLGR XPyUJmR
PHQWDGDTXHSRGHUiVHUFRPXPDWRGDVDVDOHJDo}HV FRQVXOWLYR R SDUHFHU GHYHUi VHU HPLWLGR QR SUD]R
VXEVWDQFLDOPHQWHLJXDLV Pi[LPR GH TXLQ]H GLDV VDOYR QRUPD HVSHFLDO RX
FRPSURYDGDQHFHVVLGDGHGHPDLRUSUD]R
$UW $QWHV GD WRPDGD GH GHFLVmR D MXt]R GDDXWRULGDGH
GLDQWHGDUHOHYkQFLDGDTXHVWmRSRGHUiVHUUHDOL]DGD †ž 6HXPSDUHFHUREULJDWyULRHYLQFXODQWHGHL[DUGHVHU
DXGLrQFLD S~EOLFD SDUD GHEDWHV VREUH D PDWpULD GR HPLWLGRQRSUD]RIL[DGRRSURFHVVRQmRWHUiVHJXLPHQ
SURFHVVR WRDWpDUHVSHFWLYDDSUHVHQWDomRUHVSRQVDELOL]DQGRVH
TXHPGHUFDXVDDRDWUDVR
$UW 2V yUJmRV H HQWLGDGHV DGPLQLVWUDWLYDV HPPDWpULD
UHOHYDQWHSRGHUmRHVWDEHOHFHURXWURVPHLRVGHSDUWLFL †ž 6HXPSDUHFHUREULJDWyULRHQmRYLQFXODQWHGHL[DUGH
SDomRGHDGPLQLVWUDGRVGLUHWDPHQWHRXSRUPHLRGH VHU HPLWLGR QR SUD]R IL[DGR R SURFHVVR SRGHUi WHU
RUJDQL]Do}HVHDVVRFLDo}HVOHJDOPHQWHUHFRQKHFLGDV SURVVHJXLPHQWRHVHUGHFLGLGRFRPVXDGLVSHQVDVHP
SUHMXt]R GD UHVSRQVDELOLGDGH GH TXHP VH RPLWLX QR
$UW 2V UHVXOWDGRV GD FRQVXOWD H DXGLrQFLD S~EOLFD HGH DWHQGLPHQWR
RXWURVPHLRVGHSDUWLFLSDomRGHDGPLQLVWUDGRVGHYHUmR
VHU DSUHVHQWDGRV FRP D LQGLFDomR GR SURFHGLPHQWR $UW 4XDQGR SRU GLVSRVLomR GH DWR QRUPDWLYR GHYDPVHU
DGRWDGR SUHYLDPHQWHREWLGRVODXGRVWpFQLFRVGHyUJmRVDGPLQLV
WUDWLYRV H HVWHV QmR FXPSULUHP R HQFDUJR QR SUD]R
$UW 4XDQGRQHFHVViULDjLQVWUXomRGRSURFHVVRDDXGLrQFLD DVVLQDODGRRyUJmRUHVSRQViYHOSHODLQVWUXomRGHYHUi
GHRXWURVyUJmRVRXHQWLGDGHVDGPLQLVWUDWLYDVSRGHUi VROLFLWDU ODXGR WpFQLFR GH RXWUR yUJmR GRWDGR GH
VHUUHDOL]DGDHPUHXQLmRFRQMXQWDFRPDSDUWLFLSDomR TXDOLILFDomRHFDSDFLGDGHWpFQLFDHTXLYDOHQWHV
GHWLWXODUHVRXUHSUHVHQWDQWHVGRVyUJmRVFRPSHWHQWHV
ODYUDQGRVHDUHVSHFWLYDDWDDVHUMXQWDGDDRVDXWRV $UW (QFHUUDGDDLQVWUXomRRLQWHUHVVDGRWHUiRGLUHLWRGH
PDQLIHVWDUVHQRSUD]RPi[LPRGHGH]GLDVVDOYRVH
$UW &DEH DR LQWHUHVVDGR D SURYD GRV IDWRV TXHWHQKD RXWURSUD]RIRUOHJDOPHQWHIL[DGR
DOHJDGR VHP SUHMXt]R GR GHYHU DWULEXtGR DR yUJmR
FRPSHWHQWHSDUDDLQVWUXomRHGRGLVSRVWRQR$UW $UW (P FDVR GH ULVFR LPLQHQWH D $GPLQLVWUDomR3~EOLFD
GHVWD/HL SRGHUiPRWLYDGDPHQWHDGRWDUSURYLGrQFLDVDFDXWHODGR
UDVVHPDSUpYLDPDQLIHVWDomRGRLQWHUHVVDGR
$UW 4XDQGRRLQWHUHVVDGRGHFODUDUTXHIDWRVHGDGRVHVWmR
UHJLVWUDGRV HP GRFXPHQWRV H[LVWHQWHV QD SUySULD $UW 2V LQWHUHVVDGRV WrP GLUHLWR j YLVWD GR SURFHVVR HD
$GPLQLVWUDomRUHVSRQViYHOSHORSURFHVVRRXHPRXWUR REWHU FHUWLG}HV RX FySLDV UHSURJUiILFDV GRV GDGRV H
yUJmR DGPLQLVWUDWLYR R yUJmR FRPSHWHQWH SDUD D GRFXPHQWRV TXH R LQWHJUDP UHVVDOYDGRV RV GDGRV H
LQVWUXomRSURYHUiGHRItFLRjREWHQomRGRVGRFXPHQ GRFXPHQWRVGHWHUFHLURVSURWHJLGRVSRUVLJLORRXSHOR
WRVRXGDVUHVSHFWLYDVFySLDV GLUHLWRjSULYDFLGDGHjKRQUDHjLPDJHP
6HFUHWDULDGH(VWDGRGH$ GPLQLVWUDomR3~EOLFD'LVWULWR)HGHUDO
 /HJLVODomR 

$UW 2 yUJmR GH LQVWUXomR TXH QmR IRU FRPSHWHQWHSDUD &DStWXOR;,9


HPLWLUDGHFLVmRILQDOHODERUDUiUHODWyULRLQGLFDQGRR 'D$QXODomR5HYRJDomRH&RQYDOLGDomR
'D$QXODomR5HYRJDomRH&RQYDOLGDomR
SHGLGRLQLFLDORFRQWH~GRGDVIDVHGRSURFHGLPHQWRH
IRUPXODUiSURSRVWDGHGHFLVmRREMHWLYDPHQWHMXVWLILFD $UW $$GPLQLVWUDomRGHYHDQXODUVHXVSUySULRVDWRVTXDQ
GDHQFDPLQKDQGRRSURFHVVRjDXWRULGDGHFRPSHWHQWH GRHLYDGRVGHYtFLRGHOHJDOLGDGHHSRGHUHYRJiORVSRU
PRWLYRGHFRQYHQLrQFLDRXRSRUWXQLGDGHUHVSHLWDGRV
&DStWXOR;, RVGLUHLWRVDGTXLULGRV
'R'HYHUGH'HFLGLU
'R'HYHUGH'HFLGLU
$UW 2GLUHLWRGD$GPLQLVWUDomRGHDQXODURVDWRVDGPLQLV
$UW $$GPLQLVWUDomRWHPRGHYHUGHH[SOLFLWDPHQWHHPLWLU WUDWLYRV GH TXH GHFRUUDP HIHLWRV IDYRUiYHLV SDUD RV
GHFLVmRQRVSURFHVVRVDGPLQLVWUDWLYRVHVREUHVROLFLWD GHVWLQDWiULRVGHFDLHPFLQFRDQRVFRQWDGRVGDGDWDHP
o}HVRXUHFODPDo}HVHPPDWpULDGHVXDFRPSHWrQFLD TXHIRUDPSUDWLFDGRVVDOYRFRPSURYDGDPiIp

$UW &RQFOXtGD D LQVWUXomR GH SURFHVVR DGPLQLVWUDWLYRD †ž 1RFDVRGHHIHLWRVSDWULPRQLDLVFRQWtQXRVRSUD]RGH


$GPLQLVWUDomR WHP R SUD]R GH DWp WULQWD GLDV SDUD GHFDGrQFLD FRQWDUVHi GD SHUFHSomR GR SULPHLUR
GHFLGLUVDOYRSURUURJDomRSRULJXDOSHUtRGRH[SUHVVD SDJDPHQWR
PHQWHPRWLYDGD
†ž &RQVLGHUDVH H[HUFtFLR GR GLUHLWRGHDQXODUTXDOTXHU
&DStWXOR;,, PHGLGD GH DXWRULGDGH DGPLQLVWUDWLYD TXH LPSRUWH
'D0RWLYDomR LPSXJQDomRjYDOLGDGHGRDWR
$UW 2V DWRV DGPLQLVWUDWLYRV GHYHUmR VHU PRWLYDGRVFRP $UW (PGHFLVmRQDTXDOVHHYLGHQFLHQmRDFDUUHWDUHPOHVmR
LQGLFDomRGRVIDWRVHGRVIXQGDPHQWRVMXUtGLFRVTXDQ DRLQWHUHVVHS~EOLFRQHPSUHMXt]RDWHUFHLURVRVDWRV
GR TXHDSUHVHQWDUHPGHIHLWRVVDQiYHLVSRGHUmRVHUFRQYD
OLGDGRVSHODSUySULD$GPLQLVWUDomR
 , QHJXHPOLPLWHPRXDIHWHPGLUHLWRVRXLQWHUHVVHV
 ,, LPSRQKDPRXDJUDYHPGHYHUHVHQFDUJRVRXVDQo}HV
&DStWXOR;9
 ,,, GHFLGDP SURFHVVRV DGPLQLVWUDWLYRVGHFRQFXUVRRX
'R5HFXUVR$GPLQLVWUDWLYRHGD5HYLVmR
VHOHomRS~EOLFD
 ,9 GLVSHQVHPRXGHFODUHPDLQH[LJLELOLGDGHGHSURFHVVR
$UW 'DVGHFLV}HVDGPLQLVWUDWLYDVFDEHUHFXUVRHPIDFHGH
OLFLWDWyULR
UD]}HVGHOHJDOLGDGHHGHPpULWR
 9 GHFLGDPUHFXUVRVDGPLQLVWUDWLYRV
9, GHFRUUDPGHUHH[DPHGHRItFLR
9,, GHL[HP GH DSOLFDU MXULVSUXGrQFLD ILUPDGD VREUHD †ž 2 UHFXUVR VHUi GLULJLGRjDXWRULGDGHTXHSURIHULXD
TXHVWmRRXGLVFUHSHPGHSDUHFHUHVODXGRVSURSRVWDV GHFLVmRDTXDOVHQmRDUHFRQVLGHUDUQRSUD]RGHFLQFR
HUHODWyULRVRILFLDLV GLDVRHQFDPLQKDUijDXWRULGDGHVXSHULRU
9,,, LPSRUWHPDQXODomRUHYRJDomRVXVSHQVmRRXFRQYDOL
GDomRGHDWRDGPLQLVWUDWLYR †ž 6DOYRH[LJrQFLDOHJDODLQWHUSRVLomRGHUHFXUVRDGPL
 QLVWUDWLYRLQGHSHQGHGHFDXomR
†ž $ PRWLYDomR GHYH VHU H[SOtFLWDFODUDHFRQJUXHQWH
SRGHQGRFRQVLVWLUHPGHFODUDomRGHFRQFRUGkQFLDFRP †ž 6H R UHFRUUHQWH DOHJDU TXHDGHFLVmRDGPLQLVWUDWLYD
IXQGDPHQWRV GH DQWHULRUHV SDUHFHUHV LQIRUPDo}HV FRQWUDULD HQXQFLDGR GD V~PXOD YLQFXODQWH FDEHUi j
GHFLV}HV RX SURSRVWDV TXH QHVWH FDVR VHUmR SDUWH DXWRULGDGHSURODWRUDGDGHFLVmRLPSXJQDGDVHQmRD
LQWHJUDQWHGRDWR UHFRQVLGHUDUH[SOLFLWDUDQWHVGHHQFDPLQKDURUHFXUVR
jDXWRULGDGHVXSHULRUDVUD]}HVGDDSOLFDELOLGDGHRX
†ž 1D VROXomR GH YiULRV DVVXQWRVGDPHVPDQDWXUH]D LQDSOLFDELOLGDGHGDV~PXODFRQIRUPHRFDVR
2EVDFUHVFLGRSHOD/HLQž
SRGH VHU XWLOL]DGR PHLR PHFkQLFR TXH UHSURGX]D RV
IXQGDPHQWRVGDVGHFLV}HVGHVGHTXHQmRSUHMXGLTXH $UW 2UHFXUVRDGPLQLVWUDWLYRWUDPLWDUiQRPi[LPRSRUWUrV
GLUHLWRRXJDUDQWLDGRVLQWHUHVVDGRV LQVWkQFLDV DGPLQLVWUDWLYDV VDOYR GLVSRVLomR OHJDO
GLYHUVD
†ž $ PRWLYDomR GDV GHFLV}HV GHyUJmRVFROHJLDGRVH
FRPLVV}HVRXGHGHFLV}HVRUDLVFRQVWDUiGDUHVSHFWLYD $UW 7rPOHJLWLPLGDGHSDUDLQWHUSRUUHFXUVRDGPLQLVWUDWLYR
DWDRXGHWHUPRHVFULWR 
, RVWLWXODUHVGHGLUHLWRVHLQWHUHVVHVTXHIRUHPSDUWHQR
&DStWXOR;,,, SURFHVVR
'HVLVWrQFLDH2XWURV&DVRVGH([WLQomRGR3URFHVVR
'HVLVWrQFLDH2XWURV&DVRVGH([WLQomRGR3URFHVVR  ,, DTXHOHVFXMRVGLUHLWRVRXLQWHUHVVHVIRUHPLQGLUHWDPHQ
WHDIHWDGRVSHODGHFLVmRUHFRUULGD
$UW 2LQWHUHVVDGRSRGHUiPHGLDQWHPDQLIHVWDomRHVFULWD 
GHVLVWLUWRWDORXSDUFLDOPHQWHGRSHGLGRIRUPXODGRRX ,,, DV RUJDQL]Do}HV H DVVRFLDo}HV UHSUHVHQWDWLYDVQR
DLQGDUHQXQFLDUDGLUHLWRVGLVSRQtYHLV WRFDQWHDGLUHLWRVHLQWHUHVVHVFROHWLYRV
,9 RV FLGDGmRV RX DVVRFLDo}HV TXDQWR D GLUHLWRVRX
†ž +DYHQGRYiULRVLQWHUHVVDGRVDGHVLVWrQFLDRXUHQ~QFLD LQWHUHVVHVGLIXVRV
DWLQJHVRPHQWHTXHPDWHQKDIRUPXODGR
$UW 6DOYRGLVSRVLomROHJDOHVSHFtILFDpGHGH]GLDVRSUD]R
†ž $GHVLVWrQFLDRXUHQ~QFLDGRLQWHUHVVDGRFRQIRUPHR SDUDLQWHUSRVLomRGHUHFXUVRDGPLQLVWUDWLYRFRQWDGRD
FDVRQmRSUHMXGLFDRSURVVHJXLPHQWRGRSURFHVVRVH SDUWLU GD FLrQFLD RX GLYXOJDomR RILFLDO GD GHFLVmR
D $GPLQLVWUDomR FRQVLGHUDU TXH R LQWHUHVVH S~EOLFR UHFRUULGD
DVVLPRH[LJH 
†ž 4XDQGR D OHL QmR IL[DU SUD]RGLIHUHQWHRUHFXUVR
$UW 2yUJmRFRPSHWHQWHSRGHUiGHFODUDUH[WLQWRRSURFHVVR DGPLQLVWUDWLYRGHYHUiVHUGHFLGLGRQRSUD]RPi[LPRGH
TXDQGRH[DXULGDVXDILQDOLGDGHRXRREMHWRGDGHFLVmR WULQWD GLDV D SDUWLU GR UHFHELPHQWR GRV DXWRV SHOR
VH WRUQDU LPSRVVtYHO LQ~WLO RX SUHMXGLFDGR SRU IDWR
yUJmRFRPSHWHQWH
VXSHUYHQLHQWH
  /HJLVODomR 6HFUHWDULDGH(VWDGRGH$GPLQLVWUDomR3~EOLFD'LVWULWR)HGHUDO


†ž 2SUD]RPHQFLRQDGRQRSDUiJUDIRDQWHULRUSRGHUiVHU &DStWXOR;9,


SURUURJDGRSRULJXDOSHUtRGRDQWHMXVWLILFDWLYDH[SOtFL 'RV3UD]RV
WD
$UW 2VSUD]RVFRPHoDPDFRUUHUDSDUWLUGDGDWDGDFLHQWLIL
$UW 2 UHFXUVR LQWHUS}HVH SRU PHLR GH UHTXHULPHQWRQR FDomR RILFLDO H[FOXLQGRVH GD FRQWDJHP R GLD GR
TXDO R UHFRUUHQWH GHYHUi H[SRU RV IXQGDPHQWRV GR FRPHoRHLQFOXLQGRVHRGRYHQFLPHQWR
SHGLGRGHUHH[DPHSRGHQGRMXQWDURVGRFXPHQWRVTXH
MXOJDUFRQYHQLHQWHV †ž &RQVLGHUDVHSURUURJDGRRSUD]RDWpRSULPHLURGLD~WLO
VHJXLQWH VH R YHQFLPHQWR FDLU HP GLD HP TXH QmR
$UW 6DOYRGLVSRVLomROHJDOHPFRQWUiULRRUHFXUVRQmRWHP KRXYHUH[SHGLHQWHRXHVWHIRUHQFHUUDGRDQWHVGDKRUD
HIHLWRVXVSHQVLYR QRUPDO

†~QLFR +DYHQGRMXVWRUHFHLRGHSUHMXt]RGHGLItFLORXLQFHUWD
†ž 2V SUD]RV H[SUHVVRV HP GLDVFRQWDPVHGHPRGR
FRQWtQXR
UHSDUDomR GHFRUUHQWH GD H[HFXomR D DXWRULGDGH

UHFRUULGD RX D LPHGLDWDPHQWH VXSHULRU SRGHUi GH
†ž 2VSUD]RVIL[DGRVHPPHVHVRXDQRVFRQWDPVHGHGDWD
RItFLRRXDSHGLGRGDUHIHLWRVXVSHQVLYRDRUHFXUVR
D GDWD 6H QR PrV GR YHQFLPHQWR QmR KRXYHU R GLD
HTXLYDOHQWH jTXHOH GR LQtFLR GR SUD]R WHPVH FRPR
$UW ,QWHUSRVWR R UHFXUVR R yUJmR FRPSHWHQWH SDUDGHOH WHUPRR~OWLPRGLDGRPrV
FRQKHFHUGHYHUiLQWLPDURVGHPDLVLQWHUHVVDGRVSDUD
TXH QR SUD]R GH FLQFR GLDV ~WHLV DSUHVHQWHP $UW 6DOYRPRWLYRGHIRUoDPDLRUGHYLGDPHQWHFRPSURYDGR
DOHJDo}HV RVSUD]RVSURFHVVXDLVQmRVHVXVSHQGHP

$UW 2UHFXUVRQmRVHUiFRQKHFLGRTXDQGRLQWHUSRVWR &DStWXOR;9,,


'DV6DQo}HV
 , IRUDGRSUD]R
 ,, SHUDQWHyUJmRLQFRPSHWHQWH $UW $V VDQo}HV D VHUHP DSOLFDGDV SRU DXWRULGDGHFRP
 ,,, SRUTXHPQmRVHMDOHJLWLPDGR SHWHQWHWHUmRQDWXUH]DSHFXQLiULDRXFRQVLVWLUmRHP
 ,9 DSyVH[DXULGDDHVIHUDDGPLQLVWUDWLYD REULJDomRGHID]HURXGHQmRID]HUDVVHJXUDGRVHPSUH
RGLUHLWRGHGHIHVD
†ž 1DKLSyWHVHGRLQFLVR,,VHUiLQGLFDGDDRUHFRUUHQWHD
DXWRULGDGH FRPSHWHQWH VHQGROKH GHYROYLGR R SUD]R &DStWXOR;9,,,
SDUDUHFXUVR 'DV'LVSRVLo}HV

†ž 2QmRFRQKHFLPHQWRGRUHFXVRQmRLPSHGHD$GPLQLV $UW 2VSURFHVVRVDGPLQLVWUDWLYRVHVSHFtILFRVFRQWLQXDUmRD


WUDomRGHUHYHUGHRItFLRRDWRLOHJDOGHVGHTXHQmR UHJHUVH SRU OHL SUySULD DSOLFDQGRVHOKHV DSHQDV
RFRUULGDSUHFOXVmRDGPLQLVWUDWLYD VXEVLGLDULDPHQWHRVSUHFHLWRVGHVWD/HL

$UW 2 yUJmR FRPSHWHQWH SDUD GHFLGLU R UHFXUVRSRGHUi $UW$ 7HUmRSULRULGDGHQDWUDPLWDomRHPTXDOTXHUyUJmRRX


FRQILUPDU PRGLILFDU DQXODU RX UHYRJDU WRWDO RX LQVWkQFLD RV SURFHGLPHQWRV DGPLQLVWUDWLYRV HP TXH
SDUFLDOPHQWHDGHFLVmRUHFRUULGDVHDPDWpULDIRUGH ILJXUHFRPRSDUWHRXLQWHUHVVDGR
VXDFRPSHWrQFLD
, SHVVRD FRP LGDGH LJXDO RX VXSHULRUD VHVVHQWD
†~QLFR 6HGDDSOLFDomRGRGLVSRVWRQHVWHDUWLJRSXGHUGHFRUUHU DQRV
JUDYDPH j VLWXDomR GR UHFRUUHQWH HVWH GHYHUi VHU
FLHQWLILFDGRSDUDTXHIRUPXOHVXDVDOHJDo}HVDQWHVGD ,, SHVVRDSRUWDGRUDGHGHILFLrQFLDItVLFDRXPHQWDO
GHFLVmR ,,, 9(7$'2 

$UW$ 6H R UHFRUUHQWH DOHJDU YLRODomR GH HQXQFLDGR GD ,9 SHVVRD SRUWDGRUD GH WXEHUFXORVH DWLYDHVFOHURVH
V~PXODYLQFXODQWHRyUJmRFRPSHWHQWHSDUDGHFLGLUR P~OWLSOD QHRSODVLD PDOLJQD KDQVHQtDVH SDUDOLVLD
UHFXUVR H[SOLFLWDUi DV UD]}HV GD DSOLFDELOLGDGH RX LUUHYHUVtYHOHLQFDSDFLWDQWHFDUGLRSDWLDJUDYHGRHQoD
LQDSOLFDELOLGDGHGDV~PXODFRQIRUPHRFDVR GH3DUNLQVRQHVSRQGLORDUWURVHDQTXLORVDQWHQHIURSD
2EVDFUHVFHQWDGRSHOD/HLQž
WLD JUDYH KHSDWRSDWLD JUDYH HVWDGRV DYDQoDGRV GD
$UW% $FROKLGDSHOR6XSUHPR7ULEXQDO)HGHUDODUHFODPDomR GRHQoDGH3DJHW RVWHtWHGHIRUPDQWH FRQWDPLQDomR
IXQGDGDHPYLRODomRGHHQXQFLDGRGDV~PXODYLQFXODQ SRUUDGLDomRVtQGURPHGHLPXQRGHILFLrQFLDDGTXLULGD
WHGDUVHiFLrQFLDjDXWRULGDGHSURODWRUDHDRyUJmR RX RXWUD GRHQoD JUDYH FRP EDVH HP FRQFOXVmR GD
FRPSHWHQWHSDUDRMXOJDPHQWRGRUHFXUVRTXHGHYHUmR PHGLFLQD HVSHFLDOL]DGD PHVPR TXH D GRHQoD WHQKD
DGHTXDUDVIXWXUDVGHFLV}HVDGPLQLVWUDWLYDVHPFDVRV VLGRFRQWUDtGDDSyVRLQtFLRGRSURFHVVR
VHPHOKDQWHVVRESHQDGHUHVSRQVDELOL]DomRSHVVRDOQDV
HVIHUDVFtYHODGPLQLVWUDWLYDHSHQDO †ž $SHVVRDLQWHUHVVDGDQDREWHQomRGREHQHItFLRMXQWDQ
GRSURYDGHVXDFRQGLomRGHYHUiUHTXHUrORjDXWRUL
2EVDFUHVFHQWDGRSHOD/HLQžGH GDGHDGPLQLVWUDWLYDFRPSHWHQWHTXHGHWHUPLQDUiDV
$UW 2VSURFHVVRVDGPLQLVWUDWLYRVGHTXHUHVXOWHPVDQo}HV SURYLGrQFLDVDVHUHPFXPSULGDV
SRGHUmRVHUUHYLVWRVDTXDOTXHUWHPSRDSHGLGRRXGH
RItFLRTXDQGRVXUJLUHPIDWRVQRYRVRXFLUFXQVWkQFLDV †ž 'HIHULGDDSULRULGDGHRVDXWRVUHFHEHUmRLGHQWLILFDomR
UHOHYDQWHV VXVFHWtYHLV GH MXVWLILFDU D LQDGHTXDomR GD SUySULDTXHHYLGHQFLHRUHJLPHGHWUDPLWDomRSULRULWi
ULD
VDQomRDSOLFDGD
 †ž 9(7$'2 
†~QLFR 'D UHYLVmR GR SURFHVVR QmRSRGHUiUHVXOWDUDJUDYD †ž 9(7$'2  2EVDUWLJR$LQFOXtGRSHOD/HLQž

PHQWRGDVDQomR $UW (VWD/HLHQWUDHPYLJRUQDGDWDGHVXDSXEOLFDomR


&kPDUD/HJLVODWLYDGR
  'LVWULWR)HGHUDO 'LU$GPLQLVWUDWLYR 

7(67(6 

 2FRQMXQWR
FRQMXQWRGH
GHyUJmRV
yUJmRVGHSHQGHQWHV
GHSHQGHQWHVVXERUGLQDGRV
VXERUGLQDGRVDR
DRSRGHU  $FDXVD
FDXVDDLQVSLUDomR
LQVSLUDomRSDUD
SDUDDSUiWLFD
SUiWLFDGR
GRDWR
DWRDGPLQLVWUDWLYR
3ROtWLFR
3ROtWLFRRUJDQL]DGRV
RUJDQL]DGRVPDWHULDO
PDWHULDOILQDQFHLUD
ILQDQFHLUDHKXPDQDPHQWH pXPUHTXLVLWRGH
SDUDDH[HFXomRGDVGHFLV}HVSROtWLFDVFKDPDVH D FRPSHWrQFLD
D 5HS~EOLFD E ILQDOLGDGH
E $GPLQLVWUDomR3~EOLFD F IRUPD
F (VWDGR G PRWLYR
G *RYHUQR
 7RGR
7RGR DWR
DWR DGPLQLVWUDWLYR
DGPLQLVWUDWLYR Vy
Vy VHUi
VHUi FRQVLGHUDGRLOHJtWLPRVH
FRQVLGHUDGRLOHJtWLPRVH
 2$WR $GPLQLVWUDWLYRTXH
$WR$GPLQLVWUDWLYR TXHWHP
WHPSRU
SRUSURSyVLWRGLVFLSOLQDU
SURSyVLWRGLVFLSOLQDUR KRXYHU
KRXYHUSURYD
SURYDFDEDO
FDEDOVHJXUD
VHJXUDGH
GHVHU
VHULQYiOLGR
LQYiOLGRHVWH
HVWHDWULEXW
DWULEXWR
IXQFLRQDPHQWR
IXQFLRQDPHQWR GD
GD $GPLQLVWUDomR
$GPLQLVWUDomR 3~EOLFD
3~EOLFD H D FRQGXWD GRDWRDGPLQLVWUDWLYRGHQRPLQDVH
IXQFLRQDOGRVDJHQWHVGHQRPLQDVH D SUHVXQomRGHOHJLWLPLGDGH
D $WRV3XQLWLYRV E LPSHUDWLYLGDGH
E $WRV(QXQFLDWLYRV F DXWRH[HFXWRULHGDGH
F $WRV2UGLQDWyULRV G GLVFULFLRQDULHGDGH
G $WRVGH&RQKHFLPHQWR
 7DPEpP
7DPEpPFRQKHFLGRV
FRQKHFLGRVFRPR
FRPRDWRV
DWRVUHJUDGRV
UHJUDGRVVmR
VmRDTXHOHV
DTXHOHVFXMRV
 2 $WR
$WR $GPLQLVWUDWLYR
$GPLQLVWUDWLYR TXH
TXH HPERUD
HPERUD XQLODWHUDLV
XQLODWHUDLV FRQWHP UHTXLVLWRV
UHTXLVLWRVHFRQGLo}HV
FRQGLo}HVGH
GHVXD
VXDUHDOL]DomR
UHDOL]DomRYHP
YHPHVWDEHOHFLGDV
LQWHUHVVH
LQWHUHVVH UHFtSURFR
UHFtSURFR GD
GD $GPLQLVWUDomR
$GPLQLVWUDomR H GR
GR SDUWLFXODU SHODOHLGHQRPLQDPVH
GHQRPLQDVH D YLQFXODGRV
E LPSHUDWLYRV
D $WRV3XQLWLYRV
F DXWRH[HFXWRULHGDGH
E $WRV(QXQFLDWLYRV
G GLVFULFLRQiULRV
F $WRV1HJRFLDLV
G $WRVGH&RQKHFLPHQWR
 2V
2V
2VDWRV
DWRVHP
HPTXH
TXHVHPSUH
VHPSUHKDYHUi
KDYHUiDLPSRVLomR
LPSRVLomRGH
GHXPD
XPDVDQomR
LPSRVWD
LPSRVWD SHOD
SHOD $GPLQLVWUDomR
$GPLQLVWUDomR 3~EOLFD jTXHOH TXH WLYHUHP
 $ GLIHUHQoD
GLIHUHQoD HVVHQFLDO
HVVHQFLDO HQWUH
HQWUH R DWR
DWR DGPLQLVWUDWLYR
DGPLQLVWUDWLYR H R DWR
LQIULQJLGR
LQIULQJLGR XPD SURLELomR GD OHL GR UHJXODPHQWR RX GH
MXUtGLFRp
GLVSRVLomRRUGLQDWyULDGHQRPLQDVH
D DILQDOLGDGHS~EOLFD D YLQFXODGRV
E HVVHQFLDOPHQWHRLQWHUHVVH E LPSHUDWLYRV
F HIHLWRMXUtGLFR F SXQLWLYRV
G DTXLVLomRGHXPGLUHLWRS~EOLFR G HQXQFLDWLYRV

 7RGR
7RGRDWR
DWRDGPLQLVWUDWLYR
DGPLQLVWUDWLYRGHYH
GHYHWHU
WHUXP
XPSURSyVLWR
SURSyVLWRDDOFDQoDU  2DWRDGPLQLVWUDWLYRVHUiUHYRJDGR
2DWRDGPLQLVWUDWLYRVHUiUHYRJDGR
H
HQDWXUDOPHQWH
QDWXUDOPHQWHXP
XPLQWHUHVVH
LQWHUHVVHS~EOLFR
S~EOLFRHVWH
HVWHpXP
XPUHTXLVLWR D SHORMXGLFLiULR
GH E SHOROHJLVODWLYR
D FRPSHWrQFLD F SHODSUySULDDGPLQLVWUDomRS~EOLFD
E ILQDOLGDGH G SRUTXDOTXHUSHVVRD
F IRUPD
G PRWLYRHREMHWR  2
2 3RGHU
3RGHU $GPLQLVWUDWLYR
$GPLQLVWUDWLYR TXH FRQWURODRGHVHPSHQKRGDV
IXQo}HV
IXQo}HV DGPLQLVWUDWLY
DGPLQLVWUDWLYDV
DV H R FRPSRUWDPHQWR
FRPSRUWDPHQWR LQWHUQR
LQWHUQR GH
 $ PDQHLUD
PDQHLUD SHOD
SHOD TXDO
TXDO R DWR
DWR GHYH VHU SUDWLFDGR p XP VHXV
VHXVDJHQWHV
DJHQWHVSXQLQGRRV
SXQLQGRRVSHODV
SHODVIDOWDV
IDOWDVDSXUDGDV
DSXUDGDVGHQRPLQD
UHTXLVLWRGH VH
D FRPSHWrQFLD D 3RGHU5HJXODPHQWDU
E ILQDOLGDGH E 3RGHU'LVFLSOLQDU
F IRUPD F 3RGHUKLHUiUTXLFR
G PRWLYRHREMHWR G 3RGHUGH3ROtFLD

 %
%  '
'
 &
&  $
$
 &
&  $
$
 $
$  &
&
 %
%  &
&
 &
&  %
%
&kPDUD/HJLVODWLYDGR'LVWULWR)HGHUDO
 'LU$
GPLQLVWUDWLYR

 2
2FRQMXQWR
FRQMXQWRGH
GHDWULEXLo}HV
DWULEXLo}HVFRQFHGLGDV
FRQFHGLGDVj$GPLQLVWUDomRSDUD F  GHL[DUGHSUHVWDUFRQWDVTXDQGRHVWHMDREULJDGRDID]r
GLVFLSOLQDU
GLVFLSOLQDU H UHVWULQJLU HP IDYRU GR LQWHUHVVH S~EOLFR ORUHYHODURXSHUPLWLUTXHFKHJXHDRFRQKHFLPHQWRGH
DGHTXDGR
DGHTXDGRGLUHLWRV
GLUHLWRVHOLEHUGDGHV
OLEHUGDGHVLQGLYLGXDLV
LQGLYLGXDLVGHQRPLQDVH WHUFHLURDQWHVGDUHVSHFWLYDGLYXOJDomRRILFLDOWHRUGH
D 3RGHU5HJXODPHQWDU PHGLGDSROtWLFDRXHFRQ{PLFDFDSD]GHDIHWDURSUHoRGH
E 3RGHU'LVFLSOLQDU PHUFDGRULDEHPRXVHUYLoR
F 3RGHU+LHUiUTXLFR G  WRGDVDVDOWHUQDWLYDVHVWmRFRUUHWDV
G 3RGHUGH3ROtFLD
1DV
 1DV FRPSUDVHYHQWXDLV
1DVFRPSUDV HYHQWXDLVGH
GHJrQHURV
JrQHURVDOLPHQWtFLRV
DOLPHQWtFLRVSHUHFtYHLV
 1
1R FDVR
FDVR GR
GR HQULTXHFLPHQWR
HQULTXHFLPHQWR LOtFLWR
LOtFLWR SHUGHUi
SHUGHUi R DJHQWH HP
HPFHQWUR
FHQWURGH
GHDEDVWHFLPHQWR
DEDVWHFLPHQWRRX
RXVLPLODU
VLPLODUUHDOL]DGDV
UHDOL]DGDVGLUHWD
S~EOLFRRXWHUFHLUREHQHILFLiULR
S~EOLFRRXWHUFHLUREHQHILFLiULR PHQWHFRPEDVHQRSUHoRGRGLDDOLFLWDomRVHUi
D  RVEHQVRXYDORUHVDFUHVFLGRVDRVHXSDWULP{QLR D LQGLVSHQViYHO
E  WRGRVVHXSDWULP{QLRDPHDOKDGRGXUDQWHVXDYLGD E GLVSHQViYHO
F  RVOXFURVDXIHULGRVFRPRVQHJyFLRVLOtFLWRV F REULJDWyULD
G IDFXOWDWLYD
G  WRGDVDVDOWHUQDWLYDVHVWmRFRUUHWDV

 3DUD
3DUD FRQWUDWDomRGH
3DUDFRQWUDWDomR GHSURILVVLRQDO
SURILVVLRQDOGH
GHTXDOTXHU
TXDOTXHUVHWRU
VHWRUDUWtVWLFR
 &RQVWLWXL
&RQVWLWXL
&RQVWLWXL DWR
DWR GH
GH LPSURELGDGH
LPSURELGDGH DGPLQLVWUDW
DGPLQLVWUDWLYD
LYD LPSRUWDQGR
RX
RXDWUDYpV
DWUDYpVGH
GHHPSUHViULR
HPSUHViULRH[FOXVLYR
H[FOXVLYRGHVGH
GHVGHTXH
TXHFRQVDJUDGR
HQULTXHFLPHQWRLOtFLWR
SHOD
SHOD FUtWLFD
FUtWLFD HVSHFLDOL]DGD
HVSHFLDOL]DGD RX
RX SHOD
SHOD RSLQLmR
RSLQLmR S~EOLFD
S~EOLFD D
 D  UHFHEHUSDUDVLRXSDUDRXWUHPGLQKHLUREHPPyYHORX OLFLWDomRVHUi
LPyYHORXTXDOTXHURXWUDYDQWDJHPHFRQ{PLFDGLUHWD D LQGLVSHQViYHO
RXLQGLUHWDDWtWXORGHFRPLVVmRSHUFHQWDJHPJUDWLILFD E GLVSHQViYHO
omR RX SUHVHQWH GH TXHP WHQKD LQWHUHVVH GLUHWR RX F H[LJtYHO
LQGLUHWRTXHSRVVDVHUDWLQJLGRRXDPSDUDGRSRUDomR G LQH[LJtYHO
RXRPLVVmRGHFRUUHQWHGDVDWULEXLo}HVGRDJHQWHS~EOLFR
E  SHUFHEHUYDQWDJHPHFRQ{PLFDGLUHWDRXLQGLUHWDSDUD  $
$ PRGDOLGDGH
PRGDOLGDGH GH OLFLWDomR HQWUH TXDLVTXHU LQWHUHVVDGRV
IDFLOLWDUDDOLHQDomRSHUPXWDRXORFDomRGHEHPS~EOLFR TXH
TXHQD
QDIDVH
IDVHLQLFLDO
LQLFLDOGH
GHKDELOLWDomR
KDELOLWDomRSUHOLPLQDU
SUHOLPLQDUFRPSURYHP
RXRIRUQHFLPHQWRGHVHUYLoRSRUHQWHHVWDWDOSRUSUHoR SRVVXLURVUHTXLVLWRVPtQLPRVGHTXDOLILFDomRH[LJLGRVQR
LQIHULRUDRYDORUGHPHUFDGR HGLWDOSDUDH[HFXomRGHVHXREMHWRGHQRPLQDVH
F  UHFHEHU YDQWDJHP HFRQ{PLFD GH TXDOTXHUQDWXUH]D D FRQFRUUrQFLD
GLUHWDRXLQGLUHWDSDUDWROHUDUDH[SORUDomRRXDSUiWLFD E WRPDGDGHSUHoRV
GH MRJRV GH D]DU GH OHQRFtQLR GH QDUFRWUiILFR GH F FRQYLWH
FRQWUDEDQGRGHXVXUDRXGHTXDOTXHURXWUDDWLYLGDGH G FRQFXUVR
LOtFLWDRXDFHLWDUSURPHVVDGHWDOYDQWDJHP
G  WRGDVDVDOWHUQDWLYDVHVWmRFRUUHWDV  $
$PRGDOLGDGH
PRGDOLGDGHGH
GHOLFLWDomR
OLFLWDomRHQWUH
HQWUHLQWHUHVVDGRV
LQWHUHVVDGRVGHYLGDPHQWH
FDGDVWUDGRV
FDGDVWUDGRVRX
RXTXH
TXHDWHQGHUHP
DWHQGHUHPWRGDV
WRGDVDV
DVFRQGLo}HV
FRQGLo}HVH[LJLGDV
 &RQVWLWXL
&RQVWLWXL
&RQVWLWXL DWR
DWR GH
GH LPSURELGDGH
LPSURELGDGH DGPLQLVWUDWLYD
DGPLQLVWUDWLYD TXH FDXVD SDUDFDGDVWUDPHQWRDWp
SDUDFDGDVWUDPHQWR DWpRž
žGLD
GLDDQWHULRU
DQWHULRUjGDWD
GDWDGR
GRUHFHEL
OHVmR
OHVmR DR
DR HUiULR
HUiULR TXDOTXHU DomR RX RPLVVmR GRORVD RX PHQWR
PHQWRGDV
GDVSURSRVWDV
SURSRVWDVREVHUYDGD
REVHUYDGDDQHFHVViULD
QHFHVViULDTXDOLILFDomR
FXOSRVDTXHHQVHMHSHUGDSDWULPRQLDOGHVYLRDSURSULD GHQRPLQDVH
omR
omRPDO
PDOEDUDWDPHQWR
EDUDWDPHQWRRXRXGLODSLGDomR
GLODSLGDomRGRV
GRVEHQV
EHQVRX
RXKDYHUHV D FRQFRUUrQFLD
GDVHQWLGDGHVHQRWDGDPHQWH E WRPDGDGHSUHoRV
 D  SHUPLWLURXIDFLOLWDUDDTXLVLomRSHUPXWDRXORFDomRGH F OHLOmR
EHPRXVHUYLoRSRUSUHoRVXSHULRUDRGHPHUFDGR G FRQYLWH
E  UHDOL]DURSHUDomRILQDQFHLUDVHPREVHUYkQFLDGDVQRUPDV
 $
$ PRGDOLGDGH
PRGDOLGDGH GH
GH OLFLWDomR
OLFLWDomR HQWUH
HQWUH TXDLVTXHU
TXDLVTXHU LQWHUHVVDGRV
LQWHUHVVDGRV
OHJDLVHUHJXODPHQWDUHVRXDFHLWDUJDUDQWLDLQVXILFLHQWH
SDUD
SDUD HVFROKD
HVFROKD GH
GH WUDEDOKR
WUDEDOKR WpFQLFR
FQLFR FLHQWtILFR
FLHQWtILFR RX
RX DUWtVWLFR
RXLQLG{QHD
PHGLDQWH
PHGLDQWH D LQVWLWXLomR
LQVWLWXLomR GH
GH SUrPLRV RX UHPXQHUDomR DRV
F  FRQFHGHUEHQHItFLRDGPLQLVWUDWLYRRXILVFDOVHPDREVHU
YHQFHGRUHV
YHQFHGRUHV FRQIRUPH
FRQIRUPH FULWpULRV
FULWpULRV FRQVWDQWHV
FRQVWDQWHV GH
GH HGLWDO
HGLWDO
YkQFLD GDV IRUPDOLGDGHV OHJDLV RX UHJXODPHQWDUHV
SXEOLFDGR
SXEOLFDGRQD
QDLPSUHQVD
LPSUHQVDRILFLDO
RILFLDOFRPDQWHFHGrQFLDPtQLPDGH
DSOLFiYHLVjHVSpFLH GLDVGHQRPLQDVH
GLDVGHQRPLQDVH
G  WRGDVDVDOWHUQDWLYDVHVWmRFRUUHWDV D FRQFRUUrQFLD
E WRPDGDGHSUHoRV
 &RQVWLWXL
&RQVWLWXL
&RQVWLWXL DWR
DWR GH
GH LPSURELGDGH
LPSURELGDGH DGPLQLVWUDWLYD TXH DWHQWD F FRQYLWH
FRQWUD
FRQWUD RV
RV SULQFtSLRV
SULQFtSLRV GD DGPLQLVWUDomR S~EOLFD TXDOTXHU G FRQFXUVR
DomR RX RPLVVmR TXH YLROH RV GHYHUHV GH KRQHVWLGDGH
LPSDUFLDOLGDGH
LPSDUFLDOLGDGH OHJDOLGDGH
OHJDOLGDGH H OHDOGDGH
OHDOGDGH jV
jV LQVWLWXLo}HV
LQVWLWXLo}HV H  $
$ PRGDOLGDGH
PRGDOLGDGH GH
GH OLFLWDomR
OLFLWDomR HQWUH
HQWUH TXDLVTXHU LQWHUHVVDGRV
QRWDGDPHQWH SDUD
SDUDDYHQGD
YHQGDGH
GHEHQV
EHQVLPyYHLV
LPyYHLVLQVHUtYHLV
LQVHUtYHLVSDUD
SDUDD$GPLQLVWUD
 D  SUDWLFDUDWRYLVDQGRILPSURLELGRHPOHLRXUHJXODPHQWR omR
omR RX
RX GH
GH SURGXWRV
SURGXWRV OHJDOPHQWH
OHJDOPHQWH DSUHHQGLGRV
DSUHHQGLGRV RX
RX SHQKRUD
SHQKRUD
RXGLYHUVRGDTXHOHSUHYLVWRQDUHJUDGHFRPSHWrQFLDH GRVDTXHPRIHUHFHURPDLRUODQFHLJXDORXVXSHULRUDR
GRVDTXHPRIHUHFHURPDLRUODQFHLJXDORXVXSHULRUDR
UHWDUGDU RX GHL[DU GH SUDWLFDU LQGHYLGDPHQWH DWR GH GDDYDOLDomRGHQRPLQDVH
RItFLR D WRPDGDGHSUHoRV
E  UHYHODU IDWR RX FLUFXQVWkQFLD GH TXH WHP FLrQFLDHP E FRQYLWH
UD]mRGDVDWULEXLo}HVHTXHGHYDSHUPDQHFHUHPVHJUH F OHLOmR
GR QHJDU SXEOLFLGDGH DRV DWRV RILFLDLV H IUXVWUDU D G FRQFRUUrQFLD
OLFLWXGHGHFRQFXUVRS~EOLFR

 %
% DUW/Qž
 '
'  '
' DUW,,,/Qž
 $
$ DUWž/Qž
$  $
$ DUW†ž/Qž 
 '
' DUWž/Qž
'  %
% DUW†ž/Qž
 '
' DUW/Qž
'  '
' DUW†ž/Qž
 '
' DUW/Qž
'  &
& DUW†ž/Qž
&kPDUD/HJLVODWLYDGR
  'LVWULWR)HGHUDO 'LU$GPLQLVWUDWLYR 

 $
$ DGPLQLVWUDomR
DGPLQLVWUDomR QmR SRGHUi FHOHEUDU R FRQWUDWR FRP  6HJXQGR
6HJXQGR
6HJXQGR D /HL
/HL &RPSOHPHQWDU
&RPSOHPHQWDU Qž
Qž 
 D DVVLQDWXUDGR
DVVLQDWXUDGR
SUHWHULomR
SUHWHULomRGD
GDRUGHP
RUGHPGH
GHFODVVLILFDomR
FODVVLILFDomRGDV
GDVSURSRVWDV
SURSRVWDVRX
RXFRP UHVSHFWLYR
UHVSHFWLYRWHUPR
WHUPRGR
GRTXDOGHYHPFRQVWDU
TXDOGHYHP FRQVWDUDV
DVDWULEXLo}HV
DWULEXLo}HVRV
WHUFHLURV
WHUFHLURV HVWUDQKRV
HVWUDQKRV DR
DR SURFHGLPHQWR
SURFHGLPHQWR OLFLWDWyULR
OLFLWDWyULR VRE SHQD GLUHLWRV
GLUHLWRV H RV
RV GHYHUHV
GHYHUHV LQHUHQWHV
LQHUHQWHV DR FDUJR RFXSDGR
GH GHQRPLQDVH
D QXOLGDGH D  SRVVH
E DQXODomR E  LQYHVWLGXUD
F QRYDOLFLWDomR F  SURPRomR
G DGLWDPHQWRjOLFLWDomR G  WRGDVDVDOWHUQDWLYDVHVWmRFRUUHWDV

 6mR
6mR  6HJXQGR
6HJXQGR
6HJXQGRD/HL
/HL&RPSOHPHQWDU
&RPSOHPHQWDUQž GXUDQWHRHVWiJLR
GXUDQWH
Qž
6mRFOiXVXODV
FOiXVXODVQHFHVViULDV
QHFHVViULDVHP
HPWRGR
WRGRFRQWUDWR
FRQWUDWRDV
DVTXH
TXHHVWDEH
SUREDWyULR
SUREDWyULR VmR DYDOLDGDV D DSWLGmR D FDSDFLGDGH H D
OHoDP
HILFLrQFLD
HILFLrQFLDGR
GRVHUYLGRU
VHUYLGRUSDUD
SDUDRGHVHPSHQKR
GHVHPSHQKRGRGRFDUJR
FDUJRFRP
FRPD
D RUHJLPHGHH[HFXomRRXDIRUPDGHIRUQHFLPHQWR
REVHUYkQFLDGRVIDWRUHV
E DILDQoDEDQFiULD
D  DVVLGXLGDGHHSRQWXDOLGDGH
F DFDXomRHPGLQKHLURHPWtWXORVGHGtYLGDS~EOLFD E  GLVFLSOLQDHFDSDFLGDGHGHLQLFLDWLYD
G RVSUD]RVGHLQH[HFXomR F  SURGXWLYLGDGHHUHVSRQVDELOLGDGH
G  WRGDVDVDOWHUQDWLYDVHVWmRFRUUHWDV
 2
2 FRQWUDWR
FRQWUDWR SRGHUi
SRGHUi VHU
VHU DOWHUDGR
DOWHUDGR XQLODWHUDOPHQWH
XQLODWHUDOPHQWH SHOD
$GPLQLVWUDomRTXDQGR  6HJXQGR
6HJXQGR /HL &RPSO
6HJXQGR D /HL &RPSOHPHQWDU
HPHQWDU QžQž 
 WHUmR
WHUmR FDUiWHU
D FRQYHQLHQWHDVXEVWLWXLomRGDJDUDQWLDGDH[HFXomR LQGHQL]DWyULRRYDORUGDVSDUFHODVUHODWLYDVD
E QHFHVViULDDPRGLILFDomRGDIRUPDGHSDJDPHQWRSRU D  GLiULDHSDVVDJHPSDUDYLDJHP
LPSRVLomRGHFLUFXQVWkQFLDVVXSHUYHQLHQWHV E  FRQYHUVmRGHIpULDVRXGHSDUWHGHODVHPSHF~QLD
F QHFHVViULDDPRGLILFDomRGRUHJLPHGHH[HFXomRGDREUD F  FUpGLWRVGHFRUUHQWHVGHGHPLVVmRH[RQHUDomRHDSRVHQ
G KRXYHU PRGLILFDomR GR SURMHWR RX GDV HVSHFLILFDo}HV WDGRULD RX UHODWLYRV D IpULDV DGLFLRQDO GH IpULDV RX
SDUDPHOKRUDGHTXDomRWpFQLFDDRVVHXVREMHWLYRV FRQYHUVmRGHOLFHQoDSUrPLRHPSHF~QLD
G  WRGDVDVDOWHUQDWLYDVHVWmRFRUUHWDV
 (P
(P
(PKDYHQGR
KDYHQGRDOWHUDomR
DOWHUDomRXQLODWHUDO
XQLODWHUDOGR
GRFRQWUDWR
FRQWUDWRTXH
TXHDXPHQWH
RV
RV HQFDUJRV
HQFDUJRV GR
GR FRQWUDWDGR
FRQWUDWDGR D $GPLQLVWUDomR
$GPLQLVWUDomR GHYHUi  6HJXQGR
6HJXQGR
6HJXQGR D /HL
/HL &RPSOHPHQWDU
&RPSOHPHQWDU Qž  R UHWRUQR j
UHVWDEHOHFHU
UHVWDEHOHFHU R HTXLOtEULR
HTXLOtEULR HFRQ{PLFRILQDQFHLUR
HFRQ{PLFRILQDQFHLUR LQLFLDO DWLYLGDGHGHVHUYLGRUDSRVHQWDGRGHQRPLQDVH
DWUDYpV D  UHFRQGXomR
D QRYDFOiXVXODHPFRQWUDWR E  UHLQWHJUDomR
E QRYRFRQWUDWR F  UHYHUVmR
G  WRGDVDVDOWHUQDWLYDVHVWmRFRUUHWDV
F GHDGLWDPHQWR
G FRPXQLFDomRH[SUHVVDHPHGLWDO
 6HJXQG
6HJXQGRD/HL&RPSOHPHQWDUQžDVIpULDVSRGHP
6HJXQGRD/HL&RPSOHPHQWDUQžDVIpULDVSRGHP
VHUDFXPXODGDVSRUDWp
 2
2DWUDVR
DWUDVRLQMXVWLILFDGR
LQMXVWLILFDGRQR
QRLQtFLR
LQtFLRGD
GDREUD
REUDVHUYLoR
VHUYLoRRX
RXIRUQHFL D  GRLVSHUtRGRV
PHQWRFRQVWLWXLPRWLYRSDUD E  WUrVSHUtRGRV
D UHVROXomRGRFRQWUDWR F  TXDWURSHUtRGRV
E UHVLOLomRGRFRQWUDWR G  WRGDVDVDOWHUQDWLYDVHVWmRFRUUHWDV
F UHVFLVmRGRFRQWUDWR
G H[WLQomRGRFRQWUDWR  6HJXQGRD/HL&RPSOHPHQWDUQžDFRQWDJHPGR
6HJXQGRD/HL&RPSOHPHQWDUQžDFRQWDJHPGR
GHVHUYLoR
WHPSRGH
WHPSR VHUYLoRpIHLWD
IHLWDHP
HPGLDV
GLDVTXH
TXHVmRFRQYHUWLGRVHP
 6
6HJXQGR D /HL &RPSOHPHQWDU Qž  R FRQMXQWR GH DQRVFRQVLGHUDGRRDQRFRPRGH
DWULEXLo}HV
DWULEXLo}HV H UHVSRQVDELOLGDGHV
UHVSRQVDELOLGDGHV SUHYLVWDV
SUHYLVWDV QD
QD HVWUXWXUD D  WUH]HQWRVGHVHVVHQWDGLDV
RUJ
RUJDQL]DFLRQDO
DQL]DFLRQDO H FRPHWLGDV
FRPHWLGDV D XP
XP VHUYLGRU
VHUYLGRU S~EOLFR E  WUH]HQWRVGHVHWHQWDGLDV
GHQRPLQDVH F  WUH]HQWRVHVHVVHQWDHFLQFRGLDV
D  VHUYLoRS~EOLFR G  WRGDVDVDOWHUQDWLYDVHVWmRFRUUHWDV
E  FDUJRS~EOLFR
F  IXQomRS~EOLFR  6HJXQGR
6HJXQGR
6HJXQGR D /HL
/HL &RPSOHPHQWDU
&RPSOHPHQWDU Qž
Qž 
 VmRGHYHUHVGR
G  WRGDVDVDOWHUQDWLYDVHVWmRFRUUHWDV VHUYLGRU
D  H[HUFHUFRP]HORHGHGLFDomRVXDVDWULEXLo}HV
 6HJXQGR
6HJXQGR
6HJXQGRD/HL
/HL&RPSOHPHQWDU
&RPSOHPHQWDUQžQž
RV
RVFDUJRV
FDUJRVGHVWLQD E  FXPSULUDVRUGHQVVXSHULRUHVH[FHWRTXDQGRPDQLIHVWD
GRV
GRV H[FOXVLYDPHQWH
H[FOXVLYDPHQWH jV
jV DWULEXLo}HV
DWULEXLo}HV GH
GH GLUHomR
GLUHomR FKHILD H PHQWHLOHJDLV
DVVHVVRUDPHQWR
DVVHVVRUDPHQWRVmR
VmRGH
GHOLYUH
OLYUHQRPHDomR
QRPHDomRHH[RQHUDomR
H[RQHUDomRSHOD F  GHFODUDUVHVXVSHLWRRXLPSHGLGRQDVKLSyWHVHVSUHYLVWDV
HPOHLRXUHJXODPHQWR
DXWRULGDGHFRPSHWHQWHVmRFKDPDGRV
G  WRGDVDVDOWHUQDWLYDVHVWmRFRUUHWDV
D  FDUJRVGHSURYLPHQWRSRUPHUHFLPHQWR
E  FDUJRVHPFRPLVVmR
6HJXQGRD/HL
 6HJXQGR
6HJXQGR /HL&RPSOHPHQWDU
&RPSOHPHQWDUQžQž
GHL[DU
GHL[DUGH
GHSUDWLFDU
F  FDUJRVGHGLUHomR DWR
DWRQHFHVViULR
QHFHVViULRjDSXUDomR
DSXUDomRGH
GHLQIUDomR
LQIUDomRGLVFLSOLQDU
GLVFLSOLQDUUHWDUGDU
G  WRGDVDVDOWHUQDWLYDVHVWmRFRUUHWDV LQGHYLGDPHQWH
LQGHYLGDPHQWHDVXD
VXDSUiWLFD
SUiWLFDRX
RXGDU
GDUFDXVD
FDXVDjSUHVFULomR
SUHVFULomRHP
SURFHVVRGLVFLSOLQDUpFRQVLGHUDGDLQIUDomR

 $
$ DUW/&Qž
 $
$ DUW/Qž  '
' DUW/&Qž
 $
$ DUW,,/Qž  '
' DUW/&Qž
 '
' DUW,D/Qž  &
& DUW/&Qž
 &
& DUW†ž/Qž   $
$ DUW†ž/&Qž
 &
& DUW,9/Qž   &
& DUW†ž/&Qž
 %
% DUWž/&Qž  '
' DUW/&Qž
 %
% DUWž/&Qž  &
& DUW,,,/&Qž
&kPDUD/HJLVODWLYDGR'LVWULWR)HGHUDO
 'LU$
GPLQLVWUDWLYR

D  JUDYH  2SURFHVVRDGPLQLVWUDWLYRSRGHLQLFLDUVH
2SURFHVVRDGPLQLVWUDWLYRSRGHLQLFLDUVH
E  PpGLD D  GHRItFLR
F  OHYH E  DSHGLGRGHLQWHUHVVDGR
G  WRGDVDVDOWHUQDWLYDVHVWmRFRUUHWDV F  GHRItFLRRXDSHGLGRGHLQWHUHVVDGR
G  GHSODQRRXDH[LJrQFLDGHLQWHUHVVDGR
 6HJXQGR
6HJXQGR
6HJXQGR D /HL
/HL &RPSOHPHQWDU
&RPSOHPHQWDU Qž
Qž 
 SHUPDQHFH
SHUPDQHFH
ILOLDGR
ILOLDGRDR
DR5336')
5336')QD
QDTXDOLGDGH
TXDOLGDGHGH
GHVHJXUDGR
VHJXUDGRRVHUYLGRU  6HJXQGR
6HJXQGR
6HJXQGRD/HL
/HLQž
Qž
VmR
VmROHJLWLPDGRV
OHJLWLPDGRVFRPR
FRPRLQWHUHVVD
WLWXODUGHFDUJRHIHWLYRTXHHVWLYHU GRVQRSURFHVVRDGPLQLVWUDWLYR
D OLFHQFLDGRSDUDWUDWDUGHLQWHUHVVHVSDUWLFXODUHV D  SHVVRDVItVLFDVRXMXUtGLFDVTXHRLQLFLHPFRPRWLWXODUHV
E GXUDQWH R DIDVWDPHQWR SDUD R H[HUFtFLR GH PDQGDWR GHGLUHLWRVRXLQWHUHVVHVLQGLYLGXDLVRXQRH[HUFtFLRGR
HOHWLYR GLUHLWRGHUHSUHVHQWDomR
F GXUDQWH R DIDVWDPHQWR GR SDtV SRU FHVVmR RX OLFHQoD E  DTXHOHVTXHVHPWHUHPLQLFLDGRRSURFHVVRWrPGLUHLWRV
UHPXQHUDGD RXLQWHUHVVHVTXHSRVVDPVHUDIHWDGRVSHODGHFLVmRDVHU
G  WRGDVDVDOWHUQDWLYDVHVWmRFRUUHWDV DGRWDGDHDVRUJDQL]Do}HVHDVVRFLDo}HVUHSUHVHQWDWLYDV
QRWRFDQWHDGLUHLWRVHLQWHUHVVHVFROHWLYRV
 'H
'H
'H DFRUGR
DFRUGR FRP
FRP D /HL &RPSOHPHQWDU Qž  VmR F  DV SHVVRDV RX DV DVVRFLDo}HV OHJDOPHQWHFRQVWLWXtGDV
EHQHILFLiULRV
EHQHILFLiULRVGR
GR5336')
5336')QD
QDFRQGLomR
FRQGLomRGH
GHGHSHQGHQWH
GHSHQGHQWHGR TXDQWRDGLUHLWRVRXLQWHUHVVHVGLIXVRV
VHJXUDGR G  WRGDVDVDOWHUQDWLYDVHVWmRFRUUHWDV
D RVSDLVHLUPmRV
E RLUPmRQmRHPDQFLSDGRGHTXDOTXHUFRQGLomRPHQRU 6mRFDSD]HVSDUDILQVGHSURFHVVRDGPLQLVWUDWLYR
 6mRFDSD]HVSDUDILQVGHSURFHVVRDGPLQLVWUDWLYR
GHYLQWHHTXDWURRXLQYiOLGR D  RVPDLRUHVGHGH]RLWRDQRVUHVVDOYDGDSUHYLVmRHVSHFLDO
F RF{QMXJHDFRPSDQKHLUDRFRPSDQKHLURHRILOKRQmR HPDWRQRUPDWLYRSUySULR
HPDQFLSDGRGHTXDOTXHUFRQGLomRPHQRUGHYLQWHHXP E  RV PDLRUHV GH GH]HVVHLV DQRV UHVVDOYDGDSUHYLVmR
DQRVRXLQYiOLGR HVSHFLDOHPDWRQRUPDWLYRSUySULR
G  WRGDVDVDOWHUQDWLYDVHVWmRFRUUHWDV F  RVPHQRUHVGHYLQWHHXPH PDLRUHV GH GH]RLWRDQRV
UHVVDOYDGDSUHYLVmRHVSHFLDOHPDWRQRUPDWLYRSUySULR
 5H]D
5H]D
5H]DD/HL Qž
/HLQžTXH
TXH$GPLQLVWUDomR
$GPLQLVWUDomR3~EOLFD
3~EOLFDREHGH G  WRGDVDVDOWHUQDWLYDVHVWmRFRUUHWDV
FHUiGHQWUHRXWURVDRVSULQFtSLRVGD R 
D  OHJDOLGDGHILQDOLGDGHHPRWLYDomR  1mRSRGHPVHUREMHWRGHGHOHJDomR
1mRSRGHPVHUREMHWRGHGHOHJDomR
E  UD]RDELOLGDGHSURSRUFLRQDOLGDGHPRUDOLGDGHHDPSOD D  DHGLomRGHDWRVGHFDUiWHUQRUPDWLYR
GHIHVD E  DGHFLVmRGHUHFXUVRVDGPLQLVWUDWLYRV
F  FRQWUDGLWyULR VHJXUDQoD MXUtGLFD LQWHUHVVH S~EOLFRH F  DV PDWpULDV GH FRPSHWrQFLD H[FOXVLYD GR yUJmRRX
HILFLrQFLD DXWRULGDGH
G  WRGDVDVDOWHUQDWLYDVHVWmRFRUUHWDV G  WRGDVDVDOWHUQDWLYDVHVWmRFRUUHWDV

 3DU
3DUD
3DUD HIHLWR GD /HL Qž  D XQLGDGH GH DWXDomR  6HJXQGR
6HJXQGR
6HJXQGR D /HL
/HL Qž
Qž 
 R DWR
DWR GH
GH GHOHJDomR
GHOHJDomR H VXD
LQWHJUDQWH
LQWHJUDQWH GD HVWUXWXUD GD $GPLQLVWUDomR GLUHWD H GD UHYRJDomRGHYHUmRVHU
HVWUXWXUDGD$GPLQLVWUDomRLQGLUHWDGHQRPLQDVH D  SXEOLFDGRVQRPHLREXURFUiWLFR
D  yUJmR E  SXEOLFDGRVQRkPELWRIHGHUDO
E  DXWRULGDGH F  SXEOLFDGRVQRPHLRRILFLDO
F  HQWLGDGH G  WRGDVDVDOWHUQDWLYDVHVWmRFRUUHWDV
G  WRGDVDVDOWHUQDWLYDVHVWmRFRUUHWDV
 1R
1R
1R SURFHVVR
SURFHVVR DGPLQLVWUDWLYR
DGPLQLVWUDWLYR DV
DV SURYDV
SURYDV REWLGDV SRU PHLRV
 3DUD
3DUD
3DUD HIHLWR
HIHLWR GD
GD /HL
/HL Qž
Qž R
R VHUYLGRU
VHUYLGRU RX DJHQWH LOtFLWRVVmR
S~EOLFRGRWDGRGHSRGHUGHGHFLVmRGHQRPLQDVH D  DGPLVVtYHLV
D  yUJmR E  DFHLWiYHO
E  DXWRULGDGH F  LQDGPLVVtYHLV
F  HQWLGDGH G  WRGDVDVDOWHUQDWLYDVHVWmRFRUUHWDV
G  WRGDVDVDOWHUQDWLYDVHVWmRFRUUHWDV

 $
$ REMHWLYLGDGH
REMHWLYLGDGH QR DWHQGLPHQWR GR LQWHUHVVH S~EOLFR
YHGDGD D SURPRomR SHVVRDO GH DJHQWHV RX DXWRULGDGHV Caro Candidato,
VHUiREVHUYDGD
D  QRSURFHVVRDGPLQLVWUDWLYR Alguns erros de “digitação” podem ocorrer...
E  QDILQDOLGDGHDGPLQLVWUDWLYD Pensando nisso, colocamos ao lado de cada resposta
F  HVWUXWXUDDGPLQLVWUDWLYD (de Legislação) o texto legal referente à questão.
G  WRGDVDVDOWHUQDWLYDVHVWmRFRUUHWDV Assim, em caso de dúvida quanto à resposta, você
poderá conferir na apostila - de acordo com texto legal
indicado - qual a resposta correta (o que vale é o texto
da lei - sempre!!!)

 '
' DUWž/&Qž  &
& DUWž/
 &
& DUW/&Qž  '
' DUWž/Qž
 '
' DUWž/Qž  $
$ DUW/
 $
$ DUWž†ž,/  '
' DUW/
 %
% DUWž†ž,,,/  &
& DUW/Qž
 $
$ DUWž†~/  &
& DUW/
NOÇÕES DE ADMINISTRAÇÃO
- Princípio do coleguismo - ciência de que você e todos os
demais operadores do Direito querem a mesma coisa, realizar a
justiça.
- Princípio da diligência - agir com zelo e escrúpulo em todas
funções.
- Princípio do desinteresse - relegar a ambição pessoal para
buscar o interesse da justiça.
- Princípio da confiança - cada profissional de Direito é
dotado de atributos personalíssimos e intransferíveis, sendo
escolhido por causa deles, de forma que a relação estabelecida
entre aquele que busca o serviço e o profissional é de confiança.
- Princípio da fidelidade - Fidelidade à causa da justiça, aos
valores constitucionais, à verdade, à transparência.
Noções de Administração: Ética no - Princípio da independência profissional - a maior
autonomia no exercício da profissão do operador do Direito não
serviço público.
deve impedir o caráter ético.
- Princípio da reserva - deve-se guardar segredo sobre as
informações que acessa no exercício da profissão.
- Princípio da lealdade e da verdade - agir com boa-fé e de
Ética no Serviço Público forma correta, com lealdade processual.
- Princípio da discricionariedade - geralmente, o profissional
Sobre a ética relacionada ao serviço público, vale destacar do Direito é liberal, exercendo com boa autonomia sua
que, se a Ética, num sentido amplo, é composta por ao menos profissão.
dois elementos - a Moral e o Direito (justo); no caso da - Outros princípios éticos, como informação, solidariedade,
disciplina da Ética no Setor Público a expressão é adotada num cidadania, residência, localização, continuidade da profissão,
sentido estrito. Ética corresponde ao valor do justo, previsto no liberdade profissional, função social da profissão, severidade
Direito vigente, o qual é estabelecido com um olhar atento às consigo mesmo, defesa das prerrogativas, moderação e
prescrições da Moral para a vida social. Em outras palavras, tolerância.
quando se fala em ética no âmbito do Estado não se deve pensar Em suma, é necessária obediência aos princípios éticos que
apenas na Moral, mas sim em efetivas normas jurídicas que a regem a administração pública pelos servidores para uma boa
regulamentam, o que permite a aplicação de sanções. gestão pública e, consequentemente, manutenção da
A ética no serviço público corresponde ao valor de justiça sociedade. Dessa forma, a ética consolidada no ordenamento
previsto no Direito vigente, observando-se não só os valores jurídico e inerente ao Estado, garante a preservação dos
morais, mas efetivamente as normas jurídicas que regulam essa interesses da coletividade.
moral, o que permite aplicação de sanção.
Os valores éticos na função pública visam que o interesse da Quando falamos sobre ética pública, logo pensamos em
Administração Pública sejam mantidos em prol da sociedade, corrupção, extorsão, ineficiência, etc., mas na realidade o que
ou seja, há a separação do interesse privado do interesse devemos ter como ponto de referência em relação ao serviço
público pelo servidor público. público, ou na vida pública em geral, é que seja fixado um
Assim, a desobediência da ética no âmbito da Administração padrão a partir do qual possamos, em seguida julgar a atuação
Pública gera coação, já que regulamentado por norma jurídica. dos servidores públicos ou daqueles que estiverem envolvidos
Por exemplo, o desrespeito ao princípio da moralidade, na vida pública, entretanto não basta que haja padrão, tão
caracteriza improbidade administrativa e o servidor público somente, é necessário que esse padrão seja ético, acima de tudo.
pode sofrer sanção. O fundamento que precisa ser compreendido é que os
O Estado tem que se conduzir por valores éticos morais e padrões éticos dos servidores públicos advêm de sua própria
impostos pelo Direito, o que é feito através dos agentes públicos natureza, ou seja, de caráter público, e sua relação com o
que compõe a Administração Pública e mantém a estrutura da público. A questão da ética pública está diretamente
sociedade. relacionada aos princípios fundamentais, sendo estes
Segundo Nalini1, o princípio fundamental seria o de agir de comparados ao que chamamos no Direito, de "Norma
acordo com a ciência, se mantendo sempre atualizado, e de Fundamental", uma norma hipotética com premissas
acordo com a consciência, sabendo de seu dever ético; ideológicas e que deve reger tudo mais o que estiver
tomando-se como princípios específicos: relacionado ao comportamento do ser humano em seu meio
- Princípio da conduta ilibada - conduta irrepreensível na social, aliás, podemos invocar a Constituição Federal. Esta
vida pública e na vida particular. ampara os valores morais da boa conduta, a boa-fé acima de
- Princípio da dignidade e do decoro profissional - agir da tudo, como princípios básicos e essenciais a uma vida
melhor maneira esperada em sua profissão e fora dela, com equilibrada do cidadão na sociedade, lembrando inclusive o tão
técnica, justiça e discrição. citado, pelos gregos antigos, "bem viver".
- Princípio da incompatibilidade - não se deve acumular Outro ponto bastante controverso é a questão da
funções incompatíveis. impessoalidade. Ao contrário do que muitos pensam, o
- Princípio da correção profissional - atuação com funcionalismo público e seus servidores devem primar pela
transparência e em prol da justiça. questão da "impessoalidade", deixando claro que o termo é
sinônimo de "igualdade", esta sim é a questão chave e que eleva
o serviço público a níveis tão ineficazes, não se preza pela

1
NALINI, José Renato. Ética geral e profissional. 8. ed. São Paulo: Revista dos
Tribunais, 2011.

1
NOÇÕES DE ADMINISTRAÇÃO
igualdade. No ordenamento jurídico está claro e expresso, A mudança que se deseja na Administração pública implica
"todos são iguais perante a lei". numa gradativa, mas necessária "transformação cultura" dentro
E também a ideia de impessoalidade, supõe uma distinção da estrutura organizacional da Administração Pública, isto é,
entre aquilo que é público e aquilo que é privada (no sentido uma reavaliação e valorização das tradições, valores, hábitos,
do interesse pessoal), que gera portanto o grande conflito entre normas, etc., que nascem e se forma ao longo do tempo e que
os interesses privados acima dos interesses públicos. Podemos criam um determinado estilo de atuação no seio da
verificar abertamente nos meios de comunicação, seja pelo organização.
rádio, televisão, jornais e revistas, que este é um dos principais Conclui-se, assim, que a improbidade e a falta de ética que
problemas que cercam o setor público, afetando assim, a ética nascem nas máquinas administrativas devido ao terreno fértil
que deveria estar acima de seus interesses. encontrado devido à existência de governos autoritários,
Não podemos falar de ética, impessoalidade (sinônimo de governos regidos por políticos sem ética, sem critérios de justiça
igualdade), sem falar de moralidade. Esta também é um dos social e que, mesmo após o advento de regimes democrático,
principais valores que define a conduta ética, não só dos continuam contaminados pelo "vírus" dos interesses escusos
servidores públicos, mas de qualquer indivíduo. Invocando geralmente oriundos de sociedades dominadas por situações de
novamente o ordenamento jurídico podemos identificar que a pobreza e injustiça social, abala a confiança das instituições,
falta de respeito ao padrão moral, implica portanto, numa prejudica a eficácia das organizações, aumenta os custos,
violação dos direitos do cidadão, comprometendo inclusive, a compromete o bom uso dos recursos públicos e os resultados
existência dos valores dos bons costumes em uma sociedade. dos contratos firmados pela Administração Pública e ainda
A falta de ética na Administração Pública encontra terreno castiga cada vez mais a sociedade que sofre com a pobreza, com
fértil para se reproduzir, pois o comportamento de autoridades a miséria, a falta de sistema de saúde, de esgoto, habitação,
públicas estão longe de se basearem em princípios éticos e isto ocasionados pela falta de investimentos financeiros do
ocorre devido a falta de preparo dos funcionários, cultura Governo, porque os funcionários públicos priorizam seus
equivocada e especialmente, por falta de mecanismos de interesses pessoais em detrimento dos interesses sociais.
controle e responsabilização adequada dos atos antiéticos. Essa situação vergonhosa só terá um fim no dia em que a
A sociedade por sua vez, tem sua parcela de sociedade resolver lutar para exercer os seus direitos
responsabilidade nesta situação, pois não se mobilizam para respondendo positivamente o questionamento feito por Milton
exercer os seus direitos e impedir estes casos vergonhosos de Santos "há cidadãos neste país?" E poderemos responder em alto
abuso de poder por parte do Poder Público. Um dos motivos e bom som que " SIM. Há cidadão neste pais. E somos todos
para esta falta de mobilização social se dá, devido à falta de brasileiros.".
uma cultura cidadã, ou seja, a sociedade não exerce sua Finalizando, gostaríamos de destacar alguns pontos básicos,
cidadania. A cidadania Segundo Milton Santos " é como uma que julgamos essenciais para a boa conduta, um padrão ético,
lei", isto é, ela existe mas precisa ser descoberta, aprendida, impessoal e moralístico:
utilizada e reclamada e só evolui através de processos de luta. Podemos conceituar ética, também como sendo um padrão
Essa evolução surge quando o cidadão adquire esse status, ou de comportamento orientado pelos valores e princípio morais e
seja, quando passa a ter direitos sociais. A luta por esses direitos da dignidade humana.
garante um padrão de vida mais decente. O Estado, por sua vez, O ser humano possui diferentes valores e princípios e a
tenta refrear os impulsos sociais e desrespeitar os indivíduos, "quantidade" de valores e princípios atribuídos, determinam a
nessas situações a cidadania deve se valer contra ele, e imperar "qualidade" de um padrão de comportamento ético: Maior valor
através de cada pessoa. Porém Milton Santos questiona, se "há atribuído (bem), maior ética; Menor valor atribuído (bem),
cidadão neste pais"? Pois para ele desde o nascimento as menor ética.
pessoas herdam de seus pais e ao longa da vida e também da A cultura e a ética estão intrinsecamente ligadas. Não nos
sociedade, conceitos morais que vão sendo contestados referimos a palavra cultura como sendo a quantidade de
posteriormente com a formação de ideias de cada um, porém a conhecimento adquirido, mas sim a qualidade na medida em
maioria das pessoas não sabem se são ou não cidadãos. que esta pode ser usada em prol da função social, do bem-estar
A educação seria o mais forte instrumento na formação de e tudo mais que diz respeito ao bem maior do ser humano.
cidadão consciente para a construção de um futuro melhor. A falta de ética induz ao descumprimento das leis do
No âmbito Administrativo, funcionários mal capacitados e ordenamento jurídico.
sem princípios éticos que convivem todos os dias com mandos Em princípio as leis se baseiam nos princípios da dignidade
e desmandos, atos desonestos, corrupção e falta de ética humana, dos bons costumes e da boa-fé.
tendem a assimilar por este rol "cultural" de aproveitamento em Maior impessoalidade (igualdade), maior moralidade =
benefício próprio. melhor padrão de ética.
Se o Estado, que a princípio deve impor a ordem e o respeito
como regra de conduta para uma sociedade civilizada, é o Referências Bibliográficas:
primeiro a evidenciar o ato imoral, veem esta realidade como
uma razão, desculpa ou oportunidade para salvar-se, e, assim Ética no serviço público. Disponível em:
http://www.agricultura.gov.br/arq_editor/file/Ministerio/Comissao_de_Etica/E
sendo, através dos usos de sua atribuição pública. tica_no_servico_publico.pdf.
A consciência ética, como a educação e a cultura são
aprendidas pelo ser humano, assim, a ética na administração Questões
pública, pode e deve ser desenvolvida junto aos agentes
públicos ocasionando assim, uma mudança na administração 01. (Secretaria da Criança/DF – Técnico
pública que deve ser sentida pelo contribuinte que dela se Socioeducativo – FUNIVERSA/2015) Com relação à ética
utiliza diariamente, seja por meio da simplificação de no serviço público, assinale a alternativa correta.
procedimentos, isto é, a rapidez de respostas e qualidade dos (A) O trabalho desenvolvido pelo servidor público perante
serviços prestados, seja pela forma de agir e de contato entre o a comunidade deve ser entendido como acréscimo ao seu
cidadão e os funcionários públicos. próprio bem-estar.

2
NOÇÕES DE ADMINISTRAÇÃO
(B) A decisão entre o que é legal e o que é ilegal representa utilizar livremente os bens sem qualquer prestação de contas à
o elemento ético da conduta do servidor público. sociedade.
(C) A moralidade impõe a escolha da legalidade, ainda que No entendimento de Bresser-Pereira (2001),
seja em detrimento da finalidade dos atos. patrimonialismo significa “a incapacidade ou a relutância
(D) A função pública e a vida privada do servidor devem ser de o príncipe distinguir entre o patrimônio público e seus bens
mantidas constantemente afastadas. privados”.
(E) O servidor não pode omitir a verdade, exceto quando No patrimonialismo não existiam carreiras organizadas no
ela for contrária aos interesses da pessoa interessada ou da serviço público e nem se estabeleceu a divisão do trabalho. Os
Administração Pública. cargos eram todos de livre nomeação do soberano,
que os direcionava a parentes diretos e demais amigos da
02. (Prefeitura de Fortaleza/CE – Analista de família, concedendo-lhes parcelas de poder diferenciadas, de
Gestão – Prefeitura de Fortaleza/CE/2016) João é acordo com os seus critérios pessoais de confiança. Prática
estagiário da Prefeitura de Fortaleza e foi flagrado pelo seu frequente era a troca de favores por cargos públicos (neste caso
chefe acessando sítios impróprios em um dos computadores do não se tratava de parentes e amigos, mas de interesses políticos
órgão. Nessa situação, a atitude do estagiário: ou econômicos). Regra geral, quem detinha um cargo público
(A) não foi contra a ética no serviço público, porque João o considerava como um bem próprio de caráter hereditário
não é servidor público. (passava de geração para geração). Não havia divisão do
(B) foi indiferente em relação à ética do servidor público, trabalho; os cargos denominavam-se prebendas ou
porque João estava no intervalo do almoço. sinecuras, e quem os exercia gozava de status da nobreza real.
(C) foi contra a ética no serviço público, mesmo que João Nesse modelo de administração, o soberano era tratado
seja apenas estagiário. “como um deus”. Segundo Hobbies (apud Norberto Bobbio,
(D) não foi contra a ética, pois muitos estagiários acessam 1981) “o soberano é juiz da conduta de seu súdito, mas a
sítios impróprios em computadores do órgão em questão. conduta do soberano é julgada por ele próprio”... “se o soberano
não observar as leis naturais, ninguém poderá constrangê-lo à
03. (SEAP/GO – Técnico em Saúde – obediência; ninguém poderá puni-lo”.
SEGPLAN/GO/2016) Considerando os valores fundamentais O Estado era tido como propriedade do soberano,
do serviço público, relativos à ética, marque a opção incorreta. e o aparelho do Estado (a administração) funcionava como uma
(A) Tratar com educação os cidadãos que demandam os extensão de seu poder. Em face da não distinção entre o público
serviços do Estado. e o privado, a corrupção e o nepotismo foram traços
(B) Resistir a todas as pressões de superiores hierárquicos e marcantes desse tipo de administração.
de contratantes, que visem obter favores ou vantagens do Nesse período histórico, o Estado-Administração não
Estado. pensava de forma coletiva e não procurava prestar serviços à
(C) Ter a consciência de que seu trabalho é regido por população, que era relegada ao descaso. Consequentemente, o
princípios éticos: é bom para você e para a sociedade como um foco das ações não era o atendimento das necessidades sociais
todo. e nem o desenvolvimento da nação, e os benefícios oriundos do
(D) Ser eficiente e interessado na solução das necessidades Estado e da Administração não eram destinados ao povo,
do cidadão, dentro das normas da Instituição. mas para um pequeno grupo encabeçado pelo chefe do
(E) Nenhuma das alternativas. Executivo (o soberano).
A base desse poder absoluto estava na tradição vinculada
Respostas à pessoa do soberano, que contava com um forte aparato
administrativo direcionado à arrecadação de impostos, e com
01. A/02. C/03. E uma força militar para defender seu(s) território(s) e intimidar
os opositores. Não somente o soberano agia
arbitrariamente, mas também os seus auxiliares e
servidores, que atuavam baseados na forma de agir e nas
ordens diretas recebidas de seu superior. Apenas alguns traços
da tradição eram respeitados a fim de manter a ordem em seus
territórios.
Modelos de gestão pública. Os serviços públicos (se é que podemos chamá-los assim)
consistiam, basicamente, na segurança (proteção e defesa
contra invasores) e na justiça exercida pelo soberano (ou
alguém por ele designado) de forma discricionária, mas com
MODELOS TEÓRICOS DE ADMINISTRAÇÃO respeito a certas tradições, principalmente as de caráter
PÚBLICA: PATRIMONIALISTA, BUROCRÁTICO E religioso. Eventualmente, os bons súditos contavam com
GERENCIAL2 auxílio econômico em casos de necessidade.

MODELO PATRIMONIALISTA Pode-se resumir as principais características da


Mesmo de forma desorganizada, o patrimonialismo foi administração patrimonialista:
o primeiro modelo de administração do Estado. Nele
não havia distinção entre a administração de bens públicos • confusão entre a propriedade privada e a propriedade
e bens particulares: tudo que existia nos limites territoriais de pública;
seu “reinado” era tido como domínio do soberano, que podia • impermeabilidade à participação social-privada;
• endeusamento do soberano;

2
Paludo, Augustinho. Administração pública. – 3. ed. – Rio de Janeiro:
Elsevier, 2013.

3
NOÇÕES DE ADMINISTRAÇÃO
• corrupção e nepotismo; privada, regras legais e operacionais previamente definidas,
• caráter discricionário e arbitrário das decisões; reestruturação e reorientação da administração para atender ao
• ausência de carreiras administrativas; crescimento das demandas sociais e aos papéis econômicos da
• desorganização do Estado e da Administração; sociedade da época, juntamente com o conceito de
• cargos denominados prebendas ou sinecuras; racionalidade e eficiência administrativa no atendimento às
• descaso pelo cidadão e pelas demandas sociais; demandas da sociedade.
• poder oriundo da tradição/hereditariedade. Todos os estudos a respeito da burocracia nos levam ao
sociólogo alemão Max Weber, que no início do século XX
É certo que a mudança na forma de administrar o Estado escreveu suas obras a respeito da burocracia e das organizações
não ocorre rapidamente, nem se dá mediante expedição de ato burocráticas. Weber descreveu a burocracia como um tipo de
normativo afirmando que ela mudou, mas trata-se de um poder ou dominação baseado na lei, no Direito, em que a
processo de transformação que poderá levar anos ou décadas. divisão do trabalho é realizada de forma racional e de acordo
Quanto mais longínqua for a época referida, tanto com as normas preestabelecidas. A burocracia de Weber é uma
mais conterá as características acima elencadas. Na organização em que as consequências desejadas são
medida em que a Administração Pública se aproxima do século completamente previsíveis.
XIX, algumas dessas características irão desaparecer. Para Max Weber a burocracia era a “organização
Essa forma de administração patrimonialista vigorou nos por excelência”
Estados, de forma predominante, até a segunda metade do A Burocracia corresponde a uma instituição
século XIX, quando o surgimento de organizações de grande Administrativa (pública ou privada) cujos pilares são o
porte, o processo de industrialização e as demandas sociais caráter legal das normas e procedimentos, a racionalidade, a
emergentes forçaram os governos aadotar um novo modelo de formalidade etc. Caracterizava-se como uma forma superior
administração capaz de responder tanto aos anseios dos de organização capaz de realizar, de modo eficiente e em
comerciantes e industriais, quanto aos da sociedade em geral. grande escala, as atividades administrativas, através do
Em países como o Brasil, o Estado-Administração ainda teria a trabalho de muitos funcionários, organizado de maneira
missão de alavancar o processo de desenvolvimento nacional. racional. É um tipo de organização que proporciona a
profissionalização de seus funcionários, visto que se
MODELO BUROCRÁTICO orienta pelos princípios da competência e da meritocracia, não
Em face da desorganização do Estado em termos de se prestando a favores pessoais de indicação política (típicos do
prestação de serviços públicos e da ausência de um projeto de patrimonialismo).
desenvolvimento para a nação, aliadas à corrupção e ao Estudos recentes identificaram uma dupla
nepotismo comuns na área pública, um novo modelo de racionalidade nos escritos de Weber: a formal e a
administração se fazia necessário. Era preciso reestruturar e substantiva. Segundo Hermano Thiry-Cherques (2009), a
fortalecer a Administração Pública para que pudesse cumprir racionalidade formal é constituída pela calculabilidade e
suas novas funções. O surgimento das organizações de grande predicabilidade dos sistemas jurídico e econômico. No campo
porte, a pressão pelo atendimento de demandas sociais, o das organizações, a racionalidade formal está presente em
crescimento da burguesia comercial e industrial indicavam que aparelhos como o contábil e o burocrático. Implica regras,
o Estado liberal deveria ceder seu espaço a um Estado mais hierarquias, especialização, treinamento. A racionalidade
organizado e de cunho econômico. substantiva é relativa ao conteúdo dos fins operacionais dos
A Administração Pública burocrática surge na segunda sistemas legal, econômico e administrativo. Difere da formal
metade do século XIX, na época do Estado liberal, como forma por ter uma lógica estabelecida em função dos objetivos e não
de combater a corrupção e o nepotismo patrimonialista. dos processos.
Constituem princípios orientadores do seu desenvolvimento a A racionalidade formal é fria, abstrata e universal, e não
profissionalização, a ideia de carreira, a hierarquia funcional, a leva em conta as pessoas nem suas qualidades, pois deriva das
impessoalidade, o formalismo, em síntese: o poder racional- normas em geral (leis, regulamentos, regras específicas). A
legal. Os controles administrativos visando evitar a racionalidade substantiva é que está atrelada aos
corrupção e o nepotismo são sempre a priori. Parte-se resultados, aos objetivos.
de uma desconfiança prévia nos administradores públicos e nos O termo burocracia, referindo-se a “organizações
cidadãos que a eles dirigem demandas. Por isso são sempre burocráticas”, está atrelado às grandes organizações privadas
necessários controles rígidos dos processos, como, por exemplo, ou às administrações públicas. Segundo Peter Blau (1966), “a
na admissão de pessoal, nas compras e no atendimento a grande dimensão de uma organização e a grande complexidade
demandas (Pdrae, 1995). de suas responsabilidades produzem a burocracia”. Para Max
Por outro lado, o controle – a garantia do poder do Estado Weber (1966) “a administração burocrática significa,
– transforma-se na própria razão de ser do funcionário. Em fundamentalmente, o exercício da dominação baseada no
consequência, o Estado volta-se para si mesmo, perdendo a poder”.
noção de sua missão básica, que é servir à sociedade. A Na teoria de Max originava-se a definição de sociedade
qualidade fundamental da Administração Pública legal, racional ou burocrática, fundamentada em regras
burocrática é a efetividade no controle dos abusos; seu impessoais; no formalismo, na racionalidade, na definição dos
defeito, a ineficiência, a autorreferência e a incapacidade de meios e dos fins; na profissionalização do servidor público com
voltar-se para o serviço aos cidadãos. Este defeito, entretanto, carreira e hierarquia funcional. A garantia de que as normas
não se revelou determinante na época do surgimento da seriam cumpridas provinha da autoridade institucionalizada
Administração Pública burocrática porque os serviços do Estado pela lei, visto que o poder legal permite impor obrigações e
eram muito reduzidos. Nessa época, o Estado limitava-se a normas de conduta às pessoas, pois quem governa/administra
manter a ordem e administrar a justiça, a garantir os contratos detém o poder legítimo sobre seus subordinados. Assim, o
e a propriedade. comando derivado das normas legais é tido como legítimo pelos
A administração burocrática trouxe novos conceitos à subalternos.
Administração Pública: a separação entre a coisa pública e a

4
NOÇÕES DE ADMINISTRAÇÃO
A administração burocrática pública era voltada para si e a propriedade pessoal. Os dirigentes não são os donos do
mesma, perdendo a noção de sua missão básica de negócio, e o funcionário não pode se apossar do cargo, nem tê-
instrumento do Estado para servir à sociedade, e o controle lo como sua propriedade.
dos meios transformara-se na própria razão de ser da • Profissionalização dos funcionários: nas
administração – tanto é que a qualidade era conceituada como organizações burocráticas, a administração é profissional. O
a efetividade no controle dos abusos. funcionário burocrata é um especialista, e administrar é sua
Na verdade, Weber não conceituou a burocracia, profissão. Os membros da organização são profissionais
mas apresentou características/dimensões que a caracterizam. especializados nas atividades que exercem; recebem salários e
Assim, as organizações serão ou não burocráticas, se constroem sua carreira ao longo dos anos.
apresentarem características compatíveis com as • Previsibilidade de funcionamento: nas
dimensões/características que as qualificam. Richard organizações burocráticas, as normas e os regulamentos
H. Hall (1966), compilando vários autores, apresenta as escritos e exaustivos preveem antecipadamente as possíveis
seguintes dimensões da burocracia: “hierarquia de ocorrências e padronizam a execução das atividades – o que
autoridade; divisão do trabalho (baseado na especialização assegura a completa previsibilidade de comportamento de seus
funcional); competência técnica; normas de procedimento para membros.
atuação no cargo; normas que controlam o comportamento dos A teoria burocrática foi amplamente aceita na época e
empregados; autoridade limitada ao cargo; gratificação muitas de suas características foram vantajosas para as
diferencial por cargo; impessoalidade dos contatos pessoais; organizações: a racionalidade, a hierarquia, a existência de
separação entre propriedade e administração; ênfase nas regras claras e normas de conduta, a profissionalização, o poder
comunicações escritas; e disciplina racional”. legal. Uma vez aplicada, isso deveria ser suficiente para garantir
o cumprimento das regras, que, por sua vez, deveriam gerar
A burocracia apresenta as seguintes características eficiência. Para Robert Kaplan (1966), “o principal mérito da
principais: burocracia está na sua eficiência técnica, devido à ênfase que
dá a precisão, rapidez, controle técnico, continuidade,
• Caráter legal das normas: nas organizações discrição, e por suas ótimas quotas de produção”.
burocráticas, o poder decorre da norma legal. As normas e A administração burocrática era considerada superior às
regulamentos escritos de forma exaustiva definem, demais formas de administrar. Segundo Weber (1966), a fonte
antecipadamente, o seu funcionamento padronizado. principal da superioridade da administração burocrática reside
• Caráter formal das comunicações: nas no papel do conhecimento técnico que, através do
organizações burocráticas, a comunicação é realizada por desenvolvimento da moderna tecnologia e dos métodos
escrito e de forma exaustiva: ao mesmo tempo em que os textos econômicos na produção de bens, tornou-se totalmente
formais escritos asseguram a interpretação unívoca das indispensável, indiferente que o sistema seja capitalista ou
comunicações, a forma exaustiva compreende todas as áreas da socialista. A burocracia tem um papel central na sociedade
organização. como elemento fundamental de qualquer tipo de administração
• Caráter racional e divisão do trabalho: nas de massa. A burocracia é superior em saber, tanto o da técnica
organizações burocráticas, a divisão do trabalho é horizontal e quanto o dos fatos concretos, o que normalmente é privilégio
feita de forma racional, com vistas a assegurar a eficiência e o de empresa capitalista.
alcance dos objetivos; cada componente tem atuação restrita às No Brasil, a Administração Pública burocrática
tarefas vinculadas ao seu cargo, que, por sua vez, encontram-se contemplou duas fases: a primeira, denominada modelo
descritas de forma clara, precisa e exaustiva. clássico, corresponde ao período 1930-1945. Foram
• Hierarquia da autoridade: nas organizações características marcantes desse período: a ênfase na
burocráticas, a estrutura é vertical e com muitos níveis reforma dos meios e o autoritarismo. Foi um modelo de
hierárquicos; as chefias das áreas/departamentos/seções administração fechado e autorreferido, em que os fins não eram
seguem uma escala em que a autoridade de cargo inferior relevantes. O fim do Estado Novo marca o fim dessa fase. No
obedece à autoridade de cargo superior. As regras definem a período de transição já era objeto de debate a excessiva
forma de contato entre a autoridade inferior e a superior, a fim centralização da Administração Pública brasileira. No entanto,
de garantir a unidade de comando e evitar atritos de esses questionamentos somente levaram a ações concretas a
autoridade. partir do Governo JK (1956-1961), com a denominada
• Rotinas e procedimentos padronizados: nas administração para o desenvolvimento. A segunda fase
organizações burocráticas, o ocupante de cargo guia-se por estendeu-se até as vésperas da reforma gerencial, com ênfase
regras e normas técnicas claras e exaustivas, com vistas ao no desenvolvimento da nação.
alcance dos objetivos definidos pela organização. A burocracia brasileira, no entanto, não se desenvolveu de
• Impessoalidade nas relações: nas organizações forma unânime. Núcleos ótimos coexistiram com práticas
burocráticas, a contratação de servidores e a distribuição de clientelistas. Para Luciano Martins (1995), os altos escalões da
tarefas é feita de forma impessoal; as pessoas são consideradas Administração Pública seguiram essas normas e tornaram-se a
apenas pelos cargos ou funções que exercem, e os subordinados melhor burocracia estatal da América Latina; os escalões
não obedecem a uma determinada pessoa, mas ao ocupante do inferiores foram deixados ao critério clientelista de
cargo. recrutamento de pessoal por indicação e à manipulação
• Competência técnica e meritocracia: nas populista dos recursos públicos.
organizações burocráticas, a contratação de pessoal se dá Na medida em que as organizações burocráticas
mediante critérios racionais de competência ou classificação em ganham importância e seus administradores se
concursos (e não segundo critérios pessoais), e a promoção é fortalecem, eles tendem a retirar parte do poder dos
por mérito e baseada no desempenho. políticos, o que em termos de democracia poderia se tornar
• Especialização da administração: nas organizações um grave problema. Esses administradores burocratas, segundo
burocráticas, há uma clara distinção entre o público e o privado; Bresser-Pereira (2001), “tendem a controlá-la cada vez mais e
entre o político e o administrativo; entre a propriedade pública completamente”.

5
NOÇÕES DE ADMINISTRAÇÃO
Dois termos ainda merecem destaque quanto à burocracia: contrapartida direta. Enquanto o mercado controla a
insulamento burocrático e engolfamento social: no administração das empresas, a sociedade – por meio de
primeiro caso a burocracia se isola, se autoprotege, e não políticos eleitos – controla a Administração Pública. Enquanto
permite influências políticas ou sociais em suas decisões (a a administração de empresas está voltada para o lucro privado,
burocracia realiza seu trabalho técnico, sem interferências); no para a maximização dos interesses dos acionistas, esperando
segundo, a burocracia extrapola sua área de atuação e adentra que, através do mercado, o interesse coletivo seja atendido, a
o mundo político e social. Administração Pública gerencial está explícita e diretamente
voltada para o interesse público.
MODELO GERENCIAL Neste último ponto, como em muitos outros
O mundo mudou, a sociedade mudou e as pessoas (profissionalismo, impessoalidade etc.), a Administração
mudaram, assim como a economia das nações apresentou Pública gerencial não se diferencia da Administração Pública
grandes mudanças e tecnologias inusitadas surgiram. A burocrática. Na burocracia pública clássica existe uma noção
competitividade das nações, a eficiência na administração e a muito clara e forte do interesse público. A diferença, porém,
busca por resultados se tornaram palavras de ordem. O Estado está no entendimento do significado do interesse
político estava em crise e procurava redefinir seu papel. Era público, que não pode ser confundido com o interesse do
necessário que a Administração Pública também mudasse para próprio Estado. Para a Administração Pública burocrática, o
dar conta de atender às exigências da nova ordem mundial, e à interesse público é frequentemente identificado com a
expansão das funções econômicas e sociais do Estado. afirmação do poder do Estado. Ao atuarem sob este princípio,
A principal fonte desse item é o Plano Diretor de Reforma os administradores públicos terminam por direcionar uma parte
do Aparelho do Estado de 1995 – PDRAE. substancial das atividades e dos recursos do Estado para o
A Administração Pública gerencial emerge na segunda atendimento das necessidades da própria burocracia. O
metade do século XX, como resposta, de um lado, à expansão conteúdo das políticas públicas é relegado a um segundo plano.
das funções econômicas e sociais do Estado, e, de outro, ao A Administração Pública gerencial nega essa visão do interesse
desenvolvimento tecnológico e à globalização da economia público, relacionando-o com o interesse da coletividade e não
mundial, uma vez que ambos deixaram à mostra os problemas com o do aparato do Estado.
associados à adoção do modelo anterior. A eficiência da A Administração Pública gerencial vê o cidadão como
Administração Pública – a necessidade de reduzir custos e contribuinte de impostos e como cliente dos seus serviços. Os
aumentar a qualidade dos serviços, tendo o cidadão como resultados das ações do Estado são considerados bons – não
beneficiário – torna-se então essencial. A reforma do aparelho porque os processos administrativos estão sob controle e são
do Estado passa a ser orientada predominantemente pelos seguros, como quer a Administração Pública burocrática, mas
valores da eficiência e qualidade na prestação de serviços porque as necessidades do cidadão-cliente estão sendo
públicos e pelo desenvolvimento de uma cultura gerencial nas atendidas.
organizações. Uma visão realista da reconstrução do aparelho do Estado,
A Administração Pública gerencial constitui um avanço, e, em bases gerenciais, deve levar em conta a necessidade de
até certo ponto, um rompimento com a Administração Pública equacionar as assimetrias decorrentes da persistência de
burocrática. Isso não significa, entretanto, que negue todos os aspectos patrimonialistas na administração contemporânea,
seus princípios. Pelo contrário, a Administração Pública bem como dos excessos formais e anacronismos do modelo
gerencial está apoiada na anterior, da qual conserva burocrático tradicional. Para isso, é fundamental ter clara a
alguns de seus princípios fundamentais, como a admissão dinâmica da administração racional-legal ou burocrática. Não
segundo rígidos critérios de mérito, a existência de um sistema se trata simplesmente de descartá-la, mas sim de considerar os
estruturado e universal de remuneração, as carreiras, a aspectos em que está superada e as características que ainda se
avaliação constante de desempenho, o treinamento sistemático. mantêm válidas como formas de garantir efetividade à
A diferença fundamental está na forma de controle, que deixa Administração Pública. O modelo gerencial tornou-se realidade
de basear-se nos processos para concentrar-se nos resultados. no mundo desenvolvido quando, através da definição clara de
Na Administração Pública gerencial a estratégia volta-se: objetivos para cada unidade da administração, da
para a definição precisa dos objetivos que o administrador descentralização, da mudança de estruturas organizacionais e
público deverá atingir em sua unidade; para a garantia de da adoção de valores e de comportamentos modernos no
autonomia do administrador na gestão dos recursos humanos, interior do Estado, se revelou mais capaz de promover
materiais e financeiros que lhe forem colocados à disposição o aumento da qualidade e da eficiência dos serviços
para que possa atingir os objetivos contratados; para o controle sociais oferecidos pelo setor público.
ou cobrança a posteriori dos resultados; adicionalmente,
pratica-se a competição administrada no interior do próprio O Caderno Mare n° 01 menciona as principais
Estado, quando há a possibilidade de estabelecer concorrência características da Administração Pública gerencial
entre unidades internas. No plano da estrutura (também chamada de nova Administração Pública):
organizacional, a descentralização e a redução dos níveis • orientação da ação do Estado para o cidadão-usuário ou
hierárquicos tornam-se essenciais. Em suma, afirma-se que a cidadão-cliente;
Administração Pública deve ser permeável à maior participação • ênfase no controle dos resultados através dos contratos de
dos agentes privados e/ou das organizações da sociedade civil, gestão;
e deslocar a ênfase dos procedimentos (meios) para os • fortalecimento e aumento da autonomia da burocracia
resultados (fins). estatal, organizada em carreiras de Estado, e valorização do seu
A Administração Pública gerencial inspira-se na trabalho técnico e político de participar, juntamente com os
administração de empresas privadas, mas não pode ser políticos e a sociedade, da formulação e gestão das políticas
confundida com esta. Enquanto a receita das empresas públicas;
depende dos pagamentos que os clientes fazem livremente na • separação entre as secretarias formuladoras de políticas
compra de seus produtos e serviços, a receita do Estado deriva públicas, de caráter centralizado, e as unidades
de impostos, ou seja, de contribuições obrigatórias, sem descentralizadas, executoras dessas mesmas políticas;

6
NOÇÕES DE ADMINISTRAÇÃO
• distinção de dois tipos de unidades descentralizadas: as (C) baseia-se no princípio do mérito profissional e enfatiza
Agências Executivas, que realizam atividades exclusivas de a importância do cumprimento de regras e procedimentos
Estado, por definição monopolistas, e os serviços sociais e rígidos.
científicos de caráter competitivo, em que o poder de Estado (D) baseia-se no princípio do mérito profissional e atribui
não está envolvido; grau limitado de confiança aos servidores e políticos,
• transferência para o setor público não estatal dos serviços recomendando, para isso, o contrato de gestão.
sociais e científicos competitivos; (E) foi adotada em substituição à Administração
• adoção cumulativa dos mecanismos de controle social patrimonial, que distinguia o patrimônio público do patrimônio
direto para controlar as unidades descentralizadas: do contrato privado.
de gestão em que os indicadores de desempenho sejam
claramente definidos e os resultados medidos; e da formação 04. Julgue o item seguinte referente à evolução dos
de quase mercados em que ocorre a competição administrada; modelos de administração pública.
• terceirização das atividades auxiliares ou de apoio, que O modelo burocrático foi adotado por diversos países em
passam a ser licitadas competitivamente no mercado. substituição ao modelo patrimonialista de administração
pública, no qual o patrimônio público não se distinguia do
Questões privado
( ) Certo ( ) Errado
01. A administração pública gerencial surgida no final do
século passado tem como fundamento o pressuposto de que: 05. Julgue o item seguinte referente à evolução dos
(A) atividades regulares necessárias aos objetivos da modelos de administração pública.
estrutura governada são distribuídas de forma fixa como Nas gestões que adotaram os modelos gerenciais de
deveres oficiais; administração pública, os quais surgiram como uma fase de
(B) princípios da hierarquia dos postos e dos níveis de modernização do modelo burocrático, o Estado permaneceu
autoridade significam um sistema ordenado de subordinação, responsável pela formulação e execução de serviços prestados
com supervisão dos postos inferiores pelos superiores; à sociedade de forma direta
(C) autonomia na gestão de recursos humanos, materiais e ( ) Certo ( ) Errado
financeiros é necessária para colocar foco na qualidade e
produtividade do serviço público; 06. Entre as características do modelo de gestão
(D) autoridade se distribui de forma estável, sendo administrativa patrimonialista pode ser apontado, em uma
delimitada pelas normas relacionadas com os meios de coerção; análise crítica,
(E) pessoas que atuam na administração pública têm (A) a ausência de carreiras administrativas, bem assim de
qualificações previstas por um regulamento geral, e são clara distinção entre patrimônio público e privado.
empregadas somente por meio de concurso público. (B) o excesso de verticalização e padronização dos
procedimentos.
02. A Administração pública gerencial emergiu na segunda (C) a estrutura hierárquica inflexível, afastando a
metade do século passado como estratégia para tornar a gestão meritocracia e propiciando o abuso de poder pela autoridade
pública mais eficiente. A Administração pública gerencial central.
(A) propôs a redução dos custos transferindo ao Estado a (D) o apego exagerado às regras, privilegiando a forma em
execução de serviços privados e centralizando a tomada de detrimento do interesse do cidadão.
decisão. (E) a excessiva ênfase no conceito de supremacia do
(B) buscou organizar o serviço público por meio de sanções interesse público sobre o privado, colocando o administrado a
no caso de descumprimento das regras e procedimentos serviço do Estado e não o contrário.
estabelecidos para os servidores.
(C) diminuiu a morosidade na prestação dos serviços Respostas
públicos por meio do estabelecimento de regras e
procedimentos detalhados para cada etapa da implementação 01: C / 02: E / 03: C / 04: Certo /
das políticas públicas. 05: Errado / 06: A.
(D) aumentou a eficiência da gestão dos serviços públicos
ao estabelecer remuneração por desempenho para os servidores
que exercem suas funções de forma estritamente profissional,
respeitando o devido distanciamento do cidadão.
(E) atribuiu ao Estado o papel de regulador e delegou parte
da execução dos serviços públicos à Administração indireta, às
organizações sociais e à iniciativa privada. Gestão estratégica (planejamento
estratégico no serviço público).
03. A Administração pública burocrática
(A) caracteriza-se pelo controle rígido, exercido
prioritariamente por indicadores de gestão.
(B) baseia-se no princípio do mérito profissional e enfatiza Gestão Estratégica Governamental3
a definição de metas para a atuação dos servidores públicos e,
consequentemente, a sua progressão na carreira. Primeiramente, vamos tratar do que é planejar e do que é o
planejamento estratégico. Planejar é tornar presente o futuro.

3
Planejamento Estrategico GP. CONTEÚDOS INTRODUTÓRIOS AO SALAZAR, A. A IMPORTANCIA DO PLANEJAMENTO ESTRATÉGICO
PLANEJAMENTO ESTRATÉGICO GOVERNAMENTAL. 2009. GOVERNAMENTAL NO PROCESSO DE TRANSFORMAÇÃO DO ESTADO
PFEIFFER, Peter. Planejamento estratégico municipal no Brasil: uma nova HERDADO RUMO AO ESTADO NECESSÁRIO, 2012.
abordagem. Brasília: ENAP, Texto para Discussão n° 37, 2000.

7
NOÇÕES DE ADMINISTRAÇÃO
O Planejamento é um processo contínuo e dinâmico que engloba os seguintes itens: âmbito de atuação, macropolíticas,
consiste em um conjunto de ações intencionais, integradas, políticas funcionais, macroestratégias, macroobjetivos.
coordenadas e orientadas para tornar realidade um objetivo O planejamento estratégico é uma prática essencial na
futuro, de forma a possibilitar a tomada de decisões administração, seja ela pública ou privada, devido aos
antecipadamente. benefícios que a utilização desta ferramenta traz às
Todas as organizações, públicas ou privadas, com ou sem organizações. Entre elas pode-se destacar a elevação da
fins lucrativos devem pensar em um planejamento estratégico, eficiência, eficácia e efetividade da organização, pois contribui
principalmente, porque as mesmas estão à frente de um para evitar a desorganização das operações, bem como para o
mercado totalmente competitivo e inovador. Sendo assim, é aumento da racionalidade das decisões, reduzindo os riscos e
necessário entender a importância da estratégia neste aumentando as possibilidades de alcançar os objetivos
planejamento. O objetivo do planejamento é oferecer aos traçados.
gestores e suas equipes uma ferramenta de informações para a Para Sfeiffer (2000), o planejamento estratégico tem dois
tomada de decisão, ajudando-os a atuar de forma a antecipar propósitos: por um lado, pretende concentrar e direcionar as
as mudanças que ocorrem no mercado. forças existentes dentro de uma organização, de tal maneira
que todos os seus membros trabalhem com foco na mesma
PLANEJAMENTO ESTRATÉGICO direção; por outro lado, procura analisar o entorno da
organização, ou seja, o ambiente externo, e adaptá-la a ele,
Segundo Kluyver e Pearce, estratégia diz respeito a para que seja capaz de reagir adequadamente aos desafios que
posicionar uma organização para obtenção de vantagem tiver. A é que a organização conduza o processo de
competitiva. Já para Hitt, Ireland e Hoskisson, estratégia desenvolvimento para não ser conduzida por fatores externos e
consiste em um conjunto integrado e coordenado de não controláveis.
compromissos e ações definido para explorar competências
essenciais e obter vantagem competitiva. Quando definem uma PLANEJAMENTO ESTRATÉGICO GOVERNAMENTAL E OS
estratégia, as empresas escolhem alternativas para competir. MODELOS DE GESTÃO
Neste sentido, a estratégia definida indica o que a empresa
pretende e o que não pretende fazer. Ambos os autores têm a A evolução da administração pública brasileira deu-se por
mesma opinião quanto à estratégia, que é a de chegar à frente, intermédio de três modelos/paradigmas: Modelo
estar à frente no mercado e na competição. Para isto, segundo Patrimonialista. Modelo Burocrático e Modelo Gerencial. É
afirma Sloan Jr. citado por Ansoff, “o objetivo estratégico de interessante frisar que aquele posteriormente implantado não
uma empresa é obter um retorno sobre o seu capital; se em dizimou por completo o anterior. Assim, podemos explicar
algum caso particular o retorno a longo prazo não for melhor a coexistência dos três modelos, pois se sucederam no
satisfatório, então a deficiência deverá ser corrigida, ou a tempo, mas sem que qualquer uma delas tenha sido
atividade abandonada em troca de outra que ofereça inteiramente abandonada.
perspectivas mais favoráveis”. É necessário, portanto, haver Em outras palavras, ainda há resquícios da Administração
resultados mensuráveis no curto prazo, entretanto, em se Pública Patrimonialista e da Administração Burocrática nos dias
tratando de estratégia, é necessário que seja feito um de hoje. Ressaltamos que a Administração Gerencial não foi
planejamento de longo prazo não deixando de lado que este completamente implantada. Por exemplo, na Administração
estudo deva ser constantemente analisado e se necessário Patrimonialista não existia a clara separação do que era
modificado, pois nas organizações ocorrem contingências que patrimônio público e do que era privado. E infelizmente, é
requerem remodelações. Toda organização é dinâmica e, recorrente na mídia a notícia de que recursos e/ou serviços
segundo Morgan, ela é viva. Os conceitos preconizados pelos públicos foram utilizados pelos dirigentes políticos em benefício
teóricos clássicos reduziam a empresa a um organismo fechado próprio. Vale ressaltar que a má utilização de recursos públicos,
sem qualquer interação com o meio. Para ele “a visão dos hoje é um crime chamado peculato.
sistemas abertos modificou tudo isto, sugerindo que se deveria
sempre efetuar o processo de organização tendo-se em mente o Em resumo temos que:
ambiente”. Neste sentido, Ansoff destaca que nos últimos anos
as noções relacionadas à estratégia e a sua aplicabilidade tem - O estado patrimonialista (Modelo Patrimonialista)
se tornado muito utilizada nas obras de Administração e sua predominou no Brasil até a década de 30. No Patrimonialismo
aplicabilidade tem sido em todas as áreas de uma organização. o aparelho estatal nada mais era que uma extensão do poder
Sendo assim, o Planejamento Estratégico (PE) trata-se de do soberano. Portanto o modelo organizacional patrimonialista
uma técnica de organização que procura definir qual a melhor é aquele onde há confusão da vontade do soberano com a do
maneira (estratégia) de se atingir um objetivo. Estado, onde o detentor do poder age como senhor absoluto da
A metodologia do planejamento estratégico foi introduzida coisa pública e de seus súditos.
no Brasil na metade dos anos 60. A experiência brasileira de O termo patrimonialismo, de acordo com Schwartzman4, foi
planejamento se aprofunda durante o período militar. utilizado por Max Weber para caracterizar uma forma de
Sucessivos planos são formulados e implementados a partir de dominação política tradicional, na qual “a administração
1964 seguindo o estilo autoritário, centralizador e pública é exercida como patrimônio privado do chefe político”.
economicamente concentrador que caracterizou os governos Segundo Bresser Pereira (2009), a administração pública
militares. Philip Kotler (1975) um dos defensores de sua patrimonial é caracterizada, pela ausência da separação clara
utilização, o conceituou como uma metodologia gerencial que entre o patrimônio público e o privado. Nesse modelo não há o
permite estabelecer a direção a ser seguida pela organização, perfeito entendimento de que a coisa pública pertença à
visando maior grau de interação com o ambiente. A direção coletividade, neste caso tudo se destina ao soberano, este

4
SCHWARCZ, L. M.(org). História da Vida Privada do Brasil: contrastes da modernidade: na fronteira entre o público e o privado. São Paulo: Companhia das
intimidade contemporânea. GOMES, A. C. A política brasileira em busca da Letras, 1998.

8
NOÇÕES DE ADMINISTRAÇÃO
administra o patrimônio público como seu, tudo é res principis, - são as denominações do modelo organizacional que se
como já comentado, uma vez que há confusão entre o foca no controle dos resultados (portanto a posteriori), na
patrimônio público e o particular, ocorrendo a apropriação da utilização eficaz do patrimônio público, redução de gastos,
res pública para fins pessoais. Confunde-se a vontade do melhora na qualidade dos serviços públicos prestados e
soberano com a do Estado, portanto com a da própria satisfação do cidadão-cliente frente a esses serviços.
sociedade. “Algumas características básicas definem a administração
No Patrimonialismo era aceitável que o soberano usufruísse pública gerencial. É orientada para o cidadão e para a obtenção
do tesouro público, ou seja, que seus custos fossem bancados de resultados; pressupõe que os políticos e os funcionários
pelo “capital público” tanto com necessidades pessoais (como públicos são merecedores de um grau real ainda que limitado
suas roupas), quanto com obrigações estatais (como na de confiança; como estratégia, serve-se da descentralização e
construção de estradas). do incentivo à criatividade e à inovação; o instrumento
mediante o qual se faz o controle sobre os órgãos
- no final do século XIX, começaram a ser difundidas as descentralizados é o contrato de gestão.”
ideias weberianas de administração racional-legal ou De acordo com Bresser, expõe que o Brasil, no plano político
administração burocrática (Modelo Burocrático), que tendo o é um Estado democrático, e plano administrativo encontra-se
capitalismo industrial como plano de fundo, formou um novo entre burocrático e gerencial.
Estado autoritário e burocrático. A Reforma Gerencial surgiu nos anos 1980 como uma
Com o objetivo de manter o poder, o governo de Getúlio resposta ao Estado Social. A administração pública burocrática
Vargas estabeleceu mecanismos de controle econômico, para foi apropriada para o Estado Liberal do século XIX, quando
promover o crescimento industrial e controlar a crise carga tributária estava em torno de 5 % do PIB, e que se
econômica iniciada em 1929. Estabeleceu, também, limitava a exercer as funções de polícia e justiça. No Estado
mecanismos de controle de pessoal, material e financeiro, além Democrático Liberal o tamanho do estado continuou pequeno,
de normatizar e padronizar o serviço púbico5. No plano e a administração pública burocrática continuou a se aplicar.
econômico, a política de proteção às exportações de matérias- Até que, no Estado Social, a carga a carga tributária subiu para
primas foi mantida, no entanto, o Estado passou também a aproximadamente 40% do PIB.
coordenar as grandes empresas produtoras de bens e serviços, O grande aumento do tamanho do estado deixou clara a
e para dar suporte a este desenvolvimento, foram feitos ineficiência da administração pública burocrática. Abrindo o
diversos investimentos na infraestrutura, na oferta de energia caminho para a Reforma Gerencial, que ganhou impulso a
elétrica, transporte e na produção de aço (COSTA, 2008). partir dos anos 1980 na Grã-Bretanha, e, nos anos 1990, no
Cabe ressaltar a criação do Conselho Federal do Serviço Brasil.
Público Civil em 1936, que transformou-se no DASP O Estado Social só pode ser pensado e estabelecido porque
(Departamento Administrativo do Serviço Público) em 1938, a administração pública burocrática proporcionava um mínimo
esse órgão teve grande expressão até 1945. de eficiência que o tornava economicamente viável. No entanto,
As principais atribuições do DASP consistiam em: concurso à medida que as reformas sociais avançavam, foi ficando claro
para ingresso no serviço público, critérios gerais e uniformes que esse mínimo era insuficiente. O aumento do custo dos
para classificação de cargos, organização dos serviços de serviços do estado impôs a adoção da Reforma Gerencial. Esta
pessoal e de seu aperfeiçoamento sistemático, administração imposição, porém, não era apenas fiscal, mas também política.
orçamentária, padronização das compras do Estado, Para que o Estado Social se mantivesse legitimado em face à
racionalização geral de métodos.6 ofensiva neoliberal era necessário tornar suas ações
Nesse momento, de industrialização e normatização do substancialmente mais eficientes: era necessário proceder a
serviço público, as relações de trabalho no Brasil começam a se Reforma Gerencial.
transformarem. Primeiramente, o setor industrial começa a O Estado Social torna coletivo ou público a oferta dos
tomar forma e a classe operária a ganhar força. Depois, o grandes serviços públicos de educação, saúde, e previdência
ingresso no funcionalismo público, antes por mera indicação, social. Para que esses serviços fossem gratuitos e iguais para
demanda agora de uma ação administrativa mais especializada todos e economicamente viáveis, era preciso que o estado fosse
para promover o ingresso por concurso público. capaz de oferecê-los de forma não apenas efetiva, mas também
Na organização burocrática que o homem se dá conta de ser eficiente. A administração burocrática já se havia revelado
controlado pelas relações sociais e por sua relação com a efetiva; entretanto, na medida em que a dimensão dos serviços
máquina produtiva, o que os marxistas chamam de Modo de sociais do estado aumentava a efetividade não era suficiente:
Produção”. era preciso controlar o custo dos serviços realizados
Para Weber a Burocracia é o poder suficiente para que a diretamente por servidores públicos estatutários que se
estrutura organizacional funcione de forma eficaz. As revelavam altos demais. Estes custos refletiam não apenas o
atividades são desempenhadas na com seguindo o princípio da maior volume dos serviços, mas também a ineficiência neles
racionalidade. A liderança atua por meio de regras impessoais embutida devido à rigidez da administração burocrática.
e escritas numa estrutura hierárquica rígida. A ineficiência da administração pública burocrática tornou-
Etimologicamente, bureau vem do francês, significando se uma ameaça para a legitimidade do Estado Social. A
escritório, guichê; enquanto o sufixo grego krátos refere-se a ideologia neoliberal afirmava que a oferta e o consumo privado
governo; logo, a definição remete a governo de escritório, de eram mais eficientes do que a oferta pública e o consumo
técnicos. O conceito de burocracia weberiano é, deveras, coletivo. A sociedade continuava a demandar os serviços
diverso daquele conhecido popularmente, de modo pejorativo, públicos e a tese neoliberal de que o consumo público poderia
como excesso de formalidades desnecessárias, morosidade.

5
LIMA, Júnior. A reforma administrativa no Brasil: modelos, sucessos e 6
BRESSER PEREIRA, L. C. Do estado patrimonial ao gerencial. In Pinheiro,
fracassos. Revista do Serviço Público. Brasília, ano 49, n.2, p. 5-31, abr/jun. 1998. Wilheim e Sachs (orgs.), Brasil: Um Século de Transformações. S. Paulo: Cia das
Letras, 2001.

9
NOÇÕES DE ADMINISTRAÇÃO
ser substituído com vantagem pelo privado não foi aceita pela (4) mantém o consumo coletivo e gratuito, mas transfere a
sociedade. oferta dos serviços sociais e científicos para organizações
Era preciso mudar as condições de oferta dos serviços, e sociais, ou seja, para provedores públicos não estatais que
mostrar que o estado estava usando bem os recursos dos recebem recursos do estado e são controlados através de
impostos, que os contribuintes não estavam “jogando dinheiro contrato de gestão.
bom em cima de dinheiro ruim” – uma frase típica dos Por meio dessas características e, principalmente, da quarta,
oponentes do Estado Social. A Reforma Gerencial foi a resposta o poder público garante os direitos sociais, mas transfere sua
a esse desafio ao modificar a forma de administrar a oferta dos provisão ou oferta para organizações quase estatais que são as
serviços. organizações sociais. Uma característica central da Reforma
Esse modelo é a continuação do modelo burocrático Gerencial de 1995 – a reforma brasileira – foi a distinção entre
weberiano, apenas com a adoção das ideias atuais do setor atividades exclusivas do Estado, que envolvem poder de estado,
privado, exatamente como defendia Weber (a administração e as atividades não exclusivas que devem ser realizadas por
pública buscando na área empresarial novas dinâmicas de organizações públicas não estatais. Essas organizações sociais
funcionamento para garantir sua eficiência), desse modo garantem uma flexibilidade e uma eficiência administrativa
ocorreria meramente a flexibilização da burocracia, não um maior. Os resultados alcançados pelos novos hospitais do
novo modelo organizacional. Estado de São Paulo, todos constituídos sob a forma de
Segundo Martins7, anteriormente se percebiam as organizações sociais, são definitivos a respeito.
disfunções do modelo burocrático, que não atendia Em 1995 teve início no Brasil a Reforma da Gestão Pública
satisfatoriamente as novas demandas sociais e dificultavam o ou reforma gerencial do Estado com a publicação do Plano
desenvolvimento. Entretanto, a crise da década de 1980, Diretor da Reforma do Estado e o envio para o Congresso
especialmente a vivida pelo Reino Unido e Estados Unidos Nacional da emenda da administração pública que se
propiciaram as bases da reforma que serviria de resposta aos transformaria, em 1998, na Emenda 19. Nos primeiros quatro
problemas enfrentados à época: crise do petróleo, do welfare anos do governo Fernando Henrique, enquanto Luiz Carlos
state keynesiano; término do desenvolvimento pós-guerra; crise Bresser-Pereira foi o ministro, a reforma foi executada ao nível
fiscal, aumento nas demandas estatais. federal, no MARE - Ministério da Administração Federal e
No Brasil a administração gerencial foi implementada Reforma do Estado. Com a extinção do MARE, por sugestão do
abertamente no governo Fernando Henrique Cardoso (1994- próprio ministro no final desse período, a gestão passou para o
2002), pelo Ministro da Administração Federal e da Reforma Ministério do Planejamento e Gestão, ao mesmo tempo em que
do Estado (MARE), Luiz Carlos Bresser-Pereira, por meio do estados e municípios passavam também a fazer suas próprias
Plano Diretor da Reforma do Aparelho do Estado (1995). reformas.
Com inspiração na administração privada, buscando não O Brasil, ao iniciar em 1995 sua reforma da gestão pública,
ignorar as respectivas diferenças (pois o Estado não visa lucro foi o primeiro país em desenvolvimento que tomou essa
e mantém-se, não com receita de atividade comercial, mas pelo iniciativa, menos de dez anos depois que Inglaterra, Austrália e
pagamento de impostos, o modelo brasileiro primou por Nova Zelândia iniciaram suas reformas. Desde então a Reforma
capacitar melhor a burocracia, garantir contratos e adequado da Gestão Pública de 1995 vem avançando no país,
funcionamento dos mercados, mais autonomia ao principalmente ao nível dos estados e municípios. Como a
administrador público, cidadão visto como consumidor dos reforma da gestão pública é historicamente a segunda reforma
serviços estatais e maior controle social sobre o administrador administrativa relevante do Estado moderno, mais cedo ou
público. mais tarde ela ocorrerá em todos os países. E, uma vez iniciada,
Com a Emenda Constitucional (EC) 19/98, introduziu-se na não há alternativa senão prossegui-la.
CF/88 o princípio da eficiência, ampliando os princípios da
administração pública elencados no art. 37, além de diversas O objetivo da Reforma da Gestão Pública de 1995 é
outras alterações com o intuito de tornar mais célere e flexível contribuir para a formação no Brasil de um aparelho de Estado
a administração pública; tais como a exclusão da estabilidade forte e eficiente. Ela compreende três dimensões:
do servidor público, que agora pode ser demitido quando a) uma dimensão institucional-legal, voltada à
comprovadamente ineficiente ou exonerado caso haja excesso descentralização da estrutura organizacional do aparelho do
de pessoal; também pode ser colocado à disposição; houve a Estado através da criação de novos formatos organizacionais,
descentralização das atividades estatais entre outras mudanças como as agências executivas, regulatórias, e as organizações
e acréscimos8. sociais;
Foram tentativas de desburocratizar a administração b) uma dimensão gestão, definida pela maior autonomia e
pública brasileira, deixando-a menos rígida, com menos a introdução de três novas formas de responsabilização dos
controles a priori (voltados para os procedimentos) a fim de gestores – a administração por resultados, a competição
possibilitar mais foco nos resultados. Como Característica administrada por excelência, e o controle social – em
da Administração Gerencial: substituição parcial dos regulamentos rígidos, da supervisão e
(1) os gerentes são responsáveis por resultados, ao invés de da auditoria, que caracterizam a administração burocrática; e
obrigados a seguir regulamentos rígidos; c) uma dimensão cultural, de mudança de mentalidade,
(2) os servidores são premiados por bons resultados e visando passar da desconfiança generalizada que caracteriza a
punidos pelos maus; administração burocrática para uma confiança maior, ainda
(3) realiza serviços que envolvem poder de estado através que limitada, própria da administração gerencial.
de agências executivas e reguladoras; Um dos princípios fundamentais da Reforma de 1995 é o de
que o Estado, embora conservando e se possível ampliando sua

7
MARTINS, Humberto Falcão. A ética do patrimonialismo e a modernização <https://www.academia.edu/3261499/A_etica_do_patrimonialismo_ea_moder
da administração pública brasileira. In: MOTTA, F. C. P.; CALDAS, M. P. (Orgs.). nizacao_da_administracao_publica_brasileira>. Acesso em mar 2015.
Cultura organizacional e cultura brasileira. São Paulo: Atlas, 1997a. Disponível 8
BRESSER-PEREIRA, Luiz Carlos. Reforma do Estado para a cidadania: a
em: reforma gerencial brasileira na perspectiva internacional. 2 ed. São Paulo: Editora
34, 2011.

10
10
NOÇÕES DE ADMINISTRAÇÃO
ação na área social, só deve executar diretamente as tarefas que - melhoramento do desempenho organização: a reflexão
são exclusivas de Estado, que envolvem o emprego do poder de sobre as debilidades e as forças organizacionais ajuda a
Estado, ou que apliquem os recursos do Estado. diretoria a organização a desenvolver estruturas e
Entre as tarefas exclusivas de Estado devem-se distinguir as procedimentos mais adequados;
tarefas centralizadas de formulação e controle das políticas - melhoramento da comunicação interinstitucional de das
públicas e da lei, a serem executadas por secretarias ou relações públicas: missão, visão, estratégias e objetivos que
departamentos do Estado, das tarefas de execução, que devem foram elaborados conjuntamente pelos stakeholders orientam
ser descentralizadas para agências executivas e agências melhor todos os envolvidos na sua contribuição para o objetivo
reguladoras autônomas. Todos os demais serviços que a comum;
sociedade decide prover com os recursos dos impostos não - fortalecimento do apoio político: um Plano Estratégico que
devem ser realizados no âmbito da organização do Estado, por se baseia num amplo consenso usufrui uma legitimação mais
servidores públicos, mas devem ser contratados com terceiros. sólida e pode contar com um apoio suprapartidário mais amplo;
Os serviços sociais e científicos, para os quais os respectivos
mercados são particularmente imperfeitos, já que neles impera Os benefícios mencionados serão alcançados à medida que
a assimetria de informações, devem ser contratados com se apliquem os instrumentos de maneira séria e competente.
organizações públicas não estatais de serviço, as organizações Isso significa que tem que haver uma mudança de pensamento
sociais, enquanto que os demais podem ser contratados com e na ação dos responsáveis.
empresas privadas.
Estado é dar um passo adiante e tornar o Estado mais Apesar das vantagens citadas, existem razões para não se
eficiente e mais moderno. aplicar o PEG com mais frequência:
As três formas gerenciais de controle – controle social, - a organização não dispõe de recurso para realizar qualquer
controle de resultados e competição administrada – devem ser medida que seja considerada necessária pelo planejamento;
aplicadas tanto às agências, quanto às organizações sociais. A - ausência de capacidade e competência técnica mínimas
Reforma da Gestão Pública de 1995-98 não subestimou os para conduzir o processo;
elementos patrimonialistas e clientelistas ainda existentes em - os responsáveis políticos e administrativos da organização
um Estado como o brasileiro, mas, ao invés de continuar se não assumem o compromisso para levar o processo até sua
preocupando exclusivamente com eles, como fazia a reforma etapa final.
burocrática desde que foi iniciada nos anos 1930, avançou na
direção de uma administração mais autônoma e mais A abertura econômica possibilitou a introdução de métodos
responsabilizada perante a sociedade. Embora enfrentando de gerenciamento com conceitos como: qualidade, eficiência,
paralisações previsíveis, a Reforma da Gestão Pública de 1995 gestão empreendedora e planejamento estratégico, aplicados
está sendo bem sucedida em tornar gerencial o Estado tanto a empresas privadas quanto públicas. O PEG é um
brasileiro. Sua implementação deverá durar muitos anos como instrumental flexível capaz de apresentar soluções mais
nos outros países duraram as reformas burocráticas. rápidas, atendendo de forma mais eficiente, eficaz e com maior
efetividade às demandas de uma sociedade em constante
Entre as ferramentas de gestão da Nova Gestão Pública está mutação.
o Planejamento Estratégico Governamental (PEG) entendido Apesar das inúmeras vantagens do PEG a sua aplicação
como um processo permanente no qual o ambiente da ainda é muito restrita, pois a sua implantação necessita de
organização é observado e analisado, ações são planejadas, vontade política dos governantes. A situação do funcionalismo
executadas e os seus impactos são avaliados. O Planejamento nas administrações públicas mostra também que será um
ajuda a elaborar uma visão do futuro, a tomar as decisões grande desafio capacitar pessoal tecnicamente competente para
necessárias, a esboças as mudanças almejadas, a acompanhar participar do processo. Portanto formação e qualificação são
eficientemente o processo e a organizar a cooperação do indispensáveis para o sucesso do PEG. Se a filosofia básica do
diversos atores. planejamento estratégico – conduzir o processo de
desenvolvimento, em vez de correr atrás dele – for aplicada com
A iniciação do Planejamento Estratégico nas instituições entusiasmo e de forma correta haverá novas oportunidades
públicas, segundo Pfeiffer (2000), tem como expectativa as para aproveitar melhor os recursos e potenciais existentes.
seguintes mudanças:

- fortalecimento da competência: a organização é capaz de


cumprir com os compromissos que lhe foram atribuídos por
meio de um mandato de maneira mais rápida e melhor;
- aumento da eficiência: a organização alcança os mesmos
ou melhores resultados com uma menor aplicação de recursos;
- melhoramento da compreensão e da aprendizagem: a Qualidade no serviço público.
organização e os membros compreendem melhor a sua situação Excelência no serviço público.
e o seu ambiente. A aplicação sistemática de instrumentos de
gerenciamento lhes capacita a aprender melhor. Com isso cria-
se, por um lado, uma memória coletiva da organização e, por
outro, aumenta a capacidade de aprendizagem individual;
EXCELÊNCIA NOS SERVIÇOS PÚBLICOS9
- melhores decisões: as decisões que têm de ser tomadas
passam a ter mais consistência e uma linha mais clara se os
A excelência nos serviços públicos está ligada as melhorias
futuros impactos são suficientemente analisados;
acumuladas no decorrer dos processos de modernização, à

9
Paludo, Augustinho. Administração pública. – 3. ed. – Rio de Janeiro:
Elsevier, 2013.

111
NOÇÕES DE ADMINISTRAÇÃO
utilização de ferramentas da qualidade, à situação qualitativa resulta a dificuldade em padronizar serviços e
orçamentário financeira do Estado para custeio da prestação definir objetivamente indicadores de qualidade, visto que os
dos serviços e ao padrão de relacionamento entre o Estado e a cidadãos-clientes são diferentes e reagem de formas diferentes:
sociedade. o mesmo serviço pode ter qualidade para um
A excelência na prestação de serviços públicos cidadão-usuário e não ter para o outro.
corresponde ao grau máximo/ótimo dos serviços Nesse quesito, para avaliar o serviço prestado, é
prestados – quase impossível de ser atingido –, no entanto, essencial a pesquisa de satisfação, ou ao menos a
advoga-se ser possível e atribui-se aos programas de “caixinha de sugestão” a ser utilizada para a melhoria dos
qualidade a missão de atingir essa excelência. A serviços, e é necessário que haja certa flexibilidade
excelência corresponde a uma visão existente na Administração administrativa para adaptar os serviços às novas exigências do
Pública, segundo a qual ao se utilizar ferramentas e técnicas da cidadão-usuário. Para Marianella Fadel e Regis Filho (2009), “a
qualidade para promover melhorias contínuas relacionadas aos chave para assegurar uma boa qualidade de serviço é obtida
serviços oferecidos ao cidadão – o que inclui o treinamento e a quando as percepções dos clientes excedem suas expectativas”.
motivação dos servidores – se estará caminhando rumo à Mas o despertar para a importância de fornecer serviços com
excelência. qualidade não basta, “é preciso que haja um processo contínuo
de monitoração das percepções dos clientes sobre a qualidade
A reforma gerencial de 1995 incorporou a ideia da do serviço, identificação das causas das discrepâncias
qualidade e a definiu como “a satisfação das necessidades e encontradas e adoção de mecanismos adequados para a
expectativas do usuário-cidadão”: se ele está satisfeito, é sinal melhoria”.
de que os serviços têm qualidade, e se ele não está, é sinal de Na hora de promover melhorias no serviço público, alguns
que os serviços devem ser melhorados. Portanto, não é aspectos devem ser levados em consideração. As melhorias
“simplesmente a presença ou a ausência de alguma podem abranger: a diminuição de tempo de resposta a um
propriedade” que caracteriza a qualidade no serviço público, requerimento feito pelo usuário; a inclusão de preferências dos
mas o atendimento satisfatório dos cidadãos. usuários, através de pesquisas com eles; a simplificação dos
Mas tanto a qualidade quanto o usuário-cidadão são termos trâmites; a redução dos erros; a redução de custos (Lorigados;
abrangentes. Para Marianella Fadel e Regis Filho (2009), a Lima; Sanchez, 2003).
qualidade é definida pelo cliente e transcende os parâmetros A quantidade de vezes que um cidadão tem que ir/retornar
dos produtos, contemplando aspectos relacionados com o meio a um órgão público para resolver algo também é um indicador
ambiente e com as relações humanas. A função qualidade e o de que os serviços têm qualidade, e constitui item de melhoria.
termo cliente devem ser entendidos de forma abrangente e
devem ser estendidos, também, para as relações profissionais, Histórico da qualidade na Administração Pública
sociais e familiares. Em 1990, para incentivar a busca pela qualidade e a
Além de satisfazer o cliente-cidadão, a qualidade na redução de custos, Collor lançou o Programa Brasileiro da
Administração Pública visa à redução de custos e à melhoria Qualidade e Produtividade – PBQP. O objetivo do programa
contínua dos processos. No contexto do Pdrae, a qualidade tem era apoiar o esforço de modernização da empresa brasileira,
no processo o seu centro prático de ação, e compreende: através da promoção da qualidade e produtividade, com vistas
a definição clara dos clientes (internos ou externos) e dos a aumentar a competitividade dos bens e serviços produzidos
resultados esperados; a geração dos indicadores de no país. No entanto, o programa estava direcionado para a
desempenho; e a preocupação constante em fazer certo na iniciativa privada.
primeira vez, envolvendo todos os servidores com o Em maio de 1991 cria-se o subprograma Qualidade e
compromisso de satisfazer o cliente (Caderno Mare, no 04). Produtividade na Administração Pública, voltado para
a área pública. Este programa buscava a sensibilização
Atendimentos das para a qualidade no setor público, com ênfase em ferramentas
Principal ĺ e melhorias de processo. O programa estabeleceu uma meta
necessidades do cidadão
QUALIDADE para a Administração Pública: aumento de nível de satisfação
NO Também Redução do custo dos dos usuários de 10% ao ano, e meta geral de 70% de
ĺ
SETOR visa serviços satisfação dos usuários em 2002. Em 2004 (avaliação do PPA
PÚBLICO 2000-2003), constatou-se que os órgãos que aderiram ao
Também Melhora continua dos
ĺ programa obtiveram 81,3% de satisfação dos usuários,
visa processos
quanto aos serviços prestados.
Ainda em 1991, foi criada a Fundação para o Prêmio
Para Marcelo Coutinho (2000), a reforma gerencial utiliza Nacional da Qualidade. A primeira premiação privada ocorreu
a “inovação” a serviço da qualidade. Um exemplo de inovação já em 1992, mas a premiação pública somente foi
na prestação de serviços é o guichê único. Os postos de serviços instituída em 1998. Atualmente, a Fundação Nacional da
itinerantes e as ruas da cidadania também são formas de Qualidade – FNQ –(de Direito Privado) tem como missão
inovação, que trazem agilidade e comodidade no atendimento “disseminar os fundamentos da Excelência em Gestão para o
ao cidadão. Em qualquer situação, a tecnologia da informação aumento de competitividade das organizações e do Brasil”. A
estará presente como responsável pelos meios de viabilização FNQ é um órgão brasileiro, sem fins lucrativos, de estudos sobre
dos benefícios e melhorias em geral. excelência organizacional. Foi instituída em 11 de outubro de
A qualidade dos serviços é percebível, de forma 1991, por 39 organizações privadas e públicas. A Fundação
objetiva/tangível ou subjetiva/intangível: a primeira recebe contribuições anuais de empresas que são classificadas
percepção refere-se ao que pode ser visto e constatado de plano, em membros beneméritos e mantenedores.
é o aspecto físico do local (estrutura, equipamentos) e também Em 1995, o Plano Diretor da Reforma do Aparelho do
a vestimenta adequada das pessoas; a segunda percepção Estado apresentou dois programas como instrumentos de
refere-se à forma como o cliente é tratado, como a cortesia, administração da qualidade e da melhoria das práticas na
simpatia, prontidão, agilidade. Dessa dualidade da avaliação Administração Pública Federal: o Programa de Qualidade

12
12
NOÇÕES DE ADMINISTRAÇÃO
e Participação, de aplicação geral na Administração Pública ação do Estado. Sua principal missão era implantar a gestão
Federal; e o Programa de Reestruturação e Qualidade pública por resultados na Administração Pública brasileira, com
dos Ministérios, voltado a promover a readequação das vistas a transformar o setor público em benefício do cidadão.
estruturas administrativas dos Ministérios. Tinha dois objetivos gerais:
O Programa Qualidade e Participação na 1. apoiar as organizações públicas no processo de
Administração Pública foi instituído em 1996, como transformação gerencial, com ênfase na produção de resultados
instrumento básico da modernização da gestão pública, com positivos para a sociedade, na otimização dos custos
ênfase nos princípios da qualidade e da participação dos operacionais, na motivação e participação dos servidores, na
funcionários no nível operacional. Buscava não apenas delegação, na racionalidade no modo de fazer, na definição
uma mudança na forma de gestão, mas também na clara de objetivos e no controle dos resultados; e
cultura das organizações, no que diz respeito à cooperação 2. promover o controle social.
entre administradores e funcionários: buscava introduzir novos A atuação do Programa de Qualidade no Serviço Público
conceitos e técnicas de gestão pública, baseados no encontrava-se subdividida em três áreas:
desempenho, na redução ao mínimo dos erros e na participação
dos funcionários na definição dos processos de trabalho, com • Qualidade do Atendimento ao Cidadão: visa
vistas a uma maior qualidade dos serviços, dentro da filosofia orientar as organizações públicas brasileiras no
do erro zero e da maior cooperação entre funcionários e estabelecimento dos padrões de qualidade do atendimento ao
administradores. cidadão e na realização de pesquisa de satisfação do usuário
Os objetivos gerais deste programa eram dois: 1. dos serviços públicos;
Contribuir para a melhoria da qualidade dos serviços públicos,
por meio da institucionalização de seus princípios, com ênfase • Mobilização das Organizações Públicas
na participação dos servidores; e 2. Apoiar o processo de Brasileiras: procura estimular, orientar e apoiar as
mudança de uma cultura burocrática para uma cultura organizações públicas na implementação de ações de melhoria
gerencial, fortalecendo a delegação, o atendimento ao cidadão, baseadas no Modelo de Excelência em Gestão Pública;
a racionalidade no modo de fazer, a definição clara de
objetivos, a motivação dos servidores e o controle de resultados. • Avaliação e Melhoria da Gestão: efetua o
O programa QPAP tinha como princípios: satisfação do reconhecimento das organizações públicas brasileiras
cliente; envolvimento de todos os servidores; gestão engajadas no processo de melhoria contínua da gestão por meio
participativa; gerência de processos; valorização do servidor do Prêmio de Qualidade do Governo Federal.
público; constância de propósitos; e melhoria contínua.
A implantação da gestão da qualidade foi Em 2005, foi implantado o GesPública – Programa
considerada um fator crítico para o sucesso da Nacional de Gestão Pública e Desburocratização, através do
Reforma Gerencial do Estado. Os programas implantados Decreto n° 5.378 de 23 de fevereiro de 2005 e é o resultado da
nos Ministérios contaram com um comitê estratégico, composto evolução histórica de diversas iniciativas do Governo Federal
por autoridades responsáveis pelo planejamento estratégico e para a promoção da gestão pública de excelência, visando a
um grupo Técnico de Apoio encarregado de assessorar o contribuir para a qualidade dos serviços públicos prestados ao
comitê. A metodologia proposta previu o desenvolvimento de cidadão e para o aumento da competitividade do País.
quatro grandes produtos, como coloca Bresser-Pereira: Visto como uma política pública fundamentada em um
1° – estabelecimento de um marco referencial de exame modelo de gestão específico, o Programa tem como principais
e identificação de competências e objetivos de governo. Isso características o fato de ser essencialmente público – orientado
para delimitar o alcance e a natureza das mudanças a serem ao cidadão e respeitando os princípios constitucionais da
implementadas; impessoalidade, da legalidade, da moralidade, da publicidade
2° – a proposição de um arranjo organizacional, e da eficiência –, de ser contemporâneo – alinhado ao estado-
levando em conta o conjunto do Ministério, seus órgãos e da-arte da gestão –, de estar voltado para a disposição de
entidades vinculadas. É nesse momento que a missão, a visão resultados para a sociedade – com impactos na melhoria da
de futuro do setor e os objetivos estratégicos são formulados. É qualidade de vida e na geração do bem comum – e de ser
aqui também que as funções são identificadas e distribuídas federativo – com aplicação a toda a administração pública, em
entre as organizações do setor; consequentemente, a melhor todos os poderes e esferas do governo.
estrutura organizacional pode ser escolhida em cada caso; Por sua vez, o Modelo de Excelência em Gestão Pública
3° – a avaliação do modelo de gestão — do grau de (MEGP) representa a principal referência a ser seguida pelas
atendimento e dos critérios de excelência de gestão; instituições públicas que desejam aprimorar constantemente
4° – a formulação de um Plano de Reestruturação e seus níveis de gestão. Como todo modelo de gestão, o MEGP
Melhoria da Gestão, que é elaborado especificamente para contém diretrizes expressas em seus critérios de excelência
o Ministério, e que reflete todas as decisões tomadas nos passos gerencial (liderança, estratégias e planos, cidadãos, sociedade,
anteriores. informação e conhecimento, pessoas, processos e resultados),
técnicas e tecnologias para sua aplicação (como, por exemplo,
Em 1999, surge o Programa de Qualidade no Serviço a Carta de Serviços ao Cidadão, o Instrumento Padrão de
Público, que defendeu uma nova cultura dentro do Governo, Pesquisa de Satisfação, o Guia de Gestão de Processos, o Guia
justamente abordando os pontos mais importantes da ‘d’ Simplificação Administrativa e o Instrumento de Avaliação
Administração Pública gerencial: foco no usuário-cidadão, da Gestão) e práticas de gestão implantadas com sucesso. Para
gestão por resultados, inovação nos instrumentos gerenciais e que tanto o Modelo de Excelência em Gestão Pública quanto o
o envolvimento dos servidores de todos os níveis, visando à próprio Programa GesPública acompanhem a dinâmica da
contínua melhoria na prestação dos serviços. O PQSP tinha sociedade brasileira e estejam em conformidade com as
duas finalidades principais: a melhoria da gestão e o foco necessidades dos cidadãos, são fundamentais ações contínuas
em resultados — notadamente aqueles relativos à satisfação do de inovação do modelo, de sua comunicação e de garantia de
cidadão enquanto usuário de serviços públicos e destinatário da sua sustentabilidade.

133
NOÇÕES DE ADMINISTRAÇÃO
Nos últimos anos, o GesPública utilizou-se de uma competências institucionais, de forma a gerar valor para a
estratégia de sucesso alicerçada no trabalho voluntário de sociedade e atuar em conformidade com os parâmetros
representantes de instituições públicas, desenvolvendo e constitucionais e legais impostos à Administração Pública. A
divulgando conceitos e soluções para gestão, implantando e formulação e a implementação de políticas públicas, em
mobilizando núcleos regionais e setoriais nas unidades da qualquer esfera de governo, requer o entendimento da
federação e realizando avaliações do nível de gestão das realidade, a identificação das causas críticas e as formulações
instituições, seja por meio de auto avaliações, seja nos ciclos sobre produtos e resultados que permitam a otimização do uso
anuais do Prêmio Nacional da Gestão Pública. Destaque dos recursos públicos escassos disponíveis.
também foi dado a iniciativas relacionadas ao atendimento ao A Governança está relacionada com a capacidade e as
cidadão, tais como o desenvolvimento de estudos, guias e condições internas ao governo, para exercício de suas
eventos para as centrais de atendimento integrado e a recente competências e alcance de seus objetivos. Diz respeito aos
publicação do Decreto de Simplificação do Atendimento. recursos técnicos, tecnológicos, de infraestrutura, de pessoal,
Com o Decreto Presidencial que instituiu 2009 como o Ano entre outros de que dispõem as estruturas governamentais para
Nacional da Gestão Pública, a estratégia do Programa para formular, planejar e implantar as políticas públicas, assim como
cumprimento de sua missão foi ajustada de forma a facilitar a acompanhar, avaliar e fiscalizar a sua execução e resultados
construção coletiva de uma agenda de gestão e a adesão das obtidos. Sendo assim, é mais do que uma forma eficaz e
instituições aos princípios da Carta de Brasília, documento eficiente de executar o ‘negócio governo’. Está relacionada à
elaborado em conjunto pelo Ministério do Planejamento e pelo legalidade e legitimidade, sendo mais do que valores
CONSAD e que contém um conjunto de orientações para a estritamente empresariais. Governança pública é uma atividade
melhoria da gestão pública em nosso País. Em especial, foi complexa que envolve o ‘governo’ de complexas redes sociais
criado um fórum nacional permanente de discussão do tema, nos setores políticos.
foi lançado o Portal Nacional da Gestão Pública A qualidade da governança está relacionada, também, à
(www.gespublica.gov.br) e os processos de trabalho que capacidade do sistema de liderança do órgão ou entidade em
sustentam o GesPública passaram a ser a articulação, o fomento atuar de forma coesa e orientada para o alcance dos objetivos
e a mobilização de redes de gestão, com geração de institucionais, assim como de envolver e motivar todos os
conhecimento em gestão pública. servidores ou empregados do quadro de pessoal. Os líderes, na
Tais alterações vêm promovendo mudanças culturais no gestão pública de excelência, devem orientar e direcionar o
relacionamento entre o conjunto de instituições brasileiras, tais órgão ou entidade pública ao cumprimento de suas finalidades
como: a possibilidade de adesão ao GesPública de qualquer legais – é deles o papel de promover a compreensão interna e
pessoa física ou jurídica pelo uso de instrumentos de gestão ou externa sobre o papel institucional do órgão ou entidade e
mesmo pela simples participação nos fóruns de discussão garantir o seu desempenho, na estrita observância de suas
presentes no Portal da Gestão Pública; a ampliação das competências, observadas as orientações gerais e prioridades
ferramentas disponíveis e a avaliação de sua qualidade pelos de governo.
próprios integrantes da Rede Nacional de Gestão Pública; o Para tanto, é fundamental que a alta direção disponha e
estabelecimento de parcerias com especialistas em assuntos que utilize métodos de coordenação, articulação e supervisão para
vão desde a teoria de redes e de sistemas complexos até a promover a internalização de valores e princípios da
disposição de soluções de gestão; e, principalmente, a intensa administração pública; a unidade de objetivos e a ação
comunicação entre os participantes do movimento pela integrada; a gestão dos principais riscos presentes na área de
melhoria da gestão, por meio dos mais eficientes canais (vídeos, atuação do órgão ou entidade; a transparência; a participação
ensinos à distância, redes sociais e blogs de gestão). Em uma e o controle social e, especialmente, a eficiência administrativa.
visão mais ampla do Programa, iniciativas como a proposta de
contratualização do desempenho, a lei orgânica da São aspectos de excelência institucional,
administração, o desenvolvimento e a implantação de característicos da dimensão de governança:
indicadores de desempenho, a adequação da força de trabalho Ƚ) a formação e a gestão de líderes;
das organizações e as ações dos programas de cooperação Ⱦ) a estruturação do processo decisório de forma a favorecer
internacional para a execução da agenda federativa se integram a decisão célere, concertada
no cumprimento da missão de melhorar os serviços prestados à e voltada para a geração de valor social;
sociedade. ɖ) a prática institucional de monitoramento e avaliação
sistemáticos de seu desempenho, com base em indicadores,
As dimensões do Modelo de Excelência em Gestão com vistas ao contínuo reposicionamento do órgão ou entidade
Pública - MEGP e atualização da estratégia, a fim de melhor atender às
O MEGP organiza a gestão com alto desempenho demandas e aos desafios internos e externos.
institucional e excelência gerencial e subsidia a promoção da
melhoria da Gestão do órgão/entidade pública. A dimensão governança amplia o conceito de liderança,
adotado nas versões anteriores do MEGP - amplia as dimensões
Dimensão Governança e amplitudes do papel do órgão público e da sua capacidade de
Governança pode ser entendida como o exercício de exercer a governança pública no setor em que atua.
autoridade, controle, gerenciamento e poder de governo. É a
maneira pela qual o poder é exercício no gerenciamento dos Dimensão Estratégia e Planos
recursos econômicos, políticos e sociais para o desenvolvimento Uma gestão pública de excelência deve contemplar
do país. processos formais de formulação e implementação da
Está, portanto, relacionada à capacidade de implementação estratégia, fundamentados no exercício de pensar o futuro e
das políticas públicas, em seus aspectos políticos, técnicos, integrados ao processo decisório.
financeiros e gerenciais. A estratégia deve atender aos objetivos e dispor de metas e
Esta dimensão implica a capacidade institucional de planos articulados, para as unidades internas. Deve ser
orientar-se em direção ao cumprimento de sua finalidade e formulada a partir da prospecção dos resultados institucionais

14
14
NOÇÕES DE ADMINISTRAÇÃO
que se espera alcançar, considerados os recursos internos e ɂ. à observância dos direitos alcançados por públicos
externos; assim como os fatores intervenientes, especialmente específicos;
aqueles que possam representar riscos ou oportunidades ao ɔ. à análise dos resultados de auditorias internas e externas,
desempenho organizacional. para a identificação de riscos institucionais;
São requisitos de relevância para a excelência do processo ɀ. à observância interna do regime administrativo imposto
de definição e implementação da estratégia: à atuação institucional;
a. identificação de possíveis parcerias com agentes públicos Ʉ. à manutenção de mecanismos efetivos para a atuação
e privados e potenciais conflitos de atuação; pautada pela ética pública; e
b. o conhecimento dos pontos fortes e das oportunidades de ɇ. ao estímulo à sociedade à participação e ao controle
melhoria do próprio desempenho institucional; social.
c. as necessidades de investimento e inovação de forma a
atender requisitos específicos do setor em que atua; Dimensão Informação e Conhecimento
d. o alinhamento ao Plano Plurianual; A Dimensão representa a capacidade de gestão das
e. o desdobramento da estratégia em planos específicos que informações e do conhecimento, especialmente a
atendam às outras dimensões do sistema de gestão públicas; implementação de processos gerenciais que contribuam
tais como plano de tecnologia; capacitação; melhoria e diretamente para a seleção, coleta, armazenamento, utilização,
inovação da capacidade de gestão; gestão patrimonial; gestão atualização e disponibilização sistemática de informações
do conhecimento; gestão da comunicação; e atualizadas, precisas e seguras aos usuários internos e externos,
f. a vinculação da programação orçamentária à estratégia. com o apoio da tecnologia da informação.
As informações podem ser relativas ao desempenho global
Dimensão Público Alvo institucional, aos seus processos internos, especialmente aos
Esta dimensão refere-se às práticas gerenciais direcionadas finalísticos; aos públicos alvos; aos servidores e ao ambiente
ao relacionamento do órgão/entidade com a sociedade e externo, especialmente aos referenciais comparativos. O
abrange a imagem institucional, o conhecimento que a importante é que as informações estejam relacionadas a todas
sociedade tem do órgão ou entidade e a maneira como se as partes interessadas no desempenho institucional, bem como
relaciona com a sociedade e induz sua participação. abordem os aspectos relevantes da área de atuação do
O MEGP contempla, em sua Dimensão Público Alvo, órgão/entidade, intrínsecos ou relativos à estratégia.
práticas direcionadas, entre outras: Além disso, a gestão pública de excelência deve contemplar
a. à identificação e classificação dos públicos alvos e de suas a implementação de processos gerenciais que objetivem a
necessidades e expectativas; identificação, o desenvolvimento, a geração, a proteção e o
b. ao tratamento dessas necessidades e expectativas, compartilhamento do conhecimento.
inclusive no redesenho dos processos institucionais, para a
geração de resultados mais efetivos; Dimensão Pessoas
c. ao atendimento à Lei de Acesso à Informação; A excelência da gestão pública pressupõe sistemas de
d. à comunicação ao cidadão sobre os seus serviços e trabalho estruturados, que considerem as competências, os
padrões de atendimento (Carta ao Cidadão); requisitos técnicos, tecnológicos e logísticos necessários para a
e. à manutenção de canais de comunicação acessíveis e execução dos processos institucionais, de forma a cumprir as
adequados aos perfis de seus públicos alvos; finalidades do órgão ou entidade. Inclui as adequadas
f. ao gerenciamento da qualidade do atendimento ao estruturação e alocação de cargos efetivos, funções e cargos em
público; comissão; os padrões remuneratórios e a alocação interna. São
g. ao tratamento das solicitações, reclamações e sugestões; particularmente relevantes os investimentos em adequado
h. ao fortalecimento das relações com a sociedade, inclusive dimensionamento da força de trabalho; em gestão de
por meio de instrumentos de pesquisa, ausculta e concertação. competências institucionais e profissionais; e na estruturação
de sistemas de remuneração e de gestão do desempenho
O critério também analisa a imagem e o conhecimento sintonizados com os paradigmas do gerenciamento por
mútuo existente entre o órgão/entidade e o público. O processo resultados.
é uma proposta para democratizar a administração pública, Além disso, implica a existência de processos de capacitação
valorando a legitimidade, a participação da sociedade e as e desenvolvimento dos servidores e da garantia de um ambiente
consultas aos usuários cidadãos. de trabalho humanizado, seguro, saudável e promotor do bem-
estar, da satisfação e da motivação dos agentes públicos.
Dimensão Interesse Público e Cidadania
A Dimensão Interesse Público e Cidadania diz respeito à Dimensão Processos
observância do interesse público e ao regime administrativo e a A Gestão pública de excelência exige processos finalísticos
participação e o controle social. e de apoio adequadamente estruturados, a partir da estratégia
Um sistema de gestão pública de excelência contempla, institucional, com base nos recursos disponíveis, nos requisitos
nesta Dimensão, práticas direcionadas, entre outras: dos públicos alvos e nas possibilidades e limitações jurídico-
Ƚ. à identificação e avaliação dos principais aspectos sociais, legais.
econômicos e ambientais relacionados à atuação do órgão ou O monitoramento e o controle dos processos devem induzir
entidade; mecanismos de tratamento de não conformidades e
Ⱦ. à capacidade de prevenir ou mitigar impactos adversos implantação de ações corretivas, melhoria dos processos e
na sociedade ou no ambiente, decorrentes de sua atuação; incorporação de inovações, de forma a assegurar o permanente
ɖ. à identificação da necessidade de atualização ou alto desempenho institucional.
adequação da ordem constitucional/legal;
Ɂ. à conscientização das partes interessadas quanto à Entre os processos finalísticos e de apoio, dos órgãos e
responsabilidade social e ambiental e à sustentabilidade entidades públicos, destacam-se, pela relevância do
econômica; investimento e aperfeiçoamento, os seguintes:

155
NOÇÕES DE ADMINISTRAÇÃO
Ƚ. o atendimento ao público; 03. De acordo com o art. 2.º do Decreto n. o 5.378/2005,
Ⱦ. a gestão de parcerias com entidades civis; o GesPublica deve contemplar a formulação e a implementação
ɖ. a gestão financeira; de medidas integradas em agenda de transformações da gestão,
Ɂ. a gestão de suprimentos;
necessárias à promoção dos resultados preconizados no Plano
ɂ. a gestão do patrimônio público.
Plurianual, à consolidação da administração pública
Dimensão Resultados profissional voltada ao interesse do cidadão e à aplicação de
De nada adianta o investimento nas sete primeiras instrumentos e abordagens gerenciais. Essas ações têm como
dimensões se esse investimento não gerar os resultados objetivo
esperados para a sociedade, o mercado e o próprio setor
público. A Gestão orientada para Resultados é considerada uma (A) assegurar a efetividade da ação governamental por
poderosa ferramenta metodológica de monitoramento e
meio da reestruturação das instituições de controle.
avaliação das ações dos governos em sistemas políticos
democráticos. Avaliar os resultados obtidos nas ações de (B) promover a governança, aumentando a capacidade de
governo, respeitando as dimensões de eficiência, eficácia e formulação, implementação e avaliação de políticas públicas.
efetividade, permite aos agentes políticos estabelecer correções (C) promover a governabilidade, por meio de um modelo
nos rumos dos seus processos de trabalho, como também autocrático, participativo, transparente e ético.
propicia oportunidades de desenvolver estratégias de (D) promover a inclusão da agenda de políticas públicas do
acompanhamento aos cidadãos. Estado nas organizações do primeiro setor.
(E) reduzir o endividamento da administração indireta e
Questões
aprimorar a eficácia das ações governamentais.
01. Sobre o Decreto Federal n. 5.378/2005 é correto
afirmar, EXCETO: 04. Julgue os itens a seguir, acerca do modelo do
(A) Institui o Programa Nacional de Gestão Pública e GESPUBLICA.
Desburocratização - GESPÚBLICA e o Comitê Gestor do Os objetivos do GESPUBLICA incluem a eliminação do
Programa Nacional de Gestão Pública e Desburocratização.
déficit institucional, a promoção da governança de políticas
(B) Os critérios para avaliação da gestão de que trata este
Decreto são estabelecidos em consonância com o modelo do públicas, da eficácia e da efetividade da ação governamental
Programa Nacional de Controle da Qualidade. bem como a promoção da gestão democrática, participativa e
(C) O Comitê Gestor terá a seguinte composição, entre transparente.
outros, representantes de órgãos e entidades da administração ( ) Certo ( ) Errado
pública, assim como de entidades privadas com notório
engajamento em ações ligadas à qualidade da gestão e à 05. O Modelo de Excelência na Gestão Pública, também
desburocratização, conforme estabelecido pelo Ministro de
chamado de GESPÚBLICA, é sustentado por fundamentos que
Estado do Planejamento, Orçamento e Gestão.
(D) Ao Comitê Gestor compete, entre outros, reconhecer e servem de orientação aos gestores para a prestação de um
premiar os órgãos e entidades da administração pública, serviço público de qualidade. Um deles é a geração de valor,
participantes do GESPÚBLICA, que demonstrem qualidade em que significa:
gestão, medida pelos resultados institucionais obtidos. (A) o elemento promotor da gestão, responsável pela
orientação, estímulo e comprometimento com o alcance e a
02. O Decreto n° 5.378, de 23 de fevereiro de 2005, ao melhoria dos resultados organizacionais, atuando de forma
instituir o Programa Nacional de Gestão Pública e
inspiradora e motivadora para as pessoas.
Desburocratização (GESPUBLICA), teve como finalidade
contribuir para a melhoria da qualidade dos serviços públicos (B) a busca contínua e o alcance de novos patamares de
prestados aos cidadãos e para o aumento da competitividade conhecimento, individuais e coletivos, por meio da percepção,
do País. reflexão, avaliação e compartilhamento de informações e
Nessa linha de entendimento, para a consecução dos seus experiências.
diversos objetivos, é da competência do Comitê Gestor criado (C) o alcance de resultados consistentes que assegurem o
pelo respectivo Decreto
aumento de valor tangível e intangível de forma sustentada
(A) propor ao Ministro de Estado do Planejamento,
Orçamento e Gestão o planejamento estratégico do para todas as partes interessadas.
GESPÚBLICA. (D) a criação de um ambiente favorável à criatividade, à
(B) instituir tributos que possibilitem a obtenção de experimentação e à implementação de novas ideias que possam
recursos e demais meios para a execução das ações do gerar um diferencial para a atuação da organização.
GESPÚBLICA.
(C) encaminhar projeto de lei da LOA ao Congresso Respostas
Nacional, com a finalidade de descentralizar a gestão do
GESPÚBLICA.
(D) interferir nos resultados do GESPÚBLICA. 01: B / 02: A / 03: B / 04: Certo / 05: C.
(E) vetar o reconhecimento e premiação dos órgãos e
entidades da administração pública, participantes do
GESPÚBLICA, mesmo que demonstrem qualidade em gestão,
medida pelos resultados institucionais obtidos.

16
16
NOÇÕES DE ADMINISTRAÇÃO
- Agregar valor significa realizar operações estritamente
relacionadas com a elaboração do produto ou prestação do
serviço;

Depois de eliminados ou reduzidos ao mínimo


Conceitos de eficiência, eficácia e indispensável os desperdícios, o que resta é atividade ou esforço
efetividade aplicados à que agrega valor ao produto;
A eliminação de desperdícios diminui os custos de produção
Administração Pública.
sem que o valor do produto para o cliente fique comprometido.

Veja Palavras-chave da Eficiência: Ausência de desperdícios.


EFICIÊNCIA 10 Uso econômico de recursos. Menor quantidade de recursos para
produzir mais resultados. Fazer bem alguma tarefa. Utilização
dos recursos de forma produtiva, de maneira econômica; fazer
A eficiência mede a relação entre o custo (financeiro, melhor uso dos recursos disponíveis em uma organização.
material, humano) na realização das ações governamentais e
padrões de desempenho estabelecidos. EFICÁCIA
Eficiência é a relação entre os produtos/serviços gerados
(outputs) com os insumos utilizados, relacionando o que foi A eficácia está relacionada ao atingimento de um objetivo
entregue e o que foi consumido de recursos, usualmente sob a (que fora previamente determinado). Por exemplo: um ente da
forma de custos ou produtividade. Por exemplo: uma federação coloca como sua meta construir 10 mil casas
campanha de vacinação é mais eficiente quanto menor for o populares no período de 4 anos. Se ele conseguir realizar essa
custo, ou seja, quanto menor for o custo da campanha, meta, então ele cumpriu o requisito da eficácia. O problema é
mantendoǦse os objetivos propostos. Indicadores de eficiência que nem sempre o cumprimento de metas traz benefícios
podem ser encontrados na Carta de Serviços com seus sociais. Daí a importância da efetividade na avaliação dos
elementos de custos e em informações de sistemas programas governamentais.
estruturantes do Governo, como o SIAFI.
Assim, a eficiência de uma organização depende de como A eficácia:
seus recursos são utilizados, então eficiência significa: - Consiste em fazer a coisa certa (não necessariamente da
- Realizar atividades ou tarefas da maneira certa, sem erros e maneira certa).
atrasos; - Relacionada a atingir o objetivo
- Realizar tarefas de maneira inteligente, com o mínimo de - Se desejamos fazer algo e temos êxito, somos eficazes
esforço e com o melhor aproveitamento possível de recursos. - Cumprir a missão é chegar ao resultado
- É o conceito de desempenho que envolve a comparação entre
O princípio da eficiência é o da relação entre esforço e objetivos (desempenho esperado) e resultados (desempenho
resultado: Quanto menor o esforço necessário para produzir um realizado);
resultado, mais eficiente é o processo. Autores relacionam a - Significa o grau ou taxa de realização dos objetivos finais da
eficiência com a economicidade. organização.
Assim, mede a relação entre a quantidade de - Deve ser o objetivo prioritário para todas as organizações;
produto ou serviço e custo dos insumos envolvidos. - Mede aspectos do produto ou serviço final, em termos de
A eficiência é medida na entrada do processo. Refere- alcance de metas.
se a consumo de recursos. - Significa realizar as coisas certas, pontualmente, e com os
Quantos recursos consumimos em relação ao que requisitos de qualidade especificados.
propusemos a consumir? - A eficácia está na saída do processo. A sua medida é igual
a: output obtido.
O foco da eficiência está no(a): - Grau em que se alcançam os objetivos e as metas em um
- Método, no caminho, na forma de fazermos determinado período de tempo, sem levar em conta os
- Economia de meios, o menor consumo de recursos para custos.
atingir ao objetivo
No que diz respeito à Eficácia, Maximiano conclui que a
Já a antítese da eficiência é o desperdício, que eficácia é a relação entre resultados e objetivos. Não adianta
ocorre quando: muito produzir resultados de maneira eficiente, se não forem
- Mais recursos são usados do que os necessários para realizar os resultados corretos. A diferença entre eficiência e eficácia
um objetivo; pode ser ilustrada pela história das duas principais empresas
- Consomem-se recursos e nenhum objetivo é realizado; automobilísticas do mundo: Ford e General Motors.
- Produtos e serviços (objetivos) desnecessários são realizados. Embora Henry Ford fosse um mestre da eficiência, foi a GM
que se transformou na maior e mais bem-sucedida empresa do
Eliminar desperdícios significa: ramo. Esse desempenho é o resultado de sua orientação para o
- Reduzir ao mínimo atividade que não agrega valor ao mercado e não apenas para o processo produtivo. Enquanto a
produto ou serviço; Ford tinha uma estratégia de fazer eficientemente o mesmo

10
http://alipiofilho.blogspot.com.br/2012/08/eficacia-efetividade-eficiencia- 1518-4862, Teresina, ano 18, n. 3739, 26 set. 2013. Disponível
e.html. em: <https://jus.com.br/artigos/25399>. Acesso em: 24 fev. 2017.
ALVERGA, Carlos Frederico Rubino Polari de. Eficiência, eficácia, http://www.anpad.org.br/enanpad/2006/dwn/enanpad2006-apsa-1840.pdf
economicidade e efetividade: diferença prática. Revista Jus Navigandi, ISSN

177
NOÇÕES DE ADMINISTRAÇÃO
carro, a GM se orientou para fazer um carro para cada tipo de A efetividade, na área pública, avalia em que medida os
cliente. Portanto eficácia significa: resultados de uma ação trazem benefício à população. Ou seja,
- Grau de coincidência dos resultados em relação aos objetivos; ela é mais abrangente que a eficácia, na medida em que esta
- Capacidade de um sistema, processo, produto ou serviço de indica se o objetivo foi atingido, enquanto a efetividade mostra
resolver um problema; se aquele objetivo trouxe melhorias para a população visada.
- Fazer as coisas certas; Possui uma preocupação central de averiguar a real
- Sobrevivência. necessidade e oportunidade de determinadas ações estatais,
Para avaliar o grau de eficácia de um sistema, é necessário deixando claro que setores são beneficiados e em detrimento de
saber quais são os objetivos e quais os resultados de fato que outros atores sociais. Essa averiguação da necessidade e
alcançados. oportunidade deve ser a mais democrática, transparente e
responsável possível, buscando sintonizar e sensibilizar a
Vale ressaltar também que é possível ser eficaz sem população para a implementação das políticas públicas
ser eficiente, e ser eficiente sem ser eficaz. Ou ainda, é
possível também nem ser eficaz e nem ser eficiente, ou Veja a seguir estes termos relacionados:
sem ambos.
EFICIÊNCIA EFICÁCIA EFETIVIDADE
Segundo DAFT11, às vezes a eficiência conduz à eficácia. Em - Ênfase nos - Ênfase nos - Ênfase no
outras organizações, eficiência e eficácia não são relacionadas. meios resultados impacto
Uma organização pode ser altamente eficiente e não conseguir - Fazer - Fazer o que - Medir o
seus objetivos porque fornece um serviço para o qual não existe corretamente deve ser feito desempenho
demanda, por exemplo. De maneira análoga, uma organização - Utilizar - Capacidade de (efeito) das
pode alcançar suas metas de lucros, mas ser ineficiente, tendo produtivamente atingir objetivos metas atingidas e
muitos desperdícios, custos adicionais, desorganização na sua os recursos - Cumprir metas objetivos
forma de trabalhar... - Custo- - Realizar o que alcançados
benefício foi proposto - Transformar a
Veja a seguir: - Mínimo de situação
perdas e existente
desperdícios - Mudança e
desenvolvimento

Questões

01. (BADESC - Analista Administrativo – FGV)


Levando em consideração o uso de controles e indicadores de
produtividade em um programa de educação, o percentual de
crianças matriculadas e a avaliação da qualidade por meio de
exames nacionais são, respectivamente, exemplos de:
(A) eficácia e eficiência.
(B) eficiência e eficácia.
(C) efetividade e eficiência.
(D) efetividade e eficácia.
(E) eficácia e efetividade.

Fonte: Jorgenca, 201312. 02. (Emdec - Assistente Administrativo Jr –


IBFC/2016) Em Administração, utilizar da melhor forma
EFETIVIDADE todos os recursos disponíveis denomina-se:
(A) Eficácia.
Castro afirma que a literatura especializada incorporou um (B) Eficiência.
outro conceito, mais complexo que eficiência e eficácia. Trata- (C) Desempenho.
se da efetividade, que segundo o autor, é especialmente válida (D) Gerenciamento.
para a administração pública.
Efetividade é o impacto real que os objetivos cumpridos 03. (SEFAZ-RJ - Auditor Fiscal da Receita Estadual
causam no ambiente. É quando há um impacto no todo, uma - FGV) Julgue o próximo item:
mudança na realidade. Há autores que entendem como a Não raro, as pessoas confundem os termos eficiência,
junção de eficácia e eficiência. É a satisfação, o sucesso na eficácia e efetividade. A eficácia consiste em fazer a coisa certa
prática do que é feito e quando a qualidade é percebida pelo (não necessariamente da maneira certa). Assim, está
cidadão. É um conceito mais abrangente. relacionada ao grau de atingimento do objetivo. Se desejamos
Eficiência indica se o objetivo foi alcançado (relação fazer algo (ser aprovados em um concurso público, por
custo/benefício), enquanto a efetividade apresenta se aquele exemplo) e logramos êxito nesse projeto, somos eficazes. Dessa
objetivo acarretou melhorias para a população visada forma, evidencia o cumprimento da missão, chegar ao resultado
(qualidade do resultado)13. desejado.
( ) Certo ( ) Errado

11 13
DAFT, R. L. Teoria e Projetos das Organizações. 6.ed. Rio de janeiro: LTC, 1999. CASTRO, R. B. Eficácia, Eficiência e Efetividade na Administração Pública. In:
p.39-45. 30º Encontro da ANPAD. Salvador, 2006.
12
http://jorgenca.blogspot.com.br/2013/07/eficiencia-e-eficacia.html

18
18
NOÇÕES DE ADMINISTRAÇÃO
04. (SEFAZ/RJ - Fiscal de Rendas – FGV) Com relação completamente direcionados a resultados. Contudo, tudo
aos temas eficiência e eficácia, assinale a afirmativa incorreta. depende de como é feita a abordagem dentro da organização.
(A) Eficiência é um conceito limitado. Um método de gerenciamento do conhecimento pode ser
(B) Eficiência diz respeito aos trabalhos internos de uma implementado de forma a contribuir fortemente para os
organização. resultados, enquanto que a implementação de Balanced
(C) Uma organização não pode ser eficiente se não for Scorecard pode não levar a melhoria de resultados se não
eficaz. houver uma implementação adequada.
(D) A abordagem de metas para eficácia organizacional
identifica as metas de uma organização. Seguem abaixo alguns programas selecionados de uma lista
(E) Eficácia é um conceito abrangente. dada por McNamara (2002) e que são utilizados para
gerenciamento e melhoria do desempenho organizacional, com
Respostas uma pequena descrição a respeito de cada um deles.

01. E / 02. B / 03. Certo / 04. C -Balanced Scorecard: tem como objetivo interligar
controles operacionais de curto prazo aos objetivos de longo
prazo da estratégia da organização. É focado em quatro
indicadores: perspectiva do cliente, processos internos,
perspectiva financeira e de aprendizado e crescimento.

-Benchmarking: este método utiliza a comparação de


determinados indicadores com outras organizações de modo
Avaliação e mensuração do que se tenha uma perspectiva de nível de desempenho para este
desempenho. assunto em questão. Em geral, deve-se buscar referências
comparativas de outras empresas ou organizações que sejam
notadamente reconhecidas como sendo as melhores no assunto
em questão.
Desempenho Organizacional
-ISO-9000: a busca por uma certificação pode ser
considerada um processo de melhoria de desempenho,
A principal finalidade do desempenho organizacional é
principalmente no contexto brasileiro, pois o processo traz
garantir que todos os processos e subprocessos (processos,
algumas exigências como a padronização, o estabelecimento de
departamentos, colaboradores), assim como a organização em
indicadores, a realização de medições, que não são
si, trabalhem juntos para atingir os resultados pretendidos pela
culturalmente natas em nossas empresas.
organização. O gerenciamento do desempenho trabalha
conjuntamente neste aspecto, tendo o seu foco nos resultados e
-Learning Organization: o programa é focado no
redirecionando os esforços da organização para a efetividade.
aumento de valor dos sistemas da organização, incluídas as
Um fator que é muito importante é que muitas das
pessoas, de modo a aumentar a capacidade para desempenhar.
atividades que são feitas para melhoria do desempenho como
Utiliza os princípios da teoria de sistemas, e por si só não é um
análise de desempenho dos profissionais, horas e horas de
instrumento que garanta a melhoria de desempenho. A maneira
treinamento, o trabalho extenuante de planejamento e
como a organização aplica este aumento do conhecimento e de
orçamento, muitas vezes são feitas com o objetivo único de
capacidade na busca de melhoria dos resultados é que
fazê-las, não contribuindo para os resultados que são esperados
determina sua efetividade.
pela organização.
Segundo McNamara (2002), o gerenciamento do
-Planejamento Estratégico: estabelece para a
desempenho serve para nos lembrar que "estar ocupado" não é
organização, diretrizes, estratégias e objetivos. O planejamento
gerar resultados. Todas estas atividades sozinhas não
estratégico, apesar de por si só ser como o próprio nome diz um
significam resultados. O gerenciamento do desempenho tem
planejamento, a sua implementação, se adequada traz às
como foco produzir resultados e redirecionar os esforços da
empresas melhoria de desempenho.
organização do "estar ocupado" para a efetividade.
Para tanto, todos os desempenhos de processo, funcionais,
Outros programas para melhoria do desempenho
individuais, etc., devem estar alinhados apropriadamente, de
organizacional são sugeridos e aparecem a cada dia. No
modo que influenciem positivamente os resultados de
entanto, para todos os programas sugeridos, verifica-se que
desempenho globais.
existe a necessidade da utilização de uma abordagem adequada
Não é incomum vermos organizações com excelentes
durante a implementação. Para todos eles, o processo de
resultados e desempenho para os indicadores de processo, de
implementação é muitas vezes mais importante do que os
desempenho de seus colaboradores, de treinamento, terem
resultados implementados.
desempenhos globais pobres e comprometedores para a sua
sobrevivência e enriquecimento.
Avaliação do Desempenho Organizacional
Ferramentas de Gerenciamento e Melhoria do
A avaliação - como técnica administrativa - ajuda a
Desempenho Organizacional
melhorar a gestão porque produz informações necessárias para
identificar e entender as causas dos acertos e fracassos, os
Existem muitos métodos e programas para melhoria do
problemas dos desempenhos individual e coletivo, dentro de
desempenho organizacional. Alguns métodos não apresentam
um contexto de planejamento.
foco em resultado como por exemplo, gerenciamento do
conhecimento. Outros como Balanced Scorecard são

199
NOÇÕES DE ADMINISTRAÇÃO
O processo de avaliar uma empresa envolve o Os objetivos e metas devem ser de tal forma que
gerenciamento da atividade administrativa do negócio, na possibilitem à organização obter a melhoria dos seus processos.
inferência de expectativas do mercado, no impacto de eventos Para estabelecimento de objetivos, a organização deve levar
numa empresa, em avaliar estratégias, na mensuração de ativos em consideração alguns parâmetros:
na contabilidade de uma entidade, na escolha de investimentos, As metas e objetivos precisam ser factíveis: traçar objetivos
entre outras. Sendo assim, cabe um processo de controle e que a organização não possa atingir com os recursos disponíveis
avaliação constantes. para tal causa frustração e em geral as pessoas acabam
abandonando os programas.
Certo e Peter (1993) descrevem as características gerais do As metas e objetivos, por outro lado, não podem ser
processo de controle em três etapas: atingidos sem esforços de melhoria. Os objetivos devem ser
sempre um desafio para a conquista de melhorias.
"os administradores realmente controlam segundo três Utilizar o Benchmarking é uma das formas de obtenção de
etapas gerais: medindo o desempenho, comparando o metas e objetivos para a empresa, principalmente se a mesma
desempenho medido com os padrões e tomando a atitude ainda não tem dados históricos de desempenho para a
corretiva necessária para garantir que os eventos planejados característica em questão. Neste caso, a empresa pode utilizar
realmente se materializem". referências comparativas de empresas que possuem índices que
são notadamente um exemplo. São incluídas nesta categoria,
Utilizaremos esta abordagem, de modo que consideraremos empresas que receberam o Prêmio Nacional de Qualidade,
não somente uma medição realizada, mas o processo de empresas que foram pioneiras na implementação de práticas de
controle com as três etapas que definiremos abaixo: gestão reconhecidas como de excelência, etc. Uma observação
pertinente é que é sempre necessário buscar referências de
Etapa 1 – Medição da Característica empresas de mesmo porte da organização para que os objetivos
Etapa 2 – Comparação com os Objetivos Propostos sejam adequados.
Etapa 3 – Ação Corretiva ou realimentação. A busca de objetivos coerentes é tão importante quanto a
execução de medidas confiáveis, pois os mesmos representam
Para qualquer processo de controle, estas três etapas são de o caminho para o qual a organização deve seguir. Pior do que
fundamental importância, não sendo nenhuma delas mais ou ficar parado é seguir para o caminho errado ou até mesmo
menos importante que as demais, pois as mesmas estão achar que já chegou lá.
interligadas e a má definição ou execução de cada uma delas,
compromete o processo todo. Etapa 3 – Ação Corretiva ou realimentação.

Sendo assim, a melhoria de uma avaliação só pode ocorrer Uma ação corretiva é uma atuação ou efeito implementado
pela melhoria destas etapas do processo de controle. para eliminar as causas de uma não-conformidade, defeito ou
situação indesejável detectada, de forma a evitar a sua
Etapa 1 – Medição da Característica repetição. As ações corretivas podem envolver alterações em
processos, procedimentos ou sistemas, para obter a melhoria do
Tenha mente que a medição de uma característica que desempenho como um todo da organização. Deve-se depois
tenha influência sobre o desempenho de uma organização é retomar ao início, pois é um processo contínuo.
muitas vezes um processo complexo.
Os esforços organizacionais devem ser estabelecidos de Indicadores de desempenho
modo a gerar os resultados globais da organização. Portanto, a
organização necessita definir quais características dos seus sub- Uma das maiores contribuições dos autores matemáticos foi o
processos influenciam os resultados globais da organização. Em aporte de indicadores financeiros e não financeiros - quantificados
geral, os resultados globais são uma combinação dos resultados e objetivos – para medir, monitorar ou avaliar o desempenho
que se obtém nos sub-processos. organizacional - ou de parte dele -, como indicadores
Esta combinação não é simplesmente a soma ou uma departamentais, financeiros ou contábeis, de negócios, avaliação
combinação simples dos resultados parciais da organização. do desempenho humano, etc.14
Muitas vezes existem processos alternativos que a organização Os indicadores de desempenho15 são os sinais vitais de uma
têm que percorrer, em função de escassez de recursos (como organização, pois permitem mostrar o que ela está fazendo e quais
por exemplo, falta momentânea de uma matéria-prima), em os resultados de suas ações.
função de programações existentes (por exemplo, manutenção Um sistema de medição funciona como um painel de controle
preventiva de equipamentos) ou mesmo pela ocorrência de um para que a organização - ou cada departamento - possa avaliar
problema não esperado e não previsível. seu desempenho; é um modelo da realidade e pode assumir várias
formas, como relatórios periódicos, gráficos ou sistema de
Etapa 2 – Comparação com os Objetivos Propostos informação on line, etc.
O importante é que permita que o desempenho seja analisado
A definição dos objetivos para o desempenho e as ações corretivas sejam tomadas quando necessárias,
organizacional, passa pelas mesmas etapas da definição dos proporcionando, assim, constante monitoramento e avaliação.
indicadores e normalmente é feita em conjunto com este.
Principalmente para os objetivos funcionais, a existência de Vantagens de um sistema de medição a partir de
grupos formados por pessoas de várias áreas e processos é indicadores de desempenho:
requerida. a. Avaliar o desempenho e indicar as ações corretivas
necessárias.

14
Chiavenato, I. Introdução à Teoria Geral da Administração. Editora 15
Idem.
Elsevier, 7 ed., Rio de Janeiro, 2004.

20
20
NOÇÕES DE ADMINISTRAÇÃO
b. Apoiar a melhoria do desempenho. c. Fatores críticos de sucesso. Os aspectos
c. Manter a convergência de propósitos e a coerência de fundamentais para que a organização seja bem- -
esforços na organização por meio da integração de estratégias, sucedida em seus resultados ou em seu desempenho.
ações e medições.
6-Sigma
-Os indicadores nos permitem manter, mudar ou abortar o
rumo de nossas ações, de processos empresarias, de atividades, Sigma18 é uma medida de variação estatística. Quando
etc. São ferramentas de gestão ligadas ao monitoramento e aplicada a um processo organizacional, ela se refere à
auxiliam no desenvolvimento de qualquer tipo de empresa. Alto frequência com que determinada operação a transação, utiliza
desempenho atrai o sucesso; baixo desempenho leva à direção mais do que os recursos mínimos para satisfazer o cliente.
oposta. É possível medir e monitorar até mesmo coisas abstratas A maioria das organizações está no nível "4-sigma", o que
como, por exemplo, a satisfação. significa mais de 6 mil defeitos por 1 milhão de oportunidades.
-Os indicadores de desempenho servem para medir o grau Isso representa 6 mil produtos defeituosos em cada 1 milhão de
de “atingimento” de um objetivo ou de uma meta e, portanto, produtos fabricados. Uma organização que está no nível "6-
devem ser expressos em unidade de medidas que sejam as mais sigma" registra apenas três defeitos em 1 milhão. Isso se traduz
significativas àqueles que vão utilizá-los, seja para fins de em uma vantagem de custos e, mais importante, faz com que
avaliação ou para subsidiar a tomada de decisão com base nas sobrem recursos para serem dirigidos aos processos de
informações geradas. Dessa forma, as metas estabelecidas diferenciar uma empresa 6-sigma em relação às demais.
definirão a natureza dos indicadores de desempenho16. O programa 6-sigma utiliza várias técnicas em um metódico
processo passo a passo para atingir metas bem definidas. A
Um indicador é composto de um número ou principal diferença é que, com o 6-sigma, já não se busca
percentual - que indica a magnitude (quanto) - e de qualidade pela qualidade, mas se pretende aperfeiçoar todos os
uma unidade de medida - que dá ao número ou processos de uma organização.
percentual um significado (o que, quando).
6-sigma diferencia-se da Qualidade Total em
1) Indicadores Estratégicos: Informam o “quanto” a quatro áreas básicas:
organização se encontra na direção da consecução de sua visão; a. Maior amplitude da aplicação. A maior parte do
refletem o desempenho em relação aos fatores críticos para o TQM se aplica dentro da área de produção e manufatura, e não
êxito. no projeto, finanças, etc. O 6-sigma é para a organização toda.
2) Indicadores de Produtividade (eficiência): A Motorola afixa boletins de tempo de ciclo, dados de defeitos
medem a proporção de recursos consumidos com relação às e metas de melhoria nos refeitórios e banheiros.
saídas dos processos. b. Estrutura de implementação mais simples. Os
3) Indicadores de Qualidade (eficácia): focam as faixas-preta se dedicam, inteiramente, à mudança e ficam fora
medidas de satisfação dos clientes e as características do do cotidiano. A administração é premiada ou punida pela
produto/serviço. melhoria dos negócios.
4) Indicadores de Efetividade (impacto): focam as c. Ferramentas mais profundas. Além das
consequências dos produtos/serviços. Fazer a coisa certa da ferramentas do TQM, o 6-sigma se aprofunda para descrever a
maneira certa. situação atual e prever o futuro. A estatística é a base para uma
5) Indicadores de capacidade: medem a capacidade melhor compreensão de como os processos se comportam, um
de resposta de um processo através da relação entre as saídas software para auxiliar e um mapa para a aplicação das
produzidas por unidade de tempo. ferramentas, permitindo esclarecer os problemas e melhorar
6) Indicadores de prisma de performance: medem sua solução.
a satisfação dos stakeholders (partes interessadas), as d. Forte vinculação com a saúde (financeira) dos
estratégias implementadas, os processos críticos, as negócios. O 6-sigma aborda os objetivos da empresa e se
capacidades organizacionais e as contribuições dos certifica de que todas as áreas-chave para a saúde futura
stakeholders. O modelo de prisma de performance é uma medida daquela contêm mensurações quantificáveis com metas de
e um framework (quadro) inovador. Considerado um sistema melhoria e planos de aplicação detalhados.
de avaliação da gestão da organização - de segunda geração - Quantifica o que é necessário para atingir os objetivos
atua de duas maneiras: considerando o que quer e a financeiros da organização.
necessidade dos stakeholders e, excepcionalmente, o que a
organização quer e necessita dos seus stakeholders. Desta O 6-sigma busca a eficácia organizacional em três
maneira, o relacionamento é recíproco. dimensões que devem funcionar conjuntamente:
a. Redução do desperdício. Por meio do conceito de
O que medir?17 empreendimento enxuto (lean enterprise), ou esforço de tempo
futuro, redução do ciclo de tempo ou, ainda, eliminação do
As organizações utilizam medição, avaliação e desperdício do sistema, eliminação do que não tem valor para
controle de três áreas principais: o cliente, imprimindo velocidade à empresa.
a. Resultados. Isto é, os resultados concretos e finais que b. Redução dos defeitos. É o 6-sigma em si.
se pretende alcançar dentro de um determinado período de c. Envolvimento das pessoas. Por meio da chamada
tempo, como dia, semana, mês ou ano. "arquitetura humana".
b. Desempenho. O comportamento ou os meios
instrumentais que se pretende colocar em prática.

16
Texto adaptado do Centro de Referência para Apoio de Novos 17
Chiavenato, I. Introdução à Teoria Geral da Administração. Editora
empreendimentos – Cerne, 2011. Elsevier, 7 ed., Rio de Janeiro, 2004.
18
Idem.

211
NOÇÕES DE ADMINISTRAÇÃO
O Balanced Scorecard (BSC) 19 - Ser representativo.
- Fácil de entender. A facilidade para que qualquer um
O BSC é um método de administração direcionado ao chegue às suas conclusões a partir de um indicador é
equilíbrio organizacional e se baseia em quatro perspectivas fundamental para a sua utilidade.
básicas, a saber: Finanças, clientes, processos internos e - Testado no campo. Um indicador não tem valor até
aprendizagem/crescimento organizacional. que prove que realmente funciona.
- Econômico. Indicadores que dão trabalho para serem
a. Finanças. Para analisar o negócio sob o ponto de vista calculados não funcionam.
financeiro. Envolve os indicadores e medidas financeiras e - Disponível a tempo. Dados atrasados não representam
contábeis, que permitem avaliar o comportamento da mais a situação atual. Devem estar disponíveis antes que a
organização frente a itens como lucratividade, retorno sobre situação mude.
investimentos, valor agregado ao patrimônio e outros - Compatível. Ser compatível com os métodos de coleta
indicadores que a organização adote como relevantes para seu disponíveis.
negócio.
b. Clientes. Para analisar o negócio sob o ponto de vista Assim, os indicadores, de maneira geral, são compostos por
dos clientes. Inclui indicadores e medidas como satisfação, variáveis dos seguintes grupos: custo, tempo, quantidade
participação no mercado, tendências, retenção de clientes e e qualidade.
aquisição de clientes potenciais, bem como valor agregado aos As variáveis podem ser vistas segundo quatro dimensões:
produtos/serviços, posicionamento no mercado, nível de - economicidade - mede o custo dos recursos alocados
serviços agregados à comunidade com os quais os clientes, para determinada atividade. Refere-se à Operacionalidade ao
indiretamente, contribuem, etc. mínimo custo possível.
c. Processos internos. Para analisar o negócio sob o - eficiência - mede a relação entre a quantidade de
ponto de vista interno da organização. Inclui indicadores que produto ou serviço e custo dos insumos envolvidos. A eficiência
garantam a qualidade dos produtos e processos, assim analisa é medida na entrada do processo. Refere-se a consumo de
a inovação, a criatividade, a capacidade de produção, o recursos. Quantos recursos consumimos em relação ao que
alinhamento com as demandas, a logística e a otimização dos propusemos a consumir?
fluxos, assim como a qualidade das informações, da - eficácia - mede aspectos do produto ou serviço final, em
comunicação interna e das interfaces. termos de alcance de metas. Significa realizar as coisas certas,
d. Aprendizagem/crescimento organizacional. pontualmente, e com os requisitos de qualidade especificados.
Para analisar o negócio sob o ponto de vista daquilo que é A eficácia está na saída do processo. A sua medida é igual a:
básico para alcançar o futuro com sucesso. Considera as pessoas output obtido. Grau em que se alcançam os objetivos e as metas
em termos de capacidades, competências, motivação, em um determinado período de tempo, sem levar em conta os
empowerment, alinhamento e estrutura organizacional em custos.
termos de investimentos no seu futuro. - efetividade - mede os impactos dos produtos ou serviços
em determinado aspecto do ambiente organizacional. É a
O BSC busca estratégias e ações equilibradas em todas as satisfação, o sucesso na prática do que é feito. Simplificando,
áreas que afetam o negócio da organização como um todo, ser efetivo é realizar aquilo que foi feito (eficiência) da maneira
permitindo que os esforços sejam dirigidos para as áreas de certa (eficácia) e acarretar um impacto.
maior competência, detectando e indicando as que necessitam
de eliminação de incompetências. Questões
É um sistema focado no comportamento e não no controle.
Recentemente, o BSC passou a ser utilizado para criar 01. No que se refere à construção de indicadores de
organizações focadas na estratégia. desempenho, é correto afirmar que
Alinhamento e foco são as palavras de ordem. Aquele (A) os indicadores de efetividade (impacto) informam o quanto
significa coerência da organização; este, significa concentração. a organização se encontra na direção de alcançar a própria visão.
O BSC habilita a organização a direcionar suas equipes de (B) os indicadores estratégicos medem a proporção de recursos
executivos, unidades de negócios, recursos humanos, consumidos com relação às saídas dos processos.
tecnologia da informação e recursos financeiros para sua (C) esses indicadores são formas de representação quantificável
de características de produtos e processos, utilizados para
estratégia organizacional.
acompanhar e melhorar os resultados organizacionais ao longo do
O BSC constrói um contexto para que as decisões
tempo.
relacionadas com as operações cotidianas possam ser alinhadas (D) os indicadores de capacidade medem as consequências dos
com a estratégia e a visão organizacional, permitindo divulgar produtos/serviços (fazer a coisa certa da maneira certa).
a estratégia, promover o consenso e o espírito de equipe, (E) os indicadores de produtividade (eficiência) focam as
integrar as partes da organização e criar meios para envolver medidas de satisfação dos clientes e as características do
todos os programas do negócio, catalisar esforços e motivar as produto/serviço.
pessoas.
02. O método de melhoria contínua da qualidade que visa à
Características dos Indicadores de desempenho redução das variabilidades é denominado:
(A) kaizen;
Sendo assim, as características de um indicador podem ser (B) 5 "S";
resumidas da seguinte maneira: (C) diagrama de causa e efeito;
(D) reengenharia;
(E) seis sigma.

19
Ibidem.

22
22
NOÇÕES DE ADMINISTRAÇÃO
03. Levando em consideração o uso de controles e possuem uma mesma linha de ação com o objetivo de melhorar
indicadores de produtividade em um programa de educação, o a eficiência operacional da empresa.
percentual de crianças matriculadas e a avaliação da qualidade
por meio de exames nacionais são, respectivamente, exemplos Benefícios de uma estrutura adequada.
de: -Identificação das tarefas necessárias;
(A) eficácia e eficiência. -Organização das funções e responsabilidades;
(B) eficiência e eficácia. - Informações, recursos, e feedback aos empregados;
(C) efetividade e eficiência. - Medidas de desempenho compatíveis com os objetivos;
(D) efetividade e eficácia. - Condições motivadoras.
(E) eficácia e efetividade.
Estrutura:
04. “É a capacidade de um administrador para conseguir Toda empresa possui dois tipos de estrutura: Formal e
produtos mais elevados em relação aos insumos necessários informal.
para obtê-los”. Esse conceito está relacionado a: Formal: Deliberadamente planejada e formalmente
(A) eficácia. representada, em alguns aspectos pelo seu organograma.
(B) competitividade Ênfase a posições em termos de autoridades e
(C) eficiência. responsabilidades.
(D) competência. - É estável.
- Está sujeita a controle.
05. Assinale a alternativa que corresponde a um dos - Está na estrutura.
critérios básicos de avaliação de desempenho que está - Líder formal.
relacionado ao atingimento de um objetivo proposto:
(A) Competitividade Informal: Surge da interação social das pessoas, o que
(B) Eficiência. significa que se desenvolve espontaneamente quando as
(C) Eficácia. pessoas se reúnem. Representa relações que usualmente não
(D) Produtividade. aparecem no organograma.
São relacionamentos não-documentados e não-
Respostas reconhecidos oficialmente entre os membros de uma
organização que surgem inevitavelmente em decorrência das
01. C / 02. E / 03. E / 04. C / 05. C necessidades pessoais e grupais dos empregados.
- Está nas pessoas.
- Sempre existirão.
- A autoridade flui na maioria das vezes na horizontal.
- É instável.
- Não está sujeita a controle.
- Está sujeita aos sentimentos.
- Líder informal.
- Desenvolve sistemas e canais de comunicação.
Caracterização das organizações:
tipos de estruturas organizacionais. Vantagens da estrutura informal.
- Proporciona maior rapidez no processo.
- Complementa e estrutura formal.
- Reduz a carga de comunicação dos chefes.
- Motiva e integra as pessoas na empresa.
Estruturas organizacionais
Desvantagens:
- Desconhecimento das chefias.
Deve ser delineado de acordo com os objetivos e estratégias
- Dificuldade de controle.
estabelecidos pela empresa.
- Possibilidade de atritos entre pessoas.
É uma ferramenta básica para alcançar as situações
almejadas pela empresa.
Fatores que condicionam o aparecimento da
estrutura informal.
Conceitos de estrutura organizacional:
- Interesses comuns
Organização da empresa é a ordenação e o agrupamento de
- Interação provocada pela própria estrutura formal.
atividades e recursos, visando ao alcance de objetivos e
- Defeitos na estrutura formal.
resultados estabelecidos. Djalma, 2002, p. 84.
- Flutuação do pessoal dentro da empresa.
Forma pela qual as atividades de uma organização são
- Períodos de lazer.
divididas, organizadas e coordenadas. Stoner, 1992, p.230.
- Disputa do poder.
Podemos conceituar que a estrutura organizacional é forma
como a empresa se organiza internamente, como articula suas
A estrutura informal será bem utilizada quando:
atividades e seus negócios, em outras palavras, é a dinâmica de
- Os objetivos da empresa forem idênticos aos objetivos dos
uma empresa no mercado, como ela desenvolve suas
indivíduos.
capacidades para melhor atender o público interno
- Existir habilidade das pessoas em lidar com a estrutura
(colaboradores) e externo (clientes, fornecedores, acionistas,
informal.
investidores, etc).
A departamentalização é uma forma de sistematização da
estrutura organizacional que visa agrupar atividades que

233
NOÇÕES DE ADMINISTRAÇÃO
Tipos de Estruturas Organizacionais Formais Pouca especialização dos
Fácil implantação. líderes nas funções da
Vale dizer que a estrutura organizacional atende às organização.
exigências internas e externas das organizações.
Assim, todas as empresas, desde as que se dedicam à Estrutura Funcional
produção de bens ou à prestação de serviços, possuem uma
organização própria, específica e individual. - Agrupa pessoas que exercem funções em uma determinada
área na organização.
Podemos definir três tipos tradicionais de organização: - Especialização das funções.
-1. organização linear; - A estrutura funcional é adequada para pequenas e médias
-2. organização funcional; empresas. Quando há muito crescimento da empresa, torna-se
-3. organização linha-staff ineficaz a comunicação e o controle organizacional, e torna a
manutenção das diversas áreas muito dispendiosa.
Tipos de Estruturas Organizacionais tradicionais - Indicada para empresas estáveis.

Estrutura Linear, Militar ou Tipo Linha


Chefe
Executivo
- Chefia - fonte exclusiva de autoridade;
- as ordens seguem a via hierárquica;
- Cada empregado recebe ordens e se reporta
exclusivamente com um chefe imediato a ele; Marketing Administração Área técnica
- As comunicações entre órgãos são efetuadas
exclusivamente através das linhas no organograma;
- As decisões são centralizadas na cúpula da organização Gestão de Produção de
Publicidade
Pessoas Cadeiras

Presidência
Gestão de Produção de
Promoção
Materiais mesas
Vice-
-Presidência

Preço / Praça
Diretoria

Figura: Organograma de estrutura departamental


Funcional
Departamento Departamento
Vantagens e desvantagens da Estrutura
Funcional
Divisão Divisão Divisão Divisão
Vantagens Desvantagens
Maior especialização dos
Dificulta a
Setor Setor funcionários (sabem mais
interdisciplinaridade das
sobre suas respectivas
funções;
áreas);
Aumenta o
Seção Seção
relacionamento entre Diminui a visão global da
empregados dentro de um empresa;
mesmo setor;
Figura : Organograma de estrutura departamental Linear Autoridade baseada no Dificulta a tomada de decisão;
conhecimento (e não na Líderes têm menos poder
A Estrutura Linear apresenta as seguintes pura e simples hierárquico, o que leva à
vantagens e desvantagens: hierarquia). perda de autoridade.

Vantagens Desvantagens Estrutura Linha-Staff

Clara definição das A chefia centraliza as - Fusão das estruturas (funcional e da linear). Cada órgão
responsabilidades; decisões; se reporta a apenas um órgão superior, porém também recebe
assessoria e serviços especializados de diversos órgãos de staff;
As equipes são - Separação entre órgãos operacionais (executivos) e órgãos
Maior velocidade na
preparadas para seguir de apoio (assessores) - órgãos especializados aconselham os
tomada de decisões;
ordens, não para inovar; chefes de linha;
Estrutura facilmente Lentidão nas - conquanto haja duas fontes de autoridade, apenas a dos
compreendida comunicações; chefes das unidades de linha se projeta diretamente sobre cada
empregado.

24
24
NOÇÕES DE ADMINISTRAÇÃO
4. Por fases do processo (ou processamento).
5. Por projetos.
6. De matriz (matricial)
Diretoria
Geral
Estrutura por produtos ou serviços
Auditoria
Conselho Interna Esse tipo de estrutura é utilizado quando o agrupamento de
Executivo atividades é baseado nos produtos ou serviços que a empresa
oferta.
Todas as atividades requeridas para suprir um produto ou
Gerência Gerência de Gerência de Gerência de serviço, mesmo não sendo atividades similares devem estar
Financeira Produção Marketing RH agrupadas em um mesmo departamento. Esse tipo de estrutura
facilita em muito o emprego de tecnologia, dos maquinários e
equipamentos, do conhecimento, da mão de obra, de modo que
Figura: Organograma de estrutura departamental tipo todos os esforços ficam concentrados para aumentar a
Linha-Staff eficiência dos produtos que manuseia.
Há empresas que lidam somente com serviços, e, portanto
essa estrutura é denominada como estrutura por serviços, a
Vantagens e desvantagens da Estrutura Linha- única diferença é que o agrupamento das atividades tem o foco
Staff: voltado para os serviços ao invés dos produtos. É o caso dos
hospitais, que possuem divisão de departamentos conforme
Vantagens Desvantagens suas principais atividades oferecidas aos pacientes, tais como:
Mistura a estrutura Cirurgia, Radiologia, Pediatria, Entre outros.
A estrutura por produtos separa a estrutura organizacional
funcional (mantendo a
da empresa por unidades na base de produtos.
especialização) com a Pode haver conflitos entre a
linear (mantendo a área especializada (staff) e Estrutura por produtos
autoridade). Traz as os executores (linha);
vantagens desses dois tipos
de estrutura;
Altos custos de se manter
uma assessoria dentro da
Atividade conjunta entre empresa;
linha e staff. Pouca especialização de
quem realmente toma as
decisões (linha).

Outras formas de departamentalização


Fonte: Adaptado de Chiavenato, 2011.
A departamentalização é uma forma de sistematização da
estrutura organizacional que visa agrupar atividades que Vantagens e desvantagens da estrutura por
possuem uma mesma linha de ação com o objetivo de melhorar produtos ou serviços
a eficiência operacional da empresa. Dessa forma, a empresa
Como vantagens podem citar:
consegue juntar recursos, unidades e pessoas que tenham esse
-Estimula a responsabilidade dos colaboradores para um
ponto em comum. produto ou linha de produtos específicos;
Até aqui foram visualizadas as formas de -Maior especialização e domínio sobre as especificidades
departamentalização por meio das estruturas organizacionais dos produtos que trabalha;
mais tradicionais. -Facilita o processo de comunicação, pois os profissionais
Porém, existem outras formas, uma vez que os possuem os mesmos objetivos, com foco voltado para o
departamentos são as unidades de trabalho responsáveis por produto, sendo assim o fluxo de informações entre profissionais
contribui para o desempenho do produto.
uma função ou por um conjunto de funções. É um meio de se
-Contribui para o melhoramento do produto, os
obter homogeneidade de tarefas em cada órgão, escolher profissionais especializados podem trocar ideias, sugestões,
homogeneidade das atividades, agrupando os componentes da possíveis mudanças com maior agilidade e credibilidade;
organização em departamentos e divisões. -Essa estrutura é recomendada para empresas que lidam
com fatores externos incontroláveis, situações de
Entre outros tipos de departamentalização podemos citar: imprevisibilidade e mutação, pois neste caso os esforços já estão
voltados para o constante aperfeiçoamento de melhorias e
mudanças no produto.
1. Por produtos ou serviços.
-O foco voltado para o produto pode trazer maior satisfação
2. Por localização geográfica. por parte dos clientes.
3. Por clientes.

255
NOÇÕES DE ADMINISTRAÇÃO
As desvantagens são: -Quando o ambiente externo exige da empresa um
-Aumento dos custos operacionais, uma vez que exige um posicionamento mais estratégico por parte de suas atividades
número maior de chefias ou coordenadores, já que as áreas são locais esse tipo de estrutura torna-se substancial;
divididas por produtos e, portanto necessita de uma -Os profissionais podem aproveitar o desenvolvimento e
coordenação mais efetiva; economia local ou regional para investir em novos produtos ou
-Esse tipo de estrutura não é recomendado para empresas serviços e investir cada vez mais na região;
que lidam com poucos produtos no mercado, pois acarretaria -É mais indicada para empresas de varejo;
em custos operacionais elevados; -Permite reduzir alguns custos como impostos, taxas,
-Como lida com um mercado de instabilidades, ocasiona pedágios, etc.
certa insegurança frente aos colaboradores, que receiam em -Permite localizar-se mais próximo do cliente, facilitando o
situações de alta instabilidade externa, serem demitidos ou cumprimento de prazos e entregas.
mesmo terem uma desvalorização na carreira profissional;
-É necessário maior coordenação por conta da Quanto às desvantagens:
especialização, fazendo com que a empresa tenha com isso -O enfoque territorial pode ofuscar o enfoque nos aspectos
maiores encargos salariais; de planejamento, execução ou controle da organização como
-É necessário também investir em estrutura, pois a empresa um todo;
deve investir especialização dos profissionais para conseguir -Cada gestor que coordena os departamentos tomará
obter melhor desempenho do produto/serviço. decisões baseadas na realidade regional, neste caso a
organização está mais voltada para os aspectos regionais do que
Estrutura por localização geográfica para os aspectos internos da empresa, podem ser deixados de
lado os pontos fortes e fracos que precisam ser constantemente
É também chamado de estrutura territorial, esse tipo de revisados;
organização é mais indicado quando o agrupamento de -Se não for bem estudada, essa estratégia de
atividades de acordo com a localização do trabalho onde será departamentalizar por região ou território pode tornar-se um
desempenhada uma área de mercado da empresa. grande prejuízo para a empresa, há muitos casos que a empresa
A presunção implícita nessa estratégia é que, onde os precisou retornar suas atividades descentralizadas para matriz,
mercados estão dispersos, a eficiência pode ser melhorada, por falta de estratégia e rentabilidade.
desde que todas as atividades estejam agrupadas em uma
mesma região, ou local. Estrutura por clientes

Estrutura por localização geográfica Neste tipo de estrutura ocorre quando o agrupamento de
atividades é dividido conforme o segmento de clientes, de
acordo com o perfil de clientes que são atendidos.
Divisões de
Operações O perfil do cliente é um aspecto muito importante e deve
ser levado em consideração, questões como as necessidades,
preferências, exigências e desejos dos clientes precisa ser uma
preocupação constante.
Região Região
Centro-Sul Norte
É uma forma de parceria entre o cliente e a empresa
fornecedora, pois ambas possuem benefícios, uma vez que o
cliente terá prioridade frente à empresa, pois terá profissionais
especializados em cuidar somente de suas necessidades
Agência São Agência Agência Agência específicas, portanto oferecendo maior responsabilidade,
Paulo Brasília Manaus Belém
cumprimento de prazos, cumprimento nas entregas, acordo
para melhores condições de pagamento e prazos. Bem como a
empresa fornecedora poderá ter maior segurança nas vendas de
Fonte: Adaptado de Chiavenato, 2011. seus produtos para o cliente, pois sabe que a relação
estabelecida é de longo prazo, portanto, de credibilidade e
Segundo Chiavenato20, “a estrutura por base territorial é confiança.
geralmente utilizada por empresas que cobrem grandes áreas
geográficas e cujos mercados são extensos. É especialmente Estrutura por clientes
atrativa para empresas de larga escala e cujas atividades são
geograficamente dispersas”.
Essa estratégia é bastante utilizada por empresas
multinacionais para operar suas atividades fora do país de
origem, como uma forma de facilitar as barreiras alfandegárias,
impostos e também estrategicamente melhor para os clientes.

Vantagens e desvantagens da estrutura


geográfica

As vantagens são:
-Permite maior visibilidade por região quanto aos
resultados, sejam custos, gastos, rentabilidade, etc.; Fonte: Adaptado de Chiavenato, 2011.

20
Chiavenato, Idalberto. Introdução à teoria geral da Administração. 8 – ed. –
Rio de Janeiro: Elsevier, 2011.

26
26
NOÇÕES DE ADMINISTRAÇÃO
Vantagens e desvantagens da estrutura por um sócio (como administrador de projeto), um contador sênior,
clientes e três contadores juniores para uma auditoria que está sendo
feita para um cliente. Uma empresa manufatureira, um
As vantagens são: especialista em produção, um engenheiro mecânico e um
-Quando a satisfação do cliente é o ponto mais importante químico poderiam ser indicados para, sob a chefia de um
da organização; administrador de projeto, completar o projeto de controle de
-Quando a empresa depende substancialmente do poluição.
faturamento advindo de um cliente específico; Em cada um destes casos, o administrador de projeto seria
-Quando o negócio depende de diferentes tamanhos ou designado para chefiar a equipe, com plena autoridade sobre
características de produtos ou serviços que variam conforme o seus membros para a atividade específica do projeto.
tipo ou o tamanho do cliente.
Estrutura por projetos
As desvantagens são:
-Dependência da demanda de clientes;
-O foco fica totalmente voltado aos clientes, podendo
ocasionar uma perda significativa do foco em outros objetivos
organizacionais que também são importantes;
-Se caso a economia local não apresentar significativo
crescimento pode interferir direta ou indiretamente nos
negócios.

Estrutura por processos

É quando o agrupamento das atividades está centralizado Fonte: Adaptado de Chiavenato, 2011.
nos processos de produção e equipamento. É encontrada com
mais frequência em produção, por exemplo, as atividades de Vantagens e desvantagens da estrutura por
uma fábrica podem ser grupadas em perfuração, projetos
esmerilamento, soldagem, montagem e acabamento, cada uma
de acordo com os departamentos. Podemos citar como vantagens:
-Permitem comunicação aberta e coordenação de atividades
Estrutura por processos entre os especialistas funcionais relevantes;
-Capacita a organização a responder rapidamente à
mudança;
-São abordagens orientadas para a tecnologia, portanto
contribuem para o melhor desempenho.

E como desvantagens:
-Pode haver choques resultantes das prioridades;
-Pode haver perda de controle e coordenação se a empresa
tiver muitos projetos em funcionamento.
Fonte: Adaptado de Chiavenato, 2011.
DEPARTAMENTALIZAÇÃO DE MATRIZ:
A Departamentalização de matriz é semelhante à de projeto,
Vantagens e desvantagens da estrutura por
com uma exceção principal. No caso da Departamentalização
processos
de matriz, o administrador de projeto não tem autoridade de
linha sobre os membros da equipe. Em lugar disso, a
As vantagens são:
organização do administrador de projeto é sobreposta aos
-Maior especialização de recursos alocados;
vários departamentos funcionais, dando a impressão de uma
-Possibilidade de comunicação mais rápida de informações
matriz. A organização de matriz proporciona uma hierarquia
técnicas;
que responde rapidamente às mudanças em tecnologia.
-Visão especializada do processo, sendo possível ter mais
Por isso, é tipicamente encontrada em organização de
habilidades dentro de um processo específico.
orientação técnica, como a Boeing, NASA e GE onde os
cientistas, engenheiros, ou especialistas técnicos trabalham em
As desvantagens são:
projetos ou programas sofisticados. Também é usada por
-Possibilidade de perda da visão global do andamento do
empresas com projetos de construção complexos.
processo;
-Flexibilidade restrita para ajustes no processo.
Vantagens e desvantagens da estrutura matricial
Estrutura por projetos
Vantagens: Permitem comunicação aberta e coordenação de
atividades entre os especialistas funcionais relevantes. Capacita
Neste tipo de estrutura, os profissionais trabalham em
a organização a responder rapidamente à mudança. São
função de um ou mais projetos, de forma que recebem
abordagens orientadas para a tecnologia.
atribuições temporárias levando em consideração que o projeto
Desvantagens: Pode haver choques resultantes das
tem data de início e término.
prioridades.
Terminado o projeto as pessoas são deslocadas para outras
atividades. Por exemplo: uma firma contábil poderia designar

277
NOÇÕES DE ADMINISTRAÇÃO
- O fator humano é diferente,
- A tecnologia e a natureza das atividades são diferentes,
- Os ambientes externos são diferentes,
- Os objetivos e as estratégias são diferentes.

A integração – Quanto mais atividades trabalham


integradas, maior razão para ficarem no mesmo departamento.
Fatores de integração são:
- Necessidade de coordenação.

Questões

01. Assinale como verdadeira (V) ou falsa (F) as afirmativas


a respeito dos tipos tradicionais de organização.
( ) A estrutura funcional é caracterizada por uma
autoridade funcional ou subdividida de acordo com as funções
* Destaca-se que pode ser que a organização tenha uma exercidas por cada um dentro da organização.
estrutura mista - mistura os vários tipos de estruturas para se ( ) Na estrutura linha-staff a especialização é substituída por
adaptar à realidade. uma abordagem holística da organização onde cada
departamento é simultaneamente operação e assessoria.
( ) A estrutura linear é baseada na autoridade linear, que
Departamentalização Mista significa que cada superior tem autoridade única e absoluta sem
reparti-la com ninguém.
( ) Na estrutura linha-staff as áreas responsáveis pelos
objetivos vitais da empresa estão ligadas em linha enquanto os
órgãos de assessoria não possuem uma autoridade linear.
( ) A estrutura linear é caracterizada por uma ênfase na
especialização. Cada órgão contribui com sua especialidade
para a organização sem diluição da unidade de comando.
Escolha a opção correta.
(A) V, F, V, V, V
(B) F, V, F, V, F
(C) V, V, F, F, V
(D) V, F, V, V, F
(E) F, F, V, V, V

02. Sobre estrutura organizacional, é correto afirmar:


(A) O grau de descentralização é outra decisão importante
no delineamento da estrutura; quanto mais centralização maior
É o tipo mais frequente, cada parte da empresa deve ter a será a falta de coordenação e controle.
estrutura que mais se adapte à sua realidade organizacional. (B) A formalização, explicitada em manuais de organização
Além disso, ressalta-se que devido à competição de mercados, que descrevem níveis de autoridades e responsabilidades dos
as estruturas organizacionais tendem a ficar cada vez mais vários departamentos, assegura que, na operação, não exista a
enxutas. estrutura informal.
(C) A unidade de comando, princípio da administração
A MELHOR FORMA DE DEPARTAMENTALIZAR: clássica, é aplicada em todos os tipos de estrutura quando feito
processo de departamentalização.
Para evitar problemas na hora de decidir como (D) A definição precisa de direitos e obrigações dos
departamentalizar, pode-se seguir certos princípios: membros da organização traduzidas em funções bem
- Princípio do maior uso – o departamento que faz delineadas é uma característica de organizações mecanicistas.
maior uso de uma atividade, deve tê-la sob sua jurisdição. (E) Um dos pontos a observar na estrutura é a amplitude de
- Principio do maior interesse – o departamento que controle. Quanto menor a amplitude de controle, menor o
tem maior interesse pela atividade deve supervisiona-la. número de níveis hierárquicos.
- Princípio da separação e do controle – As
atividades do controle devem estar separadas das atividades 03. A estrutura organizacional por Projetos apresenta como
controladas. vantagem:
- Principio da supressão da concorrência – Eliminar (A) possibilita economia pelo uso racional dos
a concorrência entre departamentos, agrupando atividades equipamentos.
correlatas no mesmo departamento. (B) predispõe todos os participantes da organização para a
tarefa de satisfazer os clientes.
Outro critério básico para departamentalização está (C)forma efetiva para conseguir resultados em problemas
baseado na diferenciação e na integração, os princípios são: complexos.
- Diferenciação, cujo princípio estabelece que as atividades (D)economias de escala pelo uso integrado de pessoas,
diferentes devem ficar em departamentos separados. máquinas e produção em massa.
(E)permite fixar a responsabilidade pelo desempenho no
A diferenciação ocorre quando: comportamento regional ou local.

28
28
NOÇÕES DE ADMINISTRAÇÃO
04. Assinale a alternativa que apresenta o tipo de estrutura Líder: "Um chefe, um guia, aquele que representa
organizacional que tem por essência a combinação das formas um grupo".
de departamentalização funcional e de produto ou projeto na
mesma estrutura organizacional. Chefe: "Pessoa que comanda, que dirige; o cabeça;
(A)Estrutura organizacional linha e staff. o alto escudo; tratamento irônico que geralmente é
(B)Estrutura organizacional horizontalizada. dado a um desconhecido".
(C)Estrutura organizacional verticalizada.
(D)Estrutura organizacional matricial. Enquanto a primeira definição está mais ligada com uma
(E)Estrutura organizacional informal. postura de orientação, coordenação, direção, a segunda
definição trata mais no sentido de autoritarismo, de ordem, de
05. Um Assistente em Administração, integrante de uma seguimento de regras e normas.
comissão responsável para realizar estudos do funcionamento
administrativo de uma Universidade Federal Brasileira, Enquanto o Líder é conhecido como aquele que orienta as
participou dos trabalhos para definição do que deve ser feito, pessoas a fazerem de bom grado aquilo que é proposto,
ou seja, os resultados e intenções futuras a serem alcançados geralmente pedindo e não impondo além de estar aberto a
por essa instituição de ensino, e como deve ser feito, considerar contestações para sua análise. Sua postura é mais
correspondendo aos recursos e ações necessários para alcance voltada a participação de todos segundo autores pesquisados.
dos resultados. Nesse caso a postura está mais voltada à ideia de motivação,
Neste trabalho, o servidor percebeu que a universidade respeito, conquista, atenção e a construção de um ambiente
estava estruturada em um aspecto piramidal, em decorrência harmônico em prol dos objetivos coletivos.
da centralização da autoridade no topo da organização - na
reitoria - demonstrando claramente a unidade de comando e o Nas organizações empresariais o Chefe é aquele conhecido
escalonamento hierárquico. Constatou ainda que apresentava por dar ordens diretas e incontestáveis aos demais. Faz
uma departamentalização constituída da agregação de tarefas executarem tarefas através da sua imposição, obrigando os
de acordo com as funções principais desenvolvidas dentro da demais a cumprirem, sem considerar que muitas vezes aspectos
universidade. relativos às condições humanas estão envolvidos e podem em
Esse assistente, em seu setor de trabalho e quando não certos momentos ser feito de maneira diferente ou menos
estava na comissão, era responsável pela análise de processos imediata e quase sempre de forma inflexível. Muitos defendem
administrativos para concessão de bolsas de auxílio estudantil, a necessidade dessa postura em dados momentos, mas no caso
que compreendia a conferência dos documentos presentes nos dos chefes esta é sempre a mesma postura observada.
processos administrativos, e, ao listar todos os processos,
verificou que uma bolsa era requerida por sua esposa, fato que Pessoas em particular têm uma ou outra postura
exigiu um posicionamento do Assistente. profissional e como as empresas são movidas por pessoas em
Qual o tipo de estrutura organizacional da universidade? muitas organizações chefes autoritários se chocam com líderes
(A) Linear que defendem outra forma de comportamento, sendo motivo
(B) Matricial de desentendimentos entre as partes, os colaboradores buscam
(C) Virtual atuar em um ambiente mais participativo mais propenso a
(D) Rede condução de um líder, enquanto chefes buscam exatamente o
contrário, um ambiente sem a participação nenhuma destes.
06. A estrutura organizacional que promove a retenção do
processo decisório na cúpula da organização é denominada Considere a situação onde um profissional se configure líder
estrutura: dentro de determinada empresa com a cultura voltada para a
(A)facilitada hierarquia rígida, ou então se configure chefe em uma
(B)matricial organização com a cultura voltada para a liderança
(C)indelegada participativa, pois é o que acontece na prática em muitas
(D)centralizada organizações daí o porquê dos desentendimentos entre essas
(E)monocrática partes.
Respostas
Se não houver um alinhamento entre as partes torna-se
01. D / 02.D / 03. C / 04. D / 05. A / 06: D desgastante para qualquer um tomar sobre si tais
responsabilidades e os problemas ao invés de melhorarem
acabam por surgir ou ressurgir com mais intensidade.

"O chefe existe naquelas organizações que não


precisam - ou não querem ser eficientes,
normalmente no setor público ou empresas
Aspectos comportamentais: familiares, onde os privilégios pessoais são mais
importantes que projetos de longo prazo".
liderança, motivação, clima e
cultura organizacional. "Líderes estão preocupados com o futuro e não
com o presente, ou melhor, preocupam-se em
cumprir o necessário no presente para garanti-lo e,
além disso, preparar a organização para os desafios
Liderança
vindouros".
Segundo o dicionário Aurélio, podemos classificar:

299
NOÇÕES DE ADMINISTRAÇÃO
Sentimento de posse talvez possa traduzir esta explanação - Autoritária (ou Autocrática);
a respeito dos chefes, uma vez que adquirem esse sentimento - Liberal;
só enxergam sua posição de “status” dentro da organização - Democrática.
considerando os demais como inferiores na escala de seus
próprios valores. Líder Autocrático
Já os líderes apontam com um sentimento mais coletivista
entendendo que a organização só é capaz de atingir o sucesso Determina diretrizes sem a participação do grupo, além de
se cada um doar um pouco do seu melhor. determinar as técnicas para a execução das tarefas. É ele
também que indica qual a tarefa de cada um dos colaboradores
O chefe precisa dominar seu funcionário para mostrar que e quem será o companheiro de trabalho de cada um. É inflexível
está no controle, e de certa forma, se torna um dependente, que e dominador, causando mal-estar organizacional.
na maioria das vezes não consegue resolver certos problemas Tem postura essencialmente diretiva, dando instruções
por si só. Além disso, o chefe faz com que as regras sejam sólidas, e não deixa espaço para criatividade dos liderados. Este
cumpridas. Ele manda e faz com que tudo aconteça pela ordem. líder, nos elogios ou nas críticas que faz, dirige-se diretamente
Já no caso do líder, faz uma combinação de estratégia e aos colaboradores, nominando publicamente os eventuais
caráter para atingir metas e objetivos para extrair o máximo de autores de faltas cometidas e/ou de atos dignos de elogios.
cada pessoa que trabalha ao seu lado, ouvindo ideias e gerando As consequências desta liderança estão relacionadas com a
responsabilidades. Sempre disposto a servir, aprender e sempre ausência de espontaneidade e de iniciativa, e pela inexistência
em uma posição de humildade para poder agregar cada vez de qualquer amizade no grupo. O trabalho só se desenvolve na
mais à sua equipe. presença do líder, visto que, quando este se ausenta, o grupo
produz pouco e tende a indisciplinar-se. É o estilo que produz
Toda organização deve se preocupar com esta questão e não maior quantidade de trabalho.
tentar impor a qualquer custo sua maneira de trabalho aos
colaboradores, algumas justificam a necessidade de ter em seu Líder Liberal
quadro chefes autoritários que imponham regras rígidas senão
ninguém obedece, enquanto outras dão valor à presença de Também denominado Laissez-Faire: o líder não se impõe e
líderes flexíveis mais voltados à motivação e multiplicadores de não é respeitado. Os liderados têm total liberdade para tomar
conhecimento, não há nada de errado com isso, só que alguém decisões, quase sem consulta ao líder. Não há grande
com características de um líder encontra dificuldades em se investimento na função, havendo participações mínimas e
conformar num cargo de chefe, enquanto o oposto também é limitadas por parte do líder. O grupo é que decide sobre a
verdadeiro. divisão das tarefas e sobre quem trabalha com quem. Os
elementos do grupo tendem a pensar que podem agir
O grande impasse está em mesclar os dois tipos e isso se livremente, tendo também desejo de abandonar o grupo. O
encontra com certa frequência, tratam os colaboradores de uma líder não regula e nem avalia o grupo, e faz alguns comentários
forma rígida esperando deles um comportamento participativo irregulares sobre as atividades apenas quando questionado. A
enquanto esperam que os chefes imponham regras produção não é satisfatória, visto que se perde muito tempo
incontestáveis e se comportem como líderes motivadores. com discussões e questões pessoais. Este é frequentemente
Nessas organizações a própria cultura organizacional considerado o pior estilo de liderança.
impede que seja desenvolvido um trabalho sinérgico entre
colaboradores e diretrizes. Líder Democrático
Seja como for e que tipo de organização se empreenda um
trabalho, o importante é que se tenha claro que tipo de O Líder assiste e instiga o debate entre todos os
ambiente profissional se deseja construir, se as atividades colaboradores. É o grupo que esboça as providências e técnicas
diárias e o relacionamento com o público em geral afetam de para atingir os objetivos e todos participam nas decisões. Cada
forma positiva ou negativa os resultados finais. membro do grupo decide com quem trabalhará e o próprio
grupo que decide sobre a divisão das tarefas. O líder procura
Conhecer com clareza qual o tipo de negócio e em que área ser um membro igual aos outros elementos do grupo, não se
atua, pois nem toda organização necessariamente tem de se encarregando muito de tarefas.
preocupar em formar líderes, assim como nem toda É objetivo, e quando critica limita-se aos fatos. Este tipo de
organização necessariamente deve se preocupar em promover liderança promove o bom relacionamento e a amizade entre o
chefes. grupo, tendo como consequência um ritmo de trabalho suave,
Criar um ambiente harmônico levando em consideração a seguro e de qualidade, mesmo na ausência do líder. O
importância que todos exercem, pode ser o início do sucesso de comportamento deste líder é de orientação e de apoio. É o estilo
uma organização que atenta a essas questões mudam sua que produz maior qualidade de trabalho.
maneira de enxergar seus colaboradores e passam a tomar certa Percebe-se que a liderança autocrática evidencia o líder,
cautela na hora de nomear seus líderes ou chefes. enquanto a liderança liberal evidencia os subordinados e a
O lado negativo é conhecido por muitos, descontentamento liderança democrática evidencia tanto no líder como nos
entre membros da equipe e chefias, uma queda de braço sem subordinados. Líderes influenciam liderados e liderados
efeitos positivos para ambos os lados até que o colaborador também influenciam seus líderes, portanto deve-se lembrar de
“decida” se auto demitir. que o relacionamento entre os membros da equipe pode
também interferir no estilo que o líder adotará.
De acordo com Iannini21, existem três Estilos de Sendo assim, cabe a cada líder escolher qual o estilo mais
Liderança: se adapta às suas características e situações para que os

21
IANNINI, Pedro Paulo. Chefia e liderança: capacitação gerencial. Viçosa: Aprenda
Fácil, 2000.

30
30
NOÇÕES DE ADMINISTRAÇÃO
objetivos sejam alcançados rapidamente, visto que quando O conceito de liderança deve partir de uma definição
existe uma equipe, a meta é alcançada com o esforço de todos, conceitual mais específica e menos reducionista. Para tal, é
tanto líderes quanto liderados22. necessário antes reforçar o conceito de poder, levando-se em
conta as formas inapropriadas da utilização do seu termo. O
A liderança não deve ser confundida com gerência conceito a ser utilizado deve-se ao fato de que sua sustentação
ou chefia. Para ser um líder, não precisa ser encontra-se em uma reflexão que procura distinguir o uso do
necessariamente um chefe; basta ter conhecimento e termo poder com suas formas de manifestação.
habilidades que façam com que o grupo busque a
realização dos objetivos. Assim, o poder pode ser definido como:

Liderança X Administrador (...) a capacidade que tem uma classe social (ou
uma fração ou segmento), uma categoria social ou
Movimentar as pessoas através de ações condicionadas está um grupo (social ou politicamente organizado) de
ao alcance de qualquer administrador. Mas desbloquear a definir e realizar seus interesses objetivos específicos,
sinergia potencial que elas trazem, ao ingressarem na mesmo contra a resistência ao exercício desta
organização, e canalizar a energia daí liberada, ao mesmo capacidade e independentemente do nível estrutural
tempo que providencia que ela não venha a ser estancada ou em que tal capacidade esteja principalmente
mesmo perdida por frustrações e desenganos, é atributo para fundamentada. O exercício do poder adquire
líderes. continuidade e efetividade política quando do acesso
do grupo ou da classe social ao comando das
Para tanto, espera-se que eles tenham sensibilidade principais organizações, das estruturas
suficiente para detectar as necessidades das pessoas, não para institucionais ou políticas da sociedade, inclusive
manipular ou descartar tais necessidades, mas para conviver aquelas criadas como resultado de um processo de
produtivamente com elas e ajudar seus portadores na busca da transformação, de maneira a colocar em prática ou
satisfação. Ao mesmo tempo em que assim agem, estes líderes a viabilizar tal exercício (Faria).
estão constantemente em busca de auto realização e, nesse
processo, levam consigo seus seguidores, estando sempre um O primeiro ponto importante a salientar é que o poder se
passo à frente deles. manifesta em classes sociais, categorias sociais e grupos
socialmente e politicamente organizados. Isto quer dizer que o
O líder, no novo contexto, deve ter a habilidade de tornar a poder não se manifesta somente em ambientes legalmente
atividade significativa para o subordinado, valorizando-lhe o formalizados.
esforço e conduzindo-o ao ponto desejado. Não se trata de O segundo ponto é que as classes sociais, as categorias
manipular, mas de obter a compreensão sobre a real motivação sociais ou os grupos política e socialmente organizados buscam
do outro. Portanto um bom administrador é capaz de as realizações de objetivos específicos.
desempenhar o papel do líder com sucesso, de forma a conduzir
a equipe a desenvolver a equipe e conseguir alcançar os É importante observar que apesar dos indivíduos
resultados esperados pela organização. procurarem atingir os objetivos específicos comuns, não se deve
esquecer que cada membro vincula-se a um grupo para realizar
Poder e Liderança23 seus objetivos individuais. Isto acontece devido às diferenças
pessoais de cada integrante. Aqueles que conseguem colaborar
Liderança, assim como o poder, só pode ser percebida nas de forma diferenciada para que a classe social, categoria social
suas manifestações. Entretanto, o conceito de liderança não ou grupo social atinjam os objetivos coletivos serão destacados
deve ser confundido com o conceito de poder. Ambos se pelos demais integrantes: é exatamente aqui que aparece a
relacionam por partirem da mesma fonte, a legitimidade no liderança.
âmbito coletivo, mas não possuem conotações semelhantes.
Sendo a autoridade uma das bases do poder, reduzir o conceito Por fim, o poder é uma capacidade coletiva e, como
de liderança a uma manifestação da autoridade é tentar tal, deve ser adquirida, desenvolvida e mantida. Os
compreender as expressões maiores que envolvem as relações indivíduos inserem-se em suas relações a partir de funções que
sociais para além do visível. desempenham no âmbito coletivo, de forma orgânica ou não,
podendo influir, coordenar, liderar, representar, organizar e
Segundo Schermerhorn24, liderança eficaz se apoia na conferir legitimidade. O poder e suas manifestações estão
maneira como um gerente usa o “poder” para influenciar o imbricados dentro de um processo histórico e dialético, sobre a
comportamento de outras pessoas. influência constante das mudanças sociais.

Poder é a habilidade para conseguir que outra Liderança pode ser entendida como uma manifestação de
pessoa faça alguma coisa que você quer que seja natureza tanto psicológica quanto social e política que ocorre:
feita. É a habilidade para fazer com que as coisas - no interior de uma classe social (numa fração ou segmento),
aconteçam da maneira com que você quer. categoria social ou grupos formais ou informais (social e
politicamente organizados),
Liderança é a maneira como se usa o poder e - entre classes (frações ou segmentos) categorias ou grupos
poder é o exercício da autoridade. sociais,

22
FACHADA, Maria Odete. Psicologia das Relações Interpessoais. 6. Ed. Lisboa:
Rumo. 2003. http://www.revistapsicologia.ufc.br/index.php?option=com_content&id=93%3Ali
23
FARIA, J. H; MENEGHETTI, F.K. Liderança e Organizações – Leadership and deranca-e-organizacoes&Itemid=54&la &lang=pt>.
Organizations. Volume II – número 2, 2011. 24
SCHERMERHORN Jr., JOHN R. (1999). Administração. 5. ed. Rio de
Janeiro: LTc Editora.

311
NOÇÕES DE ADMINISTRAÇÃO
- no interior de organizações e Respostas
- entre organizações.
01. Certo / 02. C / 03. Errado / 04. Certo / 05. D
A liderança apresenta-se como manifestação natural,
decorrente de delegação de autoridade ou adquirida mediante Motivação
atributos reconhecidos por outros como portadores de uma
representação real ou simbólica, com o objetivo de atingir O comportamento das pessoas dentro da organização é
objetivos imaginários e concretos (de natureza econômica, complexo, depende de fatores internos (decorrentes de suas
jurídica, política, ideológica e social), sejam eles de ordens características de personalidade, como capacidade de
individuais ou coletivas. aprendizagem, de motivação, de percepção do ambiente
A liderança difere da autoridade e do simples carisma externo e interno, de atitudes, de emoções, de valores etc.) e
porque pressupõe a prática democrática, emancipatória e fatores externos (decorrentes do ambiente que envolve as
esclarecedora, voltada sempre aos interesses de uma ética características organizacionais, como sistemas de recompensas
coletiva. Deste modo, o papel do líder requer não só e punições, de fatores sociais, de políticas, de coesão grupal
capacidades próprias como também coletivas. Uma liderança existente etc.).
não ocorre sem a legitimação dos integrantes que compõem a
coletividade que a confere. Deste modo, a liderança pode ser Com a Teoria das Relações Humanas passou-se a estudar a
transitória e requer, do líder, um constante investimento em influência da motivação no comportamento das pessoas.
sua manutenção. (Faria25) Embora este seja apenas um dos fatores internos que
influenciam o comportamento humano, a ele é dado tanta
Questões importância porque a motivação atua, em geral, sobre as
necessidades dos indivíduos, a fim de supri-las para atingir os
01. (TRT - 17ª Região/ES - Analista Judiciário - objetivos, tanto pessoais como organizacionais
Psicologia –CESPE/2015) Percepções sobre o clima de uma (CHIAVENATO26).
organização são de aprendizagem do indivíduo. Essas necessidades humanas, tidas como forças ativas e
( ) Certo ( ) Errado impulsionadoras do comportamento, apresentam uma enorme
imensidão. Isso porque as pessoas são diferentes entre si,
02. (TCE/RS - Auditor Público Externo - FMP-RS) A possuem necessidades diferentes e estas, consequentemente,
avaliação de desempenho é um sistema formal de produzem padrões de comportamento que variam de indivíduo
gerenciamento que provê a avaliação da qualidade do para indivíduo. Apesar de todas essas diferenças enormes, em
desempenho individual e/ou institucional em uma organização. pesquisas realizadas acerca do comportamento humano, foi
Assim, ela pode visar apenas ao indivíduo ou também às constatado que o processo que dinamiza o comportamento
equipes, às áreas e à organização. No nível do indivíduo, a humano é mais ou menos semelhante para todas as pessoas.
avaliação de desempenho permite:
(A) maior alinhamento das unidades da organização com Baseando-se nos pressupostos de Chiavenato27 podem ser
suas metas e objetivos estratégicos. definidas três suposições para explicar o comportamento
(B) o desenvolvimento de uma visão sistêmica por parte dos humano, das quais a análise nos é válida, pois elas estão
indivíduos em relação à organização. intimamente relacionadas com o processo de motivação:
(C) obter subsídios para a progressão na carreira, com base
em competências e desempenho, entre outros benefícios. 1- O comportamento humano é causado, ou seja,
(D) o desenvolvimento do espírito de equipe. existe uma causalidade do comportamento. Tanto a
(E) a percepção da interdependência entre áreas e pessoas. hereditariedade como o meio ambiente influem decisivamente
sobre o comportamento das pessoas. O comportamento é causado
03. (TRT - 17ª Região/ES - Analista Judiciário - por estímulos internos ou externos.
Psicologia – CESPE/2015). A conceituação de clima
organizacional é constituída por duas dimensões, que são a 2- O comportamento é motivado, ou seja, há uma
insatisfação e a satisfação do indivíduo com a organização. finalidade em todo comportamento humano. O comportamento
( ) Certo ( ) Errado não é causal, nem aleatório, mas sempre orientado e dirigido para
algum objetivo.
04. (ANAC - Analista Administrativo - Área 1 -
CESPE) A cultura organizacional consiste em um fenômeno 3- O comportamento humano é orientado para
multidimensional e multinível. objetivos pessoais. Subjacente a todo comportamento existe
( ) Certo ( ) Errado sempre um impulso, um desejo, uma necessidade, uma tendência,
expressões que servem para designar os motivos do
05. (FBN - Assistente Técnico – Administrativo – comportamento.
FGV) Quando um líder eficaz conecta os liderados, ele tem
como meta: Motivação e Desempenho
(A) dividir a equipe.
(B) aumentar sua popularidade. Sendo o comportamento humano fundamentalmente
(C) substituir os superiores. orientado por objetivos, sejam eles conhecidos ou
(D) atingir os resultados. inconscientes, fica evidente a importância de se associar

25 26
O conceito apresentado aqui pode ser encontrado de forma mais desenvolvido em CHIAVENATO, Idalberto. Gestão de pessoas: o novo papel dos recursos humanos
Faria, José Henrique de. Economia Política do Poder. 6ª. Reimpressão. Curitiba: nas organizações. Rio de Janeiro: Campus, 1999.
27
Juruá, 2010. 3 Volumes. Volume 1. CHIAVENATO, Idalberto. Recursos Humanos: edição compacta. 5. ed. São Paulo:
Atlas. 1998.

32
32
NOÇÕES DE ADMINISTRAÇÃO
atitudes motivacionais a esses objetivos. Esses fatores relacionados ao conteúdo do trabalho (denominados fatores
motivacionais vão influir diretamente no comportamento do intrínsecos ou de motivação), enquanto os aspectos
indivíduo e, consequentemente, no seu desempenho dentro da insatisfatórios diziam respeito às condições dentro das quais
organização. o trabalho era executado (fatores extrínsecos ou higiênicos).
Na verdade, tratam-se de três itens – motivação,
comportamento e desempenho – que se apresentam Fatores Fatores Higiênicos
estreitamente ligados. O desempenho é uma manifestação do Motivacionais
comportamento humano nas organizações, podendo assim Trabalho em si Condições de Trabalho
também ser motivado pelo próprio indivíduo (motivos Realização Administração da Empresa
internos) ou pela situação ou ambiente em que ele se encontra Reconhecimento Salário
(motivos externos). (MAXIMIANO28). Progresso Relações com o Supervisor
Responsabilidade Benefícios e Incentivos
Motivos Internos: são as necessidades, aptidões, Sociais
interesses e habilidades do indivíduo, que o fazem capaz de
realizar certas tarefas e não outras; que o fazem sentir-se - A satisfação no cargo depende dos fatores motivacionais.
atraído por certas coisas e evitar outras; o que o fazem valorizar
certos comportamentos e menosprezar outros. Podem ser - A insatisfação no cargo depende dos fatores higiênicos.
definidos ainda como os impulsos interiores, de natureza
fisiológica e psicológica, afetados por fatores sociológicos:
necessidades, frustração, aptidão, habilidades, atitudes e
interesses.

Motivos Externos: são os estímulos ou incentivos que o


ambiente oferece ou objetivos que a pessoa persegue porque
satisfazem a uma necessidade, despertam um sentimento de
interesse porque representam a recompensa a ser alcançada. Os
motivos externos podem ser divididos em duas categorias
principais – o trabalho e as condições de trabalho – segundo
uma classificação feita numa teoria proposta por Frederick
Herzberg, que exerceu grande influência nos estudos sobre a Em linhas gerais, Herzberg e seus colaboradores afirmam
motivação. que as pessoas são motivadas apenas pelos fatores intrínsecos,
ou seja, apenas o trabalho em si e os fatores que lhe são
Teorias Motivacionais diretamente relacionados podem motivar as pessoas
(CHIAVENATO).
Teoria de Campo de Lewin
Hierarquia das Necessidades – Maslow
A teoria de campo de Kurt Lewin (CHIAVENATO29) baseia-
se em duas suposições fundamentais: Partem do princípio de que os motivos do comportamento
- O comportamento humano é derivado da totalidade de humano residem no próprio indivíduo: sua motivação para agir
fatos coexistentes. e se comportar derivam de forças que existem dentro dele. A
- Esses fatos coexistentes têm o caráter de um campo teoria de Maslow tem uma importância relevante no estudo da
dinâmico, no qual cada parte do campo depende de uma inter- motivação humana. (CHIAVENATO30).
relação com as demais outras partes.
Em síntese, segundo Maslow, as necessidades humanas
Assim, afirma que o comportamento humano não estão arranjadas em uma pirâmide de importância e de
depende só do passado, ou do futuro, mas do campo influenciação do comportamento humano. Na base da pirâmide
dinâmico atual e presente. Esse campo dinâmico é o estão as necessidades mais baixas e recorrentes, chamadas
“espaço de vida que contém a pessoa com seu necessidades primárias – necessidades fisiológicas e de
ambiente psicológico”. segurança; enquanto no topo estão as mais sofisticadas e
intelectualizadas – necessidades secundárias: sociais, de
Foi Lewin que instituiu o termo ambiente psicológico estima e de auto realização (CHIAVENATO).
(ou ambiente comportamental) como sendo o ambiente tal
como é percebido e interpretado pela pessoa e relacionado com Realização Pessoal: criatividade, espontaneidade,
as atuais necessidades do indivíduo. desafios.
Estima: autoestima, confiança, respeito dos outros.
Teoria dos dois fatores – Frederick Herzberg Relacionamento: amizade, família, ambiente de
trabalho.
Elaborada com base em pesquisas feitas para estudar a Segurança: saúde, recursos, segurança do emprego.
relação entre a produtividade e a moral, questionavam os Fisiologia: comida, respiração, sono, sexo.
aspectos agradáveis e desagradáveis de seu trabalho. Chegaram
à conclusão de que os aspectos satisfatórios estavam mais

28 30
MAXIMIANO, Antonio César Amaru. 4. ed. Introdução à Administração. São CHIAVENATO, Idalberto. Gestão de pessoas: o novo papel dos recursos humanos
Paulo: Atlas, 1995. nas organizações. Rio de Janeiro: Campus, 1999.
29
CHIAVENATO, Idalberto. Recursos Humanos: edição compacta. 5. ed. São Paulo:
Atlas. 1998.

333
NOÇÕES DE ADMINISTRAÇÃO
O Modelo Contingencial de Motivação de Vroom Teoria Y – Concepção positiva da natureza humana

Victor H. Vroom desenvolveu uma teoria da motivação que - O dispêndio de esforço no trabalho é algo natural.
reconhece a evidência de que diferentes pessoas reagem de - O controle externo e a ameaça não são meios adequados
diferentes maneiras, conforme a situação em que estejam de se obter trabalho.
colocadas, enfatizando assim as diferenças individuais. - O homem exercerá autocontrole e auto direção, se as
(CHIAVENATO) necessidades próprias forem satisfeitas.
Segundo ele, existem três fatores que determinam em cada - A pessoa média busca a responsabilidade.
indivíduo a motivação para produzir: - O empregado exercerá e usará sua engenhosidade, quando
- Os objetivos individuais, ou seja, a força do desejo de lhe permitirem auto direção e autocontrole.
atingir objetivos.
- A relação que o indivíduo percebe entre a produtividade e Teoria de McClelland
o alcance dos seus objetivos individuais.
- A capacidade de o indivíduo influenciar seu próprio nível Segundo McClelland, as pessoas são divididas em: uma
de produtividade, à medida que acredita poder influenciá-lo. minoria desafiada pelas oportunidades e uma maioria que não
são capazes, nem dispostas a enfrentar desafios para alcançar
Teoria da Expectação - Lawler objetivos. Esse teórico identificou três necessidades
caracterizadas como pontos-chave para se obter motivação.
Em seus trabalhos sobre motivação, Lawler encontrou fortes
evidências de que o dinheiro pode motivar o desempenho e Teoria da Motivação pelo Êxito e/ou Medo
outros tipos de comportamento, como o companheirismo e
dedicação à organização. Porém, verificou-se que o dinheiro Motivos que orientam o comportamento humano:
tem apresentado pouca potência motivacional em virtude da
sua incorreta aplicação pela maior parte das organizações - Realização (competir como forma de auto realização);
(CHIAVENATO). - Afiliação (relacionar-se cordial e afetuosamente);
- Poder (exercer influência).
Em síntese, ele concluiu que o dinheiro pode ser poderoso
motivador se as pessoas acreditarem haver ligação direta ou Essas necessidades são adquiridas no decorrer da vida,
indireta entre desempenho e consequente aumento de conhecidas como secundárias, e trazem sensações que o ser
remuneração. Segundo Lawler, se essa percepção for alcançada humano gosta de experimentar, como prestígio e status. Porém,
e confirmada, as pessoas certamente terão melhor desempenho para o autor, as pessoas não são motivadas pelas mesmas
tendo em vista o resultado financeiro desejado. necessidades igualmente, variando de pessoa para pessoa.

Teoria de McGregor Questões

Descreveu os fatores motivacionais, agrupando em: 01. (Prefeitura de Palmas/TO - Técnico


Administrativo Educacional - COPESE – UFT) Marque o
Fatores Extrínsecos: salário, benefícios e promoção. fundamento que NÃO se relaciona à Teoria das Hierarquias das
Necessidades de Maslow:
Fatores Intrínsecos: auto respeito, capacidade de (A) Somente quando um nível inferior de necessidades está
solucionar problemas, autonomia, aquisição de novos satisfeito é que o imediatamente mais elevado surge no
conhecimentos. comportamento da pessoa.
(B) O comportamento funciona como um canal através do
Motivação Intrínseca ao Indivíduo: os elementos qual as necessidades são expressas ou satisfeitas.
motivadores, que apenas potencializam as suas ações, (C) Todas as pessoas conseguem chegar ao topo da
funcionam se, e somente se, o indivíduo já estiver motivado por pirâmide das necessidades.
algum fator pessoal (intrínseco). (D) Cada pessoa possui mais de uma motivação e todos os
níveis atuam conjuntamente no organismo.
A teoria de McGregor é, na verdade, um conjunto de dois
extremos opostos de suposições. Estes conjuntos foram 02. (BACEN - Analista Administrativo – FCC) Para
denominados X e Y. Por esse motivo, também é conhecida pelo Herzberg, os fatores que determinam a motivação são a
nome de Teoria X e Teoria Y. Para McGregor, se aceitarmos a realização, o reconhecimento, o trabalho em si, a
teoria X, e nos comportarmos de acordo com ela, as pessoas se responsabilidade, o avanço e o crescimento. Em outra dimensão
mostrarão preguiçosas e desmotivadas. Já, se aceitarmos a estão os fatores que determinam a desmotivação, quando não
teoria Y, as pessoas com quem interagimos se mostrarão são atendidos de acordo com as expectativas dos empregados,
motivadas. ou fatores de higiene. É pertinente, então, afirmar que:

Teoria X – Concepção negativa da natureza I. O conteúdo de uma tarefa é mais importante para a
humana motivação dos trabalhadores do que a ambiência em que ela se
desenvolve.
- O homem médio não gosta do trabalho e o evita. II. A concessão, em níveis adequados, de fatores higiênicos,
- Ele precisa ser forçado, controlado e dirigido. como segurança, salário e status, é importante para evitar que
- O homem prefere ser dirigido e tem pouca ambição. os empregados se sintam desmotivados, gerando, tal concessão,
- Ele busca apenas a segurança. motivação.
III. Para que os trabalhadores se sintam motivados a
desempenhar as suas tarefas é preciso que se dê constante

34
34
NOÇÕES DE ADMINISTRAÇÃO
atenção a fatores como reconhecimento, responsabilidade e e o grau de lealdade expresso por seus funcionários a respeito da
desenvolvimento individual, além da definição adequada da empresa. A cultura organizacional representa as percepções dos
tarefa em si. dirigentes e funcionários da organização e reflete a mentalidade
que predomina na organização (CHIAVENATO)31.
É correto o que consta em
(A) I e II, apenas. A cultura organizacional deve ser compreendida no
(B) I, II e III. contexto onde ela se insere, portanto, além de se considerar os
(C) I e III, apenas. aspectos da cultura nacional, há que se analisar os seus
(D) II, apenas. componentes, os seus tipos e as formas pelas quais é aprendida.
(E) II e III, apenas. Toda organização desenvolve uma cultura, a composição de
cada cultura depende do processo de aprendizagem ao resolver
03. (TRF - 2ª REGIÃO - Analista Judiciário - Área problemas de adaptação ao ambiente organizacional e de
Administrativa – FCC) Dentre as teorias da motivação, convivência.
aquela que, numa primeira visão, sugere que os gerentes devem A linguagem é um elemento importante que revela a
coagir, controlar e ameaçar os funcionários a fim de motivá-los adaptação do indivíduo ao ambiente organizacional. Assim,
e, numa segunda visão, acredita que as pessoas são capazes de como as palavras exame, diagnóstico, prontuário, emergência,
ser responsáveis, não precisam ser coagidas ou controladas para fazem parte da rotina do ambiente hospitalar, toda organização
ter um bom desempenho, é a teoria tem sua própria linguagem. Quando o indivíduo reconhece
(A) da motivação e higiene. essas palavras, é sinal de que está aculturada, inserido na
(B) da hierarquia das necessidades. cultura organizacional.
(C) X e Y. As culturas organizacionais dividem-se em subculturas. Na
(D) dos motivos humanos. medida em que as organizações crescem, ou a depender da
(E) do reforço positivo e de aversão. forma que se organizam, os departamentos podem se
diferenciar em termos de missão e do tipo de pessoas
04. (MPE-PI - Técnico Ministerial - Área empregadas. A cultura do departamento de engenharia pode
Administrativa – CESPE) De acordo com a teoria dos dois ser diferente do departamento de marketing, pessoas de áreas
fatores — motivação e higiene —, o oposto de satisfação não é diferentes podem desenvolver sua própria cultura ocupacional.
a insatisfação. Ainda segundo essa teoria, a eliminação dos
aspectos de insatisfação de um trabalho não o torna COMPONENTES DA CULTURA ORGANIZACIONAL
necessariamente satisfatório.
( ) Certo ( ) Errado De acordo com Chiavenato (2004), ao observar-se a cultura
organizacional, nota-se que a mesma é constituída por
05. (IBGE - Supervisor de Pesquisas - Tecnologia componentes visíveis e observáveis, orientados aos aspectos
de Informação e Comunicação CESGRANRIO/2016) O operacionais e de tarefas como, por exemplo, a estrutura
gestor de uma empresa entende que nem tudo que gera organizacional, os métodos e procedimentos, os objetivos e
insatisfação ao funcionário justifica a sua falta de motivação. estratégias, as políticas e diretrizes de pessoal, entre outros; e por
Assim, ao tratar das questões que geram a insatisfação dos componentes invisíveis e emocionais, orientados aos aspectos
funcionários, a empresa poderia ter um ambiente de trabalho sociais e psicológicos, tais como, as relações afetivas, valores e
mais pacífico, sem significar, contudo, aumento da motivação. expectativas, padrões de influenciação e de poder, normas grupais,
etc.
A perspectiva desse gestor a respeito da motivação dos Os componentes da cultura organizacional são
funcionários é compatível com a proposta da apresentados por meio de três níveis diferentes: artefatos,
(A) teoria X e teoria Y valores compartilhados e pressuposições básicas.
(B) teoria dos dois fatores Os artefatos pertencem ao primeiro nível e são mais
(C) fixação de objetivos perceptíveis, incluem os produtos, serviços e padrões de
(D) hierarquia das necessidades comportamento dos membros de uma organização, tais como,
(E) abordagem dos sistemas abertos os símbolos, as histórias, os heróis, as cerimônias anuais e os
lemas.
Respostas Os valores compartilhados constituem o segundo nível e
são formados a partir dos valores relevantes para as pessoas
01. C / 02. C / 03. C / 04. Certo / 05. B numa organização, definindo as razões pelas quais elas fazem
as suas atividades.
CULTURA ORGANIZACIONAL O terceiro nível, que se caracteriza por ser mais profundo, são
as pressuposições básicas, ou seja, as crenças inconscientes,
A cultura organizacional ou cultura corporativa é o conjunto sentimentos e percepções, nas quais as pessoas acreditam.
de hábitos e crenças, estabelecidos por normas, valores, atitudes e A cultura organizacional estabelece uma identidade
expectativas, compartilhado por todos os membros da para a organização, uma marca reconhecível pelas pessoas
organização. Ela se refere ao sistema de significados internas à organização e pelas pessoas de fora, a identidade é
compartilhados por todos os membros e que distingue uma exteriorizada revelando um modo próprio de fazer as coisas, de
organização das demais. Constitui o modo institucionalizado de categorizar, de interagir, que emerge por meio de uma
pensar e agir que existe em uma organização. A essência da estrutura interna de poder, da configuração criada
cultura de uma empresa é expressa pela maneira com que ela faz internamente, com o objetivo de obter uma vantagem
seus negócios, trata seus clientes e funcionários, o grau de competitiva sobre as outras organizações, levando em
autonomia ou liberdade que existe em suas unidades ou escritórios

31
CHIAVENATO, I. Gestão de Pessoas. Rio de Janeiro: Elsevier, 2004. 529 p.

355
NOÇÕES DE ADMINISTRAÇÃO
consideração as solicitações e peculiaridades apreendidas, culturais, e que são vistos com muito respeito, ou até adorados,
reconhecidas no meio externo, pelas pessoas das posições por grande parte dos colaboradores.
dirigentes.
CARACTERÍSTICAS DA ESSÊNCIA DA CULTURA DE
PRECEITOS DA CULTURA ORGANIZACIONAL32 UMA ORGANIZAÇÃO

A cultura organizacional envolve artefatos (padrões de Existem sete características básicas que, em conjunto,
comportamento), valores compartilhados (crenças) e capturam a essência da cultura de uma organização:
pressupostos (valores, verdades). Também pode conter
componentes visíveis, que são sempre orientados pelos aspectos Característica essenciais da Cultura
organizacionais, ou componentes ocultos, que são sempre Organizacional
orientados pela emoção e situações afetivas. Grau de Trata-se do grau de estimulo dado
inovação aos funcionários para que sejam
• Preceitos (implícitos ou explícitos) inovadores e assumam o risco da
Normas, regulamentos, costumes, tradições, símbolos, inovação.
estilos de gerência, tipos de liderança, políticas administrativas, Atenção aos Trata-se da precisão, análise e
estrutura hierárquica, padrões de desempenho. detalhes cuidado com os detalhes que se
espera dos funcionários.
• Tecnologia (instrumentos e processos Orientação para Trata-se do grau no qual o foco da
utilizados) resultados direção está direcionado aos
Máquinas, equipamentos, layout, distribuição e métodos de resultados e não aos processos e
trabalhos. técnicas utilizados para alcançá-los.
Foco na pessoa Trata-se do grau em que a direção
• Caráter (Manifestação dos indivíduos) da organização considera o impacto
Como que o indivíduo se comporta diante da sociedade. de suas decisões sobre o seu pessoal
durante o processo de tomada de
Participação, criatividade, grupos informais, medo tensão, decisões
apatia, agressividade, comodismo. Essa mesma cultura pode Trata-se do grau em que a
aparecer nas organizações de duas formas distintas. Como um Foco na equipe organização do trabalho está mais
subsistema que se liga à estrutura, à estratégia, sistemas voltada para as equipes e não para
políticos e técnicos, ou ainda como uma superestrutura que os indivíduos
determina todos os demais componentes. Alguns dos Agressividade Trata-se do grau de agressividade e
componentes da cultura são de origem histórica, do ambiente e competitividade das pessoas na
território em que ela se situa, de crenças e pressupostos (mitos, organização, em oposição à
ideologias, etc.), de regras, nomes e regulamentos, do processo tranquilidade que poderia existir
de comunicação (linguagem), de ritos, rituais e cerimônias, de Estabilidade Trata-se do grau de estabilidade
heróis e tabus, ou ainda de produtos e serviços com que está enfatizada pela organização, que
envolvida. busca a manutenção do status quo
Existem diversas funções que a cultura pode exercer dentro ao invés do crescimento
de uma organização: ela define os limites, a coerência nos atos organizacional
dos empregados; dá aos funcionários uma sensação de
identidade, de pertencer a algo grande, amplo e sério, trazendo
TIPOS DE CULTURA ORGANIZACIONAL
motivação e ainda fazendo-os se comprometer com interesses
coletivos; reduz a ambiguidade, determinando exatamente
Há inúmeros tipos de cultura organizacional, no entanto,
como os trabalhos devem ser executados. Algumas vezes ela
segundo Chiavenato (2004), destacam-se as culturas
funciona até mesmo como um vínculo entre os funcionários e a
organizacionais adaptativas e não-adaptativas.
empresa, ajudando a permanecerem unidos através de normas
As culturas organizacionais adaptativas se mostram mais
do que se deve fazer e dizer. Mas sua principal função é
maleáveis e flexíveis frente às mudanças organizacionais,
distinguir uma organização de outra.
distinguindo-se pela possibilidade de inovação e criatividade,
A cultura organizacional, assim como a gestão das
pela atenção voltada ao cliente e pelo valor atribuído às
organizações, modifica-se com o tempo, já que também sofre
pessoas.
influência do ambiente externo e de mudanças na sociedade.
As culturas organizacionais não-adaptativas são
Entretanto, a cultura de uma instituição também pode
conservadoras na manutenção de seus costumes, tradições e
influenciar essa mesma sociedade.
valores, despertando um comportamento mais burocrático
A contracultura também existe nas organizações, e nada
entre os seus administradores.
mais é do que um movimento reacionário, por parte de um
grupo pequeno, ou até mesmo grande, que quer reagir contra
Há ainda a classificação de culturas fortes e
os valores tradicionais, que está insatisfeito, e vive em busca de
fracas
mudanças e inovações na cultura atual.
Na formação da cultura há também uma forte influência dos
Culturas fortes: Seus valores são compartilhados
fundadores da instituição, que estabeleceram diretrizes
intensamente pela maioria dos funcionários e influencia
comportamentos e expectativas. É uma cultura mais

32
FERREIRA, J.M. Carvalho; et al. (1996). Psicossociologia das Organizações. CHIAVENATO, Idalberto; Gestão de Pessoas: o Novo Papel da Gestão de Pessoas. Rio
Alfragide, Editora McGraw-Hill. de Janeiro: Editora Campus, 1999.
HINDLE, Tim; Guia de Ideias e técnicas de Gestão. Editora Caminho. FLEURY, M. T. L. (Org.). As pessoas na organização. São Paulo: Gente, 2002.

36
36
NOÇÕES DE ADMINISTRAÇÃO
homogênea com valores intensamente acatados e mais aceita. mostram os acertos e erros do passado, dando fundamento às
Um resultado específico da cultura forte é um índice mais baixo práticas atuais na organização.
de rotatividade da força de trabalho. Uma cultura forte
demonstra um elevado grau de concordância entre seus Desvantagens de uma cultura organizacional bem
membros sobre os pontos de vista da organização. Essa estabelecida
unanimidade de propósitos gera coesão, lealdade e A principal desvantagem que uma cultura organizacional
comprometimento organizacional. Tais qualidades, por sua vez, forte e bem sólida pode causar é impedir ou mesmo
reduzem a propensão dos funcionários a deixar a organização. prejudicar os processos de mudanças
(ROBBINS)33. organizacionais, quando forem necessários.

Culturas fracas: São culturas mais facilmente mudadas. CLIMA ORGANIZACIONAL


Como exemplo, seria uma empresa pequena e jovem, como está
no início, é mais fácil para a administração comunicar os novos Se por um lado a gestão da cultura35 trata os valores e
valores, isto explica a dificuldade que as grandes corporações crenças que orientam o comportamento humano na
tem para mudar sua cultura. Esta cultura é mais heterogênea e organização, a gestão do clima atua na percepção global das
há poucas pessoas que aceitam a cultura ou não há pessoas que pessoas a respeito de seu ambiente de trabalho, capaz de
pensem de modo comum quanto aos valores organizacionais. É influenciar o comportamento profissional e afetar o
mais dissipada. desempenho da organização.
Segundo Fischer (1996), todas as relações entre as pessoas
Alguns fatores que influenciam na cultura e a organização devem ser consideradas.36 Na área de estudos
organizacional: sociais, o clima “refere-se à percepção da pessoa sobre quão
bem ela será recebida em dado ambiente (por exemplo o clima
FUNDADORES: deixam sua marca na empresa, da sala de aula ou no local de trabalho)”, conforme glossário
principalmente referente crenças e valores; do site da Ohio State University.
RAMO DE ATIVIDADE: alguns ramos são mais formais,
mais exigentes, com mais tecnologia e outros não, isso com “Clima Organizacional constitui o meio interno de uma
certeza irá impactar na cultura de uma empresa. organização, a atmosfera psicológica e característica que existe
DIRIGENTES ATUAIS: eles também influenciam em cada organização. O clima organizacional é o ambiente
diretamente na cultura da empresa. humano dentro do qual as pessoas de uma organização fazem
A ÁREA GEOGRÁFICA: pois há diferenças na população, os seus trabalhos. Constitui a qualidade ou propriedade do
nível cultural, padrão cultural, aspectos socioeconômicos, etc. ambiente organizacional que é percebida ou experimentada
pelos participantes da empresa e que influencia o seu
VANTAGENS E DESVANTAGENS DA CULTURA comportamento.” Idalberto Chiavenato
ORGANIZACIONAL34
Em diversos conceitos analisados, existem 3 elementos se
A partir de uma cultura empresarial bem estabelecida é repetem em quase toda definição:
possível estimular o cultivo de um ambiente de trabalho 1) Satisfação dos funcionários: que se remete ao grau
propício e favorável ao desenvolvimento efetivo das atividades de satisfação dos trabalhadores em relação ao clima de uma
diárias e, principalmente, para o alcance dos lucros e resultados empresa.
almejados pela organização. 2) Percepção dos funcionários: trata-se da percepção
dos colaboradores sobre aspectos que podem influenciá-lo
Existem diversas vantagens que uma cultura positiva ou negativamente.
organizacional pode oferecer a uma empresa 3) Cultura organizacional: cultura e clima, a cultura
• reduzir substancialmente problemas, dificuldades e conflitos influenciando o clima de uma empresa, faces complementares
internos; de uma mesma moeda.
• Favorecer o desenvolvimento de uma imagem positiva da
empresa diante do mercado; Esse clima decorre dos sentimentos reinantes no grupo, que
• Diferenciar uma empresa diante de seus concorrentes; por sua vez dependem das relações e comunicações. Há
• Definir uma identidade própria e senso de pertencimento a ingredientes que produzem clima positivo e ingredientes que
cada dos funcionários presentes; “derrubam” o clima, conforme exemplificado a seguir.
• Proporcionar uma melhor gestão e o controle eficaz de
processos e de pessoas; Ingredientes positivos que criam um clima
• Propiciar uma adaptação rápida dos colaboradores à agradável e produtivo
cultura empresarial; - Valores comuns
•Pode aumentar o comprometimento dos funcionários - Identidade positiva, autoestima grupal
atuantes. - Envolvimento de todos com tarefa
•Favorecer o processo de aprendizagem da cultura - Vitórias partilhadas pelo grupo
organizacional, o qual pode se dar de diversas formas, como - Inclusão de todos, com apoio
histórias, símbolos, rituais, linguagem e materiais. As histórias

33
ROBBINS, STEPHEN P. Comportamento Organizacional. São Paulo: Pearson Disciplina Cultura e Clima Organizacional.
Prentice Hall, 2005. http://www.fecra.edu.br/admin/arquivos/CULTURA_E_CLIMA_ORGANIZACIONA
L_APOSTILA.pdf
34 36
http://www.jrmcoaching.com.br/blog/vantagens-e-desvantagens-da-cultura- XAVIER, Ricardo. Gestão de Pessoas na prática. Desafios e soluções. São Paulo,
organizacional/ 2006.
35
LUZ, Ricardo. Gestão do Clima Organizacional. Rio de Janeiro. Qualitymark,
2003.

377
NOÇÕES DE ADMINISTRAÇÃO
Ingredientes negativos que criam um clima Melhoria na comunicação interna; - Aumento do
desagradável e improdutivo comprometimento dos funcionários com a empresa; -
- Presença de excluídos Integração; - Credibilidade
- Conflitos de valores profundos e não trabalhados
- Presença de pessoas que não se identificam com o grupo, Perguntas do tipo podem surgir: do que adianta
não têm orgulho de pertencer a ele pesquisar o clima da organização se sabemos que o grande
- Posturas críticas negativas problema da organização é o salário baixo?

Diferentemente da gestão da cultura, que trata de valores Sabemos que o salário não é o grande causador de
muitas vezes inconscientes, a gestão do clima trata de aspectos problemas de motivação (não ele sozinho). A motivação está
que são evidenciados e percebidos pelas pessoas em todas as ligada a:
relações existentes na organização. Um clima positivo favorece - Desafio
a implantação de estratégias, a execução de mudanças e o - Perspectiva de desenvolvimento
desenvolvimento de pessoas, assim como afeta todos os - Reconhecimento
processos existentes na organização. - Sentido de Utilidade
- Segurança
INDICADORES DO CLIMA ORGANIZACIONAL - Autonomia
1) Turnover - Remuneração justa
2) Absenteísmo
3) Avaliações de desempenho É importante que a área de RH gerencie o clima
4) Programas de sugestões organizacional para que sejam identificados pontos de
5) Pichações no banheiro insatisfação e desmotivação, focos de resistência a mudanças,
6) Greves problemas gerenciais e de comunicação, desconhecimento e
7) Conflitos interpessoais e interdepartamentais falta de compromisso com a estratégia, dentre outros.
8) Desperdício de material
9) Reclamações no serviço de medicina RELAÇÕES ENTRE CULTURA E CLIMA

O indicador só alerta de alguns fatores importantes, daí Entre clima e cultura há uma relação de causalidade. A
deve-se confirmar com a estratégia de realização de pesquisa cultura sendo a causa e o clima a consequência.
de clima para realmente avaliar o clima de uma empresa. Clima e cultura são fenômenos intangíveis, apesar de se
manifestarem de forma concreta. Apesar disso vemos a cultura
O QUE É UMA PESQUISA DE CLIMA? se manifestar através de arquiteturas, vestuários,
comportamentos de colaboradores, Ela irá se tangibilizar
“É o mais importante instrumento de apoio para através do relacionamento da empresa com seus parceiros
implementações consistente de processo de melhoria contínua.” comerciais.
(Luz, 200337) Clima é um fenômeno temporal, refere-se aquele
É uma ferramenta de gestão onde será analisado o ambiente dado momento. Já a cultura é decorrente de práticas
interno buscando visualizar os alavancadores e as recorrentes ao longo do tempo.
vulnerabilidades do planejamento estratégico da empresa.
Questões
A pesquisa de clima deve abordar, necessariamente,
aspectos referentes a satisfação e a motivação do público 01. (TRE-RR - Analista Judiciário - Administrativa
pesquisado, mas deve também incluir questões que a FCC/2015) A cultura organizacional
organização considera importante saber a respeito da relação (A) traz informações de satisfação e insatisfação dos
das pessoas com a empresa, tais como: comprometimento com empregados como: comunicação, reconhecimento, entre
estratégias, envolvimento com o negócio, conhecimento do outros.
produto, do mercado, da concorrência etc. (B) é um aspecto secundário e temporário, que influencia
as organizações.
OBJETIVOS DA PESQUISA DE CLIMA (C) é importante para promover as mudanças necessárias,
- Avaliar o grau de satisfação dos funcionários em relação à a partir do entendimento das crenças e valores que são
empresa (mais usual); facilmente identificáveis.
- Determinar o grau de prontidão de uma empresa para a (D) não gera impacto no comportamento, na produtividade
implementação de uma mudança. e nas expectativas dos empregados.
- Avaliar o grau de satisfação dos funcionários, decorrente (E) exige ajustes aos padrões existentes em um processo de
do impacto de algumas mudanças. intervenção, evitando-se mudanças bruscas que possam gerar
- Avaliar o grau de disseminação de determinados valores resistências.
culturais entre os funcionários.
02. TJ-AM - Analista Judiciário – Administração –
BENEFÍCIOS DO CLIMA BEM TRABALHADO NAS FGV) O conjunto de hábitos, crenças, valores e tradições,
ORGANIZAÇÕES interações e relacionamentos sociais típicos de cada
- Retenção de Talentos; - Diminuição do índice de doenças organização, é denominado
psicossomáticas; - Treinamentos sintonizados com os objetivos (A) Cultura Organizacional.
da empresa, gerando resultado; - Maior produtividade; - (B) Clima Organizacional.

37
LUZ, Ricardo. Gestão do Clima Organizacional. Rio de Janeiro. Qualitymark,
2003.

38
38
NOÇÕES DE ADMINISTRAÇÃO
(C) Diagnóstico organizacional. (E) suplanta a influência da cultura nacional, permitindo
(D) Mudança Organizacional. que organizações globalizadas mantenham sua cultura em
(E) Aprendizagem organizacional. diferentes países.

03. (COREN/PB - Agente Administrativo - Respostas


CONTEMAX) A cultura organizacional:
(A) Não produz percepções externas à organização. 01. E / 02. A / 03. B / 04. A / 05. C / 06. A
(B) Representa as normas informais e não escritas que
orientam o comportamento dos membros da organização no
dia-a-dia e que direcionam suas ações para a realização dos
objetivos organizacionais.
(C) É representado pelos sentimentos que as pessoas
partilham a respeito da organização e que afetam de maneira
positiva ou negativa sua satisfação e motivação para o trabalho. Desenvolvimento Institucional.
(D) É o conjunto de hábitos, crenças, valores e tradições
típicos do contexto social, econômico e cultural do local onde a
empresa está localizada.
(E) É a distribuição, em várias linhas, de pessoas entre Desenvolvimento Organizacional (Institucional)38
posições sociais que influenciam os relacionamentos entre os
papéis dessas pessoas. O desenvolvimento organizacional é um esforço
planejado, que abrange toda a organização, e
04. (Banco do Brasil - Nível Superior – administrado do alto, para aumentar a eficiência e a
CESGRANRIO) Uma empresa do setor bancário atua de saúde da organização através de intervenções
maneira bastante agressiva no mercado, sempre buscando planejadas nos 'procedimentos' da organização e
informações dos concorrentes para poder antecipar-se ao usando os conhecimentos fornecidos pelas ciências do
lançamento de novos produtos. Com esse objetivo, seus comportamento39
empregados são treinados constantemente para serem Na maioria das empresas ou organizações que dizem
participativos na criação de novos produtos e na identificação praticá-lo, desenvolvimento organizacional é um novo nome
das necessidades do mercado e de seus clientes. para um produto velho. De fato, a tendência da "moderna"
gerência de relações industriais é atualmente a de chamar
As características apresentadas por essa empresa são qualquer tradicional programa de treinamento gerencial como
relativas à "desenvolvimento organizacional". Ainda em outras situações,
(A) cultura organizacional desenvolvimento organizacional é qualquer coisa que aumenta
(B) estrutura organizacional a participação dos empregados nos assuntos da organização.
(C) missão organizacional Finalmente, na melhor das hipóteses, desenvolvimento
(D) lucratividade organizacional organizacional pode também constituir apenas um modelo
(E) hierarquia organizacional normativo que prescreve um certo tipo de comportamento que
se considera apropriado e efetivo para todas as organizações
05. (IF/MT – Administrador – IF/MT) É função da em todo tipo de situações.
Cultura Organizacional: Um outro tipo de desenvolvimento organizacional que vem
(A) Compreender comportamentos e objetos que se desenvolvendo com maior seriedade nos últimos 10 anos
transmitem mensagens. baseia-se, entretanto, em teorias e pesquisas sobre processos
(B) Identificar o processo por meio do qual os indivíduos interpessoais e grupais. Muito simplificadamente, Bennis definiu
aprendem e adquirem a cultura de uma organização ou este último tipo de esforço como "uma complexa estratégia
ocupação. educacional orientada a mudar as crenças, as atitudes, os
(C) Regular as relações entre os membros da organização; valores, e as estruturas das organizações de forma que estas
como devem interagir, que tratamento devem dar-se, como possam adaptar-se melhor a novas tecnologias, mercados, etc".
resolver conflitos. Esta definição, porém, deixa o leitor à vontade para imaginar a
(D) Definir a posição relativa do grupo em relação a outros natureza dos meios através dos quais se supõe que a "mudança"
grupos. acontece. Frenen e Bell, por sua vez, solucionam parcialmente
este problema indicando o seguinte: "Desenvolvimento
06. (Petrobras - Psicólogo Júnior – CESGRANRIO) organizacional é um programa educacional de longo prazo
Uma cultura organizacional forte orientado no sentido de melhorar os processos de resolução de
(A) favorece a diminuição do índice de rotatividade da força problemas e de renovação de uma organização, em particular,
de trabalho. através de uma administração mais colaborativa e efetiva da
(B) determina os valores pessoais de cada um de seus cultura desta organização (e de seus grupos internos), com a
colaboradores. assistência de um agente de mudança, ou catalizador, e o uso
(C) ajuda na adaptação a novas situações sociais e facilita o da teoria e da tecnologia pertinente à ciência do
aprendizado da organização. comportamento organizacional - incluindo pesquisa ativa
(D) requer regras e regulamentos formais claros e rigorosos (active research).
para controlar o comportamento dos colaboradores da
organização.

38
Texto adaptado de Julio Lobos, disponível em http://www.scielo.br/ 39
BECKHARD, Richard. Desenvolvimento organizacional: estratégias e
modelos. São Paulo: Blücher, 1972. 134 p

399
NOÇÕES DE ADMINISTRAÇÃO
CARACTERÍSTICAS CENTRAIS DO CONCEITO DE etc. Mencionado em segundo lugar, o agente de mudança - o
DESENVOLVIMENTO ORGANIZACIONAL qual supõe-se deva trabalhar em conjunto com os executivos
máximos da organização ou da unidade organizacional onde se
Normalmente, um programa de DO é caracterizado em localiza o esforço de mudança - deve ser um consultor externo
termos de processo ou "fluxo identificável de acontecimentos especializado, ao menos durante as etapas iniciais do programa.
inter-relacionados, orientados para a consecução de um certo Ainda que não seja nossa intenção ignorar a possibilidade
objetivo, propósito ou fim". Assim definido, o processo de DO de um funcionário pertencente à organização atuar como
pode ser observado sob duas perspectivas relativamente agente de mudança, é pouco provável que um elemento interno
diferentes: operacional e conceitual. tenha o suficiente treinamento e status e/ou permaneça alheio
O enfoque de ordem mais conceitual, no entanto, é aquele a um eventual conflito de papéis. De fato, o agente de mudança
formulado recentemente por French & Bell. Conforme esse externo está livre das restrições culturais características à
enfoque, deve estabelecer-se uma distinção entre os organização, pode aceitar maiores riscos à sua própria carreira
fundamentos e os componentes operacionais básicos do profissional e, provavelmente, possui maior treinamento e
processo de DO. experiência. Contudo, muitos teóricos e práticos na técnica de
Apesar de que tanto as definições já mencionadas como os desenvolvimento organizacional aconselham incorporar
modelos processuais aqui descritos logram caracterizar o também o pessoal de "staff e linha" de RI como parte integrante
desenvolvimento organizacional como uma área específica do programa de DO.
dentro do campo da teoria do comportamento organizacional,
é ainda conveniente comentar alguns de seus aspectos Pressupostos sobre os participantes
principais com maior detalhe.
O processo de DO apoia-se em um modelo normativo de
Em primeiro lugar, entende-se aqui desenvolvimento funcionamento organizacional que supõe que o comportamento
organizacional como um processo planejado para dos empregados baseia-se em fatores tais como: confiança,
mudar a forma em que organizações funcionam, apoio moral, entrega e recebimento de informações relevantes
tentando descongelar padrões de comportamento, (mutual feedback) e administração de conflito. Neste ponto, é
implantar as mudanças e voltar a congelar os novos necessário especificar mais claramente os pressupostos e
padrões. Em segundo lugar, o conceito de mudança valores que se acham implícitos no conceito de
organizacional já tem sido definido por outros desenvolvimento organizacional. Definidos por French & Bell
autores como um conjunto de alterações no ambiente como sendo congruentes com as teorias de McGregor, Likert,
de trabalho de uma organização, que podem ser de Argyris, Bennis, Schein, Maslow e Herzberg, alguns destes
dois tipos: estruturais e comportamentais. Apesar de pressupostos e valores são os seguintes:
que ambos tipos de mudança podem ser facilmente
reconhecidos como interdependentes, neste artigo estima-se A) O indivíduo
que o processo de desenvolvimento organizacional está - Qualquer indivíduo orienta-se para o desenvolvimento e
fundamentalmente orientado a atuar sobre o comportamento realização pessoal, sempre que localizado dentro de um
mais que sobre a estrutura - acontecendo assim as mudanças ambiente que ofereça condições para isso.
nesta segunda variável somente em decorrência das mudanças - Mais ainda, a maioria das pessoas deseja ser mais do que
já decididas no caso da primeira. é capaz de ser. As pessoas desejam e são capazes de aumentar
sua contribuição para alcançar os objetivos organizacionais,
O processo de DO - percebido já assim por K. Lewin há 30 inclusive além das possibilidades permitidas pelo ambiente da
anos - inclui qualquer das seguintes atividades: coleta de organização.
informações sobre o comportamento dos grupos e das pessoas,
diagnóstico dos problemas prioritários entre o consultor e o B) Os grupos
"cliente", definição conjunta dos problemas prioritários, - Um dos grupos de referência que possui maior relevância
determinação dos recursos internos e externos orientados para para o indivíduo é o de trabalho, incluindo seus pares e seu
a solução do problema, estabelecimento de plano de ação, e supervisor.
estabelecimento de métodos para avaliar a medida em que os - A maioria das pessoas deseja ser aceita e interagir
planos são implantados e/ou os problemas são resolvidos. Esse cooperativamente, pelo menos com um grupo de referência e,
processo, ou as atividades mais importantes que o compõem, usualmente, com mais de um grupo.
supõe-se que deva ser iterativo, no sentido de repetir-se de - Se um grupo deseja otimizar sua efetividade, o líder formal
forma contínua, independentemente de períodos de crise com não pode desempenhar sua tarefa exclusivamente e em todas
que a empresa eventualmente se confronte. as circunstâncias; por isso, os membros do grupo devem
cooperar entre si através de seu próprio comportamento.
Responsabilidades - Os sentimentos reprimidos em um indivíduo devido às
A implantação e manutenção do processo de DO é normas estabelecidas pelo seu grupo de referência chegam a
responsabilidade conjunta do pessoal de cúpula da empresa e neutralizar a sua potencialidade.
de um agente de mudança. É imprescindível que o pessoal - As soluções para maior parte dos problemas motivacionais
mencionado, em primeiro lugar, perceba pelo menos que "nem ou de atitudes nas organizações envolvem uma transação
tudo vai tão bem como deveria na organização". A seguir, é social. Isto é, tais problemas têm uma melhor chance de serem
também necessário que este pessoal-chave acredite na solucionados construtivamente se todas as partes envolvidas no
aplicabilidade da ciência do comportamento organizacional aos sistema ou subsistema alteram as suas relações mútuas. O
problemas gerais ou específicos identificados. Esta crença pode objetivo, então, não é tanto "como A pode fazer com que B
estar baseada em argumentos cognitivos ou em experiências; atinja um bom desempenho", mas "como A e B podem trabalhar
de qualquer forma, é importante que este pessoal-chave em conjunto, modificando assim suas interações para chegarem
participe e apoie as definições iniciais do programa de DO, tais ambos a ter maior efetividade".
como a localização das intervenções, o tipo das intervenções,

40
40
NOÇÕES DE ADMINISTRAÇÃO
C) Os participantes de um sistema organizacional Tipologia de intervenções baseada nos grupos
- Todo gerente ou supervisor é membro de pelo menos duas atingidos
equipes de trabalho - como superior em uma delas e como
subordinado na outra. Os comportamentos de indivíduos em A) Intervenções destinadas a melhorar a
ambas equipes de trabalho não são independentes entre si. efetividade dos indivíduos
- Estratégias de resolução de conflito do tipo "ganha ou - Plano de carreira: atividades que capacitam os
perde" entre pessoas ou grupos, segundo as quais a parte indivíduos a concentrar-se na análise dos seus objetivos
ganhadora fica jubilosa enquanto a parte perdedora adota uma profissionais e vitais e nas formas como estes podem ser
atitude defensiva, não são efetivas a longo prazo. A maioria dos atingidos. Incluem-se: balanço de vida e carreira, discussão de
problemas organizacionais são atacados de uma forma melhor objetivos, avaliação de potencial, especificação de necessidades
começando com a pergunta: "Como é que todos podemos de treinamento e detecção de áreas fortes e fracas da
ganhar?" personalidade.
-Técnicas de análise de papéis: atividades que
Pressupostos sobre a aprendizagem conduzem os indivíduos a definir seus diferentes papéis
organizacionais, identificando possíveis conflitos entre eles
O processo de DO baseia-se na ideia de que é possível e/ou com os papéis de terceiros.
aprender através da experiência, seja no trabalho diário, seja -Assessoria individual: atividades que ajudam o
mediante as próprias intervenções de DO. Supõe-se que os consultor (ou outros membros da organização) a trabalhar em
indivíduos devam ser inicialmente testados quanto à conjunto com um indivíduo, orientando-o sobre
efetividade destas mudanças de comportamento em situações 1. a definição de seus objetivos de aprendizagem;
simuladas, receber um feedback objetivo a tal respeito... e tirar 2. a forma como seu comportamento é percebido por terceiros;
conclusões da experiência. 3. o desempenho de novas formas de comportamento que
podem ajudá-lo a atingir seus objetivos mais eficientemente.
Meios disponíveis - Treinamento de sensibilidade: (T-Group) atividades
Uma grande variedade de intervenções é utilizada para orientadas à melhoria das habilidades interpessoais e do
implantar um processo de DO. Uma intervenção define-se como conhecimento psicossocial do indivíduo, situando-o
um conjunto de atividades estruturadas nas quais participa um temporariamente dentro de um grupo de estranhos sem agenda
grupo selecionado de unidades organizacionais ou equipes de preestabelecida.
trabalho, através do desempenho de uma tarefa ou sequência
de tarefas, cujos objetivos se relacionam direta ou "O T-Group é um grupo relativamente não estruturado, no
indiretamente com o aperfeiçoamento da organização. qual os indivíduos participam como aprendizes. Os dados para
Comumente uma intervenção é praticada, já que ela é uma cada aprendizagem estão dentro dos indivíduos e decorrem da
técnica ou método de aprendizagem, pelo agente de mudança. sua experiência imediata no T-Group: são as transações entre
De qualquer forma, a lista de atividades, técnicas e métodos os participantes, seu próprio comportamento no grupo, à
educacionais aplicáveis num programa de DO é extensa e acha- medida que se esforçam para criar uma organização produtiva
se disponível para ser utilizada, tanto pelo cliente como pelo e viável, uma sociedade em miniatura, e à medida que eles
consultor externo. procuram estimular e apoiar a aprendizagem recíproca dentro
Segundo French & Bell, qualquer intervenção de DO inclui dessa sociedade. Os membros do T-Group precisam estabelecer
o planejamento, execução e avaliação das consequências de um um processo de investigação no qual os dados acerca de seu
determinado tipo de ação. Neste ponto, inclusive, observa-se próprio comportamento sejam coletados e analisados,
que a ênfase que se dá à necessária adoção de um certo tipo de simultaneamente com sua experiência geradora. A
ação é condição distinta de uma intervenção de DO. De fato, a aprendizagem assim conseguida é testada e generalizada para
separação artificial existente entre o treinamento tradicional e uso contínuo."
a posterior operacionalização daquilo que foi aprendido não é - Outras atividades educacionais e de treinamento:
frequente no caso de uma intervenção de DO. diferem do caso anterior apenas porque o indivíduo situa-se
num grupo de trabalho. Neste caso, as atividades do indivíduo
As características de uma intervenção típica de são orientadas pelo consultor no exercício das habilidades
DO são as seguintes: técnicas necessárias para um melhor desempenho de uma
1. buscam-se dois tipos de objetivos: educacional e tarefa específica ou ao aprimoramento da competência
operacional; interpessoal (por exemplo, liderança, análise das
2. procura-se mais atingir os problemas realmente responsabilidades e funções dos membros do grupo, tomada de
confrontados pela organização do que os problemas hipotéticos decisões, estabelecimento de objetivos etc).
e/ou abstratos que podem ou não acomodar-se às necessidades - Desenvolvimento organizacional do tipo grid,
dos participantes; e fase I:12 durante uma semana, os indivíduos aprendem os
3. utilizam-se diversos modelos de aprendizagem. conceitos do grid, avaliam seus próprios estilos de management
utilizando o questionário grid e o esquema grid das duas
Mas, ao fim, que tipos gerais de intervenções de DO dimensões de liderança, desenvolvem habilidades para motivar
podem ser citados especificamente como exemplo? a ação de grupo, aprendem habilidades para resolução de
problemas e de crítica, trabalham na melhor das suas
French & Bell classificam um determinado número de habilidades de comunicação, e aprendem a analisar a cultura
"famílias" de intervenções de DO, conforme diversos critérios. da equipe e da organização.
O critério que melhor distingue entre os vários tipos de
intervenção existentes é, talvez, aquele baseado no grupo B) Intervenções destinadas a melhorar a
atingido. efetividade de grupos limitados de dois ou três
participantes

411
NOÇÕES DE ADMINISTRAÇÃO
- Consultoria de processos: atividades que o consultor produto de um sistema único, em vez de dois ou mais sistemas
executa com a intenção de ajudar o cliente a perceber, entender componentes em separado.
e atuar sobre uma ou mais etapas do(s) processo(s) que tomam - Espelho organizacional: é um tipo de intervenção que
lugar no ambiente organizacional. Neste enfoque de se pratica com três ou mais grupos. Neste caso, um grupo
consultoria, o cliente é orientado sobre os processos sociais que organizacional em particular decide solicitar feedback da parte
dinamizam uma organização e informado sobre a(s) forma(s) de representantes de outros grupos organizacionais sobre como
de diagnosticá-los e manejá-los. A ênfase concentra-se nos seu desempenho é percebido e considerado por estes.
aspectos de comunicações, definição dos papéis de líder e - Tecnoestrutura: este tipo de atividades está orientado
seguidores de um grupo, técnicas para solução de problemas e a melhorar a efetividade dos recursos e limitações técnicas e
tomada de decisões, normas de grupo, autoridade e influência estruturais que afetam os indivíduos ou os grupos. Elas podem
intra-organizacional, e cooperação e competição entre grupos. referir-se a
Dão-se também indicações sobre como desenvolver as 1. experimentação com novas estruturas organizacionais e
habilidades necessárias para atuar efetivamente nestes avaliação da sua efetividade em termos de objetivos específicos; e
processos. 2. desenho de novas formas, para dirigir os recursos técnicos
- Moderação induzida: atividades conduzidas pelo para a solução de problemas organizacionais de ordem
consultor (o "moderador") designadas para "ajudar dois psicossocial.
membros de uma organização a administrar seu conflito - Desenvolvimento organizacional de tipo grid,
interpessoal". Estas atividades baseiam-se em táticas de fase III: nesta etapa, a ênfase se fixa nas relações intergrupais,
confronto e na compreensão, por parte dos membros sendo o objetivo transferir os grupos desde seu inefetivo estilo
conflitantes, dos processos envolvidos tanto no conflito como de relacionamento do tipo "ganho ou perda" para um modelo
na resolução deste. colaborativo de relações intergrupais. Cada grupo analisa
- Desenvolvimento organizacional de tipo grid, separadamente o que seria um relacionamento ideal,
fase II: nesta etapa, a ênfase é dada ao trabalho em grupo. O compartilhando finalmente a sua opinião com outros grupos
objetivo é aperfeiçoar o trabalho em grupo na organização pares. A fase III também inclui o desenvolvimento de planos
através da análise da cultura e das tradições dos diversos operacionais e a designação de papéis individuais, com o
grupos, conjuntamente com o desenvolvimento de habilidades propósito de possibilitar a transferência mencionada.
de planejamento, estabelecimento de objetivos e resolução de - Consultoria de processos, moderação induzida e
problemas. Aspectos adicionais desta etapa incluem feedback survey-feedback.
entregue pelo consultor a cada manager acerca de seu
comportamento individual e como membro do grupo. Todo o E) Intervenções destinadas a melhorar a
trabalho em grupo, em qualquer caso, é feito dentro do efetividade da organização como um todo
contexto de problemas reais de trabalho. - Confrontação: projetada pelo Prof. R. Beckard, esta
intervenção consiste em uma reunião de apenas um a dois dias,
C) Intervenções destinadas a melhorar a nos quais todo o pessoal executivo de uma organização analisa
efetividade de grupos e equipes de trabalho a sua própria "saúde" organizacional. A reunião é composta de
- Construção de equipeis de trabalho: atividades várias etapas. Em resumo, trata-se de
orientadas à melhoria da efetividade operacional das equipes 1. gerar motivação pelo próprio esforço de participar da
de trabalho pertencentes à organização. Estas atividades podem reunião;
dizer respeito a tarefas específicas, apontando as habilidades 2. gerar informação sobre os principais problemas
necessárias para desempenhá-las, os recursos disponíveis, etc; contingentes;
ou podem focalizar-se na natureza e qualidade das relações 3. analisar as causas prováveis;
entre os membros da equipe de trabalho e entre estes e o líder. 4. desenvolver planos de ação para corrigir esses problemas; e
É dada especial consideração aos diferentes tipos de equipes de 5. estabelecer um cronograma incluindo os passos que devem
trabalho possíveis, isto é, equipes formais, equipes temporárias ser dados até atingir soluções satisfatórias.
ou task forces, e equipes de recente constituição. - Desenvolvimento organizacional tipo grid, fases
- Diagnóstico e/ou survey-feedback: compreendem a IV, V e VI: a fase IV do grid consiste em desenvolver um
coleta de dados destinada, em primeiro lugar, a avaliar o estado "modelo estratégico ideal de desenvolvimento da corporação".
do sistema organizacional: "como é que as coisas estão". Os Agora, o centro de atenção é o planejamento estratégico a nível
métodos disponíveis variam entre técnicas projetivas aplicadas de corporação, sendo o objetivo aprender os conceitos e as
a membros-chave do sistema e técnicas tradicionais de coleta habilidades básicas da lógica multi organizacional. O pessoal de
de dados, tais como: entrevistas, questionários, pesquisas, etc. cúpula da empresa inicia as atividades de planejamento
estratégico, ainda que seus planos e ideias sejam testados,
As atividades do tipo survey-feedback são similares às avaliados e criticados conjuntamente com outros membros da
anteriores e se diferenciam delas apenas porque procuram organização. Obtenção de dados, informações técnicas, etc, são
trabalhar com dados já coletados para uma determinada contribuições que qualquer executivo pode dar. Supõe-se que
pesquisa e elaborar planos de ação baseados em tais dados. estas atividades devam resultar em um modelo estratégico ideal
- Treinamento de sensibilidade, consultoria de de desenvolvimento especialmente desenhado para a
procedimentos, técnica de análise de papéis. organização como um todo.
A fase V, por sua vez, visa a implementação do modelo
D) Intervenções destinadas a melhorar a desenvolvido na fase anterior. Isto implica uma reorganização
efetividade das relações intergrupais simulada da situação atual, baseada na identificação de
- Relacionamento intergrupal: são atividades componentes lógicos do sistema, como centros de lucro,
orientadas para melhorar a efetividade de grupos localidades geográficas, linhas de produção, etc. Cada
interdependentes. Elas se concentram em tarefas desenvolvidas componente nomeia uma equipe de planejamento cujo trabalho
em conjunto e visam apresentar o resultado dessas tarefas como é examinar a melhor forma de contribuir para implantação do
modelo estratégico ideal. Uma equipe adicional é encarregada

42
42
NOÇÕES DE ADMINISTRAÇÃO
do planejamento de uma direção central da corporação. A partir subordinados para a identificação de problemas e necessidades
deste ponto, o modelo é implementado. de melhorias.
A fase VI compreende uma avaliação crítica das fases (E) Criação dos denominados T-groups ou learning groups,
prévias. O seu objetivo é conhecer o grau de progresso com a finalidade de realizar a primeira fase do desenvolvimento
alcançado, particularmente no que se refere ao andamento do organizacional, que é o levantamento de dados.
modelo ideal.
04. (SERPRO Técnico - Suporte Administrativo -
Questões CESPE) Com relação ao desenvolvimento organizacional,
julgue os itens subsequentes.
01 (PRODAM/AM Analista Administrativo – A cultura e o clima organizacional são fatores irrelevantes
FUNCAB) O desenvolvimento organizacional é uma técnica no desenvolvimento organizacional.
desenvolvida: ( ) Certo ( ) Errado
(A) com o objetivo de implantar mudanças de forma eficaz;
é uma complexa estratégia educacional, com base na 05. (CPTM - Analista Administrativo Júnior –
experiência, que emprega os meios mais amplos possíveis do Makiyama) Dentre os conceitos que caracterizam o
processo educacional, e que tem por finalidade fortalecer as Desenvolvimento Organizacional, assinale a alternativa que
crenças, atitudes e valores já existentes. explicita aquele que representa as interações entre as partes da
(B) com o objetivo de implantar mudanças de forma eficaz; organização, que se influenciam reciprocamente no âmbito das
é uma complexa estratégia educacional, com base na relações de trabalho bem como a estrutura e os processos
experiência, que emprega os meios mais amplos possíveis de organizacionais, objetivando que todas trabalhem juntas.
comportamento, e que tem por finalidade mudar crenças, (A) Agente de mudança.
atitudes, valores e a estrutura das organizações. (B) Retroação.
(C) com o objetivo de implantar mudanças de forma eficaz; (C) Orientação sistêmica.
é uma complexa estratégia comportamental, com base na (D) Orientação contingencial
tecnologia, que emprega os meios mais amplos possíveis de (E) Aprendizagem experiencial.
comportamento, e que tem por finalidade mudar crenças,
atitudes, valores e a estrutura das organizações. Respostas
(D) com o objetivo de implantar mudanças de forma eficaz;
é uma estratégia comportamental pouco complexa, com base 01. B / 02. B / 03. C / 04. Errado / 05. C
na experiência, que emprega os meios mais amplos possíveis do
processo educacional, e que tem por finalidade fortalecer as
crenças, atitudes e valores já existentes.
(E) com o objetivo de implantar mudanças de forma eficaz;
é uma estratégia comportamental pouco complexa, com base
na experiência, que emprega os meios mais amplos possíveis do
processo educacional, e que tem por finalidade mudar crenças,
atitudes, valores e a estrutura das organizações. Abordagens da mudança
organizacional.
02. (CEFET/MG – Administrador - CEFET-MG) Sobre
o desenvolvimento organizacional, é correto afirmar que
(A) consiste em uma abordagem de mudança na qual os
consultores externos formulam alterações necessárias.
Gestão da mudança
(B) utiliza os conhecimentos das ciências comportamentais
para melhorar a eficácia da organização.
Por volta do ano 500 a.C., o filósofo grego Heráclito
(C) é de fácil avaliação a sua eficiência e seus programas
observou que “não se pode pisar duas vezes no mesmo rio, já
demandam pouco tempo de execução.
que as águas estão em constante movimento”40. Essa afirmação
(D) adota processo dinâmico composto pelas fases de
revela que o mundo organizacional está em constante
diagnóstico, intervenção, reforço e apoio.
transformação. As organizações devem considerar o ambiente
(E) emprega técnicas de reunião de dados em nível de
interno e o ambiente externo:
intervenção entre grupos.
O ambiente interno é controlável pela organização. É o
03. (SEAP/DF Analista – Administração – IADES)
ambiente compreendido no interior das fronteiras físicas desta.
Assinale a alternativa que apresenta exemplos de características
Está sujeito à gestão direta da administração, que deve atuar
do desenvolvimento organizacional.
nele fazendo uma análise permanente das potencialidades e
(A) Procedimentos lineares e restrição a orientações
fragilidades em paralelo com as oportunidades e ameaças dos
contingenciais.
ambientes externos.
(B) Ação de intervenção composta por diagnóstico inicial,
O ambiente externo é composto pelo macroambiente,
procedimentos lineares e avaliação e acompanhamento.
também conhecido como ambiente, e o microambiente,
(C) Orientação sistêmica, foco na organização como um
chamado de mercado. Por não estarem dentro das fronteiras
todo e utilização de agentes de mudanças.
físicas da organização e não serem subordinados à sua
(D) Criação de laboratórios de sensitividade, nos quais os
administração, são ambientes incontroláveis por ela. Cabe,
gerentes conduzem e comandam as ações dos seus
entretanto, observá-los, verificar tendências e prever impactos
destes nas atividades da organização, ou seja, manter uma

40
MORGAN, Gareth. Imagens da Organização, primeira edição. São Paulo,
Atlas, 2007. 389 p.

433
NOÇÕES DE ADMINISTRAÇÃO
análise permanente, pensando em possibilidades de ações qualquer outro componente, capaz de gerar impacto
administrativas para cada uma das oportunidades e ameaças em partes ou no conjunto da organização”42.
vindas desses ambientes.
Para Lopes, Stadler e Kovaleski43, a mudança é tida como um
As mudanças no ambiente organizacional são processo natural ao longo da existência das organizações e é
cada vez mais frequentes, por isso as empresas precisam decorrente da reação destas à ação de forças exercidas pelo meio
se adaptar a variações, incertezas e oscilações, tanto do onde estão inseridas.
ambiente interno como externo. Muitas dessas mudanças são
inesperadas e ocorrem em fases de extrema dificuldade das Existe uma situação de incerteza envolvida na
empresas, tornando o processo decisório ainda mais difícil. mudança que faz com que ela seja temida nas
Conforme surgem novas exigências do mercado, as empresas organizações pelos funcionários. Nas organizações a
devem se adequar; o mercado muda e as empresas também mudança pode significar:
devem mudar para atenderem às novas necessidades, - Uma alteração de posição no mercado em que está inserida,
acompanhando o dinamismo do mercado. A competência - Mudança em sua função social,
organizacional estabelece que a mudança deve ser planejada - Modificação em seu direcionamento estratégico, com uma
em todos os níveis organizacionais como parte da propriedade possível alteração em sua missão, ou
intelectual da organização e do seu eixo competitivo. Dessa - Mudança em sua cultura, com reavaliação de seus valores e
forma, a organização emprega processos globais de melhoria práticas em diferentes níveis de autoridade e responsabilidade.
contínua.
Não podemos falar de mudança organizacional sem falar de Para Pettigrew e Whipp44, o processo de mudança deve ser
cultura organizacional, pois é esta que revela os usos, dividido em três aspectos:
costumes e valores das pessoas que trabalham na organização.
As pessoas formam uma organização informal, diferente da 1- Contexto da Mudança: representa os fatores
organização formal - que é formalizada por meio de normas e contextuais fora da empresa e que podem afetar o processo de
regulamentos escritos e detalhados, com desenho de cargos que mudança, como taxa de desemprego, leis trabalhistas, etc.;
seguem uma linha de comando orientada para atingir objetivos
organizacionais; a organização informal não possui normas e 2- Conteúdo da Mudança: significa o que vai ser
regulamentos formais, é orientada pelos relacionamentos das mudado. Pode ser desde uma máquina, equipamento ou
pessoas que possuem interesses em comum ou que material utilizado ("hard"), ou mudanças na estrutura
compartilham valores semelhantes. A organização organizacional ou maneiras e procedimentos utilizados ("soft").
informal serve para atender às necessidades sociais, McCalmann e Patton consideram que, quanto mais
de relacionamento das pessoas. relacionadas ao "hard", mais fáceis de conduzir serão as
mudanças;
Nesse contexto, utilizaremos o trabalho de Camilo, Almeida
e Mattos41, que apresenta algumas considerações sobre cultura 3- Processo de Mudança: é o aspecto relacionado ao
organizacional e mudança organizacional: estilo de liderança utilizada na mudança - que pode variar do
participativo ao autocrático - e à velocidade da mudança.
Os indicadores da cultura organizacional são a iniciativa
individual, a tolerância ao risco, a direção, a integração, os Seja qual for a mudança e o seu nível dentro da
contatos gerenciais, o controle, a identidade, o sistema de organização, é preciso45:
recompensa e a tolerância ao conflito. É certo que alguns tipos - Conhecer as razões da mudança;
culturais facilitam ou impõem barreiras para a execução do - Gerenciar o processo de mudança;
trabalho, dentre eles os que se concentram nos sistemas e não nos - Realizar um diagnóstico organizacional;
funcionários e aqueles que possuem um pensamento gerencial - Definir a direção da mudança;
visando resolver e adequar seus problemas em um curto período - Estabelecer um plano estratégico de mudança;
de tempo, transferindo a responsabilidade de algum fato ou culpa - Monitorar e avaliar o processo de mudança.
para a força de trabalho.
Existem também aquelas organizações que impõem restrições Objetivos da Mudança
a determinadas informações mantendo alguns fatos em sigilo, e
as que promovem a competição individualizada; neste último, a Shirley46 apresenta cinco tipos básicos de
cultura organizacional enfatiza o individualismo, considerando o objetivos de mudança:
trabalho em equipe como mera formalidade.
1. Estratégicos: aqueles objetivos de mudança
O conceito de mudança organizacional consiste em preocupados em alterar a relação entre a organização como um
“qualquer transformação de natureza estrutural, todo e seu ambiente; por exemplo, objetivos revisados, novo
estratégica, cultural, tecnológica, humana ou de composto de produto ou de clientes, expansão geográfica, uma
mudança na ênfase competitiva.

41
Camilo, Almeida e Mattos. Gestão da mudança e a quebra de paradigmas. 44
Pettigrew e Whipp apud LOPES, P. C. B.; STADLER, C. C.; KOVALESKI, J. C.
Disponível em: Gestão da Mudança Organizacional. Publ. UEPG Ci. Hum., Ci. Soc. Apl., Ling.,
http://revista.unilins.edu.br/index.php/cognitio/article/viewFile/35/37 Letras e Artes, Ponta Grossa, v.11, n. 1, p. 51-57, jun. 2003.
42
Wood Jr, T. (2009). Mudança organizacional: aprofundando temas atuais em 45
LOPES, P. C. B.; STADLER, C. C.; KOVALESKI, J. C. Gestão da Mudança
administração de empresas (5a ed.). São Paulo: Atlas. Organizacional. Publ. UEPG Ci. Hum., Ci. Soc. Apl., Ling., Letras e Artes, Ponta
43
LOPES, P. C. B.; STADLER, C. C.; KOVALESKI, J. C. Gestão da Mudança Grossa, v.11, n. 1, p. 51-57, jun. 2003.
Organizacional. Publ. UEPG Ci. Hum., Ci. Soc. Apl., Ling., Letras e Artes, Ponta 46
Shirley, R. Um modelo para análise da mudança organizacional. Rev. adm.
Grossa, v.11, n. 1, p. 51-57, jun. 2003. empres. 1976, vol.16, n.6, pp. 37-43. ISSN 0034-7590.
http://dx.doi.org/10.1590/S0034-75901976000600004.

44
44
NOÇÕES DE ADMINISTRAÇÃO
2. Tecnológicos: diretamente relacionados com - processo de comunicação/decisão (a maneira como
mudanças na tecnologia de produção, fábrica, equipamento e as decisões formais são tomadas e por quem, os inputs de
outras partes físicas de uma organização. informação subjacentes e os sistemas de informação
estabelecidos para fornecer os inputs para os tomadores de
3. Estruturais: objetivos de mudança preocupados com decisão);
alterações nas relações de subordinação; processos de
comunicação/decisão, relações de autoridades e aspectos - diretrizes (as regras de decisão ou linhas estabelecidas
similares da "anatomia" de uma organização cabem nesta em finanças, marketing, produção, pessoal, compra, pesquisa e
categoria. desenvolvimento e outras áreas; estas linhas servem para ligar
o desempenho de funções específicas à estratégia geral e aos
4. Comportamentais: voltados para a mudança das objetivos da firma);
crenças, valores, atitudes, relações interpessoais,
comportamento grupal, comportamento intergrupal e - sistemas de incentivo formal (características do plano
fenômenos humanos similares. de compensação, benefícios, planos de incentivos e
bonificações, critérios de promoção e outros aspectos do
5. Programa: que se destinam a alterar a estrutura ou sistema formal de recompensa usado pela organização).
aspectos dos planos de implementação técnica na produção,
marketing, pesquisa e desenvolvimento e outras áreas, como Da mesma forma, é útil subdividir os fenômenos
por exemplo, mudanças nos canais de distribuição, requisitos e humanos ou comportamentais em seus elementos
procedimentos do controle de qualidade e territórios de vendas. principais com o propósito de apontar metas de
mudança:
Quanto a esta classificação, deve-se notar que os cinco tipos
não são mutuamente exclusivos e que dois ou mais podem - o indivíduo (inclui fenômenos tais como crenças
operar simultaneamente. individuais, valores e atitudes, assim como comportamento
Por exemplo, o objetivo de mudança estratégico de aberto; também inclui considerações sobre capacidade,
"fundir", normalmente requer mudança estrutural satisfação, personalidade e outros fenômenos comportamentais
(consolidação de algumas funções administrativas) e mudanças que são de natureza individual),
de comportamento (mudança da identificação com uma
organização para a identificação com uma nova entidade - relações interpessoais (enquanto que, no caso
fundida) para que tenha êxito. Por outro lado, a mudança anterior, o foco era o indivíduo, aqui o foco são as relações
estrutural pode ser julgada apropriada devido a uma estratégia entre duas pessoas no desempenho de tarefas);
ou tecnologia não mudada, a fim de aumentar a eficiência
administrativa. A classificação anterior deveria permitir ao - comportamento grupal (esta parte se refere ao grupo
administrador apontar exatamente que tipo de mudança ele como uma unidade de análise, incluindo a consideração da
está tentando realizar, bem como sua relação com os outros presença ou ausência da coesão grupal; objetivos informais do
tipos. grupo, líderes e membros; influência do grupo sobre o
indivíduo; normas do grupo e outros fenômenos do
Alvos da mudança, segundo Shirley47: comportamento que são de natureza grupal);
Os objetivos e critérios estabelecidos no passo anterior tornam
possível a determinação de alvos da mudança, isto é, o foco de - comportamento intergrupal (enquanto que na
todos os esforços de mudança. No caso de mudança categoria anterior o foco era o trabalho do grupo singular, esta
organizacional interna, é útil classificar os alvos como estruturais diz respeito às relações de dois ou mais grupos de trabalho no
ou comportamentais. A estrutura de uma organização se refere à desempenho de tarefas).
divisão formal de trabalho (funções) entre posições, grupos,
departamentos e divisões, assim como os sistemas formais de fluxo Da maneira como são concebidos aqui, os alvos estruturais e
de trabalho, informação, incentivos e diretrizes (regras de comportamentais correspondem às principais dimensões de
decisão) necessários para coordenar as atividades e interações. qualquer organização estabelecida para alcançar uma
determinada posição estratégica com uma dada tecnologia.
Para fins de determinação de alvos num programa Quando uma decisão é tomada para alterar uma estratégia ou
de mudança é, portanto, útil classificar os principais tecnologia particular, supõe-se que um ou mais tipos de alvos
elementos ou componentes de qualquer estrutura identificados, necessariamente, irão requerer mudança, a fim de
organizacional como se segue: implementar a nova estratégia ou tecnologia.
Devemos enfatizar que uma alteração de estratégia ou
- distribuição das funções através da organização tecnologia constitui um "objetivo de mudança" neste paradigma.
(inclusive definição de funções a serem desempenhadas, Em outras palavras, certas mudanças no comportamento e
grupamento de funções e as relações de trabalho verticais e estrutura da organização são necessárias para alcançar os
horizontais entre as funções); objetivos de mudança global, e esses elementos de estrutura e
comportamento identificados anteriormente constituem os alvos,
- relações de autoridades horizontais e verticais isto é, o foco da ação.
(quem tem autoridade para fazer o que);

- relações de subordinação (definição das relações


superior/subordinado e amplitudes de controle);

47
Idem.

455
NOÇÕES DE ADMINISTRAÇÃO
Modelo de Kurt Lewin – o processo de mudança Etapa 3: Recongelamento

O modelo de Kurt Lewin, aperfeiçoado por Schein, é Os novos valores, atitudes e comportamentos são
utilizado para trabalhar a mudança nas pessoas, grupos e reforçados. O recongelamento ocorre quando o novo padrão de
organizações. O modelo possui três etapas: comportamento é aprendido e incorporado pelas pessoas.
Geralmente, há o reconhecimento de que a nova maneira de
Etapa 1: Descongelamento trabalhar é melhor que a antiga.

Busca descongelar o padrão atual de comportamento, o que Lopes, Stadler e Kovaleski apresentam também as
significa que velhos hábitos e ideias devem ser substituídos por razões da mudança e a gestão da mudança:
novas atitudes, valores e comportamentos. Geralmente, a
necessidade de mudança se torna aparente, e as pessoas Razões da mudança48
percebem a necessidade de mudança. Se não houver o
descongelamento, é provável que o padrão de comportamento Os motivos de qualquer mudança em uma organização estão
habitual não seja modificado. dentro da própria organização ou no ambiente onde
ela está inserida, ou como combinação de ambos. A
Etapa 2: Mudança mudança causada por estas forças vai depender de sua natureza
e intensidade, mas também da própria capacidade e
versatilidade da própria organização em enfrentá-las.
Quanto às forças externas à organização, pode se
destacar as influências política, econômica, legal, ética e social.
Dentre as forças internas, destacam-se os recursos
organizacionais excedentes e novos objetivos organizacionais.
Por exemplo, no ambiente externo à organização
podemos considerar os fatores sociais, econômicos, políticos e
tecnológicos. Cada um deles influencia a organização. Como
fator social podemos citar os valores do movimento feminista:
são mais mulheres no mercado de trabalho (o que aumenta a
força de trabalho e de competência técnica), e as empresas se
adaptam e criam mecanismos para combater o assédio sexual,
criam creches e direcionam seu marketing para esse novo
mercado consumidor.

Tipos de mudanças

Para Chiavenato49, existem quatro tipos de mudanças


organizacionais.
A mudança apenas na estrutura organizacional - que
afeta a estrutura propriamente dita -, as redes de comunicação
e níveis hierárquicos.
A mudança na tecnologia - que afeta diretamente as
máquinas, equipamentos, instalações, processos
organizacionais. Este tipo de mudança representa a maneira
pela qual a organização executa suas tarefas e produz seus
serviços e produtos.
Outro tipo de mudança é a dos produtos e serviços - que
afeta os resultados ou saídas da organização. E, por fim, a
mudança nas pessoas e na cultura da organização - que
provoca alterações nos comportamentos, atitudes, expectativas,
aspirações, necessidades; afeta a cultura organizacional.
As organizações que resolvem promover mudança
organizacional têm que avaliar qual dimensão ou tipo de
mudança estão dispostas a fazer. É importante ressaltar que o
quarto tipo de mudança, se bem trabalhado, proporciona um
compromisso e um empenho das pessoas durante todo o
A mudança deve ser promovida por um agente, que processo de mudança, favorecendo muito o seu sucesso.
conduzirá as pessoas aos novos valores, atitudes e
comportamentos. Para que a mudança ocorra, as pessoas
devem se identificar e perceber a eficácia desses novos valores,
atitudes e comportamentos para, então, internalizá-los.

48
LOPES, P. C. B.; STADLER, C. C.; KOVALESKI, J. C. Gestão da Mudança 49
CHIAVENATO, Idalberto. Administração nos novos tempos. Elsevier: Rio de
Organizacional. Publ. UEPG Ci. Hum., Ci. Soc. Apl., Ling., Letras e Artes, Ponta Janeiro, 2004.
Grossa, v.11, n. 1, p. 51-57, jun. 2003.

46
46
NOÇÕES DE ADMINISTRAÇÃO
Figura: Tipos de Mudanças informal. Toda mudança organizacional, para ser eficaz e
alcançar os objetivos, tem que avaliar a organização informal e
procurar trabalhá-la a favor da própria mudança. Este é, sem
dúvida, um dos maiores desafios e um dos grandes motivos de
insucesso das mudanças nas organizações.
É imprescindível que as organizações percebam que não
basta uma mudança adaptativa, mas uma mudança integrada,
sistêmica, que compreenda a essência do fenômeno da
mudança social. Partindo do princípio de que todas as
organizações estão continuamente em processo de mudança,
bem como seus ambientes, a empresa pode redefinir, mudar e
influenciar este processo a seu favor. Dessa forma, o processo
de mudança é muito mais amplo e complexo do que um simples
entendimento das dimensões ambientais. A mudança
estratégica é, antes de tudo, um processo político,
que implica na modificação da distribuição de
recursos e de poder pelos vários níveis e unidades
É a percepção da urgência da mudança por parte dos
organizacionais. Para se promover um processo de mudança
administradores que determina a velocidade da mudança
organizacional eficaz, este deve se ancorar em estratégias
organizacional50:
consistentes53.
- Mudança lenta, contínua e incremental: é
geralmente o caminho seguido pelos programas de
Gestão da mudança54
melhoria contínua e qualidade total, que costumam
receber uma diversidade de nomes. É a mudança indicada para
Seja qual for a mudança, é um processo que necessita ser
organizações que pretendem melhorar seu desempenho de
gerenciado. Dependendo do nível de mudança e a dificuldade
maneira suave e persistente, sem pressa e de maneira integrada
do seu gerenciamento, pode ser necessário definir uma equipe
e democrática, envolvendo todas as pessoas em um mutirão de
responsável pela mudança; no entanto, cada membro da
esforços de mudança.
organização deve ser envolvido. Gerentes, supervisores e chefes de
- Mudança rápida, total e radical: é o caminho
equipe devem liderar alguns aspectos do processo, cada qual no
seguido pela reengenharia. É a mudança indicada para
seu nível.
organizações que têm muita pressa e urgência para mudar e
Segundo Hodgetts, as empresas têm de ser capazes de atuar
que precisam alterar inteiramente seus rumos através de
em um ambiente no qual, independente do lugar, tempo, volume
programas mais impactantes de mudança. Nesse caso, quase
sempre, a sobrevivência da organização está em jogo. ou qualquer fator, as premissas e as regras do jogo mudem
continuamente. Para ele, a mudança, além de desejável, é
Resistência à mudança organizacional51 necessária; e os executivos precisam estar preparados para ela.
Conhecer o presente e projetar um futuro melhor é o que cria
A resistência se manifesta de várias formas, como: atraso e a tensão que impulsiona as pessoas durante o processo de
demora no início do processo de mudança, demora e ineficiência mudança.
imprevista na etapa de implantação, esforços objetivando sabotar
o processo de mudança. Os grupos resistirão à mudança na Por isso, é desejável que a equipe responsável pela
proporção dos níveis de ameaça e desconforto introduzidos pelo mudança tenha a capacidade de:
incremento corrente de mudança. A resistência, ansiedade e poder - usar eficientemente seu conhecimento e as informações;
são problemas ligados com a introdução da mudança - ser criativa;
organizacional. - projetar o futuro dentro e fora da organização;
Kotter52 afirma que a resistência à mudança diminui muito - trabalhar em equipe;
quando a organização consegue compartilhar de forma - ser flexível e se adaptar facilmente aos novos processos;
inteligível com seus funcionários os objetivos da - motivar os demais integrantes;
mesma, ou seja, os funcionários conseguem perceber a - ter ótima comunicação (interna e externa à organização);
importância da nova visão. Caso contrário, as pessoas não farão - assumir riscos e resolver conflitos;
sacrifícios, ainda que estejam insatisfeitas com o status quo; a
não ser que imaginem que os benefícios potenciais da mudança Toda mudança traz incertezas, mas também
serão atraentes ou, ao menos, acreditem realmente que a oportunidades. Em geral, criam pressões dentro de qualquer
transformação é possível. organização, especialmente quando os gerentes não têm
Problemas relativos ao poder são, portanto, fundamentais de experiência em lidar com elas. Assim, a sentença de ordem dentro
serem conhecidos durante o período de transição, uma vez que a de uma organização que decide mudar é: APRENDER A
mudança constitui uma ameaça ao equilíbrio das APRENDER.
forças entre os vários grupos de interesse, formais e Segundo Garvin, para atender às suas necessidades de
informais. mudanças, muitas empresas já incorporaram a necessidade de
As organizações em processo de mudança organizacional adquirir continuamente novos conhecimentos organizacionais,
devem trabalhar a mudança sem subestimar a organização porque o que se aprende numa escola ou universidade, ou no

50
Administrar bem, lucrar sempre. Contexto globalizado: processo de mudança, 53
Goreske, M. D. MUDANÇA ORGANIZACIONAL – CAPACIDADE DE
seus agentes e parceiros. IDEAGRI, 2015. ADAPTAÇÃO. Disponível em: http://re.granbery.edu.br/artigos/Mg==.pdf
51
Goreske, M. D. MUDANÇA ORGANIZACIONAL – CAPACIDADE DE 54
LOPES, P. C. B.; STADLER, C. C.; KOVALESKI, J. C. Gestão da Mudança
ADAPTAÇÃO. Disponível em: http://re.granbery.edu.br/artigos/Mg==.pdf Organizacional. Publ. UEPG Ci. Hum., Ci. Soc. Apl., Ling., Letras e Artes, Ponta
52
KOTTER, J. P. Liderando mudança. Rio de Janeiro: Campus, 1997. Grossa, v.11, n. 1, p. 51-57, jun. 2003.

477
NOÇÕES DE ADMINISTRAÇÃO
próprio trabalho, torna-se rapidamente obsoleto e, (B) revelam o processo de mudança como algo estável e
progressivamente, as transformações vividas pela sociedade fazem fácil de ser conduzido.
com que nenhuma organização tenha condições de garantir a um (C) sustentam a manutenção das “zonas de conforto"
empregado um trabalho específico para sempre. inerentes ao modelos de gestão tradicionais.
O que dificulta um processo de mudança e coíbe o aprendizado (D) ignoram a necessidade de mudança como algo perene.
organizacional é a capacidade de aprender das pessoas e da (E) defendem a transformação como algo presente no
equipe. cotidiano da vida organizacional.

Questões 04. Nas organizações, ocorrem permanentemente


processos de mudança, devido aos quais devem ser
01. Todo processo de mudança organizacional rompe a estabelecidas estratégias para garantir a sustentabilidade e o
rotina e impõe uma ruptura com relação ao passado. Sendo crescimento necessário ao cumprimento das missões. Com base
assim, a gestão da mudança na organização tem um significado nessas informações, julgue os próximos itens relativos ao
extremamente amplo e sistêmico. Entre a elevada gerenciamento de conflito e gestão de mudança.
responsabilidade e lucratividade da organização (o nível mais
elevado) e o grande volume de falhas em projetos, alta O processo de mudança não pode ser planejado mediante
rotatividade e perda de produtividade (o nível mais inferior), e ações proativas e propositais.
admitindo- -se a existência de cinco níveis de gestão da
mudança organizacional, assinale a alternativa que indica o 5.º ( )Certo ( )Errado
(quinto) nível (o mais elevado) nesse processo de mudança na
organização. 05. Em relação a mudança organizacional, considere:
(A) Existem projetos isolados de mudança tópica. Alguns I. São presenciados simultaneamente três campos de força
elementos de gestão de mudança são aplicados em projetos em uma organização em mudança, que se caracterizam por
isolados envolvendo diferentes áreas da organização. Logo, desejar a mudança, evitar a mudança e implementá-la.
várias táticas diferentes são usadas, mas sem consistência. II. A história da mudança mostra que, quando as condições
(B) Existe ausência de qualquer tipo de mudança ou od hoc. melhoram, as pessoas se mostram insatisfeitas; não se
Pequena ou nenhuma gestão da mudança na organização. contentam por terem progredido tanto e reclamam sobre o que
Logo, as pessoas são dependentes e sem nenhuma prática ou ainda devem percorrer.
plano formal. III. O modelo de mudança de Lewin expõe três passos para
(C) Existe competência organizacional em mudança a introdução da mudança, identificando-os sequencialmente
planejada. A gestão competente de mudança é evidente em como mudança, descongelamento e recongelamento.
todos os níveis organizacionais e é parte da propriedade IV. Uma dificuldade na gestão da mudança ocorre pelo fato
intelectual da organização e do seu eixo competitivo. Assim, de a empresa desejar ou necessitar mudar e os indivíduos,
apresenta processos globais de melhoria contínua. embora possam inicialmente colocar-se a favor, tendem a
(D) Existem padrões organizacionais de mudança resistir à medida que as mudanças ocorrem.
planejada. Padrões organizacionais amplos e métricos são Está correto o que se afirma APENAS em
amplamente desdobrados para gerir e liderar a mudança (A) I, II e III.
organizacional. Apresenta seleção de uma abordagem comum. (B) II, III e IV.
(E) Existem múltiplos projetos esparsos de mudança. (C) I e III.
Abordagem compreensiva para a gestão da mudança é aplicada (D) II e IV.
em múltiplos projetos, mas não há uma abordagem sistêmica. (E) III e IV.
Assim, exemplos de melhores práticas são evidentes.
Respostas
02. No modelo de Lewin, a mudança é um processo
sistemático de transição entre os modos antigo e novo de fazer 01. C. / 02. D. / 03. E. / 04. Errado / 05. D
as coisas. A etapa em que ocorre a passagem dos procedimentos
antigos para os novos é chamada de
(A) Congelamento.
(B) Mobilização.
(C) Fixação.
(D) Ação ou Mudança.
(E) Descongelamento.
Processo decisório: tipos de
03. “Por volta de 500 anos a.C., o filósofo grego Heráclito decisões.
observou que 'não se pode pisar duas vezes no mesmo rio’ (...)
atualmente, David Bohm (...) desenvolveu uma notável teoria
que convida à compreensão do universo como uma unidade que
flui e é indivisível." Tomada de decisão
Fonte: MORGAN, Gareth. Imagens da organização. São
Paulo: Atlas, 1996. O processo decisório é o poder de escolher, em determinada
circunstância, o caminho mais adequado para a empresa. Para
Tais ideias são importantes para que se possa compreender que um negócio ganhe a vantagem competitiva é necessário que
a natureza das organizações, porque: ele alcance um desempenho superior, e para tanto, a
(A) expressam a estabilidade que circunda o ambiente organização deve estabelecer uma estratégia adequada,
organizacional contemporâneo. tomando as decisões certas. É necessário que o administrador

48
48
NOÇÕES DE ADMINISTRAÇÃO
moderno permaneça em alerta quanto aos ambientes interno e direcionadora que torna possível a permanência das
externo da organização, para que ele possa obter a escolha mais organizações nesse contexto.
eficaz e simétrica em relação à realidade organizacional da qual É importante ressaltar que cada vez mais as organizações
faz parte. têm buscado maior aprimoramento no processo decisório, de
Tenha em mente que o processo decisório é complexo e modo a investir em técnicas atualizadas que possam servir
depende das características pessoais do tomador de como suporte ao gestor e aos seus auxiliares. Um exemplo de
decisões, da situação em que está envolvido e da técnica moderna utilizada em diversas empresas, independente
maneira como percebe a situação. do seu porte ou área de atuação é o modelo de negócios
chamado CANVAS. Esse modelo que também é chamado de
Natureza do processo decisório Business Model CANVAS foi criado primeiramente por
A Teoria das Decisões nasceu de Herbert Simon, que a Alexander Osterwalder, porém posteriormente ele foi realizado
utilizou para explicar o comportamento humano nas em conjunto com profissionais de diversas áreas para
organizações. aperfeiçoar o modelo. Tal modelo é uma ferramenta de
Simon (1916-2001) estudou a administração sob a gerenciamento estratégico e serve tanto novos empreendedores
perspectiva do processo de tomar decisões. De acordo com ele, no mercado como para empreendimentos que possuem certa
administrar é sinônimo de tomar decisões, especialmente experiência. Esse modelo funciona como uma gestão à vista,
quando se tratava das ações gerenciais. Segundo o próprio onde os envolvidos no processo decisório conseguem enxergar
autor, o processo de tomar decisões possui três fases no total: as principais estratégias traçadas pela empresa, em suas
prospecção (análise de um problema ou situação que requer principais frentes ou áreas departamentais.
solução), concepção (criação de alternativas de solução para
o problema ou situação), decisão (julgamento e escolha de O CANVAS é um diagrama ou mapa que é dividido por 9
uma das alternativas propostas). (nove) eixos composto por:
O homem econômico, de acordo com Simon, busca a 1. Atividades-chave; 2. Recursos-chave; 3. Rede de parceiros;
maximização dos ganhos por meio da racionalidade. Segundo 4. Proposição de valor; 5. Segmento de clientes; 6. Canais; 7.
o autor, a racionalidade humana é limitada e Relacionamento com os clientes; 8. Estrutura de custos; 9. Fluxos
ineficiente, e por isso ele propôs um modelo distinto – o do de receitas.
homem administrativo. Nesse modelo, as decisões tendem
ser satisfatórias ao invés de maximizadas. As decisões Com os eixos acima apresentados os gestores de uma
satisfatórias são aquelas que tendem aos requisitos mínimos organização conseguem ter maior visibilidade do andamento
desejados, fazendo com que os administradores sejam guiados do negócio podendo assim tomar decisões mais rápidas e com
pela regra de que qualquer decisão serve desde que possa mais segurança.
resolver um problema. O modelo de negócios CANVAS não será abordado com
profundidade neste tópico, pois não faz parte do presente
Os principais tipos de decisão (processo edital, apenas realizado um breve comentário para
decisório) conhecimento de uma das técnicas mais utilizadas nos
Simon distingue dois tipos de decisões: programadas e presentes dias e que têm sido um forte aliado no gerenciamento
não programadas. de informações e que tem contribuído para a competitividade
As decisões programadas são caracterizadas por serem das organizações.
repetitivas, rotineiras e estruturadas (tomadas
automaticamente). São as decisões automatizadas, sequenciais O processo decisório nas organizações
que não necessitam da intervenção do decisor. Geralmente, são Considerando o ambiente das organizações, no qual
bastante previsíveis e possíveis de serem incorporados em um diversas mudanças motivadas pelo atual cenário econômico
sistema de informação, por exemplo. O habito, a rotina, os vêm ocorrendo, podemos analisar algumas tendências se
manuais de instruções e operações padronizadas são formas de destacando em relação à tomada de decisão, tais como: o
tomar decisões programadas. estudo da concorrência, análise se cenários, uso expansivo de
As decisões não programadas não dispõem de soluções tecnologias da informação, assim como outras atividades que
automáticas, pois são desestruturadas. São as decisões não visam melhorar e facilitar a tomada de decisão dentro das
automatizadas que dependem da solução do decisor. O organizações.
lançamento de novos produtos, a redução do quadro de Portanto, o processo decisório é substancial a qualquer
funcionários e uma mudança na sede da empresa são exemplos organização, seja esta pública ou privada. Toda organização
de decisões não programadas. Para lidar com essas decisões, o necessita tomar um posicionamento frente às diversas questões,
autor indica que os gerentes devem ser capazes de desenvolver e é esse posicionamento que irá indicar como a empresa irá
sua capacidade de julgamento, intuição e criatividade. Para ele, competir no mercado.
o desenvolvimento dessas habilidades permitiria aos gerentes Para tanto, é necessário entender que cada nível estratégico
lidar de forma eficiente contra as complexidades das decisões. dentro da organização realiza seu tipo de decisões
correspondentes, afinal faz parte de um tipo de planejamento
Técnicas e Ferramentas de apoio ao processo distinto, como a seguir:
decisório
A função de decisão está essencialmente ligada às posições O nível estratégico toma decisões estratégicas e
gerenciais, ou seja, aos berços da liderança. Para diversos realiza um planejamento estratégico
autores, a liderança é importante para a eficácia das O nível tático toma decisões táticas e realiza um
organizações tendo sempre em vista as frequentes turbulências planejamento tático
e mudanças do cenário econômico em geral. Havemos de O nível operacional toma decisões operacionais e
concordar que a autoridade pode ser suficiente em épocas de realiza um planejamento operacional
estabilidade, porém, em um ambiente em constante
transformação é preciso haver liderança, pois é a força

499
NOÇÕES DE ADMINISTRAÇÃO
A tomada de decisão não basta apenas o envolvimento da As decisões que se aplicam a problemas repetitivos são
gerência ou da Alta Administração, mas todos os stakeholders55 denominadas
precisam estar inteirados com a situação-problema para que (A) coletivas
seja uma decisão estrategicamente tomada. (B) individuais
O nível operacional é composto pelos cargos que são (C) maximizadas
considerados como base a uma organização, são eles: (D) programadas
operadores de máquina, líderes de produção, supervisores, que (E) administrativas
contribuem para a formulação de objetivos e metas que fazem
04. O processo decisório compreende a decisão propriamente
parte do planejamento operacional. Que estão mais
dita. Decisão é uma escolha entre possibilidades para solucionar
relacionadas com a linha de produção, ou seja, com o produto problemas. Os principais tipos de decisões tomadas pelos gestores
que chega até o consumidor final. são as decisões programadas e as decisões não programadas. Em
O nível tático é composto pelos cargos que são relação às decisões não programadas, pode-se afirmar que:
considerados como a interligação entre o operacional e o (A) Devem ser tomadas pelos consultores externos à
estratégico, ou seja, não os níveis gerenciais, é o nível da organização quando solicitados.
gerência média ou intermediária. São compostos por cargos (B) Não é necessária a realização de um diagnóstico, a criação
como: gerentes, coordenadores administrativos, gerentes de de alternativas e a escolha de um curso de ação considerado
seção, gerentes de filiais, líderes de projetos e funções similares. original.
Os cargos e funções que fazem parte desse nível são (C) Dependem exclusivamente da reação dos liderados em
responsáveis por assegurarem que as o planejamento relação ao superior, quando envolve o estabelecimento de um
estratégico, isto é, os objetivos e metas delineados pela Alta procedimento padrão.
Administração sejam colocados em prática pelo nível (D) São tomadas para solucionar problemas que as soluções
operacional. Além de colocar em ação os processos, as pessoas padronizadas não são suficientes para resolver.
e os recursos para assegurar que seu segmento de atuação (E) Devem ser tomadas por escalões de níveis mais baixos na
estará alinhado às decisões estratégicas e para assegurar organização.
também que estará colaborando para a obtenção dos resultados
05. As decisões nas organizações podem ser programadas e
gerais da Organização.
não programadas. Sobre o processo decisório dentro das empresas,
E por fim, o nível estratégico é onde são tomadas as decisões programadas e decisões não programadas, leia as
ações estratégicas, neste nível é configurado o Planejamento sentenças abaixo e assinale a alternativa correta:
Estratégico formado pela Missão, Visão, valores e objetivos de I. As decisões programadas resolvem problemas que já foram
todas as áreas, de todos os produtos que a empresa oferta e os enfrentados anteriormente e que se comportam sempre da mesma
planos de ação para que sejam possíveis de serem alcançados. maneira.
Esse nível é composto por cargos e funções como: diretores, II. As decisões programadas economizam tempo e energia
presidentes, conselho administrativo, sócios, proprietários, evitando desgaste.
acionistas, etc. A função típica deste nível é tomar decisões III. As decisões não programadas são definidas
estratégicas. individualmente, baseiam-se em situações novas, não
Essa interação entre níveis diferentes, entre áreas e funções padronizadas.
diferentes é extremamente essencial para o processo decisório. Estão corretas as afirmativas:
Os níveis precisam ter visão sistêmica e para isso precisam unir (A) Apenas as afirmativas I e II estão corretas.
habilidades e experiências diferentes para conseguir ter o (B) Apenas as afirmativas I e III estão corretas.
máximo de precisão no momento de decidir. (C) Apenas as afirmativas II e III estão corretas.
(D) Todas as afirmativas estão corretas.
(E) Nenhuma afirmativa está correta.
Questões
Respostas
01. O processo decisório envolve decisões programadas e
decisões não programadas. Sobre as decisões programadas 01. A / 02. D / 03. D / 04. D / 05. D
assinale a alternativa incorreta.
(A) Aplicam-se a situações que não têm precedentes.
(B) Ocorrem com frequência na organização.
(C) São as mais fáceis de serem tomadas.
(D) São mais fáceis de serem estruturadas em etapas.
(E) São repetitivas.
Gerenciamento de conflitos.
02. As decisões que são tomadas na organização pela
gerência intermediária como gerentes de divisão ou de
departamento são:
(A) Estratégicas. Administração (ou Gestão) de conflitos56
(B) Operacionais.
(C) Programadas. Cada ser humano é único em seu modo de pensar
(D) Táticas. e agir; constitui-se de diferentes experiências,
vivenciadas ao longo da vida.
03. De acordo com as situações a que se aplicam, as A diversidade humana resulta em formas também distintas
decisões podem ser classificadas de diversas formas. de enxergar situações. Este fato, na convivência entre duas

55
Stakeholders: todos os agentes envolvidos no negócio de uma empresa são NASCIMENTO, M. N.; EL SAYED, K. M. Administração de Conflitos. Disponível
56

eles: fornecedores, concorrentes, clientes, colaboradores, acionistas, em: http://www.someeducacional.com.br/apz/gestao_conflitos/4.pdf.


pesquisadores, investidores, parceiros em geral.

50
50
NOÇÕES DE ADMINISTRAÇÃO
ou mais pessoas, pode gerar conflitos. A ausência destes Os Conflitos interpessoais nas organizações se dão
não significa, necessariamente, que não existem problemas. entre duas ou mais pessoas e podem ocorrer por
Entretanto, conflitos também proporcionam aos indivíduos a vários motivos: diferenças de idade, sexo, valores,
possibilidade de crescimento. crenças, por falta de recursos materiais, financeiros,
Importante saber que conflitos existem desde o início da por diferenças de papéis, diferenças de percepção em
humanidade, fazem parte do processo de evolução dos seres vários contextos, podendo ser divididos em dois tipos:
humanos e são necessários para o desenvolvimento e o
crescimento de qualquer sistema familiar, social, político e -Hierárquicos: colocam em jogo as relações com a
organizacional. autoridade existente. Ocorre quando a pessoa é responsável por
Nesse contexto, surge a administração de conflitos, algum grupo, não encontrando apoio junto ao seus subordinados,
que consiste exatamente na escolha e implementação das e vice-versa. Neste caso, as dificuldades encontradas no dia a dia
estratégias mais adequadas para se lidar com cada tipo de deixam a maior parte das pessoas envolvidas desamparada
situação. quanto à decisão a ser tomada.
Para a correta administração do conflito é importante que
sejam conhecidas as possíveis causas que levaram ao seu -Pessoais: dizem respeito ao indivíduo, à sua maneira de ser,
surgimento. Dentre elas, é possível citar: agir, falar e tomar decisões. As rixas pessoais fazem com que as
pessoas não se entendam e, portanto, não se falem. Em geral, esses
- Experiência de frustração de uma ou ambas as conflitos surgem a partir de pequenas coisas ou situações nunca
partes: incapacidade de atingir uma ou mais metas e/ou de abordadas entre os interessados. O resultado é um confronto
realizar e satisfazer os seus desejos, por algum tipo de tácito que reduz, em muito, a eficiência das relações.
interferência ou limitação pessoal, técnica ou comportamental;
- Diferenças de personalidade: são invocadas como Entre os vários aspectos do conflito, alguns podem ser
explicação para as desavenças tanto no ambiente familiar como considerados como negativos e aparecem com frequência
no ambiente de trabalho, e reveladas no relacionamento diário dentro das organizações.
através de algumas características indesejáveis na outra parte Os mais visíveis podem ser identificados nas seguintes
envolvida; situações58:
- Metas diferentes: é muito comum estabelecermos e/ou - Quando desviam a atenção dos reais objetivos,
recebermos metas/objetivos a serem atingidos e que podem ser colocando em perspectiva os objetivos dos grupos envolvidos
diferentes dos de outras pessoas e de outros departamentos, o no conflito e mobilizando os recursos e os esforços para a sua
que nos leva à geração de tensões em busca de seu alcance. A solução;
administração de conflitos consiste, exatamente, na escolha e - Quando tornam a vida uma eterna derrota para
implementação das estratégias mais adequadas para se lidar os grupos de “perdedores habituais”, interferindo na sua
com cada tipo de situação; percepção e na socialização daqueles que entram na
- Diferenças em termos de informações e organização;
percepções: costumeiramente tendemos a obter informações - Quando favorecem a percepção estereotipada a
e analisá-las à luz dos nossos conhecimentos e referenciais, sem respeito dos envolvidos, como ocorre frequentemente em
levar em conta que isto ocorre também com o outro lado com organizações. Se por um lado existem os estereótipos genéricos
quem temos que conversar e/ou apresentar nossas ideias, e que referentes às categorias profissionais, dentro de cada
este outro lado pode ter uma forma diferente de ver as coisas. organização, além dos tipos que fazem parte de sua cultura
individual, como seus heróis, mitos, tipos ideais, começam a
Para Chiavenato57, "Basicamente, existem quatro surgir seus perdedores, ganhadores, culpados e inimigos.
condições antecedentes dos conflitos:
- Ambiguidade de papel; No entanto, existem potenciais efeitos benéficos dos
- Objetivos concorrentes; conflitos, a saber:
- Recursos compartilhados; - São bons elementos de socialização, pois oferecem
- Interdependência de atividades. aos novos participantes de um grupo a sensação de
envolvimento com alguma causa;
Existem vários tipos de conflitos e sua - Ajudam a equilibrar as relações de poder dentro
identificação pode auxiliar na escolha da estratégia da organização, pois em qualquer episódio de conflito pode
mais adequada para administrá-lo: haver diferentes ganhadores (independentemente das
Conflito latente: não é declarado e não há, mesmo por percepções anteriores);
parte dos elementos envolvidos, uma clara consciência de sua - Propiciam a formação de alianças com o objetivo
existência. Eventualmente não precisam ser trabalhados; de ganhar num conflito específico, mas também de
Conflito percebido: os elementos envolvidos percebem, garantir mais poder.
racionalmente, a existência do conflito, embora não haja ainda
manifestações abertas do mesmo; Sejam eles positivos ou negativos, os conflitos
Conflito sentido: É aquele que já atinge ambas as partes podem ser considerados úteis pelo papel que
e no qual há emoção e forma consciente; desempenham na vida das pessoas.
Conflito manifesto: trata-se do conflito que já atingiu O chamado comportamento político na organização,
ambas as partes, já é percebido por terceiros e pode interferir também inevitável, tem uma forte vinculação com o conflito,
na dinâmica da organização. pois sua relação é direta, ou seja, quanto mais conflito mais
comportamento político.

57
Chiavenato, Idalberto. Gestão de pessoas 58
NASCIMENTO, M. N.; EL SAYED, K. M. Administração de Conflitos.
Disponível em:
http://www.someeducacional.com.br/apz/gestao_conflitos/4.pdf.

511
NOÇÕES DE ADMINISTRAÇÃO
Assim, quando há dúvidas sobre a sobrevivência da - tenta convencer a outra parte de que sua conclusão está
organização, sobre sua resposta às necessidades correta e a dela está equivocada;
organizacionais, sobre aspectos sucessórios, mais voltada para - leva a outra parte a aceitar a culpa por um problema
a formação de alianças e para negociações será a atuação de qualquer.
seus membros.
Lidar com o conflito implica trabalhar com grupos e tentar Colaboração
romper alguns dos estereótipos vigentes na organização, - contempla os interesses das partes envolvidas no conflito;
sabendo que essas mesmas estratégias deverão ser repetidas - busca um resultado benéfico para todas as partes
periodicamente. envolvidas.
Criar tarefas a serem executadas em conjunto por grupos
diferentes é uma forma de garantir que seu cumprimento seja Evitação
reconhecido pela potencialização do trabalho dos grupos. - trata-se de estilo considerado não assertivo e não
Quaisquer estratégias de confronto podem ser utilizadas cooperativo;
caso o conflito já seja franco, desde que exista entre as partes - evita todo e qualquer envolvimento com o conflito,
alguém que desempenhe um papel moderador. chegando a negar sua existência e o contato com as pessoas que
podem causá-lo.
Como administrar os conflitos59
Acomodação60
Os seguintes passos são considerados de suma - trata-se de estilo considerado não assertivo e cooperativo;
importância: - a parte que utiliza este estilo tende a apaziguar a situação,
a) criar uma atmosfera afetiva; chegando a colocar as necessidades e interesses da outra parte
b) esclarecer as percepções; acima dos seus.
c) focalizar em necessidades individuais e compartilhadas;
d) construir um poder positivo e compartilhado; Compromisso
e) olhar para o futuro e, em seguida, aprender com o passado; - este estilo se encontra no padrão médio de assertividade e
f) gerar opções de ganhos mútuos; cooperação, em que uma das partes envolvidas no conflito
g) desenvolver passos para a ação a ser efetivada; desiste de alguns pontos ou itens, levando a distribuir os
h) estabelecer acordos de benefícios mútuos. resultados entre ambas as partes.

Para que a negociação possa ocorrer, é necessário que Questões


ambas as partes tenham as seguintes capacidades:
01. Mudanças organizacionais frequentemente geram
Saber comunicar insegurança nas pessoas. Se as organizações comunicam
- sem diálogo não há comunicação nem solução possível constantemente seus objetivos e agem de modo transparente e
para os problemas; coerente, os conflitos e medos tendem a ser minimizados.
- a maioria dos erros, omissões, irritações, atrasos e conflitos ( ) Certo ( ) Errado
é causada por uma comunicação inadequada.
02. Uma causa frequente de conflitos nas organizações é
Saber ouvir (A) a ambiguidade de papéis.
- ouvir ativamente, pois metas e intenções não (B) a existência de objetivos compartilhados.
compreendidas levam sempre a uma resolução sem sucesso; (C) a limitação de recursos.
- demonstrar interesse genuíno pela pessoa que fala e pelo (D) a hierarquia de responsabilidades.
assunto; (E) o excesso de liberdade.
- evitar criticar ou tentar dirigir a conversa;
- adotar uma posição afirmativa, mostrando respeito pela 03. Se por um lado os conflitos são inerentes à vida social
outra pessoa. dos indivíduos, deve-se compreender a mediação de conflitos
como:
Saber perguntar (A) a atividade que estimula tais conflitos, considerando
Saber perguntar é outra faceta do ouvir ativamente, pois que o mediador, figura parcialmente envolvida, deve
quem pergunta conduz a conversa. Quanto ao estilo a ser compreender a impossibilidade de se construir consenso
adotado, é recomendável a adoção de um que leve à solução do quando as pessoas envolvidas pensam diferentemente.
conflito da forma mais pacífica possível. O que vai definir seu (B) o processo que compõe uma interlocução entre as
atual estilo de administrar conflitos está diretamente ligado a opiniões diferenciadas em que o mediador, que deve ser neutro,
duas importantes características de comportamento: assume o papel principal de incentivá-las para que possa vencer
assertividade e cooperação. o melhor argumento.
(C) a situação em que são apresentados casos antagônicos
A seguir, relacionam-se alguns estilos e algumas de e que o mediador, em uma postura imparcial, deve conduzir o
suas características: debate e se posicionar a favor da melhor ideia.
(D) o procedimento no qual os participantes com a
Competição assistência de uma pessoa imparcial – o mediador – colocam as
- busca satisfação dos interesses, independentemente do questões em disputa com o objetivo de desenvolver opções,
impacto que isto possa causar na outra parte envolvida;

59
NASCIMENTO, M. N.; EL SAYED, K. M. Administração de Conflitos. 60
NASCIMENTO, M. N.; EL SAYED, K. M. Administração de Conflitos.
Disponível em: Disponível em:
http://www.someeducacional.com.br/apz/gestao_conflitos/4.pdf. http://www.someeducacional.com.br/apz/gestao_conflitos/4.pdf.

52
52
NOÇÕES DE ADMINISTRAÇÃO
considerar alternativas e chegar a um acordo que seja estratégicas como a orçamentação por produto, contabilidade
mutuamente aceitável. gerencial e normatização do trabalho.
Portanto, no novo modelo de Estado brasileiro, adotado
04. Uma organização do trabalho embasada em atividades com a Reforma do Estado, o instrumento mais poderoso para
individualizadas e especializadas elimina a possibilidade de se trabalhar com resultados e modernizar o serviço público é o
conflitos no trabalho. Contrato, em sentido amplo e estrito.
( ) Certo ( ) Errado Amplo quando compreende o acordo de vontades dos
atores da Administração Pública e das instituições que
05. O conflito faz parte da natureza humana. Sempre que representam, e estrito no sentido de se fazer os pactos, de se
houver alguma divergência de opiniões e uma das partes restringir ou focar as atividades das organizações ao previsto e
resolver intervir na situação de forma que a sua opinião acordado (BRASIL, 1995).
prevaleça sobre a do outro, teremos um conflito. O que A contratualização é uma prática importante na
diferencia as pessoas é a forma que cada uma escolhe para Administração Pública pós-Reforma do Estado, pois possibilita
resolver os conflitos que surgem. Mas os conflitos nem sempre instituir práticas de planejamento, avaliação e
são negativos. Quando bem gerenciados, podem apresentar monitoramento da execução por parte do Estado (órgão
resultados positivos, principalmente quando estimulam os contratante) e o terceiro ou órgão público contratado. Os
participantes a criar novas técnicas e estratégias para resolução Contratos de Resultados podem ser firmados com qualquer
dos problemas. Analise as afirmações relacionadas ao tema natureza de instituição, seja ela estatal (pública), do terceiro
conflitos elencadas abaixo e assinale a que estiver correta: setor (associações e fundações) e mesmo privadas (empresas e
(A) A má comunicação pouco influencia a geração de consórcios).
conflitos. Na verdade, a proposta não é de se abandonar todos os
(B) A situação ideal em uma organização é quando não há meios de contratação já utilizados na Administração Pública,
tipo algum de conflito. mas aperfeiçoar seus instrumentos, de modo que se possa
(C) Os valores e a percepção dos indivíduos podem ser um prever com a máxima exatidão os serviços e atividades que
gerador de conflitos. estão sendo contratados, utilizando-se para isso os indicadores
(D) Quando usa a coação para resolver os conflitos, o de desempenho, que são quantificáveis para se mensurar o
mediador busca contar com a benevolência das partes atingimento dos resultados pactuados e, também, a própria
antagônicas. performance da organização.
(E) A heterogeneidade do quadro de pessoal não se Os indicadores de desempenho são variados e
constitui num fator de geração de conflitos. relacionados a um determinado serviço ou atividade, portanto,
não podem ser gerais para todas as organizações. Eles indicam
Respostas de forma quantitativa, ou por ações mensuráveis, as atividades
da organização específica.
01. Certo / 02. A / 03. D / 04. Errado / 05. C Existem vários métodos para se construir indicadores de
desempenho nas organizações, mas o mais prático é quando
você parte da identificação dos macro-processos da organização
e constrói uma cadeia de valor a partir deles até se chegar às
formas de identificação do cumprimento das atividades que lhe
são relacionadas (SIQUEIRA, 2005).
Depois de se identificar os indicadores da organização,
Novas formas de gestão de serviços parte-se para estipulação das metas, que são as quantificações
públicos: formas de supervisão e dos indicadores. As metas podem variar de acordo com a
contratualização de resultados; vontade das partes contratantes e estarão fortemente
relacionadas à etapa de negociação do instrumento contratual.
prestação de serviços públicos e novas
tecnologias. No processo de contratualização de resultados
existem 3 (três) fases importantes que são imprescindíveis
ao bom resultado da prática e não podem deixar de existir:
Negociação – esta fase inicia com a intenção de contratar
Formas de supervisão e Contratualização de os serviços por um órgão estatal e a elaboração do rol de
Resultados61 atividades a serem contratadas. A partir daí, identifica-se o
outro órgão ou entidade a ser contratada e se inicia o processo
A contratualização de resultados é uma característica que de negociação, que vai da identificação dos indicadores de
vem ganhando espaço na nova e positiva visão de gestão desempenho, estipulação das metas e a respectiva
pública, por meio de contratos entidades públicas (estatais ou orçamentação, que é a parte financeira do Contrato de
não-estatais), orientada e sem fins lucrativos são submetidas a Resultados e contém as despesas e receitas da organização
um rigoroso controle de responsabilização, diante das contratada, refletindo as obrigações de parte a parte.
entidades formuladoras de políticas públicas. Construção do Instrumento – esta fase consiste na
É apontada como uma das principais estratégias da formalização de tudo o que foi negociado, aperfeiçoando-se e
inovadora Gestão Pública, para muitos a contratualização de aferindo-se todos os pontos acordados. Um passo importante
resultados anda lado a lado de importantes ferramentas para a construção do instrumento é estabelecer as sanções e

61
http://www.administradores.com.br/artigos/economia-e-financas/gestao-por- Internacional del CLAD sobre la Reforma del Estado y de la Administración Pública,
resultados-e-contratualizacao-de-servicos-a-experiencia-de-minas-gerais/59742/ Santiago, Chile, 18 - 21 Oct. 2005
Flávio Carneiro Guedes Alcoforado. Contratualização e eficiência no setor público:
as organizações sociais Flávio Carneiro Guedes Alcoforado. X Congreso

533
NOÇÕES DE ADMINISTRAÇÃO
métodos de fiscalização e prestação de contas dos resultados do que lhes são orçados. Além disso, existem casos em que já é
instrumento contratual. possível a utilização dos recursos obtidos com a redução de
Gerenciamento – esta fase é permanente, após a despesas para investimento na própria atividade do órgão,
assinatura do instrumento contratual, compreendendo todas as como é o caso do Governo do Estado de Minas Gerais.
atividades de supervisão da entidade, monitoramento dos É importante destacar que além dos Acordos de Resultados
indicadores de desempenho pactuados e avaliação dos e Contratos de Gestão, os Convênios também podem continuar
resultados alcançados pela parte contratada. Essa fase é a ser utilizados dentro da política de Contratualização,
realizada diretamente pelo órgão público contratante. devendo-se, apenas, mudar o seu foco para desempenho
através de resultados previstos em indicadores.
Na contratualização, os instrumentos contratuais que Alguns órgãos estatais já estão utilizando isso na prática.
podem ser utilizados, em sua maioria, são os mesmos já Existe, também, uma outra forma de contratualização de
praticados pela Administração Pública, entretanto, o foco e a resultados entre órgãos do Estado que se instrumentaliza
forma de se construir o instrumento é que muda, já que se passa através do PPA – Plano PluriAnual, e que utiliza a figura de
de uma visão anterior focada no processo (meio), para uma gerentes de programa para firmar compromissos de resultados.
nova visão focada nos resultados (fins). É uma forma alternativa de contratualização, já que o controle,
neste caso, fica centralizado em um único órgão do Estado que
Desse modo, abandona-se a tradicional forma de se avaliar acompanha todos os demais.
os contratos e a prestação dos serviços somente pela correta Entretanto, a utilização desse mecanismo sem a
utilização dos recursos, através da tradicional prestação de concomitante utilização do acordo de resultados ou contrato de
contas financeira. Agora esse não é mais um critério de gestão não tem possibilidade melhoria do desempenho geral da
avaliação do Contrato e dos serviços prestados! organização, já que a sua abrangência é mais geral, não se
traduzindo em diversos indicadores de desempenho de todas as
O lema é: autonomia de gestão em troca de atividades do contratado ou acordado.
compromisso prévio com resultado.
Contratos Setor Público X Terceiro Setor
Na contratualização, avalia-se o cumprimento do contrato
pela avaliação do alcance dos resultados pactuados, Esta modalidade de contratualização é específica para os
através da verificação quanto ao atingimento das metas serviços sociais. Tem sido amplamente utilizada no Brasil,
previstas para os indicadores de desempenho. Agora a depois de já ter grande êxito em diversos outros países. Trata-
prestação de contas financeira e o adequado uso dos recursos é se da parceria estabelecida com organizações da sociedade civil,
uma obrigação da organização contratada, mas não é condição para a prestação de serviços públicos.
para se avaliar desempenho. A contratualização com as entidades do Terceiro Setor pode
ser instrumentalizada através dos Contratos de Gestão e
Contratos Setor Público X Setor Público Termos de Parceria, que são compostos de resultados,
indicadores e metas que refletem os serviços que estão sendo
A contratualização entre órgãos do próprio Estado vem contratados pelo Poder Público junto a tais organizações. Existe
sendo amplamente utilizada pelas 3 (três) esferas da desde muito tempo, a prática de se realizar convênios com as
Administração Pública, Federal, Estadual e Municipal e tem-se organizações do terceiro setor, muito embora o foco, conforme
revelado um importante avanço para a eficiência dos serviços já relatado anteriormente, era muito diferenciado, já que o
públicos. resultado não era medido por indicadores de desempenho.
Os nomes utilizados para o instrumento contratual nesse As organizações do Terceiro Setor podem ser constituídas,
caso têm variado conforme o Estado ou Governo. Alguns têm juridicamente, como Associações ou Fundações, de acordo com
adotado o nome geral de Contratos de Gestão e outros têm o disposto no Código Civil Brasileiro, em vigor. Estas, para
utilizado o Acordo de Resultados, embora ambos tenham as firmarem Contratos de Gestão ou Termos de Parceria com o
mesmas características, natureza e alcance. Estado, devem ser qualificadas como Organizações Sociais ou
Os Acordos de Resultado ou Contratos de Gestão, firmados OSCIPs – Organizações da Sociedade Civil de Interesse Público.
entre 2 (dois) órgãos da Administração Pública pressupõem a Ambos, OS – Organização Social e OSCIP, são títulos que
estipulação de compromissos assumidos de parte a parte com podem ser concedidos pelas Esferas de Governo que têm
uma determinada política pública e com os seus resultados. competência legal para tanto.
Antigamente, pelo antigo modo, o órgão só precisava funcionar, A Organização Social é apropriada para a execução dos
independente do seu desempenho relacionado à quantidade ou serviços sociais que requerem flexibilidade de gestão e
qualidade dos serviços que prestava. agilidade, sem ter que se submeter às leis de licitação públicas,
Agora ele tem que atingir resultados e prestar contas deles, nem a contratação de pessoal por concurso. Isto não quer dizer
demonstrando eficiência. O Órgão estatal contratante passa a que a organização não disponha de controles. Ao contrário,
utilizar-se de uma peça existente no Acordo de Resultados existem regras que se adequam às atividades desenvolvidas
chamada Plano de Trabalho. pela organização, mas que não acarretem demora, nem
Nessa peça, estão traduzidas em indicadores de utilizem etapas que venham a dificultar ou atrasar a prestação
desempenho todas as políticas públicas que são de competência do serviço público à população. Além disso, a Organização
dos órgãos, com as diretrizes estipuladas pelo órgão contratante Social utiliza controles de empresa privada, principalmente em
que, em regra, é da Administração Direta e é responsável por relação às áreas contábil, financeira, suprimentos e recursos
tais políticas públicas. humanos (BRASIL, 1997).
Em algumas esferas de Governo, que têm regulamentação Além disso, todas as avaliações do serviço prestado pela
própria para o Acordo de Resultados, sua assinatura confere ao organização social são monitoradas permanentemente por uma
Órgão contratado algumas flexibilidades gerenciais que podem comissão da Entidade Supervisora, no caso o órgão público que
compreender: autonomia para a gestão de RH, limites de contrata os serviços da Organização Social através do Contrato
compras diferenciados e mobilidade de utilização dos recursos de Gestão. Essa comissão de avaliação tem por objetivo

54
54
NOÇÕES DE ADMINISTRAÇÃO
acompanhar o desempenho da organização, baseando-se nos Novas práticas e ferramentas administrativas e gerenciais
indicadores de gestão, qualidade e produtividade que são surgiram nas últimas décadas como consequência dessas
previstos no Contrato de Gestão estabelecido pela mudanças; algumas delas, como o PDCA e o Diagrama de
Administração Pública com a organização. Pareto, começaram a ser utilizadas ainda na década de 1950,
Através do Contrato de Gestão, o Estado transfere recursos no Japão.
para a Organização Social executar os serviços que
anteriormente eram por ele prestados. Além disso, a BENCHMARKING
organização se obriga a captar mais recursos no mercado a fim O glossário do GesPública define benchmarking como
de ampliar a oferta dos serviços à população. Esses recursos “um processo contínuo de medição de produtos, serviços e
podem ser captados através de venda de bilheteria, projetos práticas (processos), em relação aos concorrentes mais
culturais com incentivos fiscais, doações ou mesmo competitivos, ou às empresas reconhecidas como líderes”.
contribuições de associados e empresas, dentre outras fontes A precursora na utilização dessa técnica foi a empresa
(Brasil, 1997). Xerox, em 1979. O benchmarking é um processo
contínuo para avaliar serviços, produtos e práticas da
Em geral, no Contrato de Gestão são previstos 3 tipos organização, comparando-os com seus concorrentes mais
de metas para as Organizações Sociais (ALCOFORADO, fortes ou empresas que são reconhecidas como líderes
2004): empresariais na sua área de atuação.
• Metas Organizacionais – são aquelas que dizem Procura-se identificar o “melhor do melhor”, os
respeito à gestão da organização e medem a eficiência dos fatores-chaves que influenciam a produtividade e a qualidade
administradores que a conduzem. São importantes para a dessas empresas, com a finalidade de aprimorar os produtos
profissionalização dos serviços prestados. Incluem-se os e serviços da organização.
indicadores econômico-financeiros, de organização interna e as Para utilizar o benchmarking a organização precisa:
metas de captação de recursos próprios. conhecer suas operações e avaliar seus pontos fortes e fracos
• Metas de Produção – são as relacionadas diretamente (processos e diagnósticos internos); conhecer os concorrentes e
à atividade fim da organização e medem a capacidade da organizações líderes do mercado, identificar suas habilidades e
mesma em alcançar índices adequados de prestação dos seus pontos fortes e fracos para compará-los com os pontos
serviços a que se propõe. fortes e fracos de sua empresa; e implantar na organização o
• Metas Sociais – são aquelas que promovem a difusão e “melhor do melhor”: os pontos fortes dos concorrentes – e, se
o acesso democrático dos serviços públicos executados pela possível, ultrapassá-los.
organização a parcelas mais carentes da população e podem ser
beneficiados com o acesso, promovendo inclusão social. Etapas do Processo de Benchmarking63
Dependendo do serviço público prestado, podem-se estabelecer
parcerias com entidades educacionais ou mesmo filantrópicas Etapa 1: Planejamento
para seu alcance. Na etapa do planejamento, são necessários alguns
Devido às especificidades culturais de cada setor e os procedimentos essenciais:
métodos de trabalho de cada grupo de especialistas dessas
organizações, o formato de Organizações Sociais propicia a - Identificação de áreas, processos ou atividades a serem
flexibilidade de gestão necessária a uma melhor atuação, melhoradas:
devido à possibilidade de incorporar as práticas de gestão Na primeira parte da fase do planejamento, a organização
próprias de cada campo ou setor, o que facilita, dado o respeito define quais atividades pretende aperfeiçoar por meio do
a tais peculiaridades, obtenção de ganhos de produtividade nos processo de Benchmarking. Essa seleção de atividades deve
serviços e de maior satisfação na prestação dos serviços considerar o impacto potencial que as melhorias podem trazer
públicos. à organização. Podem-se selecionar tanto atividades de gestão,
como de captação de recursos e de gestão de voluntários, como
NOVAS TECNOLOGIAS62 ações e projetos destinados à população alvo.

Com o aumento da competitividade e avanço da - Definição de critérios e indicadores para mensuração de


globalização, as novas tecnologias tornaram-se um desafio atividades:
a ser enfrentado também pelas entidades públicas. Durante a próxima esta (de coleta de dados) será necessário
Em decorrência dessas fortes mudanças oriundas das empresas estudar como as organizações realizam suas atividades. Para
privadas, constatou-se a necessidade de novas formas de tanto, e indispensável definir critérios que possibilitem alguma
administrar/gerir a coisa pública, pois esta encontrava-se comparação entre essas atividades, o que é feito na segunda
incapacitada de dar conta das novas demandas impostas pela parte da fase do planejamento. É ainda necessário definir
reestruturação produtiva e pela mundialização da economia. parâmetros e indicadores para a mensuração de custos,
As estruturas governamentais foram fortemente afetadas qualidade e prazos utilizados na execução das atividades. Tais
pelas inovações tecnológicas, pelo ritmo intenso/contínuo das indicadores serão utilizados para mensurar as atividades
mudanças, pelo surgimento de uma economia global pós- desenvolvidas pelas diferentes organizações que participarem
industrial e por uma sociedade baseada no conhecimento e na do Benchmarking.
informação. Com as inovações tecnológicas, não só as
organizações privadas, mas também as públicas, - Identificação de organizações participantes/parceiras:
mudaram suas configurações, a começar pela Inglaterra, Deve ser selecionada as organizações serão incluídas no
Estados Unidos, França, Alemanha, Canadá, Nova Zelândia e, processo de Benchmarking, o que depende diretamente do tipo
posteriormente a América Latina e o Brasil. de processo a ser efetuado. Em princípio, podem participar do

62
Paludo, Augustinho. Administração pública. – 3. ed. – Rio de Janeiro: 63
FRANCO, D. H.; RODRIGUES, R. A.; CAZELA, M. M. Tecnologias e
Elsevier, 2013. Ferramentas de Gestão. Campinas, SP: Editora Alínea, 2013.

555
NOÇÕES DE ADMINISTRAÇÃO
Benchmarking quaisquer organizações que tenham reputação Em regra, o benchmark é realizado quando a empresa está
de excelência na área estudada, não devendo ser, em crise, quando ocorrem ataques competitivos da
necessariamente, da mesma área de atuação. concorrência, quando se pretende implementar melhorias
significativas, ou quando se pretende atuar em novo ramo de
Etapa 2: Coleta de dados negócio. Um dos principais tabus à utilização do
Realizada a etapa de planejamento, inicia-se a etapa da benchmarking consiste em convencer os gestores de que seus
coleta de dados nas organizações envolvidas. O ideal é que a desempenhos podem ser melhorados a partir de experiências
organização promotora do Benchmarking obtenha, bem sucedidas de outras empresas.
inicialmente, o máximo de informações sobre as demais
entidades por meio de fontes secundárias como jornais, MELHORIA CONTÍNUA (KAIZEN)
revistas, pesquisas, bancos de dados, publicações das próprias A melhoria contínua é uma técnica de mudança
organizações e internet. organizacional lenta, suave e ininterrupta, centrada nas
Com base nesse levantamento inicial de informações a atividades em equipes. Visa aumentar a qualidade
organização pode partir para o processo de coleta de dados dos produtos e serviços dentro de programas a longo
primários, em cada uma das entidades selecionadas, prazo. Seu foco é a melhoria gradual e contínua, através da
empregando diferentes métodos: colaboração e participação das pessoas, para realizarem suas
- entrevistas com roteiros previamente elaborados; tarefas um pouco melhor a cada dia.
- realização de pesquisas empregando questionários
estruturados; Nem sempre os grupos de pessoas podem ser considerados
- oficinas ou reuniões de trabalho com gestores das uma equipe; para que sejam definidas como tal, devem
organizações; trabalhar juntas, interagir umas com as outras, possuir
- visitas de campo à organização e seus projetos. habilidades complementares e ter objetivos comuns a perseguir.
Equipes são mais que um “grupo de pessoas”. Regra
Etapa 3: Análise e comparações geral, as equipes são escolhidas, treinadas, motivadas,
Na etapa de Análises e comparações, o conjunto das avaliadas, perseguem objetivos/metas comuns, e, ao final, são
informações coletadas deve ser sistematizado e analisado, recompensadas pelos resultados obtidos.
podendo constituir um relatório sobre as atividades
desenvolvidas pelas diferentes organizações. Em seguida, é O Kaizen promove a melhoria através da
necessário comparar as diferentes atividades empregando os eliminação de problemas identificados, objetivando
parâmetros definidos na etapa de planejamento. Essa fazer melhor as atividades/tarefas e conquistar resultados
comparação resultará em uma classificação e na produção de específicos relacionados à satisfação dos clientes, ou
um ranking das atividades, segundo os diferentes fatores relacionados com a redução de custos de fabricação, estoques e
analisados, como qualidade, custo etc. distribuição.
De posse deste ranking, a organização pode comparar o
desempenho de seus próprias atividades com o das demais Dez princípios encontram-se relacionados ao uso
empresas envolvidas e identificar quais poderiam ser dessa ferramenta:
aperfeiçoadas, aplicando-se o conhecimento adquirido durante 1° promover aprimoramentos contínuos;
o processo de Benchmarking. O resultado das análises pode ser 2° enfatizar os clientes;
consolidado em um relatório de Benchmarking, a ser 3° reconhecer os problemas abertamente;
disseminado por toda a organização. 4° promover a discussão aberta e franca;
5° criar e incentivar equipes de trabalho;
Etapa 4: Elaboração e implementação do plano de 6° gerenciar projetos por intermédio de equipes
mudanças multifuncionais;
A última etapa, a elaboração e implementação do plano de 7° incentivar o relacionamento entre as pessoas;
mudanças do processo de Benchmarking inicia com a 8° desenvolver a autodisciplina;
elaboração de um plano para implementação das mudanças nos 9° comunicar e informar todas as pessoas;
processos que foram considerados prioritários. O ideal, nessa 10° treinar e capacitar todas as pessoas.
fase, é contar com a participação do máximo possível de todos
os envolvidos no processos que serão aperfeiçoados com base Downsizing (Enxugamento)
na realização do plano. A teoria que fundamenta o uso dessa ferramenta surgiu no
Além disso, é importante que o grupo envolvido no processo final do século XX. Odownsizing promove redução de
de implementação das mudanças esteja atento à necessidade de níveis hierárquicos (mediante a fusão de departamentos ou
rever objetivos e metas estabelecidos. gerências intermediárias) e o enxugamento
organizacional, para reduzir as operações ao essencial do
Os Principais Benefícios negócio (core business); e transfere as operações não
Enquanto nas organizações privadas essa técnica é utilizada essenciais para terceiros com capacidade de fazê-lo
para aumentar a competitividade, nas instituições públicas melhor e mais barato (terceirização).
utiliza-se para identificar as organizações de excelência com
vistas a promover melhorias na prestação de serviços públicos. O enxugamento substitui a cultura baseada na desconfiança
e no policiamento externo (que utiliza um contingente
Área melhorar a prestação dos excessivo de comandos e controles) por uma nova cultura, que
BENCHMARKING ĺ investe em treinamento para a qualidade e apoia a inovação, a
Pública serviços
Principal iniciativa e o comprometimento das pessoas.
Benefício Setor aumentar a O downsizing reduz custos e aumenta a flexibilidade
ĺ
Privado competitividade da organização e a capacidade de resposta às
constantes mudanças no ambiente. Ao mesmo tempo em

56
56
NOÇÕES DE ADMINISTRAÇÃO
que tem sido muito utilizada, tem sido criticada em face do promovendo mudanças nas organizações e dando sustentação
grande número de demissões decorrentes do enxugamento. aos programas de Qualidade e Produtividade.
Várias palavras diferentes vêm sendo utilizadas para definir
Ciclo PDCA as palavras japonesas que compõem os 5S: Seiri – senso de
O Ciclo PDCA teve origem na década de 1920, com utilização, seleção; Seiton – senso de ordenação,
Shewhart, nos Estados Unidos, mas tornou-se conhecido como arrumação;Seisoh – senso de limpeza; Seiketsu – senso de
ciclo de Deming a partir de 1950, no Japão. Para o glossário do asseio, saúde, padronização; Shitsuke – senso de autodisciplina,
GesPública, Ciclo PDCA é uma ferramenta que busca a harmonia, educação.
lógica para fazer certo desde a primeira vez. O 5S é um programa voltado para os funcionários,
que busca promover os bons hábitos no ambiente de trabalho,
como forma de melhorar a produtividade.
Sua utilização pelas organizações vem proporcionando
vantagens como: segurança no trabalho, hábitos em geral
mais saudáveis, maior limpeza e organização, combate ao
desperdício, melhoria nos relacionamentos, melhoria dos
produtos e serviços, espírito de equipe, e melhoria geral do
ambiente de trabalho.

Diagrama de Pareto
Diagrama ou princípio de Pareto é uma forma especial
de gráfico de barras verticais(histograma) que permite
determinar quais problemas resolver e qual a prioridade. Esse
histograma direciona os esforços para os problemas
mais importantes, visto que permite selecionar e visualizar
Fonte: http://www.ciconsulting.com.br/nossos- itens ou fatores em sua ordem crescente de importância, e
servicos/gestao-de-ativos.html utilizá-los para melhora da qualidade, redução de custos etc.

É uma técnica simples para o controle de


processos, que também pode ser utilizada para o
gerenciamento contínuo das atividades de uma organização. É
um método usado para controlar e melhorar as
atividades de um processo, desta forma, contribui com a
qualidade e com a redução de custos de forma contínua.
O PDCA padroniza as informações de controle, reduz e evita
erros lógicos, facilita o entendimento das informações, melhora
a realização das atividades e proporciona resultados mais
confiáveis.
Também chamado Ciclo da Melhoria Contínua, o PDCA é
uma “ferramenta oficial da qualidade”, utilizado em processos
de trabalho com vistas a maximizar a eficiência e
alcançar a excelência de produtos e serviços. Em regra,
quando aplicado na melhoria de processos, significa estabelecer
uma nova diretriz de controle, da qual decorre um novo
nível de controle.
O PDCA parte da insatisfação com o “estado atual
das coisas” e analisa os processos com vistas a realizá-los de
maneira otimizada. Inclui as seguintes etapas: planejamento
(Plan): estabelecer objetivos, metas e os meios para alcançá-
los; execução (Do): executar as atividades propostas no
planejamento; controle/verificação (Check/Control):
monitora/controla a execução e verifica o grau de cumprimento
do que foi planejado; Ação Avaliativa/Corretiva (Act to O diagrama de Pareto baseia-se em fatos e dados e poderá
corret): identifica eventuais falhas e corrige-as, a fim de ser usado sempre que for preciso ressaltar a importância
melhorar a execução das atividades. relativa entre os vários problemas ou condições, no sentido
O PDCA é uma ferramenta de importância de escolher o ponto de partida para a solução de um
fundamental para a análise e melhoria de processos problema: identificar a causa básica de um problema ou
organizacionais, para a eficácia do trabalho em equipe e para o avaliar o progresso de uma operação. Identifica as causas
alcance das metas estabelecidas. que mais se repetem nos problemas que estão
continuamente sendo enfrentados pela organização.
Programa 5S É uma das ferramentas mais antigas e considera que
Criado no Japão, o Programa 5S proporciona melhor nem todos os itens merecem o mesmo tratamento ou atenção
organização, arrumação, limpeza, asseio e disciplina, da administração. Esse diagrama é também denominado de “80
eliminando os desperdícios, aumentando a produtividade e por 20”. “Para ele, 80% do volume de problemas são
criando um excelente ambiente de trabalho. Senso significa constituídos por apenas 20% de eventos causadores”...
faculdade de apreciar, sentir, julgar. Os cinco sensos estão “diagrama de barras verticais para dirigir a atenção aos
problemas mais importantes e prioritários, localizados nas

577
NOÇÕES DE ADMINISTRAÇÃO
barras mais altas, deixando as barras mais baixas para d. Aprendizagem/crescimento organizacional.
constatação posterior” (Idalberto Chiavenato, 2006). Para analisar o negócio sob o ponto de vista daquilo que é
básico para alcançar o futuro com sucesso. Considera as pessoas
O BALANCED SCORECARD (BSC) 64 em termos de capacidades, competências, motivação,
empowerment, alinhamento e estrutura organizacional em
termos de investimentos no seu futuro.
Essas perspectivas podem ser tantas quanto a organização
necessite escolher em função da natureza do seu negócio,
propósitos, estilo de atuação, etc. O BSC busca estratégias e
ações equilibradas em todas as áreas que afetam o negócio da
organização como um todo, permitindo que os esforços sejam
dirigidos para as áreas de maior competência, detectando e
indicando as que necessitam da eliminação de incompetências.
É um sistema focado no comportamento e não no controle.
Recentemente, o BSC passou a ser utilizado para criar
organizações focadas na estratégia.
Alinhamento e foco são as palavras de ordem. Aquele
significa coerência da organização; este, concentração. O BSC
habilita a organização a direcionar suas equipes de executivos,
unidades de negócios, recursos humanos, tecnologia da
informação e recursos financeiros para sua estratégia
http://www.resultco.com.br/balanced-score- organizacional; constrói um contexto para que as decisões
card/ relacionadas com as operações cotidianas possam ser alinhadas
com a estratégia e a visão organizacional, permitindo divulgar
As medidas e indicadores afetam, significativamente, o a estratégia, promover o consenso e o espírito de equipe,
comportamento das pessoas nas organizações. A ideia integrar as partes da organização e criar meios para envolver
predominante é: o que se faz é o que se pode medir. O que uma todos os programas do negócio, catalisar esforços e motivar as
organização define como indicador é o que ela vai obter como pessoas.
resultados. O foco dos sistemas e medidas tradicionalmente
utilizados nas organizações - como balanço contábil, O IMPACTO DAS TECNOLOGIAS NA
demonstrativos financeiros, retorno sobre investimento, ADMINISTRAÇÃO PÚBLICA
produtividade por pessoa, etc. - concentra-se, puramente, em
aspectos financeiros ou quantitativos – e tenta controlar As novas tecnologias vêm sendo amplamente utilizadas nas
comportamentos. Esse controle, típico da Era Industrial, não organizações públicas e privadas com a finalidade de:
funciona adequadamente. Torna-se necessário construir um aumentar a competitividade, melhorar a qualidade de produtos
modelo direcionado para a organização no futuro, colocando as e serviços, reduzir custos operacionais, minimizar esforços,
diversas perspectivas em um sistema de contínua monitoração facilitar o processo de comunicação e, principalmente, atender
em substituição ao controle. às necessidades dos clientes em geral.
Em regra, o resultado mais marcante nas empresas
O BSC é um método de administração focado no equilíbrio privadas tem sido a redução do preço final dos produtos para
organizacional e se baseia em quatro perspectivas básicas, a o cliente consumidor, e na Administração Pública tem sido
saber: o aumento e a diversificação dos serviços oferecidos pela
internet.
a. Finanças. Para analisar o negócio sob o ponto de vista
financeiro. Envolve os indicadores e medidas financeiras e
ĺ
contábeis, que permitem avaliar o comportamento da Redução do preço final
Setor
organização frente a itens como lucratividade, retorno sobre do produto
investimentos, valor agregado ao patrimônio e outros Principais impactos da privado
indicadores que a organização adote como relevantes para seu tecnologia Setor
Ampliação dos serviços
negócio. público
pela internet
b. Clientes. Para analisar o negócio sob o ponto de vista ĺ
dos clientes. Inclui indicadores e medidas como satisfação,
participação no mercado, tendências, retenção de clientes e Essa transição para a era da tecnologia e do conhecimento
aquisição de clientes potenciais, bem como valor agregado aos implicou mudanças que afetaram fortemente as organizações.
produtos/serviços, posicionamento no mercado, nível de O impacto nas organizações, decorrente da utilização das novas
serviços agregados à comunidade com os quais os clientes, tecnologias, abrange duas questões centrais: a
indiretamente, contribuem, etc. estrutura/organização das empresas e as pessoas.
c. Processos internos. Para analisar o negócio sob o Nas organizações, a adoção de novas tecnologias alterou
ponto de vista interno da organização. Inclui indicadores que a estrutura organizacional em função da rapidez das
garantam a qualidade dos produtos e processos; analisa a informações, alteração no conteúdo das tarefas e integração de
inovação, a criatividade, a capacidade de produção, o todas as áreas, permitindo acesso às informações em tempo real
alinhamento com as demandas, a logística e a otimização dos pela direção. Isso possibilitou a centralização das informações
fluxos, assim como a qualidade das informações, da e a eliminação de níveis hierárquicos – sem prejudicar o
comunicação interna e das interfaces. controle sobre o desempenho das pessoas e resultados da

64
Ibidem.

58
58
NOÇÕES DE ADMINISTRAÇÃO
organização. A estrutura hierárquica piramidal foi • as mudanças eventuais tornaram-se mudanças
substituída pelo modelo horizontal em redes. No modo de constantes;
fazer (processos), as transformações afetaram a natureza do • a cultura tradicional mudou para a cultura do
trabalho, que passou de manual para eletrônico, alterando aprendizado e da inovação.
drasticamente as tarefas realizadas. Diminuiu-se o tempo de
realização das tarefas, e cada funcionário passou a ser mais Os principais impactos verificados quanto às
cobrado sobre a qualidade de seus serviços. Os empregos pessoas foram:
diminuíram nas indústrias, mas aumentaram no setor técnico- • a figura do chefe está desaparecendo e em seu lugar está
eletrônico e de serviços. Acultura tradicional de “o que está surgindo o líder: um facilitador, mobilizador e motivador da
dando certo não se mexe” mudou para a cultura do força de trabalho;
aprendizado e da inovação. • a cultura do emprego seguro/estável está cedendo lugar
Christiane Ogassawara (2009), amparada em orientações a empregos transitórios;
da Fundação Nacional da Qualidade (2006), cita alguns • exigência de ampliação das competências relacionadas ao
fatores que influenciaram as mudanças na gestão emprego, desde como utilizar o computador/tecnologias até o
organizacional: aprendizado de novas funções;
O surgimento de um novo modelo de comunicação, atuação • o relacionamento interpessoal se profissionalizou – antes
e relacionamento, tendo como base o conhecimento e a cultura o contato era mais direto; hoje é mais a distância, mais racional,
organizacional; As organizações passaram a focar em e mais frio;
competências específicas, concentrando-se em executar o que • o treinamento eventual mudou para treinamento
consideram ser excelentes e delegar a terceiros outras constante (capacitação contínua);
atividades; As tradicionais estruturas piramidais nas • o emprego burocrático (trabalho manual) cedeu lugar
organizações cedem espaço para a interdependência das para o emprego tecnológico (trabalho eletrônico);
organizações em redes; A liderança também é retratada de • o trabalho individual mudou para trabalho em grupo (ou
forma diferenciada, não sendo vista apenas nas funções de trabalho em equipe);
planejar, organizar, comandar, coordenar e controlar, mas, sim, • surgiu o trabalho virtual a distância, realizado através de
na figura de um intermediador entre as entidades que listas de discussão, fóruns, chats, teleconferência, telefone etc.
estabelecem relações com a organização, tendo o papel de – o que exige mais disciplina;
perpetuador da cultura e dos valores da organização; A • importância cada vez maior do conhecimento e de
inovação passou a ser tema central nas organizações, tanto competências intelectuais;
interna como externamente, devido às constantes mudanças; A • as redes virtuais permitem acesso direto a autoridades
educação e o aprendizado foram inseridos nas organizações, superiores, rompendo assim a hierarquia tradicional.
passando a ser um processo permanente no qual as Atenção ĺ Quanto menor for o grau de instrução
organizações aprendem e ensinam; Foco no conhecimento científica e as competências próprias dos funcionários, maior
tácito das pessoas e no conhecimento interno da organização será o impacto.
que contribuem para a geração de valor na organização; No atual contexto de mudanças e inovações, as
Entrelaçamento da sociedade e do meio ambiente, sem limites organizações que obterão os melhores resultados
determinados, no qual as organizações são vistas como sistemas são as que possuem um forte compromisso com o
vivos e interdependentes. aprendizado: de sua força de trabalho em sentido amplo e
também em nível organizacional.
Os principais impactos verificados nas
organizações foram: Questões
• as organizações mudaram de sistemas fechados para
sistemas abertos, com formas mais flexíveis de gestão; 01. Julgue os itens subsequentes, relativos a planejamento,
• o próprio negócio, a razão de ser da organização, pode ferramentas organizacionais, gestão da qualidade e assuntos
mudar (em muitas, mudou); correlatos.
• a estruturação do trabalho mudou da visão do controle O modelo de redução de custos elaborado por Deming tem
para a de facilitação do aprendizado, com diminuição de níveis como base a melhora contínua do sistema de produção com o
hierárquicos; fim de incrementar a qualidade e produtividade e assim reduzir
• a gestão por processos substituiu os departamentos, constantemente os custos.
reduzindo tempo e custos, e proporcionando melhores ( ) Certo ( ) Errado
resultados;
• o ciclo de produção foi reduzido (agora é feito em menor 02. Na gestão da qualidade da administração pública a
tempo); pesquisa, a avaliação e a apropriação dos melhores modelos de
• a estrutura real e rígida vai cedendo espaço para serviços e processos de trabalho de organizações reconhecidas
estruturas virtuais flexíveis; como representantes das melhores práticas, denomina-se
• o controle in loco cede lugar ao monitoramento realizado (A) Reengenharia.
a distância; (B) Benchmarking.
• a comunicação formal escrita desloca-se para a (C) Matriz GUT.
comunicação em rede, virtual; (D) Método Ishikawa.
• as redes virtuais estão quebrando as hierarquias formais e (E) Método de Pareto.
permitindo o acesso direto de funcionários às autoridades
superiores; 03. Entre as novas tecnologias gerenciais e organizacionais
• o principal recurso das organizações tornou-se o aplicadas ao setor público, aquela que redesenha os processos,
conhecimento (junto com as pessoas); propondo uma mudança radical, objetivando a redução de
• os relatórios periódicos cederam lugar aos relatórios custos, tempo de execução e a melhoria na qualidade de
virtuais em tempo real; serviços é denominada

599
NOÇÕES DE ADMINISTRAÇÃO
(A) Kaizen. etapas da produção, planos de correções e melhorias, etc. Ou
(B) Programa 5S. seja, a empresa utiliza o desenho de processos para
(C) Reengenharia. não somente enxergar o funcionamento de suas
(D) Balanced Scorecard. atividades, mas também para conseguir trabalhar
(E) Brainstorming. variáveis que possam trazer ao desempenho
organizacional maior eficiência e,
consequentemente, maior eficácia nos resultados.
Respostas
É importante que você, estudante, saiba que na Gestão por
01: Certo / 02: B / 03: C. processos qualquer atividade pode ser visualizada como um
processo de negócio. Por exemplo: uma empresa poder ter
definido como um processo principal “providenciar
suprimentos para as atividades ligadas à produção”. Neste caso,
alguns subprocessos podem estar envolvidos, tais como:
realizar planejamento de compras, supervisão de atividades de
compra, administração de estoques, recebimento de materiais
Instrumentos gerenciais comprados, coordenação de projetos, logística de distribuição,
almoxarifado, controle de inventários, etc., podendo cada um
contemporâneos: gestão de processos,
destes subprocessos ser subdivididos e assim por diante.
gestão de projetos, gestão por
resultados. Quanto mais se divide um processo, maior será a
visualização e entendimento de como é a dinâmica das
atividades e, consequentemente, desencadeará maior controle,
trazendo ao gestor segurança na tomada de decisão (análise de
mudanças).
Gestão de Processos
Quando se pensa em Processos de Negócio, é possível criar
Processos, dentro da Administração, podemos considerar modelos que ajudam a entender o que acontece na empresa, de
como o conjunto de atividades ou de tarefas que são realizadas forma que esse entendimento possa proporcionar melhorias aos
por uma empresa com o objetivo de criar ou adicionar valor aos processos, ou mesmo impulsionar a criação e desenvolvimento
clientes. Um processo é definido pelas etapas de início e fim, de novos processos.
cada etapa com focos associados a diferentes clientes.
Cada atividade ou trabalho que é desenvolvido nas
Uma empresa pode ser composta por vários processos, e empresas faz parte de um processo importante e pode ser
muitas vezes enxergar esse conceito como um fluxograma, desenhado por diferentes níveis hierárquicos, tanto pelo nível
obedecendo a uma sequência de atividades a serem cumpridas estratégico, tático e operacional. Esses processos são
e, portanto, com uma sequência lógica. responsáveis pela execução das tarefas ou atividades
requeridas, que envolve os mais variados recursos de uma
Dentro dos processos, as empresas podem trabalhar com empresa: recursos humanos, tecnológicos, materiais,
divisões que facilitam a visão, compreensão e organização dos financeiros que, por sua vez, são essenciais à execução e bom
processos que envolvem o negócio, para isso utilizam desempenho de tais processos.
subprocessos voltados principalmente para atividades que
possuem maior grau de complexidade. Podemos concluir, basicamente, que um processo
organizacional pode ser entendido como a dinâmica de uma
Uma empresa pode organizar seus processos de acordo com empresa, a forma que atua para conseguir alcançar um
o enfoque desejado, voltando-o: determinado fim, podendo ser um produto ou serviço.
- Para as principais atividades da empresa;
- Para os principais negócios. Segundo Davenport65, um processo seria uma
ordenação específica das atividades de trabalho no
O enfoque também pode ser voltado para os tempo e no espaço, com um começo, um fim,
departamentos - ou mesmo áreas funcionais - contidos entradas e saídas claramente identificadas, enfim,
na gestão empresarial. O que definirá o enfoque que norteará uma estrutura para ação.
os processos são os objetivos organizacionais, a formulação de
estratégias e metas, e os planos de ação. Harrington66 define como um grupo de tarefas interligadas
Sendo assim, para que uma empresa defina redefina seus logicamente, que utilizam os recursos da organização para
processos, precisa reconhecer quais são suas prioridades, gerar os resultados definidos, de forma a apoiar seus objetivos.
necessidades, potencialidades e deficiências, para que assim
consiga entender como funciona a execução de tais objetivos - O conceito de ambos confirma a importância de uma Gestão
levando em consideração a compreensão dos fluxos por Processos bem organizada e planejada.
administrativos e burocráticos, controles de estoques,
necessidades de recursos diversos, entendimento da Além do mais, as organizações crescem e, muitas vezes, os
composição do produto, qualidade do produto ou serviço, gestores perdem o controle das operações, tornando-as

65
DAVENPORT, Thomas H. Reengenharia de processos: como inovar na 66
HARRINGTON, H. James. Aperfeiçoando os processos empresarias: estratégia
empresa através da tecnologia da informação. Rio de Janeiro: Campus, 1994. revolucionária para o aperfeiçoamento da qualidade, da produtividade e da
390p. competitividade. São Paulo: Makron Books, 1993. 342p.

60
60
NOÇÕES DE ADMINISTRAÇÃO
confusas e desordenadas, sendo assim a estrutura - Processos: é um conjunto de atividades sequenciais
organizacional, organogramas e fluxogramas perdem o sentido (interligadas), ou seja, relacionam-se entre si e que tomam um
de sua existência (pois não são seguidos corretamente) e faz input com um fornecedor, acrescentam valor e depois
com que a organização fique desorientada. produzem um output para um consumidor.
A elaboração e organização de processos amenizam o
impacto do crescimento não ordenado ou não controlado. - Subprocessos: é a parte que está inter-relacionada em
Uma organização é envolvida por pessoas, que podemos uma sequência lógica com outro subprocessos, e realiza um
citar como os stakeholders - são os funcionários, clientes, objetivo específico que compõem o macroprocesso.
fornecedores, acionistas, pesquisadores, terceirizados,
investidores, ou seja, todos aqueles envolvidos direta ou - Atividades: são ações ou operações que acontecem
indiretamente com o negócio da empresa – e recursos que dão dentro do processo ou do subprocesso. Usualmente são
capacidade e suporte para o desenvolvimento das atividades, executadas por uma área específica (unidade ou
para a elaboração de produtos ou prestação de serviços. departamento), de forma que possa produzir um resultado mais
O processo é um fenômeno de transformação e mudança e específico, trabalhado a partir das habilidades e competências
é por meio de tais recursos que a empresa consegue alinhar suas que os profissionais especializados possuem.
atividades.
- Tarefas: é o dever ou função específica do trabalho.
Estrutura e Hierarquia de Processo Sendo assim, o enfoque desta é mais específico e, portanto,
menor. A tarefa compõe as atividades, podendo ser um único
elemento de uma atividade.

Essa subdivisão das etapas de um processo é importante


para que as equipes e profissionais consigam não somente
enxergar o funcionamento das atividades, mas, principalmente,
para que seja possível que o gestor possa delegar funções e
responsabilidades de forma a priorizar as operações da
organização e assim incentivar a proposição de ideias e
melhorias, além de dar um suporte maior para a tomada de
decisão.

As atividades de uma empresa são, na realidade, um


emaranhado de processos e subprocessos interagindo entre si67,
na maior parte das vezes de forma desordenada, ou quando
muito, de forma "departamentalizada", ou seja, dentro dos
compartimentos estanques chamados de "departamentos".

O ponto crucial da questão é, portanto, determinar quais os


A figura acima apresenta a estrutura de um processo por processos de maior impacto nos negócios da empresa. Quais os
meio do diagrama de Ishikawa. Com este diagrama é possível processos "críticos" da organização, aqueles que afetam
analisar as causas e efeitos possíveis de serem identificadas, diretamente ou com maior intensidade a lucratividade ou o
tanto de forma quantitativa como qualitativa. futuro dos negócios. Antes de identificá-los é preciso, porém,
Para um processo o diagrama de Ishikawa se encaixa saber claramente quais são os objetivos estratégicos do negócio
perfeitamente na necessidade de identificação dos pontos fortes ou da empresa. Eles são o ponto de partida de todo o trabalho.
e fracos que a empresa tem e que, de certa forma, ocasionam Isso não significa que o trabalho de revisão de processos só
diversos fatores tais como: perda de materiais, desperdícios de possa ser feito se a empresa tiver um plano estratégico formal.
recursos, desperdício de tempo, setup demorado, falta de O que se pede é uma clara noção dos objetivos de médio e longo
otimização dos processos, mão de obra desqualificada, prazo dos negócios.
produtividade das máquinas e equipamentos, produtividade A partir dos objetivos estratégicos deve-se sempre definir
dos funcionários, controle de produção, controle de qualidade, primeiramente o chamado "processo macro" da empresa ou do
entre outros. negócio. Este é definido pela principal sequência de atividades
desde a entrada do pedido do cliente até a entrega do produto
Por meio da figura também se percebe que é necessário ou serviço a ele.
identificar, primeiramente, os recursos envolvidos e, O processo macro é aquele que integra as atividades de
posteriormente, estabelecer uma hierarquia para organização e todos os departamentos da empresa em relação à satisfação das
uso de acordo com o nível de abrangência e formalidade que a necessidades do cliente.
empresa possui. Para melhor entendermos sobre como funciona
as classificações de um processo, faz-se necessário classificá-lo: A iniciativa de promover a melhoria de processos deve
partir da cúpula da empresa, sendo seu comprometimento
Classificação do Processo e envolvimento fundamentais para o resultado esperado.
Um dos caminhos para se proceder à análise do processo
- Macroprocessos: são processos que lidam com mais de macro e seus subprocessos críticos é a representação destes em
uma função dentro da empresa, e seu desempenho interfere fluxogramas. A vantagem desta ferramenta é permitir a
diretamente no modo como a organização funciona. compreensão clara do processo a todos os envolvidos.

67
AMARAL, R. F. Análise e Gerenciamento de Processos. Disponível em:
http://www.guiarh.com.br/PAGINA22E.htm.

611
NOÇÕES DE ADMINISTRAÇÃO
A análise do processo requer que se questione Modelagem dos Processos
constantemente por que as atividades são feitas desta forma, se
agregam valor ao negócio ou se estão na sequência correta. A Após o mapeamento dos processos é que uma organização
partir desta análise serão tomadas decisões quanto a manter, normalmente implementa a fase da modelagem, pois por meio
modificar (geralmente simplificar) ou até mesmo eliminar do mapeamento é que a empresa terá condições de identificar
alguma atividade. possíveis falhas estruturais, a dinâmica do fluxo de trabalho,
Por fim, conhecendo-se quais os recursos humanos e como melhor gerir os recursos humanos e materiais, análise do
materiais necessários para se chegar ao processo ideal, a relacionamento interpessoal, necessidade de treinamentos e
empresa deve rever seu desenho organizacional, chegando ao análise por resultados.
detalhamento das responsabilidades de cada cargo, relativas Modelagem Organizacional: Avaliar para determinar
aos processos. se as unidades organizacionais ou pessoas listadas possuem
quaisquer necessidades e interesses únicos que devam ser
Mapeamento de Processos considerados. Descreverá os papéis e funções na organização e
as maneiras com as quais as partes interessadas interagem e,
Mapear os processos é importante porque permite conhecer então, auxiliarão a identificar partes interessadas que são
e desenvolver os caminhos percorridos no desenvolvimento do afetadas por uma mudança. (BABOK pág. 32).
trabalho até chegar ao resultado pretendido, otimizando o Levando-se em conta que nenhuma estratégia pode ser
tempo e minimizando os gargalos que atrapalham o dia a dia. seguida sem uma estrutura que a suporte, o processo de
Muitas vezes as pessoas realizam atividades, mas não modelagem desempenha, então, um papel fundamental dentro
conseguem enxergar o processo global do qual o seu trabalho de qualquer organização, seja ela de pequeno, médio ou grande
faz parte. E é importante ter a visão do todo para poder chegar porte. Considerando que o foco é sempre o negócio, o processo
ao padrão desejado. de modelagem, por sua vez, deve contar a participação de todos
É preciso que todo gestor conheça, pelo menos, os na construção do conhecimento organizacional.
macroprocessos da sua organização para gerenciar melhor e de Isto porque o modelo é constituído por submodelos, onde
forma sistêmica as atividades desenvolvidas. Assim ele poderá cada um descreve diferentes aspectos da organização, os quais
estruturá-las de modo ordenado e numa lógica que favoreça a devem, indubitavelmente, estar inter-relacionados. Tudo o que
busca contínua da eficiência na produção de um serviço ou impacta no sistema deve ser verificado, já que a visão do todo
produto oferecido aos clientes. é uma condição sine qua non dentro desse processo.
A modelagem organizacional é um passo importante dado
Analisando, mapeando e aperfeiçoando de forma adequada pela organização para o melhor conhecimento do negócio, para
os seus processos, a empresa poderá alcançar: discussão de mudanças, estruturação de regras e definição dos
1. Aumento da competitividade atores e recursos envolvidos em cada atividade.
2. Maior entendimento dos procedimentos. As unidades organizacionais são constituídas de pessoas, as
3. Maior rapidez nas soluções: quais são responsáveis pela execução dos componentes das
4. Aumento nos resultados da empresa: regras do negócio. É muito importante definir quem processa
um componente da regra e quais são os atores específicos que
Para mapear e aperfeiçoar os processos nas empresas é registram eventos, avaliam condições e realizam ações (PÁDUA
preciso pensar estrategicamente, gerar uma ação contínua e et al., 2003).
indispensável para que os produtos e serviços sejam oferecidos Carvalho e Castro (2004) fazem uma crítica argumentando
ao mercado de acordo com as exigências sempre mutantes do que as técnicas de modelagem têm sua atenção central voltada
cliente. para os aspectos da funcionalidade, ou seja, no “o quê” e no
“como” fazer. Não há uma preocupação clara com os aspectos
O mapeamento de processos sugere dois momentos inerentes à capacitação (formação e uso das habilidades) dos
distintos em uma organização: atores organizacionais e, tão pouco, ênfase nos estudos quanto
1) Diagnóstico, uma "fotografia" da situação atual à criação e aproveitamento do conhecimento acumulado pela
dos processos; organização (Gestão do Conhecimento).
2) A situação proposta, enfim, o futuro baseado Em uma gestão por processos, todos os processos - ou ao
em melhorias detectadas no mapeamento inicial e menos os processos-chave - da organização deverão ser
sugeridas pelos envolvidos. monitorados, acompanhados e avaliados por métricas capazes
Na grande maioria das vezes o mapeamento de processos de mostrar se o processo está indo bem ou não.
somente utiliza softwares de fluxogramação, não envolvendo o Sendo assim, a Modelagem dos processos é uma
gerenciamento e a modelagem. metodologia para a criação de melhores processos que
possibilitem melhorar o desempenho organizacional. Existem
Para Biazzo (2000), as etapas do mapeamento de muitas técnicas de modelagem que ajudam a compreender os
processos (processo de mapeamento): processos de negócio. É necessário que se reconheça que
- Definição dos clientes, dos principais inputs e outputs e dos existem vários modelos, símbolos e formas que podem ser
stakeholders envolvidos no fluxo de trabalho; usados para criar diagramas. A abordagem inicial de diagramas
- Entrevistas com os líderes ou gestores responsáveis pelas de fluxo de trabalho foi recomendada pelo American National
atividades dentro dos processos, estudo e análise dos Standards Institute (ANSI) em 1970, e ainda é amplamente
documentos disponíveis; utilizada.
- Criação de um modelo com base na informação adquirida Porém existe uma abordagem mais recente sobre a criação
e revisão passo a passo do modelo seguindo a lógica do ciclo. de diagramas de fluxo de trabalho denominada de Notação de
Modelagem de Processos de negócio (BPMN – Business Process
Management Notation).
Ou seja, a Modelagem de processos é a expressão de
conceitos que permite que cada organização compreenda e

62
62
NOÇÕES DE ADMINISTRAÇÃO
contribua para o seu próprio desenvolvimento. Os modelos de custos, tempo e consumo de recursos inerentes a cada um
processos são de extrema importância uma vez que promovem destes componentes.
o entendimento entre as diferentes áreas de processos, tais
como gerenciamento, engenharia, produção, etc. Principais vantagens do BPM
A Modelagem de Processos é a atividade do ciclo BPM na
qual o estado atual do processo é representado de maneira - O BPM permite a definição das atividades dos processos,
completa e precisa, criando uma documentação para conforme a necessidade de uma organização; permite a
treinamentos, comunicações, discussões, medições, análises e definição, automação e gestão dos processos como vendas,
melhorias de processos. compras, produção, contratação, suporte de produtos e serviços
Os principais objetivos da Modelagem é elaborar uma de forma clara e eficaz.
documentação de processo de negócio que atenda aos fins - Acompanhamento do real andamento dos seus Processos
desejados pelo cliente, gerando material para treinamento, de Negócio: com a implantação das ferramentas do BPM é
comunicação, discussão, medição, análise e identificação de possível monitorar o andamento de um processo a partir da
oportunidades de melhoria, bem como aumentar o nível de definição das atividades e seus respectivos executores em um
maturidade e experiência da equipe do cliente através da processo específico. Dentre as ações inerentes às atividades do
execução de projetos. BPM destacam-se as seguintes: Controle de Prazos, por usuário,
para conclusão de tarefas, Controle de Andamento de Processos
em sua projeção geral, Interação com Cliente, se necessário, em
‡ˆ‹‹­  Žƒ‡Œƒ
‘•‘Ž‹†
caso de organizações que necessitam de comunicação intensa
‘†ƒ ‡–ƒŽŠƒ ‡–‘†ƒ• ‘…—‡
ƒ„”ƒ‰² ‡–‘ ƒ–‹˜‹†ƒ†‡
‘Ž‡–ƒ†‡
–ƒ­ ‘
ƒ­ ‘†ƒ com o cliente.
‹ˆ‘”ƒ­ †‘…—‡
…‹ƒ†ƒ †‘ •†‡
ه•
†‘
–ƒ­ ‘†‘ - Controle absoluto sobre as informações de um negócio:
‘†‡Žƒ‰ ‡•…‘’‘ Ž‡˜ƒ–ƒ ’”‘…‡••‘
‡ ‡–‘
”‘…‡••‘ além da gestão do fluxo de informações em sua empresa, as
ferramentas do BPM permitem controlar por quem, onde e
quando tais informações foram acessadas. Além destes, é
possível visualizar outras vantagens. Conheça-as e entenda por
O projeto de Modelagem de Processos é guiado por
que sua empresa precisa implantar os recursos do BPM.
definições de abrangência e propósito do projeto, que
BPM permite: identificar, documentar, analisar, desenhar,
direcionam o tipo de informação a ser levantada e
executar, medir, monitorar, controlar e melhorar os processos
documentada bem como seu nível de detalhamento e
da organização.
profundidade.
O detalhamento do escopo e dos limites (interfaces) do
Fases da BPM:
processo é realizado através de reuniões com patrocinadores, o
gestor do Processo e a equipe do Projeto. Com estas definições,
1. Projeto: planejar como o projeto de gestão de processos
as atividades são planejadas e inicia-se a realização de reuniões
de negócio será realizado na empresa.
e entrevistas, estudo de documentação existente e a modelagem
Nesta etapa é realizado o diagnóstico para verificar quais os
do processo.
principais problemas e a melhor forma de resolvê-los utilizando
O nível de detalhamento da documentação do processo é
a gestão de processos. Também é o momento do projeto em que
definido de acordo com o propósito da modelagem. Como
são definidos o método, meta-modelo, notação e ferramenta
resultado, a documentação de processo é consolidada e
que serão utilizados durante todo o ciclo de BPM.
validada com a Equipe do processo e participantes-chave.
O BPM (Business Process Management) ou Gestão
2. Modelagem: os processos são levantados, detalhados
do Negócio por Processos, por meio de indicadores de
no conjunto de atividades que os compõem e validados para
desempenho, fornece à administração do negócio uma visão
garantir que o seu entendimento está correto. Nesta fase, o
global dos processos da empresa e de seus resultados,
negócio da empresa é mapeado.
possibilitando a tomada de decisões racionais, com base nestes
dados.
3. Simulação: os processos são simulados com base em
Business Process Management (Metodologia BPM) é um
medidas de desempenho. Primeiramente, são planejados os
conceito que envolve gestão de negócio aliada à tecnologia da
resultados quantitativos que o processo deve alcançar. Em
informação visando à melhoria dos processos das organizações
seguida, utilizando uma ferramenta de simulação, o processo é
por meio do uso de métodos, técnicas e ferramentas para
testado para verificar se ele está sendo executado da maneira
mapear, analisar, modelar, publicar e gerenciar processos
prevista ou se estão ocorrendo possíveis filas ou gargalos.
operacionais, de apoio e de gestão, abarcando recursos
humanos, aplicações, documentos e outras fontes de
4. Execução: os processos são implantados e
informação, com o escopo de realizar o alinhamento dos
institucionalizados. Primeiramente, é necessário planejar como
processos de negócios com a estratégia, os objetivos e as cadeias
será essa implantação, considerando necessidades de
de valores das organizações, através de práticas que, aplicadas,
treinamento, aquisição de software, implantação de novos
ajudam a maximizar os resultados e o desempenho dos
sistemas, mudanças na estrutura organizacional,
processos e, com isso, fazer com que as organizações tenham
remanejamento da equipe, etc. A gestão da mudança vai
melhores resultados financeiros, vantagem competitiva,
auxiliar a organização a fazer as modificações de uma maneira
redução de custos, otimização de recursos e aumento da
que o impacto no dia a dia dos funcionários seja o menor
satisfação dos clientes, uma vez que os produtos e os serviços
possível.
também atingem um padrão melhor de qualidade.
O diferencial do BPM está na inovação através de
5. Monitoramento: os processos são acompanhados
ferramentas que permitem não somente gerenciar um processo
através de indicadores de desempenho previamente definidos
de negócio através de seus componentes (tarefas, pessoas,
para verificar se houve desvios. Caso os processos não alcancem
máquinas e softwares), como também extrair precisamente
os resultados esperados em relação aos indicadores definidos,

633
NOÇÕES DE ADMINISTRAÇÃO
é necessário tomar ações para controlar os desvios observados, 02. (CADE - Agente Administrativo - CESPE) Na
corrigindo-os. Os indicadores consideram fatores como tempo, gestão por processos, as necessidades dos clientes são
custo, qualidade, etc. traduzidas como requisitos de processos a serem desenvolvidos
pela organização.
6. Melhoria: a situação atual dos processos é avaliada, ( ) CERTO ( ) ERRADO
melhorias são levantadas e o processo futuro modelado. Nesta
fase de melhoria é que ocorre a avaliação da situação atual (AS 03. (Petrobras Técnico(a) de Suprimento de Bens
IS), para que sejam propostas melhorias e modelado o processo e Serviços Júnior – CESGRANRIO) Uma empresa do setor
futuro (TO BE) de acordo com essas melhorias. Estas podem automotivo está reestruturando suas áreas, visando a melhorar
corresponder à inclusão, modificação ou exclusão de sua eficiência e produtividade, utilizando a gestão por
atividades, papéis, sequências, documentos, ferramentas de processos como base para essa reestruturação. A gestão por
apoio, regras de negócio, entre outros. processos, em uma concepção geral, é uma abordagem da
administração, na qual
A metodologia de Aperfeiçoamento de Processos (A) as funções de uma organização são integradas, com
Empresariais (APE) base no sequenciamento de suas atividades.
(B) as organizações apresentam suas áreas de atuação
A metodologia de aperfeiçoamento de processos de separadas, em que cada uma, isoladamente, executa as
Harrington (1993) busca a melhoria dos processos empresariais atividades que estão sob sua responsabilidade.
baseando-se nos seguintes pontos: (C) as atividades da empresa são altamente burocratizadas,
- eliminação de erros; permitindo um melhor controle e com visão mecanicista do
- minimização de atrasos; trabalho que realizam.
- maximização do uso de recursos; (D) os processos críticos, que são aqueles de natureza
- promoção do entendimento; operacional, são automatizados, contribuindo para o aumento
- sejam fáceis de usar; da eficiência e da produtividade da empresa.
- sejam amistosos para os clientes; (E) os processos são vistos como fluxos de trabalho
- sejam adaptáveis às mudanças das necessidades dos estabelecidos para cada área, e cada uma dessas áreas mantém
clientes; a visão mecanicista de suas atividades.
- forneçam à organização uma vantagem competitiva;
- reduzam o pessoal necessário. 04. (ANTAQ - Técnico Administrativo – CESPE)
Qualquer atividade que utilize recursos para transformar
Gerenciamento: Implementando e garantindo os entradas em saídas pode ser caracterizada como processo.
resultados ( ) CERTO ( ) ERRADO

Uma vez definidos o processo ideal, seus recursos e as ações 05. (ANATEL - Analista Administrativo –
necessárias para sua realização, a etapa final do trabalho Administração – CESPE) No que se refere ao gerenciamento
consiste na implementação deste processo e seu constante de processos de negócio, no contexto da administração pública,
monitoramento. Este busca medir o sucesso da implementação julgue o próximo item.
do processo ideal bem como identificar eventuais mudanças no O refinamento sucessivo de processos, conhecido como drill
mesmo, em função de novos objetivos de negócio. down, desenvolve-se a partir do contexto mais abstrato, ou
Este trabalho só é viável, porém, com a adoção de medidas macroprocesso, detalhando-o sucessivamente até a parte mais
de performance que reflitam os benefícios conquistados pela rica em descrição: a atividade.
análise e gerenciamento dos processos. ( ) CERTO ( ) ERRADO
Quaisquer que sejam os parâmetros adotados -
lucratividade, tempo, redução de custos ou qualquer outro - é 06. (Petrobras - Técnico de Suprimentos de Bens
importante que sejam quantificáveis. A adoção de medidas e Serviços Júnior – Administração – CESGRANRIO)
puramente qualitativas torna o monitoramento sujeito a Para que as atividades de uma empresa sejam realizadas, é
interpretações subjetivas e não revela o resultado real da necessária a integração de diversos fatores, tais como humanos,
melhoria do processo. capitais, tecnológicos e físicos, entre outros. Visando à
excelência organizacional, as atividades inter-relacionadas
O verdadeiro objetivo: Uma filosofia de trabalho devem ser compreendidas e gerenciadas numa abordagem de
A análise e gerenciamento dos processos trazem, sem processos. Nessa abordagem, existe uma hierarquia que
dúvida, resultados significativos na eliminação ou redução dos diferencia os processos de acordo com a complexidade de sua
chamados "custos invisíveis", mas, muito mais do que isso, estrutura.
contribuem para uma mudança de filosofia de trabalho. Uma
organização orientada a processos é consciente das Associe o nível de complexidade dos processos abaixo
necessidades de seus clientes e de seus objetivos de negócio. apresentados às suas respectivas características.
I - Macroprocesso
Questões II - Subprocesso
III - Atividades
01. (MPOG - Analista Técnico Administrativo – P - Tem objetivos específicos, sendo organizado seguindo
CESPE/2015) A gestão de processos mantém maior enfoque linhas funcionais.
no produto ou no serviço oferecido aos clientes da organização, Q - Trabalho que se inicia e termina com o cliente externo.
do que na forma pela qual o trabalho é realizado. R - Trabalho tipicamente executado por um departamento
( ) CERTO ( ) ERRADO ou uma pessoa.

64
64
NOÇÕES DE ADMINISTRAÇÃO
S - Envolve mais de uma função da organização, e sua produção na fábrica, a realização de uma viagem, escrever um
operação tem impacto significativo nas demais funções da livro, criar um documento, executar uma peça de teatro...
organização.
Repare na lista acima e confirme as duas características
As associações corretas são: mencionadas anteriormente, comuns a todos os projetos: eles
(A) I - P ; II - Q ; III - R têm um fim bem definido, e são, de alguma forma, sempre únicos.
(B) I - Q ; II - R ; III – S
(C) I - R ; II - Q ; III - S Projeto é um empreendimento não repetitivo,
(D) I - R ; II - S ; III - P caracterizado por uma sequência clara e lógica de
(E) I - S ; II - P ; III – R eventos, com início, meio e fim, que se destina a
atingir um objetivo claro e definido, sendo
Respostas conduzido por pessoas dentro de parâmetros pré-
definidos de tempo, custo, recursos envolvidos e
01. Errado / 02. Certo / 03. A / 04. Certo / qualidade.
05. Errado / 06. E
É um conjunto de ações, executado de maneira coordenada
Gestão de Projetos por uma organização transitória, ao qual são alocados os
insumos necessários para, em um dado prazo, alcançar o
Antes de mais nada, o que é o PMI? objetivo determinado. Projetos são sistemas ou sequencias de
PMI - Project Management Institute | atividades finitas, com começo, meio e fim bem definidos. Uma
(www.pmi.org) – Trata-se de uma associação, sem atividade repetitiva, ou que tem duração continua não é um
fins lucrativos, de profissionais de gerência de projeto. É uma atividade funcional ou programa. No entanto, a
projetos. É um fórum de excelência na área de duração limitada é uma condição ideal, que nem sempre ocorre
gerência de projetos, promovendo seu crescimento, nem pode ser atendida.
divulgação, educação e valor nas organizações e Na prática, alguns projetos não tem prazo exato para
praticantes. Fundado por Jim Snyder em 1969 com terminar, arrastam-se indefinidamente, terminam muito tempo
cinco voluntários nos Estados Unidos. Hoje tem mais depois da data limite, ou começam sem definição clara das
de 140.000 membros. Tornou-se organização de datas de início e de conclusão. Às vezes, um projeto precisa ser
maior credibilidade mundial na área de gerência de suspenso ou prorrogado, por causa de acidentes, eventos
projetos. imprevistos, falta de recursos ou porque a estimativa do prazo
foi incorreta. Essas dificuldades podem ser contornadas em
O que é projetos: Um projeto é uma iniciativa que é única alguns casos. No entanto, os prazos são inflexíveis em outros
de alguma forma, seja no produto que gera, seja no cliente do casos. Eleições e competições oferecem o melhor exemplo. O
projeto, na localização, nas pessoas envolvidas, ou em outro que aconteceria se não fossem realizadas no dia determinado?
fator. Isto diferencia projetos de operações regulares de uma Apesar das dificuldades, a diminuição da marem de erro na
empresa – a produção em série de margarinas é uma operação previsão dos tempos e a conclusão dentro do prazo devem ser
da empresa, mas por outro lado, a criação de um móvel sob perseguidas como ideais da administração de projetos.
encomenda é um projeto. Um projeto é um empreendimento temporário, com data de
Em segundo lugar, um projeto tem um fim bem definido, início e fim, cujo objetivo é criar ou aperfeiçoar um produto ou
ou seja, tem um objetivo claro, que quando atingido, serviço. Gerenciar um projeto é atuar de forma a atingir os
caracteriza o final do projeto. Isto faz com que o objetivos propostos dentro de parâmetros de qualidade
desenvolvimento de um novo negócio, por exemplo, possa não determinados, obedecendo a um planejamento prévio de
ser considerado um projeto. prazos (cronograma) e custos (orçamento). Ou seja, dadas as
Vamos nos ater a este segundo ponto um pouco mais. metas e as restrições de recursos e tempo, cabe ao gerente de
Imagine que você tenha uma ideia para um novo produto a ser projetos garantir que ele atinja aos objetivos propostos. Muitas
lançado no mercado, e que você queira pleitear recursos para empresas estão adotando a estrutura de projetos no seu dia a
financiar o desenvolvimento deste negócio. Para isso, você dia. Desde a concepção de um novo software até a implantação
provavelmente irá desenvolver um plano de negócios, que dos procedimentos de atendimento a clientes, desde a
conterá informações sobre o produto em si, sobre as forças do construção de uma ponte até a revisão dos processos de venda
mercado que agirão sobre este negócio (clientes, concorrentes, com vistas a aumentar a taxa de fechamento de negócios,
fornecedores, etc...), irá fazer uma análise de Oportunidades e muitos empreendimentos no seio das organizações se
Ameaças, Pontos fortes e pontos fracos, apresentará planilhas enquadram na classe de projetos.
financeiras, montará um plano de Marketing, irá mostrar o
diferencial do seu produto e seu negócio, etc. Vamos entender agora outros conceitos muito
É possível dizer que a criação deste documento completo é utilizados:
um projeto, mas o conteúdo do documento em si não, uma vez
que se trata de um negócio novo, e – salvo exceções – negócios PMP
são feitos para durar indefinidamente, não para terem um final Significa: Project Management Professional – Refere-se ao
em um determinado momento. Certificado da competência do indivíduo como Gerente de
Projetos, fornecido pelo PMI. Diferencia o profissional do
Dadas estas definições, façamos uma lista de exemplos de praticante de gerência de projetos.
projetos:
A construção de uma casa é um projeto, o desenvolvimento Critérios para Certificação PMP:
de um software, a organização de um evento, a construção de - Concordar com o código de ética
um móvel sob encomenda, a implantação de uma nova linha de

655
NOÇÕES DE ADMINISTRAÇÃO
- Formação e ter experiência profissional – Graduados com organizacional; Implementação de um novo procedimento
mais de três e 4.500 horas em GP – Não graduados mas com organizacional; Estudo da viabilidade de um investimento ou
7.500 horas de experiência em GP mudança na produção.
- Prestar exame acertando mais de 70% (200 questões)
- Entidade certificadora Alguns fatores são condicionantes assim como seus
efeitos sobre as organizações no desenvolvimento de
PMBOK um projeto.
Sua sigla significa: Project Management Body of Knowledge. - Análise de mercado. Realizar um diagnóstico das
Trata-se de um documento contendo técnicas, métodos e tendências da oferta e demanda dos produtos e serviços
processos relativos a Gerência de Projetos. O Guia PMBOK tem oferecidos.
como objetivo identificar um subconjunto dos conhecimentos -Levantamento dos investimentos necessários ao projeto;
de gerenciamento de projetos que são amplamente -Levantamento dos custos operacionais do projeto e
reconhecidos como boa prática. estimativa preliminar do ponto de equilíbrio;
É necessário entender alguns termos da definição anterior. -Efeitos do projeto sobre a estrutura organizacional e de
Identificar é sinônimo de visão geral. Amplamente capital da empresa;
reconhecido significa que é aplicável na maior parte do tempo -Definir como será feito o gerenciamento do projeto;
e que existe um consenso em relação a sua utilidade. Boa -Analisar a estrutura e a projeção dos custos;
prática é aquela que pode aumentar as chances de sucesso -Definir o horizonte do projeto;
numa ampla gama de projetos. Note que o fato de ser boa -Projetar o fluxo de caixa dentro do horizonte do projeto;
prática não implica ser de uso obrigatório. -Analisar a viabilidade econômica do projeto;
O PMBOK é geral e por isso pode ser aplicado a qualquer -Estimar o retorno do investimento;
tipo de projeto. Por último, destacamos que o guia tem um
papel importante de promover um vocabulário relacionado ao Gerente de Projetos:
gerenciamento de projetos.
Tem como objetivo desenvolver o produto/serviço esperado
Gerenciamento de projetos dentro do prazo, custo e nível de qualidade desejados. Em
A administração de um projeto é o processo de tomar muitos casos, o cargo de gerente de projeto é um cargo virtual,
decisões que envolvem o uso de recursos, para realizar de existência temporária, ocupado por um funcionário da
atividades temporárias, com o objetivo de fornecer um estrutura permanente, executivo ou não. Essa pessoa recebe a
resultado. O resultado pode ser um produto físico, conceito, ou incumbência de administrar o projeto, em regime de dedicação
evento, ou, em geral, uma combinação desses três elementos. exclusiva ou acumulando essa tarefa com outras, sem
Conhecer e aplicar os princípios e as técnicas da administração desvincular-se de seu cargo original. Terminado o projeto, o
de projetos são habilidades importantes para todas as pessoas gerente volta a seu cargo permanente ou assume outro projeto.
que se envolvem com projetos. Em outros casos, como nas empresas de consultoria e de
construção, a posição de gerente de projetos é fixa, sendo
Gerenciamento de projetos é: ocupada quase sempre pelas mesmas pessoas. Qualquer pessoa,
“Aplicação de conhecimentos, habilidades, ferramentas e ocupante de um cargo gerencial ou funcionário sem posição de
técnicas às atividades do projeto a fim de alcançar seus objetivos.” gerente, pode assumir o papel de gerente de projeto. Pode ser
(PMBOK Guide, 2000). o executivo principal outros executivos, ou pessoas sem posição
gerencial, como profissionais, técnicos, cientistas, professores
O gerenciamento de um projeto varia muito em termos de ou mesmo estudantes.
finalidade, complexidade e volume de recursos empregados. O enxugamento dos quadros de pessoal e o aumento da
Apesar das variações, os princípios de administração que devem necessidade de especialização técnica têm levado muitas
ser utilizados são sempre os mesmos: empresas a recrutar no mercado profissionais por período
- A administração de projetos é uma técnica (ou conjunto determinado apenas para a execução de projetos específicos.
de técnicas que se aplica a determinadas situações). Neste contexto, entender o processo de gerenciamento de
- A aplicação das técnicas da administração de projetos projeto tem se tornado vital para organizações a medida em que
depende tanto da natureza intrínseca da situação quanto de mais e mais novos negócios vão se revestindo da aura de projeto
escolha consciente. e passam a exigir um cabedal de técnicas gerenciais que nem
- A tarefa básica da administração de projetos é assegurar a sempre estão disponíveis nas empresas.
orientação do esforço para um resultado. Controlar custos e
prazos é condição básica para realizar o resultado. Estrutura do Projeto
Assim como em uma empresa existe a estrutura
Podemos considerar projetos como um conjunto organizacional, o projeto também deverá ter essa estrutura. Um
de atividades ou medidas planejadas para serem projeto muitas vezes está vinculado a uma empresa, então esse
executadas com: projeto terá características da estrutura organizacional da
a) Responsabilidade de execução definida; empresa do qual esse projeto pertence. Se a estrutura da
b) Objetivos determinados; empresa é centralizada, naturalmente o projeto terá um sistema
c) Abrangência ou escopo definida; centralizado, pode acontecer o contrário, pode, mas é mais
d) Prazo delimitado; raro. Um projeto centralizado, ou que tem uma gerencia
e) Recursos específicos. centralizada, é o projeto que possui apenas um supervisor geral,
esse supervisor geral é aquele que estará à frente do projeto
Os projetos são desenvolvidos nas organizações com os sempre. E o andamento do projeto centralizado é aquele que o
objetivos de desenvolver novos produtos ou serviços; projeto acontece em forma de linha. Ou seja, o planejamento é
Aprimoramento de produtos ou serviços já existentes na um só e normalmente não há modificações. Essa estrutura de
empresa; Alteração ou nova implementação na estrutura projeto centralizado ocorre muito em projetos de construção.

66
66
NOÇÕES DE ADMINISTRAÇÃO
Como construção civil, montadores de automóveis. Os projetos produto. O ciclo de vida de um projeto aponta as fases que
centralizados, normalmente se utilizam de uma linha de devem ser executadas desde o início de um projeto até o seu
produção, onde cada membro desse projeto é responsável por encerramento.
um setor e nada mais. Já o ciclo de vida de um produto é determinado pelo
Linha de produção é uma teoria administrativa criada por período entre seu lançamento no mercado e sua retirada de
Winslow Taylor, onde ele acreditava que deveria existir o comercialização. Um produto pode ter um ciclo de vida
funcionário padrão, ou seja, o funcionário deveria ser indefinido (como o antigo Fusca, um produto que ficou décadas
especializado em apenas uma função dentro da empresa. Se em comercialização).
analisarmos o contexto da época, que era o início da era De acordo com o PMBOK: O ciclo de vida do produto consiste
industrial, essa teoria era bem válida. Nos dias de hoje muitas em fases do produto, geralmente sequenciais e não sobrepostas,
empresas, principalmente indústrias ainda possuem, com determinadas pela necessidade de produção e controle da
algumas adaptações a chamada linha de produção. Em se organização. A última fase do ciclo de vida de um produto é a
tratando de um projeto adotar o sistema de linha de produção retirada de circulação do produto. Geralmente o ciclo de vida de
normalmente é o mais recomendado, pois o sistema é um projeto está contido em um ou mais ciclos de vida do produto.
centralizado, para cada etapa do projeto, há um especialista e É necessário ter cuidado para distinguir o ciclo de vida do projeto
existe um comando central que delega, organiza e administra do ciclo de vida do produto.
todo o projeto. Ou seja, o ciclo de vida de um produto pode conter diversos
O outro método, o descentralizado, o projeto não possui um ciclos de projetos. O Fusca, por exemplo, passou por diversas
comando central, ou seja, não há uma pessoa, um gerente a “remodelagens”, e cada uma dessas envolveu projetos
frente do projeto, cada parte do projeto tem um gerente da área diferentes.
específica e essas cada um tenta se interligar com o outro. Essa
teoria é muito boa para projetos não físicos, ou seja, projetos Custos e Riscos no Ciclo de Projetos
que visam apenas uma abordagem mais teórica, como Quanto mais avançamos em uma fase, maiores
pesquisas, trabalhos de consultoria empresarial. Vale lembrar são os custos acumulados em um projeto. Isso é
que mesmo não sendo um projeto de estrutura centralizada, o bastante simples de entender, não é mesmo? Se você analisar o
projeto deve sempre ter uma estrutura não em termos projeto de uma casa, por exemplo, quanto mais avançarmos na
administrativos, mas sim, na forma como ele será efetuado, obra, maiores serão os gastos acumulados envolvidos.
centralizado, para que aqueles que trabalham nele, não percam Entretanto, o contrário ocorre com os riscos. Naturalmente,
o foco no que estão fazendo, afinal, um projeto não pode ficar quanto mais avançada está a fase, menores se
cada parte de uma maneira, ele deve ter um começo, um meio tornam os riscos de insucesso. Assim, quanto mais
e um fim bem definidos. “perto” de um objetivo estamos, mais seguros ficamos sobre o
sucesso da empreitada, pois existem menos variáveis que
Ciclo de vida do projeto podem dar errado.
Quando falamos em ciclo de vida do projeto Além disso, a utilização das pessoas normalmente começa
estamos nos referindo desde o começo do projeto, de um nível baixo e vai evoluindo com o passar do tempo. Ou
até o seu final. Todo projeto possui um ciclo de vida. seja, começamos os projetos com poucos
O ciclo de vida de um projeto consiste nas diversas fases por profissionais e vamos aumentando esse número.
que um projeto passa. Quando falamos em ciclo de vida Entretanto, após certo tempo, a utilização de pessoas cai
do projeto estamos nos referindo desde o começo do bruscamente até o encerramento do projeto, em que todas as
projeto, até o seu final. Todo projeto possui um ciclo de pessoas serão desmobilizadas. O mesmo ocorre com os gastos
vida. Cada projeto tem um começo, um meio e um fim. No por unidade de tempo, que começam normalmente
começo, o projeto ganha forma em papel. É a fase mais delicada mais baixos, são aumentados durante o
do projeto, pois se algo der errado, no começo, todo o resto fica desenvolvimento do projeto e caem bastante no
comprometido. É no começo que se define, o valor do projeto, final.
quais serão os custos em termos de material e humano. O tempo Outro fator importante é a capacidade e o custo de se fazer
que levará para terminar o projeto. Quem serão os responsáveis uma alteração no projeto. Em seu início, o custo é
por dirigir o projeto. A data de início do projeto. baixo e a possibilidade alta de se poder alterar
O durante, ou meio, é a parte de realização do projeto. É algum aspecto no projeto. Assim sendo, antes de começar
nessa hora que todo o planejamento do início é colocado em uma obra é razoavelmente fácil alterar o projeto e incluir algum
prática. Como já dito antes, o planejamento é extremamente ambiente, por exemplo. Já com a obra no “meio do caminho”
importante para que no momento que começa a construção do teremos de quebrar paredes e desperdiçar material, tempo e
projeto, nada saia errado, e o principal, durante a realização do dinheiro, não é mesmo? Ou seja, com o passar do tempo,
projeto, pouco coisa deve ser alterada. Ou se for, deve ser o fica cada vez mais caro e mais difícil fazermos uma
mínimo possível, para que o projeto fique exatamente como foi alteração no projeto.
planejado. E no final, a entrega do projeto. Parece meio óbvio,
mas muitos projetos tendem a não ser entregues, ou então ETAPAS
tomam um rumo completamente diferente do projeto inicial.
Temos que lembrar que as empresas muitas vezes elaboram e Processos de gerenciamento de projetos
fazem projetos para terceiros, ou seja, muitas vezes os projetos Diante disso, o gerenciamento de projetos, de acordo com o
são para outra pessoa. Então a empresa não pode no meio do PMBOK (Project Management Body of Knowledge), é realizado
caminho abandonar o projeto, ou então alterá-lo. por meio da aplicação e da integração de processos que se
relacionam com a descrição, a organização e a finalização do
Ciclo de Vida de um Projeto x Ciclo de Vida de um trabalho do projeto. Assim, os processos de gerenciamento de
Produto projetos são agrupados em cinco categorias conhecidas como
Algumas bancas gostam de confundir o candidato grupos de processos de gerenciamento de projetos (ou grupos
misturando o conceito de ciclo de projeto com o de ciclo de de processos), os quais são: 1.Processos de Iniciação,

677
NOÇÕES DE ADMINISTRAÇÃO
2.Processos de Planejamento, 3.Processos de 02. Segundo o PMBOK, as etapas de iniciação,
Execução, 4.Processos de Monitoramento e Controle e planejamento, execução, monitoração/controle e encerramento
5.Processos de Encerramento. representam apenas o
(A) ciclo de vida dos projetos ou ciclo de gerenciamento de
1.Processos de Iniciação – fase inicial em que se define projetos.
o projeto, as necessidades são identificadas, autorização do (B) grupo de processos dos projetos ou ciclo de
projeto. Em geral é uma etapa que deve ser desenvolvida em gerenciamento de projetos.
uma reunião de brainstorm. (C) grupo de processos dos projetos ou ciclo de vida dos
2.Processos de Planejamento – nessa fase se define e projetos.
refina o objetivo do projeto, planejam-se as ações necessárias (D) grupo de processos do gerenciamento de projetos ou
para atingir os objetivos e o escopo para o qual se propõe o ciclo de vida dos projetos.
projeto, além de serem desenvolvidos planos auxiliares para (E) grupo de processos do gerenciamento de projetos ou
gestão do projeto (plano de qualidade, comunicação, riscos, ciclo de gerenciamento de projetos.
atribuições e responsabilidades). São processos interativos de
definição e refinamento de objetivos e escolha dos melhores 03. Com relação ao ciclo de vida dos projetos, julgue o item
caminhos para atingir os resultados. O resultado do a seguir.
planejamento é uma lista de tarefas e/ou um gráfico de A abordagem do ciclo de vida de um projeto em fases
Gantt/cronograma. sobrepostas é adequada para a otimização do tempo.
3.Processos de Execução – integra pessoas e os outros
recursos para colocar em prática o plano do projeto. É ( ) Certo ( ) Errado
geralmente nessa fase em que ocorre a maior parte do
esforço/dispêndio do projeto. 04. A aplicação de conhecimentos, habilidades,
4.Processos de Monitoramento e Controle – ocorre ferramentas e técnicas às atividades do projeto a fim de atender
em paralelo ao processo de execução. Mede e monitora o aos seus requisitos denomina-se:
desempenho do projeto para identificar variações em relação (A) Gestão de Risco.
ao planejado para que ações corretivas sejam disparadas (B) Gestão do Conhecimento.
quando necessário, garantindo que os resultados do período (C) Gerenciamento de Projetos.
sejam alcançados. (D) Gerenciamento Contábil Financeiro.
5.Processos de Encerramento – formaliza a aceitação
do projeto, serviço ou resultado e o fechamento formal do 05. É considerado um guia de orientação para os
contrato. Analisa a evolução do projeto para que erros não se profissionais sobre o conhecimento em gerenciamento de
repitam no futuro, além disso, também é importante identificar projetos e trata-se de uma bibliografia de referência, cujo
os acertos para que eles voltem a acontecer em outros projetos. propósito é identificar e descrever conceitos e práticas do
Atualizar a base de conhecimento de lições aprendidas. gerenciamento, padronizando a terminologia dos processos
utilizados. Estamos falando do:
Os grupos de processos são ligados pelos
resultados que produzem: o resultado de um processo (A) PMI
frequentemente é a entrada de outro. Uma saída com (B) PMBOK
falhas pode comprometer a entrada de processos (C) COBIT.
dependentes. Os cinco grupos de processos possuem (D) ITIL.
conjuntos de ações que se bem aplicadas e planejadas (E) SLTI.
garantem a taxa de sucesso do projeto. Outro
destaque é que o Guia PMBOK deixa bem claro que os Respostas
grupos de processos não são fases do ciclo de vida do
projeto (as quais são apenas início, meio e fim). E / 02. E / 03. Certo / 04. C / 05. B

Questões GESTÃO PÚBLICA POR RESULTADOS

01. Um gerenciamento por projetos significa: A busca pelo aperfeiçoamento do funcionamento da


(A) gerenciar pela definição e previsão de requisitos, máquina pública é quase tão antiga quanto a existência de um
cronograma, esforço e custos em um momento permanente da quadro profissional de funcionalismo público. É recorrente na
empresa. história administrativa brasileira a criação e implementação de
(B) fazer o acompanhamento, a avaliação e o órgãos, programas e reformas que objetivem aprimorar o
monitoramento das ações do projeto. funcionamento da máquina do Estado Brasileiro. A respeito dos
(C) ter como objetivo na empresa uma infraestrutura êxitos alcançados ao longo de todo esse tempo, é nítida a
necessária, ambiente e treinamentos e a partir disso sensação de que a máquina pública ainda deve muito à
desenvolver o projeto. sociedade, sobretudo considerando que o Estado brasileiro, em
(D) gerenciar os riscos e custos da empresa. todas as suas esferas, seja federal, estadual ou municipal, é tido
(E) uma aplicação de conhecimentos, habilidades, como burocrático, lento e ineficiente.
ferramentas e técnicas às atividades do projeto a fim de O desafio é de promover as mudanças no que tange a
alcançar seus objetivos que consistem em desenvolver o necessidade de repensar a questão da governança e dos
produto/serviço esperado dentro de um prazo definido, custo e modelos de gestão, ao mesmo tempo em que vai exigir
nível de qualidade desejados. mecanismos inovadores de relacionamento com a sociedade. A
emergência do terceiro setor e dos movimentos sociais, além
dos movimentos que têm como base o voluntariado, introduz

68
68
NOÇÕES DE ADMINISTRAÇÃO
elementos desafiadores nessa nova configuração em rede do processos e assim remediar a disfunção relacionada ao apego
Estado contemporâneo68. exacerbado às normas e procedimentos, e propiciar mais
Uma das principais tendências da gestão contemporânea, eficiência e accountability.
na perspectiva do desenvolvimento organizacional, tem sido o De forma breve, esse modelo se caracterizaria, na etapa de
intercâmbio de experiências institucionais por meio da formulação da política pública, pela tradução dos objetivos em
disseminação das melhores práticas. resultados e, na etapa de implementação das ações para atingi-
Na administração pública gerencial busca-se desenvolver los, pela predominância da orientação por resultados em
uma cultura gerencial, com ênfase nos resultados, e aumentar substituição aos outros mecanismos de coordenação, como a
a governança do Estado, isto é, a sua capacidade de gerenciar coordenação por supervisão direta, a padronização de
com eficiência e efetividade. O cidadão é o principal procedimentos e a padronização de habilidades.
beneficiário dos serviços do Estado. Sendo assim, a A gestão por resultados é uma ferramenta administrativa
Administração pública gerencial é aquela construída sobre que, por meio de sua metodologia, alinha o planejamento, a
bases que consideram o Estado uma grande empresa cujos avaliação e o controle, promovendo eficiência e eficácia na
serviços são destinados aos seus clientes, outrora cidadãos; na organização. Tem foco na efetividade e na flexibilização dos
eficiência dos serviços, na avaliação de desempenho e no processos, minimizando o excessivo apego às normas e
controle de resultados, suas principais características. procedimentos, promovendo mais eficiência, efetividade e
accountability da gestão pública.
Contudo, o exame das iniciativas realizadas indica uma
trajetória quase padrão, em que é possível identificar alguns Accountability70
elementos comuns nas estratégias adotadas, como por
exemplo: Accountability é um termo inglês, originado do latim
a) fortalecimento de canais de relacionamento accomptare (tomar em conta), derivado da forma prefixada
Estado/sociedade a partir de iniciativas orientadas para a computare (computar), que por sua vez deriva de putare
melhoria da qualidade na formulação das políticas públicas; (calcular), sem tradução literal para o português – como forma
busca de maior participação cidadã; introdução de mecanismos mais próxima tem-se usado responsabilização, que se
de controle social; melhoria na entrega de serviços públicos e relaciona à obrigação de prestar contas, pelo membros de
maior envolvimento do terceiro setor; um órgão administrativo ou representativo a instâncias
b) aperfeiçoamento do marco legal, visando à controladoras ou aos seus representados. Numa tradução mais
eliminação da excessiva burocracia, que impedem a adoção de rude, pode-se confundir com o próprio termo prestar contas.
modelos de gestão orientados a resultados com maior
flexibilidade e autonomia; A Gestão por Resultados caracteriza-se pela tradução de
c) recuperação da capacidade financeira objetivos em resultados, proporcionando um marco coerente
(racionalismo econômico), a partir de iniciativas voltadas para a melhoria dos processos de trabalho na qual a informação
para a melhoria da eficiência, e introdução da cultura da sobre o desempenho é usada para melhorar a tomada de
responsabilidade fiscal; decisão. Portanto, é uma dimensão fundamental para a gestão
d) desenvolvimento das capacidades estratégica, pois constituem o fluxo de atividades necessárias
institucionais dos órgãos da administração pública, para alcançar os resultados da organização.
incluindo o fortalecimento do planejamento estratégico como
ferramenta de gestão, programas de reestruturação São consideradas funções da gestão por
administrativa, mecanismos de contratualização, intensificação resultados:
do uso da tecnologia da informação e a terceirização de serviços
de apoio; - Definir a direção estratégica da organização -
e) desenvolvimento do capital intelectual, a partir criação da missão e estabelecimento dos objetivos estratégicos,
de iniciativas voltadas para a profissionalização e modernização levando em consideração a visão e os valores da organização.
do serviço civil, intensificação de programas de capacitação de Refere-se à definição do caminho que deverá ser seguido para
servidores e de lideranças, ética na administração pública, alcançar os objetivos estratégicos. É necessário mapear os
adoção de mecanismos inovadores de remuneração de processos de trabalho e escolher os indicadores e as metas de
funcionários e introdução da gestão do conhecimento. desempenho que mostrarão se a organização está no caminho
planejado e, através de uma avaliação, o quanto de progresso
Modelo de Gestão por Resultados69 foi feito na direção dos objetivos.
- Implementar e administrar o processo de
O Modelo de Gestão por Resultados é a ferramenta que visa mudança alinhado com a direção estratégica -
levar a organização a priorizar o resultado em todas as ações, compromisso da organização com a ação;
levando a otimização do desempenho governamental. - Melhorar continuamente o desempenho das
- A Gestão por Resultados procura que as ações públicas atividades em andamento - garantia que a melhoria
alcancem um bom resultado, ou, o melhor resultado possível. contínua seja praticada em cada programa ou atividade da
O modelo de Gestão por Resultados é uma das principais organização através de planos de ação frequentemente
recomendações do receituário da Nova Gestão Pública. Isso monitorados.
decorre do argumento de ser este modelo apropriado a,
simultaneamente, focar na efetividade ou no que de fato Dessa forma, pode-se entender que a gestão por resultados
interessa ao cidadão e a sociedade, flexibilizar a condução dos é desenvolvida por meio de um ciclo que começa com o

68
MARINI, C. O contexto contemporâneo da administração pública na 70
PETER, M. G. A; MACHADO, M. V. V. Manual de contabilidade
América Latina. Revista do Serviço Público. Ano 53, Nº 4, Out-Dez 2002. Governamental. 2 Ed. – São Paulo: Atlas 2014.
69
Disponível em:
http://www.planejamento.gov.br/secretarias/upload/Arquivos/seges/eppgg/pr
oducaoacademica/tese_eduardogranha.pdf

699
NOÇÕES DE ADMINISTRAÇÃO
estabelecimento dos resultados desejados, a partir realidade, representando conceitos abstratos, como o nível de
da tradução dos objetivos; seguido do desempenho de uma gestão, as condições de vida de uma
monitoramento e da avaliação do desempenho da população, etc.
organização a partir do alcance desses resultados; e Os indicadores viabilizam a tradução dos objetivos em
retroalimenta o sistema de gestão, propiciando medidas apreensíveis, e no presente caso, a tradução dos
ações corretivas decorrentes dessa avaliação. objetivos em indicadores de resultados. Os indicadores são à
base da memória. O conjunto de indicadores deve auxiliar a
O modelo de Gestão por resultados confere grande gestão do trabalho diário. Uma série de indicadores só tem
autonomia processual aos agentes executores, e as sentido se gestores e colaboradores analisarem os resultados
características que compõe ainda sua estrutura: para saber se é preciso fazer menos, mais, ou melhor. Eles
devem ser lembrados a todo o momento, não como tarefa que
- Alinhamento de expectativas de forma clara e precisa ser feita, mas como preciosos instrumentos de análise.
transparente, entre todos os colaboradores, por meio da Os indicadores quantitativos e qualitativos devem estar ligados
tradução destas em resultados e metas a serem atingidas, o que aos objetivos e a visão da organização, devendo servir de apoio
inclui a definição de indicadores para sua apuração; para detectar as causas e os efeitos de uma ação, e não apenas
- Concessão de autonomias aos executores / seus resultados.
implementadores da gestão estratégica;
- Contratualização de resultados, autonomias e sanções; Características necessárias para a construção de
- Avaliação dos resultados e retroalimentação do sistema de indicadores:
gestão para eventuais correções de rota, constituindo assim
uma ferramenta gerencial; Comparabilidade – devem permitir a comparação
- Fortalecimento de uma modalidade de “accountability” – temporal espacial, entretanto quanto mais generalizado for o
baseada no desempenho mensurado a partir de indicadores de indicador, menores serão as especificidades que este poderá
resultados; captar da realidade estudada;
- Modificação do comportamento peculiar da burocracia, Disponibilidade da informação – as bases de dados
substituindo-o pela atenção a metas claras e contratualizadas. devem ser acessíveis e confiáveis e oferecer séries históricas;
Quantificáveis – traduzíveis em números, mas que
A vantagem esperada do modelo para o aumento da possibilitem uma análise qualitativa do desempenho da gestão;
eficiência da administração pública é bastante evidente. A Simplicidade – devem ser de fácil compreensão e
transparência dos objetivos, resultados e metas da organização aplicação, apesar de tentar retratar realidades complexas.
tende a reduzir os problemas relacionados aos gap’s de
informação e atualização dos processos de trabalho, Assim, uma vez que o modelo prescreve o aumento da
possibilitando o compartilhamento e o engajamento coletivo flexibilidade e a autonomia dos gestores sobre os meios,
em torno dos mesmos objetivos, o que favorece a racionalidade recursos e processos, estabelecendo controles a posteriori, são
do sistema e da organização. Além disso, isso ainda permite um essenciais instrumentos de contratualização entre os gestores
controle mais efetivo sobre o desempenho dos funcionários, da gestão estratégica, por meio dos quais são definidos:
pelos clientes e demais partes interessadas. Finalmente,
concede aos gestores autonomia sobre a gestão dos recursos, - Os resultados a serem alcançados, traduzidos em indicadores
viabilizando a adequação eficiente entre recursos, processos e e metas claras;
metas. - A flexibilidade ou as autonomias concedidas aos gestores e;
- As sanções, positivas e negativas, que serão aplicados quando
Avaliação de Desempenho e Resultados da avaliação do alcance dos resultados.

Indicadores de Resultados71 Implantação da Gestão por Resultados72


Após a tomada de decisão referente adoção da gestão por
Indicadores são informações mensuráveis que permitem resultados e também às alternativas para atingir os objetivos, a
verificar o alcance dos resultados. Para cada indicador deverá etapa seguinte é a implantação do modelo. Nessa etapa,
ser apresentado descrições, fórmulas, fontes, índices de compete ao gestor coordenar a implantação, procurando
referência e outras informações que se façam necessárias para vincular dinamicamente os recursos aos objetivos. Para tanto, a
o monitoramento e a gestão do conjunto de indicadores dentro função de coordenação pode ser empreendida por outro
da organização. conjunto de mecanismos, que segundo Mintzberg são os
A mensuração do desempenho por meio de indicadores é seguintes:
indispensável na gestão por resultados, pois sem ela os
funcionários tendem a esquecer quais são as atividades e como - Ajustamento mútuo - típico de tarefas que envolvem
elas devem ser realizadas. É um processo de busca, obtenção e grupos pequenos, a coordenação é obtida pelo simples processo
divulgação de informações sobre fatores críticos para o de comunicação informal. São realizadas reuniões com o
desempenho da organização, pela utilização de indicadores objetivo de discutir os processos de trabalho, ajustando-os
qualitativos e quantitativos que sinalizem o alcance dos quando necessário;
objetivos da organização. - Supervisão direta - segundo este mecanismo, uma
Os indicadores são utilizados para operacionalizar o modelo pessoa ou organização coordena o processo, por meio de
ou para permitir a avaliação do alcance das metas propostas. instruções, cobranças, alocação de recursos, etc;
São números que buscam traduzir um ou mais aspectos da

71
Disponível em: 72
Disponível em:
http://www.planejamento.gov.br/secretarias/upload/Arquivos/seges/eppgg/pr http://www.planejamento.gov.br/secretarias/upload/Arquivos/seges/eppgg/pr
oducaoacademica/tese_eduardogranha.pdf oducaoacademica/tese_eduardogranha.pdf

70
70
NOÇÕES DE ADMINISTRAÇÃO
- Padronização de normas - significa que os (E) O controle por resultados é um instrumento da
funcionários compartilham um conjunto de crenças e valores; é administração gerencial e pode envolver a contratualização de
exposta a compreensão de cada um em relação às normas, com resultados e contratos de gestão.
o objetivo de criar uma ideia coletiva de conduta, obtendo,
informalmente, a coordenação a partir delas; 02. (SUDENE-PE - Agente Administrativo –
- Padronização de processos - refere-se à prescrição FGV/2013) Na Administração Pública, a gestão por
do conteúdo do trabalho por meio de procedimentos, resultados está relacionada à
normalmente escritos, a serem seguidos. Trata-se do (A) Administração Gerencial.
mapeamento dos processos e da manualização dos (B) Administração Patrimonialista.
procedimentos. Na iniciativa privada, é muito comum em (C) Administração Centralizada.
programas de qualidade, como aqueles promovidos pela (D) Administração Burocrática.
International Organization for Standardization com a série ISO- (E) Administração Descentralizada.
9000. No caso das organizações públicas, podemos associar esta
padronização às regras formais burocráticas ou à própria 03. (PROCEMPA - Analista Administrativo -
legislação; Analista Financeiro Contábil – FGV/2014) Na
- Padronização de resultados - trata-se da Administração Pública, o grau de aceitação pela clientela dos
especificação dos resultados a serem atingidos, em substituição serviços prestados é fundamental para a imagem institucional.
à especificação dos meios como os procedimentos ou O alinhamento de expectativas de forma clara e
habilidades; transparente, entre todos os agentes políticos, externos e
- Padronização de habilidades - refere-se à designação internos; a contratualização de resultados, autonomias e
de pessoal qualificado, já possuidor de determinada habilidade sanções; a avaliação dos resultados e a retroalimentação do
adequada ao trabalho a ser feito. Não é o trabalho, mas o sistema de gestão para eventuais correções de rota; o
funcionário que é padronizado. A coordenação é obtida em fortalecimento de uma modalidade de “accountability”,
razão do funcionário já possuir determinado conhecimento. No baseada no desempenho mensurado a partir de indicadores de
setor público, podemos entender os requisitos dos concursos resultados; e a modificação do comportamento auto referido da
públicos como um esforço nesse sentido, particularmente para burocracia, substituindo-o pela atenção a metas claras e
contratação de especialistas como médicos ou dentistas. contratualizadas, são características da
Também é importante destacar outros mecanismos que (A) gestão participativa.
possuem função de coordenação e, embora estejam de alguma (B) gestão por competências.
forma associados àqueles já descritos, merecem tratamento à (C) gestão do conhecimento.
parte: (D) gestão da cultura organizacional.
A liderança e a estrutura. A perspectiva contemporânea de (E) gestão por resultados.
liderança é definida como “... um processo no qual um
indivíduo influencia outros a se comprometerem com a busca 04. (IDAF-ES - Advogado – FUNCAB) Quanto à gestão
de objetivos comuns.” E isso ocorre por meio do estímulo à de resultados no serviço público, é correto afirmar que:
autonomia dos liderados: “...o líder cria nos liderados a (A) serve como instrumento de controle da atuação dos
sensação de um espaço próprio, onde eles podem desenvolver gerentes públicos.
sua criatividade e se autorealizar na execução de suas tarefas”. (B) conduz a resultados parciais sem qualquer influência no
Ressalta-se que a busca do compartilhamento de objetivos pelo resultado final.
líder se dá por negociações constantes, à semelhança do (C) promove e desenvolve mecanismos internos que
ajustamento mútuo, e não por meio do exercício de uma melhoram o desempenho dos dirigentes e servidores públicos.
eventual autoridade hierárquica, como seria típico do (D) a gestão de resultados tem como objetivo minimizar os
mecanismo de supervisão direta. conflitos organizacionais, de modo a pacificar o ambiente de
Além disso, é também importante mencionar que trabalho.
lideranças, principalmente as carismáticas, valem-se da (E) é necessário realizar, periodicamente, encontros de
coordenação pelo compartilhamento de crenças e valores, e não diferentes grupos de trabalho, de modo a humanizar as relações
somente de objetivos. interpessoais.

Questões Respostas

01. (TRE-SE – Analista Judiciário – Área 01. E. / 02. A / 03. E / 04. C


Judiciária – FCC/2015) Sobre a gestão de resultados na
Administração pública, é correto afirmar:
(A) O estabelecimento de metas no setor público e no 
setor privado são semelhantes, na medida em que é orientado
pelo cumprimento de procedimentos e normas.

(B) A legalidade e a impessoalidade são características da
administração burocrática e, consequentemente, orientam a

elaboração de metas e o seu alcance.
(C)A contratualização de resultados é um instrumento da

administração burocrática e consiste no estabelecimento de
parcerias público-privadas.

(D) A hierarquia é essencial para o estabelecimento de um
contrato de gestão, pois essa característica assegura o próprio

cumprimento do contrato. 

711
NOÇÕES DE ADMINISTRAÇÃO
 
 
 
 
 
 
 
 
 
 
 
 
 
 
 
 
 
 
 
 
 
 
 
 
 
 
 
 
 
 
 
 
 

72
72

Вам также может понравиться